Sie sind auf Seite 1von 3817

291

CMEinfo presents a
definitive multimedia course

41ST ANNUAL
INTENSIVE REVIEW OF
INTERNAL MEDICINE
41st Annual
Intensive Review of Internal Medicine
Provided by:
Brigham and Women’s Hospital
The Department of Medicine
Harvard Medical School
Postgraduate Medical Education
______________________________________________________________________________

Date of Original Release: October 1, 2018


Termination Date: January 31, 2021
(Please note that AMA PRA Category 1 Credits™ will no longer be issued for the activity after this date)

Estimated Time to Complete the Activity: 85 hours

Media: This educational activity is in the form of an online video and MP4 and/or Audio MP3

PROGRAM DESCRIPTION
The Intensive Review of Internal Medicine course is designed to enhance internal medicine
knowledge by offering a comprehensive update in internal medicine and its subspecialties. It will
consist of a case-based review of challenging clinical problems and review of literature to guide
evidence-based practice. This course can also serve to prepare attendees for the ABIM Board
Examinations (Certification/Recertification).

LEARNING OBJECTIVES
Upon completion of this activity, participants will be able to:
• Apply current/recommended guidelines in clinical practice
• Perform differential diagnosis of complex clinical presentations
• Identify/integrate current therapeutic options for specific disorders,
including end-of-life care
• Review and interpret up-to-date literature relevant to clinical practice
• Describe pathophysiology as it applies to management of clinical problems
• Apply knowledge gained to the ABIM certification/recertification examinations

ACGME Competencies:
This course is designed to meet one or more of the following Accreditation Council for
Graduate Medical Education competencies:
• Patient Care and Procedural Skills
• Medical Knowledge
• Practice-Based Learning and Improvement

TARGET AUDIENCE
The target audience for the Intensive Review of Internal Medicine course is clinical and
academic internists, pediatricians, and primary care physicians/trainees preparing for ABIM
internal medicine certification/recertification examinations and/or seeking a comprehensive
update in internal medicine and its subspecialties.

291
METHOD OF PARTICIPATION:
Review audio/video program of conference sessions, complete the comprehensive activity
evaluation and score 70% or greater on the required post-test to assess the knowledge gained
from reviewing the program.

• Go to www.myoakstone.com and log in using your user ID and password


• Click on your program to take your exam

ACCREDITATION
The Harvard Medical School is accredited by the Accreditation Council for Continuing Medical
Education (ACCME) to provide continuing medical education for physicians.

AMA CREDIT DESIGNATION STATEMENT


The Harvard Medical School designates this enduring material for a maximum of 85 AMA PRA
Category 1 Credits™. Physicians should claim only the credit commensurate with the extent of
their participation in the activity.

RISK MANAGEMENT CREDIT


This activity meets the criteria of the Massachusetts Board of Registration in Medicine for 2.75
credits of Risk Management Study. This includes:

.75 Credits of Opioid Education and Pain Management Training


.75 Credits of End-of-Life Care Studies

Please check your individual state licensing board requirements before claiming these credits.

ABIM MOC MEDICAL KNOWLEDGE MOC POINTS


Successful completion of this CME activity enables the participant to earn up to 85 MOC points
in the American Board of Internal Medicine’s (ABIM) Maintenance of Certification (MOC)
program. Participants will earn MOC points equivalent to the amount of CME credits claimed
for the activity. It is the CME activity provider’s responsibility to submit participant completion
information to ACCME for the purpose of granting ABIM MOC points.

On the course evaluation page please indicate “Yes” when asked if you would like to receive
ABIM MOC points for your participation in this enduring activity. You will then be required to
provide your ABIM ID # and your Date of Birth. Points earned will equal the amount of AMA
PRA Category 1 CreditsTM claimed.

HMS will upload the participant data, including the points earned, directly to the ABIM so that it
will appear on the ABIM diplomates transcript. These points will not appear on your certificate
provided at the end of this enduring activity.

291
DISCLOSURE POLICY
Harvard Medical School (HMS) adheres to all ACCME Essential Areas, Standards, and Policies.
It is HMS’s policy that those who have influenced the content of a CME activity (e.g. planners,
faculty, authors, reviewers and others) disclose all relevant financial relationships with
commercial entities so that HMS may identify and resolve any conflicts of interest prior to the
activity. These disclosures will be provided in the activity materials along with disclosure of any
commercial support received for the activity. Additionally, faculty members have been instructed
to disclose any limitations of data and unlabeled or investigational uses of products during their
presentations.

Disclosure information for all individuals in control of the content of the activity is located
on the disclosure statement in the PDF and printed syllabus.

HARDWARE/SOFTWARE REQUIREMENTS:

ONLINE STREAMING:
Windows 7, 8, or 10 on PC and Mac OS X or above on Mac computers with the most current
version web browser (Internet Explorer or Edge, Firefox, Safari, and Chrome) for each.

MP4 video / MP3 Audio USB: Any computer workstation that has a standard USB-A port and
MP4 software player such as VLC (http://www.videolan.org/).

MP3 Data CDs: Computer CD-ROM drive with compatible software MP4 player such as VLC
(http://www.videolan.org/).

If you experience technical difficulties, you can contact our Customer Service hotline at
1-800-284-8433.

Disclaimer:
CME activities sponsored by Harvard Medical School are offered solely for educational
purposes and do not constitute any form of certification of competency. Practitioners
should always consult additional sources of information and exercise their best professional
judgment before making clinical decisions of any kind.

WARNING:
The copyright proprietor has licensed the picture contained on this recording for private
home use only and prohibits any other use, copying, reproduction, or performance in
public, in whole or in part (Title 17 USC Section 501 506).

CMEinfo is not responsible in any way for the accuracy, medical or legal content of this
recording. You should be aware that substantive developments in the medical field covered
by this recording may have occurred since the date of original release.

© 2018 Ebix, Inc. DBA Oakstone Publishing. LLC.


CMEinfo is a registered trademark of Oakstone Publishing, LLC.

291
ACTIVITY DISCLOSURE STATEMENT
41st Annual Intensive Review of Internal Medicine
Course Number: 732046-1901
Date of Original Release: October 1, 2018 / Termination Date: January 31, 2021

The Harvard Medical School is accredited by the Accreditation Council for Continuing Medical Education (ACCME) to provide
continuing medical education for physicians.

The Harvard Medical School designates this enduring material for a maximum of 85 AMA PRA Category 1 Credits™. Physicians
should claim only the credit commensurate with the extent of their participation in the activity.

This activity meets the criteria of the Massachusetts Board of Registration in Medicine for 2.75 credits of Risk Management Study.
This includes .75 Credits of Opioid Education and Pain Management Training and .75 Credits of End-of-Life Care Studies. Please
check your individual state licensing board requirements before claiming these credits.

Harvard Medical School has long held the standard that its continuing medical education courses be free of commercial bias.

In accord with the disclosure policy of the Medical School as well as standards set forth by the Accreditation Council for Continuing
Medical Education, course planners, speakers, and content reviewers have been asked to disclose any relevant relationship they, or
their spouse or partner, have to companies producing, marketing, re-selling or distributing health care goods or services consumed by,
or used on, patients. In addition, faculty have been asked to list any off-label uses of pharmaceuticals and/or devices for
investigational or non-FDA approved purposes that they plan to discuss.

Such disclosure is not intended to suggest or condone bias in any presentation, but is elicited to provide the course director and
participants with information that might be of potential importance to their evaluation of a given presentation.

The following planners, speakers, and content reviewers, on behalf of themselves and their spouse or partner, have reported financial
relationships with an entity producing, marketing, re-selling, or distributing health care goods or services (relevant to the content of
the activity) consumed by, or used on, patients:

NAME COMPANY RELATIONSHIP


Course Planners and Faculty:
Ajay K. Singh, MBBS, FRCP (UK) GlaxoSmithKline Consultant
MBA

Course Faculty:
Maureen Achebe, MD, MPH AMAG Pharmaceuticals Advisory Board
Luitpold Pharmaceuticals Consultant

Elizabeth M. Battinelli, MD PhD Sanofi Consultant

Carolyn Bernstein, MD Palmer Chiropractic Grant Support


Amgen Consultant

Marc P. Bonaca, MD, MPH Amgen Grant Support, Consultant


AstraZeneca Grant Support, Consultant
Merck Grant Support, Consultant
MedImmune Grant Support
Pfizer Grant Support
Aralez Consultant
Bayer Consultant
Janssen Consultant
Sanofi Consultant

Barry Brenner, MD Janssen Consultant


Jean M. Connors, MD Bristol-Myers Squibb Consultant/Scientific Advisory Board
Boehringer Ingelheim Scientific Advisory Board
Dova Pharmaceuticals Consultant
Pfizer Consultant
Portola Scientific Advisory Board
Unum Therapeutics Data Safety Monitoring Board

Paul Dellaripa, MD Genentech Principal Investigator


Bristol-Myers Squibb Principal Investigator

Akshay Desai, MD, MPH Abbott/St. Jude Medical Consultant


Amgen Consultant
AstraZeneca Consultant
Boehringer Ingelheim Consultant
DalCor Pharmaceuticals Consultant
Janssen Consultant
Novartis Consultant, Research Grants
Relypsa Consultant

Joerg Ermann, MD Boehringer Ingelheim Research Grant, Consulting


Pfizer Research Grant
Eli Lilly Scientific Advisory Board
Novartis Scientific Advisory Board

Hilary J. Goldberg, MD Genentech Member of Clinical Coordinating Center for


Industry-Sponsored Study

Samuel Z. Goldhaber, MD Boehringer Ingelheim Research Support, Consultant


Bristol-Myers Squibb Research Support, Consultant
BTG EKOS Research Support
Daiichi Research Support, Consultant
Janssen Research Support, Consultant
Thrombosis Research Institute Research Support
Agile Consultant
Bayer Consultant
Portola Consultant
Soleno Consultant

Sarah Hammond, MD Merck Research Support

Lauren Harshman, MD Bayer Advisor/Research to the Institution/Travel


Exelixis Advisor
Genentech Advisor/Research to the Institution
Dendreon/Valient Advisor/Research to the Institution
Pfizer Advisor/Research to the Institution
Medivation/Astellas Advisor/Research to the Institution
Kew Group Advisor
Theragene Advisor
Corvus Advisor
Merck Advisor/Research to the Institution
Bristol-Myers Squibb Research to the Institution
Janssen Research to the Institution
Sotio Research to the Institution
Takeda Research to the Institution
Sanofi Travel
Novartis Advisor
Craig Hersh, MD Boehringer Ingelheim Grant
Novartis Grant
23andMe Consultant

Elliot Israel, MD AstraZeneca Consultant


Entrinsic Health Solutions Consultant
GlaxoSmithKline Consultant
Merck Consultant
Novartis Consultant
4D Pharma Consultant
Pneuma Respiratory Consultant
Regeneron Consultant
Sanofi Consultant
Vorso Corp Consultant
Genentech Clinical Research Trial (BWH)
Novartis Clinical Research Trial (BWH)
Sanofi Clinical Research Trial (BWH)

David Jackman, MD AstraZeneca Consultant


CVS Caremark Consultant
More Health Consultant

Ursula Kaiser, MD Novo Nordisk Consultant

Scott Kinlay, MBBS, PhD Colorado Prevention Center Data Safety Monitoring Board

Ann LaCasce, MD Bristol-Myers Squibb Consulting


Seattle Genetics Consulting

Ming V. Lin, MD Gilead Advisory Board

James Maguire, MD Bayer Advisory Committee

Jeffrey Meyerhardt, MD Chugai Pharmaceuticals Consultant


Ignyta Consultant

Samia Mora, MD Atherotech Diagnostics Research Grant to Institution

Anju Nohria, MD Amgen Research Support


Takeda Oncology Consultant

Paul M. Ridker, MD AstraZeneca Research Support, Co-Inventor on Licensed


Patents
Novartis Research Support, Research Consultant
Pfizer Research Support
Kowa Research Support
Amgen Research Support
Siemens Co-Inventor on Licensed Patents
Quintiles Research Consultant
Corvida Research Consultant
Inflazome Research Consultant
Eisai Research Consultant
Sanofi Research Consultant
Janssen Research Consultant

Susan Ritter, MD Pfizer Spouse – Employee


Marc S. Sabatine, MD Amgen Research Grant Support, Consulting
AstraZeneca Research Grant Support, Consulting
Daiichi-Sankyo Research Grant Support
Eisai Research Grant Support
GlaxoSmithKline Research Grant Support
Intarcia Research Grant Support, Consulting
Janssen Research and Dev. Research Grant Support, Consulting
Medicines Company Research Grant Support, Consulting
MedImmune Research Grant Support, Consulting
Merck Research Grant Support, Consulting
Novartis Research Grant Support, Consulting
Pfizer Research Grant Support
Poxel Research Grant Support
Takeda Research Grant Support
Bristol-Myers Squibb Consulting
CVS Caremark Consulting
Dyrnamix Consulting
Esperion Consulting

Robert C. Stanton, MD Janssen Pharmaceuticals Consultant

Usha Tedrow, MD Abbott Medical Honorarium/Faculty, Fellows’ Course


Biosense Webster Honorarium/Faculty, Fellows’ Course
Medtronic Honorarium/Faculty, Fellows’ Course

Derrick Todd, MD Optum Consultant

Alexander Turchin, MD Merck Consultant


Monarch Medical Technologies Consultant
Eli Lilly Research Support
Brio Systems Consultant

All other individuals including course directors, planners, reviewers, faculty, staff, etc., who are in a position to control the content of
this educational activity have, on behalf of themselves and their spouse or partner, reported no financial relationships related to the
content of this activity.
41st Annual Intensive Review of Internal Medicine
Faculty List
____________________________________________________
COURSE DIRECTORS:

Ajay K. Singh, MBBS, FRCP (UK), MBA Carolyn B. Becker, MD


Senior Associate Dean for Postgraduate Associate Professor of Medicine,
Medical Education, Harvard Medical School,
Director, Master in Medical Sciences in Division of Endocrinology,
Clinical Investigation (MMSCI) Program Diabetes and Hypertension,
Harvard Medical School, Department of Medicine,
Director, Office of Postgraduate Brigham and Women’s Hospital
Medical Education, Joseph Loscalzo, MD, PhD
Brigham and Women’s Hospital Hersey Professor of the Theory and
Nancy Berliner, MD Practice of Medicine,
Professor of Medicine, Harvard Medical School,
Harvard Medical School, Division of Cardiovascular Medicine,
Chief, Division of Hematology, Chairman, Department of Medicine,
Department of Medicine, Brigham and Women's Hospital
Brigham and Women's Hospital

FACULTY:

Maureen M. Achebe, MD, MBBS Lindsey R. Baden, MD


Assistant Professor of Medicine, Associate Professor of Medicine,
Harvard Medical School, Harvard Medical School,
Division of Hematology, Division of Infectious Diseases,
Department of Medicine, Department of Medicine,
Brigham and Women's Hospital Brigham and Women’s Hospital

Dale S. Adler, MD Rebecca M. Baron, MD


Associate Professor of Medicine, Assistant Professor of Medicine,
Harvard Medical School, Harvard Medical School,
Division of Cardiovascular Medicine, Division of Pulmonary and Critical Care
Department of Medicine, Medicine, Department of Medicine,
Brigham and Women’s Hospital Brigham and Women’s Hospital

Edwin P. Alyea, III, MD Elisabeth M. Battinelli, MD


Associate Professor of Medicine, Assistant Professor of Medicine,
Harvard Medical School, Harvard Medical School,
Medical Oncology, Division of Hematology,
Dana-Farber Cancer Institute, Department of Medicine,
Department of Medicine, Brigham and Women's Hospital
Brigham and Women’s Hospital
____________________________________________________
David D. Berg, MD Wendy Y. Chen, MD, MPH
Clinical Fellow in Medicine, Assistant Professor of Medicine,
Harvard Medical School, Harvard Medical School,
Chief Medical Resident, Medical Oncology,
Department of Medicine, Dana-Farber Cancer Institute,
Brigham and Women’s Hospital Department of Medicine,
Brigham and Women’s Hospital
Carolyn A. Bernstein, MD
Assistant Professor of Neurology, Cheryl R. Clark, MD
Harvard Medical School, Assistant Professor of Medicine,
Department of Neurology, Harvard Medical School,
Brigham and Women's Hospital Director,
Health Equity Research & Intervention
Marc P. Bonaca, MD Center for Community Health and
Assistant Professor of Medicine, Health Equity,
Harvard Medical School, Division of General Internal Medicine
Division of Cardiovascular Medicine, and Primary Care,
Department of Medicine, Department of Medicine,
Brigham and Women’s Hospital Brigham and Women’s Hospital

Kari P. Braaten, MD, MPH Nathan T. Connell, MD, MPH


Instructor in Obstetrics, Assistant Professor of Medicine,
Gynecology and Reproductive Biology, Harvard Medical School,
Harvard Medical School, Division of Hematology,
Department of Obstetrics and Gynecology, Department of Medicine,
Brigham and Women's Hospital Brigham and Women's Hospital

Barry M. Brenner, MD Jean M. Connors, MD


Samuel A. Levine Distinguished Assistant Professor of Medicine,
Professor of Medicine, Harvard Medical School,
Harvard Medical School, Division of Hematology,
Renal Division, Department of Medicine, Department of Medicine,
Brigham and Women's Hospital Brigham and Women’s Hospital

Julie E. Buring, ScD Paul F. Dellaripa, MD


Professor of Medicine, Associate Professor of Medicine,
Harvard Medical School, Harvard Medical School,
Department of Medicine, Division of Rheumatology,
Brigham and Women's Hospital Immunology and Allergy,
Department of Medicine,
Walter W. Chan, MD, MPH Brigham and Women’s Hospital
Assistant Professor of Medicine,
Harvard Medical School, Bradley M. Denker, MD
Division of Gastroenterology, Associate Professor of Medicine,
Hepatology and Endoscopy, Harvard Medical School,
Department of Medicine, Renal Division,
Brigham and Women's Hospital Department of Medicine,
Beth Israel Deaconess Medical Center
____________________________________________________
Akshay S. Desai, MD, MPH Christopher H. Fanta, MD
Associate Professor of Medicine, Professor of Medicine,
Harvard Medical School, Harvard Medical School,
Division of Cardiovascular Medicine, Division of Pulmonary and
Department of Medicine, Critical Care Medicine,
Brigham and Women’s Hospital Department of Medicine,
Brigham and Women’s Hospital
Sanjay Divakaran, MD
Clinical Fellow in Medicine, Sonia Friedman, MD
Harvard Medical School, Associate Professor of Medicine,
Division of Cardiovascular Medicine, Harvard Medical School,
Department of Medicine, Division of Gastroenterology,
Brigham and Women’s Hospital Hepatology and Endoscopy,
Department of Medicine,
Todd B. Ellerin, MD Brigham and Women’s Hospital
Instructor in Medicine,
Harvard Medical School, Elizabeth B. Gay, MD
Division of Infectious Disease, Assistant Professor of Medicine,
Department of Medicine, Harvard Medical School,
South Shore Hospital Division of Pulmonary and
Critical Care Medicine,
Lawrence J. Epstein, MD Department of Medicine,
Instructor in Medicine, Part-Time, Brigham and Women’s Hospital
Harvard Medical School,
Division of Sleep Medicine, Hilary J. Goldberg, MD
Department of Medicine, Assistant Professor of Medicine,
Brigham and Women’s Hospital Harvard Medical School,
Division of Pulmonary and
Joerg Ermann, MD Critical Care Medicine,
Instructor in Medicine, Department of Medicine,
Harvard Medical School, Brigham and Women’s Hospital
Division of Rheumatology,
Immunology and Allergy, Samuel Z. Goldhaber, MD
Department of Medicine, Professor of Medicine,
Brigham and Women’s Hospital Harvard Medical School,
Division of Cardiovascular Medicine,
Brendan M. Everett, MD, MPH Department of Medicine,
Assistant Professor of Medicine, Brigham and Women’s Hospital
Harvard Medical School,
Division of Cardiovascular Medicine, Annekathryn Goodman, MD
Department of Medicine, Associate Professor of Obstetrics,
Brigham and Women’s Hospital Gynecology, and Reproductive Biology,
Harvard Medical School,
Department of Obstetrics and Gynecology,
Massachusetts General Hospital
____________________________________________________
Kathleen J. Haley, MD Elliot Israel, MD
Assistant Professor of Medicine, Professor of Medicine,
Harvard Medical School, Harvard Medical School,
Division of Pulmonary and Critical Care Division of Pulmonary and
Medicine Department of Medicine, Critical Care Medicine,
Brigham and Women’s Hospital Department of Medicine,
Brigham and Women’s Hospital
Florencia Halperin, MD
Instructor in Medicine, David M. Jackman, MD
Harvard Medical School, Assistant Professor of Medicine,
Division of Endocrinology, Diabetes Harvard Medical School,
and Hypertension, Medical Oncology,
Department of Medicine, Dana-Farber Cancer Institute,
Brigham and Women’s Hospital Department of Medicine,
Brigham and Women’s Hospital
Sarah P. Hammond, MD
Assistant Professor of Medicine, Kunal Jajoo, MD
Harvard Medical School, Assistant Professor of Medicine,
Division of Infectious Diseases, Harvard Medical School,
Department of Medicine, Division of Gastroenterology,
Brigham and Women’s Hospital Hepatology and Endoscopy,
Department of Medicine,
Lauren C. Harshman, MD Brigham and Women’s Hospital
Assistant Professor of Medicine,
Harvard Medical School, Jennifer A. Johnson, MD
Medical Oncology, Assistant Professor of Medicine,
Dana-Farber Cancer Institute, Harvard Medical School,
Department of Medicine, Division of Infectious Diseases,
Brigham and Women’s Hospital Department of Medicine,
Brigham and Women’s Hospital
Galen V. Henderson, MD
Assistant Professor of Neurology, Ursula B. Kaiser, MD
Harvard Medical School, Professor of Medicine,
Department of Neurology, Harvard Medical School,
Brigham and Women’s Hospital Division of Endocrinology,
Diabetes and Hypertension,
Craig P. Hersh, MD Department of Medicine,
Assistant Professor of Medicine, Brigham and Women’s Hospital
Harvard Medical School,
Division of Pulmonary and Sunil Kapur, MD
Critical Care Medicine, Instructor in Medicine,
Department of Medicine, Harvard Medical School,
Brigham and Women's Hospital Division of Cardiovascular Medicine,
Department of Medicine,
Brigham and Women's Hospital
____________________________________________________
Matthew I. Kim, MD Ming Valerie Lin, MD
Assistant Professor of Medicine, Instructor in Medicine,
Harvard Medical School, Harvard Medical School,
Division of Endocrinology, Division of Transplantation,
Diabetes and Hypertension, Department of Surgery
Department of Medicine, Lahey Hospital and Medical Center
Brigham and Women’s Hospital
James H. Maguire, MD
Scott Kinlay, MBBS, PhD Professor of Medicine,
Associate Professor of Medicine, Harvard Medical School,
Harvard Medical School, Division of Infectious Diseases,
Division of Cardiovascular Medicine, Department of Medicine,
Department of Medicine, Brigham and Women’s Hospital
Brigham and Women’s Hospital
Kathryn A. Martin, MD
Michael Klompas, MD Assistant Professor of Medicine,
Associate Professor of Population Medicine, Part-time, Harvard Medical School, Division
Harvard Medical School, of Women’s Health,
Division of Infectious Diseases, Department of Medicine,
Department of Medicine, Massachusetts General Hospital
Brigham and Women’s Hospital
Julia Y. McNabb-Baltar, MD
Ann S. LaCasce, MD Instructor in Medicine,
Associate Professor of Medicine, Harvard Medical School,
Harvard Medical School, Division of Gastroenterology,
Medical Oncology, Hepatology and Endoscopy,
Dana-Farber Cancer Institute, Department of Medicine,
Department of Medicine, Brigham and Women’s Hospital
Brigham and Women’s Hospital
Jeffrey A. Meyerhardt, MD
Lisa S. Lehmann, MD, PhD Associate Professor of Medicine,
Associate Professor of Medicine, Harvard Medical School,
Part-time, Harvard Medical School, Division Medical Oncology
of General Internal Medicine & Primary Dana-Farber Cancer Institute,
Care, Department of Medicine, Department of Medicine,
Brigham and Women’s Hospital Brigham and Women’s Hospital

Leonard S. Lilly, MD Tracey A. Milligan, MD


Professor of Medicine, Assistant Professor of Neurology,
Harvard Medical School, Harvard Medical School,
Division of Cardiovascular Medicine, Department of Neurology,
Department of Medicine, Brigham and Women’s Hospital
Brigham and Women’s Hospital
____________________________________________________
Elinor A. Mody, MD Kathryn M. Rexrode, MD
Assistant Professor of Medicine, Associate Professor of Medicine,
Harvard Medical School, Harvard Medical School,
Chief, Division of Rheumatology, Division of Women’s Health,
Reliant Medical Group Department of Medicine,
Brigham and Women's Hospital
Samia Mora, MD
Associate Professor of Medicine, Paul M. Ridker, MD
Harvard Medical School, Eugene Braunwald Professor of Medicine,
Division of Cardiovascular Medicine, Harvard Medical School,
Department of Medicine, Division of Cardiovascular
Brigham and Women’s Hospital Medicine, Department of Medicine,
Brigham and Women’s Hospital
Muthoka L. Mutinga, MD
Assistant Professor of Medicine, Susan Y. Ritter, MD
Harvard Medical School, Instructor in Medicine,
Division of Gastroenterology, Harvard Medical School,
Hepatology and Endoscopy Division of Rheumatology,
Department of Medicine, Immunology and Allergy,
Brigham and Women’s Hospital Department of Medicine,
Brigham and Women's Hospital
Anju Nohria, MD
Assistant Professor of Medicine, Christopher L. Roy, MD
Harvard Medical School, Assistant Professor of Medicine,
Division of Cardiovascular Medicine, Harvard Medical School,
Department of Medicine, Hospitalist Services,
Brigham and Women’s Hospital Department of Medicine,
Brigham and Women’s Hospital
M. Angela O'Neal, MD
Assistant Professor of Neurology, Anna E. Rutherford, MD, MPH
Harvard Medical School, Assistant Professor of Medicine,
Department of Neurology, Harvard Medical School,
Brigham and Women's Hospital Division of Gastroenterology,
Hepatology and Endoscopy,
Ann L. Pinto, MD, PhD Department of Medicine,
Instructor in Medicine, Brigham and Women's Hospital
Harvard Medical School,
Division of General Internal Medicine & Marc S. Sabatine, MD, MPH
Primary Care, Professor of Medicine,
Department of Medicine, Harvard Medical School,
Brigham and Women’s Hospital Division of Cardiovascular Medicine,
Department of Medicine,
Brigham and Women’s Hospital
____________________________________________________
Suzanne E. Salamon, MD David E. Sloane, MD
Assistant Professor of Medicine, Instructor in Medicine,
Harvard Medical School, Harvard Medical School,
Associate Chief for Clinical Geriatrics, Division of Rheumatology,
Beth Israel Deaconess Medical Center Immunology and Allergy,
Department of Medicine,
John R. Saltzman, MD Brigham and Women’s Hospital
Professor of Medicine,
Harvard Medical School, Benjamin N. Smith, MD
Division of Gastroenterology, Assistant Professor of Medicine,
Hepatology and Endoscopy, Harvard Medical School,
Department of Medicine, Division of Gastroenterology,
Brigham and Women’s Hospital Hepatology and Endoscopy,
Department of Medicine,
Adam Schaffer, MD Brigham and Women’s Hospital
Instructor in Medicine,
Harvard Medical School, Caren G. Solomon, MD
Department of Medicine, Associate Professor of Medicine,
Brigham and Women's Hospital Harvard Medical School,
Division of Women’s Health,
Gordon Schiff, MD Department of Medicine,
Assistant Professor of Medicine, Brigham and Women's Hospital
Harvard Medical School,
Division of General Internal Medicine & Robert C. Stanton, MD
Primary Care, Department of Medicine, Associate Professor of Medicine,
Brigham and Women’s Hospital Harvard Medical School,
Chief, Kidney & Hypertension Section,
Scott L. Schissel, MD, PhD, DPhil Joslin Diabetes Center;
Assistant Professor of Medicine, Division of Nephrology,
Harvard Medical School, Department of Medicine,
Division of Pulmonary and Beth Israel Deaconess Medical Center
Critical Care Medicine,
Chief of the Department of Medicine, Garrick C. Stewart, MD
Brigham and Women’s Hospital Faulkner Instructor in Medicine,
Harvard Medical School,
Ellen W. Seely, MD Division of Cardiovascular Medicine,
Professor of Medicine, Department of Medicine,
Harvard Medical School, Brigham and Women’s Hospital
Division of Endocrinology,
Diabetes and Hypertension, David M. Systrom, MD
Department of Medicine, Assistant Professor of Medicine,
Brigham and Women’s Hospital Harvard Medical School,
Division of Pulmonary and
Critical Care Medicine,
Department of Medicine,
Brigham and Women’s Hospital
____________________________________________________
Laura L. Tarter, MD Alexander Turchin, MD
Assistant Professor of Medicine, Associate Professor of Medicine,
Harvard Medical School Harvard Medical School,
Division of Rheumatology, Division of Endocrinology, Diabetes and
Immunology and Allergy, Hypertension,
Department of Medicine, Department of Medicine,
Brigham and Women’s Hospital Brigham and Women’s Hospital

Usha B. Tedrow, MD, MS Anand Vaidya, MD, MMSc


Associate Professor of Medicine, Assistant Professor of Medicine,
Harvard Medical School, Harvard Medical School,
Division of Cardiovascular Medicine, Division of Endocrinology,
Department of Medicine, Diabetes and Hypertension,
Brigham and Women’s Hospital Department of Medicine,
Brigham and Women’s Hospital
Manisha Thakuria, MD
Instructor in Dermatology, Anne M. Valente, MD
Harvard Medical School, Associate Professor of Pediatrics,
Department of Dermatology, Harvard Medical School,
Brigham and Women’s Hospital Division of Cardiovascular Medicine,
Department of Medicine,
Lori W. Tishler, MD Boston Children’s Hospital
Assistant Professor of Medicine, Part-Time
Harvard Medical School, Russell G. Vasile, MD
Vice President, Medical Affairs Associate Professor of Psychiatry,
Commonwealth Care Alliance Harvard Medical School,
Department of Psychiatry,
Derrick J. Todd, MD, PhD Beth Israel Medical Deaconess Hospital
Instructor in Medicine,
Harvard Medical School, Gustavo E. Velasquez, MD, MPH
Division of Rheumatology, Instructor in Medicine,
Immunology and Allergy, Harvard Medical School,
Department of Medicine, Division of Infectious Disease
Brigham and Women’s Hospital Department of Medicine,
Brigham and Women's Hospital
J. Kevin Tucker, MD
Assistant Professor of Medicine, Sushrut S. Waikar, MD
Harvard Medical School, Associate Professor of Medicine,
Renal Division, Harvard Medical School,
Department of Medicine, Renal Division,
Brigham and Women’s Hospital Department of Medicine, Brigham and
Women’s Hospital
____________________________________________________
Sarah E. Wakeman, MD, FASM
Assistant Professor of Medicine,
Harvard Medical School,
Medical Director, Mass General Hospital
Substance Use Disorder Initiative,
Division of General Internal Medicine &
Primary Care,
Department of Medicine,
Massachusetts General Hospital

Gerald L. Weinhouse, MD
Assistant Professor of Medicine,
Harvard Medical School,
Division of Pulmonary and Critical Care
Medicine
Department of Medicine, Brigham and
Women’s Hospital

Maria A. Yialamas, MD
Assistant Professor of Medicine,
Harvard Medical School,
Division of Endocrinology,
Diabetes and Hypertension,
Department of Medicine,
Brigham and Women’s Hospital
5HYLVLWLQJ(OHFWURO\WHVDQG$FLG
&.' %DVH%DVLFV
The Story of
my Seventy-Eight Year %UDGOH\0'HQNHU0'
Old Patient &OLQLFDO&KLHI
5HQDO'LYLVLRQ'HSDUWPHQWRI0HGLFLQH
$MD\.6LQJK0%)5&3 %HWK,VUDHO'HDFRQHVV0HGLFDO&HQWHUDQG
5HQDO'LYLVLRQ +DUYDUG9DQJXDUG0HGLFDO$VVRFLDWHV
%ULJKDPDQG:RPHQ¶V+RVSLWDO $VVRFLDWH3URIHVVRURI0HGLFLQH
6HQLRU$VVRFLDWH'HDQ +DUYDUG0HGLFDO6FKRRO
3RVWJUDGXDWH0HGLFDO
(GXFDWLRQ
+DUYDUG0HGLFDO6FKRRO

5HIHUHQFHV
6XJJHVWHGUHDGLQJ
ƒ :DQQHU&,Q]XFFKL6(/DFKLQ-0HWDO
(PSDJOLIOR]LQDQGSURJUHVVLRQRINLGQH\
y 5HQQNH+*'HQNHU%05HQDO3DWKRSK\VLRORJ\± 7KH
GLVHDVHLQW\SHGLDEHWHV1(QJO-0HG'2, (VVHQWLDOVWK(GLWLRQ/LSSLQFRWW:LOOLDPV :LONLQV
1(-0RD
ƒ /LQGHPDQ5'-'7RELQDQG1:6KRFN y 0RXQW'% )OXLGDQG(OHFWURO\WH'LVWUXEDQFHV,Q+DUULVRQ
V
/RQJLWXGLQDOVWXGLHVRQWKHUDWHRIGHFOLQHLQ 3ULQFLSOHVRI,QWHUQDO0HGLFLQHWK (GLWLRQ(GV/RQJR)DXFLet
UHQDOIXQFWLRQZLWKDJH-$P*HULDWU6RF al.0F*UDZ+LOOS
±
ƒ /HYH\HWDOAnn Intern Med y 'X%RVH7'-U$FLGRVLVDQG$ONDORVLV,Q+DUULVRQ
V3ULQFLSOHVRI
 ,QWHUQDO0HGLFLQHWK (GLWLRQ(GV/RQJR)DXFLet al.0F*UDZ
+LOOS
ƒ &ROOLVWHU'
KWWSZZZVFLHQFHGLUHFWFRPVFLHQFHDUWLFOHSLL
6;

$SSURDFKWR3URWHLQXULD ELECTROLYTEANDACIDBASE:
DQG+HPDWXULD ChallengingQuestionsandAnswers
%UDGOH\0'HQNHU0'
ϱůŝŶŝĐĂů^ĐĞŶĂƌŝŽƐŽŵŵŽŶƚŽƚŚĞŽĂƌĚƐ &OLQLFDO&KLHI
5HQDO'LYLVLRQ'HSDUWPHQWRI0HGLFLQH
%HWK,VUDHO'HDFRQHVV0HGLFDO&HQWHUDQG
$MD\.6LQJK0%)5&3 +DUYDUG9DQJXDUG0HGLFDO$VVRFLDWHV
3K\VLFLDQ5HQDO'LYLVLRQ
$VVRFLDWH3URIHVVRURI0HGLFLQH
%ULJKDPDQG:RPHQ¶V+RVSLWDO
6HQLRU$VVRFLDWH'HDQ +DUYDUG0HGLFDO6FKRRO
IRU3RVWJUDGXDWH0HGLFDO(GXFDWLRQ
+DUYDUG0HGLFDO6FKRRO

ZĞĨĞƌĞŶĐĞƐ 6XJJHVWHGUHDGLQJ
‡ ^ŽƵƌĐĞ͗D,DKh>K',DEͲ,DD͕͘ y 5HQQNH+*'HQNHU%05HQDO3DWKRSK\VLRORJ\± 7KH
ŵ&ĂŵWŚLJƐŝĐŝĂŶ͘ ϭϵϵϴ KĐƚ ϭ͖ϱϴ;ϱͿ͗ϭϭϰϱͲ (VVHQWLDOVWK(GLWLRQ/LSSLQFRWW:LOOLDPV :LONLQV

ϭϭϱϮ͘s,^<Z/sD͕ƌĐŚŝǀĞƐŽĨŝƐĞĂƐĞŝŶ
ŚŝůĚŚŽŽĚ͕ϭϵϴϮ͕ϱϳ͕ϳϮϵͲϳϯϬ y 0RXQW'% )OXLGDQG(OHFWURO\WH'LVWUXEDQFHV,Q+DUULVRQ
V
3ULQFLSOHVRI,QWHUQDO0HGLFLQHWK (GLWLRQ(GV/RQJR)DXFLet
al.0F*UDZ+LOOS

y 'X%RVH7'-U$FLGRVLVDQG$ONDORVLV,Q+DUULVRQ
V3ULQFLSOHVRI
,QWHUQDO0HGLFLQHWK (GLWLRQ(GV/RQJR)DXFLet al.0F*UDZ
+LOOS
dĂŬĞ,ŽŵĞDĞƐƐĂŐĞƐ
,5,0

$MD\.6LQJK0%)5&3
'LDO\VLVDQG7UDQVSODQWDWLRQ 3K\VLFLDQ5HQDO'LYLVLRQ
-.HYLQ7XFNHU0' %ULJKDPDQG:RPHQ¶V+RVSLWDO
&KLHIRI1HSKURORJ\ 6HQLRU$VVRFLDWH'HDQIRU3RVWJUDGXDWH
%ULJKDPDQG:RPHQ¶V)DXONQHU 0HGLFDO(GXFDWLRQ
+RVSLWDO +DUYDUG0HGLFDO6FKRRO

5HIHUHQFHV
5HIHUHQFHV
‡ EŶŐů:DĞĚϮϬϬϬ͖ϯϰϮ͗ϭϱϴϭͲϭϱϴϵ
‡ ůƵŵďĞƌŐĞƚĂů</͕ϰϭ͗ϯϲϵͲϯϳϰ͕ϭϵϵϮ
‡ %HUQDUGLQLet al -$P6RF1HSKURO ‡ EŶŐů:DĞĚϮϬϭϱ͖ϯϳϮ͗ϮϮϮͲϮϯϭ
 ‡ :^Eϭϰ͗ϭϬϳ͕ϮϬϬϯ
‡ .RQQHU.et al -$P6RF1HSKURO ‡ EŶŐů:DĞĚϮϬϭϰ͖ϯϳϭ͗ϮϮϲϳͲϮϮϳϲ
 ‡ EŶŐů:DĞĚϮϬϭϰ͖ϯϳϭ͗ϮϮϱϱͲϮϮϲϲ
‡ ůŝŶ:ŵ^ŽĐEĞƉŚƌŽů͘ϮϬϬϵ:Ƶů͖ϰ;ϳͿ͗ϭϭϴϯʹϭϭϴϵ͘
‡ 0HKURWUDHWDO.LGQH\,QW
‡ DZ,ϮϬϭϭ͖ϲϱ;ϰͿ͗ϮϭϯͲϮϭϱ
 ‡ ŵ&ĂŵWŚLJƐŝĐŝĂŶ͘ ϮϬϭϭ ĞĐ ϭ͖ϴϰ;ϭϭͿ͗ϭϮϯϰͲϭϮϰϮ͘
‡ 6LONHQVHQ-$P6RF1HSKURO ‡ ŶŶ/ŶƚĞƌŶDĞĚ͘ϮϬϬϯ͖ϭϯϵ͗ϭϯϳͲϭϰϳ


WƐLJĐŚŝĂƚƌLJKǀĞƌǀŝĞǁ
ĐƵƚĞ<ŝĚŶĞLJ/ŶũƵƌLJ ED/
^ƵƐŚƌƵƚ^͘tĂŝŬĂƌ͕D͕DW,
ŽŶƐƚĂŶƚŝŶĞ>͘,ĂŵƉĞƌƐ͕DŝƐƚŝŶŐƵŝƐŚĞĚŚĂŝƌŝŶZĞŶĂůDĞĚŝĐŝŶĞ
ƌŝŐŚĂŵĂŶĚtŽŵĞŶ͛Ɛ,ŽƐƉŝƚĂů DĂƵƌĞĞŶD͘ĐŚĞďĞ͕D͕DW,
ƐƐŽĐŝĂƚĞWƌŽĨĞƐƐŽƌ
,ĂƌǀĂƌĚDĞĚŝĐĂů^ĐŚŽŽů
ŝǀŝƐŝŽŶŽĨ,ĞŵĂƚŽůŽŐLJ
ƌŝŐŚĂŵĂŶĚtŽŵĞŶ͛Ɛ,ŽƐƉŝƚĂů
ƐƐŝƐƚĂŶƚWƌŽĨĞƐƐŽƌ͕,ĂƌǀĂƌĚDĞĚŝĐĂů^ĐŚŽŽů

5HIHUHQFHV ZĞĨĞƌĞŶĐĞƐ
ϭ͘ ĂŝŶ͕:͘ŝĂŐŶŽƐŝƐĨƌŽŵƚŚĞůŽŽĚƐŵĞĂƌ͘EŶŐů :DĞĚ͘ϮϬϬϱ͖ϯϱϯ͗ϰϵϴͲϱϬϳ͘
‡ &KHUWRZ*0%XUGLFN(+RQRXU0%RQYHQWUH-9 %DWHV': Ϯ͘ tĞŝƐƐ͕'͕'ŽŽĚŶŽƵŐŚƚ͕>d͘ŶĞŵŝĂŽĨĐŚƌŽŶŝĐĚŝƐĞĂƐĞ͘EŶŐů :DĞĚ͘
$FXWHNLGQH\LQMXU\PRUWDOLW\OHQJWKRIVWD\DQGFRVWVLQ ϮϬϬϱ͖ϯϱϮ͗ϭϬϭϭͲϮϯ͘
KRVSLWDOL]HGSDWLHQWV-$P6RF1HSKURO ϯ͘ 'Ž͕^͕͘tŝŶƚĞƌƐ͕:͘,Žǁ/ƚƌĞĂƚĂƵƚŽŝŵŵƵŶĞŚĞŵŽůLJƚŝĐĂŶĞŵŝĂ͘ůŽŽĚϮϬϭϳ͖
ϭϮϵ͗ϮϵϳϭͲϮϵϳϵ͘
‡ :DOG54XLQQ55/XR-/L36FDOHV'&0DPGDQL00 5D\
ϰ͘ zŽƵŶŐ͕E͘ĐƋƵŝƌĞĚƉůĂƐƚŝĐ ŶĞŵŝĂ͘:Dϭϵϵϵ:ƵůLJϮϭ͖ϮϴϮ;ϯͿ͗ϮϳϭͲϮϳϴ͘
-*&KURQLFGLDO\VLVDQGGHDWKDPRQJVXUYLYRUVRIDFXWHNLGQH\
LQMXU\UHTXLULQJGLDO\VLV-DPD ϱ͘ dŚĞDĂŶĂŐĞŵĞŶƚŽĨ^ŝĐŬůĞĞůůŝƐĞĂƐĞ͘EĂƚŝŽŶĂů/ŶƐƚŝƚƵƚĞƐŽĨ,ĞĂůƚŚ͘
EĂƚŝŽŶĂů,ĞĂƌƚ͕>ƵŶŐĂŶĚůŽŽĚ/ŶƐƚŝƚƵƚĞ͕ŝǀŝƐŝŽŶŽĨůŽŽĚŝƐĞĂƐĞƐĂŶĚ
‡ %ODQW]5&3DWKRSK\VLRORJ\RISUHUHQDOD]RWHPLD.LGQH\,QW ZĞƐŽƵƌĐĞƐ͘E/,ƉƵďůŝĐĂƚŝŽŶEŽ͘ϬϮͲϮϭϭϳ͘&ŽƵƌƚŚĞĚŝƚŝŽŶ͘
 ϲ͘ ĂůůĂƐ͕^͘ĞLJŽŶĚƚŚĞĞĨŝŶŝƚŝŽŶŽĨƚŚĞWŚĞŶŽƚLJƉŝĐĐŽŵƉůŝĐĂƚŝŽŶƐŽĨ^ŝĐŬůĞ
‡ )ULHGULFK-2$GKLNDUL1+HUULGJH06 %H\HQH-0HWDDQDO\VLV ĞůůŝƐĞĂƐĞ͗ĂŶhƉĚĂƚĞŽĨDĂŶĂŐĞŵĞŶƚ͘dŚĞ^ĐŝĞŶƚŝĨŝĐtŽƌůĚ:ŽƵƌŶĂůϮϬϭϮ͖
ORZGRVHGRSDPLQHLQFUHDVHVXULQHRXWSXWEXWGRHVQRWSUHYHQW ƌƚŝĐůĞ/ϵϰϵϱϯϱ͘
UHQDOG\VIXQFWLRQRUGHDWK$QQ,QWHUQ0HG ϳ͘ ZƵŶĚ͕͘EͲdŚĂůĂƐƐĞŵŝĂ͘EŶŐů :DĞĚ͘ϮϬϬϱ͖ϯϱϯ͗ϭϭϯϱͲϰϲ͘
‡ 6WHLQHU5:,QWHUSUHWLQJWKHIUDFWLRQDOH[FUHWLRQRIVRGLXP$P-
0HG
5HIHUHQFHV %OHHGLQJ'LVRUGHUV
7KURPERSKLOLD7HVWLQJ 7KURPERSKLOLDWHVWLQJDQGYHQRXVWKURPERVLV
&RQQRUV-01(QJO -0HG6HS  

$SL[DEDQIRUH[WHQHG WUHDWPHQWRIYHQRXVWKURPERHPEROLVP
$JQHOOL *%XOOHU+5&RKHQ$&XUWR 0*DOOXV$6-RKQVRQ03RUFDUL $ (OLVDEHWK0%DWWLQHOOL0'3K'
5DVNRE *(:HLW] -,$03/,)<(;7,QYHVWLJDWRUV
-HDQ0&RQQRUV0'
1(QJO -0HG)HE   $VVLVWDQW3URIHVVRURI0HGLFLQH
0HGLFDO'LUHFWRU$QWLFRDJXODWLRQ0DQDJHPHQWDQG6WHZDUGVKLS6HUYLFHV
+HPDWRORJ\'LYLVLRQ
%ULJKDPDQG:RPHQ¶V+RVSLWDO'DQD)DUEHU&DQFHU,QVWLWXWH
5LYDUR[DEDQRU$VSLULQIRU([WHQGHG7UHDWPHQWRI9HQRXV
7KURPERHPEROLVP
+DUYDUG0HGLFDO6FKRRO
$VVRFLDWH3URIHVVRU+DUYDUG0HGLFDO6FKRRO :HLW] -,/HQVLQJ$:$3ULQV 0+%DXHUVDFKV 5%H\HU:HVWHQGRUI -
%RXQDPHDX[ +%ULJKWRQ7$&RKHQ$7'DYLGVRQ%/'HFRXVXV +)UHLWDV $VVRFLDWH3K\VLFLDQ
0&6+ROEHUJ *.DNNDU $.+DVNHOO/YDQ%HOOHQ %3DS$)%HUNRZLW]
6'9HUKDPPH 3:HOOV363UDQGRQL 3(,167(,1&+2,&(,QYHVWLJDWRUV
1(QJO -0HG0DU  
+HPDWRORJ\'LYLVLRQ
%ULJKDPDQG:RPHQ䇻V+RVSLWDO

5HIHUHQFHV
5HIHUHQFHV ƒ 0DQQXFFL307UHDWPHQWRIYRQ:LOOHEUDQG䇻V'LVHDVH 1(QJO-0HG$XJ  

,GDUXFL]XPDE IRU'DELJDWUDQ5HYHUVDOIXOOFRKRUWDQDO\VLV ƒ*LXVHSSH/'LHJR$5REHUWR4DQG*LDQIUDQFR&8UJHQWPRQLWRULQJRIGLUHFWRUDODQWLFRDJXODQWVLQ


3ROODFN&9-U5HLOO\3$YDQ5\Q -(LNHOERRP -:*OXQG 6%HUQVWHLQ SDWLHQWVZLWKDWULDOILEULOODWLRQDWHQWDWLYHDSSURDFKEDVHGRQURXWLQHODERUDWRU\WHVWV-RXUQDORI7KURPERVL
5$'XELHO 5+XLVPDQ09+\OHN (0.DP&:.DPSKXLVHQ 3: DQG7KURPERO\VLV0D\ HSXE 
.UHX]HU -/HY\-+5R\OH *6HOONH ):6WDQJLHU -6WHLQHU7
9HUKDPPH 3:DQJ%<RXQJ/:HLW] -, ƒ &RQROO\6HWDO'DELJDWUDQYHUVXV:DUIDULQLQ3DWLHQWVZLWK$WULDO)LEULOODWLRQ1(QJO
N Engl J Med   -0HG

$QGH[DQHW $OIDIRU$FXWH0DMRU%OHHGLQJ$VVRFLDWHGZLWK)DFWRU;D ƒ 'XJD66DORPRQ2&RQJHQLWDO)DFWRU;,GHILFLHQF\DQXSGDWH6HPLQ7KURPE+HPRVW


LQKLELWRUV &RQQROO\6-0LOOLQJ7--U(LNHOERRP -:*LEVRQ&0&XUQXWWH
-7*ROG$%URQVRQ0'/X*&RQOH\3%9HUKDPPH 36FKPLGW- 6HS  GRLV(SXE$XJ
0LGGHOGRUS 6&RKHQ$7%H\HU:HVWHQGRUI -$OEDODGHMR 3/RSH]6HQGRQ
-*RRGPDQ6/HHGV-:LHQV%/6LHJDO '0=RWRYD (0HHNV% ƒ *HUQVKHLPHU7-DPHV$+DQG6WDVL5+RZ,WUHDWWKURPERF\WRSHQLDLQSUHJQDQF\%ORRG-DQ
1DNDP\D -/LP:7&URZWKHU0$11(;$,QYHVWLJDWRUV1(QJO -0HG   GRLEORRG(SXE1RY
6HS  

Centerȱfor GastrointestinalȱMotility

+HPDWRORJ\&DVHV
&RPPRQ&RPSOH[DQG5DUH (VRSKDJHDO'LVRUGHUV
EĂŶĐLJĞƌůŝŶĞƌ͕D͘͘
,͘&ƌĂŶŬůŝŶƵŶŶWƌŽĨĞƐƐŽƌŽĨDĞĚŝĐŝŶĞ
ŚŝĞĨ͕ŝǀŝƐŝŽŶŽĨ,ĞŵĂƚŽůŽŐLJ :DOWHU:&KDQ0'03+
ƌŝŐŚĂŵĂŶĚtŽŵĞŶ͛Ɛ,ŽƐƉŝƚĂů
WƌŽĨĞƐƐŽƌŽĨDĞĚŝĐŝŶĞ
,ĂƌǀĂƌĚDĞĚŝĐĂů^ĐŚŽŽů
'LUHFWRU&HQWHUIRU*DVWURLQWHVWLQDO0RWLOLW\
'LYLVLRQRI*DVWURHQWHURORJ\+HSDWRORJ\ DQG(QGRVFRS\
%ULJKDPDQG:RPHQ¶V+RVSLWDO
$VVLVWDQW3URIHVVRURI0HGLFLQH+DUYDUG0HGLFDO6FKRRO

Z&ZE^
5HIHUHQFHV
'ĞŽƌŐĞ:E͕ůͲEŽƵƌŝ >͘ŝĂŐŶŽƐƚŝĐĂŶĚƚŚĞƌĂƉĞƵƚŝĐĐŚĂůůĞŶŐĞƐŝŶƚŚĞ
ƚŚƌŽŵďŽƚŝĐƚŚƌŽŵďŽĐLJƚŽƉĞŶŝĐƉƵƌƉƵƌĂ ĂŶĚŚĞŵŽůLJƚŝĐƵƌĞŵŝĐƐLJŶĚƌŽŵĞƐ͘
,ĞŵĂƚŽůŽŐLJͬƚŚĞĚƵĐĂƚŝŽŶWƌŽŐƌĂŵŽĨƚŚĞŵĞƌŝĐĂŶ^ŽĐŝĞƚLJŽĨ ƒ /LDFRXUDV HWDO-$OOHUJ\&OLQ ,PPXQRO
,ĞŵĂƚŽůŽŐLJŵĞƌŝĐĂŶ^ŽĐŝĞƚLJŽĨ,ĞŵĂƚŽůŽŐLJĚƵĐĂƚŝŽŶWƌŽŐƌĂŵ
ϮϬϭϮ͖ϮϬϭϮ͗ϲϬϰͲϵ͘ 
>ĞŐĞŶĚƌĞD͕>ŝĐŚƚ ͕DƵƵƐ W͕ĞƚĂů͘dĞƌŵŝŶĂůĐŽŵƉůĞŵĞŶƚŝŶŚŝďŝƚŽƌ
ĞĐƵůŝnjƵŵĂď ŝŶĂƚLJƉŝĐĂůŚĞŵŽůLJƚŝĐͲƵƌĞŵŝĐƐLJŶĚƌŽŵĞ͘dŚĞEĞǁŶŐůĂŶĚ ƒ 6RXUFH)R[HWDO*XW
ũŽƵƌŶĂůŽĨŵĞĚŝĐŝŶĞϮϬϭϯ͖ϯϲϴ͗ϮϭϲϵͲϴϭ͘
^ĐŚƌĂŵ D͕ĞƌůŝŶĞƌE͘,Žǁ/ƚƌĞĂƚŚĞŵŽƉŚĂŐŽĐLJƚŝĐ ůLJŵƉŚŽŚŝƐƚŝŽĐLJƚŽƐŝƐ ŝŶ
ƚŚĞĂĚƵůƚƉĂƚŝĞŶƚ͘ůŽŽĚϮϬϭϱ͖ϭϮϱ͗ϮϵϬϴͲϭϰ
ƒ 5LFKWHUHWDO1(QJO -0HG
tĂƌŬĞŶƚŝŶ d͘dŚŝŶŬŽĨ,/d͘,ĞŵĂƚŽůŽŐLJͬƚŚĞĚƵĐĂƚŝŽŶWƌŽŐƌĂŵŽĨƚŚĞ
ŵĞƌŝĐĂŶ^ŽĐŝĞƚLJŽĨ,ĞŵĂƚŽůŽŐLJŵĞƌŝĐĂŶ^ŽĐŝĞƚLJŽĨ,ĞŵĂƚŽůŽŐLJ
ƒ )DVV HWDO1HXURJDVWURHQWHURO 0RWLO 
ĚƵĐĂƚŝŽŶWƌŽŐƌĂŵϮϬϬϲ͗ϰϬϴͲϭϰ͘
tĞŝz͕:ŝ y͕tĂŶŐzt͕ĞƚĂů͘,ŝŐŚͲĚŽƐĞĚĞdžĂŵĞƚŚĂƐŽŶĞǀƐ ƉƌĞĚŶŝƐŽŶĞĨŽƌ ƒ %DUUHWHWDO1HXURJDVWURHQWHURO 0RWLO
ƚƌĞĂƚŵĞŶƚŽĨĂĚƵůƚŝŵŵƵŶĞƚŚƌŽŵďŽĐLJƚŽƉĞŶŝĂ͗ĂƉƌŽƐƉĞĐƚŝǀĞŵƵůƚŝĐĞŶƚĞƌ
ƌĂŶĚŽŵŝnjĞĚƚƌŝĂů͘ůŽŽĚϮϬϭϲ͖ϭϮϳ͗ϮϵϲͲϯϬϮ͘ 
hdE,ZKE/WEZd/d/^
3(37,&8/&(5',6($6(
:ƵůŝĂzDĐEĂďďͲĂůƚĂƌ͕D͕DW,
‡ :K,EZ^>dDE͕D
‡ ŝƌĞĐƚŽƌŽĨŶĚŽƐĐŽƉLJ
ƐƐŽĐŝĂƚĞWŚLJƐŝĐŝĂŶ
‡ ƌŝŐŚĂŵĂŶĚtŽŵĞŶ͛Ɛ,ŽƐƉŝƚĂů ŽͲŝƌĞĐƚŽƌĞŶƚĞƌĨŽƌWĂŶĐƌĞĂƚŝĐŝƐĞĂƐĞ
‡ WƌŽĨĞƐƐŽƌŽĨDĞĚŝĐŝŶĞ ŝǀŝƐŝŽŶŽĨ'ĂƐƚƌŽĞŶƚĞƌŽůŽŐLJ͕,ĞƉĂƚŽůŽŐLJĂŶĚŶĚŽƐĐŽƉLJ
‡ ,ĂƌǀĂƌĚDĞĚŝĐĂů^ĐŚŽŽů ƌŝŐŚĂŵĂŶĚtŽŵĞŶ͛Ɛ,ŽƐƉŝƚĂů
/ŶƐƚƌƵĐƚŽƌŽĨDĞĚŝĐŝŶĞ
,ĂƌǀĂƌĚDĞĚŝĐĂů^ĐŚŽŽů

ZĞĨĞƌĞŶĐĞƐ
5HIHUHQFHV
‡ Chey ED. ACG Clinical Guideline: Treatment of Helicobacter pylori
Infection. Am J Gastroenterol 2017;112:212–238
‡ Graham DY. History of Helicobacter pylori, duodenal ulcer, gastric ƒ &URFNHWW6'$PHULFDQ*DVWURHQWHURORJLFDO
ulcer and gastric cancer. World J Gastroenterol 2014;20:5191-204 $VVRFLDWLRQ,QVWLWXWH*XLGHOLQHRQ,QLWLDO
‡ Kumar NL. Initial management and timing of endoscopy in 0DQDJHPHQWRI$FXWH3DQFUHDWLWLV
nonvariceal upper GI bleeding. Gastrointest Endosc 2016;84(1):10-7
*DVWURHQWHURORJ\
‡ Lau JY. Challenges in the management of acute peptic ulcer
bleeding. Lancet 2013;381:2033-43 ƒ ,WR7(YLGHQFHEDVHGFOLQLFDOSUDFWLFH
‡ Malfertheiner P. Management of Helicobacter pylori infection-the JXLGHOLQHVIRUFKURQLFSDQFUHDWLWLV-
Maastricht IV/ Florence Consensus Report. Gut 2012;61(5):646-64
*DVWURHQWHURO 
‡ Laine L Jensen DM. Am J Gastroenterol 2012;107:345-60
‡ Villanueva C. Transfusion strategies for acute upper gastrointestinal ƒ 0DMXPGHU6&KURQLF3DQFUHDWLWLV/DQFHW
bleeding. N Engl J Med 2013;368(1):11-21 0D\  

WƐLJĐŚŝĂƚƌLJKǀĞƌǀŝĞǁ
,ZKE/>/sZ/^^E/d^
KDW>/d/KE^ /ŶĨůĂŵŵĂƚŽƌLJŽǁĞůŝƐĞĂƐĞ
^ŽŶŝĂ&ƌŝĞĚŵĂŶD
ŶŶĂZƵƚŚĞƌĨŽƌĚ͕D͕DW, ƐƐŽĐŝĂƚĞWŚLJƐŝĐŝĂŶ͕ƌŝŐŚĂŵĂŶĚtŽŵĞŶ͛Ɛ,ŽƐƉŝƚĂů
ůŝŶŝĐĂůŝƌĞĐƚŽƌŽĨ,ĞƉĂƚŽůŽŐLJ ŝǀŝƐŝŽŶŽĨ'ĂƐƚƌŽĞŶƚĞƌŽůŽŐLJ͕ĞƉĂƌƚŵĞŶƚŽĨDĞĚŝĐŝŶĞ
ŝǀŝƐŝŽŶŽĨ'ĂƐƚƌŽŶƚĞƌŽůŽŐLJ͕,ĞƉĂƚŽůŽŐLJΘŶĚŽƐĐŽƉLJ ƐƐŽĐŝĂƚĞWƌŽĨĞƐƐŽƌ͕,ĂƌǀĂƌĚDĞĚŝĐĂů^ĐŚŽŽů
ƌŝŐŚĂŵĂŶĚtŽŵĞŶ͛Ɛ,ŽƐƉŝƚĂů
ƐƐŝƐƚĂŶƚWƌŽĨĞƐƐŽƌŽĨDĞĚŝĐŝŶĞ
,ĂƌǀĂƌĚDĞĚŝĐĂů^ĐŚŽŽů

^>dZ&ZE^ ZĞĨĞƌĞŶĐĞƐ
‡ ''ĂƌĐŝĂͲdƐĂŽĞƚĂů͘WŽƌƚĂů,LJƉĞƌƚĞŶƐŝǀĞůĞĞĚŝŶŐŝŶŝƌƌŚŽƐŝƐ͗ZŝƐŬ^ƚƌĂƚŝĨŝĐĂƚŝŽŶ͕ŝĂŐŶŽƐŝƐĂŶĚ ϭ͘ ŽŶŽǀĂƐ^͗ŝŽůŽŐŝĐƚŚĞƌĂƉŝĞƐĂŶĚƌŝƐŬŽĨŝŶĨĞĐƚŝŽŶĂŶĚŵĂůŝŐŶĂŶĐLJŝŶ
DĂŶĂŐĞŵĞŶƚ͗ϮϬϭϲWƌĂĐƚŝĐĞ'ƵŝĚĂŶĐĞĨƌŽŵ^>͘,ĞƉĂƚŽůŽŐLJϮϬϭϳ͖ϲϱ͗ϯϭϬͲϯϯϱ͘
ƉĂƚŝĞŶƚƐǁŝƚŚŝŶĨůĂŵŵĂƚŽƌLJďŽǁĞůĚŝƐĞĂƐĞ͗ĂƐLJƐƚĞŵŝĐƌĞǀŝĞǁĂŶĚŶĞƚǁŽƌŬ
‡ DDĂŶĚŽƌĨĞƌ ĞƚĂů͘EŽŶƐĞůĞĐƚŝǀĞ% ůŽĐŬĞƌƐ/ŶĐƌĞĂƐĞZŝƐŬĨŽƌ,ĞƉĂƚŽƌĞŶĂů ^LJŶĚƌŽŵĞĂŶĚĞĂƚŚŝŶWĂƚŝĞŶƚƐ
ǁŝƚŚŝƌƌŚŽƐŝƐĂŶĚ^ƉŽŶƚĂŶĞŽƵƐĂĐƚĞƌŝĂůWĞƌŝƚŽŶŝƚŝƐ͘'ĂƐƚƌŽĞŶƚĞƌŽůŽŐLJϮϬϭϰ͖ϭϰϲ͗ϭϲϴϬͲϭϲϵϬ͘ ŵĞƚĂͲĂŶĂůLJƐŝƐ͘ůŝŶ'ĂƐƚƌŽĞŶĞƚƌŽů,ĞƉĂƚŽůϭϰ͗ϭϯϴϱ͕ϮϬϭϲ͘
‡ WDĂƌƚŝŶĞƚĂů͘ǀĂůƵĂƚŝŽŶĨŽƌ>ŝǀĞƌdƌĂŶƐƉůĂŶƚĂƚŝŽŶŝŶĚƵůƚƐ͗ϮϬϭϯWƌĂĐƚŝĐĞ'ƵŝĚĞůŝŶĞďLJŵĞƌŝĐĂŶ Ϯ͘ ,Ă͕<ŽƌŶďůƵƚŚ͘ƌŝƚŝĐĂůZĞǀŝĞǁŽĨŝŽƐŝŵŝůĂƌƐŝŶ/͗dŚĞŽŶĨůƵĞŶĐĞ
ƐƐŽĐŝĂƚŝŽŶĨŽƌƚŚĞ^ƚƵĚLJŽĨ>ŝǀĞƌŝƐĞĂƐĞƐĂŶĚŵĞƌŝĐĂŶ^ŽĐŝĞƚLJŽĨdƌĂŶƐƉůĂŶƚĂƚŝŽŶ͘,ĞƉĂƚŽůŽŐLJ ϮϬϭϰ͖ϱϵ͗ ŽĨŝŽůŽŐŝĐƌƵŐĞǀĞůŽƉŵĞŶƚ͕ZĞŐƵůĂƚŽƌLJZĞƋƵŝƌĞŵĞŶƚƐ͕ůŝŶŝĐĂů
ϭϭϰϰͲϭϭϲϱ͘
‡ ZƵŶLJŽŶ͘DĂŶĂŐĞŵĞŶƚŽĨĚƵůƚWĂƚŝĞŶƚƐǁŝƚŚƐĐŝƚĞƐƵĞƚŽŝƌƌŚŽƐŝƐ͘,ĞƉĂƚŽůŽŐLJ ϮϬϭϯ͖ϱϳ͗ϭϲϱϭͲϭϲϱϯ͘ KƵƚĐŽŵĞƐ͕ĂŶĚŝŐƵƐŝŶĞƐƐ͘/ŶĨůĂŵŵŽǁĞůŝƐϮϮ͗Ϯϱϭϯ͕ϮϬϭϲ͘
‡ ,sŝůƐƚƌƵƉ ĞƚĂů͘,ĞƉĂƚŝĐŶĐĞƉŚĂůŽƉĂƚŚLJŝŶŚƌŽŶŝĐ>ŝǀĞƌŝƐĞĂƐĞ͗ϮϬϭϰWƌĂĐƚŝĐĞ'ƵŝĚĞůŝŶĞƐďLJƚŚĞ ϯ͘ :ƵůƐŐĂĂƌĚD͗ŽŶĐĞŶƚƌĂƚŝŽŶƐŽĨĂĚĂůŝŵƵŵĂďĂŶĚŝŶĨůŝdžŝŵĂďŝŶŵŽƚŚĞƌƐ
ŵĞƌŝĐĂŶƐƐŽĐŝĂƚŝŽŶĨŽƌƚŚĞ^ƚƵĚLJŽĨ>ŝǀĞƌŝƐĞĂƐĞƐĂŶĚƚŚĞƵƌŽƉĞĂŶƐƐŽĐŝĂƚŝŽŶĨŽƌƚŚĞ^ƚƵĚLJŽĨƚŚĞ ĂŶĚŶĞǁďŽƌŶƐĂŶĚĞĨĨĞĐƚƐŽŶŝŶĨĞĐƚŝŽŶ͘'ĂƐƚƌŽĞŶƚĞƌŽůŽŐLJϭϱϭ͗ϭϭϬ͕ϮϬϭϲ͘
>ŝǀĞƌ͘,ĞƉĂƚŽůŽŐLJ ϮϬϭϰ͖ϲϬ͗ϳϭϱͲϳϯϱ͘
ϰ͘ EŐ^͗'ĞŽŐƌĂƉŚŝĐĂůǀĂƌŝĂďŝůŝƚLJĂŶĚĞŶǀŝƌŽŶŵĞŶƚĂůƌŝƐŬĨĂĐƚŽƌƐŝŶ
ŝŶĨůĂŵŵĂƚŽƌLJďŽǁĞůĚŝƐĞĂƐĞ͘'ƵƚϲϮ͗ϲϯϬ͕ϮϬϭϯ͘
ϱ͘ ZĞŐƵĞŝƌŽD͗/ŶĨůŝdžŝŵĂďZĞĚƵĐĞƐŶĚŽƐĐŽƉŝĐ͕ďƵƚEŽƚůŝŶŝĐĂů͕ZĞĐƵƌƌĞŶĐĞ
ŽĨƌŽŚŶΖƐŝƐĞĂƐĞĨƚĞƌ/ůĞŽĐŽůŽŶŝĐZĞƐĞĐƚŝŽŶ͘'ĂƐƚƌŽĞŶƚĞƌŽůŽŐLJ
ϭϱϬ͗ϭϱϲϴ͕ϮϬϭϲ͘
*,%2$5'5(9,(:
$FXWHDQG&KURQLF'LDUUKHD 0XWKRND/0XWLQJD0'
$VVRFLDWH3K\VLFLDQ
'LYLVLRQRI*DVWURHQWHURORJ\+HSDWRORJ\DQG(QGRVFRS\
'HSDUWPHQWRI0HGLFLQH
%ULJKDPDQG:RPHQ¶V+RVSLWDO
%HQMDPLQ6PLWK0' $VVLVWDQW3URIHVVRURI0HGLFLQH
+DUYDUG0HGLFDO6FKRRO
$VVLVWDQW3URIHVVRU+DUYDUG0HGLFDO6FKRRO
$WWHQGLQJ3K\VLFLDQ%ULJKDP :RPHQV)DXONQHUDQG
9$0&*,)HOORZVKLS7UDLQLQJ3URJUDP

5HIHUHQFHV 6HOHFWHG5HIHUHQFHV
/D5RTXH5+DUULV-%$SSURDFKWRWKHDGXOWZLWKDFXWHGLDUUKHDLQ  %HDW\-6DQG6KDVKLGKDUDQ0$QDOILVVXUHClin Colon Rectal Surg 
UHVRXUFHULFKVHWWLQJV,Q8S7R'DWH3RVW7: (G 8S7R'DWH:DOWKDP 0DU  
0$ DFFHVVHGRQ$SULOWK  
 /LX3+&DR<.HHOH\%HWDO$GKHUHQFHWRD+HDOWK\/LIHVW\OHLV
$VVRFLDWHG:LWKD/RZHU5LVNRI'LYHUWLFXOLWLV$PRQJ0HQAm J
$PHULFDQJDVWURHQWHURORJLFDODVVRFLDWLRQPHGLFDOSRVLWLRQVWDWHPHQW Gastrenterol 
JXLGHOLQHVIRUWKHHYDOXDWLRQDQGPDQDJHPHQWRIFKURQLFGLDUUKHD  .DPDO6.KDQ06HWK$HWDO%HQHILFLDO(IIHFWVRI6WDWLQVRQWKH5DWHRI
*DVWURHQWHURORJ\ +HSDWLF)LEURVLV+HSDWLF'HFRPSHQVDWLRQDQG0RUWDOLW\LQ&KURQLF/LYHU
'LVHDVH$6\VWHPDWLF5HYLHZDQG0HWD$QDO\VLVAm J Gastroenterol
%RQLV3$/D0RQW-7$SSURDFKWRWKHSDWLHQWZLWKFKURQLFGLDUUKHDLQ 
UHVRXUFHULFKVHWWLQJV,Q8S7R'DWH3RVW7: (G 8S7R'DWH:DOWKDP  .DQZDO).UDPHU-$VFK6HWDO5LVNRI+HSDWRFHOOXODU&DQFHULQ+&9
0$ DFFHVVHGRQ0D\WK   3DWLHQWV7UHDWHG:LWK'LUHFW$FWLQJ$QWLYLUDO$JHQWVGastroenterol

&HQWHUVIRU'LVHDVH&RQWURODQG3UHYHQWLRQZZZFGFJRY  5H['.%RODQG&5'RPLQLW]-$HWDO&RORUHFWDO&DQFHU6FUHHQLQJ
5HFRPPHQGDWLRQVIRU3K\VLFLDQVDQG3DWLHQWVIURPWKH860XOWL6RFLHW\
7DVN)RUFHRQ&RORUHFWDO&DQFHUAm J Gastroenterol -XO  


/HXNHPLDDQG0'6

(GZLQ3$O\HD0' WƌŽƐƚĂƚĞĂŶĚůĂĚĚĞƌĂŶĐĞƌ͗
0HGLFDO2QFRORJ\ tŚĂƚƚŚĞ/ŶƚĞƌŶŝƐƚEĞĞĚƐƚŽ<ŶŽǁ
'DQD)DUEHU&DQFHU,QVWLWXWH
>ĂƵƌĞŶ͘,ĂƌƐŚŵĂŶD
%ULJKDPDQG:RPHQ¶V+RVSLWDO
$VVRFLDWH3URIHVVRURI0HGLFLQH ^ĞŶŝŽƌWŚLJƐŝĐŝĂŶ͕ĂŶĂͲ&ĂƌďĞƌĂŶĐĞƌ/ŶƐƚŝƚƵƚĞ
+DUYDUG0HGLFDO6FKRRO ƐƐŝƐƚĂŶƚWƌŽĨĞƐƐŽƌŽĨDĞĚŝĐŝŶĞ͕,ĂƌǀĂƌĚDĞĚŝĐĂů^ĐŚŽŽů

:ƵůLJϮϯ͕ϮϬϭϴ

6HOHFWHG5HIHUHQFHV ^ĞůĞĐƚĞĚZĞĨĞƌĞŶĐĞƐ
‡ 'RKQHU+6WLOJHQEDXHU6%HQQHU$HWDO*HQRPLFDEHUUDWLRQVDQG ͻ ŝůůͲdžĞůƐŽŶ ͕,ŽůŵďĞƌŐ>͕ZƵƵƚƵ DĞƚĂů͘ZĂĚŝĐĂůƉƌŽƐƚĂƚĞĐƚŽŵLJǀĞƌƐƵƐǁĂƚĐŚĨƵů
VXUYLYDOLQFKURQLFO\PSKRF\WLFOHXNHPLD1(QJO-0HG ǁĂŝƚŝŶŐŝŶĞĂƌůLJƉƌŽƐƚĂƚĞĐĂŶĐĞƌ͘dŚĞEĞǁŶŐůĂŶĚũŽƵƌŶĂůŽĨŵĞĚŝĐŝŶĞ͘
 ϮϬϭϭ͖ϯϲϰ;ϭϴͿ͗ϭϳϬϴͲϭϳ͘
‡ *UHHQEHUJ3&R[&/H%HDX00HWDO,QWHUQDWLRQDOVFRULQJV\VWHP ͻ ŽůůĂ D͕ĚĞZĞŝũŬĞ dD͕sĂŶdŝĞŶŚŽǀĞŶ '͕ĞƚĂů͘ƵƌĂƚŝŽŶŽĨĂŶĚƌŽŐĞŶƐƵƉƉƌĞƐƐŝŽŶŝŶƚŚĞ
IRUHYDOXDWLQJSURJQRVLVLQP\HORG\VSODVWLFV\QGURPHV>VHHFRPPHQWV@ ƚƌĞĂƚŵĞŶƚŽĨƉƌŽƐƚĂƚĞĐĂŶĐĞƌ͘dŚĞEĞǁŶŐůĂŶĚũŽƵƌŶĂůŽĨŵĞĚŝĐŝŶĞ͘
ϮϬϬϵ͖ϯϲϬ;ϮϰͿ͗ϮϱϭϲͲϮϳ͘
>SXEOLVKHGHUUDWXPDSSHDUVLQ%ORRG)HE  @%ORRG ͻ ,ƵƐƐĂŝŶD͕dĂŶŐĞŶ D͕ ĞƌƌLJ>͕ ĞƚĂů͘/ŶƚĞƌŵŝƚƚĞŶƚǀĞƌƐƵƐĐŽŶƚŝŶƵŽƵƐĂŶĚƌŽŐĞŶ
 ĚĞƉƌŝǀĂƚŝŽŶŝŶƉƌŽƐƚĂƚĞĐĂŶĐĞƌ͘EŶŐů :DĞĚ͘ ϮϬϭϯ Ɖƌϰ͖ϯϲϴ;ϭϰͿ͗ϭϯϭϰͲϮϱ͘
‡ )HQDX[30XIWL*-+HOOVWURP/LQGEHUJ(HWDO(IILFDF\RI ͻ ǀŽŶĚĞƌDĂĂƐĞ ,͕^ĞŶŐĞůŽǀ >͕ZŽďĞƌƚƐ:d͕ĞƚĂů͘>ŽŶŐͲƚĞƌŵƐƵƌǀŝǀĂůƌĞƐƵůƚƐŽĨĂ
D]DFLWLGLQHFRPSDUHGZLWKWKDWRIFRQYHQWLRQDOFDUHUHJLPHQVLQWKH ƌĂŶĚŽŵŝnjĞĚƚƌŝĂůĐŽŵƉĂƌŝŶŐŐĞŵĐŝƚĂďŝŶĞƉůƵƐĐŝƐƉůĂƚŝŶ͕ǁŝƚŚŵĞƚŚŽƚƌĞdžĂƚĞ͕ǀŝŶďůĂƐƚŝŶĞ͕
WUHDWPHQWRIKLJKHUULVNP\HORG\VSODVWLFV\QGURPHVDUDQGRPLVHG ĚŽdžŽƌƵďŝĐŝŶ͕ƉůƵƐĐŝƐƉůĂƚŝŶ ŝŶƉĂƚŝĞŶƚƐǁŝƚŚďůĂĚĚĞƌĐĂŶĐĞƌ͘:ŽƵƌŶĂůŽĨůŝŶŝĐĂůKŶĐŽůŽŐLJ͘
ϮϬϬϱ͖Ϯϯ;ϮϭͿ͗ϰϲϬϮͲϴ͘
RSHQODEHOSKDVH,,,VWXG\/DQFHW2QFRO
ͻ 'ƌŽƐƐŵĂŶ,͕EĂƚĂůĞ Z͕dĂŶŐĞŶ D͕ĞƚĂů͘EĞŽĂĚũƵǀĂŶƚ ĐŚĞŵŽƚŚĞƌĂƉLJƉůƵƐ
‡ 6DJOLR*.LP':,VVDUDJULVLO6HWDO1LORWLQLEYHUVXVLPDWLQLEIRU ĐLJƐƚĞĐƚŽŵLJ ĐŽŵƉĂƌĞĚǁŝƚŚĐLJƐƚĞĐƚŽŵLJ ĂůŽŶĞĨŽƌůŽĐĂůůLJĂĚǀĂŶĐĞĚďůĂĚĚĞƌĐĂŶĐĞƌ͘dŚĞ
QHZO\GLDJQRVHGFKURQLFP\HORLGOHXNHPLD1(QJO-0HG EĞǁŶŐůĂŶĚũŽƵƌŶĂůŽĨŵĞĚŝĐŝŶĞ͘ϮϬϬϯ͖ϯϰϵ;ϵͿ͗ϴϱϵͲϲϲ͘
 ͻ ĞůůŵƵŶƚ ũ͕ĚĞtŝƚZ͕sĂƵŐŚŶĞƚĂů͘WĞŵďƌŽůŝnjƵŵĂď ĂƐƐĞĐŽŶĚͲůŝŶĞƚŚĞƌĂƉLJĨŽƌ
‡ .DQWDUMLDQ+6KDK13+RFKKDXV$HWDO'DVDWLQLEYHUVXVLPDWLQLELQ ĂĚǀĂŶĐĞĚƵƌŽƚŚĞůŝĂů ĐĂƌĐŝŶŽŵĂ͘EŶŐů :DĞĚϮϬϭϳ͖ϯϳϲ;ϭϭͿ͗ϭϬϭϱͲϭϬϮϲ͘

QHZO\GLDJQRVHGFKURQLFSKDVHFKURQLFP\HORLGOHXNHPLD1(QJO-
0HG
%UHDVW&DQFHU8SGDWH
³$EXIIHWRIEUHDVWFDQFHUWRSLFV´

>ƵŶŐĂŶĐĞƌ
ĂǀŝĚ:ĂĐŬŵĂŶ͕D
^ĞŶŝŽƌWŚLJƐŝĐŝĂŶ͕ĂŶĂͲ&ĂƌďĞƌĂŶĐĞƌ/ŶƐƚŝƚƵƚĞ
DĞĚŝĐĂůŝƌĞĐƚŽƌŽĨůŝŶŝĐĂůWĂƚŚǁĂLJƐ͕ĂŶĂͲ&ĂƌďĞƌĂŶĐĞƌ/ŶƐƚŝƚƵƚĞ
ƐƐŝƐƚĂŶƚWƌŽĨĞƐƐŽƌŽĨDĞĚŝĐŝŶĞ͕,ĂƌǀĂƌĚDĞĚŝĐĂů^ĐŚŽŽů

:HQG\<&KHQ0'03+
'DQD)DUEHU&DQFHU,QVWLWXWH
%ULJKDPDQG:RPHQ¶V+RVSLWDO
ϭ

ZĞĨĞƌĞŶĐĞƐ 5HIHUHQFHV
 0DQVRQ-(HWDO0HQRSDXVDOKRUPRQHWKHUDS\DQGKHDOWKRXWFRPHV
GXULQJWKHLQWHUYHQWLRQDQGH[WHQGHGSRVWVWRSSLQJ SKDVHVRIWKH:RPHQ¶V
+HDOWK,QLWLDWLYHUDQGRPL]HGWULDOV-$0$
‡ *HWWLQJHU 6+RUQ/-DFNPDQ'HWDO-&OLQ 2QFR 
 0R\HU9$HWDO0HGLFDWLRQVWRGHFUHDVHWKHULVNIRUEUHDVWFDQFHULQ
‡ 1/67LQYHVWLJDWRUVNEJM 
ZRPHQUHFRPPHQGDWLRQVIURPWKH863UHYHQWLYH6HUYLFHV7DVN)RUFH
‡ 1DLGLFK HWDO5DGLRORJ\   UHFRPPHQGDWLRQVWDWHPHQW$QQ,QWHUQ0HG
‡ 3HORVRI HWDO0D\R&OLQ 3URF  
‡ 6((5&DQFHU6WDWLVWLFV5HYLHZ  3HUH]($DQG6SDQR -3&XUUHQWDQGHPHUJLQJWDUJHWHGWKHUDSLHVIRU
PHWDVWDWLFEUHDVWFDQFHU&DQFHU
KWWSRQOLQHOLEUDU\ZLOH\FRPGRLFQFUIXOO
 :RUOG&DQFHU5HVHDUFK)XQG,QWHUQDWLRQDO$PHULFDQ,QVWLWXWHIRU&DQFHU
5HVHDUFK&RQWLQXRXV8SGDWH3URMHFW5HSRUW'LHWQXWULWLRQSK\VLFDO
DFWLYLW\DQGEUHDVWFDQFHU
KWWSZFUIRUJEUHDVWFDQFHU

*,&DQFHUVIRUWKH%RDUGV
>LJŵƉŚŽŵĂ -HIIUH\0H\HUKDUGW0'03+
&OLQLFDO'LUHFWRU*DVWURLQWHVWLQDO&DQFHU&HQWHU
DƵůƚŝƉůĞDLJĞůŽŵĂ 'HSXW\&OLQLFDO5HVHDUFK2IILFHU
'DQD)DUEHU&DQFHU,QVWLWXWH
ŶŶ^͘>ĂĂƐĐĞ͕D͕DD^Đ
/ŶƐƚŝƚƵƚĞWŚLJƐŝĐŝĂŶ
$VVRFLDWH3URIHVVRURI0HGLFLQH
ĂŶĂ&ĂƌďĞƌĂŶĐĞƌ/ŶƐƚŝƚƵƚĞ
ƐƐŽĐŝĂƚĞWƌŽĨĞƐƐŽƌŽĨDĞĚŝĐŝŶĞ +DUYDUG0HGLFDO6FKRRO
,ĂƌǀĂƌĚDĞĚŝĐĂů^ĐŚŽŽů 6HQLRU3K\VLFLDQ'DQD)DUEHU&DQFHU,QVWLWXWH

6HOHFWHG5HIHUHQFHV
ZĞĨĞƌĞŶĐĞƐ
‡ 5XVWJL$DQG(O6HUDJ(VRSKDJHDO&DQFHU1(QJO-0HG

‡ &RKHQDQG/HLFKPDQ&RQWURYHUVLHVLQWKH7UHDWPHQWRI/RFDODQG/RFDOO\
‡ ^ǁĞƌĚůŽǁĞƚĂů͘dŚĞϮϬϭϲƌĞǀŝƐŝŽŶŽĨƚŚĞtŽƌůĚ,ĞĂůƚŚ $GYDQFHG*DVWULFDQG(VRSKDJHDO&DQFHUV-RXUQDORI&OLQLFDO2QFRORJ\
KƌŐĂŶŝnjĂƚŝŽŶĐůĂƐƐŝĨŝĐĂƚŝŽŶŽĨůLJŵƉŚŽŝĚŶĞŽƉůĂƐŵƐ͘ QR  -XQH 

ůŽŽĚϮϬϭϲ͘
‡ 6KDKDQG.HOVHQ*DVWULFFDQFHU$SULPHURQWKHHSLGHPLRORJ\DQGELRORJ\RIWKH
GLVHDVHDQGDQRYHUYLHZRIWKHPHGLFDOPDQDJHPHQWRIDGYDQFHGGLVHDVH-1DWO
‡ EEůŝŶŝĐĂůWƌĂĐƚŝĐĞ'ƵŝĚĞůŝŶĞƐŝŶKŶĐŽůŽŐLJ͘ &RPSU&DQF1HWZ
ǁǁǁ͘ŶĐĐŶ͘ŽƌŐ ‡ +HHVWDQG0XUSK\DQG/RZ\$SSURDFKWR3DWLHQWV:LWK3DQFUHDWLF&DQFHU
:LWKRXW'HWHFWDEOH0HWDVWDVHV-RXUQDORI&OLQLFDO2QFRORJ\QR  -XQH
‡ ǀĂŶƐ>^͕,ĂŶĐŽĐŬt͘EŽŶͲ,ŽĚŐŬŝŶůLJŵƉŚŽŵĂ͘>ĂŶĐĞƚ͘  
ϮϬϬϯϯϲϮ͗ϭϯϵͲϰϲ͘ ‡ .R3URJUHVVLQWKH7UHDWPHQWRI0HWDVWDWLF3DQFUHDWLF&DQFHUDQGWKH6HDUFKIRU
1H[W2SSRUWXQLWLHV-RXUQDORI&OLQLFDO2QFRORJ\QR  -XQH 


‡ )DNLK0HWDVWDWLF&RORUHFWDO&DQFHU&XUUHQW6WDWHDQG)XWXUH'LUHFWLRQV-RXUQDO
RI&OLQLFDO2QFRORJ\QR  -XQH 
3('97$17,&2$*8/$7,21
6DPXHO=*ROGKDEHU0'
,QWHULP&KLHI'LYLVLRQRI
&DUGLRYDVFXODU0HGLFLQH
6HFWLRQ+HDG9DVFXODU0HGLFLQH ,ĞĂƌƚ&ĂŝůƵƌĞ
%ULJKDPDQG:RPHQ¶V+RVSLWDO $QMX1RKULD0'
3URIHVVRURI0HGLFLQH $VVLVWDQW3URIHVVRU
$GYDQFHG+HDUW'LVHDVH6HFWLRQ
+DUYDUG0HGLFDO6FKRRO &DUGLRYDVFXODU'LYLVLRQ
%ULJKDPDQG:RPHQ¶V+RVSLWDO

5HIHUHQFHV ZĞĨĞƌĞŶĐĞƐ
‡ zĂŶĐLJtĞƚĂů͘ϮϬϭϯ&ͬ,ŐƵŝĚĞůŝŶĞĨŽƌƚŚĞŵĂŶĂŐĞŵĞŶƚŽĨ
‡ 1(-0 ŚĞĂƌƚĨĂŝůƵƌĞ͘:ŵŽůůĂƌĚŝŽů͘ϮϬϭϯ͖ϲϮ;ϭϲͿ͗ĞϭϰϳͲϮϯϵ͘
‡ ,ƵŶƚ^ĞƚĂů͘,&^ϮϬϭϬĐŽŵƉƌĞŚĞŶƐŝǀĞŚĞĂƌƚĨĂŝůƵƌĞƉƌĂĐƚŝĐĞ
‡ 1(-0 ŐƵŝĚĞůŝŶĞ͘:ĂƌĚŝĂĐ&Ăŝů͘ϮϬϭϬ͖ϭϲ͗ĞϭͲĞϭϵϰ͘
‡ $P-&DUGLRORJ\ ‡ DĐDƵƌƌĂLJ::sĞƚĂů͘ŶŐŝŽƚĞŶƐŝŶͲŶĞƉƌŝůLJƐŝŶŝŶŚŝďŝƚŝŽŶǀĞƌƐƵƐ
ĞŶĂůĂƉƌŝůŝŶŚĞĂƌƚĨĂŝůƵƌĞ͘EŶŐů:DĞĚ͘ϮϬϭϰ͖ϯϳϭ͗ϵϵϯͲϭϬϬϰ͘
‡ 9DQ(V17KURPE+DHPRVW ‡ ^ǁĞĚďĞƌŐ<ĞƚĂů͘/ǀĂďƌĂĚŝŶĞĂŶĚŽƵƚĐŽŵĞƐŝŶĐŚƌŽŶŝĐŚĞĂƌƚ
 ĨĂŝůƵƌĞ͗ĂƌĂŶĚŽŵŝnjĞĚƉůĂĐĞďŽͲĐŽŶƚƌŽůůĞĚƐƚƵĚLJ͘>ĂŶĐĞƚ
ϮϬϭϬ͖ϯϳϲ͗ϴϳϱͲϴϴϱ͘
‡ 3ULQV0+,67+ ‡ ďƌĂŚĂŵtdĞƚĂů͘tŝƌĞůĞƐƐƉƵůŵŽŶĂƌLJĂƌƚĞƌLJŚĞŵŽĚLJŶĂŵŝĐ
ŵŽŶŝƚŽƌŝŶŐŝŶĐŚƌŽŶŝĐŚĞĂƌƚĨĂŝůƵƌĞ͗ĂƌĂŶĚŽŵŝƐĞĚĐůŝŶŝĐĂůƚƌŝĂů͘
>ĂŶĐĞƚϮϬϭϭ͖ϯϳϳ͗ϲϱϴͲϲϲϲ͘

ƚƌŝĂů&ŝďƌŝůůĂƚŝŽŶĂŶĚŽŵŵŽŶ^ƵƉƌĂǀĞŶƚƌŝĐƵůĂƌ
ĚƵůƚŽŶŐĞŶŝƚĂů,ĞĂƌƚŝƐĞĂƐĞ dĂĐŚLJĐĂƌĚŝĂƐ
ŶŶĞDĂƌŝĞsĂůĞŶƚĞ͕D
^ƵŶŝů<ĂƉƵƌD
ŽƐƚŽŶĚƵůƚŽŶŐĞŶŝƚĂů,ĞĂƌƚŝƐĞĂƐĞĂŶĚWƵůŵŽŶĂƌLJ
,LJƉĞƌƚĞŶƐŝŽŶWƌŽŐƌĂŵ
ŝǀŝƐŝŽŶŽĨĂƌĚŝŽůŽŐLJ
ĂƌĚŝĂĐůĞĐƚƌŽƉŚLJƐŝŽůŽŐLJ
ƌŝŐŚĂŵĂŶĚtŽŵĞŶ͛Ɛ,ŽƐƉŝƚĂů͕ŽƐƚŽŶŚŝůĚƌĞŶ͛Ɛ,ŽƐƉŝƚĂů
ƌŝŐŚĂŵĂŶĚtŽŵĞŶ͛Ɛ,ŽƐƉŝƚĂů
ƐƐŽĐŝĂƚĞWƌŽĨĞƐƐŽƌ͕,ĂƌǀĂƌĚDĞĚŝĐĂů^ĐŚŽŽů
/ŶƐƚƌƵĐƚŽƌ͕,ĂƌǀĂƌĚDĞĚŝĐĂů^ĐŚŽŽů

ZĞƐŽƵƌĐĞƐ
^ƵƉƉůĞŵĞŶƚĂůZĞĨĞƌĞŶĐĞ^ůŝĚĞ
‡ ^ƚŽƵƚ<ĞƚĂů͘ϮϬϭϳͬ,'ƵŝĚĞůŝŶĞĨŽƌƚŚĞDĂŶĂŐĞŵĞŶƚŽĨ
ĚƵůƚƐtŝƚŚŽŶŐĞŶŝƚĂů,ĞĂƌƚŝƐĞĂƐĞ͘^ŽŽŶƚŽďĞƉƵďůŝƐŚĞĚ͙ϮϬϭϴ ‡ WƌŝƚĐŚĞƚƚ>͘DĂŶĂŐĞŵĞŶƚŽĨĂƚƌŝĂůĨŝďƌŝůůĂƚŝŽŶ͘EŶŐů:DĞĚ
ϭϵϵϮ͖ϯϮϲ͗ϭϮϲϰ͘
‡ sĂůĞŶƚĞD͕>ĂŶĚnjďĞƌŐD:͘ĚƵůƚŽŶŐĞŶŝƚĂů,ĞĂƌƚŝƐĞĂƐĞ͘ŚĂƉƚĞƌ ‡ ƚƌŝĂůĨŝďƌŝůůĂƚŝŽŶ͗ĐƵƌƌĞŶƚƵŶĚĞƌƐƚĂŶĚŝŶŐƐĂŶĚƌĞƐĞĂƌĐŚ
Ϯϲϰ͘,ĂƌƌŝƐŽŶ͛ƐdĞdžƚŬŽĨ/ŶƚĞƌŶĂůDĞĚŝĐŝŶĞ͕ĞĚ͘>ŽƐĐĂůnjŽ:͘ ŝŵƉĞƌĂƚŝǀĞƐ͘dŚĞEĂƚŝŽŶĂů,ĞĂƌƚ͕>ƵŶŐ͕ĂŶĚůŽŽĚ/ŶƐƚŝƚƵƚĞ
DĐ'ƌĂǁͲ,ŝůů͕ϮϬϭϴ͘ tŽƌŬŝŶŐ'ƌŽƵƉŽŶƚƌŝĂů&ŝďƌŝůůĂƚŝŽŶ͘:ŵŽůůĂƌĚŝŽůϭϵϵϯ͖
ϮϮ͗ϭϴϯϬ͘
‡ ZĞŐŝƚnjͲĂŐƌŽƐĞŬsĞƚĂů͘^'ƵŝĚĞůŝŶĞƐŽŶƚŚĞŵĂŶĂŐĞŵĞŶƚŽĨ ‡ >ŝƉ'z͕DĞƚĐĂůĨĞD:͕ZĂĞW͘DĂŶĂŐĞŵĞŶƚŽĨƉĂƌŽdžLJƐŵĂůĂƚƌŝĂů
ĐĂƌĚŝŽǀĂƐĐƵůĂƌĚŝƐĞĂƐĞƐĚƵƌŝŶŐƉƌĞŐŶĂŶĐLJdŚĞdĂƐŬ&ŽƌĐĞŽŶƚŚĞ
ĨŝďƌŝůůĂƚŝŽŶ͘Y:DĞĚϭϵϵϯ͖ϴϲ͗ϰϲϳ͘
DĂŶĂŐĞŵĞŶƚŽĨĂƌĚŝŽǀĂƐĐƵůĂƌŝƐĞĂƐĞƐĚƵƌŝŶŐWƌĞŐŶĂŶĐLJŽĨƚŚĞ
ƵƌŽƉĞĂŶ^ŽĐŝĞƚLJŽĨĂƌĚŝŽůŽŐLJ;^Ϳ͘ƵƌŽƉĞĂŶ,ĞĂƌƚ:ŽƵƌŶĂů͕ ‡ 'ĂŶnj>/͕&ƌŝĞĚŵĂŶW>͘^ƵƉƌĂǀĞŶƚƌŝĐƵůĂƌƚĂĐŚLJĐĂƌĚŝĂ͘EŶŐů:
ϮϬϭϭ͘ϯϮ;ϮϰͿ͕ϯϭϰϳͲϯϭϵϳ͘ DĞĚϭϵϵϱ͖ϯϯϮ͗ϭϲϮ͘
8SGDWHRQ$GXOW,PPXQL]DWLRQV 6H[XDOO\7UDQVPLWWHG'LVHDVHV
8SGDWH

/LQGVH\5%DGHQ0' 7RGG%(OOHULQ0'
'LUHFWRURI,QIHFWLRXV'LVHDVHV
'LYLVLRQRI,QIHFWLRXV'LVHDVHV
6RXWK6KRUH+RVSLWDO
%ULJKDPDQG:RPHQ䇻V+RVSLWDO $VVRFLDWH3K\VLFLDQ%ULJKDPDQG:RPHQ¶V+RVSLWDO
'DQD)DUEHU&DQFHU,QVWLWXWH ,QVWUXFWRULQ0HGLFLQH
+DUYDUG0HGLFDO6FKRRO +DUYDUG0HGLFDO6FKRRO
WHOOHULQ#VRXWKVKRUHKHDOWKRUJ

5HIHUHQFHV
5HIHUHQFHV
 5HG%RRN5HSRUWRIWKH&RPPLWWHHRQ
,QIHFWLRXV'LVHDVHVWK (GLWLRQ$$3 ƒ &'&6H[XDOO\WUDQVPLWWHGGLVHDVHVWUHDWPHQW
 *XLGHIRU$GXOW,PPXQL]DWLRQE\WKH$&3 JXLGHOLQHV00:51R
 00:5DWZZZFGFJRYPPZU ƒ 8SWR'DWHLQ0HGLFLQH6FUHHQLQJIRU67'¶V ODVW
± $GXOW9DFFLQH5HFRPPHQGDWLRQV XSGDWH0D\
± 'HF 1R55 3J*HQHUDO
5HFRPPHQGDWLRQVRQ,PPXQL]DWLRQ $&,3DQG
$$)3
 9DFFLQHVE\3ORWNLQ DQG2UHQVWHLQWK (GLWLRQ
(OVHFYLHU 
 9DFFLQHVDQG9DFFLQDWLRQV*$GD1(-0


&RQWUDFHSWLRQ$Q8SGDWH 0HGLFDO&RPSOLFDWLRQVRI3UHJQDQF\

.DUL3%UDDWHQ0'03+ Ellen W. Seely, M.D


'LUHFWRURI&OLQLFDO5HVHDUFK
(QGRFULQRORJ\'LDEHWHV +\SHUWHQVLRQ'LYLVLRQ
$VVRFLDWH*\QHFRORJLVW)LVK&HQWHUIRU:RPHQ¶V+HDOWK
'LUHFWRURI4XDOLW\$VVXUDQFH%:+)DPLO\3ODQQLQJ
'HSDUWPHQWRI0HGLFLQH
'HSDUWPHQWRI2EVWHWULFVDQG*\QHFRORJ\
%ULJKDPDQG:RPHQ¶V+RVSLWDO
%ULJKDPDQG:RPHQ¶V+RVSLWDO 3URIHVVRURI0HGLFLQH
+DUYDUG0HGLFDO6FKRRO
,QVWUXFWRULQ2EVWHWULFV*\QHFRORJ\DQG5HSURGXFWLYH0HGLFLQH
+DUYDUG0HGLFDO6FKRRO

5HIHUHQFHV
6HHO\(:(FNHU-/0HGLFDOFRPSOLFDWLRQVLQSUHJQDQF\,Q
5HIHUHQFHV „
6LQJK$.HGLWRU6FLHQWLILF$PHULFDQPHGLFLQH>RQOLQH@
+DPLOWRQ 21 'HFNHU,QWHOOHFWXDO3URSHUWLHV-XQH
‡ %HGQDUHNHWDO1(-0 '2,$YDLODEOHDW
KWWSZZZVFLDPPHGLFLQHFRP DFFHVVHG-XQH 
‡ +RKPDQQHWDO&RQWUDFHSWLRQ „ 6HHO\(:(FNHU-&KURQLFK\SHUWHQVLRQLQSUHJQDQF\
&LUFXODWLRQ  
‡ 3HLSHUWHWDOObstet Gynecol  „ ([HFXWLYHVXPPDU\K\SHUWHQVLRQLQSUHJQDQF\$&2*
‡ *RRGPDQHWDO&RQWUDFHSWLRQ 2EVWHW*\QHFRO
6WDQGDUGVRI0HGLFDO&DUHLQ'LDEHWHV'LDEHWHV&DUH
‡ 5REHUWVHWDO&RQWUDFHSWLRQ „
-DQ 6XSSOHPHQW 66
„ $OH[DQGHU(.3HDUFH(1%UHQW*$HWDO*XLGHOLQHVRI
WKH$PHULFDQ7K\URLG$VVRFLDWLRQIRUWKH'LDJQRVLVDQG
0DQDJHPHQWRI7K\URLG'LVHDVH'XULQJ3UHJQDQF\DQGWKH
3RVWSDUWXP7K\URLG  
$SUDFWLFDODSSURDFKWRWKH
SDWLHQWZLWKPHQRSDXVDO
V\PSWRPV (YDOXDWLRQRIWKH3DWLHQW
ZLWK0HQVWUXDO,UUHJXODULWLHV
.DWKU\Q$0DUWLQ0'
5HSURGXFWLYH(QGRFULQH8QLW'HSDUWPHQWRI
0HGLFLQH 0DULD$<LDODPDV0'
0DVVDFKXVHWWV*HQHUDO+RVSLWDO
6HQLRU'HSXW\(GLWRU(QGRFULQRORJ\DQG3DWLHQW $VVRFLDWH3URJUDP'LUHFWRU,QWHUQDO0HGLFLQH5HVLGHQF\
(GXFDWLRQ8S7R'DWH
'HSDUWPHQWRI0HGLFLQH%ULJKDPDQG:RPHQ¶V+RVSLWDO
$VVLVWDQW3URIHVVRURI0HGLFLQH+DUYDUG0HGLFDO6FKRRO

5HIHUHQFHV
x 6WXHQNHOHWDO7UHDWPHQWRIV\PSWRPVRIWKH0HQRSDXVH$Q
(QGRFULQH6RFLHW\&OLQLFDO3UDFWLFH*XLGHOLQH-&OLQ(QGRFULQRO
0HWDEO
5HIHUHQFHV
x 0R\HU9$863UHYHQWLYH6HUYLFHV7DVN)RUFH0HQRSDXVDO
KRUPRQHWKHUDS\IRUWKHSULPDU\SUHYHQWLRQRIFKURQLFFRQGLWLRQV
863UHYHQWLYH6HUYLFHV7DVN)RUFHUHFRPPHQGDWLRQVWDWHPHQW ‡ *RUGRQ&0HWDO)XQFWLRQDOK\SRWKDODPLFDPHQRUUKHD
$QQ,QWHUQ0HG   $Q(QGRFULQH6RFLHW\&OLQLFDO3UDFWLFH*XLGHOLQH-&(0

‡ 0DQVRQ-(&KOHERZVNL570HQRSDXVDOKRUPRQHWKHUDS\DQG
KHDOWKRXWFRPHVGXULQJWKHLQWHUYHQWLRQDQGH[WHQGHG ‡ /HJUR 56HWDO'LDJQRVLVDQG7UHDWHPHQW RI3RO\F\VWLF
SRVWVWRSSLQJSKDVHVRIWKH:RPHQ
V+HDOWK,QLWLDWLYHUDQGRPL]HG 2YDULDQ6\QGURPH$Q(QGRFULQH6RFLHW\&OLQLFDO3UDFWLFH
WULDOV-$0$   *XLGHOLQH-&(0
‡ +RGLV+10DFN:-9DVFXODU(IIHFWVRI(DUO\YHUVXV/DWH ‡ 1HOVRQ/0&OLQLFDOSUDFWLFH3ULPDU\RYDULDQLQVXIILFLHQF\
3RVWPHQRSDXVDO7UHDWPHQWZLWK(VWUDGLRO1(QJO-0HG 1(-0
  
‡ %RDUGPDQ+0+DUWOH\/HWDO+RUPRQHWKHUDS\IRUSUHYHQWLQJ
FDUGLRYDVFXODUGLVHDVHLQSRVWPHQRSDXVDOZRPHQ&RFKUDQH
'DWDEDVH6\VW5HY&'

%5,*+$0$1' +DUYDUG
:20(1¶6+263,7$/
0HGLFDO6FKRRO
^ůĞĞƉƉŶĞĂ͗
ŝĂŐŶŽƐŝƐΘdƌĞĂƚŵĞŶƚ
/ŶƚĞŶƐŝǀĞZĞǀŝĞǁŽĨ/ŶƚĞƌŶĂůDĞĚŝĐŝŶĞ
InterstitialLungDisease >ĂǁƌĞŶĐĞ:͘ƉƐƚĞŝŶ͕D
ƐƐŝƐŝĂŶƚůŝŶŝĐŝƌĞĐƚŽƌ
ŝǀŝƐŝŽŶŽĨ^ůĞĞƉĂŶĚŝƌĐĂĚŝĂŶŝƐŽƌĚĞƌƐ͕
HilaryJ.Goldberg,MD,MPH ĞƉĂƌƚŵĞŶƚŽĨDĞĚŝĐŝŶĞ
DivisionofPulmonaryandCriticalCareMedicine ƌŝŐŚĂŵĂŶĚtŽŵĞŶ͛Ɛ,ŽƐƉŝƚĂů
BrighamandWomen’sHospital /ŶƐƚƌƵĐƚŽƌŝŶDĞĚŝĐŝŶĞ
,ĂƌǀĂƌĚDĞĚŝĐĂů^ĐŚŽŽů

5HIHUHQFHV References
7UDYLV:'HWDO³$Q2IILFLDO$PHULFDQ7KRUDFLF6RFLHW\(XURSHDQ
5HVSLUDWRU\6RFLHW\6WDWHPHQW8SGDWHRIWKH,QWHUQDWLRQDO „ Epstein L et al. Clinical guideline for the evaluation,
0XOWLGLVFLSOLQDU\&ODVVLILFDWLRQRIWKH,GLRSDWKLF,QWHUVWLWLDO3QHXPRQLDV´ management and long-term care of obstructive sleep apnea in
$P-5HVSLU&ULW&DUH0HG9RO,VVSS± adults. J Clin Sleep Med 2009;5:263-76.
„ Collop et al. Clinical guideline for the use of unattended
&ROODUG+5HWDO³$FXWH([DFHUEDWLRQRI,GLRSDWKLF3XOPRQDU\)LEURVLV
$Q,QWHUQDWLRQDO:RUNLQJ*URXS5HSRUW´$P-5HVSLU&ULW&DUH0HG9RO
portable monitors in the diagnosis of obstructive sleep apnea in
,VVSS±
adult patients. J Clin Sleep Med 2007;3:737-47.
„ White DP. Pathogenesis of obstructive and central sleep apnea.
)UHHPHU0 .LQJ7(-U7KH$&&(666WXG\&KDUDFWHUL]DWLRQRI Am J Respir Crit Care Med. 2005;172:1363-70.
6DUFRLGRVLVLQWKH8QLWHG6WDWHV$PHULFDQ-RXUQDORI5HVSLUDWRU\DQG „ Young T et al. The occurrence of sleep-disordered breathing
&ULWLFDO&DUH0HGLFLQHYROQRSS among middle-aged adults. N Engl J Med 1993;328:1230–35
9DOH\UH'3UDVVH$1XQHV+HWDO³6DUFRLGRVLV´/DQFHW9RO „ Marin et al Long-term cardiovascular outcomes in men with
,VVSS obstructive sleep apnoea-hypopnoea with or without treatment
0DGLVRQ-0³+\SHUVHQVLWLYLW\3QHXPRQLWLV&OLQLFDO3HUVSHFWLYHV´
with continuous positive airway pressure: an observational study.
$UFKLYHVRI3DWKRORJLFDQG/DERUDWRU\0HGLFLQHYROSS Lancet 2005; 365: 1046–53
WůĞƵƌĂůĨĨƵƐŝŽŶƐ͗ĂĐĂƐĞͲ WƵůŵŽŶĂƌLJ&ƵŶĐƚŝŽŶdĞƐƚŝŶŐ
ďĂƐĞĚƌĞǀŝĞǁ ^ĐŽƚƚ^ĐŚŝƐƐĞů͕D͕WŚ
^ĐŽƚƚ^ĐŚŝƐƐĞů͕D͕WŚ ŚŝĞĨ͕ĞƉĂƌƚŵĞŶƚŽĨDĞĚŝĐŝŶĞ
ŚŝĞĨ͕ĞƉĂƌƚŵĞŶƚŽĨDĞĚŝĐŝŶĞ ƌŝŐŚĂŵĂŶĚtŽŵĞŶ͛Ɛ&ĂƵůŬŶĞƌ,ŽƐƉŝƚĂů
ƌŝŐŚĂŵĂŶĚtŽŵĞŶ͛Ɛ&ĂƵůŬŶĞƌ,ŽƐƉŝƚĂů
ŝǀŝƐŝŽŶŽĨWƵůŵŽŶĂƌLJĂŶĚƌŝƚŝĐĂůĂƌĞDĞĚŝĐŝŶĞ
ŝǀŝƐŝŽŶŽĨWƵůŵŽŶĂƌLJĂŶĚƌŝƚŝĐĂůĂƌĞDĞĚŝĐŝŶĞ ƌŝŐŚĂŵĂŶĚtŽŵĞŶ͛Ɛ,ŽƐƉŝƚĂů
ƌŝŐŚĂŵĂŶĚtŽŵĞŶ͛Ɛ,ŽƐƉŝƚĂů
ƐƐŝƐƚĂŶƚWƌŽĨĞƐƐŽƌŽĨDĞĚŝĐŝŶĞ
ƐƐŝƐƚĂŶƚWƌŽĨĞƐƐŽƌŽĨDĞĚŝĐŝŶĞ
,ĂƌǀĂƌĚDĞĚŝĐĂů^ĐŚŽŽů ,ĂƌǀĂƌĚDĞĚŝĐĂů^ĐŚŽŽů

&ƵƌƚŚĞƌZĞĂĚŝŶŐ 6HOHFWHG5HIHUHQFHV
ϭ͘ >ŝŐŚƚ͕Zt͘WůĞƵƌĂůĨĨƵƐŝŽŶ͘EŶŐů:DĞĚ ϮϬϬϮ͖ϯϰϲ͗ ‡ 0LOOHU05HWDO$76(567DVN)RUFHVWDQGDUGL]DWLRQRIOXQJ
ϭϵϳϭ IXQFWLRQWHVWLQJEur Respir JS

Ϯ͘ >ŝŐŚƚ͕Zt͘dŚĞ>ŝŐŚƚƌŝƚĞƌŝĂdŚĞĞŐŝŶŶŝŶŐĂŶĚ
‡ 3HOOHJULQR5HWDO$76(567DVN)RUFHLQWHUSUHWDWLYHVWUDWHJLHVIRU
tŚLJƚŚĞLJĂƌĞhƐĞĨƵůϰϬzĞĂƌƐ>ĂƚĞƌ͘ůŝŶŚĞƐƚDĞĚ OXQJIXQFWLRQWHVWVEur Respir JS
ϮϬϭϯ͖ϯϰ͗ϮϭͲϮϲ
ϯ͘ ,ĞĨĨŶĞƌ͕:͘ŝƐĐƌŝŵŝŶĂƚŝŶŐĞƚǁĞĞŶdƌĂŶƐƵĚĂƚĞƐ ‡ 0DF,QW\UH1HWDO$76(567DVN)RUFHVWDQGDUGL]DWLRQRIWKH
ĂŶĚdžƵĚĂƚĞƐ͘ůŝŶŚĞƐƚDĞĚ ϮϬϬϲ͖Ϯϳ͗Ϯϰϭ VLQJOHEUHDWKGHWHUPLQDWLRQRIFDUERQPRQR[LGHXSWDNHLQWKHOXQJ
Eur Respir JS
ϰ͘ DĐ'ƌĂƚŚĂŶĚŶĚĞƌƐŽŶW͘ŝĂŐŶŽƐŝƐŽĨWůĞƵƌĂů
ĨĨƵƐŝŽŶ͗ƐLJƐƚĞŵĂƚŝĐĂƉƉƌŽĂĐŚ͘ŵ:ƌŝƚŝĐĂůĂƌĞ ‡ -RQHV5/HWDO7KHHIIHFWVRIERG\PDVVLQGH[RQOXQJYROXPHV
ϮϬϭϭ͖ϮϬ͗ϭϭϵ ChestS± 
ϱ͘ WŽƌĐĞů:ŽƐĞD͘WĞĂƌůƐĂŶĚŵLJƚŚƐŝŶƉůĞƵƌĂůĨůƵŝĚ
ĂŶĂůLJƐŝƐ͘ZĞƐƉŝƌŽůŽŐLJϮϬϭϭ͖ϭϲ͗ϰϰ ‡ (VVDW0HWDO)UDFWLRQDOH[KDOHGQLWULFR[LGHIRUWKHPDQDJHPHQWRI
DVWKPDLQDGXOWVDV\VWHPDWLFUHYLHZEur Respir JHSXE

 

(YDOXDWLRQRIWKHG\VSQHLF
SDWLHQW
'DYLG06\VWURP0'
$VVRFLDWH3K\VLFLDQ
3XOPRQDU\DQG&ULWLFDO&DUH0HGLFLQH
%ULJKDPDQG:RPHQ¶V+RVSLWDO
,ĞĂĚĂĐŚĞ
$VVW3URI
+DUYDUG0HGLFDO6FKRRO
ĂƌŽůLJŶĞƌŶƐƚĞŝŶD͘͘&,^
ƐƐƚ͘WƌŽĨĞƐƐŽƌŽĨEĞƵƌŽůŽŐLJ͕,ĂƌǀĂƌĚDĞĚŝĐĂů^ĐŚŽŽů
ƐƐŽĐŝĂƚĞEĞƵƌŽůŽŐŝƐƚ͕ƌŝŐŚĂŵĂŶĚtŽŵĞŶ͛Ɛ,ŽƐƉŝƚĂů

5HIHUHQFHV tĞďƐŝƚĞƐ
¾ 0RUULV 0-HWDO 9RFDOFRUGG\VIXQFWLRQHWLRORJLHVDQGWUHDWPHQW &OLQ
3XOPRQDU\0HG ±
¾ 2OGKDP:0/HZLV*'2SRWRZVN\$5:D[PDQ$%6\VWURP '0
8QH[SODLQHGH[HUWLRQDOG\VSQHDFDXVHGE\ORZYHQWULFXODUILOOLQJ
‡ ŚƚƚƉ͗ͬͬǁǁǁ͘ŚĞĂĚĂĐŚĞƐ͘ŽƌŐͬƉĚĨͬD/^͘ƉĚĨ
SUHVVXUHVUHVXOWVIURPFOLQLFDOLQYDVLYHFDUGLRSXOPRQDU\H[HUFLVHWHVWLQJ
3XOP &LUF ‡ ŚƚƚƉ͗ͬͬǁǁǁ͘ĂŵĞƌŝĐĂŶŚĞĂĚĂĐŚĞƐŽĐŝĞƚLJ͘ŽƌŐͬ
¾ 7DLYDVVDOR 7HWDO7KHVSHFWUXPRIH[HUFLVHWROHUDQFHLQPLWRFKRQGULDO

¾
P\RSDWKLHVDVWXG\RISDWLHQWV%UDLQ
)XQFWLRQDOLPSDFWRIH[HUFLVHSXOPRQDU\K\SHUWHQVLRQLQSDWLHQWVZLWK
‡ ŚƚƚƉ͗ͬͬǁǁǁ͘ŚĞĂĚĂĐŚĞƐ͘ŽƌŐͬ
ERUGHUOLQHUHVWLQJSXOPRQDU\DUWHU\SUHVVXUH2OLYHLUD5.))DULD8UELQD
00DURQ%$6DQWRV0:D[PDQ$%6\VWURP'03XOP &LUF ‡ ŚƚƚƉ͗ͬͬǁǁǁ͘ĂĐŚĞŶĞƚ͘ŽƌŐͬ
'2,
¾ 6HJUHUD6$/DZOHU/2SRWRZVN\$56\VWURP'0:D[PDQ$%2SHQ
ODEHOVWXG\RIDPEULVHQWDQLQSDWLHQWVZLWKH[HUFLVHSXOPRQDU\
‡ ŚƚƚƉ͗ͬͬŝƚƵŶĞƐ͘ĂƉƉůĞ͘ĐŽŵͬƵƐͬĂƉƉͬŚĞĂĚĂĐŚĞͲ
K\SHUWHQVLRQ3XOP &LUF  ±
¾ :+XDQJ65HVFK5.)2OLYHLUD%$&RFNULOO'06\VWURP$%
ĚŝĂƌLJͲůŝƚĞͬŝĚϯϬϵϮϮϳϰϲϯ͍ŵƚсϴ
:D[PDQ ,QYDVLYHFDUGLRSXOPRQDU\H[HUFLVHWHVWLQJLQWKHHYDOXDWLRQRI
XQH[SODLQHGG\VSQHD,QVLJKWVIURPDPXOWLGLVFLSOLQDU\G\VSQHDFHQWHU ‡ ŚƚƚƉ͗ͬͬǁǁǁ͘ŶĐďŝ͘Ŷůŵ͘ŶŝŚ͘ŐŽǀͬƉƵďŵĞĚͬϮϮϲϳϭ
(XU -3UHY &DUG  ±'2,
¾ 0DURQ%$&RFNULOO%$:D[PDQ$%6\VWURP'07KHLQYDVLYH ϳϭϰ
FDUGLRSXOPRQDU\H[HUFLVHWHVW&LUFXODWLRQ 
^ĞŝnjƵƌĞŝƐŽƌĚĞƌƐ
KďĞƐŝƚLJDĂŶĂŐĞŵĞŶƚ
dƌĂĐĞLJ͘DŝůůŝŐĂŶ͕D͕D^͕&E
ŝƐƚŝŶŐƵŝƐŚĞĚůŝŶŝĐŝĂŶ
WƐLJĐŚŝĂƚƌLJKǀĞƌǀŝĞǁ
sŝĐĞŚĂŝƌĨŽƌĚƵĐĂƚŝŽŶ
ĚǁĂƌĚƌŽŵĨŝĞůĚƉŝůĞƉƐLJĞŶƚĞƌ
&ůŽƌĞŶĐŝĂ,ĂůƉĞƌŝŶ͕D͕DD^Đ
ĞƉĂƌƚŵĞŶƚŽĨEĞƵƌŽůŽŐLJ ŽͲŝƌĞĐƚŽƌ͕ĞŶƚĞƌĨŽƌtĞŝŐŚƚDĂŶĂŐĞŵĞŶƚĂŶĚDĞƚĂďŽůŝĐ^ƵƌŐĞƌLJ
ƌŝŐŚĂŵĂŶĚtŽŵĞŶ͛Ɛ,ŽƐƉŝƚĂů ŚŝĞĨ͕ŝǀŝƐŝŽŶŽĨŶĚŽĐƌŝŶŽůŽŐLJ͕ƌŝŐŚĂŵĂŶĚtŽŵĞŶ͛Ɛ&ĂƵůŬŶĞƌ,ŽƐƉŝƚĂů
ƐƐŝƐƚĂŶƚWƌŽĨĞƐƐŽƌŽĨEĞƵƌŽůŽŐLJ ŝǀŝƐŝŽŶŽĨŶĚŽĐƌŝŶŽůŽŐLJ͕ƌŝŐŚĂŵĂŶĚtŽŵĞŶ͛Ɛ,ŽƐƉŝƚĂů
,ĂƌǀĂƌĚDĞĚŝĐĂů^ĐŚŽŽů
/ŶƐƚƌƵĐƚŽƌŝŶDĞĚŝĐŝŶĞ͕,ĂƌǀĂƌĚDĞĚŝĐĂů^ĐŚŽŽů

ZĞĨĞƌĞŶĐĞƐ
ϭ͘ :ĞŶƐĞŶD͕ĞƚĂů͘ϮϬϭϯ,ͬͬdK^'ƵŝĚĞůŝŶĞĨŽƌƚŚĞDĂŶĂŐĞŵĞŶƚŽĨ
KǀĞƌǁĞŝŐŚƚĂŶĚKďĞƐŝƚLJŝŶĚƵůƚƐ͘ŝƌĐƵůĂƚŝŽŶ͘ϮϬϭϰ:ƵŶϮϰ͖ϭϮϵ;Ϯϱ^ƵƉƉů
‡ ƌŽƉŚLJ͕'ƌĞƚĐŚĞŶD͕͘ĞƚĂů͘Η'ƵŝĚĞůŝŶĞƐĨŽƌƚŚĞĞǀĂůƵĂƚŝŽŶĂŶĚ ϮͿ͗^ϭϬϮͲϯϴ͘
ŵĂŶĂŐĞŵĞŶƚŽĨƐƚĂƚƵƐĞƉŝůĞƉƚŝĐƵƐ͘Η EĞƵƌŽĐƌŝƚŝĐĂůĐĂƌĞ ϭϳ͘ϭ;ϮϬϭϮͿ͗
ϯͲϮϯ͘ Ϯ͘ ƉŽǀŝĂŶ D͕ĞƚĂů͘WŚĂƌŵĂĐŽůŽŐŝĐĂůDĂŶĂŐĞŵĞŶƚŽĨKďĞƐŝƚLJ͗ŶŶĚŽĐƌŝŶĞ
^ŽĐŝĞƚLJůŝŶŝĐĂůWƌĂĐƚŝĐĞ'ƵŝĚĞůŝŶĞ͘:ůŝŶ ŶĚŽĐƌŝŶŽů DĞƚĂď͘ϮϬϭϱ
‡ <ƌƵŵŚŽůnj͕͕͘ĞƚĂů͘ΗWƌĂĐƚŝĐĞWĂƌĂŵĞƚĞƌ͗ǀĂůƵĂƚŝŶŐĂŶĂƉƉĂƌĞŶƚ &Ğď͖ϭϬϬ;ϮͿ͗ϯϰϮͲϲϮ͘
ƵŶƉƌŽǀŽŬĞĚĨŝƌƐƚƐĞŝnjƵƌĞŝŶĂĚƵůƚƐ;ĂŶĞǀŝĚĞŶĐĞͲďĂƐĞĚƌĞǀŝĞǁͿZĞƉŽƌƚ
ŽĨƚŚĞYƵĂůŝƚLJ^ƚĂŶĚĂƌĚƐ^ƵďĐŽŵŵŝƚƚĞĞŽĨƚŚĞŵĞƌŝĐĂŶĐĂĚĞŵLJŽĨ ϯ͘ 'ĂƌǀĞLJtd͕ĞƚĂů͘ŵĞƌŝĐĂŶƐƐŽĐŝĂƚŝŽŶŽĨůŝŶŝĐĂůŶĚŽĐƌŝŶŽůŽŐŝƐƚƐ ĂŶĚ
EĞƵƌŽůŽŐLJĂŶĚƚŚĞŵĞƌŝĐĂŶƉŝůĞƉƐLJ^ŽĐŝĞƚLJ͘Η EĞƵƌŽůŽŐLJ ϲϵ͘Ϯϭ ŵĞƌŝĐĂŶŽůůĞŐĞŽĨŶĚŽĐƌŝŶŽůŽŐLJĐŽŵƉƌĞŚĞŶƐŝǀĞĐůŝŶŝĐĂůƉƌĂĐƚŝĐĞ
;ϮϬϬϳͿ͗ϭϵϵϲͲϮϬϬϳ͘ ŐƵŝĚĞůŝŶĞƐ ĨŽƌŵĞĚŝĐĂůĐĂƌĞŽĨƉĂƚŝĞŶƚƐǁŝƚŚŽďĞƐŝƚLJ͘ŶĚŽĐƌ WƌĂĐƚ͘
ϮϬϭϲ͖ϮϮ;^ƵƉƉů ϯͿ͗ϭͲϮϬϯ
‡ DĞĂĚŽƌ<:͘dŽ^ƚŽƉŽƌEŽƚƚŽ^ƚŽƉƚŚĞ͍ ƉŝůĞƉƐLJƵƌƌĞŶƚƐ͘
ϮϬϬϴ͖ϴ;ϰͿ͗ϵϬͲϵϭ͘ĚŽŝ͗ϭϬ͘ϭϭϭϭͬũ͘ϭϱϯϱͲϳϱϭϭ͘ϮϬϬϴ͘ϬϬϮϱϬ͘ ϰ͘ DĞĐŚĂŶŝŬ͕:/͕ĞƚĂů͘ůŝŶŝĐĂůƉƌĂĐƚŝĐĞŐƵŝĚĞůŝŶĞƐĨŽƌƚŚĞƉĞƌŝŽƉĞƌĂƚŝǀĞŶƵƚƌŝƚŝŽŶĂů͕
ŵĞƚĂďŽůŝĐ͕ĂŶĚŶŽŶƐƵƌŐŝĐĂůƐƵƉƉŽƌƚŽĨƚŚĞďĂƌŝĂƚƌŝĐƐƵƌŐĞƌLJƉĂƚŝĞŶƚͲͲϮϬϭϯ
‡ >ĂdžĞƌ͕<ĞŶŶĞƚŚ͕͘ĞƚĂů͘ΗdŚĞĐŽŶƐĞƋƵĞŶĐĞƐŽĨƌĞĨƌĂĐƚŽƌLJĞƉŝůĞƉƐLJ ƵƉĚĂƚĞ͗ĐŽƐƉŽŶƐŽƌĞĚďLJŵĞƌŝĐĂŶƐƐŽĐŝĂƚŝŽŶŽĨůŝŶŝĐĂůŶĚŽĐƌŝŶŽůŽŐŝƐƚƐ͕dŚĞ
ĂŶĚŝƚƐƚƌĞĂƚŵĞŶƚ͘Η ƉŝůĞƉƐLJΘĞŚĂǀŝŽƌ ϯϳ;ϮϬϭϰͿ͗ϱϵͲϳϬ͘ KďĞƐŝƚLJ^ŽĐŝĞƚLJ͕ĂŶĚŵĞƌŝĐĂŶ^ŽĐŝĞƚLJĨŽƌDĞƚĂďŽůŝĐΘĂŵƉ͖ĂƌŝĂƚƌŝĐ^ƵƌŐĞƌLJ͘
‡ WƵŐŚ͕DĂƌLJ:Ž͕ĂŶĚ<ĂƚŚĂƌŝŶĞ<͘DĐDŝůůĂŶ͘Η'ƵŝĚĞůŝŶĞƐĂŶĚYƵĂůŝƚLJ KďĞƐŝƚLJ;^ŝůǀĞƌ^ƉƌŝŶŐͿ͘ϮϬϭϯDĂƌ͖Ϯϭ^ƵƉƉů ϭ͗^ϭͲϮϳ
^ƚĂŶĚĂƌĚƐĨŽƌĚƵůƚƐǁŝƚŚƉŝůĞƉƐLJ͘Η EĞƵƌŽůŽŐŝĐĐůŝŶŝĐƐ ϯϰ͘Ϯ;ϮϬϭϲͿ͗ ϱ͘ ,ĞLJŵƐĨŝĞůĚ ^ĂŶĚtĂĚĚĞŶ d͘DĞĐŚĂŶŝƐŵƐ͕WĂƚŚŽƉŚLJƐŝŽůŽŐLJ͕ĂŶĚ
ϯϭϯͲϯϮϱ͘ DĂŶĂŐĞŵĞŶƚŽĨKďĞƐŝƚLJ͘EŶŐů :DĞĚϮϬϭϳ͖ϯϳϲ͗ϮϱϰͲϮϲϲ

%,267$7,67,&6%2$5'5(9,(:
ŶĚŽĨ>ŝĨĞ

>ŝƐĂ^ŽůĞLJŵĂŶŝ>ĞŚŵĂŶŶ͕D͕WŚ͕D^Đ
sEĂƚŝŽŶĂůĞŶƚĞƌĨŽƌƚŚŝĐƐŝŶ,ĞĂůƚŚĂƌĞ -8/,((%85,1*6F'
ƐƐŽĐŝĂƚĞWƌŽĨĞƐƐŽƌŽĨDĞĚŝĐŝŶĞ 6HQLRU(SLGHPLRORJLVW
'LYLVLRQRI3UHYHQWLYH0HGLFLQH'HSDUWPHQWRI0HGLFLQH
,ĂƌǀĂƌĚDĞĚŝĐĂů^ĐŚŽŽů
%ULJKDPDQG:RPHQ¶V+RVSLWDO
>njƐ>ĞŚŵĂŶŶΛŐŵĂŝů͘ĐŽŵ 3URIHVVRURI0HGLFLQH
+DUYDUG0HGLFDO6FKRRO

ZĞĨĞƌĞŶĐĞƐ 5HIHUHQFHV
ϭ͘ LJŝŶŐŝŶŵĞƌŝĐĂ͗/ŵƉƌŽǀŝŶŐYƵĂůŝƚLJĂŶĚ,ŽŶŽƌŝŶŐ/ŶĚŝǀŝĚƵĂů
WƌĞĨĞƌĞŶĐĞƐEĞĂƌƚŚĞŶĚŽĨ>ŝĨĞ͘/ŶƐƚŝƚƵƚĞŽĨDĞĚŝĐŝŶĞ͘ϮϬϭϰ͘  )OHWFKHU5+)OHWFKHU6:&OLQLFDO(SLGHPLRORJ\7KH
Ϯ͘ ůĂLJƚŽŶ:D͕,ĂŶĐŽĐŬ<D͕ƵƚŽǁ WE͕dĂƚƚĞƌƐĂůů D,͕ƵƌƌŽǁ (VVHQWLDOVWK (GLWLRQ/LSSLQFRWW:LOOLDPVDQG
͕ĚůĞƌ:͕ĞƚĂů͘ůŝŶŝĐĂůƉƌĂĐƚŝĐĞŐƵŝĚĞůŝŶĞƐĨŽƌ :LONLQV
ĐŽŵŵƵŶŝĐĂƚŝŶŐƉƌŽŐŶŽƐŝƐĂŶĚĞŶĚͲŽĨͲůŝĨĞŝƐƐƵĞƐǁŝƚŚĂĚƵůƚƐ
ŝŶƚŚĞĂĚǀĂŶĐĞĚƐƚĂŐĞƐŽĨĂůŝĨĞͲůŝŵŝƚŝŶŐŝůůŶĞƐƐ͕ĂŶĚƚŚĞŝƌ  *ODQW]6$3ULPHURI%LRVWDWLVWLFVWK (GLWLRQ0F*UDZ
ĐĂƌĞŐŝǀĞƌƐ͘DĞĚ:ƵƐƚ͘ϮϬϬϳ͖ϭϴϲ;ϭϮ^ƵƉƉůͿ͗^ϳϳ͕^ϵ͕^ϴϯͲϭϬϴ͘ +LOO
ϯ͘ ŝŶŐĨŝĞůĚ >͕<ĂLJƐĞƌ :͘/ŶƚĞŐƌĂƚŝŶŐĚǀĂŶĐĞĂƌĞWůĂŶŶŝŶŐ  +XOOH\6%&XPPLQJV65%URZQHU:6*UDG\'*
/ŶƚŽWƌĂĐƚŝĐĞ͘,^dϮϬϭϳ͖ϭϱϭ;ϲͿ͗ϭϯϴϳͲϭϯϵϯ͘
1HZPDQ7%'HVLJQLQJ&OLQLFDO5HVHDUFKUG (GLWLRQ
ϰ͘ 'ĞŚůďĂĐŚ d,ĞƚĂů͘ŽĚĞƐƚĂƚƵƐŽƌĚĞƌƐĂŶĚŐŽĂůƐŽĨĐĂƌĞŝŶƚŚĞ :ROWHUV.OXZHU
ŵĞĚŝĐĂů/h͘ŚĞƐƚϮϬϭϭ͖ϭϯϵϰ͗ϴϬϮͲϴϬϵ͘
ϱ͘ ĂůĂů ^͕ƌƵĞƌĂ ͘ŶĚͲŽĨͲ>ŝĨĞĂƌĞDĂƚƚĞƌƐ͗WĂůůŝĂƚŝǀĞĂŶĐĞƌ
ĂƌĞZĞƐƵůƚƐŝŶĞƚƚĞƌĂƌĞĂŶĚ>ŽǁĞƌŽƐƚ͘dŚĞKŶĐŽůŽŐŝƐƚ͘
ϮϬϭϳ͖ϮϮ͗ϯϲϭͲϯϲϴ͘


0RUQLQJ5HSRUW
'ĞŶĞƌĂů/ŶƚĞƌŶĂůDĞĚŝĐŝŶĞ
ĂƐĞƐĂŶĚYƵĞƐƚŝŽŶƐ
0DULD$<LDODPDV0' >ŽƌŝtŝǀŝŽƚƚdŝƐŚůĞƌ͕D
sWDĞĚŝĐĂůĨĨĂŝƌƐ
ŽŵŵŽŶǁĞĂůƚŚĂƌĞůůŝĂŶĐĞ
ŝǀŝƐŝŽŶŽĨ'ĞŶĞƌĂůDĞĚŝĐŝŶĞ
$VVRFLDWH3URJUDP'LUHFWRU,QWHUQDO0HGLFLQH5HVLGHQF\ ƌŝŐŚĂŵĂŶĚtŽŵĞŶ͛Ɛ,ŽƐƉŝƚĂů
'HSDUWPHQWRI0HGLFLQH%ULJKDPDQG:RPHQ¶V+RVSLWDO ƐƐŝƐƚĂŶƚWƌŽĨĞƐƐŽƌŽĨDĞĚŝĐŝŶĞ͕,ĂƌǀĂƌĚDĞĚŝĐĂů^ĐŚŽŽů

$VVLVWDQW3URIHVVRURI0HGLFLQH+DUYDUG0HGLFDO6FKRRO

5HIHUHQFHV ZĞƐŽƵƌĐĞƐĨŽƌdƌĂŶƐŐĞŶĚĞƌĂƌĞ

‡ %KDVLQ6HWDO7HVWRVWHURQHWKHUDS\LQPHQZLWKDQGURJHQGHILFLHQF\V\QGURPHVDQ ‡ ŚƚƚƉƐ͗ͬͬŽŝ͘ŵŐŚ͘ŚĂƌǀĂƌĚ͘ĞĚƵͬƉĐŽŝͬƉƌŝŵĂƌLJͺĐĂƌĞͺŐƵŝĚĞůŝŶĞƐͬdƌĂŶƐŐĞŶĚ
(QGRFULQH6RFLHW\FOLQLFDOSUDFWLFHJXLGHOLQH-&OLQ(QGRFULQRO0HWDE Ğƌ͘ĂƐƉηƐƵƌŐ
‡ h^&ĞŶƚĞƌŽĨdžĐĞůůĞŶĐĞĨŽƌdƌĂŶƐŐĞŶĚĞƌ,ĞĂůƚŚ
‡ %UXJDGD-HWDO5LJKWELQGOHEUDQFKEORFNDQG6WVHJPHQWHOHYDWLRQLQOHDGHV9
WKURXJK9DPDUNHUIRUVXGGHQGHDWKLQSDWLHQWVZLWKRXUGHPRQVWUDEOHVWXFWXUDO ‡ EĂƚŝŽŶĂů>'d,ĞĂůƚŚĚƵĐĂƚŝŽŶĞŶƚĞƌ
KHDUWGLVVHDVH&LUFXODWLRQ

‡ 'DQLHOV*+$PLRGDURQHLQGXFHGWK\URWR[LFRVLV-&OLQ(QGRFULQRO0HWDE

‡ .OXER*ZLH]G]LQVND-:DUWRIVN\/7K\URLGHPHUJHQFLHV0HG&OLQ1RUK$P


‡ 6KDSLUR-&OLQLFDOSUDFWLFHKDUORVVLQZRPHQ1(-0

WƐLJĐŚŝĂƚƌLJKǀĞƌǀŝĞǁ
,LJƉĞƌůŝƉŝĚĞŵŝĂ
dƌĞĂƚŝŶŐKƉŝŽŝĚ hƐĞŝƐŽƌĚĞƌ
^ĂƌĂŚ͘tĂŬĞŵĂŶ͕D͕&^D
DĞĚŝĐĂůŝƌĞĐƚŽƌ͕DĂƐƐ'ĞŶĞƌĂů,ŽƐƉŝƚĂů^ƵďƐƚĂŶĐĞhƐĞŝƐŽƌĚĞƌ/ŶŝƚŝĂƚŝǀĞ ^ĐŽƚƚ<ŝŶůĂLJ͕D^͕WŚ
WƌŽŐƌĂŵŝƌĞĐƚŽƌ͕DĂƐƐ'ĞŶĞƌĂůĚĚŝĐƚŝŽŶDĞĚŝĐŝŶĞ&ĞůůŽǁƐŚŝƉ
ůŝŶŝĐĂůŽͲ>ĞĂĚ͕WĂƌƚŶĞƌƐ,ĞĂůƚŚĐĂƌĞ^ƵďƐƚĂŶĐĞhƐĞŝƐŽƌĚĞƌ/ŶŝƚŝĂƚŝǀĞ
DĞĚŝĐĂůŝƌĞĐƚŽƌ͕Z/DĂƐƐĂĐŚƵƐĞƚƚƐ ƐƐŽĐŝĂƚĞŚŝĞĨĂƌĚŝŽǀĂƐĐƵůĂƌŝǀŝƐŝŽŶΘŝƌĞĐƚŽƌŽĨsĂƐĐƵůĂƌ
ƐƐŝƐƚĂŶƚWƌŽĨĞƐƐŽƌŽĨDĞĚŝĐŝŶĞ͕,ĂƌǀĂƌĚDĞĚŝĐĂů^ĐŚŽŽů DĞĚŝĐŝŶĞ͕sŽƐƚŽŶ,ĞĂůƚŚĐĂƌĞ^LJƐƚĞŵ
ƐƐŽĐŝĂƚĞWŚLJƐŝĐŝĂŶ͕ƌŝŐŚĂŵΘtŽŵĞŶ¶Ɛ,ŽƐƉŝƚĂů
ƐƐŽĐŝĂƚĞWƌŽĨĞƐƐŽƌŝŶDĞĚŝĐŝŶĞ͕,ĂƌǀĂƌĚDĞĚŝĐĂů^ĐŚŽŽů

ZĞĨĞƌĞŶĐĞƐ ZĞĨĞƌĞŶĐĞƐ
‡ ĂƌƌŝĞƌŝ ĞƚĂů͘ůŝŶŝĐĂů/ŶĨĞĐƚŝŽƵƐŝƐĞĂƐĞƐ͕sŽůƵŵĞϰϯ͕/ƐƐƵĞ^ƵƉƉůĞŵĞŶƚͺϰ͕ϭϱĞĐĞŵďĞƌϮϬϬϲ͕^ϭϵϳʹ^Ϯϭϱ
‡ ŚƵƚƵĂƉĞ ĞƚĂů͘ŵ:ƌƵŐůĐŽŚŽůďƵƐĞ͘ ϮϬϬϭ&Ğď͖Ϯϳ;ϭͿ͗ϭϵͲϰϰ͘
‡ ŝĐĞƌŽEŶŐů:DĞĚϮϬϭϱ͖ϯϳϯ͗ϭϳϴϵͲϭϳϵϬ
‡ ůĂƌŬZĞƚĂů͘:^ƵďƐƚ ďƵƐĞ dƌĞĂƚ͘ ϮϬϭϱKĐƚ͖ϱϳ͗ϳϱͲϴϬ
‡ ZĞǀŝĞǁŽĨŝĞƚƐĂŶĚŝĞƚ'ƵŝĚĞůŝŶĞƐ͘DŽnjĂĨĨĂƌŝĂŶ ͘ŝƌĐƵůĂƚŝŽŶϮϬϭϲ͖ϭϯϯ͗ϭϴϳ
‡ ΖKŶŽĨƌŝŽĞƚĂů͘:D ϮϬϭϱƉƌϮϴ͖ϯϭϯ;ϭϲͿ͗ϭϲϯϲͲϰϰ ‡ WZ/D͘DĞĚŝƚĞƌƌĂŶĞĂŶŝĞƚdƌŝĂů͘ƐƚƌƵĐŚ Z͕ĞƚĂů͘E:DϮϬϭϯ͖ϯϲϴ͗ϭϮϳϵ
‡ ƵƌŽƉĞĂŶDŽŶŝƚŽƌŝŶŐĞŶƚƌĞĨŽƌƌƵŐƐĂŶĚƌƵŐĚĚŝĐƚŝŽŶ;ϮϬϭϱͿ ‡ h^ŝĞƚĂƌLJ'ƵŝĚĞůŝŶĞƐĚǀŝƐŽƌLJ͘ŚƚƚƉ͗ͬͬǁǁǁ͘ŚĞĂůƚŚ͘ŐŽǀͬĚŝĞƚĂƌLJŐƵŝĚĞůŝŶĞƐͬϮϬϭϱͲ
‡ &ŝĞůůŝŶ ĞƚĂů͘ŵ:DĞĚϭϮϲ͗ϭϮϬϭϯ ƐĐŝĞŶƚŝĨŝĐͲƌĞƉŽƌƚͬϬϮͲĞdžĞĐƵƚŝǀĞͲƐƵŵŵĂƌLJ͘ĂƐƉ
‡ ,ƐĞƌ ĞƚĂů͘ĚĚŝĐƚŝŽŶ͘ϮϬϭϲƉƌ͖ϭϭϭ;ϰͿ͗ϲϵϱͲϳϬϱ͘
‡ EĞǁ,ͬ'ƵŝĚĞůŝŶĞƐ͗
‡ ,ƵƚĐŚŝŶƐŽŶĞƚĂů͘ŶŶ&Ăŵ DĞĚϮϬϭϱ͖ϭϯ͗ϮϯͲϮϲ͘
‡ <ĂŬŬŽ ĞƚĂů͘>ĂŶĐĞƚ͘ ϮϬϬϯ&ĞďϮϮ͖ϯϲϭ;ϵϯϱϴͿ͗ϲϲϮͲϴ
± ŚƚƚƉ͗ͬͬĐŝƌĐ͘ĂŚĂũŽƵƌŶĂůƐ͘ŽƌŐͬĐŽŶƚĞŶƚͬĞĂƌůLJͬϮϬϭϯͬϭϭͬϭϭͬϬϭ͘Đŝƌ͘ϬϬϬϬϰϯϳϳϯϴ͘ϲϯ
‡ <Ăƚnj:͘ϮϬϭϳ͘ŚƚƚƉƐ͗ͬͬǁǁǁ͘ŶLJƚŝŵĞƐ͘ĐŽŵͬŝŶƚĞƌĂĐƚŝǀĞͬϮϬϭϳͬϬϲͬϬϱͬƵƉƐŚŽƚͬŽƉŝŽŝĚͲĞƉŝĚĞŵŝĐͲĚƌƵŐͲŽǀĞƌĚŽƐĞͲĚĞĂƚŚƐͲĂƌĞͲƌŝƐŝŶŐͲ
ϴϱϯ͘ϳĂ͘ĐŝƚĂƚŝŽŶ
ĨĂƐƚĞƌͲƚŚĂŶͲĞǀĞƌ͘Śƚŵů͍ͺƌсϬ ± <ĞĂŶĞLJ :͕ĞƚĂů͘E:DϮϬϭϰ͖ϯϳϬ͗Ϯϳϱ
‡ >ĞĞ:ĞƚĂů͘dŚĞ>ĂŶĐĞƚ͕sŽůƵŵĞϯϵϭ͕/ƐƐƵĞϭϬϭϭϴ͕ϯϬϵͲ ϯϭϴ ‡ KůĚ'ƵŝĚĞůŝŶĞƐ
‡ >ĞŝďƐĐŚƵƚnj ĞƚĂů:D/ŶƚĞƌŶDĞĚ ϮϬϭϰƵŐ͖ϭϳϰ;ϴͿ͗ϭϯϲϵͲϳϲ
‡ >ŝŶŐt͘:EĞƵƌŽŝŵŵƵŶĞ WŚĂƌŵĂĐŽů ;ϮϬϭϲͿϭϭ͗ϯϵϰʹϰϬϬ
± dW///͘:D͘ϮϬϬϭ͖Ϯϴϱ͗ϮϰϴϲͲϮϰϴϳ͘'ƌƵŶĚLJ^͕ĞƚĂů͘ ŝƌĐƵůĂƚŝŽŶϮϬϬϰ͖ϭϭϬ͗
‡ DĂƚƚŝĐŬ ĞƚĂů͘ŽĐŚƌĂŶĞĂƚĂďĂƐĞŽĨ^LJƐƚĞŵĂƚŝĐZĞǀŝĞǁƐϮϬϬϵ͕/ƐƐƵĞϯ͘ƌƚ͘EŽ͗͘ϬϬϮϮϬϵ͘
ϮϮϳ
‡ DĐ>ĞůůĂŶ ĞƚĂů͕͘:D͕Ϯϴϰ͗ϭϲϴϵͲϭϲϵϱ͕ϮϬϬϬ͘ ‡ dĂůů͘E:DϮϬϬϲ͖ϯϱϰ͗ϭϯϭϬ
‡ DƵƌƚŚLJs͘ŚƚƚƉƐ͗ͬͬĂĚĚŝĐƚŝŽŶ͘ƐƵƌŐĞŽŶŐĞŶĞƌĂů͘ŐŽǀͬƐƵƌŐĞŽŶͲŐĞŶĞƌĂůƐͲƌĞƉŽƌƚ͘ƉĚĨ ‡ &KhZ/ZdƌŝĂůŽĨǀŽůŽĐƵŵĂď͘^ĂďĂƚŝŶĞ D^͕ĞƚĂů͘E:DϮϬϭϳ͖ϯϳϲ͗ϭϳϭϯ
‡ ^ĞĞƐĞƚĂů͘:D͘ϮϬϬϬ͖Ϯϴϯ;ϭϬͿ͗ϭϯϬϯͲϭϯϭϬ͘
‡ ^ĞƉŬŽǁŝƚnj <͘EŶŐů :DĞĚϮϬϬϭ͖ϯϰϰ͗ϭϳϲϰͲϭϳϳϮ
‡ ^ŚĂŶĂŚĂŶĞƚĂů͘:'ĞŶ/ŶƚĞƌŶDĞĚ͘ƵŐϮϬϭϬ͖Ϯϱ;ϴͿ͗ϴϬϯʹϴϬϴ
‡ ^ŽƌĚŽ ĞƚĂů͘D:ϮϬϭϳ͖ϯϱϳ͗ũϭϱϱϬ
‡ sŽůŬŽǁ EŶŐů:DĞĚ͘DĂLJϯϭ͕ϮϬϭϳK/͗ϭϬ͘ϭϬϱϲͬE:DƐƌϭϳϬϲϲϮϲ
‡ tĞŝƐƐĞƚĂů͘ϮϬϭϲ͘ŚƚƚƉƐ͗ͬͬǁǁǁ͘ŚĐƵƉͲƵƐ͘ĂŚƌƋ͘ŐŽǀͬƌĞƉŽƌƚƐͬƐƚĂƚďƌŝĞĨƐͬƐďϮϭϵͲKƉŝŽŝĚͲ,ŽƐƉŝƚĂůͲ^ƚĂLJƐͲͲsŝƐŝƚƐͲďLJͲ^ƚĂƚĞ͘ũƐƉ
‡ tĞŝƐƐĞƚĂů͘ƌƵŐůĐĞƉĞŶĚ͘ϮϬϭϱ͖ϭϱϬ͗ϭϭϮͲϵ͘
‡ tŽƌůĚ,ĞĂůƚŚKƌŐĂŶŝnjĂƚŝŽŶŚƚƚƉ͗ͬͬĂƉƉƐ͘ǁŚŽ͘ŝŶƚͬŝƌŝƐͬďŝƚƐƚƌĞĂŵͬϭϬϲϲϱͬϰϯϵϰϴͬϭͬϵϳϴϵϮϰϭϱϰϳϱϰϯͺĞŶŐ͘ƉĚĨ
8SGDWHLQ6HSVLV 3RSXODU7RSLFVIRUWKH%RDUGV

.DWKOHHQ-+DOH\0'
5HEHFFD0%DURQ0' $VVRFLDWH3K\VLFLDQ
$VVLVWDQW3URIHVVRURI0HGLFLQH+DUYDUG0HGLFDO6FKRRO 'HSDUWPHQWRI0HGLFLQH'LYLVLRQRI3XOPRQDU\DQG&ULWLFDO&DUH
$VVRFLDWH3K\VLFLDQ%ULJKDPDQG:RPHQ¶V+RVSLWDO %ULJKDPDQG:RPHQ¶V+RVSLWDO
3XOPRQDU\DQG&ULWLFDO&DUH'LYLVLRQ $VVLVWDQW3URIHVVRURI0HGLFLQH
+DUYDUG6FKRRORI0HGLFLQH

6HOHFWHG.H\5HIHUHQFHV 5HIHUHQFHV
„ 5KRGHV$HWDO6XUYLYLQJ6HSVLV&DPSDLJQ,QWHUQDWLRQDO*XLGHOLQHVIRU ‡ 0HGLFDO6WDWLVWLFV
0DQDJHPHQWRI6HSVLVDQG6HSWLF6KRFN&ULW &DUH0HG
± -$.QRWWQHUXVHWDO%ULWLVK0HGLFDO-RXUQDO
„ $QQDQH 'HWDO+\GURFRUWLVRQHSOXV)OXGURFRUWLVRQHIRUDGXOWVZLWKVHSWLF YROSS± 
VKRFN1(-0
„ 9HQNDWHVK %HWDO$GMXQFWLYHJOXFRUWLFRLG WKHUDS\LQSDWLHQWVZLWKVHSWLF ‡ $GYDQFH'LUHFWLYHVDQG0HGLFDO'HFLVLRQ
VKRFN1(-0 0DNLQJ
„ .KDQQD$HWDO$QJLRWHQVLQ,,IRUWKHWUHDWPHQWRIYDVRGLODWRU\VKRFN ± 562OLFN&KHVWYROSS± 
1(-0
„ %HQW]HU 3HWDO:LOOWKLVKHPRG\QDPLFDOO\XQVWDEOHSDWLHQWUHVSRQGWRD
‡ &KHFNSRLQW$VVRFLDWHG3QHXPRQLWLV
EROXVRILQWUDYHQRXVIOXLGV"-$0$ ± -1DLGRRHWDO-&OLQ2QFROYROSS
„ 6LQJHU0HWDO7KHUG LQWHUQDWLRQDO&RQVHQVXV'HILQLWLRQVIRU6HSVLVDQG ‡ 3URORQJHG&ULWLFDO,OOQHVV
6HSWLF6KRFN 6HSVLV -$0$
± 06+HUULGJHHWDO1(-0YROSS±
„ 7KH3UR&(66 ,QYHVWLJDWRUV$UDQGRPL]HGWULDORISURWRFROEDVHGFDUHIRU 
HDUO\VHSWLFVKRFN1(-0

DĞĐŚĂŶŝĐĂůsĞŶƚŝůĂƚŝŽŶ
ĂƐŝĐƚŽĚǀĂŶĐĞĚŽŶĐĞƉƚƐ
ƌŝƚŝĐĂůĂƌĞ͗dĂŬĞ,ŽŵĞDĞƐƐĂŐĞƐ
ĂŶĚůŝŶŝĐĂůWĞĂƌůƐ
<ĂƚŚůĞĞŶ:͘,ĂůĞLJ͕D͘͘
ƐƐŽĐŝĂƚĞWŚLJƐŝĐŝĂŶ
ĞƉĂƌƚŵĞŶƚŽĨDĞĚŝĐŝŶĞ͕ŝǀŝƐŝŽŶŽĨWƵůŵŽŶĂƌLJĂŶĚƌŝƚŝĐĂůĂƌĞ
ƌŝŐŚĂŵĂŶĚtŽŵĞŶ͛Ɛ,ŽƐƉŝƚĂů
ƐƐŝƐƚĂŶƚWƌŽĨĞƐƐŽƌŽĨDĞĚŝĐŝŶĞ ůŝnjĂďĞƚŚ'ĂLJ͕D
,ĂƌǀĂƌĚ^ĐŚŽŽůŽĨDĞĚŝĐŝŶĞ ƐƐŽĐŝĂƚĞƉƌŽŐƌĂŵĚŝƌĞĐƚŽƌ͕ƌŝŐŚĂŵĂŶĚtŽŵĞŶ͛Ɛ,ŽƐƉŝƚĂůWƵůŵŽŶĂƌLJ
ĂŶĚƌŝƚŝĐĂůĂƌĞDĞĚŝĐŝŶĞ&ĞůůŽǁƐŚŝƉ
ƐƐŝƐƚĂŶƚƉƌŽĨĞƐƐŽƌŽĨŵĞĚŝĐŝŶĞ͕,ĂƌǀĂƌĚDĞĚŝĐĂů^ĐŚŽŽů

ZĞĨĞƌĞŶĐĞƐ ZĞĨĞƌĞŶĐĞƐ
‡ 9HQNDWHVK%DODVXEUDPDQLDQHWDO$GMXQFWLYH*OXFRFRUWLFRLG7KHUDS\LQ3DWLHQWVZLWK6HSWLF
6KRFN New England Journal of Medicine 
‡ WŚĂŵdĞƚĂů͘DĂLJŽůŝŶŝĐWƌŽĐ͘ϮϬϭϳ͖ϵϮ;ϵͿ͗ϭϯϴϮ͗ϭϰϬϬ͘
‡ 6HPOHU0DWWKHZ:HWDO%DODQFHGFU\VWDOORLGVYHUVXVVDOLQHLQFULWLFDOO\LOODGXOWV New England
‡ ĂƌŶĞLJ͕ŝZŽĐĐŽ:͕EŝĞŵĂŶ'͘ƌŝƚĂƌĞDĞĚ͘ϮϬϬϱ͖ϯϯ;ϯ Journal of Medicine  

^ƵƉƉůͿ͗^ϭϮϮͲϴ͘DĞĚϮϬϬϬ͖ϯϰϮ͗ϭϯϬϭͲϭϯϬϴ͘ ‡ &KDXGKXUL'LSD\DQHWDO(DUO\5HQDO5HSODFHPHQW7KHUDS\9HUVXV6WDQGDUG&DUHLQWKH,&8$
6\VWHPDWLF5HYLHZ0HWD$QDO\VLVDQG&RVW$QDO\VLV Journal of intensive care medicine 
‡ ,ĂďĂƐŚŝED͘ƌŝƚĂƌĞDĞĚ͘ϮϬϬϱ͖ϯϯ;ϯ^ƵƉƉůͿ^ϮϮϴͲϰϬ͘
‡ *XpULQ&5HLJQLHU-5LFKDUG-&HWDO3URQHSRVLWLRQLQJLQVHYHUHDFXWHUHVSLUDWRU\GLVWUHVV
‡ ^ůƵƚƐŬLJ͕^͘ŚĞƐƚϭϵϵϵ͖ϭϭϲ͗ϵͲϭϱ͘ V\QGURPH1(QJO-0HG 

‡ <ŽŵƉĂƐ͕D͘ŵ:ZĞƐƉŝƌƌŝƚĂƌĞDĞĚϮϬϭϱ͖ǀŽůϭϵϮ͗ϭϰϮϬͲ
ϭϰϯϬ
-RVOLQ'LDEHWHV&HQWHU
'LDEHWHV)URP5HVHDUFKWR&OLQLFDO3UDFWLFH
+\SHUWHQVLRQ5HQDO'LVHDVHDQG&DUGLRYDVFXODU'LVHDVHLQ'LDEHWHV
&XUUHQW$SSURDFKWR'LDJQRVLVDQG7UHDWPHQW

ŽĂƌĚZĞǀŝĞǁWƌĂĐƚŝĐĞϯ 'LDEHWHV8SGDWH
^ĂŶũĂLJŝǀĂŬĂƌĂŶ͕D͘͘
&ĞůůŽǁ͕ŝǀŝƐŝŽŶŽĨĂƌĚŝŽǀĂƐĐƵůĂƌDĞĚŝĐŝŶĞ
ĞƉĂƌƚŵĞŶƚŽĨDĞĚŝĐŝŶĞ
ƌŝŐŚĂŵĂŶĚtŽŵĞŶ͛Ɛ,ŽƐƉŝƚĂů 52%(57&67$17210'
ůŝŶŝĐĂů&ĞůůŽǁŝŶDĞĚŝĐŝŶĞ
&KLHIRI.LGQH\DQG+\SHUWHQVLRQ6HFWLRQ
,ĂƌǀĂƌĚDĞĚŝĐĂů^ĐŚŽŽů
ƐĚŝǀĂŬĂƌĂŶΛďǁŚ͘ŚĂƌǀĂƌĚ͘ĞĚƵ -RVOLQ'LDEHWHV&HQWHU
$VVRFLDWH3URIHVVRURI0HGLFLQH
+DUYDUG0HGLFDO6FKRRO

6HOHFWHG5HIHUHQFHV
^ĞůĞĐƚĞĚZĞĨĞƌĞŶĐĞƐ
‡ $OWPDQ5$ODUFyQ*$SSHOURXWK'%ORFK'%RUHQVWHLQ'%UDQGW.HWDO ¾ *DUEHUHWDO&RQVHQVXV6WDWHPHQWE\WKH$PHULFDQ$VVRFLDWLRQRI&OLQLFDO(QGRFULQRORJLVWV
7KH$PHULFDQ&ROOHJHRI5KHXPDWRORJ\FULWHULDIRUWKHFODVVLILFDWLRQDQG DQG$PHULFDQ&ROOHJHRI(QGRFULQRORJ\RQWKH&RPSUHKHQVLYH7\SH'LDEHWHV0DQDJHPHQW
UHSRUWLQJRIRVWHRDUWKULWLVRIWKHKDQG$UWKULWLV5KHXP± $OJRULWKP6XPPDU\± ([HFXWLYH6XPPDU\(QGRFULQH3UDFWLFH
‡ $VKWRQ+$%X[WRQ0-'D\1(HWDO0XOWLFHQWUH$QHXU\VP6FUHHQLQJ
6WXG\*URXS7KH0XOWLFHQWUH$QHXU\VP6FUHHQLQJ6WXG\ 0$66 LQWRWKH ¾ ,Q]XFFKL6(HWDO0DQDJHPHQWRIK\SHUJO\FHPLDLQW\SHGLDEHWHVDSDWLHQWFHQWHUHG
HIIHFWRIDEGRPLQDODRUWLFDQHXU\VPVFUHHQLQJRQPRUWDOLW\LQPHQD DSSURDFK'LDEHWHV&DUH
UDQGRPLVHGFRQWUROOHGWULDO/DQFHW1RY  
¾ =LQPDQHWDO(PSDJOLIOR]LQ&DUGLRYDVFXODU2XWFRPHVDQG0RUWDOLW\LQ7\SH'LDEHWHV1HZ
‡ )OHPLQJ&:KLWORFN(3%HLO7/HGHUOH)6FUHHQLQJIRUDEGRPLQDODRUWLF
DQHXU\VPDEHVWHYLGHQFHV\VWHPDWLFUHYLHZIRUWKH863UHYHQWLYH (QJODQG-RXUQDORI0HGLFLQH
6HUYLFHV7DVN)RUFH$QQ,QWHUQ0HG

‡ /H*UDQG6%/HVNXVNL'=DPD,621DUUDWLYHUHYLHZIXURVHPLGHIRU
¾ 0DUVRHWDO/LUDJOXWLGHDQG&DUGLRYDVFXODU2XWFRPHVLQ7\SH'LDEHWHV1HZ(QJODQG-RXUQDO
K\SHUFDOFHPLDDQXQSURYHQ\HWFRPPRQSUDFWLFH$QQ,QWHUQ0HG
RI0HGLFLQH
  

‡ <RXQJ:)7KH,QFLGHQWDOO\'LVFRYHUHG$GUHQDO0DVV1(QJO-0HG ¾ 5LGGOHHWDO$&7DUJHWV6KRXOG%H3HUVRQDOL]HGWR0D[LPL]H%HQHILWV:KLOH/LPLWLQJ5LVNV
 'LDEHWHV&DUH

-RVOLQ 'LDEHWHV&HQWHU
'LDEHWHV)URP5HVHDUFKWR&OLQLFDO3UDFWLFH
+\SHUWHQVLRQ5HQDO'LVHDVHDQG&DUGLRYDVFXODU'LVHDVHLQ'LDEHWHV
&XUUHQW$SSURDFKWR'LDJQRVLVDQG7UHDWPHQW

dŚLJƌŽŝĚŝƐĞĂƐĞ
DĂƚƚŚĞǁ<ŝŵ͕D͘͘
'LDEHWHV0DQDJLQJ&RPPRQ ůŝŶŝĐĂůŝƌĞĐƚŽƌ
ŝǀŝƐŝŽŶŽĨŶĚŽĐƌŝŶŽůŽŐLJ͕ŝĂďĞƚĞƐĂŶĚ,LJƉĞƌƚĞŶƐŝŽŶ
&RPSOLFDWLRQV
ƌŝŐŚĂŵĂŶĚtŽŵĞŶ͛Ɛ,ŽƐƉŝƚĂů
52%(57&67$17210' ƐƐŝƐƚĂŶƚWƌŽĨĞƐƐŽƌ
&KLHIRI.LGQH\DQG+\SHUWHQVLRQ6HFWLRQ
-RVOLQ'LDEHWHV&HQWHU ,ĂƌǀĂƌĚDĞĚŝĐĂů^ĐŚŽŽů
$VVRFLDWH3URIHVVRURI0HGLFLQH
+DUYDUG0HGLFDO6FKRRO

6HOHFWHG5HIHUHQFHV
¾ 6WDQGDUGVRI0HGLFDO&DUH'LDEHWHV&DUH9ROXPH 6XSSOHPHQW
ZĞĨĞƌĞŶĐĞƐ
 
¾ ZZZ865'6RUJ ‡ ƵƌĂŶƚĞĞƚĂů͘dŚĞŝĂŐŶŽƐŝƐĂŶĚDĂŶĂŐĞŵĞŶƚ
¾ %XFNOH\/)HWDO,QWHQVLYHYHUVXVVWDQGDUGEORRGSUHVVXUHFRQWURO ŽĨdŚLJƌŽŝĚEŽĚƵůĞƐ͘:D͘ϮϬϭϴDĂƌϲ͖ϯϭϵ;ϵͿ͗
LQ635,17HOLJLEOHSDUWLFLSDQWVRI$&&25'%3'LDEHWHV&DUH ϵϭϰͲϵϮϰ
±
¾ 'H%RHUHWDO.LGQH\'LVHDVHDQG5HODWHG)LQGLQJVLQWKH'LDEHWHV ‡ WĞĞƚĞƌƐ ZW͘^ƵďĐůŝŶŝĐĂů,LJƉŽƚŚLJƌŽŝĚŝƐŵ͘EŶŐů :
&RQWURODQG&RPSOLFDWLRQV7ULDO(SLGHPLRORJ\RI'LDEHWHV DĞĚ͘ϮϬϭϳ:ƵŶϮϵ͖ϯϳϲ;ϮϲͿ͗ϮϱϱϲͲϮϱϲϱ
,QWHUYHQWLRQVDQG&RPSOLFDWLRQV6WXG\'LDEHWHV&DUH ‡ ^ŵŝƚŚd:͕,ĞŐĞĚƺƐ >͘'ƌĂǀĞƐΖŝƐĞĂƐĞ͘EŶŐů:

DĞĚ͘ϮϬϭϲKĐƚϮϬ͖ϯϳϱ;ϭϲͿ͗ϭϱϱϮͲϭϱϲϱ
¾ +ROPDQHWDO<HDU)ROORZXSRI,QWHQVLYH*OXFRVH&RQWUROLQ
7\SH'LDEHWHV1HZ(QJODQG-RXUQDORI0HGLFLQH ‡ ^ĂŵƵĞůƐD,͘^ƵďĂĐƵƚĞ͕ƐŝůĞŶƚ͕ĂŶĚƉŽƐƚƉĂƌƚƵŵ
 ƚŚLJƌŽŝĚŝƚŝƐ͘DĞĚůŝŶ Eŵ͘ϮϬϭϮ͘ϵϲ;ϮͿ͗ϮϮϯͲϯϯ
ADRENAL DISORDERS 3LWXLWDU\'LVRUGHUV
$QDQG9DLG\D0'006F
'LUHFWRU&HQWHUIRU$GUHQDO'LVRUGHUV
'LYLVLRQRI(QGRFULQRORJ\'LDEHWHV +\SHUWHQVLRQ
%ULJKDPDQG:RPHQ¶V+RVSLWDO
$VVLVWDQW3URIHVVRURI0HGLFLQH+DUYDUG0HGLFDO6FKRRO
8UVXOD%.DLVHU0'
&KLHI'LYLVLRQRI(QGRFULQRORJ\'LDEHWHV
DQG+\SHUWHQVLRQ
%ULJKDPDQG:RPHQ¶V+RVSLWDO
3URIHVVRURI0HGLFLQH
+DUYDUG0HGLFDO6FKRRO

*HQHUDO5HIHUHQFHV
6XSSOHPHQWDO5HIHUHQFHV
$GGLWLRQDO5HDGLQJ
78725,$/9,'(26
$'5(1$/3+<6,2/2*<KWWSVZZZ\RXWXEHFRPZDWFK"Y E0UK(X2W%0
z 3URODFWLQRPDVK\SHUSURODFWLQHPLD
$'5(1$/,168)),&,(1&<KWWSVZZZ\RXWXEHFRPZDWFK"Y 6JFN[.YFF.R
„ 0HOPHG6HWDO'LDJQRVLV 7UHDWPHQWRI+\SHUSURODFWLQHPLD$Q(QGRFULQH6RFLHW\&OLQLFDO
35,0$5<$/'267(521,60KWWSVZZZ\RXWXEHFRPZDWFK"Y GEYN1,L;8 3UDFWLFH*XLGHOLQHJ Clin Endocrinol Metab 2011. 96: 273-288.
3+(2&+5202&<720$KWWSVZZZ\RXWXEHFRPZDWFK"Y W=N-G1$ „ .OLEDQVNL$3URODFWLQRPDVN Engl J Med 2010. 362:1219-26.

)DVVQDFKWHWDO0DQDJHPHQWRIDGUHQDOLQFLGHQWDORPDV(XURSHDQJXLGHOLQHV z $FURPHJDO\
(XURSHDQ-RXUQDORI(QGRFULQRORJ\ „ .DW]QHOVRQ/HWDO$FURPHJDO\$Q(QGRFULQH6RFLHW\&OLQLFDO3UDFWLFH*XLGHOLQHJ Clin Endocrinol
Metab 2014. 99: 3933-3951.
<RXQJ:)-U&OLQLFDOSUDFWLFH7KHLQFLGHQWDOO\GLVFRYHUHGDGUHQDOPDVV
1HZ(QJODQG-RXUQDORI0HGLFLQH z &XVKLQJ¶V6\QGURPH
9DLG\D$+DPUDKLDQ$+$XFKXV5-*HQHWLFVRI3ULPDU\$OGRVWHURQLVP(QGRFULQH3UDFWLFH   „ 1LHPDQ/.HWDO7KH'LDJQRVLVRI&XVKLQJ¶V6\QGURPH$Q(QGRFULQH6RFLHW\&OLQLFDO3UDFWLFH
*XLGHOLQHJ Clin Endocrinol Metab 2008. 93: 1526–1540.
)XQGHU-:HWDO&DVH'HWHFWLRQ'LDJQRVLVDQG7UHDWPHQWRI3DWLHQWVZLWK3ULPDU\$OGRVWHURQLVP$Q(QGRFULQH6RFLHW\ „ 1LHPDQ/.HWDO7UHDWPHQWRI&XVKLQJ¶V6\QGURPH$Q(QGRFULQH6RFLHW\&OLQLFDO3UDFWLFH
&OLQLFDO3UDFWLFH*XLGHOLQH
-RXUQDORI&OLQLFDO(QGRFULQRORJ\DQG0HWDEROLVP
*XLGHOLQHJ Clin Endocrinol Metab 2015. 100: 2807-2831.

/HQGHUV-:0'XK4<(LVHQKRIHU*HWDO3KHRFKURPRF\WRPDDQG3DUDJDQJOLRPD$Q(QGRFULQH6RFLHW\&OLQLFDO3UDFWLFH
z +\SRSLWXLWDULVP
*XLGHOLQH „ )OHVHULX0HWDO+RUPRQDO5HSODFHPHQWLQ+\SRSLWXLWDULVPLQ$GXOWV$Q(QGRFULQH6RFLHW\&OLQLFDO
-RXUQDORI&OLQLFDO(QGRFULQRORJ\DQG0HWDEROLVP 3UDFWLFH*XLGHOLQHJ Clin Endocrinol Metab 2016. 101: 3888-3921.
%RUQVWHLQHWDO'LDJQRVLVDQG7UHDWPHQWRI3ULPDU\$GUHQDO,QVXIILFLHQF\$Q(QGRFULQH6RFLHW\&OLQLFDO3UDFWLFH*XLGHOLQH z 3HULRSHUDWLYH0DQDJHPHQW
-RXUQDORI&OLQLFDO(QGRFULQRORJ\DQG0HWDEROLVP
„ :RRGPDQVHH::HWDO$$&($&('LVHDVH6WDWH&OLQLFDO5HYLHZ3RVWRSHUDWLYH0DQDJHPHQW
)ROORZLQJ3LWXLWDU\6XUJHU\Endocrine Practice. 2015. 21: 832-838.

(QGRFULQH%RDUG5HYLHZ ZŚĞƵŵĂƚŽŝĚƌƚŚƌŝƚŝƐ͗ŝĂŐŶŽƐŝƐĂŶĚdƌĞĂƚŵĞŶƚ
$OH[DQGHU7XUFKLQ0'06
$VVRFLDWH3URIHVVRURI0HGLFLQH+DUYDUG0HGLFDO6FKRRO WĂƵů&ĞůůĂƌŝƉĂD
'LUHFWRURI4XDOLW\LQ'LDEHWHV'LYLVLRQRI(QGRFULQRORJ\ ƌŝŐŚĂŵĂŶĚtŽŵĞŶ͛Ɛ,ŽƐƉŝƚĂů
%ULJKDPDQG:RPHQ¶V+RVSLWDO
ŽƐƚŽŶD
ƐƐŽĐŝĂƚĞWƌŽĨĞƐƐŽƌŽĨDĞĚŝĐŝŶĞ
,ĂƌǀĂƌĚDĞĚŝĐĂů^ĐŚŽŽů

5()(5(1&(6
ZĞĨĞƌĞŶĐĞƐ
 :LOOLDPV7H[WERRNRI(QGRFULQRORJ\WK HGLWLRQ
‡ tĂƐŬŽ D͕ĂƐŐƵƉƚĂ ͕,ƵďĞƌƚ,ĞƚĂů͘WƌŽƉĞŶƐŝƚLJͲĂĚũƵƐƚĞĚĂƐƐŽĐŝĂƚŝŽŶŽĨŵĞƚŚŽƚƌĞdžĂƚĞǁŝƚŚŽǀĞƌĂůůƐƵƌǀŝǀĂůŝŶ
ƌŚĞƵŵĂƚŽŝĚĂƌƚŚƌŝƚŝƐ͘ϮϬϭϯ&Ğď͖ϲϱ;ϮͿ͗ϯϯϰͲϰϮ͘

 :HUQHU ,QJEDU
V 7KH7K\URLG$)XQGDPHQWDODQG ‡ ^ŝŶŐŚ:͕^ĂĂŐ<͕ƌŝĚŐĞƐ^>͕ĞƚĂů͘ŵĞƌŝĐĂŶŽůůĞŐĞŽĨZŚĞƵŵĂƚŽůŽŐLJ'ƵŝĚĞůŝŶĞĨŽƌƚŚĞdƌĞĂƚŵĞŶƚŽĨZŚĞƵŵĂƚŽŝĚ
ƌƚŚƌŝƚŝƐƌƚŚƌŝƚŝƐΘZŚĞƵŵϮϬϭϱ͖ϲϴ;ϭͿ͗ϭͲϮϲ
‡ K͛Ğůů:͕DŝŬƵůƐd͕dĂLJůŽƌd,ĞƚĂů͘dŚĞƌĂƉŝĞƐĨŽƌĐƚŝǀĞZŚĞƵŵĂƚŽŝĚƌƚŚƌŝƚŝƐĂĨƚĞƌDĞƚŚŽƚƌĞdžĂƚĞ&ĂŝůƵƌĞEŶŐ:DĞĚϮϬϭϯ
&OLQLFDO7H[WWK HGLWLRQ ϮϬϭϯ͗ϯϲϵ;ϰͿ͖ϯϬϳ
‡ tĞŝŶďůĂƚƚDĞƚĂů͘WŚĂƐĞ///^ƚƵĚLJǀĂůƵĂƚŝŶŐŽŶƚŝŶƵĂƚŝŽŶ͕dĂƉĞƌŝŶŐ͕ĂŶĚtŝƚŚĚƌĂǁĂůŽĨĞƌƚŽůŝnjƵŵĂďWĞŐŽůĨƚĞƌKŶĞ
zĞĂƌŽĨdŚĞƌĂƉLJŝŶWĂƚŝĞŶƚƐtŝƚŚĂƌůLJZŚĞƵŵĂƚŽŝĚƌƚŚƌŝƚŝƐƌƚŚƌŝƚŝƐZŚĞƵŵĂƚŽůϮϬϭϳKĐƚ͖ϲϵ;ϭϬͿ͗ϭϵϯϳʹϭϵϰϴ
 $PHULFDQ'LDEHWHV$VVRFLDWLRQ6WDQGDUGVRI ‡ ^ŽůŽŵŽŶ,͕<ƌĞŵĞƌ:D͕&ŝƐŚĞƌDĞƚĂů͘ŽŵƉĂƌĂƚŝǀĞĐĂŶĐĞƌƌŝƐŬĂƐƐŽĐŝĂƚĞĚǁŝƚŚŵĞƚŚŽƚƌĞdžĂƚĞ͕ŽƚŚĞƌŶŽŶͲďŝŽůŽŐŝĐĂŶĚ
ďŝŽůŽŐŝĐĚŝƐĞĂƐĞͲŵŽĚŝĨLJŝŶŐĂŶƚŝͲƌŚĞƵŵĂƚŝĐĚƌƵŐƐ͘ϮϬϭϰ&Ğď͖ϰϯ;ϰͿ͗ϰϴϵͲϵϳ

0HGLFDO&DUHLQ'LDEHWHV± 'LDEHWHV&DUH ‡ &ůĞŝƐĐŚŵĂŶŶZ͕DLJƐůĞƌ ͕,Ăůů^ĞƚĂů>ĂŶĐĞƚ͘ĨĨŝĐĂĐLJĂŶĚƐĂĨĞƚLJŽĨƚŽĨĂĐŝƚŝŶŝďŵŽŶŽƚŚĞƌĂƉLJ͕ƚŽĨĂĐŝƚŝŶŝďǁŝƚŚŵĞƚŚŽƚƌĞdžĂƚĞ͕


ĂŶĚĂĚĂůŝŵƵŵĂďǁŝƚŚŵĞƚŚŽƚƌĞdžĂƚĞŝŶƉĂƚŝĞŶƚƐǁŝƚŚƌŚĞƵŵĂƚŽŝĚĂƌƚŚƌŝƚŝƐ;KZ>^ƚƌĂƚĞŐLJͿ͗ĂƉŚĂƐĞϯďͬϰ͕ĚŽƵďůĞͲďůŝŶĚ͕
ŚĞĂĚͲƚŽͲŚĞĂĚ͕ƌĂŶĚŽŵŝƐĞĚ ĐŽŶƚƌŽůůĞĚƚƌŝĂů͘>ĂŶĐĞƚ ϮϬϭϳ:ƵůϮϵ͖ϯϵϬ;ϭϬϬϵϯͿ͗ϰϱϳͲϰϲϴ͘
9ROXPH6XSSOHPHQW
9DVFXOLWLV*&$305
8SGDWHLQ5KHXPDWLF'LVHDVHV ‡ ,QWHUQDO0HGLFLQH%RDUG5HYLHZ
6FOHURGHUPD6M|JUHQV0\RVLWLV ‡ 3DXO)'HOODULSD0'5KHXPDWRORJ\
%ULJKDPDQG:RPHQ¶V+RVSLWDO%RVWRQ
/DXUD/7DUWHU0'
0$
‡ $VVRFLDWH3URIHVVRURI0HGLFLQH+DUYDUG
$VVLVWDQW3URIHVVRURI0HGLFLQH
+DUYDUG0HGLFDO6FKRRO 0HGLFDO6FKRRO
'LYLVLRQRI5KHXPDWRORJ\
%ULJKDPDQG:RPHQ¶V+RVSLWDO

5HIHUHQFHV
‡ %RXUQLD9.9ODFKR\LDQQRSRXOXV3*6HOPL&0RXWVRXSRXOXV+0*HUVZLQ
5HIHUHQFHV
0(5HFHQWDGYDQFHVLQWKHWUHDWPHQWRIV\VWHPLFVFOHURVLV&OLQ5HY ‡ 6WRQH-+HWDO5LWX[LPDEYHUVXVF\FORSKRVSKDPLGHIRU$1&$DVVRFLDWHG
$OOHUJ\,PPXQRO   YDVFXOLWLV1(QJO-0HG  
‡ 7DVKNLQ'3(ODVKRII5&OHPHQWV3-HWDOIRUWKH6FOHURGHUPD/XQJ6WXG\
&\FORSKRVSKDPLGHYHUVXVSODFHERLQVFOHURGHUPDOXQJGLVHDVH1(QJ-
0HG ‡ <DWHV0:DWWV5$%DMHPD,0HWDO(8/$5(5$('7$UHFRPPHQGDWLRQVIRU
‡ 7DVKNLQ'35RWK0&OHPHQWV3-HWDO0\FRSKHQRODWHLQ6FOHURGHUPD6/6 WKHPDQDJHPHQWRI$1&$DVVRFLDWHGYDVFXOLWLV$QQ5KHXP'LV
,,/DQFHW   
‡ &RJKODQ-*3RSH-'HQWRQ&$VVHVVPHQWRIHQGSRLQWVLQSXOPRQDU\
DUWHULDOK\SHUWHQVLRQDVVRFLDWHGZLWKFRQQHFWLYHWLVVXHGLVHDVH&XUU2SLQ ‡ *XLYHOOLQ/HWDO5LWX[LPDEYHUVXVD]DWKLRSULQHIRUPDLQWHQDQFHLQ$1&$
3XOP0HG VXSSO 6 DVVRFLDWHGYDVFXOLWLV1(QJ-0HG  
‡ 0HLMHU-0HWDO(IIHFWLYHQHVVRIULWX[LPDEWUHDWPHQWLQSULPDU\6MRJUHQV
V\QGURPHDUDQGRPL]HGGRXEOHEOLQGSODFHERFRQWUROOHGWULDO$UWKULWLV ‡ 6WRQH-+HWDO7ULDORIWRFLOL]XPDELQJLDQWFHOODUWHULWLV1(QJ-0HG
5KHXP   
‡ 'DODNDV0,QIODPPDWRU\0XVFOHGLVHDVH1(QJ-0HG
‡ .LQJ7(-U%UDGIRUG:=&DVWUR%HUQDUGLQL6HWDO$SKDVHWULDORI ‡ $OLED]2QHU)$\GLQ6='LUHVNHQHOL+5HFHQWDGYDQFHVLQ7DND\DVXDUWHULWLV
SLUIHQLGRQHLQSDWLHQWVZLWKLGLRSDWKLFSXOPRQDU\ILEURVLV1(QJO-0HG ‡ (XU-5KHXPDWRO  

‡ +DWHPL*HWDO%HKoHW
VV\QGURPHDFULWLFDOGLJHVWRIWKHOLWHUDWXUH
‡ 5LFKHOGL/GX%RLV505DJKX*HWDO(IILFDF\DQGVDIHW\RIQLQWHGDQLELQ &OLQ([S5KHXPDWRO  6XSSO 6
LGLRSDWKLFSXOPRQDU\ILEURVLV1(QJO-0HG

^>ĂŶĚƚŚĞŶƚŝƉŚŽƐƉŚŽůŝƉŝĚ^LJŶĚƌŽŵĞ
0RQRDUWLFXODU-RLQW&RPSODLQWV
^ƵƐĂŶz͘ZŝƚƚĞƌD͕WŚ
ƐƐŽĐŝĂƚĞWŚLJƐŝĐŝĂŶ 'HUULFN-7RGG0'3K'
ŝǀŝƐŝŽŶŽĨZŚĞƵŵĂƚŽůŽŐLJ͕/ŵŵƵŶŽůŽŐLJĂŶĚůůĞƌŐLJ
ĞƉĂƌƚŵĞŶƚŽĨDĞĚŝĐŝŶĞ $VVRFLDWH3K\VLFLDQ
ƌŝŐŚĂŵĂŶĚtŽŵĞŶ͛Ɛ,ŽƐƉŝƚĂů 'HSDUWPHQWRI5KHXPDWRORJ\,PPXQRORJ\DQG$OOHUJ\
/ŶƐƚƌƵĐƚŽƌŝŶDĞĚŝĐŝŶĞ %ULJKDPDQG:RPHQ¶V+RVSLWDO
,QVWUXFWRURI0HGLFLQH
,ĂƌǀĂƌĚDĞĚŝĐĂů^ĐŚŽŽů
+DUYDUG0HGLFDO6FKRRO

ZĞĨĞƌĞŶĐĞƐ $GGLWLRQDO5HDGLQJIRU:LOOLDP2VOHU
‡ ůĂƐƐŝĨŝĐĂƚŝŽŶĐƌŝƚĞƌŝĂĨŽƌƐLJƐƚĞŵŝĐůƵƉƵƐĞƌLJƚŚĞŵĂƚŽƐƵƐ͗Ă  %DNHU'*6FKXPDFKHU+5³$FXWHPRQRDUWKULWLV´N
ƌĞǀŝĞǁ͘WĞƚƌŝD͕DĂŐĚĞƌ >͘ >ƵƉƵƐ͘ϮϬϬϰ͖ϭϯ;ϭϭͿ͗ϴϮϵ Engl J Med  
 6FKOHVLQJHU1³'LDJQRVLQJDQGWUHDWLQJJRXW$UHYLHZ
‡ 'ƵŝĚĞůŝŶĞƐĨŽƌƌĞĨĞƌƌĂůĂŶĚŵĂŶĂŐĞŵĞŶƚŽĨƐLJƐƚĞŵŝĐůƵƉƵƐ WRDLGSULPDU\FDUHSK\VLFLDQV´Postgrad Med.
ĞƌLJƚŚĞŵĂƚŽƐƵƐ ŝŶĂĚƵůƚƐ͘ŵĞƌŝĐĂŶŽůůĞŐĞŽĨZŚĞƵŵĂƚŽůŽŐLJ   ±
Ě,ŽĐŽŵŵŝƚƚĞĞŽŶ^LJƐƚĞŵŝĐ>ƵƉƵƐƌLJƚŚĞŵĂƚŽƐƵƐ
 6KDUII.$5LFKDUGV(3DQG7RZQHV-0³&OLQLFDO
'ƵŝĚĞůŝŶĞƐ͘ƌƚŚƌŝƚŝƐZŚĞƵŵ͘ϭϵϵϵ^ĞƉ͖ϰϮ;ϵͿ͗ϭϳϴϱͲϵϲ
0DQDJHPHQWRI6HSWLF$UWKULWLV´&XUUHQW
5KHXPDWRORJ\5HSRUWV  
‡ ŝĂŐŶŽƐŝƐĂŶĚDĂŶĂŐĞŵĞŶƚŽĨƚŚĞŶƚŝƉŚŽƐƉŚŽůŝƉŝĚ
^LJŶĚƌŽŵĞ͘'ĂƌĐŝĂĂŶĚƌŬĂŶ ͘E:D͘ϮϬϭϴDĂLJϮϰ͖ϯϳϴ  :LOVRQ-)³,QWKHFOLQLF*RXW´Ann Intern Med.
;ϮϭͿ͗ϮϬϭϬͲϮϬϭϵ͘   ,7&(UUDWXPAnn Intern Med.
  ±
ŵĞƌŐŝŶŐdŽƉŝĐƐĨŽƌƚŚĞŽĂƌĚƐ͗ƌŝŶŐŝŶŐ,ĞĂůƚŚƋƵŝƚLJ
ZĞƐĞĂƌĐŚŝŶƚŽzŽƵƌWƌĂĐƚŝĐĞ *HQHUDO'HUPDWRORJ\5HYLHZ
&OLQLFDO3HDUOVIRU&RPPRQ3UREOHPV
/ŶƚĞŶƐŝǀĞZĞǀŝĞǁŽĨ/ŶƚĞƌŶĂůDĞĚŝĐŝŶĞ
ŚĞƌLJůZ͘ůĂƌŬD͕^Đ 0DQLVKD7KDNXULD0'
ŝƌĞĐƚŽƌ͕,ĞĂůƚŚƋƵŝƚLJZĞƐĞĂƌĐŚΘ/ŶƚĞƌǀĞŶƚŝŽŶ &R'LUHFWRU0HUNHO&HOO&DUFLQRPD&HQWHURI([FHOOHQFH
ĞŶƚĞƌĨŽƌŽŵŵƵŶŝƚLJ,ĞĂůƚŚĂŶĚ,ĞĂůƚŚƋƵŝƚLJ 'HSDUWPHQWRI'HUPDWRORJ\
ŝǀŝƐŝŽŶŽĨ'ĞŶĞƌĂů/ŶƚĞƌŶĂůDĞĚŝĐŝŶĞĂŶĚWƌŝŵĂƌLJĂƌĞ %ULJKDP :RPHQ¶V+RVSLWDO
ƌŝŐŚĂŵĂŶĚtŽŵĞŶ͛Ɛ,ŽƐƉŝƚĂů &HQWHUIRU&XWDQHRXV2QFRORJ\
'DQD)DUEHU%ULJKDP :RPHQ¶V&DQFHU&HQWHU
ƐƐŝƐƚĂŶƚWƌŽĨĞƐƐŽƌŽĨDĞĚŝĐŝŶĞ͕,ĂƌǀĂƌĚDĞĚŝĐĂů^ĐŚŽŽů
,QVWUXFWRURI'HUPDWRORJ\+DUYDUG0HGLFDO6FKRRO

ZĞĨĞƌĞŶĐĞƐĂŶĚZĞƐŽƵƌĐĞƐ 6XSSOHPHQWDO5HIHUHQFH6OLGH
‡ ŶĚĞƌƐŽŶ͕ŶĚƌĞǁ͕͘ĞƚĂů͘ΗWƌŽŵŽƚŝŶŐ,ĞĂůƚŚƋƵŝƚLJŶĚ
ůŝŵŝŶĂƚŝŶŐŝƐƉĂƌŝƚŝĞƐdŚƌŽƵŐŚWĞƌĨŽƌŵĂŶĐĞDĞĂƐƵƌĞŵĞŶƚ ‡ 6FKPXOWV&'HWDO-$0$'HUP  
ŶĚWĂLJŵĞŶƚ͘Η ,ĞĂůƚŚĨĨĂŝƌƐ ϯϳ͘ϯ;ϮϬϭϴͿ͗ϯϳϭͲϯϳϳ͘ ‡ :KLWPRUH6(HWDO-$0$'HUP
‡ ^ŽĐŝĂů/ŶƚĞƌǀĞŶƚŝŽŶƐZĞƐĞĂƌĐŚΘǀĂůƵĂƚŝŽŶEĞƚǁŽƌŬ;^/ZEͿ͗ ‡ :HKQHU 05HWDO-$0$'HUP 
GRLMDPDGHUPDWRO
ŚƚƚƉƐ͗ͬͬƐŝƌĞŶĞƚǁŽƌŬ͘ƵĐƐĨ͘ĞĚƵͬ
‡ +DELI&OLQLFDO'HUPDWRORJ\$&RORU*XLGHWR'LDJQRVLV
‡ dĞƌǀĂůŽŶ͕DĞůĂŶŝĞĂŶĚ:ĂŶŶDƵƌƌĂLJͲ'ĂƌĐşĂ͘ΗƵůƚƵƌĂů DQG7UHDWPHQWWK HGLWLRQ
ŚƵŵŝůŝƚLJǀĞƌƐƵƐĐƵůƚƵƌĂůĐŽŵƉĞƚĞŶĐĞ͗ĐƌŝƚŝĐĂůĚŝƐƚŝŶĐƚŝŽŶŝŶ ± YROXPHWH[WLGHDOIRUWKH,QWHUQDO0HGLFLQHFOLQLFLDQ
ĚĞĨŝŶŝŶŐƉŚLJƐŝĐŝĂŶƚƌĂŝŶŝŶŐŽƵƚĐŽŵĞƐŝŶŵƵůƚŝĐƵůƚƵƌĂů ‡ %RORJQLD-RUL]]R6FKDIIHU'HUPDWRORJ\UG HGLWLRQ
ĞĚƵĐĂƚŝŽŶ͘Η :ŽƵƌŶĂůŽĨŚĞĂůƚŚĐĂƌĞĨŽƌƚŚĞƉŽŽƌĂŶĚ ± 'HWDLOHGWH[WDIDYRULWHRIPRVWGHUPDWRORJ\WUDLQHHV
ƵŶĚĞƌƐĞƌǀĞĚ ϵ͘Ϯ;ϭϵϵϴͿ͗ϭϭϳͲϭϮϱ͘ ‡ ZZZDDGRUJ
± ([FHOOHQWUHVRXUFHIRUFOLQLFLDQVDQGSDWLHQWV
‡ /ŵƉůŝĐĂƚŝŽŶƐƐŽĐŝĂƚŝŽŶdĞƐƚĨŽƌŝŵƉůŝĐŝƚďŝĂƐ͗ ‡ ZZZPHUNHOFHOORUJ
ŚƚƚƉƐ͗ͬͬŝŵƉůŝĐŝƚ͘ŚĂƌǀĂƌĚ͘ĞĚƵͬŝŵƉůŝĐŝƚ ± ,I\RXDUHLQWHUHVWHGLQZKDW,GRRXWVLGHRI*HQ'HUP

'HSUHVVLRQ8SGDWH
Russell G. Vasile MD

ůůĞƌŐLJͬ/ŵŵƵŶŽůŽŐLJKǀĞƌǀŝĞǁ 'LUHFWRU$IIHFWLYH'LVRUGHUV&RQVXOWDWLRQ6HUYLFH
'HSDUWPHQWRI3V\FKLDWU\
%HWK,VUDHO'HDFRQHVV0HGLFDO&HQWHU
$VVRFLDWH3URIHVVRURI3V\FKLDWU\
+DUYDUG0HGLFDO6FKRRO

ĂǀŝĚ͘^ůŽĂŶĞ͕D͕͘͘Ě͘D͘ 3V\FKLDWU\2YHUYLHZ
ƌŝŐŚĂŵĂŶĚtŽŵĞŶ¶Ɛ,ŽƐƉŝƚĂů
ĂŶĂ&ĂƌďĞƌĂŶĐĞƌ/ŶƐƚŝƚƵƚĞ
tĞƐƚZŽdžďƵƌLJsDĞĚŝĐĂůĞŶƚĞƌ
,ĂƌǀĂƌĚDĞĚŝĐĂů^ĐŚŽŽů

5HIHUHQFHV
ZĞĨĞƌĞŶĐĞƐ
‡ &KDQGOHU9*RRJOHDQGVXLFLGHVZKDWFDQZHOHDUQDERXWWKHXVHRIWKHLQWHUQHWWR
• ĂƐƚĞůůƐĞƚĂů:/ϮϬϬϴ͕ϮϬϭϮĞƐĞŶƐŝƚŝnjĂƚŝŽŶƐ SUHYHQWVXLFLGHV3XEOLF+HDOWK  
• ŽLJĐĞĞƚĂů:/ϮϬϬϵyŽůĂŝƌ ƵƐĞŝŶhƌƚŝĐĂƌŝĂ͕DĂƵƌĞƌ
ϮϬϭϯ ‡ 0F,QW\UH56HWDO7UHDWPHQWUHVLVWDQWGHSUHVVLRQ'HILQLWLRQVUHYLHZRIWKHHYLGHQFH
DQGDOJRULWKPLFDSSURDFK-RXUQDORI$IIHFWLYH'LVRUGHUV  
• ƐĐƌŝďĂŶŽ ĞƚĂůůϮϬϬϵDĂƐƚŽĐLJƚŽƐŝƐ
• ƵƌŚĂŵĞƚĂůE:DϮϬϬϴ/ŵŵƵŶŽƚŚĞƌĂƉLJĨŽƌZ ‡ *REEL HWDO$QWLGHSUHVVDQWFRPELQDWLRQYHUVXVDQWLGHSUHVVDQWSOXVVHFRQG
JHQHUDWLRQDQWLSV\FKRWLFDXJPHQWDWLRQLQWUHDWPHQWUHVLVWDQWXQLSRODUGHSUHVVLRQ
• ƵŶŶŝŶŐŚĂŵZƵŶĚůĞƐϮϬϬϳŽŵŵŽŶsĂƌŝĂďůĞ ,QWHUQDWLRQDO&OLQLFDO3V\FKRSKDUPDFRORJ\
ŝŵŵƵŶŽĚĞĨŝĞŶĐLJ
• ^ŝŵŽŶƐ&ϮϬϭϭ͗tŽƌůĚůůĞƌŐLJKƌŐĂŶŝnjĂƚŝŽŶ ‡ 1HOVRQ-&UDLJ$GMXQFWLYH=LSUDVLGRQHLQ0DMRU'HSUHVVLRQDQGWKH&XUUHQW6WDWXV
RI$GMXQFWLYH$W\SLFDO$QWLSV\FKRWLFV$P-3V\FKLDWU\
ĂŶĂƉŚLJůĂdžŝƐŐƵŝĚĞůŝŶĞƐ͗:ůůĞƌŐLJůŝŶ /ŵŵƵŶŽů
• >WƉĞĂŶƵƚĂůůĞƌŐLJƐƚƵĚLJE:DϮϬϭϱ ‡ /RHEHO HW$O/XUDVLGRQHDVDGMXQFWLYHWKHUDS\ZLWKOLWKLXPRUYDOSURDWHIRUWKH
WUHDWPHQWRIELSRODUGHSUHVVLRQ$UDQGRPL]HGGRXEOHEOLQGSODFHERFRQWUROOHG
• DĂƐƚĞůůŶĂƉŚLJůĂĐƚŽŝĚ ƌĞĐĞƉƚŽƌEĂƚƵƌĞϮϬϭϱ VWXG\$P-3V\FKLDWU\
ŝĂŐŶŽƐƚŝĐƌƌŽƌƐŝŶDĞĚŝĐŝŶĞ
ǀĂůƵĂƚŝŽŶŽĨĂƌĚŝĂĐĂŶĚWƵůŵŽŶĂƌLJZŝƐŬŝŶƚŚĞWƌĞŽƉ
*RUGRQ'6FKLII0' WĂƚŝĞŶƚ

$VVRFLDWH'LUHFWRU&HQWHUIRU3DWLHQW6DIHW\5HVHDUFKDQG3UDFWLFH ĚĂŵ͘^ĐŚĂĨĨĞƌ͕D͕DW,
%ULJKDPDQG:RPHQ
V+RVSLWDO'LY*HQHUDO0HGLFLQH ƐƐŽĐŝĂƚĞWŚLJƐŝĐŝĂŶ͕,ŽƐƉŝƚĂůDĞĚŝĐŝŶĞhŶŝƚ͕ƌŝŐŚĂŵĂŶĚ
tŽŵĞŶ͚Ɛ,ŽƐƉŝƚĂů
6DIHW\'LUHFWRU± +DUYDUG&HQWHUIRU3ULPDU\&DUH /ŶƐƚƌƵĐƚŽƌ͕,ĂƌǀĂƌĚDĞĚŝĐĂů^ĐŚŽŽů
$FDGHPLF,PSURYHPHQW&ROODERUDWLYH

$VVRFLDWH3URIHVVRURI0HGLFLQH+DUYDUG0HGLFDO6FKRRO

&ƵƌƚŚĞƌZĞĂĚŝŶŐͬZĞƐŽƵƌĐĞƐ .H\5HIHUHQFHV
‡ EĂƚŝŽŶĂůĐĂĚĞŵLJŽĨDĞĚŝĐŝŶĞ͗/ŵƉƌŽǀŝŶŐŝĂŐŶŽƐŝƐŝŶ
 )OHLVKHU/$)OHLVFKPDQQ.($XHUEDFK $'HWDO$&&$+$JXLGHOLQHRQ
,ĞĂůƚŚĂƌĞ ZĞƉŽƌƚ͘&ƌĞĞŽŶůŝŶĞǀŝĞǁŝŶŐ͘ƐƉĞĐŝĂůůLJ SHULRSHUDWLYHFDUGLRYDVFXODUHYDOXDWLRQDQGPDQDJHPHQWRISDWLHQWVXQGHUJRLQJ
ƌĞĐŽŵŵĞŶĚdžĞĐƵƚŝǀĞ^ƵŵŵĂƌLJƉƉ͘ϭͲϭϴ͘ QRQFDUGLDF VXUJHU\DUHSRUWRIWKH$PHULFDQ&ROOHJHRI&DUGLRORJ\$PHULFDQ+HDUW
$VVRFLDWLRQ7DVN)RUFHRQ3UDFWLFH*XLGHOLQHVCirculation. Dec 9
‡ ,ZYW^EĞƚtĞďƐŝƚĞÆdŽƉŝĐƐÆ^ĂĨĞƚLJdĂƌŐĞƚÆŝĂŐŶŽƐƚŝĐ 2014;130(24):e278-333.
 &RKHQ0(.R &<%LOLPRULD .<HWDO2SWLPL]LQJ$&6164,3PRGHOLQJIRU
ƌƌŽƌƐ͘hƉͲƚŽͲĚĂƚĞĐŽůůĞĐƚŝŽŶŽĨĂƌƚŝĐůĞƐŽŶdžƌƌŽƌ͘ HYDOXDWLRQRIVXUJLFDOTXDOLW\DQGULVNSDWLHQWULVNDGMXVWPHQWSURFHGXUHPL[
‡ ^ŽĐŝĞƚLJĨŽƌ/ŵƉƌŽǀŝŶŐŝĂŐŶŽƐŝƐŝŶDĞĚŝĐŝŶĞ;^/DͿ DGMXVWPHQWVKULQNDJHDGMXVWPHQWDQGVXUJLFDOIRFXVJournal of the American
College of Surgeons. Aug 2013;217(2):336-346.e331.
tĞďƐŝƚĞ͕/ŶƚĞƌŶĂƚŝŽŶĂůŽŶĨĞƌĞŶĐĞƐ͕ƌĞƐŽƵƌĐĞƐ  32,6(6WXG\*URXS'HYHUHDX[ 3-<DQJ+HWDO(IIHFWVRIH[WHQGHGUHOHDVH
PHWRSUROROVXFFLQDWHLQSDWLHQWVXQGHUJRLQJQRQFDUGLDFVXUJHU\ 32,6(WULDO D
‡ ^ĐŚŝĨĨΘZƵĂŶ͘dŚĞůƵƐŝǀĞĂŶĚ/ůůƵƐŝǀĞYƵĞƐƚĨŽƌŝĂŐŶŽƐƚŝĐ UDQGRPLVHG FRQWUROOHGWULDOLancet. May 31 2008;371(9627):1839-1847.
^ĂĨĞƚLJDĞƚƌŝĐƐ͘ :ůŽĨ'ĞŶ/ŶƚĞƌŶĂůDĞĚϮϬϭϴ  'HYHUHDX[ 3-0UNREUDGD 06HVVOHU ',HWDO$VSLULQLQSDWLHQWVXQGHUJRLQJ
QRQFDUGLDF VXUJHU\New England Journal of Medicine. Apr 17 2014;370(16):1494-
‡ ^ĐŚŝĨĨ͘ŝĂŐŶŽƐƚŝĐƌƌŽƌ͗dŝŵĞĨŽƌĂEĞǁWĂƌĂĚŝŐŵ͘ 1503.
 'RXNHWLV -'6S\URSRXORV $&.DDW] 6HWDO3HULRSHUDWLYH%ULGJLQJ
D:YƵĂůŝƚLJĂŶĚ^ĂĨĞƚLJ ϮϬϭϯ $QWLFRDJXODWLRQLQ3DWLHQWVZLWK$WULDO)LEULOODWLRQNew England Journal of Medicine.
August 27 2015;373(9):823-833.
Provided by: Brigham and Women’s Hospital
The Department of Medicine
and
Harvard Medical School
Global and Continuing Education
41st Annual Intensive Review of Internal Medicine

Book
Topic/Speaker Page #
NEPHROLOGY
Acute Kidney Injury
1
Sushrut S. Waikar, MD
Chronic Kidney Disease
20
Ajay K. Singh, MBBS, FRCP (UK), MBA
Revisiting Electrolytes and Acid-Base Basics
34
Bradley M. Denker, MD
Proteinuria, Hematuria, and Glomerular Disease
75
Ajay K. Singh, MBBS, FRCP (UK), MBA
Questions and Answers
NA
Nephrology Faculty
Electrolytes and Acid-Base: Challenging Q&A
92
Bradley M. Denker, MD
Dialysis and Transplantation
152
J. Kevin Tucker, MD
Questions and Answers
NA
Nephrology Faculty
Nephrology: Take-Home Messages and Clinical Pearls
189
Ajay K. Singh, MBBS, FRCP (UK), MBA
Nephrology Board Review
205
Ajay K. Singh, MBBS, FRCP (UK), MBA
HEMATOLOGY
Anemia
240
Maureen M. Achebe, MD, MBBS
Hypercoagulable States and New Anticoagulants
271
Jean M. Connors, MD
Bleeding Disorders
297
Elisabeth M. Battinelli, MD, PhD
Questions and Answers
NA
Hematology Faculty
Hematology Cases: Common, Complex, Rare
335
Nancy Berliner, MD
Questions and Answers
NA
Hematology Faculty
Board Review in Hematology
364
Nathan T. Connell, MD, MPH
Provided by: Brigham and Women’s Hospital
The Department of Medicine
and
Harvard Medical School
Global and Continuing Education
41st Annual Intensive Review of Internal Medicine

Book
Topic/Speaker Page #
HEMATOLOGY
Board Review Practice - 1 - - Images Part I
402
Ajay K. Singh, MBBS, FRCP (UK), MBA
GASTROENTEROLOGY
Esophageal Disorders
424
Walter W. Chan, MD, MPH
Peptic Ulcer Disease
457
John R. Saltzman, MD
Acute and Chronic Pancreatitis
486
Julia Y. McNabb-Baltar, MD
Hepatitis B and C
511
Ming Valerie Lin, MBChB
Chronic Liver Disease and Its Complications
532
Anna E. Rutherford, MD, MPH
Questions and Answers
NA
Gastroenterology Faculty
Inflammatory Bowel Disease
553
Sonia Friedman, MD
Diarrhea
585
Benjamin N. Smith, MD
Gastroenterology: Take-Home Messages and Clinical Pearls
615
Kunal Jajoo, MD
GI Board Review
627
Muthoka L. Mutinga, MD
ONCOLOGY
Oncology: Clinical Pearls
675
Wendy Y. Chen, MD
Leukemia and Myelodysplastic Syndrome
712
Edwin P. Alyea, III, MD
Prostate and Bladder Cancer
737
Lauren C. Harshman, MD
Lung Cancer
772
David M. Jackman, MD
Breast Cancer
794
Wendy Y. Chen, MD
Questions and Answers
NA
Oncology Faculty
Provided by: Brigham and Women’s Hospital
The Department of Medicine
and
Harvard Medical School
Global and Continuing Education
41st Annual Intensive Review of Internal Medicine

Book
Topic/Speaker Page #
ONCOLOGY
Lymphoma and Multiple Myeloma
829
Ann S. LaCasce, MD
Gastrointestinal Cancers
858
Jeffrey A. Meyerhardt, MD
Questions and Answers
NA
Oncology Faculty
Oncology: Take-Home Messages and Clinical Pearls
891
Ann S. LaCasce, MD
Board Review in Oncology
925
Ann S. LaCasce, MD
CARDIOVASCULAR MEDICINE
2018 Cardiology Overview
946
Leonard S. Lilly, MD
CV Prevention
964
Samia Mora, MD
Acute Coronary Syndrome Management
1008
Marc S. Sabatine, MD
Questions and Answers
NA
Cardiovascular Faculty
Pulmonary Embolism, DVT and Anticoagulation
1035
Samuel Z. Goldhaber, MD
Valvular Heart Disease
1067
Brendan M. Everett, MD, MPH
Peripheral, Aortic and Carotid Disease
1103
Marc P. Bonaca, MD
Questions and Answers
NA
Cardiovascular Faculty
Congestive Heart Failure
1133
Anju Nohria, MD
Congenital Heart Disease
1157
Ann M. Valente, MD
Questions and Answers
NA
Cardiovascular Faculty
Must-Not-Miss ECG Diagnoses
1178
Dale S. Adler, MD
Provided by: Brigham and Women’s Hospital
The Department of Medicine
and
Harvard Medical School
Global and Continuing Education
41st Annual Intensive Review of Internal Medicine

Book
Topic/Speaker Page #
CARDIOVASCULAR MEDICINE
Atrial Fibrillation and Common Supraventricular
Tachycardias 1246
Sunil Kapur, MD
Questions and Answers
NA
Cardiovascular Faculty
Bradycardias, Syncope and Sudden Death
1267
Usha B. Tedrow, MD, MS
Inflammation and CVD
1293
Paul M Ridker, MD
Cardiology: Take-Home Messages and Clinical Pearls
1318
Akshay S. Desai, MD, MPH
Board Review in Cardiology
1344
Garrick C. Stewart, MD
INFECTIOUS DISEASE
Infection in the Immunocompromised Host
1404
Sarah P. Hammond, MD
Tropical Medicine and Parasitology
1428
James H. Maguire, MD
Tuberculosis for the Non-ID Specialist
1461
Gustavo E. Velasquez, MD, MPH
Questions and Answers
NA
Infectious Diseases Faculty
Adult Immunization
1493
Lindsey R. Baden, MD
HIV Disease: An Overview
1518
Jennifer A. Johnson, MD
Pneumonia and Other Respiratory Tract Infections
1539
Michael Klompas, MD, MPH
Questions and Answers
NA
Infectious Diseases Faculty
Infectious Disease: Take-Home Messages and Clinical Pearls
1569
James H. Maguire, MD
Sexually Transmitted Diseases
1587
Todd B. Ellerin, MD
Infectious Disease Board Review
1617
Todd B. Ellerin, MD
Provided by: Brigham and Women’s Hospital
The Department of Medicine
and
Harvard Medical School
Global and Continuing Education
41st Annual Intensive Review of Internal Medicine

Book
Topic/Speaker Page #
WOMEN’S HEALTH
Contraception: An Update
1647
Kari P. Braaten, MD
Medical Complications of Pregnancy
1676
Ellen W. Seely, MD
Screening for and Preventing HPV and Cervical CA
1697
Annekathryn Goodman, MD
Questions and Answers
NA
Women’s Health Faculty
Menopause
1734
Kathryn A. Martin, MD
Evaluation of the Patient with Menstrual Irregularities
1756
Maria A. Yialamas, MD
Osteoporosis and Metabolic Bone Disease
1782
Carolyn B. Becker, MD
Women’s Health: Take-Home Messages and Clinical Pearls
1818
Caren G. Solomon, MD
Women’s Health Board Review
1835
Kathryn M. Rexrode, MD
Board Review Practice - 2
1869
David D. Berg, MD
PULMONARY MEDICINE
Pulmonary Overview
1895
Christopher H. Fanta, MD
Interstitial Lung Disease
1945
Hilary J. Goldberg, MD
COPD
1979
Craig P. Hersh, MD
Sleep Apnea
2014
Lawrence J. Epstein, MD
Questions and Answers
NA
Pulmonary Faculty
Asthma
2045
Elliot Israel, MD
Pleural Disease
2079
Scott L. Schissel, MD, PhD
CXR Interpretation for the Boards
2111
Christopher H. Fanta, MD
Provided by: Brigham and Women’s Hospital
The Department of Medicine
and
Harvard Medical School
Global and Continuing Education
41st Annual Intensive Review of Internal Medicine

Book
Topic/Speaker Page #
PULMONARY MEDICINE
Pulmonary Function Testing
2155
Scott L. Schissel, MD, PhD
Pulmonary Medicine: Take-Home Messages and Clinical Pearls
2182
Christopher H. Fanta, MD
Evaluation of the Dyspneic Patient
2211
David M. Systrom, MD
Pulmonary Board Review
2232
Christopher H. Fanta, MD
NEUROLOGY
Women’s Neurology
2281
M. Angela O'Neal, MD
Headache
2308
Carolyn A. Bernstein, MD
Stroke
2344
Galen V. Henderson, MD
Seizure Disorders
2394
Tracey A. Milligan, MD
Board Review in Neurology
2430
M. Angela O'Neal, MD
GENERAL INTERNAL MEDICINE / PRIMARY CARE
Geriatric Medicine: Hypertension, Dementia, Incontinence
2487
Suzanne E. Salamon, MD
Obesity
2541
Florencia Halperin, MD
End of Life
2564
Lisa S. Lehmann, MD, PhD
Questions and Answers
NA
General Internal Medicine Faculty
General Internal Medicine Board Review
2580
Ann L. Pinto, MD, PhD
Biostatistics Board Review
2622
Julie E. Buring, ScD
Morning Report: Instructive Cases
2658
Maria A. Yialamas, MD
Provided by: Brigham and Women’s Hospital
The Department of Medicine
and
Harvard Medical School
Global and Continuing Education
41st Annual Intensive Review of Internal Medicine

Book
Topic/Speaker Page #
GENERAL INTERNAL MEDICINE / PRIMARY CARE
Workshop: General Internal Medicine Case Studies
2704
Lori W. Tishler, MD
Addiction in Pain Management
2756
Sarah E. Wakeman, MD, FASM
Hyperlipidemia
2788
Scott Kinlay, PhD, MBBS
Questions and Answers
NA
General Medicine Faculty
CRITICAL CARE
Sepsis
2838
Rebecca M. Baron, MD
Popular ICU Topics
2868
Kathleen J. Haley, MD
Mechanical Ventilation: Basics to Advanced Concepts
2899
Kathleen J. Haley, MD
Highlights of the Pain, Agitation, Delirium, Immobility, and Sleep Guidelines 2018
2923
Gerald L. Weinhouse, MD
Cardiogenic Shock, CHF and Malignant Arrhythmias
2940
Akshay S. Desai, MD, MPH
Critical Care: Take-Home Messages and Clinical Pearls
2963
Elizabeth B. Gay, MD
Board Review in Critical Care
2976
Elizabeth B. Gay, MD
Board Review Practice - 3
3000
Sanjay Divakaran, MD
ENDOCRINOLOGY
2018 Diabetes Overview
3034
Robert C. Stanton, MD
Diabetes: Managing Common Complications
3069
Robert C. Stanton, MD
Thyroid Disease
3100
Matthew I. Kim, MD
Adrenal Disorders
3138
Anand Vaidya, MD, MMSc
Provided by: Brigham and Women’s Hospital
The Department of Medicine
and
Harvard Medical School
Global and Continuing Education
41st Annual Intensive Review of Internal Medicine

Book
Topic/Speaker Page #
ENDOCRINOLOGY
Questions and Answers
NA
Endocrinology Faculty
Pituitary Disorders
3178
Ursula B. Kaiser, MD
Endocrinology: Take-Home Messages and Clinical Pearls
3201
Carolyn B. Becker, MD
Endocrine Board Review
3218
Alexander Turchin, MD
RHEUMATOLOGY
Rheumatoid Arthritis: Diagnosis and New Treatment
3246
Paul F. Dellaripa, MD
Scleroderma/Sjogren’s and Myositis
3276
Laura L. Tarter, MD
Vasculitis/GCA/PMR
3308
Paul F. Dellaripa, MD
Questions and Answers
NA
Rheumatology Faculty
Soft Tissue Syndromes
3348
Elinor A. Mody, MD
SLE and the Antiphospholipid Syndrome
3371
Susan Y. Ritter, MD
Questions and Answers
NA
Rheumatology Faculty
Monoarticular Arthritis
3408
Derrick J. Todd, MD, PhD
Rheumatology: Take-Home Messages and Clinical Pearls
3441
Paul F. Dellaripa, MD
Rheumatology Board Review
3458
Joerg Ermann, MD
Board Review Practice - 4 - Images Part II
3512
Ajay K. Singh, MBBS, FRCP (UK), MBA
MISCELLANEOUS
Hospital Medicine: What’s New in the Literature
3536
Christopher L. Roy, MD
Provided by: Brigham and Women’s Hospital
The Department of Medicine
and
Harvard Medical School
Global and Continuing Education
41st Annual Intensive Review of Internal Medicine

Book
Topic/Speaker Page #
MISCELLANEOUS
Racial / Economic Health Disparities
3574
Cheryl R. Clark, MD
Dermatology
3597
Manisha Thakuria, MD
2018 Allergy/Immunology Overview
3656
David E. Sloane, MD
Depression Update
3688
Russell G. Vasile, MD
Diagnostic Errors in Medicine
3711
Gordon Schiff, MD
Evaluation of Cardiac and Pulmonary Risk in the Preop Patient
3744
Adam C. Schaffer, MD
Copyright © Harvard Medical School, 2018. All Rights Reserved.

Psychiatry Overview
Acute Kidney Injury
Sushrut S. Waikar, MD, MPH
Constantine L. Hampers, MD Distinguished Chair in Renal Medicine
Brigham and Women’s Hospital
Associate Professor
Harvard Medical School

Disclosures
None relevant

1
Copyright © Harvard Medical School, 2018. All Rights Reserved.

Goals
• Introduction to new terminology, definition

• Incidence, mortality, costs of AKI

• Approach to the patient with AKI

• Specific clinical scenarios

Case
• 64 year old man with HTN, DM, osteoarthritis, BPH
• Presents to ED complaining of nausea, vomiting
• Meds: lisinopril 40 mg/d, metformin 1gm bid, ibuprofen
800 mg tid
• Exam: BP 100/60, HR 104, RR 16
– Lungs clear; CV RRR nl s1 s2, no rub, no JVD
– Abd soft
– Ext no edema
• Labs
134 100 84
5.6 18 3.2*
*283 µmol/L

2
Copyright © Harvard Medical School, 2018. All Rights Reserved.

Acute Kidney Injury


or the syndrome formerly known as “Acute Renal Failure”

• “Acute”
Happening within hours to days

• “Kidney”
(more familiar than “Renal”)

• “Injury”
Not always “failure.” Refers to
organ damage…

Defining AKI
“Sudden” rise of > 2.0 mg/dL

> 0.3 mg/dL increase


0.5 mg/dL if < 1.9
1.0 mg/dL if 2.0 – 4.9
1.5 mg/dL if > 4.9

25% increase to at least 2.0 mg/dL within 48h


50% increase to at least 1.4 mg/dL
50% increase to at least 2.0 mg/dL

3
Copyright © Harvard Medical School, 2018. All Rights Reserved.

Defining AKI

“Sudden” rise of > 2.0 mg/dL


> 0.3 mg/dL increase
0.5 mg/dL if < 1.9
> 30 different 1.0 mg/dL if 2.0 – 4.9
1.5 mg/dL if > 4.9
definitions in the
nephrology literature
25% increase to at least 2.0 mg/dL within 48h
50% increase to at least 1.4 mg/dL
50% increase to at least 2.0 mg/dL

New consensus definition

• Increase in creatinine
of > 0.3* mg/dL in 48h
OR
• 1.5x baseline in 7d
OR
• Oliguria < 0.5ml/kg/h
x 6h

*26.5 µmol/L

4
Copyright © Harvard Medical School, 2018. All Rights Reserved.

AKI is deadly
70% increased odds of death

Chertow, G. M. et al. J Am Soc Nephrol 2005;16:3365-3370

AKI is increasingly common


• 5 to 7% of
admissions, up to
AKI incidence (/100,000 py)

50% of ICU pts


• Between 1988 and
2002: Four-fold
increase in AKI,
six-fold increase in
AKI-D (Waikar JASN 2006)
1Hsu Kidney Int 2007 • Community-based
estimate1: 522 AKI,
30 AKI-D per
100,000

5
Copyright © Harvard Medical School, 2018. All Rights Reserved.

AKI is expensive
• 60% increase1 in cost with post-CABG AKI
defined as 50% increase in SCr

• Additional $50002 when SCr increases by


> 0.3 mg/dL (26.5 µmol/L)

• $10 billion annually2

1Dasta et al. Nephrol Dial Transp 2008 2Chertow et al. JASN 2005

Approach to the
patient with AKI

6
Copyright © Harvard Medical School, 2018. All Rights Reserved.

Waikar & Bonventre, AKI chapter in Harrison’s Principles of Internal Medicine, 18th ed.

Approach to the
patient with AKI

• Pre-renal

• Intrinsic renal

• Post-renal

7
Copyright © Harvard Medical School, 2018. All Rights Reserved.

Check bladder scan,


insert Foley

Renal sonogram or CT

Pearl:
False negative u/s:
• Early obstruction
• Retroperitoneal fibrosis
or tumor (consider
retrograde pyelogram)

Waikar & Bonventre, AKI chapter in Harrison’s Principles of Internal Medicine , 18th edition

Pre-renal azotemia
• Serum creatinine increases due to
renal hypoperfusion
• No structural injury to kidney
• Recovery with restoration of
hemodynamics

Causes
• Hypovolemia
• Decreased cardiac output
• Decreased effective circ. volume
– Congestive heart failure, cirrhosis
• Impaired renal hemodynamics
– NSAIDs, ACE, ARB

Pearl: all NSAIDs including Cox-2 inhibitors;


Both ACE and ARB’s

8
Copyright © Harvard Medical School, 2018. All Rights Reserved.

Tubules Intratubular
Small vessels Ischemic ATN, sepsis, Myeloma
endogenous toxins proteins, uric
Glomerulonephritis, vasculitis,
(rhabdo, hemolysis), acid, debris,
TTP/HUS, DIC, atheroemboli,
exogenous toxins acyclovir, MTX.
HTN, sepsis
(contrast, cisplatin,
gent)

Interstitium
Large vessels Allergic (PCN, rifampin)
Renal artery embolus, Infection (severe pyelo,
dissection, vasculitis, Legionella, sepsis)
renal vein thrombosis,
Infiltration (lymphoma,
abdominal compartment
leukemia)
syndrome
Inflammatory (Sjogren’s,
sepsis, tubulointerstitial
nephritis with uveitis)

Waikar & Bonventre, AKI chapter in Harrison’s Principles of Internal Medicine , 18the

Common causes of AKI


Outpatient Inpatient International
• Pre-renal • Medical ICU: • Sub-Saharan
•ACE-I when ATN from sepsis, Africa
vomiting drugs • Malaria (1% of
•ACE-I + NSAID severe cases)
• Cardiac floor: • Diarrhea
• Obstruction contrast, cardiac
• BPH, stones surgery, cardio- • Tropics
renal • Leptospirosis

• SICU: • Others
rhabdomyolysis, • Post-strep GN
sepsis, postop ATN • Crush
syndrome

9
Copyright © Harvard Medical School, 2018. All Rights Reserved.

First steps
64M with HTN, DM, OA, BPH.
• Is this acute or chronic In ED with N/V.
Meds: ACE, NSAID, metformin
– Baseline SCr (if available)
Exam: BP 100/60, HR 104,
– Chronic RR 16; unrevealing exam
• Anemia (non-specific)
Labs: BUN 84, Creat 3.2 (283
• Small kidneys (< 8cm) umol/L), K 5.6, Bicarb 18
• “Increased echogenicity”
• How urgent is this?
– Hyperkalemia (K > 6.0, or rising fast)
– Anuria
– Severe hypoxemic respiratory failure
– Intoxication: methanol, ethylene glycol, salicylates,
lithium

Physical examination
• Pre-renal
– Orthostatic hypotension, tachycardia, decreased skin turgor, etc
– Hepato-renal: stigmata of liver disease
– Cardio-renal: signs of heart failure
• Post-renal GFR = 6000 ml/hour
– Palpable bladder. Pearl: normal urine output does not rule it out
• Intrinsic renal UFR = 100 ml/hour
– ATN: nonspecific (volume overload if present)
– Glomerulonephritis: (variable)
– Vasculitis: palpable purpura
– Atheroembolic disease: livedo reticularis, blue toes
– Interstitial nephritis: rash, fever (eos)—only 10% of cases

10
Copyright © Harvard Medical School, 2018. All Rights Reserved.

Studies
• Chem 7, BUN, creatinine, Ca, Phos, uric acid, CPK

• CBC with diff

• Renal ultrasound or computed tomography

• Urinalysis

• Urine sediment

• Fractional excretion of Na – pearl… BEWARE exceptions


– Traditional teaching: ATN > 1 or 2%; pre-renal < 1%
– Non-pre-renal with low FeNa: contrast, rhabdo, early sepsis,
obstruction, acute glomerulonephritis
– Pre-renal with high FeNa: diuretic use, pre-existing CKD

Case
64M with HTN, DM, OA, BPH. In ED with N/V.
Meds: ACE, NSAID, metformin
Exam: BP 100/60, HR 104, RR 16; unrevealing exam
Labs: BUN 84, Creat 3.2 (283 µmol/L), K 5.6, Bicarb 18

• Obtain baseline labs: creatinine 1.0 baseline


• Risk stratify: not emergent, but urgent
• Immediate considerations:
– Pre-renal azotemia (NSAID, ACE-I)
– Obstruction (history of BPH)
– Stop the metformin (note gap acidosis)
• Volume challenge, d/c ACE-I and NSAIDs, further evaluation of
obstruction and labs [U/A, urine sediment, FeNa]
• New appreciation of longer-term risks following an episode of AKI, even
if complete recovery

11
Copyright © Harvard Medical School, 2018. All Rights Reserved.

Further lab evaluation of


intrinsic AKI
• Pre-renal thought to be unlikely
• Post-renal ruled out
• No good cause for ATN
– No sepsis, surgery, nephrotoxin exposures,
contrast, rhabdomyolysis, tumor lysis,
ingestion, etc.

Further lab evaluation of


intrinsic AKI
• Pre-renal thought to be unlikely
• Post-renal ruled out
• No good cause for ATN
– No sepsis, surgery, nephrotoxin exposures,
contrast, rhabdomyolysis, tumor lysis,
ingestion, etc.
C3, C4 : depressed complements in diffuse GN (post-strep, lupus, cryo)
(also: atheromboli, TTP/HUS, sepsis, liver disease)
ANCA, antiGBM: time is nephrons.
ANA: lupus
LDH, haptoglobin: hemolysis, thrombotic microangiopathy
BIOPSY

12
Copyright © Harvard Medical School, 2018. All Rights Reserved.

Patterns of creatinine elevation


• Return to baseline after 48-72 hrs of fluid
replacement: pre-renal azotemia
• Rapid (> 2mg/dL per day) rise:
rhabdomyolysis [anecdotal]
• Rise within 24h, peak within 3-5d, return to
baseline ~1 wk: contrast nephropathy (note
typically non-oliguric)
• Aminoglycosides: rise after 5d therapy (can
be after discontinuation), return towards
baseline over 2-3 weeks (note, nonoliguric)

Complications of AKI
• Volume overload
• Hyperkalemia
• Metabolic acidosis
– GAP: retained anions (phosphate, urate, hippurate,
sulfate)
– NON-GAP: impaired distal H+ excretion
• Hypocalcemia, hyperphosphatemia
• Bleeding from uremic platelets
• Pericarditis
• Long term: increased risk of CKD, ESRD

13
Copyright © Harvard Medical School, 2018. All Rights Reserved.

Preventing AKI
• Avoidance of nephrotoxins
• Adequate intravascular volume
• N-acetylcysteine: no evidence for benefit
• Bicarbonate IVF: no benefit for contrast
nephropathy, conflicting data in septic shock
• Furosemide: observational studies suggest
harm, RCT suggests no benefit
– Clinical experience: widespread use

Treating AKI
• Pre-renal: improve hemodynamics
• Post-renal: relieve obstruction
• Intrinsic renal:
– Acute tubular necrosis: no proven therapies
• Specifically, no evidence for dopamine
– Acute interstitial nephritis: withdrawal of
suspected drug, ? Steroids
– Acute glomerulonephritis: (depends on type)
– Scleroderma renal crisis: ACE-inhibitors

14
Copyright © Harvard Medical School, 2018. All Rights Reserved.

Renal replacement for AKI


• “Indications”
–A
–E
–I
–O
–U

Renal replacement for AKI


• “Indications”
– Acidosis
– Electrolytes (K+)
– Ingestions (NOT digoxin)
– Overload (fluid)
– Uremia (pericarditis, encephalopathy)

15
Copyright © Harvard Medical School, 2018. All Rights Reserved.

Take home
• AKI is common, often deadly
• Pre-renal Post-renal Intrinsic renal
• Indications for dialysis
–A E I O U
• Patterns of creatinine elevation
– Pre-renal, contrast, rhabdo, aminoglycosides
• Beware the common exceptions to FeNa

Case 1
1. A 66-year-old previously healthy man is admitted to the hospital for crushing
substernal chest pain. Vital signs on admission were: blood pressure
126/78, pulse 102 beats per minute, respiratory rate 20 per minute.
Electrocardiogram reveals ST-segment elevation in leads II, III, and avf. He
is treated with aspirin, unfractionated heparin, metoprolol, nitroglycerin, and
captopril, and then taken to the cardiac catheterization laboratory where he
undergoes successful percutaneous coronary intervention of an acutely
occluded right circumflex artery. During the procedure the blood pressure
remained above 120/70, and he remained hemodynamically stable
thereafter. The serum creatinine concentration was 0.7 mg/dL (62 µmol/L)
on admission and rose to 1.4 mg/dL (124 µmol/L) on hospital day 3, when
captopril was discontinued. The serum creatinine rose to 7.8 mg/dL (690
µmol/L) by hospital day 9, when hemodialysis was initiated. Skin
examination was notable for livedo reticularis.
The renal vessels most likely involved in the pathophysiology of his acute
kidney injury are the:
• A) afferent arterioles
• B) efferent arterioles
• C) interlobular veins
• D) interlobar veins

16
Copyright © Harvard Medical School, 2018. All Rights Reserved.

Explanation
• This patient’s presentation is consistent with renal
atheroemboli. Angiography is the most common
triggering event, and anticoagulation is also a risk factor.
The clinical course is variable and includes subacute
kidney injury weeks later or severe acute kidney injury.
Urinary findings are relatively non-specific, and heavy
proteinuria is not common. Eosinophilia and
hypocomplementemia may be seen. Cholesterol crystals
lodge in small arteries of 150-200 mm in diameter
(arcuate; interlobular; occasionally afferent arterioles or
glomeruli). The answer is A. The other answers are
post-glomerular vessels which are not involved in
cholesterol embolization.

Case 2
2. A 75 year old man is admitted to the hospital for severe diarrhea for the
past four days. The past medical history is notable for hypertension and
osteoarthritis. His medications included lisinopril 40 mg daily, ibuprofen
800 mg three times daily, and metoprolol 50 mg daily. The physical
examination is notable for BP 100/60, pulse 120 beats per minute, and
decreased skin turgor. Labs show serum creatinine of 4.6 mg/dL (407
µmol/L), potassium 5.8 meq/L, and fractional excretion of Na of 0.4%.
In addition to pre-renal azotemia, the following causes of acute kidney
injury can be associated with a fractional excretion of sodium below
1.0%:
• A) rapidly progressive glomerulonephritis
• B) rhabdomyolysis
• C) contrast nephropathy
• D) none of the above
• E) All of the above (A, B, and C)

17
Copyright © Harvard Medical School, 2018. All Rights Reserved.

Explanation
• The answer is E. Although the fractional excretion of
sodium is one of the most commonly ordered tests for
the differential diagnosis of acute kidney injury and is
often presumed to denote pre-renal azotemia, low FeNa
can be seen in a number of other clinical settings. These
include rapdily progressive glomerulonephritis,
rhabdomyolysis, and contrast nephropathy.

References
• Chertow, GM, Burdick, E, Honour, M, Bonventre, JV & Bates, DW:
Acute kidney injury, mortality, length of stay, and costs in
hospitalized patients. J Am Soc Nephrol, 16: 3365-70, 2005.
• Wald, R, Quinn, RR, Luo, J, Li, P, Scales, DC, Mamdani, MM & Ray,
JG: Chronic dialysis and death among survivors of acute kidney
injury requiring dialysis. Jama, 302: 1179-85, 2009.
• Blantz, RC: Pathophysiology of pre-renal azotemia. Kidney Int, 53:
512-23, 1998.
• Friedrich, JO, Adhikari, N, Herridge, MS & Beyene, J: Meta-analysis:
low-dose dopamine increases urine output but does not prevent
renal dysfunction or death. Ann Intern Med, 142: 510-24, 2005.
• Steiner, RW: Interpreting the fractional excretion of sodium. Am J
Med, 77: 699-702, 1984

18
Copyright © Harvard Medical School, 2018. All Rights Reserved.

Disclosures
None relevant

19
Copyright © Harvard Medical School, 2018. All Rights Reserved.

CKD:
The Story of
my Seventy-Eight Year
Old Patient
Ajay K. Singh, MB, FRCP
Renal Division
Brigham and Women’s Hospital
Senior Associate Dean,
Postgraduate Medical
Education,
Harvard Medical School

Disclosures
GSK - Consultant

20
Copyright © Harvard Medical School, 2018. All Rights Reserved.

Case History
• 78 year old African American man T2D, HTN
• Scr of 1.32 mg/dL. eGFR is >60
ml/min/1.73m2.
• He is otherwise healthy.
• Past medical history - hypertension ≈15
years duration, type 2 diabetes mellitus for
8 year; Hypercholesterolemia treated with
atorvastatin.
• BP-- in the 150-160 mmHg range.
• Medications: lisinopril, ASA,
hydrochlorthiazide, atorvastatin, metformin,
glipizide.

Case History
• PE: BP 152/68 mmHg, HR 72 bpm. Weight
120 kg JVP 8 cm. Rest of the examination
negative.

• Lab values
• Dipstick urinalysis shows SG 1015, pH 6.0, trace
to 1+ alb, rest is negative
• BUN 28, Serum creatinine 1.32 mg/dL, HbA1C
7.8

• Does he have kidney disease?

21
Copyright © Harvard Medical School, 2018. All Rights Reserved.

We don’t want to call something a


disease when it isn’t

x
x
Lindeman, R. D., J. D. Tobin, and N. W. Shock. Longitudinal
studies on the rate of decline in renal function with age. J.
Am. Geriatr. Soc. 33: 278–285, 1985.

1. Does he have risk factors for CKD?


• Susceptibility Risk Factors
• Diabetes
• Hypertension
• Older age
• Family history of CKD
• Racial or ethnic minority
• Other: low income, minimal education, kidney-mass
reduction, known kidney disease

Levey et al. Ann Intern Med. 2003;139:137-147..


6

22
Copyright © Harvard Medical School, 2018. All Rights Reserved.

The Risk of Kidney Failure Is Not Uniform

Relative risks compared to Whites:

African Americans 4.45 X


Native Americans 3.57 X
Asians 1.59 X

Xue, et al., 2000

2. Obtain laboratory Data


a.) UA dipstick /Sediment
b.) Renal US

23
Copyright © Harvard Medical School, 2018. All Rights Reserved.

Is there Proteinuria-Albuminuria?
• Urine Dip a commonly used screening test
• Use ACR for quantification
Result Significance
Negative Unlikely to be proteinuria present
Trace 15-30mg/dL
1+ 30-100mg/dL
2+ 100-300mg/dL
3+ 300-1000mg/dL
4+ >1000mg/dL

Assess albuminuria – ACR

Risk for progression


or CVD

24
Copyright © Harvard Medical School, 2018. All Rights Reserved.

3. Evaluate Risk of Kidney Failure


Kidney Failure Risk Equation (KFRE)
Tangri’s laboratory based equation to predict
CKD progression
Accurately predicts CKD stage 3-5 progression
over 5 years (C statistic 0.84-0.91)

3 variable KFRE – Age, Sex, eGFR


4 variable KFRE – Age, Sex, eGFR, ACR
8 variable KFRE + Ca, Phos, Bicarb, alb

Tangri et al JAMA 2011, 2016

Rate of GFR Decline in Normals and CKD


Patients
0 10 20 30 40 50 60 70 80 90 100

1ml/min/year starting at age 45yrs


(normal ageing)
GFR ml/min

≈ 2-3 ml/min/year

ESRD
25 35 45 55 65 75
Age, years
Collister D, http://www.sciencedirect.com/science/article/pii/S027092951630033X

25
Copyright © Harvard Medical School, 2018. All Rights Reserved.

Manage Progression factors for CKD?


• Susceptibility • Progression
Risk Factors Factors
• Diabetes • Higher level of
albuminuria
• Hypertension
• Higher BP
• Older age
• Poor glycemic
• Family history of CKD control
• Racial or ethnic • Smoking
minority
• Hyperlipidemia
• Other: low income,
minimal education, • Drug use
kidney-mass reduction,
known kidney disease
Levey et al. Ann Intern Med. 2003;139:137-147.

Algorithm for this 78 year old man


eGFR ml/min/1.73m2

Does he have risk factors?


HTN, DM
High risk group

UA and sediment?

Renal US ACR

Does he have CKD? = YES


NEXT STEPS>>>>

26
Copyright © Harvard Medical School, 2018. All Rights Reserved.

Keys to managing patients with CKD

• Screen patients at high risk


• Patients with specific conditions (diabetics, HTN,
metabolic syndrome), Specific populations
(African Americans, Latinos)
• Estimate kidney function, by converting Scr
to eGFR; use UACR to stratify risk
• Treat BP, Manage diabetes
• Use ACEI/ARB, SGLT2 inhibitor (T2D)
• Consider other interventions: low protein
diet, statin

ACEi and ARBs; CCBs


• ACEi and ARB have similar antiproteinuric
effects
• Combination of ACEi and ARB: 20% reduction in
albuminuria – over and above either agent alone
(Addition of Aliskiren to maximum dose of ARB
reduces further albuminuria by 20%)
• Dual blockade not recommended
• CCBs - Strong antihypertensive agents.
• Dihydropyridine CaCBs: No effect on CKD
progression ( in absence of RAAS blockade) eg
amlodipine
• Non-dihydropyridine CaCBs: Reduce albuminuria
(use if RAAS are not tolerated) eg diltiazem

27
Copyright © Harvard Medical School, 2018. All Rights Reserved.

What about Combo therapy:


ACEi+ARB? NO…..ONTARGET
• 25,620 patients with either pre-existing vascular disease or
diabetes randomly assigned ramipril, telmisartan, or the
combination of the two
• No hint of benefit for CV endpoint with combination therapy
• Trend toward increased risk of death, 1.07 (0.98-1.16), with
combination therapy
• Many more adverse events
New Eng J Med 2008; 358:1547

•Secondary analysis in 5623 patients with eGFR<60 and/or


albuminuria
•Combination therapy reduced albuminuria over
monotherapy
•Combination therapy significantly increased ESRD or
doubling of Scr (0.79 vs. 0.56 % per year)
Circulation 2011; 123:1098

Nevertheless, multiple BP agents


required
No. of Antihypertensive Agents
Trial Target BP, mm Hg 1 2 3 4

UKPDS DBP <85


ABCD DBP <75
MDRD MAP <92
HOT DBP <80
AASK MAP <92
IDNT <135/ <85
ALLHAT <140/ <90

ABCD=Appropriate Blood Pressure Control in Diabetes; MDRD=Modification of Diet in Renal


Disease; HOT=Hypertension Optimal Treatment ; IDNT=Irbesartan Diabetic Nephropathy Trial;
DBP=diastolic blood pressure; MAP=mean arterial pressure; SBP=systolic blood pressure.

Bakris et al Am J Kidney Dis. 2000;36:646-661; Lewis et al N Engl J Med. 2001;345:851-860;


Cushman et al J Clin Hypertens. 2002;4:393-404.

28
Copyright © Harvard Medical School, 2018. All Rights Reserved.

Therapy for Hypertension based on JNC 8


Implement lifestyle interventions

No diabetes and/or CKD Diabetes and/or CKD

Age >60 Age <60 All ages

BP Goal BP Goal BP Goal


<150/90 <140/90 <140/90

Only diabetes Only CKD

Blacks <75 y >75 y


Non-Blacks

Thiazide, ACEi, ARB or CCB Thiazide or CCB ACEi or ARB CCB or Thiazide
alone or in combination alone or in combination

Source: Singh et al, Brigham Intensive Review of Internal Medicine, Oxford University Press 2014
Adapted from James PA et al: Report from the panel members appointed to the Eighth Joint National Committee (JNC 8).
JAMA 2014; DOI:10.1001/jama.2013.284427.

72.2m
32%

103.3m
46%

29
Copyright © Harvard Medical School, 2018. All Rights Reserved.

Guideline not based on RCTs


Updates JNC7 not JNC 8
Creates “disease” in ≈30 million Americans
• 32% to 46% of Americans now have HTN
• Those with “pre-hypertension, now hypertension
Integrates cardiovascular risk assessment into BP
management
BP goal same in age>65
SOURCE: Bakris G and Sorrentino M, NEJM Feb 8 2018
Muntner P, Carey RM, Gidding S, et al. Potential US population impact of the 2017
ACC/AHA high blood pressure guideline. Circulation 2018;137:109-118.

Does the SPRINT Study apply to this


patient?....No because SPRINT excluded
diabetics • RCT, N=9361, median FU
3.26 y
• Study population: pts with
SBP>130 and increased
CVD risk but not diabetic
• SBP<120 vs. <140 mmGH
• Cardiovascular composite
(MI, other coronary
syndromes, CHF,
stroke, CVD death)

30
Copyright © Harvard Medical School, 2018. All Rights Reserved.

Newer therapy for renal progression in


T2 Diabetics: YES… SGLT2 inhibitors

In clinical trials
• Average baseline HbA1c of about 7.5-8.5%, SGLT2 inhibitors reduced HbA1c by 0.5-1.5%
without inducing hypoglycaemia
• Accompanied by weight loss of 2-3kg, reflecting calorie loss via renal glucose excretion
(loss of 80-85g glucose per day) with initial changes due to altered fluid balance
plateauing at around 6 months
• SGLT2 inhibitor treatment induces an osmotic diuresis – voiding up to an extra
400ml/day, with unchanged natraemia – and a decrease in systolic blood pressure (2-
5mmHg).r

NEJM June 14, 2016


• RCT, N=7020, 590 sites, 42 countries
• Patients with T2D eGFR >30 ml/min/1.73m2
• Empagliflozin (10 or 25 mg/d) or placebo
• 39% reduction in incident or worsening nephropathy (HR 0.61,
P<0.001)
• 44% risk reduction in doubling of serum creatinine
• 55% lower relative risk of initating renal replacement therapy

31
Copyright © Harvard Medical School, 2018. All Rights Reserved.

A Change in eGFR over 192 Wk


78

Adjusted Mean eGFR (ml/ min/ 1.73 m2)


76

74
Empagliflozin, 10 mg

72
Empagliflozin, 25 mg

70
Placebo
68

66
Baseline 4 12 28 52 66 80 94 108 122 136 150 164 178 192
Week
No. at Risk
Placebo 2323 2295 2267 2205 2121 2064 1927 1981 1763 1479 1262 1123 977 731 448
Empagliflozin, 10 mg 2322 2290 2264 2235 2162 2114 2012 2064 1839 1540 1314 1180 1024 785 513
Empagliflozin, 25 mg 2322 2288 2269 2216 2156 2111 2006 2067 1871 1563 1340 1207 1063 838 524
No. in Follow-up
Analysis
Total 7020 7020 6996 6931 6864 6765 6696 6651 6068 5114 4443 3961 3488 2707 1703

Wanner C, Inzucchi SE, Lachin JM, et al. Empagliflozin and


progression of kidney disease in type 2 diabetes. N Engl J
Med. DOI: 10.1056/NEJMoa1515920.

Summary

- Use GFR equations and albuminuria to


make diagnosis of CKD
- Use a kidney failure risk equation
- Use ACEi/ARB, but not in combination
- Consider adding SGLT2 inhibitor for T2DN,
but with modest-to-well preserved GFR
- Use multiple agents for BP control

32
Copyright © Harvard Medical School, 2018. All Rights Reserved.

References
Wanner C, Inzucchi SE, Lachin JM, et al.
Empagliflozin and progression of kidney
disease in type 2 diabetes. N Engl J Med. DOI:
10.1056/NEJMoa1515920.
Lindeman, R. D., J. D. Tobin, and N. W. Shock.
Longitudinal studies on the rate of decline in
renal function with age. J. Am. Geriatr. Soc. 33:
278–285, 1985.
Levey et al. Ann Intern Med. 2003;139:137-
147..
Collister D,
http://www.sciencedirect.com/science/article/pii
/S027092951630033X

33
Copyright © Harvard Medical School, 2018. All Rights Reserved.

Revisiting Electrolytes and Acid-


Base Basics
Bradley M. Denker, MD.
Clinical Chief,
Renal Division, Department of Medicine
Beth Israel Deaconess Medical Center and
Harvard Vanguard Medical Associates
Associate Professor of Medicine
Harvard Medical School

Financial disclosures
Bradley M. Denker, MD

No conflict of interest to disclose.

34
Copyright © Harvard Medical School, 2018. All Rights Reserved.

Outline
I. Na+ disorders
- Hyper- and hyponatremia
II. K+ disorders
- Hyper- and hypokalemia
III. Acid-base disorders
- General approach
- Metabolic acidosis & alkalosis

Na+ disorders
[Na+]= Amount of Na+/Amount of H2O

Renin/Angiotensin/Aldo ADH
(RAS)

Plasma Osmolality ~ 2x [Na+]


Intracellular: Extracellular (plasma/interstitial):
[K+]=145mEq/L [Na+]=145mEq/L
[Na+]=5mEq/L [K+]=5mEq/L

35
Copyright © Harvard Medical School, 2018. All Rights Reserved.

Hypernatremia
[Na+]= Amount of Na+/Amount of H2O

Na+/H2O
or
Na+/ H2O
Renal water loss versus other sources

Hypernatremia

UOsm

< 800 mOsm/kg > 800 mOsm/kg


• Renal H2O loss • Insensible H2O loss
• GI H2O loss
+
Diabetes Osmotic
insipidus diuresis ↓ Water intake
• Na+ Intake
CDI NDI Glucose, urea,
ADH ADH mannitol
Deficiency Resistance

36
Copyright © Harvard Medical School, 2018. All Rights Reserved.

Nephrogenic diabetes insipidus


Hypokalemia
Hypercalcemia
Tubulointerstitial nephropathies
Sickle cell disease
Myeloma
Obstructive uropathy
Recovery from ATN or obstruction
Lithium

Chronic renal failure

Distinguishing central from nephrogenic DI

Water deprivation test

DDAVP (desmopressin)

↑UOsm No ∆ in UOsm
CDI NDI

37
Copyright © Harvard Medical School, 2018. All Rights Reserved.

Management of hypernatremia
Replace free water deficit (50% in first 24 hr,
no more than 0.5 mM/hr)
0.4-0.5 x BW(kg) x (SNa/140-1)

Replace ongoing free water losses


Treat underlying cause
Desmopressin for CDI
No specific Rx for NDI (attempt to reduce urine
output with Na restriction, thiazides or give
supratherapeutic dose of desmopressin)

Hyponatremia
[Na+]= Amount of Na+/Amount of H2O

Na+/H2O - Volume Depletion


or
Na+/ H2O - Water Excess;
or Euvolemic

Na+/ H2O - Volume Depletion


+ Hypotonic Fluids

RAS ADH

38
Copyright © Harvard Medical School, 2018. All Rights Reserved.

Hyponatremia

Posm

> 290 mOsm/kg Normal < 275 mOsm/kg


“Pseudohyponatremia”

↑Glucose* Lipid ↑ Hypoosmolal


Mannitol hyponatremia*
Protein ↑
*Requires ADH+
*Correct serum Na+ by 1.6
for every 100 mg/dL ∆ in glucose Water Intake

Next Slide

Hypoosmolal hyponatremia
Volume status

Hypovolemic Euvolemic Edematous


Dehydration* Psych. polydipsia† CHF*
Addison’s SIADH Liver failure*
Diuretics Hypothyroid

* UNa < 20 = Extrarenal cause of ECV depletion


†U < 100 = ADH appropriately suppressed
Osm

39
Copyright © Harvard Medical School, 2018. All Rights Reserved.

Plasma ADH (pg/mL)

Plasma Osmolality Blood Vol Depletion (%)

Rx of hyponatremia
Hypovolemia Isotonic saline

Polydipsia Water restriction

SIADH Water restriction


Hypertonic saline / Na tablets
Furosemide
Aquaretics (“vaptans”)

40
Copyright © Harvard Medical School, 2018. All Rights Reserved.

Rate of correction of hyponatremia


Acute (< 48 hr, usually due to hypotonic fluid
intake) or severely symptomatic
100 mL of 3% saline bolus to increase SNa by
2-3 mEq/L

Chronic (> 48 hr) including SIADH and


asymptomatic
0.5 mEq/l per hour

Do not exceed ∆6-8 mEq/L in 1st day

Osmotic demyelination syndrome


Central and extrapontine myelinolysis

Risk factors :
Excessive rate or amount of correction of serum Na+

Classic CPM presents with dysphagia,


quadriparesis, locked-in syndrome

Can be permanent or fatal

41
Copyright © Harvard Medical School, 2018. All Rights Reserved.

Sample board review question 1


A 26 yr-old otherwise healthy male presents with
seizure.

Na 115, K 3.5, Cl 88, CO2 23, BUN 5, Cr 0.7


Urine: Na 30 mEq/L, Osm 45 mOsm/kg

What is the most likely diagnosis:


A. SIADH
B. Hypothyroidism
C. Psychogenic polydipsia
D. Hepatic cirrhosis
E. Adrenal insufficiency

Hypoosmolal hyponatremia
Volume status

Hypovolemic Euvolemic Edematous


Dehydration* Psych. polydipsia† CHF*
Addison’s SIADH Liver failure*
Diuretics Hypothyroid

†U < 100 = ADH appropriately suppressed


Osm

* UNa < 20 = Extrarenal cause of ECV depletion

42
Copyright © Harvard Medical School, 2018. All Rights Reserved.

K+ disorders

Hyperkalemia

43
Copyright © Harvard Medical School, 2018. All Rights Reserved.

Pseudohyperkalemia
Hemolysed blood sample
Leukocytosis/thrombocytosis
Check EKG, whole blood potassium (e.g. blood gas
analyzer)

Hyperkalemia

↑ Intake Cell shift


Metabolic acidosis
Decreased urinary Hyperglycemia
K+ excretion β-blocker
24 hr urine K+ < 40 mEq Digitalis
Hyperkalemic
Next Slide periodic paralysis
Cell lysis

44
Copyright © Harvard Medical School, 2018. All Rights Reserved.

Decreased urinary K+ excretion

↓GFR ↓CCD [K+]


Renal
failure Next Slide

Cortical Collecting Duct (CCD)

Lumen Blood

45
Copyright © Harvard Medical School, 2018. All Rights Reserved.

Decreased urinary K+ excretion

↓GFR ↓CCD [K+]


Renal
failure Meds Adrenal Hyporenin
insufficiency hypoaldo
NSAIDs
ACEI/ARB Addison’s
Block Heparin
RAAS Spironolactone Next Slide
Cyclosporine

Block Na+ Amiloride


channel Trimethoprim
Pentamidine

Type IV RTA (hyporeninemic


hypoaldosteronism)

Hyperkalemia (disproportionate to level of GFR)


Non-gap metabolic acidosis with normal urine
acidifying ability
Mild CKD
Often underlying tubulointerstitial disease:
- DM
- SLE, obstruction, myeloma/amyloid, HIV etc.
- NSAIDs

46
Copyright © Harvard Medical School, 2018. All Rights Reserved.

Treatment of hyperkalemia
Stabilize membrane excitability
Calcium chloride or gluconate, 1 g IV
Increase K+ entry into cells
Glucose 25 g and insulin 10 U
β2-adrenergic agonist (albuterol 10-20 mg inh)
NaHCO3
Removal of excess K+
Cation exchange resin (Kayexalate)
Diuretics
Dialysis
Dietary K+ restriction

Hypokalemia

47
Copyright © Harvard Medical School, 2018. All Rights Reserved.

DDX of hypokalemia

Cellular shift GI cause Urinary K wasting


Alkalemia
Insulin
β-agonist
Hypokalemic periodic
paralysis

Next Slide

Features suggestive of hypokalemic


periodic paralysis

+FH or Asian male with thyrotoxicosis

Precipitated by meal or exercise

Repetitive episodes of acute profound hypokalemia

Recovery of serum K+ within hrs after each episode


without repletion, either spontaneously or with
propanolol

Low urine K+

48
Copyright © Harvard Medical School, 2018. All Rights Reserved.

DDX of hypokalemia

Cellular shift GI cause Urinary K wasting


Alkalemia Vomiting 24 hr UK > 25 mEq
Insulin Diarrhea Random UK/Creat <
β-agonist 13 mEq/g
Hypokalemic periodic
paralysis

DDX of hypokalemia

Cellular shift GI cause Urinary K wasting


Alkalemia Diarrhea 24 hr UK > 25 mEq
Insulin
β-agonist Vomiting
Hypokalemic periodic
paralysis

49
Copyright © Harvard Medical School, 2018. All Rights Reserved.

Hypokalemia/Renal K+
wasting & hypertension

Renin
AI AII Aldosterone
Substrate Renin Conv Enz

Na+ Abs
K+ Excretion
Renal Hypo-perfusion

HYPERTENSION

Hypokalemia/Renal K+
wasting & hypertension
Aldosterone
High Low
Renin

High Low

Renal artery Primary Cushing’s


stenosis hyperaldosteronism Liddle's
Reninoma Liquorice ingestion
(very rare)
Renin
AI AII Aldosterone
Substrate
Renin Conv Enz

50
Copyright © Harvard Medical School, 2018. All Rights Reserved.

Renal K wasting with normal or low BP

Osmotic Drugs Vomiting RTA Inherited


diuresis tubulopathy
Diuretics Proximal Bartter
Loop & thiazide Classic distal Gitelman
Cisplatin
Aminoglycosides
Ticarcillin Renal NaCl wasting
Amphotericin
Toluene

Use of the urine chloride


In the setting of metabolic alkalosis, urine Cl-
is a more reliable marker of hypovolemia than
urine Na+
Low urine Cl- indicates hypovolemia due to
extrarenal (GI) cause
High urine Cl- in the setting of
hypovolemia/euvolemia suggests renal salt
wasting

51
Copyright © Harvard Medical School, 2018. All Rights Reserved.

Cryptogenic hypokalemic metabolic


alkalosis

Volume Urine
Urine Cl-
status/BP diuretics
Hyperaldosteronism ↑ > 40 mEq/L -
Surreptitious
Nl or ↓ < 25 mEq/L -
vomiting
Diuretic abuse Nl or ↓ > 40 mEq/L +
Bartter/Gitelman
Nl or ↓ > 40 mEq/L -
syndrome

Sample board review question 2


A 75 yr-old male with known coronary and peripheral
vascular disease presents with worsening hypertension
despite treatment with HCTZ, amlodipine and
candesartan.
Na 141, K 3.0, Cl 108, CO2 29, BUN 25, Cr 1.9

Which diagnostic test is most likely to be useful?


A. Urine diuretic screen
B. Genetic test for mutations in NKCC2
C. CT scan of the adrenal glands
D. Urine metanephrines
E. Doppler ultrasound of the renal arteries

52
Copyright © Harvard Medical School, 2018. All Rights Reserved.

Hypokalemia/Renal K+
wasting & hypertension

Renin
AI AII Aldosterone
Substrate Renin Conv Enz

Na+ Abs
K+ Excretion
Renal Hypo-perfusion

HYPERTENSION

Acid-base disorders

53
Copyright © Harvard Medical School, 2018. All Rights Reserved.

General approach

Approach
1. Is there acidemia or alkalemia?

2. What is the primary process (metabolic


or respiratory, acidosis or alkalosis)?

3. Is there an appropriate compensatory


response?

54
Copyright © Harvard Medical School, 2018. All Rights Reserved.

1. Is there acidemia or alkalemia?

Acidemia pH < 7.35

Alkalemia pH > 7.45

2. What is the primary process?


Metabolic
processes

HCO3-
pH = 6.1 + log 0.03 x PCO
2

CO2 + H2O ↔H2CO3 ↔ H+ + HCO3–


Respiratory
processes

55
Copyright © Harvard Medical School, 2018. All Rights Reserved.

-
pH HCO3 ; PCO2 Primary disorder
Acidemia ↓ HCO3- Metabolic acidosis

↑ PCO2 Respiratory acidosis

↑ HCO3
-
Alkalemia Metabolic alkalosis

↓ PCO2 Respiratory alkalosis

HCO3-
pH = 6.1 + log 0.03 x PCO
2

3. Is there an appropriate
compensatory response?

Metabolic processes Respiratory processes


Metabolic
acidosis

↓ HCO3-

“Respiratory
alkalosis”
↓ PCO2

56
Copyright © Harvard Medical School, 2018. All Rights Reserved.

3. Is there an appropriate
compensatory response?

Metabolic processes Respiratory processes


“Metabolic
acidosis”

↓ HCO3-

Respiratory
alkalosis
↓ PCO2

3. Is there an appropriate
compensatory response?

Metabolic processes Respiratory processes


“Respiratory
acidosis”
↑ PCO2

Metabolic
alkalosis

↑ HCO3-

57
Copyright © Harvard Medical School, 2018. All Rights Reserved.

3. Is there an appropriate
compensatory response?

Metabolic processes Respiratory processes


Respiratory
acidosis
↑ PCO2

“Metabolic
alkalosis”

↑ HCO3-

Compensatory mechanisms

Remember the direction of compensation


Remember that compensation is almost
never complete

58
Copyright © Harvard Medical School, 2018. All Rights Reserved.

Metabolic acidosis

Metabolic acidosis

Ingestion Endogenous Defective acid Loss of


of acid generation excretion alkali
of acid

59
Copyright © Harvard Medical School, 2018. All Rights Reserved.

Metabolic acidosis

Ingestion Endogenous Defective acid Loss of


of acid generation excretion alkali
of acid

Ethylene glycol Oxalic acid/glycolic acid


Methanol Formic acid
Toluene Hippuric acid
Salicylic acid

Metabolic acidosis

Ingestion Endogenous Defective acid Loss of


of acid generation excretion alkali
of acid

Lactic acidosis Diabetes mellitus


Ketoacidosis
Alcohol
Rhabdomyolysis

60
Copyright © Harvard Medical School, 2018. All Rights Reserved.

Metabolic acidosis

Ingestion Endogenous Defective acid Loss of


of acid generation excretion alkali
of acid

Renal failure
Distal renal tubular acidosis

Metabolic acidosis

Ingestion Endogenous Defective acid Loss of


of acid generation excretion alkali
of acid

Diarrhea
Proximal RTA

61
Copyright © Harvard Medical School, 2018. All Rights Reserved.

Serum anion gap


[Na+] - ([Cl-] + [HCO3-])
= Unmeasured anions - Unmeasured cations
(Normal range: 8 - 12)

Serum anion gap


[Na+] - ([Cl-] + [HCO3-])
= Unmeasured anions - Unmeasured cations
(Normal range: 8 - 12)

62
Copyright © Harvard Medical School, 2018. All Rights Reserved.

High anion gap metabolic acidosis


Glycols (ethylene, propylene (lorazepam)
Oxyproline – paracetamol/women
L-Lactate
D-Lactate – short bowel syndrome
Methanol Mehta et.al,
The Lancet, Volume 372, Issue 9642,
Aspirin
Page 892, 13 September 2008
Renal Failure
Ketosis – starvation, alcohol, diabetic

Anion and osmolal gap in


diagnosis of intoxications

Anion gap
acidosis Osmolal gap

+ Normal Salicylates

Ethanol
Ethylene glycol
+ High
Propylene glycol
Methanol

- High Isopropanol

63
Copyright © Harvard Medical School, 2018. All Rights Reserved.

Serum osmolal gap


Osmolal gap = Measured Sosm - Calc Sosm

Calculated Sosm :
2 [Na+] + [glucose]/18 + [BUN]/2.8

Clues to high anion gap acidosis


syndromes

Alcoholic fetor
Papilledema
Osmolar gap
Undetectable serum ethanol

Methanol intoxication

64
Copyright © Harvard Medical School, 2018. All Rights Reserved.

Clues to high anion gap acidosis


syndromes

No fetor
Osmolar gap
Calcium oxalate dihydrate (envelope-
shaped) crystalluria
Urine fluoresces under Wood's (UV) lamp

Ethylene glycol intoxication

Clues to high anion gap acidosis


syndromes

Tinnitus/deafness
Fever, tachycardia, hyperventilation
Associated respiratory alkalosis and
metabolic acidosis

Salicylate intoxication

65
Copyright © Harvard Medical School, 2018. All Rights Reserved.

Clues to high anion gap acidosis


syndromes

Normal glucose
Serum Acetest/acetoacetate negative or
borderline
Serum β-hydroxybutyrate positive
Serum ethanol may or may not be present

Alcoholic ketoacidosis

Clues to high anion gap acidosis


syndromes

Didanosine or stavudine use


2 mth - 2 yr after start of Rx
± concurrent tenofovir use

Lactic acid elevated

Type B lactic acidosis 2° to NRTI

66
Copyright © Harvard Medical School, 2018. All Rights Reserved.

Clues to high anion gap acidosis


syndromes

Short bowel syndrome


Episodes of ∆MS associated with AG
metabolic acidosis, after CHO intake
Spontaneous resolution if NPO
Serum lactic acid level negative

D-lactic acidosis

Clues to high anion gap acidosis


syndromes

ICU patient sedated with high dose


intravenous infusion of lorazepam
Osmolar gap
Elevated serum lactic acid level

Propylene glycol intoxication

67
Copyright © Harvard Medical School, 2018. All Rights Reserved.

DDx of a non-gap metabolic acidosis

Diarrhea RTA
(bicarb loss)

I II IV
Classic distal Proximal Hyporeninemic
(bicarb loss) hypoaldosteronism

DDx of a non-gap metabolic acidosis

68
Copyright © Harvard Medical School, 2018. All Rights Reserved.

DDx of RTA
Proximal Classic distal Hyporenin
hypoaldo
Serum K Low Low High
Urine pH Variable > 5.5 < 5.5
Other Fanconi (low Nephrocalcinosis
features PO4, glycosuria) ± CaPO4 stones

Causes and Rx of RTA


Proximal Classic distal Hyporenin
hypoaldo
Common Ifosfamide Sjogren’s CKD plus:
causes NRTI (tenofovir, SLE DbM
adefovir, cidofovir) Amphotericin Obstruction
Myeloma Sickle cell dz
SLE
NSAIDs
Rx Bicarbonate (lots) Bicarbonate K+ lowering Rx:
(1 mEq/kg/day) Diuretics
Kayexalate
Low K diet

Mineralocorticoid

69
Copyright © Harvard Medical School, 2018. All Rights Reserved.

Metabolic alkalosis

Induction of metabolic alkalosis

Ingestion Loss of acid Cellular


of alkali shift
Antacids ↓ K+
GI loss Renal loss
Blood Tx
Vomiting Diuretics
NG suction Bartter/Gitelman
Hyperaldosteronism

70
Copyright © Harvard Medical School, 2018. All Rights Reserved.

Maintenance of alkalosis
Requires impairment of renal excretion of
excess bicarbonate:

• Volume contraction (e.g. vomiting, diuretics)


• Hypokalemia
• Renal failure
• Hyperaldosteronism

Changes with Metabolic Alkalosis


Excreted Bicarb exceeds reabsorption
obligate Na/K wasting in the urine
Increased Aldosterone stimulates Na+
reabsorption until no Cl- remaining in urine
Aldo stimulates K+ /H+ ATPase exacerbating
hypokalemia and metabolic alkalosis

71
Copyright © Harvard Medical School, 2018. All Rights Reserved.

Changes with Metabolic Alkalosis

UNa+ UK+ UCl- pH


(bicarb)
Day 1-3

>3 days

Correction of Alkalosis Requires Cl-


to Allow HCO3- Excretion

72
Copyright © Harvard Medical School, 2018. All Rights Reserved.

Cryptogenic hypokalemic metabolic


alkalosis

Volume Urine
Urine Cl-
status/BP diuretics
Hyperaldosteronism ↑ > 40 mEq/L -
Surreptitious
Nl or ↓ < 25 mEq/L -
vomiting
Diuretic abuse Nl or ↓ > 40 mEq/L +
Bartter/Gitelman
Nl or ↓ > 40 mEq/L -
syndrome

Take Home Messages (1)


Hypo- and Hyper-Natremia are usually water
imbalances;
Volume depletion (Na loss) stimulates RAS
Water depletion (hypersomolality) stimulates
ADH
Potassium Disorders
Most K is intracellular (intake/cellular shift)
Renal K excretion is regulated by GFR; Aldo
and UNa

73
Copyright © Harvard Medical School, 2018. All Rights Reserved.

Take Home Messages (2)


Assess pH (emia), PCO2 (Resp) and HCO3-
(metabolic)
Compensation is opposite process and direction
but not to normal pH.
Metabolic Acidosis
Elevated AG; addition of H+ with non-Cl- anion
Normal AG; addition of H+Cl- OR Bicarb loss
(GI or Renal)
Metabolic Alkalosis-vomiting and diuretics

Suggested reading
Rennke, H.G., Denker, B.M., Renal Pathophysiology – The
Essentials, 4th Edition, Lippincott Williams & Wilkins, 2014

Mount, D.B., Fluid and Electrolyte Distrubances. In Harrison's


Principles of Internal Medicine, 18th Edition, Eds. Longo, Fauci, et
al., McGraw-Hill, p. 341-359

DuBose, T.D.,Jr. Acidosis and Alkalosis. In Harrison's Principles of


Internal Medicine, 18th Edition, Eds. Longo, Fauci, et al., McGraw-
Hill,p. 363-373

74
Copyright © Harvard Medical School, 2018. All Rights Reserved.

Approach to Proteinuria
and Hematuria
5 Clinical Scenarios Common to the Boards

Ajay K. Singh, MB., FRCP


Physician, Renal Division,
Brigham and Women’s Hospital,
Senior Associate Dean
for Postgraduate Medical Education,
Harvard Medical School

Disclosures

• GSK - Consultant

75
Copyright © Harvard Medical School, 2018. All Rights Reserved.

Why Are Proteinuria and


Hematuria Important?
• As markers of kidney disease
• Insight into causality
• Prognostic indicator

Proteinuria
• Albuminuria
– Normal excretion <150 mg/24 h
• 60% is filtered plasma protein (20-40 mg of albumin)
• 40% are glycoproteins -- IgA, uromodulin
– Detected by urine dipsticks (Albustix; Ames or Miles etc)
– Detects >10-15 mg/dL; almost always (+) if urine alb > 30
mg/dL
Dip for protein
– Read-out is colorimetric; false (+) in highly alkaline urine Dip scale
tr 10 -20 mg/dl
– Microalbuminuria = 30-300 mg/24h, or 20-200 µg/min 1+ 30 mg/dl
2+ 100 mg/dl
3+ 300 mg/dl
• Quantitative assays 4+ 1000 mg/dl
poor correlation with
24° urine
– Sulfosalicylic method
• Acid precipitation, detects all proteins, sensitivity 5-10
mg/dL

76
Copyright © Harvard Medical School, 2018. All Rights Reserved.

Hematuria
• Presence of three or more red blood cells per high-power field visible
in a properly collected urine specimen without evidence of infection.
• Microscopic hematuria detected incidentally with a prevalence of 2-
31%
• Occurring along with proteinuria – strongly suggests glomerular
process

Hematuria – Major Causes


• Infection • Glomerular
– Pyelo, cystitis – GN eg IgA
• Malignancy – Hereditary
– Renal cell Ca – Thin basement membrane
– Transitional cell Ca – Alport’s
– Prostatic Ca – Vaculitis
• Metabolic/Other – Exercise
– Calculi • Interstitial
– Hypercalciuria – AIN
– Hyperuricosurua – PKD
– Coagulopathy – Papillary necrosis
– Cytoxan • Vascular
– Renal vein thrombosis,
Atheroemboli, Malignant HTN

77
Copyright © Harvard Medical School, 2018. All Rights Reserved.

#1 Young patient with routine


dipstick positivity of protein in urine

• A 14-year old young girl is noted to have


proteinuria on dipstick after a urine is sent for
urinalysis and urine c and s.
• The young girl is asymptomatic, except for some
mild frequency, has no past medical history and is
completely normal on physical examination. Wt
52 Kg, BSA 1.0 m2, BP 78/62 mmHg, no edema
• The urine culture is negative. Recheck of UA
shows 2+ protein. Spot protein to creatinine 0.8 g
prot/gram creat

Urine positive for protein

Repeat Twice

Urine transiently Urine remains


Positive for protein positive for protein

Check labs
(esp. BUN/Cr, Alb, UPCR
Routine UA and sed

F/U

Abnormal labs Normal labs Normal labs


>1g UPCR <1g UPCR

Refer to Pediatric Neg Check for Orthostatic


Nephrologist proteinuria
Check complement
ANA, antiDS DNA
Viral serologies, ASLO titer Pos
Renal US

Kidney Biopsy Trial of steroids Annual F/U

78
Copyright © Harvard Medical School, 2018. All Rights Reserved.

Orthostatic (Postural) proteinuria


– May be present normally
– Proteinuria < 1g / 24 h
– Commonly seen in adolescents; prevalence 0.6 to 6.3%
(Finish study)
– rare > age 30
– In both sexes, proteinuria increases with age.
– 90% of young men with isolated proteinuria have
“orthostatic” proteinuria; entirely benign
– Most children who test positive for proteinuria on initial
evaluation “lose” the proteinuria at follow-up.
– ≈10 percent of children have persistent proteinuria >6
months
Source: MAHMOUD LOGHMAN-ADHAM M., Am Fam
Physician. 1998 Oct 1;58(5):1145-1152. VEHASKARI VM, Archives of Disease in
Childhood, 1982, 57, 729-730

#2 Middle age person, type 1 diabetic


(x12 years), noticed to have
albuminuria
Ms. R is a 39-year old African American woman
with T1DM, HTN, atrial fibrillation on Coumadin.
Asymptomatic.
ROS- patient active, feels energetic
Medications: Coumadin, amlodipine, atorvostatin
BP 112/65 mmHg, HR 97 bpm,
NA 144, K 3.9, CL 110, CO2 24, BUN 17, CRE
0.70, EGFR 128 ml/min/1.73m2,
mAb 80 mg/g cre; Trace to 1+ prot in UA dip.

79
Copyright © Harvard Medical School, 2018. All Rights Reserved.

Natural History

Diabetic Nephropathy: KW nodules

80
Copyright © Harvard Medical School, 2018. All Rights Reserved.

Diabetic Kidney Disease


• Commonest cause of ESRD in West
• 45% of all U.S. patients with ESRD
• Screen annually for microalbuminuria in:
Type 1 diabetic patients who have had diabetes >5
years
Type 2 diabetic patients when diagnosis made.
– Test for urinary albumin excretion
Albumin to creatinine ratio (ACR)
Timed collection (e.g., 12 or 24 h) of albumin
concentration
• To reduce risk and/or slow progression of nephropathy
– optimize glucose control.
– optimize blood pressure control
– Use ARB or ACEi

Screen for MA in Diabetic Patient is positive


(repeat twice)

Urine transiently >30 mg AB


Positive for protein on UACR

Routine
F/U
MAB 30-300 mg AB >300 mg
NL Cr
Glycemic control
Manage BP
1. Glycemic control ACEi/ARB
HbA1C<7.5% T1DM age<19 or elderly AND REFER TO NEPHROLOGY
HbA1C<7% for adults
2. Manage BP <140/90 Work up
3. ACEi or ARB Typical Atypical

Manage as High Risk Patient Kidney Biopsy

81
Copyright © Harvard Medical School, 2018. All Rights Reserved.

#3 Patient with hematuria, RBC casts


and rapid loss in kidney function
• A 24-year old hairdresser from Cape Cod is
admitted with oliguria. Has a 2 day history of tea
colored urine, headaches, and feelling unwell. 1-
day history of coughing up blood and dyspnea.
• Examination 150/95 mmHg, HR 110 bpm,
afebrile, O2 sat 91% on air, reduced air entry in
lung bases.
• Labs: BUN 58 mg/dL, creatinine 2.9 mg/dL, Hb
10.2 g/dL, WBC 6.2, plats 286,000. UA 4+ bl 2+
prot, no leuks; sediment 30-60 dysmporhic RBCs,
1-2 RBC casts, 2-4 WBCs/HPF.

Glomerular Syndromes
• Nephritis: Hypertension, Azotemia,
proteinuria, hematuria, RBC casts /
dysmorphic RBCs
• Nephrosis: edema, proteinuria,
hypoalbuminemia, lipid abnormalities
• RPGN: rapid renal failure, crescents on
renal biopsy + nephritis
• Isolated urinary abnormalities: hematuria /
proteinuria

82
Copyright © Harvard Medical School, 2018. All Rights Reserved.

Pulmonary Renal Syndrome

• Main Causes
– Anti GBM Disease: Goodpasture’s
– ANCA associated vasculitis
• Granulomatosus with polyangiitis (Wegener’s)
• Microscopic polyarteritis (MPA)
• Churg Strauss
– Immune complex mediated
• Lupus nephritis
• Cryoglobulinemia
• HSP nephritis

Dysmorphic RBCs and Red Cell Cast

83
Copyright © Harvard Medical School, 2018. All Rights Reserved.

Anti-GBM Nephritis
Goodpasture’s Syndrome
• Uncommon: incidence 0.1 cases/million
• 1-2% of renal biopsy specimens
• Slight preponderance in males
• Age 1st to 9th decade
• More common in whites over African-American’s
• Year-round presentation, ? Spring and summer
• 50-75% have upper respiratory prodrome,
– plus fever, rash, myalgias, malaise, headache, weight
loss
• Renal presentation with RPGN - proteinuria but usually not
nephrotic, hypertension uncommon
• US - normal size kidneys

Pathologic Features of
Anti-GBM Nephritis

84
Copyright © Harvard Medical School, 2018. All Rights Reserved.

Goodpasture’s Syndrome
Treatment
• Treatment without methylprednisone and plasmaphereis
– 89% progress to death or dialysis; only 10% improved
• Treatment with pulse steroids, plasmapheresis, and Cytoxan
– standard-of-care
– 50% improve
– Protocol
– Pulse methylpred 1 g QD x 3 d, 1-1.5 mg/Kg prednisone
– Cyclophosphamide 3 mg/Kg/d (reduced dose in older patients, or
if GFR < 10) > 2 months
– Plasma exchange daily 4 L with albumin replacement (or FFP if
pulmonary hemorrage present) x 14 d or until ab goes away
• Patients with serum creat of >7.0 respond to treatment
– 75% with Scr <7 respond, 8% with Scr > 7 respond
– Generally no improvement in patients on dialysis

#4 19-year old white male and his military


physical

• 19-year white male is evaluated as part of health


screening for his military physical.
• Healthy asymptomatic
• One prior episode of gross hematuria, resolved
spontaneously after 3 or 4 days. Not referred
• PE normal
• UA 2+ blood, no protein, 5-10 red cells (dysmorphic) in
urine microscopy
• Biopsy considered – done
– IgA nephropathy

85
Copyright © Harvard Medical School, 2018. All Rights Reserved.

IgA nephropathy
• Epidemiology
– Most common cause of nephritis in the world (15 to 40% of
primary GN in world, 20% of primary GN in USA)
– Males > Females (2:1)
– Peak occurrence in 2nd & 3rd decades
– Asian predominance (up to 40% of biopsies compared with 20%
in European/U.S. registries)
• Clinical
– Synpharyngitic (24 to 48 hrs after URI or GI infection -- in
contrast with post-infectious nephritis 1 to 3 weeks)
– Low grade fever, loin pain
– Serum IgA levels elevated in ~50% (Test for elevated serum IgA
not diagnostic)
– Spectrum of microscopic hematuria + albuminuria to RPGN
– Can be with or w/o Henoch-Schonlein Purpura (HSP)
• Children: gross hematuria after URI
• Adults: microscopic hematuria and/or proteinuria

IgA: Therapeutic Options

• No Treatment
• Treatment
– ACEi/ARB
– Glucocorticoids
– Fish Oils
– Tonsillectomy
– Immunosuppressives
• Azathioprine + steroids
• Cyclophosphamide + steroids
• Mycophenolate mofetil

86
Copyright © Harvard Medical School, 2018. All Rights Reserved.

#5 Teenager with sudden onset face


swelling, LE edema and nephrotic
proteinuria
• 17-year white male high school student develops sudden
onset of facial swelling, LE swelling, frothy urine.
Otherwise completely asymptomatic. One week before
presentation he had a bee sting on his forearm.
Examination normal BP. 2 Kg increase in weight, swollen
eye-lids, lips, face, anasarca
• Renal function Bun 14 mg/dL, Creat 0.5 mg/dL,. serum
albumin 2.8 g/dl, total cholesterol 315 mg/dl, triglycerides
68 mg/dl, LDL cholesterol 228 mg/dl.
• UACR - 16 g alb/g cre.

Nephrotic Syndrome

• Proteinuria (> 3.5g/d) (>2 g/d is usually


glomerular origin proteinuria)
• Hypoalbuminemia (< 3 g/dL)
• Hyperlipidemia
• Lipiduria
• Edema
• Bland urine or fatty casts/ free fat / oval fat
bodies

87
Copyright © Harvard Medical School, 2018. All Rights Reserved.

Urinalysis: key features

NS: Etiology
Primary Causes Secondary Causes
• Medications
• Membranous – Gold, NSAIDs, Interferon -
alfa, Heroin, Captopril,
• Focal Segmental GS • Allergens
(FSGS) – Bee Sting, Pollen
• Minimal Change Disease • Infections
– Bacterial, Viral, Helminth
• IgA • Cancer
– Solid: Lung, colon, stomach
– Leukemia, Hodgkins,
ovarian
• Autoimmune Diseases
• Metabolic Diseases
• Pregnancy

88
Copyright © Harvard Medical School, 2018. All Rights Reserved.

Normal and Minimal Change Disease


A: Normal B: MCD

Minimal changes of glomeruli


– basic characteristics
1. Full blown nephrotic syndrome with selective
proteinuria
2. Rarely presence of hematuria, hypertension,
or renal insufficiency
3. Absence of glomerular abnormalities on LM
an IF.
4. Typical picture of damage of epithelial cells
(fusion of foot processes) in EM

89
Copyright © Harvard Medical School, 2018. All Rights Reserved.

Swelling/edema/UA pos for protein

Check labs
esp. BUN/Cr, Alb, UA,
UACR

Abnormal labs

Refer to
Nephrology
>2 g UACR

Abnormal labs Otherwise Normal labs

Further W/U Trial of steroids


For MCD
Partial response or
No response Remission
Kidney Biopsy Or remission-then-relapse
after 16 w therapy Routine F/U

Summary/Conclusion
• 6 clinical scenarios
• Proteinuria and Hematuria - GN of different
causes and different age groups

Painting by Sir Luke Fildes.

90
Copyright © Harvard Medical School, 2018. All Rights Reserved.

References
• Source: MAHMOUD LOGHMAN-ADHAM M.,
Am Fam Physician. 1998 Oct 1;58(5):1145-
1152. VEHASKARI VM, Archives of Disease in
Childhood, 1982, 57, 729-730

91
Copyright © Harvard Medical School, 2018. All Rights Reserved.

ELECTROLYTE AND ACID BASE:


Challenging Questions and Answers
Bradley M. Denker, MD.
Clinical Chief,
Renal Division, Department of Medicine
Beth Israel Deaconess Medical Center and
Harvard Vanguard Medical Associates
Associate Professor of Medicine
Harvard Medical School

Financial disclosures
Bradley M. Denker

No conflict of interest to disclose.

92
Copyright © Harvard Medical School, 2018. All Rights Reserved.

Case 1
A 35 year-old man with bipolar disorder treated on lithium,
is referred to you for chronic polyuria and polydipsia. He
complains that he has to void once every hour.
Laboratory studies:
Serum sodium 146 mEq/L
Blood urea nitrogen 35 mg/dL
Serum creatinine 1.9 mg/dL
Serum osmolality 305 mOsm/kg
24-hr urine volume 5L
Urine sodium 28 mEq/L
Urine osmolality 190mOsm/kg

Case 1
Which of the following might be appropriate in the
management of this patient?
(A) Discontinue lithium
(B) Demeclocycline
(C) Vasopressin V2 receptor antagonist
(D) Fluid restriction
(E) Furosemide

Choices B-E are all potential treatment options for


Hyponatremia but will worsen hypernatremia

93
Copyright © Harvard Medical School, 2018. All Rights Reserved.

Hypernatremia

UOsm

< 800 mOsm/kg > 800 mOsm/kg

• Renal H2O loss • Insensible H2O loss


• GI H2O loss
• Na+ intake
DI Osmotic
diuresis +
CDI NDI Glucose, urea, ↓ Water intake
mannitol

Nephrogenic diabetes insipidus


Hypokalemia
Hypercalcemia
Tubulointerstitial nephropathies
Sickle cell disease
Myeloma
Obstructive uropathy
Recovery from ATN or obstruction
Lithium

Chronic renal failure

94
Copyright © Harvard Medical School, 2018. All Rights Reserved.

Distinguishing central from nephrogenic DI

Water deprivation test

DDAVP (desmopressin)

↑UOsm No ∆ in UOsm
CDI NDI

Management of hypernatremia
Replace free water deficit (50% in first 24 hr,
no more than 0.5 mM/hr)
0.4-0.5 x BW(kg) x (SNa/140-1)

Replace ongoing free water losses


Treat underlying cause
Desmopressin for CDI
No specific Rx for NDI (attempt to reduce urine
output with Na restriction, thiazides or give
supratherapeutic dose of desmopressin)

95
Copyright © Harvard Medical School, 2018. All Rights Reserved.

Case 1
Which of the following might be appropriate in the
management of this patient?
(A) Discontinue lithium
(B) Demeclocycline; interferes with ADH action in
collecting duct-increased water excretion

(C) Vasopressin V2 receptor antagonist-blocks


V2 receptor and ADH action in collecting duct-
increased water excretion

(D) Fluid restriction; more free water needed


(E) Furosemide; blocks ability to concentrate urine;
dilute urine output

Case 2
An 85 year-old woman, who lives alone, fell in her bedroom and
broke her hip. She was unable to get up and had no access to
water. She was found 2 days later and brought into the ER. On
examination, she is drowsy but responsive. The blood pressure
is 103/51 mm Hg, pulse rate 90 per minute, weight 70 kg,
mucous membranes are very dry and skin turgor is decreased.
Laboratory studies:
Serum sodium 164 mEq/L
Blood urea nitrogen 54 mg/dL
Serum creatinine 1.2 mg/dL
Hematocrit 56%

Urine osmolality 820 mOsm/kg

96
Copyright © Harvard Medical School, 2018. All Rights Reserved.

Case 2
All of the following statements are correct except:
A. She has intracellular fluid volume depletion
B. Appropriate initial fluids would be 0.45% NaCl
C. Her serum Na+ should be lowered to a target of 152
mEq/L in the next 24 hours
D. Overly rapid correction of her hypernatremia could
cause osmotic demyelination syndrome
E. Correction of her hypernatremia could cause
cerebral edema

Hypernatremia

UOsm

< 800 mOsm/kg > 800 mOsm/kg


• Renal H2O loss • Insensible H2O loss
• GI H2O loss
• Na+ intake
DI Osmotic
diuresis
+
CDI NDI Glucose, urea, ↓ Water intake
mannitol

97
Copyright © Harvard Medical School, 2018. All Rights Reserved.

Management of hypernatremia
Replace free water deficit (50% in first 24 hr, no
more than 0.5 mM/hr)
Risk is cerebral edema due to synthesis of organic
osmolytes; CPM seen with rapid increase in Na+

Replace ongoing free water losses


Treat underlying cause
Desmopressin for CDI
No specific Rx for NDI (attempt to reduce urine output
with Na restriction, thiazides or give supratherapeutic
dose of desmopressin)

Rapid Increased Rapid Decreased


Na Na

Osmotic Demyelination Cerebral Edema


160
120
H2O H2O

Sterns, N Engl J Med 2015;372:55-65.

98
Copyright © Harvard Medical School, 2018. All Rights Reserved.

Case 2
All of the following statements are correct except:
A. She has intracellular fluid volume depletion-Yes,
appears volume and water depleted
B. Appropriate initial fluids would be 0.45% NaCl-Yes,
both salt and water depleted; water>Na so
hypotonic replacement
C. Her serum Na+ should be lowered to a target of 152
mEq/L in the next 24 hours-Ok; 0.5mEq/h~12
D. Overly rapid correction of her hypernatremia could
cause osmotic demyelination syndrome - Incorrect
E. Correction of her hypernatremia could cause
cerebral edema – Correct

Case 3
A 64 year-old woman with coronary artery disease, multiple prior
MI and ischemic cardiomyopathy, with a LV EF of 15%, is
admitted with pulmonary edema. Her medications include
aspirin, metoprolol, furosemide, spironolactone, digoxin,
isosorbide dinitrate, and lisinopril. On examination, the blood
pressure is 97/54 mm Hg, pulse rate 85 per minute, jugular
venous pressure 9 cm, moist mucous membranes, lungs with
diffuse inspiratory crackles, heart with an S3 gallop, and cool,
clammy extremities with 1+ peripheral edema.

Serum sodium 128 mEq/L Blood urea nitrogen 46 mg/dL


Serum potassium 3.6 mEq/L Serum creatinine 1.2 mg/dL
Arterial pH 7.48
Serum chloride 87 mEq/L
Serum bicarbonate 34 mEq/L

99
Copyright © Harvard Medical School, 2018. All Rights Reserved.

Case 3
Urine electrolytes (6 hrs after last diuretic dose):
Urine sodium 15 mEq/L
Urine chloride < 5 mEq/L
Urine osmolality 220 mOsm/kg

Case 3
All of the following might be appropriate in the
management of this patient EXCEPT:
(A) Intravenous 0.9% saline
(B) Restriction of free water intake
(C) Dietary sodium restriction
(D) Dobutamine
(E) Acetazolamide

100
Copyright © Harvard Medical School, 2018. All Rights Reserved.

Hyponatremia

Posm

> 290 mOsm/kg Normal < 275 mOsm/kg


“Pseudohyponatremia”

↑Glucose* Lipid ↑ Hypoosmolal


Mannitol Protein ↑ hyponatremia

*Correct serum Na+ by 1.6 for every 100 mg/dL ∆ in glucose


Next Slide

Hypoosmolal hyponatremia
Volume status

Hypovolemic Euvolemic Edematous


Dehydration* CHF*
Addison’s UOsm Nephrotic*
Diuretics Liver failure*
> 100 < 100 Renal failure
SIADH Polydipsia
Hypothyroid

* UNa < 20 = Extrarenal cause of ECV depletion

101
Copyright © Harvard Medical School, 2018. All Rights Reserved.

Intravascular Volume Depletion Will


Predominate Over Hypo-osmolality

Underfilled Arterial
Circulation

Case 3
All of the following might be appropriate in the
management of this patient EXCEPT:
(A) Intravenous 0.9% saline; NO will worsen CHF
(B) Restriction of free water intake; Yes, will reduce
hypo-osmolar hyponatremia

(C) Dietary sodium restriction; Yes, will reduce volume


expansion and reduce LVEDP

(D) Dobutamine; Yes, Β-agonist will increase C.O.


(E) Acetazolamide; Yes, CA inhibitor; Prox Tubule
inhibition of Na reabsorption

102
Copyright © Harvard Medical School, 2018. All Rights Reserved.

Case 4
A 45 year-old male smoker presents with confusion and
drowsiness. His only medications are bronchodilator and
steroid inhalers. On examination, his BP is 125/86, HR 78,
moist mucous membranes, good skin turgor, jugular venous
pressure 4 cm, lung fields clear to auscultation, no peripheral
edema. Chest radiograph shows emphysematous changes
but is otherwise normal. Laboratory studies:
Serum sodium 116 mEq/L
Serum osmolality 256 mOsm/kg
Urine sodium 85 mEq/L
Urine potassium 78 mEq/L

Urine osmolality 670 mOsm/kg

Case 4
Appropriate steps in the management of this
patient might include:
(A) Order serum protein and lipid panel
(B) Computed tomography scan of the chest
(C) Psychiatry consult for psychogenic
polydipsia
(D) Administer thiazide diuretic
(E) Order echocardiogram

103
Copyright © Harvard Medical School, 2018. All Rights Reserved.

Hyponatremia

Posm

> 290 mOsm/kg Normal < 275 mOsm/kg


“Pseudohyponatremia”

↑Glucose* Lipid ↑ Hypoosmolal


Mannitol Protein ↑ hyponatremia

*Correct serum Na+ by 1.6 for every 100 mg/dL ∆ in glucose


Next Slide

Hypoosmolal hyponatremia
Volume status

Hypovolemic Euvolemic Edematous


Dehydration* CHF*
Addison’s UOsm Nephrotic*
Diuretics Liver failure*
> 100 < 100 Renal failure
SIADH Polydipsia
Hypothyroid

* UNa < 20 = Extrarenal cause of ECV depletion

104
Copyright © Harvard Medical School, 2018. All Rights Reserved.

Case 4
Appropriate steps in the management of this patient
might include:
(A) Order serum protein and lipid panel; this is
pseudohyponatremia

(B) Computed tomography scan of the chest


(C) Psychiatry consult for psychogenic
polydipsia; would expect appropriately dilute
urine

(D) Administer thiazide diuretic; no effect on urine


dilution; Na loss will worsen hyponat

(E) Order echocardiogram; No evidence for CHF

Case 5
A 45 year-old woman with hypertension and type 2
diabetes mellitus presents with leg swelling. Her
medications are insulin, amlodipine, enalapril,
furosemide, aspirin.

Laboratory studies:
Serum sodium 136 mEq/L
Serum potassium 6.2 mEq/L
Serum bicarbonate 20 mEq/L
Blood urea nitrogen 32 mg/dL
Serum creatinine 1.9 mg/dL
24 hr urine total protein 4.8 g

105
Copyright © Harvard Medical School, 2018. All Rights Reserved.

Case 5
All of the following could be contributing to this
patient's hyperkalemia EXCEPT:

(A) Enalapril
(B) Decreased glomerular filtration rate
(C) Type 4 renal tubular acidosis
(D) Renal artery stenosis
(E) Excess dietary K intake

Hypokalemia/Renal K+
wasting & hypertension

Renin
AI AII Aldosterone
Substrate Renin Conv Enz

Na+ Abs
K+ Excretion - Hypokal
Renal Hypo-perfusion

HYPERTENSION

106
Copyright © Harvard Medical School, 2018. All Rights Reserved.

Hypokalemia/Renal K+
wasting & hypertension

Aldosterone
High Low
Renin

High Low

Renal artery Primary Cushing’s


stenosis hyperaldosteronism Liddle's
Liquorice ingestion

Hyperkalemia

↑ Intake Cell shift


Metabolic acidosis
Decreased urinary Hyperglycemia
K+ excretion β-blocker
24 hr urine K+ < 40 mEq Digitalis
Hyperkalemic
periodic paralysis
Cell lysis

107
Copyright © Harvard Medical School, 2018. All Rights Reserved.

Decreased urinary K+ excretion

↓Tubular flow ↓CCD [K+]

Renal ↓ECV Meds Adrenal Hyporenin


failure insufficiency hypoaldo
NSAIDs
ACEI/ARB Addison’s
Block Heparin 1° hypoaldo
RAAS Spironolactone
Cyclosporine

Amiloride
Block Na+
Trimethoprim
channel
Pentamidine

Type IV RTA (hyporeninemic


hypoaldosteronism)

Hyperkalemia (disproportionate to level of GFR)


Non-gap metabolic acidosis with normal urine
acidifying ability
Mild CRF
Often underlying tubulointerstitial disease:
- DM
- SLE, obstruction, myeloma/amyloid, HIV etc.
- NSAIDs

108
Copyright © Harvard Medical School, 2018. All Rights Reserved.

Case 5
All of the following could be contributing to this
patient's hyperkalemia EXCEPT:

(A) Enalapril; RAS inhibitor, increased K


(B) Decreased glomerular filtration rate;K
excretion depends on GFR
(C) Type 4 renal tubular acidosis
(D) Renal artery stenosis; causes HypoK; activated
RAS
(E) Excess dietary K intake; major cause of
increased K; Total Intracellular K ~
145x28L~4000mEq; Total Extracellular~14Lx4~60
2gms K ~ 50mEq

Case 6
A 20 year-old man, with no past medical history and on no
medications, presents with a one week history of fatigue,
nausea, vomiting, diarrhea and acute abdominal pain. On
examination, BP is 80/60, HR 110, temperature 99.8°F.
The abdomen was diffusely mildly tender.
Laboratory studies:
Serum sodium 124 mEq/L
Serum potassium 6.8 mEq/L
Serum chloride 101 mEq/L
Serum bicarbonate 18 mEq/L
Serum glucose 52 mg/dL
Blood urea nitrogen 19 mg/dL
Serum creatinine 1.1 mg/dL
24 hr urine potassium 5 mEq/L

109
Copyright © Harvard Medical School, 2018. All Rights Reserved.

Case 6
Which one of the following tests would be most
likely to reveal the underlying cause of the
hyperkalemia?

(A) Blood digoxin level


(B) Serum creatine kinase
(C) ACTH stimulation test
(D) Blood glycosylated hemoglobin level
(E) Iothalamate GFR test

Hyperkalemia

↑ Intake Cell shift


Metabolic acidosis
Decreased urinary Hyperglycemia
K+ excretion β-blocker
24 hr urine K+ < 40 mEq Digitalis
Hyperkalemic
periodic paralysis
Cell lysis

110
Copyright © Harvard Medical School, 2018. All Rights Reserved.

Decreased urinary K+ excretion

↓Tubular flow ↓CCD [K+]

Renal ↓ECV Meds Adrenal Hyporenin


failure insufficiency hypoaldo
NSAIDs
ACEI/ARB Addison’s
Block Heparin
RAAS 1° hypoaldo
Spironolactone
Cyclosporine

Block Na+ Amiloride


channel Trimethoprim
Pentamidine

Hypoosmolal hyponatremia
Volume status

Hypovolemic Euvolemic Edematous


Dehydration* Psych. polydipsia† CHF*
Addison’s SIADH Liver failure*
Diuretics Hypothyroid

* UNa < 20 = Extrarenal cause of ECV depletion


†U < 100 = ADH appropriately suppressed
Osm

111
Copyright © Harvard Medical School, 2018. All Rights Reserved.

Case 6
Which one of the following tests would be most
likely to reveal the underlying cause of the
hyperkalemia?

(A) Blood digoxin level; inhibits Na/K ATPase and


can lead to increased K
(B) Serum creatine kinase; elevated levels seen
with rhabdomyolysis and increased K
(C) ACTH stimulation test
(D) Blood glycosylated hemoglobin level;
hyperglycemia associated with K shift out of cells
(E) Iothalamate GFR test; reduced GFR
predisposes to hyperkalemia

Case 7
An 18 year-old female presents with acute muscle
weakness. She has had several previous episodes that
resolved spontaneously. BP 96/54. Rest of the exam was
unremarkable.

Laboratory studies:
Serum sodium 135 mEq/L 24 hr urine studies:
Serum potassium 2.9 mEq/L Sodium 80 mEq/d
Serum chloride 99 mEq/L (range 50-150mEq)
Serum bicarbonate 28 mEq/L Potassium 105 mEq/d
Blood urea nitrogen 8 mg/dL (range 50-100mEq/d)
Serum creatinine 0.5 mg/dL Chloride 150 mEq/Ld

112
Copyright © Harvard Medical School, 2018. All Rights Reserved.

Case 7
Which one of the following diagnoses are
compatible with this clinical picture?

(A) Gitelman’s syndrome


(B) Primary hyperaldosteronism
(C) Surreptitious amiloride use
(D) Surreptitious laxative abuse
(E) Hypokalemic periodic paralysis

DDX of hypokalemia

Cellular shift GI loss Urinary K wasting


Alkalemia Vomiting* 24 hr UK > 25 mEq
Insulin Diarrhea
β-agonist
Hypokalemic periodic
paralysis
*Also renal K+ wasting

113
Copyright © Harvard Medical School, 2018. All Rights Reserved.

Renal K wasting with normal or low BP

Osmotic Drugs Vomiting RTA Inherited


diuresis tubulopathy
Diuretics Proximal Bartter
Loop & thiazide Classic distal Gitelman
Cisplatin
Aminoglycosides
Ticarcillin
Amphotericin
Toluene

Cryptogenic hypokalemic metabolic


alkalosis

Volume Urine
Urine Cl-
status/BP diuretics

Hyperaldosteronism ↑ > 40 mEq/L -

Surreptitous
Nl or ↓ < 25 mEq/L -
vomiting
Diuretic abuse Nl or ↓ > 40 mEq/L +
Bartter/Gitelman
Nl or ↓ > 40 mEq/L -
syndrome

114
Copyright © Harvard Medical School, 2018. All Rights Reserved.

Case 7
Which one of the following diagnoses are
compatible with this clinical picture?

(A) Gitelman’s syndrome


(B) Primary hyperaldosteronism; Hypokalemia +
HTN
(C) Surreptitious amiloride use; K sparing diuretic;
expect low urine K and high serum K
(D) Surreptitious laxative abuse; causes low K but
urine K should be very low
(E) Hypokalemic periodic paralysis; cellular K
shifts, urine K not elevated

Case 8
A 74 year-old woman diagnosed with hypertension at the
age of 40 presents with worsening blood pressure control
over the past 3 years. She is now on amlodipine, lisinopril,
hydrochlorothiazide, atenolol and clonidine. Her current BP
is 156/78.
Laboratory studies:
Serum sodium 136 mEq/L
Serum potassium 3.0 mEq/L
Serum chloride 101 mEq/L
Serum bicarbonate 26 mEq/L
Blood urea nitrogen 18 mg/dL
Serum creatinine 2.0 mg/dL
Plasma renin activity 8.5 ng/mL/hr (Normal range 1-6)
Plasma aldosterone 24 ng/dl (Normal range 5-20)

115
Copyright © Harvard Medical School, 2018. All Rights Reserved.

Case 8
Which of the following tests would be the most
appropriate next step?

(A) Computed tomography scan of the adrenal


glands
(B) Dexamethasone suppression test
(C) Urine diuretic screen
(D) Doppler ultrasound of the renal arteries

Hypokalemia/Renal K+
wasting & hypertension
Aldosterone
High Low
Renin

High Low

Renal artery Primary Cushing’s


stenosis hyperaldosteronism Liddle's
Reninoma Liquorice ingestion
(very rare)
Renin
AI AII Aldosterone
Substrate
Renin Conv Enz

116
Copyright © Harvard Medical School, 2018. All Rights Reserved.

Case 8
Which of the following tests would be the most
appropriate next step?

(A) Computed tomography scan of the adrenal


glands; hyperaldo causes HTN and low K but
renin should be suppressed
(B) Dexamethasone suppression test; Rule out
Cushing’s but aldo usually normal/low
(C) Urine diuretic screen; not typical scenario/
taking diruetic
(D) Doppler ultrasound of the renal arteries

Case 9
A 28 year-old man is found unconscious in the street and
brought into the emergency room. No medical history is
available. His blood pressure is 120/75 mm Hg, respiratory
rate 12 per minute. He appears dishevelled and is comatose
and responsive only to pain. His pupils are reactive to light
and he has a non-focal neurological examination. No fetor is
noted. He is intubated, undergoes gastric lavage, and
activated charcoal is adminstered via a nasogastric tube.
Serum sodium 132 mEq/L
Serum potassium 3.5 mEq/L
Serum chloride 98 mEq/L
Serum bicarbonate 10 mEq/L
Blood urea nitrogen 32 mg/dL
Serum creatinine 1.6 mg/dL
Serum glucose 75 mg/dL Next page →

117
Copyright © Harvard Medical School, 2018. All Rights Reserved.

Case 9

Serum ethanol None detected


Acetest Negative
Serum β-hydroxybutyrate Negative
Serum lactate < 1 mmol/L
Serum salicylate None detected
Serum creatine kinase 10 mU/mL
Serum osmolality 308 mOsm/kg

Arterial blood studies on room air:


pH 7.22
PCO2 24 mm Hg

Case 9
The most appropriate next step in the
management of this patient is:

(A) Dopamine
(B) Hemodialysis
(C) Forced alkaline diuresis
(D) Thiamine
(E) Fomepizole
Blocks alcohol dehdyrogenase
which normally converts ethylene
glycol to glycolic acid and
methanol to formic acid

118
Copyright © Harvard Medical School, 2018. All Rights Reserved.

Brent; NEJM 360;21, 2009

Case 9
pH 7.22, PCO2 24 mm Hg, HCO3 10 mEq/L
Primary metabolic acidosis

Predicted PCO2 from Winter's formula = (1. 5 x {HCO3-}) + 8


= 23
Respiratory compensation is appropriate
Na 132 mEq/L, Cl 98 mEq/L
Anion gap = 132 - 98 - 10 = 24 (normal 8-12)
Anion gap metabolic acidosis

∆AG = 24 - 10 = 14
∆HCO3 = 24 - 10 = 14
∆/∆ = 14/14 = 1
Pure anion gap metabolic acidosis

119
Copyright © Harvard Medical School, 2018. All Rights Reserved.

Serum osmolal gap


Osmolal gap = Measured Sosm - Calc Sosm

Calculated Sosm :
2 [Na+] + [glucose]/18 + [BUN]/2.8

(2 x 132) + (75/18) + (32/2.8) = 280


Osmolal gap = 308 - 280 = 28 (normal < 10)

Anion and osmolar gap in diagnosis


of intoxications

Anion gap
acidosis Osmolal gap

+ Normal Salicylates

Ethanol
Ethylene glycol
+ High
Propylene glycol
Methanol

- High Isopropanol

120
Copyright © Harvard Medical School, 2018. All Rights Reserved.

Clues to high anion gap acidosis


syndromes

Alcoholic fetor
Papilledema
Osmolar gap
Undetectable serum ethanol

Methanol intoxication

Clues to high anion gap acidosis


syndromes

No fetor
Osmolar gap
Calcium oxalate dihydrate (envelope-
shaped) crystalluria
Urine fluoresces under Wood's (UV) lamp

Ethylene glycol intoxication

121
Copyright © Harvard Medical School, 2018. All Rights Reserved.

Case 9
The most appropriate next step in the
management of this patient is:

(A) Dopamine; not indicated


(B) Hemodialysis; likely will be needed; takes time to
initiate
(C) Forced alkaline diuresis; indicated for
salicylates; no utility for alcohol toxicities
(D) Thiamine; Vitamin B1; deficient in alcoholics
(E) Fomepizole; inhibits alcohol dehdyrogenase to
reduce toxic producsts

Case 10
An 18 year-old female is brought in with change in
mental status and suspected toxic ingestion. Her blood
pressure is 100/73 mm Hg, pulse rate 89, respiratory
rate 40, temperature 100.5°C. She vomited once in
the emergency room and is poorly responsive.

Laboratory studies:
Serum sodium 142 mEq/L
Serum potassium 3.6 mEq/L
Serum chloride 102 mEq/L
Serum bicarbonate 16 mEq/L
Blood urea nitrogen 21 mg/dL
Serum creatinine 1.8 mg/dL
Serum glucose 62 mg/dL Next page →

122
Copyright © Harvard Medical School, 2018. All Rights Reserved.

Case 10

Acetest Negative
Serum lactate 1.8 mmol/L
Serum osmolality 295 mOsm/kg

Arterial blood studies on room air:


pH 7.39
PCO2 25 mm Hg

Case 10
Which of the following treatments would be most
likely to be effective in the management of this
patient?

(A) Breathe into a bag


(B) Insulin drip
(C) Forced alkaline diuresis
(D) 5% dextrose
(E) Fomepizole

123
Copyright © Harvard Medical School, 2018. All Rights Reserved.

Clues to high anion gap acidosis


syndromes

Tinnitus/deafness
Fever, tachycardia, hyperventilation
Associated respiratory alkalosis and
metabolic acidosis

Salicylate intoxication

Case 10
pH 7.39, PCO2 25 mm Hg, HCO3 16 mEq/L
Primary metabolic acidosis, & probably respiratory alkalosis

Predicted PCO2 from Winter's formula = (1. 5 x 16) + 8 = 32


Concomitant respiratory alkalosis

Na 142 mEq/L, Cl 102 mEq/L


Anion gap = 142 - 102 - 16 = 24 (normal 8-12)
Anion gap metabolic acidosis

∆AG = 24 - 10 = 14
∆HCO3 = 24 - 16 = 8
∆/∆ = 14/8 = 1.8
Vomiting induced metabolic alkalosis:bicarb higher
than expected for degree of elevation of AG

124
Copyright © Harvard Medical School, 2018. All Rights Reserved.

Serum osmolal gap


Osmolal gap = Measured Sosm - Calc Sosm

Calculated Sosm :
2 [Na+] + [glucose]/18 + [BUN]/2.8

(2 x 142) + (62/18) + (21/2.8) = 295


Osmolal gap = 295 - 295 = 0

Anion and osmolar gap in diagnosis


of intoxications

Anion gap
acidosis Osmolal gap

+ Normal Salicylates

Ethanol
Ethylene glycol
+ High
Propylene glycol
Methanol

- High Isopropanol

125
Copyright © Harvard Medical School, 2018. All Rights Reserved.

Lipid
Permeable

Two benefits of alkalinization:


Increasing pH favors HS moving out of brain cell
Increasing urine pH favors salicylate excretion into the urine
(Increase urine pH from 6.5 to 8.1; 5x greater excretion)

Case 10
Which of the following treatments would be most
likely to be effective in the management of this
patient?

(A) Breathe into a bag; increasing PCO2 will


worsen acidosis
(B) Insulin drip; used for DKA
(C) Forced alkaline diuresis
(D) 5% dextrose; no benefit
(E) Fomepizole; inhibits alcohol dehydrogenase

126
Copyright © Harvard Medical School, 2018. All Rights Reserved.

Case 11
A 35 year-old male with HIV infection is maintained on
HAART therapy, with his most recent regimen being
raltegravir (INSTI), tenofovir (NRTI) and emtricitabine
NRTI). He has been doing well, but on a routine clinic visit
was found to have abnormal chemistries.

Serum sodium 135 mEq/L


Serum potassium 2.8 mEq/L Serum glucose 91 mg/dL
Serum chloride 109 mEq/L Serum calcium 8.8 mg/dL
Serum bicarbonate 18 mEq/L Serum phosphate 1.8 mg/dL
Blood urea nitrogen 13 mg/dL
Serum creatinine 1.7 mg/dL

Urinalysis: Specific gravity 1.015, pH 6, trace protein, 3+


glucose, no blood, leukocyte esterase negative

Case 11
Which of the following would be the most
appropriate next step in the management of this
patient?

(A) Stool microbiology and colonoscopy


(B) Discontinue tenofovir
(C) Discontinue raltegravir
(D) Send autoantibody panel
(E) Magnetic resonance imaging of the adrenal
glands

127
Copyright © Harvard Medical School, 2018. All Rights Reserved.

Case 11
HCO3 18 mEq/L, no ABG
Probable metabolic acidosis

Na 135 mEq/L, Cl 109 mEq/L


Anion gap = 135 - 109 - 18 = 8 (normal 8-12)
Non-gap metabolic acidosis

DDx of a non-gap metabolic acidosis

128
Copyright © Harvard Medical School, 2018. All Rights Reserved.

DDx of a non-gap metabolic acidosis

Diarrhea RTA
(bicarb loss)

I II IV
Classic distal Proximal Hyporeninemic
(bicarb loss) hypoaldosteronism

Case 11
HCO3 18 mEq/L, no ABG
Probable metabolic acidosis

Na 135 mEq/L, Cl 109 mEq/L


Anion gap = 135 - 109 - 18 = 8 (normal 8-12)
Non-gap metabolic acidosis

Urine pH 6 is inappropriately high


Renal tubular acidosis, Type I or II

Hypokalemia, hypophosphatemia, glycosuria with


normoglycemia
Fanconi syndrome, most likely with Type II (proximal) RTA

129
Copyright © Harvard Medical School, 2018. All Rights Reserved.

DDx of RTA
Proximal Classic distal Hyporenin
hypoaldo
Serum K Low Low High
Urine pH Variable > 5.5 < 5.5
Other Fanconi (low Nephrocalcinosis
features PO4, glycosuria) ± CaPO4 stones

Causes and Rx of RTA


Proximal Classic distal Hyporenin
hypoaldo
Common Ifosfamide Sjogren’s CKD plus:
causes NRTI (tenofovir, SLE DbM
adefovir, cidofovir) Amphotericin Obstruction
Myeloma Sickle cell dz
SLE
NSAIDs
Rx Bicarbonate (lots) Bicarbonate K+ lowering Rx:
(1 mEq/kg/day) Diuretics
-tenofovir disoproxil fumarate
Kayexalate
(TDF); newer alafenamide (TAF) is
Low K diet
effective at 1/30th dose so less renal
toxicity anticipated Mineralocorticoid

130
Copyright © Harvard Medical School, 2018. All Rights Reserved.

Case 11
Which of the following would be the most
appropriate next step in the management of this
patient?

(A) Stool microbiology and colonoscopy;


diarrhea leads to non-gap acidosis but intact
renal acidification (urine pH-6)
(B) Discontinue tenofovir
(C) Discontinue raltegravir; not associated with
RTA
(D) Send autoantibody panel; consider for distal RTA
and Sjogren’s syndrome
(E) Magnetic resonance imaging of the adrenal
glands; hyperaldo should cause HTN

Case 12
A 47 year-old female with known peptic ulcer disease
presents with a 3 day history of epigastric pain, profuse
vomiting and inability to tolerate oral fluids. On examination,
she is in moderate pain. Blood pressure is 88/42, pulse rate
97, and mucous membranes are dry.
Serum sodium 124 mEq/L
Serum potassium 3.0 mEq/L
Serum chloride 65 mEq/L
Serum bicarbonate 40 mEq/L
Blood urea nitrogen 56 mg/dL
Serum creatinine 2.1 mg/dL
Serum lactate 8.3 mmol/L
Arterial blood studies on room air:
pH 7.65 PCO2 38 mm Hg

131
Copyright © Harvard Medical School, 2018. All Rights Reserved.

Case 12
Which of the following best describes the acid-
base disorder in this patient?

(A) Metabolic alkalosis and respiratory acidosis


(B) Metabolic alkalosis and respiratory alkalosis
(C) Metabolic alkalosis, respiratory acidosis and
respiratory alkalosis
(D) Metabolic alkalosis, respiratory alkalosis and
metabolic acidosis
(E) None of the above

Case 12
pH 7.65, PCO2 38 mm Hg, HCO3 40 mEq/L
Metabolic Alkalosis

132
Copyright © Harvard Medical School, 2018. All Rights Reserved.

Induction of metabolic alkalosis

Ingestion Loss of acid Cellular


of alkali shift
Antacids ↓ K+
GI loss Renal loss
Blood Tx
Vomiting Diuretics
NG suction Bartter/Gitelman
Hyperaldosteronism

Case 12
pH 7.65, PCO2 38 mm Hg, HCO3 40 mEq/L
Metabolic Alkalosis
PCO2 - 0.6-0.7mm/1Meq change in bicarb or
16(0.6)=9.6 so predicted PCO2~50mmHg
Lack of respiratory compensation indicates
Respiratory Alkalosis:
Na 124 mEq/L, Cl 65 mEq/L
Anion gap = 124 - 65 - 40 = 19 (normal 8-12)
Lactate level = 8.3 mmol/L

Superimposed anion gap Metabolic Acidosis (lactic


acidosis)

133
Copyright © Harvard Medical School, 2018. All Rights Reserved.

Case 12
Which of the following best describes the acid-
base disorder in this patient?

(A) Metabolic alkalosis and respiratory acidosis


(B) Metabolic alkalosis and respiratory alkalosis
(C) Metabolic alkalosis, respiratory acidosis and
respiratory alkalosis
(D) Metabolic alkalosis, respiratory alkalosis and
metabolic acidosis
(E) None of the above

Case 13
22 year-old male with no past medical history
presents with confusion. His serum sodium is 106
mEq/L, serum osmolality 240 mOsm/kg, urine sodium
45 mEq/L and urine osmolality 40 mOsm/kg.

Select the best option (A-E) for treatment of the


serum sodium.

(A) 0.9% NaCl


(B) 3% NaCl
(C) Free water restriction
(D) Hydrocortisone
(E) No treatment

134
Copyright © Harvard Medical School, 2018. All Rights Reserved.

Hypoosmolal hyponatremia

Volume status

Hypovolemic Euvolemic Edematous


Dehydration* CHF*
Addison’s UOsm Nephrotic*
Diuretics Liver failure*
< 100 Renal failure
> 100
SIADH Polydipsia
Hypothyroid
* UNa < 20 = Extrarenal cause of ECV depletion

Case 14
52 year-old female with chronic obstructive pulmonary
disease and 2 month history of worsening dyspnea
presents with a seizure. On examination she appears
confused. BP 125/90, HR 74, mucous membranes moist,
no peripheral edema. Her serum sodium is 110 mEq/L,
serum osmolality 251 mOsm/kg, urine sodium 150 mEq/L
and urine osmolality 710 mOsm/kg.
Select the best option (A-E) for treatment of the serum
sodium.
(A) 0.9% NaCl
(B) 3% NaCl
(C) Free water restriction
(D) Hydrocortisone
(E) No treatment

135
Copyright © Harvard Medical School, 2018. All Rights Reserved.

Hypoosmolal hyponatremia

Volume status

Hypovolemic Euvolemic Edematous


Dehydration* CHF*
Addison’s UOsm Nephrotic*
Diuretics Liver failure*
< 100 Renal failure
> 100
SIADH Polydipsia
Hypothyroid
* UNa < 20 = Extrarenal cause of ECV depletion

Rate of correction of hyponatremia


Acute (< 48 hr, usually due to hypotonic fluid
intake) or severely symptomatic (seizures)
100 mL of 3% saline bolus to increase SNa by
2-3 mEq/L

Chronic (> 48 hr) including SIADH and


asymptomatic
0.5 mEq/l per hour

Do not exceed ∆8-10 mEq/L in 1st day

136
Copyright © Harvard Medical School, 2018. All Rights Reserved.

Case 15
67 year-old male with fatigue and low back pain. Serum
values were: sodium 124 mEq/L, glucose 76 mg/dL, total
protein 13 g/dL, albumin 3.6 g/dL, hemoglobin 9 g/dL.

Select the best option (A-E) for treatment of the serum


sodium.

(A) 0.9% NaCl


(B) 3% NaCl
(C) Free water restriction
(D) Hydrocortisone
(E) No treatment

Hyponatremia

Posm

> 290 mOsm/kg Normal < 275 mOsm/kg


“Pseudohyponatremia”

↑Glucose* Lipid ↑ Hypoosmolal


Mannitol hyponatremia*
Protein ↑
* Requires
ADH+ Water
Intake
*Correct serum Na+ by 1.6 for every 100 mg/dL ∆ in glucose

137
Copyright © Harvard Medical School, 2018. All Rights Reserved.

Case 16
45 year-old male with diabetes mellitus, hypertension and
ischemic cardiomyopathy maintained on aspirin, carvedilol,
captopril, glipizide and furosemide. On examination, BP is
135/94, HR 80, mucous membranes moist, jugular venous
pulsations are visible to the angle of the jaw, and there is 3+
pitting edema of the legs and thighs. His serum sodium is
123 mEq/L, urine sodium 10 mEq/L and urine osmolality
570 mOsm/kg.
Select the best option (A-E) for treatment of the serum
sodium.
(A) 0.9% NaCl
(B) 3% NaCl
(C) Free water restriction
(D) Hydrocortisone
(E) No treatment

Hypoosmolal hyponatremia
Volume status

Hypovolemic Euvolemic Edematous


Dehydration* CHF*
Addison’s UOsm Nephrotic*
Diuretics Liver failure*
> 100 < 100 Renal failure
SIADH Polydipsia
Hypothyroid

* UNa < 20 = Extrarenal cause of ECV depletion

138
Copyright © Harvard Medical School, 2018. All Rights Reserved.

Case 17
43-year-old man with Type II diabetes mellitus,
hypertension, congestive cardiac failure, nephrotic-range
proteinuria, peripheral edema and a serum creatinine of
1.6 mg/dl. His serum potassium has been in the range of
5.3-5.6 mEq/L since starting captopril, despite adhering to a
potassium-restricted diet.

Select the best option (A-E) for treatment of the serum


potassium.

(A) Thiazide diuretic


(B) Hydrocortisone
(C) Insulin
(D) Hemodialysis
(E) Sodium polystyrene sulfonate

Sites of action of natriuretics


Distal convoluted tubule (5-10%)
Thiazide diuretics

Proximal tubule Cortical collecting duct


(60-70%) (1-3%)
Osmotic diuretics K+-sparing diuretics
Carbonic anhydrase inhibitors
Thick ascending limb
(20-30%)
Loop diuretics

139
Copyright © Harvard Medical School, 2018. All Rights Reserved.

Loop and thiazide diuretics cause K+


wasting and alkalosis
↑NaCl
↓TALH Na+ ↑ Na+ ↑ K+ & H+
delivery to
reabsorption reabsorption secretion
CCD

Case 18
88-year-old woman who had partial sigmoid colectomy for
perforated diverticular abscess and septicemia two days
previously, and has been anuric since the operation. Her
serum potassium is 6.5 mg/dL but there are no
electrocardiographic changes.

Select the best option (A-E) for treatment of their serum


potassium.

(A) Thiazide diuretic


(B) Hydrocortisone
(C) Insulin
(D) Hemodialysis
(E) Sodium polystyrene sulfonate

140
Copyright © Harvard Medical School, 2018. All Rights Reserved.

Hyperkalemia

↑ Intake Cell shift


Metabolic acidosis
Decreased urinary Hyperglycemia
K+ excretion β-blocker
24 hr urine K+ < 40 mEq Digitalis
Hyperkalemic
periodic paralysis
Cell lysis

Decreased urinary K+ excretion

↓GFR ↓CCD [K+]


Renal
failure Meds Adrenal Hyporenin
insufficiency hypoaldo
NSAIDs
ACEI/ARB Addison’s
Block Heparin
RAAS Spironolactone
Cyclosporine

Block Na+ Amiloride


channel Trimethoprim
Pentamidine

141
Copyright © Harvard Medical School, 2018. All Rights Reserved.

Treatment of hyperkalemia
Stabilize membrane excitability
Calcium chloride or gluconate, 1 g IV
Increase K+ entry into cells
Glucose 25 g and insulin 10 U
β2-adrenergic agonist (albuterol 10-20 mg inh)
NaHCO3
Removal of excess K+
Cation exchange resin (Kayexalate)
Diuretics
Dialysis
Dietary K+ restriction

Case 19
18-year-old man with no prior medical history who presents
with one week of polyuria and polydipsia.

Labs:
Na 132, K 5.9, Cl 91, HCO3 16, BUN 30, Cr 1.2, glucose 330

Select the best option (A-E) for treatment of their serum


potassium.

(A) Thiazide diuretic


(B) Hydrocortisone
(C) Insulin
(D) Hemodialysis
(E) Sodium polystyrene sulfonate

142
Copyright © Harvard Medical School, 2018. All Rights Reserved.

Hyperkalemia

↑ Intake Cell shift


Metabolic acidosis
Decreased urinary Hyperglycemia
K+ excretion β-blocker
24 hr urine K+ < 40 mEq Digitalis
Hyperkalemic
periodic paralysis
Cell lysis

Case 20
26-year-old woman with acquired immune deficiency
syndrome, fatigue, weight loss, low-grade fever, and
orthostatic hypotension. Na-129; K-5.9
Serum cortisol level:
Baseline at 8 a.m. 7 µg/dL (nl 5-24 µg/dL)
30 minutes after 250 µg cosyntropin i.m. 10 µg/dL
60 minutes after 250 µg cosyntropin i.m. 11 µg/dL
Select the best option (A-E) for treatment of their serum
potassium.
(A) Thiazide diuretic
(B) Hydrocortisone
(C) Insulin
(D) Hemodialysis
(E) Sodium polystyrene sulfonate

143
Copyright © Harvard Medical School, 2018. All Rights Reserved.

Decreased urinary K+ excretion

↓GFR ↓CCD [K+]


Renal
failure Meds Adrenal Hyporenin
insufficiency hypoaldo
NSAIDs
ACEI/ARB Addison’s
Block Heparin
RAAS Spironolactone
Cyclosporine

Block Na+ Amiloride


channel Trimethoprim
Pentamidine

Case 21
For the following cases of hypokalemic metabolic alkalosis,
select the most likely cause (A-E):

16-year-old girl with amenorrhea, body mass index of 13,


and a urine chloride concentration of 5 mEq/L.

(A) Diuretic use


(B) Surreptitious vomiting
(C) Hypokalemic periodic paralysis
(D) Gitelman’s syndrome
(E) Conn’s syndrome

144
Copyright © Harvard Medical School, 2018. All Rights Reserved.

Cryptogenic hypokalemic metabolic


alkalosis

Volume Urine
Urine Cl-
status/BP diuretics
Hyperaldosteronism ↑ > 40 mEq/L -
Surreptitious
Nl or ↓ < 25 mEq/L -
vomiting
Diuretic abuse Nl or ↓ > 40 mEq/L +
Bartter/Gitelman
Nl or ↓ > 40 mEq/L -
syndrome

Case 22
For the following cases of hypokalemic metabolic alkalosis,
select the most likely cause (A-E):

35-year-old man presenting for the first time with new-onset


hypertension.

(A) Diuretic use


(B) Surreptitious vomiting
(C) Hypokalemic periodic paralysis
(D) Gitelman’s syndrome
(E) Conn’s syndrome (hyperaldosteronism)

145
Copyright © Harvard Medical School, 2018. All Rights Reserved.

Hypokalemia/Renal K+
wasting & hypertension

Aldosterone
High Low
Renin

High Low

Renal artery Primary Cushing’s


stenosis hyperaldosteronism Liddle's
Liquorice ingestion

Cortical Collecting Duct (CCD)

H+

Lumen Blood

146
Copyright © Harvard Medical School, 2018. All Rights Reserved.

Case 23
For the following cases of hypokalemic metabolic alkalosis,
select the most likely cause (A-E):

32-year-old woman with a history of bulimia. Random urine


chloride concentrations on three separate clinic visits were
40, 67 and 26 mEq/L.

(A) Diuretic use


(B) Surreptitious vomiting
(C) Hypokalemic periodic paralysis
(D) Gitelman’s syndrome
(E) Conn’s syndrome

Cryptogenic hypokalemic metabolic


alkalosis

Volume Urine
Urine Cl-
status/BP diuretics
Hyperaldosteronism ↑ > 40 mEq/L -
Surreptitious
Nl or ↓ < 25 mEq/L -
vomiting
Diuretic abuse Nl or ↓ > 40 mEq/L +
Bartter/Gitelman
Nl or ↓ > 40 mEq/L -
syndrome

147
Copyright © Harvard Medical School, 2018. All Rights Reserved.

Case 24
For the following cases of hypokalemic metabolic alkalosis,
select the most likely cause (A-E):

15-year-old girl with recurrent episodes of muscle weakness


since childhood and a urine chloride concentration is
65 mEq/L and 24 hour urine K+ is 60 mEq. Her brother has
had similar symptoms.

(A) Diuretic use


(B) Surreptitious vomiting
(C) Hypokalemic periodic paralysis
(D) Gitelman’s syndrome
(E) Conn’s syndrome

Renal K wasting with normal or low BP

Osmotic Drugs Vomiting RTA Inherited


diuresis tubulopathy
Diuretics Proximal Bartter
Loop & thiazide Classic distal Gitelman
Cisplatin
Aminoglycosides
Ticarcillin
Amphotericin
Toluene
How Can One Distinguish Bartter/Gitelman?
Urinary Calcium is Low with Na+Cl-
Cotransporter Inhibition (Thiazides)

148
Copyright © Harvard Medical School, 2018. All Rights Reserved.

Case 25
For the following cases of hypokalemia, select the most
likely cause (A-E):

20-year-old man with thyrotoxicosis and recurrent episodes


of muscle weakness after meals.

(A) Diuretic use


(B) Surreptitious vomiting
(C) Hypokalemic periodic paralysis
(D) Gitelman’s syndrome
(E) Conn’s syndrome

Features suggestive of hypokalemic


periodic paralysis

+FH or Asian male with thyrotoxicosis

Precipitated by meal or exercise

Repetitive episodes of acute profound hypokalemia

Recovery of serum K+ within hrs after each episode


without repletion, either spontaneously or with
propanolol

Low urine K+

149
Copyright © Harvard Medical School, 2018. All Rights Reserved.

Case 26
All of the following drugs can result in
Hypomagnesemia EXCEPT:

A. Ranitidine
B. Furosemide
C. Hydrocholorthiazide
D. Omeprazole
E. Gentamicin

PPI’s and Hypomag


Several large studies show association of
Hypomag with long-term PPI use (>1 year) or
If concurrently taking diuretics
GI mechanism of inhibition of GI TRP channel;
Renal losses low (FeMg<2%) consisent with non-
renal losses
Safety warning issued by FDA in 2011
Resolves with discontinuation of drug
Danziger et al., Kidney Int 2013; 83:692.

150
Copyright © Harvard Medical School, 2018. All Rights Reserved.

Suggested reading
Rennke, H.G., Denker, B.M., Renal Pathophysiology – The
Essentials, 4th Edition, Lippincott Williams & Wilkins, 2014

Mount, D.B., Fluid and Electrolyte Distrubances. In Harrison's


Principles of Internal Medicine, 18th Edition, Eds. Longo, Fauci, et
al., McGraw-Hill, p. 341-359

DuBose, T.D.,Jr. Acidosis and Alkalosis. In Harrison's Principles of


Internal Medicine, 18th Edition, Eds. Longo, Fauci, et al., McGraw-
Hill,p. 363-373

151
Copyright © Harvard Medical School, 2018. All Rights Reserved.

Dialysis and Transplantation


J. Kevin Tucker, M.D.
Chief of Nephrology
Brigham and Women’s/Faulkner
Hospital

Disclosures
• None

152
Copyright © Harvard Medical School, 2018. All Rights Reserved.

Outline
• Scope of the problem
– Demographics
• Preparing the patient for renal
replacement
• Hemodialysis
• Peritoneal dialysis
• Transplantation

Renal replacement therapy


• General nomenclature used to
designate the forms of therapy to
replace kidneys that no longer are
adequate to maintain life
– Hemodialysis
– Peritoneal dialysis
– Transplantation

153
Copyright © Harvard Medical School, 2018. All Rights Reserved.

Trends in the incidence rates of ESRD in the U.S. population

Data Source: Reference Table A.2(2) and special analyses, USRDS ESRD Database

2017 Annual Data Report


Volume 2, Chapter 1

Trends in the number of ESRD prevalent cases by


modality

Data Source: Reference Table D.1. Abbreviation: ESRD, end-stage renal disease

2017 Annual Data Report


Volume 2, Chapter 1

154
Copyright © Harvard Medical School, 2018. All Rights Reserved.

Trends in adjusted prevalence of ESRD by race

Data Source: Reference Table B.2(2) and special analyses, USRDS ESRD Database

2017 Annual Data Report


Volume 2, Chapter 1

Indications for dialysis


• Acid-base disturbances
• Electrolyte abnormalities
• Ingestions
• Fluid Overload
• Uremia
– Nausea/Vomiting
– Anorexia
– Dysgeusia
– Pruritus
– Pericarditis

155
Copyright © Harvard Medical School, 2018. All Rights Reserved.

Renal replacement decision tree

RRT No RRT

Transplant
Dialysis

Home Incenter

Renal replacement decision tree

Home
Incenter

PD Home
hemo

156
Copyright © Harvard Medical School, 2018. All Rights Reserved.

If dialysis is chosen
• Conservative (non-dialysis) therapy may
be appropriate in some patients
– No increase in survival and no
improvement in quality of life in frail elderly
who are started on dialysis
• Create a vascular access or place a
peritoneal dialysis catheter
• Manage metabolic complications
• Manage nutrition

What is the ideal time to initiate


dialysis?
• Trend towards earlier dialysis start, i.e.,
at a higher GFR through 1990’s and mid
2000’s
– Are outcomes better with earlier starts?
– Earlier starts driven by a factor that is not
GFR-dependent, for example, volume
control?

157
Copyright © Harvard Medical School, 2018. All Rights Reserved.

Trends in initiation of dialysis:


Dialysis initiated at higher GFRs over period of
1997-2007

O’Hare AM et al Arch Intern Med. 2011;171(18):1663-1669

The IDEAL Study

Cooper BA et al. N Engl J Med 2010;363:609-619

158
Copyright © Harvard Medical School, 2018. All Rights Reserved.

Kaplan–Meier Curves for Time to the Initiation of


Dialysis and for Time to Death

Cooper BA et al. N Engl J Med 2010;363:609-619

Incremental Dialysis
• Does every patient need the same
dialysis prescription irrespective of
residual kidney function?
• An incremental approach to dialysis
initiation takes into account residual
kidney function
– 1-2 days per week of HD
– 1-2 exchanges per day with CAPD

159
Copyright © Harvard Medical School, 2018. All Rights Reserved.

Putative benefits of incremental


initiation
• Better quality of life
• Preservation of residual kidney function
– Fewer hypotensive events and less
ultrafiltration lead to prolonged
preservation of native kidney function
– Native kidney function confers a survival
benefit

Hemodialysis

160
Copyright © Harvard Medical School, 2018. All Rights Reserved.

Essential Components of
Hemodialysis Procedure
• Dialyzer
• Dialysate
• Access to circulation

Hemodialysis
• Hemodialysis is a two-step procedure:
– Diffusion/Convection
– Ultrafiltration

161
Copyright © Harvard Medical School, 2018. All Rights Reserved.

Diffusion and Ultrafiltration

Dialyzers
• Most commonly used
dialyzers in the US are
hollow-fiber dialyzers.
• Dialyzer shell is a tube with
four ports.
• Two ports communicate with
the blood compartment, and
two ports communicate with
the dialysate compartment.
• Blood flows through the
fibers, and dialysate around
the outside.

162
Copyright © Harvard Medical School, 2018. All Rights Reserved.

Dialysate Composition (mEq/L)


Sodium 135-145
Potassium 0-4.0
Calcium 2.5-3.5
Magnesium 0.5-0.75
Chloride 98-124
Acetate 2-4
Bicarbonate 30-40
Dextrose 11

Hemodialysis vascular access types

• AV fistula
• AV graft
• Catheter
– Tunneled
– Non-tunneled

163
Copyright © Harvard Medical School, 2018. All Rights Reserved.

AV fistula
• The preferred vascular access
• Fewer interventions needed to maintain
patency
• Fewer infectious complications
• Tradeoff:
– More primary failures
– More patients initiating with catheters

AV grafts
• More interventions required to maintain
patency
• Higher infection rates

164
Copyright © Harvard Medical School, 2018. All Rights Reserved.

AV fistula

The AV Graft

165
Copyright © Harvard Medical School, 2018. All Rights Reserved.

Fistula First Initiative

Catheters
• Least desirable
vascular access
• Greater infection
risk
• Pro-inflammatory

166
Copyright © Harvard Medical School, 2018. All Rights Reserved.

Outpatient hemodialysis
• Once the patient is started on HD, he/she is
referred to an outpatient unit, usually closet to
his/her home.

167
Copyright © Harvard Medical School, 2018. All Rights Reserved.

Complications of hemodialysis
• Vascular access complications
– Thrombosis
– Infection
• Hypotension
• Cramping
• Air embolism
• Hemolysis

Vascular access infection


• Infection is the second leading cause of
death in ESRD patients.
• Catheters are the vascular access type
most associated with infections.
• Catheter-related bacteremia may be
due to Gram positive or Gram negative
organisms.
• Nasal carriage of Staph aureus is a risk
factor.

168
Copyright © Harvard Medical School, 2018. All Rights Reserved.

Catheter-associated bacteremia
treatment
• Removal of the catheter and treatment
with systemic antibiotics is the gold
standard
• Other approaches
– Catheter exchange with guidewire plus
systemic antibiotics
– Systemic antibiotics plus an antibiotic lock

Knowledge Check (Audience


Response)
The preferred vascular access for
hemodialysis in a 26-year-old man with
ESRD due to glomerulonephritis is:
A) AV graft
B) Tunneled catheter
C) AV fistula
D) Non-tunneled catheter

169
Copyright © Harvard Medical School, 2018. All Rights Reserved.

Peritoneal dialysis

Physiology of PD
• The peritoneal membrane can serve as
a diffusive surface for solutes to move
from areas of high concentration to low
concentration
• An osmotic gradient for fluid removal
(ultrafiltration) is provided by glucose.

170
Copyright © Harvard Medical School, 2018. All Rights Reserved.

Composition of dialysate
Volume 2, 2.5, 3L most commonly for CAPD
6L for APD
Sodium 132 mEq/L
Potassium 0
Glucose 1.5, 2.5, 4.25 g/dL
Calcium 2.5, 3.5 mEq/L
Magnesium 0.5-1.5 mEq/L
Lactate 35-40 mEq/L

171
Copyright © Harvard Medical School, 2018. All Rights Reserved.

Types of PD
• CAPD = Continuous ambulatory
peritoneal dialysis
– Manual exchanges
• CCPD = Continuous cycling peritoneal
dialysis
– Use of an automated cycler with a long
daytime dwell

Education affects Choice


National Average Patients’ Choice after
Thorough Education

100% 60%

80% 50%
40%
60%
30%
40%
20%
20% 10%
0% 0%
PD HD PD HD

Patients more likely to choose PD when provided thorough


education on treatment options.
4. Renal Data Review, Nephrology News and Issues, Sept. 1997
5. Stephenson, Kerry RN and Rosalie Villano, “Results of a Pre-
Dialysis Education Program,” Dialysis and Transplantation, Sept. 1993

172
Copyright © Harvard Medical School, 2018. All Rights Reserved.

Utilization of PD in the United


States
• PD is underutilized in the United States
– About 8% of all dialysis patients
• When pre-dialysis modality education is
provided, more patients will choose PD.
• PD patients report greater quality of life
and greater satisfaction with therapy

Barriers to utilization of PD
• Patient factors
– Lack of knowledge about PD
– “I’m afraid.”
– Poor social supports
– Technical problems, e.g., limited
manual dexterity
– Aging patient population

173
Copyright © Harvard Medical School, 2018. All Rights Reserved.

Barriers to utilization of PD
• Physician factors
– Lack of knowledge
– Concerns about
• Adequacy
• Infection
• Ultrafiltration
• Mortality

Complications of PD
• Peritonitis
– The leading cause of transfer from PD to
HD.
• Hydrothorax
• Hemoperitoneum
• Encapsulating peritoneal sclerosis

174
Copyright © Harvard Medical School, 2018. All Rights Reserved.

Clinical Signs and Symptoms


of Peritonitis
• Fever
• Abdominal pain
• Abdominal tenderness/rebound
• Cloudy effluent
• Nausea/vomiting

Laboratory studies
• Effluent cell count
– WBC > 100/uL with 50% PMNs
• Effluent Gram stain
• Effluent culture
• Blood cultures

175
Copyright © Harvard Medical School, 2018. All Rights Reserved.

Treatment of peritonitis
• Bacterial peritonitis
– Intraperitoneal antibiotics (vancomycin +
ceftazidime, for example)
– Catheter removal if infection does not clear
• Fungal peritonitis
– Prompt catheter removal
– Systemic anti-fungals

Prevention of peritonitis
• Nasal carriage of Staph aureus is a risk
factor.
• Daily application of a topical antibiotic
reduces episodes of peritonitis and exit-
site infection
– Mupirocin
– Gentamicin

176
Copyright © Harvard Medical School, 2018. All Rights Reserved.

Knowledge check (Audience


Response)
Which of the following is the leading
cause of modality change from peritoneal
dialysis to hemodialysis?
A) Exit site infection
B) Membrane failure
C) Peritonitis
D) Encapsulating peritoneal sclerosis

Kidney transplantation
• The optimal form of renal replacement
• Limited by organ availability
• Organ sources:
– Living related donors
– Living unrelated donors
– Deceased donors
• Extended criteria donors

177
Copyright © Harvard Medical School, 2018. All Rights Reserved.

Medical evaluation of the transplant


recipient
• Kidney recipients require
multidisciplinary team evaluation:
– Surgeon
– Nephrologist
– Social worker
– Pharmacist
– Psychiatrist

Medical evaluation
• Cardiovascular disease
• Malignancies
• Infections
– HIV
– Hepatitis B
– Hepatitis C
– Syphilis
– CMV
– EBV

178
Copyright © Harvard Medical School, 2018. All Rights Reserved.

Immunosuppression for
kidney transplantation
Glucocorticoids Block cytokine synthesis
Weight gain, hyperglycemia,
cataracts osteoporosis
Azathioprine Inhibits purine biosynthesis
Major interaction with allopurinol
Mycophenolate Selective effect on lymphocyte
replication
GI side effects
Cyclosporine Calcineurin inhibitor
Gingival hyperplasia
Hypertension
Hirsutism
Tacrolimus Calcineuin inhibitor
Diabetes
Hypertension
Neurotoxicity

Immunosuppression for
kidney transplantation
Sirolimus mTOR inhibitor
Bone marrow
suppression
Hyperlipidemia
Monoclonal antibodies Block activated T-cells
(basiliximab and expressing IL2 receptor
daclizumab)
Polyclonal antibodies Non-specifically block T-
(ATGAM and cells
thymoglobulin)

179
Copyright © Harvard Medical School, 2018. All Rights Reserved.

Drugs/susbtances that alter


cyclosporine metabolism
Increase level of cyclosporine Decrease level of cyclosporine
Diltiazem Barbiturates
Verapamil Phenytoin
Nicardipine Carbemazepine
Amlodipine Isoniazid
Ketoconazole Rifampin
Fluconazole
Erythromycin
Clarithromycin
Grapefruit juice

Infectious complications post-


transplant
• First month
– Surgical wound infections
– Urinary tract infections
– Catheter-related bacteremia

180
Copyright © Harvard Medical School, 2018. All Rights Reserved.

Infectious complications post-


transplant
• 1-6 months
– CMV
– EBV
– Pneumocystis jiroveci
– Nocardia
– Listeria monocytogenes
– Prophylaxis with TMP-SMX, dapsone,
valgancyclovir

Infectious complications post-


transplant
• After 6 months
– Infectious risk decreases as
immunosuppression is reduced.
– CMV disease may occur late
• Fever
• Leukopenia
• Malaise
• Allograft dysfunction

181
Copyright © Harvard Medical School, 2018. All Rights Reserved.

Non-infectious complications
of kidney transplantation
• Cardiovascular disease
• NODAT
• Malignancies
– Skin cancer
– Cervical cancer
• Post-transplant lymphoproliferative
disorder

Knowledge check (Audience


Response)
Pneumocystis jiroveci infection occurs most
commonly in kidney transplant patients during
which time period?
A) The first month post-transplant
B) 1-6 months post-transplant
C) 6-12 months post-transplant
D) 5-10 years post-transplant

182
Copyright © Harvard Medical School, 2018. All Rights Reserved.

Summary
• Renal replacement options for the patient
with advanced CKD include transplantation,
hemodialysis, and peritoneal dialysis.
• Conservative (non-dialytic) treatment may be
appropriate for some patients with advanced
CKD.
• Preparation for renal replacement shoud
include the timely placement of a vascular
access or PD catheter.

Summary
• The AV fistula is the preferred vascular
access for hemodialysis.
• Nasal carriage of Staph aureus is a risk
factor for infectious complications in
both hemodialysis and peritoneal
dialysis.
• Peritonitis is the leading cause of
transfer from HD to PD.

183
Copyright © Harvard Medical School, 2018. All Rights Reserved.

Summary
• Kidney transplantation is the optimal
form of renal replacement but limited by
organ availability.
• Infections, cardiovascular disease,
diabetes, and malignancies are
complications of transplantation for
which patient should be monitored.

Question 1
• A 52-year-old man with ESRD
secondary to diabetes is maintained on
an immunosuppressive regimen of
cyclosporine, mycophenolate mofetil,
and prednisone. He develops post-
transplant hypertension. Which of the
following antihypertensive medications
may affect cyclosporine levels?

184
Copyright © Harvard Medical School, 2018. All Rights Reserved.

Question 1
• A. Losartan
• B. Enalapril
• C. Amlodipine
• D. Hydrochlorothiazide

Question 1
• The correct answer is C. Of the choices
given, amlodipine is the only one that
increases cyclosporine levels.

185
Copyright © Harvard Medical School, 2018. All Rights Reserved.

Question 2
• A 23-year-old woman with lupus
nephritis has progressed to stage 4
CKD despite aggressive treatment with
cytotoxic agents. A recent kidney
biopsy has shown advanced fibrosis.
Her BMI is 30 kg/m2. All of the following
are true with respect to renal
replacement options EXCEPT:

Question 2
• A. She should be educated regarding
hemodialysis, peritoneal dialysis, and
transplantation.
• B. She should be encouraged to lose weight
prior to renal transplantation.
• C. Her obesity excludes her as a candidate
for peritoneal dialysis.
• D. She should be referred to a vascular
surgeon for immediate creation of an AV
fistula is her renal replacement choice is HD.

186
Copyright © Harvard Medical School, 2018. All Rights Reserved.

Question 2
• The correct answer is C. She should be
educated regarding all renal
replacement options. If HD is her
choice, she needs an AVF. She should
be encouraged to lose weight since
weight gain and diabetes are common
after transplantation. However, obesity
is not a contraindication to PD.

References
• Bernardini et al 2005, J Am Soc Nephrol
16:539-545
• Konner K et al 2003, J Am Soc Nephrol
14: 1669-1680
• Mehrotra et al 2006, Kidney Int 68: 378-
390
• Silkensen 2000, J Am Soc Nephrol 11:
582-588

187
Copyright © Harvard Medical School, 2018. All Rights Reserved.

Disclosures
• None

188
Copyright © Harvard Medical School, 2018. All Rights Reserved.

Take Home Messages


IRIM 2018

Ajay K. Singh, MB., FRCP


Physician, Renal Division,
Brigham and Women’s Hospital,
Senior Associate Dean for Postgraduate
Medical Education,
Harvard Medical School

Disclosures

GSK - Consultant

189
Copyright © Harvard Medical School, 2018. All Rights Reserved.

Clinical Scenarios In General Nephrology


• #1 CKD-Anemia
– 62-year-old woman with CKD and anemia
• #2 CKD-Hyperkalemia
– A 44-year old patient with CKD who has a K of 7.2
mEq/L
• #3 ADPKD
– 42-year old patient with known ADPKD inquiring
about ways to slow progression of PKD
• #4 Kidney Stones
– 37-year-old man. After a few twinges over past 2
months, presents with 2 hours of excruciating pain in
left groin

#1 62-year-old woman with CKD and


anemia

62-yo white woman with 10-year history of CKD


from diabetes; slowly worsening renal function.
Past medical history of a right CVA stroke. She
sees you in the office. Feels great. Working,
exercising, eating well. Lab data shows BUN 48
mg/dL, Cr 4.2 mg/dL, eGFR 18 ml/min/1.73m2, Hb
9.1 g/dL, Tsat 30%, ferritin 282.

190
Copyright © Harvard Medical School, 2018. All Rights Reserved.

Anemia Treatment in Dialysis


and Non-Dialysis Patients
• Target range - Hb 10-11 g/dL
• Make sure patient is iron replete
• TSAT >20%, Ferritin >100
• Use an ESA (2 ESA’s in US – Epo or
Darbepoietin
• Be cautious with ESA treatment in patients with
a history of stroke and/or cancer

TREAT Composite Endpoints

Pfeffer MA et al. N Engl J Med. 2009;361:2019-2032.

191
Copyright © Harvard Medical School, 2018. All Rights Reserved.

Fatal and Nonfatal Stroke


50
Darbepoetin alfa 101 (5.0%)
Patients With Events (%)

Risk of stroke doubles among


40 Placebo 53 (2.6%)
Patients on ESA with a h/o prior
Stroke (Skali, et al Circulation 2011)
30
HR: 1.92 (1.38–2.68)
20 P<0.001

10

0
0 6 12 18 24 30 36 42 48

Months
Pfeffer MA et al. N Engl J Med. 2009;361:2019-2032.

CHOIR Study: Hb 13 vs. 11.3 g/dL


in non-dialysis CKD patients
30%
125
Randomized Treatment
Hemoglobin Target 13.5 g/dL
Hemoglobin Target 11.3 g/dL
25%
97
Kaplan-Meier Failure Estimate (%)

Hazard ratio 1.337 (1.025, 1.743)


20% P= 0.0312

15%

10%

5%

0%
0 6 12 18 24 30 36
Months from Randomization
715 587 457 270 101 55 0
717 594 499 293 107 44 3
Primary Composite Endpoint:
Death, MI, CHF hosp (no RRT) and/or stroke

Singh AK et al. N Engl J Med. 2006;355:2085-2098.

192
Copyright © Harvard Medical School, 2018. All Rights Reserved.

CHOIR: Higher risk of Death


and CHF
Death 65 deaths CHF Hospitalization (where RRT
20% 20%
did not occur)
p = 0.0674
Hazard ratio 1.483 p = 0.0727
15% 15%
(0.969, 2.268) Hazard ratio 1.409
(0.967, 2.054)
10% 10%

5% 5%

0% 0%
0 6 12 18 24 30 36 0 6 12 18 24 30 36

Stroke Myocardial Infarction


20% 20%
p = 0.9803 p = 0.7836
15% Hazard ratio 1.010 15% Hazard ratio 0.915
(0.454, 2.249) (0.484, 1.729)
10% 10%

5% 5%

0% 0%
0 6 12 18 24 30 36 0 6 12 18 24 30 36
Months from Randomization Months from Randomization

Randomized Treatment Hemoglobin Target 13.5 g/dL Hemoglobin Target 11.3 g/dL
Singh AK et al. N Engl J Med. 2006;355:2085-2098.

ESAs Currently in Use in the U.S.

Source: Fishbane S et al, 2013


http://www.nephrologynews.com/articles/109496-choice-of-erythropoiesis-stimulating-agent-in-esrd

193
Copyright © Harvard Medical School, 2018. All Rights Reserved.

#2 A 44-year old patient with CKD who


has a K of 7.2 mEq/L

• A 44-year old woman with a history of stage 4


CKD is noted to have a serum K of 7.2 mEq/L.
She denies N, V and diarrhea. No new
medications
• On physical examination. Wt 72 Kg, BP 141/62
mmHg, Heart and lung examination normal,
no edema.
• EKC – see next slide….

ECG Changes of hyperkalemia

http://www.aafp.org/afp/2006/0115/p283.html
Peaked T waves
P wave wide and flat
Prolonged QRS interval with bizarre QRS morphology, High-grade AV block with slow
junctional and ventricular escape rhythms, Conduction block (bundle branch blocks,
fascicular blocks)
(Development of a sine wave appearance (a pre-terminal rhythm))

194
Copyright © Harvard Medical School, 2018. All Rights Reserved.

Causes of Hyperkalemia
• Increased intake
– K+ supplements, diet, transfusions, iatrogenic
• Decreased renal excretion
– Renal disease, particularly with type IV RTA
– DRUGS (e.g., potassium-sparing diuretics (eg,
spironolactone, triamterene, amiloride; NSAIDs)
– Adrenal insufficiency
• Intra → extracellular shifts
– Hyperosmolarity
– Insulinopenia
– Metabolic acidemia
– DRUGS (e.g., beta-blockade)
• Artifactual
– in vitro hemolysis, leukocytosis, thrombocytosis
– “pseudohyperkalemia”

Management of Hyperkalemia as an
Outpatient

K< 5.5 mEq/L K 5.5-6.0 mEq/L K>6.0 mEq/L

No structural CVD disease Do EKG Emergency treatment


Chronically on high side Hold K raising meds
Hold K raising meds Do EKG
Treat with resin If EKG changes
Recheck Ca gluconate
Then Insulin/Dex/resin
Structural CVD disease dialysis

Do EKG No EKG Changes


Hold K raising meds Recheck K
Diet Insulin/dextrose/resin
Treat with resin Diet

195
Copyright © Harvard Medical School, 2018. All Rights Reserved.

Treatment of Hyperkalemia
Mechanism Therapy Dose Onset Duration
Stabilize membrane Calcium 10% Ca-gluconate, 1-3 min. 30-60
potential 10 ml over 10 min. min
Cellular K+ uptake Insulin 10 U R with 50 ml 30 min. 4-6 h
of D50, if BS<250

β2-agonist nebulized albuterol, 30 min. 2-4 h


10 mg
+
K removal Kayexalate 30-60 g PO hours ?
ZS9 5-10 g PO hours

Hemodialysis Immediate
• ZS-9 is an inorganic cation exchanger with a high
selectivity for potassium
• Binds nine times as much potassium as Kayexalate, an
organic polymer resin.
• ZS-9 comes as a fine powder that dissolves in water
and is tasteless.

Bicarb

Blumberg et al, Am J Med, 85, 1988

196
Copyright © Harvard Medical School, 2018. All Rights Reserved.

Changes in plasma K during IV infusion of


bicarbonate in HD patients

Values= means + SE
*P<0.5, + P<0..01 vs. baseline

Blumberg et al KI, 41: 369-374, 1992

Kayexalate/SPS Complications
• Ischemic colitis and colonic necrosis
- ↑risk in enema form
- often fatal
- ↑ risk with sorbitol - but can occur without sorbitol and is
associated with intestinal SPS crystals
- post-transplant and post-op patients at ↑ risk

• Volume overload
• Reduction in serum calcium
• Iatrogenic hypokalemia

197
Copyright © Harvard Medical School, 2018. All Rights Reserved.

#3 42-year old patient with known ADPKD


inquiring about ways to slow progression of
PKD.
• A 42-year old woman with a history of ADPKD
diagnosed in her 20s with a strong family
history wants to know about treatments for
ADPKD
• On physical examination. Wt 74 Kg
(unchanged from 2 weeks back), BP 141/62
mmHg, HR 82 bpm, otherwise normal exxcept
abdominal fullness in flanks bilaterally
• BUN 34, Cre 1.5

ADPKD
• Most common genetic disease
– Incidence 1:500 – 1:1000 live births
• Mutation in ~70% located on short arm of chromosome 16p
(PKD1 locus)
• ~30% of mutation is located on chromosome 4q21-q23 milder
phenotype with PKD2 locus.
• Clinical Manifestations
– abdominal mass
– chronic flank or back pain
– gross hematuria
– recurrent UTI
– nephrolithiasis (uric acid stones)

198
Copyright © Harvard Medical School, 2018. All Rights Reserved.

Ultrasound Criteria for Diagnosis


of PKD1 in At-Risk Individuals
Positive and negative predictive values 97-100%
Ravine et al, Lancet 343:824, 1994
• Age < 30: at least 2 cysts (unilateral or bilateral)
• Age 30-59: at least 2 cysts/kidney
• Age > 60: at least 4 cysts/kidney

• For PKD2 age 30-59, use 4 or more cysts in both


kidneys for sensitivity of 96%
Pei et al, JASN 14:107A, 2003

Progressive Loss of Kidney Function


• Rate of decline of GFR
(data from MDRD study,
starting at GFR <55
ml/min)
Males 5 - 6 ml/min/year
Females 4 - 5
ml/min/year
– GFR stable for many
years, despite
progressive increase
in total kidney volume
– GFR decrease not
detected until total
kidney volume
exceeds 1500 ml

199
Copyright © Harvard Medical School, 2018. All Rights Reserved.

Treatment of ADPKD
• Progressive kidney insufficiency
– Lack of proven benefit of low protein diets or RAAS blockade
• Rigorous blood-pressure control in early PKD (Schrier
RW et al NEJM 2014)
– slower increase in total kidney volume
– no overall change in the estimated GFR
– greater decline in the left-ventricular-mass index
– greater reduction in urinary albumin excretion.
• High water intake
• Extrarenal manifestations
– Intervene as needed for symptoms
– Screen for cerebral aneurysms with + family history;
antibiotic prophylaxis for valvular regurgitation
– Avoid estrogen/progesterone in women (effect on liver cyst
disease)

#4 37-year-old man. After a few twinges


over past 2 months, presents with 2 hours of
excruciating pain in left groin
• A 37-year old man presents with 2 hrs of excrcating
pain in the left groin. Started in left flank. Episode of
gross hematuria. PMH of rt flank twinges.
• On physical examination. Looked in pain (writing,
sweaty), Wt 72 Kg, BP 155/80 mmHg, HR 105 bpm,
afebrile, JVP 8 cm, normal skin turgor, moist mucous
membranes, lungs clear, mild tenderness left flank, no
guarding, no edema.
• Na 140, K 4.2, Cl 100, BUN 24, Cre 1.1, BG 98 mg/dL.
UA SG 1015, pH 5.0, 4+ blood, 1+ leuks, rest neg. Sed
TNTC RBCs not dysmorphic, ocass WBC

200
Copyright © Harvard Medical School, 2018. All Rights Reserved.

Acute Management of a Stone Episode


Stone NO Probably uric acid
Radiopaque stone
YES

Fluid and pain Fluid and pain


medication Medication
Alkalinize urine
YES K citrate
Stone >5 mm Percutaneous
NO Lithotomy
Staghorn calculus Plus
Strain urine ESWL
Continue hydration Urologic NO
Evaluation Calyceal or upper
Ureter calculus ESWL
Stone passes
YES NO
Uretroscopy
Renal US in 2 weeks if Distal uretral or
Hydronephrosis or calcuus ESWL
Multiple stones
SOURCE: Adapted from Uptodate

Work-up
• Imaging
– Non-Contrast helical CT with Stone protocol is gold
std (detects stones not visible by KUB/IVP and has
significantly better sensitivity/specificity)
– Ultrasound: For patients needing avoidance of
radiation (pregnant, childbearing age)
– IVP: No longer favored due to lower sensitivity,
HIGHER radiation exposure
– KUB: Will miss radiolucent uric acid stones, small
stones, stones with overlying bony structures.

201
Copyright © Harvard Medical School, 2018. All Rights Reserved.

What stones don’t show up on imaging

• 85% of stones are radio-opaque


– Ca containing
– Cystine
• 15% radiolucent
– Uric acid
– Indinavir

Treatment
• Urologic Intervention?
– X<5mm : most pass spontaneously. Possible
observation and pain control
– X>5mm : less than 20% chance of passage and may
need urologic intervention
• So when to consult urology?
– If > 5mm
– For ANY size with ….
• Urosepsis, AKI, anuria, unyielding N/V/Pain ->
Inpatient consult
• Failed conservative management and stone did not
pass spontaneously -> Inpatient or Outpatient
consult depending on severity

202
Copyright © Harvard Medical School, 2018. All Rights Reserved.

Risk Factors for Stone Formation


• Dehydration
– concentrates stone forming constituents
• Anatomic abnormalities
– promote stasis, infection and/or crystal adhesion
• Changes in urinary pH
– e.g. calcium oxalate less soluble in alkaline urine
• Diet
– high protein / salt intake promotes hypercalciuria
– diet high in oxalate promotes oxaluria
• Medications
– furosemide /ca wasting; acetozolamide /bicarb

Stones
• Calcium oxalate
– 70-80%
• Uric Acid
– 10-15%
• Magnesium ammonium phosphate (struvite infection
related)
– 10-15%
• Cystine
– <1%
• Others (eg Indinavir, Triamterene)
– <1%

203
Copyright © Harvard Medical School, 2018. All Rights Reserved.

References
• N Engl J Med 2000; 342:1581-1589
• Blumberg et al KI, 41: 369-374, 1992
• N Engl J Med 2015; 372:222-231
• JASN 14:107A, 2003
• N Engl J Med 2014; 371:2267-2276
• N Engl J Med 2014; 371:2255-2266
• Clin J Am Soc Nephrol. 2009 Jul; 4(7): 1183–1189.
• MED ARH 2011; 65(4): 213-215
• Am Fam Physician. 2011 Dec 1;84(11):1234-1242.
• Ann Intern Med. 2003;139:137-147

204
Copyright © Harvard Medical School, 2018. All Rights Reserved.

Renal Board Review

Intensive Review of Internal Medicine


2018
Ajay K. Singh MBBS, FRCP, MBA
Renal Division
Brigham and Women’s Hospital Senior Associate
Dean, Postgraduate Medical Education
Harvard Medical School

Harvard Medical School

Disclosures

• Consultant
GSK

205
Copyright © Harvard Medical School, 2018. All Rights Reserved.

Question 1
A 78 year old woman with DM, CAD, and CKD (Cre=2.4 mg/dL),
has persistent HTN, despite being treated with 5 medications
(lisinopril 40 mg, amlodipine 10 mg, furosemide 40 mg bid,
metoprolol 50 mg, and diltiazem 120 mg). She is asymptomatic.
She tells you that she is often dizzy when she wakes up in the
morning. She checks her BP at home, and says it is typically 100
– 120 systolic. On exam, her BP is 164/70 mmHg, non-
orthostatic, HR 54 bpm, but otherwise unremarkable. Her UA
shows 2+ albumin. BUN=42, Cre=2.38, K=4.1.
- Which one of the following next steps is most appropriate?
A.) Increase the dose of her diltiazem to achieve better BP
control
B.) Ask her to continue to do home BP measurements and see
her again in 3 months time.
C.) Do a screen for furosemide in her urine to see if she is being
adherent with her medications
D.) Set her up for ambulatory BP monitoring
E.) Start her on a clonidine patch.

Question 1
A 78 year old woman with DM, CAD, and CKD (Cre=2.4 mg/dL),
has persistent HTN, despite being treated with 5 medications
(lisinopril 40 mg, amlodipine 10 mg, furosemide 40 mg bid,
metoprolol 50 mg, and diltiazem 120 mg). She is asymptomatic.
She tells you that she is often dizzy when she wakes up in the
morning. She checks her BP at home, and says it is typically 100 –
120 systolic. On exam, her BP is 164/70 mmHg, non-orthostatic,
HR 54 bpm, but otherwise unremarkable. Her UA shows 2+
albumin. BUN=42, Cre=2.38, K=4.1.
- Which one of the following next steps is most appropriate?
A.) Increase the dose of her diltiazem to achieve better BP control
B.) Ask her to continue to do home BP measurements and see her again
in 3 months time.
C.) Do a screen for furosemide in her urine to see if she is being adherent
with her medications
√ D.) Set her up for ambulatory BP monitoring
E.) Start her on a clonidine patch.

206
Copyright © Harvard Medical School, 2018. All Rights Reserved.

Clinic 10-40%

Pressure White Coat Sustained


Hypertension Hypertension

140/90
120/80
True Masked
Normotension Hypertension
5-20%

MH officeblood pressure (BP) level is 135/85


‘normal” but ambulatory or home BP 120/80 Ambulatory Pressure
readings are in the hypertensive range

Recommendations for Clinical Use of


ABPM: JNC 7 & WHO-ISH

JNC 7
WHO-ISH
ABPM endorsed Yes Yes
Indications:
White Coat HTN Yes Yes
Labile BP Yes Yes
R/O hypotensive episodes Yes Yes
Resistant HTN Yes Yes
Autonomic dysfunction Yes No

207
Copyright © Harvard Medical School, 2018. All Rights Reserved.

SOURCE: http://visualmed.org/relationship-between-clinic-and-ambulatory-
blood-pressure-measurements-and-mortality/

208
Copyright © Harvard Medical School, 2018. All Rights Reserved.

Question 2
A 52 year old American of Asian origin sees you for
routine follow-up. She says her blood pressure is running
in the 135-140 mmHg systolic. She feels well.
Asymptomatic.
Medications: Lisinopril 20 mg/d, HCTZ 25 mg BID,
Amlodipine 10 mg QD, atorvostatin, metformin, glipizide
PE: Looks well. BP 138/72 mmHg, HR 80 bpm, soft left
and right carotid bruit. Soft SEM aortic area. No rales. No
edema. No peripheral pulses.
UA SG 1015, pH 5.5, 1+ protein otherwise neg
Scr 1.5 mg/dL, BUN 20, K 4.4 mEq/L.

What do You do?


• A.) Add a beta blocker (atenolol) and aim for
a BP of 120/80 mmHg
• B) Add a beta blocker and aim for the JNC8
goal of 140/90
• C) Add a beta blocker and aim for <140/90
mmHg
• D) Make no changes and reassure the patient

209
Copyright © Harvard Medical School, 2018. All Rights Reserved.

Question 2
What do You do?
A.) Add a beta blocker (atenolol) and aim for
a BP of 120/80 mmHg
• B) Add a beta blocker and aim for the JNC8
goal of 140/90
• C) Add a beta blocker and aim for <140/90
mmHg
• D) Make no changes and reassure the patient

Whelton PK, et al 2017


ACC/AHA/AAPA/ABC/ACPM/AGS/APhA/ASH/ASPC/NMA/
PCNA guideline for the prevention, detection, evaluation,
and management of high blood pressure in adults: a report
of the American College of Cardiology/American Heart
Association Task Force on Clinical Practice Guidelines.
Hypertension. 2017;
Taler SJ, Initial Treatment of Hypertension, Feb 15 NEJM
NEJM 2018; 378; 636-44

210
Copyright © Harvard Medical School, 2018. All Rights Reserved.

72.2m
32%

103.3m
46%

Guideline not based on RCTs


Updates JNC7 not JNC 8
Creates “disease” in ≈30 million Americans
• 32% to 46% of Americans now have HTN
• Those with “pre-hypertension, now hypertension
Integrates cardiovascular risk assessment into BP management
BP goal same in age>65
SOURCE: Bakris G and Sorrentino M, NEJM Feb 8 2018
Muntner P, Carey RM, Gidding S, et al. Potential US population impact of the 2017 ACC/AHA
high blood pressure guideline. Circulation 2018;137:109-118.

211
Copyright © Harvard Medical School, 2018. All Rights Reserved.

Therapy for Hypertension based on JNC 8


Implement lifestyle interventions

No diabetes and/or CKD Diabetes and/or CKD

Age >60 Age <60 All ages

BP Goal BP Goal BP Goal


<150/90 <140/90 <140/90

Only diabetes Only CKD

Blacks <75 y >75 y


Non-Blacks

Thiazide, ACEi, ARB or CCB Thiazide or CCB ACEi or ARB CCB or Thiazide
alone or in combination alone or in combination

Source: Singh et al, Brigham Intensive Review of Internal Medicine, Oxford University Press 2014
Adapted from James PA et al: Report from the panel members appointed to the Eighth Joint National Committee (JNC 8). JAMA
2014; DOI:10.1001/jama.2013.284427.

Question 3
• A 22 year-old woman is admitted with a
diagnosis of Goodpasture’s syndrome. This
diagnosis is confirmed by an ELISA and
Western blot analysis demonstrating anti-
GBM antibodies. A preliminary reading of the
renal biopsy confirms the diagnosis of anti-
GBM nephritis. Her serum creatinine is 3.2
mg/dL. 2 weeks previously her serum
creatinine was 0.7 mg/dL. Which of the
following therapeutic options would be most
appropriate for her:

212
Copyright © Harvard Medical School, 2018. All Rights Reserved.

– A.) Prednisone 60 mg/day for 1 month followed by


a gradual weaning of her prednisone dose.
– B.) Pulse methylprednisone accompanied by daily
plasmapheresis with exchange, followed by high
dose oral prednisone coupled with
cyclophosphamide, until her anti-GBM titer is
undetectable.
– C.) Pulse methylprednisone followed by high dose
oral prednisone coupled with cyclophosphamide,
until her anti-GBM titer is undetectable.
– D.) Prednisone 60 mg/day plus cyclophoshamide
3 mg/Kg/day
– E.) Plasmapheresis alone

– A.) Prednisone 60 mg/day for 1 month followed by


a gradual weaning of her prednisone dose.
B.) Pulse methylprednisone accompanied by daily
plasmapheresis with exchange, followed by high
dose oral prednisone coupled with
cyclophosphamide, until her anti-GBM titer is
undetectable.
– C.) Pulse methylprednisone followed by high dose
oral prednisone coupled with cyclophosphamide,
until her anti-GBM titer is undetectable.
– D.) Prednisone 60 mg/day plus cyclophoshamide
3 mg/Kg/day
– E.) Plasmapheresis alone

213
Copyright © Harvard Medical School, 2018. All Rights Reserved.

• Ernest Goodpasture, joined the Brigham and Women’s Hospital/HMS in


1912 as a pathologist.
• In 1919 Goodpasture, while studying the influenza pandemic, described a
progressive and usually fatal disease in which glomerulonephritis was
associated with coughing of blood. Most of the patients died of kidney
failure. This has been known since as Goodpasture's syndrome.
• In 1924 he was invited to return to Vanderbilt as professor and chairman
of the Department of Pathology in the reorganized Vanderbilt University
School of Medicine.

Goodpasture’s Syndrome

• Uncommon: incidence 0.1 cases/million


• 1-2% of renal biopsy specimens
• Slight preponderance in males
• Age 1st to 9th decade
• More common in whites over African-
American’s
• Year-round presentation, ? Spring and
summer
• 50-75% have upper respiratory prodrome,
– plus fever, rash, myalgias, malaise,
headache, weight loss

214
Copyright © Harvard Medical School, 2018. All Rights Reserved.

Pathologic Features of
Anti-GBM Nephritis

Goodpasture’s Syndrome
• Renal presentation with RPGN
• proteinuria but usually not nephrotic
• Hypertension uncommon
• US - normal size kidneys
• Renal function declines rapidly

215
Copyright © Harvard Medical School, 2018. All Rights Reserved.

Goodpasture’s Syndrome
Treatment
• Treatment without methylprednisone and
plasmaphereis
– 89% progress to death or dialysis; only 10% improved
• Treatment with pulse steroids, plasmapheresis, and
Cyclophosphamide
– standard-of-care
– 50% improve
• Patients with serum creat of >7.0 respond to
treatment
– 75% with Scr <7 respond, 8% with Scr > 7 respond
– No improvement in 58 patients on dialysis

Goodpasture’s Syndrome
Treatment
• Protocol
– Pulse methyl prednisone (solumedrol) 1 g
QD x 3 d, 1-1.5 mg/Kg prednisone
– Cyclophosphamide 3 mg/Kg/d (reduced
dose in older patients, or if GFR < 10) > 2
months
– Plasma exchange daily 4 L with albumin
replacement (or FFP if pulmonary
hemorrhage present) x 14 d or until
antibody dissapears

216
Copyright © Harvard Medical School, 2018. All Rights Reserved.

Question 4

• In all of the following conditions an increase


in serum creatinine can be explained by a
reduction in GFR EXCEPT
– A.) Bilateral hydronephrosis
– B.) Severe extracellular volume contraction
caused by diarrhea
– C.) Severe congestive heart failure
– D.) Increase in serum creatinine 1 day post
arteriogram
– E.) Cimetidine treatment of peptic ulcer disease

• In all of the following conditions an


increase in serum creatinine can be
explained by a reduction in GFR
EXCEPT
– A.) Bilateral hydronephrosis
– B.) Severe extracellular volume contraction
caused by diarrhea
– C.) Severe congestive heart failure
– D.) Increase in serum creatinine 1 day post
arteriogram
E.) Cimetidine treatment of peptic ulcer
disease

217
Copyright © Harvard Medical School, 2018. All Rights Reserved.

Limitations of serum
creatinine

• Creatinine - freely filtered, secreted,


extra-renal degradation
• secretion and extra-renal elimination
increases as GFR falls
• Scr dependent on muscle mass and
meat intake
• Sensitivity of creatinine as GFR
measure approx 60%

Creatinine metabolism
• creatinine production proportional to muscle
mass
– males 20-25 mg/Kg/24h
– females 15 to 20 mg/Kg/24h
• Expected creat excretion (male) = (28-0.2[age
years](weight[kg])
• Expected creat excretion (female) = (24-
0.17[age years](weight[kg])
• Secreted by organic cation exchanger
• Drugs that interfere with proximal secretion
– Cimetidine
– Trimethoprim
– Dapsone
– Probenicid

218
Copyright © Harvard Medical School, 2018. All Rights Reserved.

SOURCE:
http://www2.kidney.org/professionals/kdoqi/guidelines
_ckd/p5_lab_g4.htm

Question 5
• You are asked to consult on a 62 year old African-
American male with acute on chronic renal
insufficiency secondary to diabetes mellitus ascribed
to contrast nephrotoxicity. Routine chemistry labs
show a potassium of 8.2 mg/dL. All of the following
would be changes seen on the EKG compatible with
hyperkalemia, except:
– A.) Peaked T waves
– B.) Prolonged QRS
– C.) Flattened p wave
– D.) Sine-wave appearing QRS complex
– E.) U wave

219
Copyright © Harvard Medical School, 2018. All Rights Reserved.

Question 5
• You are asked to consult on a 62 year old African-
American male with acute on chronic renal
insufficiency secondary to diabetes mellitus ascribed
to contrast nephrotoxicity. Routine chemistry labs
show a potassium of 8.2 mg/dL. All of the following
would be changes seen on the EKG compatible with
hyperkalemia, except:
– A.) Peaked T waves
– B.) Prolonged QRS
– C.) Flattened p wave
– D.) Sine-wave appearing QRS complex
E.) U wave

EKG changes in hyperkalemia

• Early: tenting of T waves “pinch-bottomed T


waves”; precordial leads
• Prolonged P-R interval
• ST segment depression and lengthening of
QRS
• P wave disappears, further widening of QRS
• Ventricular fibrillation

220
Copyright © Harvard Medical School, 2018. All Rights Reserved.

EKG changes in Hyperkalemia

Question 6

• 48 year old male ESRD patient presents to


the ED with a K= 7.8 mEq/L, HC03 of 22. His
EKG shows peaked T waves. Recommended
initial treatment include all of the following
EXCEPT:
– A.) Calcium gluconate 10 mls, IV
– B.) Insulin 10 units and 1 amp of 50% dextrose
– C.) Albuterol nebulizer (10-20 mg)
– D.) IV bicarbonate 8.4%, 1 to 2 amps IV
– E.) Emergent dialysis

221
Copyright © Harvard Medical School, 2018. All Rights Reserved.

• 48 year old male ESRD patient presents to


the ED with a K= 7.8 mEq/L, HC03 of 22. His
EKG shows peaked T waves. Recommended
initial treatment include all of the following
EXCEPT:
– A.) Calcium gluconate 10 mls, IV
– B.) Insulin 10 units and 1 amp of 50% dextrose
– C.) Albuterol nebulizer (10-20 mg)
D.) IV bicarbonate 8.4%, 1 to 2 amps IV
– E.) Emergent dialysis

Changes in plasma K
8.4% bicarb, epinephrine, insulin/dextrose, or HD

Blumberg et al
Am J. Med 88:507-512, 1988

222
Copyright © Harvard Medical School, 2018. All Rights Reserved.

Changes in plasma K during IV


infusion of bicarbonate in HD patients

Values= means + SE
*P<0.5, + P<0..01 vs. baseline

Blumberg et al KI, 41: 369-374, 1992

Question 7

• The most common type of kidney stone


observed in the United States is:
– A.) Cystine stone
– B.) Triple phosphate stone
– C.) Struvite stone
– D.) Calcium oxalate stone
– E.) Uric acid stone

223
Copyright © Harvard Medical School, 2018. All Rights Reserved.

• The most common type of kidney stone


observed in the United States is:
– A.) Cystine stone
– B.) Triple phosphate stone
– C.) Struvite stone
D.) Calcium oxalate stone
– E.) Uric acid stone

Nephrolithiasis
• 12% of US population affected
• Incidence rate (age 30-65)
– Male: 3/1000/yr
– Female: 1/1000/yr
• Calcium oxalate > 75%
– Hypercalciuria,hyperoxaluria, hypocitrituria,
hyperuricosuria
• Infection stone/Magnesium ammonium
phosphate/struvite/triple phosphate 7-15%
• Uric acid 2%
• Calcium phosphate 2%
• Cystine <1%

224
Copyright © Harvard Medical School, 2018. All Rights Reserved.

Common Crystals

Source: www:medstat.med.utah.edu/WebPath

Question 8

• A 60-year old man who has been previously in good


health and on no medications develops the nephrotic
syndrome. No systemic causes are identified and
serologic work up is completely negative. The most
likely histologic lesion of percutaneous renal biopsy
is:
– A.) Light chain nephropathy
– B.) Membranous glomerulopathy
– C.) Myeloma kidney
– D.) Membranoproliferative glomerulonephritis
– E.) IgA nephropathy

225
Copyright © Harvard Medical School, 2018. All Rights Reserved.

Question 8

• A 60-year old man who has been previously in good


health and on no medications develops the nephrotic
syndrome. No systemic causes are identified and
serologic work up is completely negative. The most
likely histologic lesion of percutaneous renal biopsy
is:
– A.) Light chain nephropathy
B.) Membranous glomerulopathy
– C.) Myeloma kidney
– D.) Membranoproliferative glomerulonephritis
– E.) IgA nephropathy

MN points
• Idiopathic
• Insidious onset
• M>F 2-3:1
• 80% NS at presentation
• 20% asymptomatic non-nephrotic
proteinuria
• 50% have hematuria
• HTN not common (30%)
• Increased risk of thromboembolism (15-
40%)
• Insidious decline in GFR

226
Copyright © Harvard Medical School, 2018. All Rights Reserved.

Membranous nephropathy

Primary Secondary
(Idiopathic) - Lupus
- Hepatitis B
75% 25% - NSAIDs
- Malignancy
- Toxins (Hg)
anti-PLA2R - Others
??
associated

M-type phospholipase A2 receptor


• 185 kDa type I transmembrane glycoprotein

• Expressed in human kidney, lung, placenta, WBC

• Member of the mannose receptor family


– Mannose Receptor (CD206)
– Endo180 (uPAR-associated protein or CD280)
– DEC-205 (CD205), dendritic cell receptor
– M-type phospholipase A2 receptor
– FcRY = avian yolk sac IgY receptor

• Binds certain sPLA2s, but exact function is not known

• May play a role in cellular replicative senescence

Source: Beck, et al

227
Copyright © Harvard Medical School, 2018. All Rights Reserved.

Association of anti-PLA2R with clinical status


Anti-PLA2R level
correlates with
proteinuria

Hofstra JM, Beck LH et al. (2011) Clin J Am Soc Nephrol 6: 1286-91

Question 9
• A 16-year old woman who presents with a history of a
sore throat 2 weeks previously has edema, mild
hypertension, hematuria and red cell casts on her
urine sediment. Her complements are low. The most
likely diagnosis is:
– A.)Minimal change disease
– B.)Post-infectious glomerulonephritis
– C.)IgA nephropathy
– D.)Acute interstitial nephritis
– E.) Light chain deposition disease

228
Copyright © Harvard Medical School, 2018. All Rights Reserved.

• A 16-year old woman who presents with a


history of a sore throat 2 weeks previously
has edema, mild hypertension, hematuria and
red cell casts on her urine sediment. Her
complements are low. The most likely
diagnosis is:
– A.) Minimal change disease
B.) Post-infectious glomerulonephritis
– C.) IgA nephropathy
– D.) Acute interstitial nephritis
– E.) Light chain deposition disease

Infectious agents and GN


• Bacterial • Parasitic
– Gp A-beta strep – Plasmodium malariae
– Staph aureus – Toxoplasma
– Staph epi – Filaria
– Gram neg bacilli – Schistosomia
– Strep pneumoniae – Trichenella
– Treponema pallidum – Trypanosome
– Salmonella typhi
• Rickettsial
– Meningococcus
– Scrub typhus
– Leptospirosis
• Fungal
• Viral
• Coccidioides
– Hep B and C
immites
– CMV
– Enterovirus
– Measles
– Parvovirus
– Oncovirus
– Mumps virus
– Rubella
– Varicella

229
Copyright © Harvard Medical School, 2018. All Rights Reserved.

PSGN points
• Follows pharyngeal and skin infection
• Nephritogenic strains M types Serotype 49
commonest
• Primarily disease of children, age 5 –15, rare <2 y
and > 40 y.
• Latent period 7-14 d pharyngeal, 14-28 d pyoderma
• Presentation: hematuria (70%), periorbital edema,
weight gain, HTN, oliguria
• UA: proteinuria (usually < 2g/24h), RBCs, RBC casts,
WCCs, WC casts
• FENA usually < 0.5% in acute phase
• Serology: Antistreptolysin ab (ASO),
Antistreptokinase, antideoxyribosenuclease B, and
antinicotyladenine dinucleaotidase
– Titers rise 10-14 d post strep pharyngitis, peak at 3-4 wks
– No relationship with development of nephritis or its severity

Question 10
– A 52 year old African-American female presents to
the emergency room with unstable angina. She is
noted to have a past medical history of mild chronic
renal insufficiency ( creatinine of 1.8 mg/dL). She is
transferred to the coronary care unit and therapy for
her unstable angina is initiated. A cardiac
catheterization is planned for the next day. Risk
factors that would predispose this woman to contrast
nephrotoxicty include all of the following except:
– A.) Diabetes mellitus
– B.) Pre-existing renal insufficiency
– C.) The volume of IV contrast utilized in the
procedure
– D.) Presence of extracellular volume contraction
– E.) A history of coronary artery disease

230
Copyright © Harvard Medical School, 2018. All Rights Reserved.

– A 52 year old African-American female presents to


the emergency room with unstable angina. She is
noted to have a past medical history of mild chronic
renal insufficiency ( creatinine of 1.8 mg/dL). She is
transferred to the coronary care unit and therapy for
her unstable angina is initiated. A cardiac
catheterization is planned for the next day. Risk
factors that would predispose this woman to contrast
nephrotoxicty include all of the following except:
– A.) Diabetes mellitus
– B.) Pre-existing renal insufficiency
– C.) The volume of IV contrast utilized in the
procedure
– D.) Presence of extracellular volume contraction
E.) A history of coronary artery disease

Question 11
• Her cardiologist asks you for an estimate of
her risk of developing contrast nephrotoxicity.
Which one of the following would be the
closest estimate:
– A.) 60%
– B.) <5%
– C.) 20%
– D.) >80%
– E.) >95%

231
Copyright © Harvard Medical School, 2018. All Rights Reserved.

Question 11

• Her cardiologist asks you for an estimate of


her risk of developing contrast nephrotoxicity.
Which one of the following would be the
closest estimate:
– A.) 60%
– B.) <5%
C.) 20%
– D.) >80%
– E.) >95%

Contrast Nephrotoxicity

• ARF and oliguria within 24-48 hours


• Peak serum creatininine on days 3-5
• Low fractional excretion of sodium
• Benign sediment or granular casts
• Resolution usual within 1 week
• Risk factors: CRI, diabetic nephropathy,
dose>120 cc, multiple myeloma, volume
• Prevention: hydration 75 cc 0.45% saline,
non-ionic for high risk patients

232
Copyright © Harvard Medical School, 2018. All Rights Reserved.

Contrast Nephropathy

• 12% of hospital-acquired ARF due to contrast


(Hou et al)
• Mortality of CN with Scr > 2.0, 48 h post
angiography > 34%
• 0.7% of diagnostic cath patients develop CN
– 10.9% with Scr > 1.2 mg/dL
• Overall Risk
– CRI patients 10-30%, Diabetes mellitus 24-50%

233
Copyright © Harvard Medical School, 2018. All Rights Reserved.

Contrast Nephropathy
Risk Factors

• Pre-existing renal insufficiency


• Diabetes mellitus
• CHF
• Volume Depletion
• Dose of Contrast agent

Prophylaxis for CIN

• Volume repletion – yes


• N-acetyl cysteine – no
NO
• Sodium bicarbonate - no

234
Copyright © Harvard Medical School, 2018. All Rights Reserved.

SOURCE: http://www.nephjc.com/news/2017/11/26/visual-abstracts-for-preserve

Question 12
• A 42 year otherwise healthy white male runs the
Boston Marathon in 4 hours and 4 minutes. He
complains of severe body cramps immediately after
the race and then after he returns to his hotel room.
At night, on voiding, he notices that the volume is
small and it is dark red in color. His spouse insists he
goes to the emergency room, where a blood test
shows that his serum creatinine is 4 mg/dL and his
BUN is 18. His urine shows a positive dipstick for
blood but virtually no red blood cells in the sediment.
The next step in management should be:
– A.) 800 mg indomethacin 3 times a day and return if not improved.
– B.) Starting the patient on pulse methylprednisone 500 mg/day (for
3 consecutive days).
– C.) Immediate initiation of hemodialysis
– D.) Aggressive hydration with normal saline and mannitol
– E.) Intravenous furosemide to initiate a diuresis

235
Copyright © Harvard Medical School, 2018. All Rights Reserved.

• A 42 year otherwise healthy white male runs the


Boston Marathon in 4 hours and 4 minutes. He
complains of severe body cramps immediately after
the race and then after he returns to his hotel room.
At night, on voiding, he notices that the volume is
small and it is dark red in color. His spouse insists he
goes to the emergency room, where a blood test
shows that his serum creatinine is 4 mg/dL and his
BUN is 18. His urine shows a positive dipstick for
blood but virtually no red blood cells in the sediment.
The next step in management should be:
– A.) 800 mg indomethacin 3 times a day and return if not improved.
– B.) Starting the patient on pulse methylprednisone 500 mg/day (for
3 consecutive days).
– C.) Immediate initiation of hemodialysis
D.) Aggressive hydration with normal saline and mannitol
– E.) Intravenous furosemide to initiate a diuresis

236
Copyright © Harvard Medical School, 2018. All Rights Reserved.

Epidemiology of Rhabdo
• Eighty-five percent of victims of traumatic injuries
develop rhabdomyolysis.[2]
• Of those patients with rhabdomyolysis 10-50% of
those patients will develop acute renal failure.[2]
• It is also suggested that victims of severe injury that
develop rhabdomyolysis and later acute renal failure
have a mortality of 20%.[2] An estimated 26,000
cases of rhabdomyolysis are reported annually in the
US.[2]
1Huerta-Alardin AL, Varon J, Marik P. Bench-to-beside review: Rhabdomyolysis
- an overview for clinicians. Critical Care 2005; 9: 158-169
2Harriston S. A review of rhabdomyolysis. Dimensions Of Critical Care Nursing:

DCCN [serial online]. July 2004;23(4):155-161. Available from: MEDLINE,


Ipswich, MA. Accessed March 23, 2014.

Features of Rhabdomyolysis

• Muscle pain and dark urine “Coca-Cola” color


• Orthotoludine-positive urine without RBCs
• Elevated CPK and myoglobin
• Increased K, Phos, urate, decreased Ca
• Rapid increase in serum creatinine
• Mechanism: free radicals, ferrihemate, reduced nitric
oxide
• Treatment: saline repletion, alkaline diuresis,
mannitol. Dialysis once ARF established

237
Copyright © Harvard Medical School, 2018. All Rights Reserved.

Causes of Rhabdomyolysis

• Excessive muscle activity


– seizures, delerium tremens, sport
• Direct of ischemic muscle injury
– trauma, compression syndrome, vascular occlusion
• Metabolic disorders
– hypokalemia, hyponatremia, hypophosphatemia
• Drugs or toxins
– ethanol, isopropyl alcohol, heroin, methadone
• Infections
– tetanus, legionaires, influenza

• Under normal conditions, CK levels are 45-260 U/L.


• After rhabdomyolysis, the levels of CK can be raised to 10,000-
200,000 U/L.
• No other condition except rhabdomyolysis can cause such extreme
CK elevation

SOURCE: Efstratiadis G, Voulgaridou A, Nikiforou D, et al. Rhabdomyolysis


updated. Hippokratia 2007; 11(3): 129-13

238
Copyright © Harvard Medical School, 2018. All Rights Reserved.

Disclosures

• Research Grants
GSK

239
Copyright © Harvard Medical School, 2018. All Rights Reserved.

ANEMIA

Maureen M. Achebe, MD, MPH


Division of Hematology
Brigham and Women’s Hospital
Assistant Professor, Harvard Medical School

Commercial/Faculty Disclosures

• AMAG Pharmaceuticals – Scientific advisory


board
• Luitpold Pharmaceuticals - Consulting

240
Copyright © Harvard Medical School, 2018. All Rights Reserved.

Anemia
• Hemoglobin or hematocrit below the normal
range for age and gender

• Anemia can be due to:


- ↓ red cell production (low retic count)
- ↑ red cell destruction (high retic count)
- Red cell (blood) loss (high retic count)

Evaluation of Anemia

1. Size of RBCs – MCV


2. Reticulocyte count
3. Peripheral smear

Other Labs in Evaluation

}
• LDH
+ Retics Hemolysis
• Indirect Bilirubin
• /absent Haptoglobin + Retics Ineffective
Erythropoesis

241
Copyright © Harvard Medical School, 2018. All Rights Reserved.

Evaluation of Anemia
RDW

Normal RDW AI, thal trait, aplastic anemia, renal


disease, acute blood loss

Elevated Iron deficiency, folate & B12 def,


RDW myelodysplasia, sickle cell disease

Evaluation of Anemia
Reticulocyte count
Corrected Reticulocyte Count/Reticulocyte Index

Reticulocyte Index = Retic count X Hematocrit


Normal Hct (45)

Reticulocyte Index in a normal healthy adult is between


1 and 2

242
Copyright © Harvard Medical School, 2018. All Rights Reserved.

Evaluation of Anemia
Reticulocyte Index and MCV
Iron deficiency
Anemia of inflammation Low MCV
Sideroblastic anemia
Thalassemias

Renal failure
Aplastic anemia Normal MCV
Low retic index
Hypothyroidism
B12/folate deficiency
MDS
High MCV
Alcohol liver disease
Hemolytic anemias
High retic index Blood loss

Evaluation of Anemia
Reticulocyte Index and MCV
Iron deficiency
Anemia of inflammation Low MCV
Sideroblastic anemia
Thalassemias

Renal failure
Normal MCV
Aplastic anemia
Low retic index
Hypothyroidism
B12/folate deficiency
High MCV
MDS
Alcohol liver disease

Hemolytic anemias
High retic index Blood loss

243
Copyright © Harvard Medical School, 2018. All Rights Reserved.

Case 1
58 y o woman w/ SLE, HTN and GERD presents with
intermittent headaches and fatigue.
WBC 4.7, Hgb 8, Hct 25, MCV 74, platelets 544,000,
retic count 1.5%, iron : 45, TIBC : 520, ferritin 20.
Soluble transferrin receptor is 4.2 (0.8 – 3 mg/L).

microcytic hypochromic RBCs

lymphocyte

Case 1
58 y o woman w/ SLE, HTN and GERD presents with
intermittent headaches and fatigue.
WBC 4.7, Hgb 8, Hct 25, MCV 74, platelets 544,000,
retic count 1.5%, iron : 45, TIBC : 520, ferritin 20.
Soluble transferrin receptor is 4.2 (0.8 – 3 mg/L).
Which of the following would be of most benefit?
a) Epo stimulating agent therapy
b) Better control of SLE
c) 1 unit packed RBCs
d) Intravenous iron
e) Proton pump inhibitor therapy

244
Copyright © Harvard Medical School, 2018. All Rights Reserved.

Case 2
60 y o woman w/ SLE, HTN and GERD presents 2 years
later with fatigue and dyspnea on exertion.
WBC 4.7, Hgb 7.8, Hct 23, MCV 76, platelets 267,000,
retic count 1.5%, iron : 19, TIBC : 220, ferritin 209.
Soluble transferin receptor is 0.9 (0.8 – 3 mg/L).

Case 2
60 y o woman w/ SLE, HTN and GERD presents 2 years
later with fatigue and dyspnea on exertion.
WBC 4.7, Hgb 7.8, Hct 23, MCV 76, platelets 267,000,
retic count 1.5%, iron : 19, TIBC : 220, ferritin 209.
Soluble transferin receptor is 0.9 (0.8 – 3 mg/L).
What is the most likely cause of her anemia:
a) Iron deficiency
b) Anemia of inflammation
c) Iron deficiency and Anemia of inflammation
d) None of the above

245
Copyright © Harvard Medical School, 2018. All Rights Reserved.

Iron Deficiency Anemia

http://www.cdc.gov/

Iron Homeostasis

246
Copyright © Harvard Medical School, 2018. All Rights Reserved.

Iron Homeostasis

Iron Homeostasis

247
Copyright © Harvard Medical School, 2018. All Rights Reserved.

Iron Homeostasis

Anemia of Inflammation
Iron Sequestration Syndromes
• Characterized by inappropriately high serum hepcidin

Inflammation

IL-6
Hepcidin

macrophage

248
Copyright © Harvard Medical School, 2018. All Rights Reserved.

Anemia of Inflammation
Iron Sequestration Syndromes
• Iron deficiency may exacerbate chronic diseases
leading to accelerated clinical deterioration
– Inflammatory bowel disease
– CHF
– CKD
– Rheumatologic diseases
– Infections
– Critical illness
– Malignancies

Diagnosis Iron Deficiency Anemia of


Anemia Inflammation
Iron Anemia of
deficiency Inflammation

Serum Fe Low Low

TIBC High Low

Transferrin Low Low


saturation
Ferritin Very low N/High

249
Copyright © Harvard Medical School, 2018. All Rights Reserved.

Serum Transferrin Receptor


Punnonen et al. Blood 89 (3) 1997:1052
No stainable iron n=48
129 adult patients
Hb <12.8g.dL or BM Asp/biopsy Stainable Fe + chronic infl n=64
<11.7g/dL
No stainable Fe + inflam process n=17

Management of Iron Deficiency Anemia


• Estimated to affect 2 billion people worldwide

• Oral repletion - Ferrous sulfate/gluconate PO


• Intravenous repletion

• Expected Response to Fe Treatment


Reticulocytosis – 5-7 days
Increase in Hb – 2 weeks
Ferritin repletion – 6 months

• If not, seek other cause.

250
Copyright © Harvard Medical School, 2018. All Rights Reserved.

• Retrospective comparison of safety of IV irons used at


our institution.
• 619 patients over 2 yrs : 32 adverse events (AEs), ranging
from urticaria to chest pain.
• No serious AEs or anaphylactic-type rxns.
• AE rates: LMW Dextran ≈ ferric gluconate equivalent
• Iron sucrose had higher OR of AEs (OR= 5.7; 95% CI 1.6–21.3).

• AE rates with IV iron are acceptable


Am J Hematol. 2012 Nov;87(11):E123-4.

Management of Anemia of Inflammation


• Identify specific cause.

• Epo stimulating agents (ESA) are beneficial in pts with


normal or elevated serum creatinine.1,2,3
• IV iron increases ESAs efficacy in select patient
populations.4

• Goal Hgb 10 -11 g/dL.5,6

• New therapeutic strategies – target hepcidin-


ferroportin axis.
1. Littlewood et al. JCO 2001;19:2865-74. 4. Coyne et al. J Am Soc Nephrol. 2007;18:975-84
2. Adamson, Hematol ASH Educ Prog 2008(1):159-165 5. Singh et al. N Engl J Med 2006; 255:2085-2098.
3. Hertel et al. Am J Nephrol 2006;26:149-156 6. Pfeffer et al. N Engl J Med 2009;361:2019-2032

251
Copyright © Harvard Medical School, 2018. All Rights Reserved.

Case 3
40 y o instructor at a firing range with no medical history
presents with fatigue, irritability, dyspepsia & arthralgias.
On exam: BP 145/90, pale, hearing loss and peripheral
neuropathy.
Labs: WBC 6.2, Hgb 9.3, ↑RBC protoporphyrin level.
Smear: RBCs show basophilic stippling.
Which would expect to see?
a) BM aspirate with ringed sideroblasts
b) Peripheral smear with teardrops
c) Megaloblastic RBCs & elevated methylmalonic acid
d) Elevated hepcidin level

Sideroblastic Anemias
• Heterogenous group of anemias
• Characterized by Ineffective erythropoiesis
Common Causes Clinical Associations
Myelodysplastic Syndrome Elderly person
Lead intoxication Construction workers,
Auto repairers, Scrap metal
recyclers, painters
INH without vit B6 Tuberculosis
Alcohol intoxication Alcohol abuse history

252
Copyright © Harvard Medical School, 2018. All Rights Reserved.

Sideroblastic Anemia
RBCs containing Pappenheimer bodies Ringed sideroblasts in Bone Marrow

Courtesy Arthur Skarin, MD.

Case 4
• 70 y o woman with diabetes is brought to PCP by her
family for progressive dementia. Labs are reported as
normal. Donepezil (Aricept) started. Six months f/u, no
improvement. Gait is unsteady. WBC 2.9, hgb 8.3,
platelets 85,000, retic ct 0.9%.
She is diagnosed with vitamin B12 deficiency.
Peripheral blood smear Bone marrow aspirate

Nuclear-cytoplamic asynchrony

253
Copyright © Harvard Medical School, 2018. All Rights Reserved.

Case 4
• 70 y o woman with diabetes diagnosed with vitamin B12
deficiency

Which of the following results is she likely to have?


a) ↑ folate, ↓ B12, normal MMA,↓ homocysteine
b) normal folate, ↓ B12, ↑ MMA,↓ homocysteine
c) normal folate, ↓ B12, normal MMA, ↑ homocysteine
d) normal folate, low normal B12, ↑ MMA, ↑ homocysteine
e) ↑ folate, normal B12, ↓ MMA, ↑ homocysteine

Biochemistry of Megaloblastic Anemia -


Impaired DNA Synthesis
DNA

dGTP dCTP dATP dTTP

dTDP

DHF-PG dTMP

THF-PG Thymidylate synthase


5-10-methylene
THF-PG dUMP
THF
methionine
methionine synthase
homocysteine
B12 5-Methyl THF
Cell membrane
methylTHF plasma

Dietary folate Dietary B12 Small intestine

254
Copyright © Harvard Medical School, 2018. All Rights Reserved.

Physiology of Vitamin B12

Cbl III Cobalamin II


in tissues

adenosylCbl Succinyl CoA


methylCbl

B12 Methylmalonyl CoA mutase


B12 THF
methionine
methionine synthase
homocysteine Methylmalonic acid
5-Methyl THF

DNA Synthesis for the TCA cycle

Folate vs B12 Deficiency


Folic Acid def Vitamin B12 def
Diet history EtOH, poor Vegan diet
overall intake
Neurological deficits No Paresthesias, dementia,
madness
Homocysteine High High

Methylmalonic acid Normal High

Other buzzwords pregnancy, hx of bariatric surgery


growing child fish tapeworm

255
Copyright © Harvard Medical School, 2018. All Rights Reserved.

When to get homocysteine and


methylmalonic acid levels
Cobalamin Serum folate Check
(200 – 900 pg/ml) (normal >2ng/ml) homocysteine and
MMA
200-300 >4 Yes, to R/O B12
deficiency

<200 <2 Yes, to distinguish


folate deficiency from a
combined deficiency
>300 2-4 Yes, to confirm folate
deficiency

Case 5
• 60 y o woman with Rheumatoid Arthritis presents with a
2 wk history of worsening weakness and dizziness.
• Labs: WBC 4.7, Hgb 7.3, Hct 25%, MCV 92, MCHC 39
platelet count 177,000. T bil 3.1, retic 23%, LDH 636,
direct Coombs (+) for IgG and complement.

Smear shows spherocytes

256
Copyright © Harvard Medical School, 2018. All Rights Reserved.

Case 5
• 60 y o woman with Rheumatoid Arthritis presents with a
2 wk history of worsening weakness and dizziness.
• Labs: WBC 4.7, Hgb 7.3, Hct 25%, MCV 92, MCHC 39
platelet count 177,000. T bil 3.1, retic 23%, LDH 636,
direct Coombs (+) for IgG and complement.
What would you expect?
a) Eosin-5-maleimide to be abnormal
b) Treatment with steroids to be beneficial
c) Sputum culture positive for Mycoplasma pneumonia
d) Osmotic fragility to be normal
e) Splenomegaly

Hemolytic Anemias

Hereditary Acquired

1. Defects in RBC membrane 1. Immune HA


2. Defects in RBC metabolism 2. Non-immune HA
(enzymopathies)
3. Defects in Hemoglobin
(hemoglobinopathies)

257
Copyright © Harvard Medical School, 2018. All Rights Reserved.

Autoimmune Hemolytic Anemia


Warm AIHA Cold AIHA

IgG
IgM

MAC

Splenic macrophages Intravascular hemolysis

Autoimmune Hemolytic Anemia


Warm AIHA Cold AIHA

Courtesy ASH Image bank

258
Copyright © Harvard Medical School, 2018. All Rights Reserved.

Autoimmune Hemolytic Anemia


Warm AIHA Cold AIHA
Direct IgG or IgG & C3 C3 only
Coombs
Antibody IgG IgM
Etiology 1. Drugs: Methyldopa, 1. Drugs: Quinidine
PCN, 2. Malignancy: NHL
Sulfa 3. Infection:
2. Malignancy: CLL, Mycoplasma
NHL 4. Paroxysmal cold
3. Infection hemoglobinuria

Treatment of Warm AIHA


• First line therapy –
Corticosteroids.1,2 No specific dose. Prednisone 1 mg/kg/day

• Second line
Rituximab 375 mg/m2 weekly for 4 weeks – CR in 29 - 55% pts, PR
in 50% pts. 3,4,5
Splenectomy - some response in 59 - 100% patients.6

Supportive Care
• RBC transfusions – Cross-matching is difficult because of pan-
agglutinating Abs. Use closest match possible

1. Allgood et al. Am J Med 1967;43(2):254-273 4. Bussone. Am J Hematol 2009;84(3):153-157


2. Zupanska at al. Haematologia 1981;14(4):425-433. 5. D’Arena. Eur J Haematol 2007;79(1):53-58
3. Dierickx et al. J Intern Med 2009;266(5):484-491 6.Crowther et al. Blood 2011;118 (15) 4036-4040

259
Copyright © Harvard Medical School, 2018. All Rights Reserved.

Case 6
32 y o woman presents with 3 day history of colicky
abdominal pain and fatigue.
Labs: WBC 2.9, Hct 22% MCV 78, platelets 60,000, retic
count 9%, direct and indirect Coombs (-), ferritin 10.
Abdominal USS shows portal vein thrombosis.
U/A – hemosiderin (+).

The most likely diagnosis is:


A) Factor V Leidin mutation
B) Paroxysmal cold hemoglobinuria
C) Warm autoimmune hemolytic anemia
D) Aplastic anemia
E) Paroxysmal nocturnal hemoglobinuria

Paroxysmal Nocturnal Hemoglobinuria

PIG A
GPI anchors C′ activation

C′ activation
loss of anchors
PIG A MAC
mutation CD55

260
Copyright © Harvard Medical School, 2018. All Rights Reserved.

Paroxysmal Nocturnal Hemoglobinuria


Features:
• Chronic hemolysis urine hemosiderin Fe def.
• Thrombocytopenia
• Thrombosis in unusual sites – hepatic, mesenteric,
cerebral veins.
Diagnosis: flow cytometry for absence of CD55 & CD59

Treatment: Eculizumab
Steroids
Allogeneic BM transplant

Eculizumab

Brodsky Blood 2014;124:2804-2811


©2014 by American Society of Hematology

261
Copyright © Harvard Medical School, 2018. All Rights Reserved.

Case 7
34 yr old Hispanic woman who is 20 weeks pregnant
presents with generalized aches and fatigue. She’s had
an episode once in her life before. Her pulse is 102,
temp 101, BP 120/80. She’s jaundiced. Her labs show:
Hct 22, WBC 14, platelets 420,000. LDH 574, retics 15%

What diagnostic test is most appropriate?


34 yr old Hispanic woman who is 20 weeks pregnant
presents with generalized aches and fatigue. She’s had
an episode once in her life before. Her pulse is 102,
temp 101, BP 120/80. She’s jaundiced. Her labs show:
Hct 22, WBC 14, platelets 420,000. LDH 374, retics 15%
A) Full liver function tests
B) Hemoglobin electrophoresis
C) Bone marrow biopsy
D) Rapid strep test
E) Osmotic fragility

262
Copyright © Harvard Medical School, 2018. All Rights Reserved.

Case 7 continues
8 hrs later, patient is SOB, chest pain, tachycardic, diaphoretic,
Rm air O2 sat is 83%. CXR shows bilateral lower lobe
infiltrates. What is the most appropriate next step?
A) Continue current antibiotics and exchange RBC transfusion,
send sputum cultures
B) Continue current antibiotic, IV pain medication and deliver
baby as soon as possible
C) Continue current antibiotics, send for V/Q scan, initiate
anticoagulation
D) Expand antibiotic coverage for atypicals and monitor
patient closely
E) Expand antibiotic coverage for atypicals, exchange RBC
transfusion immediately.

Sickle Cell Syndromes (Qualitative Defect)


• Due to a point mutation in β-globin chain : β6 Glu Val

Bunn, NEJM 1997;337:762

263
Copyright © Harvard Medical School, 2018. All Rights Reserved.

Sickle Cell Anemia


• Homozygous inheritance of S β-globin
• Hb electrophoresis: HbS -75-95%, HbF 2-20%, HbA2<4%
• Functional asplenia, vaso-occlusive crises

Common Complications:
• Acute chest syndrome – major cause of mortality
• Aseptic necrosis of bone
• Priaprism
• Retinopathy

Acute Chest Syndrome (ACS)


• characterized by fever, chest pain, shortness of
breath, hypoxia and a new infiltrate(s) on CXR.

Vichinsky et al. NEJM 2000; 342:1855-1865

264
Copyright © Harvard Medical School, 2018. All Rights Reserved.

Acute Chest Syndrome (ACS)


• Treatment for acute chest syndrome includes:
antibiotic coverage for typical and atypical orgs.
RBC exchange transfusion to a goal HbS <30%
Aggressive supportive care

Rx of Bone pain crisis – Fluid repletion and


liberal pain medication as needed

Rx to prevent frequent crises – Hydroxyurea


Vichinsky et al. N Eng J M 2000; 342:1855-1865

Sickle Cell Trait


• Not considered a disease state
• Vast majority are asymptomatic
• Heterozygous inheritance of HbS:
Hb electrophoresis : <50% HbS, >50% HbA

Rarely present with

• renal papillary necrosis


• painless hematuria
Ryan T Clark
• isothenuria Pittsburgh Steeler
• splenic infarcts

265
Copyright © Harvard Medical School, 2018. All Rights Reserved.

Case 8
An 18-year old woman is referred for the evaluation of
mild jaundice. She thinks she has had it intermittently
for years. She has always tired more easily than her
friends, and she has been told several times that she
was anemic. She has been treated on several occasions
with iron pills, but not in the past two years.
On physical exam, scleral icterus and a spleen tip
palpable.
Her WBC – 6,500,Hgb -11.6 g/dl, hematocrit 31%, MCV
83, retics 7%, platelets 220,000.
Blood smear: Spherocytes, increased retics

Case 8
Coombs test was negative.

The patient is most likely to respond to which of


the following:
A. Corticosteroids
B. Intravenous iron
C. Splenectomy
D. Eculizumab

266
Copyright © Harvard Medical School, 2018. All Rights Reserved.

Hereditary Spherocytosis

Courtesy Sam Lux

Hereditary Spherocytosis
• loss of RBC membrane in spleen spherocytes
• Chronic hemolysis early gall stones
• >65% are autosomal dominant – (+) family hx
• MCHC ≥ 36 in 50% patients

Diagnosis : Eosin-5-maleimide binding test by


flow cytometry.
Treatment : Folic acid
Splenectomy - limits hemolysis

267
Copyright © Harvard Medical School, 2018. All Rights Reserved.

Summary on Anemias
Iron deficiency
Anemia of chronic dx
Sideroblastic anemia Low MCV
Thalassemias

Renal failure
Low retic Aplastic anemia Normal MCV
index
Hypothyroidism
B12/folate deficiency
MDS High MCV
Alcohol liver disease

High retic Hemolytic anemias


Blood loss
index

Summary on Anemias
Diagnosis and Treatment of Iron Deficiency & AI
• Differentiating Fe Deficiency – High TIBC, low ferritin
vs. Anemia of Inflammation – Low TIBC, high ferritin

Soluble transferrin receptor – normal in AI


Bone marrow iron – normal in AI

Differentiating folate vs B12 deficiency


↑ Methylmalonic acid & neurological deficits in B12 def.
Intrinsic factor Ab - ∼99% specificity for pernicious
anemia

268
Copyright © Harvard Medical School, 2018. All Rights Reserved.

Summary on Anemias
Hemolytic anemias - ↑LDH, ↑ retic count
Spherocytes – AIHA or hereditary spherocytosis
Diagnosis AIHA – (+) direct Coombs test
Diagnosis of HS – Eosin-5-maleimide testing

Sickle Cell Disease


Urgent exchange RBC transfusion for Acute chest
syndrome. Target HbS<30%

Diagnosis and Treatment of PNH


Gold standard for diagnosis : Flow cytometry for
absence of CD55 &CD59
Treatment : with monoclonal Ab, Eculizumab

Commercial/Faculty Disclosures

• AMAG Pharmaceuticals – Scientific advisory


board
• Luitpold Pharmaceuticals - Consulting

269
Copyright © Harvard Medical School, 2018. All Rights Reserved.

References
1. Bain, BJ. Diagnosis from the Blood smear. N Engl J Med. 2005;353:498-507.
2. Weiss, G, Goodnought, LT. Anemia of chronic disease. N Engl J Med.
2005;352:1011-23.
3. Go, S,. Winters, J. How I treat autoimmune hemolytic anemia. Blood 2017;
129:2971-2979.
4. Young, N. Acquired Aplastic Anemia. JAMA 1999 July 21;282(3):271-278.
5. The Management of Sickle Cell Disease. National Institutes of Health.
National Heart, Lung and Blood Institute, Division of Blood Diseases and
Resources. NIH publication No. 02-2117. Fourth edition.
6. Ballas, S. Beyond the Definition of the Phenotypic complications of Sickle
Cell Disease: an Update of Management. The Scientific World Journal 2012;
Article ID 949535.
7. Rund, D. β-Thalassemia. N Engl J Med. 2005;353:1135-46.

270
Copyright © Harvard Medical School, 2018. All Rights Reserved.

Thrombophilia Testing

Jean M Connors MD

Medical Director, Anticoagulation Management and Stewardship Services


Hematology Division
Brigham and Women’s Hospital/Dana Farber Cancer Institute
Associate Professor, Harvard Medical School

Conflicts of Interest

Proteostasis
Consultant
Pfizer/Bristol-Meyers Squibb
Independent Review Committee
Scientific Ad Boards
Consultant
Unum Therapeutics
DSMB
Portola
Advisory Board
Dova Pharmaceuticals
Consultant
CSL Behring
Research funding to the institution

271
Copyright © Harvard Medical School, 2018. All Rights Reserved.

Agenda

• Brief overview of risks for VTE

• Thrombophilia testing
– Tests, timing, patient selection
• Inherited thrombophilias
• Antiphospholipid antibody syndrome testing

– Will not cover thrombophilia and pregnancy

Virchow's triad

Not just inherited


Kyrle P A , Eichinger S Blood 2009;114:1138-1139 factors

©2009 by American Society of Hematology

272
Copyright © Harvard Medical School, 2018. All Rights Reserved.

Seasonal
Seasonal Variation
variation ininVTE
VTE

Boulay BMJ 2001

VTE
Risks for hypercoagulable states
– Inherited
– Acquired: more common
• 35% US adults are obese, OR of 2.3 for VTE
• <10% have an inherited thrombophilia
– Mixed: all are additive or synergistic

“Provoked” vs “Unprovoked”
– Clear precipitating factor vs idiopathic or
unidentified risk factor
• Transient vs persistent provoking factor
• Unprovoked = idiopathic

273
Copyright © Harvard Medical School, 2018. All Rights Reserved.

Annual incidence of venous thromboembolism by age and sex.

Heit J A Hematology 2007;2007:127-135

©2007 by American Society of Hematology

Acquired Risk Factors for VTE


Transient/Provoked Idiopathic/
Persistent
Unprovoked
Surgery Obesity
Trauma (major trauma or Increasing Age ?????
lower-extremity injury)
Chronic Medical
Acute medical illness Illnesses
Immobilization Cancer and its
therapy
Estrogen-containing
-thalidomide
contraceptives or derivatives
hormone replacement
-Tamoxifen
therapy (drospirenone)
Inflammatory bowel
Pregnancy/puerperium disease
HIT Nephrotic syndrome
Myeloproliferative
Prolonged air travel neoplasms/PNH
Sickle cell disease

274
Copyright © Harvard Medical School, 2018. All Rights Reserved.

Acquired Deficiency
ANTITHROMBIN PROTEIN C PROTEIN S
Pregnancy Pregnancy
Liver Disease Liver Disease Liver Disease
DIC DIC DIC
Nephrotic syndrome
Major surgery Inflammation
Acute thrombosis Acute thrombosis Acute thrombosis
Treatment with:
Heparin Warfarin Warfarin
Estrogens Estrogens

• Levels decrease in the setting of above factors.


• Results drawn at initial presentation of VTE or while on anticoagulants
including DOAC often not valid.
• Abnormal results must be confirmed after discontinuation of
anticoagulation, at least 2 or more weeks after stopping warfarin, longer
for other factors such as estrogen

Established Inherited Thrombophilias


• Mutations create coagulation imbalance:
• Increased procoagulant activity
– Factor V Leiden mutation
• APC “resistance”
– Prothrombin gene G20210A mutation
• Increased prothrombin levels
– FVL and PTG comprise 50-60% of cases
• Decreased anticoagulant activity
– Multiple mutations lead to decreased level or altered
function (quantitative vs qualitative)
– Protein C: inactivates FVIII and FV
– Protein S: cofactor for protein C
– Antithrombin: inactivates thrombin, Xa

275
Copyright © Harvard Medical School, 2018. All Rights Reserved.

The “Hypercoagulable Workup”

Test for Factor V Leiden mutation


• APCR standard screen
• Genetic PCR confirm

PCR for Prothrombin G20210A mutation

Functional assay of Antithrombin


Functional assay of Protein C
Functional assay of Protein S
• Free Protein S Antigen
• Total Protein S Antigen (free + bound to C4bp)

Inherited Thrombophilias
• Extremely rare
– Dysfibrinogenemia
– Cystathionine beta synthase deficiency(homocysteinuria)

• WHAT NOT to test:


– Homocysteine and MHTFR polymophisms
• Decreased levels with B vitamins had no impact on recurrent
VTE or MI
• NORVIT (NEJM 2006) HOPE2 (NEJM 2006, Ann Int Med 2007)
VITRO (Blood 2007)
– FVIII
– XIII polymorphisms, IX, XI,XII
– PAI-1 4G/5G promoter, PAI-1

276
Copyright © Harvard Medical School, 2018. All Rights Reserved.

Thrombophilia Tests and Prevalence of Risk Factors.

Connors
Connors JM. N Engl J Med ;377:1177-1187 NEJM 2017

APLA work-up
Tests for Antiphospholipid Antibodies
• Lupus anticoagulant:
• Screen: functional clotting assays
• Sensitive PTT
• DRVVT
• Kaolin clotting time
• Confirmatory: remove APLA
• Platelet neutralization test
• Hexagonal phase phospholipids
• Anti-Cardiolipin and β2-glycoprotein I
antibodies
• IgG and IgM only
• No diagnostic role for other tests

277
Copyright © Harvard Medical School, 2018. All Rights Reserved.

Diagnostic Criteria for the Antiphospholipid Syndrome

Connors JM. N Engl J Med ;377:1177-1187

Connors NEJM 2017

Thrombophilias
• Indiscriminant testing in the inpatient or ER
setting should be avoided
• Results affected by
• Drugs—anticoagulants: heparin, warfarin, DOACS
• Acute setting—clot, inflammation, miscarriage
• Lab quality
• There is no need to know immediately—require at least
3 months anticoagulation for VTE regardless of
thrombophilia status

• Ravi Sarode at UTSW stewardship program: 63% of


inpatient thrombophilia panels cancelled after electronic
consult

278
Copyright © Harvard Medical School, 2018. All Rights Reserved.

Connors JM. N Engl J Med 2017;377:1177-1187

Connors JM. N Engl J Med ;377:1177-1187

279
Copyright © Harvard Medical School, 2018. All Rights Reserved.

Thrombophilia Testing
• Why the controversy?
– Profound controversy over the utility of testing
• No data that results should affect care
– ASH Choosing Wisely Campaign 2013: “do not test
in the setting of provoked VTE due to strong risks”
– NICE, SIGN, ACCP—no recommendations
“It is not possible to give a validated recommendation as to how such
patients (and families) should be selected.” BJH 2010
• Misinterpretation of the significance of results
– Over treatment in the case of positive results
– False sense of security with negative results
• Studies demonstrate increased VTE risk for patients with a
family history of VTE despite negative results

VTE Recurrence Risk

Leiden Thrombophilia Study

N = 474
provoked and unprovoked 1st VTE

HR = 1.3 (95% CI, 0.8 – 2.0)

Christiansen JAMA 2005

280
Copyright © Harvard Medical School, 2018. All Rights Reserved.

VTE Recurrence Risk

ELATE Trial Sub-study


661 patients unprovoked
VTE tested for
thrombophilia.
Randomized to 2 INR
ranges after initial rx.
Events are while on
warfarin therapy.

Kearon Blood 2008

Inherited Thrombophilia Testing: Why Test?

Role of testing not well defined


– Lack of studies on utility of testing
– Lack of studies on efficacy/safety of prophylaxis

When does it change care?


– Explain etiology—patients have a lower threshold for first VTE
– Prophylaxis
– Pregnancy
– OCP/hormone therapy
– Family members

When does it not change care?


– Duration of anticoagulation therapy in most provoked VTE cases
– Antiphospholipid syndrome
– Malignancy

281
Copyright © Harvard Medical School, 2018. All Rights Reserved.

Algorithm for Selecting Patients with a First Venous


Thromboembolism (VTE) for Thrombophilia Testing.

Connors NEJM 2017

Summary
• The “hypercoagulable state” is a spectrum of
risk, with many patients having multiple additive
risk factors.
• Environment and acquired events add to
baseline genetic risk and are more common
than inherited thrombophilias.
• Inherited thrombophilias provide variable
baseline risk; testing is easy, whom to test and
what to do with results more complex. No
guidelines have been established.

282
Copyright © Harvard Medical School, 2018. All Rights Reserved.

Summary
• No data for efficacy or safety for VTE prophylaxis
based on thrombophilia status alone

• Toughest decision is duration of anticoagulation in


patients with unprovoked VTE. No risk prediction
scores incorporate inherited thrombophilia
status—Vienna, DASH, HERDOO2

• No available data support different treatment for


patients with vs without inherited thrombophilia
– Data demonstrate no difference in recurrence rates
– Clinical factors more important than test results

• Consult with a specialist can be helpful

DOAC:
Paradigm shift in Secondary VTE
Prevention Strategies

Jean M Connors MD
Medical Director, Anticoagulation Management and Stewardship Services
Hematology Division
Brigham and Women’s Hospital/Dana Farber Cancer Institute
Associate Professor, Harvard Medical School

283
Copyright © Harvard Medical School, 2018. All Rights Reserved.

Duration of Anticoagulation
• What factors impact the decision for how long
to treat with anticoagulation after a venous
thromboembolic event?
• Most significant factor is etiology:
– Provoked VTE with transient risk factor
– Unprovoked or idiopathic, with no obvious risk factors

• The extremes are easy (at first)


– Provoked: 3-6 months
– Unprovoked: indefinite
The presence of an inherited thrombophilia does not
mandate indefinite duration anticoagulation.

Duration of Anticoagulation

• Treatment duration is more difficult to


determine in patients with VTE that don’t
fall neatly into the provoked or unprovoked
category, or occur in patients with
continually present/non-modifiable VTE
risk factors.

• These patients may be viewed as having


VTE as a “chronic disease”

284
Copyright © Harvard Medical School, 2018. All Rights Reserved.

Duration of Anticoagulation
Provoked
– 3 months sufficient if risk Idiopathic
gone
– 1% risk per year of
recurrence, not changed by 3 Secondary
vs 6 months

Unprovoked/Idiopathic
– Recurrence rate highest in Prandoni Haematologica 2007
first 2 years
• 10% per year in 1st 2
years
• 40% at 5 years

VTE Recurrence Risk

A= post-op within 6 weeks


B= pregnancy VTE: 0 recurrence
C= unprovoked C
D= non-surgical risks

570 patients treated for


24-26 weeks, then followed A
for 2 years.

Baglin Lancet 2003

285
Copyright © Harvard Medical School, 2018. All Rights Reserved.

VTE Recurrence Risk

Recurrence of VTE according to site of first event


and sex

male, PE or proximal DVT


male, distal DVT
female, PE or proximal DVT
female, distal DVT

Delluc ASH 2016

Unprovoked VTE
Is longer duration better?

>90% risk reduction if continue anticoagulation


(full intensity warfarin)

Reduction in risk disappears when


anticoagulation is stopped, ie not modifying
underlying risk

At 5 years, 60% will have had unnecessary


treatment

286
Copyright © Harvard Medical School, 2018. All Rights Reserved.

Duration of Treatment
Need to consider
– Patient preference and lifestyle
– Recurrence risk
– Bleeding risk: case fatality rates similar for recurrent
VTE vs bleeding: with warfarin
• Recurrent VTE case fatality rate 0.3-1%
• Major bleeds 1-3%/yr fatal bleeds 0.4-0.8%

– Alternatives? ASA, risk modification

DOAC are changing the treatment paradigm for


patients with unprovoked VTE or need for
extended secondary VTE prophylaxis.

AMPLIFY-EXTENSION

• 2468 patients with VTE treated for 6-12 months in


randomized double blind study:
– Placebo, Apixaban 2.5 mg bid, Apixaban 5 mg bid
• Followed for 12 months for recurrent VTE and
bleeding
Placebo APIX 2.5 APIX 5
Recurrent p<0.001
VTE 8.8% 1.7% I.7%
Major 0.5% 0.2% 0.1%
Bleed
CRNM 2.3% 3.0% 4.2%
Bleed
Agnelli NEJM 2013

287
Copyright © Harvard Medical School, 2018. All Rights Reserved.

Kaplan–Meier Cumulative Event Rates AMPLIFY- EXT

AMPLIFY-EXT
90-93% unprovoked
7-10% provoked

Agnelli G et al. N Engl J Med 2013;368:699-708

Agnelli NEJM 2013

EINSTEIN-CHOICE

• 3365 patients with VTE treated for 6-12 months


randomized in double blind study to:
– Asa 81, Rivaroxaban 10 mg qd, Rivaroxaban 20 mg qd
• Followed for 12 months for recurrent VTE and
bleeding

ASA RIVA 10 RIVA 20


Recurrent p<0.001
VTE 4.4% 1.2% 1.5%
Major 0.3% 0.4% 0.5%
Bleed
CRNM 1.8% 2.0% 2.7%
Bleed
Weitz NEJM 2017

288
Copyright © Harvard Medical School, 2018. All Rights Reserved.

Kaplan–Meier Rates Cumulative Event Rates EINSTEIN-CHOICE

EINSTEIN Choice
60% provoked
40% unprovoked

Patients in equipoise for


need to continue
anticoagulation

Weitz JI et al. N Engl J Med 2017;376:1211-1222

Weitz NEJM 2017

Extended Duration Anticoagulation


Summary
• The benefit-risk profile is improved with reduced dose
or prophylactic dose apixaban and rivaroxaban,
bleeding rates are similar to aspirin or placebo.
• With follow-up of one year, reduced dose appears to
confer similar decreased risk of recurrent VTE as full
dose, with better efficacy than ASA or placebo.
• Both full and reduced dose have similar low risk of
major bleeding, although event rates in all groups
were low.
• CRNM bleeding is lower with reduced dose compared
to full dose.

289
Copyright © Harvard Medical School, 2018. All Rights Reserved.

My Approach:
Extended Duration Anticoagulation
• After full intensity anticoagulation for unprovoked VTE
consider change to reduced dose apixaban or
rivaroxaban
• For provoked VTE with persistent risks and for
patients in equipoise, switch to reduced dose
apixaban or rivaroxaban
– Unclear provoking factors
– Post thrombotic syndrome or significant persistent residual
vein thrombosis
– Obesity, heart failure, immobile
• Patients who should not be on reduced dose
– Antiphospholipid syndrome
– Mechanical heart valves (should not be on a DOAC)
– Atrial fibrillation Cancer

DOAC Related Bleeding


The basics:
– Stop anticoagulant
– Assess severity
– Baseline coag tests, CBC, creatinine
– When was the last dose of drug--short half life
T ½ 10-12 hours for all DOAC with normal renal function

– Standard supportive measures


• IV fluid, RBC, platelets, fibrinogen, antifibrinolytics
– Massive hemorrhage or trauma protocols
• Localization and management of specific site

Is immediate reversal of anticoagulation needed?

290
Copyright © Harvard Medical School, 2018. All Rights Reserved.

Idarucizumab
• PRAXBIND = idarucizumab
– Humanized monoclonal antibody fragment
that binds dabigatran
– 350 x higher affinity for dabigatran than
dabigatran has for thrombin

• FDA approved Oct 16, 2015


– RE-VERSE AD: “real world” study
– 5 gram dose IV bolus
– immediate and sustained reversal of
dabigatran activity over 24 hours

– Approved for bleeding and urgent surgery

Andexanet alfa
“AndexXa”
• “decoy” recombinant FXa
molecule with mutation in
catalytic site, lacks Gla
domain
• “universal” FXai antidote
• Bolus followed by 2 hour CI
• FDA approved May 3, 2018
– For reversal of apixaban and
rivaroxaban in patients with
major bleeding

291
Copyright © Harvard Medical School, 2018. All Rights Reserved.

Use of Antidotes
idarucizumab
How should targeted reversal agents be used?
Life-threatening bleeding
Emergency Surgery
Anticipated delayed clearance and bleeding

Not for use in:


Elective surgery or surgery that can be delayed
Elevated coagulation tests but no bleeding
Bleeding managed with routine supportive care

Jean M. Connors MD
jconnors@bwh.harvard.edu

292
Copyright © Harvard Medical School, 2018. All Rights Reserved.

Question 1
Question 1
A 67 year old man is seen by his PCP for advice about
duration of anticoagulation. Two months ago he had had
urgent cholecystectomy with the development of a left
calf vein DVT. The diligent surgery intern sent a
hypercoagulable work up which revealed heterozygous
Factor V Leiden. The patient was told he requires
lifelong anticoagulation. You advise:

A. Aspirin 81 mg once daily


B. Warfarin target INR 1.5-2.0
C. Apixaban 2.5 mg twice daily
D. Continue anticoagulation for one more month

Question 1
Question 1
ANSWER: D

This patient had a clear cut surgically provoked VTE.


The presence of factor V Leiden does not affect risk of
recurrent VTE and is not a reseaon to continue life-long
anticoagulation even with Factor V Leiden mutation.
Limited duration anticoagulation of 3 months is sufficient,
unless there is significant post-thrombotic syndrome or
other indications for anticoagulation.

293
Copyright © Harvard Medical School, 2018. All Rights Reserved.

Question 2
Question 1
A 42 year old man is seeing you for a second opinion. He
has been on warfarin for 11 months for an unprovoked VTE.
He reports that he was writing a novel at the time of the PE
and would sit for 6-8 hours writing. He weighs 125 kg with a
BMI of 39.5. renal function is normal. You decide to:

A. Continue warfarin with target INR 2.0-3.0 as he has been


doing well on this and is young with low bleeding risk
B. Stop anticoagulation completely as he has had adequate
treatment for acute VTE
C. Transition to rivaroxaban 10 mg once daily
D. Transition to edoxaban 30 mg once daily

Question 2
Question 1
Answer C
This 42 yo man is at high risk for recurrent VTE, especially
PE, given age, gender, and BMI, as well as duration of
treatment of only 11 months as risk for recurrence is highest
in the first 2 years. Although he reports sitting for extended
periods of time this is really not a recognized risk factor and
he should be classified as having an unprovoked VTE

Although he could continue with warfarin, reduced dose


rivaroxaban is an attractive alternative with less lifestyle
burden. Risk of bleeding should be lower although there has
been no head to head comparison with warfarin. Edoxaban
and dabigatran have not been studied at reduced dose in
this setting.

294
Copyright © Harvard Medical School, 2018. All Rights Reserved.

References
Thrombophilia testing and venous thrombosis.
Connors JM. N Engl J Med. 2017 Sep 21;377(12):1177-1187.

Apixaban for extened treatment of venous thromboembolism


Agnelli G, Buller HR, Cohen A, Curto M, Gallus AS, Johnson M, Porcari A,
Raskob GE, Weitz JI; AMPLIFY-EXT Investigators.
N Engl J Med. 2013 Feb 21;368(8):699-708.

Rivaroxaban or Aspirin for Extended Treatment of Venous


Thromboembolism.
Weitz JI, Lensing AWA, Prins MH, Bauersachs R, Beyer-Westendorf J,
Bounameaux H, Brighton TA, Cohen AT, Davidson BL, Decousus H, Freitas
MCS, Holberg G, Kakkar AK, Haskell L, van Bellen B, Pap AF, Berkowitz
SD, Verhamme P, Wells PS, Prandoni P; EINSTEIN CHOICE Investigators.
N Engl J Med. 2017 Mar 30;376(13):1211-1222.

References
Idarucizumab for Dabigatran Reversal--full cohort analysis.
Pollack CV Jr, Reilly PA, van Ryn J, Eikelboom JW, Glund S, Bernstein
RA,Dubiel R, Huisman MV, Hylek EM, Kam CW, Kamphuisen PW,
Kreuzer J, Levy JH, Royle G, Sellke FW, Stangier J, Steiner T,
Verhamme P, Wang B, Young L, Weitz JI.
N Engl J Med 2017; 377:431-441

Andexanet Alfa for Acute Major Bleeding Associated with Factor Xa


inhibitors. Connolly SJ, Milling TJ Jr, Eikelboom JW, Gibson CM, Curnutte
JT, Gold A, Bronson MD, Lu G, Conley PB, Verhamme P, Schmidt J,
Middeldorp S, Cohen AT, Beyer-Westendorf J, Albaladejo P, Lopez-Sendon
J, Goodman S, Leeds J, Wiens BL, Siegal DM, Zotova E, Meeks B,
Nakamya J, Lim WT, Crowther M; ANNEXA-4 Investigators. N Engl J Med.
2016 Sep 22;375(12):1131-41

295
Copyright © Harvard Medical School, 2018. All Rights Reserved.

Thrombophilia Tests and Prevalence of Risk Factors.

Connors JM. N Engl J Med ;377:1177-1187

Diagnostic Criteria for the Antiphospholipid Syndrome

Connors JM. N Engl J Med ;377:1177-1187

296
Copyright © Harvard Medical School, 2018. All Rights Reserved.

Bleeding Disorders 2018

Elisabeth M. Battinelli, M.D./Ph.D.


Assistant Professor of Medicine
Harvard Medical School
Associate Physician
Hematology Division
Brigham and Women’s Hospital

Elisabeth M. Battinelli, M.D./Ph.D.

Assistant Professor of Medicine


Associate Physician

Disclosures
Sanofi, Consultant

297
Copyright © Harvard Medical School, 2018. All Rights Reserved.

http://medicinembbs.blogspot.com/2011/02/normal-hemostasis.html

Primary Hemostasis

Durkan, CVC 2008

298
Copyright © Harvard Medical School, 2018. All Rights Reserved.

Secondary Hemostasis: Coagulation


(exposed TF)
XIa
IX TF
Phospholipid
IXa VIIa
Phospholipid
X
VIIIa VII
VIII

Xa
Phospholipid
Va V

II
IIa

Fibrinogen Fibrin

Coagulation Cascade Screening Tests


Endothelial Injury

aPTT PT

TT
Soluble fibrinogen

Fibrin stabilizes platelet plug

299
Copyright © Harvard Medical School, 2018. All Rights Reserved.

Case 1: Pregnancy Panic


• A 32 yo woman from another state, pregnant with
her first child, is visiting friends and unexpectedly
goes into labor.
• She tells the covering obstetrician and OB
anesthetist at your hospital that she has von
Willebrand’s Disease and wants to know if it is
okay for her to have an epidural.
• You are asked to see her and provide advice re the
advisability of an epidural and possible treatment
for the von Willebrand’s disease.

Laboratory results
• Chemistries including BUN, creatinine and
glucose are normal
• CBC:
– Hb is 12.3 gm/dl
– Hct 35.5%
– WBC 9500
– Platelet count 450,000
– Differential is reported as normal
• Coagulation Parameters:
– PT 11.5 seconds (INR 1.1)
– PTT 28 seconds (nl < 35)

300
Copyright © Harvard Medical School, 2018. All Rights Reserved.

Your consultation
• She tells you that she has had occassional
nosebleeds since she was a child and has had
menorrhagia, also she bled a lot when she had her
wisdom teeth extracted.
• She is usually treated with the intravenous or intra-
nasal administration of ddavp for bleeding or
before surgeries.
• Her mother and one sister have similar symptoms
and respond to the same medication regimen.
• You try but are not able to reach her primary
hematologist.

301
Copyright © Harvard Medical School, 2018. All Rights Reserved.

Mannucci, 2004

Coagulation in Pregnancy
Increase Decrease
•Fibrinogen •Protein S
•vWF •Acquired resistance to
•VIII activated protein C
•VII •Fibrinolysis
•X (inhibition)
•IX •Platelet Count
•XII

302
Copyright © Harvard Medical School, 2018. All Rights Reserved.

vWF
• Large multimeric protein
– Synthesized in endothelial cells, megakaryocytes
– Stored in Weibel-Palade bodies of endothelial
cells, alpha granules of platelets
• 3 main functions
– Bind to collagen on subendothelial matrix
– Bind to platelet GPIb receptor
– Prolong half life of factor VIII from 24 mins to
12 hours

vWD
• Most common inherited hemostatic disorder,
prevalence up to 1%
• Mucocutaneous bleeding
• Epistaxis
• Easy bruising
• GI bleeding
• Post-op
• Surgery--immediately
• Dental procedures
• “Women’s health issue
• 75% women with vWD have menorrhagia
• often dx with first menstrual period or PPH

303
Copyright © Harvard Medical School, 2018. All Rights Reserved.

vWF: Laboratory Evaluation


• aPTT

• FVIII:C

• vWF:Ag

• vWF:RCo Ristocetin cofactor activity

• Multimer gel electrophoresis

vWD
• Mild vWD can be difficult to diagnose
• Levels vary and are affected by:
– Blood type
– Estrogen
– Inflammation
– Stress
– Smoking

304
Copyright © Harvard Medical School, 2018. All Rights Reserved.

vWD: Classification
– Type I: autosomal dominant, quantitative
decrease in vWF and concordant decrease in
all functions
• 70-80% of vWD cases

– Type III: homozygous recessive, almost no


detectable vWD
• Very Rare

vWD: Classification
Type II: Qualitative Abnormalities
A: decreased large mw multimers
10-15% of VWD

B: gain of function mutations, increased binding to


gpIb
M: loss of function mutations, decreased binding to
gpIb
N: loss of function mutations, decreased FVIII
binding

305
Copyright © Harvard Medical School, 2018. All Rights Reserved.

TYPE 1 vWD

• Quantitative deficiency
• Autosomal dominant trait
• 85% of patients
• Mis-sense mutations detected in some
but not all patients

vWD and Pregnancy


• Antigen levels increase with increased
estrogen
• Function does not change
• Increased risk of bleeding
– Miscarriage?
– 1st trimester
– Perineal hematomas
• 3/49 vaginal deliveries vs
• 2.2/1000 from 26,187 cohort study

306
Copyright © Harvard Medical School, 2018. All Rights Reserved.

vWD and Pregnancy


• Increased risk of post-partum hemorrhage

– Levels return to baseline over 7-21 days

– 20-25% patients with vWD have delayed PPH

– Mean onset 15.7 + 5.2 days

vWD: Treatment
DDAVP
• Release of stored vWF from Weibel-Palade bodies
• Onset of action within 30 mins
• Watch for Tachyphylaxis with repeated dosing
• Side effects include:
– Headache, HTN, flushing,N/V, hyponatremia and
seizures, uterine contractions
• Pre-procedure DDAVP challenge test to monitor
response

307
Copyright © Harvard Medical School, 2018. All Rights Reserved.

vWD:Treatment
vWF containing concentrates
• plasma derived pathogen-inactivated.
• Dose by FVIII or vWF levels.
– Humate-P
– Alphanate
Cryoprecipitate
• Pooled product from 10 donors*
• Only as last resort

Next steps
• Assume she has von Willebrand’s as
suggested by her history?
• Administer DDAVP to treat her
presumed vWD?
• Tell the patient and her doctors that you
are not certain what she has and how to
treat it and she must not have epidural
anesthesia?

308
Copyright © Harvard Medical School, 2018. All Rights Reserved.

What happens next?


• At 3 am your patient delivers a 7 lb 4 oz baby boy without
any bleeding

• You reach primary hematologist and learn that she indeed


has Type I vWD.

• Labs prior to pregnancy vWF Ag 30%, activity 35%, VIII


activity 45%

• Values two weeks before delivery vWF Ag 105%, activity


120%, VIII activity 150%

And then ….
• 48 hours after delivery you are making
rounds at the hospital and are paged by her
obstetrician because she has developed
uterine hemorrhage.
• What has happened?

309
Copyright © Harvard Medical School, 2018. All Rights Reserved.

VON WILLEBRAND’S DISEASE AND


PREGNANCY

• SELF CORRECTION IN (MOST) TYPE 1


PATIENTS
• POSTPARTUM BLEEDING IN 25% OF
PATIENTS
• INCREASED THROMBOCYTOPENIA IN
TYPE 2B PATIENTS

vWD: NHLBI Guidelines


• “We suggest that VWF ristocetin cofactor
activity and factor VIII levels of at least 50
IU/dL be achieved before delivery and
maintained for three to five days afterward (Grade
2C).”
• For patients who do not have this degree and
length of response on their own or following the
use of DDAVP, we suggest the use of VWF
concentrates (Grade 2C).”

NHLBI Guidelines for vWD 2008

310
Copyright © Harvard Medical School, 2018. All Rights Reserved.

vWD: NHLBI Guidelines


“In patients who are known to adequately
respond to DDAVP, we suggest that
DDAVP be given after the beginning of
labor and as close to the time of delivery as
can be estimated (Grade 2C).”

NHLBI Guidelines for vWD 2008

vWD: NHLBI Guidelines


“We suggest that regional anesthesia can be
considered if vWF and FVIII levels can be
maintained above 50 IU/dl (Grade 2C).”

NHLBI Guidelines for vWD 2008

311
Copyright © Harvard Medical School, 2018. All Rights Reserved.

Case 2: Can’t Stop Bleeding


• 23 year old female who presents with bleeding after wisdom tooth
removal. Patient has been referred to ED from dental clinic after the
dentist could not control the bleeding.

• Pt. has no history of epistaxis, gingival bleeding but does report that at
onset of menses her period was very heavy prompting her to regulate
menses with OCPs.

• No history of any other tooth extractions or surgical procedures.

• Family Hx: Mother with von Willebrand disease per report of patient.
No other bleeding history in the Family.

Can’t Stop the Bleeding: Labs


• WBC 8.6
• Hct 38
• Plt 235, normal appearance on smear
• PT 1.1
• PTT 32.5
• Fibrinogen 365
• von Willebrand Panel is normal
• Platelet Aggregation studies: Abnormal

312
Copyright © Harvard Medical School, 2018. All Rights Reserved.

Can’t Stop the Bleeding:


Platelet Aggregation Studies
• Absent platelet aggregation in response
to multiple agonists:
– ADP
– Thrombin
– Collagen
– Arachadonic Acid
• Normal Aggregation to Ristocetin

Signs of Thrombocytopathies
• Prolonged bleeding times
• Defective clot formation
• Bleeding tendency from childhood

• Low prevalence of these disorders but high


percentage of patients with unclassified
platelet dysfunction.

313
Copyright © Harvard Medical School, 2018. All Rights Reserved.

Classify platelet disorders by platelet


functionality
• Adhesion/Aggregation
• Activation/Signal Transduction
• Storage Granule Disorder/Release
• Aggregation

Platelet aggregation studies


• Aggregation
– Light transmission aggregometry using Chrono-
Log instrument
– ADP, arachidonic acid, collagen, epinephrine,
ristocetin
• ATP secretion
– Luciferin-Luciferase assay
– ADP, arachidonic acid, collagen,
epinephrine, ristocetin, thrombin

314
Copyright © Harvard Medical School, 2018. All Rights Reserved.

Platelet aggregation studies


• Requirements:
– Platelet count minimum 100K/uL
– 8 blue-top (citrate) tubes
• Common interferences:
– EDTA
– NSAIDs (e.g., aspirin, ibuprofen, indomethacin, COX-2
inhibitors)
– ADP receptor antagonists (e.g., clopidogrel, ticlopidine)
– GPIIb/IIIa antagonists (e.g., abciximab, eptifibatide,
tirofiban)
– Various other drugs (see Arch Pathol Lab Med.
2002;126:133–146)

Problems with Platelet Aggregation


Studies

• Numerous variables affect aggregation:


• Anticoagulant (sodium citrate best)
• Plt count in PRP
• Plt size distribution
• Time of day
• Temporal relation to meals and physical activity

315
Copyright © Harvard Medical School, 2018. All Rights Reserved.

Aggregometry

•Platelets shift from disc-like to a rounded form with


pseudopods transiently decreasing light transmission.
•Then they aggregate into clumps, increasing light
transmission.

The Aggregometer Tracings

Secondary wave (Dense granule


release, thromboxane A2
production)

Primary wave (Agonist


interacts with receptor)

316
Copyright © Harvard Medical School, 2018. All Rights Reserved.

Disorders in Platelet Aggregation

http://medicinembbs.blogspot.com/2011/02/normal-hemostasis.html

Aggregation Disorders: Glanzmann’s


Thrombasthenia

• History of muco-cutaneous bleeding

• Autosomal Recessive Inheritance

• Deficiency of GPIIb/GPIIIa complex

317
Copyright © Harvard Medical School, 2018. All Rights Reserved.

Diagnosis
• Platelet count and morphology is normal
• Bleeding time prolonged
• The hallmark of the disease is severely
reduced or absent platelet aggregation
in response to multiple agonists ie ADP,
thrombin, or collagen (except
Ristocetin)
• Flow cytometry: decreased mAb
expression of CD41 (GPIIb) and CD61
(GPIIIa)

Can’t Stop the Bleeding:


Management
• Platelet transfusions
• Supportive care: ddavp and amicar
• She improved after transfusions of platelets.

318
Copyright © Harvard Medical School, 2018. All Rights Reserved.

Case 3: Is it safe to operate?


• 75 y.o. female comes to you for pre-op
clearance for cardiac surgery.
• She is referred to your clinic for a prolonged
PTT.
• She has not seen a physician in 40 years.
• No bleeding history per the patient

Laboratory testing

• Factor XIII screen is normal


• Factor VIII level is 102%
• Factor IX level is 89%
• Factor XI level is 22%

319
Copyright © Harvard Medical School, 2018. All Rights Reserved.

Factor XI Deficiency

• Rare bleeding disorder


• Inherited as Autosomal Recessive
• Prolongs the PTT
• Characterized by variable bleeding history
• Incidence is 1 in 450 in the Ashkenazi
Jewish population and 1 in a million in non-
Jewish population.

Factor XI deficiency

• Usual sights of bleeding:


– Skin and mucousa
– Genitourinary tract
– Gastrointestinal tract

320
Copyright © Harvard Medical School, 2018. All Rights Reserved.

Factor XI deficiency
• Bleeding manifestations do not correlate with factor XI
levels
• Most bleeding episodes in patients with severe deficiency
are injury-related
• Spontaneous bleeding is rare
• May be associated with bruising, epistaxis, menorrhagia,
GI/GU bleeding, umbilical stump bleeding or bleeding
after surgery, trauma, dental procedures, pregnancy or
circumcision
• Up to 33% of patients with severe deficiency develop
inhibitors after replacement therapy

321
Copyright © Harvard Medical School, 2018. All Rights Reserved.

Treatment for Factor XI


Deficiency
• 10-20 ml fresh frozen plasma/kg, then 5-10 ml/kg
every 24 hours as necessary
• Antifibrinolytic therapy has been used in women
with factor XI deficiency and menorrhagia
• Patients with inhibitors have been treated
successfully with plasma, prothrombin complex
concentrates, and recombinant activated factor VII
• Note: factor XI concentrates available in Europe
monitor for thromboembolic complications

CASE 4
25 yo healthy woman
• CC: rash on lower
extremities
• PE:
• skin: rash
• HEENT: hemorrhagic bullae
in mouth
• Otherwise normal
• Labs:
• WBC 3.9 with normal diff; Hct 35
with normal MCV; Plts 14K
• PT/PTT normal
• electrolytes, LFTs normal

322
Copyright © Harvard Medical School, 2018. All Rights Reserved.

Thrombocytopenia
DECREASED PRODUCTION
• Marrow Failure
• Drugs
• Myelophthysis
• Nutritional deficiency (megaloblastic anemia)

SEQUESTRATION
• Splenomegaly (rarely <50,000)

INCREASED DESTRUCTION
• ITP
• TTP
• DIC

Evaluation of Thrombocytopenia
History
• Intercurrent illnesses
• Medication history
• History of autoimmune disease, LPD

Physical
• Bleeding manifestations (purpura, petechiae)
• Splenomegaly
• Adenopathy
Laboratory
• Examination of the peripheral smear
• Platelet reticulocyte count
• Antiplatelet antibodies??
• Bone marrow examination??

323
Copyright © Harvard Medical School, 2018. All Rights Reserved.

Laboratory Evaluation for


Thrombocytopenia
• CBC
• Peripheral Blood Smear
• Liver Function tests
• Thyroid Function tests
• Quantitative immunoglobulin levels
• Direct antiglobulin test
• Antiphospholipid antibodies
• ANA
• H pylori testing
• HIV, HCV, HBV
• VWD type IIb testing

ITP Therapy: Initial Treatment


Agents used for initial therapy of ITP:
• Corticosteroids
• High dose methylprednisolone
• Prednisone
• Dexamethasone
• IV Immunoglobulin
• Anti-D

324
Copyright © Harvard Medical School, 2018. All Rights Reserved.

ITP Therapy: Initial Treatment

“Standard of care”
• Prednisone 1mg/kg/day
• Taper—schedules undefined
• Response: 65-90%
• Long term response: 5-30%
• Of note, study that reported 30% assessed at
6 months.
• Most studies with longer follow-up report <10%
long term response to prednisone

ITP Therapy: Initial Treatment


Dexamethasone:
• Single course, 40mg/day X 4 (NEJM 2003; 349: 831)
• Initial response: 85%
• Relapse: 50%; 2nd course response: 100%
• Sustained: 42%
• Persistent response after D/C maintenance: 18%
• Multiple course, 40mg/day X 4 (BLOOD 2007;109: 1401)
• Two protocols: 6 cycles q28 days/4 cycles q14 days
• Initial response: 89%/86%
• RFS at 15 mos: 90% /81%
• Long term response:
• median 26mos with 6 cycles: 68%
• median 8 mos with 4 cycles: 74%
• Randomized trial Dex vs Prednisone (BLOOD 2016;127: 296)
• Durable responses with less toxicity
• Only a single course, so still need a trial of repeated courses of
dexamethasone

325
Copyright © Harvard Medical School, 2018. All Rights Reserved.

ITP Therapy: Initial Treatment


100 100

Patients achieving a remission (%)

Patients in ongoing remission (%)


75 75
p = 0.40 p = 0.007

50 50

25 25
Prednisone, n = 12
Dexamethasone, n = 14
0 0
0 1 2 3 0 30 60 90 120 150
a Months b Months

100 100
Patients achieving a remission (%)

Patients in ongoing remission (%)


75 75
p = 0.55 p = 0.014

50 50

25 25
Prednisone, n = 9
Dexamethasone, n = 13
0 0
0 1 2 3 0 30 60 90 120 150
c Months d Months

100 100
Patients achieving a remission (%)

Patients in ongoing remission (%)


75 75
p = 0.18 p = 0.053

50 50

25 25
Prednisone, n = 7
Dexamethasone, n = 12
0 0
0 1 2 3 0 30 60 90 120 150
e Months f Months

Matshke et al Acta Haem. 2016: 136:101-107

Therapy: Relapsed or Refractory ITP


Response Time to Duration of
Therapy Rate Response Toxicity Response
Splenectomy 80% 1-24 days Surgical 2/3 with no
complications, further Rx
thrombosis

Rituximab 60% response 1-8 weeks Allergic rxns; 15-20%


40% complete immune supp. sustained
Can be retreated
TPO Mimetics >80% response 2-3 weeks Unknown long- Up to 1.5-4 yrs
(plts >50K) term toxicity with continuous
Increased reticulin drug
Rapid fall of plts
with D/C of drug

Adapted from Blood. 2010;115:168-186

326
Copyright © Harvard Medical School, 2018. All Rights Reserved.

TPO Response to Thrombocytopenia

Hematol Oncol Clin North Am.23:1193, 2009.

Thrombopoietin Mimetics
Romiplostim Eltrombopag
• Fc-peptide fusion • Small molecule, oral TPO
protein (peptibody)
receptor agonist
• Binds to and activates the
TPO receptor to increase • Increases platelet
platelet counts production by increasing
• Approved for treatment of megakaryocyte growth
chronic ITP in US, EU,
Canada and Australia
TPOr binding
Fc Domain domain

Both approved for the treatment of chronic ITP

327
Copyright © Harvard Medical School, 2018. All Rights Reserved.

Take Home Messages: Case 4


• The treatment of ITP continues to be somewhat
controversial
• First-line therapy is corticosteroids—my
recommendation is pulse dexamethasone
• Second line therapy is probably still splenectomy
• However, rituximab is effective in many patients, is
probably the third most likely to give a long
remission, and can be repeated
• TPO mimetics work but have some known and
many possible unknown downsides.

Summary
• In bleeding disorders you must think about
primary and secondary hemostasis to
understand etiology.
Primary: Platelets disorder and vWD
Secondary: Coagulation factors and
fibrinolysis

History is Key for diagnosis and management.

328
Copyright © Harvard Medical School, 2018. All Rights Reserved.

Board Review Questions

Question 1
• You are asked to see a 37 yo woman
with history of vWD for consultation.
She asks: “Is it safe for me to have
elective knee surgery?”
• Pt. never had an surgery. She had
frequent epistaxis as a child and heavy
menses.
• FH: Mother has hx vWD but died young
(trauma). 5 uncles, one with bleeding
disorder.

329
Copyright © Harvard Medical School, 2018. All Rights Reserved.

Question 1

– F VIII:C 27
– vWF:ag 89
– vWF:RCo 75
– PTT 46 sec

• What type of vWD does this patient have?

Question 1
• A. Types I vWD. Treat with ddavp prior to
surgery.
• B. She does not have vWD because her
VWF:ag and :Rco are normal.
• C. You need more info. She could have type
2N or be a hemophilia A carrier.

330
Copyright © Harvard Medical School, 2018. All Rights Reserved.

2N vs. Hemophilia A carrier


• 2N should be consider in AR inheritance
with disproportionately low FVIII:C levels
compared with VWF levels.
• To prove 2N, a FVIII-VWF binding assay is
required.
• VWF gene mutation screening:R854Q at
amino-terminus of VWF subunit is most
frequent

2N vs. Hemophilia A carrier


• Treatment Plan?

• Hemophilia A carrier
– Low production FVIII
– Normal vWF
– Treat with recombinant FVIII
– Maintain levels in postop period

• vWD Type II N
– Normal FVIII production
– Endogenous vWF does not bind FVIII
– Treat with Humate-P
– Monitor levels in postop period

331
Copyright © Harvard Medical School, 2018. All Rights Reserved.

Question 1

• A. Types I vWD. Treat with ddavp at


delivery and postpartum.

• B. She does not have vWD because her


VWFag and Rco are normal.

• C. You need more info. She could have type


2N or be a hemophilia A carrier.

Question 2
• 38 y.o. Female is seen in the ER for
bleeding gums after a dental procedure.
She tells you that she has essential
thrombocytosis and takes daily
hydroxyurea and aspirin.
• A CBC is drawn and her platelet count
is 1600 X109/L million.

The ER team asks why this patient is


bleeding

332
Copyright © Harvard Medical School, 2018. All Rights Reserved.

Possible Answers

• A. The patient is on aspirin and


bleeding due to its anti-platelet effects.
• B. The patient has developed an
acquired von willebrand disease.
• C. This is a laboratory error and the
platelet count should be repeated.

Question 2
• This patient has developed an
acquired von willebrand disease in
the setting of thrombocytosis.
• Diagnosis is confirmed by von
Willebrand panel demonstrates low
ristocetin cofactor activity with normal
antigen levels.
• Aspirin should be used cautiously in
patients with platelets >1000 x 109/L

333
Copyright © Harvard Medical School, 2018. All Rights Reserved.

References
Mannucci, PM. Treatment of von Willebrand’s Disease. N Engl J Med. 2004 Aug 12;351(7):683-94.

Giuseppe L Diego A, Roberto Q, and Gianfranco C. Urgent monitoring of direct oral anticoagulants in
patients with atrial fibrillation: a tentative approach based on routine laboratory tests. Journal of Thrombosi
and Thrombolysis, May 2014 (epub).

Conolly, S. et al. Dabigatran versus Warfarin in Patients with Atrial Fibrillation. N Engl
J Med 2009; 361: 1139-1151.

Duga, S., Salomon O., Congenital Factor XI deficiency: an update. Semin Thromb Hemost. 2013
Sep;39(6):621-31. doi: 10.1055/s-0033-1353420. Epub 2013 Aug 8.

Gernsheimer, T., James, AH, and Stasi, R. How I treat thrombocytopenia in pregnancy. Blood. 2013 Jan
3;121(1):38-47. doi: 10.1182/blood-2012-08-448944. Epub 2012 Nov 13.

Elisabeth M. Battinelli, M.D./Ph.D.


Assistant Professor of Medicine
Associate Physician

No Disclosures

334
Copyright © Harvard Medical School, 2018. All Rights Reserved.

Hematology Cases:
Common, Complex, and Rare
Nancy Berliner, M.D.
H. Franklin Bunn Professor of Medicine
Chief, Division of Hematology
Brigham and Women’s Hospital
Professor of Medicine
Harvard Medical School

DISCLOSURES

Nothing to disclose

Discussion of experimental drug:


Emapalumab (Novimmune, NI-0501)

335
Copyright © Harvard Medical School, 2018. All Rights Reserved.

CASE 1

• 30 year-old woman with no significant past


medical history
• 10 days prior to admission:
– mild URI
– 1-2 days of “loose stools” (non-bloody)
• 1-2 days prior to admission:
– progressive debilitating fatigue, exertional
dyspnea, and some easy bruising

Labs

134 96 88 4.0 15.0 32.0


92 9.60 111
3.5 21 12.31 1.2
11.8

U/A:
LDH: 1566 hCG: negative
Glucose: negative
Tbili: 1.3 DAT: negative
Bili: negative
ALT/AST: 29/35 C3: 79 (normal 90-180)
Ketones: 1+
TP: 4.2 C4: 11 (10-40)
Blood: 3+
Alb: 3.0 Stool culture negative;
Protein: 3+
including E. coli O157:H7
Nitrite: negative
Leuk Est: neg
RBC: 4-10
WBC: 0-4

336
Copyright © Harvard Medical School, 2018. All Rights Reserved.

Peripheral Blood Smear

Laboratory features in TTP vs. HUS

Furlan M et al. N Engl J Med 1998;339:1578-1584.

337
Copyright © Harvard Medical School, 2018. All Rights Reserved.

Atypical HUS: Overview

• Arises through dysregulation of the alternative


complement pathway
• Multiple loss-of-function and gain-of-function
mutations in multiple complement pathway
genes are known to predispose
– Factor H, factor I, MCP, thrombomodulin, factor B, C3
– ~50% of patients have heterozygous mutations
– ~10% have more than one mutation
– Penetrance is 50%; additional genetic/environmental
modulators likely to be at play
• Renal involvement is predominant
Rosove, Sem Arthritis & Rheumatism, 2013

aHUS: The Alternative complement pathway

338
Copyright © Harvard Medical School, 2018. All Rights Reserved.

Atypical HUS: Complement Factor Mutations

Complement levels
are variable

Nester, ASH Book, 2012

Atypical HUS: Serum complement levels

Loirat et al., Ped Nephrol, 2008

339
Copyright © Harvard Medical School, 2018. All Rights Reserved.

Treatment Prior to Eculizumab for aHUS


• Plasma Exchange
– Recommended treatment prior to Eculizumab based on case
series
– No RCTs. Loirat et al. (Ped Neph 1988) showed no benefit of
plasma infusion, but did not distinguish HUS and atypical HUS
• Transplant
– High rate of recurrence with renal transplant alone unless MCP
mutation.
– Case reports of liver and renal transplant shows proof of
principle, but 2 cases with bad outcomes (Remuzzi et al. Lancet
2002)

Kim et al. Pediatric Neph 2011

Treatment of aHUS: Plasma infusion/pheresis


• No prospective controlled trials of plasmapheresis
• Plasma infusion may correct defects in patients with
quantitative complement factor deficiencies
• Plasma exchange may be required in patients with
antibodies (i.e. anti-H) or in patients with mutations
causing protein dysfunction
• Certain complement proteins (i.e. MCP) are membrane
bound and defects cannot be corrected by plasma therapy
• 2009 European Study Group guidelines for aHUS: “consensus”
– Plasmapheresis within 24 hrs of diagnosis
• Repeat daily x 5 days, then 5x weekly x 2 weeks
• With this “standard” treatment:
– ~60% initial response
– ~25 mortality rate
– ~ 50% left with CKD
– Outcomes vary by mutation type

340
Copyright © Harvard Medical School, 2018. All Rights Reserved.

Prognosis in Atypical HUS


• Prognosis
– Overall 5 year survival-52% children, 33% adults (Fremeaux-Bacchi et
al. J Am Soc Neph 2013.)
– Based on data pre-eculizumab therapy
– Genotype is predictive of outcome
• MCP-better prognosis. Only 35% with ESRD at 5 years
• CFH-worse prognosis. 77% with ESRD or died.

Treatment of aHUS: Use of eculizumab

• Humanized monoclonal antibody against C5


• First-in-class terminal complement inhibitor
• Approved for PNH in 2007
• Approved for aHUS in 2011
• World’s most expensive drug - $409,500 per year
• Requires meningococcal vaccination prior to initiation
and long-term antibiotic prophylaxis
• Has been reported to be effective for aHUS in multiple
case reports and a recent prospective trial
• No randomized trial of eculizumab vs. plasma exchange
has been conducted

Kavanagh, Semin Nephrol, 2013

341
Copyright © Harvard Medical School, 2018. All Rights Reserved.

Eculizumab blocks C5, inhibiting terminal


complement activation

Edwin et al. Molecular Immunology 2013

Treatment of aHUS: Use of eculizumab


• Two separate 26-week Phase II studies
– Trial 1: patients with aHUS and progressing TMA
– Trial 2: patients with aHUS of long duration, CKD, and on
prolonged plasmaphersis/infusion treatment

Outcomes were similar in patients in which a


complement mutation was not identified
Legendre et al., NEJM, 2013

342
Copyright © Harvard Medical School, 2018. All Rights Reserved.

Treatment of aHUS: How long?

Originally suggested that lifelong therapy should be continued


….especially by the company that makes eculizumab
But…
Study of 17 patients with aHUS treated with eculizumab
Taken off eculizumab and monitored
13/17 patients did not require further therapy
Those that recurred were successfully treated
Blood 2017 130:368

Take Home Points: Case 1


TTP
– Hereditary (Upshaw-Schulman syndrome)-ADAMTS13 deficiency
– Acquired: Antibody to ADAMTS13
• Idiopathic--usually IgG inhibitor to ADAMTS13
• May be associated with pregnancy, malignancy, lupus
• Response to plasma exchange in 90+%
• Relapse rate 10-30%
HUS
• Shiga toxin producing E. Coli
• Seen almost entirely in children
• Excellent prognosis with no therapy
Atypical HUS
• Genetic mutations, and rarely antibodies that cause inappropriate
activation of alternative complement cascade
• Grim prognosis in the pre-eculizumab era
• Prognosis with eculizumab still being defined
George et al. Am Soc Hematology 2012

343
Copyright © Harvard Medical School, 2018. All Rights Reserved.

CASE 2
48 y.o. male w/ CAD, thrombocytosis
w/ LLE extremity DVT worsening on warfarin
HPI:
• 13 yrs. PTA: RLE DVT post-vein stripping (warfarin × 6 mo)
• 10 yrs. ago RLE DVT s/p angioplasty (warfarin × 5 yr)
• off anti-coagulation × 5 yr
• 3 weeks PTA: to outside hospital w/LLE swelling, no precipitant
• Dx: LLE DVT; Rx heparin → warfarin
• D/C’d home
• Represents with pain, discoloration, and increased LLE
swelling.

PMH
• LLE DVT X1; RLE DVT X2
• s/p venous stripping
• CAD s/p angioplasty
• Hypertension
• Thrombocytosis; known JAK2 +
• MEDS: warfarin, ASA, metoprolol

CASE 2

PE
BP 112/60 HR 74 RR 16 Temp 98°F
Exam benign with exception of LLLE
LLE >> RLE; tender to deep palp, + erythema
Pulses intact

Labs
14 PT/INR 24/3.9
12 678
44 PTT 50
Electrolytes, BUN, Creatinine normal

344
Copyright © Harvard Medical School, 2018. All Rights Reserved.

CASE 2

Radiographic Studies

US:
• New occlusive thrombus L ext iliac vein to popliteal vein
• Old non-occlusive thrombus R femoral to popliteal

CT Scan:
• New DVT L ext iliac vein to femoral vein; also hypogastric
• Old DVT R ext iliac to fem
• Infra-renal IVC occlusion with collaterals
• Dilated inferior mesenteric veins
• Calcifications within the pelvic veins

CASE 2

Hospital Course
• Diagnosed as warfarin failure
• Heparin begun and tPA thrombolysis
• Initially improved, then worsened
• Transferred to MICU, Hematology consulted

Hypercoagulable workup
• JAK2 positive
• Factor V Leiden positive (homozygous)
• Bone marrow normal

345
Copyright © Harvard Medical School, 2018. All Rights Reserved.

What happened??

CAUSES OF THROMBOPHILIA

Congenital Disorders Acquired Disorders


• ATIII deficiency • Pregnancy, estrogen
• Protein C deficiency • Immobilization
• Protein S deficiency • Trauma,
• Factor V Leiden • Antiphospholipid syndrome
• Prothrombin 20210 • Malignancy
• Hyperhomocysteinemia • Nephrotic syndrome
• HIT
• Myeloproliferative Dx
• PNH
• Hyperhomocysteinemia

346
Copyright © Harvard Medical School, 2018. All Rights Reserved.

MPN: Pro-thrombotic
Thrombosis at diagnosis:
• 40% PV
• 30% ET
Arterial > venous thrombosis
Microcirculatory disorders: platelet thrombi
• Erythromelalagia
• Visual symptoms—occular migraines
• Neurologic symptoms: TIA like
Risk stratification:
• PV and ET patients at HIGH risk for thrombosis:
• Prior thrombosis
• Age > 60
• Cardiovascular risk factors? (DM, HTN, cholesterol, smoking) difficult
to distinguish disease-specific risk vs these

Venous clots respond to anticoagulation

What causes poor response to adequate anticoagulation?

Consider direct thrombin activation

347
Copyright © Harvard Medical School, 2018. All Rights Reserved.

Hematology 2006 The American Society of Hematology

Heparin-Induced Thrombocytopenia (HIT)

Diagnosis:
• HIT is more common in surgical patients, where platelet
counts typically rise postoperatively
• Typically seen at 1-2 weeks of prophylaxis or Rx with
heparin
• HIT: Relative fall to below 50% of the peak platelet count
• Rarely if ever occurs within the first five days of heparin
therapy, except with recent exposure to heparin
• Less common with LMWH, but once a patient has HIT,
antibodies are cross-reactive
• No absolute platelet number is diagnostic of HIT; more
important to consider in the appropriate clinical setting

348
Copyright © Harvard Medical School, 2018. All Rights Reserved.

HIT: Pre-test clinical score: “4 T’s”


Thrombocytopenia
• <30% drop in platelet count: 0 points
• 30-50% drop in platelet count: 1 point
• >50% drop in platelet count: 2 points
Timing
• < 4days with no prior exposure in last 100 days: 0 points
• >10 days or <1 day with exposure in last 30-100 days: 1 point
• 5-10 days after heparin exposure: 2 points
Thrombosis
• None: 0 points
• Suspected or non-necrotizing skin lesions: 1 point
• Confirmed thrombosis, skin necrosis: 2 points
oTher Likely Cause
• None: 2 points
• Possible: 1 point
• Definite: 0 points

< 3: Low; 3-5 Intermediate; >5 High Suspicion

J Thromb Haemost 2006; 4: 759–65.

HIT: Timing of Low Platelets and Thrombosis

Warkentin, T. E. Hematology 2006;2006:408-414

349
Copyright © Harvard Medical School, 2018. All Rights Reserved.

HIT

Immune mediated IgG Ab response to heparin/PF4


complex
• CVS > surgical > medical patients
• UFH (bovine>porcine) >> LMWH
8% of heparin treated patients develop Ab
1-5% develop HIT thrombocytopenia
• 30-50% of these develop thrombosis
• 20-30% morbidity and mortality rate

HIT Pathogenesis Patient makes


IgG Antibody
against the complex

Heparin then binds


to PF4,
Fc portion of this antibody
forming complex
binds to platelet receptor,
activating platelets

Heparin stimulates
PF4 release from -Thrombin generation
platelet granules -EC activation

Activated platelets
aggregate and release PF4
Images.md

350
Copyright © Harvard Medical School, 2018. All Rights Reserved.

Warfarin does NOT inhibit coagulation!

• Warfarin inhibits γ-carboxylation of clotting


factors and Proteins S and C
• It lowers the levels of factors, but does not
inhibit coagulation
• Clearly shown not to prevent thrombosis in HIT
• Can predispose to coumadin skin necrosis in the
setting of thrombin activation of any sort-
including, but not restricted to, HIT.

Heparin-Induced Thrombocytopenia

Treatment:
• STOP HEPARIN
• STOP WARFARIN
• If patient on warfarin, administer Vitamin K to reverse it
• Treatment with direct thrombin inhibitors until clinically
improved and plt count has returned to (near) normal
• Overlap warfarin and DTI X at least 5 days
• Warfarin to INR of 3-4 (depends on DTI), and overlap at
least 2 days with therapeutic INR
• Warfarin for 3-6 months in the setting of thrombosis
• Duration in the absence of thrombosis…no data

351
Copyright © Harvard Medical School, 2018. All Rights Reserved.

Take Home Points: Case 2


• Venous thrombosis usually responds to
anticoagulation
• If you see recurrent thrombosis, think of
malignancy, anti-phospholipid syndrome, or HIT
• Thrombosis in HIT may PRECEDE the
development of thrombocytopenia
• Treatment with direct thrombin inhibitor
• Therapy in patients on warfarin includes reversal
of warfarin

CASE 3
39 y/o female home maker, wife and mother of three with no
significant past medical history

• 12/12/04 – fevers, chills, rigors


– WBC 3.9, Hct 34.8, plt 148
– Fevers resolved after a 10 day course of doxycycline

• 1/05— recurrent fevers


– CBC and chemistries - normal
– bone marrow biopsy: normocellular w/small, poorly formed
granulomas

• 2/12/05 – presented with fever and back pain


– ROS – fevers, chills, rigors, back pain, generalized fatigue

352
Copyright © Harvard Medical School, 2018. All Rights Reserved.

CASE 3
Outside Hospital:
• WBC 1.1 (ANC 600), Hct 23.5, platelets 77
• ESR 9
• Blood cultures negative
• CXR normal
Transferred on vancomycin, ceftazadime, doxycycline
On arrival:
• WBC 0.9 with 18 segs, 42 bands, 26 lymphocytes, 9 monocytes, 5
metamyelocytes, 1 nucleated RBC
• Hct 22.8 with reticulocyte count of 2.3%, platelets 73
• Na+ 134, K+ 3.4 Cl- 102, HCO3- 25, BUN 12, Cr 0.8
• Tbili 0.58, ALT 55, AST 128, AP 97, alb 3.2, LDH 1350
• INR 1.47, PTT 30.5, D-dimer >1.0, FSP >40, fibrinogen 176
• Iron 50, IBC 245, % iron saturation 20, ferritin 1460
• CRP 5.3

Bone Marrow Aspirate


This image cannot currently be display ed.

353
Copyright © Harvard Medical School, 2018. All Rights Reserved.

Hemophagocytic Lymphohistiocytosis

• Related to impaired cytolytic activity of T cells and NK


cells
• Uncontrolled T-cell activation with increased secretion of
TH1 cytokines, including IFNγ, IL-12, and IL-18, which
activate macrophages
• Results in proliferation and activation of benign
macrophages, and is associated with phagocytosis of
hematopoietic elements throughout the RE system
• Familial HLH is associated with identified defects in
cytotoxic T cells and NK cells
• Mechanism of acquired forms of HLH is unknown

HLH: Diagnostic Criteria


Molecular Diagnosis consistent with HLH
OR
Clinical and laboratory criteria (5/8)
• Fever
• Splenomegaly
• Cytopenia
• Hypertriglyceridemia and/or hypofibrinogenemia
• Fasting triglycerides > 3mmol/l
• Fibrinogen < 1.5 g/l
• Ferritin > 500
• sCD25 > 2400U/ml
• Decreased or absent NK-cell activity
• Hemophagocytosis in bone marrow, CSF, or nodes

These criteria are entirely based on PEDIATRIC patients

354
Copyright © Harvard Medical School, 2018. All Rights Reserved.

Adult HLH: A newly recognized epidemic?

• In the last 10-15 years, there has been an explosion of


reports and studies of adult HLH
• Apparent increase in HLH in adults is unexplained
• Newly-prevalent vs newly-recognized
• Either way, is an increasingly common diagnosis
• Despite this, the diagnosis continues to be driven by
criteria developed in the pediatric population
• Diagnosis and treatment in adults is evolving

Hemophagocytic Lymphohistiocytosis
Genetic HLH
• Familial HLH
• Known gene defects (perforin, munc 13-4, syntaxin 11)
• Unknown gene defects
• Immune deficiency syndromes
• Chediak-Higashi syndrome
• Griscelli syndrome
• X-linked lymphoproliferative syndrome
Acquired HLH
• Infection associated hemophagocytic syndrome
• Autoimmune disease (macrophage activation syndrome)
• Malignancy (T cell lymphoma)
• Drug hypersensitivity reaction (?)

355
Copyright © Harvard Medical School, 2018. All Rights Reserved.

CTL Killing of Target Cells


• Perforin resides within the granules
Figure 12-10 of NK cells and CTLs
• Granules are released into the
“immunological synapse” at the site
of contact between a CTL and a
target cell
• The granule membrane fuses with
the cell membrane, and releases the
contents of the granules
Copyright © 2005 Elsevier Inc. (USA) All rights reserved.
• Perforin binds to and makes “pores”
in the target cell membrane.
• May allow entry of granzymes,
important mediators of apoptosis of
the target cell. Alternatively, it may
be necessary for the release of
granzymes from the endocytic
vesicle.
• Results in killing of the target cell.

Perforin in T Cell Homeostasis

• During acute T cell responses, T cells interact with


APCs, and cells may be activated, remain silent, or die
(activation-induced cell death).
• Deletion is promoted by high affinity interactions, high
dosage, and long duration of antigen stimulation.
• Perforin is required for activation-induced T cell death

Curr Opin Hematol 15: 359-67, 2008

356
Copyright © Harvard Medical School, 2018. All Rights Reserved.

Surveys of adult HLH


• 103 adult HLH patients from single hospital in China:
– 48% hematologic malignancy
– 14% autoimmune disease
– 23% infectious disease
– 23% unknown
Li et al, Medicine 2014; 93: 100
• 73 adult HLH patients from WUSL, St Louis
– 40% infectious disease
– 29% hematologic maliginancies
– 7% autoimmune disease
– 18% unknown
Otrock and Eby AmJHematol 2015; 90:220
• 68 patients in Partners hospitals, Boston
– 49% malignancy
– 33% infectious
– 28% autoimmune disease
– 15% unknown
Schram et al, BrJHaematol 2015; in press

• 62 patients at Mayo Clinic


– 62% malignancy
– 34% infectious
– 8% autoimmune disease
– 6% unknown
Parikh et al, Mayo Clinic Proc 2014; 89:484

HLH-94 Protocol

357
Copyright © Harvard Medical School, 2018. All Rights Reserved.

Therapy for Adult HLH


• Treatment in adults generally follow HLH-94
- Exception: MAS; usually respond to steroids + IST alone
- Some protocols modify the dose of etoposide in adults
- We generally use tacrolimus rather than CsA for less renal
toxicity
• Adults do less well than children
– May be delay in diagnosis and therapy
– May be more complex etiology
– Affected by much higher rate of associated malignancy

Treatment/outcome of adult HLH

Schram et al, BrJHaematol 2015; in press

358
Copyright © Harvard Medical School, 2018. All Rights Reserved.

Salvage Therapy for Adult HLH


1) Alemtuzumab (anti-CD52):
– Retrospective study of 22 pts (all ages) receiving alemtuzumab after prior
induction therapy
– 77% of pts made it to SCT
– Long term probability of survival estimated at 64%
Marsh RA et al, Pediatr Blood Cancer, 2013
2) DEP regimen:
– Multi-center, prospective study of Doxil/Etoposide/Methylpred for adult
refractory HLH
– 29 lymphoma, 22 EBV, 4 FHLH, 8 unknown
– CR in 27%, PR in 49%; 29/48 with CR/PR survived to chemo or SCT
Wang Y et al, Blood, 2015
3) Emapalumab (Novimmune, NI-0501)
– Anti IFNγ antibody
– Highly successful treatment of children with relapsed/refractory HLH with 75%
reaching transplant
– Awaiting trials in adults

HSCT for Adult HLH


• Lack of large prospective studies
• CIBMTR Query:
– 47 adults with HSCT from 2001-2012
• 37 with FHLH or acquired HLH
– Mostly MRD/MURD (1 haplo, 1 CBT)
– 56% with MAC; 44% with RIC
– Estimated 2 yr OS was 57%

Nikiforow S et al, BBMT, 2014:S243

359
Copyright © Harvard Medical School, 2018. All Rights Reserved.

Principles of Therapy for HLH

(1) Combination of immunosuppressive and cytotoxic


therapy to control hyperinflammatory state, plus
(2) disease-specific therapy, followed by
(3) Allo-SCT for familial or recurrent HLH

Schram, Berliner, Blood, 2015

Take Home Points: Case 3


• Diagnosis of HLH in adults is still based on pediatric
diagnostic criteria
• Etiology of HLH in adults is more skewed toward
malignancy-associated disease
• Treatment of HLH in adults still parallels the approach
to pediatric patients. There is increasing evidence that
most patients should undergo HSCT
• MOST IMPORTANT: It is more common than you think;
if you don’t consider the diagnosis, you won’t make it!

360
Copyright © Harvard Medical School, 2018. All Rights Reserved.

DISCLOSURES

Nothing to disclose

QUESTION 1
A 43-year old man with no past medical history presents to the
hospital with a two month history of fevers and night sweats.
Repeated cultures by his PCP have been negative, and despite three
admissions to the hospital, his illness has evaded diagnosis. He has
lost 20 pounds, and he has been unable to work for 6 weeks.
Physical examination reveals a cachectic appearing young man with a
palpable liver and spleen. He has no adenopathy. Laboratory
studies reveal pancytopenia with an ANC of 726 and a platelet
count of 15,000, elevated transaminases, and diffuse infiltrates on
chest CT. He is begun on empiric antibiotics. Which of the following
tests is LEAST likely to be helpful in making a diagnosis:

A. Bone marrow aspirate and biopsy


B. Liver biopsy
C. Serum soluble CD25
D. Serum ferritin

361
Copyright © Harvard Medical School, 2018. All Rights Reserved.

QUESTION 1
Which of the following tests is LEAST likely to be helpful in making a
diagnosis:
A. Bone marrow aspirate and biopsy
B. Liver biopsy
C. Serum soluble CD25
D. Serum ferritin

ANSWER: B
This patient almost certainly has hemophagocytic
lymphohistiocytosis (HLH). He has pancytopenia,
splenomegaly, and fever, with no obvious infectious source.
Other diagnostic features that would support the diagnosis
would be the finding of hemophagocytosis on bone marrow,
an elevated serum ferritin, an elevated soluble CD25, elevated
triglycerides, and low fibrinogen. A liver biopsy, while it might
show hemophagocytosis, is likely to be dangerous in someone
with this low a platelet count, and is not considered a
particularly helpful diagnostic procedure.

QUESTION 2
A 60 year old man undergoes coronary artery bypass grafting. He
has a history of chronic atrial fibrillation treated with warfarin.
Following surgery he is placed on a heparin bridge and restarted on
his home dose of warfarin. On the sixth postoperative day, his INR is
2.5, his heparin is stopped, and he is preparing for discharge.
However, he complains of R leg pain and swelling and is found to
have a deep vein thrombosis in the R superficial femoral vein. His
platelet count is noted to have fallen from 240K on admission to
120K. Most appropriate therapy is:

A. Restart heparin as he is a warfarin failure


B. Start low molecular weight heparin and continue the
warfarin with a longer period of bridging
C. Start a direct thrombin inhibitor
D. Give 5 mg IV Vitamin K and start a direct thrombin inhibitor

362
Copyright © Harvard Medical School, 2018. All Rights Reserved.

QUESTION 2
Most appropriate therapy is:
A. Restart heparin as he is a warfarin failure
B. Start low molecular weight heparin and continue the warfarin
with a longer period of bridging
C. Start a direct thrombin inhibitor
D. Give 5 mg IV Vitamin K and start a direct thrombin inhibitor

ANSWER: D
The patient has heparin induced thrombocytopenia with
thrombosis (HITT). Even though he is off heparin, patients
with untreated HIT can have thrombotic complications for up
to 6 weeks after discontinuing heparin. His warfarin should be
discontinued until after he has been treated with a direct
thrombin inhibitor (DTI) and his platelet count has recovered;
furthermore, the warfarin should be reversed to prevent
warfarin skin necrosis and to prevent undertreatment with
the DTI. He can then be restarted on warfarin while remaining
on the DTI until he has a therapeutic INR.

REFERENCES
George JN, Al-Nouri ZL. Diagnostic and therapeutic challenges in the
thrombotic thrombocytopenic purpura and hemolytic uremic syndromes.
Hematology / the Education Program of the American Society of
Hematology American Society of Hematology Education Program
2012;2012:604-9.
Legendre CM, Licht C, Muus P, et al. Terminal complement inhibitor
eculizumab in atypical hemolytic-uremic syndrome. The New England
journal of medicine 2013;368:2169-81.
Schram AM, Berliner N. How I treat hemophagocytic lymphohistiocytosis in
the adult patient. Blood 2015;125:2908-14
Warkentin TE. Think of HIT. Hematology / the Education Program of the
American Society of Hematology American Society of Hematology
Education Program 2006:408-14.
Wei Y, Ji XB, Wang YW, et al. High-dose dexamethasone vs prednisone for
treatment of adult immune thrombocytopenia: a prospective multicenter
randomized trial. Blood 2016;127:296-302.

363
Copyright © Harvard Medical School, 2018. All Rights Reserved.

Board Review in Hematology

Nathan T. Connell, M.D., M.P.H.


Associate Physician
Hematology Division, Department of Medicine
Brigham and Women’s Hospital

Assistant Professor of Medicine


Harvard Medical School

Disclosures
• No disclosures

364
Copyright © Harvard Medical School, 2018. All Rights Reserved.

Question 1

A 35 year-old woman comes to the emergency room


complaining of exertional dyspnea over the past several
days. Her past medical history is unremarkable. She
takes no medications and denies any other systemic
symptoms. She is afebrile and her vital signs are stable.
Physical examination is only remarkable for a few small
ecchymoses on the upper and lower extremities, mild
scleral icterus, and a grade I/VI holosystolic murmur.

Question 1

Laboratory studies reveal:


White blood cell count 9,600/mm3 (4,000-10,000)
Hematocrit 23% (36-48)
Platelets 36,000/mm3 (150,000-450,000)
PT 12 s (11-13)
PTT 26 s (22-34)
Fibrinogen 450 mg/dL (200-400)
Creatinine 0.8 mg/dL (0.7-1.3)
LDH 1535 (107-231)
Peripheral blood smear reveals decreased platelets and moderate
schistocytes.

365
Copyright © Harvard Medical School, 2018. All Rights Reserved.

Question 1

Which of the following is the appropriate next step:


A. Send serum toxicology panel and stool studies for
E. coli 0157:H7

B. Initiate therapeutic plasma exchange as soon as


possible

C. Observation for now, initiate plasma exchange if


the platelet count < 20,000/mm3

D. Observation for now, initiate plasma exchange if


the patient becomes febrile or the creatinine rises

Question 1

What are the key clinical data provided?

Thrombocytopenia, anemia, and schistocytes

366
Copyright © Harvard Medical School, 2018. All Rights Reserved.

Question 1

The following therapy is now most appropriate:


A. Send serum toxicology panel and stool studies for
E. coli 0157:H7

B. Initiate therapeutic plasma exchange as soon


as possible

C. Observation for now, initiate plasma exchange if


the platelet count < 20,000/mm3

D. Observation for now, initiate plasma exchange if


the patient becomes febrile or the creatinine rises

TTP smear: schistocytes

367
Copyright © Harvard Medical School, 2018. All Rights Reserved.

Diagnosis of TTP

Two major characteristics of the classic pentad


are enough to diagnose TTP:
Microangiopathic hemolytic anemia
Thrombocytopenia
Others are secondary manifestations of the basic
pathophysiology and may or not be present
Fever
Neurologic symptoms
Renal Insufficiency/Failure

Pathophysiology of TTP: ADAMTS13

Moake, J. L. N Engl J Med 2002;347:589-600

368
Copyright © Harvard Medical School, 2018. All Rights Reserved.

Course of TTP
About 90% of cases of TTP improve with
plasma exchange
About 30% of these patients relapse, may
respond to another course of plasma
exchange
About 10% of patients have refractory
disease after a prolonged course of
pheresis – consider rituximab, splenectomy,
cytotoxic agents

Question 2

A 54 year-old man of Greek ancestry is seen for follow-up


3 days after completing treatment for an upper respiratory
tract infection with a course of
trimethoprim/sulfamethoxazole. On review he notes that
during the past week his cough has improved, although
he now feels somewhat more short of breath with exertion
and is more fatigued than one week ago.

Laboratory studies reveal:


White blood cell count 5,860/mm3 (4,000-10,000)
Hematocrit 28% (36-48)
Reticulocyte count 11% (1-1.5%)
MCV 101 fL (80-95)
Platelets 3
175,000/mm (150,000-
450,000)

369
Copyright © Harvard Medical School, 2018. All Rights Reserved.

Question 2

The most appropriate course of action at this time is:

A. Obtain folate and vitamin B12 levels

B. Observation, return visit in a few weeks for further


laboratory studies

C. Test for glucose-6-phosphate dehydrogenase


deficiency

D. Bone marrow aspirate and biopsy

Question 2

What are the key clinical data


provided?
Anemia, with reticulocytosis, following a
course of antibiotics, in a man of Greek
ancestry

370
Copyright © Harvard Medical School, 2018. All Rights Reserved.

Question 2

The most appropriate course of action at this time is:

A. Obtain folate and vitamin B12 levels

B. Observation, return visit in a few weeks for further


laboratory studies

C. Test for glucose-6-phosphate dehydrogenase


deficiency

D. Bone marrow aspirate and biopsy

Abnormal G-6-PD Variants

A- isoform
G6PD activity declines due to enzyme instability
during the red cell life-span
present in 10% of African-American males
G6PD levels in reticulocytes are normal
May require Heinz Body identification to diagnose
during acute hemolytic episode
Mediterranean Isoform
essentially no G6PD activity in RBC
present in 5% of Mediterranean people
Can always be diagnosed

371
Copyright © Harvard Medical School, 2018. All Rights Reserved.

Heinz body prep

Heinz bodies: oxidized, denatured precipitates of hemoglobin

Oxidant Stressors in G-6-PD Deficiency


• Antibacterials • Miscellaneous
– Dapsone – Doxorubicin
– Nalidixic acid – Methylene blue
– Nitrofurantoin – Phenazopyridine
– Sulfamethoxasole (PYRIDIUM)
– Sulfapyridine – Phenylhydrazine
• Antimalarials – Probenacid
– Primaquine • Environmental/Food
– Pamaquine – Fava beans
– Naphthalene (moth
balls)
– Toluene blue

372
Copyright © Harvard Medical School, 2018. All Rights Reserved.

Physiology of G6PD Deficiency

Toxicity of Oxidative Stress in G6PD Deficiency

2H2O H2O2 Hemoglobin

Sulf-hemoglobin
GSSG 2GSH

Heinz bodies
NADPH NADP

6-PG G6P Hemolysis


G6PD

Question 3

A 35 year-old woman originally from the Dominican


Republic comes to clinic for her first visit. She has no
significant medical problems and has had two children.
After the birth of her second child three years ago she was
told to take iron tablets twice daily. Aside from an oral
contraceptive, this is her only medication.
Laboratory studies reveal:

White blood cell count 4,600/mm3 (4,000-10,000)


Hematocrit 35% (36-48)
MCV 66 fL (80-95)
Platelets 3
256,000/mm (150,000-450,000)
Fe 150 µg/dL (40-159)
TIBC 275 µg/dL (250-400)

373
Copyright © Harvard Medical School, 2018. All Rights Reserved.

Question 3

The most appropriate next steps are:

A. Phlebotomy for hemochromatosis


B. Continue current iron therapy, initiate work-
up for chronic inflammatory process
C. Switch therapy from oral iron sulfate to
intravenous ferric gluconate
D. Discontinue iron therapy, send ferritin and
hemoglobin electrophoresis

Question 3

Key Laboratory studies reveal:

Hematocrit 35% (36-48)

MCV 66 fL (80-95)

Fe 150 µg/dL (40-159)


TIBC 275 µg/dL (250-400)

374
Copyright © Harvard Medical School, 2018. All Rights Reserved.

Question 3

The most appropriate next steps are:

A. Phlebotomy for hemochromatosis


B. Continue current iron therapy, initiate work-
up for chronic inflammatory process
C. Switch therapy from oral iron sulfate to
intravenous ferric gluconate
D. Discontinue iron therapy, send ferritin and
hemoglobin electrophoresis

Smear: thalassemia

Hypochromia, microcytosis

375
Copyright © Harvard Medical School, 2018. All Rights Reserved.

Question 4

A 74 year-old man with diabetes mellitus controlled with an


oral agent presents for routine follow-up. His other medical
problems include hypertension, for which he takes an ACE
inhibitor, and benign prostatic hypertrophy. On review of his
records you note that his hematocrit has been gradually
declining over the past three years. On his visit today
laboratories reveal:

Hematocrit 28% (36-48)


MCV 84 fL (80-95)
Platelets 340,000/mm3 (150,000-450,000)
BUN 35 mg/dL (9-25)
Creatinine 1.9 mg/dL (0.7-1.3)
LDH 230 (107-231)

Question 4

The most likely etiology of his anemia is:

A. Combined iron and B12 deficiency

B. Medication effect from the ACE inhibitor

C. Erythropoietin deficiency

D. Anemia due to marrow replacement by


metastatic prostate cancer

376
Copyright © Harvard Medical School, 2018. All Rights Reserved.

Question 4

CLINICAL SCENARIO

Progressive anemia, with a high creatinine

Question 4

The most likely etiology of his anemia is:

A. Combined iron and B12 deficiency

B. Medication effect from the ACE inhibitor

C. Erythropoietin deficiency

D. Anemia due to marrow replacement by


metastatic prostate cancer

377
Copyright © Harvard Medical School, 2018. All Rights Reserved.

Regulation of Erythropoietin

Renal/Hepatic Epo production

Hypoxia

Iron
Folate
B12

Increased Red Cell Production

Anemia of Renal Disease


Decline in renal function is associated with decreased
erythropoietin production
Anemic patients with diabetes mellitus and even modestly
abnormal renal function are often erythropoietin deficient
The normal range for erythropoietin values is for patients
who are not anemic
Anemia of any significant degree should lead to a rise in the
erythropoietin level out of the normal range
normal response to moderately severe anemia is a rise in
the epo level

378
Copyright © Harvard Medical School, 2018. All Rights Reserved.

Question 5

A 23 year-old man with sickle cell anemia is admitted for


management of pneumonia. He presented with a two-day
history of a dry non productive cough, fever to 101°F and
was found to have a right lower lobe infiltrate. On admission
his oxygen saturation was 94% on room air, and he was not
short of breath. He is placed on cefuroxime and given IV
hydration along with 1 unit of red blood cell transfusion.

Question 5

CBC on admission:
White blood cell count 18,000/mm3 (4,000-10,000)
Hematocrit 21% (36-48)
Platelets 247,000/mm3 (150,000-
450,000)

One day later he complains of increasing shortness of breath


and is found to have an oxygen saturation of 86% on room
air. Chest radiograph reveals bilateral lower lobe opacities.

379
Copyright © Harvard Medical School, 2018. All Rights Reserved.

Question 5

In addition to supplemental oxygen, the


appropriate next step is to:
A. Obtain V/Q scan
B. Administer furosemide
C. Transfuse RBCs
D. Add coverage for atypical organisms and
transfuse RBCs
E. Add coverage for atypical organisms, perform
exchange transfusion

Acute Chest Syndrome

Clinical syndrome of fever, hypoxia, and pulmonary


infiltrates
Often associated with infection with atypical
organisms such as chlamydia or mycoplasma
Blood transfusion-simple or exchange- may be life-
saving
Exchange transfusion:
demonstrated to improve outcome in acute chest syndrome
in adults
Indications for exchange: stroke, acute chest syndrome,
priapism

380
Copyright © Harvard Medical School, 2018. All Rights Reserved.

Question 5

In addition to supplemental oxygen, the


appropriate next step is to:

A. Obtain V/Q scan


B. Administer furosemide
C. Transfuse RBCs
D. Add coverage for atypical organisms and
transfuse RBCs
E. Add coverage for atypical organisms, perform
exchange transfusion

Question 6

A 76 year-old woman on warfarin for chronic atrial fibrillation


normally anticoagulated to an INR of 2.5 is found to have an INR
of 5.8 on routine testing. She is otherwise asymptomatic. The
most appropriate next step is:

A. Decrease dose of warfarin by 50%


B. Hold warfarin, administer 1 mg vitamin K subcutaneously
C. Hold warfarin, administer 2.5 mg vitamin K orally
D. Hold warfarin, recheck INR in 1 to 2 days prior to
restarting therapy
E. Hold warfarin for one day, restart at 50% of previous dose
the next day

381
Copyright © Harvard Medical School, 2018. All Rights Reserved.

Question 6

A 76 year-old woman on warfarin for chronic atrial fibrillation


normally anticoagulated to an INR of 2.5 is found to have an INR
of 5.8 on routine testing. She is otherwise asymptomatic. The
most appropriate next step is:

A. Decrease dose of warfarin by 50%


B. Hold warfarin, administer 1 mg vitamin K subcutaneously
C. Hold warfarin, administer 2.5 mg vitamin K orally
D. Hold warfarin, recheck INR in 1 to 2 days prior to
restarting therapy
E. Hold warfarin for one day, restart at 50% of previous dose
the next day

Warfarin (COUMADIN, JANTOVEN)

• Inhibits Vitamin K dependent gamma- carboxylation of


coagulation Factors II, VII, IX, and X

• Half life is 40 hours (highly variable)

• Duration of effect is 2-5 days

382
Copyright © Harvard Medical School, 2018. All Rights Reserved.

Non-urgent warfarin reversal


Administration of vitamin K

Oral:
Poorly bioavailable in some patients
Takes 12-24 hours to take effect

SC:
Variable absorption (not recommended)

IV:
Takes effect in 4 to 6 hours
Anaphylactic risk is very small

Urgent Warfarin Reversal


Prothrombin complex concentrate 4-factor
(US: KCENTRA; Canada: BERIPLEX and OCTAPLEX)

Indicated for acute major bleeding or urgent surgery

Dosing is based on the INR

Repeat dosing not indicated.


Administer Vitamin K concurrently

383
Copyright © Harvard Medical School, 2018. All Rights Reserved.

Direct Oral Anticoaguants (DOACs)

Dabigatran (PRADAXA)

Rivaroxaban (XARELTO)

Apixaban (ELIQUIS)

Edoxaban (SAVAYSA)

Reversing DOAC-Associated Bleeding


FFP: No benefit
PCCs Possible benefit
Factor VIIa Possible benefit

Idarucizumab Dabigatran (PRADAXA)


(PRAXBIND)

Andexanet alfa Direct Xa inhibitors


(ANDEXXA)

384
Copyright © Harvard Medical School, 2018. All Rights Reserved.

Question 7

A 24 year-old Caucasian female is found to


have a right popliteal deep venous thrombosis
while taking oral contraceptives.
Of the following heritable conditions, which is
most likely to be found on diagnostic evaluation:

A. Antithrombin III deficiency


B. Protein C deficiency
C. Homocysteinemia
D. Factor V Leiden
E. Prothrombin gene mutation (G20210A)

Question 7

A 24 year-old Caucasian female is found to


have a right popliteal deep venous thrombosis
while taking oral contraceptives.
Of the following heritable conditions, which is
most likely to be found on diagnostic evaluation:

A. Antithrombin III deficiency


B. Protein C deficiency
C. Homocysteinemia
D. Factor V Leiden
E. Prothrombin gene mutation (G20210A)

385
Copyright © Harvard Medical School, 2018. All Rights Reserved.

Common Inherited Thrombophilias


Factor V Leiden
G1691A mutation (arg to glu amino acid 506)
Prevents inactivation of factor V
Present in 4-6% of Caucasian population, 0.05% of
Asians and Africans
Prothrombin Gene Mutation
G20210A mutation (3’-untranslated region)
Leads to elevated levels of prothrombin
Present in 2-3% of Caucasians, 0.0% of Asians and
Africans

Other Inherited Causes of Thrombophilia

Rare (<1% of population combined)


Antithrombin deficiency
Protein C deficiency
Protein S deficiency
Very Rare
Dysfibrinogenemia
Homozygous homocystinuria
Possibly Inherited
Elevated factor VIII, IX, XI levels

386
Copyright © Harvard Medical School, 2018. All Rights Reserved.

Clinical Frequency of Thrombophilia

In patients with VTE who are:


less than age 50 years
have a family history of thrombosis
a history of recurrent events
no acquired risk factors except for pregnancy or oral
contraceptive:

Factor V Leiden 40%


Prothrombin Gene Mutation 16%
Protein C, S, or Antithrombin 13%

Question 8

Which of the following statements is true regarding oral


replacement with Vitamin B12?

A. It is only effective in patients who possess intrinsic


factor
B. Since it costs more than parenteral replacement, it
should be discouraged
C. It is generally effective, though requires monitoring
for compliance
D. When used in appropriate situations, the dose and
frequency are similar to parenteral B12
E. Methylmalonate and homocysteine levels cannot
be used for the monitoring of therapy

387
Copyright © Harvard Medical School, 2018. All Rights Reserved.

Question 8

Which of the following statements is true regarding oral


replacement with Vitamin B12?

A. It is only effective in patients who possess intrinsic


factor
B. Since it costs more than parenteral replacement, it
should be discouraged
C. It is generally effective, though requires monitoring
for compliance
D. When used in appropriate situations, the dose and
frequency are similar to parenteral B12
E. Methylmalonate and homocysteine levels cannot
be used for the monitoring of therapy

Cobalamin (Vitamin B12) Deficiency


Diagnosis:
Levels of less than 200 pg/mL diagnostic
homocysteine and methylmalonate levels can
be helpful in such situations (both are elevated
in B12 deficiency)
Clinical Features:
Hematopoiesis: pancytopenia
Neurological: peripheral neuropathy>posterior
column > dementia

388
Copyright © Harvard Medical School, 2018. All Rights Reserved.

Cobalamin (Vitamin B12) Deficiency


Pathophysiology:
Malabsorption
Intrinsic factor deficiency: gastric bypass
Resection of distal ilieum; celiac disease, ileitis
Competition from parasites: blind loop,
intestinal divericulum
Dietary deficiency is extremely rare
Treatment
Parenteral: 1000 mg/month IM; no need to
monitor
Oral: daily, absorbed by mass action

Folic acid deficiency


Clinical Features:
No neurologic sequelae
Hematopoietic and GI manifestations same as
cobalamin
Pathophysiology:
Decreased intake:
poor diet, alcoholism, infancy
Impaired absorption: gastric by-pass; sprue, etc.
Increased requirements:
growth spurts: infancy, adolescence
pregnancy
Hemolysis
marrow hyperplasia, bulky tumors

389
Copyright © Harvard Medical School, 2018. All Rights Reserved.

Question 9

An asymptomatic 35 year-old woman is seen in the office and is found


to have the following CBC:
White blood cell count 6,200/mm3 (4,000-10,000)
Hematocrit 36% (36-48)
Platelets 758,000/mm3 (150,000-450,000)

Which of the following statements is true:

A. Therapy with hydroxyurea is indicated


B. Warfarin therapy is indicated to prevent thrombotic
complications
C. The possibility of iron deficiency, an inflammatory state or
CML should be investigated
D. The patient has a very high risk of thrombosis or
hemorrhage during the next five years

Question 9

An asymptomatic 35 year-old woman is seen in the office and is found


to have the following CBC:
White blood cell count 6,200/mm3 (4,000-10,000)
Hematocrit 36% (36-48)
Platelets 758,000/mm3 (150,000-450,000)

Which of the following statements is true:

A. Therapy with hydroxyurea is indicated


B. Warfarin therapy is indicated to prevent thrombotic
complications
C. The possibility of iron deficiency, an inflammatory state or
CML should be investigated
D. The patient has a very high risk of thrombosis or
hemorrhage during the next five years

390
Copyright © Harvard Medical School, 2018. All Rights Reserved.

Thrombocytosis

Reactive Thrombocytosis
Infection
Inflammation
Rebound
Iron Deficiency
Myeloproliferative Disorders
Myelodysplasia (5q- syndrome)

Smear: Myeloproliferative disorders

CML: neutrophilia, CML: increased basophils


myeloid precursors

Essential thrombocythemia

391
Copyright © Harvard Medical School, 2018. All Rights Reserved.

Essential thrombocytosis

No platelet lowering treatment necessary in


asymptomatic individuals, age < 60, with platelet
counts of < 1,500,000/mm3

Older individuals, those with a history of


complications, and those with higher counts may
benefit from hydroxyurea

Low-dose aspirin (81 mg daily) is generally


recommended

Question 10
A 67 year-old man with a history of unstable angina is admitted to the
coronary care unit for further management of chest pain. His complete
blood count is normal on admission and he is started on unfractionated
heparin. On hospital day 5 his platelet count is noted have drifted down to
80,000/mm3. An ELISA assay for antibodies to the heparin-PF4 complex is
sent.

What is the appropriate next step:

A. Continue unfractionated heparin pending the ELISA results


B. Switch to a low molecular weight heparin pending the ELISA
results
C. Discontinue unfractionated heparin and begin anticoagulation with
a direct thrombin inhibitor pending ELISA results
D. Discontinue unfractionated heparin and begin warfarin to target an
INR 2-3 pending the ELISA results

392
Copyright © Harvard Medical School, 2018. All Rights Reserved.

Question 10
A 67 year-old man with a history of unstable angina is admitted to the
coronary care unit for further management of chest pain. His complete
blood count is normal on admission and he is started on unfractionated
heparin. On hospital day 5 his platelet count is noted have drifted down to
80,000/mm3. An ELISA assay for antibodies to the heparin-PF4 complex is
sent.

What is the appropriate next step:

A. Continue unfractionated heparin pending the ELISA results


B. Switch to a low molecular weight heparin pending the ELISA
results
C. Discontinue unfractionated heparin and begin anticoagulation with
a direct thrombin inhibitor pending ELISA results
D. Discontinue unfractionated heparin and begin warfarin to target an
INR 2-3 pending the ELISA results

Heparin-Induced Thrombocytopenia
Relative fall to below 50% of the peak platelet
count

Rarely if ever occurs within the first five days of


heparin unless there has been a recent pre-
exposure to heparin

Less common with LMWH, but once a patient has


HIT, antibodies may be cross-reactive

There is no absolute platelet number that is


diagnostic of HIT, and is more important to consider
in the appropriate clinical setting

393
Copyright © Harvard Medical School, 2018. All Rights Reserved.

HIT MANAGEMENT

STOP HEPARIN

BEGIN A DIRECT THROMBIN


INHIBITOR

HIT MANAGEMENT

If patient was on warfarin, administer Vitamin K

When platelet count has returned to normal overlap


warfarin and DTI for at least 5 days

Continue the overlap at least 24-48 hours with


therapeutic INR (DTI may artificially elevate the INR)

Continue warfarin for 3-6 months in the setting of


thrombosis

Duration in the absence of thrombosis…no data

394
Copyright © Harvard Medical School, 2018. All Rights Reserved.

Heparin-Induced Thrombocytopenia (HIT)


Pathophysiology:
Anti-PF4/heparin antibodies:
Activation of platelets
formation of platelet
microparticles
neutralization of heparin
increased thrombin generation
• Activation of endothelium and
monocytes
cell surface TF expression
Indirect through platelet
activation.
Can lead to arterial as well as
venous thrombosis

Semin Thromb Hemost 2004; 30: 273-283

Question 11

A 28 year-old woman is seen for preconception counseling. Her past


medical history is notable for significant bleeding after extraction of her
wisdom teeth. She takes no medications. Her family history is notable for
the fact that her mother required blood transfusions several days after the
birth of each of her two children. Her sister also had major bleeding
several days after the birth of her child.

An appropriate evaluation at this time would include:

A. Thrombin and reptilase times


B. Von Willebrand antigen level, ristocetin cofactor levels, and factor
VIII activity level
C. Factor XIII screen
D. Euglobulin clot lysis time
E. Factor XI activity level

395
Copyright © Harvard Medical School, 2018. All Rights Reserved.

Question 11

A 28 year-old woman is seen for preconception counseling. Her past


medical history is notable for significant bleeding after extraction of her
wisdom teeth. She takes no medications. Her family history is notable for
the fact that her mother required blood transfusions several days after the
birth of each of her two children. Her sister also had major bleeding
several days after the birth of her child.

An appropriate evaluation at this time would include:

A. Thrombin and reptilase times


B. Von Willebrand antigen level, ristocetin cofactor levels, and factor
VIII activity level
C. Factor XIII screen
D. Euglobulin clot lysis time
E. Factor XI activity level

Von Willebrand Disease Screening Panel

von Willebrand Antigen

Factor VIII activity

Ristocetin cofactor activity level

Note: Those with Type O blood have a


VW:Ag level ~25% lower

396
Copyright © Harvard Medical School, 2018. All Rights Reserved.

Von Willebrand Factor


A 270 kD monomer that assembles into multimers ranging
from 600 kD to 20,000 kD in size

vWF binds to subendothelial collagen and to platelets via


GPIb and GPIIb/IIIa receptors

Facilitates platelet-platelet interactions

Acts as a carrier for Factor VIII, protecting it from proteolysis


and prolonging its half life

The ristocetin co-factor assay measures the functional


activity of vWF binding to platelets: causes a conformational
change in vWF so that it can bind to the GPIb on platelets

vWD Genetic Mutations

Type 1: partial vWF deficiency


Type 2: qualitative defects in vWF
2A: decreased platelet dependent
function
2B: increased affinity of vWF for platelets
2N: defective factor VIII binding site
Type 3: complete vWF deficiency

397
Copyright © Harvard Medical School, 2018. All Rights Reserved.

Question 12

A 67 year-old man with a history of hypertension is seen for routine physical


examination. Physical exam is notable for a ruddy complexion and a grade
II/VI systolic flow murmur. CBC is notable for:

White blood cell count 9,800/mm3 (4,000-10,000)


Hematocrit 58% (36-48)
Platelets 430,000/mm3 (150,000-450,000)

The serum erythropoietin level is low. Which of the following is the most
appropriate next step in evaluation:

A. Serum iron panel with HFE gene mutation analysis


B. JAK2 V617F mutation analysis
C. Polysomnography (sleep study)
D. Hemoglobin electrophoresis with p50 dissociation curve

Question 12

A 67 year-old man with a history of hypertension is seen for routine physical


examination. Physical exam is notable for a ruddy complexion and a grade
II/VI systolic flow murmur. CBC is notable for:

White blood cell count 9,800/mm3 (4,000-10,000)


Hematocrit 58% (36-48)
Platelets 430,000/mm3 (150,000-450,000)

The serum erythropoietin level is low. Which of the following is the most
appropriate next step in evaluation:

A. Serum iron panel with HFE gene mutation analysis


B. JAK2 V617F mutation analysis
C. Polysomnography (sleep study)
D. Hemoglobin electrophoresis with p50 dissociation curve

398
Copyright © Harvard Medical School, 2018. All Rights Reserved.

Causes of an Elevated Hematocrit

Polycythemia Vera
~95% of cases have a JAK2 V617F mutation
Relative or spurious erythrocytosis
Absolute Erythrocytosis from Hypoxia
Renal Disease
Tumors
Hemoglobinopathies
Epo receptor mutations

Question 13
A 28-year-old man with no significant past medical history
presents with several weeks of worsening fatigue and darkened
urine. On exam, he is tachycardic (130 beats/min), tachypneic
(24/minute), and jaundiced. His CBC shows:

White blood cell count 9,800/mm3 (4,000-10,000)


Hematocrit 12% (36-48)
Platelets 430,000/mm3 (150,000-450,000)
MCV 106 fL (80-100)

The peripheral blood smear shows numerous spherocytes and


his Direct Coombs (DAT) is positive for IgG. The patient’s
antibody screen is reactive to all cells tested. The blood bank
says that it may be 4-6 hours before a fully compatible cross-
match unit is available.

399
Copyright © Harvard Medical School, 2018. All Rights Reserved.

Question 13

In addition to supportive care with folic acid and initiation


of corticosteroids, which of the following is the next
appropriate step in management:

A. Transfusion of emergency release AB-negative red cell


units until fully crossmatch compatible red cell units
are available
B. Transfusion with the least incompatible type-specific
red cell units
C. Observation until fully compatible type-specific red
cells units are available
D. Red cell exchange transfusion with concurrent
plasmapheresis

Question 13

What are the key clinical data provided?

Immune-mediated hemolysis with hemodynamic


instability and difficulty with crossmatch

400
Copyright © Harvard Medical School, 2018. All Rights Reserved.

Autoimmune Hemolytic Anemia

Due to antibody mediated destruction of red


cells leading to shortened RBC survival
Compensation by the bone marrow (retics)
Corticosteroids
Decrease removal by reticuloendothalial system
Decrease antibody production
Supportive Care:
Folic Acid
Transfusion support

Question 13

In addition to supportive care with folic acid and initiation


of corticosteroids, which of the following is the next
appropriate step in management:

A. Transfusion of emergency release AB-negative red cell


units until fully crossmatch compatible red cell units
are available
B. Transfusion with the least incompatible type-specific
red cell units
C. Observation until fully compatible type-specific red
cells units are available
D. Red cell exchange transfusion with concurrent
plasmapheresis

401
Copyright © Harvard Medical School, 2018. All Rights Reserved.

Board Review Practice


Images
Part 1
2018
Ajay K. Singh, MBBS, FRCP
Brigham and Women’s Hospital
Harvard Medical School

Disclosures
• GSK - Consultant

402
Copyright © Harvard Medical School, 2018. All Rights Reserved.

Question 1
A 52 year old homeless man
on chronic dialysis with a
history of non-compliance
presents with chest pain. CXR
and EKG show the following.
The next step should be:
A. Administer predinisone 60
mg/day tapered over 1 month.
B. Emergency
pericardiocentesis and
initiate heparin-free daily
hemodialysis
C. Anticoagulate the patient
immediately with heparin
D. Administer colchicine
E. Administer vitamin B6

B. Emergency pericardiocentesis
and initiate heparin-free daily
hemodialysis
Uremic pericarditis w/ tamponade
• inflammation of the visceral and parietal
membranes of the pericardial sac.
• BUN is usually >60 mg/dL
• Risk factors: inadequate dialysis and/or fluid
overload .
• Fever and pleuritic chest pain - worse in the
recumbent position. pericardial rub is generally
audible, but is frequently transient.
•Signs of cardiac tamponade may be seen,
particularly in patients with rapid pericardial fluid
accumulation.
Electrical alternans Sinus Tachy w/ • EKG does not show the typical diffuse ST and T
beat to beat variation in QRS wave elevations observed with other causes of acute
appearance (best seen V2 and V4) pericarditis. This results from the lack of penetration
Source: Ary Goldberger, MD of the inflammatory cells into the myocardium
cmbi.bjmu.edu.cn

403
Copyright © Harvard Medical School, 2018. All Rights Reserved.

Question 2
A 68 year old woman has
a long history of
rheumatoid arthritis (RA).
Which one of the
following deformities is
not characteristically
seen in RA?

A. Boutonnière deformity
B. swan- neck deformity
C. Ulnar deviation of the
metacarpophalangeal
joints
D. Bouchard’s nodes
E. Mallett Finger

SOURCE: https://www.cedars-sinai.edu/Patients/Health-Conditions/Arthritis---Rheumatoid-
Arthritis-Osteoarthritis-and-Spinal-Arthritis.aspx

Question A 68 year old woman has


a long history of
rheumatoid arthritis (RA).
Which one of the
following deformities is
not characteristically
seen in RA?

A. Boutonnière deformity
B. Swan- neck deformity
C. Ulnar deviation of the
metacarpophalangeal
joints
D. Bouchard’s nodes
E. Mallett Finger

SOURCE: https://www.cedars-sinai.edu/Patients/Health-Conditions/Arthritis---Rheumatoid-
Arthritis-Osteoarthritis-and-Spinal-Arthritis.aspx

404
Copyright © Harvard Medical School, 2018. All Rights Reserved.

Osteoarthritis (OA) hand abnormalities

http://www.womens-health-
advice.com/photos/osteoarthritis.html

RA hand abnormalities

SOURCE:
http://morphopedics.wikidot.com/rheumatoid-
arthritis

405
Copyright © Harvard Medical School, 2018. All Rights Reserved.

Question 3
Serum levels of which one
of the following laboratory
tests would be expected to
be most abnormal in this
patient?

A. 17-hydroxyprogesterone
B. Angiotensin-converting
enzyme
Q: C. Anti-tissue
transglutaminase antibody
D. Prolactin
E. Vitamin B6

SOURCE:Graham McMahon and


http://emedicine.medscape.com/article/361490-
overview

Question
B. Angiotensin-converting
enzyme

Lupus pernio is a manifestation of


sarcoidosis that involves the
nasal bridge and cheeks. The
CXR shows bilateral hilar
lymphadenopathy. Serum levels
of the angiotensin-converting
Q:
enzyme are elevated in the
majority of patients with untreated
sarcoidosis.

SOURCE:Graham McMahon and


http://emedicine.medscape.com/article/361490-
overview

406
Copyright © Harvard Medical School, 2018. All Rights Reserved.

ACE Levels
• ACE levels may be elevated in ≈ 60% of patients at the time of
diagnosis.
• Non-caseating granulomas (NCGs) secrete ACE: the enzyme is
secreted by epithelioid cells at the periphery of the granulomas,
and the level is usually elevated in patients with “active”
sarcoidosis
• Serum ACE levels may correlate with total body granuloma load.
• Levels may be increased in fluid from bronchoalveolar lavage or
in cerebrospinal fluid.
Also in pts with miliary tuberculosis, silicosis, asbestosis, biliary cirrhosis,
leprosy, histoplasmosis, hepatitis, lymphoma, berylliosis, diabetic
retinopathy and hyperthyroidism.
• Sensitivity and specificity as a diagnostic test is limited (60% and
70%, respectively). There is no clear prognostic value.
• Serum ACE levels may decline in response to therapy.
• Decisions on treatment should not be based on the ACE level
alone.

Question 4
What is the most likely
diagnosis?

A. Amyloidosis
B. Celiac disease
C. Hypothyroidism
D. Kawasaki disease
E. Type 2 diabetes

Q:

Pastore L, et al, N Engl J Med 2007; 356:2547


http://www.nejm.org/doi/full/10.1056/NEJMc070
200#t=article

407
Copyright © Harvard Medical School, 2018. All Rights Reserved.

B. Celiac disease

Atrophic glossitis is a
typical manifestation of
celiac disease.

The tongue was the most


frequently affected site in
a series of 128 patients
with celiac disease who
were examined for oral
mucosal lesions and
symptoms, with 29.6% of
Pastore L, et al, N Engl J Med 2007; 356:2547 the patients describing
http://www.nejm.org/doi/full/10.1056/NEJMc070 soreness or a burning
200#t=article
sensation and 8.6%
having erythema or
atrophy. 1
1R.
Docimo, M. Costacurta, P. Maturo, L. Di Iorio, F.M. Paone. (2009) Malattia celiaca e
manifestazioni intraorali. Prevenzione & Assistenza Dentale 35, 26-33

Other Manifestations of Celiac Disease


in the Oral Cavity1
• Dental enamel defects with a various degree of advancement:
discolorations, horizontal groves and pits, and structural
destruction causing the change of the dental crown.
• Symmetric location of defects within all dentition sections, and
within the same anatomic groups of teeth (the most
frequently: incisors and first permanent molars), is specific for
celiac disease.
• Recurrent aphthae and other disorders of the oral mucosa
such as ulceration, erythema, atrophic glossitis, as well as
dryness and a burning sensation (particularly of the tongue)
• Delayed tooth eruption may also be a consequence of
alimentary deficiency in celiac disease.

1Krzywicka B, Herman K, Kowalczyk-Zając M, Pytrus T. Celiac disease and its


impact on the oral health status - review of the literature. Adv Clin Exp Med.
2014 Sep-Oct;23(5):675-81.

408
Copyright © Harvard Medical School, 2018. All Rights Reserved.

Leffler, DA et al Extraintestinal manifestations of coeliac disease


Nature Reviews Gastroenterology & Hepatology 12, 561–571 (2015)

Question 5
• This 53-year-old college
professor could no longer
lecture because her tongue
kept getting in the way. Her
tongue was enlarged and
had serrations, reflecting
imprints of her teeth. Her
upper-torso muscles were
grossly hypertrophied and
hard as wood
The most likely diagnosis is:
A. Acromegaly
B. Hypothyroidism
C. Amyloidosis
D. Pernicious anemia
E. An allergic reaction to
toothpaste
Source: http://cnx.org/content/m14953/latest/

409
Copyright © Harvard Medical School, 2018. All Rights Reserved.

Question
C. Amyloidosis

An enlarged, serrated tongue


suggests amyloidosis,
acromegaly, or hypothyroidism.
The “shoulder pad” sign,
however, is relatively specific for
amyloid disease. Thus, the
combination of an enlarged,
serrated tongue with the
"shoulder pad" sign is
pathognomonic of systemic
amyloidosis.

Source: http://cnx.org/content/m14953/latest/

Localized Amyloidosis

• Tongue is most frequently affected site in forms of


localised amyloidosis
• Tongue biopsy possess a highly diagnostic value for
amyloidosis.
• No consensus regarding the management of lingual
amyloidosis, although numerous therapies have
been proposed, including surgical excision and
pharmacological treatment. However lesions often
persist or recur.

Angiero F, et al Amyloid deposition in the tongue: clinical and histopathological


profile. Anticancer Res. 2010 Jul;30(7):3009-14.

410
Copyright © Harvard Medical School, 2018. All Rights Reserved.

Question 6
What is the diagnosis?

A. Central retinal artery


occlusion
B. Diabetic retinopathy
C. Ocular toxoplasmosis
D. Optic neuritis
E. Malignant
hypertension
Q:

Source: http://www.aoa.org/diabetic-retinopathy.xml

B. Diabetic retinopathy

The fundus photograph


shows findings consistent
with a diagnosis of
diabetic retinopathy.
Features
• Microaneurysms
• Dot and blot hemorrhages
• Flame-shaped hemorrhages
• Retinal edema and hard
exudates
• Cotton-wool spots
• Venous loops and venous
beading
• Intraretinal microvascular
abnormalities
• Macular edema

Source: http://www.aoa.org/diabetic-retinopathy.xml

411
Copyright © Harvard Medical School, 2018. All Rights Reserved.

Microaneurysms: The earliest clinical sign of diabetic retinopathy; these occur


secondary to capillary wall outpouching due to pericyte loss; they appear as small, red
dots in the superficial retinal layers

Dot and blot hemorrhages: Appear similar to microaneurysms if they are small; they
occur as microaneurysms rupture in the deeper layers of the retina, such as the inner
nuclear and outer plexiform layers

Flame-shaped hemorrhages: Splinter hemorrhages that occur in the more superficial


nerve fiber layer

Retinal edema and hard exudates: Caused by the breakdown of the blood-retina
barrier, allowing leakage of serum proteins, lipids, and protein from the vessels

Cotton-wool spots: Nerve fiber layer infarctions from occlusion of precapillary arterioles;
they are frequently bordered by microaneurysms and vascular hyperpermeability

Venous loops and venous beading: Frequently occur adjacent to areas of


nonperfusion - reflect increasing retinal ischemia, and their occurrence is the most
significant predictor of progression to proliferative diabetic retinopathy (PDR).

Intraretinal microvascular abnormalities: Remodeled capillary beds without


proliferative changes; can usually be found on the borders of the nonperfused retina
Macular edema: Leading cause of visual impairment in patients with

Source: Medscape.com/article/1225122-overview

Question 7
The urine sediment
shows what form of
crystals?

A. Oxalate crystals
B. Cystine crystals
C. Uric acid crystals
D. Struvite crystals
E. Acyclovir crystals

412
Copyright © Harvard Medical School, 2018. All Rights Reserved.

B. Cystine crystals

• The crystals are usually hexagonal,


translucent, white.
• Cystinuria is an autosomal recessive
disorder caused by mutations in the
SLC3A1 and SLC7A9 genes.
• These genes encode two parts of a
transporter protein in the kidneys.
•Under normal circumstances, this
protein allows certain amino acids,
including cysteine, to be reabsorbed into
the blood from the filtered fluid that will
become urine.
•As the levels of cystine in the urine
increase, the crystals typical of
cystinuria are able to form, resulting in
kidney stones.

Source: http://en.wikipedia.org/wiki/Cystinuria

Question 8
A 22-year old woman
presents with joint and
abdominal pain and a
rash. Rectal exam is
positive for occult
blood. Urine shows
hematuria. What is the
most likely diagnosis?

A. Renal cell cancer


B. Renal infarction
C. Kidney stone
D. Goodpasture’s
syndrome
E. Henoch-Schonlein
purpura

413
Copyright © Harvard Medical School, 2018. All Rights Reserved.

E. Henoch-Schonlein
purpura

The patient has palpable


purpura and hematuria. The
the red cells appear crenated -
- dysmorphic red cells. The
combination of the clinical
presentation and the hematuria
makes HSP nephritis the most
likely etiology.
HSP is an IgA-mediated, small-
vessel vasculitis that
predominantly affects children
but also is seen in adults. HSP
is a subset of necrotizing
vasculitis characterized by
fibrinoid destruction of blood
vessels and leukocytoclasis.

Glomerular Syndromes

• Nephritis: Hypertension, Azotemia,


proteinuria, hematuria, RBC casts /
dysmorphic RBCs
• Nephrosis: edema, proteinuria,
hypoalbuminemia, lipid abnormalities
• RPGN: rapid renal failure, crescents on
renal biopsy + nephritis
• Isolated urinary abnormalities:
hematuria / proteinuria

414
Copyright © Harvard Medical School, 2018. All Rights Reserved.

Pathology of IgAN/HSP nephritis

HSP nephritis
Clinical Features
Dermal Renal
Purple, nonblanching, 33% children, 63% adults
urticarial, purpuric papules Hematuria, macroscopic / microscopic
may become confluent Proteinuria
Bx: leukocytoclastic vasculitis Azotemia

GI
Abd pain (2/3rds of cases)
may precede rash Joints
Vomiting Arthralgias and periarticular edema (2/3)
Diarrhea knees, ankles, elbows, wrists
Periumbilical pain
Major complications (5%)
intussusception
bowel ischemia
necrosis

415
Copyright © Harvard Medical School, 2018. All Rights Reserved.

Question 9
What is the most likely
diagnosis?

A. Renal tubular acidosis


B. Primary
hypoparathyroidism
C. Familial hypocalciuric
hypercalcemia
D. Salicylate overdose
Q: E. Paget's disease

Slide courtesy of Graham McMahon, BWH

A. Renal tubular acidosis

The film reveals bilateral


Answer:
symmetric calcification of the
renal parenchyma, sparing
only the renal pelvis. This
patient had been diagnosed
with renal tubular acidosis at
9 years of age, but did not
undergo medical follow-up for
20 years. The other listed
choices are not common
causes of nephrocalcinosis.

416
Copyright © Harvard Medical School, 2018. All Rights Reserved.

Distal RTA
1. Classical form of RTA (described first)
2. Failure of alfa intercalated cells to secrete H+ and K+
3. Hypokalemia, hypocalecemia, hyperchloremia
4. Urinary stone formation (related to alkaline urine,
hypercalciuria and low urine citrate
5. Nephrocalcinosis
6. Bone demineralization 9rickets in children, osteomalacia in
adults)

Question 10 A 78-year-old male


presented with headache
of 4 weeks' duration.
Pain was excruciating,
constant and
predominantly over the
right hemicranium, with
the maximum being over
the right temple. There
was jaw and tongue
claudication. The patient
felt weak and there was
SOURCE:
https://www.google.com/search?q=temporal+arteritis&biw=132 low grade fever. He also
7&bih=661&source=lnms&tbm=isch&sa=X&sqi=2&ved=0ahUK complained of proximal
EwjdyPjXmufNAhUCDT4KHWfWD9kQ_AUIBigB#imgrc=1Boe
GCHqqtZAPM%3A
and axial joint arthralgias.
He had tenderness
tenderness over his
superficial temporal
artery and over his scalp.
SOURCE: http://www.ncbi.nlm.nih.gov/pmc/articles/PMC2771971/

417
Copyright © Harvard Medical School, 2018. All Rights Reserved.

A diagnosis of giant cell arteritis (GCA) was confirmed


by a superficial temporal artery (STA) biopsy. Which
one of the following is atypical of GCA?

A.) Jaw claudication


B.) Small-Medium vessel involvement
C.) Tongue claudication
D.) Proximal and axial joint arthralgias suggesting
polymyalgia rheumatica (PMR)
E.) Scalp tenderness

A diagnosis of giant cell arteritis (GCA) was confirmed


by a superficial temporal artery (STA) biopsy. Which
one of the following is atypical of GCA in this
presentation?

A.) Jaw claudication


B.) Small-Medium vessel involvement
C.) Tongue claudication
D.) Proximal and axial joint arthralgias suggesting
polymyalgia rheumatica (PMR)
E.) Scalp tenderness

418
Copyright © Harvard Medical School, 2018. All Rights Reserved.

• Most common vasculitis


GCA
• Elderly women
• Small, medium size arteries
• Cranial Arteritis: temporal, facial, ophthalmic arteries. Also aortic
arch (giant cell aortitis, uncommon)
• Throbbing headache that doesn’t respond to NSAIDs, tender
temporal area, firm & nodular temporal artery
• Facial pain, loss of taste, tongue pain/claudication, scalp
tenderness
• Jaw claudication
• Visual abnormailities: amaurosis fugax, blurred vision, double
vision, blindness
• 50% cases => Polymyalgia Rheumatica
• Necrotizing vasculitis with granulomas: fragementation of the
internal elastic lamina due to the presence of multinucleated Giant
Cells & intimal fribrosis.

GCA Treatment

Stone JH et al N Engl J Med 2017; 377:317-328July 27, 2017

N=251
Pred (26 w course) + tocilizumab (162 mg SC weekly/QOW)
vs.
Pred (26 w or 52 w course) +placebo
Outcome: Rate of remission
At 52 weeks: ≈56% remission in combination therapy group
versus 18% in the prednisone alone
More side-effects in the prednisone alone group

419
Copyright © Harvard Medical School, 2018. All Rights Reserved.

Question 11
This 26-year old patient
presented with a facial rash
and diffuse arthralgias. She is
not on any drug treatment.
Which one of the following
tests would confirm the
diagnosis?

A. ESR
B. CRP
C. Anti-ds DNA ab
D. Anti-microsomal antibody
measurement
E. Anti-RNP ab measurement

C. Anti-ds DNA ab
DDx
• SLE
• Pellagra
• Bloom syndrome (BLM)

Bloom = autosomal recessive


characterized by short stature and a
butterfly facial rash that develops shortly
after first exposure to sun. Other clinical
features include a high-pitched voice;
distinct facial features, such as a long,
narrow face, pigmentation (hypo/hyper),
moderate immune deficiency, sub-fertility
in females.

Pellagra most commonly caused by a


chronic lack of niacin (vitamin B3).
Characterized by 4 D’s: diarrhea,
dermatitis, dementia, and death
Source cure4lupus.org

420
Copyright © Harvard Medical School, 2018. All Rights Reserved.

Anti-ds DNA Antibodies


• Group of anti-nulcear Abs that target double stranded DNA
• Low sensitivity for SLE, high specificity (absence of the
antibodies does not rule out the disease)
• Anti-dsDNA antibodies are highly associated with lupus
nephritis
• Some patients with high titers of anti-dsDNA antibodies do not
develop renal disease -most likely because anti-dsDNA are a
heterogeneous population and some non-pathogenic.
• Patients with rheumatoid arthritis can develop anti-dsDNA
antibodies, however they are usually treatment related. Anti-
TNFα biological therapies, such as adalimumab, infliximab,
etanercept.
• Infection with viral pathogens can induce anti-dsDNA
antibodies transiently e.g., HIV, human parvovirus B19, BK
virus.

Question 12
A 46-year-old woman with
metastatic sarcoma who had
been treated with five cycles of
doxorubicin, ifosfamide, and
mesna chemotherapy
presented with two
symmetrical, horizontal white
lines on all of her fingernails
but not on her toenails. Which
Q: one of the following is the
most likely diagnosis?

SOURCE: Morrison-Bryant M, Gradon JD. A. Chronic renal failure


N Engl J Med. 2007 Aug 30;357(9):917 B. Iron deficiency
C. Graves' disease
D. Chemotherapy treatment
E. Psoriasis

421
Copyright © Harvard Medical School, 2018. All Rights Reserved.

D. Chemotherapy treatment

• A diagnosis of Muehrcke's lines related to chemotherapy


treatment.
• Muehrcke's lines are the two smooth white bands that run parallel
to the lunula across the width of the nail.
• The lines are nonpalpable and, unlike Beau's lines, do not indent
the nail itself.
• Muehrcke's lines are a nonspecific finding that may be associated
with periods of metabolic stress, which transiently impairs protein
synthesis. Muehrcke's lines are also caused by infections and
trauma.

Question 13
A 56-year-old woman with
hypertension presented
with a sudden onset
painless impaired vision in
the right eye. On the
previous day, she
had undergone cardiac
catheterization for
evaluation of hypertensive
Q: emergency. On physical
examination, the visual
acuity in the right eye was
20/100 with significant
visual-field defect in the
inferior temporal quadrant.

SOURCE: Meyer CH, Holz FG. Images in clinical medicine. Blurred vision after cardiac
catheterization. N Engl J Med. 2009 Dec 10;361(24):2366.

422
Copyright © Harvard Medical School, 2018. All Rights Reserved.

Q: What is the diagnosis? B. Cholesterol embolism

The image demonstrates


Answer: cholesterol emboli with surrounding
white lucency representing retinal
edema. Fluorescein angiography
confirmed occlusion of the
cilioretinal artery with nonperfusion
of the tissue bed in the
hypofluorescent areas. Cholesterol
emboli are crystals that are released
in the arterial bloodstream from
ulcerated or disrupted
atherosclerotic plaques and can be
an initial sign of vascular disease.

• Cholesterol embolism syndrome should be suspected in a


patient who develops worsening renal function, hypertension,
distal ischemia or acute multisystem dysfunction after an
invasive procedure.
• middle-aged to elderly Step wise rise in Scr
• men >women Bland urine classic
cholesterol embolus in a medium sized artery of the kidney

SOURCE: https://en.wikipedia.org/wiki/Cholesterol_embolism

423
Copyright © Harvard Medical School, 2018. All Rights Reserved.

Center for Gastrointestinal Motility

Esophageal Disorders

Walter W. Chan, MD, MPH

Director, Center for Gastrointestinal Motility


Division of Gastroenterology, Hepatology and Endoscopy
Brigham and Women’s Hospital
Assistant Professor of Medicine, Harvard Medical School

Disclosures
No relevant disclosure.

424
Copyright © Harvard Medical School, 2018. All Rights Reserved.

Outline
Clinical presentations of esophageal disorders
Diagnostic tests for esophageal and swallowing
symptoms
Esophageal disorders
– Gastroesophageal reflux disease and complications
– Esophageal motility disorders
– Eosinophilic esophagitis
– Other esophagitis (infectious, pill-induced)

CLINICAL PRESENTATION OF
ESOPHAGEAL DISORDERS

425
Copyright © Harvard Medical School, 2018. All Rights Reserved.

Clinical Presentation
Symptoms
– Dysphagia
• Mucosal inflammation, mechanical obstruction, dysmotility
• Solids only: suggests mechanical obstruction
• Solids + liquids: suggests dysmotility
– Odynophagia
• Inflammation (infectious, caustic), ulceration
– Chest pain
– Heartburn
– Regurgitation
– Others:
• Weight loss, cough, choking, hoarseness, dyspnea

Clinical Presentation
Physical Examination
– Often normal
– Signs of dehydration or malnutrition
• Hypotension, tachycardia, dry skin/mucousa, poor skin
turgor, etc.
• Electrolytes derangement
– Symptoms/signs associated with underlying primary
illness
• Oropharyngeal or dental (erythema, edema, erosions,
plaques, halitosis, etc.)
• Pulmonary/airway (wheezing, dyspnea, cough, hoarseness)
• Skin (rash, scales, hardened skins, etc.)
• Muscoloskeletal (joint pain, swelling)

426
Copyright © Harvard Medical School, 2018. All Rights Reserved.

Etiologies of
Esophageal Dysphagia

Mechanical / Mucosal Motility Functional


Obstructive

• Mass or • GERD / reflux • Achalasia • Visceral


malignancy esophagitis • GERD hypersensitivity
• Stricture • Eosinophilic • DES • Functional
• Ring esophagitis • Nutcracker dysphagia
• Web • Infection • Hypomotility /
• Diverticulum • Caustic injury / IEM
• Fistula pills • Scleroderma /
• External • Radiation connective
compression • Autoimmune tissue disorders

DIAGNOSTICS FOR
ESOPHAGEAL AND
SWALLOWING DISORDERS

427
Copyright © Harvard Medical School, 2018. All Rights Reserved.

Imaging
Videofluoroscopic swallow
study/modified barium swallow
– Assess oropharyngeal bolus transport
– Evaluate structural abnormalities of
oropharynx and upper esophagus

Barium swallow
– Evaluate for high-grade esophageal
obstructive lesion, strictures, diverticuli,
or retained foreign body
– May demonstrate characteristic changes
of certain motility disorders, hiatal
hernia, or reflux

Endoscopy
Rule out mechanical
obstruction
– Malignancy
– Foreign body
– Stricture, ring, web

Assess and biopsy abnormal


mucosa
– Infection (viral, fungal)
– Inflammation (GERD, caustic)
– Eosinophilic esophagitis

428
Copyright © Harvard Medical School, 2018. All Rights Reserved.

Esophageal Manometry
Transnasal passage of catheter
with pressure sensors into the
esophagus and stomach

Indications
– Assess esophageal contractile
activity and anatomical landmarks
=> diagnosis of motility disorders
– Assist in placement of pH probe
– Pre-operative evaluation for anti-
reflux surgery or other
esophageal/chest surgeries

Ambulatory pH Monitoring
Continuously records the distal esophageal pH to
quantify acid reflux over study period (24-48 hrs)

Two modalities:
– Transnasal probe with pH sensor
– Endoscopically placed pill-sized capsule (Bravo)

A battery-powered device records data from the


probe/capsule and patient-reported symptoms

Allows correlation between patient-reported


symptoms and reflux events

429
Copyright © Harvard Medical School, 2018. All Rights Reserved.

Intraluminal Impedance
Multichannel intraluminal
impedance detects bolus
movement in the esophagus
– Impedance = resistance to
changing electrical current from
transit of bolus
• Liquid: ↓ impedance
– Combined with manometry or pH
probe
• EM-MII: Measures bolus transit
• pH-MII: Measures bolus reflux

ESOPHAGEAL DISORDERS:
GASTROESOPHAGEAL
REFLUX DISEASE

430
Copyright © Harvard Medical School, 2018. All Rights Reserved.

GERD
25-40% healthy adult Americans experience GERD
symptoms at least once per month
– 7-10% of adults experience symptoms daily
– Likely underestimated due to self-treatment/OTC PPI

Gender: Male ≈ Female


– Reflux esophagitis: 2-3:1 (M:F)
– Barrett’s esophagus: 10:1 (M:F)

GERD prevalence increases with age


– Mean ≈ 40 years old
– Intensity of symptoms may decrease after age 50, but
prevalence of erosive esophagitis increases with age
Richter et al. N Engl J Med 1992.

GERD: Pathophysiology
Gastroesophageal reflux is physiologically normal
– Normally with meals
– Refluxate
• Gastric acid, bile, pancreatic secretion, food matter
– Pathologic reflux results when irritation of the
esophagus (symptom ± inflammation) occurs from
refluxate exposure
• Increased acid/refluxate exposure
• Decreased barrier to irritation

Areas of dysfunction: esophagus, lower


esophageal sphincter (LES), or stomach

431
Copyright © Harvard Medical School, 2018. All Rights Reserved.

GERD: Pathophysiology
Factors associated with GERD
– Esophagus: Impaired esophageal motility or epithelial
barrier function
– LES: Weak LES or inappropriate relaxation
– Stomach: Delayed emptying, increased acid
production, hiatal hernia
– Others:
• Obesity
• Pregnancy / hormonal changes
• Medications
• Ingestions: Food, alcohol, tobacco

GERD: Clinical Presentation


Esophageal Symptoms Extraesophageal
– Typical Manifestations
• Heartburn – ENT
• Regurgitation • Hoarseness, sorethroat,
– Atypical cough, post-nasal drip
• Chest pain – Pulmonary
• Dysphagia • Asthma
• Odynophagia • COPD
• Nausea • Interstitial lung disease
• Globus sensation – Dental erosions
• Epigastric pain
– Waterbrash

432
Copyright © Harvard Medical School, 2018. All Rights Reserved.

GERD: Management
Lifestyle modification
– Dietary modification
• Avoid trigger food items
– Spicy food, citrus food, chocolate, caffeine, carbonated rinks,
alcohol, peppermint
• Multiple, small meals
– Avoid recumbency for 2-3 hours after eating
– Tobacco cessation
– Elevating the head of bed Strategies with the best
evidence for improving
– Weight loss
GERD symptoms

GERD: Management
Empiric acid suppression therapy
– Trial x 8 weeks and titrate dose to severity of symptoms
– Administer 15-30 min before meals
– Always try to taper dose when symptoms respond
• Goal: manage symptoms with lowest possible dose

Risk factors for PPI failure: longer duration of


disease, hiatal hernia, extraesophageal symtoms,
low compliance

Pregnancy: Antacids, H2 receptor antagonists, PPI


(except omeprazole – category C) are safe to use

433
Copyright © Harvard Medical School, 2018. All Rights Reserved.

GERD: Management
PPI Adverse Effects
– Acute side effects in <2%
• Headache, diarrhea, constipation, abdominal pain
– Increased risk for community-acquired pneumonia in
elderly with short term usage (upon initiation)
– Chronic PPI use:
• Malabsorption of B12 (elderly), calcium, magnesium
• Osteoporosis and hip fractures in high risk groups
• Bacterial gastroenteritis and C. difficile
• Small intestinal bacterial overgrowth
– Suggested association
• Cardiovascular events?? => no evidence on further investigation
• Chronic renal insufficiency?? => further investigation needed
• Dementia?? => no evidence on multiple subsequent studies

GERD: Management
When should endoscopy and further diagnostic
testing be performed to evaluate GERD?
– Alarming features
• Dysphagia / odynophagia
• Weight loss
• Signs of GI bleeding/anemia
• Vomiting
• Abnormal imaging
• Family history of upper GI malignancy
– Age > 50
– Nonresponder to medical therapy

434
Copyright © Harvard Medical School, 2018. All Rights Reserved.

GERD: Management
Limited options for PPI nonresponders:
– Optimize timing of PPI and compliance
– Esophageal function testing (manometry, pH monitor)
– Addition of H2 blockers for nocturnal symptoms
• Most beneficial if pH testing shows overnight acid reflux
– No role for metoclopramide without gastroparesis
– Neuromodulator for hypersensitivity: TCA, Trazodone
– Consider GABA-B receptor agonists as reflux inhibitors
• Baclofen 5-20 mg tid (not FDA approved for GERD)
– Decreases acid exposure, reflux episodes, symptoms, and TLESR
– Side effects: dizziness, somnolence, constipation

Van Herwaarden et al. Aliment Pharmacol Ther 2002


Shaheen et al. Gut 2012

GERD: Management
Anti-reflux Surgery
– Indications
• Failed/intolerant of medical therapy, medication dependence
– Fundoplication
• Relieves GERD symptoms in over 90% of patients
• Laparoscopy has comparable 10-year outcome to laparotomy
• Adverse events: gas-bloat syndrome (15-20%), dysphagia
– Bariatric surgery may be more effective than
fundoplication among obese patients
• RYGB >> adjustable gastric band and sleeve gastectomy
• Gastric banding has also previously been shown to cause or
worsen GERD in some patients, and should be avoided in
obese patients with GERD
Broeders et al. Ann Surg. 2009
Pallati et al. Surg Obes Relat Dis. 2014

435
Copyright © Harvard Medical School, 2018. All Rights Reserved.

Non-Cardiac Chest Pain


Esophageal
Etiologies Motor
Disorders
(10-18%) Esophageal motility
disorder or spasm
only accounts for a
GERD
Functional
(29-60%)
small minority of
Chest Pain patients with non-
(32-60%)
cardiac chest pain!

Evaluation:
• RULE OUT CARDIAC CAUSE
• Lifestyle modification, empiric PPI
• EGD, pH study, manometry

Fass et al. Neurogastroenterol Motil 2006


Barret et al. Neurogastroenterol Motil 2016

ESOPHAGEAL DISORDERS:
BARRETT’S ESOPHAGUS

436
Copyright © Harvard Medical School, 2018. All Rights Reserved.

Barrett’s Esophagus
Replacement of normal distal esophageal squamous
epithelium by specialized intestinal metaplasia

Chronic GERD -> esophagitis -> metaplastic change

Overall prevalence in US: ~5.6%


– Patients with GERD: ~5-10%

Risk factors:
– Caucasian, age, male, obesity, tobacco, alcohol

Increased risk of esophageal adenocarcinoma


El-Serag et al. Gastrointest Endosc. 2006
Hayeck et al. Dis Esophagus. 2010

Barrett’s Esophagus
Barrett’s screening
– No screening recommended for general population
– Screening indicated in patients with multiple risk
factors associated with Barrett’s (AGA
recommendations):
• Age 50 or older
• Male sex
• White race
• Chronic GERD
• Hiatal hernia
• Elevated BMI
• Intra-abdominal distribution of body fat

Spechler et al. Gastroenterol. 2011

437
Copyright © Harvard Medical School, 2018. All Rights Reserved.

Barrett’s Esophagus
Barrett’s surveillance
– Endoscopic surveillance with biopsies is indicated for
signs of dysplastic progression
– All biopsies should be examined by pathologists with
expertise in esophageal histopathology
No Dysplasia Low-grade Dysplasia High-grade Dysplasia
Surveillance every 3-5 * Referral for endoscopic * Referral for endoscopic
years therapy therapy
or or
Surveillance every 6-12 Referral for surgery
months if therapy not or
performed Surveillance every 3
months if therapy not
performed

Barrett’s Esophagus
Treatment
– Endoscopic therapy
• Endoscopic mucosal resection
• Radiofrequency ablation
• Indication: Dysplastic Barrett’s
– All post-endoscopic therapy patients require
continued surveillance to ensure eradication of
Barrett’s tissues and to monitor for recurrence
– Chemoprevention
• PPI has been shown to reduce neoplastic progression in
patients with Barrett’s
• Aspirin and NSAIDs are associated with reduced risk for
esophageal cancer

Kastelein et al. Clin Gastroenterol Hepatol. 2013

438
Copyright © Harvard Medical School, 2018. All Rights Reserved.

ESOPHAGEAL DISORDERS:
MOTILITY DISORDERS

Esophageal Motility
Disorders

Disorders of Inhibitory Disorders of Excitatory


Innervation Innervation and Smooth
Muscle

Decreased Increased Hypotensive Hypertensive

Achalasia Transient lower Hypotensive Hypertensive


esophageal peristalsis (IEM) peristalsis
Diffuse sphincter (nutcracker,
esophageal relaxation Hypotensive LES jackhammer)
spasm (DES) (TLESR)
Scleroderma Hypertensive
LES

439
Copyright © Harvard Medical School, 2018. All Rights Reserved.

Achalasia
Preferential degeneration of post-ganglionic
inhibitory neurons in myenteric plexus
– Insufficient lower esophageal sphincter relaxation with
swallowing and aperistalsis

Annual Incidence: 1 in 100,000

Most commonly between ages 25-50

Prevalence: Male = Female

16-fold increase in risk of squamous cell carcinoma


– Average of 14 years after achalasia diagnosis
– Routine surveillance endoscopy not recommended

Achalasia
Primary achalasia: Idiopathic, ? viral, ? Autoimmune
– Gradual onset and insidious progression
– Mean duration of symptoms -> diagnosis = 4.7 years

Secondary achalasia: Chagas disease, malignancy,


amyloidosis, sarcoidosis, eosinophilic gastroenteritis,
neurofibromatosis
– Often acute onset and more rapidly progressive

Signs/symptoms
– Dysphagia (nearly 100%), regurgitation (60-90%)
– Difficulty belching (85%), chest pain/heartburn (40%),
weight loss, pulmonary/ENT symptoms (especially when
recumbent)

440
Copyright © Harvard Medical School, 2018. All Rights Reserved.

Achalasia Diagnosis
Laboratory studies
– Anti-neuronal antibodies, T cruzi assay

Endoscopy
– Rule out mechanical obstruction

Barium swallow
– Bird’s beak
– Dilated esophagus
– No peristalsis
– Poor esophageal emptying

Manometry
– Abnormal relaxation of the LES with
swallowing
– Aperistalsis of the esophageal body
Source: Fox et al. Gut 2008.

Achalasia Treatment
Pneumatic Surgicalmyotomy POEM Botulinum toxin CCB / Nitrates
dilation injection
Response 60-90% at 1 yr 90-93% at 1 yr 98% at 1 yr 90% at 1 mo 50-70% initial
86% at 2 yrs 90% at 2 yrs 91% at 2 yrs 60% at 1 yr <50% at 1 yr
82% at 5 yrs 85% at 5 yrs
Complications 2-5% 10% symptomatic 37% increased reflux 20% rash, transient 30% headache,
perforation reflux; on 24-hr pH; chest pain hypotension
11% mucosal tear 18% reflux
esophagitis
Advantages Good response Excellent, Excellent response; Low morbidity Rapidly initiated
rates prolonged Less invasive than
response; surgery;
Laparoscopic Long myotomy
Disadvantages Risk of Surgical risk Lack of controlled Frequent repeat; Poor effect on
perforation; and long-term data Loss of response; esophageal
~25% require Fibroinflammatory emptying;
re-treatment reaction at LES Tachphylaxis

441
Copyright © Harvard Medical School, 2018. All Rights Reserved.

Achalasia Treatment

Low Surgical High Surgical


Risk Risk

Pneumatic
Myotomy Botox Injection
Dilation Failure

Failure

Nitrates
Repeat myotomy or dilation
Calcium channel
Esophagectomy
blockers

Diffuse Esophageal Spasm


Impairment of inhibitory innervation in the
esophageal body
– Excessive simultaneous or premature contractions
– Normal LES relaxation

Symptoms and Signs


– Chest pain (at rest, with eating, stress)
– Dysphagia, regurgitation

Imaging
– Barium swallow
• Non-propagated (tertiary) contractions
• “corkscrew” appearance

442
Copyright © Harvard Medical School, 2018. All Rights Reserved.

Diffuse Esophageal Spasm


Manometry
– Spastic esophageal body contractions
with normal LES relaxation

Treatment
– Acid suppression: PPI or H2 Blockers
– Smooth muscle relaxants: nitrates,
CCB
– Anticholinergics
– Pain modulators: TCA
– Dilation, botox injection, myotomy

Clinical course/Prognosis
Source: Zerbib et al. J Clin Gastroenterol 2015. – Intermittent and non-progressive
– 3-5% transition to achalasia

Other Motility Disorders


Motor Pathology Etiologies Clinical Study Treatment
Disorders Presentation Findings
Hypomotility Impaired • Idiopathic • GERD • Decreased • Anti-reflux
Disorders excitatory • Medications - Esophagitis contraction therapy
• Hypotensive LES innervation - Anticholinergic - Stricture • Increased • Avoid
• Absent or muscle - Smooth • Dysphagia failed offending
contractility contractility muscle • Chest pain peristalsis meds
• Ineffective relaxant • Impaired • Baclofen
Esophageal - Hormone
Motility (IEM)
esophageal • Bethanechol
• Pregnancy
clearance
• CTD (Scl)
• GERD
Hypermotility Overactive • CNS stress • Chest pain • Increased • Anti-reflux
Disorders excitatory • GERD • Dysphagia contraction therapy
• Hypertensive LES nerves or • Normal • Anticholinergic
• Hypercontractile smooth peristalsis • Smooth
LES muscle • Nutcracker muscle
• Nutcracker/ response esophagus relaxant
Jackhammer
• Elevated • TCA
LES • Cognitive-
pressure behavioral
therapy

443
Copyright © Harvard Medical School, 2018. All Rights Reserved.

ESOPHAGEAL DISORDERS:
EOSINOPHILIC ESOPHAGITIS

Eosinophilic Esophagitis
Clinically characterized by symptoms related
to esophageal dysfunction
Pathologically, ≥ 1 biopsy specimens must
show eosinophil-predominant inflammation
– ≥15 eos/hpf on esophageal biopsies
Disease is isolated to the esophagus and
other causes of esophageal eosinophilia are
excluded

Liacouras et al. J Allergy Clin Immunol 2011

444
Copyright © Harvard Medical School, 2018. All Rights Reserved.

Eosinophilic Esophagitis
Mean age: 37 years Symptoms:
Gender: 72% Male – Dysphagia 82%
– Heartburn: 29%
Mean symptom
duration: 5 years – Chest pain: 8%

Atopic history: 74%


Food allergy history:
19%

Eosinophilic Esophagitis
Food impaction
– EoE found in 11-55% of
adult with food impaction
– 30-55% of EoE patients
experience food impaction

Endoscopy
– Ringed esophagus
– Longitudinal furrows
– White patches
– Small caliber esophagus
– Strictures

445
Copyright © Harvard Medical School, 2018. All Rights Reserved.

PPI-REE/PPI-Responsive EoE
25-50% of patients with EGD + biopsies
esophageal eosinophilia
have histologic resolution ≥15 eos/hpf
of eosinophilia with PPI
Twice daily PPI for
8-12 weeks
Clinically indistinguishable
from PPI-non-responsive
EoE EGD + biopsies

≥15 eos/hpf <15 eos/hpf


? underlying GERD vs anti-
inflammatory properties of EoE PPI-REE/
PPI (EoE variant) PPI-REoE

Treatment for PPI-Non-


Responsive EoE
Elimination diet
– Empiric vs allergy test-directed

Topical therapy
– Swallowed fluticasone, budesonide slurry

Systemic therapy
– Steroids
– ? Biologics

Endoscopic dilation of strictures

446
Copyright © Harvard Medical School, 2018. All Rights Reserved.

ESOPHAGEAL DISORDERS:
OTHER ESOPHAGITIS

Infectious Esophagitis
Odynophagia is the most common
symptoms
Most often develops in immunosuppressed
patients
– HIV infection, chemotherapy, organ
transplantation, chronic immunosuppressive
medication use

447
Copyright © Harvard Medical School, 2018. All Rights Reserved.

Infectious Esophagitis
Candidial esophagitis
– Most frequent cause of infectious esophagitis
– Often associated with oral candidiasis
– Upper endoscopy with biopsy is the most sensitive
and specific test
• Characteristic appearance: whitish plaques
– Treatment typically involves systemically active
oral azoles (e.g. fluconazole)

Infectious Esophagitis
Viral esophagitis
– Herpes simplex virus, cytomegalovirus
– Most often seen in transplant recipients
– Diagnosis requires upper endoscopy with
esophageal brushing, viral culture, or biopsies
• Characteristic appearances: mucosal ulcerations
– Treatment:
• HSV: acyclovir, foscarnet
• CMV: gangiclovir, foscarnet, cidofovir

448
Copyright © Harvard Medical School, 2018. All Rights Reserved.

Pill Esophagitis
Dysphagia or odynophagia
Most common drugs:
– Antibiotics, NSAIDs, bisphophonates, potassium
choloride, quinidine, ferrous sulfate
Most patients have normal underlying
esophageal structure and function
Risk factors: advanced age, esophageal
dysmotility, extrinsic esophageal compression,
large pill size, swallowing pills without water
and in supine position

Summary
The main symptoms of esophageal disorders
include dysphagia, odynophagia, heartburn,
chest pain, and regurgitation.

Barium esophogram, endoscopy, esophageal


manometry, and ambulatory pH monitoring are
the primary diagnostic modalities for esophageal
disorders.

449
Copyright © Harvard Medical School, 2018. All Rights Reserved.

Summary
Lifestyle modification and empiric acid
suppression trial are first-line approach to
uncomplicated GERD.

In management of GERD symptoms, endoscopy


and other diagnostic studies should be reserved
for patients with alarm features, older age
individuals (>50), or those who fail medical
therapy.

Summary
Dose tapering of PPI should always be
attempted once symptom response is achieved.

The most common cause of non-cardiac chest


pain is gastroesophageal reflux disease.
Management strategies for GERD should be
followed once cardiac cause has been ruled out.

450
Copyright © Harvard Medical School, 2018. All Rights Reserved.

Summary
Secondary or pseudoachalasia should be ruled
out in patients diagnosed with achalasia.

Management strategy of primary achalasia


depends on the patient’s surgical risk.
– Low surgical risk patients should undergo pneumatic
dilation or myotomy (surgical or endoscopic).

Summary
Esophageal biopsies to assess for eosinophilia
should be performed for work-up of dysphagia.

When esophageal eosinophilia is found, an 8-12-


week trial of high-dose PPI followed by repeat
biopsies should be performed to determine
responsiveness to PPI

Management of PPI-non-responsive EoE mainly


involves elimination diet or steroid therapy.

451
Copyright © Harvard Medical School, 2018. All Rights Reserved.

Summary
Odynophagia is the most common symptom for
infectious esophagitis.

Infectious esophagitis usually occur in


immunosuppressed patients, with candida being
the most common agent.

Medications at highest risk for causing pill


esophagitis include NSAID, antibiotics,
bisphosphonate, potassium tablets.

Question 1
A 40 year-old woman with no significant past medical
history presented to the Emergency Department with
chest pain episodes over the past week. She described
a sharp pain in her substernal region, with no
association with exertion, positional changes, or eating.
She was admitted for management and cardiac
evaluation, including an exercise stress test, which were
all negative. Her symptoms were felt to most likely be
of an esophageal origin. However, she denied any
dysphagia, odynophagia, nausea, vomiting, heartburn,
reflux, or regurgitation symptoms. Her other laboratory
evaluations were unremarkable and she had no other
complaints.

452
Copyright © Harvard Medical School, 2018. All Rights Reserved.

Question 1
What is the most appropriate next step for the
management of this patient?

A. Barium swallow.
B. Empiric trial of proton pump inhibitor.
C. Upper endoscopy with mucosal biopsies.
D. Esophageal manometry with impedance study.
E. Empiric trial of smooth muscle relaxant (calcium
channel blockers).

Question 1: Answer
B. Empiric trial of proton pump inhibitor.
The patient in question presented with chest pain symptoms and
her cardiac evaluation was unremarkable. The most common
cause of non-cardiac chest pain is gastroesophageal reflux disease
(GERD), which should be evaluated/treated next. In a young and
healthy patient without alarming signs/symptoms, the initial steps
of GERD management include lifestyle modification and an empiric
trial of proton pump inhibitor (PPI). Diagnostic studies (answers A,
C, D) should be considered only after the patient fails her PPI trial
or if she has any alarming signs/symptoms. An empiric trial of
smooth muscle relaxant (answer E) would not be appropriate
without an objective diagnosis of hypercontractile motility disorder,
as GERD is a significantly more common cause of non-cardiac
chest pain than esophageal dysmotility, and smooth muscle
relaxants would worsen reflux.

453
Copyright © Harvard Medical School, 2018. All Rights Reserved.

Question 2
A 20 year-old man with a history of eczema and chronic
dysphagia presents for follow-up after an incidence of food
impaction for which he underwent urgent upper endoscopy in
the emergency department. A repeat upper endoscopy was
performed :

Mucosal biopsies obtained revealed >80 eosinophils/hpf. He


continues to report intermittent dysphagia symptoms when
eating solid food, but no further food impaction episode. He
denies any chest pain, heartburn, reflux, or regurgitation.

Question 2
What is the most appropriate next step in management?

A. Start six-food elimination diet.


B. Schedule esophageal manometry.
C. Initiate treatment with swallowed fluticasone
D. Start twice daily proton pump inhibitor.
E. Repeat upper endoscopy for biopsies.

454
Copyright © Harvard Medical School, 2018. All Rights Reserved.

Question 2: Answer
D. Start twice daily proton pump inhibitor.
The patient in question (young man with a history of atopy)
presented with intermittent dysphagia and a history of food
impaction, raising suspicion for eosinophilic esophagitis. His upper
endoscopy revealed ringed esophagus and esophageal biopsies
showed an increase in eosinophilic infiltration. The initial step of
management after diagnosis of esophageal eosinophilia is a trial of
twice daily proton pump inhibitor (PPI) for 8-12 weeks, followed by
repeat upper endoscopy with biopsies to assess histologic response
(E). This allows differentiation between PPI-responsive esophageal
eosinophilia if eosinophilic infiltration resolves and non-PPI
responsive eosinophilic esophagitis (EoE) if eosinophilia persists.
Elimination diet (A) and steroids (C) should be considered only
after post-PPI biopsies confirm lack of responsiveness. Esophageal
manometry (B) is not part of the management algorithm for
esophageal eosinophilia.

References
Liacouras et al. J Allergy Clin Immunol
2011
Source: Fox et al. Gut 2008.
Richter et al. N Engl J Med 1992
Fass et al. Neurogastroenterol Motil 2006
Barret et al. Neurogastroenterol Motil
2016

455
Copyright © Harvard Medical School, 2018. All Rights Reserved.

Thank You

Center for Gastrointestinal Motility

456
Copyright © Harvard Medical School, 2018. All Rights Reserved.

PEPTIC ULCER DISEASE


• JOHN R SALTZMAN, MD
• Director of Endoscopy
• Brigham and Women’s Hospital
• Professor of Medicine
• Harvard Medical School

Disclosures
• No disclosures

457
Copyright © Harvard Medical School, 2018. All Rights Reserved.

Overview
• Causes of peptic ulcer disease
• Clinical manifestations
• Current diagnostic tests
• Recommended therapies
• New strategies to treat H. pylori
• Complications including GI bleeding

Peptic ulcer disease


• Ulcers are defects in gastrointestinal mucosa that
extend beyond muscularis mucosa
• Arise when mucosal defenses are impaired in
presence of gastric acid and pepsin
• Overall incidence declining worldwide
• Lifetime prevalence 5-10%
• Complications
– GI bleeding (10-20%)
– Perforation (5%)
– Obstruction (2%)

458
Copyright © Harvard Medical School, 2018. All Rights Reserved.

Causes of peptic ulcers


• Helicobacter pylori infection
• Aspirin and NSAIDs
• Rare causes:
– Neoplasia (carcinoma, lymphoma, leiomyosarcoma)
– Acid hypersecretion (Zollinger-Ellison syndrome)
– Granulomatous disease (Crohn’s, sarcoidosis)
– Systemic mastocytosis
– Infectious (CMV, HSV, TB)
– Familial
• Idiopathic (No H. pylori, NSAID or other identifiable
cause)

Additional Risk Factors


• Older age
• Prior history of peptic ulcers
• Smoking – proportional to amount
smoked
• Medications in combination with NSAIDs
– Corticosteroids, anticoagulants, anti-
platelets, SSRIs, bisphosphonates

459
Copyright © Harvard Medical School, 2018. All Rights Reserved.

Helicobacter pylori
• Gram-neg spiral, urease producing bacteria
• H. pylori is the most common chronic bacterial
infection in humans
• Infection mainly acquired in childhood (<10 years)
• Risk factors include
– Low socioeconomic status
– Household crowding
– Country of origin and ethnicity

Cover TL, Blaser MJ. Gastroenterology 2009;136(6):1863-73;


Bruce MG Epidemiol Infect 2015;143(6):1236-4

H. pylori and peptic ulcers

• H. pylori associated with up to 50% to


80% duodenal ulcers
• H. pylori associated with up to 60% to
70% gastric ulcers
• Prevalence of H. pylori
– Undeveloped countries 80% by age 20
– Developed countries 10% by age 20
• Incidence of peptic ulcers declining in
developed countries due to less H. pylori

460
Copyright © Harvard Medical School, 2018. All Rights Reserved.

H. pylori eradication results in long-term


remission of peptic ulcer disease
Duodenal ulcer Gastric ulcer
% Patients in remission % Patients in remission
100 100
H. pylori eradicated H. pylori eradicated
75 75

50 50
H. pylori-positive
25 25
H. pylori-positive
0 0
0 0.5 1 1.5 2 0 2 4 6 8 10 12
Years after termination Months after termination
of treatment of treatment

Miehlke S. Eur J Gastroenterol Hepatol 1995;7(10):975-8;


Axon AT. BMJ 1997;314(7080):565-8

NSAIDs and peptic ulcers


• Responsible for majority of ulcers not due to H. pylori
(>100,000 hospitalizations/year)
• Increased risk for significant GI event (bleeding,
perforation and obstruction) and “silent” ulcers
• Surreptitious NSAID use is common, especially in
refractory / complicated ulcers
• Co-therapy of NSAIDs with corticosteroids,
anticoagulants, other NSAIDs, low dose aspirin,
selective serotonin reuptake inhibitors (SSRIs) and
alendronate increases the risk of ulcers

461
Copyright © Harvard Medical School, 2018. All Rights Reserved.

Risk of ulcer complications with


ASA and NSAIDs

ASA + ASA + ASA +


NSAIDs > COX-2 inhibitor > placebo

• ASA alone causes a 2-3 fold increase in ulcer bleeding


• Long term ASA use does not reduce risk of ASA-induced
ulcer bleed (Meta-analysis of 24 RCTs)
• Enteric coated or buffered ASA does not protect against
ulcers due to systemic effects

Garcia Rodriguez LA. Gastroenterology 2007;132(2):498-506;


Derry S, Loke YK. BMJ 2000;321(7270):1183-7

Risk factors for NSAID-


induced peptic ulcers
Risk Factor Relative Risk 95 Percent CI
Overall 2.74 2.54-2.97
Anticoagulants 12.7 6.3-25.7
High dosage 10.1 4.6-22.0
(>2x normal)
Age (>60) 5.52 4.63-6.60
Prior PUD 4.76 4.05-5.59
Corticosteroids 4.4 2.0-9.7

462
Copyright © Harvard Medical School, 2018. All Rights Reserved.

Other causes of peptic ulcers


• Gastric acid hypersecretion
– Zollinger-Ellison syndrome
• Cigarette smoking
– Increased rate of ulcers in patients with H.
pylori
– More difficult to treat ulcers
• Neoplasia (carcinoma, lymphoma)
• Familial (polygenic inheritance)

Zollinger-Ellison Syndrome
• Neuroendocrine tumor (duodenum, pancreas)
• Hypergastrinemia ↑ acid secreLon
• PUD (duodenum, jejunum), diarrhea, GERD
• Usually sporadic
– MEN1 syndrome in 25% of cases
• Diagnosis – gastrin, cross-sectional imaging
– Ddx hypergastrinemia – hypochlorhydria from PPI or
atrophic gastritis

463
Copyright © Harvard Medical School, 2018. All Rights Reserved.

Clinical manifestations of
peptic ulcer disease
• Dyspepsia: epigastric pain most common
• Duodenal ulcer symptoms
– Epigastric pain (burning, gnawing or hunger-like), 2 to 5
hours after meal or on empty stomach (without a food
buffer)
– Nocturnal between 11 PM and 2 AM (circadian
stimulation of acid maximal)
– Symptom relief with food or antacids
• Gastric ulcer symptoms: Pain soon after meals
with less relief by food or antacids
• Ulcers can be asymptomatic (up to 70%)

Differential diagnosis of
peptic ulcer disease

• Functional nonulcer dyspepsia


• Gastroesophageal reflux disease
• Gastric cancer
• Pancreatic and biliary diseases

464
Copyright © Harvard Medical School, 2018. All Rights Reserved.

Alarm symptoms
• Weight loss
• Persistent vomiting
• Dysphagia
• Anemia
• Hematemesis
• Palpable abdominal mass
• Family H/O upper GI carcinoma
• Previous gastric surgery

Diagnosis of peptic ulcer disease


• Suspect by symptoms and history
• Ask about alarm symptoms
• Consider blood tests
– CBC, LFT’s, calcium and lipase
• Test for H. pylori infection
• Consider endoscopy

465
Copyright © Harvard Medical School, 2018. All Rights Reserved.

Upper endoscopy for PUD


• Upper endoscopy gold standard test
• Best in patients over the age of 45 years or
with alarm symptoms
• Allows biopsy (for H. pylori or to R/O
cancer)
• Sensitive, specific and safe
• Best initial test in most patients

Gastric ulcer

466
Copyright © Harvard Medical School, 2018. All Rights Reserved.

Antisecretory therapy for PUD


• H2-blockers 90% effective after 8 weeks
• Proton pump inhibitors
– More rapid healing than H2-blockers
– 90% effective at 4 weeks
• Large ulcers (>2 cm) take longer to heal
• Gastric ulcers—confirm healing and exclude
malignancy in 8 to 12 weeks
• Continue maintenance PPIs in patients with non H.
pylori, non NSAID ulcers

Diagnostic tests for H. pylori infection

467
Copyright © Harvard Medical School, 2018. All Rights Reserved.

Endoscopy-based methods of
detecting H. pylori
Method of Sensitivity Specificity
Diagnosis Main Indication (%) (%)

Histology Diagnosis 90 90-95

Culture H. pylori antibiotic 80-90 95-100


sensitivities

Rapid urease Rapid results in 90 90


test (CLO) endoscopy room

Gastric biopsies with H. pylori

468
Copyright © Harvard Medical School, 2018. All Rights Reserved.

Noninvasive methods of
detecting H. pylori
Method of Sensitivity Specificity
Diagnosis Main Indication (%) (%)

Serology Screening 85 79

Urea breath Screening and confirm 95 96


test eradication

Stool antigen Screening and confirm 93 94


test eradication

Principle of 13C + 14C-urea breath test

469
Copyright © Harvard Medical School, 2018. All Rights Reserved.

General principles of treating PUD


Heal ulcer by decreasing gastric
acid using PPIs or H2RAs

Treat the underlying cause:


• Test and eradicate H. pylori infection
• For NSAID-related ulcers
- Discontinue or switch to non-NSAID analgesic
- For patients who require chronic NSAIDs, consider
co-therapy with PPI (or misoprostol)

Established recommendations
for H. pylori eradication
• Strong evidence to treat
– Gastric or duodenal ulcers
(current or H/O untreated H. pylori)
– MALT lymphoma
• Also recommended (limited evidence of benefit)
– Atrophic gastritis / intestinal metaplasia
– Gastric adenocarcinoma in early stages
– 1st degree relatives of patients with gastric cancer
Maastricht IV Consensus Report. Gut 2012;61(5):646-64;
Liang X. Clin Gastroenterol Hepatol 2013;11(7):802-7

470
Copyright © Harvard Medical School, 2018. All Rights Reserved.

New recommendations for


H. pylori eradication
• Any patient with a positive test not previously treated
• Non-ulcer dyspepsia
– Under the age of 60 years without alarm features who undergo non-
endoscopic testing and found to have H. pylori
– Those who undergo endoscopy and are found to have H. pylori
• Prior to long-term NSAID or ASA use to reduce risk of bleeding
(in patients with H. pylori)
• Patients with unexplained iron deficiency anemia despite an
appropriate evaluation (in patients with H. pylori)
• Adults with idiopathic thrombocytopenic purpura (ITP) should
be tested for and treated if H. pylori present

Chey WD. Am J Gastroenterol 2017;112:212–238

Treatment of H. pylori
• Multiple regimens and durations evaluated
• Treatment must be effective, have acceptable costs,
side effects and ease of administration
• Commonly used regimens have frequent side effects
(usually mild) including metallic taste, diarrhea and
allergic reactions
• Be aware of clarithromycin and metronidazole
resistance and do not repeat Rx with these drugs
• Best regimens eradicate organism > 90%

471
Copyright © Harvard Medical School, 2018. All Rights Reserved.

H. pylori eradication therapies


• Clarithromycin triple therapy
• PPI BID+ 2 antibiotics including clarithromycin
– Determine if patient had previous macrolide exposure
– Clarithromycin may be used if local resistance low
(resistance <15%)
• Bismuth quadruple therapy
– PPI BID+ Bismuth salt + 2 antibiotics
• Other first line regimens
• Salvage therapy
– Avoid initial antibiotics and offer to those with persistent
infection after initial therapy

Liang X. Clin Gastroenterol Hepatol 2013;11(7):802-7;


Chey WD. Am J Gastroenterol 2017;112:212–238

Clarithromycin triple therapy

• Proton pump inhibitor twice daily


• Amoxicillin 1 gram twice daily
• Clarithromycin 500 mg twice daily
• All given for 14 days
• Can substitute Metronidazole 500 mg twice
daily if penicillin allergic

472
Copyright © Harvard Medical School, 2018. All Rights Reserved.

Bismuth quadruple therapy


• Proton pump inhibitor twice daily
• Bismuth subcitrate or subsalicylate
2 tablets 4 times daily
• Tetracycline 500 mg 4 times daily
• Metronidazole 250-500 mg 4 times daily
• All given with meals for 10-14 days
• First line therapy if prior macrolide exposure
or penicillin allergy

2017
H. pylori
Treatment
Guidelines
American College of
Gastroenterology (ACG)

Chey WD. Am J Gastroenterol 2017;112:212-238

473
Copyright © Harvard Medical School, 2018. All Rights Reserved.

Levofloxacin triple therapy

• Proton pump inhibitor twice daily


• Amoxicillin 1 gram twice daily
• Levofloxacin 500 mg once daily
• All given for 10-14 days

Concomitant therapy
• Proton pump inhibitor twice daily
• Amoxicillin 1 gram twice daily
• Clarithromycin 500 mg twice daily
• Nitroimidazole (metronidazole or
tinidazole) 500 mg twice daily
• All given for 10-14 days

474
Copyright © Harvard Medical School, 2018. All Rights Reserved.

Confirmation of H. pylori eradication

• Recommended for all treated patients


– Serologic testing not useful
– Urea breath testing or stool antigen testing

• Repeat testing 4-6 weeks after therapy


– Patients need to avoid using antibiotics, bismuth compounds and PPIs
(for 1-2 weeks) which can cause false negative results

Salvage regimens
• Use for recurrent or persistent H. pylori
• If clarithromycin used as first line therapy
– Bismuth quadruple therapy x 14 days
– Levofloxacin triple therapy x 14 days
• If Bismuth used as first line therapy
– Clarithromycin triple therapy x 14 days
– Levofloxacin triple therapy x 14 days
• Concomitant therapy alternative option

475
Copyright © Harvard Medical School, 2018. All Rights Reserved.

Treatment recommendations for


NSAID-related peptic ulcers
Heal ulcer with acid lowering medications

Test and eradicate any H. pylori infection

Discontinue or For patients requiring chronic


switch to NSAIDs consider strategies
non-NSAID analgesic to prevent recurrent ulcers

Preventing recurrent ulcers in patients


requiring chronic NSAIDs
• Use lowest dose of NSAID and provide co-
therapy with either PPI or misoprostol
(prostaglandin E1 analogue)
• Switch to a selective COX-2 inhibitor
• Effective anti-inflammatory agents
• Less ulcer complications than nonselective NSAIDs
– Increases cardiovascular events (RR =1.42)

Sung JJ. Gut 2011;60(9):1170-7;


Laine L. Aliment Pharmacol Ther 2009;30(7):767-74

476
Copyright © Harvard Medical School, 2018. All Rights Reserved.

Complications of peptic ulcers


• Gastrointestinal bleeding 10-20%
• Gastric outlet obstruction 2%
• Perforation/fistulalization 5%
• Usually occur in chronic peptic ulcers
• May develop in days - weeks of NSAIDs
• May occur heralded by symptoms or may
occur in “silent” ulcers

Bleeding peptic ulcers


• Accounts for over 50% severe UGI bleeding
• Overall mortality rate 2-14%
• Resuscitation is critical
• Restrictive transfusion strategy (Hgb < 7 g/dL)
• Acid suppression with PPIs
• Endoscopy within 24 hours

Crooks C. Gastroenterology 2011;141(1):62-70;


Villanueva C. N Engl J Med 2013;368(1):11-21

477
Copyright © Harvard Medical School, 2018. All Rights Reserved.

Restrictive vs. liberal transfusion strategy

921 patients with acute upper GI bleeding

Restrictive: Transfuse Liberal: Transfuse


when Hgb < 7 g/dL when Hgb < 9 g/dL

Further GI bleed 10% 16% (P=0.01)

Adverse events 40% 48% (P=0.02)

Hazard ratio: 0.55


Survival at 6 weeks 95% 91% (95%CI:0.33 to 0.92 ; P=0.02)

Villanueva C. N Engl J Med 2013;368(1):11-21

Survival according to
transfusion strategy
Overall Survival (%)

Days

478
Copyright © Harvard Medical School, 2018. All Rights Reserved.

RCT of aspirin vs. placebo after


peptic ulcer bleed
Event rate (%)

Sung JJ. Ann Intern Med 2010;152:1-9

Resumption of aspirin after


upper GI bleeding

American College of Gastroenterology Guidelines

“If given for secondary prevention (i.e. established


CV disease) then aspirin should be resumed as soon
as possible after bleeding ceases in most patients:
ideally within 1-3 days and certainly within 7 days”

Laine L, Jensen DM. Am J Gastroenterol 2012;107:345-60

479
Copyright © Harvard Medical School, 2018. All Rights Reserved.

Summary
• NSAIDs/ASA and H. pylori are the major causes
of peptic ulcers
• Test for and eradicate H. pylori using
established treatments regimens
• For ulcer patients requiring chronic NSAIDs
– Treat with PPI (misoprostol) or COX2 NSAID
• For bleeding ulcer patients requiring ASA
– Restart ASA within 7 days with a PPI

Questions

480
Copyright © Harvard Medical School, 2018. All Rights Reserved.

Question 1
A 47 year old man presents with a bleeding gastric ulcer with
biopsies showing H. pylori infection. He is treated with
clarithromycin triple therapy including a PPI, amoxicillin and
clarithromycin. At 8 weeks repeat upper endoscopy shows that the
ulcer has healed but H. pylori is still present. His next course of H.
pylori treatment should avoid use of the following medication:

a) Proton pump inhibitors


b) Amoxicillin
c) Clarithromycin
d) Metronidazole
e) Bismuth subsalicylate

Question 1
A 47 year old man presents with a bleeding gastric ulcer with
biopsies showing H. pylori infection. He is treated with
clarithromycin triple therapy including a PPI, amoxicillin and
clarithromycin. At 8 weeks repeat upper endoscopy shows that the
ulcer has healed but H. pylori is still present. His next course of H.
pylori treatment should avoid use of the following medication:

a) Proton pump inhibitors


b) Amoxicillin
c) Clarithromycin
d) Metronidazole
e) Bismuth subsalicylate

481
Copyright © Harvard Medical School, 2018. All Rights Reserved.

The answer is C
• Commonly used regimes to treat H. pylori include
clarithromycin triple therapy (PPI BID + 2 antibiotics
including clarithromycin) and Bismuth quadruple
therapy (PPI BID, Bismuth + 2 antibiotics)
• Clinicians need to be aware of clarithromycin and
metronidazole resistance
• If a patient is previously treated with clarithromycin or
metronidazole, a repeat treatment for H. pylori should
not use these drugs
Maastricht IV Consensus Report. Gut 2012;61(5):646-64;
Chey WD. Am J Gastroenterol 2017;112:212–238

Question 2
A 62 year old male presents with abdominal pain, nausea, and
passage of dark black stools. He has diabetes, hypertension, and
coronary artery disease and takes aspirin daily for secondary
prophylaxis. An upper endoscopy shows a duodenal ulcer with a
visible vessel that is treated with endoscopic hemostasis. Biopsies of
the stomach show no H. pylori present. He remains stable for 3 days
and starts treatment with oral PPI therapy. What are your
recommendations about further antiplatelet therapy?

a) Discontinue aspirin therapy


b) Switch aspirin therapy to clopidogrel
c) Continue an oral PPI and restart aspirin at this time
d) Continue an oral PPI and restart aspirin therapy after 4 weeks

482
Copyright © Harvard Medical School, 2018. All Rights Reserved.

Question 2
A 62 year old male presents with abdominal pain, nausea, and
passage of dark black stools. He has diabetes, hypertension, and
coronary artery disease and takes aspirin daily for secondary
prophylaxis. An upper endoscopy shows a duodenal ulcer with a
visible vessel that is treated with endoscopic hemostasis. Biopsies of
the stomach show no H. pylori present. He remains stable for 3 days
and starts treatment with oral PPI therapy. What are your
recommendations about further antiplatelet therapy?

a) Discontinue aspirin therapy


b) Switch aspirin therapy to clopidogrel
c) Continue an oral PPI and restart aspirin at this time
d) Continue an oral PPI and restart aspirin therapy after 4 weeks

The answer is C
• The patient has significant cardiovascular disease
requiring continued anti-platelet therapy
• The use of a PPI given with aspirin is superior to
switching to clopidogrel alone in reducing
recurrent ulcer bleeding
• He should continue oral PPI therapy and restart
aspirin as soon as possible
• This strategy significantly reduces mortality as
compared to restarting aspirin several weeks later

Sung JJ. Ann Intern Med 2010;152:1-9

483
Copyright © Harvard Medical School, 2018. All Rights Reserved.

Disclosures
• No disclosures

References
• Chey ED. ACG Clinical Guideline: Treatment of Helicobacter pylori
Infection. Am J Gastroenterol 2017;112:212–238
• Graham DY. History of Helicobacter pylori, duodenal ulcer, gastric
ulcer and gastric cancer. World J Gastroenterol 2014;20:5191-204
• Kumar NL. Initial management and timing of endoscopy in
nonvariceal upper GI bleeding. Gastrointest Endosc 2016;84(1):10-7
• Lau JY. Challenges in the management of acute peptic ulcer
bleeding. Lancet 2013;381:2033-43
• Malfertheiner P. Management of Helicobacter pylori infection-the
Maastricht IV/ Florence Consensus Report. Gut 2012;61(5):646-64
• Laine L Jensen DM. Am J Gastroenterol 2012;107:345-60
• Villanueva C. Transfusion strategies for acute upper gastrointestinal
bleeding. N Engl J Med 2013;368(1):11-21

484
Copyright © Harvard Medical School, 2018. All Rights Reserved.

485
Copyright © Harvard Medical School, 2018. All Rights Reserved.

ACUTE AND CHRONIC PANCREATITIS


Julia Y McNabb-Baltar, MD, MPH
Associate Physician
Co-Director Center for Pancreatic Disease
Division of Gastroenterology, Hepatology and Endoscopy
Brigham and Women’s Hospital
Instructor of Medicine
Harvard Medical School

Disclosure
I have no disclosures

486
Copyright © Harvard Medical School, 2018. All Rights Reserved.

Outline
Acute Pancreatitis
– Epidemiology
– Diagnosis
– Etiology
– Severity
– Treatment
– Complications
Chronic Pancreatitis
– Etiology
– Diagnosis
– Etiology
– Treatment
– Complications

ACUTE PANCREATITIS

487
Copyright © Harvard Medical School, 2018. All Rights Reserved.

Epidemiology
>275,000 patients are hospitalized for AP
annually
Aggregate cost of >$2.6 billion per year
Incidence 5-30 / 100,000
Case fatality is 5 % overall

Diagnosis
2 of the 3 following criteria:

1) Abdominal pain consistent with the


disease
2) Serum amylase and/or lipase > 3 times
ULN
3) Characteristic findings on abdominal
imaging
Tenner, Baillie, DeWitt. American College of Gastroenterology
Guideline: Management of Acute Pancreatitis. Am J Gastro. 30 july 2013

488
Copyright © Harvard Medical School, 2018. All Rights Reserved.

Abdominal Imaging
CECT: 90% Sn, Sp
– Routine use of CT unwarranted
CT or MRI recommended if unclear dx or
patient fail to improve within 48-72h, to
assess local complications
– Guideline recommendation: Strong
recommendation, low quality of evidence
MRCP may help detect CBD stones and
PD disruption
Tenner. Am J Gastro. 2013
Stimac. Am J Gastro 2007

Imaging
Abdominal
ultrasound should
be performed in all
patients to assess
gallstone
pancreatitis
Guideline
recommendation:
Strong
recommendation,
low quality of
evidence

Tenner. Am J Gastro. 2013


http://upload.wikimedia.org/wikipedia/commons/8/8a/Gallstone_0414092027109.jpg

489
Copyright © Harvard Medical School, 2018. All Rights Reserved.

Etiology
Common Uncommon/Rare
Gallstone Vascular/vasculitis
Alcohol CTD, PPT
Hypertriglyceridemia Cancer
ERCP Hypercalcemia
Trauma Pancreas Divisum
Drugs (AZA, 6-MP, Hereditary pancreatitis
sulfonamide, estrogen, Infections (mumps, coxsackie
tetracycline, valproic acid, virus, CMV, echovirus,
HAART) parasites)
SOD Autoimmune (Sjogren, AIP)

Tenner. Am J Gastro. 2013


Yadav. J Clin Gastroenterol. 2003

Severity -Mild
75-80%
Absence of organ failure and pancreatic
necrosis
Substantial improvement by 48 hours

Banks, Bollen, Dervenis. Classification of acute pancreatitis- 2012: revision of the


Atlanta classification and definitions by international consensus. Gut. 2012.

490
Copyright © Harvard Medical School, 2018. All Rights Reserved.

Moderately Severe AP
Local or systemic complications without
persistent organ failure
– Necrotizing Pancreatitis
– Acute Peripancreatic Fluid Collection (APFC)
– Acute Necrotic Collection (ANC)

Transient organ failure (<48h)

Banks, Bollen, Dervenis. Classification of acute pancreatitis- 2012: revision of the


Atlanta classification and definitions by international consensus. Gut. 2012.

Severe AP
Persistent organ failure (>48h)
– Single organ failure
– Multiple organ failure

Banks, Bollen, Dervenis. Classification of acute pancreatitis- 2012: revision of the


Atlanta classification and definitions by international consensus. Gut. 2012.

491
Copyright © Harvard Medical School, 2018. All Rights Reserved.

Risk Stratification
Serial bedside evaluation
– Vitals, orthostatics
– Volume overload, ARDS
RANSON > 3
APACHE II > 8
HAPS
CRP : 48- 72h to become accurate
– CRP > 150
BISAP
PASS

Lankisch. Clin Gastro Hep. 2009


Tenner. Am J Gastro. 2013

Initial Assessment
• Goal directed therapy using isotonic crystalloid solution (
LR or NS)
– Conditional recommendation, low quality of evidence
• Crystalloid replacement fluid is prefered over
colloid
– Conditional recommendation, low quality of evidence
• Early aggressive IV hydration is most beneficial in the first
12-24h, and may be of little benefit beyond
– Conditional recommendation, moderate quality of evidence
Tenner. Am J Gastro. 2013
Crocketts. Gastro. 2018

492
Copyright © Harvard Medical School, 2018. All Rights Reserved.

Nutrition
In AP, AGA recommends early oral feeding ( within 24h)
– Strong recommendation, moderate quality (AGA 2018)
In mild AP initiation of diet with low-fat diet appears as safe
as clear liquid diet
– Conditional recommendation, moderate quality (ACG 2013)
If oral intake not tolerated, enteral nutrition is recommended.
Parenteral nutrition should be avoided unless EN is not
available, not tolerated, or not meeting the caloric
requirements
– Strong recommendation, moderate quality (AGA 2018)
NG and NJ delivery of enteral feeding appear comparable in
efficacy and safety
– Strong recommendation, moderate quality (AGA 2018)

Tenner. Am J Gastro. 2013


Crocketts. Gastro. 2018

PYTHON trial
Multicentric, RC superiority T early enteral
feed vs. oral diet at 72 hour.
N=208 with APACHE II score ≥8
Outcome: composite outcome of major
infection or death at 6 month
30% enteral feeding vs. 27% in on
demand group
– ( OR=1.07 CI: 0.79-1.44)
– Mortality 11 EN vs. 7 on demand % (p=0.33)

Bakker. NEJM. 2014

493
Copyright © Harvard Medical School, 2018. All Rights Reserved.

ERCP in AP
Patients with AP and concomitant cholangitis
should undergo ERCP within 24h of admission
– Strong recommendation, moderate quality
ERCP is not needed in most patients with gallstone
pancreatitis who lack laboratory or clinical
evidence of ongoing biliary obstruction
– Conditional recommendation, low quality
In absence of cholangitis and/or jaundice, MRCP
or EUS should be used to screen for CBD stone if
highly suspected
– Conditional recommendation, low quality
Tenner. Am J Gastro. 2013
Crocketts. Gastro. 2018

Role of Antibiotics
Should be given for an extrapancreatic
infection such as cholangitis
– Strong recommendation, high quality
Routine use of prophylactic antibiotics in
patients with predicted severe acute
pancreatitis is not recommended
– Strong recommendation, moderate quality

Tenner. Am J Gastro. 2013


Crocketts. Gastro. 2018

494
Copyright © Harvard Medical School, 2018. All Rights Reserved.

Infected Necrosis
Infected necrosis should be considered in patients
with necrosis who deteriorate or fail to improve after
7-10 days. CT-FNA should be done prior to ATBx if
possible
– Strong recommendation, low quality
In infected necrosis, the use of antibiotics known to
penetrate pancreatic necrosis, such as carbapenem,
quinolones, and metronidazole, may be useful in
delaying or avoiding intervention
– Conditional recommendation, low quality
Routine administration of antifungal along with
prophylactic or therapeutic ATBx is not recommended
– Conditional recommendation, low quality

Tenner. Am J Gastro. 2013

Role of Surgery
In biliary AP, cholecystectomy should be performed
before discharge to prevent recurrent AP
– Strong recommendation, moderate quality (AGA 2018)
In patients with necrotizing gallstone pancreatitis, in
order to prevent infection, cholecystectomy should be
deferred until active inflammation subsides and fluid
collections resolve or stabilize
– Strong recommendation, moderate quality (ACG 2013)
The presence of asymptomatic pseudocysts and
pancreatic, or extrapancreatic necrosis do not warrant
intervention, regardless of the size, location, extension
– Strong recommendation, moderate quality (ACG 2013)

Tenner. Am J Gastro. 2013


Crocketts. Gastro. 2018

495
Copyright © Harvard Medical School, 2018. All Rights Reserved.

Infected Necrosis
Randomized prospective trial – 19 hospitals, 88
patients
– primary open necrosectomy
– step-up approach
• percutaneous or endoscopic drainage
• minimally invasive retroperitoneal necrosectomy
Advantages of step-up approach
– 35% - did not require retroperitoneal necrosectomy
– less new-onset multiple organ failure
– less diabetes
– less need for pancreatic enzymes
– less costly
No decreased mortality (19% vs. 16%)

van Santvoort et al. NEJM 2010; 362: 1491-1502

Acute Alcoholic Acute


Pancreatitis
AGA recommends brief alcohol
intervention during admission
– Strong recommendation, moderate quality
(AGA 2018)
– RCT comparing repeated-6 months interval
intervention for 2 years vs single intervention
during admission.
– No statistical significant difference but a trend
towards reduction in hospitalization
Crocketts. Gastro. 2018
Nordback. Gastro. 2009

496
Copyright © Harvard Medical School, 2018. All Rights Reserved.

CHRONIC PANCREATITIS

Definition
Irreversible damage to the pancreas and
development of histologic evidence of
inflammation and fibrosis with eventual
loss of pancreatic exocrine and endocrine
tissue
Calcifications

Fibrosis

Inflammation

497
Copyright © Harvard Medical School, 2018. All Rights Reserved.

Epidemiology
Incidence 3-12/100,000
Prevalence 50/100,000
Men > Women
Median age of diagnosis 30-40 years
old
80,000 yearly admission in US
10 year survival : 70%

Clinical Presentation

Bloating, Diabetes
cramping Recurrent
pancreatitis
Gas, foul
smelling,
oily stool Nausea,
Chronic Vomiting
Pancreatitis
Stool
frequency Abdominal
pain
Weight
Back
loss
pain

498
Copyright © Harvard Medical School, 2018. All Rights Reserved.

Pain Pattern
No Pain
Usually pain
16%
free, but
episodes of
mild to
moderate pain
Constant 13%
severe pain
Constant mild
4%
to moderate
pain
Constant mild 4%
to moderate [CATEGORY
pain with NAME]
epsiodes of [PERCENTAGE]
severe pain
44%
C. Mel Wilcox, Clin Gastro Hep. 2015

Laboratory
IgG4 in
Autoimmune
Pancreatitis
Low fecal
Lipase elastase
Chronic
Pancreatitis

Elevated
Low albumin Hba1c

499
Copyright © Harvard Medical School, 2018. All Rights Reserved.

STEP 1: CT scan

D Conwell, Pancreas 2014

Differential diagnosis
Calcifications Atrophy
– Islet cell tumor - NET – Aging
– Serous cystadenoma – Prior necrotizing
– Mucinous cystic pancreatitis
neoplasm – Cystic Fibrosis
– Solid pseudopapillary – Shwachman-
neoplasm Diamond syndrome
– Granulomatous – Hemochromatosis
infection
– Hyperparathyroidism

500
Copyright © Harvard Medical School, 2018. All Rights Reserved.

STEP 2: MRI/MRCP with or without secretin

STEP 3: EUS
EUS criteria
Clin Gastro Hep,
D Conwell, 2012

Parenchymal Abnormalities Ductal Abnormalities


– Hyperechoic foci (C) – Main duct dilation
– Hyperechoic stranding (A) – Duct irregularities (D)
– Cysts – Side branch dilation
– Lobularity of contour of – Stones (E)
gland – Hyperechoic duct margins
(B)

501
Copyright © Harvard Medical School, 2018. All Rights Reserved.

STEP 4: Pancreatic Function Test

Direct pancreas function test.


A cholecystokinin B secretin
Clin Gastro Hep, D Conwell, 2012

Conditions associated with


pancreatic exocrine insufficiency
Condition Presumed mechanism
Chronic pancreatitis Acinar cell loss or injury
Severe acute pancreatitis Acinar cell loss
Shwachman-Diamond syndrome Acinar cell dysfunction
Pancreatic surgery Acinar cell removal
Intestinal surgery Asynchronous delivery of
enzymes with meals
Ductal dysfunction Cystic fibrosis
Duct blockage Ductal carcinoma, IPMN,
stricture
Zollinger-Ellison syndrome Enzyme destruction
C.E. Forsmark, Gastroenterology. 2013

502
Copyright © Harvard Medical School, 2018. All Rights Reserved.

Determination of Etiology
TIGAR-O
Toxic-Metabolic
Idiopathic
Genetic
Autoimmune
Recurrent/severe AP
Obstructive
Whitcomb. Gastroenterology, 2001

Hereditary factors

TRYPSINOGEN TRYPSIN SPINK-1


PRSS-1
TAP

Trypsinogen Trypsin

503
Copyright © Harvard Medical School, 2018. All Rights Reserved.

Autoimmune Pancreatitis
Diffusely enlarged pancreas
Featureless borders: Sausage-shape
Delayed enhancement w/w/o capsule-like rim

Courtesy Dr Sainani

Assess complications
Pancreatic cancer
Overall lifetime risk is 4%, increased if smoke,
coexisting diabetes and in hereditary pancreatitis
Pseudocyst
Pancreatic ascites and pleural effusion
Bile duct obstruction
– From inflammatory mass in the head
Duodenal obstruction
Osteopenia / Osteoporosis
– similar risk as IBD, in exocrine insufficiency

504
Copyright © Harvard Medical School, 2018. All Rights Reserved.

Assess for complications


Gastroparesis
Small Bowel Bacterial Overgrowth
– Multifactorial: intestinal motility from pain and
opioids, inflammatory changes, obstruction,
surgically altered anatomy
Variceal bleed and splenic vein
thrombosis
Pseudoaneurysm

Therapy
Measure pain, quality of life
Counsel smoking and alcohol cessation
Nutrition counselling, Calcium and Vitamin D
supplement, baseline Bone Mineral Density
Analgesia- start tylenol, NSAIDS, tramadol
Adjunctive therapy for those with worsening
pain, increasing requirements
– Pregabalin, Gabapentin, SSRI, SNRI
Assess exocrine and endocrine function: fecal
elastase or serum trypsin, Hb1ac , GTT

Clin Gastro Hep, D Conwell, 2012

505
Copyright © Harvard Medical School, 2018. All Rights Reserved.

Pancreatic Enzymes
Replacement Therapy
Normal pancreas produces 900 K per
meal, 10% needed for normal
digestion
Start 50-75 K /meal and 20-35 K/
snack
Temperature-sensitive
Enzyme degradation over time
Limited time of effect

Side Effects of Pancreatic


Enzymes Replacement
Abdominal pain, gas
Headache
Mouth ulcers possible
Hypersensitivity to porcine product
Hyperuricemia
Bowel perforation and obstruction
Fibrosing colonopathy
– Theoretical risk : concern > 6,000 IU/kg/day

506
Copyright © Harvard Medical School, 2018. All Rights Reserved.

Endoscopic and surgical


treatment
Inflammatory mass in pancreatic head
Assess anatomy
Whipple operation, Berger, Frey or Berne operation

Dilated pancreatic duct


Endoscopic therapy: Pancreatic and biliary sphincterotomy,
Stricture dilation and stenting, lithotripsy, stone extraction
Surgical Therapy: Modified Puestow

Small duct disease


Medical management
Surgical options: V-plasty, total pancreatectomy with islet
cell auto transplantation
Clin Gastro Hep, D Conwell, 2012

Take Home Points


Acute Pancreatitis
– 2/3 criteria for diagnosis
– Ultrasound to assess for gallstones
– Wait after initial 48-72h to obtain a CT scan because necrosis
can take longer to appear
– Favor enteral route
Chronic Pancreatitis
– Think of the etiology as it is not just caused by alcohol
– Autoimmune pancreatitis can be treated with steroids
– Remember the diagnostic and treatment approach according to
the anatomy
– Monitor for complication and treat accordingly

507
Copyright © Harvard Medical School, 2018. All Rights Reserved.

Question 1
You are admitting a patient with acute
necrotizing alcoholic pancreatitis. What
nutrition do you prescribe?
a) Parenteral nutrition in 1 week
b) Enteral nutrition via nasogastric tube now
c) Oral feeding as tolerated using low fat
diet
d) Enteral nutrition via nasojejunal tube in 3
days

Question 1
c ) Oral feeding as tolerated using low fat
diet

Python trial and AGA recommendation

508
Copyright © Harvard Medical School, 2018. All Rights Reserved.

Question 2
What gene is most associated with
hereditary pancreatitis and what genetic
inheritance pattern does it follow
(depicted below)
a) PRSS1, autosomal dominant
with incomplete penetrance
b) CTRC, autosomal recessive
c) CFTR, mitochondrial
inheritance
d) SPINK1, X-linked recessive

Question 2
a) PRSS1, autosomal dominant
with incomplete penetrance

509
Copyright © Harvard Medical School, 2018. All Rights Reserved.

References
Crockett, SD. American Gastroenterological
Association Institute Guideline on Initial
Management of Acute Pancreatitis.
Gastroenterology 2018;154:1096-1101
Ito, T. Evidence-based clinical practice
guidelines for chronic pancreatitis 2015. J
Gastroenterol 2016;51:85-92
Majumder, S. Chronic Pancreatitis. Lancet
2016, May7; 387 (10031): 1957-66

510
Copyright © Harvard Medical School, 2018. All Rights Reserved.

Psychiatry Overview

Hepatitis B and C
M. Valerie Lin, MD
Transplant Hepatologist
Division of Transplantation, Department of Surgery
Lahey Hospital and Medical Center
Tufts University School of Medicine

Disclosure
• Gilead HBV advisory board

511
Copyright © Harvard Medical School, 2018. All Rights Reserved.

HEPATITIS B

Learning Objectives
• Understand the natural history (phases of
infection) of chronic HBV
• Recognize when to initiate treatment for HBV
• Identify currently approved drugs for HBV and
their efficacy
• Describe the novel drug targets and inhibitors
currently in pipeline

512
Copyright © Harvard Medical School, 2018. All Rights Reserved.

Case
34 year old Asian male who was tested positive
for HBsAg. Additional labs showed positive anti-
HBc, positive HBeAg, HBV DNA > 170m IU/mL
and normal LFT. Clinically asymptomatic.

1) Which phase of HBV infection?


2) Is treatment indicated?

Serological Tests for HBV and


Interpretation

Source: AASLD – LiverLearning

513
Copyright © Harvard Medical School, 2018. All Rights Reserved.

Phases of Chronic Hepatitis B

Immune Tolerant CHB

514
Copyright © Harvard Medical School, 2018. All Rights Reserved.

HBeAg Positive “Active” CHB

Immune Inactive CHB

515
Copyright © Harvard Medical School, 2018. All Rights Reserved.

HBeAg Negative “Active” CHB

Resolved CHB

516
Copyright © Harvard Medical School, 2018. All Rights Reserved.

Interpretation of HBV Serology


Status HBsAg HBeAg HBc IgM HBc IgG HBs Ab HBe DNA LFT
Ab
Immune Tolerance + + - + - - +++ N

Active infection + + + (acute) - (acute) - - +++


- (chronic) + (chronic)

Carrier + - - + - +/- + or UD N

E-Ag neg chronic HBV + - - + - - ++

Resolution - - - + + - UD N

Window Period - - + - - - +++

Immunization - - - - + - - N

Algorithm to Identify HBV


Treatment Candidates

AASLD Practice Guidelines

517
Copyright © Harvard Medical School, 2018. All Rights Reserved.

Current Available HBV Treatment


Medication Administration Dosing Effectiveness* Risk of
Resistance
First line
Entecavir Oral 0.5mg daily >90% in naïve <1% in naïve
patients 30% at 5 yrs in lam-
experienced pts

Tenofovir Oral 25mg or 300mg >90% <1%


daily

Peg-interferon Injectable 180ug weekly 30% None

Second line
Lamivudine Oral 100mg daily <50% 70% at 5 years

Adefovir Oral 10mg daily <50% 30-40%

Telbivudine Oral 600mg daily <70% 30% at 5 years

Responses to HBV Treatment


• Virologic/Serologic Response
– Decrease in HBV DNA
– HBeAg loss and sero-conversion to anti-Hbe
– HBsAg loss
• Biochemical response
– Normalization of ALT
• Histological response
– Decrease in hepatic inflammation

518
Copyright © Harvard Medical School, 2018. All Rights Reserved.

HBV DNA Reduction After 1 Year


of Treatment

Scaglione SJ, Lok AS. Gastro 2012; 142: 1360-1368

Rate of HBeAg Seroconversion and


HBsAg Loss after 1 to 5 years of Rx
HBeAg Seroconversion HBsAg Loss

519
Copyright © Harvard Medical School, 2018. All Rights Reserved.

Novel Drug Targets and Inhibitors

Liver International 2017; 37(Suppl. 1): 33–39

Case
34 year old Asian male who was tested positive
for HBsAg. Additional labs showed positive anti-
HBc, positive HBeAg, HBV DNA > 170m IU/mL
and normal LFT. Clinically asymptomatic.

1) Which phase of HBV infection? Immune


Tolerant
2) Is treatment indicated? No

520
Copyright © Harvard Medical School, 2018. All Rights Reserved.

Take Home Points


• Age of infection influences natural history
• The phases of HBV disease help determine treatment
needs
• There are 8 approved HBV treatments - 2 types of
interferons and 6 nucleos/tide analogues
• Currently available HBV treatment are effective in
HBV suppression but not eradication
• Entecavir and tenofovir are the preferred treatment
because of low risk of drug resistance

HEPATITIS C

521
Copyright © Harvard Medical School, 2018. All Rights Reserved.

Learning Objectives
• Describe the recommendations for HCV
testing
• Understand the nature history of HCV
• Describe the goal of HCV treatment
• Understand the general concept of prescribing
HCV therapy

CDC and USPSTF


Recommendations for HCV Testing
• Risk-based
– Injection or intranasal drug users
– Received blood products prior to 1992
– Exposure to hemodialysis
– Infants born to HCV-infected mothers
– Unregulated tattoo
– History of incarceration
• Medical Indications
– Persistently elevated ALT
– HIV
• Birth Cohort
– One-time test for persons born 1945-1965

Ly et al. Ann Int Med 2012

522
Copyright © Harvard Medical School, 2018. All Rights Reserved.

Testing Sequence For Identifying


Active HCV Infection

CDC. Testing for HCV infection: MMWR 2013;62(18)

HCV Natural History


Acute hepatitis C
55 - 85%
Chronic infection
70%
Chronic hepatitis 1 - 4%/yr
20% HCC
Cirrhosis
Decompensation
Time 4 - 5%/yr
(yr)
10 20 30

523
Copyright © Harvard Medical School, 2018. All Rights Reserved.

Factors Associated with Faster


Fibrosis Progression
Non-modifiable Modifiable

HCV Can Be Cured


• Cure is defined by
HCV Life Cycle
sustained virological
response (SVR)
• I.e. undetectable HCV
RNA at least 3 months
post-Rx
• SVR is associated with:
– ALT normalization
– Disease progression
– Improved mortality

524
Copyright © Harvard Medical School, 2018. All Rights Reserved.

FDA Approved Oral HCV Direct


Acting Anti-virals (DAAs)

AASLD HCV modules

Case
50 year old man with history of hyperlipidemia (on
simvastatin) and GERD (on omeprazole) presented to
discuss treatment for chronic HCV. He is treatment
naïve. Lab showed genotype 1a, HCV RNA 1 million
IU/ml. Liver elastography showed stage 1 fibrosis.

1) Which HCV Rx would you recommend? And for how


long?
2) What if he has stage 3-4 fibrosis?

525
Copyright © Harvard Medical School, 2018. All Rights Reserved.

General Principles of Prescribing Oral


DAAs
• Regimen selection
• Drug-drug interaction Genotype • 1 to 6
• Monitoring
– Side effects
– Labs Prior Rx • Naïve
Experience • Rx Experienced

Fibrosis • Stage 1-3


Stage • Cirrhosis

Commonly Used DAA


Combinations
Generic Type of inhibitor Brand Name

Ledipasvir + Sofosbuvir NS5A + NS5B Harvoni


Velpatasvir + Sofosbuvir NS5A + NS5B Epclusa
Velpastasvir + Sofosbuvir + Voxilaprevir NS5A + NS5B + PI Vosevi
Elbasvir + Grazoprevir NS5A + PI Zepatier
Pibrentasvir + Glecaprevir NS5A + PI Mavyret

NS5A = NS5A inhibitor


NS5B = NS5B Nucleoside inhibitor
PI = Protease inhibitor

AASLD HCV modules

526
Copyright © Harvard Medical School, 2018. All Rights Reserved.

Case
50 year old man with history of hyperlipidemia (on
simvastatin) and GERD (on omeprazole) presented to
discuss treatment for chronic HCV. He is treatment
naïve. Lab showed genotype 1a, HCV RNA 1 million
IU/ml. Liver elastography showed stage 1 fibrosis.
1) Which HCV Rx would you recommend? And for how
long? All the DAAs listed in the table for either 8 or
12 weeks
2) What if he has stage 4 fibrosis? Extend Rx duration
to 12 weeks

Things to Note When Prescribing HCV


DAAs
• Regimen selection
• Drug-drug interaction
• Monitoring
– Side effects
– Labs

527
Copyright © Harvard Medical School, 2018. All Rights Reserved.

Things to Note When Prescribing HCV


DAAs
• Regimen selection
• Drug-drug interaction
• Monitoring CNS GI
Headache Nausea
– Side effects
– Labs
Fatigue Diarrhea

Insomnia

Things to Note When Prescribing HCV


DAAs
• Regimen selection • CBC, LFTs, Cr/GFR
• Drug-drug interaction Pre-Rx • HCV RNA and Genotype
• HBsAg and HBcAb
• Monitoring
– Side effects During
• CBC, LFT, Cr/GFR
– Labs Rx • HCV RNA
(week 4)

• HCV RNA at end of Rx


Post- Rx • HCV RNA at 12 weeks
post-Rx

528
Copyright © Harvard Medical School, 2018. All Rights Reserved.

Post-Marketing HBV Reactivation


with DAA Regimens
• 29 cases identified
• Occurred within 4-8 weeks (mean 53 days)
• Heterogeneous in HCV genotype, DAA received and
baseline HBV parameters
• Outcomes:
– 2 fatalities and 1 liver transplant
– 16 pts received HBV treatment (usually delayed)
• FDA advised to screen HBV pre-therapy and monitor
during and post- therapy

Bersoff-Matcha SJ, et al. Ann Int Med June 2017

Take Home Messages


• Chronic HCV infection leads to cirrhosis and HCC
• HCV direct-acting antivirals (DAAs) are highly
effective and well-tolerated
• The main hurdle for HCV treatment is linkage of care
• When selecting HCV regimen, consider genotype,
prior treatment experience and fibrosis stage
• HBV reactivation could occur in patients with HBV
coinfection, and these patients needs to be
monitored closely or prophylactically treated
• Refer to HCV Guidelines ww.hcvguidelines.org

529
Copyright © Harvard Medical School, 2018. All Rights Reserved.

Question 1
Which of the following is an indication for
stopping HBV treatment?
A. After completing 6 months interferon and serum HBV DNA is
undetectable
B. When serum HBV DNA becomes undetectable in patients receiving
nucleos/tide analogue
C. When HBsAg is cleared (HBsAg-) in a non-cirrhotic patients receiving
nucleos/tide analogue
D. After completing 5 years nucleos/tide analogue treatment
regardless of response
E. After completing 5 years interferon treatment regardless of
response

Question 1
Which of the following is an indication for
stopping HBV treatment?
A. After completing 6 months interferon and serum HBV DNA is
undetectable
B. When serum HBV DNA becomes undetectable in patients receiving
nucleos/tide analogue
C. When HBsAg is cleared (HBsAg-) in a non-cirrhotic patients receiving
nucleos/tide analogue
D. After completing 5 years nucleos/tide analogue treatment
regardless of response
E. After completing 5 years interferon treatment regardless of
response

530
Copyright © Harvard Medical School, 2018. All Rights Reserved.

Question 2
Which of the following is associated with the
higher risk for liver-related complications
secondary to HCV?
A. HCV genotype 2
B. Obesity
C. Immune compromise
D. Histology with advanced fibrosis

Question 2
Which of the following is associated with a
higher risk for liver-related complications
secondary to HCV?
A. HCV genotype 2
B. Obesity
C. Immune compromise
D. Histology with advanced fibrosis

531
Copyright © Harvard Medical School, 2018. All Rights Reserved.

CHRONIC LIVER DISEASE AND ITS


COMPLICATIONS

Anna Rutherford, MD, MPH


Clinical Director of Hepatology
Division of Gastronterology, Hepatology & Endoscopy
Brigham and Women’s Hospital
Assistant Professor of Medicine
Harvard Medical School

I have no disclosures relevant to this presentation.

532
Copyright © Harvard Medical School, 2018. All Rights Reserved.

Learning Objectives

• Natural history and outcome of cirrhosis

• Initial management of complications of cirrhosis

• Appropriate timing of referral for liver transplant

Chronic Liver Compensated Decompensated Death


Disease Cirrhosis Cirrhosis

Ascites
Jaundice
Encephalopathy
Variceal Hemorrhage

533
Copyright © Harvard Medical School, 2018. All Rights Reserved.

Probability of Hepatic Decompensation:


58% over 10 years

Ascites
Jaundice
Encephalopathy
Variceal bleeding

Hepatology 1987; 7: 122-128

Risk Factors for Hepatic Decompensation

• GI Bleeding
• Infection
• Alcohol intake
• Medications
• Dehydration
• Constipation
• Obesity (Hepatology 2011; 54: 555)

534
Copyright © Harvard Medical School, 2018. All Rights Reserved.

Hepatic Decompensation Reduces Survival

9 years

1.6 years

Hepatology 1987; 7: 122-128

Prediction of 3-Month Survival in Cirrhotics

MELD score = 3.8 ln (bilirubin) + 11.2 ln (INR) + 9.6 ln (creatinine) + 6

535
Copyright © Harvard Medical School, 2018. All Rights Reserved.

Prediction of 1 Year Survival in Cirrhotics:


Child Turcotte Pugh Score
Points 1 2 3
Encephalopathy None 1 and 2 3 and 4

Ascites Absent Slight Moderate


Bilirubin (mg/dL) 1-2 2-3 >3
Albumin (g/dL) 3.5 2.8-3.5 <2.8
INR <1.7 1.7-2.3 >2.3

5-6 A 100% 1 year survival


7-9 B 80% 1 year survival
10-15 C 45% 1 year survival

Natural History of Cirrhotic Ascites


Portal Hypertension
No Ascites

Uncomplicated
Ascites

Ascites +
Hyponatremia

Refractory Ascites

536
Copyright © Harvard Medical School, 2018. All Rights Reserved.

Management of Ascites
• 50% of compensated cirrhotics will develop ascites 10 years
from diagnosis
• Ascites most common complication of cirrhosis that leads
to hospital admission
• 15% 1 year mortality, 44% 5 year mortality after first
appearance of ascites
• New-onset ascites requires diagnostic paracentesis
• Bleeding complications in less 1/1,000 who require
paracentesis
• Use of blood products (FFP/platelets) not data supported
• SAAG of ≥ 1.1 is 97% accurate for portal hypertension

AASLD Guidelines: Hepatology 2013; 57: 1651–1653.

Treatment Options for Cirrhotics with Ascites


• Cessation of Alcohol use
• Sodium restricted diet (2000mg/day) and education
• Dual diuretics: Spironolactone and Furosemide, orally with single daily dosing
• Most patients do not need fluid restriction
• Ratio Spironolactone 100mg: Furosemide 40mg, increase every 3-5 days
• Maximum doses are Spironolactone 400mg: Furosemide 160mg
• Amiloride 10-40mg substituted for Spironolactone for tender gynecomastia
• Consider stopping NSAIDS, ACE Inhibitors, Angiotensin Receptor blockers,
Propranolol
• Liver transplant evaluation

537
Copyright © Harvard Medical School, 2018. All Rights Reserved.

Management of Refractory Ascites

• 10% of cirrhotics with ascites

• Unresponsive to 2g sodium diet and high dose diuretics


OR
Clinically significant complications of diuretics (encephalopathy, creatinine > 2g/dL,
sodium < 120 mmol/L, potassium > 6 mmol/L)

• Options include serial LVPs vs. TIPS liver transplant

• In LVP ≥ 5L, albumin infusion of 6-8g/L removed improves survival and prevents post-
paracentesis circulatory dysfunction

• Use of nonselective beta blockers may be associated with increased mortality in


patients with refractory ascites

Hepatology 2012; 55: 1172-1181.


J Hepatology 2014; 60: 643.

Large Scale Randomized Controlled Trials of TIPS


vs. LVP for Refractory Ascites
N Control of Ascites Survival Encephalopathy
60 61% vs. 18% (p=.006) 69% vs. 52% (p=.11) 58% vs. 48%*

70 51% vs. 17% (p=.003) 41% vs. 35% (p=.29) 77% vs. 66%

109 58% vs. 16% (p<.001) 40% vs. 37%* 60% vs. 34% (p=.058)

66 79% vs. 42% (p=.001) 77% vs. 52% (p=.021) Severe (p=0.39)

* P value not significant.

AASLD Guidelines: Hepatology 2013; 57: 1651–1653

538
Copyright © Harvard Medical School, 2018. All Rights Reserved.

Natural History of Cirrhotic Ascites


Portal Hypertension
No Ascites

Uncomplicated
Ascites

Ascites +
Hyponatremia

Refractory Ascites

Hepatorenal
Syndrome

Hepatorenal Syndrome – clinical features

• Cirrhosis with ascites


• Serum creatinine > 1.5mg/dL
• No creatinine improvement after 2 days diuretic
withdrawal
• No creatinine improvement after volume expansion
with albumin (1g/kg up to 100g)
• Absence of shock, nephrotoxins
• Bland urine sediment/no parenchymal kidney
disease
AASLD Guidelines: Hepatology 2013; 57: 1651–1653.

539
Copyright © Harvard Medical School, 2018. All Rights Reserved.

Natural History of Hepatorenal Syndrome


6
Type 2 HRS SBP Type 1 HRS
5

4
Therapeutic
Creatinine paracenteses Cefotaxime
(mg/dL) 3

0
0 -6 -4 -2 0 1 2 3
Months Weeks

Gastroenterology 2002; 122:1658.

Hepatorenal Syndrome – management

• Treatment of underlying liver disease (alcoholic hepatitis, HBV)


• Prevention with albumin infusion in SBP
• Cessation of nonselective beta blockers in SBP
• Albumin/Octreotide/Midodrine
• In ICU, Norepinephrine (or Vasopressin) + Albumin
• Terlipressin – not approved in U.S.
• Hemodialysis as bridge to liver recovery or transplant
• TIPS
• Liver transplantation

540
Copyright © Harvard Medical School, 2018. All Rights Reserved.

Natural History of Cirrhotic Ascites


Portal Hypertension
No Ascites

Uncomplicated
Ascites

Ascites + SBP
Hyponatremia

Refractory Ascites

Hepatorenal
Syndrome

Spontaneous Bacterial Peritonitis:


Treatment
Systemic antibiotics for Community Acquired SBP
• Ceftriaxone or Cefotaxime
• Beta lactam/beta-lactamase inhibitor combination
• Avoid aminoglycosides
• Most patients will respond to 5 day course of treatment

Cessation of nonselective beta blockers

Albumin IV on Day 1 and Day 3 with any of following:


• BUN > 30mg/dL
• Creatinine > 1.0 mg/dL
• Serum bilirubin > 4mg/dL
Best Pract Res Clin Gastroenterol 2007; 21: 77-93.
Gastroenterology 2014; 146: 1680-1690.

541
Copyright © Harvard Medical School, 2018. All Rights Reserved.

Spontaneous Bacterial Peritonitis:


Primary & Secondary Prophylaxis

• Prior episode of SBP

• Ascites total protein <1.5/dL AND


Creatinine ≥ 1.2, BUN ≥ 25 or serum Na ≤130

• Child score ≥ 9 or Bilirubin ≥ 3

Gastroenterology 2007; 133: 818-824.

Treatment of Hepatic Encephalopathy

• Determine precipitant of hepatic encephalopathy and treat


(Infection, Electrolytes, GI Bleeding, Constipation, Dehydration, Sedatives)
• Lower ammonia level (only if elevated in setting of altered
mental status): Lactulose Rifaximin
• No need to follow blood ammonia levels
• Do not restrict protein: Maintain dietary protein intake of 1.2
to 1.5g/kg/day
• Probiotics may be as effective as Lactulose in treatment of
chronic hepatic encephalopathy

542
Copyright © Harvard Medical School, 2018. All Rights Reserved.

Management of Gastroesophageal Varices

• GEV present in 50% cirrhotics: 30-40% compensated, 85% decompensated


• In compensated cirrhotics, varices develop at rate of 7-8%/year
• LS < 20kPA and platelet count > 150,000 very unlikely to have GE varices
• EGD to screen for gastroesophageal varices recommended in anyone with
new diagnosis of cirrhosis
• If no varices on original EGD, repeat every 2 years with ongoing liver injury
(obesity, alcohol) or 3 years (abstinence, viral elimination)
• Small varices (grade 1) on original EGD repeat 1-2 years
• EGD at time of other clinical decompensation (ascites, encephalopathy)
• If cirrhosis but no varices prevent clinical decompensation
• Grade 1 varices Non-selective beta blocker
• Grade 2-3 varices Non-selective beta blocker or variceal ligation (not both)

Hepatology 2017; 65: 310-332.

Management of Acute Variceal Hemorrhage


• Admit to ICU
• Blood volume resuscitation
Hemoglobin ~7 g/dL > 9g/dL: Risk of Rebleeding 11% vs. 22% in Cirrhotics (p=0.02)
NEJM 2013; 368: 11-21

• Short-term antibiotic prophylaxis (maximum 7 days)


Decreases rebleeding and improves survival
IV Ceftriaxone 1g/day >> Norfloxacin 500mg BID in Childs B/C
Gastroenterology 2006; 131: 1049-1056

• Somatastatin, Octreotide, Vapreotide for 3-5 days


• Upper endoscopy within 12 hours

• TIPS
AASLD Guidelines: Hepatology 2007; 46: 922-938.

543
Copyright © Harvard Medical School, 2018. All Rights Reserved.

Early TIPS An Option for Variceal Hemorrhage

• 63 Child B/C cirrhotics with acute variceal bleeding


• All had vasoactive drugs and endoscopic therapy, then
randomized:
• 32 underwent TIPS within 72 hours of admission
• 31 continued vasoactive drugs for 3-5 days
Nadolol/Propranolol and continued banding
• 7/31 needed rescue TIPS
• Median follow-up 16 months
• At 1 year, episodes of encephalopathy 18% TIPS vs.
10% pharmacologic (p=.80)
• No significant differences in adverse events

NEJM 2010; 362: 2370-2379.

Transjugular Intrahepatic Portosystemic Shunt

Hepatic Vein

TIPS

Splenic
Portal Vein Vein

Superior Mesenteric
Vein

544
Copyright © Harvard Medical School, 2018. All Rights Reserved.

Early TIPS An Option for Variceal Hemorrhage

Rebleeding Survival

97% vs. 50% 86% vs. 61%

NEJM 2010; 362: 2370-


2379.

Management of Bleeding Gastric Varices

• Vasoactive drugs, restrictive transfusion, antibiotic prophylaxis


• Banding gastric varices can be technically difficult (IGV1> GOV2>
GOV1)
• Cyanoacrylate glue injection
-Polymerizes into firm clot within varix
- Risk of distal embolization
- Not approved by FDA for use in US, center dependent expertise
• TIPS
• Balloon-occluded retrograde obliteration (BRTO)
- Balloon catheter in draining vessel then instill sclerosant/sponge
- 90% long-term bleeding control
- Can increase portal pressure: worsen esophageal varices, ascites

545
Copyright © Harvard Medical School, 2018. All Rights Reserved.

Balloon Retrograde Transcatheter Obliteration

Optimal Timing of Referral for LT


Clinical Decompensation + Biochemical Decompensation (MELD >15)

- Encephalopathy
- Ascites
- Variceal Hemorrhage or chronic GI bleed from portal hypertensive gastropathy
- Hepatocellular Carcinoma
- Hepatorenal syndrome
- Hepatopulmonary syndrome or Portopulmonary Hypertension

Other considerations:
- Acute Liver Failure
- Poor quality of life or Recurrent, resistant infections in PSC/PLD
- Liver based metabolic conditions with systemic manifestations

546
Copyright © Harvard Medical School, 2018. All Rights Reserved.

Timing: What is MELD?

• Model for End-stage Liver Disease


• Originally created to predict short term mortality
post TIPS
• Basis for liver allocation in U.S. since 2/2002
• MELD-Sodium used since 1/2016
• 4 objective lab tests (Sodium, Total bilirubin, Creatinine, INR)
• Highly predictive of 3-month mortality in cirrhotics
• MELD of 15 is threshold patient survival with
transplantation > survival without transplantation

Liver Disease Etiology of Adult Transplant Recipients

HCV
Malignancy
Alcohol
Others
Cholestatic
ALF

SRTR.transplant.hrsa.gov 2012

547
Copyright © Harvard Medical School, 2018. All Rights Reserved.

Exclusions for Liver Transplantation


MELD Score <15
Severe cardiac or pulmonary disease
AIDS
Ongoing alcohol or illicit substance abuse
HCC with metastatic spread
Uncontrolled sepsis
Anatomic abnormality that precludes liver transplant
Intrahepatic cholangiocarcinoma
Extrahepatic malignancy
Persistent non-compliance
Lack of adequate social support system

Hepatology 2014; 59: 1144-1165.

Living Donor Liver Transplantation


• ~4% of liver transplants in U.S. in 2017
• Patient must be listed for deceased donor transplant
• Anticipated prolonged time on wait list with MELD >15
• Recipients of LDLT are less sick: MELD 15-20
• Has family member or acquaintance with close
relationship – no coercion
• In adult, take the right lobe (2/3 mass of liver) from donor
recipient
• Pediatric cases use left lobe living donor transplant

548
Copyright © Harvard Medical School, 2018. All Rights Reserved.

Liver Transplants in 2017 in 11 UNOS Regions


REGION 2017
1 336
2 997
3 1251
4 815
5 1235
6 228
7 670
8 535
9 420
10 773
11 822

SUMMARY
• Hepatic decompensation reduces survival
• MELD and CTP scores predict 3-month and 1-year mortality
in hospitalized cirrhotics
• TIPS > LVP in management of refractory ascites
• Prevention of HRS includes antibiotics in UGI bleeding, IV
albumin in SBP/LVP
• Nonselective beta blocker cessation in SBP
• Consider early TIPS in Childs B/C variceal bleeds
• Gastric varices: cyanoacrylate glue, TIPS, BRTO in select
patients
• Refer for liver transplantation: MELD ≥ 15 + clinical
decompensation

549
Copyright © Harvard Medical School, 2018. All Rights Reserved.

SELECT REFERENCES
• G Garcia-Tsao et al. Portal Hypertensive Bleeding in Cirrhosis: Risk Stratification, Diagnosis and
Management: 2016 Practice Guidance from AASLD. Hepatology 2017; 65: 310-335.
• M Mandorfer et al. Nonselective B Blockers Increase Risk for Hepatorenal Syndrome and Death in Patients
with Cirrhosis and Spontaneous Bacterial Peritonitis. Gastroenterology 2014; 146: 1680-1690.
• P Martin et al. Evaluation for Liver Transplantation in Adults: 2013 Practice Guideline by American
Association for the Study of Liver Diseases and American Society of Transplantation. Hepatology 2014; 59:
1144-1165.
• B Runyon. Management of Adult Patients with Ascites Due to Cirrhosis. Hepatology 2013; 57: 1651-1653.
• H Vilstrup et al. Hepatic Encephalopathy in Chronic Liver Disease: 2014 Practice Guidelines by the
American Association for the Study of Liver Diseases and the European Association for the Study of the
Liver. Hepatology 2014; 60: 715-735.

A 61 year old HCV cirrhotic is admitted with


abdominal pain and confusion. His admission
creatinine is 2.1 (baseline 1.0), and he has evidence
of SBP on tap (>250 PMNs). In addition starting
Ceftriaxone, your next steps in management
should include all except:

A) Stop Nadolol
B) Albumin infusion 1.5mg/kg IV
C) Check urinalysis
D) Start Pentoxyfylline 300mg TID

550
Copyright © Harvard Medical School, 2018. All Rights Reserved.

A 61 year old HCV cirrhotic is admitted with


abdominal pain and confusion. His admission
creatinine is 2.1 (baseline 1.0), and he has evidence
of SBP on tap (>250 PMNs). In addition starting
Ceftriaxone, your next steps in management
should include all except:

A) Stop Nadolol
B) Albumin infusion 1.5mg/kg IV
C) Check urinalysis
D) Start Pentoxyfylline 300mg TID
The correct answer is D. This cirrhotic man has what appears to be acute kidney injury in the setting of spontaneous bacterial
peritonitis (SBP). The urinalysis is recommended to evaluate the acute kidney injury, which could be hepatorenal syndrome (HRS)
or something else. In SBP there is data to support cessation of non-selective beta blockers while the infection is being treatment,
as well as giving albumin infusion of 1.5 mg/kg of body weight to prevent HRS on the first day of treatment. There is no data to
support the use of Pentoxyfylline in cirrhotics with acute kidney injury or SBP.

On hospital day #2, this patient has massive hematemesis.


Urgent endoscopy shows grade III esophageal varices with red
wale signs. He has 3 bands placed. Next steps should include:

A) Octreotide IV for 72 hours


B) Consider BRTO before he rebleeds
C) Transfuse to keep hemoglobin > 10g/dL
D) Referral for urgent liver transplantation

551
Copyright © Harvard Medical School, 2018. All Rights Reserved.

On hospital day #2, this patient has massive hematemesis.


Urgent endoscopy shows grade III esophageal varices with red
wale signs. He has 3 bands placed. Next steps should include:

A) Octreotide IV for 72 hours


B) Consider BRTO before he rebleeds
C) Transfuse to keep hemoglobin > 10g/dL
D) Referral for urgent liver transplantation

The correct answer is A. Management of an acute esophageal variceal bleeding includes upper endoscopy within 12 hours,
vasoactive medications such as Octreotide for 72 hours and antibiotic prophylaxis. A hemoglobin threshold of > 7g/dL reduces the
risk of rebleeding when compared to a transfusion threshold of > 9g/dL. BRTO is used in management of isolated gastric variceal
bleeding. Acute variceal bleeding is not an indication for urgent liver transplantation, and it does not give additional points to the
MELD score.

552
Copyright © Harvard Medical School, 2018. All Rights Reserved.

Psychiatry Overview

Inflammatory Bowel Disease


Sonia Friedman MD
Associate Physician, Brigham and Women’s Hospital
Division of Gastroenterology, Department of Medicine
Associate Professor, Harvard Medical School

Conflict of Interest

• No conflicts of interest

553
Copyright © Harvard Medical School, 2018. All Rights Reserved.

Overview of Inflammatory Bowel Disease


(IBD)
IBD
>1.4 million persons in US

ULCERATIVE1 INDETERMINATE CROHN’S


COLITIS COLITIS2 DISEASE3
10%-20% of IBD patients

Proctitis Pancolitis Ileitis Colitis


28% 47% % at time 22% 32%
of diagnosis
Left-sided Ileocolitis
disease 45%
25%

1. Loftus EV, et al. Gut. 2000;46:336-343; 2. Marion JF, et al. In: Kirsner JB, ed.
Inflammatory Bowel Disease. 5th ed. Philadelphia, Pa: WB Saunders Co; 2000:315-
325; 3. Loftus EV, et al. Gastroenterology. 1998;14:1161-1168.

Age-Specific Incidence of IBD*


Ulcerative Colitis Crohn’s Disease
10 10

8 8

6 6

4 4

2 2

0 0
0 20 40 60 80 0 20 40 60 80
Age (yrs) Age (yrs)
*Per 100,000 population
Reprinted with permission from Lashner BA. In: Stein SH and Rood RP, eds. Inflammatory Bowel
Disease: A Guide for Patients and Their Families. Philadelphia, Pa: Lippincott-Raven Publishers;
1999:23-29.

554
Copyright © Harvard Medical School, 2018. All Rights Reserved.

UC: Presenting Symptoms


• Bloody diarrhea
• Abdominal cramping
• Tenesmus
• Weight loss
• Fevers
• Symptoms depend upon extent
and severity of inflammation

Endoscopic Spectrum of Severity


UC – Spectrum of Disease

Normal Mild

Moderate Severe

Reprinted with permission from AGA Clinical Teaching Project. IBD. 3rd ed. 2002.

555
Copyright © Harvard Medical School, 2018. All Rights Reserved.

Classification of UC Severity

FULMINANT
SEVERE • >10 stools/day
• >6 bloody • Continuous bleeding
stools/day • Toxicity
MODERATE • Fever • Abdominal
• ≥4
• Tachycardia tenderness/distension
stools/day
• Anemia or • Transfusion requirement
± blood
MILD
• Minimal ↑ ESR • Colonic dilation on x-ray
• <4
signs of
stools/day
toxicity
± blood
• Normal ESR
• No signs of
toxicity

Truelove SC, Witts LJ. Br Med J. 1955;2(4947):1041-8;


Kornbluth A, Sachar DB. Am J Gastroenterol. 2010;105(3):501-23.

CD: Presentation
• Diarrhea
• Chronic abdominal pain and
tenderness
• Weight loss
• Fever
• Perianal disease
• Symptoms vary with location of
disease

556
Copyright © Harvard Medical School, 2018. All Rights Reserved.

CD: Clinical Features


Inflammation Obstruction Fistulization

Abdominal pain Cramps Diarrhea


Tenderness Distention Pain
Diarrhea Vomiting Air/feces in urine
Weight loss Types
– Enteroenteric
– Enterovesical
– Retroperitoneal
– Enterocutaneous

Adapted with permission from AGA Gastroenterology Teaching Project. 3rd ed. 2002.

Crohn’s Disease: Severity


• Remission: Asymptomatic either spontaneously or after medical or
surgical intervention and not on steroids.
• Mild to moderate: Ambulatory and able to tolerate an oral diet, no
dehydration, toxicity, abdominal tenderness, mass, obstruction, or
> 10 percent weight loss.
• Moderate to severe: Failed treatment for mild to moderate or
prominent symptoms such as fever, weight loss, abdominal pain
and tenderness, intermittent nausea or vomiting, or anemia.
• Severe-fulminant disease: Persisting symptoms despite
conventional glucocorticoids or biologics as outpatients or
individuals presenting with high fevers, persistent vomiting,
intestinal obstruction, significant peritoneal signs, cachexia or
evidence or an abscess

557
Copyright © Harvard Medical School, 2018. All Rights Reserved.

AGA Clinical Pathway for Crohn’s Disease:


Characterizing Risk
Low Risk High Risk
>30 years Age at diagnosis <30 years

Limited Anatomic involvement Extensive


Perianal and/or
No severe Yes
rectal disease
Superficial Ulcers Deep
Prior surgical
No Yes
resection
Stricturing and/or
No Yes
penetrating behavior

Sandborn WJ. Gastroenterology. 2014;147(3):702-5.

558
Copyright © Harvard Medical School, 2018. All Rights Reserved.

The image demonstrates a new gastrocolic fistula (solid white


arrows). Multifocal involvement of the small bowel and
terminal ileum is also present (dashed white arrows).

559
Copyright © Harvard Medical School, 2018. All Rights Reserved.

Treatment of inflammatory bowel disease


(IBD) 20 - 5 years ago
Surgery

• Biological therapy was


Severe
considered only for Biological
those who have a Therapy

moderate-severe disease Immunomodulators


Moderate
or failed other (thiopurines/
medications methotrexate)

• It was introduced for Prednisone/Budesonide

treatment of IBD in the Mild

late 90s Anti-inflammatory drugs (5-ASA)

Treatment of moderate to severe IBD


in 2018

Surgery

Total Parenteral Nutrition (Crohn’s


disease only)
Small molecules (UC only)

Prednisone/Budesonide

Biologics + Immunomodulators

560
Copyright © Harvard Medical School, 2018. All Rights Reserved.

Goals of IBD Management


• Clarify disease and severity

• Induce Remission
– Defined by both patient reported
outcomes as well as biological
parameters

• Maintain a steroid-free remission

• Change the natural course of IBD


– Avoid hospitalization and surgery
– Avoid disease-related
complications
– Reduce costs of care Rubin DT, et al. Am J Gastroenterol Suppl. 2016;3:4‐7.
Peyrin‐Biroulet L, et al. Am J Gastroenterol.
2015;110(9):1324‐38.

Newer Management Concepts For


Moderate to Severe IBD
• Treat early in disease
• Treat aggressively with “top down”
biologics
• Check drug levels
• Dual therapy (immunomodulator +
biologic)
• Aim for deep remission (histologic and
endoscopic remission)

561
Copyright © Harvard Medical School, 2018. All Rights Reserved.

Pharmacology of
Thiopurines
Azathioprine 6-MP

2:1

2.5 – 3.0 mg/kg 1.0 – 1.5 mg/kg


100mg 50mg
Cuffari C, et al. Aliment Pharmacol Ther. 2000;14:1009-1014.

What are the data on efficacy and


safety of biologic agents?

562
Copyright © Harvard Medical School, 2018. All Rights Reserved.

Currently Approved Biologics for


IBD
Certolizumab Ustekinumab
Pegol approved approved for
for Crohn’s Crohn’s
disease (2008) Disease (2017)
Infliximab Adalimumab Natalizumab Vedolizumab
Initial cA2 approved for approved for approved for approved for
(Infliximab) Crohn’s disease Crohn’s disease Crohn’s disease Crohn’s
NEJM (1997) (1998) (2007) (2008) Disease (2014)

Infliximab Adalimumab Golimumab Vedolizumab


approved for approved for approved for approved for
ulcerative colitis ulcerative ulcerative ulcerative
(2005) colitis (2012) colitis (2013) colitis (2014)

Currently Approved Small Molecule


Therapy for IBD

Tofacitinib
approved for
ulcerative
colitis (2018)

563
Copyright © Harvard Medical School, 2018. All Rights Reserved.

Anti-TNF Therapies – Approved


for IBD

CD and UC CD UC
• Infliximab (IFX): chimeric mouse/human anti-TNF-⍺ antibody
• 75% human IgG1 isotype
• Adalimumab (ADL)/Golimumab (GOL): fully human monoclonal anti-TNF-⍺ antibodies
• 100% human IgG1 isotype
• Certolizumab pegol (CIMZIA): antigen-binding fragment (Fab') of a humanized
monoclonal antibody coupled to polyethylene glycol (No Fc portion)
• 95% human IgG1 isotype

Clinical Remission in UC
Patients failing 5-ASA/Steroids/Immunomodulators

ACT: Infliximab 8 Weeks1 ULTRA 2 Adalimumab PURSUIT: Golimumab


40% ** 40% 8 Weeks2 40% 6 Weeks3
30%
** 30% 30%
20% 20% * 20% ** **
10% 10% 10%
0% 0% 0%
Infliximab Infliximab Placebo Adalimumab Placebo Golimumab Golimumab Placebo
10 mg/kg 5 mg/kg 400/200 mg 200/100 mg

ACT: Infliximab 54 Weeks1 ULTRA 2: Adalimumab PURSUIT: Golimumab


40% ** 40% 52 Weeks2 40% 54 Weeks4
**
30% 30% 30% **
20% 20% ** 20%
10% 10% 10%
0% 0% 0%
Infliximab Infliximab Placebo Adalimumab Placebo Golimumab Golimumab Placebo
10 mg/kg 5 mg/kg 100 mg 50 mg

1Rutgeerts P,et al. N Engl J Med. 2005;353(23):2462-76; 2Sandborn WJ, et al. Gastroenterology. *P<.05 vs. placebo;
2012;142(2):257-65; 3Sandborn WJ, et al. Gastroenterology. 2014;146(1):85-95; 4Sandborn WJ, et al.
Gastroenterology. 2014;146(1):96-109. **P <.01 vs. placebo

564
Copyright © Harvard Medical School, 2018. All Rights Reserved.

Vedolizumab for UC
Induction Week 6
60
53 N=374
Proportion of Patients

50 47*
* P<.0001
40 ** P=.001

30 26
23
20 17 **

10 5

0
Remission Response
Vedo Anti-TNF Naïve - Vedo Placebo

Feagan BG, et al. N Engl J Med. 2013;369(8):699-710.

Vedolizumab for UC
Maintenance Week 52
60 57*
52* N=373
Proportion of Patients

50 45*
42* * P<.0001
40

30
24

20 16

10

0
Remission Response
Vedo q 4 weeks Vedo q 8 weeks Placebo

Feagan BG, et al. N Engl J Med. 2013;369(8):699-710.

565
Copyright © Harvard Medical School, 2018. All Rights Reserved.

Cyclosporine vs. Infliximab for UC

There were no significant differences between IFX and cyclosporine in


adverse drug-related events, post-operative complications, or mortality.
Narula N, et al. Am J Gastroenterol. 2016;111(4):477-91.

UC Success Trial: Week 16 Results


Concomitant Immunomodulators in Patients with UC
*P<.05 compared to IFX
#P<.05 compared to AZA
100
#
# 77
80
69 #
# 63
60 55
50
*40#
37
40
24 22
20

0
Steroid-free remission Response Mucosal healing
AZA N=76 IFX N=77 IFX+AZA N=76

Panaccione R. Gastroenterology. 2011;140 (5 Suppl 1):S134.

566
Copyright © Harvard Medical School, 2018. All Rights Reserved.

Efficacy of Anti-TNF Therapies for Crohn’s Disease


ACCENT I*
(infliximab)2
80
58.5

Patients (%)
60
CHARM** 39.0
40
(adalimumab)1 22.8
80 60 20
Patients (%)

60 40 0
24 Week 2 Response Week 30 Remission Overall Remission
40
Week 30
20
0
Week 4 Response Week 26 Overall Remission PRECISE 2
Remission Week 26 (certolizumab)3
80
64.1

Patients (%)
60 47.9
40 30.7

20

0
*5 mg/kg dose. Week 6 Response Week 26 Overall Remission
**Maintenance trial with 80/40 mg induction dosing. Randomized Remission Week 26
responders = CR-70 at week 4. 1Colombel JF, et al. Gastroenterology. 2007;132(1):52-65;
Week 26 remission among randomized responders on 40 mg every 2Hanauer SB, et al. Lancet. 2002;359(9317):1541-9;
other week dosing. 3Schreiber S, et al. N Engl J Med. 2007;357(3):239-50.

Crohn’s Disease: SONIC Study Design


Randomization of Patients
Azathioprine 2.5 mg/kg Infliximab 5 mg/kg Infliximab 5 mg/kg
+ placebo infusions + placebo capsules + Azathioprine
2.5 mg/kg
Visits
Week 0* • • •
Week 2 • • •
Week 6 • • •
Week 10
Main

Week 14 • • •
Week 18
Week 22 • • •
Week 26* Primary Endpoint (Corticosteroid-free Remission at Week 26)
Week 30 • • •
Week 34
Extension

Week 38 • • •
Week 42
Week 46 • • •
Week 50 Secondary Endpoint (Week 50)
Week 54 • Infusions
* Endoscopy performed at Weeks 0 & 26 Colombel JF, et al. N Engl J Med. 2010;362(15):1383-95.

567
Copyright © Harvard Medical School, 2018. All Rights Reserved.

Crohn’s Disease: SONIC Clinical Remission


Without Corticosteroids: Week 26
100
p<0.001
Proportion of Patients (%)

80
p=0.009 p=0.022

60 57
45
40
30

20
52/170 75/169 96/169
0
AZA + placebo IFX + placebo IFX+ AZA

Colombel JF, et al. N Engl J Med. 2010;362(15):1383-95.

GEMINI 3:
Week 10 Results Vedolizumab in CD
TNF Failure TNF-naive
100 (Secondary Endpoint) 100 (Exploratory Endpoint)
Patients with Clinical

Patients with Clinical

80 80
Remission (%)

P=0.0012b
Remission (%)

60 60

27 35
40 40 16
12
20 20

0 0
Placebo Vedolizumab Placebo Vedolizumab
N=157 N=158 N=50 N=51
A 95% CI for difference from placebo.
BP value vs. placebo.

Clinical remission was defined as CDAI score ≤150.


Sands BE, et al. Gastroenterology. 2014;1479(3):618-627.

568
Copyright © Harvard Medical School, 2018. All Rights Reserved.

Vedolizumab: Maintenance in Crohn’s

GEMINI II
100% Vedolizumab q 4w
Week 52
Vedolizumab q 8w
Placebo
75%

P=.005
P=.004
P<.001
50% 45.5% 43.5% P=.04
P=.02
36.4% 39.0%
30.1% 28.8% 31.7%
25% 21.6%
15.9%

0%
Clinical Response Clinical Remission Steroid-Free Remission
P values vs. placebo

Feagan BG, et al. N Engl J Med. 2013;369(8):699-710.

Infections and Mortality in the TREAT


Registry: 15,000 Patient-Years of Experience

Multivariate Analysis
4.5

4.0 Mortality Serious infections


3.5
Adjusted Odds Ratio

Steroids
3.0

2.5
AZA
AZA IFX 6-MP Steroids
2.0
6-MP MTX
1.5 IFX MTX

1.0
P<.001 P=.006 P=.002
0.5

0.0

AZA = azathioprine; IFX = infliximab; MTX = methotrexate.

Lichtenstein GR, et al. Am J Gastroenterol. 2012;107(9):1409-22.

569
Copyright © Harvard Medical School, 2018. All Rights Reserved.

Minimizing Toxicity for Anti-TNF Therapy


• Rule out evidence for TB prior to initiation
– If immunocompromised, PPD and Quantiferon may be
negative
• Rule out active infection prior to initiation
– Abscess, C. difficile, CMV
• Check serology for Hepatitis B
• Vaccination for age-appropriate disease
– Influenza, Hepatitis A and B, pneumococcal pneumonia,
herpes zoster
• Assess for signs/symptoms of:
– Uncontrolled heart failure
– Demyelinating disorders
– Skin cancers/suspicious moles

Kane S. Curr Gastroenterol Rep. 2010;12(6):502-6.

Vedolizumab: Safety

• Safety data from 6 studies in 2,830 patients (4,811 patient-years of


exposure)
• Adverse events and serious adverse events not higher than placebo
• No cases of progressive multifocal leukoencephalopathy (PML)
• Most frequent AEs were GI events and infections (more frequent in
placebo groups)
• Immunogenicity (anti-vedolizumab antibody): 4-18%
• Infusion reactions: <5% of patients (3 serious)
• Malignancies: 18 in all studies (including 11 open label studies)

Colombel JF, et al. Gut. 2016. pii: gutjnl-2015-311079.

570
Copyright © Harvard Medical School, 2018. All Rights Reserved.

Ustekinumab: Induction

Clinical Remission
UNITI-1 UNITI-2
Patients in Clinical Remission (%)

Feagan BG, et al. N Engl J Med. 2016;375(20):1946-60.

Ustekinumab: Maintenance
Primary and Major Secondary End Points in IM-UNITI
Patients (%)

Feagan BG, et al. N Engl J Med. 2016;375(20):1946-60.

571
Copyright © Harvard Medical School, 2018. All Rights Reserved.

Ustekinumab: Adverse Events


• Ustekinumab and Malignancy
– Rate of Malignancy: 0.2% prevalence in the
UNITI trials (similar to general population)

• Serious Infection (SI)


– UNITI Trials: 2.3% (UST q8) vs. 5.3% (UST q12)
and 2.3% in the placebo group
– Psoriasis Longitudinal Assessment (PSOLAR)
Registry
• 2.3% (12,093 patient) had co-diagnosis of
IBD
• Numerically lower rates of SI in patients
treated with UST compared to IFX: 1.38 vs
5.75

• Reversible Posterior Leukoencephalopathy Syndrome


(RPLS)
– Characterized by headaches, altered
consciousness, and seizures
– Self-limited with supportive care

• Limited data in IBD, but low risk of increased serious


infection or risk of malignancy in UST Feagan BG, et al. N Engl J Med. 2016;375(20):1946-60
Loftus EV et al. Gastoenterology, 2016;150(4):S805
Gratton D et al. JAMA Dermatology, 2017 Feb 1;16(2):177-179

Advantages of Small Molecules in IBD


• Low-molecular weight
• Oral administration
• Resist gastric degradation
• Rapidly enter the systemic circulation
• Short half-life
• Lack immunogenicity
• Easier to manufacture than biologics, which may
improve cost effectiveness
• Bind to specific intracellular targets.

572
Copyright © Harvard Medical School, 2018. All Rights Reserved.

Tofacitinib in UC: OCTAVE Phase 3 Trial


Clinical Response and Remission: Week 8

OCTAVE 1 OCTAVE 2
Clinical Clinical Clinical Clinical
Response Remission Response Remission
80 P<.00 80 P<.00
1 1
59.
60 60 55
9 P<.01 P<.001

40 40
32.
8 25. 29 22.
20 2 20 1
12. 12
8.2 4
6
0 0
Placebo 10 mg Placebo TNF TNF Placebo 10 mg Placebo TNF TNF
BID Naive Rxed BID Naive Rxed
Tofacitinib Tofacitinib
10 mg BID 10 mg BID
Sandborn WF, et al. NEJM. May 2017.

Phase 3 OCTAVE Tofacitinib Trial: Safety

OCTAVE Induction 1 OCTAVE Induction 2

Tofacitinib Tofacitinib
Placebo Placebo
10 mg BID 10 mg BID
N=122 N=112
N=476 N=429

Treatment –emergent AEs, N(%) 269 (56.6) 73 (59.8 232 (54.1) 59 (52.7)

Treatment –emergent SAEs, N(%) 16 (3.4) 5 (4.1) 18 (4.2) 9 (8.0)

Discontinuation due to AEs, N(%) 18 (3.8) 2 (1.6) 17 (4.0) 8 (7.1)

Infections, N(%) 111 (23.3) 19 (15.6) 78 (18.2) 17 (15.2)

Serious infections N(%) 6 (1.3) 0 (0.0) 1 (0.2) 0 (0.0)

Sandborn WF, et al. NEJM. May 2017.

573
Copyright © Harvard Medical School, 2018. All Rights Reserved.

Phase 3 OCTAVE Tofacitinib Trial


Maintenance Week 52

100 Placebo
P<.001 vs. placebo, all comparisons Tofacitinib 5 mg
80 Tofacitinib 10 mg
Percent of Patients

59
60
47 49

40 33 35
25 28
22 19
20
5 7 5
0
Sustained remission Sustained mucosal Sustained steroid- Sustained clinical
healing free remission* response
*Among remitters at baseline;
Placebo N=59, Tofacitinib 5 mg N=65, Tofacitinib 10 mg N=55
Sandborn WF, et al. NEJM. May 2017.

Biosimilars
• Biosimilar is highly similar to the reference product with
minor differences in clinically inactive components
• Price reductions up to 80%
• No clinically meaningful differences between the biological
product and the reference product in terms of biological
activity, safety, efficacy, purity, and potency
• Same strength, dosage form, and route of administration as
original
• Can be alternated with original without loss of efficacy or
change in risk
• Can be substituted at the pharmacy without intervention of
healthcare provider

574
Copyright © Harvard Medical School, 2018. All Rights Reserved.

Non-response: Definitions
• Primary non-response: A patient does not respond to a
loading dose of a biological agent when he/she receives it for
the first time. Is this patient a non-responder to all drugs
targeting the same pathway?

• Secondary non-response: A patient has responded to


biological therapy at onset, but loses response or becomes
intolerant to the molecule. This patient may respond to dose
adjustment or is likely to benefit from other agents targeting
the same pathway. If antibodies to initial drug, can switch
within class.

Reactive Testing Algorithm: Anti-TNF


Therapy
Secondary loss of response
(disease activity confirmed)

Sub-therapeutic
Infliximab/Adalimumab
Therapeutic concentration
infliximab/adalimumab
concentration Drug antibody
Drug antibody
positive
negative

Low level of antibody High level of antibody

Change drug class or Dose escalate


surgery Consider dose Change to
escalation, addition of different
immunomodulator, or anti-TNF
change anti-TNF

Modified from Khanna R, et al. Aliment Pharmacol Ther. 2013;38(5):447-59.

575
Copyright © Harvard Medical School, 2018. All Rights Reserved.

Changing Class More Efficacious Than Dose


Escalation in Patients with Adequate ADA
Levels

Yanai H, Clin Gastroenterol Hepatol. 2015 Mar;13(3):522-530

Proactive Monitoring
• Outcomes of patients who
had proactive therapeutic
Probability on Infliximab

concentration monitoring
(TCM) of IFX (N=48), and
those without (N=78) TCM
No TCM
• Target 5 to 10 µg/mL IFX
• Pts with IFX <3 fell from 27%
to 10%
• 86% of TCM group remained Weeks
on IFX, versus 52% without
monitoring

Vaughn BP, et al. Inflamm Bowel Dis. 2014;20(11):1996-2003.

576
Copyright © Harvard Medical School, 2018. All Rights Reserved.

IBD: Systemic Complications


Growth failure
Eye
in children
inflammation*

Lower Kidney
bone density* stones

Liver and Subfertility*


bile duct
inflammation Ovaries

Uterus
Gallstones
Arthritis and
joint pains
Skin lesions

*Higher incidence in women.

577
Copyright © Harvard Medical School, 2018. All Rights Reserved.

578
Copyright © Harvard Medical School, 2018. All Rights Reserved.

Musculoskeletal Disorders in IBD


• Peripheral arthritis
Reprinted with permission
from Berens DL. Roentgen
Features of Ankylosing
• Sacroiliitis Spondylitis. Clin Ortho.
1971;74:20-33.

Reprinted from the Clinical


Slide Collection on the
Rheumatic Diseases,
• Ankylosing copyright 1991, 1995, 1997.
Used by permission of
spondylitis the American College of
Rheumatology.

579
Copyright © Harvard Medical School, 2018. All Rights Reserved.

AGA Recommendations
for Managing Osteoporosis
Basic Prevention:
-Ca/Vit D
-exercise
T score >-1 -smoking cessation
-avoid alcohol
IBD patient: -minimize
Any of: corticosteroids
-Prolonged steroid use T score -2.5 to -1 -treat hypogonadism
(>3mo consec or recurrent
courses) Prevention and:
-Low trauma, fragility fracture DXA -repeat DXA 2 years
-Postmenopausal or male age -Prolonged CS consider BP
>50 and DXA 1 year
-Hypogonadism
T score <-2.5
Prevention and:
-Screen other causes low BMD
Vert Fracture -Bisphosphonate therapy or
Regardless of DXA -Refer to bone specialist
Gastroenterology 2003;124:795-841

Relative Risk of Colon Cancer Based on


Extent of UC (and Crohn’s colitis)
20
18
16

14
12

10

8
6

0
All Cases Proctitis Left-Sided Pan-colitis

Ekbom A, et al. N Engl J Med. 1990;323:1228-1233.

580
Copyright © Harvard Medical School, 2018. All Rights Reserved.

Risk Factors for Colon Cancer in IBD

• Strictures that cannot be passed


• Extensive colitis
• Long duration of disease
• Active disease
• Primary sclerosing cholangitis
• Family history of colon CA

Take Home Messages


• Treat early
• Treat aggressively for moderate to severe IBD
• Use dual therapy
• Use drug levels to guide therapy
• Treat to endoscopic and histologic remission
• The risk of a serious side effect is much less than
the risk of a complication from the IBD itself

581
Copyright © Harvard Medical School, 2018. All Rights Reserved.

Question
Which of the therapies for IBD has the highest
risk for immunogenicity (drug antibody
development)?
A. Vedolizumab
B. Ustekinumab
C. Infliximab
D. Adalimumab
E. Golimumab

Question
Which of the therapies for IBD has the highest
risk for immunogenicity (drug antibody
development)?
A. Vedolizumab
B. Ustekinumab
C. Infliximab
D. Adalimumab
E. Golimumab

Infliximab is 25% mouse protein and thus has the highest immunogenicity

582
Copyright © Harvard Medical School, 2018. All Rights Reserved.

Question
A 38 year old man with a medical history notable for Crohn’s
Disease, currently on Infliximab at 5mg/kg every 8 weeks, presents
in follow up with worsening abdominal pain, frequent bowel
movements up to 15 daily, to discuss next options. Serum CRP is
56mg/dL. Infliximab trough level obtained prior to his last infusion
was 12μg/mL (adequate therapeutic level is >5μg/mL); no antibody
level detected.

After providing induction therapy, what would be the next best


therapeutic option in managing his disease?

A. Start azathioprine
B. Increase the infliximab dose to 10mg/kg
C. Discontinue infliximab and start the biosimilar: inflectra
D. Discontinue infliximab and start ustekinumab
E. Decrease interval between infliximab infusions to every 4 weeks

Question
A 38 year old man with a medical history notable for Crohn’s
Disease, currently on Infliximab at 5mg/kg every 8 weeks, presents
in follow up with worsening abdominal pain, frequent bowel
movements up to 15 daily, to discuss next options. Serum CRP is
56mg/dL. Infliximab trough level obtained prior to his last infusion
was 12μg/mL (adequate therapeutic level is >5μg/mL); no antibody
level detected.

After providing induction therapy, what would be the next best


therapeutic option in managing his disease?

A. Start azathioprine
B. Increase the infliximab dose to 10mg/kg
C. Discontinue infliximab and start the biosimilar: inflectra
D. Discontinue infliximab and start ustekinumab
E. Decrease interval between infliximab infusions to every 4 weeks
IFX does not work here even when the level is therapeutic so it is
necessary to switch drug class

583
Copyright © Harvard Medical School, 2018. All Rights Reserved.

References
1. Bonovas S: Biologic therapies and risk of infection and malignancy in
patients with inflammatory bowel disease: a systemic review and network
meta-analysis. Clin Gastroenetrol Hepatol 14:1385, 2016.
2. Ha C, Kornbluth A. A Critical Review of Biosimilars in IBD: The Confluence
of Biologic Drug Development, Regulatory Requirements, Clinical
Outcomes, and Big Business. Inflamm Bowel Dis 22:2513, 2016.
3. Julsgaard M: Concentrations of adalimumab and infliximab in mothers
and newborns and effects on infection. Gastroenterology 151:110, 2016.
4. Ng SC: Geographical variability and environmental risk factors in
inflammatory bowel disease. Gut 62:630, 2013.
5. Regueiro M: Infliximab Reduces Endoscopic, but Not Clinical, Recurrence
of Crohn's Disease After Ileocolonic Resection. Gastroenterology
150:1568, 2016.

584
Copyright © Harvard Medical School, 2018. All Rights Reserved.

Acute and Chronic Diarrhea

Benjamin Smith, MD
Assistant Professor, Harvard Medical School
Attending Physician: Brigham & Womens, Faulkner and
VAMC GI Fellowship Training Program

Commercial/Faculty
Disclosures

None

585
Copyright © Harvard Medical School, 2018. All Rights Reserved.

Goals
• Acute and Chronic Diarrhea
• Focus on essentials
• New developments

Topics Covered
• Definitions
• Approach to acute diarrhea
– Differential diagnosis
– Diagnostic approach
• When to watch
• When to test
• When to treat
• Approach to chronic diarrhea
– Mechanisms
– Diagnostic evaluation
• Initial testing
• Follow-up testing

586
Copyright © Harvard Medical School, 2018. All Rights Reserved.

Diarrhea: Definitions

• Objective definition
– Excessive stool weight: >200gm/day
• Subjective definition
– Excessive frequency of defecation (>3 stools)
– Less-than-normal form and consistency
• Acute diarrhea: < 4 weeks duration
• Chronic diarrhea: > 4 weeks duration
• Persistent diarrhea: 2-4 weeks duration

Normal Intestinal Physiology

• 10 L of fluid enters the jejunum daily


– 2 L: Food and drink
– 8L: Salivary, gastric, biliary, pancreatic,
intestinal secretions
• 1L enters the colon
• 80-100 mL excreted daily

587
Copyright © Harvard Medical School, 2018. All Rights Reserved.

Abnormal Intestinal Physiology

• Loosening of stools: 50-60mL increase daily


fecal water
• Diarrhea: 100mL (1-2%) increase in fecal water
• Many disorders disrupt intestinal fluid and
electrolyte absorption by at least this amount
– Frequent event
– Extensive differential diagnosis

Acute Diarrhea: Major Causes

• Infectious: Most common


– Viral: Noroviruses , Rotavirus
– Bacterial: Food poisoning, C. difficile
– Parasitic/protozoal
• Noninfectious
– Medications
– Consumption of poorly absorbed sugars (e.g.
sorbitol)
– Enteral feeding
– Ischemic colitis
– Fecal impaction: “paradoxical diarrhea”

588
Copyright © Harvard Medical School, 2018. All Rights Reserved.

Diagnostic Evaluation

• Majority of cases are mild and self-


limited
• 90% of cases need no diagnostic
evaluation

Situations Requiring Additional Testing

– Bloody diarrhea
– Profuse diarrhea leading to dehydration
– Duration >48 hrs or > 6 unformed stools/24
hrs
– Severe abdominal pain: Over age 50
– Temperature > 38.5°C (101.3°F)
– Immunocompromised or elderly (>70 yrs)

589
Copyright © Harvard Medical School, 2018. All Rights Reserved.

Initial Diagnostic Evaluation

• Stool assessment:
Wright’s stain
– Fecal leukocytes
– Occult blood
Fecal lactoferrin assay
• Presence of both: Dysentery
– Supports a bacterial etiology for acute diarrhea
– Campylobacter, Salmonella > E.Coli 0157:H7, Shigella

• Exception: Nosocomial diarrhea


– Testing for C. difficile toxin higher yield than fecal leukocytes

Stools Negative for FOBT and


Leukocytes

• Not severely ill


• Treat symptomatically for several days
• Likely etiologies:
– Viral (>75%)
• Noroviruses (71%), Rotavirus,
– Bacteria elaborating preformed toxin
• S. aureus, B. cereus, C. perfringens
• Stool culture positive: 1.5-5.6%

590
Copyright © Harvard Medical School, 2018. All Rights Reserved.

Further Testing: Stool Culture


• Sick/bloody diarrhea
– Stool culture positive: Up to 87%
• Food handlers
• Patients with IBD
• Routine culture:
– Campylobacter, Salmonella, Shigella
• Notify lab:
– E. Coli O157:H7, Yersinia, Aeromonas
• Single specimen: Bacteria shed continuously
• Clostridium difficile toxin assay
– Antibiotic/Chemotherapy within preceding 2 weeks
– Hospital-acquired diarrhea

Stool For Ova and Parasites


• Cost-effective in high risk groups
– Persistent diarrhea (>14 days)
– Community waterborne outbreak
– Exposure to untreated water (e.g.
streams)
– Daycare center exposure
– Travel to Russia, Nepal
• Giardia, Cryptosporidia
– Homosexual men (Giardia, E.histolytica)
– Patients with AIDS (Giardia, E.histolytica
and others)
• Intermittent shedding
– 3 specimens on consecutive days

591
Copyright © Harvard Medical School, 2018. All Rights Reserved.

Who needs Endoscopic Evaluation?

• Bloody diarrhea
– IBD versus infectious diarrhea
– Suspected ischemic colitis
• Pseudomembranous colitis
• Immunocompromised or other high-risk patients:
Look for CMV
• Flexible sigmoidoscopy versus colonoscopy

Supportive Therapy
• Rehydration: Glucose-NA+ co transporter
– WHO oral rehydration solution (per liter of water)
• 20gm glucose or 40gm sucrose, NaCl, NaHCO3, KCL
– Alternative rehydration solution (per liter of water)
• 4 tablespoon sugar
• ½ teaspoon salt
• ½ teaspoon baking soda
– Rice-based oral rehydration solution (e.g. Cera-lyte)
• Fluids for sweat replacement (e.g. Gatorade,
Powerade, Propel)
– Not equivalent to ORS. Sufficient for mild cases
– Diluted fruit juice plus saltine crackers
• Dietary modification
– Lactose free diet for several weeks
– Avoid food with high fat content
– Boiled starches or cereals with salt
– BRAT diet: Bananas, rice, apple sauce, toast

592
Copyright © Harvard Medical School, 2018. All Rights Reserved.

Supportive Therapy: Other Measures

• Stop all sugar substitutes


• Review medications
• Assess for fecal impaction and treat
• Adjust tube feeds: Dilute, decrease rate or add
fiber

Antidiarrheal agents
• Stools nonbloody and fever low-grade
• Antimotility agents: Decrease peristalsis
– Loperamide (Imodium)
• 4mg initially, then 2mg after each loose movement
• Maximum: 16mg/day for 2 days
– Diphenoxylate atropine (Lomotil)
• Central opiate and anti-cholinergic side-effects (atropine)
• 1-2 tabs tid/qid
• Other agents
– Pepto-Bismol: 2 tabs every 30 minutes; Can also help
vomiting
– Kaopectate

593
Copyright © Harvard Medical School, 2018. All Rights Reserved.

Empiric Antibiotic Therapy


• Severely ill immunocompetent individuals
– Fever, bloody diarrhea
– Dehydration
– >8 stools/day or symptoms for > 1 week
• Immunocompromised patients
– AIDS, malignancy, transplant recipients
• Drugs of choice
– Quinolone: Ciprofloxacin, levofloxacin, norfloxacin (3-5 days)
– Alternatives: Azithromycin (3 days) and erythromycin (5 days)
• Suspect fluoroquinolone resistance
• Campylobacter infection
– Metronidazole: C. difficile suspected

Probiotics: Mixed Results


• Prevention of travelers diarrhea
– Supportive studies/meta-analysis
– Lactobacillus GG and acidophilus
– Saccaromyces boulardii: N. Africa, Turkey
• Infectious diarrhea
– Mixed data: Lactobacillus species
– Reduce duration of diarrhea: about 24 hours
– 10 billion CFU within first 48 hours
• Antibiotic associated diarrhea: Need more
studies

594
Copyright © Harvard Medical School, 2018. All Rights Reserved.

Acute Diarrhea

Select Infectious Causes

Salmonella
• Most common
• 1.2 million illnesses
• 23,000 hospitalizations
• 450 deaths in US each year
• Single species: Salmonella cholerasuis
– 2200 different serotypes
– Nontyphoidal serotypes
• S. enteritidis, S. typhimurium, S. heidelberg
– Typhoidal serotypes
• S. typhi, S. paratyphi CDC 2018

595
Copyright © Harvard Medical School, 2018. All Rights Reserved.

Salmonella Gastroenteritis
• Source: Contaminated eggs and poultry
• Leukocytosis: Predominant mononuclear cells
• Achlorhydric individuals at increased risk
• Should not receive antibiotics
– Do not alter rate of clinical recovery
– Increases incidence and duration of intestinal
carriage

Salmonella Complications
• Bacteremia (5%): Risk of endovascular invasion
– Risk factors:
• Aortic aneurysm
• Vascular graft
• Prosthetic heart valve
• Osteomyelitis
– Risk factors:
• Orthopedic prosthetics
• Sickle cell disease
• These patients should receive antibiotics
– Ciprofloxacin 500mg BID for 10-14 days
– Multi-drug resistant: Chloramphenicol

596
Copyright © Harvard Medical School, 2018. All Rights Reserved.

Typhoid Fever

• Caused by Salmonella typhi or paratyphi


• Uncommon: 500 cases/yr in the U.S.
• Spiking fever
– Relative bradycardia,
• “Rose spot” rash: upper anterior trunk
• Hepatosplenomegaly
• RLQ abdominal pain
• Diarrhea: Varies in severity
• Duration 3-5 weeks

Campylobacter jejuni

• A leading cause of bacterial


diarrhea: 4-11%
• Source - Contaminated poultry:
50-70%
• Relapsing course: 15-20%
• Drugs of choice:
– Fluoroquinolones
– Azithromycin
– Rising rates of resistance
• Check sensitivities

597
Copyright © Harvard Medical School, 2018. All Rights Reserved.

Campylobacter: Complications

• Reactive arthritis/Reiter’s Syndrome: 1%


– 1-2 weeks after diarrhea onset
– Self-limited: several months
• Guillain-Barré Syndrome
– Responsible for 25% cases
– Symptoms within 3 months of diarrhea onset
– May be Culture negative but serology positive
• Antibiotics do not prevent these complications

Enterohemorrhagic E. coli (EHEC)

• Most prevalent in U.S.: E. coli 0157: H7


– Undercooked meat ingestion
– Predisposing factors: Age < 10 or > 50
– Watery diarrhea bloody diarrhea
– Often afebrile
– April 2017: Local food establishment that serves
chicken and rice
• 15 reported cases, 10 hospitalized

• May elaborate 2 proteins:


– Shiga toxin = verotoxine
– Intimin= Adhesine protein

598
Copyright © Harvard Medical School, 2018. All Rights Reserved.

EHEC Infections: Complications


• EHEC with both proteins
– Potentially lethal
– Increase risk of Hemolytic-uremic Syndrome
(HUS)
• Acute renal failure
• Microangiopathic hemolytic anemia
• Thrombocytopenia
– Selective culture required
– Avoid Antibiotics: Increased risk of HUS

Chronic Diarrhea:
A Systematic Approach

599
Copyright © Harvard Medical School, 2018. All Rights Reserved.

Chronic Diarrhea

• Afflicts 5% of the population


• Myriad of disorders
• Order of prevalence varies
– Practice setting
• Optimum evaluation strategy
– Not established
– Expert opinion

MAJOR CAUSES IN DEVELOPED


COUNTRIES
• Irritable bowel syndrome
• Inflammatory bowel disease
– Ulcerative colitis, Crohn’s disease
– Microscopic colitis
• Malabsorption syndromes
– Lactose intolerance
– Celiac disease
• Medications
• Chronic infections
– Immunocompromised
– Bacterial, parasitic

600
Copyright © Harvard Medical School, 2018. All Rights Reserved.

Categorization of Diarrhea

Osmotic
• Watery
Secretory

• Inflammatory
• Fatty

• Several mechanisms may coexist

Osmotic Diarrhea

• Hallmarks
– Diarrhea stops with fasting
– Large osmotic gap: > 125 mOsm/kg

• Mechanism
– Ingestion of osmotically active solutes
– Retention of water in intestinal lumen
– Electrolyte absorption (Na+, K+) is normal
– Large osmotic gap between expected (290mOsm)
and calculated

• Stool osmotic gap = 290 – 2([Na+] + [K+])

601
Copyright © Harvard Medical School, 2018. All Rights Reserved.

Osmotic or Malabsorptive Diarrhea


• Exogenous Causes:
– Antacids: Mg+
– Laxatives:
• Polyethylene glycol (PEG)
• Poorly absorbed anion (PO4-3, SO4-2, citrate)
– Sugar substitutes: Sugar-free candy/gum or medication
elixirs:
• Sorbitol, mannitol, Splenda, lactulose
– Nonabsorbable fats: Olestra

• Endogenous Causes:
– Congenital: Disaccharide deficiencies: Lactose intolerance
– Acquired
• Post-enteritis: Lactose intolerance
• Pancreatic insufficiency
• Celiac disease

Secretory Diarrhea

• Hallmarks
– Persists with fasting
– Nocturnal diarrhea
– Large volume, watery
– Small stool osmolar gap < 50 mOsm/kg

602
Copyright © Harvard Medical School, 2018. All Rights Reserved.

Secretory Diarrhea

• Exogenous:
– Stimulant laxatives: Bisacodyl, senna
– Prostaglandins, theophylline, colchicine
– Dietary secretagogues: Ethanol, caffeine, colas

• Endogenous:
– Bile acid malabsorption:
• Crohn’s ileitis, SB resection, bacterial
overgrowth, cholecystectomy
– Hormone-producing tumors: VIPoma, gastrinoma

Bile Acid Diarrhea


• IBS-D: up to 50% with bile acid malabsorption
• Post cholecystectomy: 5-12%; Often improves over
weeks to months
• Bile acids: Stimulate colonic motility and secretion
• Treatment: Bile acid sequestrants
– Colestipol (tablet): 1 gm twice daily
– Cholestyramine (powder): 4 gms daily
– Colesevelam/Welchol (tablet): 1.875gm twice daily
• Average delay in transit time of 4 hours c/w placebo
• Potential side effects: Bloating, flatulence, abdominal
discomfort and constipation
– Need to be separated from other medication by > 2 hours.

603
Copyright © Harvard Medical School, 2018. All Rights Reserved.

Inflammatory Diarrhea
• Hallmarks
– Mucoid, bloody stool
– Tenesmus, abdominal pain, fever
– FOBT positive
– Fecal leukocytes:
• Low sensitivity (70%) /specificity (50%)
• Fecal calprotectin: Zn/Ca binding protein
– Derived from neutrophils & monocytes
– Levels increased in intestinal inflammation
– Distinguish inflammatory from noninflammatory causes
of chronic diarrhea

Inflammatory Diarrhea

• Chronic infections
– C. difficile, amebiasis, TB, parasitic pathogens
• IBD: Crohn’s, ulcerative colitis
• Radiation or chemo-induced mucositis
• Colonic ischemia

604
Copyright © Harvard Medical School, 2018. All Rights Reserved.

Fatty Diarrhea
• Oil droplets in stool, floating stool
• Diagnosis
– Positive Sudan III stain: Qualitative
– 72 hour stool collection
• Abnormal: > 7gm fat/day
• Rarely done (limited reproducibility)
– Stool acid steatocrit
• Acidify stool
• Separate fecal homogenate into lipid, water, solid phases;
Measure lipid
• Good correlation with quantitative fecal fat
– NIRA: Near infrared reflectance analysis
• Simultaneous measurement of fecal fat, carbohydrates,
nitrogen
• Expanding use in Europe, starting in U.S.

Fatty Diarrhea: Causes

• Pancreatic insufficiency
• Crohn’s disease
• Short bowel syndrome
• Bacterial overgrowth

605
Copyright © Harvard Medical School, 2018. All Rights Reserved.

History
• Stool characteristics: Watery, bloody, oily
• Epidemiological factors: Travel, sick contacts
• Aggravating/mitigating factors: Diet, stress
• Presence or absence:
– Fecal incontinence, abdominal pain, weight loss
• Past history:
– Diabetes, Hyperthyroidism, surgery, XRT, CAD
• Medication history
• Sexual history: Risk factors for AIDS
• Family history: IBD, neoplasm, celiac disease
• Markers of eating disorder, malingering

PHYSICAL EXAM
• Extent of fluid and nutritional depletion
• Skin rashes or flushing
• Mouth ulcers
• Thyroid masses or exophthalmos
• Arthritis
• Hepatomegaly or abdominal masses
• Anorectal exam: sphincter tone, perianal
fistula/abscess
• Scars (suggesting prior abdominal surgery)

606
Copyright © Harvard Medical School, 2018. All Rights Reserved.

Initial Laboratory Testing


• Minimum
– CBC with differential
– Electrolyte panel
– Total protein & albumin
– Thyroid function tests
– ESR

• Strongly consider
– Iron studies, vitamin B12, Folate, Prothrombin time
– Sprue serology: TTG IgA and IgA level

Initial Stool Testing

• Fecal occult blood testing


• Fecal leukocytes
• C. difficile toxin: Antibiotic history
• Stool culture
• Stool examination for O & P: Three samples
• ELISA for Giardia antigen

607
Copyright © Harvard Medical School, 2018. All Rights Reserved.

Endoscopic Evaluation
• Required for evaluation of many patients
• Flexible Sigmoidoscopy
– Reasonable exam for some patients
• Colonoscopy
– Patients with iron deficiency anemia
– Older patients: >50 yrs
– Patients with suspected Crohn’s disease
– Biopsy normal-appearing mucosa
• Collagenous/lymphocytic colitis
• 10% right-sided only
• Upper Endoscopy
– May be useful to rule-out sprue or Whipple’s

Difficult to Diagnose Cases

• Common problems overlooked


– Lactose intolerance
– Fecal incontinence
– Review medications again
• Stool culture: Aeromonas and pleisiomonas
• O&P: Cryptosporidium, Microsporidia
• Breath test for bacterial overgrowth
• Calculate the osmotic gap

608
Copyright © Harvard Medical School, 2018. All Rights Reserved.

Difficult Cases: Osmotic

• Laxative screen
– Inadvertent or surreptitious laxative use
• Stool pH < 5.3
– Carbohydrate malabsorption (e.g. lactulose, sorbitol)

Difficult Cases: Secretory

• Plasma peptides:
– VIP, gastrin, glucagon, calcitonin, tryptase
• 24 hour urine collection: 5-HIAA
• Imaging: CT scan:
– Pancreatic neoplasm, intestinal lymphoma,
TB

609
Copyright © Harvard Medical School, 2018. All Rights Reserved.

Difficult Cases: Inflammatory and Fatty


• Fecal fat assessment
– Stool sudan stain
• Suspect pancreatic insufficiency
– Pancreatic imaging: Chronic pancreatitis
• MRI/MRCP
– Stool: Fecal elastase, chymotrypsin
• Reliable only in moderate to severe insufficiency
– Urine collection: Pancreolauryl test
– Trial of pancreatic enzymes
• Small bowel follow through, MR enterography
– Rule-out IBD
• Small bowel biopsies for sprue, Whipple’s
• Breath test for SIBO
– Trial of antibiotics

SYMPTOMATIC THERAPY

• Therapeutic Options:
– Opiates: most effective
– Empiric trial of antimicrobial therapy
– Cholestyramine and colestipol
– Octreotide

610
Copyright © Harvard Medical School, 2018. All Rights Reserved.

Summary: Acute Diarrhea


• Infections: Most common cause of acute diarrhea
– Viral more common than bacterial etiology
– 90% self-limited, require no further evaluation
• Further evaluation: Sick, bloody diarrhea
– Sick: Stool for Fecal leukocytes, occult blood
– Bloody diarrhea: Stool culture
– Exception: Antibiotic exposure, hospitalized C. Diff. Toxin
• Empiric Antibiotics: Severely ill, immunocompromised
– Quinolone
– Second line: Azithromycin or erythromycin
– Metronidazole: C. difficile suspected
• Avoid antibiotics: E. coli 0157:H7

Summary (2): Chronic Diarrhea

• Broad differential diagnosis


• Evaluation assisted by:
– Careful history and physical
– Categorize the diarrhea:
• Watery, Inflammatory or Fatty
• Keep in mind possibility of fecal incontinence
• Many patients with chronic diarrhea require endoscopic
evaluation
• Acute and chronic diarrhea:
– Supportive measures: Rehydration, antidiarrheal agents

611
Copyright © Harvard Medical School, 2018. All Rights Reserved.

Question 1:
All of the following may cause chronic
inflammatory diarrhea except:

a. Ulcerative Colitis
b. Radiation
c. Clostridia difficile infection
d. Bacterial overgrowth
e. Chemotherapy

Question 1: Answer d
• Bacterial overgrowth
– Bile acid malabsorption
• Watery secretory diarrhea
• Fatty diarrhea
– Not inflammatory diarrhea

612
Copyright © Harvard Medical School, 2018. All Rights Reserved.

Question 2:
In patients with acute diarrhea, empiric
antibiotic therapy should be considered in all
but one of the following contexts:

a. Fever and bloody diarrhea


b. Moderate to severe travelers diarrhea
c. Known or suspected E. coli 0157: H7
d. Elderly and immunocompromised
e. Hospitalization under consideration

Question 2: Answer C
• Known or suspected E. coli 0157: H7
• Most common EHEC infection
• Avoid antibiotics:
• Increases production of shiga toxin
• Increases risk of Hemolytic-uremic Syndrome (HUS)
• Acute renal failure
• Microangiopathic hemolytic anemia
• Thrombocytopenia

613
Copyright © Harvard Medical School, 2018. All Rights Reserved.

References
1. LaRoque R, Harris JB , Approach to the adult with acute diarrhea in
resource-rich settings In: UpToDate, Post TW (Ed), UpToDate, Waltham,
MA (accessed on April 7th 2018).

2. American gastroenterological association medical position statement:


guidelines for the evaluation and management of chronic diarrhea.
Gastroenterology 1999;116: 1461-1464.

3. Bonis PA, LaMont JT, Approach to the patient with chronic diarrhea in
resource rich settings In: UpToDate, Post TW (Ed), UpToDate, Waltham,
MA (accessed on May 28th 2017).

4. Centers for Disease Control and Prevention: www.cdc.gov

Commercial/Faculty
Disclosures

None

614
Copyright © Harvard Medical School, 2018. All Rights Reserved.

GI TAKE HOME MESSAGES and CLINICAL PEARLS

Intensive Review of Internal Medicine

Kunal Jajoo, MD
Clinical Director
Division of Gastroenterology, Hepatology and Endoscopy
Brigham and Women’s Hospital
Assistant Professor of Medicine
Harvard Medical School

Disclosures
• None relevant to this presentation

615
Copyright © Harvard Medical School, 2018. All Rights Reserved.

GI Take Home Messages


• Whopping dose of GI information
• How will you metabolize this?
• Acronyms?
• Pathways?
• Clinical vignettes?
• Systems-based approach

GI Take Home Messages


• Whopping dose of GI information
• How will you metabolize all this?
• Acronyms?
• Pathways?
• Clinical vignettes?
• Systems-based approach

616
Copyright © Harvard Medical School, 2018. All Rights Reserved.

Esophagus
• GERD is quite common
• Treatment includes lifestyle changes and acid
suppression (H2RA / PPI) or sometimes
surgery
• EGD and pH testing (alarm / new symptoms,
treatment failure)
• Longterm effects of PPI need further
investigation

Esophagus
• Barrett’s esophagus is a result of GERD and a
precursor of adenocarcinoma
• Progression rate is much lower than initially
thought
• Screening indicated when multiple risk factors
present
• Surveillance indicated as treatment for
dysplasia is effective

617
Copyright © Harvard Medical School, 2018. All Rights Reserved.

Esophagus
• Eosinophilic esophagitis is an increasingly
recognized allergic disorder most commonly
presenting as dysphagia
• PPI therapy, elimination diet, food allergy
testing and swallowed steroids are mainstays
of therapy

Esophagus
• Motility disorders can be difficult to diagnose
• Must rule out structural causes of dysphagia
• High resolution manometry can distinguish
amongst the dysmotility disorders and direct
therapy

618
Copyright © Harvard Medical School, 2018. All Rights Reserved.

Peptic Diseases
• PUD most commonly caused by NSAIDs or H
pylori (or both - synergistic)
• NSAIDs often under-reported in the history
• PPIs have a more rapid response in healing
peptic ulcers than H2RAs
• Restrictive transfusion regimen for bleeding
PUD
• 2° ASA should be re-started within one week

Peptic Diseases
• Newest H pylori guidelines favor a test and
treat strategy for non-ulcer dyspepsia, ITP, iron
deficiency anemia and prior to long-term
NSAID use
• Treatment algorithm for H pylori hinges upon
macrolide exposure / PCN allergy
• Bismuth, clartihro, levofloxacin based
regimens; sequential regimens

619
Copyright © Harvard Medical School, 2018. All Rights Reserved.

IBD
• UC and Crohn’s
• Clinical parameters to define severity of
disease
• Goals of therapy
– Induce remission (clinical and biologic)
– Steroid free therapies
– Avoid surgery/hospital

IBD
• Paradigm shift: step-up therapy replaced by
top-down
• Treat early in moderate to severe disease
• Dual therapy – immunomodulator and
biologic mAb
• Be aware of longterm complicatons –
osteoporosis, cancer risk (highest in pan-
colitis), need for surveillance

620
Copyright © Harvard Medical School, 2018. All Rights Reserved.

Liver
• HBV • HCV
– Phases of Hep B – Risk based testing
determine treatment • Baby boomers should be
need: treat when tested
evidence of – progression to cirrhosis,
inflammation (ALT 2x), HCC
replication or fibrosis – Direct acting anti-virals
– Multiple treatment high sustained virologic
regimens: suppression response
– Entecavir / Tenofovir – Risk of re-activation HBV
preferred

Liver
• Decompensation occurs in the majority of
patients with cirrhosis
• Ascites>jaundice > encephalopathy > variceal
bleeding
• New ascites warrants paracentesis (SAAG > 1.1
= pHTN)
• Treatment – Na restriction,
furosemide/spironolactone; LVP/TIPS

621
Copyright © Harvard Medical School, 2018. All Rights Reserved.

Liver
• Hepatorenal syndrome
– Renal impairment that does not respond to
holding diuretics or volume expansion
– Type 1: acute precipated by SBP, alc hep, surgery
– Type 2: chronic, refractory ascites, Cr > 1.5
– Poor prognosis
– Albumin/Octreotide/Midodrine
– TIPS

Liver
• Varices
– Screening indicated; if seen, then non-selective
beta blocker or banding for esophageal
– Acute hemorrhage managed in ICU, banding, early
TIPS for CPT B/C
– Gastric variceal hemorrhage: BRTO (IR),
coiling/glue (GI)

622
Copyright © Harvard Medical School, 2018. All Rights Reserved.

Liver
• Transplant
– MELD score
• Highly predictive of 3 month mortality, used to triage
transplantation, MELD > 15
– OLT at MELD > 15 and decompensation
– Allocation is regional – variation
– LDLT
– Exclusions

Pancreas
– Acute Pancreatitis
• Most commonly due to gallstones or alcohol
• 2/3 criteria: typical pain, amylase/lipase > 3 x normal,
imaging
• Severity graded on presence of local complications and
degree of organ failure
• Early, aggressive fluid hydration, colloid
• Oral feeding > enteral feeding, avoid TPN; NGT ≈ NJT

623
Copyright © Harvard Medical School, 2018. All Rights Reserved.

Pancreas
– Acute Pancreatitis
• ERCP only in patients with concomitant cholangitis or
evidence of ongoing biliary obstruction
• Antibiotics for cholangitis, but not for prophylaxis of
severe AP
• Cholecystectomy in the same hospitalization if biliary
AP without necrosis

Pancreas
– Chronic Pancreatitis
• Toxic/Metabolic, Idiopathic, Genetic, Auto-Immune,
Recurrent, Obstructive
• Exocrine pancreatic insufficiency treated with
pancrealipase
• Opioid sparing pain regimen
• Steroids for AIP

624
Copyright © Harvard Medical School, 2018. All Rights Reserved.

Diarrhea
• Acute
– Viral > bacterial
– Norovirus, Rotavirus
– Salmonella (avoid abx), Campylobacter, C Diff,
EHEC (avoid abx)
– O&P only with > 14 days of symptoms, travel or
exposure

Diarrhea
• Chronic
– Inflammatory
– Medications
– EPI
– Celiac disease
– Microscopic colitis

625
Copyright © Harvard Medical School, 2018. All Rights Reserved.

GI - Take Home

626
Copyright © Harvard Medical School, 2018. All Rights Reserved.

GI BOARD REVIEW

Muthoka L. Mutinga, MD
Associate Physician
Division of Gastroenterology, Hepatology and Endoscopy
Department of Medicine
Brigham and Women’s Hospital
Assistant Professor of Medicine
Harvard Medical School

DISCLOSURES

• Muthoka Mutinga, MD- No disclosures

627
Copyright © Harvard Medical School, 2018. All Rights Reserved.

1. A 22 year old woman whose medical history is


only notable for moderate to severe acne, presents
for evaluation of worsening and persistent moderate
to severe retrosternal pain for the past two days. She
has had no similar symptoms in the past and does not
use illicit drugs. She has no personal or family
history of cardiovascular disease.

Which of the following is her likely diagnosis?

A. Dissecting aortic aneurysm

B. Esophageal spasms

C. Pill esophagitis

D. Reflux esophagitis

628
Copyright © Harvard Medical School, 2018. All Rights Reserved.

1. The answer is C
• Persistent retrosternal pain in a patient who may be taking
doxycycline for moderate to severe acne is highly
suggestive of pill induced esophagitis

• Other symptoms associated with pill esophagitis include


heartburn, odynophagia, dysphagia

• A discrete esophageal ulcer with normal surrounding


mucosa is typically seen endoscopically

2. A 20 year old man with ileocolonic Crohn’s disease


managed with 6-mercaptopurine presents for urgent
evaluation of anal pain and rectal bleeding. He is recovering
from a recent exacerbation of Crohn’s disease associated with
frequent non-bloody, watery stools. He reports acute onset of
sharp pain with defecation two days ago and also noticed a
small amount of bright red blood when wiping after
defecation. He has not been using suppositories or enemas to
manage the Crohn’s disease flare and reports no anal trauma,
fever, chills or abdominal pain.

629
Copyright © Harvard Medical School, 2018. All Rights Reserved.

Which of the following would you recommend for initial


management of his likely diagnosis?

A. Lateral internal sphincterotomy

B. Stool softener

C. Botulinum toxin injection

D. Sitz bath

2. The answer is D
• Patients with symptomatic anorectal disease, such as an anal fissure in
this case, often have elevated anal sphincter tone
• A warm sitz bath relaxes the internal anal sphincters and would be an
appropriate initial nonsurgical therapy for this patient
• Other nonsurgical approaches to manage anal fissures in Crohn’s
disease patients include measures to decrease diarrhea and bulk the
stools as well as topical medications such as nitrates and calcium
channel blockers to relax the internal sphincter
• The majority of acute anal fissures resolve without requiring surgical
therapy

630
Copyright © Harvard Medical School, 2018. All Rights Reserved.

3. A 41 year old man recently diagnosed with acute


uncomplicated sigmoid diverticulitis is referred for a follow-
up colonoscopy six weeks after completion of antibiotic
therapy. The colonoscopy exam is only notable for mild
sigmoid diverticulosis without evidence of inflammation.

Which of the following is associated with increased


risk of diverticulitis in men?

A. Tobacco use

B. Consumption of seeds and nuts

C. High fiber diet

D. Vigorous physical activity

631
Copyright © Harvard Medical School, 2018. All Rights Reserved.

3. The answer is A
• The incidence of diverticulitis has risen dramatically in the
United States
• Smoking is associated with increased risk of diverticulitis
• Consumption of seeds and nuts have not been shown to
increase the risk of diverticulitis contrary to popular belief
• Other lifestyle factors associated with a lower risk of
diverticulitis include:
– Low red meat consumption (<4 servings/week)
– Normal body weight (BMI 18.5-24.9)
– High dietary fiber intake (>23gm/day)

4. A 48 year-old man with a long history of


gastroesophageal reflux disease undergoes an
EGD which reveals a 5 centimeter long
segment of salmon-colored mucosa extending
proximally from the gastroesophageal junction.
Biopsies reveal Barrett’s esophagus without
dysplasia. Another endoscopy one year later
notes similar findings.

632
Copyright © Harvard Medical School, 2018. All Rights Reserved.

What is the proper subsequent surveillance


protocol for this patient?

A. Biopsy of the Barrett’s segment every 3-6


months
B. Biopsy of the Barrett’s segment annually
C. Biopsy of the Barrett’s segment every 3-5
years
D. No further surveillance is necessary

4. The answer is C.
• Barrett’s esophagus
– Intestinal metaplasia of the esophagus
– Associated with an increased risk of esophageal adenocarcinoma

• No dysplasia: repeat EGD in 3-5 years

• Indefinite for dysplasia: intensify acid suppressive therapy


and repeat EGD in 3-6 months

• Low grade dysplasia: confirm with 2nd pathologist and do


ablation therapy (preferred) or repeat EGD annually

• High grade dysplasia: confirm with 2nd pathologist and do


ablation therapy and endoscopic mucosal resection (EMR)
for discrete nodules.

633
Copyright © Harvard Medical School, 2018. All Rights Reserved.

5. A 21 year old college student is referred for evaluation of


rectal bleeding and intermittent loose stools associated with
urgency. His weight is stable and he reports no abdominal
pain. Stool culture do not reveal infection. The symptoms
have been ongoing for 1 month, but he delayed seeking
medical attention thinking symptoms would resolve. A
colonoscopy is performed and reveal inflammation localized
to the rectum. Biopsies reveal chronic active proctitis and
normal mucosa in all other regions of the colon.

Which of the following is associated with a poor


outcome in patients with ulcerative colitis?

A. Pancolitis
B. Older age
C. Normal c-reactive protein (CRP) level
D. Current smoker
E. Normal hemoglobin level

634
Copyright © Harvard Medical School, 2018. All Rights Reserved.

5. The answer is A.
• Extensive or pancolitis is associated with a higher risk of colectomy
• Younger age is associated with more severe disease, shorter time to
relapse and increased risk of colectomy
• Elevated inflammatory markers such as CRP and the erythrocyte
sedimentation rate (ESR) are associated with a higher risk of
colectomy
• Current smokers typically have a lower rate of relapse and fewer
hospitalizations
• On the other hand, nonsmokers and ex-smokers typically have more
extensive disease and a lower likelihood of disease regression
• Low hemoglobin or fibrinogen levels are independently associated
with treatment failure in patients with severe colitis.

6. A patient with hepatic cirrhosis due to


alcohol abuse is found to have large
esophageal varices on upper endoscopy,
but has no history of gastrointestinal
bleeding.

635
Copyright © Harvard Medical School, 2018. All Rights Reserved.

What is the appropriate therapy to


prevent future variceal bleeding?

A. Ursodiol
B. Sucralfate
C. Nonselective β-blocker
D. Endoscopic sclerotherapy
E. Proton pump inhibitor

6. The answer is C.
• Patients with portal hypertension and large esophageal
varices benefit from pharmacological prophylaxis (non-
selective β-blockers such as propranolol, nadolol and
timolol)
• Endoscopic sclerotherapy has been shown to be ineffective
and possibly harmful for prophylaxis of variceal
hemorrhage
• Sucralfate, ursodiol and acid inhibition therapy have no
role for prophylaxis of variceal hemorrhage
• Variceal band ligation has shown initial promise in patients
with large varices and may be considered in patients with
contraindications for nonselective β-blocker therapy

636
Copyright © Harvard Medical School, 2018. All Rights Reserved.

7. A 67 year-old man with a history of


congestive heart failure and peripheral
vascular disease presents with 48 hours of
left lower quadrant abdominal pain,
diarrhea with intermittent bleeding, and
low grade fever. Stool cultures reveal no
infection and ischemic colitis is
considered as a possible cause of his
symptoms.

Which of the following is true of this


disorder?
A. Digoxin may predispose patients to bowel
ischemia
B. Urgent angiography is indicated in order to
identify a culprit blood vessel
C. This disorder has the highest mortality rate of
the ischemic intestinal disorders
D. The diagnosis can be made on CT scan.
E. Serum lactate levels are markedly elevated in
this disorder

637
Copyright © Harvard Medical School, 2018. All Rights Reserved.

7. The answer is A.
• Ischemic colitis is a form of intestinal ischemia most often affecting
the descending and sigmoid colon (“watershed area”)
• Usually results from low vascular flow to a colonic segment, rather
than emboli or large vessel thrombosis
• Generally associated with low mortality
• Digoxin, a splanchnic vasoconstrictor, can predispose to both
mesenteric ischemia and ischemic colitis
• CT scan- may show colonic thickening, but can’t distinguish this from
other inflammatory processes
• Angiography- useful for suspected mesenteric ischemia, but of limited
value in ischemic colitis
• Serum lactate elevation- uncommon and usually associated with bowel
infarction and necrosis

8. A 58 year old man with no history of alcoholism,


chronic viral hepatitis or obesity is seen by his
primary care provider for evaluation of increased
abdominal girth as well as scrotal and lower
extremity edema. Abdominal ultrasound imaging is
notable for moderate ascites, patent hepatic and
portal veins and a normal appearing liver. Lab tests
to evaluate for chronic viral hepatitis and other forms
of chronic liver disease are unrevealing.
Arrangements are made for a diagnostic
paracentesis.

638
Copyright © Harvard Medical School, 2018. All Rights Reserved.

Ascites is classified via the serum-ascites albumin


gradient (SAAG), which is obtained by subtracting
the albumin level in ascites from that in the serum.
Which of these conditions will cause ascites with a
low SAAG gradient (<1.1mg/dL)?

A. Budd-Chiari Syndrome
B. Cirrhosis due to chronic hepatitis C
C. Peritoneal carcinomatosis
D. Congestive heart failure
E. Acute alcoholic hepatitis

8. The answer is C.
• The serum-ascites albumin gradient (SAAG)
differentiates between ascites due to portal
hypertension (SAAG>1.1) and that due to non-
portal hypertensive states (SAAG<1.1), with 97%
accuracy.
• Low gradient ascites (SAAG<1.1) can be seen in
conditions such as peritoneal carcinomatosis,
tuberculous peritonitis and fungal infections of the
peritoneum.

639
Copyright © Harvard Medical School, 2018. All Rights Reserved.

9. A 47 year-old woman presents with a 4-


month history of watery diarrhea and weight
loss of 25 pounds. Laboratory examination
reveals a mild iron deficiency anemia and a low
serum calcium. Stool cultures and examination
for ova and parasites are unremarkable. A
colonoscopy with random biopsies is normal. A
tissue transglutaminase (tTG) antibody is
strongly positive.

Which of the following statements


regarding this disorder is true?

A. Adherence to a gluten-free diet may reduce the risk of


developing risk of small intestinal lymphoma
B. This disease is more common in patients with
Type II diabetes mellitus
C. This disease is seen most commonly in patients of
Mediterranean background
D. Small intestinal biopsy in those who are compliant with a gluten
free diet typically reveals villous atrophy, deepened crypts and
intraepithelial lymphocytes
E. There is no association between this disease and
autoimmune thyroid disorders

640
Copyright © Harvard Medical School, 2018. All Rights Reserved.

9. The answer is A.
• The constellation of symptoms, evidence of nutrient
malabsorption and positive tissue transglutaminase (tTG)
antibody are consistent with celiac sprue.
• Primarily affects people of Northern European descent,
though it has worldwide distribution.
• May be associated with other autoimmune disorders such
as Type I diabetes mellitus and autoimmune thyroid
disease (e.g. Hashimoto’s thryoiditis).
• Treatment-- gluten free diet; this eliminates symptoms in
most people and decreases cancer risk, such as small
intestinal lymphoma and adenocarcinoma.
• Histological abnormalities also typically resolve with
adherence to a gluten free diet.

10. A 35 year-old woman is found to


have the following iron studies:
• Serum iron: 186
• Total iron binding capacity: 255
• Serum ferritin: 280 (normal 25-240)
• Transferrin saturation: 73%

Her liver function tests are normal, she


is asymptomatic, and has a normal physical
examination

641
Copyright © Harvard Medical School, 2018. All Rights Reserved.

Which of the following tests would be the most


ideal to obtain to confirm her diagnosis?

A. Percutaneous liver biopsy


B. Serum B12 and folate levels
C. Glucose tolerance testing
D. Testing for hereditary hemochromatosis
gene mutation
E. Abdominal CT scan with IV contrast

10. The answer is D.


• A transferrin saturation >55% is suggestive of an iron
overload syndrome.
• Transferrin saturation= serum iron/TIBC
• The ferritin may also be elevated in patients with
hemochromatosis-- less likely in menstruating women
• HFE gene analysis (hemochromatosis genetic test) would
be the next test to obtain; can detect 2 of the most common
mutations (C282Y & H63D)
• CT scanning is not sensitive enough to determine degree of
iron overload.
• A liver biopsy is not needed in a young, asymptomatic
patient.

642
Copyright © Harvard Medical School, 2018. All Rights Reserved.

11. A 28 year old woman with recent onset of moderate upper


abdominal pain exacerbated by eating and associated with
mild nausea and early satiety is seen for further evaluation by
her primary care provider. She also reports mild weight loss
associated with the symptoms. She does not take aspirin or
NSAIDs. She uses an inhaler for mild asthma and does not
take any other medications. Laboratory testing including a
complete blood count, liver function tests and TSH are
normal and the H. pylori breath test is negative.
Subsequently an esophagogastroduodenoscopy is performed.
Mild gastric mucosal erythema is noted, but biopsies are
histologically normal as are duodenal biopsies.

Which of the following would you order next to


establish her diagnosis?

A. Breath test to evaluate for small intestinal


bacterial overgrowth
B. Gastric emptying study
C. Abdominal ultrasound
D. Magnetic resonance cholangiopancreatography
(MRCP)

643
Copyright © Harvard Medical School, 2018. All Rights Reserved.

11. The answer is B


• Upper abdominal pain is often overlooked as a
symptom associated with gastroparesis
• In this case the symptoms of nausea and early
satiety in the absence of a mechanical cause of or
explanation on upper endoscopic examination
further favors the diagnosis of gastroparesis
• A solid phase gastric emptying study would help
to establish the diagnosis

12. A 42 year old presents to a travel clinic


requesting advice about reducing risk of
exposure to Hepatitis E. He is planning to
travel to an endemic area.

644
Copyright © Harvard Medical School, 2018. All Rights Reserved.

Which of the following statements regarding


Hepatitis E is true?

A. The virus is endemic in Southeast Asia


B. The clinical features of hepatitis E are similar to those
of hepatitis B
C. The virus is transmitted primarily via percutaneous
blood exposure
D. Hepatitis E is associated with a high rate of fulminant
hepatic failure in most people who acquire the
infection.

12. The answer is A.

• Hepatitis E is endemic in Southeast Asia, Africa,


the Middle East and Central America.
• It is transmitted via the fecal-oral route
• The infection is generally associated with a self-
limited icteric illness, with course similar to
Hepatitis A.
• Pregnant women have a high rate of fulminant
hepatic failure- up to 25%.
• A Hepatitis E vaccine is in development.

645
Copyright © Harvard Medical School, 2018. All Rights Reserved.

13. A 34 year old woman is seen by an ENT


specialist for evaluation of a chronic sore
throat. A diagnosis of gastroesophageal
reflux (GERD) is suspected.

Which of the following is another atypical


manifestations of GERD?

A. Exertional asthma

B. Hemoptysis

C. Hoarseness

D. Chronic diarrhea

646
Copyright © Harvard Medical School, 2018. All Rights Reserved.

13. The answer is C.

• A variety of clinical syndromes have been associated with


GERD such as hoarseness, chronic laryngitis, chronic non-
productive cough, nocturnal asthma and non-cardiac chest
pain.
• A 24-hour intraesophageal pH monitor can be used to
confirm the presence of acid reflux.
• Alternatively an empiric trial of a proton pump inhibitor
could be tried.
• There is no association between simple GERD and
exertional asthma, hemoptysis and chronic diarrhea.

14. A 62 year old man with recently diagnosed


compensated cirrhosis due to nonalcoholic
steatohepatitis (NASH) is advised to discontinue
statin therapy by his primary care provider due to
concerns about possible hepatoxicity. He is obese,
has hypertension and dyslipidemia, but does not
have diabetes mellitus or coronary heart disease.

647
Copyright © Harvard Medical School, 2018. All Rights Reserved.

Which of the following is believed to be true


concerning the effects of statin therapy in patient
with chronic liver disease?

A. Statins may decrease overall survival


B. Statins may increase the risk of hepatic
decompensation
C. Statins may decrease the risk of developing
hepatocellular carcinoma
D. Statins may retard the progress and/or
development of cirrhosis

14. The answer is D


• Recent data shows no evidence that statins are deleterious
in patients with chronic liver disease including
compensated cirrhosis
• In fact, statins appear to retard the progression and/or
development of cirrhosis, likely by inhibiting profibrotic
cytokines
• Statins appear to improve overall survival in patients with
chronic liver disease and decrease the risk of hepatic
decompensation
• Statins are not known to have any direct effect on the
development of hepatocellular carcinoma

648
Copyright © Harvard Medical School, 2018. All Rights Reserved.

15. A 62 year old man is referred for upper


endoscopy for evaluation of persistent
epigastric pain. He does not take aspirin or
NSAIDs and has not experienced melena or
weight loss. He does an internet search and is
worried that he may have a H. pylori infection.

Which of the following is a potential sequela of


chronic H. pylori infection?

A. Gastric varices

B. Gastric fundic gland polyps

C. Gastric MALT lymphoma

D. Duodenal adenocarcinoma

649
Copyright © Harvard Medical School, 2018. All Rights Reserved.

15. The answer is C.

• H. pylori is a gram-negative, urease-producing bacterium that


colonizes the gastric mucosa.
• There is an association between MALT lymphoma and chronic H.
pylori infection. In addition, chronic H. pylori infection can be
associated with a risk for developing gastric adenocarcinoma
• There is a clear association between H. pylori infection and duodenal
ulcer disease eradication reduces ulcer recurrence by 80%.
• Chronic H. pylori infection does not increase the risk of developing
gastric varices, fundic gland polyps or duodenal adenocarcinoma
• Duodenal adenocarcinomas typically arise from adenomatous polyps
in the duodenum

16. A 64 year old woman with COPD presents for evaluation


of recurrent diarrhea and mild diffuse abdominal pain. Of
note was diagnosed and treated for C. difficile infection three
months ago. This first episode occurred after receiving
antibiotic therapy for acute bronchitis. She had a confirmed
recurrence of C. difficile infection six weeks later from which
she promptly recovered after antibiotic therapy. She was well
until the current presentation and had not received other
antibiotics in the interim. Stool testing is again positive for C.
difficile toxin, and no other pathogens are identified. Of note,
the first episode of C. difficile infection was treated with oral
metronidazole for 10 days and the second with oral
vancomycin for 10 days.

650
Copyright © Harvard Medical School, 2018. All Rights Reserved.

…continued
Her labs are normal except for a white blood count of 16,000
cells/mm3. She is afebrile, normotensive and not tachycardic.
She is unwilling to consider a fecal microbiota transplantation
after treatment of the current C. difficile infection.

Which of the following would be the most


appropriate treatment for this second recurrence of
C. difficile infection?

A. Oral vancomycin for 14 days

B. Rifaximin for 14 days

C. Fidaxomicin for 10 days

D. Metronidazole for 14 days

651
Copyright © Harvard Medical School, 2018. All Rights Reserved.

16. The answer is C


• A second recurrence of C. difficile infection should be treated with
Fidaxomicin 200mg orally BID for 10 days or a pulsed or tapering
dose of oral vancomycin
• Fidaxomicin has the advantage of being associated with a lower risk of
recurrence of C. difficile infection compared to currently available
antimicrobial agents, but it is also very expensive
• Fecal microbiota transplantation is a reasonable consideration after a
second recurrence of C. difficile infection
• Oral metronidazole (500mg TID for 10- 14 days) or oral vancomycin
(125mg QID for 10-14 days) are the preferred treatment for mild to
moderate initial C. difficile infection and can be repeated for the first
recurrence.

17. An 86 year-old man develops abdominal


cramps, watery diarrhea and low grade fevers.
Within two days, his stools become bloody, and
he feels weak and light-headed. He is seen in
the office and is noted to have a temperature of
100.8 and is normotensive but mildly
orthostatic. He has a diffusely tender abdomen
with no peritoneal signs, and bloody stool in the
rectal vault. He also has a fine petechial rash
on his lower extremities. Labs are notable for
hematocrit of 27%, platelet count of 48K/µL,
and creatinine of 3.5mg/dL. Of note, his prior
labs 6 months ago were all normal.

652
Copyright © Harvard Medical School, 2018. All Rights Reserved.

Which of the following statements


regarding this disorder is true?
A. The hematologic and renal complications of
this disorder are seen most often in middle-aged
adults
B. The illness is primarily transmitted through
the ingestion of poorly-cooked meat products
C. Antibiotics are effective in preventing
complications in this illness
D. In general, the diarrheal illness is severe
E. The disease is caused by Gram-positive
bacteria

17. The answer is B.


• Hemolytic-uremic syndrome (HUS) is a sequela of
E. coli O157:H7 infection (EHEC), a gram
negative bacterium.
• Transmitted via poorly cooked meat products.
• Generally self limited, except in the very young
and in the elderly.
• Antibiotics have not been shown to prevent HUS,
and in fact, may promote the development of
HUS.

653
Copyright © Harvard Medical School, 2018. All Rights Reserved.

18.A screening colonoscopy on a 50 year


old man with no family history of colon
cancer reveals two small, flat polyps in
the ascending colon which are removed.
The pathology report indicates that they
are sessile serrated adenomas.

Which statement regarding serrated


adenomas is true?
A. They are not associated with an increased risk
of metachronous colon cancer
B. The prevalence of serrated polyps is very low
(<1%)
C. They disproportionally contribute to interval
colon cancers
D. They are easily detected on colonoscopic
examination

654
Copyright © Harvard Medical School, 2018. All Rights Reserved.

18. The answer is C.


• Serrated polyps are:
– Characterized histologically by serrated (saw-tooth) appearance of
crypt epithelium
– More easily missed on colonoscopic exam due to their flat
morphology and ambiguous color, especially in the proximal colon

• A recent metanalysis of data published between 2009-2014


estimate that the prevalence of serrated polyps ranged
from 5.6-28.7%

• Serrated polyps are associated with an increased risk of


metachrounous cancer and certain types are also
associated with increased risk of synchronous advanced
neoplasia

• They also disproportionally contribute to interval colon


cancers

19. A 52 year old man with cirrhosis due to chronic


hepatitis C, genotype 3, recently had confirmed
sustained virologic response (SVR) after completion
of direct acting antiviral therapy. He is seen for an
annual physical exam and reports no interval
symptoms. He has no history of complications
related to cirrhosis. He is concerned about his future
risk of developing hepatocellular cancer (HCC).

655
Copyright © Harvard Medical School, 2018. All Rights Reserved.

Which of the following statements concerning


HCC is true?

A. Alcohol induced cirrhosis is the leading risk factor for


HCC in the U.S.

B. Following SVR, cirrhotic patients’ risk for developing


HCC is similar to non-cirrhotic patients

C. Hepatocellular cancer is the fastest growing cause of


cancer death in the U.S.

D. Direct acting antiviral therapy eliminates subsequent risk


of developing HCC in patients who achieve SVR

The answer is C.

• Hepatocellular cancer (HCC) is the fastest growing cause of cancer


death in the U.S.
• Chronic hepatitis C is the leading risk factor for HCC in the U.S., not
alcohol induced cirrhosis
• Cirrhotic patients who achieve SVR after receiving direct acting
antiviral therapy for chronic hepatitis C, still have a >4 fold greater
incidence of HCC compared to non-cirrhotic patients
• The cumulative incidence of HCC is lower in patients with chronic
hepatitis C who achieve SVR following therapy with direct acting
antivirals compared to those who fail to achieve SVR

656
Copyright © Harvard Medical School, 2018. All Rights Reserved.

20. A 31 year old with recently diagnosed


ulcerative colitis is concerned about the
long term risk of developing colon cancer.
He is experiencing a mild exacerbation of
ulcerative colitis and requests and
appointment for evaluation and to discuss
his concerns about colon cancer risk.

Which one of the following factors in patients with Crohn’s


or ulcerative colitis is associated with a low risk of
developing colorectal cancer?

A. Extensive and active colonic disease

B. Isolated proctitis

C. Primary sclerosing cholangitis (PSC)

D. Prior dysplasia or colonic stricture

657
Copyright © Harvard Medical School, 2018. All Rights Reserved.

20. The answer is B.


• Isolated proctitis is associated with a low risk of
colorectal cancer.
• Current guidelines recommend annual surveillance
colonoscopy in patients with inflammatory bowel
disease who are at high risk of developing
colorectal cancer.
• These high risk groups include patients with:
– Prior dysplasia or colonic stricture
– Family history of colorectal cancer in a first-degree
relative age <50
– Primary sclerosing cholangitis (PSC), regardless of
duration of inflammatory bowel disease
– Extensive, active disease

21. A 58 year old presents with a 5 year history of


progressive dysphagia for liquids and solids. She
describes occasional nocturnal regurgitation of food. A
barium esophagram reveals a dilated esophagus with
beak-like narrowing at the level of the
gastroesophageal junction. An upper endoscopy
reveals no masses. High resolution esophageal
manometry confirms that she has type II achalasia.

658
Copyright © Harvard Medical School, 2018. All Rights Reserved.

Which therapy would likely result in the highest


likelihood of relief of dysphagia symptoms?

A. Pneumatic dilation

B. Isosorbide mononitrate

C. Botulinum toxin injection of the LES

D. Nifedipine

21. The answer is A.


• Achalasia results from progressive loss of ganglion cells in the
myenteric plexus.

• Treatment that disrupts the muscles of the lower esophageal sphincter


such as pneumatic dilation and surgical myotomy provide the greatest
symptom relief in patients with type II achalasia (achalasia with
pressurization of the esophageal body).

• Medical therapy such as botulinum toxin injections of the lower


esophageal sphincter, nitrates and calcium channel blockers are less
effective in reducing dysphagia symptoms.

• Role of esophagogastroduodenoscopy (EGD) is to rule out causes of


pseudoachalasia such as adenocarcinoma of the stomach or distal
esophagus.

659
Copyright © Harvard Medical School, 2018. All Rights Reserved.

22. A 35 year old with mild Crohn’s disease


presents with symptoms of intense epigastric
pain. He doesn’t drink alcohol. His only
medication is oral mesalamine. Laboratory
studies are notable for lipase of 2500 U/L (nl
<60 U/L), total bilirubin 2.5mg/dL (nl: 0.2-
1.2mg/dL), direct bilirubin 0.5 mg/dL, AST
17 U/L, ALT 25 U/L, alkaline phosphatase 72
U/L, creatinine 0.7mg/dL and hematocrit is
38% after hydration. He does not meet any
systemic inflammatory response syndrome
(SIRS) criteria .

Which of the following imaging tests would


be the most appropriate at this time?

A. ERCP

B. Abdominal ultrasound

C. MRCP

D. KUB

660
Copyright © Harvard Medical School, 2018. All Rights Reserved.

22. The answer is B.


• Patients with Crohn’s disease are at increased risk of
developing gallstones.
• Since he has no definite cause for pancreatitis, an
abdominal ultrasound to assess for gallstones is appropriate
at this time.
• Mesalamine infrequently causes acute pancreatitis.
• ERCP is indicated in patients with suspected ascending
cholangitis or high grade biliary obstruction.
• The patient’s indirect hyperbilirubinemia is suggestive of
Gilbert’s syndrome rather than an obstructive biliary
process, hence MRCP is not indicated.

23. A 42 year old man is found to have occult blood


in the stool on routine digital rectal exam during an
physical exam. He has no upper or lower
gastrointestinal symptoms and does not take aspirin
or NSAIDs. There is no family history of
gastrointestinal malignancy and his hematocrit is
normal.

661
Copyright © Harvard Medical School, 2018. All Rights Reserved.

Which of the following could potentially cause


occult gastrointestinal blood loss?

A. Meckel’s diverticulum

B. Barrett’s esophagus

C. Colonic diverticulosis

D. Cameron lesions

23. The answer is D.


• Cameron lesions are erosions or ulcers that develop on the lower
margin of a large hiatal hernia sac and may be associated with acute
and chronic blood loss as well as iron deficiency anemia
• A Meckel’s diverticulum is a congenital vestigial remnant of the
omphalomesenteric duct, located in the ileum, usually within 2 feet of
the ileocecal valve. Most are asymptomatic, but they can occasional
cause overt bleeding, rather than occult blood loss
• Colonic diverticulosis are a fairly common cause of overt lower
gastrointestinal hemorrhage, but are associated with occult blood loss
• Barrett’s esophagus, characterized by intestinal metaplasia of the
esophagus, is a non-inflammatory lesion and is not associated with
occult blood loss

662
Copyright © Harvard Medical School, 2018. All Rights Reserved.

24. A 42 year old woman with hypertension, type II diabetes


mellitus, dyslipidemia and obesity is noted to have multiple
small gallstones on ultrasound imaging performed to evaluate
transaminase elevation (ALT and AST <100 IU/L). The
common bile duct measures 3mm and there is no intrahepatic
ductal dilation. She reports no history of episodic, severe
upper abdominal pain. She is very anxious about the
possibility of developing symptoms or complications from the
gallstones especially since she has diabetes mellitus.

What is the recommended course of action for this


patient?

A. Elective cholecystectomy

B. Ursodeoxycholic acid

C. Lithotripsy

D. No intervention

663
Copyright © Harvard Medical School, 2018. All Rights Reserved.

24. The answer is D


• The risk of gallstone related complications
in a person with asymptomatic cholelithiasis
is approximately 20% over 15 years of
follow-up
• The risk of cholecystectomy outweighs the
benefits, even in diabetic patients

25. A 32 year old man who is new to your practice


is seen for follow-up after hospitalization for asthma
exacerbation. He recently found out that his 58 year
old father was diagnosed with Stage II colon cancer.
There are no other first or second degree relatives
with colon cancer or polyps. He seeks your advise
regarding when he should commence colorectal
cancer screening. He is asymptomatic.

664
Copyright © Harvard Medical School, 2018. All Rights Reserved.

At what age should he begin colorectal cancer


screening?

A. Age 35

B. Age 40

C. Age 48

D. Age 50

25. The answer is B


• Colorectal cancer screening for individuals
with a first degree relative with colorectal
cancer should begin at age 40, or 10 years
before the age of diagnosis of the youngest
affected relative, whichever is earlier.

665
Copyright © Harvard Medical School, 2018. All Rights Reserved.

26. A 33 year old man with no significant past


medical history presents to emergency department
for evaluation of persistent right lower quadrant
pain associated with anorexia, nausea and subjective
fever. His symptoms began acutely the night before
and he initially had generalized lower abdominal
pain. He has a low grade fever and is not ill
appearing. A clinical suspicion of acute appendicitis
is confirmed with ultrasound imaging. He is fearful
of surgery and insists on nonoperative management.

Which one of the following statements is true regarding


nonoperative management of acute uncomplicated
appendicitis?

A. It is the optimal strategy for patients with multiple


medical comorbidities

B. Very few patients who pursue this option require


subsequent appendectomy

C. It is a safe option for patients with fecaliths seen on


imaging

D. After recovery, colonoscopy should be performed on


patients >40 years old, who have had no prior exam

666
Copyright © Harvard Medical School, 2018. All Rights Reserved.

26. The answer is D


• Timely open or laparoscopic surgery remains the recommended
treatment of choice for adults with uncomplicated appendicitis
• Colonic neoplasms can mimic appendicitis, therefore colonoscopy
screening should be performed on those age 40 or above, after
recovery
• Patients with multiple medical comorbidities were excluded from trials
of nonoperative management of acute appendicitis, thus the efficacy of
this strategy is unknown in these patients
• Up to 30% of patients who were initially successfully treated with
antibiotics require appendectomy for recurrent appendicitis or for pain
• Up to 40% of patients with fecaliths seen on imaging have complicated
appendicitis

27. A 44 year old man with poorly controlled type II diabetes


mellitus and morbid obesity presents to the Emergency
Department for evaluation of severe acute epigastric pain
radiating to his back for the past 3 hours. He is afebrile, heart
rate is 92 beats/minute, BP 154/90 mm Hg, respiratory rate of
16 breaths/min, and oxygen saturation is 98% on room air .
His exam is notable for hypoactive bowel sounds and
moderate epigastric tenderness with light palpation. His
sclera are anicteric. He has no occult blood in his stool.

667
Copyright © Harvard Medical School, 2018. All Rights Reserved.

LABS:
White blood count: 14,000 mm3 (nl: 4,500-11,000 mm3)
Hematocrit: 43%
ALT: 84 U/L (nl: 7-56 U/L)
AST: 55 U/L (nl: 0-40 U/L)
ALK: 99 IU/L
Total bilirubin: 0.4 mg/dL
Sodium: 126 mEq/L (nl: 135-145 mEq/L)
Glucose: 95mg/dL
Amylase: 55 U/L (nl: 23-85 U/L)
Lipase: 4589 U/L (nl: 0-160 U/L).

An abdominal ultrasound is notable for increased echogenicity of the


liver suggestive of fatty deposits, normal gallbladder without stones or
sludge, common bile duct measures 3mm in diameter and there is no
intrahepatic duct dilation. An abdominal/pelvic CT scan confirms
interstitial pancreatitis with peripancreatic fat stranding and no necrosis.
He does not consume alcohol.

What is the likely case of the acute pancreatitis?

A. Gallstones

B. Pancreatic mucinous neoplasm

C. Hypertriglyceridemia

D. Sphincter of Oddi dysfunction

668
Copyright © Harvard Medical School, 2018. All Rights Reserved.

27. The answer is C


• Hypertriglyceridemia is an important risk factor for acute
pancreatitis when levels exceed 1000mg/dL
• Hypertriglyceridemia-induced acute pancreatitis is the
third most common cause of acute pancreatitis after
alcohol and gallstones.
• Excess triglyceride in serum can displace water containing
sodium and cause pseudo-hyponatremia, a clue in this case
• Serum triglyceride levels >500mg/dL may interfere with
colorimetric reading resulting in falsely normal amylase,
another clue in this case. Lipase results are unaffected.
• Secondary causes of hypertriglyceridemia include poorly
controlled diabetes mellitus, medications, pregnancy,
alcohol and hypothyroidism.

28. A busy Gastroenterology practice begins


providing quarterly reports of adenoma detection
rates (ADR) to each provider, in an effort to
motivate providers to improve their adenoma
detection rates

669
Copyright © Harvard Medical School, 2018. All Rights Reserved.

28. Which of the following statements is true


regarding the adenoma detection rate (ADR)?

A. The bowel preparation quality has little bearing on the


ADR

B. ADR in the proximal colon is higher than in the distal


colon

C. ADR increases with procedures performed late in an


Endoscopist’s schedule

D. Short colonoscopy withdrawal time (< 6 minutes) is


associated with lower ADR

28. The answer is D.


• Adenoma detection rate (ADR), defined as the percent of
colonoscopies in which one or more adenomas are
detected, increases with longer colonoscopy withdrawal
time
• The optimal colonoscopy withdrawal time is unknown, but
some studies suggest at least 6 minutes is optimal
• Serrated adenomas are more likely to be located in the
proximal colon, may be quite flat, and thus may be more
difficult to detect
• Optimal cleansing of the colon enhances the adenoma
detection rate
• Fatigue may contribute to lower adenoma detection rates
for later procedures in a Endoscopist’s schedule

670
Copyright © Harvard Medical School, 2018. All Rights Reserved.

29. A 23 year old returning foreign traveler is seen


in urgent care with complaints of unilateral knee
pain and swelling. She reports no trauma and is not
sexually active. She had a self-limited diarrheal
illness during her trip. She is well appearing and lab
tests are unrevealing.

Which one of the statements regarding reactive


arthritis following an enteric infection is true?

A. Intestinal infection with ameba has been


associated with this syndrome

B. Many affected people are HLA DQ2 and DQ8


antigen-positive

C. There is a high female to male ratio

D. It may be associated with a triad of arthritis,


conjunctivitis and urethritis

671
Copyright © Harvard Medical School, 2018. All Rights Reserved.

29. The answer is D


• Post-enteric reactive arthritis, formerly known as Reiter’s syndrome,
usually develops 2-4 weeks after an acute diarrheal illness

• It is more common in men than women and many affected patients are
HLA B27 antigen positive

• It may be associated with a triad of arthritis, conjunctivitis and


urethritis

• Shigella sp. is the most commonly associated enteric organism,


although Salmonella sp, Campylobacter jejuni, Yersinia enterocolitica
and even Clostridium difficile have been implicated.

• Amebic intestinal infections are not typically associated with reactive


arthritis

30. A 36 year old woman is seen in the office for


evaluation of a sensation of a lump in her throat
ongoing for several months. She reports no
dysphagia or odynophagia. Her head and neck exam
is unremarkable.

672
Copyright © Harvard Medical School, 2018. All Rights Reserved.

Which one of the following is a common cause of a


globus sensation?

A. Parathyroid adenoma

B. Gastroesophageal reflux disease

C. Depression

D. Myasthenia gravis

30. The answer is B.


• Globus refers to the sensation of a lump or foreign body in
the throat in the absence of dysphagia and odynophagia
• Gastroesophageal reflux, goiter, early hypopharyngeal
cancer and anxiety disorders can all be associated with the
globus sensation
• Myasthenia gravis can result in oropharyngeal dysphagia,
but not the globus sensation
• Parathyroid adenomas are generally very small and
unlikely to cause a globus sensation

673
Copyright © Harvard Medical School, 2018. All Rights Reserved.

DISCLOSURES
• Muthoka Mutinga, MD – No disclosures

Selected References
1. Beaty JS and Shashidharan M. Anal fissure. Clin Colon Rectal Surg 2016
Mar;29(1):30-37.
2. Liu P-H, Cao Y, Keeley B, et al. Adherence to a Healthy Lifestyle is
Associated With a Lower Risk of Diverticulitis Among Men. Am J
Gastrenterol 2017;112:1868-1876.
3. Kamal S, Khan M, Seth A, et al. Beneficial Effects of Statins on the Rate of
Hepatic Fibrosis , Hepatic Decompensation and Mortality in Chronic Liver
Disease: A Systematic Review and Meta-Analysis. Am J Gastroenterol
2017;112: 1495-1505.
4. Kanwal F, Kramer J, Asch S, et al. Risk of Hepatocellular Cancer in HCV
Patients Treated With Direct-Acting Antiviral Agents. Gastroenterol
2017;153:996-1005.
5. Rex DK, Boland CR, Dominitz JA, et al. Colorectal Cancer Screening:
Recommendations for Physicians and Patients from the U.S. Multi-Society
Task Force on Colorectal Cancer. Am J Gastroenterol 2017 Jul;112(7):1016-
1030

674
Copyright © Harvard Medical School, 2018. All Rights Reserved.

Oncology Pearls for the Boards


Wendy Y. Chen, MD MPH
Senior Physician
Department of Medical Oncology
Psychiatry Overview
Dana Farber Cancer Institute
Brigham and Women’s Hospital
Harvard Medical School

DISCLOSURE

None related to this lecture

675
Copyright © Harvard Medical School, 2018. All Rights Reserved.

“Pearl” - a beautiful thing produced as the


result of a long, irritating process which seems
pointless at the time it is being endured

Oncology Pearls
• Oncologic emergencies
• Important issues for common cancers
• Screening in different populations
• Oncology issues in primary care

676
Copyright © Harvard Medical School, 2018. All Rights Reserved.

Case #1

60 y.o. man with 80 pack-yr smoking hx c/o 1


week history of facial swelling and cough
Physical examination: distended neck veins,
prominent chest wall venous distention,
facial edema
Chest CT: bulky right hilar/mediastinal
lymphadenopathy

Case #1
A. Proceed with mediastinoscopy or bronchoscopy
to obtain tissue diagnosis
B. Initiate heparin and radiation with plan to biopsy
when swelling improves
C. Initiate steroids and heparin with plan for biopsy
when swelling improves
D. Initiate heparin and emergent stent placement
then biopsy once swelling improves
E. Explain to patient this is metastatic cancer and
he should consider hospice

677
Copyright © Harvard Medical School, 2018. All Rights Reserved.

Oncology pearl
• Describe the signs and symptoms of
and optimal treatment for superior vena
cava syndrome.

– Ref. Wilson LD et al. SVC syndrome with


malignant causes. N Engl J Med 2007;
356: 1862-9.

Superior Vena Cava Syndrome


• Majority due to malignancy
• External compression -mediastinal mass
– Most common malignancies – lung cancer
(both NSCLC and small cell) and lymphoma
• Thrombosis
– More common now with catheters

678
Copyright © Harvard Medical School, 2018. All Rights Reserved.

Signs and symptoms


• Physical examination
– Facial edema and plethora
– Jugular venous distension
– Venous distension of superficial veins on
chest
• Symptoms
– Shortness of breath
– Cough
– Hoarseness

Management of SVC syndrome


• Tissue diagnosis critical for Rx decisions
– Prognosis depends on underlying disease
• Treatment plan depends on tumor histology
– Chemo-insensitive cancers (e.g. NSCLS)
treated with upfront XRT
– Chemo-sensitive tumors (e.g. small cell,
lymphoma) treated with upfront chemo
– Stent placement not usually done 1st line in
cancers that may respond to chemo/RT
=> Rarely fatal

679
Copyright © Harvard Medical School, 2018. All Rights Reserved.

Case #2:

64 y.o. male with metastatic prostate


cancer to bone with metastases to ribs,
pelvis, and multiple vertebrae. Currently
on docetaxel chemo. Calls c/o of 1-2
weeks increasing mid-back pain. He has
no weakness, radicular pain, bowel or
bladder difficulties. Neurologic exam is
normal.

Case #2:
A. Order a bone scan and CT scan to
look for disease progression.
B. Prescribe NSAIDs and oxycodone as
needed for pain.
C. Obtain an MRI of the spine.
D. Recommend radiation to the spine.

680
Copyright © Harvard Medical School, 2018. All Rights Reserved.

Oncology pearl
• Describe the signs and symptoms and
optimal management of spinal cord
compression.
– Ref: Ropper AE and Ropper AH. Acute
spinal cord compression. N Engl J Med
2017; 376: 1358-69

Spinal Cord Compression


• Initiate treatment earlier to prevent
neurologic deficits
– Once neurologic deficits occur, often
irreversible
• Usually from epidural compression from
vertebral body metastases
– Most common tumors: lung, breast, prostate,
myeloma, lymphoma, renal cell
– Thoracic spine most common location (60%)
– Intramedullary metastases less common

681
Copyright © Harvard Medical School, 2018. All Rights Reserved.

Ropper N Engl J Med 2017

Symptoms
• BACK PAIN!
– New or worsening back pain with known
vertebral mets mandates further evaluation
– Pain may be radicular, but not always
• Weakness
– Motor deficits more common than sensory
• Bowel and bladder symptoms occur late
• Neurologic exam may be normal
– Key is early diagnosis
• MRI is imaging of choice

682
Copyright © Harvard Medical School, 2018. All Rights Reserved.

Treatment
• Corticosteroids to ↓ edema
– Only short-term benefit
• Should not be used if diagnosis unknown
– Typically 10 mg IV load then 4 mg po q 6hrs
• Radiation 1st line treatment for most
• Upfront surgery reserved for:
– Unknown diagnosis
– Progression during or after radiation
– Spinal instability
– One RCT showed improved function with
immediate surgery for less radiosensitive tumors
with single area of compression

Case #3:

62 y.o. man presents to ER with temperature


of 101.4°F, malaise, and cough 15 days
after receiving chemotherapy for refractory
lymphoma. Physical exam is unrevealing
for source. WBC is 640 cells/mm3 with 45%
polys. CXR is negative. Cultures are
obtained.

683
Copyright © Harvard Medical School, 2018. All Rights Reserved.

Case #3:

A. Admit patient and start cefepime


B. Admit patient, start cefepime and G-
CSF
C. Admit patient and observe
D. Discharge patient home on oral
quinolone and G-CSF
E. Discharge patient home on oral quinolone

Oncology pearl
• Describe the optimal management of
febrile neutropenia.

– Ref: Freifeld AG et al. Executive summary:


clinical practice guideline for the use of
antimicrobial agents in neutropenic
patients with cancer: 2010 update by the
Infectious Diseases Society of America.
Clin Infect Dis 2011; 427-31

684
Copyright © Harvard Medical School, 2018. All Rights Reserved.

Fever and Neutropenia


Fever
• Oral temperature > 38.3°C (101°F) or
38.0 °C (100.4 °F) for > 1 hr

Neutropenia
• Absolute neutrophil count < 500 cells/µL
or ANC < 1,000 with predicted nadir of <
500 in next 48 hrs

Fever and Neutropenia


• Risk for occult infection and mortality ↑ as
ANC falls below 1,000/mm3
– Greatest risk with ANC < 500/mm3
– Mortality rate for solid tumors less than heme
malignancies
• Growth factors (GCSF)
– Modestly ↓ duration of neutropenia and
hospitalization
⇒No impact on mortality
⇒No significant benefit to empiric use of GCSF in
uncomplicated F & N

685
Copyright © Harvard Medical School, 2018. All Rights Reserved.

Risk Factors for F&N

• Rapid decline in ANC


• Prolonged duration of neutropenia (>7-10d)
• Leukemic induction
• Uncontrolled cancer
• Hematologic cancers

Typical infectious sources of F&N

• Catheters
• Skin
• Respiratory tract
• Sinuses
• GI tract
=>Source identified in less than 30% of
cases
– Endogenous flora in 80% of cases

686
Copyright © Harvard Medical School, 2018. All Rights Reserved.

Likely Organisms
• Gram-positive infections (50-60%)
– Staph epidermidis
– Streptococcus
– Enterococcus faecalis/faecium
• Gram-negative rods (more likely to
cause death)
– Enterbacteriaceae (E. coli, Klebsiella)
– Pseudomonas aeruginosa

Routine evaluation

• History
• Physical exam
• CBC, chemistries, LFTs, urine analysis
• Blood/sputum/urine cultures
• CXR
• Consider directed radiology:
chest/abd/sinus CT if warranted by
symptoms

687
Copyright © Harvard Medical School, 2018. All Rights Reserved.

Treatment
Empiric antibiotics: broad spectrum with gram positive
and gram negative coverage (especially
Pseudomonas)
• 3rd generation cephalosporin (cefepime or ceftaz)
– May depend upon local hospital bacteriology
• Alternatives:
– Imipenem cilastatin or meropenem
• Higher rate C.diff colitis than cephalospoin
– Beta-lactam allergy: cipro+clinda or aztreonam+vanco
• <1% cross-reactivity between 3rd generation ceph and PCN/1st gen
cephalosporin

Vancomycin for F & N

• Empiric initial use does not improve


morbidity or mortality

• Encourages development of vancomycin


resistant Enterococuus

• Reserve for high suspicion of line/skin


infection, mucositis, h/o MRSA, severe PCN
allergy along with quinolone or aztreonam

688
Copyright © Harvard Medical School, 2018. All Rights Reserved.

“Low risk” F & N


• Outpatient antibiotic treatment reasonable
for low risk (Cipro/levo + Amox/Clav)
– anticipated duration of neutropenia < 7 days
– solid tumor
– clinically stable
– no major morbidities
– adequate oral intake and social supports
– malignancy responding to current treatment
– observe for a few hours

Case #4:
45 y.o. female with mantle cell lymphoma
admitted day 9 after chemotherapy with
F & N. Her fever resolved promptly with
institution of empiric antibiotics. One
day later, blood cultures grow E. coli
sensitive to amoxicillin. ANC is
currently 246.

689
Copyright © Harvard Medical School, 2018. All Rights Reserved.

Case #4:
A. Stop IV antibiotics and discharge her
on p.o. amoxicillin.
B. Continue IV antibiotics until ANC > 500.
C. Obtain echocardiogram to evaluate for
endocarditis
D. Stop IV antibiotics and start amoxicillin,
but continue to observe her for 48 hours
in hospital.

F & N – part 2
• When can antibiotics be stopped or
changed?
– Only when BOTH fever and neutropenia
have resolved
– If ANC > 500 and afebrile and source
isolated => complete course of abx for
infection
– If ANC>500 and afebrile and no source
isolated=> discontinue abx

690
Copyright © Harvard Medical School, 2018. All Rights Reserved.

Case #5:

57-year-old woman with metastatic lung


cancer is brought to the emergency room
with confusion. Serum calcium is 17.2
mg/dl and creatinine is 1.5 mg/dl. Four
liters of normal saline and IV furosemide
are administered. What is the next most
appropriate therapeutic intervention that
should be undertaken first?

Case #5:

A. Another 2 liters normal saline infusion IV


and furosemide
B. Denosumab 120 mg IV
C. Zoledronate 4 mg IV
D. Chemotherapy for lung cancer

691
Copyright © Harvard Medical School, 2018. All Rights Reserved.

Oncology pearl
• Describe optimal management of
hypercalcemia.

– Ref: Stewart AF. Hypercalcemia


associated with cancer. N Engl J Med
2005; 352: 373-9.

Hypercalcemia

• Occurs in up to 20% of cancer patients


– Both solid tumors and leukemia
– Most common: breast, lung, myeloma
• Incidence ↓↓ among metastatic breast and
myeloma pts with routine bisphophonate use
• Urgent rx for hypercalcemia important for
palliation, but long-term control requires
effective anti-cancer therapy

692
Copyright © Harvard Medical School, 2018. All Rights Reserved.

Causes of Hypercalcemia
• Humoral hypercalcemia of malignancy
– Tumors secrete PTHrP
– Most common cause
• Local osteolytic hypercalcemia
– Mainly in breast, myeloma, and lymphoma
• 1,25 (OH)2D-production by tumor
– Rare and occurs only in lymphoma
• Ectopic PTH
– Extremely rare – isolated case reports

Drug therapy for hypercalcemia


• Inhibit osteoclastic bone resorption
– Bisphosphonates (2-4 days for max effect)
– Calcitonin (immediate; tachyphylaxis in 2-3d)
• Increase urinary calcium excretion
– Normal saline to volume replete
• Loop diuretic can be added as needed to manage volume
• Antibody to RANKL inhibits osteoclast activity
– Denosumab
• Dialysis
– Only in rare circumstances – e.g. CHF does not allow
aggressive volume repletion or oliguric renal failure

693
Copyright © Harvard Medical School, 2018. All Rights Reserved.

Bisphosphonates vs denosumab
• 1st line treatment after volume repletion
– Oral agents much less potent so not used for
hypercalcemia Rx
• Zolendronate vs. Denosumab
– Have not been directly compared for
hypercalcemia
• Both associated with osteonecrosis of jaw
– Denosumab not approved for hypercalcemia
• Much more expensive (given SQ not IV)
• Can be associated with refractory hypocalcemia
• Can be used in severe renal impairment

Case #6
Which of the following pairs a proven cancer
prevention action and the cancer it
prevents?

A. Tobacco cessation and bladder cancer


B. Hepatitis C vaccine and hepatocellular
carcinoma
C. Decreasing soy consumption and breast
cancer
D. Human papilloma virus and uterine
cancer

694
Copyright © Harvard Medical School, 2018. All Rights Reserved.

Oncology pearl
• Recognize important epidemiologic
associations for cancer

Important associations
Tobacco Ionizing Radiation
• Head and neck • Thyroid
• Pancreas • Hodgkin's
• Bladder • Breast
• Lung • Lung
• Esophagus • Leukemia
• Kidney
Infectious causes
• Hepatitis B and C => hepatocellular cancer
• Epstein-Barr virus =>post-transplant lymphoma and
nasopharyngeal cancer
• Human papilloma virus => cervical and anal cancer

695
Copyright © Harvard Medical School, 2018. All Rights Reserved.

Case #7:

53 y.o man with Burkitt’s lymphoma


begins induction chemotherapy. He
starts on allopurinol and hydration.
Four days later, his creatinine has
increased from 1.0 to 5.4 mg/dL. Urine
output is 20 cc over the last 24 hours.
Labs include K+ 6.0, Ca 6.1, PO4 8.3,
uric acid 15.0. He complains of muscle
cramps and paresthesias.

Case #7:

A. Rasburicase, aggressive IV
hydration, alkalinize urine, kayexalate
(sodium polystyrene sulfonate)
B. Aggressive IV hydration, alkalinize
urine, kayexelate, IV calcium.
C. Dialysis
D. Rasburicase, furosemide, kayexelate,
oral calcium.

696
Copyright © Harvard Medical School, 2018. All Rights Reserved.

Oncology pearl
• To recognize and treat tumor lysis
syndrome.

– Ref: Howard SC et al. Tumor lysis


syndrome. N Engl J Med 2011; 364: 1844-
54

Tumor lysis syndrome


• Large tumor burden with rapid cell kill
• More common with aggressive leukemia
and lymphomas, e.g Burkitt’s
– Uncommon with solid tumors
– Risk highest during induction chemo when
tumor burden greatest
• Fatal complications include arrhythmias
and renal failure
• Oliguria poor prognostic sign

697
Copyright © Harvard Medical School, 2018. All Rights Reserved.

Laboratory abnormalities
• Serum potassium > 6.0 mg/dL
• Serum uric acid > 8 mg/dL
• Serum phosphate ≥ 4.5 mg/dL
• Serum calcium < 7 mg/dL

Management

• Prophylaxis/prevention key
– Aggressive hydration to maintain urine output
– Rasburicase vs allopurinal for prevention
• Treatment
– Rasburicase - recombinant urate oxidase
• Converts uric acid to allantoin which is more
soluble in urine than uric acid
• Contraindicated in G6PDH deficiency
• Dialysis if oliguric or persistent metabolic
abnormalities or severe symptoms

698
Copyright © Harvard Medical School, 2018. All Rights Reserved.

Case #8:
32 y.o. man in good health presents for
routine physical exam. His mother died
of colon cancer at 37, and his 39 y.o.
brother was just diagnosed with colon
cancer. Physical examination and fecal
occult blood test are negative. Labs are
normal.

What do you recommend?

A. Colonoscopy now and, if negative,


every 10 years.
B. Sigmoidoscopy now and, if negative,
every 5 years.
C. Annual fecal occult blood testing
D. Prophylactic colectomy
E. Genetic counseling with screening
recommendations to take place after
that has been performed.

699
Copyright © Harvard Medical School, 2018. All Rights Reserved.

Oncology pearl
• Identify different populations for
colorectal cancer screening.

– Wolf AMD et al. Colorectal cancer


screening for average‐risk adults: 2018
guideline update from the American
Cancer Society. CA J Clin 2018

Screening for Colorectal Cancer


• Risk assessment to determine screening
– Average risk
• Age 50 or older without personal or family history of
colorectal cancer or adenoma
– Increased risk
• Personal history of polyps or colorectal cancer
• Family history of polyps or colorectal cancer
• History of inflammatory bowel disease
• Known hereditary syndrome (e.g. Lynch, familial
adenomatous polyposis

700
Copyright © Harvard Medical School, 2018. All Rights Reserved.

Screening for average risk


Age 50 to 75 (could consider starting age 45)
• Annual Fecal Occult Blood Test or Fecal
Immunochemical or Stool DNA Test
• Flex sig q 5 yrs +/- FOBT/FIT q 1-3 yrs
• Colonoscopy q 10 yrs
• CT colonography q 5 yrs

USPTF, ACS

Screening if family history


• Increased risk: One 1st degree relative
with history of colorectal cancer > age
50 or 2 or more 2nd degree relatives with
colon cancer
– Begin screening at age 40 or 10 years
before youngest age of diagnosis then
follow standard screening intervals

701
Copyright © Harvard Medical School, 2018. All Rights Reserved.

High risk for screening (HNPCC)


• Amsterdam criteria (all criteria)
– At least 3 relatives (2 must be FDR) with
history of colorectal cancer
– At least one relative <50 at diagnosis
– At least 2 successive generations affected
• Bethesda criteria
– FDR with colon cancer < 45 or adenoma < 40
– Two HNPCC-related cancer (endometrial,
small intestine, ovarian, gastric)

Screening for high risk


• Familial adenomatous polyposis
– Consideration of colectomy
• Hereditary non-polyposis colorectal cancer
– Colonoscopy q1-2 yrs beginning age 20-25
• Personal history of colorectal cancer
– Colonoscopy at 1, 3, then q 5 yrs
– Modified for rectal cancer
• Personal history of polyps
– Depends upon number and type of polyps
(tubular vs villous, high grade vs low grade)

702
Copyright © Harvard Medical School, 2018. All Rights Reserved.

Case #9:
Which of the following patients has resectable non-small cell
lung cancer?
A. 10 cm RLL mass with ipsilateral mediastinal and
subcarinal LN involvement.
B. 2 cm LUL mass with ipsilateral hilar and
supraclavicular LN involvement.
C. 2 cm RUL mass with ipsilateral hilar LN
involvement and small malignant pleural effusion.
D. 4 cm RLL mass with positive ipsilateral
mediastinal LN and invasion of carina
E. 2 cm LLL mass with positive contralateral hilar LN,
but negative contralateral mediastinal LN

Oncology pearl
• Identify which patients with non-small
cell lung cancer are operable.

703
Copyright © Harvard Medical School, 2018. All Rights Reserved.

Lung cancer staging

Stage IIIB and IV and any T4 unresectable


N0 N1 N2 N3 M1

T1
IIIB IV
T2

T3

T4 IIIB

Lung cancer staging - nodes

• N1 – ipsilateral peribronchial or hilar

• N2 – ipsilateral mediastinal or subcarinal

• N3- contralateral mediastinal or hilar,


ipsilateral or contralateral scalene, or
supraclavicular LN

704
Copyright © Harvard Medical School, 2018. All Rights Reserved.

Lung cancer staging - tumor


• T3 – Tumor of any size extending into chest
wall, diaphragm, mediastinal pleura, or
pericaridum without involving heart, great
vessels, trachea, esophagus, vertebrae,
main stem bronchus, or carina; separate
tumor nodule in same lobe
• T4 – Tumor of any size involving heart,
trachea, esophagus, vertebrae, or carina;
separate tumor nodule in different ipsilateral
lobe
• Malignant pleural/pericardial effusion M1

Case #10
65 y.o. male notes worsening back and
hip pain. Bone scan and CAP CT scan
show widespread blastic bone
metastases without a clear primary.
Which of the following blood tests would
be most helpful to identify the primary
site?

705
Copyright © Harvard Medical School, 2018. All Rights Reserved.

Case #10
A. PSA (prostate specific antigen)
B. CEA (carcinoembryonic antigen)
C. AFP (alphafetoprotein)
D. hCG (human chorionic gonadotropin)

Oncology pearl
• How to evaluate cancer of unknown
primary
– Ref: Varadhachary GR and Raber MN,
Cancer of unknown primary site. N Engl J
Med 2014; 371:757-765

706
Copyright © Harvard Medical School, 2018. All Rights Reserved.

Approach to unknown primary


• Chest/abdominal/pelvic CT with contrast
– Unclear if PET-CT superior
• Focused tumor markers
– Often non-specifically elevated
– Man with adenocarcinoma and bone mets
• PSA
– Man with mediastinal/midline carcinoma
• hCG and AFP for germ cell tumor
• Immunhistochemistry vs tissue of origin
molecular profiling

Histology of unknown primary


• Adenocarcinoma (70%)
– Lung, hepatobiliary, pancreatic, renal most common
• Poorly differentiated cancer (25%)
– Lymphoma, melanoma, sarcoma, germ cell
• Squamous cell carcinoma (5%)
– Cervical => head and neck primary
– Inguinal => genital or anorectal primary
• Neuroendocrine tumors (1%)

707
Copyright © Harvard Medical School, 2018. All Rights Reserved.

Find non-metastatic cancers


• Woman with axillary lymph nodes
– Node positive breast cancer
• Neck nodes with squamous cell ca
– Locally advanced head and neck
• Young man with
mediastinal/retroperitoneal LN
– Germ cell tumor
• Woman with peritoneal disease
– Stage III ovarian cancer

Find treatable metastatic cancer

• Breast
• Ovarian
• Prostate
• Germ cell
• Lymphoma
• Melanoma
• Head and neck
• Colorectal

708
Copyright © Harvard Medical School, 2018. All Rights Reserved.

Case #11
45 y.o. premenopausal women has
severe hot flashes during the day and
frequent night sweats. She has a
history of a stage I ER negative breast
cancer diagnosed two years ago treated
with lumpectomy and radiation. She is
not on hormonal therapy. Which of the
following is the best choice for treating
her hot flashes?

Case #11
A. Amitriptyline
B. Lorazepam
C. Soy protein
D. Paroxetine
E. Conjugated estrogens without
progesterone

709
Copyright © Harvard Medical School, 2018. All Rights Reserved.

Oncology pearl
• Counsel cancer survivors regarding hot
flash treatment
– Ref: Fisher MA et al. Risk factors,
pathophysiology, and treatment of hot
flashes in cancer. CA Cancer J Clin 2013;
63: 167-92.

Hot flash treatment


• Placebo-controlled RCT key since placebo
has 25-30% response rate
• RCTs show 30-50% ↓ in frequency and
intensity of hot flashes with:
– SSRI’s (paroxetine, fluoxetine)
– SNRI’s (venlafexine)
– Gabapentin
– Clonidine
=> No consistent effect of soy protein, black
cohosh

710
Copyright © Harvard Medical School, 2018. All Rights Reserved.

HT for breast cancer survivors


• Menopausal symptoms common among
breast ca survivors
– Treatment induced menopause and side effects
of treatments
• Two RCT among breast cancer survivors
– Both showed ↑ recurrence risk
– Results similar regardless of ER status
=>No systemic HT for br ca survivors
=> Limited vaginal HT (e.g. ring, suppositoryk
or cream) OK JCNI 2008, Lancet Oncol 2009

Summary
• Study what is important for the internist to
know (and a little bit of trivia)
– Know your oncologic emergencies
– Cancer risk assessment
– Screening recommendations
– Common issues among common cancers

711
Copyright © Harvard Medical School, 2018. All Rights Reserved.

Leukemia and MDS

Edwin P, Alyea, MD
Medical Oncology
Dana Farber Cancer Institute
Brigham and Women’s Hospital
Associate Professor of Medicine
Harvard Medical School

Leukemia and MDS


Edwin P. Alyea

• Chronic Lymphocytic Leukemia


• Acute Lymphocytic Leukemia
• Myelodysplastic Syndromes
• Acute Myeloid Leukemia
• Chronic Myelogeneous Leukemia

712
Copyright © Harvard Medical School, 2018. All Rights Reserved.

Disclosures
Edwin P. Alyea

• None

Objectives
• Incidence
• Diagnosis
– Histology
– Immunophenotyping
– Cytogenetics
– Molecular genetics
– Minimal Residual Disease
• Prognosis/Risk Stratification
• Treatment Options
– New agents

713
Copyright © Harvard Medical School, 2018. All Rights Reserved.

Chronic Lymphocytic Leukemia


• Most common form of leukemia
• Heterogeneous disease
• median age at diagnosis is 65
• involves the bone marrow, lymph nodes and spleen at
diagnosis
• malignant cell is a B lymphocyte-CD19, dimCD20, CD5,
CD23
• common complications include autoimmune hemolytic
anemia and hypogammaglobulinemia
• increased risk of secondary malignancies

Prognostic Factors: CLL-


International Prognostic Index (IPI)
Variable Adverse Factor Grading
TP53(17p) Del and/or 4
mutated

IGHV Unmutated 2

Beta-2 >3.5 2
microglobulin

Clinical Stage Binet B/C 1


Age >65 1
a a GE
Lancet Oncology, 2016

714
Copyright © Harvard Medical School, 2018. All Rights Reserved.

Who to treat and when?


Risk Group Score Treatment 5 yrs Overall
Recommendations survival

Low 0-1 Do not treat 93%

Intermediate 2-3 No treat unless 79%


symptoms

High 4-6 Treat unless 63%


asymptomatic

Very High 7-10 Novel inhibitor or 23%


clinical trial
Lancet Oncology, 2016

-No improvement in survival with early treatment (IWCLL 2008)

Frontline Treatment for CLL


Under 65
• Ibrutinib
• FCR if healthy; BR is a reasonable alternative
• Del 17p –treatment naïve- ibrutinib. Role of RIC
allogeneic stem cell transplant unclear.
• Unmutated IGHV – consider ibrutinib or FCR

Over 65
• Del 17p/TP53 mut, del 11q, unmutated IGHV:
ibrutinib
• Others:
Bendamustine/Rituximab; CLB + obinutuzumab, CLB +
ofatumumab; ibrutinib

715
Copyright © Harvard Medical School, 2018. All Rights Reserved.

Fludarabine Ibrutinib
• Purine analogue
• BTK inhibitor
• higher response rate with • oral
combination chemotherapy • Combinations
– Fludarabine, being explored
Cyclophosphamide and • Lymphocyte count
Rituxan (FCR) often rises when
• Side Effects therapy initiated
– myelosuppression
– tumor lysis syndrome Side Effects
– hemolytic anemia • Myelosuppression
– opportunistic infections • infections
• should not combine • A fib (6%)
with Prednisone • Bleeding (6%)
• PCP prophylaxis
should be given

Ibrutinib prolongs PFS/OS in frontline CLL


PFS OS

• 18-month PFS rate: 90% with ibrutinib vs. 52% with chlorambucil
• 24-month OS rate: 98% with ibrutinib and 85% with chlorambucil
• Median follow-up: 18.4 months
Burger et al., N Eng J Med, 2016

716
Copyright © Harvard Medical School, 2018. All Rights Reserved.

Venetoclax therapy has durable benefits in CLL

FDA approved
for patients
with 17p and
as second line
therapy

11
Roberts AW , Davids MS et al. N Engl J Med 2015

Infectious Complications in CLL


• Hypogammaglobulinemia common
– infections with encapsulated organisms
– replacement therapy indicated for patients with recurrent
infections

• Progressive granulocytopenia

• Severe T cell immune suppression


– infections with listeria, PCP, etc.
– Fludarabine leads to reduction in CD4+ T cell counts
– need PCP prophylaxis
– CMV reactivation with alemtuzumab
– Fungal infections seen in patients with ibrutinib

717
Copyright © Harvard Medical School, 2018. All Rights Reserved.

Other Complications of CLL


• Autoimmune Complications
Hemolytic anemia
– treat with steroids, IvIg
– seen after treatment with fludarabine
ITP
– steroids, IvIg
• secondary malignancies- increased risk
• risk of transformation to more aggressive disease
– rapidly enlarging nodes, rapid increase in LDH, B symptoms

Acute Lymphocytic Leukemia

• Most common form of childhood cancer


• peak incidence 3-5 years, bimodal peak at 50
• increased incidence in patients with Down’s
syndrome, Bloom’s Syndrome and Ataxia-
Telangiectasia
• represents about 15% of leukemia in adults

718
Copyright © Harvard Medical School, 2018. All Rights Reserved.

Adverse Prognostic Features in ALL


Adverse Feature

Age >35-40
Clinical Features WBC >25,000
ETP T-ALL
25% of adults
Cytogenetics Ph+ (9;22), Ph-Like >40% in patients
4:11, complex over 60

Delayed Response >4-6 weeks to


to induction therapy obtain remission

Minimal Residual Disease defined milestones

NCCN Guidelines, March 2018

Treatment of ALL

• Induction Therapy- multiagent chemotherapy


– (e.g. cytoxan, adriamycin, L-aspariginase, vincristine
and prednisone)
• Consolidation therapy-
– CNS prophylaxis
– Maintenance chemotherapy extends for about 2 years

• 5 year leukemia free survival: 30%- 55%

719
Copyright © Harvard Medical School, 2018. All Rights Reserved.

Risk Adapted Therapy


First Complete Remission
• High Risk Patients
– Poor Risk Cytogenetic Abnormalities (i.e. 9;22)
– Adverse features (MRD positive)
– Allogeneic Bone Marrow Transplantation in first
remission
• Standard Risk (T cell or No adverse features)
• chemotherapy
Relapse
• Bone Marrow Transplantation
•CAR T cells

Ph+ ALL
• Incidence: 25% of adults, >40% -50% of patients over 60
• Diagnosis: FISH or cytogenetics
• Minimal residual disease monitoring by PCR
• 9;22 translocation
– Bcr-abl; chromosome 9-ABL and chromosome 22 BCR
– p190 in 70%
– p210 in 30%
• Treatment
– Chemotherapy plus TKI
– TKI plus steroids
– Stem Cell Transplantation
– If PCR negative—TKI alone? (Data to support this in young patients)

720
Copyright © Harvard Medical School, 2018. All Rights Reserved.

Chimeric Antigen Receptor (CAR)

Abbreviations: CAR, chimeric antigen receptor; GMP, good manufacturing practice; TCR, T-cell receptor Barrett et al, 2014

Anti-CD19 CARs in ALL


2015 Co-stim Age Clinical Outcomes Toxicity
Abstract# Domain
Author
682 – Park CD28 Adults 82% CR Severe CRS
MSKCC n-=46 6mo OS >65% 24%, Neuro
Phase 2 28%
681-Grupp 41BB Age <24 93% CR 88% CRS, 1
UPENN N=59 1yr OS 78% Grade 4, 27%
Phase 2 tx’d
684 – Lee CD28 Kids and 59% CR 6% Gr ¾ CRS
NIH young LFS 17.7 mos if MRD after CTX
Phase 2 adults neg CR stratification
N=39
FDA Approved: Tisagenlecleucel.
Patients up to age 25 with B-cell precursor acute lymphoblastic leukemia that
is refractory or in second or later relapse

721
Copyright © Harvard Medical School, 2018. All Rights Reserved.

Special Issues in ALL


• Infections
– use of prednisone increases risk for infections such
as PCP. PCP prophylaxis should be given.
• Avascular Necrosis
– related to prior steroid therapy
• Sites of Relapse
– Both the CNS and testicles are potential sites of
relapse.

Myelodysplastic Syndrome
Heterogenous group of clonal disorders characterized
by inadequate and dysmorphic hematopoiesis

• Stem cell disorder


• Increased incidence with age
• Increased risk with prior chemotherapy or
radiation
• Cytopenias present at diagnosis
– Anemia 45-90%
– Thrombocytopenia 20-40%
– Neutropenia 25-40%

722
Copyright © Harvard Medical School, 2018. All Rights Reserved.

Bone Marrow
Findings in MDS
-hypercellular marrow
-trilineage dysplasia

-irregular nuclear
formations in red cell
precursors

From Heaney, NEJM 1999

Differential Diagnosis of MDS


• Anemia of chronic disease
• marrow infiltration
• drug toxicity (ETOH, chemotherapy, INH)
• viral infections (HIV)
• vitamin deficiency (B12, folate)

The MCV is often elevated in patients with MDS

723
Copyright © Harvard Medical School, 2018. All Rights Reserved.

IPSS-R calculation
Parameter Categories and Associated Scores

Cytogenetic risk Very good Good Intermediate Poor Very Poor


group 0 1 2 3 4

Marrow blast ≤2% >2 - <5% 5 - 10% >10%


proportion 0 1 2 3

≥10 g/dL 8 - <10 g/dL <8 g/dL


Hemoglobin
0 1 1.5
≥0.8 x 109/L <0.8 x 109/L
Absolute
neutrophil count 0 0.5

≥100 x 109/L 50 - 100 x 109/L <50 x 109/L


Platelet count
0 0.5 1

Possible range of summed scores: 0-10


Steensma D Hematologist 2013; from Greenberg P et al Blood 2012 Sep 20;120(12):2454-65. Epub 2012 Jun 27

IPSS-R (see: http://www.mds-foundation.org/ipss-r-calculator/)


Time until
% patients Median
25% of
(n=7,012; Median survival for
Risk group Points patients
AML data on survival, years pts under 60
develop AML,
6,485) years
years
Very low 0-1.5 19% 8.8 Not reached Not reached
Low 2.0-3.0 38% 5.3 8.8 10.8
Intermediate 3.5-4.5 20% 3.0 5.2 3.2
High 5.0-6.0 13% 1.5 2.1 1.4
Very high >6.0 10% 0.8 0.9 0.7

Only valid for:


Adults
De novo disease
Treated with supportive care
At the time of diagnosis

From: Greenberg P et al Blood 2012 Sep 20;120(12):2454-65. Epub


2012 Jun 27

724
Copyright © Harvard Medical School, 2018. All Rights Reserved.

Mutations and Outcome


in Patients with MDS

Prognostic Genes:

TP53, EZH2, ETV6, RUNX1,


or ASXL1

Behar et al., N Engl J Med. 2011

Treatment of MDS
• Treatment: Age, Performance status and IPSS
• Supportive Care
– Chelation therapy for iron overload
• RARS-may respond to pyridoxine
• Growth Factors:
– EPO-helpful if EPO level is less than 500
– G-CSF for patients with recurrent infections
• Chemotherapy
– 5-azacytidine-hypomethylation agent
– Decitibine-inhibits DNA methylation
– Lenalidomide (5q-)
• Allogeneic Bone Marrow Transplant
– only curative option

725
Copyright © Harvard Medical School, 2018. All Rights Reserved.

AZA-001 Survival Study (Higher risk MDS):


Azacitidine vs “Conventional Care”
1.0 Log-Rank p=0.0001
0.9 HR = 0.58 [95% CI: 0.43, 0.77]
0.8 Deaths: Aza = 82, Control = 113
Proportion surviving

0.7
Difference: 9.4 months
0.6
0.5
24.4 months
0.4
15 months
0.3 Azacitidine
0.2
0.1 Control arm
0.0 (BSC, 3&7, LDAC)
0 5 10 15 20 25 30 35 40
Time (months) from randomization

Probably better to give 7 days than <7 days


IV same as SC Fenaux P, et al. Lancet Oncology 2009 10: 223–232

Special Issues in MDS


• 5q- syndrome
– older women with anemia and high platelet counts
– SF3B1 mutation
– prolonged course
– Lenolidomide associated with response
• Hypoplastic MDS
– hypocelluar bone marrow
– DDX with aplastic anemia. Cytogenetics are helpful.
• Therapy related MDS
– prior radiation and chemotherapy exposure
– 11q23 abnormalities associated with epipodophylotoxins
(VP-16)

726
Copyright © Harvard Medical School, 2018. All Rights Reserved.

Acute Myelogenous Leukemia

• Incidence of 2.4/100,000
• median age of 65-70
• most common acute leukemia in adults
• associated with prior radiation and toxin
exposure
• may arise from prior MDS

Bone Marrow
Findings in AML
Histology
-large blasts
-nucleoli often visible
- granules often present
-Auer rods may be
present
Immunophenotyping
demonstrates myeloid
makers
CD13, CD33, CD11
From Lowenberg, NEJM 1999

727
Copyright © Harvard Medical School, 2018. All Rights Reserved.

Cytogenetics in AML
• Favorable outcomes
– t(8;21), t(15;17), inversion 16
– t(8;21) and inv16 involve AML1-CBFb
• Adverse outcomes:
– -5, -7, 11q23, trisomies 8 and 13, 6;9 translocation
– >3 cytogenetic abnormalities

• Cytogenetic abnormalities have the most significant


impact on prognosis and are used to guide therapy

Current Risk Assessment in AML


Key Prognostic Data in AML in 2016
Patient age
Cytogenetics / karyotype
Primary versus secondary disease
(secondary = post-antecedent hematologic disorder, or therapy-related)

Molecular studies:

• FLT3 ITD (internal tandem duplication) mutation Unfavorable


• NPM1 mutation Favorable
• CEBPA biallelic mutation Favorable

• KIT mutation [in ~25% of t(8;21) or inv(16) AML] Unfavorable

Of Future Importance: mutation status of IDH1/2, DNMT3A, TET2, etc.

728
Copyright © Harvard Medical School, 2018. All Rights Reserved.

Treatment of AML
• Induction Therapy-
– anthracycline and Ara-C. Complete remission rate 60%-
80%.
– Complete remission lower in older adults, high toxicity
– Low intensity regimens used in older adults and those with
P53 defects
• Consolidation therapy- chemotherapy or BMT
– High dose Ara- C most commonly used
– randomized trial has shown no significant difference in
overall survival between consolidation chemotherapy,
allogeneic BMT, and autologous BMT

Risk Adapted Therapy for AML


1st complete remission
• Favorable cytogenetics/Molecular Profile
– t(8;21), t(15;17) and inv16
– Positive for NPM1 or CEBPA
– chemotherapy
• Unfavorable cytogenetics/Molecular Profile/MRD+
– t(9;22), -7, -5, 11q23
– FLT 3 ITD (not TKD), other high risk markers
– Allogeneic BMT
• Intermediate risk- normal cytogenetics
– chemotherapy or BMT
– Meta analysis favors BMT (Koreth et al, JAMA 2009)

729
Copyright © Harvard Medical School, 2018. All Rights Reserved.

APML- M3
t(15;17)

• All trans-retinoic acid (Atra) is combined with


induction therapy
• Atra improves complete remission rate
• Atra reduces the complications associated with
DIC
• Arsenic trioxide is used to treat relapsed patients
• Atra + Arsenic associated with >85% CR

Issues in AML
Hyperleukocytosis
• associated with very high blast count
• manifestations can include respiratory
compromise and/or altered mental status
• medical emergency
• treatment:
– IV hydration
– rapid initiation of chemotherapy
– leukopheresis
– radiation therapy

730
Copyright © Harvard Medical School, 2018. All Rights Reserved.

Chronic Myelogenous Leukemia

A myeloproliferative disease characterized


by the 9;22 chromosomal translocation
• median age at diagnosis of 53
• sex ratio of 1:1
• increased risk associated with prior radiation
exposure
• median survival is approximately 5 years

CML
clinical presentation

• ~50% asymptomatic at presentation


• fatigue (80%)
• weight loss (60%)
• abdominal discomfort (40%)
• easy bruising (35%)
• leukostasis, priapism and thrombosis are
rare

731
Copyright © Harvard Medical School, 2018. All Rights Reserved.

Ph Chromosome

• 90% of patients with hematologically


acceptable CML have the Ph chromosome
• 5% have a variant of the Ph chromosome
• 5% are Ph chromosome negative
– 40% of these patients will have the bcr-abl
translocation detected by PCR

Imatinib (STI571)
Gleevec

• BCR-ABL tyrosine kinase inhibitor (TKI) inhibits


proliferation of bcr-abl containing cells
• oral
• minimal side effects
– nausea (55%), myalgias (49%), edema (60%), rash (32%),
diarrhea (29%)

732
Copyright © Harvard Medical School, 2018. All Rights Reserved.

CML Treatment Options


• Chronic Phase Low Risk/Intermediate:
– Determine Risk Score:
– Imatinib or 2nd generation TKI
– 95% 5 year survival
– Assess Response Milestones (PCR log reduction)
– May be able to stop TKI in some patients

• Accelerated/ Blast Crisis


• Evaluate for BMT
• Chemotherapy + TKI for blast crisis in some cases
NCCN guidelines Jan 2018 ; Lancet Hematology 2015; Blood 2017

Question 1
42 yo female presents with a high white blood cell count (>100K)and
is found to have ALL. Her cytogenetics return 2 weeks later and she
has a 9;22 translocation.

The treatment associated with the best chance of long term survival is:

• Gleevec
• Dasatinib
• Intensive Consolidation Chemotherapy
• Allogeneic Stem Cell Transplantation

733
Copyright © Harvard Medical School, 2018. All Rights Reserved.

Question 2
64 yo physician is found to have an elevated lymphocyte count, HCT and Plts
normal. Flow cytometry demonstrates the lymphocytes to be CD20+, CD23+
and CD5+. The patient is observed. After developing anemia, therapy with
ibrutinib is started. His nodes decline but he has rapid rise in his WBC. He is
concerned about his rising WBC and wants to change therapy. What option
would you recommend?

a) Change to Venetoclax
b) Re-check FISH studies
c) Change to Fludarabine
d) Continue ibrutinib

Question 2a
The patient remains on ibrutinib and has an excellent response.
He complains of easy bruising. He is planning to have his hip
replaced. What would you suggest regarding his treatment for
CLL.

• Continue ibrutinib
• Stop ibrutinib and change to Venetoclax
• Hold ibrutinib for 3-7 days before and after surgery
• Continue ibrutinib and add warfarin after surgery to
prevent DVT

734
Copyright © Harvard Medical School, 2018. All Rights Reserved.

Summary
• Cytogenetics/Molecular Profiles predict outcome
• Risk adapted approach used to guide therapy
• New approaches using targeted therapies

Disclosures
Edwin P. Alyea

• None

735
Copyright © Harvard Medical School, 2018. All Rights Reserved.

Selected References
• Dohner H, Stilgenbauer S, Benner A, et al. Genomic aberrations and
survival in chronic lymphocytic leukemia. N Engl J Med.
2000;343:1910-1916.
• Greenberg P, Cox C, LeBeau MM, et al. International scoring system
for evaluating prognosis in myelodysplastic syndromes [see comments]
[published erratum appears in Blood 1998 Feb 1;91(3):1100]. Blood.
1997;89:2079-2088.
• Fenaux P, Mufti GJ, Hellstrom-Lindberg E, et al. Efficacy of
azacitidine compared with that of conventional care regimens in the
treatment of higher-risk myelodysplastic syndromes: a randomised,
open-label, phase III study. Lancet Oncol. 2009;10:223-232.
• Saglio G, Kim DW, Issaragrisil S, et al. Nilotinib versus imatinib for
newly diagnosed chronic myeloid leukemia. N Engl J Med;362:2251-
2259.
• Kantarjian H, Shah NP, Hochhaus A, et al. Dasatinib versus imatinib in
newly diagnosed chronic-phase chronic myeloid leukemia. N Engl J
Med;362:2260-2270.

736
Copyright © Harvard Medical School, 2018. All Rights Reserved.

Prostate and Bladder Cancer:


What the Internist Needs to Know
Lauren C. Harshman MD
Senior Physician, Dana-Farber Cancer Institute
Assistant Professor of Medicine, Harvard Medical School

July 23, 2018

Confluence of Interests
• Advisory: Bayer, Genentech, Dendreon, Pfizer,
Medivation/Astellas, Kew Group, Theragene, Corvus,
Merck, Exelixis, Novartis

• Research to the institution: Bayer, Sotio, Bristol-Myers


Squib, Merck, Takeda, Dendreon/Valient, Jannsen,
Medivation/Astellas, Genentech, Pfizer

737
Copyright © Harvard Medical School, 2018. All Rights Reserved.

Prostate Cancer in the US


• Incidence: 220,800 in 2015
161, 360 in 2017 to 164,690 in 2018
• Mortality: 29, 430
• 2nd cause of cancer death in 2018 up
from #3 in 2017
• 1 in 7 men
• Risk Factors:
- Age
- Family history
- Race: African American
- Genetic syndromes
• BRCA 1/2, Lynch syndrome
Siegel J CA Cancer Statistics 2018, www.cancer.org (ACS)

The Spectrum of Prostate Cancer


Organ Confined Organ Confined Metastatic Castration Resistant
Low Risk - Risk of Mets Disease Prostate Cancer

Prostate Cancer

Risk of cancer Rising PSA no mets Rising PSA no/min mets

Varied spectrum from low volume, low grade disease that would never
kill the patient to intermediate risk disease to high risk, high volume
localized disease to frank metastatic disease that is incurable.

Treatment decisions require balancing the risk of progression and


morbidity secondary to treatment toxicity with life expectancy.

738
Copyright © Harvard Medical School, 2018. All Rights Reserved.

Randomized Screening Studies for Prostate Cancer


• Rationale: identify cancer early increase chances of cure
• Screening tests: PSA (FDA 1994), digital rectal exam (DRE)
PLCO (US)** European Goteborg
Sample size 150,000 162,243 20,000
Median F/u 13 yrs 13 yrs 14 yrs
Prostate cancer No difference 21% reduction 44% reduction,
mortality p=0.001 p=0.0002
NNT* 37 12
*NNT: number needed to treat to prevent 1 death; **52% contamination

Why not screen?

Risks of Biopsy Risk of Overdiagnosis & Tx Toxicity


• Infection
• Pain
• Anxiety

Sanda et al, NEJM 2008


www.auanet.org/content/health-policy/quality/pdf/AUA-SUNA-PNBWhitePaper.pdf

739
Copyright © Harvard Medical School, 2018. All Rights Reserved.

Screenshot June 2016 & June 2017

For the C recommendation for men aged 55 to 69 years, the USPSTF’s


Screenshot
intention is to convey that each man’s values may shift June to
the balance 2017
a
net benefit of screening and to promote the importance of informed
decision making prior to screening.

The USPSTF continues to find that the benefits of screening do not


outweigh the harms in men 70 years and older and recommends against
screening in these men.

https://screeningforprostatecancer.org

Recommendations 2013 on…


American Urologic Association (AUA)
•No PSA screening in men under age 40. No routine screening in men between ages
40 to 54 years at average risk
•“For men ages 55 to 69 years the Panel recognizes that the decision to undergo PSA screening involves
weighing the benefits of preventing prostate cancer mortality in 1 man for every 1,000 men screened over a
decade against the known potential harms associated with screening and treatment. For this reason, the
Panel strongly recommends shared decision-making for men age 55 to 69 years that are considering PSA
screening, and proceeding based on a man’s values and preferences”

•To reduce harms of routine screening, prefer interval of two years or more
•No PSA screening: >age 70 OR any man with < 10 -15 year life expectancy

American Cancer Society (ACS)


• All men >50 with 10 yr life expectancy
• Discuss age >45 for African Americans and men with first degree relative
diagnosed <65 yo
• Discuss age >40 for those with multiple family members diagnosed young
• Screen yearly for PSA >2.5, every other year for <2.5
Auanet.org; cancer.org

740
Copyright © Harvard Medical School, 2018. All Rights Reserved.

≥ 50
Effects of 2012 USPTF
Recommendations
• Decreased PSA testing
• Significant decrease in prostate cancer incidence
• Too short follow-up to know effects on mortality 50-74
rate
• Pendulum still swinging…

≥75

Sammons JAMA 2015, Jemal JAMA 2015, Penson JAMA 2015

Grading and Staging Prostate Cancer


• Gleason Score (GS):
• Most prominent pattern +
Next most prominent pattern
- GS: 3 + 3 = 6
- GS: 4 + 3 = 7
- GS: 4 + 5 = 9

• PSA: <10, 10-20, >20

• TNM clinical staging


- Staging of bones
and nodes

741
Copyright © Harvard Medical School, 2018. All Rights Reserved.

Treatment Options for Early Stage Prostate Cancer

Radical Radiation Therapy


Prostatectomy +/-ADT

Active Surveillance Watchful Waiting

Overview: Treatment Options by Stage


Low Risk Intermediate Risk High Risk
GS 6 GS 7 GS 8-10
PSA <10 PSA 10-20 PSA >20
T1 T2 T3-4
Active surveillance Radical prostatectomy Radiation + 2-3 yrs CAB
Radical prostatectomy Radiation + 6 mo. CAB Radical prostatectomy +/-
Radiation: external beam, adjuvant radiation
brachytherapy
Stage: CT/MRI pelvis,
Bone scan

• Life expectancy <10 yrs: consider watchful waiting


• Surgery and radiation prevent PC deaths and increase overall survival in men
with clinically significant tumors and reasonable life expectancy
• Weigh side effect profiles, comorbidities, and risk of overtreatment
• Adding CAB to radiation improves outcomes but at expense of toxicity

GS: Gleason Score; CAB: combined androgen blockade (e.g., Lupron + bicalutamide)

742
Copyright © Harvard Medical School, 2018. All Rights Reserved.

Active Surveillance for Low Risk Prostate Cancer


• No initial treatment; treat only those men who need it
• Close monitoring: PSA, physical exam, serial re-biopsy
• Treat to cure at a sign of more aggressive disease
– e.g., Increase in Gleason score, PSADT
• Cons: anxiety, chance of progression to non-curative state

• Grey: death to other cancer


• Black: death due to PC
• GS <7: few deaths from PC
• GS ≥7 and >10 yr LE: no AS

Parker et al, BJC 2006

Outcome of Surgically Treated Patients


• Gleason 8-10: 10% of all cases
– 49% 15-year PCSM G8-10
– 45% of all cancer deaths
• Gleason 7: 40% of all cases
– 8% 15-year PCSM
– 50% of all cancer deaths G4+3
• Gleason 6: 50% of all cases G3+4
– <1% 15-year PCSM G3+3
– 1 of 3756 patients with
organ-confined, Gleason 6
cancer died from prostate
Stephenson et al J. Clin Onc 2009
cancer
• Surgery induces high cure rate in GS ≤7
• Surgery for select cases of GS 8-10
PCSM: prostate-cancer specific mortality

743
Copyright © Harvard Medical School, 2018. All Rights Reserved.

Prostate Cancer Exquisitely Dependent on Androgens


…Initially Orchiectomy

Brain LH
FSH
Testosterone

(LHRH) DHT

Lupron
Antiandrogens Prostate
Abiraterone Cancer

•Medical or surgical castration: testosterone suppression


DHT: dihydrotestosterone

Intermediate Risk: Radiation plus 4 - 6 mo ADT

• RTOG Trial
- Enrolled all risk patients
- XRT +/- Short term ADT
- Intermediate risk
• Decrease PC mortality
• Increase OS
- No benefit for low risk
- High risk likely need longer
course ADT

• Two other trials with similar


results
- TROG Lancet 2011
- DFCI JAMA 2008
Jones NEJM 2011

744
Copyright © Harvard Medical School, 2018. All Rights Reserved.

High Risk Disease: Radiation plus Longterm


Testosterone Suppression

36 months versus 6 months ADT


• all deaths (increased OS)
• Prostate Ca deaths
• 5 yr mortality: 19% vs. 15.2%
HR: 1.42 (95.7% CI 1.79, p0.65

Bolla et al NEJM 2009

ProtecT Trial
• Evaluated the effectiveness of
surgery, radiation or active
monitoring (AM) on PC mortality
• PSA detected clinically localized
cancers (n=1643)
• Outcomes equivalent: high cure rates
• Death from PC ~1% at 10 years
irrespective of treatment
• Treatment over AM 50% less:
– Rate of disease progression
– Development of metastatic disease
• 25% of men on AM went on to
receive radical therapy within 3 yrs
and 50% by 10 yrs

AM = active monitoring Hamdy NEJM 2016

745
Copyright © Harvard Medical School, 2018. All Rights Reserved.

Weighing Toxicity
Surgery Radiation ADT
• Surgical complications • Irritative urinary Sx • Hot flashes
- Wound healing • Bowel dysfunction • Fatigue
- Infection • Erectile dysfunction • Weight gain
- Anesthesia risks • Risk of second cancer • Bone density loss
• Erectile dysfunction • Loss of libido
• Urinary Incontinence • Emotional changes
• Metabolic insults:
insulin resistance
• ?Cardiac toxicity
• ?Dementia

• Surgery: radical prostatectomy (robotic, open, laparoscopic)


• Radiation: external beam, brachytherapy
• ADT: GnRH agonist or antagonist +/- antiandrogen
Sanda NEJM 2008, Resnick NEJM 2013

Summary: Treatment of localized disease

• Low risk disease:


– Active surveillance for > 70 yrs with low risk disease
– Shared decision making re: surgery vs. radiation for younger men

• Intermediate risk disease:


– Surgery or Radiation plus 4-6 months androgen suppression

• High grade but localized cancer:


– Radiation plus long course androgen suppression (2-3 yrs) OR
– Radical prostatectomy (+/- adjuvant radiation)

746
Copyright © Harvard Medical School, 2018. All Rights Reserved.

The spectrum of prostate cancer


Organ Confined Organ Confined Metastatic Castration Resistant
Low Risk - Risk of Mets Disease Prostate Cancer

Prostate Cancer

Risk of cancer Rising PSA no mets Rising PSA no/min mets

• Biochemical Recurrence: Rising PSA after local therapy


- Post prostatectomy: >0.2
- Post-radiation: nadir + 2 (still have normal prostate tissue making PSA)

Natural History of Prostate Cancer: deferred ADT

Makarov et al J. Urol; 2008, 179 (156-162)

747
Copyright © Harvard Medical School, 2018. All Rights Reserved.

ADT for Biochemical relapse


• Non-inferiority Phase 3 Study:
• Post XRT as primary or local Rx
• 8 month ADT, hold and restart
with PSA rise vs. continuous
• Intermittent not inferior, better QOL

Overall survival by Treatment Arm


10 0
P e rce n ta g e of P a tie n ts
75

Crook NEJM 2012


50

• TOAD trial: early vs. delayed ADT


25

Treatment = A:Delayed
- Early use = increase OS (HR 0.55)
Treatment = B:Immediate
- Median OS not reached at 8 yrs
0

0 2 4 6 8
Years
Duchesne ASCO 2015, Lancet 2016

Hormone Sensitive Metastatic Prostate Cancer

Organ Confined Organ Confined Metastatic Castration Resistant


Low Risk - Risk of Mets Disease Prostate Cancer

Prostate Cancer

Risk of cancer Rising PSA no mets Rising PSA no/min mets

748
Copyright © Harvard Medical School, 2018. All Rights Reserved.

Metastatic Prostate Cancer Starting ADT: Prognosis

Hussain et al. J Clin Oncol 2006

Eisenberger et al. S8894 NEJM

• Worse prognosis: high volume bone or


liver/lung metastases Harshman et al J Clin Oncol 2017

• Better prognosis: node only or < 4 • Patients with deeper PSA nadir
bone metastases have improved survival by years

Metastatic Disease: Intermittent vs. Continuous ADT


• Men with metastatic prostate
cancer starting ADT (n=3040)
• PSA < 4 in 1535 men =
selected group

• Randomized to continuous vs.


intermittent ADT

• Median 6 mo survival benefit with


continuous testo suppression
• HR for death with intermittent
tx: 1.10; 90% CI - 0.99 to 1.23
• Exceeded the upper boundary
for noninferiority
• Thus, “not non-inferior” Hussain et al NEJM 2013

Standard Remains Continuous ADT… in those that can tolerate it

749
Copyright © Harvard Medical School, 2018. All Rights Reserved.

Adding Docetaxel to ADT Increases Survival


High Volume Metastases Low Volume Metastases
1.0 1.0

0.9 0.9

0.8 0.8

0.7 0.7

0.6 0.6

0.5 0.5
p=0.0006

Probability
Probability

p=0.1398
0.4 HR=0.60 (0.45-0.81) 0.4
HR=0.63 (0.34-1.17)
Median OS: Median OS:
0.3 0.3
ADT + D: 49.2 months ADT + D: Not reached
0.2
ADT alone: 32.2 months 0.2 ADT alone: Not reached
0.1 0.1

0.0 0.0

0 12 24 36 48 60 72 84 0 12 24 36 48 60 72 84

OS (Months) OS (Months)
Arm TOTAL DEAD ALIVE MEDIAN
• Overall: 57.6 vs. 44 mo, HR:0.61, 95% CI: 0.47-0.80, p <0.001
• High volume = liver, lung, > 4 bone with one beyond spine and pelvis
• High volume metastatic dz: 17 month OS benefit: 32.2 mo 49.2 mo
Sweeney ASCO 2014, NEJM 2015

Hormone therapy works….for a While

Flare!

• Ultimately almost all patients become “castration-resistant”


• CRPC = rising PSA/progression despite testosterone <50

750
Copyright © Harvard Medical School, 2018. All Rights Reserved.

Progression of Prostate Cancer Despite


Testosterone Suppression:
Castration-resistant Disease

Organ Confined Organ Confined Metastatic Castration Resistant


Low Risk - Risk of Mets Disease Prostate Cancer

Prostate Cancer

Risk of cancer Rising PSA no mets Rising PSA no/min mets

Current Treatment of Castration-resistant Prostate Cancer

Bone-targeted:
Second line • Radium-223
Androgen blockade: • Antiresorptives
• Abiraterone
• Enzalutamide
• (nilutamide, flutamide,
ketoconazole/HC)
Chemotherapy
• Docetaxel
Immunotherapy: • Cabazitaxel
Sipuleucel-T

• Median overall survival from time of second line agent: 18-35 months
• Clinical trials remain imperative as there are no cures!

751
Copyright © Harvard Medical School, 2018. All Rights Reserved.

Take Home Messages:


Castration Resistant Prostate Cancer
• 6 agents that improve survival after progression on initial
androgen deprivation therapy
• Range from immunotherapy to second line antiandrogens to
chemotherapy to bone metastasis homing agents
• Incremental improvements in survival: median 2-5 months
each (these add up when given sequentially!)
• Best sequence unclear: consider tolerability, toxicity, co-
morbidities (e.g., seizures; diabetes given need for steroids)
• Clinical trials imperative as still no cure!

Summary of Prostate Cancer


• Prostate cancer mortality has decreased significantly in the PSA era
– Screening appears most beneficial in the 55-69 age group with life
expectancy >10 yrs (shared decision making is key)
– Overtreatment is a valid concern, but diagnosis does not equate to
definite need to treat
• Active surveillance reasonable in low risk disease—goal: delay or prevent
toxicity
• Both surgery and radiation + ADT can improve outcomes in younger
patients with intermediate or high risk disease
• Wealth of agents that improve survival in metastatic disease but no cures
– ADT +/- docetaxel chemotherapy
– 2nd line hormone suppression: abiraterone, enzalutamide
– Immunotherapy: first anticancer vaccine sipuleucel-T
– Bone homing agents like Radium-223
– Clinical trials remain imperative to improve outcomes

752
Copyright © Harvard Medical School, 2018. All Rights Reserved.

Slides for Reference on Advanced


Treatments for Castration-Resistant
Prostate Cancer That Your Patients May
Receive

Extinguishing AR axis after Androgen Suppression


Abiraterone *Steroid biosynthesis
Pregnenolone
*Intratumoral
CYP17 & Systemic
Enzalutamide
17-OH-Pregnenolone
CYP17

DHEA

Hormone regulated
AR
genes
* AR upregulated in CRPC tissue
plus
* Tumors make own androgens
=
Rationale for 2nd Line Hormonal Rx

753
Copyright © Harvard Medical School, 2018. All Rights Reserved.

Abiraterone/Prednisone & Enzalutamide in CRPC

• First approved in docetaxel-refractory setting


- 4-5 month overall survival benefit
• Later tested pre-chemotherapy with even greater benefit over
placebo in progression-free and overall survival
deBono NEJM 2011, Scher NEJM 2012, Ryan NEJM 2013, Beer NEJM 2014

Docetaxel Chemotherapy Increases Overall Survival

• Cabazitaxel (2nd gen taxane) vs. mitoxantrone:


- Median OS: 15.1 vs. 12.7 mo, HR 0.7, p<0.0001

754
Copyright © Harvard Medical School, 2018. All Rights Reserved.

Sipuleucel-T: First Anticancer Treatment Vaccine

Antigen (PAP-
GMCSF) is
exposed to an Antigen is Fully activated,
Antigen APC takes up processed and the APC is now
Presenting the antigen presented on sipuleucel-T
Cell (APC) surface of the and is collected
APC

INFUSE
PATIENT

T-cells proliferate and attack sipuleucel-T activates T-


cancer cells cells in the body

IMPACT Trial: Sipuleucel-T Improves Overall Survival


36.5 mo median f/u
HR = 0.759 (95% CI, 0.606, 0.951)
P = 0.017 (Cox model)
Median survival benefit = 4.1 mo

Sipuleucel-T (n = 341)
Median survival: 25.8 mo.
36 mo. survival: 32.1%

Placebo (n = 171)
Median survival: 21.7 mo.
36 mo. survival: 23.0%

No. at Risk

Sipuleucel-T 341 274 142 56 18 3


Placebo 171 123 59 22 5 2 Kantoff et al NEJM 2010

755
Copyright © Harvard Medical School, 2018. All Rights Reserved.

Bone Metastases & Radium-223


• >90% of patients with mCRPC have bone metastases
• Most frequent cause of morbidity and mortality
• Incur high treatment-related costs in this disease

• Radium-223 dichloride: radiotherapeutic


– Calcium mimetic: hones in on bone metastases due to their high rate
of bone turnover
– Short path length alpha particles limits toxicity to surrounding healthy
tissue
– Well tolerated with the most frequently observed adverse events:
low grade myelosuppression, GI toxicity, and fatigue
– Compared to placebo, radium-223 significantly :
• Median overall survival: 11.3 14.9 mo; HR 0.70
• Median time first symptomatic skeletal events

Parker N Engl J Med 2013

100
ALSYMPCA Overall Survival
90 HR 0.695; 95% CI, 0.552-0.875
80 P = 0.00185
70
60
Radium-223, n = 541
% 50 Median OS: 14.0 months
40

30

20
Placebo, n = 268
Median OS: 11.2 months
10
0
Month 0 3 6 9 12 15 18 21 24 27

Radium- 223 541 450 330 213 120 72 30 15 3 0


Placebo 268 218 147 89 49 28 15 7 3 0

Benefit in patients with and without prior chemotherapy

756
Copyright © Harvard Medical School, 2018. All Rights Reserved.

Bladder Cancer for Internists: A to Z

Bladder Cancer US Statistics in 2018


• 81,190 cases will be diagnosed
– 62,3800 males/ 18,810 females (3:1)
– #4 cancer in incidence
– 50-60% superficial disease
– Median age: 73-74 yrs

• 17,240 estimated deaths


– 10% present with metastatic disease
– 4% of all cancer deaths

Siegel CA Cancer Stat 2012, 2018

757
Copyright © Harvard Medical School, 2018. All Rights Reserved.

Risk Factors

Types of Bladder Cancer

TCC 90%
Squamous (6-8%)
Adenocarcinoma(2%)
Other (3%)

Urothelium
• Urothelial cell or transitional cell-carcinoma (TCC) (90%) •Renal pelvis
• Squamous (6-8%) •Ureters
– Schistosomiasis •Bladder
– Non-schistosomiasis cases: chronic catheterization patients •Urethra
• Adenocarcinoma (2%)
– Urachal
– Non-urachal (chronically irritated transitional epithelium)
• Small cell (<1%, treat like lung cancer)

758
Copyright © Harvard Medical School, 2018. All Rights Reserved.

Diagnosis: Internist Often First To See


• Hematuria: gross or microscopic
• UTI like or irritative symptoms: frequency, dysuria, blood
– Patients are often treated with 2-3 courses of antibiotics before they
are referred to a urologist

• If not clearly a UTI on UA or culture:


– Refer to urology for cystoscopy and
upper tract evaluation
– Send urine cytology
– Consider CT abdomen/pelvis

Staging of Bladder Cancer: TNM

• T1 = superficial, non-invasive
• T2 = muscle invasion
• T3 = into/through perivesciular fat
• T4b = invasion of pelvic/abdom wall

• N1 = Involved regional lymph nodes

• M1 = Distant metastasis

Critical that TURBT samples the


muscle layer!

Harshman et al Surg Pathol Clin 2015

759
Copyright © Harvard Medical School, 2018. All Rights Reserved.

Spectrum of Bladder Cancer


Bladder Cancer

Non-muscle Muscle Invasive Metastatic


Invasive (60%) (30%) (10%)

Recur

• TURBT • Radical cystectomy +/- • Chemotherapy


• Intravesical tx neo/adjuvant chemo • Immunotherapy
if high grade OR
• Chemoradiation

Non-Muscle Invasive Urothelial Cancer


• Most common stage (60%)
• Risk of progression to muscle invasive
– High grade, T1a - 48% rate of progression
– Low grade Ta - 2% rate of progression
• Managed by cystoscopy and TURBT Webmd.com
– Transurethral resection of bladder tumor
– BCG instillations weekly x 6 then q 3 months for “high risk” lesions
• Improved disease free survival compared with no therapy
• Decreases need for “salvage” cystectomy
• No improvement in overall survival because often cured with
cystectomy

760
Copyright © Harvard Medical School, 2018. All Rights Reserved.

Muscle Invasive Disease: Outcomes after Cystectomy

Overall survival

Can adding chemotherapy improve surgical outcomes??

• Organ confined (T2, ~35%): 5 yr OS: 74%, median OS ~15 yrs


• Extravesicular (>T2): 5 yr OS: 37%, median OS ~4.5 yrs
• Regional nodal disease: 5 yr OS: 31%, median: OS ~2 yrs
OS: overall survival Stein J Clin Oncol 2001

SWOG 8710: Cystectomy +/- Neoadjuvant MVAC

T2-4aN0M0 disease

Median Alive at 5 yrs p-value


n=307
Survival (2-sided)

RC 46 mos 43%
.06
MVAC +RC 77 mos 57%

• Disease-Specific Survival: 1.66 in favor of chemotherapy (p= 0.002)


• Overall Survival: 1.33 in favor of chemotherapy (p=0.06)
• Clinically significant increase in median overall survival
• Supported by meta-analyses
MVAC: methotrexate, vinblastine, adriamycin, cisplatin Grossman NEJM 2003
RC: radical cystectomy ABC Collaboration Eur Urol 2005

761
Copyright © Harvard Medical School, 2018. All Rights Reserved.

Adjuvant Chemo: Immediate vs. Delayed


Phase 3 EORTC Trial: pT3-4 or Node + Urothelial Cancer
100 5 yr PFS: 46.8% vs. 39.5% 100 5 yr OS: 53.6% vs. 47.7%
90
90
80
HR 0.52, p<0.0001 80
HR 0.78, p =0.13
70 Median: 2.9 vs. 0.9 yrs 70 Median: 6.8 vs. 4.6 yrs
60 60
PFS 50 OS 50
40
40
30
30
20
20
10
10
0 (years)
0 (years)
0 2 4 6 8 10 12
0 2 4 6 8 10 12
O N Number of patients at risk : Treatment
O N Number of patients at risk : Treatment 66 141 95 70 44 25 3 Immediate
73 141 71 56 39 21 3 Immediate 82 143 83 67 42 21 4 Deferred
103 143 50 43 30 18 4 Deferred

• Largest randomized adjuvant study to date but closed early


• Immediate cisplatin-based chemo improves PFS and non-significant 22.2%
reduction in risk of death (underpowered)
• Multiple meta-analyses suggest DFS and OS benefit to cisplatin-based chemo
• Clinical Practice: consider if pT3-T4 or node positive and cisplatin eligible
Sternberg ASCO 2014, Leow Eur Urol 2013

Trimodality Bladder Preservation Therapy:


Maximal TURBT + Chemoradiation
• Optimal candidates: small solitary lesions <5cm, T2 or
T3 disease, no hydronephrosis, ability to perform a
complete TURBT, and minimal to no CIS

• Bladder preservation approach:


– Maximal TURBT to remove all residual disease
– Radiation: 64-65 Gy bladder, 44-45 Gy adj pelvic nodes
– Cisplatin based therapy: weekly vs. every 3 wks
– Cisplatin unfit: 5FU/Mitomycin, carbo/taxol
– Repeat TURBT: if residual cancer salvage cystectomy

• Outcomes
– ~50% of patients long term control with intact bladder
Shipley et al Cancer 2003
– 20-30% require salvage cystectomy Mak JCO 2009

762
Copyright © Harvard Medical School, 2018. All Rights Reserved.

Chemoradiation in Patients with Muscle


Invasive UC Unfit for Surgery or Cisplatin
• 2001-2008: 360 T2-4aN0 patients
– Majority T2 (82%)
– Median age: 72 yrs
• TURBT: complete only in 55%
• XRT 55 Gy in 20 fx or 64 Gy in 32 fx
• +/- Chemo: 5-FU plus Mitomycin C

• Addition of chemo to XRT:


– 43% reduction in risk of
locoregional relapse
– Trend to benefit in OS

James et al NEJM 2012

Metastatic Disease: MVAC vs. Gem/Cis (GC)

mOS ~14-15 mo.


PFS: ~8 mo.
RR: 46-49%

• Cisplatin is the best drug for urothelial cancer


• Non-inferiority trial (not equivalency or superiority)
• GC was not inferior to MVAC in all respects
• GC probably superior in terms of toxicity: less mucositis, neutr. fever/sepsis

MVAC: methotrexate, vinblastine, adriamycin, cisplatin, GC: gemcitabine/cisplatin Von der Maase JCO 2000, & JCO 2005

763
Copyright © Harvard Medical School, 2018. All Rights Reserved.

Some Metastatic disease is CURABLE with Chemotherapy

# Factors % pts CR (%) Med. Surv 5-yr 10 yr

0 32 38 33 mo 33% 24%

1 45 25 13.4 mo 11% 6%

2 23 5 9.3 mo 0% 0%

• N = 203 patients
• Risk factor: KPS < 80; any visceral metastasis - lung, liver, bone
• ~20% of patients can have longevity/durable responses to chemo
Bajorin; JCO 1999

Cisplatin Unfit Patients


• Up to 50% of patients are “unfit” for cisplatin
– Performance status, renal dysfunction, neuropathy, heart failure
• No standard therapy in this setting: carboplatin, gemcitabine, taxanes

• Carboplatin based regimens:


- ORR: 30-40%
- CRs: 10% or less
Galsky J Clin Oncol 2011

764
Copyright © Harvard Medical School, 2018. All Rights Reserved.

Second line Chemotherapy

• Before 2016: No standard


– Doublets/Triplets likely not better than single agents
– Increased response rates but not necessarily OS
– General outcomes from many trials of different chemos:
• Progression-free survival: 3-5 months
• Overall survival: 5-9 months

• 2016 New Standard—Immunotherapy with PD-1 pathway


blockade

PD-1 Blockade: Reinvigorate the CTLs

Killer T cell

Increased
cytokine

IFN-γ
PD-1 Increased
TCR killing
mAb
Pembrolizumab PD-L1 MHC
Atezolizumab
Nivolumab
Durvalumab
Avelumab Tumor cell
G. Freeman

765
Copyright © Harvard Medical School, 2018. All Rights Reserved.

Boon of Checkpoint Inhibitor Immunotherapy


Approvals for Urothelial Cancer
Durvalumab
1108 Trial
2L mUC

Avelumab
JAVELIN Trial
2L mUC

Atezolizumab Pembrolizumab
ImVigor211 (-) KeyNote-052
2L mUC 1L Cis-Inel mUC
Pembro
MVAC Pembrolizumab
KeyNote-052
Improves OS KeyNote-045
1L Cis-Inel
1L mUC 2L mUC
mUC
Atezolizumab
CG Atezolizumab Nivolumab Pembro Nivolumab Atezolizumab
ImVigor210
Less toxic than ImVigor210 CheckMate275 KeyNote-045 CheckMate275 ImVigor210/211 2L
1L Cis-Inel
MVAC 1L mUC 2L mUC 2L mUC 2L mUC 2L mUC mUC
mUC

1992 - 2000 May 2016 Feb 2017 April 2017 May 2017 June 2017 Sept 2017

Figure N. Hahn

KEYNOTE-045: Open-Label, Phase 3 Study of Pembrolizumab vs


Investigator’s Choice of Paclitaxel, Docetaxel, or Vinflunine for
Overall Survival: Total
Previously Treated Advanced Urothelial Cancer

100
90 Events HR (95% CI) P
80 Pembro 155 0.73 0.0022
70 Chemo 179 (0.59-0.91)
43.9%
O S, %

60 30.7%
50
40
30
Median (95% CI)
20 10.3 mo (8.0-11.8)
10 7.4 mo (6.1-8.3)
0
0 2 4 6 8 10 12 14 16 18 20 22 24
Time, months
No. at risk
270 226 194 169 147 131 87 54 27 13 4 0 0
272 232 171 138 109 89 55 27 14 3 0 0 0

Data cutoff date: Sep 7, 2016. Bellmunt J et al, NEJM 2017


Data cutoff date: Sep 7, 2016.

766
Copyright © Harvard Medical School, 2018. All Rights Reserved.

2017: Atezolizumab & Pembrolizumab Approved Plimack ASCO 2016

Multiple First line trials exploring IO/IO or chemo-IO

767
Copyright © Harvard Medical School, 2018. All Rights Reserved.

Multiple First line trials exploring IO/IO or chemo-IO

Conclusions
• Multidisciplinary approach essential to cure

• MIBC: can be highly sensitive to chemotherapy and immunotherapy


– Cisplatin remains gold standard first line agent in fit/eligible patients
– PD-1/PD-L1 blockade in first line cisplatin-ineligible or platinum-refractory
disease (2nd line +)…but maybe only if PD-L1+…evolving story

• Cure/long term control possible in node positive (St IV) disease

• Modest but real survival benefit to neoadjuvant cisplatin-based


chemotherapy

• Novel therapies imperative to improve outcomes: SUPPORT TRIALS

768
Copyright © Harvard Medical School, 2018. All Rights Reserved.

Board Review Question 1


• Which pairing of treatment and clinical scenario is incorrect?
A. Prostatectomy for 57 yo with T1c (no nodule on DRE), Gleason score
7 disease in 4 of 6 cores and PSA 5
B. Active surveillance for 73 yo with Gleason score 6 in 5% of 1 core
out of 12 cores and PSA 4.5, T1c (no nodule on DRE)
C. 2-3 years of androgen suppression and radiation for 65 yo with 6 of
12 cores with Gleason score 8 and T2b (nodule on DRE)
D. Radiation alone for 60 yo with Gleason score 4+3 =7 disease in 8 of
12 cores, PSA 8, and T2a (small nodule) disease

Board Review Question 1


• Which pairing of treatment and clinical scenario is incorrect?
A. Prostatectomy for 57 yo with T1c (no nodule on DRE), Gleason
score 7 disease in 4 of 6 cores and PSA 5
B. Active surveillance for 73 yo with Gleason score 6 in 5% of 1 core
out of 12 cores and PSA 4.5, T1c (no nodule on DRE)
C. 2-3 years of androgen suppression and radiation for 65 yo with 6 of
12 cores with Gleason score 8 and T2b (nodule on DRE)
D. Radiation alone for 60 yo with Gleason score 4+3 =7 disease in 8 of
12 cores, PSA 8, and T2a (small nodule) disease
Answer:
– D is incorrect: Radiation plus short term testosterone suppression
alone for intermediate risk prostate cancer results in superior
prostate cancer specific and superior overall survival compared to
radiation alone.

769
Copyright © Harvard Medical School, 2018. All Rights Reserved.

Board Review Question 2


• Which pairing of treatment and clinical scenario is incorrect?
A. Neoadjuvant cisplatin combination therapy followed by cystectomy
for muscle invasive urothelial bladder cancer in a “fit” 60 yo
B. Transurethral resection of bladder tumor (TURBT) alone for grade 1
Ta solitary papillary urothelial cancer
C. Concurrent mitomycin plus 5-FU with radiation for multifocal high
grade T1 (lamina propria invasion but not muscle) urothelial cancer
D. Mitomycin plus 5FU plus XRT for 78 yo female with muscle invasion
and not “fit” for cystectomy

Board Review Question 2


A. Neoadjuvant cisplatin combination therapy followed by cystectomy
for muscle invasive urothelial bladder cancer in a “fit” 60 yo
B. Transurethral resection of bladder tumor (TURBT) alone for grade 1
Ta solitary papillary urothelial cancer
C. Concurrent mitomycin plus 5-FU with radiation for multifocal high
grade T1 (lamina propria invasion but not muscle) urothelial cancer
D. Mitomycin plus 5FU plus XRT for 78 yo female with muscle invasion
and not “fit” for cystectomy
• Answer:
– C is incorrect: TURBT with intravesical therapy (most commonly
BCG to decrease risk of recurrence) is appropriate management for
non-muscle cancers at high risk for recurrence. Radiation is not
appropriate for T1 (superficial) disease outside of a clinical trial.

770
Copyright © Harvard Medical School, 2018. All Rights Reserved.

Selected References
• Bill-Axelson A, Holmberg L, Ruutu M et al. Radical prostatectomy versus watchful
waiting in early prostate cancer. The New England journal of medicine.
2011;364(18):1708-17.
• Bolla M, de Reijke TM, Van Tienhoven G, et al. Duration of androgen suppression in the
treatment of prostate cancer. The New England journal of medicine.
2009;360(24):2516-27.
• Hussain M, Tangen CM, Berry DL, et al. Intermittent versus continuous androgen
deprivation in prostate cancer. N Engl J Med. 2013 Apr 4;368(14):1314-25.
• von der Maase H, Sengelov L, Roberts JT, et al. Long-term survival results of a
randomized trial comparing gemcitabine plus cisplatin, with methotrexate, vinblastine,
doxorubicin, plus cisplatin in patients with bladder cancer. Journal of Clinical Oncology.
2005;23(21):4602-8.
• Grossman HB, Natale RB, Tangen CM, et al. Neoadjuvant chemotherapy plus
cystectomy compared with cystectomy alone for locally advanced bladder cancer. The
New England journal of medicine. 2003;349(9):859-66.
• Bellmunt j, de Wit R, Vaughn D et al. Pembrolizumab as second-line therapy for
advanced urothelial carcinoma. N Engl J Med 2017; 376(11): 1015-1026.

Extra Slides For Reference

771
Copyright © Harvard Medical School, 2018. All Rights Reserved.

Lung Cancer
David Jackman, MD
Senior Physician, Dana-Farber Cancer Institute
Medical Director of Clinical Pathways, Dana-Farber Cancer Institute
Assistant Professor of Medicine, Harvard Medical School

Disclosures

• Consultant:
– AstraZeneca
– CVS Caremark
– MOREHealth

772
Copyright © Harvard Medical School, 2018. All Rights Reserved.

Question 1

Which of the following statements is true?


A. Lung cancer is the third most common cause of cancer death, after
breast and colon cancer.
B. For former smokers, it takes ~ 10 years for the risk of lung cancer to
decrease to that of a nonsmoker.
C. More than half of new lung cancers are already metastatic at the time of
diagnosis.
D. Tumor grade (based on appearance under the microscope) is a more
important predictor than tumor stage (extent of disease on scans)
E. Small cell lung cancer is both more aggressive and more common than
non-small cell lung cancer.

Question 1

Which of the following statements is true?


A. Lung cancer is the third most common cause of cancer death, after
breast and colon cancer.
B. For former smokers, it takes ~ 10 years for the risk of lung cancer to
decrease to that of a nonsmoker.
C. More than half of new lung cancers are already metastatic at the time
of diagnosis.
D. Tumor grade (based on appearance under the microscope) is a more
important predictor than tumor stage (extent of disease on scans)
E. Small cell lung cancer is both more aggressive and more common than
non-small cell lung cancer.

773
Copyright © Harvard Medical School, 2018. All Rights Reserved.

5 things to know about … Lung Cancer Epidemiology

Topics to Address

Overview and
1
Epidemiology
2 CT Screening U.S. Deaths/year

3 Diagnosis and
Staging
1. Lung Cancer: 154,050

4 Treatment Overview
2. Colon Cancer: 50,630
A Localized Disease
3. Pancreatic Cancer: 44,330
Locally Advanced
B Disease 4. Breast Cancer: 41,400

Metastatic Disease
5. Prostate Cancer: 29,430
C

Total: 165,790

Siegel et al, CA: A Cancer Journal for Clinicians, 2018 5

5 things to know about … Lung Cancer Epidemiology

Topics to Address

Overview and
2
1
Epidemiology
3
2 Diagnosis and
CT Screening
Staging
4 Diagnosis and
3 Treatment
Staging Overview
5
4 Treatment
CT Screening
Overview • 85% of lung cancers in the U.S. occur in
smokers.
B Localized Disease
• Risk increases with number of cigs/day and
C
Locally Advanced
Disease
number of years smoking.
• Smokers have a 15-30-fold higher risk of lung
Metastatic Disease
C
cancer compared to nonsmokers.
• Quitting decreases risk, but it never normalizes.

774
Copyright © Harvard Medical School, 2018. All Rights Reserved.

5 things to know about … Lung Cancer Epidemiology

Topics to Address

Overview and
2
1
Epidemiology
3
2 Diagnosis and
CT Screening
Staging
4 Diagnosis and
3 Treatment
Staging Overview
5
4 Treatment
CT Screening
Overview

B Localized Disease

Locally Advanced
C Disease Unknown Early
(8%) (15%)
C Metastatic Disease Locally Advanced
Metastatic (22%)
(55%)
SEER 2000
7

5 things to know about … Lung Cancer Epidemiology

Topics to Address Stage 5-year survival


Overview and Localized 55%
2
1
Epidemiology
3
2 Diagnosis and
CT Screening
Staging Regional 28%
4 Diagnosis and
3 Treatment
Staging Overview
Distant 4%
5
4 Treatment
CT Screening
Overview

B Localized Disease

Locally Advanced
C Disease

C Metastatic Disease

SEER Cancer Statistics Review, 1975-2013


8

775
Copyright © Harvard Medical School, 2018. All Rights Reserved.

5 things to know about … Lung Cancer Epidemiology

Topics to Address

Overview and
2
1
Epidemiology
3
2 Diagnosis and
CT Screening
Staging
4
3
Diagnosis and
Treatment
Staging Overview 88% 12%
Non-small Cell
5
Small Cell
4 Treatment
CT Screening
Overview

B Localized Disease

Locally Advanced
C Disease

C Metastatic Disease

DIAGNOSIS AND STAGING

10

776
Copyright © Harvard Medical School, 2018. All Rights Reserved.

5 things to know about …Diagnosis and Staging

Topics to Address
1. Stage and Approach
Overview and
1
Epidemiology NSCLC SCLC
Treatment Approach
2 Diagnosis and Stage Stage
Staging
IA,
Surgical resection
3 Treatment Overview IB (< 4cm)
IB (> 4cm), Surgical resection,
4 CT Screening
IIA, IIB +/- chemotherapy

IIIA Multimodality approach

IIIB Limited Chemotherapy and Radiation

IV Extensive Chemotherapy

11

5 things to know about …Diagnosis, Staging and


Screening
Topics to Address 2. Initial Staging Workup
Overview and
1
Epidemiology Which of the following is not indicated in
Diagnosis and
2
Staging
the initial workup of EVERY new lung
3 Treatment Overview
cancer patient?
4 CT Screening A. Abdominal imaging
B. Bone imaging (PET/CT or bone scan)
C. Brain imaging (MRI or CT) without
contrast
D. Chest CT with IV contrast
E. All are indicated

12

777
Copyright © Harvard Medical School, 2018. All Rights Reserved.

5 things to know about …Diagnosis, Staging and


Screening
Topics to Address 2. Initial Staging Workup
Overview and
1
Epidemiology Which of the following is not indicated in
Diagnosis and
2
Staging
the initial workup of EVERY new lung
3 Treatment Overview
cancer patient?
4 CT Screening A. Abdominal imaging
B. Bone imaging (PET/CT or bone scan)
C. Brain imaging (MRI or CT) without
contrast
D. Chest CT with IV contrast
E. All are indicated

13

5 things to know about …Diagnosis and Staging

Topics to Address Common Sites of Metastasis

Overview and Liver


1
Epidemiology Bone
2 Diagnosis and Adrenals
Staging
Brain
3 Treatment Overview
Brain MRI w/ gadolinium (preferred), or
4 CT Screening CT head with and without contrast

CT chest with contrast, specified for lung cancer

PET-CT (preferred) or
bone scan

14

778
Copyright © Harvard Medical School, 2018. All Rights Reserved.

5 things to know about …Diagnosis and Staging

Topics to Address
3. The Value of Mediastinal Staging
Overview and
1
Epidemiology • Appropriateness of resection
Diagnosis and
2
Staging
• Guiding radiation therapy
3 Treatment Overview
• Ways and means:
4 CT Screening • Mediastinoscopy
• Endobronchial ultrasound
• Surgical lymph node dissection

15

5 things to know about …Diagnosis and Staging

Topics to Address
4. Making a Diagnosis
Overview and
1
Epidemiology GET AS MUCH TISSUE AS YOU
Diagnosis and
2
Staging SAFELY CAN
3 Treatment Overview • Diagnosis
4 CT Screening
• Genomic analysis
• Immunotherapy assessment
• Clinical trial eligibility

Caveats:
• Core over FNA
• Non-bone over bone

16

779
Copyright © Harvard Medical School, 2018. All Rights Reserved.

5 things to know about …Diagnosis and Staging

Topics to Address 5. Paraneoplastic syndromes


Overview and
1
Epidemiology Match the paraneoplastic syndrome
2 Diagnosis and with the most likely corresponding
Staging
thoracic tumor histology.
3 Treatment Overview
Hyponatremia (SIADH) Small cell lung cancer
4 CT Screening
Cushing syndrome Squamous cell lung cancer

Lambert-Eaton Adenocarcinoma of lung

Hypercalcemia Mesothelioma

Myasthenia Gravis Thymoma

17

5 things to know about …Diagnosis and Staging

Topics to Address 5. Paraneoplastic syndromes


Overview and
1
Epidemiology Match the paraneoplastic syndrome
2 Diagnosis and with the most likely corresponding
Staging
thoracic tumor histology.
3 Treatment Overview
Hyponatremia (SIADH) Small cell lung cancer
4 CT Screening
Cushing syndrome Squamous cell lung cancer

Lambert-Eaton Adenocarcinoma of lung

Hypercalcemia Mesothelioma

Myasthenia Gravis Thymoma

18

780
Copyright © Harvard Medical School, 2018. All Rights Reserved.

5 things to know about …Diagnosis and Staging

Topics to Address 5. Paraneoplastic Syndromes


Overview and Endocrine/ Mechanism Symptoms
1
Epidemiology Paracrine
2 Diagnosis and Hypercalcemia Multiple Altered mental status, ataxia.
Staging Cardiac concerns
3 Treatment Overview SIADH ADH/aVP Symptomatic hyponatremia
Cushing’s ACTH Muscle weakness, hyperglycemia,
4 CT Screening syndrome hylokalemia. Infection.

Neurologic Mechanism Symptoms


Lambert-Eaton Anti-voltage gated Fatigue, large muscle
calcium channels weakness
Paraneoplastic Anti-Hu Ataxia, diplopia,
cerebellar Anti-Yo dysphagia
degeneration
Myasthenia gravis Anti-Acetylcholine Fatigable weakness of
receptor voluntary muscles
Pelosof et al. Mayo Clin Proc 2010. 85(9): 838-854 19

MANAGEMENT OF LUNG
CANCER

20

781
Copyright © Harvard Medical School, 2018. All Rights Reserved.

5 things to know about …Lung Cancer Treatment

NSCLC SCLC Treatment 1. Anatomic Resection


Stage Stage Approach
IA,
remains the Gold
IB (< 4cm) Surgical resection Standard
IB (> 4cm), Surgical resection, Alternatives include:
IIA, IIB +/- chemotherapy
• Wedge resection
IIIA Multimodality • Radiation
approach
• Ablation
IIIB Limited Chemotherapy and
Radiation
Chemotherapy /
IV Extensive Targeted Therapy /
Immunotherapy

21

5 things to know about …Lung Cancer Treatment

2. Role of Radiation
Radiation is appropriate for all but which of the following?
A. Curative therapy for a localized lung cancer in a patient with poor
operative risk
B. Combined with chemotherapy as part of a multimodality strategy
for a right upper lobe mass with hilar and mediastinal nodal
involvement.
C. Whole lung irradiation for a patient with multiple lung lesions
throughout the left lung.
D. Palliative therapy for brain metastases
E. Palliative therapy for SVC syndrome

22

782
Copyright © Harvard Medical School, 2018. All Rights Reserved.

5 things to know about …Lung Cancer Treatment

2. Role of Radiation
Radiation is appropriate for all but which of the following?
A. Curative therapy for a localized lung cancer in a patient with poor
operative risk
B. Combined with chemotherapy as part of a multimodality strategy
for a right upper lobe mass with hilar and mediastinal nodal
involvement.
C. Whole lung irradiation for a patient with multiple lung lesions
throughout the left lung.
D. Palliative therapy for brain metastases
E. Palliative therapy for SVC syndrome

23

5 things to know about …Lung Cancer Treatment

NSCLC SCLC Treatment


Stage Stage Approach
IA,
IB (< 4cm) Surgical resection

IB (> 4cm), Surgical resection,


IIA, IIB +/- chemotherapy

IIIA Multimodality
approach 3. Genomically
IIIB Limited Chemotherapy and targeted therapy
Radiation has been a major
Chemotherapy /
IV Extensive Targeted Therapy / advance in non-
Immunotherapy small cell lung
cancer
24

783
Copyright © Harvard Medical School, 2018. All Rights Reserved.

5 things to know about … Lung Cancer Treatment

Moving beyond Chemo Targeted


therapies
chemotherapy
Response rate 20-30%
100 Median 8-12 months
80 survival
Percent Alive

60

40

20

0
0 5 10 15 20 25 30
Months
Schiller et al. N Engl J Med. 2002;346:92.

25

5 things to know about … Lung Cancer Treatment


Chemo Targeted
Moving beyond therapies
chemotherapy Response rate 20-30% 60-80%
Median 8-12 months 2-3 years
100 survival

80
Percent Alive

60

40

20

0
0 5 10 15 20 25 30
Months

Schiller et al. N Engl J Med. 2002;346:92.


26
Rosell R, et al. Lancet Oncol. 2012;13:239-246.

784
Copyright © Harvard Medical School, 2018. All Rights Reserved.

5 things to know about … Lung Cancer Treatment

Also:
ROS1
RET
NTRK
MET

Johnson BE, et al.


ASCO 2013.

27

5 things to know about … Lung Cancer Treatment

4. Immunotherapy

Ribas, NEJM 2012

For metastatic non-small cell lung cancer patients treated with


nivolumab, a PD-1 inhibitor, the 5-yr survival: 15% !

Gettinger S, Horn L, Jackman D, et al. J Clin Onco 2018


28

785
Copyright © Harvard Medical School, 2018. All Rights Reserved.

5 things to know about …Treating Local Disease

5. What’s the current standard of care for additional testing at


time of diagnosis?
Genomic, Immunohisto- Coming?
approved chemical,
approved
Non-small cell, non- EGFR, BRAF, PD-L1 MET, NTRK,
squamous ALK, ROS1 RET, HER2
TMB
NSCLC, squamous - PD-L1 TMB
SCLC - - TMB

29

CT SCREENING

30

786
Copyright © Harvard Medical School, 2018. All Rights Reserved.

5 things to know about … Lung Cancer Screening

Topics to Address 1. Does it save lives?

Overview and • 20% decrease in lung


1
Epidemiology cancer mortality
2 Diagnosis and
Staging • 6.7% decrease in all
cause mortality
3 Treatment Overview

4 CT Screening

NLST investigators, NEJM 2011. 365: 395-409 31

5 things to know about … Lung Cancer Screening

Topics to Address 1. Does it save lives? 2. Whom to screen?


• 20% decrease in lung • Ages 55-80
Overview and
1
cancer mortality • Smokers, heavy and
Epidemiology
recent:
Diagnosis and
2
• 6.7% decrease in all • > 30 pack-yrs
Staging
cause mortality • Quit within last
3 Treatment Overview 15 yrs

4 CT Screening

Uspreventiveservicestaskforce.org. Lung Cancer:Screening, Release Date Dec 2013 32

787
Copyright © Harvard Medical School, 2018. All Rights Reserved.

5 things to know about … Lung Cancer Screening

Topics to Address 1. Does it save lives? 2. Whom to screen?


• 20% decrease in lung • Ages 55-80
Overview and
1
cancer mortality • Smokers, heavy and
Epidemiology
recent:
Diagnosis and
2
• 6.7% decrease in all • > 30 pack-yrs
Staging
cause mortality • Quit within last
3 Treatment Overview 15 yrs

4 CT Screening
3. How and
•when?
Annual LD-CT until:
• > 15 yrs since smoking
cessation, or
• Age 80, or
• Unwilling or too ill to
undergo curative
resection for detected CA

Uspreventiveservicestaskforce.org. Lung Cancer:Screening, Release Date Dec 2013 33

5 things to know about … Lung Cancer Screening

Topics to Address 1. Does it save lives? 2. Whom to screen?


• 20% decrease in lung • Ages 55-80
Overview and
1
cancer mortality • Smokers, heavy and
Epidemiology
recent:
Diagnosis and
2
• 6.7% decrease in all • > 30 pack-yrs
Staging
cause mortality • Quit within last
3 Treatment Overview 15 yrs

4 CT Screening
3. How and 4. Is it cost-
•when?
Annual LD-CT until: effective?
• > 15 yrs since smoking
cessation, or • Estimates of $80K/QALY
• Age 80, or gained
• Unwilling or too ill to • Cost-effectiveness
undergo curative improves if tied to effective
resection for detected CA smoking cessation
program
34

788
Copyright © Harvard Medical School, 2018. All Rights Reserved.

5 things to know about … Lung Cancer Screening

Topics to Address 1. Does it save lives? 2. Whom to screen?


• 20% decrease in lung • Ages 55-80
Overview and
1
cancer mortality • Smokers, heavy and
Epidemiology
recent:
Diagnosis and
2
• 6.7% decrease in all • > 30 pack-yrs
Staging
cause mortality • Quit within last
3 Treatment Overview 15 yrs
5. Follow-up
4 CT Screening for detected
3. How and lesions 4. Is it cost-
•when?
Annual LD-CT until: effective?
• > 15 yrs since smoking
cessation, or • Estimates of $80K/QALY
• Age 80, or gained
• Unwilling or too ill to • Cost-effectiveness
undergo curative improves if tied to effective
resection for detected CA smoking cessation
program
35

5 things to know about … Lung Cancer Screening

Topics to Address Incidental Solid Pulmonary Nodules found on


Overview and
Non-Screening CT: Fleischner Guidelines
1
Epidemiology
Nodule Size Low Risk Pt High Risk Pt
2 Diagnosis and
< 4 mm No follow-up needed Follow-up CT at 12
Staging
mo; if unchanged, no
3 further follow-up
Treatment Overview
> 4 – 6 mm Follow-up CT at 12 Initial follow-up CT at
mo; if unchanged, no 6-12 mo, then at 18-24
4 CT Screening further follow-up mo if no change
> 6 – 8 mm Initial follow-up CT at Initial follow-up CT at
6-12 mo, then at 18-24 3-16 mo, then at 9-12
mo if no change and 24 mo if no
change
> 8 mm Follow-up CT at Same as for low-risk
around 3, 9, and 24 patient
mo, dynamic contrast-
enhanced CT, PET,
and/or biopsy

MacMahon et al, Radiology 2005. 237(2): 395-400 36

789
Copyright © Harvard Medical School, 2018. All Rights Reserved.

5 things to know about … Lung Cancer Screening


Incidental Subsolid Pulmonary Nodules found on Non-Screening CT:
Topics to Address
Fleischner Guidelines
Nodule
Overview and
Size Management Recommendations Additional Remarks
2
Epidemiology
Solitary Pure GGNs
3 < 5CT
mmScreening No CT follow-up required Obtain contiguous 1 mm-thick sections
to confirm that nodule is truly a pure
GGN
4 Diagnosis and
> 5Staging
mm Initial follow-up CT at 3 mo to confirm persistence, then FDG-PET is of limited value, potentially
annual surveillance CT for a minimum of 3 yrs misleading, and not recommended.
5 Treatment
Solitary part-solid nodules
OverviewInitial follow-up CT at 3 mo to confirm persistence. If Consider PET-CT for part-solid nodules
persistent and solid component < 5 mm, then yearly > 10 mm.
surveillance CT for minimum of 3 yrs. If persistent and
B Localized Disease solid component > 5 mm, then biopsy or surgical resection

Multiple Subsolid nodules


Locally Advanced
Pure CGGNs < 5 mm
Disease Obtain follow-up CT at 2 and 4 years Consider alternate causes for multiple
GGNs < 5 mm
Pure C
GGNs > 5 mm DiseaseInitial follow-up CT at 3 months to confirm persistence and
Metastatic FDG-PET is of limited value, potentially
without a dominant then annual surveillance CT for a minimum of 3 yrs misleading, and not recommended.
nodule
Dominant nodule(s) with Initial follow-up CT at 3 mo to confirm persistence. If Consider lung-sparing surgery for
part-solid or solid persistent, biopsy or surgical resection is recommended, patients with dominant lesion(s)
component especially for lesions with > 5 mm solid component suspicious for lung cancer.

Naidich et al, Radiology 2013. 266(1): 304-17 37

NCCN Guidelines for Solid Nodule on


Initial screening LDCT

38

790
Copyright © Harvard Medical School, 2018. All Rights Reserved.

NCCN Guidelines for solid nodules found on follow-up


scans or annual LDCT

39

NCCN Guidelines for Part-Solid Nodules on


Initial screening LDCT

40

791
Copyright © Harvard Medical School, 2018. All Rights Reserved.

NCCN Guidelines for Part-Solid Nodules on


Follow-up or Annual LDCT

41

Summary Points

Topics to Address
5 Key Points for Boards:
Overview and
1
Epidemiology
2 Diagnosis and • Smoking cessation
Staging
• Importance of Stage
3 Treatment Overview
• Staging tests to get
4 CT Screening
• Paraneoplastic Syndromes
• Who Should be screened

42

792
Copyright © Harvard Medical School, 2018. All Rights Reserved.

Summary Points

Topics to Address
5 Key Points for Practice:
Overview and
1
Epidemiology
2 Diagnosis and • Common confusion in staging tests:
Staging • PET/CT doesn’t obviate need for
3 Treatment Overview dedicated brain imaging
• Brain imaging needs contrast
4 CT Screening
• Get enough tissue the first time
• Refer to oncology for evaluation and
consideration of therapy
• Start appropriate CT screening

43

References

• Gettinger S, Horn L, Jackman D, et al. J Clin Onco 2018


• NLST investigators, NEJM 2011. 365: 395-409
• Naidich et al, Radiology 2013. 266(1): 304-17
• Pelosof et al. Mayo Clin Proc 2010. 85(9): 838-854
• SEER Cancer Statistics Review, 1975-2013

44

793
Copyright © Harvard Medical School, 2018. All Rights Reserved.

Breast Cancer Update


“A buffet of breast cancer topics”

Wendy Y. Chen, MD MPH


Dana-Farber Cancer Institute
Brigham and Women’s Hospital

Wendy Y. Chen, MD MPH


Dana-Farber Cancer Institute
Brigham and Women’s Hospital

• Disclosures:

– None related to this presentation

794
Copyright © Harvard Medical School, 2018. All Rights Reserved.

Breast Cancer
• Epidemiology and Genetics

• Screening

• Prevention

• Primary (non-metastatic) breast cancer

• Metastatic breast cancer

Cancer cases

Cancer deaths

Siegel et al, CA Cancer


J Clin 2018

795
Copyright © Harvard Medical School, 2018. All Rights Reserved.

Established breast cancer risk factors

• Age
• Overweight
• Alcohol use
• Physical activity
• Menopausal hormone use
• Late age at first birth (> 30 y.o.)
• Early age at menarche
• Late age at menopause
• BRCA1/2 and Family history

Established breast cancer risk factors

• Age
• Overweight
• Alcohol use
• Physical activity
• Menopausal hormone use
• Late age at first birth (> 30 y.o.)
• Early age at menarche
• Late age at menopause
• BRCA1/2 and Family history

796
Copyright © Harvard Medical School, 2018. All Rights Reserved.

Modifiable breast cancer risk factors

Body mass index

797
Copyright © Harvard Medical School, 2018. All Rights Reserved.

Body mass index


• ↑↑ Postmenopausal breast cancer risk
– 20-30% higher with BMI > 30
– No increase in premenopausal breast ca
• Main source of estrogen after
menopause = peripheral conversion of
androgens into estrogens by aromatase
in adipose tissue

Body mass index and sex steroid levels


60

50

40

Estradiol
30
SHBG

20

10

0
<22.5 22.5-24.9 25-27.4 27.5-29.9 30+
Body Mass Index

798
Copyright © Harvard Medical School, 2018. All Rights Reserved.

Physical activity

Physical activity and breast cancer

• ↓↓ breast cancer risk with physical


activity
– Prevent weight gain and ↓ body fat
– ↓ estradiol, estrone, and free estradiol
and ↑ sex hormone binding globulin
– ↓ insulin levels

McTiernan, JAMA 2003; Canc Res 2004

799
Copyright © Harvard Medical School, 2018. All Rights Reserved.

Exercise and postmenopausal breast ca

1.25
P for trend =.03
=> No strong
association
1
with
moderate or
strenuous
activity
0.75

0.5
0 <=5 5.1-10 10.1-20 20.1-40 >40

met-hours of recreational physical activity McTiernan, JAMA 2003

Alcohol

800
Copyright © Harvard Medical School, 2018. All Rights Reserved.

Alcohol Intake and Breast Cancer


1.8

1.6
Relative Risk

1.4

1.2

0.8
0 5 10 15 20 25 30 35 40 45 50 55 60
Alcohol Intake (g/d)

Alcohol and estrogen levels


20
P for trend= 0.001
18 0g
1-4 g
16
5-15 g
15 -24 g
14
25+ g

12 P for trend= 0.03


10

8
Estrone Estradiol
Onland-Mouret 2006,
J Clin Endocrinol Metab

801
Copyright © Harvard Medical School, 2018. All Rights Reserved.

Family History/Genetics

Who should get BRCA1/2 testing?


Risk for BRCA1/2 mutation is greater if:

Age < 50 at diagnosis

Bilateral breast cancer

Male breast cancer

Triple negative breast cancer < age 60

=> Person with cancer should be tested first

802
Copyright © Harvard Medical School, 2018. All Rights Reserved.

BRCA 1 & 2
• ↑↑ risk of breast and ovarian cancer
– BRCA1 : 55-70% breast , 40-50% ovarian ca
– BRCA2 : 45-70% breast , 15-20% ovarian ca
– ↑↑ male breast cancer with BRCA2
– ↑ pancreatic and prostate cancer
– Explains 5-10% of breast cancers

• Multigene/expanded panel testing


– CDH1, ATM, p53, PALB2, CHK2, pTEN, NBN,
NF1, STK11, etc
– Clinical recommendations unclear

Screening mammography

To screen or not to screen?

803
Copyright © Harvard Medical School, 2018. All Rights Reserved.

Screening question
A 55 y.o. woman wants to know more about breast cancer
screening. You would advise her that:
1. Mammograms do not reduce mortality for women aged
40-49

2. MRI reduce breast cancer mortality more than


mammograms do

3. Virtually all DCIS found on mammography will turn into


invasive cancer if not removed

4. Mammographically detected lesions can be accurately


evaluated by core needle biopsy

Answer to screening question


A 55 y.o. woman wants to know more about breast cancer
screening. You would advise her that:
1. Mammograms DO reduce mortality for women aged 40-
49.

2. MRI has NOT been shown to reduce breast cancer


mortality

3. Only 15-40% of DCIS will become invasive.

4. Mammographically detected lesions can be accurately


evaluated by core needle biopsy

804
Copyright © Harvard Medical School, 2018. All Rights Reserved.

What we know about screening


Women aged 50-74 should get routine
screening mammogram

– Randomized controlled trials and meta-


analyses => 15-20% ↓↓ mortality

– Recommended screening interval 1-2 years

Mammography for women 40-49


Arguments AGAINST Arguments FOR
• Large RCT’s (100,000+) • Although individual trials
=>15-25% ↓↓ mortality but not significant, meta-
not statistically significant analyses => 7-23%
significant ↓↓ mortality
• Absolute benefits smaller • Relative benefits similar to
because breast cancer less women 50+
common < age 50

• Older studies done before • Older studies used outdated


modern treatments so not techniques, so older RCT’s
applicable to present underestimated benefit

• More false positives in • Breast cancer leading cause


women 40-40 of death age 40-49

805
Copyright © Harvard Medical School, 2018. All Rights Reserved.

False positive mammograms –


provider perspective
Definition varies: additional mammo to
biopsy

– ~5% of mammograms “false positive”


• 20-50% cumulative risk over 10 mammograms

– More common in women younger than 50

=> Problem from population and health care


cost perspective
Elmore, NEJM 2004

False-positive mammograms –
patient perspective

806
Copyright © Harvard Medical School, 2018. All Rights Reserved.

Overdiagnosis
Detection of disease that would not be
clinically apparent during life
– Overdiagnosis => Overtreatment!!
– Difficult to determine in individual
• Ideally estimated from control arm of RCT
• Estimated at 5-50%
– Can be DCIS or small invasive cancers

=> Focus on increasingly sensitive radiologic


techniques worsens problem

Ductal Carcinoma In-Situ (DCIS)


• Rare before screening mammography
– 60,290 cases in 2015
– ~20% of all mammographically detected lesions

• Uncertainty re: natural history of DCIS


– 14-40% estimated to recur if untreated
• Largest study = 80 women (Eusebi 1994)
– Cannot identify those that recur so treat all
• Mastectomy/lumpectomy +/- RT +/-tamoxifen
– Is it really cancer? Confusing nomenclature
• Cells look similar to invasive disease, but do not
invade basement membrane
• Cervical carcinoma in situ => CIN III

807
Copyright © Harvard Medical School, 2018. All Rights Reserved.

Lobular Carcinoma in Situ (LCIS)


• Often incidental finding

• ↑ risk of invasive carcinoma bilaterally


without predictable location - ~1%/yr
– Generally does not need surgical excision

• Management options
– observation
– tamoxifen

Breast density

Fatty breast Dense breast

808
Copyright © Harvard Medical School, 2018. All Rights Reserved.

Breast Density
• Breast density associated with ↑
breast cancer risk => unclear what
screening modality best
– 34 states have mandatory notification
– Ultrasound best for targeted areas, not
whole breast
– MRI high rate of false positives and not
necessarily covered by insurance

Breast tomosynthesis (3-D mammogram)


Breast stays in place and x-ray
machine moves to take multiple
images of breast

Park, 2007
Radiographics,

809
Copyright © Harvard Medical School, 2018. All Rights Reserved.

Breast MRI
• More sensitive – especially in young women
– Does not replace mammograms
– Many more false positives
– Impractical as screening tool
• Current indications for MRI from ACS
• Recommended for:
BRCA1/2, Li-Fraumeni (p53), Cowden’s (pTEN)
Radiation to chest between ages 10 and 30
Lifetime risk of >20-25%

• Insufficient evidence for or against:


Personal history of breast cancer
Dense breasts on mammography
= >Never been shown to reduce mortality!
Saslow CA J Clinic 2007

3D (tomo) vs 2D

• No randomized trial data

• Prospective comparison trials


– ↓↓ recall rate
– ↑↑ cancer detection rate
– Unclear if superior in dense breasts

• Already widely adopted


Friedenwald JAMA 2014; Ciatto Lancet Oncol 2013

810
Copyright © Harvard Medical School, 2018. All Rights Reserved.

Prevention

Prevention question
A healthy 45 y.o. premenopausal woman has
a mother who was diagnosed with breast
cancer at age 60 and passed away at 65 with
metastatic disease. She would like to
discuss breast cancer prevention. You
recommend that she consider:

A. Aromatase inhibitor
B. Raloxifene
C. Tamoxifen
D. All of the above depending upon side
effects

811
Copyright © Harvard Medical School, 2018. All Rights Reserved.

Breast cancer chemoprevention

• Age 60 or older
• Personal history of lobular carcinoma
in situ or atypical hyperplasia
• Age 35-59 with 5 yr predicted risk of
breast cancer >1.66% (Gail model)
http://www.cancer.gov/bcrisktool/

=>Both tamoxifen and raloxifene FDA


approved for chemoprevention

Randomized placebo controlled


prevention trials

• Tamoxifen, raloxifene, aromatase


inhibitors – 40-60% ↓↓ breast ca risk
– Only prevents ER+ breast cancer
– No survival benefit seen
– Raloxifene and AI’s only after menopause
Why isn’t breast cancer chemoprevention done
more?
Absolute risk of breast cancer low
Side effects
Screening available for breast cancer

812
Copyright © Harvard Medical School, 2018. All Rights Reserved.

Tamoxifen vs Raloxifene vs AI

Tamoxifen Raloxifene Aromatase


inhibitors

Hot Flashes Yes Yes No

Uterine effects Yes No No

Bone Density ↑↑ ↑↑ ↓↓
Thrombotic risk Yes Yes No

Premenopaual Yes No No
use

Breast Cancer Treatment

813
Copyright © Harvard Medical School, 2018. All Rights Reserved.

Treatment question
A 60 y.o. woman was diagnosed with a 1.5
cm, node negative, ER positive, HER2
negative breast cancer and had a
mastectomy. What will have the biggest
impact on 10 year overall survival?

1. Hormonal therapy
2. Radiation
3. Chemotherapy
4. Trastuzumab

Treatment of Non-metastatic breast cancer

• Surgery

• Radiation

• Systemic therapy
– Adjuvant chemotherapy
– Hormonal therapy
– HER2 based therapy

814
Copyright © Harvard Medical School, 2018. All Rights Reserved.

Breast cancer surgery - 2018

• Sentinel node biopsy routinely done


– Axillary dissection only for palpable
lymph nodes at diagnosis
• Bilateral mastectomy more common
– No impact on survival (except BRCA/12)
– More preoperative MRI => occult disease
– Celebrities having bilateral mastectomies

Rationale for radiation


More recurrences with Lumpectomy + RT or
lumpectomy alone mastectomy survival same

Fisher, N Engl J Med 2002

815
Copyright © Harvard Medical School, 2018. All Rights Reserved.

Primary Breast Cancer – Systemic Rx


• Patients with disease confined to breast and
axilla can be cured by surgery/RT alone, but
some may develop distant metastases

• Decision for adjuvant treatment based on:


– Tumor characteristics and likelihood of
developing metastases

– Overall life expectancy

Risk factors for developing metastatic


breast cancer

• Biology of tumor
– ER status
– HER status

• Tumor size and grade

• Axillary lymph node involvement

816
Copyright © Harvard Medical School, 2018. All Rights Reserved.

Breast Cancer is More than 1 Disease

1. ER positive + HER2 negative (Luminal A)


Hormonal therapy +/- chemo

2. ER positive + HER2 positive (Luminal B)


Hormonal therapy +/- trastuzumab + chemo

3. ER negative + HER2 positive (HER2 enriched)


Chemo + trastuzumab

4. Triple negative (Basaloid)


No targeted therapies

Gene expression profiling of tumor

Sorlie, PNAS, 2001

817
Copyright © Harvard Medical School, 2018. All Rights Reserved.

OncotypeDX®
• RT-PCR gene expression array of 21 genes
– To estimate recurrence risk - compare pattern to 668 women
treated with tamoxifen in 1980’s on NSABP B-14
• ER positive tumors
– Strongest data for node-negative and 1-3 positive lymph node
• Generally covered by insurance if medically indicated

Paik
NEJM
2004

Risk of distant recurrence by OncotypeDX®

• Low score is good!


• Node negative
– TailoRx: 5 yr Distant met 0.7%
– SEER: 5 yr Br Ca death 0.4%
– TransATAC: 9 yr Distant met
4%
• Node positive
– PlanB: 3 yrs DFS 98%
– SEER: 5 yr Br Ca death 1.0%
– TransATAC : 9 yr distant emt
17%

Paik ,,
New Engl
J Med
2004 Sparano, N Engl J Med 2015; Petkov NPJ
Breast Cancer 2016; Gluz, J Clin Oncol 2016

818
Copyright © Harvard Medical School, 2018. All Rights Reserved.

OncotypeDX® and chemotherapy benefit

Paik J Clin Oncol 2006


Paik, JCO 2006

TAILORx study
• Randomized 10,273 ER positive, node negative
with OncotypeDX 11-25 to hormonal rx +/- chemo
– Overall survival 93.9 vs 93.8% (median f/u 8 yrs)

⇒ ? Benefit to chemo for women < age 50


⇒ ? Node positive patients
Sparano N Engl J Med 2018

819
Copyright © Harvard Medical School, 2018. All Rights Reserved.

Adjuvant breast cancer chemotherapy

• Improves cure rate and overall


survival in appropriate populations
– Triple negative
– HER2 positive when given with
trastuzumab
– ER positive if poorly differentiated or
multiple involved lymph nodes

Flavors of Adjuvant Chemotherapy


• AC – doxorubicin (Adriamycin) +
cyclophosphamide
• AC + T – AC + Taxane (paclitaxel or docetaxel)
• TAC - docetaxel (Taxotere) + AC
• TC – doctaxel + cyclophosphamide
• CAF – AC + 5-FU
• CMF –cyclophosphamide +methotrexate + 5-FU

820
Copyright © Harvard Medical School, 2018. All Rights Reserved.

HER2/neu and Trastuzumab


• HER2/neu (c-erbB2) proto-oncogene - 185-kd
transmembrane factor receptor
– 20-25% of breast cancers overexpress HER2
– Immunohistochemistry (+3) or FISH assay (ratio > 2.0)

• Trastuzumab - monoclonal antibody to HER2


– Can cause congestive heart failure
– Not administered with an anthracycline

• HER2+ tumors have worse prognosis, if not


treated with trastuzumab
– With treatment, prognosis similar to HER2 negative

AC + paclitaxel +/- Trastuzumab

50% Reduction in Recurrence

Perez, NEJM, 2005

821
Copyright © Harvard Medical School, 2018. All Rights Reserved.

pertuzumab
Trastuzumab/
T-DM1

lapatinib

Hudis, NEJM, 2007

Tyrosine kinase needs homo or heterodimerization for activation

Adjuvant Hormonal therapy


• 1st targeted therapy!
– ↓↓ breast cancer mortality by 1/3
– ↓↓ contralateral breast cancer by 1/2
– 5 yrs for stage I, 10 yrs for node positive
• Tamoxifen
– Can be used for both pre and postmenopausal
– No data for raloxifene in adjuvant setting
• Aromatase inhibitors
– Preferred for postmenopaual women
• Ovarian suppression
– Considered for high risk

822
Copyright © Harvard Medical School, 2018. All Rights Reserved.

Metastatic Breast Cancer

Metastatic Breast Cancer Treatment Decision Tree

Luminal A Luminal B HER2 enriched Triple negative

ER +, HER2 - ER +, HER2 + ER -, HER2 + ER -, HER2 -

Hormonal Rx Hormonal Rx Chemo + pertuz


Trastuzumab + Trastuzumab

TDM1
Chemo +
Chemotherapy Chemo + ?novel agents
Trastuzumab Chemo +
Trastuzumab
or lapatinib

823
Copyright © Harvard Medical School, 2018. All Rights Reserved.

Metastatic breast cancer – Hormonal Rx

• 1st line except for visceral crisis


– AI +/- palbociclib for postmenopausal
– Tamoxifen + ovarian suppression for premenopausal
• Aromatase inhibitor
– Anastrozole, letrozole, exemestane
• Fulvestrant
– Pure estrogen receptor antagonist

Targeted therapy
• Palbociclib, ribociclib, and abemaciclib
– Oral Cyclin D Kinase (CDK) 4/6 inhibitor
• Everolimus Improve PFS,
– Oral mTOR inhibitor
but not OS,
add toxicity

• Immunotherapy
– PDL1/PD1 inhibitors in clinical trials
– Some activity in triple negative, but still
unclear how to maximize response

824
Copyright © Harvard Medical School, 2018. All Rights Reserved.

Chemotherapy for Metastatic Breast Cancer


• Doxorubicin • 5-fluorouracil
• Paclitaxel • Capecitabine
• Docetaxel • Vinorelbine
• Cyclophosphamide • Gemcitabine
• Methotrexate • Liposomal
• Eribulin doxorubicin
• Ixabepilone

Principles of metastatic breast cancer

• Chemotherapy => modest improvements in


overall survival
– Main goal of treatment = symptom control/
palliation
• Single agent chemotherapy less toxic than
doublet with similar efficacy
• Targeted therapy (ER and HER2) changed
natural history of metastatic breast cancer
– 20-30% survive > 5 years with metastatic disease
– Wide variation in survival for metastatic breast
cancer

825
Copyright © Harvard Medical School, 2018. All Rights Reserved.

Bisphopshonates for bone metastases

• 70-75% of women with metastatic breast


cancer will develop bone mets
• Complications of bone metastases
– Pain
– Hypercalcemia
– Pathologic fracture
– Cord/nerve compression
=> Routine bisphosphonate use for bone
metastases has significantly improved
quality of life for metastatic pts

Precision or personalized medicine

Breast cancer treatment already targets


several known pathways
– Examples: ER, HER2, mTOR, CDK4/6

826
Copyright © Harvard Medical School, 2018. All Rights Reserved.

Precision medicine
Promises Pitfalls
• Large amount of tumor- • Available target ≠ anti-tumor
specific personalized effect
genetic information – e.g. Hormonal therapy
ineffective for ER+ lung cancer
• Some mutations
already have FDA • Multiple mutations
approved treatments – Pathways have multiple
redundancies, which one is
rate limiting?
• Many mutations do not
currently have effective
targeted therapy

Breast Cancer Summary


• Epidemiology –Overweight and alcohol use increase
breast ca risk, physical activity decreases risk

• Prevention – Tamoxifen , raloxifene, and AI reduce


breast ca risk, but no effect on mortality

• Screening – Mammography leads to early diagnosis of


breast ca, but overdiagnosis a problem

• Primary, non-metastatic, breast cancer –Biologic


subtypes of breast cancer drive appropriate therapy

• Metastatic breast cancer – New biologic agents offer


hope for better survival

827
Copyright © Harvard Medical School, 2018. All Rights Reserved.

Thank You!

References
1. Manson JE et al. Menopausal hormone therapy and health outcomes
during the intervention and extended poststopping phases of the Women’s
Health Initiative randomized trials. JAMA 2013; 310: 11353-68.

2. Moyer VA et al. Medications to decrease the risk for breast cancer in


women: recommendations from the U.S. Preventive Services Task Force
recommendation statement. Ann Intern Med 2013; 159: 698-708

3. Perez EA and Spano JP. Current and emerging targeted therapies for
metastatic breast cancer. Cancer 2012; 118: 3014-25.
http://onlinelibrary.wiley.com/doi/10.1002/cncr.26356/full
4. World Cancer Research Fund International/American Institute for Cancer
Research Continuous Update Project Report: Diet, nutrition, physical
activity and breast cancer. 2017.
http://wcrf.org/breast-cancer-2017

828
Copyright © Harvard Medical School, 2018. All Rights Reserved.

Lymphoma
Multiple Myeloma
Ann S. LaCasce, MD, MMSc
Institute Physician
Dana Farber Cancer Institute
Associate Professor of Medicine
Harvard Medical School

Disclosures for
Ann S. LaCasce, MD
No relevant conflicts of interest to declare
Research Support/P.I.

No relevant conflicts of interest to declare


Employee

No relevant conflicts of interest to declare


Consultant

No relevant conflicts of interest to declare


Major Stockholder

No relevant conflicts of interest to declare


Speakers Bureau

No relevant conflicts of interest to declare


Honoraria

Membership in Advisory Seattle Genetics, BMS


Board
Presentation includes a No relevant conflicts of interest to declare
description of the following
off-label use of a drug or
medical device

829
Copyright © Harvard Medical School, 2018. All Rights Reserved.

Agenda
• Classification of lymphoid malignancies
• Presentation
• Work- up and staging
• Serious complications
• Non-Hodgkin lymphoma
– Diffuse large B-cell lymphoma
– Follicular lymphoma
• Hodgkin lymphoma
– Therapy
– Complications of therapy
• Multiple Myeloma

Classification of lymphoma
• Malignancies of normal lymphoid cells which reside
predominantly in lymphoid tissues (nodes, spleen, marrow)
• WHO classification based on morphology,
immunophenotype, cytogenetics and clinical factors
• Non-Hodgkin lymphoma
– B-cell
• Precursor vs mature
– T and NK-cell
• Precursor vs mature
• Hodgkin lymphoma
– Classical (nodular sclerosis, mixed cellularity, lymphocyte
rich, lymphocyte delpleted)
– Nodular lymphocyte predominant Hodgkin lymphoma

830
Copyright © Harvard Medical School, 2018. All Rights Reserved.

Lymphocyte maturation and lymphoma


subtypes

Jaffe. Blood. 2008

Presentation
• Lymphadenopathy (2/3)

• B symptoms - fever (>38), drenching night sweats,


weight loss > 10% in 6 months

• Extra nodal sites - GI tract, skin, bone

• Rare - kidney, bladder, adrenal, heart, lungs, breast,


testes, thyroid

831
Copyright © Harvard Medical School, 2018. All Rights Reserved.

Differential diagnosis of
lymphadenopathy
• Infection

• Medication (dilantin, sulfonamides, penicillin,


hydralazine)

• Rheumatologic (Lupus, RA, Still’s disease, Churg-


Strauss)

• Other (sarcoid, Kikuchi disease, amyloidosis, chronic


granulomatous disease, Castleman’s disease)

Biopsy
• Supraclavicular > cervical/axillary > inguinal

• Excisional biopsy when possible

• CT guided core needle

• Send for pathology, immunohistochemisty/flow


cytometry

832
Copyright © Harvard Medical School, 2018. All Rights Reserved.

Work-up
• CT scans chest/abdomen/pelvis
• PET scan
• Bone marrow biopsy
• CBC/diff
• BUN/creatinine
• LFTs
• Uric acid
• Electrolytes/calcium
• B2 microglobulin (indolent)
• LDH
• SPEP (CLL/SLL/waldenstrom’s/lymphoplasmacytoid)

Staging

A – asymptomatic; B- fever, night sweats, 10% wt loss

833
Copyright © Harvard Medical School, 2018. All Rights Reserved.

Serious complications

• Cord compression
• Pericardial disease/tamponade
• Hypercalcemia
• SVC/airway compromise
• Hyperviscosity
• Intestinal obstruction
• Ureteral obstruction
• Tumor lysis syndrome
• ITP/AIHA

Risk factors

Exposures: Immune dysfunction:

Occupational Autoimmune disease


Environmental Immunodeficiency
Prior RT, chemotherapy Immune suppression

Genetics:
Exposures:

Occupational
Environmental
Prior RT, chemotherapy

834
Copyright © Harvard Medical School, 2018. All Rights Reserved.

Infectious associations

EBV:
Burkitt lymphoma
DLBCL
NK-T cell lymphoma
Hodgkin lymphoma
Plasmablastic lymphoma

HTLV-1:
Adult T-cell leukemia/lymphoma Marginal zone lymphoma :
H pylori
HHV-8: B burgdorferi
Primary effusion lymphoma C jejuni
Large B cell lymphoma associated Hepatitis C
with Castlemans

Non-Hodgkin Lymphoma

835
Copyright © Harvard Medical School, 2018. All Rights Reserved.

Clinical behavior of non-Hodgkin


lymphoma

Indolent Aggressive Highly


aggressive
Survival Years Months Weeks
untreated
Response to Not curable Curable Curable
chemotherapy
Example Follicular Diffuse large Burkitt
lymphoma B-cell lymphoma
lymphoma

Indolent lymphomas

• B-cell lymphomas • T-cell lymphomas


- B-cell CLL/SLL - T-cell LGL leukemia
- lymphoplasmacytic - Mycosis fungoides
- Hairy cell leukemia
- Follicular (gr 1-2)
- Marginal zone
- Nodal
- Extranodal (MALT)
- Splenic
- Mantle cell*
- Plasma cell myeloma

836
Copyright © Harvard Medical School, 2018. All Rights Reserved.

Aggressive lymphomas

• Diffuse large B-cell lymphoma

• Follicular lymphoma (grade 3)

• Peripheral T-cell lymphoma

• Anaplastic large cell lymphoma

• NK/T cell lymphoma

Highly aggressive lymphomas

Burkitt Lymphoma

Precursor B lymphoblastic lymphoma

Precursor T lymphoblastic lymphoma

Adult T-cell lymphoma/leukemia

837
Copyright © Harvard Medical School, 2018. All Rights Reserved.

Diffuse large B-cell lymphoma

•Most common subtype NHL – 25%


• Median age 65
• Male predominance

Therapy for DLBCL


• 1970’s - CHOP (cyclophosphamide, doxorubicin,
vincristine, prednisone)

• 1980’s – 2nd and 3rd generation regimens (addition


of other active agents, modification of doses and
schedules) with improved CR rates and survivals in
pilot studies

838
Copyright © Harvard Medical School, 2018. All Rights Reserved.

Therapy for large cell lymphoma 1990’s

CHOP

Fisher et al. NEJM 1993

Rituximab – anti CD20 monoclonal antibody


NK- NK-
cell cell
NK-
CD20 CD20 cell

ADCC

CDC
B-cell

CD20

direct killing

839
Copyright © Harvard Medical School, 2018. All Rights Reserved.

Overall
CHOP vs RCHOP Survival patients with
in older
DLBCL

Coiffier, B. et al. N Engl J Med 2002;346:235-242

International Prognostic Index

Pre-Rituxan Era Rituxan Era


Risk 5 yr Risk 4 yr 4 yr
factors OS factors DFS OS
0-1 73% 0 94% 94%

2 51% 1-2 80% 79%


3 42%
3-5 53% 55%
4-5 26%

Risk factors: age > 60, stage III/IV, >1 EN site, PS, LDH

840
Copyright © Harvard Medical School, 2018. All Rights Reserved.

Gene expression profiling in DLBCL

Dunleavy and Wilson. Oncology. 2014 Lenz et al. NEJM. 2008

Follicular lymphoma
• Second most common NHL (20%)
• Median age at presentation - 60
• Male to Female – 1:1.7

841
Copyright © Harvard Medical School, 2018. All Rights Reserved.

Indolent B-cell lymphoma:


clinical management

Localized Advanced Advanced


Low tumor High tumor
burden burden
Involved Field Observation Therapy
RT

Observation vs early therapy


• Prospective randomized and retrospective studies

• No survival disadvantage

• 3 year median progression to treatment


• Grade 1: 48 months
• Grade 2: 16.5 months

• Same rate of histological transformation


• Is an active process, requires periodic monitoring

• Spontaneous remissions can occur

842
Copyright © Harvard Medical School, 2018. All Rights Reserved.

Indications for therapy

• Cytopenias secondary to BM infiltration


• Threatened end-organ function
• Symptoms attributable to disease
• Bulk at presentation
• Steady progression during a period of observation
>6 months
• Presentation with concurrent histologic
transformation
• Massive splenomegaly

Therapy follicular lymphoma


• Not curable with conventional therapy
• Rituximab +/- chemotherapy:
– Bendamustine
– CHOP
• Maintenance
• Novel agents (lenalidomide, idelalisib)
• Stem cell transplants may be curative in subset of
patients with relapsed/refractory disease

843
Copyright © Harvard Medical School, 2018. All Rights Reserved.

Hodgkin lymphoma

Epidemiology

Increased incidence in
industrialized countries

NS subtype associated
with high standard of
living

MC/LD in economically
disadvantaged
countries (EBV
associated)

844
Copyright © Harvard Medical School, 2018. All Rights Reserved.

Clinical presentation

• Fever including classic Pel-Ebstein


• Drenching night sweats (25%)
• Pruritus (10-15%)
• Abdominal/groin pain from retroperitoneal disease
• Alcohol induced pain
• Rarely jaundice
• Adenopathy – neck/mediastinum most common

Competing risks in Hodgkin lymphoma

Minimize
late effects

Maximize
cure

Armitage JO. N Engl J Med 2010

845
Copyright © Harvard Medical School, 2018. All Rights Reserved.

HL complications of therapy

• Fertility
– ABVD low risk of infertility
• Pulmonary Toxicity
– Bleomycin
– Mediastinal RT
• Cardiac toxicity
– Mediastinal RT – pericardial, valvular and CAD
– Doxorubicin - cardiomyopathy
• Second malignancies
– solid tumors from radiation - breast, lung, etc

Hodgkin lymphoma
• Chemotherapy: ABVD developed (adriamycin, bleomycin,
vinblastine, dacarbazine)
• RT: involved site radiotherapy

• Stage I and II Disease


– Chemotherapy with or without radiation
– Approximately 85%-90% cured with initial chemotherapy
• Stage III and IV Disease
– Chemotherapy always required
– Role of radiation therapy to sites of bulky disease
uncertain
– 75% cured with initial therapy depending on risk

846
Copyright © Harvard Medical School, 2018. All Rights Reserved.

Brentuximab Vedotin
ORR 75% (34% CR) with 96% disease control in relapsed HL

Monomethyl auristatin E (MMAE), microtubule-disrupting agent


Protease-cleavable linker
Anti-CD30 monoclonal antibody

CD-30
ADC binds to CD30
ADC-CD30 complex
is internalized and
traffics to lysosome
MMAE is released G2/M cell
MMAE disrupts cycle arrest
microtubule network
Apoptosis

BV+AVD associated with 5% improvement in PFS with


more toxicity

Connors et al.
NEJM 2017

847
Copyright © Harvard Medical School, 2018. All Rights Reserved.

Immunotherapy and Hodgkin lymphoma


Classical HL (cHL) is characterized pathologically by a
failed immune response.
Near uniform amplification at 9p24.1
leading to overexpression of PD-L1 and PD-L2
cHL has a genetically driven vulnerability to
PD-1 blockade.

PD-L1 expression in cHL

Chen et al. Clin Cancer Res. 2013


Ansell et al. N Engl J Med. 2014

On-going research in early HL


• Early stage disease:
– Given excellent prognosis, focus on preventing
long-term complications
– Less chemotherapy
– Less radiation
– Chemotherapy only
• Advanced stage disease:
– Incorporation of upfront brentuximab vedotin,
PD-1 inhibitors
• Risk adapted therapy using PET scans

848
Copyright © Harvard Medical School, 2018. All Rights Reserved.

Multiple Myeloma

Multiple Myeloma

High incidence in
African Americans,
Pacific Islanders

Etiology: MGUS,
irradiation,
exposures

849
Copyright © Harvard Medical School, 2018. All Rights Reserved.

Criteria for Diagnosis of MM

• Monoclonal plasma cells in bone marrow and/or presence of


biopsy-proven plasmacytoma

• Myeloma-related organ dysfunction (1 or more)


(C) serum calcium >10.5 mg/L or ULN
(R) Renal insufficiency (SCr >2 mg/dL)
(A) Anemia (hemoglobin <10 g/dL or 2g <normal)
(B) Lytic bone lesions or osteoporosis

Durie BGM et al. Hematol J.


2003;4:379

Initial diagnostic evaluation


Blood:
CBC with diff and platelet counts
BUN, SCr
Electrolytes, calcium, albumin
Quantitative immunoglobulins
Serum protein electrophoresis (SPEP) and immunofixation
β2-microglobulin
Serum free light chain
Urine:
24-hr protein electrophoresis (UPEP) and immunofixation
Other
Skeletal survey
Bone marrow aspirate and biopsy with cytogenetics
PET in selected situations

850
Copyright © Harvard Medical School, 2018. All Rights Reserved.

International Staging System


Stage Criteria Median Survival
Serum β2M <3.5 g/dL
I 62 mo
Serum albumin ≥3.5 g/dL
II Serum β2M <3.5 g/dL 44 mo
Serum albumin <3.5 g/dL
OR
Serum β2M 3.5 to <5.5 mg/dL*
III Serum β2M ≥5.5 g/dL 29 mo

Complications
• Bone disease/hypercalcemia
• Hyperviscosity-IgM, IgG3, IgA
• Recurrent infections
• Renal failure: hypercalcemia, myeloma kidney,
hyperuricemia, IV urography, dehydration, plasma
cell infiltration, pyelonephritis, amyloidosis
• Cardiac failure: amyloid, hyperviscosity, anemia
• Anemia: BM tumors, renal dysfunction,
myelosuppression, low endogenous erythropoietin
• Neuropathy: sensory ±motor, amyloid, anti-myelin
Ab

851
Copyright © Harvard Medical School, 2018. All Rights Reserved.

MM initial therapy
Non-transplant: Transplant:
• dexamethasone/bortezomib/ • 3 drug combinations*:
cyclophosphamide – dexamethasone/
• dexamethasone/bortezomib/ bortezomib
lenalidomide Plus:
• dexamethasone/ – Lenalidomide or
lenalidomide – cyclophosphamide or
– doxorubicin

Bisphosphonates for bony disease

Stem cell transplantation

• Autologous transplant:
– Survival benefit compared with standard therapy

• Maintenance with lenalidomide improves outcome


but increased secondary malignancies

• Myeloablative allo-transplant rarely performed due


to treatment related mortality

852
Copyright © Harvard Medical School, 2018. All Rights Reserved.

Targeting the hallmarks

Hanahan and Weinberg Cell 2011

Summary
Non-Hodgkin lymphoma:
– Often presents with lymphadenopathy but any
organ may be involved
– Excisional or core biopsy to determine subtype
– Staging with CT +/- PET and bone marrow biopsy
– Aggressive lymphoma is curable in > half of
patients with combination chemotherapy
– Indolent lymphoma is not curable with standard
chemotherapy, but patients may have long
remissions and survival

853
Copyright © Harvard Medical School, 2018. All Rights Reserved.

Hodgkin lymphoma:
– Often presents in neck and mediastinum
– High cure rates
– Early stage disease treated with combined
modality therapy, advanced disease treated with
chemotherapy
– Significant long term toxicities of therapy
Multiple myeloma:
– Diagnosis requires malignant plasma cells in
marrow or plasmacytoma + CRAB criteria
– Initial therapy includes IMIDS/proteasome
inhibitors/steroids with upfront consolidation
with ASCT

Question #1
26 year old college student presents with cough, night
sweats and 20 lb weight loss. On exam she has bilateral
cervical and left supraclavicular lymphadenopathy.
Chest CT scan confirms a 4 cm left supraclavicular node
and a large mediastinal mass.
The most likely diagnosis is:
a. Follicular lymphoma
b. T-cell LGL
c. Hodgkin lymphoma
d. Small lymphocytic lymphoma
e. Burkitt’s lymphoma

854
Copyright © Harvard Medical School, 2018. All Rights Reserved.

Question #1
a. Follicular lymphoma commonly presents in older
adults with asymptomatic lymphadenopathy.
b. T-cell LGL presents with cytopenias and
splenomegaly.
c. Hodgkin lymphoma affects young adults and
presents with adenopathy in the neck and chest. B
symptoms are common.
d. Small lymphocytic lymphoma also presents with
asymptomatic adenopathy with frequent spleomegaly
in older adults.
e. Burkitt’s lymphoma typically presents with rapidly
progressive adenopathy and high LDH.

Question #2
Which of the following are indications for therapy in
the indolent lymphomas?

a. thrombocytopenia
b. bulky lymphadenopathy
c. weight loss
d. transformation to diffuse large B-cell lymphoma
e. all of the above

855
Copyright © Harvard Medical School, 2018. All Rights Reserved.

Question #2
All of the above are indications for initiating therapy
in follicular lymphoma. Early therapy in the
absence of symptoms has not been shown to
prolong overall survival.

References

• Swerdlow et al. The 2016 revision of the World Health


Organization classification of lymphoid neoplasms.
Blood 2016.

• NCCN Clinical Practice Guidelines in Oncology.


www.nccn.org

• Evans LS, Hancock BW. Non-Hodgkin lymphoma. Lancet.


2003 362:139-46.

856
Copyright © Harvard Medical School, 2018. All Rights Reserved.

Disclosures for
Ann S. LaCasce, MD
No relevant conflicts of interest to declare
Research Support/P.I.

No relevant conflicts of interest to declare


Employee

No relevant conflicts of interest to declare


Consultant

No relevant conflicts of interest to declare


Major Stockholder

No relevant conflicts of interest to declare


Speakers Bureau

No relevant conflicts of interest to declare


Honoraria

Membership in Advisory Seattle Genetics, BMS


Board
Presentation includes a No relevant conflicts of interest to declare
description of the following
off-label use of a drug or
medical device

857
Copyright © Harvard Medical School, 2018. All Rights Reserved.

GI Cancers for the Boards


Jeffrey Meyerhardt, MD, MPH
Clinical Director, Gastrointestinal Cancer Center
Deputy Clinical Research Officer
Dana-Farber Cancer Institute

Associate Professor of Medicine


Harvard Medical School
Senior Physician, Dana-Farber Cancer Institute

Disclosures

• Advisory board: Ignyta


• Consultant: Chugai Pharmaceutical

858
Copyright © Harvard Medical School, 2018. All Rights Reserved.

Common GI Malignancies:
Esophageal Cancer
Gastric Cancer
Pancreatic Cancer
Colorectal Cancer
Hepatocellular Carcinoma

Less Common GI Malignancies:


Biliary Cancer/Gallbladder
Pancreatic Islet Cell/Carcinoid Tumors
Anal Cancer
Small bowel
Total Annual Cases in US 310,440
Total Deaths in US 157,700

CA Cancer J Clin 2018;68:7‐30

Annual Incidence and Mortality of


Gastrointestinal Malignancies
United States (2017) Worldwide (2012)
Siegel et al CA Cancer J Clin 2017 Globocan2012 WHO

Cancer New Cases Deaths New Cases Death


Esophageal 17,290 15,850 455,784 400,169

Gastric 26,240 10,800 951,594 723,073

Pancreas 55,440 44,30 337,872 330,391

Colorectal 140,250 50,630 1,360,602 693,933

Liver 42,220 30,200 782,451 745,533

Breast 268,670 41,400 1,671,149 521,907

Lung 253,290 158,770 1,824,701 1,589,925

859
Copyright © Harvard Medical School, 2018. All Rights Reserved.

Esophageal Cancer

Esophageal Cancer: Risk Factors


Risk Factor Squamous-Cell Carcinoma Adenocarcinoma
Tobacco use +++ ++
Alcohol use +++ —
Barrett’s esophagus — ++++
Weekly reflux symptoms — +++
Obesity — ++
Poverty ++ —
Achalasia +++ —
Caustic injury to the esophagus ++++ —
Nonepidermolytic palmoplantar keratoderma
(tylosis) ++++ —
Plummer–Vinson syndrome ++++ —
History of head and neck cancer ++++ —
History of breast cancer treated with XRT +++ +++

Adapted from Enzinger and Mayer, N Engl J Med. 2003 Dec 4;349(23):2241-52

860
Copyright © Harvard Medical School, 2018. All Rights Reserved.

* Per 100,000 persons

5
*
4.5
4
3.5
3
2.5 ADC/Esophagus
2 SCC
1.5
1
0.5
0
1972 80 88 1996

Per Capita Cigarette Consumption, Major Smoking & Health Events,


and Incidence of Esophageal Cancer - United States

Temporal Trends in Incidence Rates and Survival Rates for Esophageal


Adenocarcinoma.

Rustgi AK, El-Serag HB. N Engl J Med 2014;371:2499-2509.

861
Copyright © Harvard Medical School, 2018. All Rights Reserved.

Neoplastic Progression of Barrett’s Esophagus

GERD 0.005%/yr
1:7 Americans 0.5%/yr
1%/yr
4% 5%
Squamous 10% /yr Low- ? High- /yr 85+%
epithelium
Metaplasia Grade Grade ADC METS
Dysplasia Dysplasia

Early Genetic Events: c-erbB2


Oxidative stress
Inflammation
17pLOH p53; 9pLOH p16
cyclin D1 E-cadherin-
G1&G2 catenin
2nd Tier Genetic Events:
COX-2 p53 mutation; p16 mutation/methylation
BCL-2 EGF(R), telomerase RNA
Late Genetic Events:
4 N (G2) aneuploidy of 5q/13q
and LOH of 5q/13q(Rb)/18q

Local Esophageal Cancer:


Treatment Options
• Surgery alone
– Resectability rates 54-69%
– Perioperative mortality 4-10%
– Perioperative complications (cardiac,
infection, anastomotic leaks) 26-41%

– 2 year survival 35-42%


– 5 year survival 15-24%

862
Copyright © Harvard Medical School, 2018. All Rights Reserved.

Local Esophageal Cancer:


Treatment Options
• At least 3 randomized trials have shown no
benefit to postoperative radiation or
postoperative chemotherapy
(Fok Surgery 1997, Tenier Surg Gyn Onc 1991, Ando J Thor Cardiovasc Surg 1997)

• Not standard practice in US – postoperative


therapy reserved for R1 resections

• 2 neoadjuvant chemotherapy trials with


conflicting results
(Kelson NEJM 1998; MRC Lancet 2002)

Radiation Therapy vs. Chemotherapy + Radiation


Therapy for Localized SCC or ADC of the Esophagus

Herskovic. N Engl J Med 1992, Al-Sarraf. J Clin Oncol 1997

863
Copyright © Harvard Medical School, 2018. All Rights Reserved.

Preoperative Chemoradiation followed by Surgery


versus Surgery Alone in Esophageal Cancer

Sjoquist Lancet
Oncology. 2011.
12 (7), p 681–692

Esophageal Cancer:
Treatment Algorithm

Early stage disease (T1-2, N0) Surgery

Preoperative
Locally advanced disease chemoradiation
regimen followed
(T3, N0-1)
by surgery

Palliation of local
Metastatic Disease symptoms
Palliative chemo if
appropriate

864
Copyright © Harvard Medical School, 2018. All Rights Reserved.

Esophageal Cancer: Survival


Stage Tumor Node Mets 5 year
survival
0 Tis N0 M0 > 95%
1 T1 N0 M0 50-80%
IIA T2-3 N0 M0 30-40%
III T3 N1 M0 10-25%
T4 Any N M0
IVA Any T Any N M1a <5%
IVB Any T Any N M1b < 1%

Adapted from Enzinger and Mayer, N Engl J Med. 2003 Dec 4;349(23):2241-52

Chemotherapy for Metastatic Esophageal


and Gastric Cancer

• No standard therapy for metastatic


disease – typically platinum based
combination regimen
• Expectations of current treatment options
– Response rates 30-60%
– Relief of dysphagia (without XRT) up to 80%
– Median overall survival 7-10 months

865
Copyright © Harvard Medical School, 2018. All Rights Reserved.

Gastric Cancer

Incidence of Gastric Cancer


(Cases per 100,000 population)

35
30
25
20
Men
15 Women
10
5
0
1930 1990

866
Copyright © Harvard Medical School, 2018. All Rights Reserved.

Risk Factors for Gastric Adenocarcinoma


Nutritional
Low fat or protein consumption
Salted meat or fish
High nitrate consumption
Environmental
Poor food preparation (smoked)
Lack of refrigeration
Poor drinking water (well water)
Occupation (rubber, coal workers)
Smoking (1.6x)
Low social class
Medical
Prior gastric surgery
Helicobacter pylori infection (2x)
Gastric atrophy and gastritis
Hereditary
E cadherin mutation families

Gastric Cancer Survival by Stage


National Cancer Data Base 1985-1996

1
0.9
0.8 IA

0.7
Survival (%)

0.6 IB

0.5
0.4 II
0.3
IIIA
0.2
IIIB
0.1
IV
0
0 1 2 3 4 5
Time, years

867
Copyright © Harvard Medical School, 2018. All Rights Reserved.

Proportion of H.pylori-Positive and H.pylori-Negative Patients who


Remained Free of Gastric Cancer

Uemura et al, N Engl J Med,345,2001; 345: 784-789

Chemoradiotherapy After Surgery Compared with Surgery


Alone for Adenocarcinoma of the Stomach or
Gastroesophageal Junction

MacDonald et al, N Engl J Med, 2001;345: 784-9

868
Copyright © Harvard Medical School, 2018. All Rights Reserved.

CALGB 80101: Overall Survival

O ve ra ll S u rviv al b y Arm
1.0

ECF
0.8

5 -F U
Proportion Surviving

0.6
0.4
0.2

P, log rank = 0.80


0.0

0 1 2 3 4 5 6 7
Y e a rs fro m S tu d y E n try

Fuchs et al J Clin Oncol 29(suppl):256s, abstr 4003.

MAGIC: Peri-operative Chemotherapy


R A
A ECF Surgery only ECF
Deemed surgically
N
resectable gastric
D
cancer
O
N=503 M
I Surgery only
Z B
E

36% v 23% at
5 years

Notes: 88% of patients randomized to chemotherapy arm had surgery but only 42% of
patients completed all protocol treatment
Cunningham N Engl J Med. 2006 Jul 6;355(1):11-20

869
Copyright © Harvard Medical School, 2018. All Rights Reserved.

Chemotherapy for Metastatic Esophageal


and Gastric Cancer
• No standard therapy for metastatic
disease – typically platinum based
combination regimen
• Expectations of current treatment options
– Response rates 30-60%
– Relief of dysphagia (without XRT) up to 80%
– Median overall survival 7-10 months

Trastuzumab + Chemotherapy Versus


Chemotherapy only for HER2-positive
Advanced Gastric or GEJ Junction cancer

Overall Survival Progression-free Survival

Bang et al The Lancet, Volume 376, Issue 9742, 2010, 687 - 697

870
Copyright © Harvard Medical School, 2018. All Rights Reserved.

Ramucirumab in Previously Treated


Advanced Gastric or GEJ adenocarcinoma

Overall Survival Progression-Free Survival

Ramucirumab Placebo
Median Overall Survival 5.2 months 3.8 months
Median Progression Free Survival 2.1 months 1.3 months

Fuchs et al. The Lancet, Volume 383, Issue 9911, 2014, 31 - 39

Checkpoint Inhibitors in Gastric Cancer

Fuchs et al. JAMA Oncol. 2018;4(5):e180013. doi:10.1001/jamaoncol.2018.0013

871
Copyright © Harvard Medical School, 2018. All Rights Reserved.

Take Home Points of Gastric Cancer

• Gastric cancer decreasing incidence in


past few decades
• ~50% of patients diagnosed will pass
away from gastric cancer
• Standard of care for localized disease is
surgery and often chemoRT
postoperatively
• Metastatic disease median survival < 1 yr

Board Question #1
Your patient is a 72 year old carpenter who presents with several
month history of difficulty swallowing, primarily just certain
foods. He has lost approximately 10 pounds in the past
month, but he claims that he was trying to diet. You order an
EGD and there is a mass in his midesophagus. Biopsies
demonstrate squamous cell carcinoma. He consults a
thoracic surgeon who performs an esophagectomy and final
pathology reveals invasion through the muscle to subserosa
(T3) and 2 positive lymph nodes. Margins were all negative.
You now recommend:

A. Adjuvant chemotherapy
B. Adjuvant radiation
C. Combined chemotherapy and radiation
D. Observation and intermittent surveillance

872
Copyright © Harvard Medical School, 2018. All Rights Reserved.

Board Question #1

A. Adjuvant chemotherapy
B. Adjuvant radiation
C. Combined chemotherapy and radiation
D. Observation and intermittent surveillance

Since patient has esophageal cancer, no role for adjuvant therapy


based on multiple trials of either modality
If tumor was GE junction, adjuvant therapy would be reasonable
to offer

Pancreas Cancer

873
Copyright © Harvard Medical School, 2018. All Rights Reserved.

Pancreatic Cancer: Survival


Data from National Cancer Database 1985-1990

Relative %
at each time
point

Niederhuber et al. (2006) Cancer. 76 (9): 1671-7

Risk Factors for Pancreatic


Adenocarcinoma
• Hereditary
– Hereditary chronic pancreatitis
– Peutz-Jeghers syndrome
– Ataxia-telangiectasia
• Chronic pancreatitis – mixed data
• Diabetes (relative risk 2)
• Smoking
• Obesity
• History of partial gastrectomy

874
Copyright © Harvard Medical School, 2018. All Rights Reserved.

Pancreatic Cancer: Treatments

• Local disease: Surgery + adjuvant therapy

• Borderline resectable: Neoadjuvant


chemotherapy, reassess, +/- radiation,
reassess, surgery if resectable

• Locally advanced: Chemotherapy +/-


chemoradiation

• Metastatic: Chemotherapy

Pancreatic Cancer: Local Disease

• Curative-intent resection possible in only 15%


of all patients with pancreatic cancer
• 15% of resected patients survive 5 years
• Adjuvant Therapy trials
– ChemoRT after surgery beneficial in 1 trial of 43
patients in US; not beneficial in 2 European trials
– Chemotherapy beneficial in 2 trials in European
– Nonetheless improvements are modest

Yang et al. CA Cancer J Clin. 2005 55(6):352-67.

875
Copyright © Harvard Medical School, 2018. All Rights Reserved.

Pancreatic Cancer: Borderline Resectable

P R R R
E F
R E E E
Patient with N m GEM O
E- S 50.4g S S
BLR PDAC R FOLFIRINOX L
RE T EBRT T SURGERY T
(Intergroup O 2 L
GI A + CAPE A A
Definition) ST
L 2 months months O
G G G
ER L W
E E E

• 27% RECIST response

• R0 operations possible in 64% patients

• 33% resections <5% viable cells, 13% pCR

• median OS of all enrolled patients: 22 months

Katz et al JAMA Surg. 2016 Aug 17; 151(8): e161137.

Pancreatic Cancer: Metastatic Disease

Response rate 5% 0%

Burris HA, et al J Clin Oncol 1997;15:2403-2413.

876
Copyright © Harvard Medical School, 2018. All Rights Reserved.

Metastatic Pancreatic Cancer: FOLFIRINOX


R
A
Gemcitabine
N
D
O
M FOLFIRINOX
I
(5-FU, LV, Irinotecan, Oxaliplatin)
Z N = 842
E

Response Median Median 1 year OS Grade 3/4 Grade 3/4


Rate PFS OS WBC Emesis

Gemcitabine 9% 3.3 m 6.8 m 21 % 19 % 5%

FOLFIRINOX 33 % 6.4 m 11.1 m 48 % 46 % 15 %

P value 0.0001 <0.0001 <0.0001 <0.0001 0.0001 0.002


Conroy et N Engl J Med. 2011 May 12;364(19):1817-25.

Metastatic Pancreatic Cancer:


Gemcitabine + nab Paclitaxel
R
A
Gemcitabine
N
D
O
M
I
Gem-nab Paclitaxel
Z N = 861
E

Response Median Median 1 year Grade 3/4 Grade 3/4


Rate PFS OS OS WBC Neuropathy

Gemcitabine 7% 3.7 m 6.7 m 22 % 27 % 1%

Gemcitabine- 23 % 5.5 m 8.5 m 35 % 38% 17 %


nab Paclitaxel
P value <0.0001 <0.0001 <0.0001 0.0002 0.001 0.001

Von Hoff N Engl J Med 2013 Oct 31;369(18):1691-703

877
Copyright © Harvard Medical School, 2018. All Rights Reserved.

Take Home Points of Pancreatic Cancer

• Only 15% of patients diagnosed are


eligible for surgery and only 15% of those
patients will be cured (cure rate 2-3%)
• Locally advanced disease chemoRT
• Metastatic disease treated with
chemotherapy and median survival 6-11
months – some increased choices but
more toxic
• Yet to find “targeted” therapy options

Colorectal Cancer

878
Copyright © Harvard Medical School, 2018. All Rights Reserved.

Risk Factors for Developing Colorectal Cancer

Decrease Risk Increase Risk Uncertain Impact


Screening Family history Statins
Exercise IBD Fiber
Vitamin D Diabetes Glycemic index
Aspirin / NSAIDs Obesity Fruits/Vegetables
Post-menopausal Red meat Folic Acid
estrogen Western diet
Calcium Alcohol
Smoking

Colorectal Cancer: Risk Factors


• Hereditary
– Familial syndromes (~5% of CRC)
• Familial adenomatous polyposis
• Hereditary nonpolyposis cancer syndrome

– Family history (or personal history) (~10-15%)


• ~ 2 x risk of developing CRC
• Depending on age of relative at dx and # of relatives

879
Copyright © Harvard Medical School, 2018. All Rights Reserved.

Colorectal Cancer: Screening


• Why screen?

Winawer, S. J. et al. N Engl J Med 1993;329:1977-1981

Colorectal Cancer: Screening


• American Cancer Society recommendations for
average risk patients (updated 2008)

Tests that find polyps and cancer


• Flexible sigmoidoscopy every 5 years *
• Colonoscopy every 10 years
• Double contrast barium enema every 5 years *
• CT colonography (virtual colonoscopy) every 5 years *
Tests that mainly find cancer
• Fecal occult blood test (FOBT) every year *
• Fecal immunochemical test (FIT) every year *
• Stool DNA test (sDNA), interval uncertain *

*Colonoscopy should be done if test results are positive.


Levin et al CA Cancer J Clin 2008; 58:130-160

880
Copyright © Harvard Medical School, 2018. All Rights Reserved.

Colorectal Cancer: Screening


• Screening still not universal enough
– Any screening 54% in 2002 65% in 2010
– 23 million Americans age 50-75 never screened

Centers for Disease Control and Prevention (CDC). Behavioral Risk Factor Surveillance System Survey Data. Atlanta,
Georgia: U.S. Department of Health and Human Services, Centers for Disease Control and Prevention, 2010.

Colorectal Cancer: AJCC 8


Primary tumor (T)

Tx Primary tumor cannot be assessed


Tis Carcinoma in situ
T1 Tumor invades submucosa
T2 Tumor invades muscularis propia
T3 Tumor invades through the muscularis propia into the subserosa
T4 Tumor directly invades other organs or structures, and/or perforates

Nodal status (N)


N1a = 1 node
N1b = 2-3 nodes
Nx Regional lymph nodes cannot be assessed N1c = tumor deposits in
N0 No regional lymph node metastases subserosa or mesentery
N1 Metastases in 1 to 3 regional lymph nodes
N2 Metastases in 4 or more regional lymph nodes
N2a = 4-6 nodes
N2b = 7+ nodes
Distance Metastases (M)

Mx Distant metastases cannot assessed M1a = single organ


M0 No distant metastases detected M1b = more than 1 organ
M1 Presence of distant metastases M1c = peritoneal mets (with or without
other organs)

881
Copyright © Harvard Medical School, 2018. All Rights Reserved.

Colorectal Cancer: Staging

Stage 5 year survival


Stage I > 90%
Stage II 70-85%
Stage III 30-70%
Stage IV 8-10 %

Colorectal Cancer:
Standard Therapy Algorithm
Stage Colon Rectal
I (T1-T2, N0, M0) Surgery only Surgery only
Surgery +/- Chemoradiation
II (T3-T4, N0, M0)
Chemotherapy Surgery
Chemotherapy
OR
Surgery
III (Tany, N+, M0) Surgery
Chemotherapy
Chemoradiation &
Chemotherapy
Chemotherapy +/- Chemotherapy +/-
IV (Tany, Nany, M1)
Surgery Surgery

National Comprehensive Cancer Network. NCCN Clinical Practice


Guidelines in Oncology: Colon Cancer V3.2012. Available at:
www.nccn.org. Accessed April 12, 2012.

882
Copyright © Harvard Medical School, 2018. All Rights Reserved.

Colorectal Cancer: Surgery


• Surgical resection cures a large number of patients
with early disease
– 80% of patients present without detectable mets

• Colon cancer
– At least 12-14 nodes should be included in sample
– Increasing evidence for equivalent outcomes with
laparoscopic colectomy

• Rectal cancer
– Low anterior resection – maintains sphincter
– Abdominoperineal resection – low tumors permanent
colostomy

Colorectal Cancer: Radiation


• Why beneficial in rectal and not colon?

– Related to risk of local recurrence (radiation is a


local treatment)

• In colon cancer – <2% risk of local recurrence


• In rectal cancer – up to 30% local recurrence
rate with surgery alone

883
Copyright © Harvard Medical School, 2018. All Rights Reserved.

Colorectal Cancer: Chemotherapy


• Stage III colon cancer
– Adjuvant 5-FU/LV reduces the risk of disease
recurrence by 40% and overall mortality by 33%.
– 1990 consensus statement from NCI committee
recommended adjuvant 5-FU and leucovorin for
stage III patients
– 3 trials demonstrated improvements with adding
oxaliplatin to fluoropyrimidine
• Downside: Neurotoxicity
• Ongoing studies testing 3 v 6 months therapy
Sun W, Haller DG. Seminars in Oncology. 2005;32(1):95-102.
Andre T, et al. N Engl J Med 2004;350(23):2343-2351.

Colorectal Cancer: Chemotherapy


• 5 yr overall survival for stage III colon cancer
surgery surgery w/chemo

Stage IIIA 52% 71%


Stage IIIB 37% 51%
Stage IIIC 21% 32% using AJCC 6

Surgery only Surgery + Adjuvant 5-FU


Greene FL, et al. Ann Surg. 2002;236:416

884
Copyright © Harvard Medical School, 2018. All Rights Reserved.

FOLFOX Adjuvant Therapy –


Duration Based on Risk Groups
T1-3 N1 (58.7%) T4 or N2 (41.3%)
100 100
Duration Duration
90 3 Months 3 Months
90
6 Months 6 Months
80 80

70 70
Percent Without Event

Percent Without Event


60 60
3-yr DFS% (95%
Duration HR (95% CI)
50 CI) 50
Duration 3-yr DFS% (95% CI) HR (95% CI)
40 1.01 (0.90 – 40
3m 83.1 (81.8 – 84.4)
1.12) 3m 62.7 (60.8 – 64.6) 1.12 (1.03 – 1.23)
30 30

20 6m 83.3 (82.1 – 84.6) Ref 20 6m 64.4 (62.6 – 66.4) Ref


10 10
3-yr DFS diff. = -0.2%, 95% CI, (-1.9 to 1.5%) 3-yr DFS diff. = -1.7%, 95% CI, (-4.3 to 0.9%)
0
0
0 1 2 3 4 5 6
0 1 2 3 4 5 6

Years from Randomization


Years from Randomization
3744 3313 2796 1934 1064 527 211
2634 2099 1640 1044 531 292 107
3727 3336 2788 1949 1081 566 221
2622 2151 1655 1094 586 301 110

Interaction p-value = 0.11

Shi et al N Engl J Med 2018; 378:1177-1188

Colorectal Cancer: Chemotherapy


• Stage II colon cancer

– No randomized trials with enough stage II patients to


draw definitive recommendations.
– 2 meta-analyses give conflicting results
– ASCO Consensus panel concluded that based on
available evidence, benefit would not exceed 5% - a
benefit between 0-5% possible (JCO 2004)
– High risk patients may be appropriate – T4 lesions,
obstruction or perforation, few lymph nodes in sample.
– Various molecular marker panels are prognostic but not
predictive

885
Copyright © Harvard Medical School, 2018. All Rights Reserved.

Colorectal Cancer: Chemotherapy


• Metastatic disease
– In general, not curable
• Patients with isolated metastases that can be
surgically resected – up to 30% long term
survival.
– Chemotherapy known to prolong life and
improves symptoms
• Median survival without therapy ~ 6 months

Colorectal Cancer: Chemotherapy


1998
• FDA approved chemotherapy option for
metastatic colorectal cancer

5-FU
Median survival with therapy 10-12 months

886
Copyright © Harvard Medical School, 2018. All Rights Reserved.

Colorectal Cancer: Chemotherapy options for


Metastatic Colorectal Cancer 2018
– First line – Previously treated with irinotecan
• 5-FU +/- bevacizumab • Cetuximab (RAS wildtype)
• Capecitabine • Cetuximab + Irinotecan (RAS
• FOLFIRI or IFL +/- wildtype)
bevacizumab or cetuximab • Panitumumab (RAS wildtype)
(RAS wildtype) or
panitumumab (RAS wildtype) – All prior therapies
• FOLFOX +/- bevacizumab or • Regorafenib
cetuximab (RAS wildtype) or
panitumumab (RAS wildtype) • TAS102

– Second-line – MSI-H tumors


• Irinotecan (5-FU progression) • Pembrolizumab
• FOLFOX (Irinotecan • Nivolumab
progression)
• FOLFIRI with ziv-aflibercept – BRAF mutated
• FOLFIRI with ramucirumab • Vemurafenib/Irinotecan
/Cetuximab

Le DT et al. N Engl J Med 2015;372:2509-2520.

887
Copyright © Harvard Medical School, 2018. All Rights Reserved.

Colorectal Cancer: Progress

Prior to any active chemotherapy 6 months

Fluoropyrimidine only 10-12 months

Fluoropyrimidine + 1 other active cytotoxic 14-16 months


chemotherapy (irinotecan or oxaliplatin)

Fluoropyrimidine + irinotecan + oxaliplatin 20-22


(at some point in course of treatment) months

All three cytotoxic chemotherapy during


course of treatment with exposure
to VEGF and EGFR inhibitors 2+ years

Adapted from Meyerhardt and Mayer, NEJM 2005

Take Home Points of Colorectal Cancer


• Disease common – some risk factors modifiable

• 80% patients without detectable mets at diagnosis

• Treatment depends on stage of disease – surgery is


considered in all patients

• Stage III colon cancer adjuvant chemo


Stage II and III rectal neoadj chemoRT or post op
chemoRT and chemo adjuvant either way
Stage II colon ? Chemo adjuvant?

• Metastatic disease treatment mainly palliative but


median survival > 2 years

888
Copyright © Harvard Medical School, 2018. All Rights Reserved.

Board Question #2
Your patient is a 56 year old school teacher who presented with
about 2 months of intermittent blood in the toilet bowl with
bowel movements. A colonoscopy is performed which
demonstrates a mass in the mid sigmoid and biopsy confirm
adenocarcinoma. He undergoes laparoscopic hemicolectomy
and final pathology reveals a 3 cm, moderately differentiated
adenocarcinoma through the muscle layer into the serosa
with 2 of 9 lymph nodes positive. The next step would be:
A. Re-operate for more complete lymph node dissection
B. Referral to medical oncologist for consideration of
chemotherapy
C. Followup colonoscopy in one year
D. Referral to radiation oncologist for postoperative radiation

Board Question #2
A. Re-operate for more complete lymph node dissection
B. Referral to medical oncologist for consideration of
chemotherapy
C. Followup colonoscopy in one year
D. Referral to radiation oncologist for postoperative radiation

• Patient has stage III colon cancer


• Lymph node sampling is not ideal (12 is preferred #) – can
request pathologist to further search
• Patient should be considered / offered chemotherapy
• Colonoscopy in 1 yr is appropriate surveillance but not next
step
• Radiation reserved for rectal and rarely colon (T4)

889
Copyright © Harvard Medical School, 2018. All Rights Reserved.

Selected References
• Rustgi A and El-Serag. Esophageal Cancer. N Engl J Med 2014;371:2499-509.

• Cohen and Leichman. Controversies in the Treatment of Local and Locally


Advanced Gastric and Esophageal Cancers. Journal of Clinical Oncology 33,
no. 16 (June 2015) 1754-1759.

• Shah and Kelsen. Gastric cancer: A primer on the epidemiology and biology of the
disease and an overview of the medical management of advanced disease. J Natl
Compr Canc Netw. 2010; 8: 437-47.

• Heestand, Murphy and Lowy. Approach to Patients With Pancreatic Cancer


Without Detectable Metastases. Journal of Clinical Oncology 33, no. 16 (June
2015) 1770-1778.

• Ko. Progress in the Treatment of Metastatic Pancreatic Cancer and the Search for
Next Opportunities. Journal of Clinical Oncology 33, no. 16 (June 2015) 1779-
1786.

• Fakih. Metastatic Colorectal Cancer: Current State and Future Directions. Journal
of Clinical Oncology 33, no. 16 (June 2015) 1809-1824.

Disclosures

• Advisory board: Ignyta


• Consultant: Chugai Pharmaceutical

890
Copyright © Harvard Medical School, 2018. All Rights Reserved.

Take Home Points


Ann S. LaCasce, MD, MMSc
Institute Physician
Dana Farber Cancer Institute
Associate Professor of Medicine
Harvard Medical School

Disclosures for
Ann S. LaCasce, MD
No relevant conflicts of interest to declare
Research Support/P.I.

No relevant conflicts of interest to declare


Employee

No relevant conflicts of interest to declare


Consultant

No relevant conflicts of interest to declare


Major Stockholder

No relevant conflicts of interest to declare


Speakers Bureau

No relevant conflicts of interest to declare


Honoraria

Membership in Advisory Seattle Genetics, BMS


Board
Presentation includes a No relevant conflicts of interest to declare
description of the following
off-label use of a drug or
medical device

891
Copyright © Harvard Medical School, 2018. All Rights Reserved.

Management of SVC syndrome

• Tissue diagnosis critical for Rx decisions


– Prognosis depends on underlying disease
• Treatment plan depends on tumor histology
– Chemo-insensitive cancers (e.g. NSCLS) treated
with upfront XRT
– Chemo-sensitive tumors (e.g. small cell,
lymphoma) treated with upfront chemo
– Stent placement not usually done 1st line in
cancers that may respond to chemo/RT
=> Rarely fatal

Spinal Cord Compression

• Initiate treatment earlier to prevent neurologic deficits


– Once neurologic deficits occur, often irreversible
• Usually from epidural compression from vertebral body
metastases
– Most common tumors: lung, breast, prostate,
myeloma, lymphoma
– Thoracic spine most common location
– Intramedullary metastases less common

892
Copyright © Harvard Medical School, 2018. All Rights Reserved.

Treatment
• Corticosteroids to decrease edema
– Only short-term benefit
– Caution if diagnosis unknown
– Typically 10 mg load then 4 mg q 6hrs
• Radiation 1st line treatment for most
• Upfront surgery reserved for:
– Unknown diagnosis
– Progression during or after radiation
– Spinal instability
– One RCT showed improved function with immediate surgery
for less radiosensitive tumors with single area of
compression

F+N likely organisms

• Gram-positive infections (60-75%)


– Staph epidermidis
– Streptococcus
– Enterococcus faecalis/faecium
• Gram-negative rods (more likely to cause death)
– Pseudomonas aeruginosa
– E. coli
– Klebsiella pneumonia

893
Copyright © Harvard Medical School, 2018. All Rights Reserved.

Treatment
Empiric antibiotics: broad spectrum with gram positive and
gram negative coverage (especially Pseudomonas)
• 3rd generation cephalosporin (ceftaz or cefepime)
– May depend upon local hospital bacteriology
• Alternatives:
– Imipenem cilastatin or meropenem
• Higher rate C.diff colitis than cephalospoin
– Beta-lactam allergy: Quinolone with gram pos
• <1% cross-reactivity between 3rd generation ceph
and PCN/1st gen cephalosporin

Hypercalcemia
• Tumor secretion of PTHrP
• Local osteolytic hypercalcemia
• 1,25 (OH)2D-production by tumor (lymphoma)
• Ectopic PTH (very rare)
• Increase urinary calcium excretion
– Normal saline to volume replete then add loop
diuretic (immediate effect)
• Inhibit osteoclastic bone resorption
– Bisphosphonates (2-4 days for max effect)
– Calcitonin (immediate; tachyphylaxis in 2-3d)

894
Copyright © Harvard Medical School, 2018. All Rights Reserved.

Tumor lysis syndrome


• Large tumor burden with rapid cell kill
• More common with aggressive leukemia and lymphomas, e.g
Burkitt’s
• Occurs during first cycle
• Fatal complications include arrhythmias and renal failure
• High uric acid does not equal TLS
• Prophylaxis/prevention key
– Allopurinol and aggressive hydration to maintain urine output
• Treatment
– Rasburicase - recombinant urate oxidase
• Converts uric acid to allantoin which is more soluble in
urine than uric acid
• Dialysis if oliguric or persistent metabolic abnormalities or severe
symptoms

Breast cancer
• Risk factors:
– Age
– Obesity/sedentary lifestyle
– Alcohol
– Exposure to hormones
– BRCA 1/2
• BRCA 1/2
– Responsible for 5-10% breast CA
– BRCA1: 50-70% risk breast and 40-50% ovarian
– BRCA2: 40-60% breast (including male) and 10-25% ovarian
• Prevention
– Tamoxifen , raloxifene, and AI reduce breast ca risk, but no effect on
mortality
• Screening
– Women aged 50-74 should get routine screening mammogram every
1-2 years

895
Copyright © Harvard Medical School, 2018. All Rights Reserved.

Primary Breast Cancer

• Patients with disease confined to breast and axilla can


be cured by surgery/RT alone, but some may develop
distant metastases

• Decision for adjuvant treatment based on:


– Size of tumor
– Tumor grade
– Axillary nodal involvement
– ER status
– HER2/neu status
– Overall life expectancy

Breast cancer subtypes dictate therapy


• ER positive + HER2 negative (Luminal A)
Hormonal therapy +/- chemo

• ER positive + HER2 positive (Luminal B)


Hormonal therapy +/- trastuzumab + chemo

• ER negative + HER2 positive (HER2 enriched)


Chemo + trastuzumab

• Triple negative (Basaloid)


No targeted therapies

896
Copyright © Harvard Medical School, 2018. All Rights Reserved.

Risk of distant recurrence by OncotypeDX®

RCT Oncotype Dx 11-25

Paik ,,
New Engl
J Med
2004 Sparano NEJM 2018

Adjuvant breast cancer therapy


• Chemotherapy improves overall survival in:
– Triple negative
– HER2 positive when given along with trastuzumab
– ER positive if poorly differentiated or multiple involved lymph
nodes
• Hormonal therapy:
– 1st targeted therapy!
– ↓↓ breast cancer mortality by 1/3
– ↓↓ contralateral breast cancer by 1/2
• Tamoxifen
– Selective estrogen receptor modulator – 5 vs10 yrs
– No data for raloxifene in adjuvant setting
• Aromatase inhibitors
– Preferred for postmenopaual women
• Ovarian suppression
– Considered for high risk

897
Copyright © Harvard Medical School, 2018. All Rights Reserved.

Metastatic Breast Cancer Treatment Decision Tree

Luminal A Luminal B HER2 enriched Triple negative

ER +, HER2 - ER +, HER2 + ER -, HER2 + ER -, HER2 -

Hormonal Rx Hormonal Rx Chemo + pertuz


Trastuzumab + Trastuzumab

TDM1
Chemotherapy Chemo +
Chemo + ?novel agents
Trastuzumab Chemo +
Trastuzumab
or lapatinib

Lung cancer epidemiology

• Leading cause of cancer deaths at 154,000/yr


• Smoking:
– 85% of cases in smokers
– 15 – 30 risk of developing lung ca
– Quitting reduces risk but not to baseline
• More than half of patients diagnosed with lung ca
have metastatic disease
• 90% of cases are non-small cell lung cancer

898
Copyright © Harvard Medical School, 2018. All Rights Reserved.

Survival correlates with stage

Stage 5-year survival


(Clinical)
Localized 55%

Regional 28%

Distant 4%

Lung cancer screening

• 20% decrease in lung cancer mortality


• Data supports annual low dose CT screening
– Ages 55-80
– > 30 pack years
– Quit within the last 15 years
• Fleischner guidelines for f/u imaging lung nodules
• NCCN guidelines for management of lung nodules on
screening CT

899
Copyright © Harvard Medical School, 2018. All Rights Reserved.

Presenting symptoms
Endocrine/Parac Mechanism Symptoms
rine
Hypercalcemia Multiple Altered mental status, ataxia. Cardiac
Local Symptoms concerns
Cough SIADH ADH/aVP Symptomatic hyponatremia

Hemoptysis Cushing’s ACTH Muscle weakness, hyperglycemia,


syndrome hylokalemia. Infection.
Chest Pain
Dyspnea
Hoarseness Neurologic Mechanism Symptoms

Pleural involvement Lambert-Eaton Anti-voltage gated Fatigue, large muscle


calcium channels weakness
SVC syndrome
Paraneoplastic Anti-Hu Ataxia, diplopia, dysphagia
Pancoast syndrome cerebellar Anti-Yo
degeneration
Myasthenia gravis Anti-Acetylcholine Fatigable weakness of
receptor voluntary muscles

Biopsy/staging
• Tissue:
– More is better
– Genomic anaylsis
– Core preferred to FNA
– Non-bone sites preferred
• Staging
– mediastinoscopy
– Chest CT with contrast
– PET/CT
– Brain MRI with gadolinium (in selected cases)

900
Copyright © Harvard Medical School, 2018. All Rights Reserved.

Treatment dictated by stage

NSCLC SCLC
Treatment Approach
Stage Stage
IA,
Surgical resection
IB (< 4cm)
IB (> 4cm), Surgical resection,
IIA, IIB +/- chemotherapy

IIIA Multimodality approach

IIIB Limited Chemotherapy and Radiation

Chemotherapy, Targeted
IV Extensive
therapy, Immunotherapy

Primary therapy

• Localized disease
– Anatomic resection gold standard
– Steriotactic radiosurgery in selected cases
• Locally advanced disease
– Surgery and radiation: sequential or concurrent

901
Copyright © Harvard Medical School, 2018. All Rights Reserved.

Standard
5 things chemotherapy
to know in met
about …Treating lungDisease
Local cancer
Chemo Targeted
therapies
Rosell R, et al. Lancet Oncol. 2012
Response rate 20-30% 60-80%
Median survival 8-12 months 2-3 years

100

80
Percent Alive

60

40

20

0
0 5 10 15 20 25 30
Months
Schiller et al. N Engl J Med. 2002;346:92.

5 things to know about …Treating Local Disease


Genomics in lung cancer

Johnson BE, et al. ASCO 2013.

902
Copyright © Harvard Medical School, 2018. All Rights Reserved.

Novel approaches in lung cancer

Ribas, NEJM 2012

Genomic Immunohisto- Coming


chemical
staining
Non-small cell, EGFR, ALK, (PD-L1) BRAF, MET,
non-squamous ROS1 RET, NTRK,
HER2
NSCLC, - (PD-L1)
squamous
SCLC - - DLL3

Prostate cancer epidemiology

• 221,000 new cases per year (1 in 7 men)


• 28,000 deaths per year
• Risk factors:
– Age
– Family history
– African American
– BRCA 1/2, Lynch syndrome

903
Copyright © Harvard Medical School, 2018. All Rights Reserved.

The Spectrum of Prostate Cancer


Organ Confined Organ Confined Metastatic Castration Resistant
Low Risk - Risk of Mets Disease Prostate Cancer

Prostate Cancer

Risk of cancer Rising PSA no mets Rising PSA no/min mets

Varied spectrum from low volume, low grade disease that would never
kill the patient to intermediate risk disease to high risk, high volume
localized disease to frank metastatic disease that is incurable.

Treatment decisions require balancing the risk of progression and


morbidity secondary to treatment toxicity with life expectancy.

Prostate cancer screening


PLCO (US)** European Goteborg
Sample size 150,000 162,243 20,000
Median F/u 13 yrs 13 yrs 14 yrs
Prostate cancer No difference 21% reduction 44% reduction,
mortality p=0.001 p=0.0002
NNT* 37 12

American Urologic Association (AUA)


•No PSA screening in men under age 40. No routine screening in men between ages
40 to 54 years at average risk
•Routine screening interval of two years or more may be preferred
•No PSA screening: >age 70; any man with < 10 -15 year life expectancy

American Cancer Society (ACS)


• All men >50 with 10 yr life expectancy
• Discuss age >45 for African Americans and men with family history at < age 65
• Discuss age >40 for those with multiple family members diagnosed young
• Screen yearly for PSA >2.5, every other year for <2.5

904
Copyright © Harvard Medical School, 2018. All Rights Reserved.

Grading and Staging Prostate Cancer


• PSA:
– <10, 10-20, >20
• TNM clinical staging
– Staging of bones and nodes

• Gleason Score (GS):


• Most prominent grade + next most
prominent
- GS: 3 + 3 = 6
- GS: 4 + 3 = 7
- GS: 4 + 5 = 9

Overview: Treatment Options by Stage


Low Risk Intermediate Risk High Risk
GS 6 GS 7 GS 8-10
PSA <10 PSA 10-20 PSA >20
T1 T2 T3-4
Active surveillance Radical prostatectomy Radiation + 2-3 yrs CAB
Radical prostatectomy Radiation + 6 mo. CAB Radical prostatectomy +/-
Radiation: external beam, adjuvant radiation
brachytherapy
Stage: CT/MRI pelvis,
Bone scan

• Life expectancy <10 yrs: consider watchful waiting


• Surgery and radiation prevent PC deaths and increase overall survival in men
with clinically significant tumors and reasonable life expectancy
- No head to head trials: radiation vs. surgery
• Weigh side effect profiles, comorbidities, and risk of overtreatment
• Adding CAB to radiation likely improves outcomes but at expense of toxicity
GS: Gleason Score; CAB: combined androgen blockade (e.g., Lupron + bicalutamide)

905
Copyright © Harvard Medical School, 2018. All Rights Reserved.

Weighing Toxicity

Surgery Radiation ADT


• Surgical complications • Irritative urinary Sx • Hot flashes
- Wound healing • Bowel dysfunction • Fatigue
- Infection • Erectile dysfunction • Weight gain
- Anesthesia risks • Risk of second cancer • Bone density loss
• Erectile dysfunction • Loss of libido
• Urinary Incontinence • Emotional changes
• Metabolic insults:
insulin resistance

• ?Cardiac toxicity

• Surgery: radical prostatectomy (robotic, open, laparoscopic)


• Radiation: external beam, brachytherapy
• ADT: GnRH agonist or antagonist +/- antiandrogen
Sanda NEJM 2008, Resnick NEJM 2013

Metastatic prostate cancer

• Continuous androgen deprivation therapy


with better outcomes than intermittent
• Adding docetaxel improves overall survival

906
Copyright © Harvard Medical School, 2018. All Rights Reserved.

Current Treatment of Castration-resistant


Prostate Cancer

Bone-targeted:
Second line • Radium-223
Androgen blockade: • Antiresorptives
• Abiraterone
• Enzalutamide
• (nilutamide, flutamide,
ketoconazole/HC)
Chemotherapy
• Docetaxel
Immunotherapy: • Cabazitaxel
Sipuleucel-T

• Median overall survival from time of second line agent: 18-35 months
• Clinical trials remain imperative as there are no cures!

Bladder cancer epidemiology


• 81,000 cases per year in the US (3 to 1 M/F)
• Median age in 70’s
• 17,000 deaths per year
• Risk factors
– Age
– Tobacco
– Occupational exposures
– Parasitic infection of bladder
– Cyclophosphamide
– Male
– More common in whites
– Family history
• 90% of cases urothelial or transitional cell
• Rare squamous (schistosomiasis, chronic catheter), adeno, small cell

907
Copyright © Harvard Medical School, 2018. All Rights Reserved.

Presentation

• Hematuria
• UTI type symptoms
• If no infection:
– Refer for cystoscopy
– Urine cytology
– Possible imaging with CT

Staging of Bladder Cancer

• T1 = superficial, non-invasive
• T2 = muscle invasion; path
specimen must have muscularis
propria
• Stage IV encompasses any of
these factors:
- T4b = invasion of
pelvic/abdominal wall
- Involved regional lymph
nodes
- Distant metastasis

Harshman Surg Pathol Clin 2015

908
Copyright © Harvard Medical School, 2018. All Rights Reserved.

Spectrum of Bladder Cancer

Bladder Cancer

Non-muscle Muscle Invasive Metastatic


Invasive (60%) (30%) (10%)

Recur

• TURBT • Radical cystectomy +/- • Chemotherapy


• Intravesical tx neo/adjuvant chemo
if high grade OR
• Chemoradiation

Non-muscle invasive urothelial cancer

• 60% of cases
• Risk of progression to muscle invasive
– High grade, T1a - 48% rate of progression
– Low grade Ta - 2% rate of progression
• Managed by cystoscopy and TURBT
– Transurethral resection of bladder tumor
– BCG instillations weekly x 6 then q 3 months for
“high risk” lesions

909
Copyright © Harvard Medical School, 2018. All Rights Reserved.

Muscle invasive disease


• Organ confined: 5 yr OS: 74%,
• Extravesicular: 5 yr OS: 37%
• Regional nodal disease: 5 yr OS: 31%
• Adjuvant therapy:
• Multiple meta-analyses suggest
survival benefit to cisplatin-based
chemo
• Clinical Practice: consider if T3-T4
or nodes + and cisplatin eligible
• Bladder preservation:
• small lesions without CIS, T2 and
T3
• maximal TURBT
• cisplatin plus RT
• 50% long term control
• 20-30% require cystectomy

Metastatic urothelial cancer

• Cisplatin is the most active drug

• Gemcitabine plus cisplatin most commonly used

• Second line or first line in patients not eligible for


platin – immunotherapy with PD-1/PDL-1 inhibitor

910
Copyright © Harvard Medical School, 2018. All Rights Reserved.

Annual Incidence and Mortality of


Gastrointestinal Malignancies
United States (2017) Worldwide (2012)
Siegel et al CA Cancer J Clin 2017 Globocan2012 WHO

Cancer New Cases Deaths New Cases Death


Esophageal 17,290 15,850 455,784 400,169

Gastric 26,240 10,800 951,594 723,073

Pancreas 55,440 44,30 337,872 330,391

Colorectal 140,250 50,630 1,360,602 693,933

Liver 42,220 30,200 782,451 745,533

Breast 268,670 41,400 1,671,149 521,907

Lung 253,290 158,770 1,824,701 1,589,925

Esophageal cancer
• Risk factors:
– SCC: etoh, tbco, prior RT
– Adeno: Barretts’s, reflux, obesity
• Localized disease
– Surgery alone (5 year survival 15-24%)
– No benefit to post-op chemo or RT
– 2 neo-adjuvant studies with conflicting results
• Metastatic disease
– Survival less than one year
– Palliative RT
– Palliative chemo (platinum based, Her2 directed therapy
if over-expressed)

911
Copyright © Harvard Medical School, 2018. All Rights Reserved.

Gastric cancer
• Risk factors: H.pylori, salted meats, nitrates,
lack of refrigeration, occupation, E cadherin
mutation
• Standard for localized disease is surgery often
plus post-op chemotherapy/RT
• Palliative chemo for metastatic disease
(platins most active)

Pancreatic cancer

• Risk factors: hereditary, DM, obesity, tbco


• Localized disease (15% of patients):
– surgery with adjuvant chemo/RT
– 15% survive 5 years
• Locally advanced:
– Chemotherapy + RT
• Metastatic disease:
– Chemotherapy (FOLFIRINOX, gemcitabine/nab-
paclitaxel)

912
Copyright © Harvard Medical School, 2018. All Rights Reserved.

Colorectal cancer

• Risk factors:
– Increase risk: family hx, IBD, DM, tbco, obesity,
western diet
– Decrease risk: screening, exercise, vit D, ASA,
NSAIDS, calcium
• Hereditary (5% cases):
– FAP
– HNPCC

Colorectal Cancer: Screening


• American Cancer Society recommendations for average risk patients
(updated 2008)
Tests that find polyps and cancer
• Flexible sigmoidoscopy every 5 years *
• Colonoscopy every 10 years
• Double contrast barium enema every 5 years *
• CT colonography (virtual colonoscopy) every 5 years *
Tests that mainly find cancer
• Fecal occult blood test (FOBT) every year *
• Fecal immunochemical test (FIT) every year *
• Stool DNA test (sDNA), interval uncertain *

*Colonoscopy should be done if test results are positive.

Levin et al CA Cancer J Clin 2008; 58:130-160

913
Copyright © Harvard Medical School, 2018. All Rights Reserved.

Stage predicts 5 year survival

Stage I > 90%


Stage II 70-85%
Stage III 30-70%
Stage IV 8-10 %

Therapeutic approach CRC

Stage Colon Rectal


I (T1-T2, N0, M0) Surgery only Surgery only
Surgery +/- Chemoradiation
II (T3-T4, N0, M0)
Chemotherapy Surgery
Chemotherapy
OR
Surgery
III (Tany, N+, M0) Surgery
Chemotherapy
Chemoradiation &
Chemotherapy
Chemotherapy +/- Chemotherapy +/-
IV (Tany, Nany, M1)
Surgery Surgery

National Comprehensive Cancer Network. NCCN Clinical Practice Guidelines in


Oncology: Colon Cancer V3.2012. Available at: www.nccn.org. Accessed April 12,
2012.

914
Copyright © Harvard Medical School, 2018. All Rights Reserved.

CLL
• most common form of leukemia
• mature B-cell lymphoma
– CLL > 5000 circulating tumor cells
– SLL < 5000 circulating cells
• median age at diagnosis is 65
• commonly involves nodes diffusely, spleen, liver
• common complications include autoimmune
hemolytic anemia and hypogammaglobulinemia
• increased risk of secondary malignancies

Risk factors CLL

• cytogenetics
– 13q favorable
– 11q unfavorable
– 17p most unfavorable
• IGVH
• stage

915
Copyright © Harvard Medical School, 2018. All Rights Reserved.

Therapy for CLL


• No benefit to early treatment
• Indications – anemia, thrombocytopenia, bulky LAN or
splenomegaly, Richter’s
• Initial therapy:
– Chemo-immunotherapy: FCR (younger with unmutated
IGVH), BR (not for high risk cytogenetics or unmutated
disease), obinituzumab + chlorambucil, ibrutinib
– Ibrutinib for 17p and selected others (afib, bleeding)
• Relapsed disease:
– Inbrutinib
– Idelalisib + rituximab
– Vanetoclax (17p)

ALL

• Most common childhood cancer


• 15% of adult leukemias
• Prognostic factors:
– Philadelphia chromosome
– Other cytogenetics/molecular profiles
– high white cell count at diagnosis

916
Copyright © Harvard Medical School, 2018. All Rights Reserved.

Treatment of ALL

• Induction/consolidation/CNS
prophylaxis/maintenance
• Pediatric regimens in younger adults
• Ph+ patients with TKI plus steroids
• Transplant in first remission in Ph+ and other
high risk patients
• Relapsed disease
– Novel drugs blinatumomab, antibody drug
conjugates
– Chimeric antigen T cells (treatment related toxicity)

MDS
• heterogeneous diseases or stem cells
• incidence increases with age
• Prior chemo/RT predisposing factor
• Presents wit cytopenias (anemia most common)
• Marrow hypercellular with dysplasia
• Prognosis – IPSS
– % blasts
– Cytogenetics
– Number of cytopenias
• Numerous genetic mutations recently identified
• Can transform to AML

917
Copyright © Harvard Medical School, 2018. All Rights Reserved.

Treatment MDS

• Supportive care

• Growth factors

• Hypomethylating agents (decitabine and


azacytidine)

• Allogeneic stem cell transplant only curative


approach

AML

• 2.4 cases/100,000
• Median age late 60’s-70
• Associated with prior chemo/RT/toxin
• Pre-existing MDS
• Prognosis:
– Age
– Cytogenetics (ie t(15;17)
– Complex chromosomes adverse
– Mutations (ie FLT-3, NPM-1)

918
Copyright © Harvard Medical School, 2018. All Rights Reserved.

Therapy AML

• Induction with anthracycline plus cytarabine


• APML atra plus arsenic or chemotherapy
depending on risk
• Consolidation with chemotherapy or
allogeneic transplant depending on risk

CML

• Myeloproliferative disorder characterized by t(9;22)


• Median age 53
• Presents with leukocytosis and splenomegaly
• Therapy with imatinib or other tyrosine kinase
inhibitors
• Risk for transformation to AML or ALL (blast crisis)

919
Copyright © Harvard Medical School, 2018. All Rights Reserved.

Non-Hodgkin's Lymphoma

• Most common hematologic malignancy


• 75,000 cases/year in the US
• 5th most common cause of cancer deaths
• 2nd fasting growing malignancy in terms of
mortality
• 85% are of B-cell origin

Non-Hodgkin’s lymphoma

Indolent Aggressive Highly


aggressive
Survival Years Months Weeks
untreated
Response to Not curable Curable Curable
chemotherapy
Example Follicular Diffuse large Burkitt
lymphoma B-cell lymphoma
lymphoma

920
Copyright © Harvard Medical School, 2018. All Rights Reserved.

Biopsy
• Supraclavicular > cervical/axillary > inguinal

• Excisional biopsy when possible

• CT guided core needle

• FNA

• Send for pathology, immunohistochemisty/flow


cytometry

Indolent B-cell lymphoma:


clinical management

Localized Advanced Advanced


Low tumor High tumor
burden burden
Involved Field Observation Therapy
RT

921
Copyright © Harvard Medical School, 2018. All Rights Reserved.

Hodgkin lymphoma
• Stage I and II Disease
– Combined modality therapy with chemotherapy and
radiation
– Increasing role for chemotherapy alone
– Approximately 85%-90% cured with initial chemotherapy

• Stage III and IV Disease


– Chemotherapy always required
– Role of radiation therapy to sites of bulky disease
uncertain
– 75% cured with initial therapy depending on risk

HL complications of therapy
• Fertility
– ABVD low risk of infertility
• Pulmonary Toxicity
– Bleomycin
– Mediastinal RT
• Cardiac toxicity
– Mediastinal RT – pericardial, valvular and CAD
– Doxorubicin - cardiomyopathy
• Second malignancies
– solid tumors from radiation - breast, lung, etc

922
Copyright © Harvard Medical School, 2018. All Rights Reserved.

Multiple Myeloma
• 20,000 new cases per year
• Risk factors: African Americans, RT, MGUS
• Mean age 65
• Diagnosis:

Monoclonal plasma cells in bone marrow and/or


presence of biopsy-proven plasmacytoma

Myeloma-related organ dysfunction (1 or more)


(C) serum calcium >10.5 mg/L or ULN
(R) Renal insufficiency (SCr >2 mg/dL)
(A) Anemia (hemoglobin <10 g/dL or 2g <normal)
(B) Lytic bone lesions or osteoporosis

Complications
• Bone disease/hypercalcemia
• Hyperviscosity-IgM, IgG3, IgA
• Recurrent infections
• Renal failure: hypercalcemia, myeloma kidney,
hyperuricemia, IV urography, dehydration, plasma
cell infiltration, pyelonephritis, amyloidosis
• Cardiac failure: amyloid, hyperviscosity, anemia
• Anemia: BM tumors, renal dysfunction,
myelosuppression, low endogenous erythropoietin
• Neuropathy: sensory ±motor, amyloid, anti-myelin
Ab

923
Copyright © Harvard Medical School, 2018. All Rights Reserved.

MM initial therapy
Non-transplant: Transplant:
• dexamethasone/bortezomib/ • 3 drug combinations*:
cyclophosphamide – dexamethasone/
• dexamethasone/bortezomib/ bortezomib
lenalidomide Plus:
• dexamethasone/ – Lenalidomide or
lenalidomide – cyclophosphamide or
– doxorubicin

Bisphosphonates for bony disease

924
Copyright © Harvard Medical School, 2018. All Rights Reserved.

Oncology Board Review


Ann S. LaCasce, MD, MMSc
Institute Physician
Dana Farber Cancer Institute
Associate Professor of Medicine
Harvard Medical School

Question 1
63 year old woman has completed 5 years of tamoxifen following
lumpectomy and RT for a 1.5 cm, ER +, node negative breast
cancer. She presents to your office with severe, localized back
pain. Physical examination is normal including the neurologic
exam. The alkaline phosphatase is 330 (elevated) and the CA27.29
is 156 (elevated). A bone scan is positive in several areas of the
thoracic and lumbar spine, as well as in several ribs. The course
of action at this point should be:

A. Combination chemotherapy

B. Tamoxifen therapy

C. MRI scan of the spine

D. Radiation therapy to areas of localized disease

E. Stem cell transplantation

925
Copyright © Harvard Medical School, 2018. All Rights Reserved.

Question 1 - Answers
A. Combination chemotherapy – might be appropriate at some time
but not initially

B. Tamoxifen therapy – might be appropriate some time but not


initially

C. MRI scan of the spine – you must rule out or in spinal cord
compression – and outcome is best if it is discovered prior to
the onset of neurologic findings

D. Radiation therapy to areas of localized disease – the might be


appropriate at some time, and would be for spinal cord
compression if that is present on MRI

E. Stem cell transplantation – unfortunately doesn’t work for patients


with metastatic breast cancer

Question 2
68 year old male presents with back pain, anemia and fevers. The
patient has no lymphadenopathy or splenomegaly. Laboratory
studies are notable for HCT of 34% total protein of 9.8 gm/dl,
creatinine of 3.2 mg/dl, and calcium of 12.3 mg/dl. Plain x-rays of
the spine show generalized osteoporosis, without focal defects.
Which of the following is true:

A. Fever is a worrisome sign and infection is a life threatening risk in


this disease

B. Renal failure is likely a consequence of dehydration

C. The patient should undergo CT scans with contrast to evaluate


source of fever

D. The lack of lytic lesion argues against an underlying diagnosis of


myeloma

E. IgA and IgG paraproteins have similar serum viscosities

926
Copyright © Harvard Medical School, 2018. All Rights Reserved.

Question 2 - Answers
A. Fever is a worrisome sign and infection is a life threatening
risk – for myeloma patients and infection is the most common
cause of death

B. Renal failure is a common complication of myeloma due to


myeloma kidney, hypercalcemia and can be acutely and
irreversibly worsened by dehydration.

C. Patients with myeloma should not receive IV contrast even in the


setting of apparently normal renal function

D. The most common bone lesion seen in patients with myeloma is


generalized osteoporosis, not lytic bone lesions

E. IgA and IgG paraproteins have different serum viscosities – IgA


paraproteins have an unpredictable but higher viscosity than IgG
paraproteins.

Question 3
46 year old woman presents to your office for routine health care.
She is concerned about the possibility of developing breast
cancer, and asks you about her risk factors. Which statement is
correct:
A. A previous history of LCIS does not substantially increase
her risk of developing breast cancer

B. Presence of a BRCA-1 germ line mutation will substantially


increase her risk of developing breast cancer

C. A maternal aunt with post-menopausal breast cancer will


substantially increase her risk of developing breast cancer

D. The majority of women with breast cancer have identifiable


risk factors for the development of breast cancer

E. Duration and degree of estrogen (endogenous and


exogenous) exposure is not associated with increased of
developing breast cancer

927
Copyright © Harvard Medical School, 2018. All Rights Reserved.

Question 3 - Answers
A. A previous bx which revealed LCIS will substantially increase her risk of
developing breast cancer – LCIS substantially increases the likelihood of
future breast cancer, with an incidence of about 1% per year.

B. BRCA 1 and 2 mutations are associated with an increased risk of


breast cancer

C. A maternal aunt with post-menopausal breast cancer will substantially


increase your risk of developing breast cancer – it is first degree relatives
and relatives who develop breast cancer an a younger age who confer
the greatest risk on family members

D. 80-85% of patients have no identifiable risk factors other than being a


woman

E. Duration and degree of estrogen (endogenous and exogenous) exposure


will increase your risk of developing breast cancer – early menarche,
HRT, etc

Question 4
67 year old male smoker presents headaches, forgetfulness, and
poor coordination. Several times over the past few weeks he has
had periods of confusion and urinary incontinence. Head CT scan
reveals multiple round enhancing lesions. Chest x-ray shows a 2
cm lesion in the right mid lung field. The most likely diagnosis is:

A. Prostate cancer metastatic to lung and brain

B. Pneumonia with brain abscesses

C. Colon cancer with lung and brain metastases

D. Adenocarcinoma of the lung with brain metastases

E. Gastric cancer with lung and brain metastases

928
Copyright © Harvard Medical School, 2018. All Rights Reserved.

Question 4 - Answers
A. Prostate cancer commonly metastasizes to bone, and
rarely to lung and brain

B. Pneumonia with brain abscesses – possible, rare, and


this is an oncology review session

C. Brain metastases are an uncommon complication of


colon cancer

D. Brain metastases are very common in patients with


adenocarcinoma of the lung, at presentation and
later in the course of illness

E. Brain metastases are less common with gastric cancer

Question 5
26 year old woman with Hodgkin lymphoma and bulky
mediastinal disease is treated with ABVD (doxorubicin,
bleomycin, vinblastine, dacarbazine) and radiation to the
mediastinum. Which of the following is most true:

A. She is more likely to die of causes other then HL

B. Given she is over 20, she is not at increased risk of breast


cancer

C. She has an increased risk of leukemia

D. She is likely to have difficulty with fertility in the future

E. She is not at increased risk for heart disease

929
Copyright © Harvard Medical School, 2018. All Rights Reserved.

Question 5 - Answers
A. HD is highly curable but there is increased mortality from
other diseases and she is more likely to die of other
causes, including heart disease and second cancers

B. She has an increased risk of breast cancer – women under 30


who receive chest radiation have an increased risk of breast
cancer

C. ABVD does not increase the risk of leukemia as MOPP did

D. She is likely to remain fertile without increased risk of children


with birth defects – ABVD and chest radiation do not impair
fertility or decrease the likelihood of having a normal child

E. She is at increased risk for heart disease – she is at increased


risk for cardiomyopathy from doxorubicin and coronary artery
disease from radiation

Question 6
46 year old Asian woman, who never smoked, is diagnosed
with stage IV non-small cell lung cancer, metastatic to
liver and bone. Which of the following is correct:

A. She is potentially curable with intensive modern


chemotherapy

B. The likelihood of responding to an EGFR kinase inhibitor


is related to the presence of a mutation in the kinase
region

C. The likelihood of having a mutation of the kinase region


of EGFR is random and not related to gender or ethnic
background

D. Cytotoxic chemotherapy is the only potentially beneficial


treatment

E. Resistance to kinase inhibitors is rare

930
Copyright © Harvard Medical School, 2018. All Rights Reserved.

Question 6 - Answers
A. She is incurable and modern therapy has not changed this
fact

B. The likelihood of responding to an EGFR kinase inhibitor


is related to the presence of a mutation in the kinase
region

C. The likelihood of having a mutation of the kinase region of


EGFR is related to being a woman, a non-smoker and Asian

D. If her tumor has a EGFR kinase mutation she has a good


chance of responding to an inhibitor

E. Development of resistance to EGFR kinase inhibitors is


related to further acquired mutations in the tumor

Question 7
22 year old man presents with left a supraclavicular mass, and an
otherwise normal physical examination. Chest x-ray shows a
widened mediastinum. Aspiration cytology of the supraclavicular
mass demonstrates undifferentiated carcinoma. The next clinical
action should be:

A. Institution of multi-agent chemotherapy

B. MRI scan of the chest

C. Mediastinoscopy and biopsy of the para-tracheal nodes

D. Testicular ultrasound

E. Institution of radiation therapy to the mediastinum and


supraclavicular areas

931
Copyright © Harvard Medical School, 2018. All Rights Reserved.

Question 7 - Answers
A. Institution of multi-agent chemotherapy – always better to
know what you are treating and to identify potentially curable
diseases

B. MRI scan of the chest – will not add helpful information

C. Mediastinoscopy and biopsy of the para-tracheal nodes – will


not add helpful information since you already have pathology

D. Testicular ultrasound – this is correct – you can see


occult testicular cancers with this pattern of spread and
they are highly curable and relatively common in this age
group – so the burden of proof is on you to find curable
diseases

E. Institution of radiation therapy to the mediastinum and


supraclavicular areas – better to know what you are treating –
this would be clearly palliative

Question 8
Which of the following is true about the epidemiology of
lung cancer:

A. Adenocarcinoma has become the most common histologic


subtype of lung cancer.

B. Women who smoke, develop lung cancer with a similar


incidence and at a similar age as men who smoke.

C. Asbestos does not add to the risk of developing lung cancer


in smokers

D. Cigarette filters reduce the carcinogenic effect of cigarettes.

E. 90% of patients diagnosed with stage I non-small cell lung


cancer will survive their cancer.

932
Copyright © Harvard Medical School, 2018. All Rights Reserved.

Question 8 - Answers
A. Adenocarcinoma has become the most common histologic
subtype of lung cancer

B. Women who smoke, develop lung cancer at an earlier age than


men who smoke

C. Asbestos and smoking are additive risk factors for lung cancer

D. Cigarette filters do not reduce the carcinogenic effect of


cigarettes – and they apparently increase the risk of
adenocarcinomas in the lung periphery.

E. Only slightly more than half the patients diagnosed with stage I
non-small cell lung cancer will survive their cancer – even
apparently localized lung cancer has a high recurrence rate and
mortality.

Question 9
28 year old man is admitted to the hospital with newly
diagnosed acute lymphoblastic leukemia. Which of
the following clinical characteristics would convey the
worst prognosis:

A. Peripheral blood blast count of 200,000/mm3

B. T-cell phenotype

C. Mediastinal mass

D. Philadelphia chromosome t(9;22)

E. Thrombocytopenia

933
Copyright © Harvard Medical School, 2018. All Rights Reserved.

Question 9 - Answers
A. A high peripheral blood blast count does not decrease
chance of cure

B. T-cell phenotype – with modern therapy this is a very


treatable and potentially curable disease

C. Mediastinal mass – most of these patients have T-cell


phenotype and are potentially curable as above

D. Philadelphia chromosome t(9;22) – this is a poor


prognostic finding requiring more intensive
therapy

E. Thrombocytopenia – does not predict outcome

Question 10
51 year old man underwent surgical resection for rectal cancer. On
pathology evaluation the tumor penetrates the serosa of the bowel
and one regional lymph node shows involvement with metastatic
carcinoma. There is no evidence of distant metastases. Optimal
therapy should include:

A. No post-operative therapy

B. Re-resection of pelvic tissue surrounding the area of the


original tumor

C. Radiation therapy to the pelvis

D. Systemic chemotherapy

E. Both radiation to the pelvis, and chemotherapy

934
Copyright © Harvard Medical School, 2018. All Rights Reserved.

Question 10 - Answers
A. Post-operative therapy is indicated as in E

B. Maximal tissue is usually resected during primary


surgery but because of the tight area and surrounding
structures this is limited

C. Radiation therapy to the pelvis will decrease local but


not systemic recurrence

D. Systemic chemotherapy will reduce systemic


recurrence but not local recurrence

E. Both radiation to the pelvis, and chemotherapy –


will reduce both local and systemic recurrence

Question 11
46 year old woman with epithelial ovarian cancer at surgical
debulking is found to have billateral ovarian masses with multiple
peritoneal and omental nodules, as well as ascitic fluid. All tumor
that can be removed is removed, but tumor masses of 2-3 cm
remain. There is no evidence of disease outside of the peritoneal
cavity. Postoperatively, the patient is treated with paclitaxel and
carboplatin for 6 cycles. Which of the following is true:

A. A very low chance of response to chemotherapy and a very low


chance of cure.

B. A high chance of complete clinical response, but a low chance for


cure.

C. A high chance of response and a high chance for cure.

D. The need for radiation therapy delivered to the whole abdomen.

E. The need for localized radiation therapy to the pelvis.

935
Copyright © Harvard Medical School, 2018. All Rights Reserved.

Question 11 - Answers
A. Ovarian cancer is a tumor that responds to many
chemotherapy agents but has a high recurrence and
death rate

B. A high chance of complete clinical response, but a


low chance for cure – as above

C. As in “B”

D. Whole abdominal radiation is very toxic, limits ability to


give chemotherapy, and does not improve outcome

E. Localized radiation therapy to the pelvis limits ability to


give chemotherapy and does not improve outcome

Question 12
A 72 year old man presents with hematuria. Cystoscopy reveals
multiple bladder nodules which are biopsied and reveal
transitional cell carcinoma. The likelihood of developing
metastatic bladder cancer is most closely related to:

A. The size of the tumors in the bladder.

B. The number of tumors in the bladder.

C. History of smoking.

D. Family history.

E. Bladder wall muscle invasion by the tumor.

936
Copyright © Harvard Medical School, 2018. All Rights Reserved.

Question 12 - Answers
A. Tumors without muscle invasion can be large and not
metastasize.

B. The number of tumors in the bladder can be a “pain” to remove


and treat, but does not affect outcome.

C. History of smoking – this is a risk factor for developing bladder


cancer but does not determine likelihood of developing
metastases.

D. Family history – weak risk factor for developing bladder cancer but
does not affect risk of metastases.

E. Bladder wall muscle invasion by the tumor – this is an


important risk factor for the development of metastases.

Question 13

A 32 year old man presents with acute myelogenous leukemia.


Allogeneic bone marrow transplantation will most likely be
recommended if a suitable donor can be found if:

A. His initial blast count > 100,000/mm3

B. He is septic at presentation

C. He has M3/Acute promyelocytic leukemia and has DIC

D. His leukemic blasts have a 7q- chromosomal deletion

E. His leukemic blasts have a t(8;21) chromosomal translocation

937
Copyright © Harvard Medical School, 2018. All Rights Reserved.

Question 13 - Answers
A. His initial blast count of > 100,000/mm3 does not affect outcome

B. Being septic at presentation affects initial mortality but not ultimate


response to chemotherapy

C. M3/Acute promyelocytic leukemia with DIC affects initial mortality


but not ultimate response to chemotherapy and in fact thse
patients ultimately have a better prognosis than average

D. Leukemic blasts wth a 7q- chromosomal deletion portend a


poor prognosis with standard chemotherapy

E. Patients whose leukemic blasts have a t(8;21) chromosomal


translocation have a better prognosis than average

Question 14
You are evaluating a 52 year old man with newly
diagnosed non-small-cell carcinoma of the right lung.
In which of the following scenarios is the patient
resectable?

A. Contralateral (N3) mediastinal adenopathy.

B. Enlarged (4 cm) left adrenal gland.

C. Ipsilateral pleural effusion.

D. Ipsilateral (N2) mediastinal adenopathy.

E. Enlarged supraclavicular lymph nodes.

938
Copyright © Harvard Medical School, 2018. All Rights Reserved.

Question 14 - Answers

A. Patients with contralateral (N3) mediastinal adenopathy are not


resectable.

B. 25% of pts with NSCLC will have adrenal metastases early on,
and these pts are not helped by resection – and an adrenal mass
of this size is more likely malignant than representing a benign
adenoma.

C. Pleural effusions make resection unsuccessful.

D. Patients with ipsilateral (N2) mediastinal adenopathy are


potentially resectable, and curable, particularly after
preoperative chemotherapy and radiation.

E. Enlarged supraclavicular lymph nodes are indicative of metastatic


disease, and these patients are not helped by resection.

Question 15

Which of the following is true regarding tamoxifen and


raloxifene?

A. Raloxifene is a bone strengthening agent but


tamoxifen is not

B. Both increase the risk of endometrial cancer.

C. Both decrease the risk of developing a future breast


cancer

D. Tamoxifen increases risk of hot flashes, but


raloxifene doesn’t

939
Copyright © Harvard Medical School, 2018. All Rights Reserved.

Question 15 - Answers
A. Both increased bone density

B. Tamoxifen increases the risk of endometrial


cancer, but raloxifene does not

C. Both decrease the risk developing breast


cancer

D. Both increase the likelihood of hot flashes

Question 16
42 year old woman presents with a 3 cm poorly differentiated
breast, ER/PR negative, HER2 positive cancer with 5 involved
axillary lymph nodes. Which of the following is true:

A. The presence of HER2 on breast cancer cells does not affect


prognosis

B. Adjuvant chemotherapy is not effective in reducing the


subsequently development of metastatic breast cancer

C. Trastuzumab, when added to chemotherapy, substantially


reduces the risk of developing metastatic disease in the
future.

D. Letrozole, an aromatase inhibitor, would further improve the


cure rate for this patient.

E. The addition of trastuzumab to chemotherapy is safe, without


short and long-term complications.

940
Copyright © Harvard Medical School, 2018. All Rights Reserved.

Question 16 - Answers
A. Tumors which overexpress HER2 are more likely to metastasize

B. Adjuvant chemotherapy substantially reduces the risk of her


subsequently developing metastatic breast cancer for both HER2
positive and negative tumors.

C. Trastuzumab, when added to chemotherapy, further reduces


the risk of developing metastatic disease in the future

D. Letrozole, an aromatase inhibitor, is not effective for pts with


estrogen receptor negative tumors

E. Trastuzumab, when administered after doxorubicin, is associated


with increased risk of cardiomyopathy

Question 17
56 year old man with a history of a primary melanoma
on the right forearm 3 years ago, presents with
hepatomegaly and is found to have metastatic
melanoma in the liver. Which of the following is the
best treatment option:

A. Dacarbazine and tamoxifen

B. No therapy is effective or warranted and the patient


should be place in hospice care

C. Nivolumab plus ipilimumab.

D. Prophylactic brain irradiation

941
Copyright © Harvard Medical School, 2018. All Rights Reserved.

Question 17 - Answers
A. Dacarbazine and tamoxifen was a favored treatment in the
past, but tamoxifen was shown to be ineffective, and the
benefit from dacarbazine is marginal

B. Metastatic melanoma is a bad disease, and in the past


placement on hospice as initial treatment could be considered
appropriate

C. Nivolumab plus ipilimumab has been shown to be highly


active in this disaease

D. Patients with metastatic melanoma do frequently develop


brain metastases but prophylactic brain radiation has not
been shown to improve outcome

Question 18
64 yo woman has a routine CBC showing a WBC of 14,500/mm3
with 75% mature-appearing lymphocytes, Hct 41%, and Plt of
180,000/mm3. She has no adenopathy or splenomegaly and feels
well. Which of the following is true:

A. The diagnosis of CLL can only be made on a bone marrow


aspirate and biopsy

B. She is at increased risk for infection

C. She is in need of urgent chemotherapy

D. She is likely to die of CLL before her 70th birthday

E. Splenomegaly is rare in patients such as this one

942
Copyright © Harvard Medical School, 2018. All Rights Reserved.

Question 18 - Answers
A. The diagnosis of CLL can be made on testing of the
peripheral blood, by flow cytometry, looking for the co-
expression of CD20 and CD5

B. She is at increased risk for infection, from decreased


antibody production in response to infection, and infection
is the leading cause of death for patients with CLL

C. Many people live with CLL without therapy for years and
reasons to treat include severe anemia, thrombocytopenia,
bulky adenopathy, or systemic symptoms and she has none of
these

D. Statistically she is likely to be alive at age 75

E. Splenomegaly is a common finding in patients with CLL

Questions 19-23 - Answers


A. Bevacizumab
B. Cyclophosphamide
C. Paclitaxel
D. Trastuzumab
E. Ipilimumab

19. Stabilization of microtubule assembly


20. Inhibition of HER2
21. Inhibition of VEGF
22. Alkylation of DNA
23. Increase T cell response versus tumor

943
Copyright © Harvard Medical School, 2018. All Rights Reserved.

Questions 19-23 - Answers


A. Bevacizumab - 21
B. Cyclophosphamide - 22
C. Paclitaxel - 19
D. Trastuzumab - 20
E. Ipilimumab- 23

19. Stabilization of microtubule assembly


20. Inhibition of HER2
21. Inhibition of VEGF
22. Alkylation of DNA
23. Increase T cell response versus tumor

Questions 24-28 - Answers


A. Bevacizumab
B. Cyclophosphamide
C. Paclitaxel
D. Trastuzumab
E. Ipiliumumab
24. Congestive heart failure
25. Hypertension
26. Colitis, pneumonitis, hypophysitis
27. Acute hypotensive and bronchospasm
28. Hematuria

944
Copyright © Harvard Medical School, 2018. All Rights Reserved.

Questions 24-28 - Answers


A. Bevacizumab - 25
B. Cyclophosphamide - 28
C. Paclitaxel - 27
D. Trastuzumab - 24
E. Ipiliumumab- 26
24. Congestive heart failure
25. Hypertension
26. Colitis, pneumonitis, hypophysitis
27. Acute hypotensive and bronchospasm
28. Hematuria

Oncology Board Review

Ann S. LaCasce, MD

Dana-Farber Cancer Institute


Brigham and Women’s Hospital

• Disclosures:
– No relevant conflicts of interest to declare

945
Copyright © Harvard Medical School, 2018. All Rights Reserved.

Cardiology Overview
Leonard S. Lilly, M.D.
Professor of Medicine
Harvard Medical School
Chief, Brigham/Faulkner Cardiology
Brigham and Women’s Hospital

BRIGHAM AND HARVARD


WOMEN’S HOSPITAL MEDICAL SCHOOL

Faculty Disclosure/Conflicts

None

946
Copyright © Harvard Medical School, 2018. All Rights Reserved.

Recent Cardiology Highlights


• 2017 Hypertension Guidelines: new treatment
strategies and goals
• PCSK9 inhibitor reduces cardiovascular events
in 2nd major outcomes trial (ODYSSEY Outcomes)
• PCI no better than sham procedure for
improving symptoms or exercise capacity in
single vessel CAD and stable angina (ORBITA)
• Anti-inflammatory therapy with canakinumab
(IL-1β inhibitor) reduced cardiac events in patients
with prior MI and hsCRP > 2 mg/L (CANTOS)
• FDA approves new DOAC reversal agent

2017 Hypertension Guidelines

947
Copyright © Harvard Medical School, 2018. All Rights Reserved.

• 56 year old black businessman presents to


office for annual evaluation.
• Blood pressure is 138/84, similar to other
recent office visits and at home.
• He doesn’t smoke and is not diabetic. There is
a strong family history of hypertension. He
consumes 3-4 alcoholic beverages per week.
BMI is 28.
• Medication: Atorvastatin 10 mg daily.
• Predicted 10-year risk for first cardiovascular
event is 12% using ACC/AHA pooled cohort
equations.

Which recommendation is consistent with


current hypertension guidelines?
A. His blood pressure is not in a range that
requires intervention; continue routine
monitoring of BP
B. Recommend life style modifications, including
weight loss, but antihypertensive drug therapy
should not yet be initiated
C. He should follow life style modifications and
begin drug therapy using a beta-blocker or an
ACE inhibitor
D. His target on-therapy blood pressure should be
< 130/80

948
Copyright © Harvard Medical School, 2018. All Rights Reserved.

2017 Hypertension Guidelines


Category Systolic Pressure Diastolic Pressure
(mmHg) (mmHg)
Normal < 120 and <80
Elevated 120-129 and <80
Stage 1 Hypertension 130-139 or 80-89

Stage 2 Hypertension ≥140 or ≥90

Class I recommendation in primary prevention:


Initiate BP-lowering meds for:
• SBP ≥140 mm Hg or DBP ≥90 mm Hg
• SBP ≥130 mm Hg or DBP ≥80, if 10-year ASCVD risk ≥10%

On-therapy BP target: < 130/80


Whelton PK, et al. 2017 Guideline for the prevention,
detection, evaluation, and management of high blood
pressure in adults. J Am Coll Cardiol 2018; 71:2199.

First line drug therapies for Stage 1 hypertension:


• Diuretics (thiazide or chlorthalidone)
• Calcium channel blockers
• ACE inhibitors
• Angiotensin receptor blockers
In patients with CAD or HFrEF:
• Beta-blockers

For black patients, without HFrEF or chronic kidney


disease, initial therapy should include:
• Diuretic (thiazide or chlorthalidone) and/or
• Calcium channel blocker

Whelton PK, et al. 2017 Guideline for the prevention,


detection, evaluation, and management of high blood
pressure in adults. J Am Coll Cardiol 2018; 71:2199.

949
Copyright © Harvard Medical School, 2018. All Rights Reserved.

DOAC Reversal Agents

• 62 year old woman with chronic


nonvalvular atrial fibrillation takes
dabigatran 150 mg twice daily.
• 30 minutes ago she sustained trauma as
passenger in a motor vehicle collision.
• In the Emergency Department she is
intubated, unresponsive and hypotensive.
CT imaging shows small intracranial
subdural hematoma and retroperitoneal
hemorrhage that requires urgent
exploratory laparotomy.

950
Copyright © Harvard Medical School, 2018. All Rights Reserved.

Which is the most appropriate


anticoagulation reversal agent for this
patient on dabigatran with life-threatening
bleeding in need of emergency surgery?

A. Vitamin K
B. Protamine
C. Idarucizumab
D. Andexanet alfa

Direct Oral Anticoagulants (DOACs)


Compared to warfarin:
• At least as effective in nonvalvular AF
and venous thromboembolism, with
fewer bleeding complications
• Predictable anticoagulation
• Do not require routine blood test
monitoring / more convenient
• Shorter half-lives – usually sufficient to
simply withhold drug for minor bleeding
• Vitamin K is ineffective for reversal

951
Copyright © Harvard Medical School, 2018. All Rights Reserved.

DOAC Emergency Reversal of Anticoagulation


Rapid Reversal Agent Alternatives
Direct Thrombin Inhibitor
• 4 Factor prothrombin
Idarucizumab 1
• Dabigatran complex concentrate
(Fab monoclonal antibody fragment
directed at dabigatran) • Hemodialysis

Factor Xa Inhibitors
• Apixaban Andexanet alfa 2 • 4 Factor prothrombin
• Rivaroxaban (Recombinant factor Xa complex concentrate
decoy protein)
• Edoxaban

1N Eng J Med 2017;377:431.


2N Eng J Med 2016;375:1131.

Heart Failure

952
Copyright © Harvard Medical School, 2018. All Rights Reserved.

Switching to sacubitril/valsartan in place of an ACE


inhibitor or ARB would be most appropriate for which of
the following heart failure patients?

A. 72 y.o. man hospitalized for heart failure decompensation,


LVEF 35% and severe (NYHA class IV) symptoms despite
ACE inhibitor, beta-blocker and spironolactone
B. 64-year-old man with nonischemic cardiomyopathy, LVEF
38%, NYHA class II symptoms, comes for routine office
visit on torsemide and optimal doses of metoprolol
succinate and lisinopril
C. 62-year-old woman with ischemic cardiomyopathy, LVEF
40%, NYHA class II symptoms who developed
angioedema previously on an ACE inhibitor
D. 76-year-old woman with heart failure and preserved
ejection fraction (HFpEF), LVEF 50%, NYHA class III
symptoms, taking metoprolol, valsartan, and furosemide

AHA/ACC 2017 Heart Failure Guidelines1


Class I indication:
For patients with chronic HFrEF with
class II-III symptoms who tolerate an
ACE inhibitor or ARB, replacement with
an angiotensin receptor neprilysin
inhibitor (e.g., sacubitril/valsartan) is
recommended to further reduce
morbidity and mortality

1J Am Coll Cardiol 2017;70:776.

953
Copyright © Harvard Medical School, 2018. All Rights Reserved.

PARADIGM-HF: Cardiovascular Death or Heart Failure


Hospitalization (Primary Endpoint)
40

1117
32 Enalapril
Cumulative Rates (%)

(n=4212) 914

24
Sacubitril / Valsartan
(n=4187)
16
HR = 0.80 (0.73-0.87)
P = 0.0000002
8 Number needed to treat = 21

0
0 180 360 540 720 900 1080 1260
Days After Randomization
Stopped early (27 m)
because of marked benefit
N Eng J Med 2014;371:993.

Candidates for Sacubitril/valsartan


Chronic symptomatic HFeF (EF≤ 40%)
Not hospitalized with acute decompensated failure

NYHA Class I IV
II-III

Tolerating
ACEi/ARB

Safety Screen
K ≤ 5.4 mmol/L
eGFR ≥ 30 mL/min/m2
SBP ≥ 95 mm Hg

Yes No

Candidate for
Valsartan/sacubitril
Contraindications:
• History of angioedema on
ACEI
• Pregnancy

J Am Coll Cardiol 70:776-803, 2017.

954
Copyright © Harvard Medical School, 2018. All Rights Reserved.

ECG Interpretation

52 year old malpractice attorney presents to


the Emergency Department with left-sided
chest pain since awakening. Pain is less
intense sitting upright.
• No history of angina
• Father sustained MI at age 56
• Had URI 3 weeks ago

An electrocardiogram is obtained
immediately...

955
Copyright © Harvard Medical School, 2018. All Rights Reserved.

What is most appropriate immediate action?


A. Send directly to cardiac cath lab, if can achieve
door-to-coronary intervention time < 90 min
B. Administer fibrinolytic therapy if < 6 hours since
onset of pain
C. Administer calcium channel blocker and nitrates
D. Administer aspirin and obtain echocardiogram

ST Which ST-T PR
Coving Leads? Evolution Segment

Pericarditis Diffuse Days- ↓


weeks
(STE < 5 mm)

Acute Localized Hours Normal


STEMI

956
Copyright © Harvard Medical School, 2018. All Rights Reserved.

Management of Idiopathic Acute


Pericarditis

• Aspirin (e.g., 650 mg q 4-6 h)


• NSAIAs (e.g., ibuprofen 400-800 mg q 6-8 h)
• Colchicine (off-label: 0.5 mg BID;
0.5 mg daily if weight ≤70 kg)
ICAP Trial. N Engl J Med 2013; 369:1522.
CORP Trial. Ann Int Med 2011;155:409.

• Prednisone 0.25-0.5 mg/kg/d (last resort)

What is most appropriate immediate action?


1. Send directly to cardiac cath lab, if can achieve
door-to-coronary intervention time < 90 min
2. Administer fibrinolytic therapy if < 6 hours since
onset of pain
3. Administer calcium channel blocker and nitrates
4. Administer aspirin and obtain echocardiogram

957
Copyright © Harvard Medical School, 2018. All Rights Reserved.

What is most appropriate immediate action?


1. Send directly to cardiac cath lab, if can achieve
door-to-coronary intervention time < 90 min
2. Administer fibrinolytic therapy if < 6 hours since
onset of pain
3. Administer calcium channel blocker and nitrates
4. Administer aspirin and obtain echocardiogram

STEMI – Initial Management

Invasive Strategy Preferred Fibrinolysis Acceptable


• Skilled PCI lab available • Non-PCI hospital, and delay
• Can achieve PCI within to transfer for PCI > 120 min
90 min of medical contact • No contraindications to
• STEMI with cardiogenic fibrinolysis
shock, regardless of timing • Should be administered
< 30 min after arrival

N Engl J Med 2017; 376:2053.

958
Copyright © Harvard Medical School, 2018. All Rights Reserved.

Endocarditis Prophylaxis

36 y.o. woman is scheduled for a tooth


extraction. She has a history of a heart murmur
and her dentist calls to determine if
premedication is needed. For which one of the
following conditions is antibiotic prophylaxis
recommended?
A. Atrial septal defect with surgical repair 8 years ago
B. Hypertrophic cardiomyopathy with outflow tract
obstruction
C. Bicuspid aortic valve with severe aortic regurgitation
D. History of MVP, s/p mitral valve repair (annuloplasty
ring), with no residual regurgitation

959
Copyright © Harvard Medical School, 2018. All Rights Reserved.

Antibiotic Prophylaxis in Endocarditis


American Heart Association Guidelines
“recommended only for patients with underlying cardiac
conditions associated with the highest risk of adverse outcome”

1. Prosthetic heart valve (including TAVR) or valve


repair with prosthetic material
2. Prior occurrence of endocarditis
3. Certain congenital heart diseases:
• Unrepaired cyanotic lesions (e.g., Tetralogy of Fallot)
• Prior repair with residual defects adjacent to prosthetic material
• Complete repair with prosthetic material, for first 6 months
4. Valve abnormality after heart transplant

J Am Coll Cardiol 2017; 70:252.


Circulation 2007; 116:1736-1754.

Antibiotic Prophylaxis in Endocarditis


American Heart Association 2007 Guideline
• Recommended for invasive dental procedures
• Not recommended for upper respiratory tract
procedures, unless involves incision or biopsy of
mucosa (e.g., tonsillectomy, bronchoscopy with biopsy)
• Not recommended for GU or GI procedures in
absence of infection
Standard regimen: Amoxacillin 2 g 30-60 min prior
If allergy: Clindamycin, azithromycin, clarithromycin, cephalexin
If parenteral required: Ampicillin, cefazolin, ceftriaxone, clindamycin

Circulation 2007;116:1736-1754.

960
Copyright © Harvard Medical School, 2018. All Rights Reserved.

Statins and Muscle Injury

A 62 year old woman with history of


hypertension, diabetes and dyslipidemia takes
simvastatin 40 mg daily.
Which one of the following medications would
not be expected to increase the risk of muscle
toxicity if added to her regimen?

A. Gemfibrozil
B. Ketoconazole
C. Azithromycin
D. Diltiazem
E. Amiodarone

961
Copyright © Harvard Medical School, 2018. All Rights Reserved.

Statin Muscle Side Effects


CK Incidence
Myalgia Normal 1-20%
Myositis Elevated < 0.5%
Rhabdomyolysis >10-40 x < 0.01%
ULN
Related to:
• Drug metabolism
• Dose of drug
• Concurrent medications

Commonly Used Statins


Metabolism

Atorvastatin CYP3A4
Lovastatin CYP3A4
Simvastatin CYP3A4
Fluvastatin CYP2C9
Rosuvastatin CYP2C9
Pitavastatin CYP2C9

Pravastatin Neither

962
Copyright © Harvard Medical School, 2018. All Rights Reserved.

Increased Risk of Myopathy


Potent inhibitors of CYP3A4:
⋅ cyclosporine ⋅ ketoconazole
⋅ erythromycin ⋅ protease inhibitors
⋅ clarithromycin ⋅ grapefruit juice

Moderate inhibitors of CYP3A4:


⋅ amiodarone
⋅ verapamil
⋅ diltiazem

Gemfibrozil

Additional Reading
1. Whelton PK, Carey RM, Aronow WS, et al. 2017 ACC/AHA/AAPA/
ABC/ACPM/AGS/APhA/ASH/ASPC/NMA/PCNA Guideline for the prevention,
detection, evaluation, and management of high blood pressure in adults: Executive
summary. J Am Coll Cardiol 2018; 71:2199.
2. Tomaselli GF, Mahaffey KW, Cuker A, et al. 2017 ACC expert consensus decision
pathway on management of bleeding in patients on oral anticoagulants. J Am Coll
Cardiol 2017; 70:3042.
3. Yancy CW, Jessup M, Bozkurt B, et al. 2017 ACC/AHA/HFSA focused update of
the 2013 ACCF/AHA guideline for the management of heart failure. J Am Coll
Cardiol 2017;70:776.
4. 2015 ESC Guidelines for the diagnosis and management of pericardial diseases. Eur
Heart J 2015; 6:2921.
5. Anderson JL and Morrow DA. Acute myocardial infarction. N Engl J Med 2017;
376:2053.
6. 2017 AHA/ACC focused update of the 2014 AHA/ACC guideline for the
management of patients with valvular heart disease. J Am Coll Cardiol 2017;
70:252.

963
Copyright © Harvard Medical School, 2018. All Rights Reserved.

CV Prevention
Samia Mora, MD, MHS

Associate Physician
Divisions of Preventive and Cardiovascular Medicine
Department of Medicine
Brigham and Women’s Hospital
Associate Professor, Harvard Medical School
Name)

Disclosures
• Dr. Mora has received institutional research support
from Atherotech Diagnostics. NIDDK, FDA, Roche,
Abbott

964
Copyright © Harvard Medical School, 2018. All Rights Reserved.

Objectives
1. To review current challenges for CVD prevention
2. To review (briefly) recent evidence / guidelines on:
• CV risk assessment
• Cholesterol
• Blood pressure
• Aspirin
• Lifestyle

Take-home messages

• Lifestyle interventions should underlie


all preventive efforts

• The intensity of preventive interventions


should match baseline cardiovascular
risk

965
Copyright © Harvard Medical School, 2018. All Rights Reserved.

Primordial, Primary, Secondary Prevention

Vaduganathan et al JACC 2015

Trends In Cardiovascular Death Rates Over Time

Nabel E, Braunwald E. NEJM 2012;366:54-63.

966
Copyright © Harvard Medical School, 2018. All Rights Reserved.

Impact of CVD in Women


1. More strokes in women, more coronary heart
disease (CHD) in men
2. Impact of obesity is greater in women than men
3. Gestational diabetes (2-10% of pregnancies)
increases risk of future diabetes by ~ 50% (30-60%)
3. Unique risk factors for stroke in women:
Pregnancy; Hormone Therapy
More hypertension after age 65

Objectives
1. To review current challenges for CVD prevention
2. To review (briefly) recent evidence / guidelines on:
• CV risk assessment
• Cholesterol
• Blood pressure
• Aspirin
• Lifestyle

967
Copyright © Harvard Medical School, 2018. All Rights Reserved.

Risk stratification is the key to prevention


High Risk Individuals
Established CHD
Cerebrovascular disease
Peripheral arterial disease
Abdominal aortic aneurysm
End-stage or CKD
Diabetes mellitus
(10 year Framingham risk > 20%)
10 year ASCVD risk (Pooled Cohorts) ≥ 7.5% *

* http://my.americanheart.org/cvriskcalculator

The information required to estimate ASCVD risk includes:


age, sex, race, TC, HDL-C, SBP, BP Rx, diabetes, smoking
10-yr risk of MI, fatal or nonfatal stroke, CHD death
lifetime risk

968
Copyright © Harvard Medical School, 2018. All Rights Reserved.

ASCVD Risk Calculator

http://tools.acc.org/ascvd-risk-estimator-plus/#!/calculate/estimate/

Importance of disease prevalence in applying


clinical risk scores
• Clinical prediction scores are developed to help clinicians
make more accurate probability estimates

• Caveat: clinical risk scores may not perform as well in a new


setting as they did in the setting in which they were
developed due to differences in disease prevalence in
different populations

969
Copyright © Harvard Medical School, 2018. All Rights Reserved.

Importance of disease prevalence in applying risk


scores
• To adjust for differences in disease prevalence in
clinical settings, Bayes’ Theorem can be applied
to risk scores (similar to diagnostic tests)

“High risk” score

“Low risk” score

13
Mora AHA 2015

Pooled Cohort Risk Equations


• Since the publication of the PCE, numerous
studies reported apparent overestimation of
risk with PCE (i.e. predicted > observed risk)
• Potential explanations:
– Differences in study populations
– Increasing use of statins and aspirin
– Decreasing ASCVD event rates
– Under-ascertainment of event rates in studies that
rely mainly on self-report

Mora et al AHA 2017

970
Copyright © Harvard Medical School, 2018. All Rights Reserved.

Pooled Cohort Risk Equations in the Women’s Health Initiative (WHI)


Observed vs predicted risk in women ≥65 years
before and after adding CMS events
20

15
Percent

10

5 Observed, unadjusted
Observed adj. asa, statin, CMS
0 Predicted
<7.5% 7.5 - <10% ≥ 10%
10-year risk category

Without including surveillance for ASCVD events using CMS, observed risks in
the WHI were lower than predicted by PCE as noted in several other US cohorts

Adjustment for time-dependent changes in statin and aspirin use resulted in small
increases in observed risks

Mora et al AHA 2017

Objectives
1. To review current challenges for CVD prevention
2. To review (briefly) recent evidence / guidelines on:
• CV risk assessment
• Cholesterol
• Blood pressure
• Aspirin
• Lifestyle

971
Copyright © Harvard Medical School, 2018. All Rights Reserved.

4 Statin Benefit Groups


( 1-3) High Risk:
1. Clinical ASCVD*
2. LDL–c >190 mg/dL, Age >21 years
3. Primary prevention – Diabetes:
Age 40-75 years, LDL–c 70-189 mg/dL
(4) Primary prevention† 4. No Diabetes:
≥7.5% 10-year risk, Age 40-75 years, LDL–c 70-189 mg/dL
* ACS, MI, angina, coronary or other arterial revascularization, stroke,
TIA, atherosclerotic PAD
† Requires risk discussion between clinician and patient

Stone et al JACC 2014;63:2889-934

Individuals Not in a Statin Benefit Group


Additional factors may inform clinical decision making

• Family history of premature ASCVD*


• High lifetime risk
• LDL–c ≥160 mg/dL
• hs-CRP ≥ 2 mg/L
• CAC score ≥ 300 or ≥75th percentile
• Ankle brachial index (ABI) < 0.9

* onset <55 y first degree male or <65 first degree female

Stone et al JACC 2014;63:2889-934

972
Copyright © Harvard Medical School, 2018. All Rights Reserved.

Statins and CVD Prevention by Baseline Risk

Cholesterol Treatment Trialists CTT Collaborators. Lancet. 2012;380:581–590.

PCSK9i lower LDL-c by ~ 60%

Robinson NEJM 2015


Sabatine NEJM 2015

973
Copyright © Harvard Medical School, 2018. All Rights Reserved.

PCSK9i Clinical Outcomes Studies


Alirocumab
n=18,000; Post-acute MI or hospitalized UA w/in 12 months
Rx w/ atorva 40/80 mg/d, rosuva 20/40 mg/d or max tolerated; LDL >
ODYSSEY Outcomes 70, nonHDL > 100, or apo B > 80
Randomized to alirocumab 75-150 mg SC Q2W
Completion ~ December 2017

Bococizumab
High risk CVD event; LDL 70-100 or nonHDL 100-130; on LLRx;
SPIRE-1 Randomized to Boco 150 mg SC Q2W vs placebo; n=17,000

SPIRE-2 Same as above except LDL > 100 or nonHDL > 130; n=9000

Evolocumab
N=27,564; History of MI, CVA, or PAD + RF;
Rx with atorva ≥ 20 mg or equivalent;
FOURIER LDL > 70 or nonHDL > 100;
Rx w/ evo 140 Q2W or 420 mg QM vs placebo

FOURIER: Eligibility Criteria


Clinical ASCVD (MI or stroke) within 5 years plus additional
risk factors:

Sabatine et al AHJ 2016; 173: 94

974
Copyright © Harvard Medical School, 2018. All Rights Reserved.

FOURIER: Primary Endpoint

Sabatine et al NEJM 2017; 376: 1713

FOURIER Trial—Key Efficacy and Safety End Points

Binding and neutralizing antibody rates for evolocumab of 0.35 and 0.0%, respectively
Sabatine MS et al. NEJM. 2017

975
Copyright © Harvard Medical School, 2018. All Rights Reserved.

FOURIER – PRIOR MI
Outcomes by High Risk Features
≥1 High Risk Feature No High Risk Feature

• 67% of FOURIER • 37% of FOURIER


• 22% RRR • 6% RRR
• 2.5% ARR • 0.5% ARR

SPIRE: Bococizumab

Trials stopped due to immunogenicity

Ridker et al NEJM 2017 376: 1522

976
Copyright © Harvard Medical School, 2018. All Rights Reserved.

Alirocumab
ODYSSEY-Outcomes Study Design
• Patient population: • Primary endpoint:
• Recent ACS (1-12 mo before randomization) – CHD death
• Lipids not optimally controlled on optimal statin: – Non-fatal MI
LDL-C ≥70 mg/dL, non-HDL-C ≥100 mg/dL, or apo B – Ischemic stroke
≥80
– Unstable angina requiring hospitalization

Run-In Period Double-Blind Treatment Period


(~ 2 to 5 years)

At Month 2 and beyond:


75 mg or 150 mg
every 2w adjusted in blinded fashion to
Adjust statin, achieve LDL-C<50 mg/dl
Check lipids
Alirocumab (n=9000)
R
Randomization
Index ACS Placebo (n=9000)
event

Schwartz GG et al., Am Heart J 2014; 168:682-689

Stegg G et al., ACC 2018

977
Copyright © Harvard Medical School, 2018. All Rights Reserved.

Guidance on Use of Nonstatin Therapy


4 statin benefit groups

• Adherence and lifestyle


• Statin intolerance
• All patients
Control with
of other risk ASCVD
factors
• Discussion about non-statin
• Primary
therapies
• Choosing
• >50% alternative goalsinofLDL-C
reduction
therapy
•• Secondary (may consider)
Monitoring response to therapy
• • LDL-C
Referral <70 mg/dL
to a specialist/nutritionist
• Ezetimibe
• Non-HDL-C <100 mg/dL
• Bile acid sequestrant
• PCSK9 inhibitor
• Other therapies (LDL apheresis,
lomitapide, mipomersen)

PCSK9, proprotein convertase subtilisin/kexin type 9 serine protease.


Lloyd-Jones DM, et al. JACC. 2017;70(14):1785-1822.

2016 ESC/EAS Guidelines on the Management


of Dyslipidemias: Pharmacological Treatment
of Hypercholesterolemia

Task Force for the Management of Dyslipidaemias of the ESC and EAS. Eur Heart J. 2016 prepub.

978
Copyright © Harvard Medical School, 2018. All Rights Reserved.

AACE 2017 Lipid Goals

ENDOCRINE PRACTICE Rapid Electronic Article in Press DOI:10.4158/EP171764.GL

2017

R 20. Lipids, including TG, can be measured


in the non-fasting state if fasting
determinations are impractical
32
Jellinger et al. Endocrine Practice 2017;Epub ahead of print Feb 3

979
Copyright © Harvard Medical School, 2018. All Rights Reserved.

Take-home messages
• Statins are the most effective traditional LDL-cholesterol lowering medication
and should be used at the maximally tolerated intensity in those at highest
risk (e.g. clinical ASCVD)

• Currently approved PCSK9 inhibitors (alirocumab and evolocumab) achieve


significant reductions in LDL-cholesterol to previously unseen levels

• The PCSK9 inhibitors significantly and safely reduce the rate of adverse events
when added to statin therapy in high risk secondary prevention patients with
additional risk factors

• Incremental benefit for ASCVD (but not mortality) associated with lowering of
LDL-cholesterol levels well below current targets

Objectives
1. To review current challenges for CVD prevention
2. To review (briefly) recent evidence / guidelines on:
• CV risk assessment
• Cholesterol
• Blood pressure
• Aspirin
• Lifestyle

980
Copyright © Harvard Medical School, 2018. All Rights Reserved.

2017 Guideline for the Prevention, Detection, Evaluation


and Management of High Blood Pressure in Adults
BP Classification (JNC 7 and ACC/AHA Guidelines)
SBP DBP 2003 JNC7 2017 ACC/AHA
<120 and <80 Normal BP Normal BP

120–129 and <80 Prehypertension Elevated BP

130–139 or 80–89 Prehypertension Stage 1 hypertension

140–159 or 90-99 Stage 1 hypertension Stage 2 hypertension

≥160 or ≥100 Stage 2 hypertension Stage 2 hypertension

• Blood Pressure should be based on an average of ≥2 careful readings on ≥2 occasions


• Adults with SBP or DBP in two categories should be designated to the higher BP category
Whelton PK et al. Hypertension. 10.1161/HYP.0000000000000065. [Epub ahead of print].
Whelton PK et al. J Am Coll Cardiol. 2017; doi: 10.1016/j.jacc.2017.11.006. [Epub ahead of print].

Direct relationship of CVD with urinary Sodium


2,275 individuals with Prehypertension

Cook et al, Circulation 2014:129:981

981
Copyright © Harvard Medical School, 2018. All Rights Reserved.

2017 Guideline for the Prevention, Detection, Evaluation


and Management of High Blood Pressure in Adults
BP Thresholds for Treatment
SBP DBP CVD Risk/other circumstances Recommended Treatment
<120 and <80 N/A Healthy Lifestyle
120–129 and <80 N/A Nonpharmacological therapy

130-139 or 80-89 No CVD/10-yr ASCVD risk <10%* Nonpharmacological therapy

130–139 or 80–89 CVD/10-year ASCVD risk ≥ 10%

≥130 or ≥80 Diabetes or CKD Antihypertensive drug therapy


(plus nonpharmacological therapy)
≥130 Age ≥65 years

≥140 or ≥90 N/A

Whelton PK et al. Hypertension. 10.1161/HYP.0000000000000065. [Epub ahead of print]. * AHA/ACC 2013 Pooled
Whelton PK et al. J Am Coll Cardiol. 2017; doi: 10.1016/j.jacc.2017.11.006. [Epub ahead of print]. Cohort CVD Risk Equations

2017 Guideline for the Prevention, Detection, Evaluation


and Management of High Blood Pressure in Adults
High BP Treatment Target
SBP DBP CVD Risk Recommended Treatment
<120 and <80 N/A N/A

120–129 and <80 N/A N/A

130-139 or 80-89 No CVD and 10-year


ASCVD risk <10%
130–139 or 80–89 Clinical CVD or 10-year
ASCVD risk ≥ 10%
SBP <130 and DBP <80 mm Hg
≥130 or ≥80 Diabetes or CKD

≥140 or ≥90 N/A

≥130 Age ≥65 years SBP <130 mm Hg

Whelton PK et al. Hypertension. 10.1161/HYP.0000000000000065. [Epub ahead of print].


Whelton PK et al. J Am Coll Cardiol. 2017; doi: 10.1016/j.jacc.2017.11.006. [Epub ahead of print].

982
Copyright © Harvard Medical School, 2018. All Rights Reserved.

Lifestyle Modifications for BP Control

Modification Recommendation Approximate SBP


Reduction Range

Weight Maintain normal body weight 5-20 mmHg/10 kg


reduction (BMI=18.5-25) weight lost

DASH eating Diet rich in fruits, vegetables, low 8-14 mmHg


plan fat dairy and reduced in fat

Restrict sodium <2.4 grams of sodium per day 2-8 mmHg


intake

Physical Regular aerobic exercise for at 4-10 mmHg


activity least 30 minutes at least 5 days of
the week
Moderate <2 drinks/day for men and <1 2-4 mmHg
alcohol drink/day for women

Chobanian AV et al. JAMA 2003;289:2560-2572

Included: SBP 130-180


Age >50 plus one CVD risk factor
SPRINT Excluded: Diabetes, CVA, HF

Target Systolic BP

Intensive Treatment Standard Treatment


Goal SBP < 120 mm Hg Goal SBP < 140 mm Hg

Screened
(n=14,692)

Randomized
(n=9,361)

Intensive treatment Standard treatment


SPRINT trialists. NEJM 2015;737:2103-16.
(n=4,678) (n=4,683)

983
Copyright © Harvard Medical School, 2018. All Rights Reserved.

SPRINT: Change in Blood Pressure

134.6 mmHg
(1.8 medications)

121.5 mmHg
(2.8 medications)

DBP: 66 vs. 75 mmHg


SPRINT trialists. NEJM 2015;737:2103-16.

SPRINT
Median f/u = 3.3 yrs
Number needed to treat = 61

SPRINT trialists. NEJM 2015;737:2103-16.

984
Copyright © Harvard Medical School, 2018. All Rights Reserved.

Summary of SPRINT Findings


• Difference of 13 mmHg SBP between Rx groups
• 3 vs. 2 BP meds
• 25% reduction primary outcome
• NNT 61
• 27% reduction all-cause mortality
• NNT 90
• Rx effect similar across all 6 pre-specified subgroups
• Patients >75 y/o with benefit
• 28% >75 yrs of age (mean age 80)
• No overall difference in serious adverse events
• Increased hypotension, electrolyte abnormalities, AKI, but not injurious falls

HOPE-3

~12K pts
No CVD
Annual risk ~1%
(10-yr risk ~ 10%)
Avg SBP ~138

Randomized:
Placebo vs
Canda 16 + HCTZ 12.5

~5.6 yrs f/u

Primary outcome:
CV death, MI, CVA

Upper BP tertile
benefit

HOPE-3 NEJM 2016

985
Copyright © Harvard Medical School, 2018. All Rights Reserved.

HOPE-3

HOPE-3 NEJM 2016

Causes of Secondary Hypertension

Medical Conditions Drugs


Chronic kidney disease NSAIDS
Primary hyperaldosteronism Oral contraceptives
Renovascular disease Adrenal steroids
Chronic steroid therapy Sympathomimetics
Cushing’s syndrome Cyclosporine or tacrolimus
Pheochromocytoma Erythropoietin
Aortic coarctation Ephedra, mu huang, bitter orange
Thyroid or parathyroid disease Cocaine or amphetamines
Sleep apnea Alcohol

Chobanian et al. JAMA 2003;289:2560-2572

986
Copyright © Harvard Medical School, 2018. All Rights Reserved.

Take-home messages

1. Risk-based assessment*

2. For most patients, goal is <130/ <80

3. Use any of: CCB, thiazides, ACE/ ARB, BB, taking into
account CKD, CAD, HF, aortopathy

*ACC/AHA Pooled Cohort Equations (http://tools.acc.org/ASCVD-


Risk-Estimator/) to estimate 10-year risk of atherosclerotic CVD.

Objectives
1. To review current challenges for CVD prevention
2. To review (briefly) recent evidence / guidelines on:
• CV risk assessment
• Cholesterol
• Blood pressure
• Aspirin
• Lifestyle

987
Copyright © Harvard Medical School, 2018. All Rights Reserved.

What about Aspirin?


• Aspirin for CVD prevention
• Review overall estimates of benefit
• Review estimates in men, women
• Review the 2016 USPSTF guidelines

Aspirin: Mechanism of Action

Membrane Phospholipids

Arachadonic Acid

COX-1 Aspirin
Prostaglandin H2

Thromboxane A2 Prostacyclin
↑ Platelet Aggregation ↓ Platelet Aggregation
Vasoconstriction Vasodilation
Spite, Serhan Circulation Research 2010; 107:1170-1184

988
Copyright © Harvard Medical School, 2018. All Rights Reserved.

2016 U.S. Preventive Task Force Services


updated meta-analysis

Outcome No. trials No. individuals Summary Relative


Risk
Nonfatal MI 10 114,734 0.78 (0.71-0.87)
8 (≤100 mg) 87,524 0.83 (0.74-0.94)
Nonfatal stroke 10 99,655 0.95 (0.85-1.06)
7 (≤100 mg) 68,734 0.86 (0.76-0.98)
CVD mortality 11 118,445 0.94 (0.86-1.03)
8 (≤100 mg) 87,524 0.97 (0.85-1.10)
Total mortality 11 118,445 0.94(0.89-0.99)
8 (≤100 mg) 87,524 0.95 (0.89-1.01)

Guiruis-Blake JM et al. 2016 www.uspreventiveservicestaskforce.org

Aspirin in Primary Prevention:


Sex differences?

Mora, Manson. JAMA Internal Med 2016; 176


Antithrombotic Trialist Collaboration. Lancet 2009;373:1849

989
Copyright © Harvard Medical School, 2018. All Rights Reserved.

2016 U.S. Preventive Services Task Force


Recommendations for low dose aspirin
Population Recommendation Grade
Adults age 50-59 yrs For primary prevention of ASCVD and colorectal B (Moderate)
cancer if:
- ≥ 10% ASCVD risk *
- Not at increased risk of bleeding
- Life expectancy of at least 10 yrs
- Willing to take aspirin for at least 10 yrs

Adults age 60-69 yrs Individualize the decision if: C


- ≥ 10% ASCVD risk,
- Not at increased risk of bleeding
- Life expectancy of at least 10 yrs
- Willing to take aspirin for at least 10 yrs

Adults < 50 yrs Insufficient evidence I


Adults ≥ 70 yrs Insufficient evidence I

Guiruis-Blake JM et al. 2016 www.uspreventiveservicestaskforce.org

2016 ADA Aspirin Recommendations for Patients with Diabetes


• Aspirin 75 to 162 mg/day for primary ASCVD prevention
in diabetic patients at increased ASCVD risk and not
increased risk of bleeding
– Those at risk for ASCVD (10-year risk >10%)—men >50 yrs,
women >50 yrs, with >1 additional risk factor
– Family history of premature ASCVD
– HTN
– Smoking
– Dyslipidemia
– Albuminuria
• Not recommended for primary prevention in low risk
groups (10-year risk <5%)
• Clinical judgement for intermediate risk (5 to 10%) or
<50 yrs and risk factors
ADA Diabetes Care 2016;39:S1-S106

990
Copyright © Harvard Medical School, 2018. All Rights Reserved.

Longterm low-dose aspirin reduces cancer incidence and


metastasis, in particular for gastrointestinal cancers (colorectal)
Cancer incidence Cancer metastasis

Placebo
Aspirin

HR 0.88 (0.80-0.98), P=0.017 HR 0.81 (0.70-0.93), P=0.003

Years to Notification Years to Notification

Rothwell PM. et al. 2012 Lancet 13:518

Bleeding risks with aspirin


Risk factors for bleeding
Proton Pump Inhibitors (PPIs) may
Age
decrease risk of GI bleeding on aspirin
Male sex
GI hospitalization
Excess alcohol use
Current smoking
Hypertension
Diabetes
Liver / renal disease

Concomitant meds Tran-Duy A. et al. 2015 Int J Clin Pract doi:10.1111/ijcp.12634


(NSAIDs, anticoagulants)

Whitlock E et al. 2015 www.uspreventiveservicestaskforce.org


GI: Gastrointestinal

991
Copyright © Harvard Medical School, 2018. All Rights Reserved.

Primary Prevention with Aspirin:


Tipping the Scales
www.aspiringuide.com
Nonfatal GI or Intracranial
CVA/MI Bleed

Older age
ASCVD Risk Male
NSAID/anticoag
Ulcers
Prior GIB

Mora et al JAMA 2016; 316:709


Mora and Manson JAMA Internal Med 2016; 176

Aspirin-Guide

www.
aspiringuide.com

JAMA 2016; 316:709


JAMA Internal Med 2016; 176

992
Copyright © Harvard Medical School, 2018. All Rights Reserved.

Objectives
1. To review current challenges for CVD prevention
2. To review (briefly) recent evidence / guidelines on:
• CV risk assessment
• Cholesterol
• Blood pressure
• Aspirin
• Lifestyle

Preventability of
Heart Disease, Stroke, and Diabetes
With lifestyle
modifications*

-83% -79% -91%

* Physical activity, not smoking, weight control, healthy diet (high in whole grains, fiber,
fruit/veg, fish, low in saturated and trans fats), moderate alcohol.
Stampfer, et al. NEJM 2000; Chiuve, et al. Circulation 2008; Hu, et al. NEJM 2001.

993
Copyright © Harvard Medical School, 2018. All Rights Reserved.

AHA Life’s Simple 7


1. ≥150 minutes moderate activity /week
or ≥75 minutes vigorous activity/week
2. Eat a healthy diet (4-5 components of
healthy diet score*)
3. Have a normal body weight (BMI < 25)
4. Never smoked or quit >1 year ago
5. Total cholesterol <200 mg/dL
6. Blood pressure <120/80 mm Hg
7. Fasting blood glucose <100 mg/dL
* 1) 4.5 cups or more of fruits and vegetables per day 2)
two or more 3.5-oz servings of fish per week 3) three
servings per day of whole grains 4) less than 1500 mg of
sodium per day 5) 36 ounces or less of sugar-sweetened
beverages per week
Lloyd-Jones et al. Circulation 2010; 121:586-613

AHA Life’s Simple 7 predicts mortality

Yang et al. JAMA 2012; 307:1273-83

994
Copyright © Harvard Medical School, 2018. All Rights Reserved.

Activity Recommendations for CV/General Health


• Goal: 30 minutes 7 days per week (minimum 5 days
per week)
• Assess risk with history and/or exercise test: I (B)
• 30-60 min moderate aerobic + increase in daily
activities: I (B)
• Encourage resistance training 2/week: IIb (C)
• Medically supervised programs for high-risk (ACS,
revascularization, heart failure): I (B)
Courtesy of Donna Polk, MD

Physical Activity Recommendations for LDL and BP

3 to 4 sessions a week, lasting on average 40


minutes per session, and involving moderate-to-
vigorous intensity physical activity.

Eckel RH et al. Circulation 2013: 10.1161/01.cir.0000437740.48606.d1

995
Copyright © Harvard Medical School, 2018. All Rights Reserved.

2014 ACC/AHA/TPC Obesity Guideline Goals

• BMI 18.5 to 24.9 kg/m2

• Women: WC <35 inches


• Men: WC <40 inches

• 3 to 5% sustained weight reduction in


overweight/obese individuals with CVD risk
factors
Jensen et al. Circulation 2014: 129: S102

Diet Recommendations for Weight Reduction


I IIa IIb III
Prescribe a diet to achieve reduced calorie intake for obese or
overweight individuals who would benefit from weight loss, as part
of a comprehensive lifestyle intervention. Any one of the following
methods can be used to reduce food and calorie intake:
a. Prescribe 1,200–1,500 kcal/d for women and 1,500–1,800
kcal/d for men (kilocalorie levels are usually adjusted for the
individual’s body eight);
b. Prescribe a 500-kcal/d or 750-kcal/d energy deficit; or
c. Prescribe one of the evidence-based diets that restricts
certain food types (such as high-carbohydrate foods, low-
fiber foods, or high-fat foods) in order to create an energy
deficit by reduced food intake.

996
Copyright © Harvard Medical School, 2018. All Rights Reserved.

Scientific Report of the 2015 Dietary Guidelines


Advisory Committee (Key Recommendations)

Three beneficial dietary patterns

1. Healthy US-style pattern


2. Healthy Mediterranean-style pattern
3. Healthy Vegetarian pattern

https://health.gov/dietaryguidelines/2015-scientific-report/pdfs/scientific-report-of-the-2015-dietary-guidelines-advisory-committee.pdf

Dietary Priorities

Mozaffarian Circulation 2016: 129: S102

997
Copyright © Harvard Medical School, 2018. All Rights Reserved.

Women’s Health Initiative (WHI)

Howard et al JAMA 2006;295:655

998
Copyright © Harvard Medical School, 2018. All Rights Reserved.

Lyon Diet Heart Study

De Lorgeril M et al. Arch Intern Med 1998;158:1161 De Lorgeril M et al. Circ 1999;99:779-785

PREDIMED STUDY

N=7447
57% women
High CVD risk
No prior CVD

RRR of Mediterranean diet (EVVO or raw nuts)


reduced CVD by 30% compared with control diet
Estruch et al NEJM 2013;368:1279

999
Copyright © Harvard Medical School, 2018. All Rights Reserved.

Statins

Cholesterol Treatment Trialists CTT Collaborators. Lancet. 2012;380:581–590.

PREDIMED Study Design. Participants

Men: 55-80 y 1. Smoking


2. Hypertension
Women: 60-80 y
3. ↑ LDL-C
High risk without CVD 4. ↓ HDL-C
with type 2 diabetes or 5. Overweight / obesity
3+ CVD risk factors 6. Family history of early-onset CHD

Random
N=7447

Estruch et al NEJM 2013;368:1279

1000
Copyright © Harvard Medical School, 2018. All Rights Reserved.

PREDIMED: 3 Randomized Groups


Mediterranean diet + Extra-Virgin Olive Oil
Advice on MeDiet + EVOO ≥ 50 ml/day (~4 Tbsp/d)
(1 gallon/m)

Mediterranean diet + Nuts


Advice on MeDiet +
3 whole walnuts (15 g)
+ 30 g of raw nuts
8 hazelnuts (7.5 g) per day
+
6 almonds (7.5 g)

Control group: Advice on a low-fat diet (Reduce fat, discourage


use of nuts and olive oil; Non-food items)
Estruch et al NEJM 2013;368:1279

PREDIMED: Compliance with Mediterranean Diet

Estruch R. et. al. N Engl J Med 2013;368:1279-90

1001
Copyright © Harvard Medical School, 2018. All Rights Reserved.

Food Differences between Mediterranean


and Control Diets
KEY FOODS MeDiet + EVOO MeDiet + nuts Control P value
diet
Grams/day

Virgin olive oil 29 10 3 < 0.001

Refined-mixed olive oil - 17 -6 -4 < 0.001

Nuts 0 18 -3 < 0.001

Legumes 2.4 2.4 0 < 0.001

Fish or Seafood 1.25 2.50 -4 ≤ 0.01

The change is follow-up minus baseline; the last available follow-up FFQ of each
participant was used.

Differences in consumption of other key foods were not statistically significant.

Topics discussed
1. To review current challenges for CVD prevention
2. To review (briefly) recent evidence / guidelines on:
• CV risk assessment
• Cholesterol
• Blood pressure
• Aspirin
• Lifestyle

1002
Copyright © Harvard Medical School, 2018. All Rights Reserved.

Summary Slide Take-home messages


1. First step in prevention is to assess ASCVD risk
(risk factors, global risk score)
2. Lifestyle improvement is the most important component of ASCVD
prevention and risk factor treatment (Life’s Simple 7)
3. Statins added to lifestyle to reduce risk of ASCVD in higher risk
individuals (4 statin benefit groups); PCSK9i in the very highest
risk patients
4. Blood pressure control: Target BP for most patients <130/80
mmHg; risk-based assessment
5. Aspirin (81-162 mg/d) in higher risk individuals if benefit outweighs
risk of bleeding (avoid in low risk individuals and elderly)

Question
According to the 2013 ACC/AHA guideline on treatment of blood
cholesterol to reduce ASCVD risk in adults, what is the
definition of high-intensity statin therapy? (select the best
answer) A. Daily dose lowers LDLC by ≥75%

B. Daily dose lowers LDLC by ≥50%


C. Daily dose lowers LDLC by 30 to 50 %
D. Daily dose lowers LDLC by 25%

1003
Copyright © Harvard Medical School, 2018. All Rights Reserved.

Answer / Discussion
According to the 2013 ACC/AHA guideline on treatment of blood cholesterol to reduce
ASCVD risk in adults, what is the definition of high-intensity statin therapy? (select
the best answer) Discussion:
• High intensity statin usually
A. Daily dose lowers LDLC by ≥75%
lowers LDLC by 50%
B. Daily dose lowers LDLC by ≥50% • Moderate intensity statin
daily dose lowers LDLC by
C. Daily dose lowers LDLC by 30 to 50 % 30-50%
D. Daily dose lowers LDLC by 25% • Low intensity statin daily
dose lowers LDLC by <30%

Statin Intensity
High: Moderate: Low: Lowers LDL-C
Lowers LDL-C by ≥50% Lowers LDL-C 30 to <30%
<50%
Atorvastatin 40, 80 Atorvastatin 10, 20
Rosuvastatin 20, 40 Rosuvastatin 5, 10
Simvastatin 20, 40 Simvastatin 10
Pravastatin 40, 80 Pravastatin 10, 20
Lovastatin 40 Lovastatin 20
Fluvastatin XL 40 BID Fluvastatin 20, 40

Pitavastatin 2, 4 Pitavastatin 1
Stone et al JACC 2014;63:2889-934

1004
Copyright © Harvard Medical School, 2018. All Rights Reserved.

Question / Case
26 yo Hispanic M, smoker, multiple borderline risk factors,
BMI 33, Lp(a) 70 (uln 30 mg/dL) - one day prior to ACS:
• Does he have any of the other factors that the 2013
ACC/AHA guidelines recommend can be considered if a
risk decision is not certain? (select the best answer)
A. Smoking
B. Obesity (his BMI 33)
C. High lifetime risk
D. High Lp(a) (his Lp(a) 70 mg/dL)

Answer / Case Discussion


yo Hispanic M, smoker, multiple borderline risk factors, BMI 33, Lp(a) 70 (uln 30 mg/dL) -
one day prior to ACS:
• Does he have any of the other factors that the 2013
ACC/AHA guidelines recommend can be considered if a
risk decision is not certain? Discussion:
A. Smoking • High lifetime risk is
a factor that the
B. Obesity (his BMI 33) guidelines say can
C. High lifetime risk inform treatment
decision (2013
D. High Lp(a) guidelines would
support moderate
statin Rx here)

1005
Copyright © Harvard Medical School, 2018. All Rights Reserved.

One day prior to his ACS, what was his risk?


According to the 2013 ACC/AHA guidelines

60
Predicted Risk (%)

40
20
0
10-yr risk Lifetime risk
Jose Optimal RF
* Optimal risk factors: TC 170, HDL-c 50, SBP 110, No DM, No HTN or Rx
*
http://my.americanheart.org/cvriskcalculator 85

Question
The 2016 USPSTF recommend considering aspirin for primary
prevention of ASCVD for individuals 50-69 years who are not
at increased risk of bleeding if (select the best answer)
A. 10-yr ASCVD ≥7.5%
B. 10-yr ASCVD ≥10%
C. 10-yr ASCVD ≥10% AND age >70 y
D. None of the above

1006
Copyright © Harvard Medical School, 2018. All Rights Reserved.

Answer / Discussion
The 2016 USPSTF recommend considering aspirin for primary
prevention of ASCVD for individuals 50-69 years who are not
at increased risk of bleeding if
Discussion:
A. 10-yr ASCVD ≥7.5% • 7.5% is the threshold for the
cholesterol guidelines
B. 10-yr ASCVD ≥10%
• Age > 70 Grade I (Insufficient
C. 10-yr ASCVD ≥10% AND age >70 y evidence) [also increased
risk bleeding]
D. None of the above

Supplemental References
1. Risk assessment guidelines. Goff et al JACC
2014;63:2935-59
2. Cholesterol guidelines. Stone et al JACC
2014;63:2889-934
3. Lifestyle guidelines. Eckel et al Circulation
2014;129:S76-99
4. Obesity guidelines. Jensen et al Circulation
2014;129:S102
5. 2017 ACC/AHA BP guidelines. Whelton PK et al. JACC
2017 ; doi: 10.1016/j.jacc.2017.11.006.
6. 2016 USPSTF aspirin guidelines.
www.uspreventiveservicestaskforce.org

1007
Copyright © Harvard Medical School, 2018. All Rights Reserved.

Acute Coronary Syndrome Management

HMS & BWH


Intensive Review of Internal Medicine

Marc S. Sabatine, MD, MPH


Chairman, TIMI Study Group
Lewis Dexter, MD, Distinguished Chair in Cardiovascular Medicine, BWH
Professor of Medicine, Harvard Medical School

Disclosures
Research Grant Support through BWH:
Amgen; AstraZeneca; Daiichi-Sankyo; Eisai; GlaxoSmithKline; Intarcia; Janssen Research and
Development; Medicines Company; MedImmune; Merck; Novartis; Pfizer; Poxel; Takeda

Scientific Advisory Boards & Consulting:


Consulting for: Amgen; AstraZeneca, Bristol-Myers Squibb; CVS Caremark; Dyrnamix; Esperion;
Intarcia; Janssen Research and Development; Medicines Company; MedImmune; Merck; Novartis

An Academic Research Organization of


Brigham and Women’s Hospital and Harvard Medical School

1008
Copyright © Harvard Medical School, 2018. All Rights Reserved.

ACUTE CORONARY SYNDROMES

Non-ST elevation ACS ST elevation ACS


CK-MB

Tn

UA NSTEMI STEMI

UA NQWMI QwMI

ACS: ECG
• What to look for
– STE or LBBB not known to be old
– ST depression ≥0.5 mm; TWI >1 mm
– Coronary distribution
• What else to look for
– Q waves or poor R wave progression (PRWP)
• How to look for it
– 12-lead ECG w/in 10 mins of presentation
– Compare to prior ECGs
– Obtain serial ECGs (initial ⊕ in <50% ACS Pts)
An Academic Research Organization of
Brigham and Women’s Hospital and Harvard Medical School

1009
Copyright © Harvard Medical School, 2018. All Rights Reserved.

ACS: Biomarkers
Era Assay Measure at presentation
+…

Ancient Historsy CK-MB q8 hrs × 3


(1980s)

Dawn of modern Troponin q8 hrs × 3


cardiac markers
(1990s)

Recent past Troponin 3-6 hrs after sx onset


Now hs-Troponin 1 (to 3) hours later
(If presentation ≥3 h after sx onset)
Examine absolute and ∆
An Academic Research Organization of
Brigham and Women’s Hospital and Harvard Medical School

Other Causes of ⊕ Troponin


• Very rarely “false positive” (ie, no true
myocardial injury - ? renal failure)
• “Type 2 MI” = myocyte injury not due to ACS
– Supply-demand mismatch not due to ∆ in CAD
• HoTN, tachycardia or HTN crisis
• HCM, severe AS
• Coronary spasm
– Non-ischemic injury: myocarditis, contusion
– Multi-factorial
• CHF exacerbation
• PE, ARDS, severe pneumonia
• Sepsis, stroke
An Academic Research Organization of
Brigham and Women’s Hospital and Harvard Medical School

1010
Copyright © Harvard Medical School, 2018. All Rights Reserved.

“Minor” Elevations of cTn in ACS


and Clinical Outcomes
99th %ile AMI per CK-MB

16
Normal "Leaklet” AMI
12.1
D/MI/ACS thru 30 days

12 11.3
10.5

8
5.5
4

n=734 n=181 n=213 n=693


0

<0.1 0.1-0.4 0.4-1.5 >=1.5


An Academic Research Organization of
Brigham and Women’s Hospital and Harvard Medical School Troponin I Morrow DA et al. JAMA 2001;286:2405

ACS Likelihood
Feature High Intermediate Low
History • Chest or L arm pain or • Chest or L arm pain or • Prob ischemic sx w/o
discomfort as chief sx ≈ discomfort as chief sx intermed-likelihood
prior doc angina • Age >70 y characteristics
• Known h/o CAD • Male sex • Recent cocaine use
• Diabetes mellitus

Exam • Transient MR murmur, HoTN, • Extracardiac vascular • Chest discomfort


diaphoresis, pulm edema, or disease reproduced by palp
rales

ECG • New, or presumably new, • Fixed Q waves • Tw flattening or


transient ST deviation • ST depression 0.5-1 mm inversion <0.1 mV in
(≥1 mm) or TWI (≥2 mm) in or TWI >1 mm leads w/ dominant R
multiple precordial leads waves
• Normal ECG

Biomarkers • Elevated • Normal • Normal

ACC/AHA 2007 UA/NSTEMI Guidelines. Circulation 2007;116:e148


An Academic Research Organization of
Brigham and Women’s Hospital and Harvard Medical School

1011
Copyright © Harvard Medical School, 2018. All Rights Reserved.

CCTA vs Perfusion Imaging


Parameter Standard of Care

CCTA

N 501 449 P
Cost Effectiveness
Median Time to dx (hr, IQR) 5.8 (4.0-9.0) 21.0 (8.5-23.8) <.001
Length of stay (hr, IQR) 8.6 (6.4-27.6) 26.7 (21.4-30.6) <.001
Mean Total Cost $4026 ± 6792 $3874 ± 5298 .75
Procedures
Angiography 11% 7% .06
Revasc (PCI or CABG) 6% 4% .14
Safety
MACE 2 6 .18
Radiation exposure (mSv/pt) 13.9 ± 10.4 4.7 ± 8.4 <.001
An Academic Research Organization of
Brigham and Women’s Hospital and Harvard Medical School
Hoffmann et al. NEJM 2012;367:299

ST-Elevation MI (STEMI)
• Consider immediate reperfusion therapy
• In whom?
– Within 12 hrs of sx onset, or
– 12-24 hrs after sx onset if clinical or ECG evidence of
ongoing ischemia
• How?
– Primary PCI (including transfer to PCI-capable hosp
if door-in to door-out time will be <30 min &
1st med contact to PCI anticipated <120 min)
– Fibrinolytic (barring contraindications*)
*Absolute: prior ICH; intracranial neoplasm, aneurysm, or AVM; stroke or head trauma w/in 3 mos; active
internal bleeding or diathesis; suspected AoD
*Relative: severe HTN; stroke; prolonged CPR; recent bleed, surgery or trauma; noncompressible vasc
puncture; pregnancy; current use of anticoagulants
An Academic Research Organization of
Brigham and Women’s Hospital and Harvard Medical School

1012
Copyright © Harvard Medical School, 2018. All Rights Reserved.

Preventive PCI in STEMI


PRAMI: 465 Pts w/ STEMI CvLPRIT: 296 Pts w/ STEMI
Immediate PCI of all other lesions >50% PCI of all other lesions >70% during index hosp.
25
Cardiac death, MI, refractory angina (%)

Death, reMI, HF, IDR (%)


20

P=0.009
15

10

0
Culprit only Complete

NEJM 2013;369:1115 JACC 2015;65:963


An Academic Research Organization of
Brigham and Women’s Hospital and Harvard Medical School

What To Do after Fibrinolysis?


• If it fails (persistent STE [<50% resolution] or sx, development of
shock, evidence of infarct-related artery reocclusion): PCI
• If it succeeds:
• Non-invasive ischemia testing (ie, stress test), OR
• Transfer high-risk pts w/in 3-24 hrs for elective PCI
(high-risk = anterior MI, inferior MI w/ low EF or RV infarct,
extensive STE or LBBB, HF, hypotension or tachycardia)
D, MI, RI, CHF/shock

88.7% cath
• 1059 high-risk STEMI median time 32.5 h (1/3 w/in 12 h) P=0.004
Pts Rx’d with lytic 67.4% PCI, 22.7 h from lytic

• Rand. to immed transfer 98.5% cath


w/ PCI w/in 6 h or rec for median time 2.8 h
cath w/in 2 wks (earlier 84.9% PCI; 3.9 h from lytic

if needed)
Transfer-AMI, NEJM
2009;360:2705-18

1013
Copyright © Harvard Medical School, 2018. All Rights Reserved.

Anti-Ischemic Therapy
• Nitrates
– Sx relief; no mort benefit (GISSI-3 & ISIS-4)
• Beta-blockers
– ↓ ischemia, ↓ D/MI (in AMI trials)
• Calcium channel blockers
– If ischemia despite max βB or βB contra.
• Morphine
– Pain, CHF, agitation; don’t mask angina
• Oxygen

Beta-Blockers Acutely in ACS


Class I
Oral βB should be initiated in the first 24 hrs if w/o any of following:
1) heart failure,
2) low output state,
3) ↑ risk for cardiogenic shock, or
4) other relative contraindications (PR >0.24 sec, 2° or 3° AVB,
active asthma, or reactive airways disease)

Class III Harm


IV βB at time of presentation if risk factors for shock

Risk factors for cardiogenic shock (the greater the number of risk factors present, the higher the risk)
are age >70 yrs, SBP <120 mm Hg, HR >110 bpm or <60 bpm, and ↑ time since onset of symptoms.

Circ 2013;127:e362

1014
Copyright © Harvard Medical School, 2018. All Rights Reserved.

Management Strategy in NSTEACS


INVASIVE
(ie, angiography for all in ~48 hrs)
PCI / CABG
NSTEACS
anatomy
Long-term
Initial Med Rx
Med Rx
high-risk low-risk

recurrent
angina

Cont’d
Med Rx

CONSERVATIVE
An Academic Research Organization of
Brigham and Women’s Hospital and Harvard Medical School
(ie, selective angiography)

Benefit of INV vs CONS Strategy

An Academic Research Organization of


Brigham and Women’s Hospital and Harvard Medical School O’Donoghue M, et al. JAMA 2008;300:71-80

1015
Copyright © Harvard Medical School, 2018. All Rights Reserved.

Troponin Treatment Interaction


CONS INV

OR=0.41
30
D/MI/ACS at 30 days (%)

Interaction (0.28-0.61)
25 P<0.001 p<0.001
20 17.6
OR=1.60 16.5 15.6
15 (0.83-3.0)
P=NS 8.8
10
6.6
4.3 4.4 5.4
5

0
<0.1 0.1-0.4 0.4-1.5 >=1.5
N=734 N=181 N=213 N=693
TnI Level at Presentation
An Academic Research Organization of
Brigham and Women’s Hospital and Harvard Medical School Cannon CP et al. NEJM 2001;344:1879

TIMACS
3031 Patients with NSTEACS
Cath w/in 24 h (median 14 h) or >36 h (median 50 h)
D, MI, refract ischemia
D, MI, stroke

Mehta SR et al. NEJM 2009;360:2165-75

1016
Copyright © Harvard Medical School, 2018. All Rights Reserved.

2014 ACC/AHA NSTEACS Guidelines:


Early Invasive
Immediate Early Invasive Delayed Invasive Ischemia-Guided
(w/in 2 h) (w/in 24 h) (w/in 25-72 h)

• Refractory angina • GRACE score >140 • TIMI Risk Score ≥2 • TIMI Risk Score 0-1
• Signs or symptoms • Temporal ∆ in Tn • GRACE score >109- • GRACE score <109
of HF or new or • New or presumably 140 • Low-risk Tn-neg
worsening MR new ST depression • Diabetes female patient
• Recurrent angina or • GFR <60 • Patient or clinician
ischemia at rest or mL/min/1.73m2 preference in
with low-level activity absence of high-risk
despite intensive • EF <0.40
features
med Rx • Early postinfarction
angina
• PCI w/in 6 mo
• Prior CABG

An Academic Research Organization of


Brigham and Women’s Hospital and Harvard Medical School Circulation 2014;130:2354-94

Antithrombotics

• Antiplatelet drugs
– Start with COX Inhibitor (ie, aspirin)
– Almost always add: P2Y12 ADP Receptor Blocker (eg, clopidogrel,
prasugrel, ticagrelor)
– Sometimes also add: glycoprotein IIb/IIIa inhibitors (eg, abciximab,
eptifibatide, tirofiban)

• Anticoagulants (one of the following)


– Unfractionated heparin (UFH)
– Low-molecular-weight heparin: enoxaparin or dalteparin
– Direct thrombin inhibitors: bivalirudin
– Pentasaccharide blockers: fondaparinux

An Academic Research Organization of


Brigham and Women’s Hospital and Harvard Medical School

1017
Copyright © Harvard Medical School, 2018. All Rights Reserved.

Antiplatelet Therapy
Clopidogrel
Prasugrel
Ticagrelor
ADP Receptor (P2Y12)
Abciximab
Eptifibatide Ticlopidine
Tirofiban ADP

ADP
GP IIb/IIIa
receptor
Collagen
Activation Thrombin
TXA2
Fibrinogen COX

TXA2

Aspirin

Give ASA 162-325 mg PO to all patients w/ ACS


(barring contraindication)
ADP = adenosine diphosphate, TXA2 = thromboxane A2, COX = cyclooxygenase.
Adapted from Schafer AI. Am J Med. 1996;101:199-209.

Clopidogrel in Acute Coronary Syndromes


CURE CLARITY-TIMI 28
12,563 Pts w/ NSTEACS 3,491 Pts w/ STEMI
Predominantly managed conservatively Fibrinolytic therapy
14
15

Placebo Placebo
CV Death, MI, or Urg Revasc (%)

12
CV Death, MI, Stroke (%)

10
10

Clopidogrel
8 Clopidogrel

6
Odds Ratio 0.80
5

RR 0.80
4 (95% CI 0.65-0.97)
P=0.001 P=0.026
2
0

0
0 3 6 9 12 0 5 10 15 20 25 30
months days
Yusuf et al. NEJM 2001;345:494 Sabatine MS et al. NEJM 2005;352:1179

1018
Copyright © Harvard Medical School, 2018. All Rights Reserved.

13,608 Patients with ACS and Planned


PCI Randomized to Prasugrel (60/10)
vs. Clopidogrel (300/75)

15

Clopidogrel
12.1 HR 0.81
CV Death / MI / Stroke (0.73-0.90)
10 9.9 P=0.0004
Endpoint (%)

Prasugrel

5
TIMI Major Prasugrel
Non-CABG Bleeds HR 1.32
2.4
(1.03-1.68)
1.8
P=0.03
Clopidogrel
0
0 30 60 90 180 270 360 450
Days
Wiviott SD et al. NEJM 2007;357:2001-15

Primary efficacy endpoint: CV death, MI or stroke


18,624 Patients w/in 24 hrs of onset of ACS
13
12 HR 0.84
Clopidogrel 11.7
11 (95% CI 0.77–0.92)
Cumulative incidence (%)

10 P=0.0003 9.8
9
Ticagrelor
8
7
6
5
4
3 Cardiovascular Death: 4.0% vs. 5.1%, HR 0.79 (0.69-0.91), P=0.001
2 All-cause Mortality: 4.5% vs. 5.9%, HR 0.78 (0.69-0.89), P<0.001
1
0
0 60 120 180 240 300 360
Days after randomisation
No. at risk
Ticagrelor 9,333 8,628 8,460 8,219 6,743 5,161 4,147
Clopidogrel 9,291 8,521 8,362 8,124 6,743 5,096 4,047

K-M = Kaplan-Meier; HR = hazard ratio; CI = confidence interval


Wallentin L, et al. NEJM 2009;361:1045-57

1019
Copyright © Harvard Medical School, 2018. All Rights Reserved.

Bleeding
13 NS
Ticagrelor
12 11.6 Clopidogrel
11.2
11
NS
K-M estimated rate (% per year)

10
NS 8.9 8.9
9
7.9
8 7.7

7 NS
5.8 5.8
6
5
4 p=0.025
2.8
3
2.2
2
NS
1 0.3 0.3
0
PLATO major TIMI major Non-CABG Red cell PLATO life- Fatal bleeding
bleeding bleeding TIMI major transfusion* threatening/
bleeding fatal bleeding
Major bleeding and major or minor bleeding according to TIMI criteria refer to non-adjudicated events analysed with the use
of a statistically programmed analysis in accordance with definition described in Wiviott SD et al. NEJM 2007;357:2001–15;
*Proportion of patients (%); NS = not significant

2014 ACC/AHA NSTEACS Guidelines:


P2Y12 Inhibitors
Recommendation COR LOE

Clopidogrel 300-600 mg load (latter preferred before PCI) → 75 mg/d I B

Prasugrel 60 mg loading dose → 10 mg/d (PCI only) I B

Ticagrelor 180 mg loading dose → 90 mg/bid (initial Rx or PCI) I B

Reasonable to use ticagrelor in preference to clopidogrel (initial Rx IIa B


or PCI)

Reasonable to use prasugrel in preference to clopidogrel (PCI only) IIa B

Prasugrel should not be given if prior stroke or TIA III: Harm B

An Academic Research Organization of


Brigham and Women’s Hospital and Harvard Medical School Circulation 2014;130:2354-94

1020
Copyright © Harvard Medical School, 2018. All Rights Reserved.

Glycoprotein IIb/IIIa Inhibitors


• Potent intravenous antiplatelet drugs

• Typically consider giving at time of PCI

• UA/NSTEMI
– INV Strategy: give at time of PCI; upstream use (ie, prior to
angiography) w/o clear efficacy and increases risk of bleeding
– CONS Strategy: usually no role unless Pt goes for PCI

• STEMI
– Primary PCI: give at time of PCI (not before)
– Fibrinolysis: CONTRAINDICATED

Coagulation Cascade & Anticoagulants


AMPLIFICATION (“Intrinsic”) INITIATION (“Extrinsic”)
(FXII, FXI, platelet membrane) (Tissue Factor, FVIIa, Ca2+) Warfarin

Intrinsic Extrinsic
IX Tissue Factor IXa Tenase Tenase Rivaroxaban (PO)
Apixaban (PO)
Edoxaban (PO)
VIII Thrombin VIIIa

V Thrombin Va
V X Xa ATIII Fondaparinux
Prothrombinase
ATIII LMWH

ATIII UFH

Prothrombin Thrombin Fibrin Formation

Bivalirudin (IV)
Platelet Dabigatran (PO)
An Academic Research Organization of
Brigham and Women’s Hospital and Harvard Medical School
Activation

1021
Copyright © Harvard Medical School, 2018. All Rights Reserved.

Anticoagulants Acutely
• Unfractionated heparin (UFH)
– Most commonly used; fast on & fast off; reversible
– Wt-based dosing; unpredictable PD, requiring PTT
– Compared with no anticoagulation: D/MI by ~33%

• Low-molecular weight heparin (LMWH)


– More predictable PD; not as reliably reversed
– Compared with UFH: 9% D/MI, may bleeding
– Consider in conservatively managed patients

• Bivalirudin
– Fast on & fast off
– Compared with heparin: 9% MACE, 38% stent thrombosis,
bleeding (especially if compared with UFH+GP Iib/IIIa inhibitor)
– Consider in invasively managed Pts, espec if at high risk for bleeding
An Academic Research Organization of
Oler et al. JAMA 1996;276:811; JAMA 2004; 292: 45, 55, & 89
Brigham and Women’s Hospital and Harvard Medical School
Cavender MA and Sabatine MS. Lancet 2014;384:599-606

CURE: Long-term benefit of clopidogrel


12,562 Patients with NSTEACS (mostly conservatively managed)

CV Death, MI, or Stroke CV Death, MI, or Stroke


First 30 Days >30 Days–1 Year
1.00 1.00
Proportion Event-Free
Proportion Event-Free

Clopidogrel .98 Clopidogrel


.98

.96 .96
Placebo Placebo
.94 .94
RRR: 21% RRR: 18%
.92 95% CI, 0.67–0.92 .92 95% CI, 0.70–0.95
P=.003 P=.009
.90 .90
Week 0 1 2 3 4 Month 1 4 6 8 10 12
No. at Risk No. at Risk
Clopidogrel 6259 6145 6070 6026 5990 5981 5481 4742 4004 3180 2418
Placebo 6303 6159 6048 5993 5965 5954 5390 4639 3929 3159 2388

Yusuf S, et al. Circulation. 2003;107:966-972.

1022
Copyright © Harvard Medical School, 2018. All Rights Reserved.

Continued Divergence of Event Curves With


More Potent Long-Term P2Y12 Inhibition
8 8
6.9
6.6
Clopidogrel Clopidogrel

CV Death, MI, or Stroke (%)


CV Death, MI, or Stroke (%)

6 6

5.6 5.3
4
4
Ticagrelor
Prasugrel

2
2 HR 0.80 HR 0.80
P=0.003 P<0.001
1

0
0
3 30 60 90 180 270 360 450 31 90 150 210 270 330
Days after randomisation Days after randomisation
Wiviott SD et al. NEJM 2007;357:2001-15 Wallentin L, et al. NEJM 2009;361:1045-57

Ticagrelor in Patients w/ Prior MI


10
21,162 Patients w/ MI 1-3 years prior
9 All on low-dose ASA Placebo (9.0%)
Median follow-up 33 months
8 Ticagrelor 90 (7.8%)
CV Death, MI, or Stroke (%)

Ticagrelor 60 (7.8%)
7

Ticagrelor 90 mg
4
HR 0.85 (95% CI 0.75 – 0.96)
3 P=0.008
Ticagrelor 60 mg
2
HR 0.84 (95% CI 0.74 – 0.95)
1 P=0.004

0
0 3 6 9 12 15 18 21 24 27 30 33 36
Months from Randomization

An Academic Research Organization of


Brigham and Women’s Hospital and Harvard Medical School Bonaca MP et al. and Sabatine MS. NEJM 2015; 372:179

1023
Copyright © Harvard Medical School, 2018. All Rights Reserved.

Efficacy of Adding Low-dose NOAC to


DAPT post-ACS

CV Death / MI / Stroke Cardiovascular Death All Cause Death


5% 5%
HR 0.85 HR 0.62 HR 0.64
12% Placebo Placebo Placebo
Estimated Cumulative incidence (%)

mITT 10.4% mITT mITT 4.5%


p=0.04 p<0.001 4.2% p<0.001
9.0%
ITT ITT ITT
p=0.01 p<0.001 p<0.001
2.7%
2.5%

Rivaroxaban Rivaroxaban Rivaroxaban


2.5 mg BID 2.5 mg BID 2.5 mg BID
NNT = 71 NNT = 59 NNT = 56
0 12 24 0 12 24 0 12 24
Months Months Months

Mega JL et al. NEJM 2012;366:9-19

Long-term Low-Dose Anticoagulation

COMPASS Trial: 27,395 patients with stable ASCVD

5 mg bid

2.5 mg bid

Years

An Academic Research Organization of NEJM 2017;377:1319


Brigham and Women’s Hospital and Harvard Medical School

1024
Copyright © Harvard Medical School, 2018. All Rights Reserved.

“Triple Therapy” (DAPT + OAC)


Favors less intensive antithrombotic Rx Favors more intensive
(eg, dose NOAC and/or antiplt monoRx) antithrombotic Rx

Coronary Risk Acute MI


Elective PCI
Multiple, long, narrow (antiplt)
Single next gen DES
stents

Stroke Risk Prior stroke or high CHA2DS2-VASc


CHA2DS2-VASc = 1 LV thrombus
(anticoag)
Large VTE

HAS-BLED ≥3 Bleed Risk


(esp. h/o bleed, ICH, HAS-BLED ≤2
elderly, renal dysfxn)

An Academic Research Organization of


Brigham and Women’s Hospital and Harvard Medical School

Beta-Blockers: Clinical Data


1884 Patients 1-4 weeks after acute MI
Randomized to β-blocker vs. placebo

45% risk reduction 28% risk reduction


P=0.0001 P=0.0006

Norwegian Multicenter Study Group. NEJM 1981;304:801.

1025
Copyright © Harvard Medical School, 2018. All Rights Reserved.

PROVE IT – TIMI 22
4162 patients hospitalized w/in prior 10 d for ACS

30 Pravastatin 40 mg
Death or Major CV Events (%)

(avg achieved LDL = 95 mg/dl)


25

20
Atorvastatin 80 mg
15 (avg achieved LDL = 62 mg/dl)

10
16% RR
5
(P = 0.005)

0
0 3 6 9 12 15 18 21 24 27 30
Months of Follow-up
Cannon et al. NEJM 2003; 350: 1495

Primary Endpoint — ITT


Cardiovascular death, MI, documented unstable angina requiring
rehospitalization, coronary revascularization (≥30 days), or stroke
HR 0.936 CI (0.887, 0.988) Simva alone
p=0.016 (achieved LDL-C 34.7%
69.5 mg/dL) NNT= 50
32.7%

EZE + Simva
(achieved LDL-C
53.7 mg/dL)

7-year event rates


NEJM 2015;372:2387-97

1026
Copyright © Harvard Medical School, 2018. All Rights Reserved.

IMPROVE-IT vs. CTT:


Ezetimibe vs. Statin Benefit

. IMPROVE-IT

CTT Collaboration.
Lancet 2005; 366:1267-78; Using CTT methods: LDL difference between groups using baseline LDL for Pts without blood
Lancet 2010;376:1670-81. samples. Endpoint of CV Death, MI, stroke or revasc >30days post Rand. Cox HR reported.

Summary of Effects of
PCSK9i Evolocumab
• ↓ LDL-C by 59% down to a median of 30 mg/dl
• ↓ CV outcomes in patients on statin
• Safe and well-tolerated HR 0.85 (0.79-0.92)
100
Placebo P<0.0001
15 14.6
80 12.6 HR 0.80 (0.73-0.88)
LDL Cholesterol (mg/dl)

59% reduction P<0.0001


KM Rate (%) at 3 Years

P<0.00001
60 9.9
10
Absolute 56 mg/dl 7.9
40

5
20 Evolocumab
(median 30 mg/dl, IQR 19-46 mg/dl)

0 0
0 24 48 72 96 120 144 168 CVD, MI, stroke CVD, MI, stroke
Weeks after randomization UA, cor revasc
An Academic Research Organization of
Brigham and Women’s Hospital and Harvard Medical School
Sabatine MS et al. NEJM 2017;376:1713-22

1027
Copyright © Harvard Medical School, 2018. All Rights Reserved.

ACC 2018

Non-Statin Therapy

1028
Copyright © Harvard Medical School, 2018. All Rights Reserved.

LDL Cholesterol
2034 patients w/ baseline LDL-C<70 mg/dL
100

90
Placebo
80 (median 66 mg/dl, IQR 56-78 mg/dl)
(median 1.7 mmol/L, IQR 1.4-2.0 mmol/L)
LDL Cholesterol (mg/dl)

70

60

50
66% mean reduction (95%CI 62-69), P<0.00001
40

30

20
Evolocumab
10 (median 21 mg/dl, IQR 11.5-37 mg/dl)
(median 0.5 mmol/L, IQR 0.3-1.0 mmol/L)
0
0 12 24 36 48 60 72 84 96 108 120 132 144
Weeks
An Academic Research Organization of
Brigham and Women’s Hospital and Harvard Medical School
Giugliano RP et al. and Sabatine MS. JAMA Cardiol 2017;2:1385-91

Clinical Outcomes
by Baseline LDL-C
CVD, MI, stroke, UA, or cor revasc HR (95% CI) Pinteraction

All Patients 0.85 (0.79-0.92)

Baseline LDL-C <70 mg/dL 0.80 (0.60-1.07)


0.65
Baseline LDL-C ≥70 mg/dL 0.86 (0.79-0.92)

0.4 1.0 2.5


CVD, MI, or stroke

All Patients 0.80 (0.73-0.88)

Baseline LDL-C <70 mg/dL 0.70 (0.48-1.01)


0.44
Baseline LDL-C ≥70 mg/dL 0.81 (0.73-0.89)

0.4 1.0 2.5


EvoMab better Pbo better

An Academic Research Organization of


Brigham and Women’s Hospital and Harvard Medical School
Giugliano RP et al. and Sabatine MS. JAMA Cardiol 2017;2:1385-91

1029
Copyright © Harvard Medical School, 2018. All Rights Reserved.

CV Death, MI, Stroke

P = 0.0001

mM
19 39 58 77 97 116 135 155 174 mg/dL

An Academic Research Organization of


LDL-C at 4 weeks
Brigham and Women’s Hospital and Harvard Medical School
Giugliano RP et al. & Sabatine MS. Lancet 2017;309:1962-71

ACE Inhibitors Post-MI with EF <40% but asx

19% Reduction in
Mortality

Pfeffer MA et al. NEJM 1992;327:669-677.

1030
Copyright © Harvard Medical School, 2018. All Rights Reserved.

ACE Inhibitors in All Acute MI

Greater apparent benefit


7% Reduction in
Mortality in anterior STEMI
than in inferior STEMI or
NSTEACS

ISIS-4. Lancet 1995;345:669.

Aldosterone Antagonists
6632 patients with recent MI and EF <40% w/ either HF sx or diabetes
All-cause mortality

Pitt B et al. NEJM 2003;348:1309-1321.

1031
Copyright © Harvard Medical School, 2018. All Rights Reserved.

Summary
• Diagnose ACS using H&P, 12-lead ECG, troponin
• For STEMI: select Primary PCI vs Lytic
• For UA/NSTEMI: select Invasive (eg, ⊕ Tn) vs. Conservative Strategy
• Anti-ischemic Rx: beta-blocker, nitrates
• Select Antiplatelet Regimen
– ASA
– + P2Y12 Inhibitor: ticagrelor, prasugrel, or clopidogrel
– ? + GP IIb/IIIa inhibitor (typically at time of PCI)
• Select Anticoagulant: UFH, LMWH, or bivalirudin (or fondaparinux)
• Long-term therapy
– ASA, P2Y12 inhibitor (at least 12 mos, if not longer)
– β-blocker, statin (+ EZE + PCSK9i)
– ? ACEI, ? Aldo inhibitor
An Academic Research Organization of
Brigham and Women’s Hospital and Harvard Medical School

Question #1
A 67 year old diabetic woman presents with substernal chest pain
at rest for 15 minutes that, after beta-blocker and nitrates, has
partially but not completely resolved. A 12-lead ECG reveals
inferior ST-segment depressions. Cardiac troponin is elevated.

In addition to ASA, the most appropriate treatment strategy would


be:

a. UFH, GP 2b/3a inhibitor, stress test in 24-48 hrs


b. Clopidogrel, bivalirudin, stress test in 24-48 hrs
c. Enoxaparin, cardiac catheterization in 48 hrs
d. UFH, cardiac cath w/in 12-24 h, ticagrelor ± GP 2b/3a if PCI

1032
Copyright © Harvard Medical School, 2018. All Rights Reserved.

Question #1
A 67 year old diabetic woman presents with substernal chest pain at rest for 15
minutes that, after beta-blocker and nitrates, has partially but not completely
resolved. A 12-lead ECG reveals inferior ST-segment depressions. Cardiac
troponin is elevated. In addition to ASA, the most appropriate treatment strategy
would be:

a. UFH, GP 2b/3a inhibitor, stress test in 24-48 hrs


[no, GP 2b/3a for INV managed patients]
b. Clopidogrel, bivalirudin, stress test in 24-48 hrs
[no, bival for INV managed patients]
c. Enoxaparin, cardiac catheterization in 48 hrs
[no, need to add P2Y12 inhib]
d. UFH, cardiac cath w/in 12-24 h, ticagrelor ± GP 2b/3a if PCI
[yes, approp antiplt, anticoag, and management]

Question #2
You are considering the optimal lipid management in this same 67
year old diabetic woman.
LDL-C on admission (not on any lipid-lowering Rx) was 180 mg/dL.
She was started on atorva 80 mg.
What else would you recommend?

a. Target LDL-C reduction of 50%


b. Target LDL-C of 70 mg/dL
c. Add ezetimibe
d. Add PCSK9 inhibitor
e. Add ezetimibe and a PCSK9 inhibitor

1033
Copyright © Harvard Medical School, 2018. All Rights Reserved.

Question #2
You are considering the optimal lipid management in this same 67 year old
diabetic woman.
LDL-C on admission (not on any lipid-lowering Rx) was 180 mg/dL.
She was started on atorva 80 mg.
What else would you recommend?

a. Target LDL-C reduction of 50%


[OK, but after A80, LDL-C still likely >70 mg/dL]
b. Target LDL-C of 70 mg/dL
[OK, but we know clinical benefit to adding EZE or PCSK9i in
those with LDL-C <70]
c. Add ezetimibe
[yes, but only ~24% LDL-C reduction beyond statin]
d. Add PCSK9 inhibitor
[yes, ~60% LDL-C reduction & ~20% MACE reduction]
e. Add ezetimibe and a PCSK9 inhibitor
[yes, cannot be too low!]

Key References

• O’Gara et al. Circulation 2013;127:e362

• Amsterdam et al. Circulation 2014;130:2354

• Bonaca et al. N Engl J Med 2015;372:179

• Sabatine et al. N Engl J Med 2017;376:1713

1034
Copyright © Harvard Medical School, 2018. All Rights Reserved.

PE, DVT, ANTICOAGULATION


Samuel Z. Goldhaber, MD
Interim Chief, Division of
Cardiovascular Medicine
Section Head, Vascular Medicine
Brigham and Women’s Hospital
Professor of Medicine
Harvard Medical School

DISCLOSURES
Research Support:
Boehringer-Ingelheim; BMS; BTG
EKOS; Daiichi; Janssen; NHLBI;
Thrombosis Research Institute
Consultant:
Agile; Bayer; Boehringer-Ingelheim;
BMS; Daiichi; Janssen; Portola;
Soleno

1035
Copyright © Harvard Medical School, 2018. All Rights Reserved.

LEARNING OBJECTIVES
• Optimal duration and intensity of
anticoagulation after provoked or
unprovoked PE/ DVT
• Anticoagulation Rx of patients with
cancer and VTE—role of edoxaban
• Advanced Rx for acute PE beyond
anticoagulation—low-dose TPA
• Prevention of VTE in hospitalized
medically ill patients--betrixaban

EXTENDED DURATION
ANTICOAGULATION
• A 44 y.o. obese (BMI=32) man suffered
a flare of ulcerative colitis requiring
hospitalization. He suffered acute
submassive PE and was treated with
rivaroxaban for 6 months. Negative
hypercoag workup.
• At his 6-month visit in the office, he asks:
“May I discontinue anticoagulation?”
“Can you switch me to ‘baby aspirin’?”

1036
Copyright © Harvard Medical School, 2018. All Rights Reserved.

VOTING CASE #1:


Based on the EINSTEIN-CHOICE
Trial (NEJM 2017), you advise:

A.Rivaroxaban 20 mg with dinner


B.Rivaroxaban 10 mg with dinner
C.Aspirin 81 mg daily
D.No further medical therapy
E.Obtain a D-dimer to help decide

EINSTEIN CHOICE: 60%


PROVOKED and 40% UNPROVOKED
VTE—PREVENTING RECURRENCE

N=3,396

(Weitz JI. Thromb Haemost 2015; 114: 645-650)

1037
Copyright © Harvard Medical School, 2018. All Rights Reserved.

RECURRENT VTE:
PROVOKED vs UNPROVOKED
Riva 20 Riva 10 mg ASA 100
mg mg
Provoked 1.4% 0.9% 3.6%

Unprovoked 1.8% 1.5% 5.6%

(Weitz JI. NEJM 2017; March 17)

EFFICACY: RIVAROXABAN
20 MG VERSUS 10 MG

(Weitz JI. NEJM 2017; March 17)

1038
Copyright © Harvard Medical School, 2018. All Rights Reserved.

SAFETY: RIVAROXABAN 20 MG
VERSUS 10 MG

(Weitz JI. NEJM 2017; March 17)

PROVOKED, MINOR PERSISTING


RISK FACTORS PREDISPOSE TO
RECURRENCE
• Inflammatory bowel disease
• Lower limb paresis or paralysis
• Congestive heart failure
• Obesity
• Family history of VTE
• Hereditary thrombophilia

1039
Copyright © Harvard Medical School, 2018. All Rights Reserved.

PROVOKED, MINOR TRANSIENT


RISK FACTORS PREDISPOSE TO
RECURRENCE
• Immobilization
• Travel > 8 hours
• Estrogen, pregnancy
• Leg injury with impaired mobility

RECURRENT VTE:
EINSTEIN CHOICE/ EXTENSION
Riva 10/20 Placebo/ ASA
mg, N=2,832 100 mg, N=1,721
Unprovoked 1.6% 6.5%
Cancer 0% 4.6%

Minor 1.5% 4.9%


Persisting
Min. Transient 0.4% 4.5%
Major Surgery 0% 0%
(Prins MH, ISTH 2017)

1040
Copyright © Harvard Medical School, 2018. All Rights Reserved.

High VTE
(Prandoni. Recurrence
Haematolo- Rate
gica 2007;
92: 199-
205)

(N=1,626
DVT
patients)

CHEST ACCP GUIDELINES 2016:


DURATION OF RX
If unprovoked with low to moderate
bleeding risk, we suggest extended
anticoagulant therapy (no
scheduled stop date) over 3 months
of therapy (Grade 2B).
If provoked by surgery or a
nonsurgical transient risk factor,
anticoagulate for 3 months (Grade
1B).
(CHEST 2016; 149: 315-352)

1041
Copyright © Harvard Medical School, 2018. All Rights Reserved.

PE: FILLED WITH WBCs and


PLATELETS—INFLAMMATION

(Savchenko AS.
J Thromb Haemostas
2014; 12: 860-870)

THROMBIN-
INDUCED
INFLAMMATION
LEADS TO Aspirin
THROMBOSIS
(Croce K, Libby P.
Intertwining of
thrombosis and
inflammation in
atherosclerosis.
Curr Opin Hematol
2007;14: 55)

1042
Copyright © Harvard Medical School, 2018. All Rights Reserved.

ASPIRE & WARFASA: 32% LESS


VTE WITH LOW-DOSE ASPIRIN!
(N=1,224)

(Brighton TA et al. NEJM 2012; 367: 1979-1987)

D-DIMER DISAPPOINTS re:


EXCLUDING RECURRENT VTE:
Unprovoked VTE; 5 months warfarin;
Average 2.2 years follow-up (N=319)
Gender Negative D-dimer: Positive D-dimer:
Recurrence Recurrence

Men 8% per year 16% per year

Women 5% per year 10% per year

(Kearon C. Ann Intern Med 2015; 162: 27-34)

1043
Copyright © Harvard Medical School, 2018. All Rights Reserved.

NOACS FOR EXTENDED


TREATMENT OF VTE
RE- Dabigatran vs. ↓93%
SONATE placebo

RE-MEDY Dabigatran vs. Non-inferior


warfarin, INR 2-3
EINSTEIN Rivaroxaban vs. ↓82%
-EXT placebo
AMPLIFY- Apixaban vs. placebo ↓81%
EXT

CORE EXPLANATIONS: HIGH


VTE RECURRENCE RATE
• Persistent VTE risk factors (such as
obesity, diabetes, metabolic
syndrome, sedentary lifestyle)
• Persistent inflammation (that may
wax and wane in intensity)
• Persistent hypercoagulability:
genetic or acquired (e.g.,
anticardiolipin antibodies)

1044
Copyright © Harvard Medical School, 2018. All Rights Reserved.

EINSTEIN CHOICE LESSONS


1. VTE: chronic inflammatory
disease, high recurrence rate
after discontinuing
anticoagulation
2. For extended duration
anticoagulation, Rivaroxaban:
70% more effective and as safe
as aspirin to prevent recurrent
provoked/ unprovoked VTE

VOTING CASE #1:


Based on the EINSTEIN-CHOICE
Trial (NEJM 2017), you advise:

A.Rivaroxaban 20 mg with dinner


B.Rivaroxaban 10 mg with dinner
C.Aspirin 81 mg daily
D.No further medical therapy
E.Obtain a D-dimer to help decide

1045
Copyright © Harvard Medical School, 2018. All Rights Reserved.

CAN DOACS REPLACE


LMWH IN CANCER
PATIENTS WITH VTE?

CANCER AND ACUTE VTE


• A 65 y.o. woman with esophageal
cancer was hospitalized with difficulty
swallowing and newly discovered lung
nodules. She had a 40 PPD cigarette
smoking history and had quit 2 years
ago.
• During hospitalization, she became
SOB and was diagnosed with
submassive PE.

1046
Copyright © Harvard Medical School, 2018. All Rights Reserved.

VOTING CASE #2:


Based on the HOKUSAI VTE Cancer
Trial (NEJM 2017), you advise:
A.Rivaroxaban 20 mg with dinner
B.Rivaroxaban 10 mg with dinner
C.Enoxaparin as monotherapy
D.Warfarin (after enoxaparin bridge)
E.Edoxaban (after enoxaparin
bridge)

CLOT TRIAL for cancer pts:


Reduction in Recurrent VTE
25
Risk
Recurrent VTE, %

Recurrent reduction =
20
VTE 52% WARFARIN
15
p-value =
10 0.0017
DALTEPARIN
5

0
Lee et al.
NEJM 2003; 0 30 60 90 120 150 180 210

349: 146 Days Post Randomization

1047
Copyright © Harvard Medical School, 2018. All Rights Reserved.

HOKUSAI VTE CANCER


TREATMENT TRIAL

(Van Es N. Thromb Haemost 2015; 114: 1268-


76)

HOKUSAI VTE CANCER:


EDOXABAN vs. DALTEPARIN
(N=1046)
• Primary outcome: Recurrent VTE or major
bleeding 12 months after randomization
• Edoxaban had more major bleeding
(especially upper GI) and fewer recurrent
VTE events.
• Edoxaban was noninferior to dalteparin
with respect to the primary outcome.
(Raskob GE. NEJM 2017; epubl December 12)

1048
Copyright © Harvard Medical School, 2018. All Rights Reserved.

PRIMARY OUTCOME

(Raskob GE. NEJM 2017; epubl December 12)

RECURRENT VTE

(Raskob GE. NEJM 2017; epubl December 12)

1049
Copyright © Harvard Medical School, 2018. All Rights Reserved.

MAJOR BLEEDING

(Raskob GE. NEJM 2017; epubl December 12)

HOKUSAI VTE CANCER:


EDOXABAN vs. DALTEPARIN
• The increase in upper GI bleeding
occurred mainly in patients with GI
cancer.
• These data provide clinicians and
cancer/ VTE patients with a rationale to
discuss prescription of oral edoxaban
rather than LMWH monotherapy
injections.
(Raskob GE. NEJM 2017; epubl December 12)

1050
Copyright © Harvard Medical School, 2018. All Rights Reserved.

VOTING CASE #2:


Based on the HOKUSAI VTE Cancer
Trial (NEJM 2017), you advise:
A.Rivaroxaban 20 mg with dinner
B.Rivaroxaban 10 mg with dinner
C.Enoxaparin as monotherapy
D.Warfarin (after enoxaparin bridge)
E.Edoxaban (after enoxaparin
bridge)

ADVANCED MANAGEMENT
OF PE:

WHEN ANTICOAGULATION
ALONE MIGHT NOT SUFFICE

1051
Copyright © Harvard Medical School, 2018. All Rights Reserved.

62 y.o. woman; BP=102/60; HR=134


RR=30; Oxygen saturation=89%;
Thrombolysis or Heparin Alone?

1052
Copyright © Harvard Medical School, 2018. All Rights Reserved.

VOTING CASE #3:

A.Continue IV heparin monotherapy


B.Switch to enoxaparin monotherapy
C.TPA 100 mg/ 2h—peripheral vein
D.TPA 50 mg/ 2h—peripheral vein
E.Ultrasound-facilitated catheter-
directed thrombolysis—24 mg TPA
or lower OPTALYSE dose

WHAT ABOUT PE DEATH RATE


AFTER HOSPITAL DISCHARGE?

• 8% die within 30 days (mostly of


recurrent PE)
• 20% die within 6 months

(Minges KE. Am J Cardiol 2015; 116: 1436-1442)

1053
Copyright © Harvard Medical School, 2018. All Rights Reserved.

HIGH PE MORTALITY at 30d


and 6 months; HIGH 30d
READMISSION RATES

(Minges KE. Am J Cardiol 2015; 116: 1436-1442)

PE THROMBOLYSIS: TREND
TOWARD LOWER DOSES
OVER THE PAST 30 YEARS
• Lower doses: associated with less
major and less minor bleeding
• Lower doses and lower durations of
infusion: lower the cost of therapy
and lower the hospital length of stay

1054
Copyright © Harvard Medical School, 2018. All Rights Reserved.

PE THROMBOLYSIS
1. Systemic thrombolysis:
a) 100 mg/2h (“full dose”) TPA
(FDA Approved 1990)
b) 50 mg TPA (“half-dose”)
(MOPETT (AJC 2013; 111: 273)
2. Catheter-directed, Ultrasound-facilitated
TPA 24 mg (“1/4 dose”)
(FDA Approved 2014)
3. OPTALYSE-PE Dose: TPA 8 mg/2h
(“< one-tenth dose”)
(JACC Cardiovasc Interventions 2018)

LYSIS IN SUBMASSIVE PE:


MORTALITY META-ANALYSIS

(JAMA 2014; 311: 2414-2421)

1055
Copyright © Harvard Medical School, 2018. All Rights Reserved.

SUBMASSIVE PE: 50 mg TPA


versus heparin (MOPETT) (N=121)
TPA 10mg/1 min followed by 40 mg/2 h
No bleeding complications/ no ICH
PA Pressure (ECHO) lower with TPA
TPA PA systolic Controls P value
Admission 50 51 0.4
Within 48h 34 41 < 0.001
6 months 31 49 < 0.001
28 months 28 43 < 0.001
(Am J Cardiology 2013; 111: 273-277)

1056
Copyright © Harvard Medical School, 2018. All Rights Reserved.

TPA 24 mg/12h = total TPA


dose

TPA 12 mg TPA 12 mg

ULTRASOUND-FACILITATED CDT:
Improves Blood Flow in Distal Pulmonary
Arteries (Microvasculature)

(Piazza G, et al. AHA 2017)

1057
Copyright © Harvard Medical School, 2018. All Rights Reserved.

CDT VS. HEPARIN: SUBMASSIVE PE


CDT HEPARIN

Lyse PE Yes No

Rapidly reverse Yes No


pulmonary
hypertension
Rapidly reduce RV Yes No
size
Clean out Probably No
pulmonary
microvasculature

1058
Copyright © Harvard Medical School, 2018. All Rights Reserved.

IVC FILTERS VS. CONTROLS

(Bikdeli B. JACC 2017; 70: 1587-97)

1059
Copyright © Harvard Medical School, 2018. All Rights Reserved.

VOTING CASE #3:

A.Continue IV heparin monotherapy


B.Switch to enoxaparin monotherapy
C.TPA 100 mg/ 2h—peripheral vein
D.TPA 50 mg/ 2h—peripheral vein
E.Ultrasound-facilitated catheter-
directed thrombolysis—24 mg TPA
or lower OPTALYSE dose

AN OUNCE OF
PREVENTION IS
WORTH A POUND
OF CURE

1060
Copyright © Harvard Medical School, 2018. All Rights Reserved.

53

VTE AFTER HOSPITAL DISCHARGE


• Despite in-hospital VTE prophylaxis
with UFH or LMWH, > 400,000 VTE
events occur in this patient population
annually.

• About half occur within 6 weeks of


discharge. Prior to betrixaban, no
agents were FDA approved for
extended duration prophylaxis.

BWH: READMISSION with NEW VTE within


30 DAYS OF HOSPITAL DISCHARGE

1061
Copyright © Harvard Medical School, 2018. All Rights Reserved.

BETRIXABAN
• Factor Xa inhibitor

• 19-25 hour half life

• Low peak-to-trough ratio

• 17% renal clearance

• Antidote (Andexanet)

APEX STUDY DESIGN


Standard
Prophylaxis Extended Prophylaxis
10 ± 4 days 35 – 42 days Evaluation

Enoxaparin
Subjects enrolled

Placebo Ultrasound Follow-up


40 mg & Visit 3 safety visit
(N=7,513)

Double blind, double dummy


1:1
R

30 Days
Day 35 After Visit 3
Betrixaban Betrixaban
(+7 days) (+5 days)
80 mg 80 mg

Loading dose
160 mg

(NEJM 2016; 375: 534-544)

1062
Copyright © Harvard Medical School, 2018. All Rights Reserved.

Baseline Characteristics:
Primary Admission Diagnoses
Enoxaparin Betrixaban
(n=3,754) (n=3,759)
44.6%
Acute CHF NYHA III-IV, % (n) 44.5% (1,672)
(1,677)
29.6%
Acute infection, % (n) 28.2% (1,058)
(1,112)
Acute respiratory failure, % (n) 12.6% (474) 11.9% (448)
Acute ischemic stroke w/ immobilization,
% (n) 11.5% (432) 10.9% (411)

Acute rheumatic disorder, % (n) 3.1% (117) 2.9% (109)

(NEJM 2016; 375: 534-544)

SUMMARY: BETRIXABAN vs
ENOXAPARIN

• 24% reduction in VTE


• 46% reduction in symptomatic VTE
• 30% reduction fatal, irreversible
events
• 56% reduction VTE-related
rehospitalization
• 56% reduction in stroke
• No increase in major or fatal bleeding

1063
Copyright © Harvard Medical School, 2018. All Rights Reserved.

TAKE HOME POINTS


1. For extended duration therapy,
Rivaroxaban: 70% more effective and
as safe as aspirin to prevent recurrent
provoked/ unprovoked VTE
2. Edoxaban is an alternative to LMWH in
non-GI cancer patients.
3. Critically ill PE patients warrant
thrombolysis or embolectomy.
4. Betrixaban: a new paradigm for VTE
prevention during/ after hospitalization

What parameters are used to


determine advanced PE therapy?
A. Clinical assessment
B. Right ventricular size
C. Right ventricular function
D. Troponin elevation
E. All of the above

1064
Copyright © Harvard Medical School, 2018. All Rights Reserved.

What parameters are used to


determine advanced PE therapy?
A. Clinical assessment
B. Right ventricular size
C. Right ventricular function
D. Troponin elevation
E. All of the above

After completing 6 months of full


anticoagulation for acute VTE,
A. Aspirin is no more effective than
placebo.
B. Aspirin has significantly fewer major
bleeding complications than NOACs.
C. Aspirin is as effective as NOACs.
D. Aspirin is more effective than placebo
E. Aspirin does not cause major
bleeding

1065
Copyright © Harvard Medical School, 2018. All Rights Reserved.

References
• (NEJM 2016; 375: 534-544)
• (NEJM 2016; 375: 534-544)
• (Am J Cardiology 2013; 111: 273-277)
• (Van Es N. Thromb Haemost 2015; 114:
1268-76)
• (Prins MH, ISTH 2017)

1066
Copyright © Harvard Medical School, 2018. All Rights Reserved.

Valvular and Congenital Heart


Disease

Brendan M. Everett, MD, MPH


Brigham and Women’s Hospital
Harvard Medical School

Disclosures
• I have no financial disclosures relevant
to the topics of valvular and congenital
heart disease

1067
Copyright © Harvard Medical School, 2018. All Rights Reserved.

Aortic Stenosis

Aortic Stenosis

1068
Copyright © Harvard Medical School, 2018. All Rights Reserved.

Aortic Stenosis
Obstruction

Afterload Hypertrophy Cor flow

Diastolic O2
dysfunction mismatch

Natural History of Aortic Stenosis


Onset severe
Angina
symptoms
100 Syncope
Failure
80 period
Latent period,
(increasing
increasing obstruction,
60 myocardial overload) 0 2 4 6
hypertrophy
Avg
Yrs survival (yr)
after onset
40
Average death
20
age ( )
0
0 40 50 60 70 80
Age (yr)
Ross J Jr. and Braunwald E: Circ 38(Suppl 5):61, 1968
CP1061785-6

1069
Copyright © Harvard Medical School, 2018. All Rights Reserved.

Aortic Stenosis
Classification of Severity

MILD MODERATE SEVERE

Jet Velocity (m/s) < 3.0 3.0 - 4.0 > 4.0

Mean Gradient (mmHg) < 25 25-40 > 40

Valve Area (cm2) > 1.5 1.0-1.5 ≤ 1.0

Valve Area Index (cm2/m2) ≤ 0.6

Zoghbi W et al. J Am Soc Echocardiogr 2003; 16:777.

Severe Aortic Stenosis


Indications for Replacement

Indication ACC/AHA1 ESC2


2014 2012

Symptoms I(B) I(B)


Need for CABG/Ao Surgery I(B) I(C)
LVEF < 0.50 I(B) I(C)

Abnormal ETT IIa(B) IIa or IIb(C)


Rapid Progression/Delay IIb(C) IIa(C)
Very severe AS IIa(B) IIa(C)
Severe LVH (≥15mm) ----- IIb(C)
1. Nishimura 2014 ACC/AHA guidelines JACC 2014
2. Vahanian A et al. European Heart Journal 2012

1070
Copyright © Harvard Medical School, 2018. All Rights Reserved.

Surgery in Asymptomatic Severe AS


Class LOE

LVEF <0.50 not due to another cause I C

Symptoms on ETT due to AS I C

Fall in BP on ETT IIa C

Very severe AS (Vmax>5.5m/s); rapid IIa C


progression (>0.3m/s/y) & severe
calcification
Markedly elevated BNP; exercise increase in IIb C
gradient >20mm Hg; excessive LVH
ESC 2012 VHD Guidelines. Euro Heart J. 2012; 33: 2451-2496

TAVR has not been evaluated for asymptomatic patients with severe AS. These
patients should have SAVR if at low or intermediate surgical risk.

Severe AS (AVA < 1cm2)

High-Gradient AS Low-Gradient AS

Mean gradient > 40 mm Hg AVA < 1.0 cm2


Peak velocity > 4 m/s

AVR Reduced EF Preserved EF

Clinical question: Clinical question:


Is the EF low because Am I ignoring AS
of the AS? Or is AVA because the gradients
small because of the are low? Is this low-
low EF? gradient, low-flow AS?

Otto CM, et al J Am Coll Cardiol. 2017 Mar 14;69(10):1313–46.

1071
Copyright © Harvard Medical School, 2018. All Rights Reserved.

Severe AS with Low EF


Look at the Valve!

Dobutamine Stress Echo (DSE)

↑ CO ↑ CO ↔ CO
↑ Gradient ↑ Gradient ↔ Gradient
↑ AVA > 1.0 cm2 AVA < 1.0 cm2 AVA < 1.0 cm2
CR ?
AVR, TAVR ?

Paradoxical Low Flow Aortic Stenosis

1. Look at the valve!

2. Look at the SVi!

Stroke Volume Stroke Volume


index > 35 mL/m2 index ≤ 35 mL/m2

Pibarot P et al. J Am Coll Cardiol Img 2009;2:400-403

1072
Copyright © Harvard Medical School, 2018. All Rights Reserved.

AVR in Low-Flow/Low-Gradient AS

Recommendations Class LOE

AVR is reasonable in symptomatic patients IIa C


with low-flow/low-gradient severe AS with
reduced LVEF when low-dose dobutamine
stress shows AV velocity ≥ 4 m/s (or mean
gradient ≥ 40 mm Hg) with an AVA ≤1.0 cm2
at any dobutamine dose

AVR is reasonable in symptomatic patients IIa C


with low-flow/low-gradient severe AS,who
are normotensive and have LVEF ≥ 50%

ACC/AHA Guideline: J Am Coll Cardiol. 2014 Mar 3;63(22):2438–88.

PARTNER Cohort B: Medical Therapy vs. TAVR


All Cause Mortality

Standard Rx
TAVR
HR [95% CI] =
100 0.54 [0.38, 0.78]
All-cause mortality (%)

P (log rank) < 0.0001


80

60

40

20

0
0 6 12 18 24
Months
Numbers at Risk
TAVR 179 138 122 67 26
Standard Rx 179 121 83 41 12

1073
Copyright © Harvard Medical School, 2018. All Rights Reserved.

TAVR vs. SAVR:


Partner A and CoreValve

A: Sapien 3 (Partner A)
B: CoreValve (U.S. Pivotal Trial)

Vahl TP, et al.. J Am Coll Cardiol. 2016;67:1472–1487.

TAVR or SAVR? 2017 Guidelines


Nishimura, et al.
2017 VHD Focused Update
Figure 1. Choice of TAVR Versus Surgical AVR in the Patient With Severe Symptomatic AS

3 to <15% 15 to < 50%


T

30 day mort 30 day mort


≥50% 30
P

day mort
C RI
US

AS indicates aortic stenosis; AVR, aortic valve replacement; and TAVR, transcatheter aortic valve replacement.

1. Nishimura RA, et al. 2017 AHA/ACC Focused Update of the 2014 AHA/ACC Guideline for the Management of Patients
With Valvular Heart Disease. J Am Coll Cardiol. 2017;
2. Risk calculator available at the ACC or STS websites.

1074
Copyright © Harvard Medical School, 2018. All Rights Reserved.

Aortic Regurgitation

Aortic Regurgitation
Etiology
VALVE ROOT
• BAV DISEASE • CT DISORDER
• RHEUMATIC • DISSECTION
• IE • IE
• MYXOMATOUS • AORTITIS
• APLA • HTN
• FEN-PHEN ? • OTHER
• TRAUMA (Congenital)

1075
Copyright © Harvard Medical School, 2018. All Rights Reserved.

Acute Severe AR

Aortic Regurgitation
Classification of Severity

MILD MODERATE SEVERE


ANGIO GRADE 1+ 2+ 3-4+

DOPPLER JET AREA < 25% LVOT > MILD > 65% LVOT

VENA CONTR (cm) < 0.3 0.3-0.6 > 0.6

REGURG VOL (ml/b) < 30 30-59 > 60

REGURG FRAC (%) < 30 30-49 > 50

EROA (cm2) < 0.10 0.10-0.29 > 0.30

Am Soc Echocardiography 2003

1076
Copyright © Harvard Medical School, 2018. All Rights Reserved.

Aortic Regurgitation
Vasodilator Therapy
100
Aortic Valve Replacement (%)

80
P=0.29 Enalapril
60
Nifedipine
40
Control
20

0
0 1 2 3 4 5 6 7 8 9 10

Years

Mean SBP 142-147


Evangelista A et al. NEJM 2005; 353:1342-9

Severe Aortic Regurgitation


Indications for Surgery
Class I Class II
• Symptoms • LVESD > 50 mm or
• EF < 0.50 > 25 mm/m2
• Need for Ao surgery • LVEDD > 65 mm if
surgical risk low
• LVESD > 50 mm or
LVEDD > 70 mm and
progressively ↑

ESC Valve Guidelines 2012


ACC/AHA Valve Guidelines 2014

1077
Copyright © Harvard Medical School, 2018. All Rights Reserved.

BAV Aortopathy

Indications for Surgery


BAV with Dilated Aortic Root
Class I
• Maximal dimension > 5.5 cm or annual
increase in size > 0.5 cm / year.*
• Maximal dimension > 4.5 cm and
surgery indicated for severe AS or AR.*

* Consider lower threshold values for patients of small


stature of either gender
ACC/AHA Valve Guidelines 2014

1078
Copyright © Harvard Medical School, 2018. All Rights Reserved.

Mitral Regurgitation

Mitral Regurgitation Etiology


Acute MR Chronic MR
• Acute MI (Inf-Post) • Myxomatous
• Endocarditis • Ischemic
• Trauma • DCM
• “Acute on chronic” • Rheumatic
• MAC
• HOCM
• Other (APLS, etc.)

1079
Copyright © Harvard Medical School, 2018. All Rights Reserved.

Symptomatic Primary MR
• If Primary MR
• AND Severe
– Vena contracta ≥ 0.7 cm
– Regurgitant volume ≥ 60 mL
– Regurgitant fraction ≥ 50%
– Effective regurgitant orifice ≥ 0.4 cm2
– LV dilation
• LVEF > 30% or LVESD ≥ 40mm: MV surg (Class I)
• LVEF ≤ 30%: MV Surgery (Class IIB)
• Progressive increase in LVESD or decrease in EF:
MV surgery (Class IIA)

ACC/AHA VHD guidelines 2014

Asymptomatic Primary MR

1080
Copyright © Harvard Medical School, 2018. All Rights Reserved.

Chronic MR

Medical Therapy
•No
ABx prophylaxis
role when and if indicated
for vasodilator therapy in
asymptomatic,
• Management of AF normotensive
•patients
Managementwith chronic severe MR
of CAD
and
• ACE-I normal
or ARB LVreduced
for HTN, function
EF

Surgery in Asymptomatic Severe Primary MR


Class LOE
LVEF <0.60 or LVSD >45mm – ESC I C
LVEF 0.30-0.60 and/or LVESD ≥ 40 mm - AHA
Preserved EF and new onset AF or PA HTN at rest (>50 mm IIa C
Hg)
Preserved EF, high likelihood durable repair, low risk and IIa C
flail leaflet, LVESD >40mm (AHA/ACC < 40 mm)
LVEF ≥60 but progressive decease or LVESD ≥ 40 but IIa C
progressive increase
Preserved EF, high likelihood durable repair, low risk, and IIb C
LASVI >60mL/m2 or exercise PA HTN (>60 mmHg)
LVEF ≤ 30% IIb C

ESC 2012 VHD Guidelines. Euro Heart J. 2012; 33: 2451-2496


ACC/AHA VHD guidelines 2014 and 2017

1081
Copyright © Harvard Medical School, 2018. All Rights Reserved.

Ischemic MR

Mitral regurgitation
Indications for mitral valve surgery
for functional MR:

• Severe MR, persistent


symptoms despite optimal class IIb
medical therapy, including
CRT

1082
Copyright © Harvard Medical School, 2018. All Rights Reserved.

Mitral Stenosis

1083
Copyright © Harvard Medical School, 2018. All Rights Reserved.

Mitral Stenosis
Diastolic Filling Period

Mitral Stenosis
Anticoagulation

• AF: paroxysmal, persistent, permanent

• Hx TIA/CVA, systemic embolus

• Presence of LA thrombus

• LA > 5.5cm or spontaneous contrast

1084
Copyright © Harvard Medical School, 2018. All Rights Reserved.

POP QUIZ!
What would you recommend for
anticoagulation for someone with
PAF and mitral stenosis?
a. Apixaban
b. Dabigatran
c. Warfarin
d. Rivaroxaban
e. Edoxaban

POP QUIZ!
What would you recommend for
anticoagulation for someone with
PAF and mitral stenosis?
a. Apixaban
b. Dabigatran
c. Warfarin
d. Rivaroxaban
e. Edoxaban

1085
Copyright © Harvard Medical School, 2018. All Rights Reserved.

PMBV
CLASS 1 Indications
• Symptoms
• PA HTN (PA > 50 rest, > 60 ex)
Predicated on:
1. Favorable morphology
2. Operator and Lab experience

Absent:
1. Moderate to severe MR
2. LA thrombus
3. Inability to perform trans-septal puncture

PMBV

MVA = 1.5 cm2 MVA = 3.5 cm2


Mean Gradient = 8.0 mm Hg Mean Gradient = 4.0 mm Hg
PCWP = 16-18 mm Hg PCWP = 9 mm Hg

1086
Copyright © Harvard Medical School, 2018. All Rights Reserved.

Tricuspid Regurgitation

Tricuspid Regurgitation
Etiology
• Primary
– Congenital: Ebstein’s
– Carcinoid (metastatic)
– Rheumatic
– Myxomatous
– Radiation
– Trauma
– Infectious endocarditis
• Secondary
– TV annular dilation
– PA HTN

1087
Copyright © Harvard Medical School, 2018. All Rights Reserved.

Prosthetic Valves

Prosthetic Valves

Tissue Mechanical

1088
Copyright © Harvard Medical School, 2018. All Rights Reserved.

Prosthetic Heart Valves


Complications
• Structural valve deterioration
• Para-valvular regurgitation
• Hemolytic anemia
• Infective endocarditis
• PV thrombosis
• Thromboembolism; bleeding
• Pannus formation
• Prosthesis-patient mismatch

Anti-Thrombotic Therapy
Aspirin Warfarin Warfarin Bridge
75-100mg INR 2.0-3.0 INR 2.5-3.5

Mechanical Valve

1. AVR- low risk* Class I Class I None needed

2. AVR- high risk* Class I Class I UFH or SC


LMWH
3. MVR Class I Class I UFH or SC
LMWH

* These recommendations apply to bileaflet mechanical valves


Class III: oral direct thrombin inhibitors or anti-Xa agents should NOT be
used in patients with mechanical prostheses
Risk Factors: AF, previous thromboembolism, LV dysfunction,
hypercoagulable condition, older generation mechanical AVR
ACC/AHA VHD guidelines 2014, 2017

1089
Copyright © Harvard Medical School, 2018. All Rights Reserved.

Anti-Thrombotic Therapy
Aspirin Clopidogrel Warfarin Warfarin No
75-100 mg 75 mg INR 2.0- INR 2.5- Warfarin
3.0 3.5
Biological Valve

1. AVR

< 3 months Class IIa Class IIb

> 3 months Class IIa

2. MVR

< 3 months Class IIa Class IIa

> 3 months Class IIa

3. TAVR

< 6 months Class IIa Class IIb

> 6 months Class IIa

ACC/AHA VHD guidelines 2014, 2017

Endocarditis Prophylaxis
Endocarditis Prophylaxis Recommend- Level of
ation Evidence
Prosthetic cardiac valve or prosthetic material IIa C-LD
for valve repair (including TAVR)
Previous infective endocarditis IIa C-LD
Unrepaired cyanotic congenital heart disease IIa C-LD
(CHD)
Repaired CHD with prosthetic material, first 6 IIa C-LD
months
Repaired CHD with residual defects at site of IIa C-LD
patch or device
Cardiac transplant with valve regurgitation IIa C-LD
due to structurally abnormal valve
For dental procedures that involve manipulation of either gingival tissue or the
periapical region of teeth or perforation of the oral mucosa.
ACC/AHA VHD guidelines 2017

1090
Copyright © Harvard Medical School, 2018. All Rights Reserved.

Antibiotic Regimens
Situation Agent Single Dose 30-60 min
before procedure
Oral Amoxicillin 2g
Cannot take oral Ampicillin 2 g IM or IV
Allergic to PCN or AMP Cephalexin 2g
Clindamycin 600 mg
Azithro or clarithro 500 mg
Allergic to PCN or AMP Cefazolin or ceftriaxone 1 g IM or IV
and unable to take oral
Clindamycin 600 mg IM or IV

Bonow RO et al. JACC. 2008;52:e1–e142.

https://www.nytimes.com/2018/04/29/health/drugs-opioids-addiction-heart-endocarditis.html

1091
Copyright © Harvard Medical School, 2018. All Rights Reserved.

ATRIAL SEPTAL DEFECT

1. Secundum
2. Primum
3. Sinus venosus
4. Coronary sinus

ATRIAL SEPTAL DEFECT


• Types: secundum, primum , sinus venosus,
coronary sinus

• Exam: Grade 2 MSM, fixed splitting S2

• ECG: IRBB. Left axis deviation = primum.

• ECHO: RV volume overload, shunt flow,


associated findings (MVP, cleft MV, APVD)

1092
Copyright © Harvard Medical School, 2018. All Rights Reserved.

SECUNDUM ASD

PATENT FORAMEN OVALE


• Present in 25-30% of population
• Sometimes associated with interatrial
septal aneurysm
• Implied role in
– Cryptogenic stroke
– Migraine
– Platypnea-orthodeoxia
– Decompression sickness

1093
Copyright © Harvard Medical School, 2018. All Rights Reserved.

TETRALOGY OF FALLOT

1. VSD
2. Overriding Ao
3. RVOT
obstruction
4. RVH

TETRALOGY OF FALLOT
• VSD (NON-RESTRICTIVE, LARGE)

• OVERRIDING AORTA

• RVOT OBSTRUCTION

• RVH

1094
Copyright © Harvard Medical School, 2018. All Rights Reserved.

COARCTATION

BAV DISEASE

Lembeke A et al.Circulation 2003;107:e80

Question 1
A 70 year old woman presents with
mild dyspnea on exertion.
HR 72 reg, BP 156/80. Grade 3 late
peaking murmur of aortic stenosis.
ECG: NSR. LVH
TTE: Calcified aortic valve, mean
gradient 50 mm Hg, AVA=0.8cm2. Wall
thickness 1.5 cm. LVEF 0.75.

1095
Copyright © Harvard Medical School, 2018. All Rights Reserved.

Question 1
Which of the following treatments would
you recommend?
a. Diuretic and ACE-inhibitor with close
follow-up
b. Bioprosthetic aortic valve
replacement
c. Mechanical aortic valve replacement

Question 1
Which of the following treatments would
you recommend?
a. Diuretic and ACE-inhibitor with close
follow-up
b. Bioprosthetic aortic valve
replacement
c. Mechanical aortic valve replacement

1096
Copyright © Harvard Medical School, 2018. All Rights Reserved.

Answer: This woman has symptomatic AS, with a mean


gradient and valve area in the severe range. Diuretics could
Question 1
well be harmful, and an ACEI is not going to alter her fixed
stenosis. She will require elective AVR. Given her age of 70,
Which of the following treatments would
the guidelines suggest a bioprosthetic aortic valve would be
appropriate. Without significant comorbidity, the AVR should
you recommend?
be performed via an open surgical route.
a. Diuretic and ACE-inhibitor with close
follow-up
b. Bioprosthetic aortic valve
replacement
c. Mechanical aortic valve replacement

Question 2
A previously asymptomatic 50 yr old
woman with mitral valve prolapse and
mild mitral regurgitation presents with
NYHA Function Class II dyspnea of 6
months duration. She had dental work
done 2 weeks before symptom onset
without antibiotic prophylaxis.

HR 84 reg, BP 102/76. T 98.2. Grade 3


systolic murmur at the apex.

1097
Copyright © Harvard Medical School, 2018. All Rights Reserved.

Question 2

Echocardiography now shows


moderate-severe MR with a
partially flail posterior leaflet.
LVEF is 65%. 6 blood cultures are
negative.

Question 2

Which of the following strategies


would you pursue?

• Careful clinical follow-up with


echo studies at 3-month intervals
• ACE inhibitor therapy
• Mitral valve replacement
• Mitral valve repair

1098
Copyright © Harvard Medical School, 2018. All Rights Reserved.

Question 2

Which of the following strategies


would you pursue?

• Careful clinical follow-up with


echo studies at 3-month intervals
• ACE inhibitor therapy
• Mitral valve replacement
• Mitral valve repair

Answer: This woman has chronic mitral valve prolapse and


Question 2
myxomatous MV disease and has ruptured a chord. This is
what caused her mitral regurgitation to worsen and led to her
symptoms. She has symptomatic severe MR and will require
surgery. Mitral valve repair is the preferred operation.

Which of the following strategies


would you pursue?

• Careful clinical follow-up with


echo studies at 3-month intervals
• ACE inhibitor therapy
• Mitral valve replacement
• Mitral valve repair

1099
Copyright © Harvard Medical School, 2018. All Rights Reserved.

Question 3
• A 71 year old woman with a St. Jude
MVR and AF is scheduled for
laparoscopic cholecystectomy. She
had a TIA 2 years ago. She takes
warfarin, low dose aspirin, metoprolol
succinate, and furosemide. Labs
include INR 3.4, BUN 42, serum
creatinine 2.1 (CrCl 27 mL/min).

Question 3
• Which of the following strategies for
anticoagulation management would
you advise?
– a. Taper warfarin and operate when INR 2.0
– b. Hold warfarin for 5 days and operate
– c. Hold warfarin, admit for IV UFH when
INR < 2.5
– d. Hold warfarin, bridge with enoxaparin, 1
mg/kg bid

1100
Copyright © Harvard Medical School, 2018. All Rights Reserved.

Question 3
• Which of the following strategies for
anticoagulation management would
you advise?
– a. Taper warfarin and operate when INR 2.0
– b. Hold warfarin for 5 days and operate
– c. Hold warfarin, admit for IV UFH when
INR < 2.5
– d. Hold warfarin, bridge with enoxaparin, 1
mg/kg bid

Answer: This woman has a mechanical mitral valve prosthesis and atrial
fibrillation, a high risk feature for thromboembolism. Her INR goal is 2.5-3.5.
Anti-Thrombotic Therapy
While recommendations vary, ACC/AHA and ESC guidelines would suggest
UFH for someone with mechanical MVR + one other risk factor for
thromboembolism. Her eGFR precludes the use of LMWH.
Aspirin Warfarin Warfarin Bridge
75-100mg INR 2.0-3.0 INR 2.5-3.5

Mechanical Valve

1. AVR- low risk Class I Class I None needed

2. AVR- high risk Class I Class I UFH or SC


LMWH
3. MVR Class I Class I UFH or SC
LMWH

Class III: oral direct thrombin inhibitors or anti-Xa agents should NOT be
used in patients with mechanical prostheses

Risk Factors: AF, previous thromboembolism, LV dysfunction,


hypercoagulable condition, older generation mechanical AVR

ACC/AHA VHD guidelines 2014

1101
Copyright © Harvard Medical School, 2018. All Rights Reserved.

References
• ACC/AHA 2008 Valvular Heart Disease
Guidelines. www.acc.org/guidelines
• ACC/AHA 2014 and 2017 Valvular Heart
Disease Guidelines. www.acc.org/guidelines
• ESC 2012 Valvular Heart Disease Guidelines.
www.esc.org/guidelines
• ACC/AHA 2009 Adult Congenital Heart Disease
Guidelines. www.acc.org/guidelines

1102
Copyright © Harvard Medical School, 2018. All Rights Reserved.

Peripheral, Aortic and Carotid Disease

Marc P. Bonaca, MD, MPH


Medical Director Aortic Center
Vascular Medicine, Cardiology
Brigham and Women’s Hospital
Assistant Professor of Medicine
Harvard Medical School

Disclosures

Research Grant Support to TIMI Study


Group: Amgen, AstraZeneca, MedImmune,
Merck

Consulting: Aralez, AstraZeneca, Merck,


Bayer, Janssen, Sanofi

1103
Copyright © Harvard Medical School, 2018. All Rights Reserved.

Peripheral Artery Disease (PAD)

• The presence of a stenosis


or occlusion in the aorta or
arteries of the limbs
• Usually caused by
atherosclerosis
• Associated with an
increased risk of death,
myocardial infarction, and
stroke
• May impair walking or
cause critical limb
ischemia
3

Clinical Presentation of PAD


~15%
Typical
Claudication
50%
Asymptomatic

~ 33%
Atypical
Limb Symptoms

2-3%
Critical
Limb Ischemia
4

1104
Copyright © Harvard Medical School, 2018. All Rights Reserved.

Physical Findings in PAD

• Absent or diminished peripheral pulses


• Absent posterior tibial pulse > 90% specific
for diagnosis of PAD
• Bruits
• Hair loss
• Dystrophic nail changes
• Rapid elevation pallor or dependent rubor
of the limb
• Evidence of tissue loss (ulceration,
gangrene)

Diagnostic Testing for Suspected PAD

Colors correspond to Class of


Recommendation in Table 1.

ABI indicates ankle-brachial index; CLI,


critical limb ischemia; CTA, computed
tomography angiography; GDMT, guideline-
directed management and therapy; MRA,
magnetic resonance angiography; PAD,
peripheral artery disease; and TBI, toe-
brachial index.

Gerhard-Herman et al. Circulation 2016

1105
Copyright © Harvard Medical School, 2018. All Rights Reserved.

Imaging Tests for PAD

• Duplex ultrasonography
• Magnetic resonance angiography
• Computed tomographic angiography
• Conventional contrast angiography

1106
Copyright © Harvard Medical School, 2018. All Rights Reserved.

PAD Risk-reduction Therapies

• Lifestyle modifications
– Weight maintenance/reduction
• Smoking
– Complete cessation
• Diabetes mellitus
– HbA1c goal, target specific agents
• Dyslipidemia
– High intensity statin + other agents (lower is better)
• Hypertension
– Therapies to achieve target, ACE inhibitors (HOPE Trial)
• Antithrombotic therapy

Summary of Effects of
PCSK9i Evolocumab
• ↓ LDL-C by 59% to a median of 30 mg/dL
• ↓ CV outcomes in patients on statin
• Safe and well-tolerated HR 0.85 (0.79-0.92)
Placebo P<0.0001
15 14.6
12.6 HR 0.80 (0.73-0.88)
59% reduction P<0.0001
KM Rate (%) at 3 Years

P<0.00001
9.9
10
Absolute 56 mg/dl 7.9

5
Evolocumab
(median 30 mg/dl, IQR 19-46 mg/dl)

0
CVD, MI, stroke CVD, MI, stroke
UA, cor revasc
An Academic Research Organization of
Brigham and Women’s Hospital and Harvard Medical School
Sabatine MS et al. NEJM 2017;376:1713-22

1107
Copyright © Harvard Medical School, 2018. All Rights Reserved.

Major Adverse Limb Events in Patients


with PAD and no MI or Stroke
Placebo PAD
Evolocumab (no MI/stroke, N=1505)
3.0%
57% RRR
2.6%
Major Adverse Limb Events

2.5% HR 0.43
(0.19 – 0.99)
P=0.042
2.0% 1.3% ARR

1.5%
1.3%

1.0%

0.5%

0.0%
0 90 180 270 360 450 540 630 720 810 900
Days from Randomization
An Academic Research Organization of
Brigham and Women’s Hospital and Harvard Medical School

Achieved LDL-C and Major


Adverse Limb Events

P=0.026 for beta coefficient

adjusted for significant (p<0.05) predictors of LDL-C cholesterol at 1 month after randomization including age,
BMI, LDL-C at baseline, male sex, race, randomized in North America, current smoker, high intensity statin.
An Academic Research Organization of
Brigham and Women’s Hospital and Harvard Medical School

1108
Copyright © Harvard Medical School, 2018. All Rights Reserved.

MACE or MALE
In Patients with PAD and no MI or Stroke
Placebo
Evolocumab
PAD
(no MI/stroke, N=1505)

14% 48% RRR 12.8%

12% HR 0.52
PAD
(0.35 – 0.76)
6.3% ARR
MACE or MALE

10% P=0.0006
NNT2.5y 16

8%
6.5%
6%

4%

2%

0%
0 90 180 270 360 450 540 630 720 810 900

Days from Randomization


An Academic Research Organization of
Brigham and Women’s Hospital and Harvard Medical School

Targets for Antithrombotic Therapy

Marc P. Bonaca, and Mark A. Creager Circulation


Research. 2015;116:1579-1598

Copyright © American Heart Association, Inc. All rights reserved.

1109
Copyright © Harvard Medical School, 2018. All Rights Reserved.

New Guidelines!
For “Symptomatic PAD”

• Similar to 2005 COR Therapy Comment


Guideline LOE
I Monotherapy CAPRIE showed
A clopidogrel
• Recommendations ASA 75-325 mg superior to ASA
for therapy to or Clopidogrel

reduce MACE based IIb DAPT Refer to the


on presence of B-R DAPT guideline
ASA+ for CAD
claudication (not Clopidogrel
MACE risk factors)
IIb DAPT or TAPT Benefit for
B-R MACE and ALI
Vorapaxar but increase in
bleeding so
(added to ASA overall benefit
and/or clopi) uncertain
Therapies Approved by the US FDA for PAD in BLUE
Gerhard-Herman et al. Circulation 2016 15

COMPASS Trial

Reductions in CV Death
Mortality

HR for Major Bleeding 1.70


Adapted from: N Engl J Med 2007;357:217-27.
(1.40 – 2.05), p<0.001

>90% with CAD, large subgroup with


Concomitant PAD, consistent benefits for both

1110
Copyright © Harvard Medical School, 2018. All Rights Reserved.

Anand et al. Lancet 2017

COMPASS PAD – Limb Risk By PAD History

4% 3.80%
Incidence of MALE (%)

3%

2%
1.37%
1%
0.50%
N=86 N=37 N=5
0%
Prior Revascularization or Claudication but no Asymtomatic low ABI
Amputation History of (<=0.90)
Revascularization or
Amputation

N=2264 N=2705 N=1422


36% of Population 42% of Population 22% of Population

1111
Copyright © Harvard Medical School, 2018. All Rights Reserved.

ASA+Ticagrelor in PAD with Prior MI

Placebo
25%
Ticagrelor 60 mg BID
CV Death, MI, or Stroke (%)

PAD
20% 19.3%
ARR 5.2% Absolute Risk
HR
NNT 20 Difference at
(95% CI)
3 Years
15% 14.1% 0.69
CVD / MI / Stroke – 5.2 (0.47 – 0.99)
P=0.045
0.47
CV Death – 5.4 (0.25 – 0.86)
10% No PAD P=0.014
0.52
Mortality – 5.7 (0.32 – 0.84)
P=0.0074
5% TIMI Major Bleeding 1.18
0.02 (0.29 – 4.70)
P-interaction NS
P=0.82

0% 0.1 Ticagrelor Better Placebo Better 10


1.0
Days from Randomization
An Academic Research Organization of
Brigham and Women’s Hospital and Harvard Medical School Bonaca MP et al. JACC 2016

A Framework for Optimizing Antithrombotic Intensity in PAD

Symptomatic PAD

High MACE Risk?


• Prior MI (mortality benefit)
High Bleeding Risk? • Severe CAD
• Recent major bleeding • Others (eg, DM)
• Prior stroke/ICH No
High MALE Risk?
• Indication for AC (e.g. AF)
• Prior bypass/stenting
• Others (eg, frailty, anemia)
• Prior thrombotic complication
• Undergoing intervention
• Others*(eg, severe disease)
Yes
Yes
No
(either)

Consider a More
Intensive
Monotherapy Antithrombotic
Strategy

*Risk scores may be helpful!

1112
Copyright © Harvard Medical School, 2018. All Rights Reserved.

Intermittent Claudication:
Exercise Therapy

• Frequency: 3–5 supervised sessions/week


• Duration: 35–50 minutes of exercise/session
• Type of exercise: treadmill or track walking
to near-maximal claudication pain
• Length: ≥6 months
• Results: 100%–150% improvement in maximal
walking distance
• Improvement in QoL
NOW COVERED BY CMS!
Stewart KJ et al. N Eng J Med. 2002;347:1941-1951.
21

Exercise in PAD
P<0.001
Change in Pain Free Walking Time (Minutes) From

P<0.04
7

5.8
Baseline to Six Months

6
Mean Difference P=0.02
5 1.01 Minutes
95% CI 0.07 – 1.95
4 P=0.04 3.7

2
1.43
1.2
1
0.42
0
Control Home Based Optimal Medical Stent Supervised
Exercise Care Revascularization Exercise
McDermott et al. JAMA 2013 Murphy et al. Circulation 2012

1113
Copyright © Harvard Medical School, 2018. All Rights Reserved.

Structured Exercise Therapy

COR LOE Recommendations


In patients with claudication, a supervised exercise
I A program is recommended to improve functional status and
QoL and to reduce leg symptoms.
A supervised exercise program should be discussed as a
I B-R treatment option for claudication before possible
revascularization.
In patients with PAD, a structured community- or home-
based exercise program with behavioral change
IIa A
techniques, can be beneficial to improve walking ability
and functional status.
In patients with claudication, alternative strategies of
exercise therapy, including upper-body ergometry, cycling,
IIa A and pain-free or low-intensity walking that avoids
moderate-to-maximum claudication while walking, can be
beneficial to improve walking ability and functional status.

Gerhard-Herman et al. Circulation 2016

Pharmacotherapy for Claudication


FDA Approved Drugs
Pentoxifylline
• Methylxanthine
• Approved August 1984
• Decreases plasma viscosity, improves RBC deformability,
some vasodilation
Cilostazol
• Phosphodiesterase III inhibitor derivative
• Approved January 1999
• Platelet inhibitor, vasodilation, ↑HDL-cholesterol,
↓triglycerides
• Contraindicated if history of CHF of any severity
24

1114
Copyright © Harvard Medical School, 2018. All Rights Reserved.

Indications and Options for


Revascularization
• Persistent, lifestyle-limiting claudication
despite maximal medical therapy
• Rest pain
• Nonhealing ulcer, gangrene

• Endovascular reconstruction options


• Angioplasty / Stenting
• Surgical reconstruction options
• Bypass
Gerhard-Herman et al. Circulation 2016 25

Endovascular Revascularization and Supervised


Exercise (ERASE) for Claudication Study
N = 212 patients with aorto-iliac and/or femoral-popliteal artery disease randomized
to supervised exercise training (SET) or SET plus endovascular revascualrization

Fakhry et al. Presented at 2013 AHA Scientific Sessions,


26

1115
Copyright © Harvard Medical School, 2018. All Rights Reserved.

PAD Risk-reduction Therapies


Therapies for MACE Reduction in all Patients
• Lifestyle Modification
• Tobacco Cessation Therapies
• Targeting blood pressure goals with preference for ACEi
• LDL-C lowering with statin ± ezetimibe and/or PCSK9i
• Antiplatelet monotherapy (symptomatic), preference for P2Y12 inhibition
Therapies for MACE Reduction in Selected Patients
Diabetes
• Glucose lowering to reduce microvascular risk
• GLP-1 , SGLT2 inhibitors but caution with canagliflozin

Prior MI or CAD (Polyvascular Disease) and low bleeding risk


• ASA + rivaroxaban 2.5 BID (broad polyvascular definition)
• ASA + ticagrelor 60 mg BID (prior MI or other need for DAPT)
• ASA and/or clopidogrel with vorapaxar (prior MI or diabetes)

Therapies for MALE Reduction in all Patients


• LDL-C lowering with statin ± ezetimibe and/or PCSK9i
Therapies for MALE Reduction in Selected Patients
Prior peripheral revascularization & low bleeding risk
• ASA + rivaroxaban 2.5 BID
• ASA + ticagrelor 60 mg BID (prior MI or other need for DAPT)
• ASA and/or clopidogrel with vorapaxar

Therapies for Claudication


Symptomatic Patients
• Cilostazol 100 mg BID (only if no history of heart failure)

Abdominal Aortic Aneurysms


• Annual incidence 40.6 to 49.3 per 100,000
men and 6.8 to 12 per 100,000 women
• Accounts for ~16,000 deaths annually in
U.S., 13th leading cause of death in US
• Majority of pts asymptomatic with AAA
incidentally found
• Primary complication is rupture which is
associated with high mortality (~50% reach
hospital, 30-50% who reach hospital die)
• Rupture is predictable by size and rate of
change
• Medical therapy is targeted at reducing
overall cardiovascular risk and reducing the
rate of expansion

2011 ACCF/AHA Focused Update of the Management of Patients With Peripheral Artery Disease Guideline (Updating the 2005 Guideline) 28

1116
Copyright © Harvard Medical School, 2018. All Rights Reserved.

Risk of AAA Rupture


Relation to Size

Maximum Diameter (cm) 5-Year Rupture Rate (%)

<4.0 2
4.0–4.9 3–12
5.0–5.9 25
6.0–6.9 35
>7.0 75

Lederle FA, et al. Arch Intern Med 2000;160:1425.


29

Screening for AAA


ACC/AHA Guidelines

I IIa IIb III • Men 60 years of age or older who are


either the siblings or offspring of
patients with AAAs should undergo
physical examination and ultrasound
screening for detection of aortic
aneurysms

I IIa IIb III • Men who are 65-75 years of age who
have ever smoked should undergo a
physicial examination and 1- time
ultrasound screening for detection of
AAAS
Rooke TW, Hirsch AT, et al. JACC, 2013; 61:1555-70.
30

1117
Copyright © Harvard Medical School, 2018. All Rights Reserved.

Aortic Aneurysm
AAA – Medical / Lifestyle Therapy

Primary goal to reduce overall cardiovascular risk

• Aspirin
• Statin therapy

• Blood pressure control


• Beta-blockers in patients planned for repair
• ACEi/ARB, diuretics, CCB

• Lifestyle counseling
• Smoking Cessation
• Exercise is ok! (moderate activity)

Surgical Repair for Infrarenal AAA

Creager, MA ed. Atlas of Vascular Medicine, Current Science, 2002 32

1118
Copyright © Harvard Medical School, 2018. All Rights Reserved.

Endovascular Repair Technique and


Devices

Components of endograft device:


• Delivery system to allow placement via femoral artery approach
• Attachment system that forms a blood tight seal between graft and abdominal aorta
• Graft excludes aneurysm from flow and minimizes rupture risk

Infra-renal attachment Deliver contralateral


Device delivery
iliac endoleg Complete deployment
33

Abdominal Aortic Aneurysm


ACC/AHA Guidelines
I IIa IIb III • Patients with AAA >5.5 cm in diameter should
undergo repair to eliminate the risk of rupture.

I IIa IIb III • Patients with AAA 4.0 to 5.4 cm in diameter


should be monitored by ultrasound or CT scans
every 6-12 months to detect expansion
I IIa IIb III
• For AAA <4.0 cm in diameter, monitor by
ultrasound every 2-3 years.
I IIa IIb III
• In patients with symptomatic aortic aneurysms,
repair is indicated regardless of diameter.

Rooke TW, Hirsch AT, et al. JACC, 2013; 61:1555-70.


34

1119
Copyright © Harvard Medical School, 2018. All Rights Reserved.

Abdominal Aortic Aneurysm


ACC/AHA Guidelines

I IIa IIb III • Open or endovascular repair of


infrarenal AAAs is indicated in
patients who are good surgical
candidates.

• Periodic long-term surveillance


I IIa IIb III imaging should be performed to
monitor for endoleak, confirm
graft position, and document
shrinkage or stability of the
excluded aneurysm sac
Rooke TW, Hirsch AT, et al. JACC, 2013; 61:1555-70.
35

Thoracic Aortic Aneurysm


Epidemiology
• Degenerative disease (cystic medial necrosis)

• Incidence 10.4 cases / 100,000 pt-yrs, 2:1 male:female

• Risk factors: HTN, smoking, COPD

• Associated with genetic syndromes, bicuspid AV, inflammatory


diseases, infection, prior trauma

• Most common site is descending thoracic aorta (>50%)

• Peak incidence occurs at age 70 in men and age 80 in women,


infrequent in persons <50 years

• Most asymptomatic but if large it may present with heart failure


(if AI), dysphagia, hoarseness, chest pain, cough or wheezing
Hiratzka et. al: 2010 Consensus Guidelines – Thoracic Aortic Disease

1120
Copyright © Harvard Medical School, 2018. All Rights Reserved.

Thoracic Aortic Aneurysm


Screening
• Marfan Syndrome – TTE every 6 months
• Loeys-Dietz – complete aortic imaging every 6 months
• Turner Syndrome – imaging of heart and aorta every 5-10 yrs
• Bicuspid aortic valve – valve, aortic root, ascending aorta

• Anyone with a 1st degree relative with thoracic aortic aneurysm


or dissection

• If the mutant gene (FBN1, TGFBR1, TGFBR2, COL3A1, ACTA2, MYH11)


associated with aortic aneurysm and/or dissection is identified in a
patient, first degree relatives should undergo counseling and testing.
Then, only the relatives with the genetic mutation should undergo aortic
imaging.

Hiratzka et. al: 2010 Consensus Guidelines – Thoracic Aortic Disease

Thoracic Aortic Disease


`
When to Intervene?

• Generally 5.5 cm, or growth > 0.5 cm/y


including bicuspid AoV (unless RF for
dissection such as FH)

• For genetic syndrome (MFS, LDS, EDS,


familial syndrome) may undergo at smaller
diameters (4.0-5.0) – for MFS
contemplating pregnancy > 4.0 cm

• If aortic valve surgery and > 4.5 cm


2010 ACCF/AHA/AATS/ACR/ASA/
SCA/SCAI/SIR/STS/SVM Guidelines for the Diagnosis and Management of Patients with Thoracic Aortic Disease

1121
Copyright © Harvard Medical School, 2018. All Rights Reserved.

Thoracic Aortic Aneurysm


Diameter and Risk of Complication

Hiratzka et. al: 2010 Consensus Guidelines – Thoracic Aortic Disease

Pathogenesis of Stroke
Ischemic Stroke (80%) Hemorrhagic Stroke (20%)
Atherothrombotic
Cerebrovascular Intracerebral
Disease (20%) Cryptogenic (30%) Hemorrhage (70%)

?
Lacunar (25%)
(small vessel disease) Cardioembolic (20%) Subarachnoid Hemorrhage (30%)

Albers GW et al. Chest. 1998;119:683S-698S.


Albers GW Personal communication. February 27, 2003.
Rosamond WD et al. Stroke. 1999;30:736-743.
40

1122
Copyright © Harvard Medical School, 2018. All Rights Reserved.

Risk Factors and Newer Therapies


Modify Risk
Risk Factor Increased Risk Bosch J. BMJ
2002; 324:699
ACE Inhibitors / ARBs 25-34% PROGRESS
Hypertension 2X Collaborative
reduction in stroke Group. Lancet
2001:358; 1033
B Dahlof et al.
High Cholesterol 2X Statins 25% reduction in stroke Lancet
2002;359:995-1003
MRC/BHF HPS
Investigators
IGT / DM 2X / 3X Weight loss, diet, meformin Lancet 2002; 360
(9326): 7

UKPDS 10-year
NOACs as effective as warfarin with less outcomes
Atrial Fibrillation 5X
bleeding – broader population for rx

Smoking 2-4 X Reductions in prevalence of smoking

OSA 2X Increased identification and treatment

Lloyd-Jones et al. Circulation Heart Disease and Stroke Statistics 2012

Mechanisms of Stroke in Patients with


Carotid Artery Stenosis

42

1123
Copyright © Harvard Medical School, 2018. All Rights Reserved.

Stroke Attributable
Asymptomatic Carotid Disease
In the Current Era
Proportion of ischemic stroke Population Attributable Risk for Stroke

subtypes: NOMASS

< 15% due to asymptomatic carotid disease

White, H. Circ 2005;111:1327 Lloyd-Jones et al. Circulation Heart Disease and


Stroke Statistics 2012

Duplex Ultrasonography:
Internal Carotid Artery Stenosis

44

1124
Copyright © Harvard Medical School, 2018. All Rights Reserved.

Carotid Duplex Ultrasonography (DUS)


ACC/AHA Guidelines

I IIa IIb III


• DUS is recommended as the initial diagnostic test
to detect hemodynamically significant carotid
stenosis in asymptomatic patients with known or
suspected carotid stenosis.

I IIa IIb III • DUS is recommended to detect carotid stenosis in


patients who develop focal neurological symptoms
corresponding to the territory supplied by the left or
right internal carotid artery.
I IIa IIb III
• DUS is not recommended for routine screening of
asymptomatic patients who have no clinical
manifestations of or risk factors for atherosclerosis.

Medical Therapy for Carotid Stenosis

• Goal to Reduce Overall Cardiovascular Risk


• Lifestyle modification
• Antiplatelet therapy
• Lipid lowering (high-intensity statin)
• Antihypertensive therapy

46

1125
Copyright © Harvard Medical School, 2018. All Rights Reserved.

Symptomatic Carotid Disease

• Focal neurologic symptoms, retinal artery embolization


on fundoscopic exam, amaurosis fugax

• Imaging (CUS, CTA/MRA)

• Revascularization for stenosis of 50% or greater (CEA


vs. CAS) time sensitive
• Within 6 hours urgent revasc
• Fixed deficit >6 hours within 2 weeks once stable

• Antiplatelet therapy

47

Medical Intervention for Prevention of Ipsilateral Stroke


With Asymptomatic Severe Carotid Stenosis

Average annual stroke (+/-TIA) rates decreased by later publication years

Abbott, A. L. Stroke 2009;40:e573-e583


48

1126
Copyright © Harvard Medical School, 2018. All Rights Reserved.

Carotid Revascularization Endarterectomy


vs. Stenting Trial (CREST)

2522 symptomatic and asymptomatic


patients randomly assigned to
CEA or stent. >50% stenosis for
symptomatic and >60% for
asymptomatic

Results for Asymptomatic Patients


CAS CEA p
MI 7 (1.2%) 13 (2.2%) 0.2
All Stroke 15 (2.5%) 8 (1.4%) 0.15
Stroke/Death/MI 21 (3.5%) 21 (3.5%) 0.96

Brott TG et al. N Engl J Med 2010;363:11-23. Silver, FL. Stroke 2011, 42:675-680

Asymptomatic Carotid Disease


Medical Therapy is Improving – Should we Revasc?
• Stroke rates are decreasing due to medical therapy

• % stroke due to asymp. CAS is small and shrinking

• Asymptomatic CAS is a potent predictor of MI and


Death (and not a very good predictor of stroke)

• Trials of CEA show benefit (for stroke) in era of high


rates and no medical therapy…lack of benefit for
women, high risk patients, broad endpoints, etc.

• No one knows if CEA provides additional benefit on


top of optimal medical therapy…but there is risk
(ACST 3.1% risk of peri-op stroke or death at 30 days)

1127
Copyright © Harvard Medical School, 2018. All Rights Reserved.

Management Choices for Patients with


Unilateral Asymptomatic Carotid Stenosis
At Least Everyone is Confused…

Klein, A. NEJM 2008, 358:e23

Asymptomatic Carotid Disease


Considerations for Revascularization
• Patient Factors
• Life expectancy (>5 years)
• Comorbidities
• Treatment of concomitant risk factors

• Anatomic Factors
• High risk plaque characteristics, grade of stenosis
• TCD – is there active embolization?
• Anatomic issues informing procedural risk (hostile neck)

• Procedural Factors
• Volume, local outcomes (procedural risk < 3%)
• CEA (generally preferred especially in >70 YO) vs. Stenting (if
high surgical risk but must take DAPT for at least 30 days).

1128
Copyright © Harvard Medical School, 2018. All Rights Reserved.

Indications for Carotid Revascularization


ACC/AHA Guidelines
I IIa IIb III
• Patients at average or low surgical risk who
experience nondisabling ischemic stroke or TIA within
6 months should undergo CEA if the ipsilateral ICA
I IIa IIb III stenosis is >70%.
• CAS is indicated as an alternative to CEA for
symptomatic patients at average or low risk of
complications.
I IIa IIb III
• It is reasonable to perform CEA in asymptomatic
patients who have >70% stenosis of the ICA if the risk
of perioperative stroke, MI, and death is low.
I IIa IIb III • It is reasonable to choose CAS over CEA when
revascularization is indicated in patients with neck
anatomy unfavorable for CEA
Brott et a. JACC. 2011;57:1002-44

Question 1

A 64 year old man has been experiencing left calf discomfort for the past
three months. It occurs whenever he walks a distance of more than three
blocks. The physical examination reveals a blood pressure in each arm of
152/88 mm Hg. Both femoral pulses are present. The right dorsalis pedis
and posterior tibial pulses are present; the left dorsalis pedis and posterior
tibial are not palpable. The right ankle systolic pressure, measured with a
Doppler instrument, is 158 mm Hg; the left ankle pressure is 120 mm Hg.

Which of the following treatments should be considered next to improve


walking distance?

(A) Clopidogrel
(B) Supervised walking program
(C) Left iliac-femoral artery bypass graft
(D) Left iliac PTA/stent

1129
Copyright © Harvard Medical School, 2018. All Rights Reserved.

Question 1

(A) Clopidogrel
(B) Supervised walking program
(C) Left iliac-femoral artery bypass graft
(D) Left iliac PTA/stent

The ACC/AHA PAD guidelines recommend supervised exercise training


as an initial treatment modality for intermittent claudication. Clopidogrel
reduces the risk of adverse cardiovascular events but does not improve
walking time in PAD. Revascularization is indicated for life-style limiting
claudication despite optimal medical therapy. Moreover, the patient’s
physical examination suggests infra-inguinal disease, so it is unlikely
that iliac artery revascularization would be helpful.

Question 2
• A 78 year old woman presents with
a 5 minute episode of left arm
weakness. Her medications include
aspirin and atorvastatin. Her
angiogram is shown. Which of the
following additional treatments is
recommended?

A) Heparin
B) Dipyridamole
C) Carotid endarterectomy Right carotid artery
D) Carotid-subclavian bypass

1130
Copyright © Harvard Medical School, 2018. All Rights Reserved.

Question 2

A) Heparin
B) Dipyridamole
C) Carotid endarterectomy
D) Carotid-subclavian bypass

Carotid endarterectomy is indicated to treat symptomatic hemodynamically


significant carotid artery stenosis. The patient is already taking an
antiplatelet drug, and although addition of dipyridamole may be more
effective than aspirin alone in reducing recurrent stroke, it should not replace
carotid endarterectomy in this patient. There is no evidence that heparin
reduces risk of stroke in patients with symptomatic carotid artery stenosis.
Carotid-subclavian artery bypass is not a surgical procedure used in lieu of
carotid endarterectomy to treat internal carotid artery stenosis.

References
Aboyans et al: Measurement and interpretation of the ankle-brachial index: a
scientific statement from the American Heart Association. Circulation 126: 2890,
2012

Gerhard-Herman et al: ACC/AHA 2016 guidelines for the management of


patients with peripheral artery disease

Rooke et al: 2011 ACCF/AHA Focused Update of the Guideline for the
Management of Patients With Peripheral Artery Disease (updating the 2005
guideline): a report of the American College of Cardiology Foundation/American
Heart Association Task Force on Practice Guidelines." J Am Coll Cardiol 58:
2020, 2011.

Brott et al: 2011 ASA/ACCF/AHA Guideline on the Management of Patients With


Extracranial Carotid and Vertebral Artery Disease A Report of the American
College of Cardiology Foundation/American Heart Association Task Force on
Practice Guidelines. J Am Coll Cardiol 57: e16-9, 2011

1131
Copyright © Harvard Medical School, 2018. All Rights Reserved.

References

2010 ACCF/AHA/AATS/ACR/ASA/SCA/SCAI/SIR/STS/SVM Guidelines for the


Diagnosis and Management of Patients With Thoracic Aortic Disease

2011 ACCF/AHA Focused Update of the Guideline for the Management of Patients
With Peripheral Artery Disease (Updating the 2005 Guideline) : A Report of the
American College of Cardiology Foundation/American Heart Association Task
Force on Practice Guidelines

ACCF/AHA/ACR/SCAI/SIR/SVM/SVN/SVS 2010 Performance Measures for Adults


With Peripheral Artery Disease

Fleming C, Whitlock EP, Beil TL, Lederle FA. Screening for abdominal aortic
aneurysm: a best-evidence systematic review for the U.S. Preventive Services Task
Force. Ann Intern Med 2005;142:203-11.

1132
Copyright © Harvard Medical School, 2018. All Rights Reserved.

Heart Failure
Anju Nohria, MD
Assistant Professor
Advanced Heart Disease Section
Cardiovascular Division
Brigham and Women’s Hospital

Disclosures
• Amgen: research funding
• Takeda Oncology: consultant

1133
Copyright © Harvard Medical School, 2018. All Rights Reserved.

Outline
• Discuss pathogenesis and prevalence of heart failure

• Outline approach to diagnosis and evaluation of heart


failure patients

• Apply evidenced-based therapy to the population with


heart failure and reduced EF (HFrEF)

• Outline a management approach to heart failure with


preserved EF (HFpEF)

Pathophysiology of Heart Failure


Coronary Disease Cardiomyopathy Cardiac Overload

Left Ventricular Dysfunction

Neurohormonal
Vasoconstriction
Activation

↓ Peripheral Organ Bloodflow Cardiac Remodeling

↓Skeletal ↓RBF, Na LV LV
bloodflow retention Dilatation Hypertrophy
Arrhythmias
Symptoms, Fluid retention, Death

1134
Copyright © Harvard Medical School, 2018. All Rights Reserved.

Pathology of Heart Failure

HFpEF normal HFrEF


Concentric Remodeling Eccentric Remodeling
↑ Thickness ↔ Thickness
↔ Volume ↑ Volume
↓ Volume / Mass ↑ Volume / Mass

Images Courtesy of William Little and Marvin Konstam J Card Fail 9:1-3, 2003
Aurigemma, Zile, Gaasch Circulation 2006; 113; 296-304

HFpEF has Similar Outcomes as HFrEF


HF Admissions

• Approximately 50% of all HF


admissions are for HFpEF
• Pts w/ HFpEF are older, female,
HTN, CKD, Afib, ↓CAD

Cheng et al. Am Heart J 2014;168:721-30.e3

1135
Copyright © Harvard Medical School, 2018. All Rights Reserved.

Staging Heart Failure


ACC/AHA Classification
A. At risk patients without
structural heart disease
NYHA Classification
Progressive Disease

B. Structural heart disease I. Cardiac disease without


without symptoms functional limitation

Worsening QOL
C. Structural heart disease with II. Slight limitation of physical
prior or current symptoms activity

D. Refractory heart failure III. Marked limitation of physical


activity

IV. Inability to carry on physical


activity without discomfort

The Stage A Patient:


2013 ACC/AHA Recommendations
• Control hypertension w/ target < 130/80 mm Hg
– SPRINT trial: Intensive BP control reduces HF hospitalizations

• Treat lipid disorders in accordance with guidelines


• Control metabolic syndrome and diabetes
– EMPA-REG trial: Empagloflozin reduces incident HF

• Encourage smoking cessation and regular exercise


• ACE-I/ARB in appropriate patients (e.g. diabetes, vascular disease)
• Discourage excessive alcohol intake/illicit drug use
• Surveillance for LV dysfunction in high risk patients
– STOP-HF: Can use BNP > 50 pg/ml to refer to CV specialist to reduce incident HF

Yancy CW et al. CIR.0b013e31829e8776

1136
Copyright © Harvard Medical School, 2018. All Rights Reserved.

Goals of Therapy for Stage B HF


• Identify and treat reversible causes of LV dysfunction
• Prevent progressive LV remodeling and onset of
symptoms
– BP control in asymptomatic LVH
• Thiazide, CCB, or ACE-I
– Neurohormonal blockade in asymptomatic LV
systolic dysfunction
• ACE-I and Beta-blockers
• Aldosterone antagonist in post-MI LV dysfunction + DM

Goals of Therapy for Symptomatic HF:


Stage C HF
• Address precipitating • Identify and treat the
factors causative/inciting factor
• Initiate Rx to prevent dz • Neurohormonal blockade
progression and mortality • Manage related risks
(sudden cardiac death)
• Improve sxs and end-organ • Lower PCWP
perfusion • Increase CO
• Reduce LOS and re- • Pt education
hospitalization • Longitudinal dz
management programs

1137
Copyright © Harvard Medical School, 2018. All Rights Reserved.

Outcome Trials of
ACE Inhibitors in Heart Failure
NYHA Placebo
Patients Class Mortality Hazard ratio

V-HeFT II 804 I-III 25% 0.72


(Hyd/Iso)
CONSENSUS I 253 IV 44% 0.66

SOLVD Tx 2569 II-III 40% 0.84

SOLVD Px 4228 I 16% 0.91

SAVE 2231 Post MI 25% 0.81


EF<40%
ISIS-4 58,050 24h post MI 7.7% 0.93

ARB Trials in Heart Failure


ELITE I/II OPTIMAAL CHARM VALIANT ValHEFT

Patients NYHA II-IV Acute MI/CHF NYHA II-IV Acute MI/CHF NYHA II-IV
(n)
722/3152 5477 2548 14,808 5010

Study Losartan Losartan Candesartan Valsartan, Valsartan


Design or or and Captopril, or and
Captopril Captopril ACEI both ACEI
β-blocker
16% / 23% 79 % 55 % 70 % 35 %

Mortality No Captopril No No No difference


difference better difference differences

HF Hosp No Capt better Both better Both better Both better


difference
Other Losartan Losartan ↓ Mort. w/ ↓ BP w/ both ↑ Mort. w/
better better β-blker β-blker
tolerated tolerated

1138
Copyright © Harvard Medical School, 2018. All Rights Reserved.

β-Blocker Trials in Symptomatic HF


Annual Mortality

Trial Target Dose Mean Dose Control β-blocker RRR


(mg/d) (mg/d)) (%)
Bisoprolol
CIBIS I 5 3.8 11.0 8.7 NS
CIBIS II 10 7.5 13.2 8.8 34
Bucindolol
BEST 100-200 76 17 15 NS
Metoprolol
MDC 100-150 108 11.1 11.9 NS
MERIT-HF 200 159 11.0 7.2 34
Carvedilol
US Carvedilol 12.5-100 45 14.4 5.9 65
COPERNICUS 50 37 18.5 11.4 38

Aldosterone Antagonists in HF
Trial N LVEF NYHA End-pt HR

RALES1 1663 ≤ 35% III-IV All cause 0.7,


mortality p<0.001
EPHESUS2 6632 Post-MI EF II or All cause 0.85,
< 40% I w/ DM mortality P=0.008
EMPHASIS- 2737 EF < 30% II CV death 0.63,
HF3 or or HF p<0.0001
EF 30-35% hosp.
w/ QRS >
130

1Pitt, B et al. NEJM 1999;341:709-17; 2Pitt B et al. NEJM 2003;348:1309-21;


3Zennad F et al. NEJM 2011;364:11-21

1139
Copyright © Harvard Medical School, 2018. All Rights Reserved.

Sacubitril/Valsartan (LCZ696)
Mechanism of Action

Vardeny O et al. JACC: Heart Failure 2014;2:663-70.

PARADIGM-HF
Death from CV Causes or HF Hosp.
• N=8,442
• RCT:
– Enalapril 10mg bid vs. LCZ696
NNT=21 @ 27 mths
200mg bid
• Inclusion Criteria:
– EF ≤ 40% → 35%
– NYHA II-IV
– BNP ≥ 150, NT-BNP ≥ 600
– BNP ≥ 100 if hosp w/in 12 mths
– On optimal Rx
– SBP ≥ 95
* Increased hypotension, less renal
– eGFR > 30 ml/min dysfunction, no increase in
– K < 5.4 hyperkalemia, angioedema, or cough.

McMurray et al. NEJM 2014; 371:993-1004.

1140
Copyright © Harvard Medical School, 2018. All Rights Reserved.

Guideline Update
COR LOE Recommendations

I B-R ACEi OR ARB OR ARNI in conjunction with beta-blockers + MRA (where appropriate) is
recommended for patients with chronic HFrEF to reduce morbidity and mortality.
I B-R In patients with chronic, symptomatic HFrEF NYHA class II or III who tolerate and ACE
inhibitor or ARB, replacement by an ARNI is recommended to further reduce morbidity
and mortality
III B-R ARNI should NOT be administered concomitantly with ACEi or within 36 hours of last
ACEi dose
III C=EO ARNI should NOT be administered to patients with a history of angioedema

Population Initial Dose


Enalapril > 10 mg/d or 49/51 mg twice
Valsartan > 160 mg/d daily
Assess tolerability
Low dose ACE-I/ARB
Ensure 36 hrs off ACEi Up-titrate in step-wise
ACE-I/ARB naïve
Adequate BP fashion to 97/103 mg bid
eGFR<=30 mL/min/m2
eGFR ≥ 30 ml/min/1.73 m2 Re-assess BP, K and Cr
24/26 mg twice
Moderate Hepatic after each dose increase
daily
Impairment
(Child-Pugh Class B)
Elderly
Yancy et al. Circulation 2016; Yancy et al. JACC 2018;71(2):201-30.

Effect of Hydralazine/Isordil in
Symptomatic HF

Pts who are unable to take ACE-I/ARB should be placed


on hydralazine and isordil to reduce mortality and
improve symptoms

RR at 2 years: V-HeFT I 34% (p=0.028) V-HeFT II 28% (p=0.016)


Cohn et al. NEJM 1986;314:1547-52; Cohn et al. NEJM 1991;325:303-10.

1141
Copyright © Harvard Medical School, 2018. All Rights Reserved.

Alternative Vasodilator Strategies:


The A-HeFT Trial (Hydralazine/Isordil)
• 1050 NYHA III/IV AA pts
• Composite endpt (death, HF hosp,
QOL)
• Bidil (Hydralazine 37.5 mg + Isordil
20 mg) 2 tablets tid
– 68% at target
– Mean dose 3.8 tablets
• Contemporary background Rx
– ACEI/ARB 87 %
– Beta blkers 75 %
– Spironolactone 40 %
• Adverse events common
– HA 44% - Dizziness 29%
Taylor et al. NEJM 2004; 351:2049-57

Digoxin Reduces HF Hospitalization


But Not Mortality
30 50
Placebo n=93
DIGOXIN OVERALL MORTALITY
Withdrawal 40
20
No incremental benefit30
(and potential harm) at
p = 0.001 Placebo
10 Levels > 20
1.0 ng/mL
n=3403 p = 0.8

DIGOXIN n=85 10 DIGOXIN


0 n=3397
0
0 20 40 60 80 0 12 24 36 48
RADIANCE DIG Trial
N Engl J Med 1993;329:1 N Engl J Med 1997;336:525

1142
Copyright © Harvard Medical School, 2018. All Rights Reserved.

Ivabradine: A selective If Inhibitor


closed open

If
RR
Channel
Pure
0 mV heart rate
reduction

-40 mV

-70 mV
Ivabradine

If inhibition reduces the diastolic depolarization slope, thereby lowering heart rate
No effect on myocardial contractility or relaxation
Use-dependent block = low risk of bradycardia

Thollon et al. Br J Pharmacol. 1994;112:37-42.

SHIFT
Ivabradine (If inhibitor in SA node)
• N=6,558
• EF ≤ 35%, NYHA II-IV
• Resting HR ≥ 70 bpm
• On optimal med Rx
– ACE-I (91%)
– B-blocker (89%)
• Ivabradine: 5 bid → 7.5 bid
• Average HR: 64 vs 75 bpm @1 yr
• Greater the reduction in HR, greater
the benefit
• Side effects
– Symptomatic bradycardia: 5 vs 1%
– Phosphenes: 3 vs 1%
Swedberg et al. Lancet 2010;376:875-85.

1143
Copyright © Harvard Medical School, 2018. All Rights Reserved.

Guideline Update
COR LOR
IIa B-R Ivabradine may be beneficial to reduce HF
Moderate
hospitalization for patients with symptomatic
stable chronic HFrEF who are receiving GDMT,
including a beta blocker at maximum
tolerated dose, and who are in sinus rhythm
with a heart rate of ≥ 70 bpm at rest.

Age ≥ 75 Ivabradine HR < 50 Reduce to 2.5 mg


Re-assess whether b- 2.5 mg bid bid or stop
blockers maximized HR 50-60 Continue 5 mg bid
Assess eligibility for Age ≤ 75 Ivabradine 5
ivabradine mg bid HR > 60 Increase to 7.5 mg
bid

Yancy et al. Circulation 2016; Yancy et al. JACC 2018;71(2):201-30.

Iron Repletion in HF
• 50% HF patients have iron deficiency, with or without anemia
• Iron deficiency in HF is associated with ↑ mortality, independent of anemia
• No improvement in all-cause mortality and HF hospitalization with darbopoeitin
• No improvement in functional capacity or QOL with oral iron

Ponikowski et al. Eur Heart J 2015;36(11):657-68.

1144
Copyright © Harvard Medical School, 2018. All Rights Reserved.

2017 Guideline Update for Iron


Deficiency in HF

Yancy et al. Circulation 2017. DOI:10.1161/CIR0000000000000509

Heart Failure Management:


More Than Just Drugs
• Dietary counseling
• Patient education
• Physical activity
• Medication compliance
• Nonpharmacologic Therapies
– CRT
– Sleep Disordered Breathing
• Management of Related Risks
– Sudden Death (ICD implantation)
– Thromboembolism/Stroke

1145
Copyright © Harvard Medical School, 2018. All Rights Reserved.

Cardiac Synchronization Therapy in


HF
• In pts w/ EF < 35%, NSR, QRS > 120 ms, and on
optimal medical therapy, CRT:
• Promotes reverse remodeling (even in NYHA I)
• Reduces HF hospitalizations (even in NYHA I)
• Improves symptoms
• Improves survival
• Predictors of benefit:
• LBBB morphology
• QRS duration ≥ 150 msec

CRT Guideline Update


• Class I: EF ≤ 35%, NYHA II-IV, NSR, LBBB w/ QRS ≥ 150 msec,
optimal med Rx
• Class IIa: EF ≤ 35%, NYHA III-IV, NSR, LBBB w/ QRS 120-150
msec, optimal med Rx
• Class IIa: EF ≤ 35%, NYHA III-IV, NSR, non-LBBB w/ QRS ≥
150 msec, optimal med Rx
• Class IIb: EF ≤ 30%, CAD, NYHA I, NSR, LBBB w/ QRS ≥ 150
msec

• *Not indicated in NYHA I-II, non-LBBB w/ QRS < 150 msec.

Circulation 2012; 126: 1784-1800

1146
Copyright © Harvard Medical School, 2018. All Rights Reserved.

Indications for ICD Therapy in HF


• Cardiac Arrest
• Sustained VT
• EF<40%, CAD, NSVT, inducible VT
• EF<30%, > 40d post-MI or 3mths post-
revascularization, NYHA I-III
• EF<35%, Non-ischemic CMP, NYHA II-III
• Contraindicated in NYHA IV, unless bridge to
advanced therapies

Anticoagulation in Patients with Heart


Failure and Sinus Rhythm (WARCEF)
N=2305;
HR=0.93, p=0.40 ASA 325 mg daily vs.
Warfarin (INR 2-3.5)

Reduced risk of
ischemic stroke
with warfarin offset
by increase in major
hemorrhage

Homma S, et al. N Engl J Med 2012;366: 1859-69.

1147
Copyright © Harvard Medical School, 2018. All Rights Reserved.

Symptomatic HF is a Clinical
Diagnosis

Nohria et al. JACC 2003;41:1797-1804.

BNP to Assist Diagnosis of HF


Pts presenting to EW with dyspnea

Maisel AS, et al. NEJM 2002;347:161


Januzzi J et al. Am Heart J 2005;149:744.

1148
Copyright © Harvard Medical School, 2018. All Rights Reserved.

Treatment of
Decompensated HF
DRY WET

WARM A Diuretics B
Vasodilators
Nitroprusside
Nitroglycerin
or
COLD L C Nesiritide

Low/Marginal BP Adequate BP

Inotropic Drugs
Dopamine
Dobutamine
Milrinone

Stevenson LW. Eur J Heart Failure 1999

DOSE Trial
Low High
• N=308 pts with ADHF, < 24 hrs admission
Dose Dose

HIGH vs. LOW DOSE Diuretics Q12 1X oral 2.5X oral


• ↑ improvement in dyspnea @ 72 hrs Bolus
• ↑ net diuresis and weight loss @ 72 hrs 1X oral 2.5X oral
Continuous
• ↑ proportion w/ WRF (↑Cr > 0.3 mg/dL)
• No diff in death, re-hospitalization, or ED visits @ 60d

BOLUS vs. CONTINUOUS Diuretic Infusion


• No difference in any outcomes

Felker et al. NEJM 2011;364:797-805.

1149
Copyright © Harvard Medical School, 2018. All Rights Reserved.

Inotropes Increase Mortality in ADHF


• ESCAPE trial
• 433 pts w/ ADHF, EF < 30%, SBP ≤ 125 mm Hg, 1 sign and
symptom of HF, HF hosp. w/in previous yr

• Inotropes↑ myocardial oxygen demand, arrhythmias, neurohormonal activation

Elkayam et al. Am Heart J 2007;153:98-104.

Who Should be Referred for


Advanced Therapies?
• Escalating diuretic dose requirements
• Progressive renal dysfunction
• Increasing frequency of HF hospitalization
• Increasing burden of ventricular arrhythmias
• Intolerance of standard medical therapy
• Refractory HF symptoms
• Need for inotropic support
• > 5% non-fluid related weight loss (cachexia)
• Peak oxygen consumption ≤ 10 ml/kg/min w/ RER >1

1150
Copyright © Harvard Medical School, 2018. All Rights Reserved.

HF-PEF: A Heterogeneous Clinical


Syndrome
• Must Exclude
• Primary Infiltrative, restrictive, or hypertrophic heart
disease
• Pericardial Disease
• Critical coronary artery obstruction
• Hemodynamically significant valvular disease
• Advanced renal (renovascular) disease
• Hypertensive Emergency
• Uncontrolled atrial arrhythmias
• Primary pulmonary hypertension

2017 Guideline Update for HFpEF

Yancy et al. Circulation 2017. DOI:10.1161/CIR0000000000000509

1151
Copyright © Harvard Medical School, 2018. All Rights Reserved.

Congestive Heart Failure: Summary


• Heart failure is a clinical diagnosis
• BNP can be helpful when diagnosis is uncertain, but should
not replace clinical assessment
• ACEi and beta-blockers are cornerstones of HF therapy and
should be titrated to target doses if tolerated
• ARBs should be used in ACEi intolerant patients
• Substitution w/ ARNI should be considered in pts on ACEi or
ARB to reduce HF mortality and hospitalization
• Beta blockers should not be started in acutely
decompensated patients

Congestive Heart Failure: Summary


• Aldosterone antagonists are increasingly the favored
‘second-line’ after ACEi/ARB and beta-blocker
• Hydralazine/Isordil is an alternative for the ACEi/ARB
intolerant and may be added for those still symptomatic on
ACEi/Beta-blocker/aldosterone antagonist
• Digoxin and ivabradine can be considered to reduce HF
hospitalization
• Device Therapy (ICD +/- CRT) is appropriate for many HF
patients with LVEF ≤ 35%
• Heart failure with preserved EF remains a poorly
understood, heterogeneous disorder with limited
therapeutic options

1152
Copyright © Harvard Medical School, 2018. All Rights Reserved.

Disclosures
• Amgen: research funding
• Takeda Oncology: consultant

References
• Yancy CW et al. 2013 ACCF/AHA guideline for the management of
heart failure. J Am Coll Cardiol. 2013;62(16):e147-239.
• Hunt S et al. HFSA 2010 comprehensive heart failure practice
guideline. J Cardiac Fail. 2010;16:e1-e194.
• McMurray JJV et al. Angiotensin-neprilysin inhibition versus
enalapril in heart failure. N Engl J Med. 2014;371:993-1004.
• Swedberg K et al. Ivabradine and outcomes in chronic heart
failure: a randomized placebo-controlled study. Lancet
2010;376:875-885.
• Abraham WT et al. Wireless pulmonary artery hemodynamic
monitoring in chronic heart failure: a randomised clinical trial.
Lancet 2011;377:658-666.

1153
Copyright © Harvard Medical School, 2018. All Rights Reserved.

Question 1
• A 45 yo woman with non-ischemic CMP (EF 30%) presents for a
routine clinic visit. She reports dyspnea after climbing 1 flight of stairs
but is otherwise asymptomatic.
• Medications include lisinopril 20 mg daily, carvedilol 12.5 mg bid,
spironolactone 25 mg daily, and furosemide 40 mg bid.
• Exam reveals HR 70 bpm BP 120/80. She has no JVD, clear lungs, RRR,
Nl S1, S2, soft systolic murmur @ the apex, no hepatosplenomegaly
and no edema
• Labs are notable for Na 136, K 4.8, BUN/Cr 20/1.2
• EKG shows NSR @ 70 bpm, LBBB with QRS duration 130 msec.

Question 1 contd.
• What would be the best next step in the
management of this patient?
A) Placement of a biventricular pacemaker –
defibrillator
B) Up-titration of lisinopril
C) Up-titration of carvedilol
D) Stop lisinopril and start valsartan/sacubitril

1154
Copyright © Harvard Medical School, 2018. All Rights Reserved.

Rationale
• All the options presented are appropriate,
however, the intervention that is likely to give
her the greatest morbidity and mortality
benefit would be to stop her Lisinopril and
start sacubitril/valsartan. Since she is
tolerating Lisinopril 20 mg daily, we would
start her on 49/51 mg bid and then up-titrate
to 97/103 mg bid if tolerated by BP, K, and Cr.

Question 2
• A 60 yo man with known ischemic CMP (EF 35%) presents with new
onset atrial fibrillation and rapid ventricular response.
• His medications include losartan 50 mg daily, metoprolol succinate
200 mg daily, torsemide 20 mg daily, aspirin 81 mg daily, and
atorvastatin 80 mg daily.
• His exam is notable for a HR 110 bpm, BP 90/60, JVP 12 cm of water
sitting upright, bibasilar crackles, Irreg, irreg Nl S1, S2, no
hepatosplenomegaly and no edema.
• Labs with Na 137, K 4.8, BUN/Cr 25/1.4
• EKG with atrial fibrillation and VR 110 bpm. Old AMI. Non-specific ST-
T abnormalities.

1155
Copyright © Harvard Medical School, 2018. All Rights Reserved.

Question 2 contd.
• What would be the next best step in the
management of this patient?
A) Add digoxin
B) Add ivabradine
C) Change to carvedilol
D) Emergent electrical cardioversion

Rationale
• He is hemodynamically stable and therefore
does not require urgent cardioversion. He is
on maximal dose metoprolol succinate with
marginal BP and therefore changing to
carvedilol is unlikely to help with rate control
and may cause hypotension. Ivabradine is not
indicated since the pt is in Afib. Digoxin is the
best option since it may help with rate control
without compromising BP.

1156
Copyright © Harvard Medical School, 2018. All Rights Reserved.

Adult Congenital Heart Disease


Anne Marie Valente, MD

Boston Adult Congenital Heart Disease and Pulmonary


Hypertension Program
Division of Cardiology
Brigham and Women’s Hospital, Boston Children’s Hospital
Associate Professor, Harvard Medical School

Disclosures

• No disclosures

1157
Copyright © Harvard Medical School, 2018. All Rights Reserved.

Outline
• To recognize the increasing prevalence of adults
living with congenital heart disease
• To emphasize key points about common ACHD
conditions
• To focus on the hemodynamic changes in pregnancy
and the risk factors for women with heart disease

CHD Patients Reaching Adulthood


2,600,000 –
– 2,300,000
20,000 new patients/yr
2,200,000 –
5% increase/yr

U.S. 1,800,000 –
Adult –
CHD 1,400,000 –
Patients – 1,400,000
1,000,000 –
– 1,000,000
600,000 –
750,000
– 500,000 Hoffman 1978 Kaiser
200,000 – 325,000 Fyler 1980 New England
2-0 – Ferencz 1985 Baltimore-DC
Gilboa 2016 Quebec
1970 1980 1990 2000 2010 2020

1158
Copyright © Harvard Medical School, 2018. All Rights Reserved.

Breakdown of Severity of CHD in Adults


Heterotaxy
Complex Single Ventricle
Transposition
Conduits
Cyanotic
Eisenmenger
15%
47%
38%

Simple Simple ASD


Simple Aortic Disease Tetralogy of Fallot
Simple Mitral Disease
Simple PDA
Moderate Coarctation
AV Canal Defects
Ebstein

Warnes C et al. J Am Coll Cardiol 2001


Marelli A et al. Am Heart J 2009

ACHD: Multisystem Disorder


Neurologic: Increased incidence of occult or clinically evident strokes
Decreased level of executive functioning skills
Anxiety, post-traumatic stress disorder, depression
Psychosocial disorders
Lungs: Restrictive lung disease
Pulmonary vascular disease
Renal: Decreased perfusion
Hepatic: Liver fibrosis
Peripheral Vasculature: Increased chronic venous insufficiency
Orthopedic: Scoliosis
Kyphosis
Hematologic: Anemia
Coagulopathies

1159
Copyright © Harvard Medical School, 2018. All Rights Reserved.

ACHD: Causes of Death

n = 6933 subjects CONCOR registry


n = 197 deaths; median age 48 yrs

Verheugt CL et al. Eur Heart J 2010

Genetic Considerations
Syndrome Common CHD Genetic Abnormality
DiGeorge syndrome Aortic arch anomalies, TOF, 22q11.2 deletion
Truncus arteriosus

Down syndrome ASD, VSD, AVC, TOF Trisomy 21

Holt-Oram syndrome ASD, VSD, MV disease TBX5

Noonan syndrome PS, ASD, HCM PTPN11, KRAS, SOS1, RAF1,


NRAS, BRAF, MAP2K1
Turner syndrome Coarctation, BAV, 45X
Aortopathy

Williams syndrome Supravalvar AS, 7q11.23 deletion


Arteriopathy

1160
Copyright © Harvard Medical School, 2018. All Rights Reserved.

Patent Foramen Ovale (PFO)

• Patency of the flap valve


of the fossa ovalis
• Present in 25% of adults
• Not associated with R
heart dilation

Echocardiography in Pediatric and Congenital Heart Disease, ed. Geva T . Wiley, 2016

PFO and Cryptogenic Stroke


CLOSE REDUCE

<60 years age, cryptogenic stroke PFO, large shunt or aneurysm, no cerebrovascular
disease
At 5.3 years, PFO closure had a lower risk • At 3.2 years, PFO closure had a
of recurrent stroke than those lower risk of recurrent stroke than
maintained on antiplatelet therapy (0% antiplatelet therapy (1.4% vs.
vs. 6%; HR, 0.03) 5.4%; HR, 0.23).

N Engl J Med 2017: 377:1011–1021 N Engl J Med 2017: 377:1033–1042

1161
Copyright © Harvard Medical School, 2018. All Rights Reserved.

PFO Management

Mojadidi MK et al. J Am Coll Cardiol 2018

Atrial Septal Defect (ASD)

• Left-to-right shunt causing pulmonary


overcirculation and R heart dilation
• Palpitations, dyspnea, exercise
intolerance
• Exam findings: Fixed, split S2
• EKG: RBBB, RAD*

*for secundum ASDs (most common)


If LAD -> think primum ASD

Harrison’s Textbook of Internal Medicine, ed. Loscalzo J. McGraw-Hill, 2018

1162
Copyright © Harvard Medical School, 2018. All Rights Reserved.

Types of ASD

• Secundum ASD: most common


• Primum ASD: on the spectrum of
AVC defects; associated with a
cleft mitral valve
• Sinus venosus defect*: associated
with partial anomalous
pulmonary venous return

* Not in the atrial septum

Echocardiography in Pediatric and Congenital Heart Disease, ed. Geva T. Wiley, 2016

Partial Anomalous Pulmonary Venous Return

• Left-to-right shunt often discovered


incidentally
• Results in mild R heart dilation
• Often asymptomatic or presents
with symptoms similar to ASD

Harrison’s Textbook of Internal Medicine, ed. Loscalzo J. McGraw-Hill, 2018

1163
Copyright © Harvard Medical School, 2018. All Rights Reserved.

Ventricular Septal Defect


• Left-to-right shunt causing left
heart dilation
• Large VSDs cause symptoms of
heart failure and poor growth in
children
• Many VSDs found in adults are
pressure-and flow-restrictive and
do not cause LV dilation or
symptoms
• If not restrictive -> Eisenmenger
syndrome
Harrison’s Textbook of Internal Medicine, ed. Loscalzo J. McGraw-Hill, 2018

Types of VSD

• Muscular: often pressure- and


flow-restrictive
• Membranous: “outlet”
• AV canal type: associated with
abnormalities of the AV valves
• Subpulmonary: “conal septal”
associated with R coronary cusp
prolapse and aortic insufficiency

Echocardiography in Pediatric and Congenital Heart Disease, ed. Geva T . Wiley, 2016

1164
Copyright © Harvard Medical School, 2018. All Rights Reserved.

Pressure-Restrictive VSD

The key is to have a HIGH


velocity on Doppler
interrogation of the VSD flow

4v2
~5 m/s = 4(25) = 100 mmHg
gradient between the RV and
LV
Essential Atlas of Cardiovascular Disease, ed Libby P. Springer, 2009

Patent Ductus Arteriosus

• Exam: continuous murmur


• Large PDAs cause left heart
dilation and may lead to
Eisenmenger syndrome
• Transcatheter closure preferred
if no evidence of pulmonary
hypertension

Harrison’s Textbook of Internal Medicine, ed. Loscalzo J. McGraw-Hill, 2018

1165
Copyright © Harvard Medical School, 2018. All Rights Reserved.

Expected Chamber Enlargement with Cardiac Shunts

Shunt RA RV PA LA LV Aorta
ASD + + +
VSD +/- + + +
PDA + + + +

Eisenmenger Syndrome

ASD, VSD, or complex Over time, PVR PVR ↑’s: shunt reverses:
defect ↑ resulting in R-to-L → Eisenmenger
↑ Qp and/or PAp, with L-to- bi-directional flow syndrome: ↑ cyanotic
R shunting

1166
Copyright © Harvard Medical School, 2018. All Rights Reserved.

Eisenmenger Syndrome: Complications


• Hyperviscosity symptoms
• Bleeding
• Thrombosis, including stroke
• Arrhythmias
• Heart failure
• Gout
• Gallstones, cholecystitis
• Renal dysfunction
• Hypertrophic osteoarthropathy
• Sudden death

Ebstein Anomaly
• Failure of delamination of the
septal leaflet of the TV
• Results in “atrialized” portion of
the RV
• Symptoms depend on severity
of TR and associated lesions
– Signs of R heart failure
– Cyanosis (if ASD/PFO)
• Exam: Widely split S1 “sail sign”
• 20% of patients have WPW

Harrison’s Textbook of Internal Medicine, ed. Loscalzo J. McGraw-Hill, 2018

1167
Copyright © Harvard Medical School, 2018. All Rights Reserved.

Ebstein Anomaly

Essential Atlas of Cardiovascular Disease, ed Libby P. Springer, 2009

Tetralogy of Fallot (TOF)


• The most common form of
cyanotic CHD
• Spectrum of severity
• Surgical repairs have changed
over time
– Palliative shunts
– Transannular patch
– RV-PA conduit
• Many adults undergo PVR
later in life

Harrison’s Textbook of Internal Medicine, ed. Loscalzo J. McGraw-Hill, 2018

1168
Copyright © Harvard Medical School, 2018. All Rights Reserved.

Repaired TOF: Sequelae in Adulthood


• RA/RV dilation
• RV dysfunction
• Pulmonary regurgitation
• Tricuspid regurgitation
• Branch PA stenosis
• Residual VSD
• LV dysfunction
• Aortic root dilation
• Arrhythmias
• Sudden death
Harrison’s Textbook of Internal Medicine, ed. Loscalzo J. McGraw-Hill, 2018

Transposition of the Great Arteries (TGA)

Complete TGA Congenitally Corrected TGA


D-loop TGA L-loop TGA
Harrison’s Textbook of Internal Medicine, ed. Loscalzo J. McGraw-Hill, 2018

1169
Copyright © Harvard Medical School, 2018. All Rights Reserved.

TGA s/p Atrial Switch

• Senning Procedure (1957)


• Mustard Procedure (1963)
• Redirects blood flow to allow
systemic venous return to get
to the lungs
• Long-term complications:
– Systemic RV dysfunction
– Tricuspid regurgitation
– Atrial baffle complications
– Atrial arrhythmias
Harrison’s Textbook of Internal Medicine, ed. Loscalzo J. McGraw-Hill, 2018

TGA s/p Arterial Switch

• Jatene procedure (1975)


• The great arteries are
transected and switched – the
coronaries are re-implanted
• Long-term complications:
– Branch PA stenosis
– Neo-aortic root dilation
– Neo-aortic regurgitation
– Coronary artery stenosis

Harrison’s Textbook of Internal Medicine, ed. Loscalzo J. McGraw-Hill, 2018

1170
Copyright © Harvard Medical School, 2018. All Rights Reserved.

Congenitally Corrected TGA

• AV and VA discordance
• Symptoms depend on associated
lesions:
– ASD
– Tricuspid valve dysplasia ± TR
– Pulmonary stenosis
• 1% incidence of complete heart
block per year in adulthood
• May be diagnosed in adulthood

Harrison’s Textbook of Internal Medicine, ed. Loscalzo J. McGraw-Hill, 2018

Coarctation of the Aorta


• Presents with hypertension
• Exam: Brachial-femoral delay
and diminished LE pulses
• Associations:
– Bicuspid aortic valve
– Ascending aortic dilation
– Aneurysms
– LVH
– Coronary artery disease
– Cerebral aneurysm

Harrison’s Textbook of Internal Medicine, ed. Loscalzo J. McGraw-Hill, 2018

1171
Copyright © Harvard Medical School, 2018. All Rights Reserved.

Pregnancy Hemodynamics
50 Plasma volume
Stroke volume
% change from prepregnancy value

Heart rate
40 Cardiac output
SVR
30

20

10

-20

-40
4 8 12 16 20 24 28 32 36 Post-partum
Duration of pregnancy (weeks)

Risk Score for Pregnant Women with HD


PREDICTOR POINTS

Prior cardiac events or arrhythmias 3


Baseline NYHA III-IV or cyanosis 3
Mechanical valve 3
Ventricular dysfunction 2
High risk left-sided valve disease / left ventricular 2
outflow tract obstruction
Pulmonary hypertension 2
Coronary artery disease 2
High risk aortopathy 2
No prior cardiac intervention 1
Late pregnancy assessment 1

Silversides C et al. J Am Coll Cardiol 2018

1172
Copyright © Harvard Medical School, 2018. All Rights Reserved.

ACHD Question #1
A 30-year-old man is seen for systemic hypertension,
not controlled on two medications. His blood pressure
is 190/90 in the right arm. He has diminished femoral
pulses. Which of the following is not associated with
his condition?
a) Bicuspid aortic valve
b) Differential cyanosis
c) Risk of premature coronary artery disease
d) Intracranial aneurysm

ACHD Answer #1
A 30-year-old man is seen for systemic hypertension,
not controlled on two medications. His blood pressure
is 190/90 in the right arm. He has diminished femoral
pulses. Which of the following is not associated with
his condition?
a) Bicuspid aortic valve
b) Differential cyanosis
c) Risk of premature coronary artery disease
d) Intracranial aneurysm

1173
Copyright © Harvard Medical School, 2018. All Rights Reserved.

ACHD Answer #1
• Differential cyanosis
(cyanosis of the toes but not
the fingers)
• Occurs with a PDA and
Eisenmenger syndrome
• Right-to-left shunt bypasses
the upper limb vessels and
flows directly into the
descending aorta
• Does not occur with CoA

ACHD Question #2
A 28-year-old woman presents with a history of
childhood heart surgery. She is 22 weeks pregnant.
Which condition places her at highest risk for a
maternal cardiac event during pregnancy?

a) Unrepaired atrial septal defect with normal PA


pressure
b) Mild pulmonary stenosis, unrepaired
c) Repaired tetralogy of Fallot
d) Congenital mitral stenosis, s/p mechanical valve

1174
Copyright © Harvard Medical School, 2018. All Rights Reserved.

ACHD Answer #2
A 28-year-old woman presents with a history of
childhood heart surgery. She is 22 weeks pregnant.
Which condition places her at highest risk for a
maternal cardiac event during pregnancy?

a) Unrepaired atrial septal defect with normal PA


pressure
b) Mild pulmonary stenosis, unrepaired
c) Repaired tetralogy of Fallot
d) Congenital mitral stenosis, s/p mechanical valve

ACHD Answer #2
• Women with heart disease have a 15% incidence of
maternal cardiac complications during pregnancy
• Mechanical heart valves are included in the CARPREG
II risk score
• Complications include: valve thrombosis, valve
failure/dysfunction, endocarditis, cerebral vascular
accident, hemolysis, bleeding, ventricular
dysfunction, heart failure and arrhythmias
• Anticoagulation strategies must be carefully
individualized with meticulous monitoring during
pregnancy

1175
Copyright © Harvard Medical School, 2018. All Rights Reserved.

Summary

• The number of adults living with congenital heart


disease is growing
• Surgical strategies have evolved over time and long-
term complications depend on associated lesions
and surgical era of repair
• Many women with CHD tolerate the hemodynamic
changes of pregnancy. However, morbidity is
common, particularly with high-risk conditions

Resources
• Stout K et al. 2017 ACC/AHA Guideline for the Management of
Adults With Congenital Heart Disease. Soon to be published… 2018

• Valente AM, Landzberg MJ. Adult Congenital Heart Disease. Chapter


264. Harrison’s Textbook of Internal Medicine, ed. Loscalzo J.
McGraw-Hill, 2018.

• Regitz-Zagrosek V et al. ESC Guidelines on the management of


cardiovascular diseases during pregnancy The Task Force on the
Management of Cardiovascular Diseases during Pregnancy of the
European Society of Cardiology (ESC). European Heart Journal,
2011. 32(24), 3147-3197.

1176
Copyright © Harvard Medical School, 2018. All Rights Reserved.

Thank You!
Boston Adult Congenital Heart & Pulmonary Hypertension Program

Amvalente@partners.org

1177
Copyright © Harvard Medical School, 2018. All Rights Reserved.

Intensive Review of Internal Medicine

Dale S Adler, MD
Executive Vice Chair
Department of Medicine
Brigham and Women’s Hospital
No Disclosures

All 15 cases are real


Nothing unusual
Can be figured out with a history, good cardiac examination & EKG

Think about epidemiology and physiology

Coronary, valvular, myocardial, pericardial and vascular


conditions

1178
Copyright © Harvard Medical School, 2018. All Rights Reserved.

Usually
occurs at
night as she
Case #1:
lies down
in bed
No SOB

It happened once
when I was walking
up the stairs
carrying my
briefcase

Rxxx59744

Case #1:
Medical Chronic left rotator cuff
difficulties.

History
Strong family history of
premature CAD.

BMI 32

Hypertension,
vigorously treated, and
controlled.

Dyslipidemia treated
with statin, with TC185
Rxxx59744 HDL50 LDL98 TG 211.

1179
Copyright © Harvard Medical School, 2018. All Rights Reserved.

Case #1: Medications

Aspirin Atenolol Lisinopril Furosemide Pravastatin Metformin

Rxxx59744
Basal insulin

Case #1: Exam


150/80 mmHg HR 100 bpm RR 14

JVP not elevated

Carotid upstrokes and volumes normal

Clear lungs

PMI non-displaced

S1 normal S2 Ø

2/6 systolic murmur, LSB, early peaking, decreases with Valsalva

Left arm hurts with rotation

Has neck sensation during exam


Rxxx59744

1180
Copyright © Harvard Medical School, 2018. All Rights Reserved.

What are the Appropriate Next


Steps?

Case #1: EKG

Rxxx59744

1181
Copyright © Harvard Medical School, 2018. All Rights Reserved.

Case #1: EKG

Rxxx59744

Rxxx59744
Multifactorial
Intervention

Gaede P, Lund-Anderson H, Parving HH, Pedersen O. Effect of a multifactorial intervention on mortality in type 2 diabetes. NEJM 2008; 358:580-91.

1182
Copyright © Harvard Medical School, 2018. All Rights Reserved.

Anatomy
Normal left main

95% LAD stenosis


after first
diagonal

Non dominant
LCX without
irregularities

80% RCA stenosis


after A.M. 1 60%
R-PDA stenosis

Rxxx59744

Revascularization
for ACS is
appropriate.

Stents or surgery
associated with equal
survival in non DM pts
with multi-vessel
disease and good LV
function.

For multi-vessel
disease with non
complex lesions,
stents or surgery
equal in DM pts.

1183
Copyright © Harvard Medical School, 2018. All Rights Reserved.

More complex anatomy,


greater benefit to CABG

32.4
20.3
14.2
24.3 6.1
10.5
4.8 2.2
0.004 0.001 13.5 0.04
12.2
5.4 0.001 4.1
Non DM
0.003 0.004
DM
CABG PES p CABG PES p CABG PES p

Composite MACE Cardiac Death Repeat Revasc

13

FREEDOM Trial
Future REvascularization Evaluation in Patients with Diabetes Mellitus: Optimal Management of
Multivessel Disease

Farkouh, ME et al
Freedom Trial, NEJM 2012; 367:2375-84

1184
Copyright © Harvard Medical School, 2018. All Rights Reserved.

Adjusted annualized rates of cardiovascular death and heart failure admission among patients referred for
coronary angiography by coronary flow reserve (CFR) and early revascularization (Revasc) strategy
(coronary artery bypass grafting [CABG], percutaneous coronary intervention [PCI], or neither).

Lower reserve needs more extensive/reliable


revascularization

Taqueti V R et al. Circulation. 2015;131:19-27

Crohn’s
Disease
Family History Stable
Father & paternal
grandfather had
MI’s in 6th decade

Impaired Glucose
Tolerance
A1c 6.3%
Treated with metformin
Angina
>1 year Smoking
quit in 5th decade

Cholesterol Hypertension
TC 180 Treated with Beta-blocker
HDL 70 ACE Inhibitor
LDL 92
TG 91
Atorvastatin 40mg

Case Study: MXXR - A 75 Year Old Woman with Easily Precipitated


Exertional…..

1185
Copyright © Harvard Medical School, 2018. All Rights Reserved.

Exercise Study CTA


(outside institution)
• 7 minutes, 30 seconds
(outside institution)
• Standard Bruce protocol
• Stops due to chest pain & SOB • 40-50% proximal LAD and LCx
• HR 118 stenosis
• BP 164/70 mmHg • 30% mid RCA stenosis
• 1.5mm horizontal ST segment
depression (resolves after 3
minutes of recovery)
• Perfusion images normal
• EF 77%

Case Study: MXXR

Exam
• 127 pounds (trim)
• BP 140/70 mmHg (both arms)
• HR 58
• RR 12
• JVP not elevated (<8)
• Carotid upstrokes & volumes normal
• Clear lungs
• PMI not displaced
• Single S1, physiologically split S2 with
mid systolic click at apex that moves
earlier in cardiac cycle with Valsalva

Case Study: MXXR

1186
Copyright © Harvard Medical School, 2018. All Rights Reserved.

The story & studies are The story & studies are
consistent with important consistent with
epicardial coronary microvascular angina Which of the
disease, and if symptoms with an outstanding
cannot be managed, prognosis and her statements
coronary angiography is medical regimen can be
indicated. reduced. are true?

The story is consistent The story is consistent


with mitral valve with the non-GI effects
prolapse. of Crohn’s Disease.

Case Study: MXXR

Cath
• Dominant RCA with
narrow right PDA and
PLV
• 30% LAD non-calcified
plaque
• FFR across LAD 0.96

Case Study: MXXR

1187
Copyright © Harvard Medical School, 2018. All Rights Reserved.

Sex Differences in CAD Outcomes

481 Men
72% had >50% stenosis
p>0.0001 p>0.0001

LJ Shaw, et al
CIRC Cardiovasc Imaging 2010 3:473-481

Sex Differences in CAD Outcomes


Non-obstructive CAD warrants attention in all, and especially in women.

LJ Shaw, et al
CIRC Cardiovasc Imaging 2010 3:473-481

1188
Copyright © Harvard Medical School, 2018. All Rights Reserved.

Extent Of Epicardial CAD, In This Population Referred For PET Exercise


Testing Then Angiography, Did Not Predict Global CFR.

Figure 1.

Taqueti V R et al. Circulation. 2015;131:19-27

Freedom From Cardiovascular Death Or Heart Failure Admission According


To Coronary Flow Reserve (CFR) And Angiographic Score (Coronary Artery Disease
Prognostic Index [CADPI]).

Taqueti V R et al.
Circulation. 2015;131:19-27

1189
Copyright © Harvard Medical School, 2018. All Rights Reserved.

Case Study:
61 year old man OX-500
Sudden onset SOB three
days post left total knee Anxious & diaphoretic
BP 108/60 mmHg
replacement
HR 98
RR 20
History Pulse Ox 79%
Hypertension
Dyslipidemia
Previous radiacal
prostatectomy JVP not elevated
Traumatic splenectomy Carotid upstrokes & volumes normal
Crackles at left lung base
PMI not displaced
Troponin-I 0.75 No sternal lift
CK 105 Normal S1, physiologically split S2, P2
MB 3.7 is not increased
Left leg swollen, above & below knee

Case Study: OX-500 - EKG

1190
Copyright © Harvard Medical School, 2018. All Rights Reserved.

This man, with multiple CAD risk factors, is most likely


having an acute coronary syndrome.
He should be heparinized and he should have coronary
angiography.

This man, s/p orthopedic surgery, is most likely


struggling with a pulmonary embolism.
He should be heparinized and have an echocardiogram.

This man should receive lytic therapy


immediately which will be helpful in either
circumstance

Case Study: OX-500

Case Study: OX500 - Echo

Dilated RV with paradoxic septum

1191
Copyright © Harvard Medical School, 2018. All Rights Reserved.

Case Study: OX500 - CT

Case Study: OX500


What are the logical next steps?
Lytic agents, given size of PE

IVC filter and lytic agents

IVC filter and ongoing heparinization

IVC filter and catheter or surgical embolectomy and


ongoing heparinization

1192
Copyright © Harvard Medical School, 2018. All Rights Reserved.

Case Study: Ox500 - Outcomes

Before Procedure After


• HR: 115
• BP: 100/70 Suction • HR : 87
• O2 Sat: 94% 100% embolectomy on • BP: 130/80
non-rebreather. right side; then • O2 Sat: 98% on
• PA: 50/20 placement of 40% face mask.
• Angiogram: Like CT. retrievable IVC filter. • PA: 40/

ECG & Pulmonary Embolism Prognosis

Vanni, S. et al.
A JMed 2009;122: 257-64.

1193
Copyright © Harvard Medical School, 2018. All Rights Reserved.

Pictorial View of PEITHO Results

Meyer G et al
N Engl J Med 2014;370:1402-1411

Seattle II Study

150
Patients

Age 59

RV/LV 1.55

PA 51
mmHg

1194
Copyright © Harvard Medical School, 2018. All Rights Reserved.

Case Study: JD47 Age 40


episode of chest pain and
shortness of breath. No further
treatment.

Age 43
47 y.o. man for progressive shortness of breath, had
CT-PE, and told of large clot in lungs →
with several Anticoagulated
weeks of
increasing
dyspnea Reported HTN,
Medications
Diltiazem
dyslipidemia, family
Furosemide history of “clots”
Atorvastatin
Warfarin

Case Study – JD47


Vitals: 120/82 mmHg HR 72 RR 14 94%- RA

Carotids: JVP 8-9 cm, prominent “a” wave

Lungs: Clear

Heart Sounds:
Left parasternal lift. PMI displaced laterally.
Normal sounding S1, physiologically split S2,
with P2 heard at apex, as well as in pulmonic
and left lower sternal border regions

1195
Copyright © Harvard Medical School, 2018. All Rights Reserved.

Case Study: JUD47 - EKG

06-17-11

37

Case Study – JD47


LABS

EKG

ED Reports

1196
Copyright © Harvard Medical School, 2018. All Rights Reserved.

Case Study: JD47 - CT


06-17-11

39

Case Study: JD47

Lytic agents for acute large PE, with EKG,


RV changes and risk of early
decompensation.

Heparinization and change to LMWH for


small PE atop chronic thromboembolic
pulmonary HT.

Echocardiogram, and if significant


pulmonary HT, TR confirmed, elective
surgical RX for CTEPH.
40

1197
Copyright © Harvard Medical School, 2018. All Rights Reserved.

Case Study: JD47 - Echo

06-17-11

41

Case Study: JD47 - Echo


06-17-11

42

1198
Copyright © Harvard Medical School, 2018. All Rights Reserved.

Race and Epidemiology of Venous


Thromboembolism
• Incidence of VTE is 30-60% higher in blacks than
whites.
• Incidence of pulmonary embolism is higher in
blacks, and percentage of black VTE patients who
have PE is higher.
• Incidence of fatal PE is higher in blacks, 3.73 vs. 1.15
deaths/100,000 people per year.
• Idiopathic PE is seen in 18% of blacks, vs. 10% of
whites. Sickle cell trait, factor VIII, VWF ↓ protein C
may play roles.
Tyler W. Buckner, MD
Nigel S. Key, MD
CIRC 2012;125:837-839

Case Study: LX
Impaired glucose
tolerance

ESRD secondary to
Dyslipidemia
prior lithium toxicity

58 yo woman Catheterization 15
brought from months prior for chest
Bipolar disorder and in from dialysis pain and reported
problematic marriage center with left NSTEMI: PA 45/10
with alcohol arm and chest mmHg, No gradient
dependent husband discomfort and across outflow tract or
“not feeling aorta. EF 65%. 30% LAD
and LCx stenoses.
well.”

1199
Copyright © Harvard Medical School, 2018. All Rights Reserved.

EKG - End of dialysis

1200
Copyright © Harvard Medical School, 2018. All Rights Reserved.

The Echo
Which Of Following
Technician is Statements Are True:
on the way!

Echo: Cath: V-gram:


Extensive anteroapical,
No coronary artery
inferoapical, apicolateral Like echo
akinetic region irregularities

Vigorous contraction of LV
outflow tract region

Mitral regurgitation

1201
Copyright © Harvard Medical School, 2018. All Rights Reserved.

Takotsubo.
Extensive mid anterior apical and infero apical akinesis.

EKG – 48 hours after admission

1202
Copyright © Harvard Medical School, 2018. All Rights Reserved.

Diagnosis:
Takotsubos or Stress Induced Cardiomyopathy

Seen in association with:

• Emotional or physiologic stress, including sepsis, cerebral events,


pheochromocytoma.
• Characterized by sympathetic stimulation
• Can have full range of ST-T wave abnormalities.
• Can be any region of myocardium.
• Can be associated with transient outflow tract obstruction and MR.
• Rare cases of VF or rupture.
• Usually, LV gets entirely better, with variable time course.
• Can recur in 10 – 15% of patients.

Takotsubos Frequency

2001 Enrollment 2013


Period
4325 Consecutive
ACS Patients

9/90 90 (2.1%) Confirmed


Arrhythmia Takotsubo’s
Ventricular
Potential?
Arrhythmia

Full recovery of
NSVT:VT Always within WMA
3:1 1st 48 Hours

No coronary lesion
≥50%
Auzel, O et al
AmJCardiol 2016;117:1242-1247

1203
Copyright © Harvard Medical School, 2018. All Rights Reserved.

Takotsubo.
Does not have to be apical.

Templin C et al
NEJM 2015;373:929-38

Takotsubo’s is NOT benign

Though most get better, and recurrence rate is ~ 15%,


Templin C et al there can be stroke, arrhythmia, death…over 30 days.
NEJM 2015;373:929-38

1204
Copyright © Harvard Medical School, 2018. All Rights Reserved.

Case Study: RP53 - Background

Case Study: RP53 - History


Works everyday as a hands-on
Quit Smoking in 3rd Decade
manager in a family business.

Received TPA, then


1988 Evolving Anterior MI under went LIMA-OMB,
SVG-AM, sequential
SVG-D1-LAD.
Patent LIMA-OMB, SVG-AM
occluded, Sequential SVG- 1999 Accelerating Angina
D1-LAD with 99% stenosis.
Received SVG-A.M, SVG-LAD

1205
Copyright © Harvard Medical School, 2018. All Rights Reserved.

Case Study: RP53 - Exam

Case Study: RP53

1206
Copyright © Harvard Medical School, 2018. All Rights Reserved.

Most likely reason (s) for this patient’s easily


precipitated angina (worse post prandial)
Case Study: RP53

Hiatal hernia + Para esophageal hernia without erosive


esophagitis.
Aortic stenosis combined with or without native or graft
coronary disease.
Mitral regurgitation and hypertensive/diabetic
microvascular impairment.

Worsening anemia and outflow tract obstruction.

Left subclavian stenosis jeopardizing flow to LIMA.

Case Study: RP53 - Echo

EDD 5.6
ESD 4.1
Wall thickness 1.4cm
EF 50%
No wall motion abnormalities.
Heavily calcified aortic valve.
Mean gradient 44
VMAX 4.4 m/sec
TVI below the valve 23, above
the valve 119
AVA 0.7cm²

1207
Copyright © Harvard Medical School, 2018. All Rights Reserved.

Case Study: RP53 - Cath


• Patent grafts.
• RCA occluded after acute marginal.
– Good L→R collaterals.
– Acute marginal mid 95% stenosis.
• Occluded left main.
• Far distal LAD 80% stenosis.
• Fully patent, large ileofemoral systems.

Case Study: RP53 – Surgery?


Patient States He Is Willing To Abide By Recommendation,
Convene Surgeons And Interventionalists:

CABG and SAVR


PCI and TAVR
PCI and SAVR
TAVR alone
Medical management
All of the above
All except E, F.

1208
Copyright © Harvard Medical School, 2018. All Rights Reserved.

TAVR vs SAVR,
Well Established In High Risk Patients
Partners-Sapien balloon 2014 Core Valve-self expanding
expanded valve valve

• Equal mortality early and at • TAVR with overall lower


2 yrs in high risk pts. mortality and stroke rate
• Higher stroke and vascular • Perivalvular leak a concern.
complication rate with • LBBB and CHB a concern.
TAVR.
• Smaller catheter systems.
• Perivalvular leak a concern.
• But developed smaller
catheter systems, and in
2015: medium risk pts
equal.

Intermediate Risk also


reasonable for TAVR
Patients 2032
Prior CABG,
24%
Age 81 Frail, 44%

Cerebrovas
STS 5.8
Disease,
32%

NYHD III/IV 77% Moderate
MR, 17%

Mean Gradient 45±13


Atrial
COPD, 32%
Fibrillation,
Ejection Fraction 56% 31%

Leon, MB et al
PARTNER 2 NEJM 2016;374:1609-20

1209
Copyright © Harvard Medical School, 2018. All Rights Reserved.

Freedom from Degeneration

DVIR, D
Euro PCR 2016

Case Study: S99


39 year old woman who feels well.

Occasional orthostatic lightheadedness.

Maternal Grandfather died at age 55.

No history of heart murmur.

Medications: estrogen/progesterone, zolpidem.

1210
Copyright © Harvard Medical School, 2018. All Rights Reserved.

Case Study: S99

Vitals Exam Cardiac


Height 68” Neck Veins – not elevated Loud S1

Carotids – Upstrokes brisk;


Volumes normal, not Physiologically split S2
Weight 120 lbs increased
III/VI systolic murmur
Lungs – Clear loudest just left of lower left
sternal border
BP 100/70 HR 62
Chest – Minimal pectus With Valsalva, murmur
excavatum becomes minimally louder
With handgrip, extra sound
BP 90/60 HR 70 PMI – 5th intercostal space;
heard after S1, murmur
Mid clavicular line; No thrill
moves later

Case Study: S99 - EKG

1211
Copyright © Harvard Medical School, 2018. All Rights Reserved.

Case Study: S99 - Diagnosis


Autonomic insufficiency, unclear
etiology.

Mitral stenosis.

Obstructive hypertrophic
cardiomyopathy.

Mitral valve prolapse and mild mitral


regurgitation. Can follow with yearly
exams.

Posterior mitral valve prolapse and


significant mitral regurgitation.

a and d.

Movement of MV Clicks

Devereux, R et al
CIRC 1976;54:3-14

1212
Copyright © Harvard Medical School, 2018. All Rights Reserved.

34 year old woman Medical History Exam


Abdominal Pain Paraplegia in aftermath BP 124/84 mmHg
of MVA
Nausea Abdominal distention
Colostomy
& Emesis for 9 hours
Suspect bowel obstruction

CIRC 2016;133:1132-34
Hibbs, J et al
Troponin T Negative

Echo
Ejection Fraction 65%
No Wall Motion Abnormalities

Chest CT
No Pericardial Effusion
No Mass

EKG

Hibbs, J et al
CIRC 2016;133:1132-34

1213
Copyright © Harvard Medical School, 2018. All Rights Reserved.

Discussion

Represents Giant J Waves (Osborne Waves)

Sometimes seen with hypothermia

ST elevations

Due to gastric distention

Key is prominent J Waves that worsen with normal


echo (not ischemia/myocarditis/pericarditis)

Hibbs, J et al
CIRC 2016;133:1132-34

CASE STUDY S17


51 Year Old
Woman

Abrupt onset pleuritic chest


discomfort – worse supine
Hodgkins Disease Stage IIA – 22 yrs previously – treated
with Mantle RT and MOPP following splenectomy

Treated hypothyroidism

Soleal DVT – 3 years previously

Contrast allergy

1214
Copyright © Harvard Medical School, 2018. All Rights Reserved.

CASE STUDY S17


Vitals Neck Veins
BP 110/80-10mmHg paradox Mid neck level seated upright
HR 110 beats per minute Collapse with inspiration
RR 20

Lungs Carotids
Clear Upstrokes brisk
No palpable PMI Volumes diminished

Heart Sounds No Peripheral Edema


Normal S1, single S2
II/VI early peaking
systolic murmur
along LLSB

Case Study S17

1215
Copyright © Harvard Medical School, 2018. All Rights Reserved.

Which of the Following are True?


Pleuritic discomfort against her background of
DVT suggests pulmonary embolism as
possible diagnosis
Radiation & symptoms and EKG suggest
pericardial processes that could include
effusive disease or effusive constrictive disease
Prior radiation & symptoms suggest chronic
constrictive pericarditis

Appropriate testing would include ventilation


perfusion study and surface echo-doppler

All except c

All except b

Case Study S17: ECHO


Large circumferential pericardial
effusion

No diastolic collapse of RV or RA

Transmitral early filling falls 35%


with inspiration

Transtricuspid early filling increases


75% with inspiration

1216
Copyright © Harvard Medical School, 2018. All Rights Reserved.

Case Study S17: Hemodynamics

All diastolic
pressures
between 18-
20 mmHg

Blunted “y”
descent in RA

Blunted early
filling in RV

Case Study S17: Hemodynamics

Pericardial Pressure =

A B C

18-20 5-10
Zero
mmHg mmHg

1217
Copyright © Harvard Medical School, 2018. All Rights Reserved.

EKG Changes in Acute Pericarditis


• Diffuse ST segment elevation with upright T-waves (lead aVR may have ↓ ST)
• PR segment depression, except in lead aVR, where there is no elevation
• Jpoint/T wave peak ration often > 25%
• NO “reciprocal changes” except in R

• Resolution
• Normalization of PR segments and ST segments, followed by T wave inversions
• Ultimately resolution of T wave inversions
• Time course highly variable, effusions/myocarditis alter QRS amplitude
• Tamponade (especially from a complex effusion)
• Accompanied by QRS electrical alternans

Case 234

1218
Copyright © Harvard Medical School, 2018. All Rights Reserved.

Imazio, et al
CIRC 2011;124:1270-75

Results of the
COlchicine for acute
PEricarditis (COPE) Trial

Sample Size 120 patients

85%
Idiopathic
Diagnosis Acute Pericarditis 15%
Autoimmune/post
Pericardiotomy

Group I Group 2
Randomization Conventional
Conventional Treatment &
Treatment & ASA Colchicine

1219
Copyright © Harvard Medical School, 2018. All Rights Reserved.

Colchicine for Pericarditis

Imazio, et al
CIRC 2005;112:2012-16

• 31 year old man


• Dyspnea
• Sense of vibration
• Patient and mother have extreme wariness

HXS:NXJ regarding testing and results as well as


medications
• Father had atrial fibrillation and mitral
regurgitation
• Cousin received a pacer for HCM
• No history of sudden death

1220
Copyright © Harvard Medical School, 2018. All Rights Reserved.

HXS:NJX
BP 130/80

HR 83

RR 12

No fall in BP with standing

JVP<7cm H2O: No HJR

Brisk carotid upstrokes and volumes – post Valsalva in 2 components

Hypertrophic precordium, normal S1, S2 physiologically split, P2 not increased

Non displaced PMI – apical thrill

IV/VI systolic murmur across the outflow tract, not quite to neck

Definite increment with Valsalva (Stage III)

Chirping posteriorly directed holo-systolic murmur that increments with Valsalva

HXS:NJX: June
2017

1221
Copyright © Harvard Medical School, 2018. All Rights Reserved.

HXS-NJX
2011 2017
• Septum 1.9 cm, nl 1.0 cm. • Septum 2.9-3.1 cm.
• Vmax outflow and Ao valve • Vmax outflow and Ao valve
1.6 m/sec, 11 mmHg, nl 1.0 3.3 m/sec, 45 mmHg.
and 0. • E’s 0.07.
• E’s 0.11m/sec, nl >0.8. • SAM, and important MR.
• No SAM. • Valsalva Vmax 6.3 m/sec,
• Valsalva not done. 120 mmHg.

1222
Copyright © Harvard Medical School, 2018. All Rights Reserved.

1223
Copyright © Harvard Medical School, 2018. All Rights Reserved.

Case Study 085


Cardiac Testing Patient Presentation
Holter
8-10 beat runs of • 55 year old woman
ventricular tachycardia,
rate 170,
• Symptoms
clear-cut AV – Exertional dyspnea
dissociation – Heart pounding
RBBB
Superior axis • Family History
configuration – SCD in 18 year old grand
daughter of father
Echo • No history of HTN
EF 65%
Septal thickness 2.3 cm
Posterior wall 1.3 cm

Case Study 085

1224
Copyright © Harvard Medical School, 2018. All Rights Reserved.

Case Study 085

Case Study 085

Which
Ventricular ectopy with this
EKG, good EF: can be managed of the This patient has an infiltrative
process and should undergo
with a Beta-blocker and EMB, looking for sarcoid or
reassurance.
following amyloid.

statements
are true?

This patient has an excellent The combination of verapamil


story for hypertrophic and Beta-blockers may be used
cardiomyopathy and an MRI to quiet the ectopy and she can
might be helpful. be followed conservatively.

1225
Copyright © Harvard Medical School, 2018. All Rights Reserved.

Stratification of Hypertrophic
Cardiomyopathy Patients

Maron, B
JAMA Cardiology
Published online 02MAR2016

56 year old woman


with sudden onset Case Study 33
exertional dyspnea
and pleuritic
discomfort

2 months of
intermittent chills

Fit runner and busy


teacher

No history of
hypertension,
diabetes, elevated
lipids

1226
Copyright © Harvard Medical School, 2018. All Rights Reserved.

Case Study 33
3 Months Prior Subsequently
• Fell while running • “Not feeling well”
– “ brief double vision” – episodic chills
– had a neck injury • Day of Admission
– Seen in local ED
– noted exertional dyspnea
– Multiple attempts to
– pleuritic discomfort when
place IV
supine.
– Then underwent head
CTA • Medical Regimen
• unremarkable – Levothyroxine 50
micrograms daily.

heard throughout precordium,


3/6 slightly
Case
and loudest at apex and JVP < 6 cm
posteriorly. Moves minimally
later with hand grip
late systolic
murmur
H20 Study 33

Loud S1,
Carotid
S2Ǿ and P2
volumes
not heard
normal
at LLSB

102° BP 130/60 HR 110 RR 18 96% room air

Systolic murmur
PMI not heard left of
displaced spine, near
scapula

Hyperdyna
mic No pectus
precordium

1227
Copyright © Harvard Medical School, 2018. All Rights Reserved.

Case
Study 33

Case Study 33
Pleuropericarditis, with fever, chills, and rub.

Pleuropericarditis in patient with myxomatous mitral


regurgitation (MVP), with an expected loud S1 and murmur that
moves later in cardiac cycle with handgrip.

Subacute bacterial endocarditis atop myxomatous mitral valve


disease and suspect damage to the anterior mitral valve leaflet.
Perhaps an earlier embolic event, musculoskeletal and immune
symptoms.

Acute myocardial infarction 2 months previously, with


subsequent Dresslers syndrome and posteromedial papillary
muscle head rupture.

1228
Copyright © Harvard Medical School, 2018. All Rights Reserved.

Echo: Case
Nearly flail AMVL with redundancy
8-10mm vegetation
Study 33
Severe MR
Blood cultures
EF 65%
Strep mitis 4/4
Mild left atrial dilation

CASE 33: Next Steps

1. Antibiotics for 4-6 weeks.


Surgery for heart failure, embolic, recurrent
fevers.
2. Antibiotics, early mitral valve replacement
for destroyed valve and vegetation.
3. Mitral e-clip early, with antibiotics.
4. Early surgery if valve seems like it can be
repaired. Otherwise, follow conventional
indications for surgery.

1229
Copyright © Harvard Medical School, 2018. All Rights Reserved.

Early surgery vs. conventional treatment for infective


endocarditis. (EASE)
Conventional approach:
Operate for persistent fever, embolic events while on Rx, hemodynamic
compromise. Special considerations for STAPH and fungi.
“EASE”:
18-80 y.o. (mean 48), 76 patients definite endocarditis (largely viridans,
few staph, enterococci.
Severe AI or MR
Vegetation > 10mm

Surgery within 48 hours (37 pts) Surgery for conventional indications (39 pts)

Excluded major strokes, with risk of hemorrhagic transformation, prosthetic valves, right
sided vegetations
Kang, D et al
NEJM2012;366:2466-73

Large difference in embolic events 8 late vs 0 early 105

Guidelines – “Expert Opinion”


Antibiotic prophylaxis for heart conditions was considered excessive.

Without data, 2007 suggestion:


prophylaxis only for prosthetic heart valves, shunts, or people with prior SBE.

1 case of SBE for every 277


non-Rx’s pts in the UK. 35 new
cases/mo. above expected.
Drayer, MJ., et al. Lancet 2015;385: 1219-28

1230
Copyright © Harvard Medical School, 2018. All Rights Reserved.

Case Study JD-61

Symptoms Exam
• Squeezing, left-sided chest • BP 90/60 mmHg
pain • HR 85
• Lightheadedness • RR 12
• JVP 8 cm H20
• Diaphoresis
• Carotid upstrokes normal,
volumes diminished
• Clear lungs
• Left parasternal lift
• Muffled S1
• Physiologically split S2
Namana V, et al
NEJM 2016;374:872

Case Study JD-61

Namana V, et al
NEJM 2016;374:872

1231
Copyright © Harvard Medical School, 2018. All Rights Reserved.

Case Study JD-61


Which of the following statements is true?

This patient is having an


The EKG is consistent V2, V3, ST segment
MI related to a
with acute pericarditis depression has to
dominant circumflex
and the story raises represent anterior
and leads V7,8,9 would
question of tamponade. ischemia, usually LAD.
be helpful.

This patient is having an RV involvement cannot


IMI with RV be ascertained without
involvement. obtaining a lead V4R.

Namana V, et al
NEJM 2016;374:872

Case Study JD-61

Namana V, et al
NEJM 2016;374:872

1232
Copyright © Harvard Medical School, 2018. All Rights Reserved.

CASE STUDY 82
64 year old man
Awoke with right arm numbness and back pain.
The back pain resolved, and shortly, the arm
numbness resolved.
However, he noticed that his heart rate was 40

5 years prior
LIMA-LAD, SVG- Diagonal
SVG-OMB

Increased weight
(BMI 39)

obstructive sleep apnea


Hyperlipidemia

CASE Study 82

Home Exam Labs


medications • 170/60 both arms
• HR 78, RR 18, Pulse Ox • Cr 1.4
• ASA 93% on 3 liters 0² • HCT 40
• Metropolol • JVP not elevated • WBC 9.5
• Symmetric carotid brisk • TR-T 0.02
• Clonidine upstrokes and normal
• Furosemide volumes • Lactate 1.6
• Spironolactone • Clear lungs
• Lovastatin • S1 S² physiologically split
• Non-tender, protuberant
abdomen
• Normal distal pulses
• No edema

1233
Copyright © Harvard Medical School, 2018. All Rights Reserved.

CASE STUDY 82

CASE STUDY 82
Differential diagnosis includes which of the following:

Acute ST segment
Massive acute
elevation
pulmonary Aortic dissection
myocardial
embolism
infarction

Pericardial
Esophageal spasm
tamponade

1234
Copyright © Harvard Medical School, 2018. All Rights Reserved.

CASE STUDY 82

Clinical presentations and signs of type A dissection


No previous cardiac surgery Previous cardiac surgery
Chest pain or
back pain 88% 68% p <0.05
Abrupt onset
pain 92% 84% p <0.05
Any pulse
deficit 28% 30% NS
Pericardial
effusion 48% 26% p <0.05
Tamponade 16% 3% NS
*Widest diameter
of ascending Ao 5.28 (1.34) 5.40 (1.57) NS

Collins, JS et al IRAD
CIRC 2004;110:II-237-242

Klodell, CT et al
Ann Thorac Surgery 2012;93:1206-14

1235
Copyright © Harvard Medical School, 2018. All Rights Reserved.

Case 234
Visits because his 35
year old brother died
suddenly 3 weeks
ago.
Participated in
36 year old man • Post mortem showed
multiple triathlons
rupture of non-coronary
sinus of valsalva aneurysm
into the pericardium, and
a forme fruste of a
bicuspid valve.

Case 234

1236
Copyright © Harvard Medical School, 2018. All Rights Reserved.

Case 234

Bicuspid Aortic Valve and Aortic Dilation (Bicuspid Aortopathy)


BAV Incidence 1.3%.
1/3 will have attention to valve.
? 1/2 to Aorta

Three common types:


1. Anterior-posterior or “horizontal” (R-L fusion);
dilation of right side of aorta. Aortic issues (> 4.5-
5.0cm) usually with age >50, and almost always
with some amount of aortic stenosis.
2. Vertical orientation (R-non fusion), with
ascending Aorta and Arch dilation.
3. Aortic root or sinus dilation. Age <40, with or
without valve; more often genetic.

1237
Copyright © Harvard Medical School, 2018. All Rights Reserved.

Case Study 322


Patient History Exam
• 21 year old man • BP 127/70 mmHg
• 3 days of intermittent flu-like • HR 88
symptoms
• “Lung burning” – worse supine, • RR 12
non-pleuritic • Pain free
• MVP • JVP not elevated
• Father 7th decade likely vascular
disease • Carotids normal
• Clear lungs
• Pectus excavatum
• Normal S1, click not heard
• Physiologically split S2
• TrT 1.41 -1.79 mg/dl
• CK 1570-900

1238
Copyright © Harvard Medical School, 2018. All Rights Reserved.

Case Study 322, 3 years prior


2 0 1 1
E K G

Case Study 322


2 0 1 6
E K G

1239
Copyright © Harvard Medical School, 2018. All Rights Reserved.

Case Study 322

Story is consistent with acute MI, coronary dissection,


coronary embolism, vasospasm

Suggestive of pericarditis

Suggestive of myocarditis, perhaps subtle pericarditis

Next step would include coronary angiography, emergently

Case Study 322


2 0 1 6
E c h o

1240
Copyright © Harvard Medical School, 2018. All Rights Reserved.

Case Study 322


2 0 1 6
M R I

XXX424
Heart Transplant Free Survival

Anzini M.
Circulation 2013;128:2384-94

1241
Copyright © Harvard Medical School, 2018. All Rights Reserved.

81 Year Old Man XXX675


with 3 Days of
Chest Discomfort.
Pain radiates to left jaw

GERD & esophageal strictures,


and has had multiple
dilatations.

DM
HT
Longstanding LBBB

BP 147/80
HR 114
Crackles and LVS3,
Paradoxically split S2

XXX675

1242
Copyright © Harvard Medical School, 2018. All Rights Reserved.

XXX675

EKG & History Suggest?


Ischemia based on history; cannot interpret “more”
on EKG secondary to LBBB.

Ischemia based on history and marked, global ST


segment changes that are significant even with a
LBBB.

Ischemia based on history and leads V3 and V4


concordant changes.

This patient needs an echo to decide next steps, for


he may simply have GERD symptoms and EKG
changes consistent with LBBB and tachycardia.

XXX675

Cath: Severe ramus stenosis


Stented

1243
Copyright © Harvard Medical School, 2018. All Rights Reserved.

PM631 Case Study

Premature Told of
Vascular Chest Pain –
Disease
LBBB following
mechanical
Right fall related
Cerebral Dyslipidemia left-hip
Event – Hypertension fracture and
soon after open reduction
initiating and internal
estrogen fixation

PM631
EKG Comparison

1244
Copyright © Harvard Medical School, 2018. All Rights Reserved.

PM631
Outcome
Underwent Emergent Catheterization

Occluded left
circumflex: stented

Occluded LAD:
collateralized

1245
Copyright © Harvard Medical School, 2018. All Rights Reserved.

Atrial Fibrillation and Common Supraventricular


Tachycardias

Sunil Kapur MD

Cardiac Electrophysiology
Brigham and Women’s Hospital
Instructor, Harvard Medical School

No disclosures

1246
Copyright © Harvard Medical School, 2018. All Rights Reserved.

Cardiac Conduction:
SVT = down to (and including) the AVN

Outline
• Atrial fibrillation
– Basics
– Atrial Flutter
• Regular Paroxysmal SVTs
– Atrial tachycardia
– AV Node Reentrant Tachycardia
– AV Reentrant Tachycaria (Accessory pathway
mediated)
• Questions

1247
Copyright © Harvard Medical School, 2018. All Rights Reserved.

Basics
• Atrial Fibrillation (AF) is the most common clinical arrhythmia

Normal Sinus Rhythm AF

• In AF, the normal regular electrical impulses of the atria are overwhelmed
by disorganized electrical impulses

JHeuser 06:08, 28 November 2005

1248
Copyright © Harvard Medical School, 2018. All Rights Reserved.

• AF is dramatically increasing in prevalence

Rahman. Nat. Rev. Cardiol. 2014

“AF–omics”

Is all atrial fibrillation the same?


Are there subtypes with different
genetic causes?

J Am Coll Cardiol. 2008 Jul 8; 52(2): 117–123.

1249
Copyright © Harvard Medical School, 2018. All Rights Reserved.

Clinical Risk Factors for the Development of Atrial Fibrillation


Non-Modifiable Modifiable Behavioral
Age Valvular heart disease Alcohol use
Gender Cardiac and Non-Cardiac Surgery Caffeine Use
Race Hypertension Medications
Familial/Genetic Dyslipidemia and Coronary Artery Physical Activity
Disease
Birth Weight Obesity, Diabetes Mellitus, and CLINICAL BOTTOM LINE
Metabolic Syndrome Initial assessment should
Pericardial Fat Heart Failure include laboratory tests
Obstructive Sleep Apnea (electrolytes, thyroid-
Chronic Kidney Disease stimulating hormone, and
Thyroid disease renal and hepatic function)
Pulmonary / Pulmonary Vascular to rule out underlying
Disease disorders and
Pericarditis/Inflammation echocardiogram to look for
Congenital Heart Disease structural heart disease.
Psychiatric Conditions

J Am Coll Cardiol 2014;63:2335–45

1250
Copyright © Harvard Medical School, 2018. All Rights Reserved.

• The macro-electro-pathophysiology of AF remains unclear

Nishida. JACC. 2014, 823 - 831

Definitions of Atrial Fibrillation: A Simplified Scheme1


Paroxysmal • AF that terminates (spontaneously or with intervention) within 7 days
of onset.
• Episodes may recur with variable frequency.
Persistent • Continuous AF that is sustained >7 days.
Long-standing persistent • Continuous AF >12 months in duration.
Permanent • When the patient and clinician make a joint decision to stop further
attempts to restore and/or maintain sinus rhythm.
• Acceptance of AF represents a therapeutic attitude on the part of the
patient and clinician rather than an inherent pathophysiological
attribute of AF.
• Acceptance of AF may change as symptoms, efficacy of therapeutic
interventions, and patient and clinician preferences evolve.
CLINICAL BOTTOM LINE
If the diagnosis is suspected and ECG is normal, longer-term monitoring
with a Holter monitor or loop recorder can be helpful.

1251
Copyright © Harvard Medical School, 2018. All Rights Reserved.

• The macro-electro-pathophysiology of AF remains unclear

Nishida. JACC. 2014, 823 - 831

• AF is associated with significant morbidity and mortality

Go AS. N Engl J Med 2009;360:2127-2129.


Eur Heart J. 2013 Apr;34(14):1061-7.

1252
Copyright © Harvard Medical School, 2018. All Rights Reserved.

Clinical Management
Priority– Risk Factor management

Clinical Risk Factors for the Development of Atrial Fibrillation


Non-Modifiable Modifiable Behavioral
Age Valvular heart disease Alcohol use
Gender Cardiac and Non-Cardiac Surgery Caffeine Use
Race Hypertension Medications
Familial/Genetic Dyslipidemia and Coronary Artery Physical Activity
Disease
Birth Weight Obesity, Diabetes Mellitus, and
Metabolic Syndrome
Pericardial Fat Heart Failure
Obstructive Sleep Apnea
Chronic Kidney Disease
Thyroid disease
Pulmonary / Pulmonary Vascular
Disease
Pericarditis/Inflammation
Congenital Heart Disease
Psychiatric Conditions

Clinical Management
Priority #2 – Cardioembolic stroke prevention

1253
Copyright © Harvard Medical School, 2018. All Rights Reserved.

• The evidence does not support the use of aspirin as


monotherapy for the prevention of thromboembolic events in
patients with AF.

• The issue of whether aspirin could be a reasonable antithrombotic


monotherapy in very low-risk patients (CHADS2 = 0) has not been well
addressed, as the individual trials enrolled very few such patients.
• A 2007 meta-analysis found that aspirin, compared to placebo or no
therapy, reduced the risk of stroke by about 20 percent, although this
effect was not statistically significant (relative risk reduction 19 percent;
95% CI -1.0 to 35.0).
• Further, aspirin had little effect on reducing the risk of disabling stroke.

1254
Copyright © Harvard Medical School, 2018. All Rights Reserved.

Priority #2 – Cardioembolic stroke prevention

Priority #3 – Electrical Management

• Rate Control-A rate-control strategy uses drugs that block (slow conduction through) the
atrioventricular (AV) node such as beta blockers, rate-slowing calcium channel blockers, or
digoxin. AV nodal ablation plus ventricular pacing to control symptoms is also considered
when pharmacologic therapy is ineffective.

• Rhythm Control-A rhythm-control strategy uses antiarrhythmic drug therapy,


radiofrequency catheter ablation, and/or a surgical procedure performed at the time of
open heart surgery to maintain sinus rhythm (SR).

1255
Copyright © Harvard Medical School, 2018. All Rights Reserved.

Atrial Fibrillation - no benefit of antiarrhythmic drug


therapy to maintain sinus rhythm?

Rate
Control Antiarrhythmic
drugs
AFFIRM poor efficacy
poor tolerance
PIAF toxicity
RACE
STAF
Rhythm- and rate-control strategies are associated with similar rates
of mortality and serious morbidity, such as embolic risk, which is best
addressed using anticoagulation based on the CHADS2 or
CHA2DS2-VASc criteria.

Priority #3 – Electrical Management

• So who gets rhythm control:

– Failure of rate control

– Symptomatic Afib

– Increasing indications ?

1256
Copyright © Harvard Medical School, 2018. All Rights Reserved.

Mixed effects of antiarrhythmic drugs

IA: quinidine procainamide disopyramide

III
sotalol
IC 1 ibutilide**Ikr + plateau Na
Ito ICa
Flecainide 2 dofetilide
Propafenone amiodarone*
Moricizine azimilide
0 Ikr
INa
IB 3 Iks
Lidocaine
Mexiletine
Tocainide
phase 4
IK1
IKACh

QRS T

Clinical Management: Priority #3 – Electrical Management

Wavelength (ƛ) =
Refractory Period (RP) x
Conduction Velocity (CV)

FibNAF = L / ƛ

Iwasaki et al. Circulation. 2011; 124: 2264-2274

1257
Copyright © Harvard Medical School, 2018. All Rights Reserved.

Amiodarone Vs Sotalol vs Placebo for Persistent


Atrial Fibrillation
Singh, B. N. et al. N Engl J Med 2005;352:1861

Persistent AF on warfarin
Patients 665
Age 67 yrs
Duration of AF < 1 yr 77%
AF symptoms 62%
Ischemic heart disease 26%

Time to Recurrence of AF among Patients in Whom


Sinus Rhythm Was Restored on Day 28

Rate Versus Rhythm Control

Rate and possibly Rhythm control

Priority #3 – Electrical Management

• So who gets rhythm control:

– Failure of rate control

– Symptomatic Afib

– heart failure ?

– Increasing indications ?

N Engl J Med 2018; 378:417-427

1258
Copyright © Harvard Medical School, 2018. All Rights Reserved.

Clinical Management: Priority #2 – Electrical Management

Pulmonary Vein Isolation:


60 – 70% efficacy for paroxysmal AF
Poor efficacy for persistent / permanent AF
Oral et al Circulation 2002;105:1077

Freedom from any AF >90% reduction in AF


with no drug therapy with or without drugs

mean follow-up: 150 days

1259
Copyright © Harvard Medical School, 2018. All Rights Reserved.

Outline
• Atrial fibrillation
– Basics
– Atrial Flutter
• Regular Paroxysmal SVTs
– Atrial tachycardia
– AV Node Reentrant Tachycardia
– AV Reentrant Tachycaria (Accessory pathway
mediated)
• Questions

Atrial Flutter and Atrial Fibrillation: Frequently Associated, but Not the
Same Arrhythmia

• TYPICAL Atrial flutter vs ATYPICAL ATRIAL FLUTTER

1260
Copyright © Harvard Medical School, 2018. All Rights Reserved.

Typical CTI dependent Isthmus Dependent


Atrial Flutter (counterclockwise)

P-waves:
II, III, F = negative
V1 = positive

Outline
• Atrial fibrillation
– Basics
– Atrial Flutter
• Regular Paroxysmal SVTs
– Atrial tachycardia
– AV Node Reentrant Tachycardia
– AV Reentrant Tachycaria (Accessory pathway
mediated)
• Questions

1261
Copyright © Harvard Medical School, 2018. All Rights Reserved.

Regular PSVT

Atrial tachycardia
Distinction from atrial flutter:
-Focal (“Focal” is Not a Mechanism)

–AT Mechanisms:
–Triggered
–Automatic
–Reentrant (Micro)

-Sinus tachycardia

-Multifocal atrial tachycardia

-Spectrum to atrial fibrillation

1262
Copyright © Harvard Medical School, 2018. All Rights Reserved.

AVNRT
-Typical and Atypical

-”Dual AVN
physiology”

-Influence AVN
physiology to alter
the arrhythmia

-Hidden retrograde
P waves

-Management

AVRT
Atrioventricular Reentrant Tachycardia =
utilization of an accessory pathway

-Orthodromic versus antidromic

-Manifest versus Concealed

-Wolf Parkinson White Pattern versus


Syndrome

-Management

1263
Copyright © Harvard Medical School, 2018. All Rights Reserved.

Summary
• Atrial fibrillation is the most common
arrhythmia and associated with morbidity and
mortality
– Management include decisions regarding (1) risk
factor management (2) anticoagulation (3) rhythm
management
• Paroxysmal SVTs including AVNRT, AT and
AVRT are often symptomatic yet less
commonly associated with mortality

Question #1
A 70-year-old woman with type 2 diabetes mellitus and rheumatic mitral stenosis is
evaluated for dyspnea and fatigue. She has a history of atrial fibrillation that has
resulted in these symptoms in the past. She has had successful cardioversions, most
recently about 2 years ago. She has hypertension controlled with medication. She also
has mild left-ventricular dysfunction related to coronary artery disease and history of
myocardial infarction. Her current medications include atenolol, lisinopril, aspirin,
atorvastatin, and insulin. Physical examination demonstrates an irregularly irregular
rhythm with a heart rate of 78 beats per minute. Blood pressure is 130/80 mm Hg. The
cardiovascular and pulmonary examinations are otherwise unremarkable.
What medication should this patient receive for at least 3-4 weeks before
cardioversion?
A. Warfarin
B. Clopidogrel
C. Rivoraxaban
D. No additional medication is needed

1264
Copyright © Harvard Medical School, 2018. All Rights Reserved.

Answer #1
A. Warfarin
B. Clopidogrel
C. Rivoraxaban
D. No additional medication is needed

Prior to cardioversion, anticoagulation prevents cardioembolic


risk. With rheumatic heart disease warfarin is the preferable
choice. Dual antiplatelet therapy is inferior to warfarin for
anticoagulation in atrial fibrillation.

Question #2
Which one of the following statements about atrial fibrillation is
correct?
A. Lone atrial fibrillation is a common cause of atrial fibrillation.
B. Atrial fibrillation is more common in younger women than in
older men.
C. Anticoagulation is not indicated in patients who have
nonrheumatic heart disease and atrial fibrillation.
D. Many patients who have atrial fibrillation do not require
antiarrhythmic therapy.
E. Atrial fibrillation is a serious and common problem in patients
with AVNRT.

1265
Copyright © Harvard Medical School, 2018. All Rights Reserved.

Answer #2
A. Lone atrial fibrillation is a common cause of atrial fibrillation.
Most atrial fibrillation occurs in the context of comorbidities

B. Atrial fibrillation is more common in younger women than in older men.


Atrial fibrillation is more common in elderly and more common in men at all age groups

C. Anticoagulation is not indicated in patients who have nonrheumatic heart


disease and atrial fibrillation.
Atrial fibrillation anticoagulation is indicated in nonrheumatic heart disease based on cardioembolic risk. The type
of anticoagulant is influenced by the rheumatic diagnosis

D. Many patients who have atrial fibrillation do not require antiarrhythmic


therapy.
Rate control is a common and acceptable strategy

E. Atrial fibrillation is a serious and common problem in patients with AVNRT


AVNRT is seen in many healthy patients both with and without atrial fibrillation. In WPW, atrial fibrillation can be
serious and more common, however in AVNRT there is no specific enrichment or alternative concern.

Supplemental Reference Slide


• Pritchett EL. Management of atrial fibrillation. N Engl J Med
1992; 326:1264.
• Atrial fibrillation: current understandings and research
imperatives. The National Heart, Lung, and Blood Institute
Working Group on Atrial Fibrillation. J Am Coll Cardiol 1993;
22:1830.
• Lip GY, Metcalfe MJ, Rae AP. Management of paroxysmal atrial
fibrillation. Q J Med 1993; 86:467.
• Ganz LI, Friedman PL. Supraventricular tachycardia. N Engl J
Med 1995; 332:162.

1266
Copyright © Harvard Medical School, 2018. All Rights Reserved.

Bradycardia and Sudden Death Prevention


Usha B. Tedrow, MD, MSc
Director, Clinical Cardiac Electrophysiology Program
Cardiovascular Division
Brigham and Women’s Hospital
Associate Professor of Medicine
Harvard Medical School

Disclosures

• St Jude Medical, Faculty, fellows’ course,


Catheter ablation, arrhythmia
• Boston Scientific, Faculty, fellows’ course,
Catheter ablation, ventricular tachycardia
• Biosense Webster, Funded Research,
Catheter ablation, arrhythmia

1267
Copyright © Harvard Medical School, 2018. All Rights Reserved.

Bradycardia Examples
Sinus Bradycardia Mobitz I Block

Mobitz II Block

•Indicative of conduction
disease below the AV node
•Needs urgent temporary
Complete Heart Block (and then permanent)
pacing

Indications for Pacing

Symptomatic bradycardia

SA node disease AV node disease


(or distal conduction disease)

(spontaneous or drug-induced)

1268
Copyright © Harvard Medical School, 2018. All Rights Reserved.

What do pacemakers do?

What Pacemakers Do
• Electronic pacemakers deliver electrical
impulses that depolarize the myocardial cells
near the lead tip, such that the signal
propagates into the contiguous myocardium
• Pulses can be delivered in many ways,
depending on how the pacemaker is
programmed

1269
Copyright © Harvard Medical School, 2018. All Rights Reserved.

Types of Pacemakers
• Lead in the right atrium

• Lead in the right ventricle

• Leads in both RA and RV

• Leads pacing RV, LV = biventricular pacer

A single chamber ventricular


pacemaker is a clock

VVI at 60 ppm

1 second

1270
Copyright © Harvard Medical School, 2018. All Rights Reserved.

A VVI Pacemaker Is A Clock


(that resets)

0.7 seconds

Clock reset 1 second

A DDD Pacemaker is 2 Clocks


DDD at 60 ppm = rate clock
AV delay 200ms = PR interval clock

200 ms 140ms

0.7 seconds

1271
Copyright © Harvard Medical School, 2018. All Rights Reserved.

AS VS AS VP

AP VS AP VP

1272
Copyright © Harvard Medical School, 2018. All Rights Reserved.

What is mode switching?

Leadless pacing: Micra Transcatheter Pacing System


Positioned in the Right Ventricle.

Reynolds D et al. N Engl J Med 2016;374:533-541.

1273
Copyright © Harvard Medical School, 2018. All Rights Reserved.

His Bundle Pacing: Anatomy

1274
Copyright © Harvard Medical School, 2018. All Rights Reserved.

Main Consequences of LBBB


• Widening of QRS complex (>120
msec)
• Increased time to complete
ventricular contraction
• Septal activation much before
free wall of ventricle, leading to
discoordinated contraction and
wasted energy
• Mitral valve may leak as a result
of sequential activation of
papillary muscles

Deleterious Effects of Ventricular


Dyssynchrony on Cardiac Function
Reduced diastolic
filling time 1
+ Weakened
contractility 2
+ Protracted mitral
regurgitation 2
+ Post systolic regional
contraction 3
= Diminished stroke
1. Grines CL, et al Circulation 1989;79: 845-853
volume 2. Xiao HB, et al Br Heart J 1991;66: 443-447
3. Søgaard P, et al. J Am Coll Cardiol 2002;40:723–730

1275
Copyright © Harvard Medical School, 2018. All Rights Reserved.

Achieving Cardiac
Resynchronization
Goal: Pace Right and Left Ventricles

• Epicardial Approach
• Requires thoracotomy
• Transvenous Approach
• Requires access to the
coronary sinus
• Requires leads developed
for LV application

1. Bakker et al. J of Cardiac Failure 1998; 4[3]:1-35


2. Saxon et al. PACE 1998; 21 Part II: 914
3. Daubert et al. PACE 1997; 20[II]
4. Gras et al. PACE 1998; 21[II]:824

Surgical Left Ventricular Lead

1276
Copyright © Harvard Medical School, 2018. All Rights Reserved.

Pre-op EKG: QRS 216 msec

Post-op EKG: QRS 152 msec

1277
Copyright © Harvard Medical School, 2018. All Rights Reserved.

CRT Indications
• Class I
– LVEF less than or equal to 35%, sinus rhythm
– LBBB with a QRS duration greater than or equal to 150 ms, and NYHA
class II, III, or ambulatory IV symptoms on optimal medical therapy
• Class IIa
– LVEF less than or equal to 35%, sinus rhythm, LBBB with a QRS
duration 120 to 149 ms, and NYHA class II, III, or ambulatory IV
symptoms
– LVEF less than or equal to 35%, sinus rhythm, a non-LBBB pattern
with a QRS duration greater than or equal to 150 ms, and NYHA class
III/ambulatory class IV symptoms

Tracy CM et al. Heart Rhythm. 2012 Oct;9(10):1737-53

CRT Indications
• Class IIa
– Atrial fibrillation and LVEF less than or equal to 35% on
GDMT if a) the patient requires ventricular pacing or
otherwise meets CRT criteria and b) AV nodal ablation or
pharmacologic rate control will allow near 100%
ventricular pacing with CRT
– LVEF less than or equal to 35% and are undergoing new or
replacement device placement with anticipated
requirement for significant (40%) ventricular pacing

Tracy CM et al. Heart Rhythm. 2012 Oct;9(10):1737-53

1278
Copyright © Harvard Medical School, 2018. All Rights Reserved.

CRT Indications
• Class IIb
– LVEF less than or equal to 30%, ischemic etiology of heart failure, sinus
rhythm, LBBB with a QRS duration of greater than or equal to 150 ms,
and NYHA class I symptoms
– LVEF less than or equal to 35%, sinus rhythm, a non-LBBB pattern with
QRS duration 120 to 149 ms, and NYHA class II, III/ambulatory class IV
• Class III: No Benefit
– NYHA class I or II symptoms and non-LBBB pattern with QRS duration
less than 150 ms
– CRT is not indicated for patients whose co-morbidities and/or frailty
limit survival with good functional capacity to <1 year

Tracy CM et al. Heart Rhythm. 2012 Oct;9(10):1737-53

Sudden Cardiac Arrest


• >400,000 out-of-hospital cardiac arrests per year in
U.S.
• ~50% of these arrests are in patients having a
myocardial infarction
• 95% out-of-hospital mortality
• Goals:
– Prevention
– Recognition of high risk populations

1279
Copyright © Harvard Medical School, 2018. All Rights Reserved.

Survival in Cardiac Arrest


5% estimated out-of-hospital survival2,3 even
in the best ambulance/EMS programs
Many events are not witnessed
There is difficulty reaching victims within 6-8
minutes.
Early availability of defibrillation is very
important!

1 Swagemakers V. J Am Cardiol. 1997;30:1500-1505


2 Ginsburg W. Am J Emer Med. 1998;16:315-319.
3 Cobb LA. Circ. 1992;85:I98-102.

AEDs Improve Survival


60%

50%
53%
45% 49%

40%

35%

30%
29%
25%

20% 24%

15%

10%

5%
5%
0%
National Average Boston,MA Seattle,WA Rochester,MN Casino Study
White RD. Ann Emer Med. 96;28:480-485. Cobb LA. Circ. 92;85:I98-102.
Smith SC. Circ. 97;13:1321-1324. Valenzuela TD. N Engl J Med. 2000;343:1206-1209.

1280
Copyright © Harvard Medical School, 2018. All Rights Reserved.

Patients at risk for sudden cardiac death


General population

CAD risk factors

Prior coronary event

EF < 35% and CHF

Prior out-of-hospital
cardiac arrest

Prior MI, low EF, VT

Huikiri et al, NEJM 2001

How can we prevent morbidity and


mortality from Sudden Cardiac Death?
• Primary prevention of coronary disease
• Early identification of structural heart disease
• Identification and avoidance of medications that
increase risk
• Community resuscitation programs (CPR, emergency
response systems)
• Community-based automated defibrillators (AEDs)
• Implantable cardiac defibrillators (ICD)

1281
Copyright © Harvard Medical School, 2018. All Rights Reserved.

Evaluation after Arrest


• Evaluate for acute causes
– Myocardial ischemia/infarction
– Electrolytes
– Drugs
• characterize underlying heart disease
– echocardiogram
• LV or RV dysfunction, areas of infarct or scar
– coronary angiography / exercise testing
– Consider cardiac MRI
– Consider EPS

+ ICD

SVC coil
Shock
vector
_
RV coil

1282
Copyright © Harvard Medical School, 2018. All Rights Reserved.

How does it know what to shock?

Subcutaneous ICD

1283
Copyright © Harvard Medical School, 2018. All Rights Reserved.

CE Mark Approved.
Caution: S-ICD is an investigational device limited
Burke, S-ICD, HRS 2012, Boston, MA to investigational use only under US federal law. Not for sale.

1284
Copyright © Harvard Medical School, 2018. All Rights Reserved.

ICD guidelines summary


• Class I
– LVEF ≤ 35% due to prior MI at least 40 days post-MI and
are in NYHA Functional Class II or III
– LV dysfunction due to prior MI, at least 40 days post-MI,
with LVEF ≤ 30%, and are in NYHA Functional Class I
– survivors of cardiac arrest due to VF or hemodynamically
unstable sustained VT after evaluation to define the cause
of the event excludes any completely reversible causes

Epstein AE et al.J Am Coll Cardiol. May 27, 2008;51(21)

ICD guidelines summary


• Class I
– Nonischemic DCM who have an LVEF ≤ 35% and NYHA
Functional Class II or III
– With nonsustained VT due to prior MI, LVEF < 40%, and
inducible VF or sustained VT at electrophysiological study
– Structural heart disease and spontaneous sustained VT,
whether hemodynamically stable or unstable
– Syncope of undetermined origin with clinically relevant,
hemodynamically significant sustained VT or VF induced at
electrophysiological study

Epstein AE et al.J Am Coll Cardiol. May 27, 2008;51(21)

1285
Copyright © Harvard Medical School, 2018. All Rights Reserved.

ICD guidelines summary


• Class IIa
– Long QT syndrome with syncope or ventriuclar arrhythmia on beta
blocker
– unexplained syncope, significant LV dysfunction, and nonischemic
DCM
– Sustained VT and normal or near-normal ventricular function
– • With catecholaminergic polymorphic VT who have syncope and/or
documented sustained VT while receiving beta blockers
– • For the prevention of SCD in patients with HCM, Brugada syndrome
ARVD/C who have one or more risk factors for SCD
– Cardiac sarcoidosis, giant cell myocarditis, or Chagas disease
– Nonhospitalized patients awaiting transplantation

Epstein AE et al.J Am Coll Cardiol. May 27, 2008;51(21)

Sudden Death before the age of 35 Yrs


Corrado et al NEJM 339:364, 1998

Total - 0.8 per 100,000 pt yrs 269


Atherosclerotic CAD 17 %
Myocarditis / Cardiomyopathy 12 %
Arrhythmogenic RV Dysplasia 11 %
Mitral Valve Prolapse 10 %
Conduction System Disease 9%
Hypertrophic Myopathy 6%
Aortic Dissection 5%
Anomalous Coronary 3%

1286
Copyright © Harvard Medical School, 2018. All Rights Reserved.

Familial syndromes associated with syncope


and sudden death:

• 1. Hypertrophic cardiomyopathy
• 2. Congenital long QT syndrome
• 3. Arrhythmic right ventricular dysplasia
• 4. Brugada syndrome
• 5. Others:
- familial dilated cardiomyopathy
- congenital heart block
- catecholaminergic polymorphic VT (CPVT)

• 1. The ECG suggests which of the following:


• A. Hypertrophic cardiomyopathy
• B. Arrhythmogenic right ventricular dysplasia
• C. Brugada syndrome
• D. Congenital long QT syndrome
• E. Familial Dilated Cardiomyopathy

1287
Copyright © Harvard Medical School, 2018. All Rights Reserved.

Hypertrophic Cardiomyopathy
• Prevalence 1 in 500
• LVH or repolarization abnormalities in 85% of
patients
• Palpitations due to atrial fibrillation
• Sudden death risk 1 – 3% per year

4. The ECG suggests which of the following:


A. Hypertrophic cardiomyopathy
B. Arrhythmogenic right ventricular dysplasia
C. Brugada syndrome
D. Congenital long QT syndrome
E. Familial dilated cardiomyopathy

1288
Copyright © Harvard Medical School, 2018. All Rights Reserved.

4. The ECG suggests which of the following:


A. Hypertrophic cardiomyopathy
B. Arrhythmogenic right ventricular dysplasia
C. Brugada syndrome
D. Congenital long QT syndrome
E. Familial dilated cardiomyopathy

Arrhythmogenic right ventricular


dysplasia (cardiomyopathy)
• Prevalence: 1/5000
• T-wave inversions in precordial leads in 85%
• fibrofatty infiltration of RV / LV
• VT with LBBB morphology – often exercise provoked
• Estimated mortality risk 3% per year
• Autosomal dominant and recessive forms
• at least 8 causative genes:
Cell adhesion proteins: plakophilin (27%), plakoglobin
ryanodine receptor, TGF-beta

Hulot et al. Natural history and risk stratification of ARVD. Circulation. 2004;110:1879.
Sen-Chowdhry et al. ARVC: clinical presentation, diagnosis, and mgmt. Am J Med
2004;117:685. Gerull, B.et al. Mutations in the desmosomal protein plakophilin-2 are
common in ARVC. Nat Genet 36(11): 1162-4.

1289
Copyright © Harvard Medical School, 2018. All Rights Reserved.

1. The ECG suggests which of the following:


A. Hypertrophic cardiomyopathy
B. Arrhythmogenic right ventricular dysplasia
C. Brugada syndrome
D. Congenital long QT syndrome
E. Familial dilated cardiomyopathy

Brugada syndrome

• Sudden death due to PMVT – VF


typically during sleep, fever
Average age at first event: 41 yrs (range 2 - 77 yrs)
• atrial arrhythmias and AV block also occur
• Approx 20% have a sodium channel (SCN5A) mutation
• Male and Asian predilection
• Risk of cardiac arrest approximately 10% per year - (may
be substantially less if no arrhythmia symptoms at time of
diagnosis)
• ICD is protective

Brugada Syndrome 2nd Consensus Conference Antzelevitch et al Circulation 2004


Eckardt L, et al. Long-term prognosis …. Circulation. 2005;111:257-63.

1290
Copyright © Harvard Medical School, 2018. All Rights Reserved.

2. The ECG suggests which of the following:


• A. Hypertrophic cardiomyopathy
• B. Arrhythmogenic right ventricular dysplasia
• C. Brugada syndrome
• D. Congenital long QT syndrome
• E. Familial dilated cardiomyopathy

Congenital Long QT
syndrome
• mutations in genes coding for cardiac potassium or sodium
channels
• autosomal dominant and recessive forms, variable
penetrance.
• Presenting symptoms: syncope, sudden death
• ECG: QTc usually > 0.46s

Torsade de pointes

1291
Copyright © Harvard Medical School, 2018. All Rights Reserved.

References
• Epstein AE et al.J Am Coll Cardiol. May 27, 2008;51(21)
• Bardy GH. N Engl J Med. 2005;352:225-237.
• Tracy CM et al. Heart Rhythm. 2012 Oct;9(10):1737-53
• Moss AJ, et al. N Engl J Med. 2009;361:1329–1338
• Linde C et al. J Am Coll Cardiol. 2008;52:1834–1843

1292
Copyright © Harvard Medical School, 2018. All Rights Reserved.

Inflammation and Cardiovascular Disease

Paul M Ridker, MD, MPH


Director, Center for Cardiovascular Disease Prevention
Divisions of Cardiovascular Medicine and Preventive Medicine
Brigham and Women’s Hospital
Eugene Braunwald Professor of Medicine
Harvard Medical School

Conflicts of Interest Paul M Ridker, MD, MPH

Dr Ridker reports having received research funding support from multiple not-
for-profit entities including the National Heart, Lung, and Blood Institute, the
National Cancer Institute, the American Heart Association, the Doris Duke
Charitable Foundation, the Leducq Foundation, the Donald W Reynolds
Foundation, and the James and Polly Annenberg La Vea Charitable Trusts. Dr
Ridker also reports having received investigator-initiated research support
from Astra-Zeneca, Novartis, Pfizer, and Kowa, as well as non-financial
research support from Amgen. Dr Ridker is listed as a co-inventor on patents
held by the Brigham and Women's Hospital that relate to the use of
inflammatory biomarkers in cardiovascular disease that have been licensed to
Siemens and AstraZeneca, and has served during the past year as a
research consultant to Quintiles, Novartis, Corvidia, Inflazome, Easai, Sanofi,
and Jansen. Neither Dr. Ridker nor the BWH receives any royalties
attributable to sales of the hsCRP test used in connection with the CIRT or
CANTOS trials.

1293
Copyright © Harvard Medical School, 2018. All Rights Reserved.

Inflammation and Cardiovascular Disease : Summary

1. Atherosclerosis is a disorder driven by lipid accumulation and inflammation.

2. hsCRP, a clinical biomarker of inflammation, is as powerful a risk marker for heart


attack and stroke as is LDL cholesterol, HDL cholesterol, or blood pressure, and has
similar variability over time. Diet, exercise, and smoking cessation all lower hsCRP
and all lower cardiovascular risk.

3. In primary prevention, the JUPITER trial has proven that patients with hsCRP
>2mg/L greatly benefit from statin therapy even if lipid levels are low.

4. In secondary prevention, the CANTOS trial has proven that lowering hsCRP, at least
with canakinumab, significantly reduces cardiovascular events.

5. Quality cardiovascular care requires us to recognize the distinction between


“residual cholesterol risk” and “residual inflammatory risk” as these patient groups
have different reasons for recurrent events.

Residual Inflammatory Risk:


Addressing the Obverse Side of the Atherosclerosis Prevention Coin
Ridker PM. Eur Heart J 2016;37:1720-22

Known Cardiovascular Disease


LDL 150 mg/dL (3.8 mmol/L)
hsCRP 4.5mg/L

High Intensity Statin

“Residual Cholesterol Risk” “Residual Inflammatory Risk”


LDL 110 mg/dL (2.8 mmol/L) LDL 70 mg/dL (1.8 mmol/L)
hsCRP 1.8 mg/L hsCRP 3.8 mg/L

Additional Additional
LDL Reduction Inflammation Reduction

IMPROVE-IT : Ezetimibe 6% RRR No Prior Proof of Concept


FOURIER/SPIRE: PCSK9 Inhibition q2 weeks 15% RRR

1294
Copyright © Harvard Medical School, 2018. All Rights Reserved.

Sabatine et al, NEJM 2017;376:1713-1722

Residual Inflammatory Risk:


Addressing the Obverse Side of the Atherosclerosis Prevention Coin
Ridker PM. Eur Heart J 2016;37:1720-22

Known Cardiovascular Disease


LDL 150 mg/dL
hsCRP 4.5mg/L

High Intensity Statin

“Residual Cholesterol Risk” “Residual Inflammatory Risk”


LDL 110 mg/dL LDL 80 mg/dL
hsCRP 1.8 mg/L hsCRP 3.8 mg/L

Additional Additional
LDL Reduction Inflammation Reduction

IMPROVE-IT : Ezetimibe 6% RRR CANTOS


FOURIER/SPIRE: PCSK9 Inhibition q2 weeks 15% RRR Canakinumab 150mg SC q 3 months 15%RRR

1295
Copyright © Harvard Medical School, 2018. All Rights Reserved.

Göran K. Hansson. NEJM 2005; 352:1685-95.

Inflammation in atherosclerosis: from pathophysiology to practice

Libby P et al JACC 2009;54:2129-38

1296
Copyright © Harvard Medical School, 2018. All Rights Reserved.

Low Grade Systemic Inflammation Precedes By Many Years the Onset of Vascular Events

Ridker et al NEJM 1997; 336:973-9

IL-6 and Risk of Future MI in Apparently Healthy Men

P Trend = 0.001

3
P=0.01
P=0.003
Relative Risk of MI

2
P=0.3

0
1 2 3 4
≤1.04 1.04-1.46 1.47-2.28 ≥2.28
Quartile of IL-6 (range, pg/dL)
Ridker et al Circulation 2000;101:1767-1772

1297
Copyright © Harvard Medical School, 2018. All Rights Reserved.

Event-Free Survival According to Baseline Quintiles of


hs-CRP and LDL Cholesterol
Quintiles of hsCRP Quintiles of LDL

1.00
1.00
CVD Event-Free Survival Probability

0.99
0.99

1
1
2
2

0.98
0.98

3
sICAM1 3
VCAM
4
P-selectin
4

0.97
0.97

Eselectin
IL-6, IL-18
IL1ra 5
5
TNF / YKL-40 0.96
0.96

0 2 4 6 8 0 2 4 6 8
Years of Follow-Up Years of Follow-Up

Ridker et al N Engl J Med. 2002;347:1157-1165.

The magnitude of independent risk associated with inflammation is


at least as large, if not larger, than that of BP and cholesterol

Risk Ratio (95%CI)

hsCRP 1.37 (1.27-1.48)

Systolic BP 1.35 (1.25-1.45)

Total cholesterol 1.16 (1.06-1.28)

Non-HDLC 1.28 (1.16-1.40)

0.5 1.0 1.2 1.4 1.8

Risk Ratio (95%CI) per 1-SD higher usual values

Adjusted for age, gender, smoking, diabetes, BMI, triglycerides, alcohol, lipid levels, and hsCRP

Emerging Risk Factor Collaborators, Lancet January 2010 CR-12

1298
Copyright © Harvard Medical School, 2018. All Rights Reserved.

High Sensitivity C-Reactive Protein (hsCRP) : A Test In Context


hsCRP 1 mg/L 3 mg/L 10 mg/L

Lower Risk Moderate Risk Higher Risk Possible Acute Phase Response
Repeat in 2 to 3 weeks
Relative Risk of Future CV Events

hsCRP (mg/L)
Ridker PM. JACC 2016;16:67:712-23

Variability, Tracking Over Time, and Additive Clinical Utility in Risk Prediction
for hsCRP is Almost Identical to That of Cholesterol

hsCRP
hsCRP(mg/L)
mg/L
Variable Intra-Class 95 % CI
15

Correlation
10

hsCRP 0.54 0.53-0.55


5

LDL-C 0.57 0.56-0.58


0

12 Month 24 Month 36 Month 48 Month

LDLC (mmol/L)
LDL Cholesterol mmol/L
4.5

.004 .005 .004


3.0
1.5

Non-lipid risk factors


plus TC
0.0

12 Month 24 Month 36 Month 48 Month

plus TC plus HDLC


HDLC (mmol/L)
HDL Cholesterol mmol/L
plus TC plus HDLC plus hsCRP
2.5
2.0
1.5

0.0 0.01 0.02


1.0

Change in C-statistic
0.5

(as compared with non-lipid-based model)


12 Month 24 Month 36 Month 48 Month

Glynn R et al, Clin Chem 2009;55:305-312 Emerging Risk Factor Collaborators, NEJM 2012;367:1310-1320

1299
Copyright © Harvard Medical School, 2018. All Rights Reserved.

Inflammation, Statin Therapy, and hsCRP: Initial Observations

P Trend = 0.005 -21.6% (P=0.004)


3
0.25

Median hs-CRP (mg/dL)


0.24 Placebo
Relative Risk

2 0.23

0.22

0.21
1 Pravastatin
0.20

0.19

0 0.18
Pravastatin Placebo Pravastatin Placebo Baseline 5 Years
Inflammation Absent Inflammation Present

Ridker et al Circulation. 1998;98:839–844. Ridker et al Circulation. 1999;100:230-235.


15

Minimal Relationship Between Achieved LDL and


Achieved CRP After Initiation of Statin Therapy
r = 0.18
Variance = 3 percent
100
Achieved CRP (mg/L)

10

.1

30 80 130 180
Achieved LDLC (mg/dL)
Ridker et al NEJM 2005;352:20-28. PROVE IT – TIMI 22

1300
Copyright © Harvard Medical School, 2018. All Rights Reserved.

Achieving Low Levels of BOTH Cholesterol and Inflammation


is Crucial for Best Clinical Outcomes: the Concept of “Dual Goals”

0.10

0.40
Recurrent Vascular Events (%)

Recurrent Vascular Events (%)


0.075

0.30
0.05

0.20
0.025

0.10
0.00

0.00
0 0.5 1.0 1.5 2.0 2.5 00 11 22 33 4 4 5 5 6 6 7
Years
Follow-Up (Years)
PROVE-IT IMPROVE-IT
Ridker et al NEJM 2005;352:20-8 Bohula et al, Circulation 2015;132:1224-33

LDL >70 mg/dL LDL <70 mg/dL LDL > 70 mg/dL LDL <70 mg/dL
hsCRP > 2mg/L hsCRP > 2mg/L hsCRP < 2mg/L hsCRP < 2mg/L

Neither Goal LDL Goal hsCRP Goal Dual Goals


Achieved Achieved Achieved Achieved

JUPITER Ridker et al NEJM 2008;359:2195-2207


Trial Design
JUPITER
Multi-National Randomized Double Blind Placebo Controlled Trial of
Rosuvastatin in the Prevention of Cardiovascular Events
Among Individuals With Low LDL and Elevated hsCRP

MI
Rosuvastatin 20 mg (N=8901) Stroke
No Prior CVD or DM
Men >50, Women >60 Unstable
LDL <130 mg/dL Angina
4-week Placebo (N=8901)
CVD Death
hsCRP >2 mg/L run-in CABG/PTCA

Argentina, Belgium, Brazil, Bulgaria, Canada, Chile, Colombia, Costa Rica,


Denmark, El Salvador, Estonia, Germany, Israel, Mexico, Netherlands,
Norway, Panama, Poland, Romania, Russia, South Africa, Switzerland,
United Kingdom, Uruguay, United States, Venezuela

Mean LDLC 104 mg/dL, Mean HDLC 50 mg/dL, hsCRP 4 mg/L


18

1301
Copyright © Harvard Medical School, 2018. All Rights Reserved.

JUPITER Ridker et al NEJM 2008;359:2195-2207

Primary Trial Endpoint : MI, Stroke, UA/Revascularization, CV Death

HR 0.56, 95% CI 0.46-0.69 Placebo 251 / 8901


0.08
P < 0.00001

- 44 %
0.06
Cumulative Incidence
0.04

Rosuvastatin 142 / 8901


0.02
0.00

0 1 2 3 4

Number at Risk Follow-up (years)


Rosuvastatin 8,901 8,631 8,412 6,540 3,893 1,958 1,353 983 544 157
Placebo 8,901 8,621 8,353 6,508 3,872 1,963 1,333 955 534 174

JUPITER Glynn et al, NEJM 2009;360:1851-61


Total Venous Thromboembolism
HR 0.57, 95%CI 0.37-0.86
0.025

P= 0.007
0.020

Placebo 60 / 8901
Cumulative Incidence

0.015

- 43 %
0.010
0.005

Rosuvastatin 34 / 8901
0.000

0 1 2 3 4
Follow-up (years)
Number at Risk
Rosuvastatin 8,901 8,648 8,447 6,575 3,927 1,986 1,376 1,003 548 161
Placebo 8,901 8,652 8,417 6,574 3,943 2,012 1,381 993 556 182

1302
Copyright © Harvard Medical School, 2018. All Rights Reserved.

Can Inflammation Reduction, in the Absence of Lipid


Lowering, Reduce Cardiovascular Event Rates?

From CRP to IL-6 to IL-1: Moving Upstream to Identify novel Targets for Atheroprotection

Sarilumab

Ridker PM. Circ Res 2016;118:145-156.

1303
Copyright © Harvard Medical School, 2018. All Rights Reserved.

NLRP3 Inflammasome, Caspase-1, and IL-1β Maturation


Role of Cholesterol Crystals

Canakinumab Anti-inflammatory Thrombosis Outcomes Study (CANTOS)

Stable CAD (post MI) N = 10,061


Residual Inflammatory Risk 39 Countries
(hsCRP > 2 mg/L) April 2011 - June 2017
1490 Primary Events

Randomized Randomized Randomized Randomized


Canakinumab 50 mg Canakinumab 150 mg Canakinumab 300 mg Placebo
SC q 3 months SC q 3 months SC q 3 months SC q 3 months

Primary Endpoint: Nonfatal MI, Nonfatal Stroke, Cardiovascular Death (MACE)

Secondary Endpoint: MACE plus Unstable Angina Requiring Urgent Revascularization (MACE+)

Additional Adjudicated Endpoints: Cancer, Infection

Ridker et al N Engl J Med. 2017;377:1119-31

1304
Copyright © Harvard Medical School, 2018. All Rights Reserved.

Canakinumab (Novartis)
• high-affinity human monoclonal anti-human
interleukin-1β (IL-1β) antibody currently
indicated for the treatment of IL-1β driven
inflammatory diseases (Cryopyrin-Associated
Period Syndrome [CAPS], Muckle-Wells
Syndrome)
• designed to bind to human IL-1β and
functionally neutralize the bioactivity of this
pro-inflammatory cytokine
• long half-life (4-8 weeks) with CRP and IL-6
reduction for up to 3 months
25

Interleukin-1 Beta as a Target for Atherosclerosis: Biologic Basis for


CANTOS and Beyond

Libby P. JACC 2017;70:2278-89

1305
Copyright © Harvard Medical School, 2018. All Rights Reserved.

CANTOS: Dose-Dependent Effects on Inflammatory Biomarkers and Lipids (48 Months)

Placebo SC q 3 mth

hsCRP
Percent Change from Baseline (median)

Canakinumab 50mg SC q 3 mth


Canakinumab 150mg SC q 3 mth
Canakinumab 300mg SC q 3 mth
LDLC
HDLC
TG

Months

Placebo Canakinumab 50 Canakinumab 150 Canakinumab 300

Ridker et al N Engl J Med. 2017;377:1119-31

CANTOS: Primary Cardiovascular Endpoints


Placebo SC q 3 months
Canakinumab 150/300 mg SC q 3 months

MACE MACE - Plus

HR 0.85 HR 0.83
Cumulative Incidence (%)

Cumulative Incidence (%)

95%CI 0.76-0.96 95%CI 0.74-0.92


P = 0.007 P = 0.0006

0 1 2 3 4 5
Follow-up Years Follow-up Years

35 - 40% reductions in hsCRP and IL-6


No change in LDLC

Ridker et al N Engl J Med. 2017;377:1119-31

1306
Copyright © Harvard Medical School, 2018. All Rights Reserved.

CANTOS : Consistency of Effect Across All patient Groups


Defined By Baseline Clinical Characteristics
Group
MACE MACE Plus
Women
Men

Age < 60 yrs


Age ≥ 60 yrs

Diabetes
No diabetes

Non Smoker
Smoker

BMI < 30 kg/m2


BMI ≥ 30 kg/m2

LDLC < 80 mg/dL


LDLC ≥ 80 mg/dL

hsCRP < 4 mg/L


hsCRP ≥ 4 mg/L

HDLC > 45 mg/dL


HDLC ≤ 45 mg/dL

TG < 150 mg/dL


TG ≥ 150 mg/dL

Overall

0.5 1.0 0.5 1.0


Canakinumab Canakinumab Canakinumab Canakinumab
Superior Inferior Superior Inferior
Ridker et al Lancet 2018;391:319-328

CANTOS: Greater Risk Reduction Among Those


With Greater hsCRP Reduction (MACE)

HR (95% CI) P
Placebo 1.0 (ref) (ref)
On-treatment hsCRP: ≥ 2.0 mg/L 0.95 (0.81, 1.09) 0.48
On-treatment hsCRP: < 2.0 mg/L 0.75 (0.66, 0.85) <0.0001

Placebo
On-treatment hsCRP ≥ 2mg/L

On-treatment hsCRP < 2mg/L

MACE
25% reduction in risk for those achieving hsCRP < 2 mg/L
5 % reduction in risk for those achieving hsCRP ≥ 2 mg/L
(No change in LDL cholesterol)

Ridker et al Lancet 2018;391:319-328

1307
Copyright © Harvard Medical School, 2018. All Rights Reserved.

CANTOS : 31% Reduction in Cardiovascular Mortality and All-Cause Mortality


Among Participants with Robust Inhibition of the Inflammatory Response

CANTOS - Cardiovascular Mortality CANTOS - All Cause Mortality


0.20

0.20
Placebo 1.0 (referent) (referent) Placebo 1.0 (referent) (referent)
Canakinumab, hsCRP ≥2mg/L 0.99 (0.82-1.21) 0.95 Canakinumab, hsCRP ≥2mg/L 1.05 (0.90-1.22) 0.56
Canakinumab, hsCRP <2mg/L 0.69 (0.56-0.85) 0.0004 Canakinumab, hsCRP <2mg/L 0.69 (0.58-0.81) <0.0001
0.15

0.15
Cumulative Incidence

Cumulative Incidence
0.10

0.10
0.05

0.05
0.00

0 1 2 3 4 5 0.00 0 1 2 3 4 5
Years Years

35 - 40% reductions in hsCRP and IL-6


No change in LDLC
Ridker et al Circulation 2018;137:1763-1766

Lisa Coussens LM, Zena Werb.


Nature. 2002 420(6917):860-7.

“Inflammation is a critical component of


tumour progression (and) it is now becoming
clear that the tumour microenvironment,
which is largely orchestrated by inflammatory
cells, is an indispensable participant in the
neoplastic process, fostering
proliferation, survival, and migration”.

1308
Copyright © Harvard Medical School, 2018. All Rights Reserved.

CANTOS: Additional Non-Cardiovascular Clinical Benefits


Incident Lung Cancer
3.0
HR (95%CI) P
Placebo 1.0 (referent) (referent)
Canakinumab 50 mg 0.77 (0.49-1.20) 0.25
Canakinumab 150 mg 0.61 (0.39-0.97) 0.034
Canakinumab 300 mg 0.33 (0.18-0.59) 0.00008

P-trend across groups = 0.0003


2.0
Cumulative Incidence (%)
1.0

Canakinumab 300 mg
67% reduction
in incident lung cancer
0.0

P=0.00008

0 1 2 3 4 5
Follow-up Years
Lancet. 2017;390:1833-1842

CANTOS: Increased Incidence Rates of MACE, Cardiovascular Death,


and All-Cause Mortality Among Patients with Moderate CKD as
Compared to Those with Normal Renal Function.

P < 0.0001
Incidence Rate / 100 person-years

P < 0.0001

P < 0.0001

Moderate CKD Normal Renal Function

1309
Copyright © Harvard Medical School, 2018. All Rights Reserved.

CANTOS : Effects of Canakinumab on Major Vascular Events Among


Those With and Without Moderate CKD at Study Entry

eGFR < 60 mL/min/1.73m2 eGFR > 60 mL/min/1.73m2


Confirmed MACE+Urgent Revascularization Confirmed MACE+Urgent Revascularization
Among Subjects whose baseline eGFR <60 Among Subjects whose baseline eGFR >=60

0.4
0.4

HR (95% CI) P
_______________________________________________ HR (95% CI) P
_______________________________________________
Placebo 1.0 (ref) (ref) Placebo 1.0 (ref) (ref)
Active Canakinumab 0.82 (0.68,1.00) 0.054 Active Canakinumab 0.86 (0.77,0.97) 0.012

HR 0.82 HR 0.86

0.3
0.3

Cumulative Incidence
Cumulative Incidence

95%CI 0.68-1.00 95%CI 0.77-0.97


P = 0.05 P = 0.01

0.2
0.2

0.1
0.1
0.0

0.0
0 1 2 3 4 5 0 1 2 3 4 5
No. at risk: Follow-up (years) No. at risk: Follow-up (years)
Placebo 626 561 499 Years423 182 27
Placebo 2717 2546 2422 Years
2155 1056 179
Canakinumab 1249 1139 1047 894 410 58
Canakinumab 5467 5177 4940 4410 2155 382

Moderate CKD Normal Renal Function


(N = 1875) (N = 8184)

Residual Inflammatory Risk:


Addressing the Obverse Side of the Atherosclerosis Prevention Coin
Eur Heart J 2016;37:1720-22

Known Cardiovascular Disease


LDL 150 mg/dL
hsCRP 4.5mg/L

High Intensity Statin

“Residual Cholesterol Risk” “Residual Inflammatory Risk”


LDL 110 mg/dL LDL 80 mg/dL
hsCRP 1.8 mg/L hsCRP 3.8 mg/L

Additional Additional
LDL Reduction Inflammation Reduction

IMPROVE-IT : Ezetimibe 6% RRR CANTOS


FOURIER/SPIRE: PCSK9 Inhibition q2 weeks 15% RRR Canakinumab 150mg SC q 3 months 15%RRR

1310
Copyright © Harvard Medical School, 2018. All Rights Reserved.

CANTOS : Adding a New Axis to the Oxford LDL Lowering Line

60

50
JUPITER
Relative Risk Reduction (%)

40

30

20

10

0 10 20 30 40 50 60
% Reduction in LDL-C
Ridker ACC 2018

How Common is Residual Inflammatory Risk?


Following Following
High-Intensity Statins High-Intensity Statins
Plus Ezetimibe

Residual Inflammatory Risk Residual Cholesterol Risk Both Neither


hsCRP ≥ 2 mg/L hsCRP < 2 mg/L hsCRP ≥ 2 mg/L hsCRP < 2 mg/L
LDLC < 70 mg/dL LDLC ≥ 70 mg/dL LDLC ≥ 70 mg/dL LDLC < 70 mg/dL

Circulation Res 2017;120:617-9.

1311
Copyright © Harvard Medical School, 2018. All Rights Reserved.

Relationships of hsCRP Levels With Future Cardiovascular Events


Among High-Risk Patients Treated with Both Statins and PCSK9 Inhibitors

FOURIER SPIRE 1/2

Incidence rate / 100 person years


Incidence rate / 3 years

hsCRP hsCRP

Bohula et al, Circulation 2018 Pradhan et al, Circulation 2018

How Common is Residual Inflammatory Risk?


Following Following Following
High-Intensity Statins High-Intensity Statins High-Intensity Statins
Plus Ezetimibe Plus PCSK9 Inhibition

Residual Inflammatory Risk Residual Cholesterol Risk Both Neither


hsCRP ≥ 2 mg/L hsCRP < 2 mg/L hsCRP ≥ 2 mg/L hsCRP < 2 mg/L
LDLC < 70 mg/dL LDLC ≥ 70 mg/dL LDLC ≥ 70 mg/dL LDLC < 70 mg/dL

Circulation Res 2017;120:617-9.


Pradhan et al Circulation 2018 (on line).

1312
Copyright © Harvard Medical School, 2018. All Rights Reserved.

FOURIER: Residual Inflammatory Risk is Present


Irrespective of On-Treatment LDL-C

18.2%
16.4%
14.7% 15.4%
13.1%
20% 14.9%
Incidence Rate at 3 Years

12.6% 13.2%
10.8% 12.0%
15% 12.3%
10.4% 10.9%
9.0% 9.8%
10%
>3
5%
1-3
<1 hsCRP
0% (mg/L)
<20 20-49 50-69 70-99 ≥100
LDL-C at 1 month (mg/dL)

Bohula E. Circulation 2018. DOI: 10.1161/CirculationAHA. 118.034032

Inflammation and Atherothrombosis :


Clinical Approaches

Intervention Reduces Reduces Event Rates?


Inflammation?

Smoking Cessation Yes Yes

Exercise Yes Yes

Diet Yes Yes

Weight Loss Yes Yes

Statins Yes Yes

IL-1β inhibition Yes Yes

1313
Copyright © Harvard Medical School, 2018. All Rights Reserved.

Sustained Weight Loss Reduces hsCRP Irrespective of Diet : NIH Pounds Lost Trial

Median Percent Change 6 months 24 months

Nicklas JM, et al; Obesity 2013;21(4):681-9.

Cardiovascular Inflammation Reduction Trial (CIRT)


Primary Aims (NHLBI - Ridker PI)

Stable CAD • To directly test the


On Statin, ACE/ARB, BB, ASA inflammatory hypothesis of
atherothrombosis
• To evaluate in a randomized,
Persistent Evidence of Inflammation double-blind, placebo-
Diabetes or Metabolic Syndrome controlled trial whether MTX
given at a target dose of 20
mg po weekly over a three
year period will reduce rates
MTX 15-20 mg Placebo of recurrent myocardial
Weekly infarction, stroke, or
cardiovascular death among
patients with a prior history of
atherosclerosis and either
Nonfatal MI, Nonfatal Stroke, type 2 diabetes or metabolic
Cardiovascular Death syndrome.

N = 5,500 NHLBI-Sponsored
350 US and Canadian Sites

1314
Copyright © Harvard Medical School, 2018. All Rights Reserved.

Ongoing Trials

LoDoCo2
Colcot

Nidorf, SM, et al; JACC 2013; 61:404-10.

Conclusions:
The Canakinumab Anti-Inflammatory Thrombosis Outcomes Study (CANTOS)

1. CANTOS demonstrates that targeting the IL-1b to IL-6 pathway of innate


immunity with canakinumab reduces cardiovascular event rates and
potentially reduces rates of incident lung cancer and lung cancer mortality.

2. CANTOS thus provides critical proof-of-concept that inflammation


inhibition, in the absence of lipid lowering, can improve atherothrombotic
outcomes. This is the first hard evidence in 40 years of an effective therapy
for atherosclerosis not directly related to cholesterol reduction.

3. The magnitude of hsCRP reduction following a single dose of canakinumab


may provide a simple clinical method to identify individuals most likely to
accrue the largest cardiovascular and cancer benefits from continued
treatment, including 31% reductions in cardiovascular as well as all-cause
mortality.

1315
Copyright © Harvard Medical School, 2018. All Rights Reserved.

Known Cardiovascular Disease

High Intensity Statin

Biologic Residual Residual Residual Residual Residual


Issue Cholesterol Risk Inflammatory Risk Thrombotic Risk Triglyceride Risk Lp(a) Risk
Critical
Biomarker
LDL-C >100mg/dL hsCRP >2mg/L No simple TG >200mg/dL Lp(a) >50mg/dL
biomarker HDL <40mg/dL

Potential Targeted Targeted Targeted Targeted Targeted


Intervention LDL / Apo B Inflammation Antithrombotic Triglyceride Lp(a)
Reduction Reduction Reduction Reduction Reduction

Randomized IMPROVE-IT CANTOS COMPASS Ongoing Planned


Trial FOURIER, SPIRE
Evidence ODDYSEY

Inflammation and Cardiovascular Disease : Summary

1. Atherosclerosis is a disorder driven by lipid accumulation and inflammation.

2. hsCRP, a clinical biomarker of inflammation, is as powerful a risk marker for heart


attack and stroke as is LDL cholesterol, HDL cholesterol, or blood pressure, and has
similar variability over time. Diet, exercise, and smoking cessation all lower hsCRP
and all lower cardiovascular risk.

3. In primary prevention, the JUPITER trial has proven that patients with hsCRP
>2mg/L greatly benefit from statin therapy even if lipid levels are low.

4. In secondary prevention, the CANTOS trial has proven that lowering hsCRP, at least
with canakinumab, significantly reduces cardiovascular events.

5. Quality cardiovascular care requires us to recognize the distinction between


“residual cholesterol risk” and “residual inflammatory risk” as these patient groups
have different reasons for recurrent events.

1316
Copyright © Harvard Medical School, 2018. All Rights Reserved.

Inflammation, Atherothrombosis, and Vascular Prevention:


Three Translational Questions

Is there evidence that individuals with elevated levels of


inflammatory biomarkers are at high vascular risk even when
other risk factors are acceptable? Yes (hsCRP, 1997)
Is there evidence that individuals identified at increased risk
due to inflammation benefit from a therapy they otherwise
would not have received? Yes (statins, JUPITER 2008)
Is there evidence that reducing inflammation per se will
reduce vascular events? Yes (CANTOS, 2017)
“Lower is better” appears to be true for both LDLC and
hsCRP in both primary and secondary prevention
49

1317
Copyright © Harvard Medical School, 2018. All Rights Reserved.

Take Home Messages in


Cardiology
Akshay Desai, MD
Brigham and Women’s Hospital

PREVENTION

1318
Copyright © Harvard Medical School, 2018. All Rights Reserved.

Core Principles

• Lifestyle interventions are appropriate for


everyone

• Assess long term cardiovascular risk in all

• Match intensity of preventive interventions to


baseline cardiovascular risk

Fundamentals of Prevention

1. ≥150 minutes moderate activity /week


or ≥75 minutes vigorous activity/week
2. Eat a healthy diet (4-5 components of
healthy diet score*)
3. Have a normal body weight (BMI < 25)
4. Never smoked or quit >1 year ago
5. Total cholesterol <200 mg/dL
6. Blood pressure <120/80 mm Hg
7. Fasting blood glucose <100 mg/dL
* 1) 4.5 cups or more of fruits and vegetables per
day 2) two or more 3.5-oz servings of fish per
week 3) three servings per day of whole grains 4)
less than 1500 mg of sodium per day 5) 36 ounces or
less of sugar-sweetened beverages per week

Lloyd-Jones et al. Circulation 2010; 121:586-613

1319
Copyright © Harvard Medical School, 2018. All Rights Reserved.

4 Statin Benefit Groups


( 1-3) High Risk:
1. Clinical ASCVD*
2. LDL–c >190 mg/dL, Age >21 years
3. Primary prevention – Diabetes:
Age 40-75 years, LDL–c 70-189 mg/dL
(4) Primary prevention†
4. No Diabetes:
≥7.5% 10-year risk, Age 40-75 years, LDL–c 70-189
mg/dL
Generally use high-intensity statin (e.g. atorvastatin 40-80 mg or
rosuvastatin 20 mg) to achieve LDL reduction >= 50% from baseline

Stone et al JACC 2014;63:2889-934

2017 Hypertension Guidelines


Category Systolic Pressure Diastolic Pressure
(mmHg) (mmHg)
Normal < 120 and <80
Elevated 120-129 and <80
Stage 1 Hypertension 130-139 or 80-89

Stage 2 Hypertension ≥140 or ≥90

Class I recommendation in primary prevention:


Initiate BP-lowering meds for:
• SBP ≥140 mm Hg or DBP ≥90 mm Hg
• SBP ≥130 mm Hg or DBP ≥80, if 10-year ASCVD risk ≥10%

On-therapy BP target: < 130/80


Whelton PK, et al. 2017 Guideline for the prevention, detection, evaluation, and
management of high blood pressure in adults. J Am Coll Cardiol 2018; 71:2199.

1320
Copyright © Harvard Medical School, 2018. All Rights Reserved.

Lifestyle Modifications for BP Control

Modification Recommendation Approximate SBP


Reduction Range

Weight Maintain normal body weight 5-20 mmHg/10 kg


reduction (BMI=18.5-25) weight lost

DASH eating Diet rich in fruits, vegetables, 8-14 mmHg


plan low fat dairy and reduced in fat

Restrict <2.4 grams of sodium per day 2-8 mmHg


sodium intake

Physical Regular aerobic exercise for at 4-10 mmHg


activity least 30 minutes at least 5 days
of the week
Moderate <2 drinks/day for men and <1 2-4 mmHg
alcohol drink/day for women

Chobanian AV et al. JAMA 2003;289:2560-2572

Summary

1. First step in prevention is to assess ASCVD risk


(risk factors, global risk score)
2. Lifestyle improvement is the most important component of ASCVD
prevention and risk factor treatment (Life’s Simple 7)
3. Statins added to lifestyle to reduce risk of ASCVD in higher risk
individuals (4 statin benefit groups); PCSK9i in the very highest
risk patients
4. Blood pressure control: Target BP for most patients <130/80
mmHg; risk-based assessment
5. Aspirin (81-162 mg/d) in higher risk individuals if benefit outweighs
risk of bleeding (avoid in low risk individuals and elderly)

1321
Copyright © Harvard Medical School, 2018. All Rights Reserved.

VASCULAR MEDICINE

11

1322
Copyright © Harvard Medical School, 2018. All Rights Reserved.

PAD Risk-reduction Therapies

• Lifestyle modifications
– Weight maintenance/reduction
• Smoking
– Complete cessation
• Diabetes mellitus
– HbA1c goal, target specific agents
• Dyslipidemia
– High intensity statin + other agents (lower is better)
• Hypertension
– Therapies to achieve target, ACE inhibitors (HOPE Trial)
• Antithrombotic therapy
– Aspirin or Clopidogrel for symptomatic PAD

* Supervised Exercise program may improve


walking distance and relieve symptoms, consider
prior to revascularization

AAA: Summary Points

• Screen for AAA (exam + U/S) in


• Men > 60 who are siblings or offspring of
patients with AAA
• Men 65-75 who have ever smoked

• Intervene for symptomatic patients or


according to maximal diameter
• ≥ 5.5 cm Repair
• 4.0-5.5 cm Surveillance q 6-12 mos
• < 4.0 cm Surveillance q 2-3 years
13

1323
Copyright © Harvard Medical School, 2018. All Rights Reserved.

Thoracic Aortic Disease

When to Intervene?

• Generally 5.5 cm, or growth > 0.5 cm/y


including bicuspid AoV (unless RF for
dissection such as FH)

• For genetic syndrome (MFS, LDS, EDS,


familial syndrome) may undergo at smaller
diameters (4.0-5.0) – for MFS contemplating
pregnancy > 4.0 cm

2010 ACCF/AHA/AATS/ACR/ASA/
SCA/SCAI/SIR/STS/SVM Guidelines for the Diagnosis and Management of Patients with Thoracic Aortic Disease

Indications for Carotid Revascularization


ACC/AHA Guidelines
I IIa IIb III
• Patients at average or low surgical risk who
experience nondisabling ischemic stroke or TIA within
6 months should undergo CEA if the ipsilateral ICA
stenosis is >70%.
I IIa IIb III
• Carotid stenting is indicated as an alternative to CEA
for symptomatic patients at average or low risk of
complications.

Asymptomatic Carotid Stenosis is still an area of controversy,


routine revascularization likely not indicated

Brott et a. JACC. 2011;57:1002-44

1324
Copyright © Harvard Medical School, 2018. All Rights Reserved.

ACUTE CORONARY
SYNDROMES

ST-Elevation MI (STEMI)
• Consider immediate reperfusion therapy
• In whom?
– Within 12 hrs of sx onset, or
– 12-24 hrs after sx onset if clinical or ECG evidence of
ongoing ischemia
• How?
– Primary PCI (including transfer to PCI-capable hosp
if door-in to door-out time will be <30 min &
1st med contact to PCI anticipated <120 min)
– Fibrinolytic (barring contraindications*)
*Absolute: prior ICH; intracranial neoplasm, aneurysm, or AVM; stroke or head trauma w/in 3 mos; active
internal bleeding or diathesis; suspected AoD
*Relative: severe HTN; stroke; prolonged CPR; recent bleed, surgery or trauma; noncompressible vasc
puncture; pregnancy; current use of anticoagulants
An Academic Research Organization of
Brigham and Women’s Hospital and Harvard Medical School

1325
Copyright © Harvard Medical School, 2018. All Rights Reserved.

Management Strategy in NSTEACS


INVASIVE
ASA 162-325 mg
(ie, angiography for all in ~48 hrs)
P2Y12 inhibitor
NSTEACS PCI / CABG
Anticoagulant
anatomy
Long-term
Initial Med Rx
Med Rx
high-risk low-risk

recurrent
angina

Cont’d
Med Rx
INVASIVE APPROACH
PREFERRED IN MOST
PATIENTS CONSERVATIVE
An Academic Research Organization of
(ie, selective angiography)
Brigham and Women’s Hospital and Harvard Medical School

Discharge Checklist

Risk Factor Modification Medical Therapy


1. Low chol (<200 mg/d) and low 1. Aspirin 81 mg/d for life
fat (<7% saturated) diet 2. P2Y12 Inhibitor for 12 mos
2. LDL goal <70 mg/dl 3. β-Blocker
3. ? HDL >40 md/dl 4. Statin high-intensity lipid-lowering
4. BP <140/90, <130/80 if DM or (eg, atorva 80 mg qd)
CKD 5. ACEI (or ARB if ACEI intol) if CHF,
5. Smoking cessation EF<0.40, HTN, DM; ? in all CAD
6. If DM, HbA1c cntl 6. Aldo antag if EF <40% & CHF or
(? avoid TZDs) DM
7. Exercise (≥30 min 3-4 x per 7. Nitrates standing if sx, prn for all
wk)
8. BMI goal 18.5-24.9 kg/m2

1326
Copyright © Harvard Medical School, 2018. All Rights Reserved.

ST Which ST-T PR
Coving Leads? Evolution Segment

Pericarditis Diffuse Days- ↓


weeks
(STE < 5 mm)

Acute Localized Hours Normal


STEMI

1327
Copyright © Harvard Medical School, 2018. All Rights Reserved.

Management of Idiopathic (post-viral)


Acute Pericarditis

• Aspirin (e.g., 650 mg q 4-6 h)


• NSAIAs (e.g., ibuprofen 400-800 mg q 6-8 h)
• Colchicine (off-label; Class IIb: evidence favors efficacy)
CORP. Ann Int Med 2011;155:409.
COPE. Circulation 2005;112:2012.
CORE. Arch Intern Med 2005;165:1987.

• Prednisone 0.25-0.5 mg/kg/d (last resort)

• Sinus tach • Low voltage • Electrical alternans

1328
Copyright © Harvard Medical School, 2018. All Rights Reserved.

Cardiac Tamponade
(Beck’s Triad)

• Jugular venous distention


• Hypotension with pulsus paradoxus
• “Small, quiet heart”

HEART FAILURE

1329
Copyright © Harvard Medical School, 2018. All Rights Reserved.

Guideline-Directed Medical
Management of HF: 2018

Relief of EF≤40% NYHA II-IV Still


Congestive NYHA I-IV Symptomatic?
Symptoms
Consider
additional
therapies

MRA
ACEi/ARB/ Beta-Blocker
ARNI Ivabradine
Spironolactone Hydral/ISDN
Diuretics Lisinopril, etc. Carvedilol Digoxin
Eplerenone
Loop Valsartan, etc. Metoprolol
(thiazide) Sacubitril/Val Bisoprolol

Yancy C, et al. Circulation. 2013;128:e240-e327


McMurray JJV, et al. Eur Heart J. 2012;33:1787-1747

Yancy C, et al. Circulation 2017


Yancy, et al. J Am Coll Cardiol 2017

1330
Copyright © Harvard Medical School, 2018. All Rights Reserved.

Device Therapy
• ICD
– Prior Cardiac Arrest, Sustained VT
– Symptomatic HF, EF≤35%
– Asymptomatic, LVEF<30%, CAD
– High risk groups (HCM, Long QT, Brugada)

• CRT (Biventricular Pacing)


– EF≤35%, symptomatic HF, QRS>130 ms, LBBB

VENOUS THROMBOEMBOLISM

1331
Copyright © Harvard Medical School, 2018. All Rights Reserved.

DEFINITIONS OF PE:
AHA PE Guidelines 2011
• Massive PE (5-10%): sustained
hypotension, pulselessness, or
persistent bradycardia
• Submassive PE (20-25%): RV
dysfunction or myocardial necrosis,
without hypotension
• Low Risk PE (70%): no markers of
adverse prognosis
(Circulation 2011; 123: 1788-1830)

RISKS FOR POOR PROGNOSIS


1. Elevated biomarkers (troponin)
(European Heart J 2010; 31: 1836)
2. RV enlargement/ hypokinesis: ECG
RV/ LV ratio > 0.9
CT—(JACC Cardiovasc Imaging
2011; 4: 841-849)
ECHO—(Circulation 2010;122: 1124)

1332
Copyright © Harvard Medical School, 2018. All Rights Reserved.

RISK STRATIFY TO GUIDE


MANAGEMENT STRATEGY
5% 10% 15% 70%
High Risk Submassive Submassive Low Risk
High: RVD+Tn Low: RVD or Tn
Reperfuse: Reperfuse, Hospitalize, ? Early
Lysis/ Or Watch Anticoa- Discharge;
Embo- And gulate Anticoa-
lectomy Wait gulate
(ESC Guidelines. European Heart J 2014;
35: 3033-3080)

ACUTE VTE TREATMENT:


NOAC EFFICACY
• All 4 NOACs are noninferior to
LMWH/ warfarin for efficacy,
regardless of weight, PE vs. DVT,
CKD, and cancer.
• Edoxaban: prespecified submassive
PE subgroup showed superiority.

(van Es N, et al. Blood 2014; 124: 1968-1975)

1333
Copyright © Harvard Medical School, 2018. All Rights Reserved.

2014 ESC GUIDELINES


Recommendation Class Level

Extended oral IIa B


anticoagulation should
be considered for
patients with a first
episode of unprovoked
PE and low bleeding
risk.
(Konstantinides SV, et al. Eur Heart J 2014 Aug 29)

ECG / ARRHYTHMIAS

1334
Copyright © Harvard Medical School, 2018. All Rights Reserved.

1335
Copyright © Harvard Medical School, 2018. All Rights Reserved.

18 year old male with palpitations

1336
Copyright © Harvard Medical School, 2018. All Rights Reserved.

Management of Atrial Fibrillation

• Treat predisposing factors


• Prevent thromboembolism / stroke
• Rate control (target HR<110 bpm)
• Restore/maintain sinus rhythm?

CHADS2 – VASc Score for Stroke Risk in AF

Risk Factors Score Implications of the Score

Congestive HF 1 0 = stroke risk 1.2%/yr


- Rx: no oral
Hypertension 1 anticoagulation or asa
Age > 75 yrs 2
1 = stroke risk 2.2%/yr
Diabetes mellitus 1
- Rx oral antiicoagulant
or asa
Prior stroke or TIA 2
2 or more =
Vascular Disease 1 stroke risk >3% / yr
- Rx oral anticoagulant
Age 65 – 75 1

Sex category = AF guidelines of the European Society of


1
female Cardiology. European Heart J (2010) 31, 2369

1337
Copyright © Harvard Medical School, 2018. All Rights Reserved.

Anticoagulation - Platelet inhibitors

• Aspirin is inferior to warfarin

• Aspirin + clopidogrel is superior to asa


alone but with increased bleeding risk

• Aspirin + clopidogrel is inferior to warfarin

Direct Oral Anticoagulants

• All are non-inferior to warfarin in “non-valvular” AF*


Apixaban was superior for stroke prevension in one trial

* Should not be used in mitral stenosis (not evaluated) or


mechanical valves (dabigatran is inadequate, others
have not been evaluated)

• Lower risk of intracranial bleeding than warfarin


• Rapid onset of action, reliable pharmacokinetics, and
fewer drug interactions than warfarin
– Bridging with parenteral heparin is not needed for initiating
therapy

1338
Copyright © Harvard Medical School, 2018. All Rights Reserved.

VALVULAR HEART DISEASE

Natural History of Aortic Stenosis


Onset severe
Angina
symptoms
100 Syncope
Failure
80 period
Latent period,
(increasing
increasing obstruction,
60 myocardial overload) 0 2 4 6
hypertrophy
Avg
Yrs survival (yr)
after onset
40
Average death
20
age ( )
0
0 40 50 60 70 80
Age (yr)
Ross J Jr. and Braunwald E: Circ 38(Suppl 5):61, 1968
CP1061785-6

1339
Copyright © Harvard Medical School, 2018. All Rights Reserved.

Aortic Stenosis
Classification of Severity

MILD MODERATE SEVERE

Jet Velocity (m/s) < 3.0 3.0 - 4.0 > 4.0

Mean Gradient (mmHg) < 25 25-40 > 40

Valve Area (cm2) > 1.5 1.0-1.5 < 1.0

Valve Are Index (cm2/m2) < 0.6

Zoghbi W et al. J Am Soc Echocardiogr 2003; 16:777.

Severe Aortic Stenosis


Indications for Surgery

Indication ACC/AHA1 ESC2


2006 2012

Symptoms I(B) I(B)


Need for CABG/Ao Surgery I(C) I(C)
LVEF < 0.50 I(C) I(C)

Abnormal ETT IIb(C) IIa or IIb(C)


Rapid Progression/Delay IIb(C) IIa(C)
Very severe AS IIb(C) IIa(C)
Severe LVH (>15mm) ----- IIb(C)*
1. Bonow R et al. J Am Coll Cardiol 2006
2. Vahanian A et al. European Heart Journal 2012

1340
Copyright © Harvard Medical School, 2018. All Rights Reserved.

BAV Aortopathy

Chronic MR

Medical Therapy
• ABx prophylaxis
No role when and if indicated
for vasodilator therapy
in
asymptomatic, normotensive patients
• Management of AF
with chronic severe
• Management of CAD MR and normal LV
function
• ACE-I or ARB for HTN, reduced EF

1341
Copyright © Harvard Medical School, 2018. All Rights Reserved.

Endocarditis Prophylaxis
Endocarditis Prophylaxis Recommend- Level of
ation Evidence
Prosthetic cardiac valve or prosthetic material IIa C-LD
for valve repair (including TAVR)
Previous infective endocarditis IIa C-LD
Unrepaired cyanotic congenital heart disease IIa C-LD
(CHD)
Repaired CHD with prosthetic material, first 6 IIa C-LD
months
Repaired CHD with residual defects at site of IIa C-LD
patch or device
Cardiac transplant with valve regurgitation IIa C-LD
due to structurally abnormal valve
For dental procedures that involve manipulation of either gingival tissue or the
periapical region of teeth or perforation of the oral mucosa.
ACC/AHA VHD guidelines 2017

ATRIAL SEPTAL DEFECT


• Types: secundum, primum , sinus venosus,
coronary sinus

• Exam: Grade 2 MSM, fixed splitting S2

• ECG: IRBB. LAD = primum.

• ECHO: RV volume overload, shunt flow,


associated findings (MVP, cleft MV, APVD)

1342
Copyright © Harvard Medical School, 2018. All Rights Reserved.

PATENT FORAMEN OVALE


• Present in 25-30% of population
• Sometimes associated with interatrial
septal aneurysm
• Implied role in
– Cryptogenic stroke
– Migraine
– Platypnea-orthodeoxia
– Decompression sickness

GOOD LUCK!

1343
Copyright © Harvard Medical School, 2018. All Rights Reserved.

Board Review in
Cardiology
Psychiatry Overview
Garrick C. Stewart, MD, MPH
Associate Physician, Center for Advanced Heart Disease
Division of Cardiovascular Medicine
Brigham and Women’s Hospital
Harvard Medical School

Disclosures of Conflicts of Interest

•No disclosure

1344
Copyright © Harvard Medical School, 2018. All Rights Reserved.

Question #1
A 78-year old man with bioprosthetic mitral valve replacement and
10 year history of diabetes mellitus presents with new onset atrial
fibrillation with heart rate of 82 beats/minute and blood pressure of
156/96 mm Hg. Echocardiogram reveals LVEF 56%. Which of the
following is a true statement?

A. Restoration of sinus rhythm reduces risk of stroke compared with rate control
and anticoagulation
B. Patient can be treated with rivaroxaban instead of warfarin
C. Warfarin should be started to reduce risk of stroke
D. The combination of aspirin and clopidogrel is equivalent to warfarin
E. Patient can be cardioverted now and complete 4 weeks of dabigatran

Answer #1
A 78-year old man with bioprosthetic mitral valve replacement and
10 year history of diabetes mellitus presents with new onset atrial
fibrillation with heart rate of 82 beats/minute and blood pressure of
156/96 mm Hg. Echocardiogram reveals LVEF 56%. Which of the
following is a true statement?

A. Restoration of sinus rhythm reduces risk of stroke compared with rate control
and anticoagulation
B. Patient can be treated with rivaroxaban instead of warfarin
C. Warfarin should be started to reduce risk of stroke
D. The combination of aspirin and clopidogrel is equivalent to warfarin
E. Patient can be cardioverted now and complete 4 weeks of dabigatran

1345
Copyright © Harvard Medical School, 2018. All Rights Reserved.

Anticoagulation Indicated if Score ≥2


(Consider Anticoagulation if Score 1)

Chua SK et al. J Thorac Cardiovasc Surg. 2013 Apr 26


http://clincalc.com/Cardiology/Stroke/CHADSVASC.aspx

Novel Oral Anticoagulants

Nonvalvular Atrial Fibrillation: AF in the absence of rheumatic mitral


stenosis, a mechanical or bioprosthetic heart valve, or mitral valve repair.

Novel/Direct Oral Anticoagulants have only been studied in nonvalvular AF.

Fontana et al. Eur Heart J 2014; doi:10.1093/eurheartj/ehu027

1346
Copyright © Harvard Medical School, 2018. All Rights Reserved.

Question #2
A 42 year old man presents with several days of sharp substernal
chest pain that is relieved by sitting forward. He had a similar
presentation two years ago. Physical exam reveals flat neck veins,
regular rhythm, friction rub and clear lung fields. A 12-lead ECG is
obtained:

© 2015 ABIM

Question #2
Randomized, placebo-controlled trials have
shown which of the following agents to be
beneficial in the treatment of this condition?
A. Aspirin
B. Colchicine
C. Corticosteroids
D. Cyclophosphamide
E. Ibuprofen

1347
Copyright © Harvard Medical School, 2018. All Rights Reserved.

Question #2
Randomized, placebo-controlled trials have
shown which of the following agents to be
beneficial in the treatment of this condition?
A. Aspirin
B. Colchicine
C. Corticosteroids
D. Cyclophosphamide
E. Ibuprofen

Survival Free of Incessant or


Recurrent Pericarditis

Imazio M et al. N Engl J Med 2013;369:1522-1528.

1348
Copyright © Harvard Medical School, 2018. All Rights Reserved.

Question #3
A 45-year-old man is admitted with heart failure. He has noted orthopnea
and progressive severe fatigue over the past 6 months. He had been
previously healthy and takes no medications. On exam, he appears chronically
ill. His blood pressure is 90/75 mm Hg; heart rate is 100 bpm.

Numerous ecchymoses are scattered over his extremities. Jugular venous


pressures are elevated to the angle of the jaw in the seated position. Carotid
upstrokes are low volume. Both lung bases are dull. There is a soft
holosystolic murmur. Summation gallop is present with prominent P2. Ascites
and hepatomegaly are present. 2+ edema and cold extremities are noted.

ECG shows diffusely low QRS voltages. Echocardiogram shows left ventricular
ejection fraction (LVEF) 50% and septal and posterior wall 2 cm. N-terminal
pro–B-type natriuretic peptide is 5000.

Question #3
Which of the following is the most likely cause of his heart
failure?
A. Coronary artery disease
B. Cardiac amyloid
C. Thiamine deficiency
D. Constrictive pericarditis
E. Aortic stenosis

1349
Copyright © Harvard Medical School, 2018. All Rights Reserved.

Answer #3
Which of the following is the most likely cause of his heart
failure?
A. Coronary artery disease
B. Cardiac amyloid
C. Thiamine deficiency
D. Constrictive pericarditis
E. Aortic stenosis

Cardiac Amyloidosis
Normal Amyloid

Quarta et al. Circulation 2012; 126: e178-82.

1350
Copyright © Harvard Medical School, 2018. All Rights Reserved.

Question #4
Cardiac troponin are frequently used in medicine. Each
of the following statements regarding troponin elevation
is true EXCEPT:
A. Cardiac troponin T 0.09 (ULN 0.03) is associated with worse
outcomes in an acute coronary syndrome
B. Troponin is commonly elevated in asymptomatic patients
with chronic kidney disease
C. Non-ischemic cardiomyopathy patients admitted with mild
troponin elevations in the setting of decompensated heart
failure have a worse prognosis than patients with normal
troponins
D. Troponin does not increase with acute pulmonary embolism
E. Troponin elevation is a predictor of mortality in setting of
sepsis

Answer #4
Cardiac troponin are frequently used in medicine. Each
of the following statements regarding troponin elevation
is true EXCEPT:
A. Cardiac troponin T 0.09 (ULN 0.03) is associated with worse
outcomes in an acute coronary syndrome
B. Troponin is commonly elevated in asymptomatic patients
with chronic kidney disease
C. Non-ischemic cardiomyopathy patients admitted with mild
troponin elevations in the setting of decompensated heart
failure have a worse prognosis than patients with normal
troponins
D. Troponin does not increase with acute pulmonary
embolism
E. Troponin elevation is a predictor of mortality in setting of
sepsis

1351
Copyright © Harvard Medical School, 2018. All Rights Reserved.

Acute Right Ventricular Strain Can Lead to


Troponin Elevation in PE

Akram et al. QJM 2009; 102: 407-414

Meta-Analysis of Sepsis and


Troponin Elevation

Bessiere et al. Intensive Care Med 2013;39:1181-89

1352
Copyright © Harvard Medical School, 2018. All Rights Reserved.

Question #5
A 64 year old woman with chronic systolic heart failure, EF
25%, in NYHA Class 2 and 3 prior HF hospitalizations
presents for a routine visit. She is euvolemic with heart
rate is 60bpm in sinus rhythm, BP 110/62. She is on
carvedilol 25mg bid, enalapril 5mg bid and eplerenone
25mg daily. Creatinine is 1.2mg/dL, potassium 4.1. What
is the most appropriate step to reduce cardiovascular
death and HF hospitalization?
A. Transition from eplerenone to spironolactone 25mg daily
B. Start digoxin 0.125mg every other day
C. Initiate ivabridine 5mg bid
D. Stop enalapril and start sacubitril-valsartan 24-26mg bid no earlier
than 36 hours after stopping enalapril
E. Add losartan 25mg daily

Answer #5
A 64 year old woman with chronic systolic heart failure, EF
25%, in NYHA Class 2 and 3 prior HF hospitalizations
presents for a routine visit. She is euvolemic with heart
rate is 60bpm in sinus rhythm, BP 110/62. She is on
carvedilol 25mg bid, enalapril 5mg bid and eplerenone
25mg daily. Creatinine is 1.2mg/dL, potassium 4.1. What
is the most appropriate step to reduce cardiovascular
death and HF hospitalization?
A. Transition from eplerenone to spironolactone 25mg daily
B. Start digoxin 0.125mg every other day
C. Initiate ivabridine 5mg bid
D. Stop enalapril and start sacubitril-valsartan 24-26mg bid no
earlier than 36 hours after stopping enalapril
E. Add losartan 25mg daily

1353
Copyright © Harvard Medical School, 2018. All Rights Reserved.

PARADIGM-HF: CV Death or HF
Hospitalization (Primary Endpoint)

McMurray JJ et al. N Engl J Med. 2014;371:993-1004.

2016 HF Guideline Update


COR LOE Recommendation
I B-R ACEI or ARB or ARNI in conjunction with β blockers + MRA (where
appropriate) is recommended for patients with chronic HFrEF to
reduce morbidity and mortality
I B-R In patients with chronic, symptomatic HFrEF NYHA class II or III who
tolerate and ACEI or ARB, replacement by an ARNI is recommended to
further reduce morbidity and mortality
III B-R ARNI should NOT be administered concomitantly with ACEI or within
36 hours of last ACEI dose
III C-EO ARNI should NOT be administered to patients with a history of
angioedema

1. Yancy CW et al. J Am Coll Cardiol. 2016;68:1476-1488.

1354
Copyright © Harvard Medical School, 2018. All Rights Reserved.

Question #6
A previously healthy 38 year-old woman presents to the
emergency room with lightheadedness, palpitations and
generalized fatigue. She just returned from a 2-week
vacation to Martha’s Vineyard and noticed a rash on her left
thigh. ECG is performed:

Question #6 (cont)
Each of the following statements regarding this patient’s
condition is true EXCEPT:

A. This is a common cause of cardiomyopathy


B. Steroids may improve symptoms
C. AV block often requires temporary cardiac pacing
D. Pericarditis is common and responds to indomethicin
E. Myocarditis may occur in 10% of patients

1355
Copyright © Harvard Medical School, 2018. All Rights Reserved.

Answer #6
Each of the following statements regarding this patient’s
condition is true EXCEPT:

A. This is a common cause of cardiomyopathy


B. Steroids may improve symptoms
C. AV block often requires temporary cardiac pacing
D. Pericarditis is common and responds to indomethicin
E. Myocarditis may occur in 10% of patients

Lyme and the Heart

Krause PJ et al. Circulation 2013;127:e451

1356
Copyright © Harvard Medical School, 2018. All Rights Reserved.

Question #7
A 33-year old female presents to the emergency room at 25-
weeks gestation with progressive dyspnea (now at rest),
orthopnea, and fatigue. She has no prior medical problems.
Physical exam is notable for opening snap after S2 with a I/IV
diastolic murmur heard best in left lateral decubitus position.
HR is 108 beats/min and BP is 130/84 mm Hg. She receives
oxygen, IV furosemide, and diltiazem. Echocardiogram
demonstrates mitral valve area of 0.5 cm2 and normal left
ventricular ejection fraction. Which is the best approach for
management?
A. Start carvedilol
B. Start lisinopril
C. Balloon valvuloplasty
D. Immediate Cesarean section
E. Start digoxin

Answer #7
A 33-year old female presents to the emergency room at 25-
weeks gestation with progressive dyspnea (now at rest),
orthopnea, and fatigue. She has no prior medical problems.
Physical exam is notable for opening snap after S2 with a I/IV
diastolic murmur heard best in left lateral decubitus position.
HR is 108 beats/min and BP is 130/84 mm Hg. She receives
oxygen, IV furosemide, and diltiazem. Echocardiogram
demonstrates mitral valve area of 0.5 cm2 and normal left
ventricular ejection fraction. Which is the best approach for
management?
A. Start carvedilol
B. Start lisinopril
C. Balloon valvuloplasty
D. Immediate Cesarean section
E. Start digoxin

1357
Copyright © Harvard Medical School, 2018. All Rights Reserved.

Mitral stenosis
Left-sided symptoms
Hemoptysis, right-heart failure, hoarseness
10-20 years post-rheumatic fever
Echocardiogram useful to determine therapies
Exclude atrial myxoma

Mitral stenosis in pregnancy


• NYHA Class III and IV—Treat with
percutaneous valvotomy before delivery
• Judicious use of diuretics and beta-
blockers for milder forms of MS
• Propranolol (classic) vs. cardio-selective
(e.g., atenolol, metoprolol)
• Beware of bradycardia and hypoglycemia
in newborn
• Avoid ACE-inhibitors

Maisano F. Eur Heart J 2014; 35: 1575-7.

1358
Copyright © Harvard Medical School, 2018. All Rights Reserved.

Question #8
A 48-year old man with history of hypertension presents to
emergency room with acute chest pain radiating to back with
hypotension and a new, large left effusion on chest x-ray.
Systolic blood pressure differs in both arms by 22 mm Hg.
Appropriate steps in management include:

A. Treat with sodium nitroprusside alone


B. Start a loop diuretic alone
C. Refer for urgent transthoracic echocardiogram
D. Refer for urgent surgical repair for proximal dissection
E. Narcotics for pain relief alone

Answer #8
A 48-year old man with history of hypertension presents to
emergency room with acute chest pain radiating to back with
hypotension and a new, large left effusion on chest x-ray.
Systolic blood pressure differs in both arms by 22 mm Hg.
Appropriate steps in management include:
A. Treat with sodium nitroprusside alone
B. Start a loop diuretic alone
C. Refer for urgent transthoracic echocardiogram
D. Refer for urgent surgical repair for proximal dissection
E. Narcotics for pain relief alone

1359
Copyright © Harvard Medical School, 2018. All Rights Reserved.

Aortic Dissection

European Heart Journal 2014; 35:2873-2926

Acute Management of Aortic Dissection


• Stabilization and pain relief
• Combination of sodium nitroprusside and beta-
blocker to reduce BP and wall stress
• Immediate surgery for life threatening
complications
• Urgent confirmation of dissection
• Distal dissections are often in older patients and
associated with CAD

1360
Copyright © Harvard Medical School, 2018. All Rights Reserved.

Question #9
A 58 year old African-American man presents to primary care
office for routine physical examination. He expressed
concern about his cardiovascular risk due to his strong family
history of coronary artery disease. He has no medical
problems, exercises once weekly, and review of systems is
negative for any symptoms. BP is 142/78, HR 75, BMI 25.8.
Exam unremarkable. Lipid profile: Cholesterol 211, HDL 49,
LDL 122, Triglycerides 144. He denies smoking or illicit drug
use. Which is the incorrect statement?
A. Cardiovascular death rates increase as the patient ages
B. Cardiovascular death rates have decreased recently
C. Routine aerobic exercise may reduce his risk for MI
D. Death from cardiovascular disease peaked in the 1970s
E. Black race is not associated with increased stroke risk

Answer #9
A 58 year old African-American man presents to primary care
office for routine physical examination. He expressed
concern about his cardiovascular risk due to his strong family
history of coronary artery disease. He has no medical
problems, exercises once weekly, and review of systems is
negative for any symptoms. BP is 142/78, HR 75, BMI 25.8.
Exam unremarkable. Lipid profile: Cholesterol 211, HDL 49,
LDL 122, Triglycerides 144. He denies smoking or illicit drug
use. Which is the incorrect statement?
A. Cardiovascular death rates increase as the patient ages
B. Cardiovascular death rates have decreased recently
C. Routine aerobic exercise may reduce his risk for MI
D. Death from cardiovascular disease peaked in the 1970s
E. Black race is not associated with increased stroke risk

1361
Copyright © Harvard Medical School, 2018. All Rights Reserved.

Answer #9
• Plateau of cardiovascular death in
1970s CVD=CHD, HF, stroke, HTN
100
• CVD deaths decreased over past 90
decade 80
70

% of population
60
• HTN affects ~74 million in US
50
40
• MI or Fatal coronary heart disease 30
(CHD) affects 30,000 men and 20
10,000 women between ages 35-44 10
(0.8% of population) 0
20-39 40-59 60-79 80+
Age (years)
• Stroke hospitalization highest
Men Women
among blacks (4.0% vs 2.4% whites)
and 2-fold risk of 1st stroke
AHA 2013 Statistics

Question #10
A 21 year old male with a history of bicuspid aortic valve
presents to office with significant hypertension (192/102).
ECG demonstrates LVH. Brachial-femoral delay in noted
on pulse exam. In addition to blood pressure assessments
in arms and legs, what diagnostic tools should be
considered as routine parts of the evaluation?
A. Chest x-ray
B. Echocardiogram
C. MRI/MRA
D. All of the above
E. None of the above

1362
Copyright © Harvard Medical School, 2018. All Rights Reserved.

Answer #10
A 21 year old male with a history of bicuspid aortic valve
presents to office with significant hypertension (192/102).
ECG demonstrates LVH. Brachial-femoral delay in noted
on pulse exam. In addition to blood pressure assessments
in arms and legs, what diagnostic tools should be
considered as routine parts of the evaluation?
A. Chest x-ray
B. Echocardiogram
C. MRI/MRA
D. All of the above
E. None of the above

Coarctation of Aorta

Brickner ME et al. N Engl J Med 2000;342:256-263.

1363
Copyright © Harvard Medical School, 2018. All Rights Reserved.

Coarctation of Aorta
– Associated with BAV, subaortic stenosis, circle of Willis aneursym, and
VSD
– Recognized risk in hypertension for young patients
– HTN in upper arms (especially right) compared with legs
– Death related to CHF, aortic rupture, endocarditis, MI, IC bleed

Warnes et al. Circulation 2008;118;e714-e833

Question #11
A 49 year old female with heart failure and ventricular
tachycardia presents with a new pericardial friction rub
and anti-histone antibodies. Which drug is most
associated with drug-induced lupus?

A. Verapamil
B. Hydralazine
C. Amiodarone
D. Lidocaine
E. None of the above

1364
Copyright © Harvard Medical School, 2018. All Rights Reserved.

Answer #11
A 49 year old female with heart failure and ventricular
tachycardia presents with a new pericardial friction rub
and anti-histone antibodies. Which drug is most
associated with drug-induced lupus?

A. Verapamil
B. Hydralazine
C. Amiodarone
D. Lidocaine
E. None of the above

Side Effect Profiles


Verapamil Hydralazine
-Constipation -Drug-induced SLE
-Skin reaction -Dizziness
-Gingival hyperplasia
Lidocaine
Perioral numbness
Diplopia Amiodarone
Seizures -Photosensitivity
Cholestatic jaundice -Lung toxicity
Hyperacusis -Hepatitis
Slurred speech -Thyroid abnormalities

1365
Copyright © Harvard Medical School, 2018. All Rights Reserved.

Question #12
A 69-year old man returns for a follow-up visit after an anterior
myocardial infarction 2 months ago complicated by heart
failure. On maximal medical therapy, he currently has no
symptoms of heart failure. A follow-up echocardiogram reveals
an improved left ventricular ejection fraction from 20% at the
time of MI to 33% today. What would you do next?
A. Consider ICD only if he becomes symptomatic (NYHA 2 or 3)
B. Refer for an implantable cardioverter-defibrillator (ICD)
C. Repeat echocardiogram in another month to see if LVEF improves
prior to implanting ICD
D. Consider cardiac MRI to assess LVEF
E. Educate patient about the contraindication of ICD due to his age

Answer #12
A 69-year old man returns for a follow-up visit after an anterior
myocardial infarction 2 months ago complicated by heart
failure. On maximal medical therapy, he currently has no
symptoms of heart failure. A follow-up echocardiogram reveals
an improved left ventricular ejection fraction from 20% at the
time of MI to 33% today. What would you do next?
A. Consider ICD only if he becomes symptomatic (NYHA 2 or 3)
B. Refer for an implantable cardioverter-defibrillator (ICD)
C. Repeat echocardiogram in another month to see if LVEF improves
prior to implanting ICD
D. Consider cardiac MRI to assess LVEF
E. Educate patient about the contraindication of ICD due to his age

1366
Copyright © Harvard Medical School, 2018. All Rights Reserved.

Epstein et al. Circulation 2008:117:2820-40

Question #13
A 63-year old man has recurrent non-sustained ventricular
tachycardia (up to 9-beats) in the setting of an acute
myocardial infarction. Mild signs of heart failure are present
and successful stent is placed in left anterior descending
artery. Ejection fraction is 25%. Which statement is true?
A. ACE-inhibitors do not improve outcomes in this population
B. Implantable defibrillator should be performed prior to discharge
C. Lidocaine should be started at this time
D. Beta blockers may improve survival and should be started once
congestion is resolved and prior to discharge
E. Eplerenone is not associated with improved survival

1367
Copyright © Harvard Medical School, 2018. All Rights Reserved.

Answer #13
A 63-year old man has recurrent non-sustained ventricular
tachycardia (up to 9-beats) in the setting of an acute
myocardial infarction. Mild signs of heart failure are present
and successful stent is placed in left anterior descending
artery. Ejection fraction is 25%. Which statement is true?
A. ACE-inhibitors do not improve outcomes in this population
B. Implantable defibrillator should be performed prior to discharge
C. Lidocaine should be started at this time
D. Beta blockers may improve survival and should be started
once congestion is resolved and prior to discharge
E. Eplerenone is not associated with improved survival

CAPRICORN
Carvedilol on outcome after myocardial infarction in patients
with LVEF<40% (n=1959)

Death from all causes


23 % risk reduction, p = 0.031
Post-MI Management
• ACE inhibitors improve outcomes
(SAVE, AIRE, TRACE)

• EPHESUS: 30% risk reduction in


mortality with eplerenone

• ICD should not be used within 40 days


post MI +/- revascularization

• Excess risk with type 1 anti-arrhythmic


agents

Lancet 2001;357:1385-1390

1368
Copyright © Harvard Medical School, 2018. All Rights Reserved.

Question #14
An 35-year old woman presents to the emergency room
following a motor vehicle accident in which she was wearing a
seatbelt. She noted mild dizziness while driving and awakens
finding herself on the side of the road. Past medical history is
notable for heart failure secondary to sarcoidosis. She takes
lisinopril 5 mg daily, metoprolol succinate 25 mg daily, and
prednisone 5 mg daily. Physical exam is remarkable for mild
facial lacerations. What is the most likely cause of this event?
A. Hysterical fainting
B. Epilepsy
C. Sinus bradycardia
D. Neurocardiogenic syncope
E. Ventricular tachycardia

Answer #14
An 35-year old woman presents to the emergency room
following a motor vehicle accident in which she was wearing a
seatbelt. She noted mild dizziness while driving and awakens
finding herself on the side of the road. Past medical history is
notable for heart failure secondary to sarcoidosis. She takes
lisinopril 5 mg daily, metoprolol succinate 25 mg daily, and
prednisone 5 mg daily. Physical exam is remarkable for mild
facial lacerations. What is the most likely cause of this event?
A. Hysterical fainting
B. Epilepsy
C. Sinus bradycardia
D. Neurocardiogenic syncope
E. Ventricular tachycardia

1369
Copyright © Harvard Medical School, 2018. All Rights Reserved.

History is the key for syncope


Event Classic scenario

Cardiac syncope - Rapid onset without aura; clear sensorium


- History of CAD, LV dysfunction

Bradyarrhythmias - History of conduction disease, heart transplant

Neurological syncope - Preceded by aura; clouded sensorium


- Incontinence, tongue biting, seizure activity

Hysterical fainting - Not accompanied by change in pulse, blood


pressure, or skin color
- Paresthesias of hands/face, hyperventilation
- Dyspnea, signs of anxiety

Event Classic scenario

Neurocardiogenic - 50% of syncopal episodes


- Precipitants: emotional distress, pain,
fatigue, fear, decreased venous return

Catecholamine state

Sympathetic stimulation of heart

Stimulates cardiac mechanoreceptors


(vagal afferent C fibers)

Vasodilatation and bradycardia

1370
Copyright © Harvard Medical School, 2018. All Rights Reserved.

Question #15
Which of the following is FALSE related to patients
undergoing percutaneous coronary intervention?

A. In patients with CKD (creatinine clearance<60 mL/min), the


volume of contrast media should be minimized
B. Administration of N-acetyl-L-cysteine is useful to prevent
contrast-induced acute kidney injury
C. Patients with prior reaction to contrast can proceed with
catheterization with appropriate prophylaxis
D. Administration of high-dose statin is reasonable before PCI
to reduce risk of peri-procedural MI
E. Patients not on aspirin should be given non-enteric aspirin
325 mg before PCI

Answer #15
Which of the following is FALSE related to patients
undergoing percutaneous coronary intervention?

A. In patients with CKD (creatinine clearance<60 mL/min), the


volume of contrast media should be minimized
B. Administration of N-acetyl-L-cysteine is useful to prevent
contrast-induced acute kidney injury
C. Patients with prior reaction to contrast can proceed with
catheterization with appropriate prophylaxis
D. Administration of high-dose statin is reasonable before PCI
to reduce risk of peri-procedural MI
E. Patients not on aspirin should be given non-enteric aspirin
325 mg before PCI

1371
Copyright © Harvard Medical School, 2018. All Rights Reserved.

-N=2308
-2 doses before and after
procedure
-Contrast-induced AKI (13.8% vs.
14.7% control, p=0.64)

Question #16
A 39 year old woman who is 30-weeks pregnant and
presents to emergency room with mild dyspnea on exertion.
Examination reveals BP 110/70 mm Hg, HR 95 beats/minute,
clear chest, jugular venous pressure 7 cm water, and III/VI
holosystolic murmur at apex. Echocardiography confirms
moderate mitral regurgitation. Estimated PA pressure is 30+
right atrial pressure. Management option includes which of
the following?
A. Start low-dose furosemide
B. Arrange for emergent delivery of fetus
C. Plan mitral valve repair prior to delivery
D. Left and right heart catheterization
E. Proceed with pregnancy with close follow-up

1372
Copyright © Harvard Medical School, 2018. All Rights Reserved.

Answer #16
A 39 year old woman who is 30-weeks pregnant and
presents to emergency room with mild dyspnea on exertion.
Examination reveals BP 110/70 mm Hg, HR 95 beats/minute,
clear chest, jugular venous pressure 7 cm water, and III/VI
holosystolic murmur at apex. Echocardiography confirms
moderate mitral regurgitation. Estimated PA pressure is 30+
right atrial pressure. Management option includes which of
the following?
A. Start low-dose furosemide
B. Arrange for emergent delivery of fetus
C. Plan mitral valve repair prior to delivery
D. Left and right heart catheterization
E. Proceed with pregnancy with close follow-up

Valvular Heart Lesions Associated With High


Maternal and/or Fetal Risk During Pregnancy

1. Severe AS with or without symptoms


2. AI with NYHA functional Class III–IV symptoms
3. MS with NYHA functional Class II–IV symptoms
4. MR with NYHA functional Class III–IV symptoms
5. Aortic and/or mitral valve disease resulting in severe pulmonary
hypertension (pulmonary pressure >75% of systemic
pressures)
6. Aortic and/or mitral valve disease with severe LV dysfunction
(EF<40%)
7. Mechanical prosthetic valve requiring anticoagulation
8. AI in Marfan’s syndrome

Nishimura et al. J Am Coll Cardiol. 2014;63(22):e57-e185

1373
Copyright © Harvard Medical School, 2018. All Rights Reserved.

Question #17
A 39-year old woman presents to the office with newly
diagnosed hypertension (BP 151/92). Lifestyle
modifications, as recommended by the JNC VII
guidelines, can produce blood pressure reductions
which are equivalent to:

A. One antihypertensive agent


B. Two antihypertensive agents
C. Three antihypertensive agents
D. None of the above

Answer #17
A 39-year old woman presents to the office with newly
diagnosed hypertension (BP 151/92). Lifestyle
modifications, as recommended by the JNC VII
guidelines, can produce blood pressure reductions
which are equivalent to:

A. One antihypertensive agent


B. Two antihypertensive agents
C. Three antihypertensive agents
D. None of the above

1374
Copyright © Harvard Medical School, 2018. All Rights Reserved.

Adapted from Chobanian et al. JAMA 2003;289: 2560-2572 and JNC 7

JAMA. 2014;311(5):507-520.

1375
Copyright © Harvard Medical School, 2018. All Rights Reserved.

Question #18
A 38-year-old female attorney has experienced palpitations over
past 2 years. The episodes are not associated with dizziness,
syncope, or diaphoresis. She is otherwise healthy and
examination is completely normal with an exception of heart rate
of 108 beats/minute which increases to 128 beats/minute with
standing and when she presents to an audience. An
electrocardiogram reveals sinus tachycardia and is otherwise
normal. Stress reduction and caffeine reduction did not change
symptom burden. What is the best treatment for her
palpitations?
A. Dofetilide
B. Nadolol
C. Radiofrequency Ablation
D. Amiodarone
E. Nifedipine

Answer #18
A 38-year-old female attorney has experienced palpitations over
past 2 years. The episodes are not associated with dizziness,
syncope, or diaphoresis. She is otherwise healthy and
examination is completely normal with an exception of heart rate
of 108 beats/minute which increases to 128 beats/minute with
standing and when she presents to an audience. An
electrocardiogram reveals sinus tachycardia and is otherwise
normal. Stress reduction and caffeine reduction did not change
symptom burden. What is the best treatment for her
palpitations?
A. Dofetilide
B. Nadolol
C. Radiofrequency Ablation
D. Amiodarone
E. Nifedipine

1376
Copyright © Harvard Medical School, 2018. All Rights Reserved.

Interventions and Indications


Amiodarone Dofetilide

-Paroxysmal atrial fib/flutter -Blockade of the cardiac ion


channel carrying the rapid
-PSVT component of the delayed
-Sustained VT rectifier potassium currents
-Atrial fib/flutter

Beta-adrenergic blocker Radiofrequency Ablation


-Sinus tachycardia -Treatment for atrial fib/flutter
-Palpitations -Pulmonary vein isolation
-Various CV conditions -Atrial tachycardia

Question #19
A 72-year old man with a history of severe mitral regurgitation
from anterior leaflet prolapse with a LVEF of 50% with LVEDD
59 mm presents for a second opinion. Dyspnea occurs when
walking up one flight of stairs and is otherwise healthy. Which
is the best option for management?
A. Refer for mitral valve surgery
B. Refer for percutaneous MitraClip
C. Follow serial echocardiograms and plan surgery when LVEF
decreases below 35%
D. Start ACE-inhibitor to attenuate the progression of LV dilatation
E. Refer for cardiac resynchronization to reduce mitral regurgitation

1377
Copyright © Harvard Medical School, 2018. All Rights Reserved.

Answer #19
A 72-year old man with a history of severe mitral regurgitation
from anterior leaflet prolapse with a LVEF of 50% with LVEDD
59 mm presents for a second opinion. Dyspnea occurs when
walking up one flight of stairs and is otherwise healthy. Which
is the best option for management?
A. Refer for mitral valve surgery
B. Refer for percutaneous MitraClip
C. Follow serial echocardiograms and plan surgery when LVEF
decreases below 35%
D. Start ACE-inhibitor to attenuate the progression of LV dilatation
E. Refer for cardiac resynchronization to reduce mitral regurgitation

Recommendations for Mitral Valve Surgery in


Severe Primary Mitral Regurgitation

J Am Coll Cardiol. 2014;63(22):e57-e185

1378
Copyright © Harvard Medical School, 2018. All Rights Reserved.

Question #20
Please review the following ECG:

Question #20
Each of the following statements about this ECG finding
is true EXCEPT:

A. Exercise stress testing should include imaging


B. This can represent end-organ damage from
systemic hypertension
C. Repolarization will normalize with exercise
D. Can be seen with both eccentric and concentric
ventricular remodeling
E. This ECG is diagnostic for left ventricular
hypertrophy

1379
Copyright © Harvard Medical School, 2018. All Rights Reserved.

Answer #20
Each of the following statements about this ECG finding
is true EXCEPT:

A. Exercise stress testing should include imaging


B. This can represent end-organ damage from
systemic hypertension
C. Repolarization will normalize with exercise
D. Can be seen with both eccentric and concentric
ventricular remodeling
E. This ECG is diagnostic for left ventricular
hypertrophy

Question #21
A 53-year old woman who has not seen a doctor in 10 years
presents to clinic for a routine visit given vague chest pain and
fatigue. She takes no medicines and denies allergies or past
medical history. She noticed her BP was 170/100 at a drug store 4
weeks ago prompting the visit. She has tried decreasing salt
intake. Exam demonstrates BP 166/106, HR 76, AV nicking and
loud S2. JVP is 7 (normal). Routine labs and ECG are normal.
She has uncontrolled diabetes, proteinuria, and LVEF is 40%.
While arranging for catheterization, which is the best next option?

A. Start enalapril
B. Further lifestyle change and recheck BP in 4-6
weeks
C. Start hydrochlorothiazide
D. Refer for implantable defibrillator
E. Start diltiazem

1380
Copyright © Harvard Medical School, 2018. All Rights Reserved.

Answer #21
A 53-year old woman who has not seen a doctor in 10 years
presents to clinic for a routine visit given vague chest pain and
fatigue. She takes no medicines and denies allergies or past
medical history. She noticed her BP was 170/100 at a drug store 4
weeks ago prompting the visit. She has tried decreasing salt
intake. Exam demonstrates BP 166/106, HR 76, AV nicking and
loud S2. JVP is 7 (normal). Routine labs and ECG are normal.
She has uncontrolled diabetes, proteinuria, and LVEF is 40%.
While arranging for catheterization, which is the best next option?

A. Start enalapril
B. Further lifestyle change and recheck BP in 4-6
weeks
C. Start hydrochlorothiazide
D. Refer for implantable defibrillator
E. Start diltiazem

Hypertension Treatment
• First line therapy is often a diuretic
• Tailor choice of drug to patient population
ACE-inhibitor: Diabetes, CHF, MI, proteinuria, and low LVEF
Beta-blockers: CAD
Long acting drugs and combination drugs for non-compliant patients

• Multiple drugs are often required


• Screen for secondary causes of hypertension

1381
Copyright © Harvard Medical School, 2018. All Rights Reserved.

Question #22
A 69-year old woman with history of cardiac transplantation
and normal appearing heart on recent echocardiogram
presents for routine office visit. She is asymptomatic and
has a past medical history of controlled hypertension,
diabetes, and dyslipidemia without history of endocarditis.
She is scheduled for a dental extraction next week. What
is the best recommendation for SBE prophylaxis?

A. Amoxicillin 2 grams orally 1 hour before procedure


B. Clindamycin 600 mg orally 1 hour before procedure
C. Ampicillin 2 grams IV 30 minutes before procedure
D. All are acceptable options
E. None of the above

Answer #22
A 69-year old woman with history of cardiac transplantation
and normal appearing heart on recent echocardiogram
presents for routine office visit. She is asymptomatic and
has a past medical history of controlled hypertension,
diabetes, and dyslipidemia without history of endocarditis.
She is scheduled for a dental extraction next week. What
is the best recommendation for SBE prophylaxis?
A. Amoxicillin 2 grams orally 1 hour before procedure
B. Clindamycin 600 mg orally 1 hour before procedure
C. Ampicillin 2 grams IV 30 minutes before procedure
D. All are acceptable options
E. None of the above

1382
Copyright © Harvard Medical School, 2018. All Rights Reserved.

Endocarditis Prophylaxis
Endocarditis Prophylaxis No longer recommended:
Class IIa – Acquired valvular dysfunction (RHD)
– Prosthetic cardiac valves – Hypertrophic cardiomyopathy
– Previous bacterial endocarditis – MVP with valvular
– Unrepaired cyanotic congenital heart regurgitation/thickened leaflets
disease
– Completely repaired cyanotic
congenital heart disease using
prosthetic material or within 6 months
of catheter intervention
– Repaired CHD with residual defect or
adjacent to site of a prosthetic patch
– Cardiac transplantation with valve
regurgitation due to structurally
abnormal valve

Nishimura et al. ACC/AHA Guidelines Circulation 118:887 (2008)

Question #23
A 56 year old male presents for a new visit for
management of his chronic heart failure with LVEF of 30%
and NYHA Class III functional class. Medications include
lisinopril 40mg once daily, carvedilol 25 mg twice daily, and
furosemide 40 mg daily. Which of the following statements
regarding therapy to improve survival is correct?
A. Hydralazine plus isosorbide dinitrate improves survival in self-described African-
American heart failure patients who are taking ACE-inhibitors and beta-blockers
B. Aldosterone-receptor antagonist reduces all-cause mortality in patients with NYHA
Class III or IV heart failure, MI patients with heart failure, and older patients with
milder forms of heart failure following hospitalization
C. Cardiac resynchronization therapy improves survival in patients with mild heart
failure patients (NYHA Class I/II) with left bundle branch block and QRS of 160
milliseconds
D. Angiotensin II receptor blockers in addition to ACE-inhibitors is associated with an
improvement in cardiovascular survival
E. All of the above are correct

1383
Copyright © Harvard Medical School, 2018. All Rights Reserved.

Answer #23
A 56 year old male presents for a new visit for
management of his chronic heart failure with LVEF of 30%
and NYHA Class III functional class. Medications include
lisinopril 40mg once daily, carvedilol 25 mg twice daily, and
furosemide 40 mg daily. Which of the following statements
regarding therapy to improve survival is correct?
A. Hydralazine plus isosorbide dinitrate improves survival in self-described African-
American heart failure patients who are taking ACE-inhibitors and beta-blockers
B. Aldosterone-receptor antagonist reduces all-cause mortality in patients with NYHA
Class III or IV heart failure, MI patients with heart failure, and older patients with
milder forms of heart failure following hospitalization
C. Cardiac resynchronization therapy improves survival in patients with mild heart
failure patients (NYHA Class I/II) with left bundle branch block and QRS of 160
milliseconds
D. Angiotensin II receptor blockers in addition to ACE-inhibitors is associated with an
improvement in cardiovascular survival
E. All of the above are correct

A-HeFT: Overall Survival


100
43% Decrease in Mortality

Fixed-dose HYD/ISDN
Survival (%)

95

90
Placebo

Hazard ratio=0.57
P=.01
85
0 100 200 300 400 500 600
Days Since Baseline Visit Date
HYD/ISDN 518 463 407 359 313 251 13
Placebo 532 466 401 340 285 232 24

Adapted from Taylor AL, et al. N Engl J Med. 2004;351:2052.

1384
Copyright © Harvard Medical School, 2018. All Rights Reserved.

EMPHASIS-HF in “Mild HF”


Event Eplerenone (%) Placebo(%) Adjusted HR p-value
CV Death/HF Hospitalization 18.3 25.9 0.63 (0.54-0.74) <0.001
CV Death 10.8 13.5 0.76 (0.61-0.94) 0.01
HF Hospitalization 12.0 18.4 0.58 (0.47-0.70) <0.001
Hyperkalemia Hospitalization 0.3 0.2 1.15 (0.25-5.31) 0.85

N Engl J Med. 2010 Nov 14.

CHARM-Alternative: Primary outcome CV death or


CHF hospitalization
%
50
406 (40.0%)
Placebo
40
334 (33.0%)
30
Candesartan
20

10 HR 0.77 (95% CI 0.67-0.89), p=0.0004


Adjusted HR 0.70, p<0.0001
0
Number at risk
0 1 2 3 3.5 years
Candesartan 1013 929 831 434 122
Placebo 1015 887 798 427 126

1385
Copyright © Harvard Medical School, 2018. All Rights Reserved.

Question #24
You are asked to see a 34-year-old woman with history of dental
abscess and IV drug use who has been admitted to the hospital
with fevers, chills, and dyspnea for 5 days. Temperature is 102.3.
Heart rate is 120 beats/minute, and blood pressure is 89/72 mm
Hg. Jugular venous pressure is 14 cm. Rales are heard
bilaterally. Cardiac examination reveals a soft S1 and a grade
II/IV early diastolic decrescendo murmur that is loudest at the
upper left sternal border; the apical impulse is normal and
extremities are cool. Echocardiogram reveals severe aortic
insufficiency with evidence of vegetation. Blood cultures are
pending.

Question #24
Which of the following is the best next step for this
patient?
A. Attempt to stabilize the patient with intra-aortic balloon counterpulsation
B. Perform emergency cardiac catheterization, and then refer for emergency
aortic valve replacement
C. Start IV dobutamine, IV furosemide, and IV antibiotics, and refer for
emergency aortic valve replacement
D. Start IV dobutamine, IV furosemide, and IV antibiotics and consider aortic
valve replacement if blood cultures do not clear with antibiotics in 2 weeks
E. Perform transesophageal echocardiogram to determine if surgery is
required

1386
Copyright © Harvard Medical School, 2018. All Rights Reserved.

Answer #24
Which of the following is the best next step for this
patient?
A. Attempt to stabilize the patient with intra-aortic balloon counterpulsation
B. Perform emergency cardiac catheterization, and then refer for emergency
aortic valve replacement
C. Start IV dobutamine, IV furosemide, and IV antibiotics, and refer for
emergency aortic valve replacement
D. Start IV dobutamine, IV furosemide, and IV antibiotics and consider aortic
valve replacement if blood cultures do not clear with antibiotics in 2 weeks
E. Perform transesophageal echocardiogram to determine if surgery is
required

Baddour et al. Circulation 2005;111:e394-434.

1387
Copyright © Harvard Medical School, 2018. All Rights Reserved.

Definition of Infective Endocarditis “Modified Duke Criteria”

Question #25
A 52-year old male presents to clinic for perioperative
evaluation prior to elective cholecystectomy. He works full
time in construction and exercises by walking ~40 minutes
several times per week. His past medical history is notable for
hypertension, hypercholesterolemia, and a myocardial
infarction 3 years ago. What is the best management?

A. Coronary angiography prior to surgery


B. Perform cholecystectomy only if emergency
C. Stress test with MIBI or echocardiogram
D. Proceed with surgery and ensure beta-blocker use
E. Stress test without imaging

1388
Copyright © Harvard Medical School, 2018. All Rights Reserved.

Answer #25
A 52-year old male presents to clinic for perioperative
evaluation prior to elective cholecystectomy. He works full
time in construction and exercises by walking ~40 minutes
several times per week. His past medical history is notable for
hypertension, hypercholesterolemia, and a myocardial
infarction 3 years ago. What is the best management?

A. Coronary angiography prior to surgery


B. Perform cholecystectomy only if emergency
C. Stress test with MIBI or echocardiogram
D. Proceed with surgery and ensure beta-blocker use
E. Stress test without imaging

Goldman Criteria
Estimates of Metabolic equivalents (METs)
Can take care of self, such as eat, dress, or use the
toilet (1 MET).

Can walk up a flight of steps or a hill (4 METs).

Can do heavy work around the house such as


scrubbing floors or lifting or moving heavy furniture
(between 4 and 10 METs).

Can participate in strenuous sports such as


swimming, singles tennis, football, basketball, and
skiing (>10 METs).

≥4 METs Very Reassuring for


Low Perioperative CAD Risk

Goldman L et al. NEJM 1977

1389
Copyright © Harvard Medical School, 2018. All Rights Reserved.

AHA/ACC Guidelines 2014


Major predictors requiring intensive management and may lead to
delay/cancellation of surgery
• Unstable coronary syndromes including unstable or severe angina or recent MI

• Decompensated HF including NYHA functional class IV or worsening or new-onset HF

• Significant arrhythmias including high grade AV block, symptomatic ventricular


arrhythmias, supraventricular arrhythmias with ventricular rate >100 bpm at rest, symptomatic
bradycardia, and newly recognized ventricular tachycardia

• Severe heart valve disease including severe aortic stenosis or symptomatic mitral stenosis
Other clinical predictors that warrant careful assessment of current status
• History of ischemic heart disease
• History of cerebrovascular disease
• History of compensated heart failure or prior heart failure
• Diabetes mellitus
• Renal insufficiency

Fleisher et al. JACC 2014; 62:e77-e137.

Question #26
A 31-year old woman with history of mechanical mitral
valve replacement, on warfarin 7.5mg daily, who is 10
weeks pregnant presents to your office for consultation
regarding management of her anticoagulation. Which
situation may be considered for the use of low
molecular weight heparin?
A. When administered concomitantly with aspirin or dipyridamole
B. When administered twice daily at a dose adjusted for anti-Xa levels
C. When administered daily at a dose adjusted for activated partial
thromboplastin time
D. When unfractionated heparin has caused heparin-induced thrombocytopenia

1390
Copyright © Harvard Medical School, 2018. All Rights Reserved.

Answer #26
A 31-year old woman with history of mechanical mitral
valve replacement, on warfarin 7.5mg daily, who is 10
weeks pregnant presents to your office for consultation
regarding management of her anticoagulation. Which
situation may be considered for the use of low
molecular weight heparin?
A. When administered concomitantly with aspirin or dipyridamole
B. When administered twice daily at a dose adjusted for anti-Xa levels
C. When administered daily at a dose adjusted for activated partial
thromboplastin time
D. When unfractionated heparin has caused heparin-induced thrombocytopenia

Anticoagulation for Mechanical Valves During


Pregnancy

Nishimura et al. J Am Coll Cardiol. 2014;63(22):e57-e185

1391
Copyright © Harvard Medical School, 2018. All Rights Reserved.

Question #27
A 46 year old man from Vermont presents to the hospital with
an aborted cardiac arrest due to ventricular tachycardia. He
has a history of pacemaker placed at age 40 for complete
heart block. Echocardiogram revealed EF of 45% with
inferoseptal hypokinesis. There are normal epicardial
coronary arteries on angiography. Cardiac PET scan reveals
metabolic uptake in the septum and inferior wall with perfusion
defect. What is the most likely diagnosis?
A. Arrhythmogenic right ventricular cardiomyopathy
B. Cardiac sarcoidosis
C. Lyme disease
D. Coronary vasospasm
E. Chagas disease

Answer #27
A 46 year old man from Vermont presents to the hospital with
an aborted cardiac arrest due to ventricular tachycardia. He
has a history of pacemaker placed at age 40 for complete
heart block. Echocardiogram revealed EF of 45% with
inferoseptal hypokinesis. There are normal epicardial
coronary arteries on angiography. Cardiac PET scan reveals
metabolic uptake in the septum and inferior wall with perfusion
defect. What is the most likely diagnosis?
A. Arrhythmogenic right ventricular cardiomyopathy
B. Cardiac sarcoidosis
C. Lyme disease
D. Coronary vasospasm
E. Chagas disease

1392
Copyright © Harvard Medical School, 2018. All Rights Reserved.

Cardiac Sarcoidosis
Noncaseating Granuloma

Major Cardiac Sequelae


1. AV Block
2. Ventricular Arrhythmias
3. Heart Failure

Tadamura F et al. Circulation 2006; 113:e771-3.

Question #28
A 73-year old man with a 25 pack year history of smoking
(stopped 6 years ago), hypertension, and diabetes mellitus
presents to your office for an initial visit. Fasting lipid profile
reveals total cholesterol 233 mg/dL, HDL 51 mg/dL, LDL131
mg/dL, and triglycerides 140 mg/dL. In addition to diet
modifications, routine exercise, and blood pressure control,
what would you do to manage this patient?
A. Refer for exercise test and start statin if positive
B. Start atorvastatin 20mg nightly
C. Reassure her that she is at target cholesterol goals
D. Start ezetimibe 10mg daily
E. Start niacin 1 gram daily with aspirin

1393
Copyright © Harvard Medical School, 2018. All Rights Reserved.

Answer #28
A 73-year old man with a 25 pack year history of smoking
(stopped 6 years ago), hypertension, and diabetes mellitus
presents to your office for an initial visit. Fasting lipid profile
reveals total cholesterol 233 mg/dL, HDL 51 mg/dL, LDL131
mg/dL, and triglycerides 140 mg/dL. In addition to diet
modifications, routine exercise, and blood pressure control,
what would you do to manage this patient?
A. Refer for exercise test and start statin if positive
B. Start atorvastatin 20mg nightly
C. Reassure her that she is at target cholesterol goals
D. Start ezetimibe 10mg daily
E. Start niacin 1 gram daily with aspirin

4 Major Statin Benefit Groups:


1) With clinical ASCVD*
2) Primary elevations of LDL–C >190
mg/dL,
3) Diabetes aged 40 to 75 years with
LDL– C 70 to189 mg/dL and without
clinical ASCVD
4) Without clinical ASCVD or diabetes
with LDL–C 70 to189 mg/dL and
estimated 10-year ASCVD risk >7.5%

*ASCVD= Atherosclerotic
Cardiovascular Disease

1394
Copyright © Harvard Medical School, 2018. All Rights Reserved.

Question #29
A 39 year old woman who is 30-weeks pregnant and
presents to emergency room with mild dyspnea on exertion.
Examination reveals BP 110/70 mm Hg, HR 95 beats/minute,
clear chest, jugular venous pressure 7 cm water, and III/VI
holosystolic murmur at apex. Echocardiography confirms
moderate mitral regurgitation. Estimated PA pressure is 30+
right atrial pressure. Management option includes which of
the following?
A. Start low-dose furosemide
B. Arrange for emergent delivery of fetus
C. Plan mitral valve repair prior to delivery
D. Left and right heart catheterization
E. Proceed with pregnancy with close follow-up

1395
Copyright © Harvard Medical School, 2018. All Rights Reserved.

Answer #29
A 39 year old woman who is 30-weeks pregnant and
presents to emergency room with mild dyspnea on exertion.
Examination reveals BP 110/70 mm Hg, HR 95 beats/minute,
clear chest, jugular venous pressure 7 cm water, and III/VI
holosystolic murmur at apex. Echocardiography confirms
moderate mitral regurgitation. Estimated PA pressure is 30+
right atrial pressure. Management option includes which of
the following?
A. Start low-dose furosemide
B. Arrange for emergent delivery of fetus
C. Plan mitral valve repair prior to delivery
D. Left and right heart catheterization
E. Proceed with pregnancy with close follow-up

Valvular Heart Lesions Associated With High


Maternal and/or Fetal Risk During Pregnancy

1. Severe AS with or without symptoms


2. AI with NYHA functional Class III–IV symptoms
3. MS with NYHA functional Class II–IV symptoms
4. MR with NYHA functional Class III–IV symptoms
5. Aortic and/or mitral valve disease resulting in severe pulmonary
hypertension (pulmonary pressure >75% of systemic
pressures)
6. Aortic and/or mitral valve disease with severe LV dysfunction
(EF<40%)
7. Mechanical prosthetic valve requiring anticoagulation
8. AI in Marfan’s syndrome

Nishimura et al. J Am Coll Cardiol. 2014;63(22):e57-e185

1396
Copyright © Harvard Medical School, 2018. All Rights Reserved.

Question #30
A 67-year old man with known diabetes and coronary artery
disease presents with acute chest pain, dyspnea, and mild
hypotension with electrocardiogram demonstrating 2.4 mm
ST elevations in leads V1-V4 and I and avL. He had a
percutaneous coronary angioplasty with placement of a drug
eluting stent in the proximal left anterior descending artery 25
days ago after a positive stress test. Which is not a common
causes of in-stent thrombosis?
A. Multiple, consecutive long stents in the artery
B. Use of clopidogrel instead of prasugrel
C. Diseased coronary vessel with poor distal run-off
D. Inadequate expansion of the stent
E. Non-adherence to clopidogrel and aspirin

Answer #30
A 67-year old man with known diabetes and coronary artery
disease presents with acute chest pain, dyspnea, and mild
hypotension with electrocardiogram demonstrating 2.4 mm
ST elevations in leads V1-V4 and I and avL. He had a
percutaneous coronary angioplasty with placement of a drug
eluting stent in the proximal left anterior descending artery 25
days ago after a positive stress test. Which is not a common
causes of in-stent thrombosis?
A. Multiple, consecutive long stents in the artery
B. Use of clopidogrel instead of prasugrel
C. Diseased coronary vessel with poor distal run-off
D. Inadequate expansion of the stent
E. Non-adherence to clopidogrel and aspirin

1397
Copyright © Harvard Medical School, 2018. All Rights Reserved.

https://www.nhlbi.nih.gov/health/health-topics/topics/stents/risks

Porto et. al. The American Journal of Cardiology, Volume 105,


Issue 12, 15 June 2010, Pages 1710–1715

Question #31
81-year old woman presents for a routine clinical visit for a
physical and her exam is notable for splitting of the second
heart sound. Which statement regarding splitting of the
second heart sounds is NOT true?

A. Paradoxical splitting of S2 is expected in patients with a RV paced rhythm


B. Delayed closure of the pulmonic valve with inspiration contributes to
physiologic splitting
C. Fixed splitting of S2 is present with an ostium secundum atrial septal
defect
D. Severe pulmonary hypertension is associated with an accentuated P2
E. Right bundle branch block is associated with paradoxical splitting of S2

1398
Copyright © Harvard Medical School, 2018. All Rights Reserved.

Answer #31
81-year old woman presents for a routine clinical visit for a
physical and her exam is notable for splitting of the second
heart sound. Which statement regarding splitting of the
second heart sounds is NOT true?

A. Paradoxical splitting of S2 is expected in patients with a RV paced rhythm


B. Delayed closure of the pulmonic valve with inspiration contributes to
physiologic splitting
C. Fixed splitting of S2 is present with an ostium secundum atrial septal
defect
D. Severe pulmonary hypertension is associated with an accentuated P2
E. Right bundle branch block is a/w with paradoxical splitting of S2

Answer (Continued)

• Increased filling during inspiration delays the closure of the P2

Soft S2: Valvular stenosis


Fixed splitting: Atrial septal defect
Widened splitting: RBBB
Paradoxical splitting: LBBB, RV pacing

Joseph D. Sapira The Art and Science of Bedside Diagnosis. Williams and Wilkins 1990

1399
Copyright © Harvard Medical School, 2018. All Rights Reserved.

Question #32
An 45-year old man presents to the emergency room with
dyspnea. His blood pressure was 228/110 mm Hg and the
remainder of the physical examination was only notable for
faint crackles. Which of the following findings are
characteristics of a hypertensive crisis?
A. Retinal hemorrhages
B. Microangiopathic hemolytic anemia
C. Azotemia and proteinuria
D. Pulmonary edema and jugular venous distension
E. All are characteristics of hypertensive crisis

Answer #32
An 45-year old man presents to the emergency room with
dyspnea. His blood pressure was 228/110 mm Hg and the
remainder of the physical examination was only notable for
faint crackles. Which of the following findings are
characteristics of a hypertensive crisis?
A. Retinal hemorrhages
B. Microangiopathic hemolytic anemia
C. Azotemia and proteinuria
D. Pulmonary edema and jugular venous distension
E. All are characteristics of hypertensive crisis

1400
Copyright © Harvard Medical School, 2018. All Rights Reserved.

Features of Hypertensive Crisis

- Renal insufficiency with proteinurea


- Hematuria
- Azotemia
- Microangiopathic hemolytic anemia
- Heart failure
- Nausea and vomiting

Hypertensive encephalopathy
- Confusion, irritability, headaches, stupor, neurologic deficits,
seizures, coma
- Sudden rise in blood pressure causes acute damage to blood
vessels
- Not always present with malignant hypertension
- Failure of cerebral autoregulation causing excess cerebral blood
flow and damage to the wall leading to increased vascular
permeability

1401
Copyright © Harvard Medical School, 2018. All Rights Reserved.

Question #33
A 52-year old male presents to clinic for perioperative
evaluation prior to elective cholecystectomy. He works full
time in construction and exercises by walking ~40 minutes
several times per week. His past medical history is notable for
hypertension, hypercholesterolemia, and a myocardial
infarction 3 years ago. Risk factors associated with cardiac
complications after major non-cardiac surgery include all of
the following EXCEPT:
A. Severe mitral stenosis
B. Stable class II angina
C. Acute myocardial infarction 2 months ago
D. More than 5 premature ventricular contractions
(PVC)/minute on a pre-operative ECG
E. Presence of a S3

Answer #33
A 52-year old male presents to clinic for perioperative
evaluation prior to elective cholecystectomy. He works full
time in construction and exercises by walking ~40 minutes
several times per week. His past medical history is notable for
hypertension, hypercholesterolemia, and a myocardial
infarction 3 years ago. Risk factors associated with cardiac
complications after major non-cardiac surgery include all of
the following EXCEPT:
A. Severe mitral stenosis
B. Stable class II angina
C. Acute myocardial infarction 2 months ago
D. More than 5 premature ventricular contractions
(PVC)/minute on a pre-operative ECG
E. Presence of a S3

1402
Copyright © Harvard Medical School, 2018. All Rights Reserved.

Goldman Criteria

Goldman L et al. NEJM 1977

Thank you!

1403
Copyright © Harvard Medical School, 2018. All Rights Reserved.

INFECTION IN THE IMMUNOCOMPROMISED HOST

Sarah P Hammond, MD
Associate Physician
Division of Infectious Diseases, Department of Medicine
Brigham and Women’s Hospital
Assistant Professor
Harvard Medical School

Disclosures
• I have research funding from Merck

1404
Copyright © Harvard Medical School, 2018. All Rights Reserved.

Overview
• Introduction and Basic Principles
• Drug-induced Immunodeficiency
– Corticosteroids
– TNF-α inhibitors
– Anti CD-20 therapy
• Asplenia
• Transplant

Introduction and Basic Principles

1405
Copyright © Harvard Medical School, 2018. All Rights Reserved.

Infection in Compromised Host


• The immunocompromised host (ICH) is
susceptible to opportunistic infections and
community-acquired infections
– Infection can result from exposure to a lower number
of organisms
• The inflammatory response to infection is
suppressed in ICH
– Attenuated signs and symptoms of infection
• ICH with infection typically has high burden of
organisms once infection established

Rubin, RH. N Engl J Med. 1987;317:1151-3.

Basic Principles
• Net state of immunosuppression
– Dose and duration of suppressive medications
– Mechanical factors
– Infections contributing to compromise

• Important epidemiologic exposures


– New exposures
– Remote exposures with possibility of reactivation

Fishman JA, Rubin, RH. N Engl J Med. 1998;338:1741-51.


Fishman JA. N Engl J Med. 2007;357:2601-14.

1406
Copyright © Harvard Medical School, 2018. All Rights Reserved.

Drug-Induced
Immunosuppression

Mechanism, Indication, Dose & Duration


• In last 2 decades the number of immunosuppressants
approved to treat malignant and auto-immune
disorders has expanded!
• Mechanism of an immunosuppressive agent is key to
predicting host susceptibility to infection
– Particularly true of biologic and targeted agents
• Indication for which an agent is given is important
when considering impact on host defenses
• Dose and duration over which the agent is given
impacts vulnerability
– Effect of some biologic agents last for 6 months or more

1407
Copyright © Harvard Medical School, 2018. All Rights Reserved.

Immunosuppressive Drugs
• Corticosteroids • Monoclonal antibodies
• Antimetabolites – Tumor necrosis factor-α (TNF-
– Methotrexate α) inhibitors
– Azathioprine, 6-mercatopurine • Infliximab
• (Etanercept)
– Mycophenolate
• Adalimumab
• T lymphocyte agents • Certolizumab pegol
– Tacrolimus, Cyclosporine • Golimumab
– Sirolimus – CD-20 antibodies
• ‘Targeted’ agents • Rituximab
• Ofatumumab, Obinutuzumab
– Janus kinase inhibitors • Ocrelizumab
• Ruxolitinib
• Tofacitinib
– IL12/23 antibody: Ustekinumab
• (Baricitinib) – CD 52 antibody: Alemtuzumab
– Proteosome inhibitors • Biologic agents
• Bortezomib – IL1 inhibitor: Anakinra
– PI3 kinase inhibitors – T cell costimulation inhibitor
• Idelalisib • Abatacept, Belatacept

Corticosteroids
• Acute effect of corticosteroids on host
defenses includes
– Neutrophil demargination and reduced
chemotaxis
– Lymphopenia and depletion of T lymphocytes
• Long term effect of corticosteroids on host
defenses includes
– Skin weakening and poor wound healing

Klein NC, et al. Infect Dis Clin North Am 2001;15:423-32.

1408
Copyright © Harvard Medical School, 2018. All Rights Reserved.

Corticosteroids and infection


• In general, steroids increase risk of infection
– In study of rheumatoid arthritis (RA), patients on
steroids vs. non-immunosuppressive therapy
• Rate ratio of mild infection: 1.15 (95% CI 1.11, 1.19)
• Rate ratio of severe infection: 1.9 (95% CI 1.75, 2.05)
– In metanalysis of 71 controlled trials of steroid vs.
non-steroid therapy, steroid recipients had
significantly higher rate of infection (12.7% vs.
8.0%) and lethal infection (1.2% vs. 0.5%)
• Included studies of GI, pulmonary, renal, neurologic
and rheumatic conditions
Lacaille D, et al Arthritis Rheum 2008;59:1074-81.
Stuck AE, et al. Rev Infect Dis 1989; 11:954-62.

Corticosteroids: dose and duration


• In metanalysis by Stuck, et
al., dose and duration of
steroids impacted infection
risk
• In large database study of
RA, IBD and psoriasis,
prednisone doses of <5, 5-
10, and >10 mg resulted in
a sequential increase in risk
of serious infection in RA
and psoriasis
• Dose-dependent risk of <10
mg of prednisone per day
has been shown in other
observational studies Stuck AE, et al. Rev Infect Dis 1989; 11:954-62.
Grijalva CJ, et al. JAMA 2011;306:2331-9.
Youssef J, et al. Rheum Dis Clin North Am 2016;42:157-76.

1409
Copyright © Harvard Medical School, 2018. All Rights Reserved.

Corticosteroids & specific infections


• Specific types of infection associated with
steroids
– Opportunistic pathogens
• Bacterial (eg. Listeria)
• Fungal (eg. Aspergillus)
• Viral (eg. zoster)
• Reactivation of latent tuberculosis
• Strongyloides hyperinfection syndrome
• Pneumocystis jiroveci pneumonia (PCP)

Corticosteroids and PCP


• Steroids are a major risk factor for PCP www.dpd.cdc.gov/dpdx/HTML/Pneumocystis.htm

– In a study of 116 non-HIV patients, 105


(91%) were on steroids
– Similarly, in a study of 134 non-HIV cancer
patients with PCP, 116 (87%) were on
steroids
• While PCP in non-HIV patients is a rare event, it is
associated with significant mortality
– Mortality rate among 114 non-HIV cancer patients
was 49% in one study
– Among 223 patient with connective tissue disease
with PCP the in-hospital mortality was 45.7%
Yale SH, Limper AH. Mayo Clin Proc 1996;71:5-13.
Sepkowitz KA, et al. JAMA 1992;267:832-7.
Ward MM, Donald F. Arthritis and Rheum 1999;42:780-9.

1410
Copyright © Harvard Medical School, 2018. All Rights Reserved.

Corticosteroids and PCP


• PCP risk depends on steroid dose and duration
– Among 105 non-HIV patients with PCP on steroids
• Median dose: 30 mg prednisone per day (25% on 16 mg/day or less)
• Median duration: 12 weeks (25% on for 8 weeks or less)
• PCP risk may not be uniform among patient types
– In general, risk in lupus, temporal arteritis and rheumatologic
disease all linked to ‘high dose’ steroids (30-50 mg/day)
– Risk may be especially high in patients with metastatic cancer
on steroids
– PCP risk is elevated, but occurs infrequently in IBD
• At what steroid dose/duration should prophylaxis start?
– There are no studies directly assessing this
– Extrapolating from available data many clinicians start
prophylaxis after 3-4 weeks of 20-30 mg prednisone per day
Yale SH, Limper AH. Mayo Clin Proc 1996;71:5-13. Chew LC, et al. J Clin Rheumatol 2015;21: 72–75
Sepkowitz KA, et al. JAMA 1992;267:832-7. Cotter TG, et al. Clin Gastroenterol Hepatol 2017;15:850-6.
Youssef J, et al. Rheum Dis Clin North Am 2016;42:157-76. Schoovaerts K, et al. Acta Clin Belg. 2017;27:1-4.

Case Presentation #1
• 65 year old woman with rheumatoid arthritis (RA)
presents with 3 days of productive cough and
progressive dyspnea
– She has had fever as high as 100.9 for last 2 days
• Past Medical History
– RA diagnosed 25 years ago
• Treated with infliximab and methotrexate for the last 15
years
– Hypertension
– Up-to-date on vaccines and cancer screening
• Vaccinated with influenza and conjugate pneumococcal
vaccine at her last PCP visit 4 month ago

1411
Copyright © Harvard Medical School, 2018. All Rights Reserved.

Case Presentation #1 (con’t)


• Social history
– She lives in CT with 2 indoor/outdoor cats
– No recent travel; she takes care of a 4 year old grandchild 2
days a week
• Exam
– T 100.7 F, P 112, R 21, O2 sat 93% 4L nasal canula
– Somewhat chronically ill-appearing women; tachycardic;
chest has coarse crackles in both lungs; the abdomen is
non-tender
• Labs: WBC 15.3 with 79% neutrophils, 7% bands
• Radiology: Chest x-ray shows dense infiltrates in right
upper and left lower lobes

Chest CT Scan

1412
Copyright © Harvard Medical School, 2018. All Rights Reserved.

Which is the most likely cause of


pneumonia?
A. Legionella pneumophilia
B. Cryptogenic organizing pneumonia
C. Aspergillus fumigatus
D. Influenza
E. Mycobacterium tuberculosis

TNF-α inhibitors
• TNF-α is an inflammatory cytokine produced by
macrophages
– Important for control of infection with intracellular organisms
and for granuloma formation
• There are 5 FDA-approved drugs that inhibit TNF-α
– Monoclonal antibodies that bind TNF-α
• Infliximab (1998), Adalimumab (2002), Golimumab (2009)
– Pegylated portion of a monoclonal antibody that binds TNF-α
• Certolizumab pegol (2008)
– Soluble TNF-α receptor that binds soluble TNF-α
• Etanercept (1998)
• ‘Biosimilars’ are biological products that are very similar to
approved agents– improved cost/availability
– Between 2016-2018 FDA has approved multiple biosimilars for
infliximab, etanercept and adalimumab
https://www.fda.gov/Drugs/DevelopmentApprovalProcess/HowDrugsareDevelopedandApproved/
ApprovalApplications/TherapeuticBiologicApplications/Biosimilars/ucm580432.htm, accessed 3/17/18

1413
Copyright © Harvard Medical School, 2018. All Rights Reserved.

TNF-α inhibitors and infection


• Several studies and registries have assessed if
TNF-α inhibitors increase overall risk of infection
– Many studies have assessed risk for bacterial
infection; results have been mixed
– Metanalysis including 9 trials assessed infection risk in
RA patients treated with infliximab or adalimumab vs.
non-TNF-α therapy
• Pooled odds ratio for serious infection: 2.0 (95% confidence
interval 1.3, 3.1)
• Most infections in this study were bacterial (114 of 126)
• Limited by control comparison: All controls got placebo (+/-
methotrexate, at same dose as TNF- α recipients)
Bongartz T, et al. JAMA. 2006;295:2275-85.
Curtis JR, et al. Arthritis Rheum. 2007;56:1125-33.
Schneeweiss S, et al. Arthritis Rheum. 2007;56:1754-64.

TNF-α inhibitors and infection


• Grijalva, et al. assessed infection requiring hospitalization in
autoimmune disorder patients treated with infliximab,
etanercept or adalimumab vs. comparators
– Large propensity score analysis study that included 16,022
patients; more than half had RA
• No difference in serious infection between those treated
with TNF-α inhibitor and comparators for patients with RA,
IBD, or psoriasis/spondyloarthropathy
• Among the 10,484 patients
with RA, those treated with
infliximab had higher risk for
serious infection than those
treated with non-biologics or
other TNF-α agent

Grijalva CJ, et al. JAMA 2011;306:2331-9.

1414
Copyright © Harvard Medical School, 2018. All Rights Reserved.

TNF-α inhibitors & Opportunistic Infection


• TNF-α inhibitors have been associated with several
opportunistic infections (OIs)
– Mycobacteria: Tuberculosis (TB), Non-tuberculous mycobacteria
– Bacteria: Listeria, Legionella, Nocardia
– Fungi: Pneumocystis, Cryptococcosis, Endemic mycoses
• TB is the most common OI
internationally
• In a recent large US
database study that
included 33,324 new TNF-α
users, Pneumocystis was
the most common OI
Winthrop KL, et al. Ann Rheum Dis. 2013;72:37-42..
Slifman NR, et al. Arthritis Rheum. 2003;48:319-24.
Perez-Alvarez R, et al. Medicine. 2011;90:359-71.
Salmon-Ceron D, et al. Ann Rheum Dis. 2011;70:616-23.
Baddley JW, et al. Ann Rheum Dis. 2014;73:1942-8.

TNF-α inhibitors & Opportunistic Infection


• In French registry study of infliximab, adalimumab
and etanercept incidence of OI was low (152 OIs per
100,000 patient-years)
– Severity of illness was high
– Median time to OI was 16.2 months (range 6-26)
– OI risk significantly higher in patients treated with
infliximab compared to etanercept
• In US database study of same TNF-α inhibitors,
incidence of non-viral OIs was also low (2.7 OIs per
1000 patient-years), but was significantly higher
than patients treated with non-biologic therapy
• In both studies OI risk significantly higher in those on
current steroids (>10 mg prednisone/day)
Winthrop KL, et al. Ann Rheum Dis. 2013;72:37-42.. Perez-Alvarez R, et al. Medicine. 2011;90:359-71.
Slifman NR, et al. Arthritis Rheum. 2003;48:319-24. Salmon-Ceron D, et al. Ann Rheum Dis. 2011;70:616-23.
Baddley JW, et al. Ann Rheum Dis. 2014;73:1942-8.

1415
Copyright © Harvard Medical School, 2018. All Rights Reserved.

TNF-α inhibitors and TB


• Keane, et al. analyzed all cases of TB associated with infliximab
reported to FDA 1998-2001
– 40/70 (57%) had extrapulmonary TB
– Median time to TB: 12 weeks
• Increased risk was again demonstrated in more recent large
database studies from Europe and US
– These studies have demonstrated differences in TB risk after
monoclonal antibodies vs. etanercept
• TB risk after infliximab is comparable to adalimumab but both are higher than
after etanercept
• TB develops earlier after infliximab than etanercept (~3 vs 12 months)
• When TNF-α therapy is planned, patients should be screened with
PPD (≥ 5mm is reactive) or interferon gamma release assay (IGRA)
– Those with reactive PPD and/or positive IGRA should be assessed for
active vs. latent TB and treated
Keane J, et al. N Engl J Med. 2001;345:1098-104.
Winthrop KL, et al. Arthritis Rheum. 2005;52:2968-74.
Winthrop, KL. Nat Clin Pract Rheumatol. 2006;2:602-610.
Salmon-Ceron D, et al. Ann Rheum Dis. 2011;70:616-23.
Winthrop KL, et al. Ann Rheum Dis. 2013;72:37-42.

Clinical Case #2
• A 58-year-old woman is diagnosed with diffuse
large B cell lymphoma (DLBCL) after she presents
with several weeks of fevers, night sweats, weight
loss and left neck swelling
• Past medical history
– Hypercholesterolemia
– An episode of ‘hepatitis’ requiring a brief hospital stay
after she was stuck by a needle when she was training
to be a phlebotomist 35 years ago
• Plans are made to initiate chemotherapy with
rituximab, cyclophosphamide, doxorubicin,
vincristine, and prednisone (R-CHOP)

1416
Copyright © Harvard Medical School, 2018. All Rights Reserved.

Clinical Case #2
• Physical exam is notable for a thin woman with
marked left neck lymphadenopathy
• Labs are notable for normal BUN, Cr, AST, ALT, and
bilirubin
– Hepatitis B surface antibody (HBsAb)– 27 IU/L
– Hepatitis B core IgG (HBcAb) – positive
– Hepatitis B surface antigen (HBsAg) – negative
– Hepatitis B virus PCR – negative
– Hepatitis C virus antibody – positive
– Hepatitis C virus PCR – negative
– HIV screening - negative

What is the best management for her


previous hepatitis exposures?
A. Monitor hepatitis C PCR and HBsAb every 2-4
weeks while she is on chemotherapy
B. Start prophylactic low dose ribavirin and
lamivudine to continue for 1 year
C. Start prophylactic entecavir to continue until
12 months after chemotherapy
D. No special care is indicated since both
hepatitis B and C are old infections that have
resolved

1417
Copyright © Harvard Medical School, 2018. All Rights Reserved.

CD-20 Antibodies
• Rituximab is a monoclonal antibody directed at CD-20, a
B lymphocyte marker
– Immunologic effect is depletion of B lymphocytes
– Functionally this leads to inability to mount antibody
response to new and recall antigens
• FDA approved for treatment of some lymphomas,
chronic lymphoid leukemia (CLL), and RA
– Used off-label to treat immune-mediated anemia and
thrombocytopenia, lupus, and pemphigus
• Ofatumumab and Obinutuzumab
– Newer CD-20 antibodies used in hematologic malignancy
only; thought to have same infection risks as rituximab
• Ocrelizumab
– The newest CD-20 antibody approved for multiple sclerosis
(MS) in early 2017-- likely has same side effects
Gelinck LBS, et al. Ann Rheum Dis. 2007;66:1402-3.
Van der Kolk LE, et al. Blood. 2002;100:2257-9.

Rituximab and Infection


• Pre-marketing trials showed no increase in infection in RA
or cancer patients on rituximab vs. non-rituximab regimens
– Similarly, ocrelizumab trials show no increase in infection in MS
• With clinical use, there have been several case reports of
various opportunistic infections
– PCP and Progressive multifocal leukoencephalopathy due to JC
virus are most commonly cited in case reports/series
– Others include CMV reactivation, pure red cell aplasia due to
parvovirus, and enterovirus encephalitis
– Interpretation of case reports is difficult since patients often on
multiple immunosuppressants
• Rituximab is definitively associated with
– Hepatitis B virus (HBV) reactivation
– Increased severity of babesiosis (in exposed patients)
Coiffier B, et al. N Engl J Med. 2002;346:235-42. Suzan F, et al. N Engl J Med. 2001;345:1000.
Edwards JCW, et al. N Engl J Med. 2004;350:2572-81. Sharma VR, et al. Blood. 2000;96:1184-6.
Montalban X, et al. N Engl J Med 2017;376:209-20. Padate BP, Keidan J. Clin Lab Haem. 2006;28:69-71.
Hauser SL, et al. N Engl J Med 2017;376:221-34. Perez-Alvarez R, et al. Medicine. 2011;90:359-71.

1418
Copyright © Harvard Medical School, 2018. All Rights Reserved.

HBV reactivation
Resolved HBV Reactivated HBV
Exposure
• Perinatal infection infection
RITUXIMAB
• Percutaneous • HBsAg+ HBsAb+ • HBsAg+
• Sexual • usually HBeAg+
• HBV DNA-
• HBV DNA high
• HBcAb+
• Abnormal LFTs

RITUXIMAB
Acute HBV
Infection
• HBsAg+
• HBV DNA+ Inactive Carrier
• HBsAg+
• usually HBeAb+
• HBV DNA low (or -)
Chronic HBV
• normal LFTs
infection
• HBsAg+ >6 months
• HBV DNA+
• HBcAb+

Chronic active
infection
• HBsAg+
• HBV DNA+

Rituximab and HBV Reactivation


• Fulminant hepatic failure and death have been reported
with reactivation
• Risk of reactivation is higher in chronic carriers (HBsAg+)
than in patients with resolved HBV
• Prevention
– All patients in whom rituximab, TNF-α inhibitor, or high-dose
steroid therapy is being considered should be screened
– Antiviral prophylaxis typically given to inactive carriers
• Agents with high barrier to resistance recommended
– Management of HBcAb+ patients varies
• Guidelines advocate for prophylaxis, but a few recent studies have
shown that careful monitoring with preemptive therapy is effective
• Note: risk in this population with TNF-α inhibitor appears to be very
low– prophylaxis not typically necessary
Terrault NA, et al. Hepatology. 2018;67:1560-99.
Perez-Alvarez R, et al. Medicine. 2011;90:359-71.
Kusumoto S, et al. Clin Infect Dis 2015;61:719-29.
KR Reddy, et al. Gastroenterol. 2015;148:215–219.

1419
Copyright © Harvard Medical School, 2018. All Rights Reserved.

Rituximab and Babesiosis


• A tick-borne malaria-like parasitic infection
– Northeastern US, Wisconsin and Minnesota
– Causes fevers, malaise, anemia (due to
Courtesy of Francisco Marty
– hemolysis) and thrombocytopenia
• Patients treated with rituximab who acquire babesiosis
shortly before or after treatment have worse illness
• In case-control study of persistent babesiosis vs.
uncomplicated infection, over half cases had received
rituximab
• Rituximab-treated patients with babesiosis:
– Often have a higher parasite burden (median 8% vs. 2.5%)
– More likely to develop severe complications including ARDS
– More likely to have relapse requiring repeat or prolonged
treatment courses (typically ~6 weeks)
Krause PJ, et al. Clin Infect Dis. 2008;46:370-6.
Vannier E and Krause PJ. N Engl J Med 2012;366:2397-407.
Wormser GP, et al. Clin Infect Dis. 2010;50:381-6.

Asplenia

1420
Copyright © Harvard Medical School, 2018. All Rights Reserved.

Asplenia
• Immunolgically the spleen plays an important role in
filtering blood
– Important for clearance of extracellular bacteria and
intracellular parasites (eg. babesia)
• Many important medical reasons for surgical
splenectomy including trauma, malignancy, and
refractory hematologic disorders (eg. ITP)
• Other conditions result in asplenia or ‘hyposplenism’
– Asplenia: Congenital absence, autoinfarction (eg. In sickle
cell disease)
– Hyposplenism: Advanced HIV, chronic graft-versus-host
disease
LG Rubin,W Schaffner. NEJM 2014;371:349-56

Asplenia and bacterial infection


• Asplenic patients are at risk for overwhelming bacterial infection
due to encapsulated organisms
– Streptococcus pneumoniae
– Haemophilus influenzae
– Neisseria meningitidis
• In recent study of severe sepsis in asplenic patients, S. pneumoniae
caused 42% of infections (most invasive infection), more than any
other pathogen
– No cases of H. influenzae or N. meningitidis observed
• Progression of infection can be rapid and mortality approaches 50%
in some studies
• Risk may vary by reason for and time since splenectomy
• Understanding of risk for bacterial infection impacts risk
– In one study rate of overwhelming infection was 16.9% among 79
asplenic patients with poor knowledge vs. <2% among 142 patients
with good knowledge
LG Rubin,W Schaffner. NEJM 2014;371:349-56
MS El-Alfy, MH El-Sayed. Hematol Journal 2004;5:77–80
Theilacker C, et al. Clin Infect Dis. 2016;62:871–8

1421
Copyright © Harvard Medical School, 2018. All Rights Reserved.

Vaccination of Asplenic Patients


• Vaccination for S. pneumoniae, H. influenzae, and N.
meningitidis crucial to prevent infection
• Timing with splenectomy
– Vaccinate at least 2 week pre-splenectomy in planned cases and
1-2 weeks after (or before hospital discharge) in unplanned
cases
• Vaccination algorithms
– S. pneumoniae: two vaccines recommended for use in asplenic
patients
• 23 valent polysaccharide vaccine (PPSV23, 1983)
• 13 valent protein conjugate vaccine (PCV13, 2011)
– Contains 12 serotypes in the PPSV23 vaccine
– N. meningitidis: the two vaccines types now also recommended
include conjugate meningococcal ACWY and (new)
meningococcus type B
Davidson RN, Wall RA. Clin Microbiol Infect 2001;7:657-60.
LG Rubin, W Schaffner. NEJM 2014;371:349-56
Bennett NM, et al. MMWR 2012;61:816-9.

Management of fever
• Asplenic patients
should be educated
about risk of severe
bacterial infection
– Survey studies show
5-25 % of asplenic
patients have poor
knowledge of
infection risk
• “Pill in pocket”
approach

Rubin LG , Schaffner W. N Engl J Med. 2014;371:349-56


MS El-Alfy, MH El-Sayed. Hematol Journal 2004;5:77–80
Wilkes A, et al. ANZ J Surg. 2008;78:867-70

1422
Copyright © Harvard Medical School, 2018. All Rights Reserved.

Asplenia and other infections


• Capnocytophaga canimorsus
– Gram-negative bacterial colonizer of canine mouth
– Cause of severe infection/sepsis in asplenic patients
– Usually susceptible to beta-lactam/beta-lactamases
– Asplenic patients should be educated about careful
cleaning after animal bite (especially dog) and medical
evaluation
• Babesiosis
– Severity of babesiosis is increased and parasite burden
higher in asplenic patients
– May require prolonged or repeated courses of
treatment
Stiegler D, et al. Am J Forensic Med Pathol. 2010;31:198-9.
Davidson RN, Wall RA. Clin Microbiol Infect. 2001;7:657-60.
Krause PJ, et al. Clin Infect Dis. 2008;46:370-6.

Transplantation

1423
Copyright © Harvard Medical School, 2018. All Rights Reserved.

Case Presentation #3
• A 39-year-old woman who underwent a renal
transplant 7 years ago presents with presents with 3
days of diarrhea and fever to 101.2° F
– Generally feels unwell with fevers and frequent diarrhea
but no SOB, cough, nausea, or vomiting
• Past medical history: HTN, history of IgA nephropathy
s/p renal transplant 7 years ago (baseline Cr 1.1)
– Donor CMV-/recipient CMV+; never had CMV post-
transplant
– Never had rejection since transplant
• Medications: Tacrolimus, Mycophenolate mofetil,
Lisinopril

Case Presentation #3 (con’t)


• Social History
– Accountant
– Married and lives with wife and 7-year-old son
who has a pet hedgehog at home
– Likes to garden
• Vital signs: T 100.9 P 110 BP 119/73
• Exam: Soft abdomen with mild diffuse
tenderness without rebound or guarding

1424
Copyright © Harvard Medical School, 2018. All Rights Reserved.

The most likely cause of her illness is:


A. Salmonellosis
B. CMV colitis
C. Cryptosporidium
D. Norovirus
E. Mycophenolate toxicity

Timing of Infection after Transplant


Time after 0-1 Month 1-6 Months* >6 Months
transplant
Type of Nosocomial Opportunistic Community-
infection MOSTLY BACTERIA OR VIRAL acquired
CANDIDA CMV VIRAL
Cather-related UTI VZV Influenza
Line infection BK HPV
Pneumonia BACTERIAL Norovirus
Nocardia HBV reactivation
Procedure- Listeria BACTERIAL
related FUNGAL S. pneumoniae
Wound infection PCP Listeria
Fever+ neutropenia Aspergillus Legionella
Donor-derived PARASITIC FUNGAL
Strongyloides Cryptococcus
RARE
Toxoplasma Endemic fungi

* Patients treated for rejection or graft-versus-host disease Fishman JA, Rubin RH. N Engl J Med. 1998:338;1741-51.
after 6 months are also at risk for these infections Fishman JA. N Engl J Med. 2007;357:2601-14.

1425
Copyright © Harvard Medical School, 2018. All Rights Reserved.

Summary
• ICH with infection often presents with attenuated signs
and symptoms of infection
• When treating an ICH with infection consider:
– Net state of immunosuppression
• Includes mechanism, indication, dose/duration for patients on
suppressive medications
– Epidemiologic exposures
• Asplenia increases risk for overwhelming bacterial
infection vaccinate and educate about fever
• Transplant recipients are susceptible to a wide range of
infections
– Risk depends on: 1) when the transplant occurred. 2)
whether significant rejection has been treated recently

Selected References
• Fishman JA, Rubin RH. N Engl J Med.
1998:338;1741-51.
• Stuck AE, et al. Rev Infect Dis 1989; 11:954-62.
• Youssef J, et al. Rheum Dis Clin North Am
2016;42:157-76.
• Rubin LG , Schaffner W. N Engl J Med.
2014;371:349-56.

1426
Copyright © Harvard Medical School, 2018. All Rights Reserved.

Disclosures
• I have research funding from Merck

1427
Copyright © Harvard Medical School, 2018. All Rights Reserved.

Tropical Medicine and Parasitology


James H. Maguire, MD
Senior Physician
Division of infectious Disease and Department of Medicine
Brigham and Women’s Hospital
Professor of Medicine
Harvard Medical School

Disclosures
Bayer: Chagas disease advisory committee

1428
Copyright © Harvard Medical School, 2018. All Rights Reserved.

Outline
• Febrile illnesses
• Diarrhea
• Important infections of immigrants and
returning long-term expatriates
• Skin lesions

40 year old man with fever


and mental status changes
• 3 week stay in Nigeria 10 days earlier
• x1 week: malaise, feverishness
• Unusual behavior at home after work
• ER next morning:
– Temp 102o F
– Agitated, unable to follow commands
• Hct 32, WBC 6000, platelets 110,000,
creatinine 2.2
Differential diagnosis?

1429
Copyright © Harvard Medical School, 2018. All Rights Reserved.

Fever in returned US travelers


• 9624 visits 2000-2012 (Geosentinel clinics)
• 18.2% with fever
Malaria (27.4%)
Viral syndrome (18.5%)
Dengue (12%)
Mononucleosis syndrome (8.7%)
Rickettsial disease (4.7%)
Enteric fever (6.1%)
Fam Pract 31:678, 2014

• Also; fever with diarrhea or respiratory


infection, acute hepatitis, acute HIV

Further history:
- Immigrated from Nigeria 6 years earlier
- Prior to travel: no antimalarial
chemoprophylaxis, no vaccinations.
- Visited family
Further studies?

Plasmodium falciparum Rapid blood test for


malaria (dipstick) was
(8% of RBCs infected) also positive

1430
Copyright © Harvard Medical School, 2018. All Rights Reserved.

1485 reported cases of imported malaria in 2015

11 deaths in 2015

Purpose of travel: Visiting


friends and relatives 70%

MMWR Morb Mortal Wkly Rep. 2018;67(1-28)

Plasmodium falciparum: life-threatening


– High parasitemia
– Only rings on smears:
RBCs infected with later
stages adhere to
endothelium of small
blood vessels
– Pathophysiology
• Severe anemia
• Blockage of blood flow
• Systemic and local
cytokine production

1431
Copyright © Harvard Medical School, 2018. All Rights Reserved.

Plasmodium falciparum: life-threatening


– High parasitemia
– Only rings on smears:
RBCs infected with later
Falciparum
stages adheremalaria
to
Progressesofrapidly
endothelium small
blood vessels
Life threatening (especially non-immune)
– Pathophysiology
Mimics other infections
• Severe anemia
• Blockage of blood flow
• Systemic and local
cytokine production

Falciparum malaria: complications


• Cerebral malaria, hypoglycemia
• Pulmonary edema
• Renal failure, blackwater fever
• Jaundice, tender
hepatomegaly
• Diarrhea, dysentery
• Lactic acidosis
• Severe anemia
• Placental dysfunction

1432
Copyright © Harvard Medical School, 2018. All Rights Reserved.

40 year old man with fever:


• ICU: Intravenous quinidine
• CDC: artesunate (770-488-7788; -7100 after
hours)
• Artesunate 2.4 mg/kg i.v. at 0, 12, 24; 48 hr
• 24 hr: alert, oriented; parasitemia <1%
• 72 hours: parasitemia cleared
• Atovaquone 250 mg, proguanil 100 mg:
4 tablets daily x4
• Full recovery

Treatment of falciparum malaria


• Oral therapy
– Artemether-lumefantrine
– Alternatives:
• Chloroquine (sensitive strains)
• Quinine + doxycycline or
clindamycin
• Atovaquone-proguanil
• Severe falciparum malaria
– IV artesunate (CDC)
– Alt: IV quinidine gluconate +
doxycycline or clindamycin

1433
Copyright © Harvard Medical School, 2018. All Rights Reserved.

Treatment of falciparum malaria


• Oral therapy
– Artemether-lumefantrine
Artemether-lumefantrine
– Alternatives:
• Chloroquine (sensitive strains)
• Quinine + doxycycline or
clindamycin
• Atovaquone-proguanil
• Severe falciparum malaria
IVIV
– artesunate
artesunate (CDC)
(CDC)
– Alt: IV quinidine gluconate +
doxycycline or clindamycin

Artemisinin combination therapy


• Artemisinins: most rapidly acting antimalarials
• Active against resistant P. falciparum
• Improved survival compared with quinine
• Given in combination with second
drug (mefloquine, atovaquone-
proguanil, lumefantrine, others)
• Resistance spreading in SE Asia

Drugs of choice for treatment


of falciparum malaria
Cochrane Database Syst Rev. 2007 Oct 17;(4):CD005967; WHO Global malaria Program 2017

1434
Copyright © Harvard Medical School, 2018. All Rights Reserved.

Plasmodium vivax, ovale


• Infect only young cells
• No vascular sequestration
• Death unusual
Plasmodium vivax
• Relapses (4+ yrs)
• Treatment:
– Chloroquine* and
– Primaquine (prevent
relapses)
Plasmodium ovale
• 30 mg/day x 2 weeks
• G-6-PD screen
*Chloroquine-sensitive areas only

What prophylaxis do you recommend


for 59-year-old who will spend 10 days
traveling to Hanoi, Bangkok, and
Singapore for business?
• Weekly choloroquine
• Daily doxycycline
• Weekly mefloquine
• Daily atovaquone-proguanil (MalaroneR)
• Daily primaquine
• None of the above

1435
Copyright © Harvard Medical School, 2018. All Rights Reserved.

Malaria: chemoprophylaxis
• No chloroquine resistance:
weekly chloroquine
• Chloroquine-resistance:
– Doxycycline
– Atovaquone-proguanil
– Mefloquine (not border
areas in SE Asia)
• No chloroquine resistance
(predominantly P. vivax):
primaquine Mefloquine resistance

59-year-old will spend 10 days traveling


to Hanoi, Bangkok, and Singapore for
business
What do you recommend?
• Weekly choloroquine
• Daily doxycycline
• Weekly mefloquine
• Daily atovaquone-
proguanil (MalaroneR)
• Daily primaquine
• None of the above
Mefloquine resistance

1436
Copyright © Harvard Medical School, 2018. All Rights Reserved.

http://cdc.gov/malaria/map/

Province Prophylaxis for this State/Province/


Malaria in Province
Name District
None in the Red River
Hanoi delta and none in the Not applicable
city of Hanoi.

23-year old woman with fever and rash 3


days after returning from vacation in
Bahia, Brazil
• Abrupt onset: fever, chills, headache,
myalgia, arthralgia
• Temperature 102o F, diffuse
rash, lymphadenopathy
• WBC 3400 with normal
differential, platelets
120,000
Differential diagnosis?

1437
Copyright © Harvard Medical School, 2018. All Rights Reserved.

Diagnosis?
• Dengue • Enteroviruses
• Chikungunya • Rickettsial infection
• Zika • Gr A streptococcus
• Acute HIV • Syphilis
• EBV • Typhoid
• Measles • Leptospirosis
• Rubella • Drug reaction
• Parvovirus • Other

23 year old woman with fever and rash


• Clinical diagnosis: dengue
• Confirmed retrospectively with serology
(IgG, IgM)
• Afebrile on 7th day; prolonged fatigue for
several weeks

1438
Copyright © Harvard Medical School, 2018. All Rights Reserved.

Classic dengue
• Incubation period 1-7 days, up to 14+ days
• 5-7 days of fever; often biphasic
• Rash in ~50%: flushlike, later macular or
morbilliform
• Serologic tests negative during first 5-7 day
– CDC: symptoms < 5 days: PCR
– CDC: symptoms > 5 days: IgM antibody
test
• Dengue shock syndrome/dengue
hemorrhagic fever after second infections:
rare in travelers

Chikungunya Zika
• ‘That which bends up’ • Like dengue + joint
in the Kimakonde swelling, conjunctivitis
language of
Mozambique’ • Infection during
• Dengue-like illness pregnancy:
except >1/3 patients microcephaly, fetal brain
have incapacitating defects
arthralgia/arthritis
lasting months or years • Increase in Guillain-
Barre syndrome
• Sexual transmission

Clinical differentiation of dengue, zika,


and chikungunya, difficult: avoid NSAIDs

1439
Copyright © Harvard Medical School, 2018. All Rights Reserved.

Dengue 2018

Geographical limits
for year-around
survival of the
principle vector,
Aedes aegypti

Countries and territories Countries and territories


where chikungunya cases where Zika cases have
have been reported been reported

Areas with risk of Zika

CDC.gov Zika: Americas 2015-17 (www.paho.org)

1440
Copyright © Harvard Medical School, 2018. All Rights Reserved.

Approximate ranges of Aedes aegypti and A. albopictus


in the United States, 2017 (CDC)

N Engl J Med 2016. DOI: 10.1056/;CDC.gov; CDC and Denguevirusnet.com

1441
Copyright © Harvard Medical School, 2018. All Rights Reserved.

42 year old man with diarrhea after


travel to Thailand
• Chills, cramps, watery diarrhea on return
flight
• Self-treated with levofloxacin
• Markedly improved after 7 days; one loose
stool/day
• 24 hours after stopping levofloxacin: chills,
diarrhea, crampy abdominal pain

42 year old man with recurrent diarrhea


after travel to Thailand

Most likely diagnosis?

• Treatment failure
• Amebic dysentery
• Clostridium difficile colitis
• Falciparum malaria

1442
Copyright © Harvard Medical School, 2018. All Rights Reserved.

42 year old man with recurrent diarrhea


after travel to Thailand
• Fecal leukocytes: few/hpf
• Stool for ova and parasites: negative
• Assay for Clostridium difficile toxin: negative
• Stool culture: Campylobacter resistant to
quinolones

Treatment: azithromycin 1 gm po
Acute illness subsides, but 6 months later
he still has frequent loose stools, cramps

Persistent travelers’ diarrhea


• Diarrhea lasting > 2-3 weeks
• Six months following bout of diarrhea, 18%
experience chronic GI symptoms
• Three major categories:
– Infections
– Chronic gastrointestinal disease unmasked by
enteric infection (e.g., inflammatory bowel disease,
celiac sprue)
– Post-infectious processes

(Amer J Gastroenterol 2004;99:1774-8; Trop Dis Travel Med Vaccines 2017; 3:9)

1443
Copyright © Harvard Medical School, 2018. All Rights Reserved.

Persistent travelers’ diarrhea:


infectious causes
• Protozoan infections most common (Giardia,
Cryptosporidium, Cyclospora, Cystoisospora,
Entamoeba histolytica)
• Helminths less common: Strongyloides,
Schistosoma
• Bacteria: C. difficile,
Enteroadherent E. coli;
tropical sprue
• Multi-pathogen molecular
assays available CDC-DPDx

Persistent travelers’ diarrhea: post-


infectious causes
• Post-infectious malabsorption
– Lactase deficiency
– Bacterial overgrowth
• Post-infectious irritable bowel syndrome

Our patient: probable irritable bowel syndrome

1444
Copyright © Harvard Medical School, 2018. All Rights Reserved.

Diarrhea: strategies for travelers


Prevention Empirical treatment
• Food & beverages • Fluids
– “Boil it, cook it, peel it or • Loperamide 4 mg x1, then 2
forget it!” mg for each loose stool up
– Easier said than done to 16 mg/day; not in
• Hand hygiene dysentery)
• Chemoprophylaxis (high • Bismuth subsalicylate
risk travelers) +/-
– Bismuth subsalicylate • Antibiotics (severe cases)
(65% protection) – Azithromycin 1gm x1;
– Rifaximin (70-80% 500 mg x 1 + loperamide;
protection) 500 mg x 3days
– Fluoroquinolones – Nondysenteric cases
(effective, not Rifaximin 200 mg TID x3
recommended) days (max); fluoro-
– ETEC vaccines quinolones x1 or x3d

J Travel Med 2017; 24:s57, Am J Gastroenterol 2016, 111:602

In South Asia, 80% of travelers who


received antibiotics for treatment of
travelers’ diarrhea became colonized
with ESBL-producing bacteria
Clin Infect Dis 2015; 60:837

1445
Copyright © Harvard Medical School, 2018. All Rights Reserved.

Subclinical chronic infections of


immigrants and returning long-term
expatriates
• Tuberculosis
• HIV
• Hepatitis B
• Chagas’ disease
• Strongyloidiasis
• Schistosomiasis
• Cysticercosis

Case history
Recent immigrant from rural
El Salvador receives a letter
after donating blood at his
local blood bank. His blood
tested positive for infection with
Trypanosoma cruzi.

He has no symptoms. Physical examination


is negative. CBC, diff, LFTs are all normal
Does he have Chagas disease?
Does he need further evaluation or treatment?

1446
Copyright © Harvard Medical School, 2018. All Rights Reserved.

Chagas disease (American trypanosomiasis)


• ~250,000 immigrants with
T. cruzi infection in US
• Most unaware of infection
• Most US blood banks
screening blood donors
• Diagnosis Triatomine bug
– History of exposure: Latin
America, not Caribbean;
poor housing; history of
blood transfusion; infected
mother or sibling
– Serology: 2 or more tests
(IFA, ELISA, Western blot,
etc.)

Chronic Chagas disease


– 70-80%: subclinical
– 20-30%: decades later
• Chronic cardiomyopathy:
CHF, arrhythmias (sudden
death), heart block,
thromboembolism; RBBB
• Mega-esophagus, -colon
• Reactivation in HIV-
infected, other immune
suppressed: CNS lesions
mimic toxoplasmosis, acute
myocarditis
– Congenital transmission
– Treatment: nifurtimox (CDC),
benznidazole (FDA-approved) Megaesophagus

1447
Copyright © Harvard Medical School, 2018. All Rights Reserved.

32 y.o. man from Ecuador with renal failure


• Living in US for 8 years
• Evaluation for kidney transplant
– Epigastric pain-? gastritis
– WBC 7900, 9% eosinophils
• 6 weeks after renal allograft:
– Temp 102.5o
– Abdominal pain
– Diarrhea (heme +)
– Blood cultures+ for E. coli
– Broad spectrum antibiotics
- Progressive respiratory failure

32 y.o. man from Ecuador with renal failure


• Living in US for 8 years
• Evaluation for kidney transplant
Diagnosis?
– Epigastric pain
– WBC 7900, 9% eosinophils
• 6 weeks after renal allograft:
– Temp 102.5o
– Abdominal pain
– Diarrhea (heme +)
– Blood cultures+ for E. coli
– Broad spectrum antibiotics
- Progressive respiratory failure

1448
Copyright © Harvard Medical School, 2018. All Rights Reserved.

Bronchoscopy
• Filariform larvae of
Strongyloides
stercoralis
• Immunosuppression
held
• Daily ivermectin per
N-G tube
• E.coli meningitis
• Post-mortem: larvae
throughout gut,
lungs, brain, heart

Strongyloides stercoralis
• Able to complete life cycle in host
without exogenous reinfection
– Infection life-long
– Eosinophilia in >75%
– Potential for life threatening
hyperinfection/dissemination
(corticosteroids, HTLV-1>>>HIV,
malnutrition)
• Diagnosis: microscopic examination
(stool, secretions, CSF), serology
• Treatment: ivermectin

1449
Copyright © Harvard Medical School, 2018. All Rights Reserved.

25 year old woman with seizure


• Spent a semester in Ghana
• Avoided swimming in fresh water
• 4 years later: seizure
• MR of brain: suspicious for tumor
• Craniotomy
• After diagnosis made
recalled walking barefoot
along the side of a river
one afternoon

Schistosoma mansoni

Schistosomiasis
• Contact with fresh water infested
with snail intermediate host
• Chronic disease: response to eggs
in tissue: acute inflammation, Schistosoma mansoni
granulomas, fibrosis
• Multiple complications including
ectopic egg deposition with
disastrous complications (spinal
cord, central nervous system)
• Diagnosis:
– Eggs in stool/urine
– Serology
• Treatment: praziquantel

1450
Copyright © Harvard Medical School, 2018. All Rights Reserved.

Cysticercosis (Taenia solium)


– Ingestion of pork tapeworm
eggs (not pork)
– Most common presentation:
seizures several years after
infection due to inflammation
around degenerating cysts

Cysticercosis (Taenia solium)


• Diagnosis:
– CT or MR pathognomonic if
scolex visualized
– ELISA and immunoblot (CDC)
• Treatment (when indicated):
– High dose albendazole or praziquantel
– Funduscopic examination
– Corticosteroids, anticonvulsants
– Surgery, shunts
• Search for tapeworm carrier in household

1451
Copyright © Harvard Medical School, 2018. All Rights Reserved.

Skin lesions in returning travelers


• Teenager with an itchy rash
• Developed after 10-day vacation in Trinidad
• Worsened despite 7-days of amoxicillin

Diagnosis?

1452
Copyright © Harvard Medical School, 2018. All Rights Reserved.

Cutaneous larva migrans (creeping eruption)

Cutaneous larva migrans


(creeping eruption)
• Infection with dog or cat hookworms
(Ancylostoma braziliensis, others)
• Contact of bare skin with contaminated,
usually sandy, soil
• Diagnosis: clinical
• Treatment
– ivermectin 200 mcg/kg PO QD x1-2 days
– albendazole 400 mg PO QD x3 days

1453
Copyright © Harvard Medical School, 2018. All Rights Reserved.

Cutaneous lesions in returning travelers


• Cutaneous larva migrans — 25 percent
• Pyoderma — 18 percent
• Arthropod-reactive dermatitis —10 percent
• Myiasis — 9 percent
• Tungiasis — 6 percent
• Urticaria — 5 percent
• Fever and rash — 4 percent
• Cutaneous leishmaniasis – 3 percent
Clin Infect Dis. 1995;20(3):542

Healthy 47-year-old man with skin lesions.


• One week earlier he left West Africa
• 5 days in five-star hotel, ate only hotel food,
slept in an air-conditioned room with the
windows closed
• He walked on the beach outside the hotel,
swam in the chlorinated hotel pool, and
lounged on wooden cots near the pool
• No history of insect bites, but sensation of
bites and small red bumps on his buttocks
and thighs

1454
Copyright © Harvard Medical School, 2018. All Rights Reserved.

• On flight to US: numerous


boils several centimeters
in diameter; unable to sit
because of pain
• Back in US, the lesions
increasing in size in front
of his eyes
• In EW reports things
moving inside, blowing
bubbles in bath tub

Diagnosis?

Furuncular myiasis
(infestation with maggots--fly larvae)

• Tumbu fly: (Cordylobia


anthrophaga): sub-Saharan Africa
– Eggs deposited on sandy soil
or clothes contaminated with urine or sweat
– Larvae hatch, penetrate skin to subcutaneous
tissue where they feed and grow
– Larval chamber becomes boil-like and inflamed
– 10-14 days later mature larvae falls out of lesion,
pupates on ground
• Human botfly (Dermatobia hominis): Latin America
• Treatment: extraction (squeezing, suffocation;
excision)

1455
Copyright © Harvard Medical School, 2018. All Rights Reserved.

60-year old biologist: painless, slowly


expanding ulcer for 2 months after
returning from forest in Peru

Cutaneous leishmaniasis
(vector: sand flies)

1456
Copyright © Harvard Medical School, 2018. All Rights Reserved.

Leishmaniasis (Leishmania braziliensis)


• Diagnosis: biopsy,
scrapings, aspirate for
intracellular organisms
• Treatment: lipid-
associated amphotericin
compounds, parenteral
pentavalent antimony,
miltefosine
• Complication: mucosal
leishmaniasis

Take home points


• Include travel-related infections when formulating
differential diagnoses
• Falciparum malaria can be rapidly fatal and should be
ruled out in all febrile patients returning from malaria-
endemic areas regardless of clinical presentation
• Infectious diseases among immigrants and short-term
travelers often differ; important infections of
immigrants are often subclinical at the time of entry
• Most travel-associated infectious diseases can be
prevented by inquiring about patients’ travel plans
frequently and providing advice, vaccines and
prophylactic medication

1457
Copyright © Harvard Medical School, 2018. All Rights Reserved.

A young man returns from Haiti with fever and


headache. His temperature is 103o F but he
appears well. Laboratory studies show mild
anemia, thrombocytopenia, and normal renal
function . A blood smear shows a few rings
of Plasmodium falciparum.

Treatment of choice: a. Chloroquine


b. Mefloquine
c. Quinine + doxycycline
d. Quinidine +clindamycin
e. Artemether /lumefantrine

Treatment of choice: a. Chloroquine


b. Mefloquine
c. Quinine + doxycycline
d. Quinidine +clindamycin
e. Artemether /lumefantrine
*
Artemisinin combination therapy is the
treatment of choice for all cases of falciparum
malaria because of more rapid action and
increased survival compared to other agents.

1458
Copyright © Harvard Medical School, 2018. All Rights Reserved.

Failure to treat which of the following can lead


to death from overwhelming infection in a
person receiving high doses of corticosteroids?

a. Falciparum malaria
b. Babesiosis
c. Giardiasis
d. Strongyloidiasis
e. Schistosomiasis

Failure to treat which of the following can lead


to death from overwhelming infection in a
person receiving high doses of corticosteroids?
a. Falciparum malaria
b. Babesiosis
c. Giardiasis
d. Strongyloidiasis *
e. Schistosomiasis
Splenectomy (not steroid therapy is associated
with overwhelming malaria and babesiosis;
steroids accelerate strongyloides replication;
schistosomes do not replicate in the host.

1459
Copyright © Harvard Medical School, 2018. All Rights Reserved.

References
• CDC: Travelers’ Health
– www.cdc.gov/travel
– CDC “Yellow Book”
• World Health Organization
– www.who.int/int
– WHO “Green Book”
• Ashley EA et al. Malaria. Lancet 2018;391:1608-21.
• Paixão ES, et al Zika, chikungunya and dengue: the
causes and threats if new and re-emerging arboviral
diseases. BMJ Glob Health 2018;3:suppl 1.
• Riddle MS, et al. Guidelines for the prevention and
treatment of travelers’ diarrhea: a graded expert panel
report. J Travel Med 2017; 24:s63-80.
• Thwaites GE, Day NPJ. Approach to fever in the
returning travelers. N Engl J Med 2017; 376:548-60.

1460
Copyright © Harvard Medical School, 2018. All Rights Reserved.

Tuberculosis for the Non-ID Specialist


Gustavo E. Velásquez, MD, MPH

Associate Physician
Division of Infectious Diseases, Department of Medicine
Brigham and Women’s Hospital

Instructor in Medicine
Research Associate in Global Health and Social Medicine
Harvard Medical School

No financial disclosures

1461
Copyright © Harvard Medical School, 2018. All Rights Reserved.

Objectives
• Review diagnostic approaches for latent
tuberculosis infection (LTBI)
• Review treatment of LTBI
• Approved regimens
• Side effects, monitoring, contraindications
• Prevention/management of complications
• Review new approaches to the diagnosis and
management of active tuberculosis

Tuberculosis burden
• 1/3 of the world’s population is infected
• ~10.4 million new cases each year
• ~1.6 million deaths last year
• The 9th leading cause of death worldwide
• The leading cause of death from a single
infectious agent (ranking above HIV/AIDS)
• The leading cause of death among PLHIV

WHO/HTM/TB/2017.23

1462
Copyright © Harvard Medical School, 2018. All Rights Reserved.

WHO/HTM/TB/2017.23

Tuberculosis progress

1463
Copyright © Harvard Medical School, 2018. All Rights Reserved.

Clinical scenarios
• Latent TB
• Suspected active TB
• Active TB
• Special scenarios

Diagnosis and treatment of


latent tuberculosis

1464
Copyright © Harvard Medical School, 2018. All Rights Reserved.

What is latent TB?

Latent Active
Sterile Quiescent Subclinical Percolator Chronic Pulmonary Fulminant

Pathologic

Protective

Pathologic

Skin testing for TB (TST)


• PPD (purified protein derivative) contains multiple
antigens
• Sensitized inflammatory cells migrate to the site
causing a delayed-type hypersensitivity reaction
• Sensitivity and specificity impossible to determine
• A positive reaction is usually indicative of current
infection
• Sometimes negative even in active disease;
therefore, not a good diagnostic aid

1465
Copyright © Harvard Medical School, 2018. All Rights Reserved.

Who should receive TST testing?


• High-risk individuals
• Close contacts of active TB patients
• Immigrants
• HIV or other immunosuppression
• IV drug use
• Abnormal CXR
• Healthcare workers

What is a positive TST?


• 5 mm
• HIV, close contacts of TB cases
• 10 mm
• Other high-risk individuals
• 15 mm
• Everyone else
• (0 mm)
• Immunocompromised high-risk close contacts

1466
Copyright © Harvard Medical School, 2018. All Rights Reserved.

Lewinsohn et al. Clin Infect Dis 2017;


64(2):e1–e33

BCG and TST


• BCG administration can lead to positive TST
• The risk of positive TST resulting from BCG
varies with administration schedule
• Infancy vs. adolescence
• However, the rate is low and most positive
TSTs represent TB infection even in
vaccinated individuals

1467
Copyright © Harvard Medical School, 2018. All Rights Reserved.

Interferon γ release assays


(IGRAs)
• Measure memory response to TB and M.
avium antigens using stimulation of
lymphocytes to produce interferon
• Only sensitized (i.e., immune individuals)
produce interferon
• Two formats
• ELISPOT (T-SPOT.TB)
• ELISA (QuantiFERON-TB Gold)

Sensitivity/specificity of IGRAs
• Current generation of tests (Quantiferon-TB
Gold, T-SPOT.TB) utilize antigens found in M.
tuberculosis but not M. bovis BCG
• Assays have similar sensitivity/specificity to
TST for diagnosis of LTBI

1468
Copyright © Harvard Medical School, 2018. All Rights Reserved.

Interferon γ release assays


(IGRAs)
• Advantages (over TST)
• Testing can be completed in a single visit
• Not subjective
• No interference from previous BCG vaccination
• Disadvantages
• Cost
• Availability
• Logistics
• Current CDC recommendation is that IGRA can be
used in all settings where TST is currently used

When to perform IGRA over TST?


• 5 years or older
• Are likely to be infected with Mtb
• Low or intermediate risk of disease progression
• Testing for LTBI is warranted
• Either have
• A history of BCG vaccination OR
• Are unlikely to return to have their TST read
(TST acceptable if IGRA not available, too costly, or
too burdensome)
Lewinsohn et al. Clin Infect Dis 2017;64(2):e1–e33

1469
Copyright © Harvard Medical School, 2018. All Rights Reserved.

Treatment options for LTBI


• 9H: INH x 9 months (with B6)
• 6H: INH x 6 months (with B6)
• 4R: Rifampin x 4 months
• 2RZ: Rifampin + Pyrazinamide x 2 months (*high
risk of hepatotoxicity*)
• 3HP: INH + Rifapentine x 3 months (with B6)

• No age cutoff (except for 3HP if aged <2 years)


• Must be evaluated for active TB
• Generally no need to retreat if previously treated

INH + Rifapentine (3HP)


• Supervised regimen of INH + RPT + B6, one
weekly dose for 12 weeks
• Equivalent efficacy to INH alone (6H or 9H)
• Currently recommended by CDC as an
alternative to INH or RIF monotherapy
• Cannot be used in pregnancy or age <2 years
• Now recommended with DOT or self-
administered therapy (SAT)!
• Belknap et al. Ann Intern Med 2017;
167(10):689–97

1470
Copyright © Harvard Medical School, 2018. All Rights Reserved.

Hot new CDC guidelines


• 3HP recommended for LTBI in 2011
• Additional indications in 2018
– In persons with LTBI aged 2-17 years
– In persons with LTBI with HIV infection
including AIDS, and taking ARVs with acceptable
drug-drug interactions with RPT
– By DOT or self-administered therapy (SAT) in
persons aged ≥ 2 years

Borisov AS et al. Morb Mortal Wkly Rep. 2018;67(25):723–6

When to perform baseline LFTs?


• Underlying liver disease (HBV, HCV, EtOH,
cirrhosis)
• Pregnant and within 3 months postpartum
• Regular EtOH use
• Concomitant medications with potential
hepatotoxicity

1471
Copyright © Harvard Medical School, 2018. All Rights Reserved.

Stopping rules based on LFTs


• Symptomatic and transaminases >3x ULN
• Asymptomatic and transaminases >5x ULN

• If any of the above, do not rechallenge!

Diagnosis and treatment of


suspected active TB

1472
Copyright © Harvard Medical School, 2018. All Rights Reserved.

AFB smear
• Rapid, first bacteriologic evidence of
mycobacterial disease
• Quickly identifies infectious “spreaders” of
TB
• Stained smears may be used as a rapid
monitor of patient response to treatment

Drawbacks of AFB smear


diagnosis
• Least sensitive detection method. Detects
10,000 AFB per mL of sputum while culture
detects 10-100 AFB per mL
• Is not specific for M. tuberculosis. Other
organisms such as Nocardia spp. may also
be acid-fast

1473
Copyright © Harvard Medical School, 2018. All Rights Reserved.

Mycobacterial culture
• More sensitive and specific for TB diagnosis
than AFB smear
• True sensitivity and specificity is difficult to
calculate due to culture negative TB:
– 80 - 90% of cases reported in the US are
culture proven
– Specificity = 98% (false-positives due to
contamination)
• Major limitation - Slow

“Rapid” culture of TB
• Liquid culture methods (e.g., BACTEC,
MGIT) - 7-12 days
• Diagnosis in fluids other than sputum
remains problematic

1474
Copyright © Harvard Medical School, 2018. All Rights Reserved.

ATS/IDSA/CDC guidelines
• If suspect pulmonary TB
• AFB smear should be performed
• 3 specimens strongly recommended and
improve sensitivity
• At least 3mL sputum volume, optimally 5-10mL
• Concentrated specimens and fluorescence
microscopy preferred
• Both solid and liquid AFB cultures for every
specimen
• NAAT should be done on the initial respiratory
specimen
Lewinsohn et al. Clin Infect Dis 2017;64(2):e1–e33

Alternative diagnostics
• Several groups are trying to develop rapid
diagnostics based on detecting bacterial
components
– Lipoarabinomannan (LAM)
– Volatile lipids
– Nucleic acid amplification

1475
Copyright © Harvard Medical School, 2018. All Rights Reserved.

Urinary lipoarabinomannan
• LAM is an abundant cell wall glycolipid
• LAM is shed in the urine of many infected
patients
• Can be detected using a urine dipstick
• Early results suggest good specificity,
relatively high sensitivity even in HIV
patients

Volatile lipids
• Metabolism in M. tuberculosis produces
distinctive lipid molecules some of which
are volatile
• These can be detected in exhaled breath
by:
– Mass spectrometry
– “Electronic nose”
– Giant rats

1476
Copyright © Harvard Medical School, 2018. All Rights Reserved.

GeneXpert MTB/RIF Performance


• Very high specificity for TB
• Very high sensitivity for smear-positive
disease
• Sensitivity 50-60% for smear-negative
disease but improves with repeated
samples
• Excellent rates of detection for rifampin
resistance

1477
Copyright © Harvard Medical School, 2018. All Rights Reserved.

Guidelines for treatment of


smear-negative suspected TB
• Start treatment while awaiting results of
culture
• When culture results become available
– Continue if culture positive
– Continue if response to treatment or high
suspicion
– Discontinue if no response and low suspicion

Treatment of active TB

1478
Copyright © Harvard Medical School, 2018. All Rights Reserved.

TB pneumonia treatment
• Current CDC recommendation:
– INH x 6 months (add pyridoxine)
– Rifampin x 6 months
– Ethambutol x 2 months
– Pyrazinamide x 2 months

Nahid P et al. Clin Infect Dis. 2016 Oct 1;63(7):e147–95

Continuation phase
• INH + Rifampin for 4 months
• Intermittent dosing (2-3 doses/week) with
DOT is possible, generally avoid if:
– HIV co-infection
– Smear positive
– Cavitary disease
• Major consideration is adherence

Nahid P et al. Clin Infect Dis. 2016 Oct 1;63(7):e147–95

1479
Copyright © Harvard Medical School, 2018. All Rights Reserved.

Why so many drugs?


• Drug resistance occurs at a rate of 1/105 -
1/109
• Cavitary lesions contain from 109 - 1011
organisms per gram of tissue
• Drug resistance can either be primary or
arise de novo during treatment

Treatment modifications
• INH intolerance
– Add PZA for the duration of treatment
• PZA intolerance
– Continue therapy for 9 months
• RIF or multiple drug intolerance
– Expert consultation
• HIV
– No change, but beware of interactions
between rifamycins and antiretrovirals
(and rifabutin is less effective than rifampin)

1480
Copyright © Harvard Medical School, 2018. All Rights Reserved.

Directly observed therapy


• Standard in most of the world
• Local standards vary throughout US
• Massachusetts takes a graded approach
ranging from unsupervised drug
administration to institutionalization

Practical treatment options


• Write prescriptions
• Refer to public health clinic
• Refer for inpatient therapy

1481
Copyright © Harvard Medical School, 2018. All Rights Reserved.

Public health considerations


• Only active disease is transmissible
• The frequency of transmission is roughly
proportional to the burden of organisms in
expectorated sputum
• Transmission is largely restricted to
contacts of smear positive patients
• All cases should be reported for contact
tracing

Should patients be isolated


(Massachusetts)?
• Adherent patients can go home
• Contacts should be screened
• Patients with positive smears are
considered infectious
• Most smear positive patients will convert
to smear negative after 2 weeks of
treatment

1482
Copyright © Harvard Medical School, 2018. All Rights Reserved.

Special scenarios
• High risk groups
• Drug-resistant tuberculosis

Who is at risk for developing


disease?
• Increased exposure
• Decreased cell-mediated immunity
– Congenital immune suppression
– Cancer, chemotherapy
– Pregnancy
– HIV - mechanism unknown but striking
increase in susceptibility

1483
Copyright © Harvard Medical School, 2018. All Rights Reserved.

Clinical presentation of TB in HIV


CD4 Disease
• PLHIV are more High Lung
susceptible to both
primary disease and
reactivation at all CD4
levels
• Patients with AIDS are
much more likely to
develop
extrapulmonary
disease
Low Other

TB therapy in HIV
• Rifampin has significant interactions with many
drugs, particularly protease inhibitors
• Can consider rifabutin, but less favorable
pharmacokinetics
• Timing of therapy remains poorly defined
• Regimens that do not contain rifamycins could
help considerably

1484
Copyright © Harvard Medical School, 2018. All Rights Reserved.

TNF and tuberculosis


• Mice with mutations in TNF or its receptor
are very susceptible to TB
• Patients receiving TNF blocking agents
develop reactivation TB at very high rates
– Infliximab (Remicade) seems to produce a
higher risk than Etanercept (Enbrel)

Vaccination against TB
• BCG vaccine derived from Mycobacterium
bovis by years of serial passage
• In use in much of the world since the
1930’s
• Studies of efficacy show variable results;
however, not very effective against adult
TB

1485
Copyright © Harvard Medical School, 2018. All Rights Reserved.

Multidrug resistance
• Defined as resistance to INH and Rifampin
• Treatment requires expert consultation

New antibiotics
Bedaquiline

Delamanid Pretomanid

1486
Copyright © Harvard Medical School, 2018. All Rights Reserved.

New drugs in MDR-TB


• Both delamanid and bedaquiline have been
tested in MDR-TB
• Both randomized controlled trials of best
regimen vs. best regimen plus new agent

Delamanid Bedaquiline

Delamanid and Bedaquiline

1487
Copyright © Harvard Medical School, 2018. All Rights Reserved.

But a big red flag

Bedaquiline black box warning

1488
Copyright © Harvard Medical School, 2018. All Rights Reserved.

Attempts to shorten therapy

Nimmo C et al. Lancet Infect Dis. 2015 Feb;15(2):141–3

Summary
• New diagnostics and treatment options are
now available for latent TB
• New diagnostics for active TB will likely
soon be available
• While promising, new treatment options
for active TB are several years away

1489
Copyright © Harvard Medical School, 2018. All Rights Reserved.

Question 1
• Which of the following are acceptable
treatments for latent tuberculosis infection
a. Isoniazid and B6 daily for 5 months
b. Rifampin daily for 4 months
c. Isoniazid and Rifapentine and B6 daily for 12 weeks
d. All of the above
e. None of the above

Question 1
• Which of the following are acceptable
treatments for latent tuberculosis infection
a. Isoniazid and B6 daily for 9 months
b. Rifampin daily for 4 months
c. Isoniazid and Rifapentine and B6 weekly for 12 weeks
d. All of the above
e. None of the above

1490
Copyright © Harvard Medical School, 2018. All Rights Reserved.

Question 2
• Which of the following is true of treatment for
active tuberculosis
a. Treatment should be initiated after the results of
antibiotic sensitivity tests are available
b. Pregnant women should not receive isoniazid
c. Antibiotic choice or duration should be altered in the
presence of drug resistance
d. Antibiotic resistance is found only in those previously
treated with drugs
e. All of the above

Question 2
• Which of the following is true of treatment for
active tuberculosis
a. Treatment should be initiated before the results of
antibiotic sensitivity tests are available
b. Pregnant women may receive isoniazid
c. Antibiotic choice or duration should be altered in the
presence of drug resistance
d. Antibiotic resistance is found not only in those
previously treated with drugs
e. All of the above

1491
Copyright © Harvard Medical School, 2018. All Rights Reserved.

References
• World Health Organization. Global Tuberculosis Report 2016. Geneva, Switzerland: World
Health Organization; 2017. Report No.: WHO/HTM/TB/2017.23. Available at:
http://www.who.int/tb/publications/global_report/en/
• Lewinsohn DM, Leonard MK, LoBue PA, Cohn DL, Daley CL, Desmond E, et al. Official
American Thoracic Society/Infectious Diseases Society of America/Centers for Disease
Control and Prevention Clinical Practice Guidelines: Diagnosis of Tuberculosis in Adults and
Children. Clin Infect Dis. 2017 Jan 15;64(2):e1–e33. Available at:
https://academic.oup.com/cid/article-pdf/64/2/e1/13132770/ciw694.pdf
• Nahid P, Dorman SE, Alipanah N, Barry PM, Brozek JL, Cattamanchi A, et al. Official
American Thoracic Society/Centers for Disease Control and Prevention/Infectious Diseases
Society of America Clinical Practice Guidelines: Treatment of Drug-Susceptible
Tuberculosis. Clin Infect Dis. 2016 Oct 1;63(7):e147–95. Available at:
https://academic.oup.com/cid/pdf-lookup/63/7/e147
• Borisov AS, Bamrah Morris S, Njie GJ, Winston CA, Burton D, Goldberg S, et al. Update of
Recommendations for Use of Once-Weekly Isoniazid-Rifapentine Regimen to Treat Latent
Mycobacterium tuberculosis Infection. Morb Mortal Wkly Rep. 2018;67(25):723–6. Available
at: https://www.cdc.gov/mmwr/volumes/67/wr/mm6725a5.htm

1492
Copyright © Harvard Medical School, 2018. All Rights Reserved.

Update on Adult Immunizations


2018
Lindsey R. Baden, MD
Division of Infectious Diseases
Brigham and Women’s Hospital
Dana Farber Cancer Institute
Harvard Medical School

Disclosures
• None

1493
Copyright © Harvard Medical School, 2018. All Rights Reserved.

Greatest Medical Triumph of


the 20th Century
Annual 20thCentury 1998 Morbidity % Decreased
Morbidity
Smallpox 48164 0 100
Diphtheria 175885 1 100
Pertusis 147271 7405 95
Tetanus 1314 41 97
Paralytic Polio 16316 1 100
Measles 503282 100 100
Mumps 152209 666 >99
Rubella 47745 364 >99
HiB 20000 61 >99
Total 1112186 8639 >99

AAP, Red Book: 2000, 2012

Types of Vaccines
Live Attenuated Killed Whole Purified Protein or Genetically
Organism Polysaccharide Engineered
Smallpox, 1798

Rabies, 1885 Typhoid, 1896


Cholera, 1896
Plague, 1897

BCG, 1927 Pertussis, 1926 Diphtheria, 1923


Yellow fever, 1935 Influenza, 1936 Tetanus, 1927
Rickettsia, 1938
After World War II (advent of cell culture)
Polio Polio Pneumococcus Hepatitis B
Measles Rabies Meningococcus Pertussis
Mumps Japanese enceph HiB
Rubella Hepatitis A Hepatitis B
Adenovirus Tick-borne enceph
Ty21a Typhoid (Vi)
Varicella Pertussis
Rotavirus

Plotkin and Ornstein, Vaccine: 1999, 2012

1494
Copyright © Harvard Medical School, 2018. All Rights Reserved.

Recommended Adult Immunization Schedule


by vaccine and age group

MMWR 2018

Recommended Vaccinations Indicated for Adults


based on medical and other indications

MMWR 2018

1495
Copyright © Harvard Medical School, 2018. All Rights Reserved.

Spacing the Administration of


Killed and Live Antigens
• > 2 Killed antigens
– May be given at any interval between doses
• Killed and live antigens
– May be given at any interval between doses
– Exception: cholera and yellow fever
• > 2 Live antigens
– 4 weeks if not given simultaneously
– Exception: OPV in relation to MMR or Ty21a

MMWR 2011;60(RR-2):1-64

Mitigating Factors to Response


• Preformed antibodies
– IVIg, blood, plasma, and platelet products
• MIG, TIG, VZIG, RIG, HBIG, RSV-Ig, anti-Rh(D)
– Impairs live viral replication
• Varicella, measles >rubella, mumps >>yellow fever (YF),
typhoid
– Recommendations
• Ig and killed antigen: no time
• Ig and live vaccines (depends on dose of IG)
– Ig live: none-OPV, YF, Ty21a; 3m–mumps, RA27/3; 5m–
measles,VZV
– Live Ig: 2wk – MMR; 3wk VZV
– Consider follow-up serologic testing
• Antibiotic use
– Live bacterial vaccines (e.g., Ty21a)

1496
Copyright © Harvard Medical School, 2018. All Rights Reserved.

Contraindications
• Bleeding risks (e.g., IM injections)
– Hemophilia, anti-coagulation
• Immunosuppressed
– Medications, pregnancy, HIV, transplantation
• Infection, rejection, diminished responsiveness
• Live antigens: MMR, YF, OPV, VZV
• Fetal risk: rubella, VZV (registry: 800-986-8999)
• Household contacts: OPV, ??VZV
• Prednisone (>20mg qd for >14 days, wait 1m post discontinuation)
• Hypersensitivity
– Egg: influenza, YF, measles, mumps
– Gelatin: MMR, YF, VZV
– Abx: neomycin, streptomycin, or polymyxin B
– Thimerosal: DTP, HBV, influenza, Japanese encephalitis
• Current moderate/severe illness

Specific Vaccines

• dT, Tdap
• MMR
• Influenza
• Pneumoccocus, Menningococcus, HiB
• Varicella and Zoster
• HAV, HBV
• HPV

1497
Copyright © Harvard Medical School, 2018. All Rights Reserved.

dT Tdap
• Dosing
• Boost every 10 years
• 5 years if high risk behaviors anticipated
• Who: all adults
• 45-65 cases tetanus annually in US w/ 60% in adults
• Serologic studies show that >40% and 40-80% of 60+yo and
11% and 62% of 18-39yo w/o neutralizing antibody to tetanus
and diphtheria respectively
• PEP: Wound management
• History of <3 doses adsorbed tetanus toxoid
– Clean, minor wounds: dT
– Other wounds: TIG (250 U IM) and dT
• History of >3 doses
– Clean, minor wounds: boost if >10 years since last dT
– Other wounds: boost if >5 years

MMWR:Vol 55 RR-17; Dec 15, 2006

1498
Copyright © Harvard Medical School, 2018. All Rights Reserved.

Pertussis
N=2781, 1:1 randomization
Acellular pertussis vaccine
Median 22 months follow-up
0.7 to 5.7% of illness due to
pertussis
Incidence of pertussis 370-450
per 100,000 pt-years
Vaccine 92% effective

Ward et al. NEJM 2005;353:1555

Tdap
• 2005 two formulations approved in US
– ADACEL (sanofi pasteur); licensed 7/10/05; single booster; 11-64 year
olds. 0.5 mL IM
– BOOSTRIX (GSK); single booster; 10-18 year olds
• Recs
– 19-64 yr olds -- use as next tetanus booster
• Can be given as close as 2 years from last boost
• Priority to boost following groups
– Close contacts of infants/elderly and health care workers
– Not licensed for decennial re-booster
– Contraindications
• Allergy to prior tetanus vaccination;
• Adults with history of unexplained encephalopathy w/in 7 days of a vaccine
with a pertussis component;
• ?GBS w/in 6 weeks of prior tetanus vaccination;
• ?Ongoing/evolving neurologic condition

MMWR:Vol 55 RR-17; Dec 15, 2006

1499
Copyright © Harvard Medical School, 2018. All Rights Reserved.

Patients with Measles According to the Day of Onset of Rash, Indiana, May to June 2005

Parker A N Engl J Med 2006;355:447-455

Estimated Direct Costs of Containing Measles during the Outbreak in Indiana

Parker A. N Engl J Med 2006;355:447-455

1500
Copyright © Harvard Medical School, 2018. All Rights Reserved.

Fluctuation in the Number of Persons Susceptible to Measles in a Community Where


Measles Virus Is Circulating

Mulholland E. N Engl J Med 2006;355:440-443

MMR I
• General
• Contains neomycin and gelatin, made from chick-embryo cell
culture, live attenuated
• Mumps (Jeryl Lynn strain)
• ~78% w/ Ab after 1 dose; ~88% (range 66-95%) after 2 doses
• AEs: rarely orchitis, parotitis
• Before vaccination, in 1967, about 186,000 cases/yr
– In 2012 - 229 cases
– As of June 6, 2016- 1,272 cases in 33 states (>100 in MA)
• Rubella (RA 27/3)
• >95% w/Ab after 1 dose
• About 10% of young adults are not immune
• AEs: Fever (5-15%), rash (5%), joint pain (up to 25% of young
women 7-21d post vaccination)
• Avoid in pregnancy

1501
Copyright © Harvard Medical School, 2018. All Rights Reserved.

MMR II
• Measles
– 95-98% w/ Ab after 1st dose, >99% post 2nd dose
• 1963-7 inactivated vaccine – atypical measles
– Recent IgG use interferes w/ immunogenicity
– AEs: 5-15% w/ T>103 5-12d post vaccination, CNS
<1/million doses, decreased plat seen 2-3 wks post
vaccination, may decrease ppd reactivity for 4-6 weeks
– Who
• Everyone born after 1956, colleges typically require
documentation of 2 dose
• Recommended for HIV+ patients if not severely
immunosuppressed
– PEP: Can vaccinate w/in 72h of exposure, also
consider Ig (0.25 – 0.5 mL/kg, max 15 mL) w/in 6d

Influenza: 3 vs 4 valent
• Inactivated, multivalent (tri: 2 A’s and 1 B and now quad:
2 A’s and 2 B’s) adapted periodically to the strains
expected to circulate in the winter (A/H1N1, A/H3N2, B-Victoria
+/-Yamagata)
– 1 dose annually in autumn
– Efficacy: 56% resp illness, 50% pneumonia hosp, 68% death
• Who -ALL
– Age>50, CRF, DM, cardiac dz, pulm dz, HIV,
immunosuppressed, nursing home residents
– Health care workers, household contacts of at risk patients
• AEs: local reaction 10%
– Caution in the setting of egg allergy
• PEP: consider chemoprophylaxis (rimant/amant, oseltamivir/zanamivir)

MMWR 62 (RR-07):1-
43;2013

1502
Copyright © Harvard Medical School, 2018. All Rights Reserved.

FluMist
Not recommended for 2017-18 season

• Cold adapted, live attenuated, intranasal, 2.5ml/nares


• Decrease febrile URI 24%, HCW visit 18-37%, Abx use
41-45%
• FDA approved (6/03) for healthy people, age 5-49
• AE: runny nose (45%), sore throat (28%), tiredness
(26%), cough (14%), chills (9%)
• Contraindications: >50yo, pregnancy, egg allergy, h/o
GBS, immunocompromised, recent resp illness (72h), or
those w/cardiac, pulmonary, metabolic, or renal dz
• Possible transmission of vaccine strain thus avoid in
those who are in contact w/ immunosuppressed patients
• Avoid antiviral therapy for 2 weeks
• Expensive
MMWR 62 (RR-07):1-43;2013

Pneumococcus
• Pneumovax
– 23 valent polysaccharide vaccine
• 88% of strains causing bacteremia/menningitis
– 0.5mL IM (25ug of each polysaccharide)
– About 70-81% effective, bacteremia
– Consider boost in 5yrs
• Prevnar
– 13+ valent protein conjugated vaccine, rec for children
• Who?
– Lung disease, CHF, age>65, immunosuppressed, DM, cirrhosis,
corticosteroid use, transplantation, asplenic, nephrotic
syndrome, renal failure, HIV, CSF leaks
– About 1/3 of 50-64yo have an indication

MMWR 46(RR-8):1-24;1997 and 49(RR09):1-38;2000

1503
Copyright © Harvard Medical School, 2018. All Rights Reserved.

Ada NEJM 2001;345:1045

Menningococcus and HiB


• Menningococcus
– Serogroups A, C, Y, and W-135 (50ug of each)
– ?Boost in 5 years
– Who: asplenic, hypocomplementemia, properidin
deficiency, travelers, college students
– Consider in setting of serogroup A/C outbreak
– Types of vaccine
• Polysaccharide
• Conjugate (preferred in adults <55 years old)
– MenB
• HiB
– Part of the childhood vaccination schedule
– Various conjugate formulations: HbOC, PRP-OMP,
PRP-T, PRP-D
– Who: consider in asplenic patients

1504
Copyright © Harvard Medical School, 2018. All Rights Reserved.

Meningococcal B Vaccines
• Oct 2014 -MenB-FHbp (Trumenba)
– 3 dose series (0, 2, 6 months)
• Jan 2015 – MenB-4C (Bexsero)
– 2 dose series (0, 1-6 months)
• Approved for
– 10-25 year olds
• Those at increased risk
– Complement deficiency, receive eculizumab, asplenia
– Microbiologists, persons associated with a MenB
outbreak

MMWR Vol 64 No. 22 -- June 12, 2015

Varicella Vaccine
• Oka strain, ~1350 pfu/dose
• Given SC, 0.5 mL
• Varivax (Merck) licensed 1995
• ProQuad (MMR-Varicella) licensed 2005
• Store frozen

1505
Copyright © Harvard Medical School, 2018. All Rights Reserved.

Burden of Varicella over Time

MMWR:Vol 56 RR-4; June 22, 2007

VZV: Varicella Vaccine


• Live attenuated, Oka strain, licensed 3/95
• 0.5mL SQ/IM, 2 doses 4-8 weeks apart
• Seroconversion
– 78-82% after 1 dose
– >99% after 2nd dose
• Issues
– Mild varicella syndrome in 1-4%, 5-26 days post vaccination
• Who
– All children, occupation exposure, non-pregnant women of
childbearing age, travelers
– ?Immunocompromised patients and household contacts
– 70-90% of individuals w/o h/o infection are seropositive

1506
Copyright © Harvard Medical School, 2018. All Rights Reserved.

Varicella Vaccine Recs


• 2 doses in children (12-15 m and 4-6 yrs of age)
• Catch-up for those who received 1 dose
• > 13 years old w/o evidence for varicella immunity
• HIV infected individuals (CD4>200)
– 2nd dose at 3 months
• Outbreak control
• PEP (within 3-5 days of exposure)
• Postpartum women
• School requirements

Zoster (HZ)
• Oxman et al NEJM 2005;352:2271-84

– N= 38,546
– Used Oka/Merck strain
• Decreased incidence of zoster by 51.3%
• Decreased postherpetic neuralgia by 66.5%

1507
Copyright © Harvard Medical School, 2018. All Rights Reserved.

NEJM 2005;352:2271-84

HZ Vaccine Recs
• Who
– Persons > 60 years old with a history of
varicella

• Contraindications
– Significant immunosuppression

1508
Copyright © Harvard Medical School, 2018. All Rights Reserved.

HZ/su- VZV glycoprotein E/ASO1E

Lal et al. NEJM May 2015

HAV
• Vaccines
– Havrix (1440 ELU) and Vaqta (50U):
• 2 doses 1mL IM at 0 and 6-12m
• Seroconversion at 15d 88-93% and 28d 95-99%
• AEs: local discomfort
• Who
– Travelers, individuals under custodial care, persons
with hepatitis B+C infection, chronic liver disease,
high-risk behaviors (IVDU, MSM), or receive clotting
factors
• PEP
– Ig w/in 14d of exposure (85% effective, 0.02 mL/kg)
– Vaccination

1509
Copyright © Harvard Medical School, 2018. All Rights Reserved.

HBV I
• Recombinant vaccine to HBsAg
– Recombivax and Engerix-B
• 10-20 ug/mL of antigen w/ aluminum hydroxide
• 40 ug/mL formulation for dialysis and immunosuppressed
patients
• Series: 3 doses at 0, 1, and 6 months
– Donot vaccinate in buttock (diminished
immunogenicity)
• Obtain f/u serology in persons at risk thus PEP
affected (e.g., IVDU, MSM, STDs, health care
workers)
– If seronegative then revaccinate up to 3 more times
– Boost in hemodialysis pts when titer <10 mIU/mL

HBV II
• Who
– Children, travel, hepatitis C infection, liver disease,
household contacts of HBV infected persons, health
care workers, high-risk behaviors (e.g., IVDU, MSM)
• PEP
– Unimmunized or immunized nonresponder: HBIG
(0.06 mL/kg) w/in 14d and vaccinate
– Immunized and known responder: nothing
– Immunized and unknown response: check titer if
negative then Rx as nonresponder
• First vaccine preventable cancer: hepatoma

1510
Copyright © Harvard Medical School, 2018. All Rights Reserved.

HPV Associated Disease


• Estimated in US in
2007
– 6.2 million new HPV
infections
– 11,000 new cases
of cervical cancer
– 3,700 cervical
cancer deaths
MMWR:Vol 56 RR-2; Mar 2007

Age-Standardized Rates of New Cases of Cervical Cancer per 100,000 Women, 2002

Agosti J and Goldie S. N Engl J Med 2007;356:1908-1910

1511
Copyright © Harvard Medical School, 2018. All Rights Reserved.

HPV Vaccine(s)
• Gardasil (Merck)
– Licensed 8/18/06
– Quadravalent HPV L1 protein (major capsid protein)
adjuvanted with alum
– Serotypes 6, 11, 16, 18
– Given 0.5 mL, IM at 0, 2, 6 months
– 9-valent
• Serotypes 31, 33, 45, 52, and 58. Joura et al NEJM 2015; 372:711-23
• Cervarix (GSK)
– Bivalent HPV L1 protein adjuvanted with AS04
– Serotypes 16, 18
– Given 0, 1, 6 months

1512
Copyright © Harvard Medical School, 2018. All Rights Reserved.

Times to End-Point Events in the Intention-to-Treat Population

Garland S N Engl J Med 2007;356:1928-1943

Some HPV Vaccine Unknowns


• Efficacy appears to be linked to vaccination prior
to serotype exposure
– Age of vaccination
• Duration of protection
• Role of vaccinating males
• Serotype replacement
• Impact on non-cervical HPV associated
malignancies
• Impact on cervical cancer screening

1513
Copyright © Harvard Medical School, 2018. All Rights Reserved.

Contraindications to Vaccines

MMWR 2018

Recommended Adult Immunization Schedule


by vaccine and age group

MMWR 2018

1514
Copyright © Harvard Medical School, 2018. All Rights Reserved.

Recommended Vaccinations Indicated for Adults


based on medical and other indications

MMWR 2018

Q1) Which vaccine is not live


attenuated?

A) MMR
B) Hepatitis B virus
C) Zoster Vaccine
D) Yellow Fever
E) Oral Polio Virus

1515
Copyright © Harvard Medical School, 2018. All Rights Reserved.

Q1) Which vaccine is not live


attenuated?

A) MMR
B) Hepatitis B virus
C) Zoster Vaccine
D) Yellow Fever
E) Oral Polio Virus

Correct is B-hepatitis B virus

Q2) The shingles vaccine is indicated for


which patient group?

A) Those who have had zoster in the past


B) Those who are immunosuppressed
C) Anyone older than 50 years
D) Anyone older than 60 years
E) Anyone older than 50 or 60 years

1516
Copyright © Harvard Medical School, 2018. All Rights Reserved.

Q2) The shingles vaccine is indicated for


which patient group?

A) Those who have had zoster in the past


B) Those who are immunosuppressed
C) Anyone older than 50 years
D) Anyone older than 60 years
E) Anyone older than 50 or 60 years

Correct is E –anyone over 50 or 60 years

References
1) 2012 Red Book, Report of the Committee on
Infectious Diseases, 29th Edition, AAP
2) Guide for Adult Immunization by the ACP
3) MMWR at www.cdc.gov/mmwr
– 2018 Adult Vaccine Recommendations
– Dec 2006;55(No.RR-15) Pg1-48 General
Recommendations on Immunization (ACIP and
AAFP)
4) Vaccines by Plotkin and Orenstein. 6th Edition,
Elsecvier 2012
5) Vaccines and Vaccinations, G Ada, NEJM
2001;345:1042-50

1517
Copyright © Harvard Medical School, 2018. All Rights Reserved.

HIV DISEASE: AN OVERVIEW


Jennifer A. Johnson, MD

Brigham and Women’s Hospital


Division of Infectious Disease
Harvard Medical School

No financial disclosures

1518
Copyright © Harvard Medical School, 2018. All Rights Reserved.

Estimated HIV Incidence among Persons Aged ≥13 Years, by Transmission Category,
2010–2015—United States

Note. Estimates were derived from a CD4 depletion model using HIV surveillance data. Data have been statistically adjusted to account for missing
transmission category. Heterosexual contact is with a person known to have, or to be at high risk for, HIV infection.
* Difference from the 2010 estimate was deemed statistically significant (P < .05).

Diagnoses of HIV Infection among Men Who Have Sex with Men,
by Age at Diagnosis, 2010–2015—United States and 6 Dependent Areas

Note: Data have been statistically adjusted to account for missing transmission category. Data on men who have sex with men do not include
men with HIV infection attributed to male-to-male sexual contact and injection drug use.

1519
Copyright © Harvard Medical School, 2018. All Rights Reserved.

Estimated HIV Incidence among Persons Aged ≥13 Years, by Area of Residence,
2015—United States
Total = 38,500

Note. Estimates were derived from a CD4 depletion model using HIV surveillance data. Estimates rounded to the nearest 100 for estimates >1,000 and to
the nearest 10 for estimates ≤1,000 to reflect model uncertainty.

Estimated HIV Prevalence among Persons Aged ≥13 years, by Area of Residence,
2015—United States
Total = 1,122,900

Note. Estimates were derived from a CD4 depletion model using HIV surveillance data. Estimates rounded to the nearest 100 for estimates >1,000 and
to the nearest 10 for estimates ≤1,000 to reflect model uncertainty.

1520
Copyright © Harvard Medical School, 2018. All Rights Reserved.

Diagnosed Infection among Persons Aged ≥13 Years Living with Diagnosed or
Undiagnosed HIV Infection, 2010–2015—United States

Note. Estimates were derived from a CD4 depletion model using HIV surveillance data.
*Difference from the 2010 estimate was deemed statistically significant (P < .05).

HIV Diagnosis

CDC Laboratory Testing for the Diagnosis of HIV Infection:


Updated recommendations, 2014

1521
Copyright © Harvard Medical School, 2018. All Rights Reserved.

HIV prognosis
Prognosis is EXCELLENT!
Life expectancy of HIV-positive patients on
antiretroviral therapy (ART) with undetectable viral
load who maintain or recover CD4 count to ≥ 500
have NO INCREASED MORTALITY above the
general population
SMART and ESPRIT studies – analysis of non-IDU
participants in continuous ART control arms, includes
2380 participants over 12,357 person-years of follow-
up
Rodger AJ. AIDS 2013.

Not quite there yet, but closing the gap

1522
Copyright © Harvard Medical School, 2018. All Rights Reserved.

Case 1
26yo man with depression, MSM, presents for discussion new HIV
diagnosis. Presented 2 weeks ago for STI testing for rectal
discharge, + for rectal gonorrhea by NAAT on rectal swab, and HIV
screening same day was positive. Received IM ceftriaxone and oral
azithromycin last week, and had additional testing sent at that time:
CD4 470, HIV-1 VL 12,743, no resistance mutations. Syphilis
serology, HCV Ab and HBSAg all negative. Which of the following
would be appropriate for today’s visit?

A) Initiate antiretroviral therapy with


tenofovir/emtricitabine/cobicistat/elvitegravir
B) Initiate sulfamethoxazole/trimethoprim daily for prophylaxis and
to ensure medication adherence, return in 6 months to discuss ART
C) Initiate post-exposure prophylaxis with
tenofovir/emtricitabine/raltegravir for 28 day course
D) No medications, follow-up in 12 months for repeat bloodwork

Case 1
26yo man with depression, MSM, presents for discussion new HIV
diagnosis. Presented 2 weeks ago for STI testing for rectal
discharge, + for rectal gonorrhea by NAAT on rectal swab, and HIV
screening same day was positive. Received IM ceftriaxone and oral
azithromycin last week, and had additional testing sent at that time:
CD4 470, HIV-1 VL 12,743, no resistance mutations. Syphilis
serology, HCV Ab and HBSAg all negative. Which of the following
would be appropriate for today’s visit?

A) Initiate antiretroviral therapy with


tenofovir/emtricitabine/cobicistat/elvitegravir
B) Initiate sulfamethoxazole/trimethoprim daily for prophylaxis and
to ensure medication adherence, return in 6 months to discuss ART
C) Initiate post-exposure prophylaxis with
tenofovir/emtricitabine/raltegravir for 28 day course
D) No medications, follow-up in 12 months for repeat bloodwork

1523
Copyright © Harvard Medical School, 2018. All Rights Reserved.

Case 1
A) Initiate antiretroviral therapy with
tenofovir/emtricitabine/cobicistat/elvitegravir
All HIV-positive patients should be offered ART. In this case his CD4 cell count is
> 350 so not immediate risk of opportunistic infections, but evidence supports
benefits of initiation of ART even at high CD4 cell count, and risks of drug
resistance and ART-toxicities are low with current treatments. From public health
perspective this also decreases his risk of transmission to others which could be
significant given recent new rectal gonorrhea.
B) Initiate sulfamethoxazole/trimethoprim daily for prophylaxis and to ensure
medication adherence, return in 6 months to discuss ART
SMX/TMP prophylaxis for PCP not necessary or of benefit at CD4 count > 200, and
there is no evidence to support test of adherence prior to initiation of ART
C) Initiate post-exposure prophylaxis with tenofovir/emtricitabine/raltegravir
for 28 day course
Patient already has documented HIV infection, too late for PEP
D) No medications, follow-up in 12 months for repeat bloodwork
12 months is too long to wait for follow-up, even for patient who elects not to start ART
immediately.

Who should be treated with ART?

ALL HIV-POSITIVE PERSONS

1524
Copyright © Harvard Medical School, 2018. All Rights Reserved.

How urgently should ART be initiated?

Urgently There’s time to breath


For those with risk of AIDS- CD4 > 500, asymptomatic,
related complications not pregnant
CD4 < 200-350
Current opportunistic infection
(OI) – start within 2 weeks Patients starting ART should
For pregnant women or others at ideally be willing and able to
high risk of transmission commit to lifelong treatment;
in stable patients clinicians
(Acute HIV) may postpone therapy if
Hep B or hep C coinfection needed to manage problems
If any HIV-related complications that may interfere with
HIV Associated Neurocognitive medication adherence (eg.
Disorder (HAND) Psychiatric, substance
HIV Associated Nephropathy abuse…).
(HIVAN)

What ART to start?


Currently > 25 antiretroviral formulations available
(including co-formulations)

First-line regimens = 3 drugs from 2 classes:


2 nucleos(t)ide reverse transcriptase inhibitors (NRTIs) +
1 non-nucleoside reverse transcriptase inhibitor (NNRTI), or
1 integrase inhibitor (INSTI), or
1boosted protease inhibitor (PI)

1525
Copyright © Harvard Medical School, 2018. All Rights Reserved.

Single Tablet Regimens

Antiretroviral agents – not STRs

1526
Copyright © Harvard Medical School, 2018. All Rights Reserved.

A few selected antiretroviral toxicities

Tenofovir disproxil fumarate – nephrotoxicity (rare),


osteopenia (cumulative exposure)
Tenofovir alafenamide – lower risk of
nephrotoxicity and osteopenia than with TDF
Abacavir – hypersensitivity syndrome (possible if
HLA B5701 positive)
Efavirenz – neuropsychiatric side effects
(depression, poor sleep, vivid dreams)
Ritonavir – diarrhea, GI upset

A few selected drug/food interactions

Rilpivirine – requires fatty meal for optimal


absorption, not absorbed well in presence of any
antacids (H2B, PPI, etc)
Atazanavir – not absorbed well in presence of any
antacids (H2B, PPI, etc)

Ritonavir and cobicistat –


potent CYP 450 inhibitors,
interact with MANY
medications

1527
Copyright © Harvard Medical School, 2018. All Rights Reserved.

HIV as a chronic disease

http://www.thelancet.com/journals/lancet/article/PIIS0140-6736(13)61809-7/abstract

Treatment failure
Assess adherence (discuss with patient, call
pharmacy, review viral load trend)
Assess drug-drug interactions
Drug resistance testing:
HIV genotype
HIV Integrase Resistance genotype
New regimen should include ideally 3 active agents

1528
Copyright © Harvard Medical School, 2018. All Rights Reserved.

Complications of HIV

On ART and Off ART Off ART

Cardiovascular disease Opportunistic infections


Malignancies (lung, (PCP, MAI, cryptococcal
meningitis, toxoplasmosis)
anal, oropharyngeal,
liver, skin) Malignancies (Kaposi’s
sarcoma, non-Hodgkin
Toxicities of ART: bone lymphoma)
demineralization (TDF),
HIV-associated dementia
renal toxicity (TDF),
(HAD)
metabolic syndrome

CD4 count Opportunistic Infections


Any CD4 TB, bacterial pneumonia, other
STIs
< 200 Pneumocystis pneumonia,
candidal infections
< 100 Toxoplasma encephalitis

< 50 CMV infections, disseminated


MAI, cryptococcal meningitis

1529
Copyright © Harvard Medical School, 2018. All Rights Reserved.

Case 2
22yo man, generally healthy, MSM, presents for evaluation of rash and
fever. Found to have secondary syphilis with RPR 1:128. Treated with
IM penicillin. In discussion reports > 6 sex partners in last 6 months,
50% condom use. Interested in HIV risk reduction, including PrEP. The
next best steps are:

A) Initiate tenofovir/emtricitabine today, return in 3 mo for next HIV test


B) HIV Ag/Ab test today, initiate tenofovir/emtricitabine/raltegravir x
28 days for PEP then transition to tenofovir/emtricitabine for PrEP
C) HIV Ag/Ab test today, initiate tenofovir/emtricitabine if HIV-negative,
return in 3 months for repeat HIV test
D) HIV Ag/Ab test today, risk reduction counseling and condoms for now,
return in 3 months to start PrEP if still interested

Case 2
22yo man, generally healthy, MSM, presents for evaluation of rash and
fever. Found to have secondary syphilis with RPR 1:128. Treated with
IM penicillin. In discussion reports > 6 sex partners in last 6 months,
50% condom use. Interested in HIV risk reduction, including PrEP. The
next best steps are:

A) Initiate tenofovir/emtricitabine today, return in 3 mo for next HIV test


B) HIV Ag/Ab test today, initiate tenofovir/emtricitabine/raltegravir x
28 days for PEP then transition to tenofovir/emtricitabine for PrEP
C) HIV Ag/Ab test today, initiate tenofovir/emtricitabine if HIV-
negative, return in 3 months for repeat HIV test
D) HIV Ag/Ab test today, risk reduction counseling and condoms for now,
return in 3 months to start PrEP if still interested

1530
Copyright © Harvard Medical School, 2018. All Rights Reserved.

Case 2
A) Initiate tenofovir/emtricitabine today, return in 3 mo for next HIV
test
Do not initiate PrEP before confirming that patient is HIV-negative
B) HIV Ag/Ab test today, initiate tenofovir/emtricitabine/raltegravir
x 28 days for PEP then transition to tenofovir/emtricitabine for PrEP
Unclear date of last exposure, no indication for PEP, should start PrEP once
confirmed HIV-negative
C) HIV Ag/Ab test today, initiate tenofovir/emtricitabine if HIV-
negative, return in 3 months for repeat HIV test
Baseline HIV test is critical to confirm patients starting PrEP are not already
infected with HIV. Other routine baseline bloodwork and STI screening is
also important. Once these are reviewed and HIV-negative confirmed, no
reason to delay PrEP for this high-risk individual, should start right away.
Needs HIV testing every 3 months while on PrEP and other STI testing at least
every 6 months.
D) HIV Ag/Ab test today, risk reduction counseling and condoms for
now, return in 3 months to start PrEP if still interested
No reason to delay PrEP if HIV-negative and interested.

HIV Prevention

Risk reduction counseling


Needle exchange programs
Barrier protection (condoms)
Pre-exposure prophylaxis (PrEP) http://www.cdc.gov/actagainstaids/pdf/campaigns/starttalk
ing/stsh-prep-infographic-basics.pdf
Post-exposure prophylaxis (PEP)
Prevention of mother-to-child transmission (PMTCT)
Treatment as prevention (TASP)
Male circumcision
(HIV vaccines? Someday…)

1531
Copyright © Harvard Medical School, 2018. All Rights Reserved.

Treatment as prevention

730 serodiscordant couples


60% heterosexual
HIV+ partner on ART, VL < 200
> 44,000 condomless sex acts

ZERO HIV transmissions

U=U: Undetectable = Untransmittable

https://www.preventionaccess.org/undetectable

1532
Copyright © Harvard Medical School, 2018. All Rights Reserved.

PrEP

https://wwwn.cdc.gov/hivrisk/
http://www.cdc.gov/vitalsigns/pdf/2015-11-24-vitalsigns.pdf

Pre-exposure prophylaxis (PrEP)


Indicated for MSM, heterosexual men and women,
and IDU with “substantial risk of acquiring HIV”
Screen for HIV, hep C and hep B, basic labs
Tenofovir/emtricitabine once daily (≤ 3 month
supply at a time)
Clinic visits q3 months: counseling, HIV screening,
other STI screening renal function monitoring,
pregnancy testing if indicated

US Public Health Service PrEP Clinical Practice Guideline, 2014

1533
Copyright © Harvard Medical School, 2018. All Rights Reserved.

PrEP

540 HIV- ppt at 13 STI clinics in England


Randomized immediate TDF/FTC or defer 1 yr
3 vs. 20 HIV infections relative risk reduction 86%
No difference in other STIs

PrEP works in a real world setting

On-demand PrEP?

1534
Copyright © Harvard Medical School, 2018. All Rights Reserved.

PEP
If potential exposure, evaluate and offer PEP within
72 hours
In most cases PEP = tenofovir/emtricitabine +
raltegravir or dolutegravir x 28 days
Kuhar DT, Infect Control Hosp Epidemiol 2013.

https://www.aids.gov/hiv-aids-basics/prevention/reduce-your-risk/post-exposure-prophylaxis/

Despite the progress…

1535
Copyright © Harvard Medical School, 2018. All Rights Reserved.

Still a long way to go

Take home points


HIV incidence is decreasing in US but still > 35,000
annually, and risk is quite high for MSM
Prognosis with HIV is excellent on ART
All HIV-positive patients should be offered ART
Many antiretroviral drugs available, including
several single-tablet once-daily regimens
Fewer AIDS diagnoses and opportunistic infections
as more patients take effective ART
There are many strategies for HIV prevention –
PrEP should be discussed with high-risk patients

1536
Copyright © Harvard Medical School, 2018. All Rights Reserved.

References
CDC, Division of HIV/AIDS Prevention. “Epidemiology of HIV Infection through
2014” slide set. http://www.cdc.gov/hiv/library/slideSets/
Centers for Disease Control and Prevention and Association of Public Health
Laboratories. Laboratory Testing for the Diagnosis of HIV Infection: Updated
Recommendations. Available at http://dx.doi.org/10.15620/cdc.23447 .
Published June 27, 2014.
Panel on Antiretroviral Guidelines for Adults and Adolescents. Guidelines for the
use of antiretroviral agents in HIV-1-infected adults and adolescents. Department
of Health and Human Services. Available
at http://aidsinfo.nih.gov/contentfiles/lvguidelines/AdultandAdolescentGL.pdf.
Kuhar DT, Henderson DK, Struble KA, et al. Updated US Public Health Service
guidelines for the management of occupational exposures to human
immunodeficiency virus and recommendations for postexposure prophylaxis. Infect
Control Hosp Epidemiol 2013;34(9):875-92. doi: 10.1086/672271.
US Public Health Service. Preexposure prophylaxis for the prevention of HIV
infection in the United States – 2014 Clinical Practice Guideline. Updated May
2014. Available at: http://www.cdc.gov/hiv/pdf/PrEPguidelines2014.pdf

No financial disclosures

1537
Copyright © Harvard Medical School, 2018. All Rights Reserved.

Thank you!

1538
Copyright © Harvard Medical School, 2018. All Rights Reserved.

Pneumonia in Hospitalized Patients

41st Intensive Review of Internal Medicine


July 25, 2018

Michael Klompas MD, MPH, FIDSA, FSHEA


Hospital Epidemiologist, Brigham and Women’s Hospital
Professor, Harvard Medical School & Harvard Pilgrim Health Care Institute

Disclosures

o Grant funding from CDC

1539
Copyright © Harvard Medical School, 2018. All Rights Reserved.

Outline

o How accurate are clinical signs for pneumonia?


o What kind of imaging should we get?
o Is there a role for procalcitonin?
o What pathogens cause pneumonia?
o Do we need to get cultures?
o Do we need to include atypical coverage?
o Should we add steroids?
o How long should we treat?

Case Study
o A 68 year old woman with a history of congestive heart
failure and mild dementia is admitted to hospital from a
nursing home with confusion. She appears a little bit
tachypneic but denies shortness of breath. She has an
intermittent non-productive cough but it’s not clear
whether this is different from baseline.
o On exam, she is lethargic but easily arousable.
Temperature is 100.0, HR 110, BP 108/64, RR 24, SaO2
90% RA. JVP difficult to visualize. Possible crackles in
the bases. Mild lower extremity edema.
o Labs are notable for WBC count of 10.2, hct 32, plt 240,
Na 130, creatinine 1.4, LFTs normal.
o Urinalysis with 4-6 WBC/hpf
o Portable chest x-ray with edema +/- LLL infiltrate

1540
Copyright © Harvard Medical School, 2018. All Rights Reserved.

Does this patient have pneumonia?

1541
Copyright © Harvard Medical School, 2018. All Rights Reserved.

Would you start antibiotics?

Why is Pneumonia So Difficult to Diagnose?

o Many medical conditions in hospitalized patients


present with the same clinical signs as
pneumonia

o Radiographic opacities
o Fever
o Abnormal white blood cell count
o Impaired oxygenation
o Increased pulmonary secretions

1542
Copyright © Harvard Medical School, 2018. All Rights Reserved.

Accuracy of clinical signs for VAP


Relative to autopsy, systematic review, 14 studies, 655 pts

0.1 0.2 0.5 1 2 5 10

Fever

Abnormal WBC

Purulent sputum

Crepitations

Hypoxemia

New infiltrate

Negative Positive
Likelihood Ratio Likelihood Ratio

JAMA 2007; 297:1583

Accuracy of Clinical Diagnosis of VAP


Relative to 253 autopsies

100% Loose definition:


Positive Predictive Value

Infiltrate and 2 of
80% temp / wbc / purulence
Sensitivity /

60% Strict definition:


Infiltrate and 3 of
40% temp / wbc / purulence

20%

0%
Sensitivity Positive
Predictive
Value
Tejerina et al., J Critical Care 2010;25:62

1543
Copyright © Harvard Medical School, 2018. All Rights Reserved.

Would further imaging help?

Chest X-Ray vs CT Scan


319 patients with clinically suspected pneumonia

Initial pneumonia classification following chest x-ray

150

120
No. of Patients

90

60

30

0
Definite Probable Possible
Pneumonia Pneumonia Pneumonia

AJRCCM 2015;192:974-982

1544
Copyright © Harvard Medical School, 2018. All Rights Reserved.

Chest X-Ray vs CT Scan


319 patients with clinically suspected pneumonia

Revised pneumonia classification following CT chest

Excluded Possible Probable Definite


150

120
No. of Patients

90

60

30

0
Definite Probable Possible
Pneumonia Pneumonia Pneumonia

AJRCCM 2015;192:974-982

Chest X-Ray vs CT Scan


319 patients with clinically suspected pneumonia

Final Pneumonia Probable


Classification:
Possible

Definite

Excluded

AJRCCM 2015;192:974-982

1545
Copyright © Harvard Medical School, 2018. All Rights Reserved.

Intensive Care Med 2016;42:1159-63

Is this pneumonia more likely


bacterial or viral?

1546
Copyright © Harvard Medical School, 2018. All Rights Reserved.

Etiology of Community-Acquired Pneumonia


2,259 adults admitted to 5 hospitals in Chicago and Nashville

Viruses
23%

No
pathogen
isolated
62%
Bacteria
11%

Bacteria + Virus 3%
Fungus or AFB 1%

N Engl J Med 2015;373:415-427

Prevalence of Viruses in CAP

Pooled Prevalence
24%

Eur Respir Rev 2016;25:178-88

1547
Copyright © Harvard Medical School, 2018. All Rights Reserved.

Could procalcitonin help?

Procalcitonin and Pneumonia Etiology


1,735 adults admitted to 5 U.S. hospitals with pneumonia

100%
No pathogen
isolated
75%

50%

Bacteria
25%
Virus

0%
<0.1 0.1-0.24 0.25-0.49 ≥0.5
Procalcitonin Level
Clin Infect Dis 2017;65:183-90

1548
Copyright © Harvard Medical School, 2018. All Rights Reserved.

o Population: 1,656 patients with suspected lower


respiratory tract infection and in whom clinicians
were unsure whether to start antibiotics or not

o Intervention: randomized to procalcitonin &


guidelines on interpretation versus usual care

N Engl J Med 2018;ePub ahead of print

Procalcitonin for ?Pneumonia


1656 patients with possible pneumonia randomized to PCT vs routine care

Antibiotic Starts in the ED Duration of Antibiotics


8
40% NS

6
30%
Days of Antibiotics
Percent of Patients

NS
4
20%

2
10%

0
0%

Usual Care Procalcitonin

N Engl J Med 2018;ePub ahead of print

1549
Copyright © Harvard Medical School, 2018. All Rights Reserved.

Should we culture for bacteria


and test for viruses?

Positive cultures will allow you to tailor therapy.

Negative cultures may help facilitate stopping abx early.

Cultures essential to help us track pathogen frequency and


resistance rates.

1550
Copyright © Harvard Medical School, 2018. All Rights Reserved.

Rapid Viral PCR Panels

o 720 patients with acute respiratory illness, fever


>37.5C, or both presenting to the emergency
department in the winter

o Randomized to rapid respiratory virus PCR


panel vs usual care

Lancet Repir Med 2017;5:401-11

Rapid Viral PCR Panels


720 patients randomized to rapid viral PCR panels vs usual care

o No difference in antibiotic starts


o No difference in mean duration of antibiotics

But…

o More patients in the viral PCR group received very brief


(<48h) courses of antibiotics (17% vs 9%)
o Length-of-stay shorter in the PCR group
(mean 5.7 vs 6.8 days)
o Appropriate antiviral treatment for flu more common in
the PCR group (91% vs 65%)
Lancet Repir Med 2017;5:401-11

1551
Copyright © Harvard Medical School, 2018. All Rights Reserved.

This patient is from a nursing home.


Should we include coverage for
MRSA and Pseudomonas?

Trends in Antimicrobial Use


128 VA Medical Centers, 2006-2010

40%
Percent of Hospitalized Patients

30%

20%

10%

0%
2006 2007 2008 2009 2010

Clin Infect Dis 2015;61:1403-10

1552
Copyright © Harvard Medical School, 2018. All Rights Reserved.

Pathogen Prevalence
128 VA Medical Centers, 2006-2010

5.0%
% of Patients with Positive Cultures

Treatment:
4.0%
Prevalence Ratio
(Blood, Sputum, Lung)

3.0%
50:1
2.0%

1.0%

0.0%
MRSA Pseudomonas Acinetobacter

Clin Infect Dis 2015;61:1403-10

Would a nasal swab for MRSA help?

1553
Copyright © Harvard Medical School, 2018. All Rights Reserved.

Nasal MRSA Culture/PCR

o Can a nasal swab screen MRSA predict the


presence or absence of MRSA pneumonia?
o Meta-analysis of 22 studies, 5163 patients

Sensitivity 85%
Positive predictive value 57%
Negative predictive value 98%

Clinical Infectious Disease 2018;ePub ahead of print

Do we need to include coverage for


atypicals?

1554
Copyright © Harvard Medical School, 2018. All Rights Reserved.

βlactam vs βlactam+macrolide vs βlactam+quinolone


Cluster randomized trial of 2,283 non-ICU patients with CAP in the Netherlands

12.0%
NS

NS NS
90-day Mortality

8.0%

4.0%

0.0%
Beta lactam Beta lactam + Beta lactam +
macrolide quinolone

N Engl J Med 2015372:1312-23;

βlactam vs βlactam+macrolide vs βlactam+quinolone


Cluster randomized trial of 2,283 non-ICU patients with CAP in the Netherlands

8.0
Median Hospital Length of Stay

6.0

4.0

2.0

0.0
Beta lactam Beta lactam + Beta lactam +
macrolide quinolone

N Engl J Med 2015372:1312-23;

1555
Copyright © Harvard Medical School, 2018. All Rights Reserved.

βlactam alone vs βlactam+macrolide


Randomized controlled trial of 580 patients with CAP in Switzerland

Mild Illness Moderate-Severe Illness

βlactam βlactam + macrolide

JAMA Internal Med 2014;174:1894-1901

Should we add steroids?

1556
Copyright © Harvard Medical School, 2018. All Rights Reserved.

Steroids and Mortality


Severe pneumonia

Less severe pneumonia

Combined

Ann Intern Med 2015;163:519-522

1557
Copyright © Harvard Medical School, 2018. All Rights Reserved.

Need for Mechanical Ventilation

Risk for ARDS

Ann Intern Med 2015;163:519-522

Length of Stay
Low Risk of Bias

-1.0 days (-1.8 to -0.2)


High Risk of Bias

Ann Intern Med 2015;163:519-522

1558
Copyright © Harvard Medical School, 2018. All Rights Reserved.

Hyperglycemia

RR 1.5 (95% CI 1.0-2.2)

Ann Intern Med 2015;163:519-522

How long should we treat for?

1559
Copyright © Harvard Medical School, 2018. All Rights Reserved.

8 vs 15 Days Rx for VAP


Double-Blind, Randomized Trial in 51 French ICUs, N=401
8 Days 15 Days
35
Mortality % or No. of Days

NS
30

25
NS
20

15
NS NS
10

0
All Cause Ventilator-Free Organ Failure- ICU Length-of-
Mortality Days Free Days Stay
JAMA 2003;290:2588

Clinical Outcomes in Patients with


Non-Fermenting Gram Negative Bacilli

8 Days 15 Days
35
Mortality % or No. of Days

30

25

20

15

10

0
All Cause Ventilator-Free Organ Failure- ICU Length-of-
Mortality Days Free Days Stay
JAMA 2003;290:2588

1560
Copyright © Harvard Medical School, 2018. All Rights Reserved.

Is less than 8 days feasible?

o Randomized controlled trial in 4 hospitals in Spain


o 312 patients with CAP randomized to stopping antibiotics on
day 5 vs usual care (median 10 days)
o Antibiotics stopped if temperature ≤37.8°C for ≥48hrs and ≤1
sign of clinical instability
o SBP<90mm Hg, HR >100/min, RR>24/min, SaO2≤90%, PaO2≤60mm Hg

JAMA Internal Medicine 2016;176:1257-65

1561
Copyright © Harvard Medical School, 2018. All Rights Reserved.

5 vs 10 Days for Community Acquired Pneumonia


Randomized controlled trial, 312 patients, 4 hospitals in Spain

5-Day Rx (N=150) 10-Day Rx (N=162)


100%
Percent of Patients

75%

50%

25%

0%
Clinical Success (Day 10) Clinical Success (Day 30)

JAMA Internal Medicine 2016;ePub ahead of print

Is less than 5 days feasible?


Image: www.she.stir.ac.uk/env-carbon-management/challenge.php

WE

1562
Copyright © Harvard Medical School, 2018. All Rights Reserved.

o Randomised controlled trial amongst patients admitted to 9 hospitals in


the Netherlands with community acquired pneumonia
o Excluded if immunocompromised, recent hospital or nursing home
admission, recent exposure to systemic antibiotics, PaO2 < 50mm Hg, ICU
admission, empyema, isolation of Staph aureus, Klebsiella sp., or an
atypical organism

3 Days vs 8 Days of Amoxicillin for Patients


Hospitalized with Pneumonia

3-Day Rx (N=56) 8-Day Rx (N=63)


100%
Percent of Patients

75%

50%

25%

0%
Clinical Cure Bacteriological Radiologic
success Success

BMJ 2006;332:1355

1563
Copyright © Harvard Medical School, 2018. All Rights Reserved.

Could procalcitonin help?

Procalcitonin Surveillance: SAPS


1575 critically ill patients, open label RCT, 15 ICUs, Netherlands

Procalcitonin (n=761) Control (n=785)


25

20 VAP Subgroup (N=102)


Median Duration of Treatment
15
Procalcitonin Arm: 4.0 days (IQR: 2-7)
Control Arm: 7.0 days (IQR: 4-11)
10
***
5

0
Median Duration ICU of Stay
ICU Length Mortality
Mortality (28-day)
Antibiotics Length-of-Stay (28-day)

Lancet Infect Dis 2016;16:819-827

1564
Copyright © Harvard Medical School, 2018. All Rights Reserved.

Summary
o Diagnosing pneumonia is challenging. We’re often wrong.
CT and procalcitonin may help.

o Many (?most) pneumonias are caused by viruses. Test for


them.

o Know your antibiogram. Vancomycin not necessary for most


patients. If you start it, stop at 48hrs if MRSA not isolated.
Routine coverage for atypicals probably not necessary for
most immunocompetent patients with non-severe disease.

o Steroids may confer a mortality benefit in patients with


severe disease

o Short course regimens (3-5 days) usually adequate. Serial


procalcitonin measurements may facilitate short courses.

Question 1
A 54 year old woman is admitted to the ICU with an episode of
pancreatitis. 7 days after admission she develops a new fever, has
increased respiratory secretions, and requires increased ventilator
support. You start empiric treatment with vancomycin and cefepime. An
endotracheal aspirate is culture positive for Pseudomonas aeruginosa.
How long will you treat her?

A. 3 days
B. 7 days
C. 15 days
D. 21 days

1565
Copyright © Harvard Medical School, 2018. All Rights Reserved.

Answer 1
A 54 year old woman is admitted to the ICU with an episode of
pancreatitis. 7 days after admission she develops a new fever, has
increased respiratory secretions, and requires increased ventilator
support. You start empiric treatment with vancomycin and cefepime. An
endotracheal aspirate is culture positive for Pseudomonas aeruginosa.
How long will you treat her?

RCT data suggests that while microbiologic


A. 3 days
persistence / recurrence is higher with short
B. 7 days course vs long course therapy, clinical
C. 15 days outcomes are identical. Short course therapy
is therefore preferred even for patients with
D. 21 days
Pseudomonas.

Question 2
64 year old gent with diabetes, rheumatoid arthritis (on prednisone 5mg
po qd), and a remote history of stroke is admitted from an assisted living
facility with fever, cough, and shortness of breath. Temp 37.8, BP 100/64,
HR 90, RR 22, SaO2 90% RA. Chest x-ray is clear. WBC count 10.8.
What tests would you obtain?

A. CT scan chest, procalcitonin, sputum Gram stain & culture


B. Procalcitonin, sputum stain & culture, NP swab for viruses
C. CT scan chest, sputum stain & culture, NP swab for viruses
D. CT chest alone
E. Procalcitonin alone
F. None of the above

1566
Copyright © Harvard Medical School, 2018. All Rights Reserved.

Question 2
64 year old gent with diabetes, rheumatoid arthritis (on prednisone 5mg
po qd), and a remote history of stroke is admitted from an assisted living
facility with fever, cough, and shortness of breath. Temp 37.8, BP 100/64,
HR 90, RR 22, SaO2 90% RA. Chest x-ray is clear. WBC count 10.8.
What tests would you obtain?

A. CT scan chest, procalcitonin, sputum Gram stain & culture


B. Procalcitonin, sputum stain & culture, NP swab for viruses
C. CT scan chest, sputum stain & culture, NP swab for viruses
D. CT chest alone
E. Procalcitonin alone 1. CT can both rule in and rule out
pneumonias not seen on CXR
F. None of the above
2. Procalcitonin not helpful for diagnosis in
intermediate probability cases. More useful
for informing duration of therapy
3. Bacteria and viruses are equally common
in CAP – test for them!

Disclosures

o Grant funding from CDC

1567
Copyright © Harvard Medical School, 2018. All Rights Reserved.

Thank You!

mklompas@partners.org

1568
Copyright © Harvard Medical School, 2018. All Rights Reserved.

Infectious Diseases: Take-Home


Messages and Clinical Pearls
James H. Maguire, MD
Senior Physician
Division of infectious Disease and Department of Medicine
Brigham and Women’s Hospital
Professor of Medicine
Harvard Medical School

Disclosures

Bayer: Chagas disease advisory committee

1569
Copyright © Harvard Medical School, 2018. All Rights Reserved.

Infections and immunodeficiency


Organism Humoral Complement Phagocyte Cellular

Bacteria Pneumococcus, Meningococcus S. aureus, Salmonella,


meningococcus, (terminal enterics, P. Listeria.
H. influenzae components) aeruginosa, Legionella
Viruses Enterovirus - - Herpesviruses
Hepatitis B,C
Fungi - - Candida, Cryptococcus,
Aspergillus, endemic fungi,
Mucor Candida,
Pneumocystis
Mycobacteria - - - TB, atypicals
Protozoa Giardia - - Toxoplasma,
Cryptosporidium,
T. cruzi,
Leishmania
Helminths - - - Strongyloides

Opportunistic infections in HIV


• Previously undiagnosed HIV, not taking
antiretroviral therapy (ART) for financial,
psychosocial or medical reasons
• Recently started on (effective) ART
(immune reconstitution inflammatory
syndrome (IRIS)
• Exceedingly rare in longstanding well-
controlled HIV infection

1570
Copyright © Harvard Medical School, 2018. All Rights Reserved.

Fever in HIV
• Unrelated to HIV; drug fever
• Acute HIV
• CD4 200-500: pneumococcal infection,
other respiratory infections, tuberculosis,
lymphoma (“ARC”-zoster, thrush)
• CD4 <200: PCP, toxoplasmosis, endemic
fungi, cryptocococcosis
• CD4 <50: M. avium complex, CMV
• IRIS

Immune reconstitution
inflammatory syndrome (IRIS)
• Recommend initiation of ART within 2
weeks of diagnosis of opportunistic
infection (exception=CNS infections)
• IRIS: paradoxical worsening of infection or
disease process following initiation of
effective ART and restoration of immune
response
• Most common with mycobacterial, viral
and fungal infections; incidence 10-20%

1571
Copyright © Harvard Medical School, 2018. All Rights Reserved.

Immune reconstitution
inflammatory syndrome (IRIS)
• Fever, adenopathy common; localized
symptoms and signs
• Usually occurs within a week to a few
months after starting ART
• Continue ART and directed therapy for
opportunistic infection in most cases
• NSAIDS, steroids

http://aidsinfo.nih.gov/contentfiles/lvguidelines/adult_oi.pdf (2018)
Expert Rev Anti Infect Therapy 2015,13:751

Infections and immunodeficiency


• Neutrophils
– Decreased numbers
– Impaired function
• Neutropenia
– Cytotoxic chemotherapy, drug
hypersensitivity, marrow disorders, other
– Severe: ANC <500; profound <100
– Signs of infection often masked
– Infections can progress rapidly, life-
threatening

1572
Copyright © Harvard Medical School, 2018. All Rights Reserved.

Neutropenia and infections


• Common pathogens
– Gram-negative enterics (E. coli, Klebsiella,
Enterobacter) > Pseudomonas aeruginosa
– Gram-positive organisms (intravascular
catheters): S. aureus, coagulase-negative
staphylococci, enterococci
– Fungi (prolonged neutropenia, antibiotics):
Candida spp., Aspergillus, Zygomycetes

Neutropenia and infections


• High risk patients-empirical therapy:
– Antipseudomonal β-lactam (cefepime, others)
– Vancomycin – only when indicated
– Prolonged fever : echinocandin (e,g.,
micafungin)
• Low risk: oral therapy (e.g., amoxicllin-
clavulanate/ciprofloxacin; moxifloxacin)

Clin Infect Dis. 2011;52(4):e56.

1573
Copyright © Harvard Medical School, 2018. All Rights Reserved.

Impaired neutrophil function


• Genetic: Chronic granulomatous disease,
hyper-IgE (Job) syndrome, others
• Acquired: steroids, diabetes
• Infections in diabetics
– Impaired phagocytes
– Vascular insufficiency
– Neuropathy
– Colonization (S. aureus, Candida)
(Impaired cellular immunity (TB, Coccidioides))

Infections in diabetics
• S. aureus-skin and soft tissue, pneumonia,
bacteremia
• Pseudomonas aeruginosa: malignant otitis
externa
• Rhinocerebral mucormycosis (acidosis)
• Candidiasis: cutaneous, mucosal, urinary
• Enteric organisms: UTIs, emphysematous
cystitis/pyelonephritis, papillary necrosis
• Diabetic foot infections
• Necrotizing soft tissue infections

1574
Copyright © Harvard Medical School, 2018. All Rights Reserved.

Skin and soft tissue infections


• Erysipelas (superficial): β-hemolytic
streptococci>Staphylococcus aureus
• Cellulitis: β-streptococci, S. aureus
• Necrosis, purulence, abscess: S. aureus,
especially MRSA
• Recurrent cellulitis (β-streptococci)
• Lymphedema
• Strategy: edema control, antifungals,
foot care, prophylactic penicillin

Skin and soft tissue infections:


special cases

Dog/cat bites Pasteurella multocida,


Capnocytophaga canimorsus
Saltwater Vibrio vulnificus
Freshwater Aeromonas, Pseudomonas
Hot tubs Pseudomonas
Human bites Skin and mouth flora
Fish, raw meat,
poultry Erysipelothrix rhusiopathiae

1575
Copyright © Harvard Medical School, 2018. All Rights Reserved.

Treatment of MRSA skin and soft


tissue infections
• Incision and drainage
• Oral antibiotics
– TMP-SMX
– Doxycycline }
Not uniformly active
against streptococci
– Clindamycin if susceptible
– Linezolid
• Intravenous antibiotics: vancomycin,
daptomycin, ceftaroline, linezolid
• Attempt decolonization for recurrences

Necrotizing soft tissue infections


• Necrotizing fasciitis
– Group A β-hemolytic streptococci
– Polymicrobial including anaerobes
– Marine Vibrios, Aeromonas, rare MRSA
• Clostridium perfringens necrotizing
cellulitis, myonecrosis
• Pain → erythema, edema → necrosis
• Treatment: antibiotics, surgical
debridement, supportive therapy

1576
Copyright © Harvard Medical School, 2018. All Rights Reserved.

Toxic shock syndromes


• S. aureus: menstrual, non-menstrual
• Group A streptococci: skin, soft tissue
infections, other
• Exotoxin, super-antigen
• Hypotension, diffuse erythematous rash, end
organ failure
• Treatment
• Clindamycin plus beta-lactam antibiotic or
vancomycin +/-IVIG
• Source control: remove foreign body, surgery
• Supportive care

Respiratory infections
• 22 year old: 6 day history sore throat,
difficulty swallowing admitted to ICU with
fever, shortness of breath, hypotension,
leukocytosis with left shift , ↑creatinine
• CXR/CT of chest: nodular opacities, effusion
• CT of neck: paratonsillar abscess,
thrombosis of right internal jugular vein
• Cultures: Fusobacterium necrophorum
• Diagnosis: Lemierre syndrome
• Pressors, surgery, antibiotics, anticoagulation

1577
Copyright © Harvard Medical School, 2018. All Rights Reserved.

Ann Int Med 2015;162:2412

• Young adults with pharyngitis


– 21% Fusobacterium necrophorum
– 10% group A streptococcus
• Fusobacterium is resistant to macrolides
• Recommend: treat severe sore throat (3 or
4 of following: fever, no cough, tender
anterior nodes, tonsillar exudate) with
penicillin not azithromycin

Extrapulmonic tuberculosis
• Seen in 50% or more of patients with HIV/TB
coinfection but also in not HIV-infected
• Clues
– Sterile pyuria
– Chronic lymphadenopathy (especially cervical)
– Monarthritis, spinal osteomyelitis (esp thoracic)
– Lymphocytic predominant ascites
– Chronic lymphocytic meningitis
– Exudative pleural effusion
– Pericarditis

1578
Copyright © Harvard Medical School, 2018. All Rights Reserved.

Nontuberculous mycobacterial infection


• Acquired from environment
• Immune compromised; immune competent
often with risk factor (lung disease, trauma,
surgery)
• Sensitivity testing helpful, but not always
predictive of response
• Treatment: combination therapy, prolonged,
may require surgery

Nontuberculous mycobacterial infection


Disseminated (esp. M. avium-intracellulare (MAC)
HIV) M. kansasii
Pulmonary MAC
M. kansasii
M. abscessus
Skin and soft tissue M. marinum
M. abscessus, fortuitum, chelonae
Catheter-associated M. abscessus, fortuitum, chelonae
Cervical lymphadenitis MAC, M. scrofulaceum
Leprosy M. leprae
Buruli ulcer M. ulcerans

1579
Copyright © Harvard Medical School, 2018. All Rights Reserved.

Fungal infections of the lungs


• “Endemic” mycoses (dimorphic fungi)
– Histoplasmosis: river valleys, guano in soil
– Coccidioidomycosis: SW USA, Latin America
– Blastomycosis: USA (south, south central)
– South American blastomycosis (CA, SA)
– Sporotrichosis (sphagnum moss, thorns)
– Penicilliosis [talaromycosis] (China, SE Asia)
• Cryptococcosis (soil contaminated with bird
droppings)

Tick-borne infections in the USA


Bacteria • Relapsing fever
• Borrelia burgdorferi Borrelia
• Anaplasma • Borrelia miyamotoi
phagocytophilum Viruses
• Ehrlichia chafeensis • Colorado Tick Fever
• Rickettsia rickettsii • Powassan virus
• Rickettsia 364D, • Heartland virus
others • Bourbon virus
• STARI Protozoa
• Francisella tularensis • Babesia spp

1580
Copyright © Harvard Medical School, 2018. All Rights Reserved.

Lyme disease CDC

• Suspect in persons with possible exposure


• Most do not report tick bite
• Early Lyme disease
– Diagnose clinically (serology often negative)
– Erythema chronicum migrans
– Fever, myalgia, Bell’s palsy, meningitis,
heart block, arthralgia
• Late Lyme disease: arthritis, neurological
(encephalitis, neuropathy, radiculitis)

Lyme disease
• Diagnosis: screening ELISA, confirmatory
Western blot
• Treatment
– Oral doxycycline, amoxicillin or cefuroxime
– Intravenous ceftriaxone: certain syndromes
– Treatment > 28 days not indicated for
chronic Lyme
– No antibiotics for post-Lyme syndrome
• Prophylaxis for attached tick in Lyme-
endemic area: doxycycline 200 mg PO x1

1581
Copyright © Harvard Medical School, 2018. All Rights Reserved.

Ticks may carry and transmit transmit multiple


pathogens to same host (e.g., Ixodes scapularis
(Lyme, anaplasmosis, babesiosis, Powassan
virus, Borrelia miyamotoi)

Distribution of tick-borne infections (CDC)

Clinical approach to tick-borne infections


• Tick-borne infections can be rapidly fatal
(e.g., Rocky Mountain Spotted Fever)
• Unexplained fever, chills, headache, fatigue,
myalgia, arthralgia, rash, neurologic
symptoms
• Consider place, season,
activities, pets
• Rapid diagnostic testing
often not available
• Early, empirical treatment CDC.gov

can be life-saving (doxycycline)

1582
Copyright © Harvard Medical School, 2018. All Rights Reserved.

Salmonella bacteremia
• Typhoid and other enteric fevers
• Transient bacteremia with gastroenteritis
• Endovascular infections
– Atherosclerosis-aneurysms
– Heart valves, especially prosthetic
– Schistosomiasis
• Persistent infection, localized infections:
immune deficiency: AIDS, transplant, etc
• Usually do not treat salmonella gastroenteritis
unless at risk of localized or persistent
infection

Typhoid
• Risk in South Asia 6 to 30 times higher than
East and Southeast Asia, Africa, the
Caribbean, and Central and South America
• Increasing antibiotic resistance
• Oral live attenuated Ty21a vaccine: 1
capsule orally q 48 hrs x4; boost every 5
years
• Vi capsular polysaccharide vaccine:
intramuscular injection every 2 years
• Protection: 50-80%; not paratyphoid fever

1583
Copyright © Harvard Medical School, 2018. All Rights Reserved.

Vaccinations for travel


Routine adult immunization, plus:
Cholera Plague
Hepatitis A Meningococcal
Hepatitis B Rabies
Inactivated polio Typhoid
Japanese Yellow Fever
encephalitis Tick-borne
Influenza encephalitis

Yellow fever vaccine


• Travel to endemic areas or requiring proof of
vaccination for entry to country
• Contraindications: hypersensitivity to egg or
chicken proteins, immune suppression, age
under 9 months
• Precautions (weigh risks and benefits): age >
60 years, asymptomatic HIV infection,
pregnancy, breastfeeding
• Vaccinate at least 10 days before travel; WHO:
no need to revaccinate after 2016: but every
10 years, if required for entry, special cases

1584
Copyright © Harvard Medical School, 2018. All Rights Reserved.

Areas with Risk of


Yellow Fever Virus
Transmission

CDC.gov

Yellow Fever Vaccine Outbreak of Yellow Fever


in Brazil
shortage mid-2017 to 2016 – present
mid- 2018: consult CDC
(cdc.gov/travel/page/search-for-
stamaril-clinics)

CDC.gov

1585
Copyright © Harvard Medical School, 2018. All Rights Reserved.

Wild poliovirus & cVDPV cases, 2018


01 January – 22 June

Inactivated poliomyelitis
vaccine:
All travelers to endemic or
epidemic areas, including
Wild poliovirus type 1 (N=11)
those with recent wild virus
cVDPV1 (N=14)

Reintroduction of wild type


circulation and neighboring
virus in Venezuela not
confirmed
countries
• Booster if received
primary series
http://www.polioeradication.org/Dataand monitoring.aspx

1586
Copyright © Harvard Medical School, 2018. All Rights Reserved.

Sexually Transmitted Diseases


Update 2018

Todd B Ellerin, MD
Director of Infectious Diseases
South Shore Hospital
Associate Physician, Brigham and Women’s Hospital
Instructor in Medicine
Harvard Medical School
tellerin@southshorehealth.org

Disclosure
None

1587
Copyright © Harvard Medical School, 2018. All Rights Reserved.

What’s New
More than 2 million cases of chlamydia,
gonorrhea, and syphilis were reported in the US
in 2016, the highest number ever
Majority of these (1.6 million) were Chlamydia
470,000 cases of GC and 28,000 cases of
primary and secondary syphilis

CDC. Sexually Transmitted Disease Surveillance 20016

Case 1

24 yo black male HIV+ diagnosed in 2000, last


CD4 count 380 cells/mm3 and VL of 50K
copies/ml. Lost to follow up for 5 years. No ARV
therapy.
H/o gonorrhea and genital herpes-type 2
(treated episodically approx 2 outbreaks/yr).
Presents with dysuria, painful ulcers surrounding
his edematous penile foreskin, and blood at the
tip of his urethral meatus.

1588
Copyright © Harvard Medical School, 2018. All Rights Reserved.

Case 1

Previous herpetic outbreaks had been unilateral,


small vesicles on the foreskin which crusted
over in days.
Last herpetic outbreak several years ago.
HIV acquired through unprotected heterosexual
intercourse.
Last protected sexual intercourse was 4 days
prior with his girlfriend.
No recent unprotected intercourse.

1589
Copyright © Harvard Medical School, 2018. All Rights Reserved.

What’s the most likely cause of


these painful ulcers?
A) Syphilis with bacterial superinfection
B) Chancroid
C) Genital herpes in the setting of declining cell-
mediated immunity
D) Lymphogranuloma venereum (LGV)
E) Gonorrhea

Viral culture + HSV-2


CD4 count 100 cells/mm3

1590
Copyright © Harvard Medical School, 2018. All Rights Reserved.

STD Treatment Guidelines 2015

https://www.cdc.gov/std/tg2015/tg-2015-print.pdf

Genital HSV - Epidemiology


At least 50 million persons, 12 and older, in the US are
infected with HSV-2
HSV-2 is more common in women (~25%) than men
Most (as many as 90%) of HSV-2-infected individuals are
undiagnosed and unaware of their infection; those
periodically shed virus in their genital tract (2-28% of
days)
Most transmission is from asymptomatic carriers
5-50% of clinical genital HSV is caused by HSV-1;
Recurrence and asymptomatic shedding are more
common with genital HSV-2

1591
Copyright © Harvard Medical School, 2018. All Rights Reserved.

Type-Specific HSV-2 serology

Testing for antibodies directed against HSV type-


specific glycoproteins (GP-G1 & G2 for HSV 1 or 2)
Sensitivity 80-98%, Specificity > 96%
Point-of-care tests are available
Evolving role in clinical practice: assessment of
partners, diagnosis of culture-negative ulcers, STD
screen, identification of asymptomatic carriers,
vaccine candidates
Limitations: lag after initial exposure, a positive test
only indicates previous exposure

Genital HSV Prevention

1592
Copyright © Harvard Medical School, 2018. All Rights Reserved.

Antiviral Therapy for Prevention of HSV-2


Transmission

Prospective placebo-controlled study of heterosexual HSV-


2 sero-discordant couples
HSV-2 positive partner given valacyclovir 500 mg daily or
placebo
HSV-2 susceptible partners assessed monthly
Condoms and safe sex counseling provided at all visits
After 8 months of therapy Symptomatic HSV-2 transmission
was reduced by 77%, and asymptomatic acquisition by
50%
Valacyclovir is now FDA approved for this indication

Characteristics of common ulcerative


diseases

HSV Syphilis Chancroid

Ulcer Painful, often Painless Painful,


many, papules purulent,
irregular, deep
LN Rare except Small 1-2 weeks later
primary infxn

Comments COMMON Must test for Unusual in US


and treat

1593
Copyright © Harvard Medical School, 2018. All Rights Reserved.

Characteristics of less common ulcerative


diseases

LGV Donovanosis

Ulcer Small, painless, Large,


heal before LAD irregular,
bleeding
LN Large, draining None,
nodes hypertrophied
tissue

Comments Rare in US, Rare in US


except MSM

Evaluation of the Patient with Genital Ulcer


A diagnosis based on history/physical is often inaccurate
Obtain a serologic test for syphilis
If serology negative, repeat at 1 & 3 months
Culture (or antigen test) for herpes simplex
HSV PCR more sensitive than culture
In endemic areas or during outbreak: GS/Culture of edge
of ulcer for H. ducreyi (53-88% sensitive)
MSM and endemic areas: serology for LGV and
Chlamydia PCR / NAAT
Consider empiric therapy based on most likely
diagnosis

1594
Copyright © Harvard Medical School, 2018. All Rights Reserved.

Case 3: Penile Funk

1595
Copyright © Harvard Medical School, 2018. All Rights Reserved.

1596
Copyright © Harvard Medical School, 2018. All Rights Reserved.

Syphilis Fast Facts


Painless ulcer, think syphilis, test and treat empirically
Rash involves palms and soles, think syphilis, test,
and treat empirically
Early syphilis (primary/secondary) treat all sex
partners with past 90 days and sex partners after 90
days can be tested
Follow pts to make sure RPR has fallen at least 4 fold
Screening withy RPRs leads to false +’s so +RPRs
need to be confirmed by treponemal test
Screening with specific Treponemal test avoids false + RPR
but introduces scenario +Treponemal test with negative RPR

1597
Copyright © Harvard Medical School, 2018. All Rights Reserved.

Syphilis Serology Sensitivities

Test Stage
0
1 20 30

VDRL 70% 99% 56%


RPR 80% 99% 56%

FTA-abs 85% 100% 98%


MHA-TP/TPHA 65% 100% 95%
TPI 50% 97% 95%

Syphilitis Condylomata Lata, female

1598
Copyright © Harvard Medical School, 2018. All Rights Reserved.

The Prozone reaction


False Negative Syphilis Screening Tests
The RPR testing methodology uses charcoal particles,
added to a cardiolipin Ag suspension
A test is positive when an optimal Ab:Ag ratio creates a
precipitate (lattice, also referred to as the zone of
equivalence). Charcoal is trapped in this lattice and
allows the reaction to be visible macroscopically
Antibody excess (prozone) or antigen excess
(postzone) will yield false negative test results because
they do not form the lattice needed to trap the charcoal
particles

Neurosyphilis
May occur during any stage Indication for CSF Analysis
Symptoms: asymptomatic, Neurologic or ophthalmic/otic
meningitis, cranial nerve signs/sx
palsies, general paresis, tabes Active tertiary disease
dorsalis, meningovascualr
disease, auditory symptoms, Treatment failure
optic neuritis, cognitive Controversial: HIV infection
dysfunction,
CSF-VDRL is specific but Controversial: nontreponemal
insensitive titer of >1:32
Elevated CSF protein and ‘CSF should be followed after
WBC (>4/mL) is supportive but treatment until normal (6m, 2y)
non-specific; When in doubt consider
>20cells/mL may be more empiric therapy
specific in HIV-infected

1599
Copyright © Harvard Medical School, 2018. All Rights Reserved.

Syphilis
Follow Response to Treatment

Re-examine clinically and follow non-treponemal test


titer (RPR, VDRL)
Goal: sustained ≥ 4-fold decrease in titer and
symptom resolution
Testing intervals:
Primary/secondary syphilis: 6 and 12 months
Late syphilis: 6, 12, 18, and 24 months
HIV-infection: 3,6,9,12, and 24 months (most
respond to therapy)

Syphilis
Treatment Failure
Definition of treatment failure:
Signs/symptoms persist
Failed to achieve 4-fold decrease in titer within 6-12
months
Sustained 4-fold increase in titer (e.g. 1:8 1:32; repeat
1:32)
Management of treatment failure:
HIV test (should have been done at diagnosis)
CSF analysis
If CSF normal administer benzathine pcn weekly x 3 wks
If titers don’t decline after repeat therapy (re-infection not
suspected and the CSF examination is normal) no
additional therapy indicated

1600
Copyright © Harvard Medical School, 2018. All Rights Reserved.

Case 4: 34 y.o. male with fever, rash,


headache

Well until 7 days prior, when he developed sore


throat, anorexia, diarrhea; + non-pruritic rash
PE: Ill-appearing. T = 103.4. Oroph with ulcer left
tonsil. Mild generalized lymphadenopathy. Diffuse
maculopapular exanthem over body, greatest on
neck, back, trunk, and arms.
LAB: WBC 5.1 (53 P, 11 B, 28 L, 8 M, 0 Atyp), PLT =
125; ALT 88, remainder of labs normal

1601
Copyright © Harvard Medical School, 2018. All Rights Reserved.

34-y.o. male with fever, rash:


Evaluation
Throat culture no beta strep; blood
cultures heterophile, RPR neg;
CMV IgG pos, IgM neg; hep A, B:
no evidence acute infection.

HIV antigen/antibody negative

1602
Copyright © Harvard Medical School, 2018. All Rights Reserved.

What’s the most likely diagnosis?


A) Syphilis
B) Epstein-Barr Virus
C) Acute HIV
D) Primary HSV gingivostomatitis

HIV Viral Load >750,000 copies/ml

1603
Copyright © Harvard Medical School, 2018. All Rights Reserved.

27 yo male with tender swelling in left groin,


multiple sex partners

Lymphogranuloma Venereum
Primary lesions (3-30d incubation): papule of small ulcer,
Indistinct from HSV or syphilis, often painless
Secondary stage (2-6w)
Anogenitorectal: proctitis (purulent, mucous, bloody)
Inguinal: buboes, cellulitis, periadnitis, dissemination
Long-term complications: chronic ulceration, fistulae,
strictures, genital elephantiasis
Diagnosis
Culture: positive in <30% of cases
Nucleic acid testing: may be positive in early stages,
not FDA approved, and requires lab validation
Serology: complement fixation (CF ≥ 1:64) or
microimmunofluorescence (MIF ≥1:128)

1604
Copyright © Harvard Medical School, 2018. All Rights Reserved.

Clinical Manifestations of LGV

Weir. CMAJ 2005;


172:185

Primary Lesion

LGV
Complications

Anal Stricture

Williams et al. BMJ, 2006; 332:99-100

1605
Copyright © Harvard Medical School, 2018. All Rights Reserved.

LGV Therapy

Treat suspected cases, as diagnosis is challenging


Treatment:
doxycycline, 100 mg BID x21 days
alternative: azithromycin 1g/week x 3 weeks
Asymptomatic sex partners:
doxycycline 100 mg bid x 7 days
azithromycin 1g x1

Non-Ulcerative Sexually Transmitted


Diseases

1606
Copyright © Harvard Medical School, 2018. All Rights Reserved.

Clinical Manifestations of Gonorrhea

• Urethritis (men or women)


• Epididymitis
• Cervicitis
• PID
• Proctitis
• Pharyngitis
• Disseminated infection (DGI) (complement
deficiency, women> men, tenosynovitis, dermatitis)
• Conjunctivitis
• Rare: meningitis, hepatitis, endocarditis

Images: CDC

Diagnosing Gonorrhea
• Gram stains: 95-100% sensitive 98%
specific in male urethritis; o/w low
sensitivity
• Culture: for sites not approved for NAAT
(rectal, pharyngeal) or if susceptibility
testing required). 35-370c, 3-5% co2,
Thayer-Martin
• Nucleic acid amplification tests (NAATs)
approved for vaginal, endocervical,
urethral, & urine specimen; 97-99%
sensitive and 99% specific for cervical &
urethral swabs. Some labs validated for
oral and rectal; not FDA cleared

1607
Copyright © Harvard Medical School, 2018. All Rights Reserved.

Use of FQ is NOT
recommended unless
susceptibility results available
Antimicrobial susceptibility
results only available with
culture tests
Perform culture & susceptibility if
GC infection persists or recurs
Most common cause is re-
infection
True Rx failures or resistant GC
isolates should be reported

Treatment of Gonorrhea
IM ceftriaxone (250 mg) plus either doxy 100 mg po bid x 7d or azithromycin 1g po x1
is the treatment of choice
PO cefixime 400 mg is alternative if Ceftriaxone not available plus add doxy x 7d or
azithro 1g po x1 plus test of cure in 1 week; cefuroxime not adequate
True beta-lactam allergy:
Azithromycin 2 g po x1 dose (some resistance reported), testof cure in 1 week
Gemifloxacin 320 mg po x1 plus azithro 2g po x1 (2014 draft guidelines)
Gentamicin 240 mg IM x1 plus azithromycin 2 g po x1 (2014 draft guidelines)
Recent shift towards higher ceftriaxone MICs hence 250mg dose
Higher dose for DGI
Rescreen at 3m or within 12m (NOT A TEST OF CURE)

MMWR August 10, 2012 / 61(31);590-594

1608
Copyright © Harvard Medical School, 2018. All Rights Reserved.

Epidemiology of Chlamydia
trachomatis (serotypes D-K)

Most frequently reported infectious disease in


the US
At least 75% of women and 50% of men have
no symptoms
Rapid diagnostic tests allow for easier office
testing

Screening for Chlamydia


trachomatis
Up to 40% of women with untreated chlamydia will
develop PID. Of those with PID:
20% will become infertile
18% will experience chronic pelvic pain
9% will have a tubal pregnancy
Two CDC asymptomatic screening studies have resulted
in decline in overall infection rates
Chlamydia screening followed by treatment not only
reduced the prevalence of lower genital tract infection,
but also complication rates and cost

1609
Copyright © Harvard Medical School, 2018. All Rights Reserved.

Screening for Chlamydia trachomatis


Screening for Women:
<25 yrs screen annually
>24 screen at least once a year if at risk:
inconsistent use of barrier method
new or more than one partner in last 3 months
other STD
Screening for Men: no routine screening. May be
considered in high risk setting (adolescent and STD
clinics, correctional facilities)

Chlamydia
Rescreening and test-of-cure
CDC guidelines: rescreen all women with
Chlamydia infection 3-4 months after
treatment or when they next present for care
Rescreening is distinct from early retesting to
detect therapeutic failure (test-of-cure)
Except in pregnant women, test-of-cure is not
recommended unless therapeutic compliance
is in question

1610
Copyright © Harvard Medical School, 2018. All Rights Reserved.

Screening Guidelines for MSM*

HIV test if not done in the past year


Syphilis serology with confirmatory testing
Urethral or urinary GC and Chlamydia (if insertive)
Rectal GC and Chlamydia (NAAT preferred, if
receptive)
Pharyngeal GC (NAAT preferred, if receptive oral)
Consider serologic evaluation for HSV-2
HBsAg
HCV if also using drugs (receptive anal? not in
guidelines)
Vaccines for HAV and HBV
Screen q3-6m if multiple anonymous partners

*sexually active

What’s New with 2015 STD Guidelines


Mycoplasma genitalium causes 15-20% of male
NGU and 30% of recurrent persistent urethritis
Pathogenic role in females less clear given
majority asymptomatic but seen in 10-30% of
female clinical cervicitis
Use Azithromycin. Doxy ineffective.
Moxifloxacin 400 mg po daily x 7-14d
Trichomonas vaginalis + test in female rescreen
after 3 months. NAAT testing more sensitive than
wet mount

1611
Copyright © Harvard Medical School, 2018. All Rights Reserved.

What’s New with 2015 STD Guidelines


Annual HCV testing for HIV+ patients
Pts diagnosed and treated for urethritis
should abstain from sex for 7d after treatment
(of patient and partner) and be retested at 3
months to detect repeat infection
HPV9 vaccine recommended for girls ages 9-
26 (ideally given age 11-12) and boys 9-21
(ideal 11-12) and up to 26 years in MSM and
immunocompromised men

Question 1
A pregnant woman is referred for a lab finding
of an RPR titer of 1:8 and a positive FTA
during her first prenatal visit. She reports no
h/o syphilis, and is asymptomatic. She reports
an allergy to beta-lactam antibiotics (hives
during a course each of of amoxicillin and
cephalexin). What is the most appropriate
treatment?

1612
Copyright © Harvard Medical School, 2018. All Rights Reserved.

What is the most appropriate treatment?

a. This is a false positive test due to pregnancy,


no treatment recommended
b. Doxycycline 100 mg orally bid, for 14 days
c. Doxycylcine 100 mg orally bid, for 28 days
d. Azithromycin 2g in a single dose
e. Desensitization, and treatment with penicillin

What is the most appropriate treatment?

a. This is a false positive test due to pregnancy,


no treatment recommended
b. Doxycycline 100 mg orally bid, for 14 days
c. Doxycylcine 100 mg orally bid, for 28 days
d. Azithromycin 2g in a single dose
e. Desensitization, and treatment with penicillin

1613
Copyright © Harvard Medical School, 2018. All Rights Reserved.

Syphilis in Pregnancy
Screen in first prenatal visit
Screen high risk mothers again around 28 weeks
and at delivery
Treat for appropriate stage of syphilis (although
some expert endorse additional doses for early
syphilis)
Treat with penicillin, even if desensitization
required
Repeat RPR titer at 3 months

Question 2

A 57 y/o MSM with allergies to beta-lactams and


azithromycin is seen for a penile discharge and
dysuria. NAAT is positive for both GC and
Chlamydia. He receives a dose of ciprofloxacin
(500mg) and a one week treatment course of
doxycyline. He returns to urgent care at the end of
the treatment course reporting no improvement. A
probe is repeated and is again positive for both
organisms.
Which of the following statements is incorrect?

1614
Copyright © Harvard Medical School, 2018. All Rights Reserved.

Question 2
Which of these statements is incorrect?

a. Re-infection is a common cause of relapsed


symptoms
b. One week is too early for a test of cure (repeat
probes)
c. FQ-resistant gonorrhea should be suspected and
culture and sensitivities are recommended
d. FQ-resistant Chlamydia should be suspected, culture
is recommended, and azithromycin should be
prescribed

Question 2
Which of these statements is incorrect?
Correct answer (incorrect statement) is d

a. Re-infection is a common cause of relapsed


symptoms
b. One week is too early for a test of cure (repeat
probes)
c. FQ-resistant gonorrhea should be suspected and
culture and sensitivities are recommended
d. FQ-resistant Chlamydia should be suspected,
azithromycin should be prescribed

1615
Copyright © Harvard Medical School, 2018. All Rights Reserved.

References
CDC. Sexually transmitted diseases treatment
guidelines, 2015. MMWR 2015;64 No.3
Up to Date in Medicine: Screening for STD’s (last
update May 2017)

1616
Copyright © Harvard Medical School, 2018. All Rights Reserved.

Infectious Diseases Board


Review
Todd Ellerin, MD
Infectious Diseases
South Shore Hospital
Brigham and Women’s Hospital
Harvard Medical School
todd_ellerin_md@sshosp.org

Disclosures
• NONE

1617
Copyright © Harvard Medical School, 2018. All Rights Reserved.

HSV-1 encephalitis; Treatment with high dose IV


acyclovir

50 yo female with 5 days of fevers, increasing confusion, and


personality changes. LP with 350 WBCs (90% lymphocytes),
glucose normal, CSF gram stain and cultures negative

Fish tank granuloma with nodular lymphangitis d/t


Mycobacterium marinum

55 yo male no sig PMHx slammed his hand in a drawer and


developed this nodular rash a couple of weeks later. Not a
gardener. Has 2 pets named sushi and bubbles. What’s the dx?

1,2

1618
Copyright © Harvard Medical School, 2018. All Rights Reserved.

Unexplained seizure
• 36 yo male from Brazil lives near Boston
h/o 1 seizure 20 years before had a 2nd
witnessed seizure.
• Felt well before seizure
• No additional medical hx, no ETOH, no
illicit meds, HIV negative
• Labs normal

1619
Copyright © Harvard Medical School, 2018. All Rights Reserved.

What’s most likely dx


• A) Tuberculous meningitis
• B) Calcified meningioma
• C) Neuroschistosomiasis
• D) Cryptococcoma in immunocompetent pt
• E) Neurocysticercosis

1620
Copyright © Harvard Medical School, 2018. All Rights Reserved.

• A) Tuberculous meningitis
• B) Calcified meningioma
• C) Neuroschistosomiasis
• D) Cryptococcoma in immunocompetent pt
• E) Neurocysticercosis

1621
Copyright © Harvard Medical School, 2018. All Rights Reserved.

1622
Copyright © Harvard Medical School, 2018. All Rights Reserved.

Crusted (Norwegian) Scabies

What’s the diagnosis and the fastest


way to make the diagnosis? What’s the
treatment?

1623
Copyright © Harvard Medical School, 2018. All Rights Reserved.

Crusted (Norwegian) Scabies


• Skin scraping looking for mites/eggs
• Given high burden of mite infestation hundreds
to thousands of mites as opped to 5-15 in
typical cases, treatment:
– ●Topical 5% permethrin or topical 5% benzoyl
benzoate applied daily for seven days, then twice
weekly until cure
• AND
– ●Oral ivermectin (200 mcg/kg/dose) given on days
1, 2, 8, 9, and 15

1624
Copyright © Harvard Medical School, 2018. All Rights Reserved.

Name 3 first line therapies for


cystitis in a woman of child
bearing age

• A) Amoxicillin
• B) Levofloxacin
• C) Nitrofurantoin
• D) Fosfomycin
• E) Trimethoprim/Sulfamethoxazole

1625
Copyright © Harvard Medical School, 2018. All Rights Reserved.

• A) Amoxicillin
• B) Levofloxacin
• C) Nitrofurantoin 100 mg po bid x 5d
• D) Fosfomycin 3g mixed in 4 oz water po
x1
• E) Trimethoprim/Sulfamethoxazole DS
po bid (as long as community sulfa-
resistant E coli <20%)

Enterovirus (Aseptic) meningitis

•34 yo school teacher develops 3-4 days of worsening


HA, stiff neck, and mild photophobia after only being
back to school for 2 weeks after summer break.
No confusion
•No PMHx
•LP reveals 120 WBCs, 90% lymphocytes,
protein 60 mg/dl, glucose normal
What’s the most likely dx?

1626
Copyright © Harvard Medical School, 2018. All Rights Reserved.

Hand Foot and Mouth Disease

•What illness is spreading around the school?

CMV infectious mononucleosis; 10% atypical


lymphocytes

40 yo male develops 3 weeks of unexplained fevers,


sweats, and 10 pound weight loss. Mild pharyngitis early
on that he forgot about. Exam nl x spleen tip.
WBC count 3K with 50% polys and 40% lymphocytes
on diff. HCT nl. PLTs 120K. LFTs elevated
AST 260, ALT 400, TB 2.5, and AP 290.
Monospot negative. What’s the most likely dx?

1627
Copyright © Harvard Medical School, 2018. All Rights Reserved.

1628
Copyright © Harvard Medical School, 2018. All Rights Reserved.

Sporotrichosis and either oral Itraconazole solution or


Fluconazole

51 yo female gardener who was making wreaths from boxwood


prior to Christmas and also works with sphagnum moss
Likely injured fingers but doesn’t recall specific trauma
Biopsy of finger revealed acute and chronic inflammation, loosely
formed granulomas, and numerous oval shaped yeast forms
What’s the likely dx and what’s the treatment

1629
Copyright © Harvard Medical School, 2018. All Rights Reserved.

1630
Copyright © Harvard Medical School, 2018. All Rights Reserved.

Ferritin 6K; Adult-onset Still’s disease; Important ID


principle the longer a patient has bonafide fevers without
a diagnosis, the less likely the dx is infectious (e.g fevers
beyond 6 months rarely infectious!)
55 yo male architect designs marinas reports 9 months
of periodic fevers ranging from 100-103F. Fevers and
polyarthralgias every few weeks. Duration several
days. Intermittent truncal rash and periodic pharyngitis.
Ankles swelled recently. No oral/genital ulcers
No diarrhea or red eye. WBC count 18K (90% polys)
ESR 100. Blood and pharyngeal cxs off abxs all
negative. HIV negative, blood smear for parasites,
ANA, RF, ANCA all negative. What is a key test to
help with diagnosis?

• 45 yo female from Wisconsin notes 10 days of


worsening HA and mild neck stiffness w/o photophobia
• No fevers and no sick contacts
• USOH until 4 weeks ago when she had a left
popliteal fossa cellulitis treated with cephalexin which
led to resolution in less than 1 week
achier than usual but attributes it to workouts

1631
Copyright © Harvard Medical School, 2018. All Rights Reserved.

Lyme meningitis; Ceftriaxone 2g IV q24h x 2-4 weeks

CSF Results

• 100 WBCs (80% lymphs)


• Protein=60 mg/dl
• Glucose= normal
• CSF gram stain negative
• CSF culture negative

• What’s the diagnosis and treatment?

1632
Copyright © Harvard Medical School, 2018. All Rights Reserved.

Parvovirus B19; adults get the lacy rash more than


slapped cheek; usually self-limited polyarthritis of small
joints. Need to make sure she is not pregnant given
potential pregnancy complications; +Lyme IgM false +

28 yo school teacher
develops febrile illness a/w HA and mild diarrhea
and after 8 days develops a rash on her lower extremities.
Her rash fades but she develops swelling in both hands
which persists after 5 weeks.
Her Lyme screen is reactive and her IgM western blot is +
What is the most like dx and what are complications?

Plasmodium falciparum malaria; coArtem (artemether


20mg/lumefantrine 120 mg) time 0, 8 hrs later, then bid x 2d (6
doses total) or malarone (atovaquone 250 mg/proguanil 100 mg) 4
tabs po daily x 3d (assuming no malarone prophylaxis)

40 yo male from Kenya living in Boston returns to Kenya


for 1 month visiting friend and family. He spends a lot of
time outdoors. 2 weeks after returning home he develops
fevers, chills, aches and malaise. No abdominal pain
or diarrhea. No rash. Exam shows moderately ill
appearing male with normal physical exam. Labs
notable for thrombocytopenia (70K) and mild elevation in
transaminases (AST 80, ALT 90). If you only had
1 diagnostic test what would you send and what’s
treatment of choice?

1633
Copyright © Harvard Medical School, 2018. All Rights Reserved.

TMP-SMX is the only antibiotic that lacks anaerobic


activity

Which antibiotic does not belong?

Clindamycin
Imipenem
Trimethoprim/Sulfamethoxazole
Piperacillin/Tazobactam
Cefoxitin

1634
Copyright © Harvard Medical School, 2018. All Rights Reserved.

•20 yo male previously healthy p/w 2 wks fevers, HA, increasing


neck stiffness, and double vision. Immigrated from India 9
months ago, lives in Austin,TX visiting family in Massachusetts.
Given BCG as child
•No sick contacts, exposure to animals/birds, recent travel,
monogamous with wife, business consultant
•Exam reveals temps (99-101F) and b/l CN 6th palsies
•Labs normal except Na 127

•Head CT w/o contrast normal


•Lumbar puncture:
– 775 WBCs/mm3 (56% lymphocytes, 37% polys)
– Protein 169 mg/dl
– Glucose 19 mg/dl
– OP elevated 280 mm water (nl<200)

1635
Copyright © Harvard Medical School, 2018. All Rights Reserved.

1636
Copyright © Harvard Medical School, 2018. All Rights Reserved.

Results
• Bacterial cultures negative
• CSF cryptococcal antigen negative
• CSF fungal cultures negative
• CSF VDRL negative
• CSF HSV PCR
• CSF Tb PCR negative
• CSF AFB cultures negative x 4 wks
• CSF cytology normal
• HIV antibody negative

1637
Copyright © Harvard Medical School, 2018. All Rights Reserved.

Results
• Bronchoscopy revealed negative AFB
smears and fungal cultures
• Mediastinoscopy revealed non-necrotizing
granulomas and fibrosis with special stains
negative for AFB and fungi. No malignancy
• TB PCR on lymph node negative

TB meningitis; TB PCR of CSF low sensitivity and CSF


AFB culture may take 6 weeks to grow

What’s the most likely dx?

1638
Copyright © Harvard Medical School, 2018. All Rights Reserved.

1639
Copyright © Harvard Medical School, 2018. All Rights Reserved.

What’s the dx and cause?

Oral Hairy Leukoplakia


EBV

What’s more important than that?

HIV+

1640
Copyright © Harvard Medical School, 2018. All Rights Reserved.

Rash and periorbital HA after


travel?
• 30 yo female traveled to Brazil 3 mos
before
• Traveled to St. John island 10d before and
spent 5d there and felt well
• 2d after returning home developed HA at
her eyes, mild sore throat, itchy rash starting
on her face and spreading down her chest
and abdomen. No aches. No red eye
• Labs normal x WBC count 4K

1641
Copyright © Harvard Medical School, 2018. All Rights Reserved.

1642
Copyright © Harvard Medical School, 2018. All Rights Reserved.

What’s the most likely dx?


• A) Measles
• B) Dengue
• C) Chikungunya
• D) Zika

1643
Copyright © Harvard Medical School, 2018. All Rights Reserved.

• A) Measles
• B) Dengue
• C) Chikungunya
• D) Zika

1644
Copyright © Harvard Medical School, 2018. All Rights Reserved.

Stage 1 Lyme disease and Babesiosis; Send either blood smear for
parasites or babesia or whole blood DNA testing and consider
Anaplasma whole blood PCR testing; Treat with 10-20 days of
doxycycline and 7-10 days of atovaquone 750 mg suspension bid
and azithromycin 500 mg po d#1 then 250 mg for remainder

40 yo male presents 2 wks after camping in NE in July with


expanding, red, painless circular rash on trunk. He has noted
fevers and HA for about 5 days and rigors over
past 48 h. Labs show normal WBC count
with left shift, PLT count 90K, and
LFTs 2-3x normal with bilirubin of 2.
Hematocrit shows new anemia (30%).
What’s the diagnosis, what diagnostics would
you send, and what’s the tx?

1645
Copyright © Harvard Medical School, 2018. All Rights Reserved.

Blood culture + for gram + cocci in chains=Subacute


bacterial endocarditis. Blood cultures much more
sensitive than TTE for dx!

60 yo male presents with 3 weeks of fevers and


drenching night sweats. Exam normal except resting
tachycardia around 100 bpm. CBC normal x WBC
count 12.5K. CMP normal. UA and CXR both normal.
Referred for TTE which was normal. If you could only
order 1 test what would it be?

1646
Copyright © Harvard Medical School, 2018. All Rights Reserved.

Contraception: An Update
Kari P. Braaten, MD, MPH
Associate Gynecologist, Fish Center for Women’s Health
Director of Quality Assurance, BWH Family Planning
Department of Obstetrics and Gynecology
Brigham and Women’s Hospital

Instructor in Obstetrics, Gynecology and Reproductive Medicine


Harvard Medical School

I have no financial disclosures

1647
Copyright © Harvard Medical School, 2018. All Rights Reserved.

Why we need a fresh look at contraception

• Most Americans want two children.

• Women spend about 5 years pregnant, postpartum or trying to


get pregnant and about 30 years trying to avoid pregnancy.

• About half (49%) of pregnancies in the U.S are unintended


(3.1 million per year).

• The U.S. has one of the highest unintended pregnancy rates in


the developed world

• U.S. women are less likely to use highly effective reversible


contraceptive methods than women in other developed
countries.

Objectives
• Understand differences in real-life effectiveness of
various contraceptive methods

• Understand the benefits and risks of long-acting


reversible contraceptive methods (LARC)
• Reduce barriers and increase access to highly effective
methods

• Utilize CDC contraceptive guidelines


• Medical Eligibility (MEC) and Practice Management (SPR)

1648
Copyright © Harvard Medical School, 2018. All Rights Reserved.

Contraceptive Effectiveness

Contraceptive Effectiveness
% experiencing unintended preg. in 1st yr
Tier Method Typical Use Perfect Use
No method 85 85
Diaphragm 12 6
III
Condom (male) 18 2
Combined pill and minipill 9 0.3
Combined patch (Evra) 9 0.3
II
Combined ring (NuvaRing) 9 0.3
DMPA (Depo-Provera) 6 0.2
IUD
Copper T (Paragard) 0.8 0.6
I
Lng-IUS (Mirena) 0.2 0.2
Etonogestrel implant (Nexplanon) .05 .05
Female Sterilization 0.5 0.5
Male Sterilization 0.15 0.10

Trussell J. Contraceptive Efficacy. In: Hatcher et.al. Contraceptive Technology 20th edition, 2009

1649
Copyright © Harvard Medical School, 2018. All Rights Reserved.

Contraceptive Use and


Unintended Pregnancy
• 46% of unintended pregnancies occur in
contraceptive users1

• The majority contraceptive failures come from


inconsistent or incorrect use
• Only 5% of unintended pregnancies are
related to true method failure1

The Alan Guttmacher Institute. Fact Sheet: Contraceptive Use in the United States, 2016

Each year, one third of U.S. women at risk of


unintended pregnancy are not fully protected

14%

18%

68%

The Alan Guttmacher Institute. Fact Sheet: Contraceptive Use in the United States, 2016

1650
Copyright © Harvard Medical School, 2018. All Rights Reserved.

Continuation rates
Method % Continuation at 1 year
Fertility awareness methods 47
Diaphragm 57
Condom (male) 43
Combined pill and minipill 67
Combined patch (Evra) 67
Combined ring (NuvaRing) 67
DMPA (Depo-Provera) 56
IUD
Copper (ParaGard) 78
LNG-IUS (Mirena) 80
Etonogestrel implant (Nexplanon) 84
Female Sterilization 100
Male Sterilization 100

The Case for Long-Acting


Reversible Contraception (LARC)
• Need for effective contraceptive methods
that are “forgettable,” user-independent

• Traditionally, sterilization has been very


popular in the U.S. (approximately 1/3 of
U.S. contraceptive users)

• 20% of women selecting sterilization at


age < 30 years later express regret
Hillis et al. Obstet Gynecol 1999
Stanwood, NL. Obstet Gynecol 2002
Spinelli et al. Am J Public Health 2000;90:1403
Abma et al. Vital Health Stat 23 1997;19:1

1651
Copyright © Harvard Medical School, 2018. All Rights Reserved.

Characteristics of IUDs and Implants


• Highly effective
• Safe
• Long-term protection
• Single motivational act for insertion (‘forgettable’)
• Not related to coitus
• Immediately effective
• Rapid return to fertility upon removal
• Highest satisfaction among methods (IUDs)
• Appropriate for women with a variety of medical
problems Belhadj H, et al. Contraception. 1986
Skjeldestad F, Bratt H. Advances in Contraception. 1988
Arumugam K, et al. Med Sci Res. 1991

US use of LARC is increasing

1652
Copyright © Harvard Medical School, 2018. All Rights Reserved.

LARC: Effectiveness

Winner B., NEJM 2012

Intrauterine Contraception

• What’s available in the U.S?


• Benefits?
• Side Effects and Risks?

1653
Copyright © Harvard Medical School, 2018. All Rights Reserved.

The IUDs you already know…

CuT380A (Paragard®) Lng-IUS 52 (Mirena®)

TCu380A (Paragard®)
• Contains 380mm2 of copper
• Mechanism of action:
• Foreign body effect – sterile inflammatory response
which is toxic to sperm/ova and impairs implantation
• Copper ions enhance this response
• Efficacy:
• 0.5-0.8% pregnancy rate in first year
• 1.6% cumulative rate at 7, and 2.2% at
8 and 12 years
• Approved for 10 years of use
• Evidence based used for 12 years
1. Trussell J. ContraceptiveEfficacy. In: Hatcher et.al. Contraceptive Technology 20th edition
2. Sivin I, Fertil Steril 1994
3. Rowe P. Contraception 2016

1654
Copyright © Harvard Medical School, 2018. All Rights Reserved.

Levorgestrel-IUS20 (Mirena®)
• Contains 52 mg of levonorgestrel
• Releases 20 mcg daily
• Mechanism of action:
• Foreign body effect
• Endometrial thinning/decidualization
• Cervical mucous thickening
• Efficacy
• 0.1-0.2% pregnancy in first year
• 0.5-1% cumulative pregnancy rate
at 7 years
• Approved for 5 years of use
• Evidence-based use for 6-7 years

The IUDs you maybe don’t know…

Three newer levonorgestrel (progestin)


IUDs
• Skyla – approved Feb 2013
• Liletta – approved Feb 2015
• Kyleena – approved Sept 2016

1655
Copyright © Harvard Medical School, 2018. All Rights Reserved.

Skyla

• Lower dose of levonorgestrel (14mcg /day)


• Approved for 3 years of use
• IUD & inserter smaller than Mirena™(28x32 mm)
• Less amenorrhea than Mirena™
• Possibly less pain with insertion, lower expulsion
• Efficacy:
• First year failure rate 0.3 per 100 women1
• Three year cumulative failure rate 0.9 per 100
women1
1. Nelson A et al. Obstet Gynecol 2013

Liletta
• SAME size and dose of levonorgestrel as Mirena™,
different inserter
• Developed by non-profit pharmaceutical company
• CHEAPER*
The Liletta IUD is the progestin-
• Currently approved for 4 years
containing
• Under FDA review IUDforof choice of the
5 years
• Efficacy, bleeding profile, side effects appear identical
BWH Family
in clinical trials Planning Division
• Efficacy:
• First-year failure rate: 0.15 per 100 women1
• 3-year failure rate 0.55 per 100 women1

1. Eisenberg D et al, Contraception 2015

1656
Copyright © Harvard Medical School, 2018. All Rights Reserved.

Kyleena
• Dose of levonorgestrel between Mirena/Liletta
and Skyla (19.5 mg)
• Rates of amenorrhea higher than Skyla, lower
than Mirena/Liletta
• Same size as Skyla (30 x 28 mm)
• Approved for five years of use
• Efficacy:
• First year failure rate 0.16 per 100 women
• Cum. 5-year failure rate 1.45 per 100 women*
• Kyleena will likely take the place of Skyla

LNG IUS: Side Effects


• Disrupted bleeding patterns
• Extra bleeding and spotting for first 3-6 months

• Mirena/Liletta:
• After 8 months ~50% of women have no menstrual bleeding, just occasional
spotting
• 20% of women have amenorrhea (no periods) at the end of the first year,
increases to ~40% at 3 years
• Overall decrease in bleeding by ~80-90%
• Skyla:
• Initial increase in bleeding/spotting also decreases, but amenorrhea less
common (6% at 1 year)
• Kyleena
• Bleeding decreases over time, amenorrhea ~13% at the end of year 1, 20%
at year 3 and 23% at year 5.
• Overall bleeding > Mirena but < Skyla
Lahteenmaki et.al., Steroids 2000
Eisenberg et al Contraception 2015
Gemzell-Danielsson K. Fertil Steril 2012

1657
Copyright © Harvard Medical School, 2018. All Rights Reserved.

LNG IUS: Side Effects


• Lower abdominal pain
• Possible hormonal side effects (~10% of women)
• Mood changes
• Acne
• Headache
• Persistent ovarian follicles ( 52 mg IUS)
• Breast tenderness (uncommon)
• Nausea (uncommon)
• Most hormonal side effects decrease after 3 months of use

• No reported weight gain

Lahteenmaki et.al., Steroids 2000; 65:693

Cu T380-A (Paragard®)

• Side effects
• No hormonal side effects (no hormones)
• Heavier menstrual bleeding
• Most women experience increased blood loss
• Approximately 50% increase in blood loss
• Especially heavy in first few months post insertion
• More cramping and/or pain
• Menstrual pain, discomfort with intercourse, backache
• Anemia

1658
Copyright © Harvard Medical School, 2018. All Rights Reserved.

Potential IUD (device) complications


• Expulsion (1st year)
• Cu-T380A 3-10%
• LNG-IUS 3-6%
• Perforation (~1/1000)
• Malposition within the uterus
• Embedded in uterine muscle, rotated, too low
• Infection in uterus (pelvic inflammatory disease)
• Risk only increased in first 20 days after insertion
(1-10 / 1000 women)
• After that PID risk only related to risk of acquiring
an STI and rare (1.4 / 1000)
Dean and Goldberg, Up to Date 2013

Contraceptive Implant

• What’s available in the U.S?


• Benefits?
• Side Effects and Risks?

1659
Copyright © Harvard Medical School, 2018. All Rights Reserved.

Implantable Contraceptive Device


Available in the United States

• Etonogestrel Implant
(Nexplanon™)
• Replaced Implanon ™
• Approved for 3 years
use, evidence based
use for 4-5 years
• Differences:
• Radio-opaque
• New easier inserter

Etonogestrel Implant
(NEXPLANON)
• Inserted subdermally in the groove between the
biceps and triceps muscles
• Can only be inserted and removed by clinicians
completing FDA mandated training

• Highly effective, 0.38 pregnancies/100 woman-


years

• Very few absolute contraindications to use.


• Only women with current breast cancer can’t use
• OK with any other medical problems or risk factors
Darney P et. al, Fertil Steril May 2009

1660
Copyright © Harvard Medical School, 2018. All Rights Reserved.

Side effects with Nexplanon


Irregular bleeding is expected.
Patterns are unpredictable. Usually not heavy.
Most discontinuation for irregular bleeding.

10-14% of users:
• Acne
• Headache
• Weight increase

<10% of users:
• Breast Pain
• Emotional changes
• Abdominal Pain
• Decreased Libido
• Nausea

Contraception 1998; 58: 99S-115S

Potential device complications

• Deep insertion / difficult removal


• Nexplanon™ is second generation product
• Easier to use inserter, makes insertion fool-proof
• Visible on X-ray

• Infection at insertion site


• Expulsion (very rare)

1661
Copyright © Harvard Medical School, 2018. All Rights Reserved.

Extended use of LARC


• Strong data for extended use of etonogestrel implant,
copper IUD and 52 mg levonorgestrel IUDs beyond
their FDA approval durations
• Etonogestrel implant: no pregnancies reported up to 5
years of use in 550 women1,2
• Recommended by BWH/PPFA for 4-5 years

• Lng-IUS 52 (Mirena and Liletta): 0.53% cumulative


pregnancy rate at 7 years3
• Recommended by BWH/PPFA 6-7 years

• Copper IUD: 12 years: 2.2% cumulative pregnancy rate


at 12 years, with no pregnancies in years 10 or higher4
• Recommended by BWH/PPFA 12 years
1. Ali et al. Hum Reprod 2016
2. McNicholas et al. Am J Obestet Gynecol 2017
3. Rowe P. Contraception 2016
4. United Nations Development Programme, Contraception 1997

Extended use of LARC


Women do not need to rush to replace!

• Decisions should be individualized with patients


• Consider age, baseline fertility, time to menopause

• Important to consider whether they will replace with


another highly effective method
• Non-LARC methods still less effective

32

1662
Copyright © Harvard Medical School, 2018. All Rights Reserved.

Contraceptive Eligibility

Who is eligible (which patients) for each method?

CDC guidelines:
Medical Eligibility for Contraceptive Use
Updated 2016

1663
Copyright © Harvard Medical School, 2018. All Rights Reserved.

CDC MECs
• Available as free iPhone application
• Can search by medical condition or
contraceptive method

WHO & CDC


Medical Eligibility Criteria (MEC)
• Recommendation Grading System
• Category 1: No restrictions
• Category 2: Generally use (Benefits usually outweigh risks)

• Category 3: Not recommended, unless other


methods not available or
appropriate. (Risks usually outweigh benefits-
use requires careful clinical judgment and access to
clinical services)
• Category 4: Absolute contraindication

1664
Copyright © Harvard Medical School, 2018. All Rights Reserved.

Examples from CDC MEC


COC,
patch, TCu-380A LNG-IUS
ring CDC Risk CDC Risk
Medical Conditions
CDC Risk Category Category
Category
Hypertension
Adequately controlled HTN 3 1 1
Elevated blood pressure levels 3 1 1
SBP 140-159 or DBP 90-99
SBP ≥ 160 or DBP ≥ 100 4 1 2
Headaches
Non-migraine 1 1 1

Migraine
2 1 1
Without aura

With aura
4 1 1

MMWR, July 2016

CDC Selected Practice


Recommendations (SPR) for
Contraceptive Use, 2016

How to initiate and manage each method?

1665
Copyright © Harvard Medical School, 2018. All Rights Reserved.

Examples from CDC SPR, 2016


• Initiating contraception
• How to be reasonably
certain a woman is not
pregnant?
• Switching from one
method to another
• When back-up method
needed?
• Examinations and
tests before starting
• i.e. blood pressure
measurement before
starting an estrogen-
containing method

1666
Copyright © Harvard Medical School, 2018. All Rights Reserved.

Examples from
CDC SPR, 2016

Reducing barriers
Expanding access to LARC

1667
Copyright © Harvard Medical School, 2018. All Rights Reserved.

Removing barriers and expanding access

• Removing restrictions on who is eligible


• Removing requirements for screening tests
• What screening is truly necessary before insertion?

• Provision at point of care


• At time of current office visit (no need to return or
reschedule)
• At the time of abortion
• At the time of delivery

Who can get an IUD? Eligibility


• Nearly all reproductive age women
• OK regardless of age
• OK if nulliparous (or never been pregnant, or never
had sex!)
• OK with nearly all medical conditions
• OK if had a sexually transmitted infection in the past

1668
Copyright © Harvard Medical School, 2018. All Rights Reserved.

Who should not get an IUD?


(Contraindications)
• Women who are currently pregnant
• Women with a current uterine infection (PID)
• Women with current untreated gonorrhea or
chlamydia
• Women with current cervical cancer (pre-
treatment)
• Women with current breast cancer
• OK for Copper IUD, non-hormonal

• Women with uterine anomalies (i.e. bicornuate


uterus)

When can an IUD be inserted?


• IUD can be inserted anytime in the cycle if1
• Patient is consistently using reliable method
• Patient has been abstinent since last period
• Within 5 days of a single act of unprotected IC
• Copper T380A only (as emergency contraception)

• IUDs can be inserted immediately after1


• Abortion or miscarriage
• At the time of procedure if having uterine evacuation
• Once pregnancy has passed (if medical tx or spontaneous)
• Delivery
• Right after the placenta, at vaginal birth or c-section.

1. Carusi and Goldberg, Up To Date, 2010

1669
Copyright © Harvard Medical School, 2018. All Rights Reserved.

Impact of post-abortion
IUD insertions
• Increased utilization at 6 months
• Reduced repeat unintended pregnancy

Bednarek et.al., NEJM 2011 Goodman et.al., Contraception 2008


Hohmann et.al., Contraception 2012 Roberts et.al., Contraception 2010
Cremer et.al., Contaception 2011

Professional Organization Statements

• ACOG 2012: LARC methods are safe and


appropriate for most women including nulliparous
women and adolescents. Adolescents should be
encouraged to consider LARC1

• AAP 2014: LARC methods should be considered


first-line for adolescents2

1. ACOG Committee opinion no. 539. Obstet Gynecol 2012


2. Contraception for adolescence. Pediatrics 2014

1670
Copyright © Harvard Medical School, 2018. All Rights Reserved.

Impact of LARC:
Contraceptive CHOICE Project
• 9,256 adolescents and women in the St Louis area offered
all contraceptive methods at no cost, 2007-2011
• Counseling focused on efficacy
• 75% of women chose LARC, including 2/3 of adolescents
• 12-month continuation rates >80%, high rates of
satisfaction1
• LARC users were 22 times LESS likely to have an
unintended pregnancy than non-LARC users2
• Abortion rates in the cohort <50% that of regional and
national rates.3
• Significant reduction in the percentage of abortions
that were repeat abortions.3 1. Rosenstock JR et al. Obstet Gynecol 2012
2. McNicholas et. al. Clin Obstet Gynecol 2014
3. Peipert JF et al, Obstet Gynecol 2012

Greater use of long-acting


reversible contraception holds
great promise as a means to
reduce unintended pregnancy in
the United States.

1671
Copyright © Harvard Medical School, 2018. All Rights Reserved.

Take-home points
• Challenges with correct use, adherence and
continuation of short-acting contraceptive methods are
leading contributors to undesired pregnancy rates in
the US
• Guidelines are available to help clinicians determine,
who, when and how contraceptive methods can be
used.
• Increased use of LARC methods holds significant
promise in reducing unintended pregnancy rates
• Women want to use LARC, and we must work to
reduce barriers restriction women’s use of these
methods

Question 1
Which of the following LARC methods does not have
evidence to support extended use?
A. Lng IUS 52 (Mirena)
B. Lng IUS 13.5 (Skyla)
C. CuT380A (Paragard)
D. Etonogesterel contraceptive implant (Nexplanon)

1672
Copyright © Harvard Medical School, 2018. All Rights Reserved.

Question 1

Lng IUS 13.5 (Skyla) does not have evidence that it


protects from pregnancy at greater than 3 years.
Exchange should be encouraged at the three year
mark.

Question 2
Which patient is NOT eligible for a LNG IUD?

a) 19 y.o, G0, monogamous woman

b) 40 y.o. G2P2, with recent new partner. History of


chlamydia as a teen.

c) 21 y.o, G4P2, diagnosed with chlamydia a few weeks


ago, but never took the antibiotics.

d) 35 y.o. G3P3, monogamous woman with fibroids

1673
Copyright © Harvard Medical School, 2018. All Rights Reserved.

Question 2
Untreated genital tract infection is a contraindication to IUD
insertion.
This patient may be eligible for insertion after completion of
treatment.
Adolescence, nulliparity, previous STI, fibroids are not
contra-indications.

References
• Bednarek et.al., NEJM 2011
• Hohmann et.al., Contraception 2012
• Peipert et.al., Obstet Gynecol 2012
• Goodman et.al., Contraception 2008
• Roberts et.al., Contraception 2010

1674
Copyright © Harvard Medical School, 2018. All Rights Reserved.

I have no financial disclosures

Thank you
Questions?

1675
Copyright © Harvard Medical School, 2018. All Rights Reserved.

Medical Complications of Pregnancy

Ellen W. Seely, M.D


Director of Clinical Research
Endocrinology, Diabetes & Hypertension Division
Department of Medicine
Brigham and Women’s Hospital
Professor of Medicine
Harvard Medical School

Potential Conflicts:
None

1676
Copyright © Harvard Medical School, 2018. All Rights Reserved.

Objectives
•To understand the implications of common
medical conditions on pregnancy
•To be able to appropriately change medical
management of patients prior to pregnancy
•To understand difference in disease
management goals in pregnancy

Common Complicating Medical Conditions


that Precede Pregnancy
Hypertension ~5-10%

Diabetes Mellitus ~1-2%

Hypothyroidism~2-3%
Leading causes of hospitalization in pregnancy:
■ Preterm labor
■ Hypertension
■ Diabetes Mellitus
■ Bleeding

1677
Copyright © Harvard Medical School, 2018. All Rights Reserved.

Why Is it Important for Medical Caregivers


to Know about these?

Prepregnancy Planning and Counseling


Key to Successful Pregnancy Outcome

Most pregnancies are unplanned

Hypertensive Disorders in Pregnancy


Chronic Hypertension:
Hypertension before 20 weeks gestation
Preeclampsia:
Hypertension with proteinuria or other end organ manifestations
after 20 weeks gestation
Gestational Hypertension:
Hypertension after 20 weeks gestation , without signs of
preeclampsia
Superimposed Preeclampsia:
Preeclampsia in woman with pre-existing hypertension

ACOG Hypertension in Pregnancy, 2013 Ob Gyn

1678
Copyright © Harvard Medical School, 2018. All Rights Reserved.

Risks of Hypertension in Pregnancy

Fetus/Neonate
Intrauterine growth restriction-Small for gestational
age infant

Prematurity (induced)

Mother
Exacerbation of hypertension and risk of stroke, MI,
renal failure

Superimposed preeclampsia

Chronic Hypertension and


Preeclampsia Rates

General population 4-5%


Preexisting hypertension 25 %
< 4 y duration 22%
> 4 y duration 31%
DBP <100 mm Hg 24%
DBP 100-110 mm Hg 42%
Based on BP at visit regardless of antihypertensive Rx

Adapted from Sibai Am Obstet Gyn 2002;100:369

1679
Copyright © Harvard Medical School, 2018. All Rights Reserved.

Preexisting Hypertension in Pregnancy

Pregnancies do well unless superimposed


preeclampsia develops

Goal SBP 120-160; DBP 80-105 mm Hg *

Antihypertensives generally can be tapered


during pregnancy to lower fetal exposure

* ACOG Hypertension in Pregnancy 2013, Ob Gyn

Control of Hypertension in Pregnancy Study


CHIPS Results
987 women; 74.6% had preexisting hypertension
Tight control DBP goal 85 mmHg (vs less tight DBP
100 mmHg) did not affect rates
Primary outcome: pregnancy loss/high-level
neonatal care >48 hours 31.4% and 30.7%, respectively;
adjusted odds ratio, 1.02; 95% confidence interval [CI], 0.77 to 1.35
Secondary outcomes: serious maternal
complications 3.7% and 2.0%, respectively; adjusted odds ratio,
1.74; 95% CI, 0.79 to 3.84
DID lower rates of severe hypertension (≥160/110 mm Hg) (40.6% less-tight,
27.5% of tight)

NEJM 2015

1680
Copyright © Harvard Medical School, 2018. All Rights Reserved.

Pharmacologic Treatment of Preexisting


Hypertension in Pregnancy

CEIs and ARBs prior to conception


fetal anuria and oligohydramnios (2nd & 3rd trimester)
neonatal renal failure (2nd & 3rd trimester)
birth defects? (1 trimester Cooper et al. N Engl J Med, 354: 2443-51
st

2006 versus Li DK et al. BMJ. 2011 Oct 18;343:d5931; Bateman et al, Am J Ob


Gyn 2015)

Antihypertensive Options

Methydopa
used since 1960s
infant F/U up to 7.5 yrs (Cockburn J et al. Lancet 1982;1(8273):647)
First line option
Alpha-Beta blocker
labetolol alternative first line (ACOG/Canadian)
Beta blockers
Concern with growth restriction with atenolol
Calcium channel blockers
Only long acting ones
Diuretics (concern of dehydration expressed by ACOG)

1681
Copyright © Harvard Medical School, 2018. All Rights Reserved.

Summary: Chronic Hypertension and


Pregnancy
Importance of family planning with all
women with hypertension
Awareness re need to switch from certain
antihypertensives (ACEIs, ARBs)
Knowledge that BP goals differ during
pregnancy
Awareness of hypertensive complications
of pregnancy

Diabetes Mellitus

Birth defect risk rises related to level of


HbA1C at conception- as high as 20%

Preconception glucose control can reduce


birth defects to ~1% over general
population

1682
Copyright © Harvard Medical School, 2018. All Rights Reserved.

Risks of Diabetes Mellitus in Pregnancy

Fetus/Neonate
Birth defects

Macrosomia

Prematurity (induced)

Birth Trauma

Neonatal Hypoglycemia

Mother
Progression of diabetic complications: retinopathy, nephropathy

Preeclampsia

Increased chance for C-section

Prepregnancy Planning for Women with


Diabetes
Discuss family planning at least yearly and stress the
importance of prepregnancy glucose control

If HbA1C not in normal range, plan with patient 6 months


to achieve glucose control prior to conception

Counseling re risks

Prepregnancy determination of retinopathy, nephropathy


and other complications (i.e. gastroparesis)

Adjustment of diabetes medications (including BP meds)

1683
Copyright © Harvard Medical School, 2018. All Rights Reserved.

Optimal Diabetes Control for Conception

American Diabetes Association control goal:


HbA1C < 6.5 %

Diabetes Care 2018

1684
Copyright © Harvard Medical School, 2018. All Rights Reserved.

Management of Diabetes Mellitus


Preconception and During Pregnancy
Diet and home glucose monitoring are the mainstay
of management
For Type 2 DM, if on other agents, change to
insulin- insulin remains the gold standard for
treatment of diabetes during pregnancy.
“Insulin is the preferred medication ; Noninsulin
medications lack long-term safety data” ADA. Standards
of medical care in diabetes-2015.
Diabetes Care 2015;38 (suppl 1):S1-S93

Oral agents for Type 2 DM - the jury is out.

Insulin in Management of Diabetes Mellitus


Preconception and During Pregnancy
Only human insulin should be used
Long experience with NPH and regular
Shorter acting i.e. lispro and aspart are commonly
used
Longer acting:
Levemir
Glargine insulin controversial (Category C) increased IGF-1-
receptor affinity and mitogenic potency in human
osteosarcoma cell-culture model
Pumps can be used

1685
Copyright © Harvard Medical School, 2018. All Rights Reserved.

Insulin resistance during pregnancy

Insulin resistance increases as pregnancy


progresses
Insulin requirements typically increases in
pregnancy but may decrease just before
delivery therefore doses needed to be
adjusted throughout pregnancy

Glucose goals during pregnancy

FBS <95 mg/dl


Post meals
1 hr <140 mg/dl
2 hr <120 mg/dl
HbA1C <6% (if achievable without too
much hypoglycemia)

ADA and ACOG

1686
Copyright © Harvard Medical School, 2018. All Rights Reserved.

Gestational Diabetes Mellitus and the


Primary Care Physician

GDM: Diabetes first diagnosed in pregnancy


after the 1st trimester, resolves with delivery

Risk of future Type 2 DM as high as 60-70%

ADA recommends regular screening of all


women with a hx of GDM for Type 2 DM (yearly
and if normal can move to q 3 yrs)

Gestational diabetes and the incidence of type 2 diabetes

80
Cumulative incidence of type 2 diabetes (%)

70

60

50

40

30 Latin American
Mixed or other
20 Boston cohort
Zuni
10 Navajo

0
0 5 10 15 20 25 30
Length of follow up after delivery (years)

Adapted from Kim C et al. Diabetes Care. 2002 Oct;25(10):1862-8.

1687
Copyright © Harvard Medical School, 2018. All Rights Reserved.

Summary: DM and Pregnancy


Need for pregnancy planning to normalize
HbA1c
May need to switch medications (both
glycemic and antihypertensive meds) before
conception
Insulin doses require adjustment throughout
pregnancy
Women with a history of GDM should
receive regular screening for Type 2 DM

Hypothyroidism

1688
Copyright © Harvard Medical School, 2018. All Rights Reserved.

Thyroid Changes in Pregnancy

Diagnosis often complicated by the changes in thyroid


function that take place in normal pregnancy.

Early pregnancy, rise in HCG (has thyroid stimulatory


activity), leads to compensatory fall in TSH.

TSH concentration is usually within the normal range;


in some women falls into the suppressed range.

Relationship between TSH and hCG


1.5— —50

TSH
TSH (uIU/mL)

—40
hCG (IU/L x 103)

1.0—

—30

—20
0.5—
hCG
—10

—0
0 10 20 30 40
Weeks of Gestation
Adapted from Glinoer et al. JCEM 1990; 71(2):282

1689
Copyright © Harvard Medical School, 2018. All Rights Reserved.

Serum TSH in Pregnancy: Mean and 95% CI


4.5

3.5
TSH (mIU/mL)

2.5

1.5

0.5

0.03

1st trimester 2nd trimester 3rd trimester


10 20 30 40
Weeks Gestation
Adapted from Panesar et al. Ann Clin Biochem 2001; 38: 329

TSH normal range in pregnancy


■ Use pregnancy specific normal range of
your lab
■ If trimester-specific reference ranges for
TSH are not available in the laboratory, the
following reference ranges are
recommended:
■ First trimester, 0.1-2.5 mIU/L
■ Second trimester, 0.2-3.0 mIU/L
■ Third trimester, 0.3-3.0 mIU/L.
Guidelines of the American Thyroid Association for the Diagnosis and Management of Thyroid
Disease During Pregnancy and Postpartum

1690
Copyright © Harvard Medical School, 2018. All Rights Reserved.

Hypothyroidism: Diagnosis In
Pregnancy
2-10% of pregnancies
Universal screening not recommended by ACOG,
the Endocrine Society, or American Association of
Clinical Endocrinologists
Diagnosis made by elevated TSH
Symptoms:
weight gain
constipation (typical of normal pregnancy)
cold intolerance (not typical of pregnancy)

Clinical Hypothyroidism: Risks to


Pregnancy

Fetal/Infant
Poor growth

Premature Birth

Pregnancy loss

decrease in IQ

Maternal
Increases risk for preeclampsia, placental
abruption (preterm delivery)

1691
Copyright © Harvard Medical School, 2018. All Rights Reserved.

Subclinical Hypothyroidism: Risks to


Pregnancy

Fetal/Infant
Poor growth

Premature Birth conflicting data

Early Pregnancy loss +

decrease in IQ controversial

Maternal
Increases risk for preeclampsia, placental
abruption (preterm delivery)

Hypothyroidism: Treatment Prior to and


During Pregnancy
Aim for TSH <2.5 at conception

Check TSH as soon as pregnancy test positive

Requirement for thyroid hormone characteristically


increases in pregnancy and returns to prepregnancy
requirement postpartum

Titrate thyroid hormone dose to maintain TSH <2.5 1st and


2nd trimesters, <3.5 3rd trimester

Reduce thyroid hormone dose to prepregnancy dose at


delivery and check TSH at 6 wk pp visit

1692
Copyright © Harvard Medical School, 2018. All Rights Reserved.

Summary Hypothyroidism and Pregnancy

TSH values vary through pregnancy and may be


suppressed in first trimester due to rise in HCG

TSH levels run lower in pregnancy so aim for T4


replacement is a lower TSH.

Overall Summary

Women with medical problems should receive


regular family planning counseling

Adjustment of medications is often necessary prior


and during to pregnancy

Goals for treatment of medical problems may differ


during pregnancy

1693
Copyright © Harvard Medical School, 2018. All Rights Reserved.

Potential Conflicts:
None

Question 1
You see a 34 yo women with Type 2 diabetes and
hypertension who has recently stopped using
contraception. She is taking glyburide and lisinopril. Her
last HbA1c was 9%.

Your recommendations include all of the following except:


a) Aim for HbA1c <6.5%.

b) D/C lisinopril and substitute labetolol before she tries to become


pregnant

c) D/C lisinopril and substitute an angiotensin 2 receptor blocker (ARB)


to help protect her kidneys during pregnancy

d) Eye exam for retinopathy assessment

1694
Copyright © Harvard Medical School, 2018. All Rights Reserved.

Question 1-Answer
Your recommendations include all of the following except: Answer C

a) Aim for HbA1c<6.5%. This is a correct goal for conception per ADA.

b) D/C lisinopril and substitute labetolol before she tries to become


pregnant. Lisinopril is contraindicated in pregnancy and labetolol is a
1st line medication.

c) D/C lisinopril and substitute an angiotensin 2 receptor blocker (ARB)


to help protect her kidneys during pregnancy. Both ACEIs and ARBs
cause fetal renal damage.

d) Eye exam for retinopathy assessment. An eye exam should be


performed prior to pregnancy in women with diabetes.

Question 2

A 24 yo women reports a home positive pregnancy test


and having missed her period. Because of fatigue, another
clinician ordered a TSH to rule out hypothyroidism and
which returned at 5.0 mIU/L. She has no symptoms.

You:
a) Recommend recheck of TSH in 4 wks.

b) Explain that TSH values may increase over the course of


normal pregnancy due to the rise in HCG.

c) Start T4 with a goal TSH < 2.5.

d) Start T4 with a goal TSH < 4.5.

1695
Copyright © Harvard Medical School, 2018. All Rights Reserved.

Question 2 Answer
You: Answer D
a) Recommend recheck of TSH in 4 wks. Incorrect:T4
requirement rises over the course of pregnancy.

b) Explain that TSH values usually increase in 1st trimester of


normal pregnancy due to rise in HCG. Incorrect:TSH levels
typically fall in the 1st trimester due to rise in HCG.

c) Start T4 with a goal TSH < 4.5. Incorrect: TSH goals are
lower in pregnancy. Do not use non pregnant nl range.

d) Start T4 with a goal TSH < 2.5. Correct: TSH goals are
lower in pregnancy.

References
Seely EW, Ecker JL. Medical complications in pregnancy. In:
Singh AK, editor. Scientific American medicine [online].
Hamilton (ON): Decker Intellectual Properties; June 2016.
DOI: 10.2310/7900.1041. Available at:
http://www.sciammedicine.com (accessed June 4, 2018).
Seely EW, Ecker J. Chronic hypertension in pregnancy.
Circulation. 2014;129(11):1254-61.
Executive summary: hypertension in pregnancy. ACOG.
Obstet Gynecol 2013;122: 1122.
Standards of Medical Care in Diabetes 2018. Diabetes Care.
2018 Jan; 41(Supplement 1): S137-S143.
Alexander EK, Pearce EN, Brent GA, et al. 2017 Guidelines of
the American Thyroid Association for the Diagnosis and
Management of Thyroid Disease During Pregnancy and the
Postpartum. Thyroid. 2017; 27(3):315.

1696
Copyright © Harvard Medical School, 2018. All Rights Reserved.

Global Burden Of HPV-Related Disease


The Challenges of HPV Vaccine
Implementation

Annekathryn Goodman, MD, MPH

Disclosures
• none

1697
Copyright © Harvard Medical School, 2018. All Rights Reserved.

OUTLINE
• Background
– cervical cancer stats
– Human Papillomavirus Infection (HPV)
• Vaccination
• Screening
• Management of abnormal screening tests
• Management of abnormal cervical biopsies

Cervical Cancer
• Cervical cancer is the most common HPV-
associated cancer among women
– 500,000+ new cases and 275,000 attributable deaths
world-wide in 2012
– 13,240 new cases and 4,170 attributable deaths in
estimated for 2018 in the U.S.
– 25.9% cervical cancers occur in women who are
between the ages of 35 and 44
– 14% between 20 and 34
– 23.9% between 45 and 54

1698
Copyright © Harvard Medical School, 2018. All Rights Reserved.

Cervical Cancer
HPV-associated Disease
• Types 16, 18: US ETIOLOGY
– ~62% of HPV-associated cancers
• Human Papillomavirus
– ~50% of ≥ CIN*2
• Types 31, 33, 45, 52, 58
Infection (HPV)
– Worldwide: ~20% of invasive cervical • Persistence of HPV
cancer
– US • Lack of treatment of
~14% of HPV-related cancers in
women pre-invasive disease
• ~15% of invasive
cervical cancer
• ~25% of ≥ CIN2
~5% in men
• Types 6, 11
– 90% of anogenital warts

*CIN = cervical intraepithelial neoplasia 5


Serrano B, et al. Infect Agent Cancer. 2012;7:38.

HPV Transmission

• 80% of people will be infected with HPV in their


lifetime.a
– The most common route of transmission is sexual
intercourse: genital-genital, anal-genital, oral-genital,
manual-genital.
• Nearly 50% of high school students have engaged
in sexual (vaginal-penile) intercourse.b
– 1/3 of 9th graders and 2/3 of 12th graders have
engaged in sexual intercourse.b
– 24% of high school seniors have had sexual intercourse
with 4 or more partners.b

a. Dunne EF, et al. JAMA. 2007;297:813-819. 6


b. Jemal A, et al. J Natl Cancer Inst. 2013;105:175-201.

1699
Copyright © Harvard Medical School, 2018. All Rights Reserved.

HPV (High Risk): Natural History

• 3-8 month incubation period

• 80% cleared in 12 months

• 95% cleared by three years

• Less than 1% of all HPV High


Risk infections lead to
invasive cancer

HPV Types Differ in Their


Disease Associations

Mucosal Cutaneous
~40 Types Sites of infection Sites of infection ~ 80 Types

High risk (oncogenic)


HPV 16, 18, 31, 33, 45, 52, 58 Low risk (non-oncogenic)
HPV 6, 11

Cervical Cancer
Anogenital Cancers Genital Warts “Common”
Oropharyngeal Cancer Laryngeal Papillomas Hand and Foot
Cancer Precursors Low Grade Cervical Disease Warts
Low Grade Cervical Disease

1700
Copyright © Harvard Medical School, 2018. All Rights Reserved.

New Cancers Probably Caused by HPV,


United States, 2006-2010

Oropharynx Vagina Vulva Anus Cervix Penis

n=
n = 2200 n = 700
600 13% n = 2600 8%
3% 15%
nn == 1800
1800
10%
10%
n = 10,400
59% n = 7200
77%

Women (n = 17,600) Men (n = 9300)

9
CDC. http://www.cdc.gov/vaccines/who/teens/for-hcp/hpv-resources.html.

HPV Vaccines

• Safe, well tolerated, and effective HPV


vaccines are available that could prevent
most HPV-related cancers.
• There are clear recommendations from
ACIPa,b and AAPc for use of HPV vaccines in
11 through 12-year-old girls and boys.

a. Markowitz LE, et al. MMWR Recomm Rep. 2014;63:1-30.


b. Petrovsky E, et al. MMWR Morb Mortal Wkly Rep. 2015;64:300-304. 10
c. AAP. Red Book. 2015.

1701
Copyright © Harvard Medical School, 2018. All Rights Reserved.

Available HPV Vaccines

Bivalent Quadravalent 9-valent


(Ceravix®) (Gardasil®) (Gardasil®9)
Manufacturer GlaxoSmithKline Merck Merck

6,11,16,18
L1 VLP types 16,18 6,11,16,18
31,33,45,52,58

Trichoplusia ni insect cell line Saccharomyces Saccharomyces


Manufacturing infected with L1 encoding cerevisiae (Baker’s cerevisiae (Baker’s
recombinant baculovirus yeast) – expressing L1 yeast) – expressing L1

Females 9-26 y Females 9-26 y


Licensed Females 9-25 y
Males 9-26 y Males 9-15 y

11
Petrovsky E, et al. MMWR Morb Mortal Wkly Rep. 2015;64:300-304.

HPV Vaccine Efficacy


Results of Selected Clinical Trials

Outcome Vaccine Sex Efficacy, %

Bivalent and
Cervical precancer Females > 93
quadrivalent

Vaginal/vulvar
Quadrivalent Females 100
precancer

Anal precancer Quadrivalent Males 75

Females 99
Anogenital warts Quadrivalent
Males 98

12
Dunne EF, et al. MMWR Morb Mortal Wkly Rep. 2014;63:1-84.

1702
Copyright © Harvard Medical School, 2018. All Rights Reserved.

ACIP/AAP Recommendations for Use of


HPV Vaccines
• Routine vaccination at age 11 or 12 years
– Can start at age 9 years

• Vaccination for females age 13 through 26 years and males


age 13 through 21 years* who have not completed the
series
• Males age 22 through 26 years* may be vaccinated
• Males age 22 through 26 years who are
immunocompromised and MSM should be vaccinated
*Recommendation for males ≥ 16 years is off label for the
9vHPV vaccine.
Markowitz LE, et al. MMWR Recomm Rep. 2014;63:1-30.; Petrovsky E, et al. 13
MMWR Morb Mortal Wkly Rep. 2015;64:300-304.; AAP. Red Book 2015.

ACIP/AAP Recommendations for Use of


HPV Vaccines

• Females: 2vHPV, 4vHPV (as long as available), or


9vHPV vaccine can be used
• Males: 4vHPV (as long as available) or 9vHPV
vaccine can be used
• 3-dose schedule
– 2nd at least 1 month after 1st, 3rd at least 6 months after
1st

• Complete series with same HPV product when


possible
– 9vHPV vaccine can be used to complete the 4vHPV 14
vaccine series

1703
Copyright © Harvard Medical School, 2018. All Rights Reserved.

Guidance on ACIP Website:


Additional HPV 9 Vaccine Doses
Completing HPV vaccination
http://www.cdc.gov/vaccines/who/teens/downloads/9vHPV-guidance.pdf
• There is NO ACIP recommendation for
routine additional 9vHPV vaccination for
anyone that’s already completed 4vHPV or
2vHPV vaccination series.
• No serious safety concerns giving 9vHPV
after 3 dose 4vHPV series (but more injection
site reactions)
• Additional HPV 9v protection: mostly limited
to females (cervical cancers & pre-cancers)

HPV Vaccine in
Immunocompromised Host
• Transplant recipient & HIV infection with <200
CD4 counts higher risk of HPV-related disease
• Vaccine safe and immunogenic
• ACIP recommends Vaccination through age 26
years

16

1704
Copyright © Harvard Medical School, 2018. All Rights Reserved.

HPV Vaccine and Adult Women


• Placebo-controlled randomized trial of HPV4 underway
– 38,000 women (ages 24-34, 35-45)
– Excluded women with h/o LEEP, biopsy-proven cervical
HPV, vulva/ vaginal pre-cancers, h/o genital warts
– At enrollment, 1/3 positive for exposure to >1 vaccine
type
– So far (>3 yrs): good vaccine efficacy (overall 89%)
against persistent infection, dysplasia, and genital warts
– Efficacy better in younger women
– Cost effectiveness of vaccine decreases as age
increases
– Still considered “off label for women > 26 years
LEEP- Loop Electrosurgical Excision Procedures
Data – presented to ACIP 2/10

HPV Vaccine and Adult Women


Lancet Infectious Dis 2016

• Double blind vaccine (2vHPV) vs placebo


• Women > 25 years; 5747 participants
• HPV 16/18 vaccine continues to protect against
infections, cytological abnormalities, and lesions
associated with HPV 16/18 and CIN1+ irrespective
of HPV type, and infection with non-vaccine
types HPV 31 and HPV 45 over 7 years of follow-
up.
18

1705
Copyright © Harvard Medical School, 2018. All Rights Reserved.

HPV Vaccine FAQ


Can you get HPV infection from getting the
vaccine?
No, the vaccine does not contain any viral DNA so
there is no way to become infected with the virus
by getting the vaccine.
Do you have to do pregnancy test before
giving vaccine?
No, but neither vaccine should be given to women
who are pregnant or are planning to get pregnant
soon.

HPV Vaccine FAQ


Can you get HPV vaccine while nursing?
Yes, ACIP says lactating women can receive HPV
vaccine.
Can the HPV vaccine be used to treat
abnormal pap smear?
NO. They are not meant to be a treatment for HPV
infection or HPV related disease.

1706
Copyright © Harvard Medical School, 2018. All Rights Reserved.

HPV vaccine FAQ


• Do you still have to get cervical cancer
screening if you get HPV vaccine?
– Yes. HPV Vaccines are prophylactic vaccines. They
work best if given before exposure to HPV virus.
They are not meant to be a treatment for HPV
infection or HPV related disease. Women must still
get regular cervical cancer screening. (Begin at age 21)

HPV Vaccine
Adverse Events

• Local
– Injection site swelling (29.1-40.3%)
– Injection site erythema (25.8-34%)
• Systemic
– Pyrexia (10.1% vaccine vs 8.4% placebo)
– Headache, dizziness, myalgia, arthralgia, GI (0.5%
difference between vaccine & placebo)

1707
Copyright © Harvard Medical School, 2018. All Rights Reserved.

HPV Vaccine
Severe Adverse Effects
• No SAE
• No relationship between exposure to HPV
vaccination and autoimmune outcomes, Multiple
sclerosis, demyelinating conditions, venous
thromboembolism
• No deaths considered vaccine related
• No increase SAE with combo with other vaccines

International uptake
of 3 doses HPV vaccine

Australia UK Canada Netherlands USA

Brotherton, Lancet 2011; Cuzick BJC 2010; Ogilvie et al., 2010; Marc et al., 2010, NIS-Teen 2011

1708
Copyright © Harvard Medical School, 2018. All Rights Reserved.

HPV Vaccine Impact:


High HPV Vaccine Coverage in Australia
• 80% of school-age girls in Australia are fully
vaccinated
• High-grade cervical lesions have declined in women
less than 18 years of age
• For vaccine-eligible females, the proportion of genital
warts cases declined dramatically by 93%
• Genital warts have declined by 82% among males of
the same age, indicating herd immunity

Garland et al, Prev Med 2011


Ali et al, BMJ 2013

Summary
• HPV is the most common STI in adolescents and is
directly linked to anogenital warts and cervical cancer
• To date, HPV vaccine is safe and highly efficacious in
preventing precursors to cervical cancer
• Routine vaccination of 11-12 year-old girls is supported
by the CDC, ACIP and AAP, with catch-up for women
through age 26
• Males can now be offered vaccination with Gardasil
• Parents are generally accepting of this vaccine,
especially if counseled correctly

1709
Copyright © Harvard Medical School, 2018. All Rights Reserved.

Why does Cervical Cancer Happen?


• Social Ecology
• Risk Factors High Income Countries
– Poverty
– Access to health care
– Lack of medical insurance
• Risk Factors LMIC
– Poverty
– Lack of screening

Social Ecology of Cervical Cancer

1710
Copyright © Harvard Medical School, 2018. All Rights Reserved.

Social Ecology Levels


• Intrapersonal
• Interpersonal
• Organizational
• Community
• Society

Challenges to Cervical Cancer


Screening
General Influences INTRAPERSONAL LEVEL
Barriers to Screening
Personality Depression, anxiety
Comprehension Ability to navigate healthcare system
Genetics Less common high risk subtype
Family Family history of cervical cancer
Home environment Shame, role of family decision maker

1711
Copyright © Harvard Medical School, 2018. All Rights Reserved.

Challenges to Cervical Cancer


Screening
General Influences Barriers to Screening
INTERPERSONAL LEVEL
Culture Fatalistic beliefs
Social Mores Smoking
Employment Unemployed

Challenges to Cervical Cancer


Screening
General Influences ORGANIZATIONAL LEVEL
Barriers to Screening
School Lack of healthcare awareness
Health Insurance Lack of healthcare coverage
Race, Ethnicity Minority status, discrimination, trust

1712
Copyright © Harvard Medical School, 2018. All Rights Reserved.

Challenges to Cervical Cancer


Screening
General Influences Barriers to Screening
COMMUNITY LEVEL
Socio-economic status Low socio-economic status
Public resources Citizenship status
Healthcare facilities Delay of services
Economics Poverty

Challenges to Cervical Cancer


Screening
General Influences Barriers to Screening
LEVEL
Educational system Utilization of healthcare
Governmental policy Lack of financial support

1713
Copyright © Harvard Medical School, 2018. All Rights Reserved.

Cervical Cancer Prevention


• Primary: HPV Vaccination
• Secondary: Screening
– Pap , colposcopy, biopsies, treatment
– Visual Inspection with Acetic Acid (VIA), colpo, RX
– VIA, treatment
– HPV testing, colpo, treatment
– VIA & HPV testing, treatment

Guidelines
• www.asccp.org/Portals/9/docs/AS
• Guidelines are only for women at CCP%20Management%20Guideli
nes_August%202014.pdf
average risk for cervical cancer.
• App available for iPad, iPhone and
Android
• Guidelines should never
• These guideline do not apply to replace clinical judgment.
women with:
– History of cervical cancer
– In Utero exposure to DES
– Immuno-compromised
– HIV positive

1714
Copyright © Harvard Medical School, 2018. All Rights Reserved.

The Dangers of Guidelines


• 67 year old nurse with h/o DES exposure
• s/p cryotherapy and LEEP in her 20’s
• Last pap 2014 normal
• Medicare denied approval of repeat pap in 2017
• 2018 presented with enlarged left inguinal node.
Biopsy squamous cell ca
• Exam exophytic 10 cm cervical mass with
extension to sidewall

When To Perform A Pap Smear

• Screening should start at age 21.

• Screening guidelines are age dependent.

• Annual Pap Smears in women without a history of


premalignant or malignant lower genital disease are no
longer recommended.

• Recommended screening practices should not change


on the basis of HPV vaccination status.

1715
Copyright © Harvard Medical School, 2018. All Rights Reserved.

When to perform a Pap Smear


• Guidelines for management of abnormal
Pap smears are different by the following
age categories:
– Ages 21- 24
– Ages over 30

• Guidelines for management of abnormal


pap smears are different for the pregnant
woman.

Cervical Cancer:
Screening Guidelines

• Ages 21- 29 years: PAP SMEAR screening


every three years:

– No screening HPV testing.

– HPV testing only for evaluation of atypical


squamous cells of uncertain significance (ASCUS).

1716
Copyright © Harvard Medical School, 2018. All Rights Reserved.

Cervical Cancer:
Screening Guidelines

• Ages 30 – 65 years:
– Screening with both PAP SMEAR and HPV
testing every five years (preferred).

– Or PAP SMEAR testing every three years


(accepted).

– Or HPV testing alone with Cobas HPV test


every three years for women over 25.

Cervical Cancer: Screening


Guidelines
• Guidelines for management of abnormal
pap smears are different by the following
age categories:
– Ages 21- 24
– Ages over 30

• Guidelines for management of abnormal


pap smears are different for the pregnant
woman.

1717
Copyright © Harvard Medical School, 2018. All Rights Reserved.

When To Stop?
• Age 65 and not at high risk for cervical cancer (USPSTF,
ACOG, ASCCP).
• Discontinuation of screening assumes adequate prior
negative screening.
• Three consecutive negative cytology results or 2
negative cytology results with (-) co-testing within the
prior 10 yrs with one within 5 yrs.
• No prior hx of cervical cancer, CIN2/3.
• If any present, continue screening for 20 yrs.

When to stop?

• After hysterectomy (with cervix removal) if no


CIN 2/3 or cervical
cancer.
• If CIN 2+, and cervix removed, continue screening
of vaginal cuff for
20 years, even if > 65 years old.
• If hysterectomy and cervix not removed, continue
screening as
recommended for age.

1718
Copyright © Harvard Medical School, 2018. All Rights Reserved.

Essential Changes From Prior Management


Guidelines
• Genotyping triages HR HPV positive women to
colposcopy earlier after negative cytology
– Colposcopy indicated for ASCUS +HPV regardless of
genotyping

• HPV negative ASCUS


– Follow up at 3 years with co-testing
– Not sufficient for exiting women from screening at age
65

Essential Changes From Prior


Management Guidelines
• Cytology reported as negative, but lacking
endocervical cells can be managed without early
repeat.

• Cytology reported as unsatisfactory requires


repeat even if HPV negative.

• Genotyping triages HPV (+) women with 16 or 18


to earlier colposcopy.

1719
Copyright © Harvard Medical School, 2018. All Rights Reserved.

Essential Changes From Prior


Management Guidelines
• HPV negative ASCUS
– Follow up at 3 years with co-testing
– Not sufficient for exiting women from screening at age 65

• Women aged 21 – 24 are managed conservatively

• CIN 2+ follow up is more clearly defined with


incorporation of co-testing.

• Incorporate co-testing post colposcopy.

The Consequences of Over-Screening:


Long Term Changes
• LEEP Procedure and Preterm Birth • 80 percent of low
– one LEEP: 7.2% preterm grade lesions will
deliveries (between 28 and 37
weeks) spontaneously
regress.
– No LEEP: 4.6%

– Two LEEPs: preterm risk • 63 percent of CIN 2


increases 3x lesions regress by
three years.

Obstet Gynecol vol121:1063-1067, 2013

1720
Copyright © Harvard Medical School, 2018. All Rights Reserved.

Case 1

• 58 year old G2P2


• Menopause at age 52
• No history of abnormal Pap testing
• Pap test with physical shows:
– Insufficient cellularity. HPV co-testing is negative.

• Now what?

Unsatisfactory Cytology
• 1% or less across all preparations.

• Decreased with use of liquid based pap.

• Most cases now due to insufficient squamous


cells.

1721
Copyright © Harvard Medical School, 2018. All Rights Reserved.

Case 2

• 58 year old G2P2


• Menopause at age 52
• No history of abnormal Pap testing
• Now pap test shows normal results, but no
EC/TZ
• HPV remains negative

• Now what?

Cytology Normal
EC/TZ Absent/Insufficient
• Suggests squamo-columnar junction may not have
been adequately sampled.

• Reported rates 10-20%.

• More prevalent in older women and adolescents.

• HPV testing is independent of TZ sampling


– Adds margin of safety when co-testing is performed.

1722
Copyright © Harvard Medical School, 2018. All Rights Reserved.

Management

• Age 21-29: routine screening.

• Age 30-64
– HPV negative: Routine screening
– HPV unknown: Test for HPV or repeat cytology in 3
years
– HPV positive:
• Cytology and HPV testing in 1 year or
• HPV genotyping (HPV 16 and 18)

Management
Negative Cytology, HPV positive

• Due to increased risk for CIN 3+ if hrHPV positive


guidelines balance risk of observation vs.
intervention.

• Two options:
1. Cytology and HPV co-testing at 12 months.

2. HPV genotyping
• Positive HPV 16/18: Colposcopy and ECC.
• Negative HPV 16/18: Repeat cytology and HPV co-testing in
one year.

1723
Copyright © Harvard Medical School, 2018. All Rights Reserved.

Risk of HSIL with + HPV HR


• 2% (52 of 2562 over 10 years) Khan 2005

• 3% (88 of 2941 over 10 years) Castle 2002

• 1.2% (30 or 2562 over 10 years) Miller 2002

Atypical Squamous Cells: Pap Smear Diagnosis


• ASC-US
– 50% will be positive for high risk HPV
– 5-7% presence of CIN 2-3

• ASC-H
– 24-94% presence of CIN 2-3

1724
Copyright © Harvard Medical School, 2018. All Rights Reserved.

ASC-US in Pregnancy

• Pregnant women
– Identical to non-pregnant women.

– Acceptable to defer colposcopy until 6 weeks


postpartum.

– ECC is unacceptable.

– If no suspected CIN 2+ at initial colposcopy, follow up


postpartum.

Case 3

• 22 year old G0.


• No previous Pap smear.
• Asymptomatic.
• Seen for routine Gyn exam.
• Pap test shows LSIL.

• What next?

1725
Copyright © Harvard Medical School, 2018. All Rights Reserved.

Women Ages 21-24


• No screening before age 21.

• Routine screening with initial normal pap test is every 3


years:
– Cervical CA risk is low through age 25
– HPV is common
– Most lesions will regress

• Less intensive management.

• Encourage HPV vaccination, smoking cessation.

Women Ages 21-24

• ASCUS/LSIL:
– Cytology every 12 months preferred

– HPV reflex is acceptable


• Follow up is repeat cytology if positive
• Routine screening if negative

• Colposcopy only if ASC-H, AGC, HSIL at follow


up.

1726
Copyright © Harvard Medical School, 2018. All Rights Reserved.

Low-grade Squamous
Intraepithelial Lesions (LSIL)
• ALTS Trial showed natural history to be similar
to ASC-US HPV+.

• Women 21-24 have lower risk CIN 3+.

• Estimated 77% of LSIL are HPV positive.

LSIL Management

• Colposcopy (recommended):
– Manage based on colposcopic findings

• If co-test is negative, repeat co-test in 1 year


– If cytology negative and HPV negative
• Repeat co-testing in 3 years

– If >ASC or HPV positive


• Colposcopy

1727
Copyright © Harvard Medical School, 2018. All Rights Reserved.

ASC-H Management

• Colposcopy for all women.

• High rate of HPV + makes reflex testing


unsuitable.

• 5 year cancer risk among ASC-H, HPV negative


is 2%.

High-Grade Squamous
Intraepithelial Lesion (HSIL)
• CIN 2+ identified in 60% of women at
colposcopy.
• Consider immediate excision of
transformation zone.
• Cervical cancer found in 2% at colposcopy
– Risk rises with age
– Risk modifies with HPV result
• HPV result from co-test may help inform
choice.

1728
Copyright © Harvard Medical School, 2018. All Rights Reserved.

Management HSIL

• Immediate LEEP

• Colposcopy
– Diagnostic excisional procedure recommneded for
inadequate colposcopy
• Except if pregnant

HSIL in Young Women

• Colposcopy

– If no CIN 2+ observe with colposcopy and cytology


at 6 month intervals for 24 months.

– If CIN 2/3 present manage with colposcopy and


biopsy or treat

1729
Copyright © Harvard Medical School, 2018. All Rights Reserved.

Endometrial Cells with Cervical cytology


• No further evaluation in asymptomatic
premenopausal women.

• If postmenopausal, endometrial assessment is


recommended regardless of symptoms.

• No further evaluation with benign glandular cells


following hysterectomy

Ages 21-24: Biopsy CIN I

• Treatment not recommended.

• After ASC-US or LSIL pap: Repeat pap smear.

• After ASC-H or HSIL pap: Repeat pap smear and


colposcopy every six months for one year.
– If colposcopy is inadequate: Excisional
procedure.

1730
Copyright © Harvard Medical School, 2018. All Rights Reserved.

Biopsy: CIN 2-3:


Recommend excisional
procedure

Biopsy: AIS
-Excisional procedure
-Hysterectomy is preferred treatment

1731
Copyright © Harvard Medical School, 2018. All Rights Reserved.

Special Screening Situations:

Pregnancy

• Pap smear is performed at first prenatal visit


and at the six week post partum visit.

• Abnormal Pap smears are evaluated in a


similar manner to non-pregnant women.

CIN in Pregnant Patient


• Colposcopy should have exclusion of invasive cancer as its primary
goal.

• Unless cancer is identified or suspected, treatment of CIN in


pregnancy is contraindicated.

• A diagnostic excisional procedure is recommended only if invasion


is suspected.

• Initial evaluation of AGC is the same except NO ECC or endometrial


biopsy.
• Reassess with cytology and colposcopy no sooner than 6 weeks
postpartum.

1732
Copyright © Harvard Medical School, 2018. All Rights Reserved.

FINAL THOUGHTS
for the over 65 crowd
• 25 % of cervical cancers occur in women over
age 65
• 40 % of cervical cancer deaths occur in
women over age 65
• Cervical cancer statistics artificially lower in
over 65 age because because statistics include
women who have had hysterectomies
• Stay tuned for possible guideline changes
• Maintain high clinical level of suspicion

1733
Copyright © Harvard Medical School, 2018. All Rights Reserved.

A practical approach to the


patient with menopausal
symptoms

Kathryn A Martin, MD
Reproductive Endocrine Unit, Department of
Medicine
Massachusetts General Hospital
Senior Deputy Editor, Endocrinology and Patient
Education, UpToDate

Conflict of Interest Disclosure

Kathryn A Martin, MD
Senior Deputy Editor, UpToDate

1734
Copyright © Harvard Medical School, 2018. All Rights Reserved.

Menopausal hormone therapy (MHT):


Major messages

• Indicated for treatment of moderate to


severe symptoms (hot flashes) in younger
menopausal women (50s or <10 years
postmenopause- if no contraindications)
• Safety in this group is well
established
• Under prescribed
• Difficult for women to get treated
• Untreated hot flashes have important
consequences

Major messages

• Hot flashes last a LONG time


• Duration of therapy should be
individualized

• Vulvovaginal atrophy a common symptom


• Low dose vaginal estrogen
• WHI 2017 safety data

1735
Copyright © Harvard Medical School, 2018. All Rights Reserved.

Trends in oral MHT use

WHI

Sprague et al
Obstet Gynecol 2012

Why are prescription rates so low?

• Lack of awareness of most current MHT


evidence
• Difference in guideline recommendations
• Current generation of new trainees have no
experience (Manson NEJM 2016; Santen et
al 2014)

• 10-12 million women in US are candidates;


not receiving MHT

1736
Copyright © Harvard Medical School, 2018. All Rights Reserved.

Consequences of not treating PMW

Untreated hot flashes:


• Quality of life
• Sleep disturbances
• Economic impact (Sarrel Menopause 2015)
– Lost productivity
– Increased healthcare costs
• Increase in osteoporosis-related fractures

Consequences of not treating PMW


• Increase in use of compounded “bioidentical
hormones” (as many women on BHT as MHT)
–Perceived to be safer
–A high percentage of women believe they are
FDA-approved products (Pinkerton and
Santoro, Menopause 2015)
–Concerns re: standardization and purity;
expensive
–Endocrine Society Scientific Statement

1737
Copyright © Harvard Medical School, 2018. All Rights Reserved.

Women’s Health Initiative: Program Design


Effect of MHT on prevention of chronic disease (CHD, OP)

Hysterectomy
YES NO
N=10,739 N=16,608

CEE CEE 0.625 Placebo


Placebo mg/d + MPA
0.625
mg/d 2.5 mg/d

Trial stopped in 2002: breast


Trial stopped in 2004:
No CHD or breast ca; cancer risk and apparent
stroke concern. *Role of MPA CV risks. (HR vs AR)

Current thinking
• Age is important (WHI mean age 63)
Timing of exposure: Probable underlying
atherosclerosis and vulnerable plaque older
but not younger PMW
Don’t initiate MHT in older women!
• Safety in younger postmenopausal women
well established (ages 50-59, < 10 years
postmenopause)
• WHI follow-up analyses, coronary
calcification, 2 RCTs (KEEPS, ELITE),
Cochrane, WHI mortality data 2017

1738
Copyright © Harvard Medical School, 2018. All Rights Reserved.

Current thinking
Younger PMW – prevention of CHD is NOT
established
• No evidence for increased risk CHD, but
data in the two RCTs are inconsistent
(KEEPS and ELITE using E2 and P)
• Use for symptom management only
• WHI – with less favorable regimen:
• Additional 2.5 cases/1000 women/5
years of use; 5.5 fewer cases for E only

KEEPS: KRONOS Early Estrogen Prevention Study

• 726 women ages 42-58


• Oral CE 0.45 mg, transdermal E2 50 mcg,
placebo; Micronized P 200 mg x 12 (4 yrs)
• Endpoints: carotid IMT, coronary artery calcium
scores
• E improved markers of CVD risk
• Carotid IMT no different oral or transdermal
estrogen, vs placebo Harman Ann Intern Med 2014
• No rebound increase thickness 3 years after
stopping E (Menopause 2018)

1739
Copyright © Harvard Medical School, 2018. All Rights Reserved.

ELITE Trial

• Early Versus Late Intervention Trial With Estradiol


• 643 postmenopausal women
• < 6 yrs or ≥ 10 yrs from menopause (mean age 55
or 65)
• Oral E2 1 mg vs placebo for 6 years (vaginal P 12
days if uterus)
• Carotid IMT: (data support timing hypothesis)
• ≥10 yrs from menopause no diff from placebo
• < 6 yrs slower progression of atherosclerosis
Hodis N Engl J Med 2016; 374:1221

Early vs Late Initiation of Hormone Therapy


and Carotid Intima Media Thickness

Hodis et al,
NEJM, 2016

1740
Copyright © Harvard Medical School, 2018. All Rights Reserved.

Mortality all ages: 18 years of follow-up

Manson
JAMA
2017

Overall risk vs benefit: WHI data (women 50-59)


Number of events per 1000 women per 5 years use

1741
Copyright © Harvard Medical School, 2018. All Rights Reserved.

Endocrine Society Clinical Practice Guideline 2015


• Individualize therapy based upon clinical factors
and patient preference
• Medical history – contraindications: breast ca,
CHD, stroke, VTE, active liver disease, high risk
endometrial ca, TIA, undiagnosed vaginal
bleeding
• Before initiating MHT, estimate patient’s:
• 10-year cardiovascular risk
• 5-year breast cancer risk

Stuenkel et al JCEM 2015

Choosing Candidates: Evaluate CVD risk

10-year CVD risk MHT recommendation if <10


years since menopause
Low (<5%) MHT ok

Moderate (5-10%) MHT ok, but use transdermal

High (>10%) Avoid MHT

Calculated using ACC/AHA risk calculator Stuenkel et al, JCEM 2015;


Adapted from Manson, Fertil Steril 2014

1742
Copyright © Harvard Medical School, 2018. All Rights Reserved.

Breast Cancer Risk Cutoffs for MHT


Risk category 5-y NCI or IBIS Suggested
Breast cancer risk approach
assessment, %
Low <1.67 MHT ok

Intermediate 1.67-3 Caution

High >3 Avoid

Stuenkel et al, JCEM 2015

Menopausal symptoms
Hot flashes
• Peak in late menopausal transition/ early
postmenopause (longitudinal cohort studies about
85%)
• Varies by ethnicity
• Risk factors: obesity, PMS, genetic variants tachykinin
receptor 3 (TACR3)
Duration
• For many/most women: 7 to 10 years
• Long duration of symptoms has important implications
for duration of therapy
• Patterns/trajectories

1743
Copyright © Harvard Medical School, 2018. All Rights Reserved.

Trajectories – hot flashes

26%

18% 29%

27%

Case 1
• A 51 year old woman with 4 months of amenorrhea

• She is awakened at least six to seven times a night


and has frequent episodes during the day that are
interfering with her ability to function at work

• No history VTE, stroke. Calculated 10-year risk CVD


5%, breast cancer risk 1.2% in 5 years

• What would you recommend?

1744
Copyright © Harvard Medical School, 2018. All Rights Reserved.

Case 1
• A. Black cohosh

• B. Conjugated estrogen with continuous


medroxyprogesterone

• C. 17-B estradiol (oral or transdermal) with cyclic


micronized progesterone

• D. Bazedoxifene/Conjugated estrogen

Case 1
• A. Black cohosh – is ineffective in trials and meta-analyses,
and this patient has significant symptoms.

• B. Conjugated estrogen with continuous


medroxyprogesterone – this could be used, but a continuous
progestin will result in breakthrough bleeding (patient still has
ovarian function. A cyclic progestin regimen is preferred
initially

• C. 17-B estradiol (oral or transdermal) with cyclic micronized


progesterone- this would be my first choice: a cyclic progestin
to minimize BTB and 17-B estradiol, the same estrogen made
by the ovary

• D. Bazedoxifene/Conjugated estrogen – this is not a first line


therapy for MHT, but it can be helpful for women who are
unable to tolerate progestins

1745
Copyright © Harvard Medical School, 2018. All Rights Reserved.

Other menopausal symptoms


• Mood disorders (perimenopausal)
• Sleep disturbances
– 40% even in the absence of HF: contributors are
primary sleep disorders, depression and
anxiety.
• Joint pain – similar to the MS side effects of
aromatase inhibitors (AIs), improves with estrogen
• Vulvovaginal atrophy (VVA) - Genitourinary
syndrome of menopause (GSM)--tends to develop
in the years or decades after menopause

• Long-term consequences: bone loss, CVD

Perimenopausal mood disorders


• Depression/mood disorders (35-40% in menopausal
transition)
• More common in women with hot flashes
• Particularly nighttime HF associated with mood
symptoms (not just sleep deprivation)
• Other risk factors: personal or family history of
mood disorders, PMS Freeman EW Arch Gen Psych 2006

• Choose initial therapy based upon predominant


symptom (depression vs VMS) – often need both E and
SSRI (citalopram, escitalopram, duloxetine)
• Usually progestin intolerant

1746
Copyright © Harvard Medical School, 2018. All Rights Reserved.

Estrogen for vasomotor symptoms

• 80-95% decrease in VMS (dose)


• Which estrogen? - 17-B estradiol
• Dose – start low unless sx are severe.
• Lower doses effective in many women
• Lower risk of VTE and stroke

Route: transdermal vs oral


Transdermal estrogens have less effect on:
•Clotting factors, triglycerides, C-reactive protein
•SHBG
Lower risk of:
• VTE (Canonico Circ 07, BMJ 08)
• Stroke (if dose < 50 mcg) Renoux BMJ 2010)
• Increase in mammographic density
Most important for women with high TG,
gallbladder disease, and risks for CVD or
VTE

1747
Copyright © Harvard Medical School, 2018. All Rights Reserved.

Which progestin?
Advantages of micronized progesterone
• Bioidentical, physiologic
• Neutral metabolic effect. (eg does not negate
benefits of oral E)
• Vascular –Unlike MPA, P does not negate
vasodilatory effect of E (primate data). Differential
effects on endothelial function
• Ischemic stroke risk (Canonico Stroke 2016) MP
better than MPA
• Breast cancer - ? lowest risk with micronized P ;
(European observational studies; Endo Society metaanalysis JCEM 2015)

My approach
Late transition/early postmenopause
• Transdermal E2 0.025 mg (25 mcg) + cyclic MP 200 mg
days 1-12 (**higher dose 50 mcg for severe symptoms)

Postmenopausal (≥ 2 years)
• Transdermal E2 0.025 mg (25 mcg) + continuous MP
100 mg (higher dose E2 for severe symptoms)

• Oral E2 ok for healthy women without DVT risks who


prefer oral preps (0.5-1 mg)
• Consider low dose OC in some women in 40s with
heavy bleeding who desire contraception
• POI/POF

1748
Copyright © Harvard Medical School, 2018. All Rights Reserved.

Women who cannot tolerate oral progestins

• 20% side effects: BTB and mood issues


• Vaginal progesterone (45 mg daily x 10 days/month)
• Progestin IUD (levonorgestrel) – off-label use in US.
Breast cancer concerns
• Quarterly progestin regimens: safety not established
• Tissue selective estrogen complex (TSEC)
–Oral conjugated estrogen 0.45 mg with bazedoxifene
20 mg
–SERM antagonizes E effect on endometrium, so no P
needed

Duration of therapy and stopping


• Extended use: ACOG, NAMS statements
–Extended use (beyond age 60 [or 65]) may be
appropriate for women with severe vasomotor
symptoms and low risk for cardiovascular
disease/breast cancer

–Periodic weaning to assess need (anticipate recurrent


VMS, use nonhormonal agents while weaning)
–Tapering vs not tapering

Obstet Gynecol 2014 123:202; Menopause 2015; 22:693)

1749
Copyright © Harvard Medical School, 2018. All Rights Reserved.

Case 2
• A 53 year old woman with breast cancer. She
has undergone surgery, chemotherapy, RT,
and is now on tamoxifen

• Since starting tamoxifen she has had severe


hot flashes, They are most bothersome at
night

• She is having trouble functioning at work

• What would you suggest?

Case 2

A. Black cohosh

B. Gabapentin

C. Acupuncture

D. Paroxetine

1750
Copyright © Harvard Medical School, 2018. All Rights Reserved.

Case 2
A. Black cohosh – ineffective for hot flashes; binds to
estrogen receptors (although there is no evidence
that it increases recurrence risk)

B. Gabapentin – this would be my first choice.


Gabapentin is better than placebo in trials, although
not as effective as estrogen. A single bedtime dose is
often helpful and avoids daytime sleepiness

C. Acupuncture – the best clinical trial evidence


suggests that acupuncture is ineffective for HF

D. Paroxetine – avoid in women on tamoxifen; it inhibits


CYP2D6, the enzyme the converts TAM to endoxifen

Nonhormonal alternatives
Phytoestrogens, soy, red clover
• Inconsistent data
• Large effects unlikely
Black cohosh (Cimicifuga): widely used
• HALT trial - Black cohosh alone no more effective than
placebo. CEE effective as expected. (Newton, Ann Intern
Med 2006)
• Cochrane 2012 – no evidence that better than placebo but
heterogeneity
Concerns about estrogen agonist effects
Acupuncture – both acupuncture and sham acupuncture
work

1751
Copyright © Harvard Medical School, 2018. All Rights Reserved.

Nonhormonal therapies: Pharmacologic


SSRIs
• Citalopram 20 mg
• Escitalopram 10 to 20 mg
• Paroxetine 7.5 mg (Brisdelle- low dose mesylate salt; FDA
approved) or paroxetine HCl 20 mg, CR 12.5-25 mg
• Sertraline and fluoxetine are less effective
• Women on tamoxifen: Avoid paroxetine! (CYP2D6
inhibitor); escitalopram weakest inhibitor
SNRIs
• Venlafaxine 75 mg; Desvenlafaxine 50 mg

Nonhormonal therapies: Pharmacologic


• Gabapentin 100 to 1200 mg one hour before bedtime.
Titrate dose up. (good choice for night time symptoms)
• Clonidine – use limited by side effects
• Range of effect all drugs: only a 40 to 50% reduction (e.g.
only 20% better than placebo (25-30%) Loprinzi et al J Clin Oncol
2009

• Response not explained by mood effect

• Promising new approach: NK3R antagonist


administration- 80% decrease frequency vs 30 % for
placebo Prague et al Lancet 2017; 389 epub

1752
Copyright © Harvard Medical School, 2018. All Rights Reserved.

Vulvovaginal atrophy:Low-dose vaginal estrogen


Do not need systemic dosing for vulvovaginal sx
• E2 or CEE creams: difficult to administer low
doses
• E2 tablets: 10 µg twice weekly (serum
E2<10pg/ml; 37 pmol/L)
• Vaginal ring 8 ug/day (serum E2 = 7 pg/mL; 26
pmol/L)
• Improved sexual QOL (Setty, Menopause August
2015), no endometrial thickening

Vulvovaginal atrophy:Low-dose vaginal estrogen

• *Breast cancer patients (discuss with


oncologist, no vaginal E if taking an
aromatase inhibitor)
• **WHI 2017: no increase endometrial or
breast cancer, stroke or VTE
• Ospemifene, DHEA

1753
Copyright © Harvard Medical School, 2018. All Rights Reserved.

Key points
• For many/most symptomatic women <age 60 or <10
years postmenopause, the benefits of MHT outweigh the
risks
• Women with moderate to severe menopausal symptoms
are currently undertreated
• Reluctance to prescribe coincides with an increase in
use of bioidentical hormone therapy
• Individualized approach: assess baseline CVD and
breast cancer risks
• Start with low dose E and titrate up, unless severe
symptoms
• Preferred regimen: E2 and MP

Key points
• Hot flashes last a long time – has implications for
duration of therapy
• Extended use (beyond age 60) may be
appropriate for women with severe vasomotor
symptoms and low risk cardiovascular
complications
• After stopping, if recurrent VMS, try SSRIs/SNRIs
or gabapentin
• Vaginal estrogen should be discussed with ALL
women, particularly when systemic estrogen is
stopped

1754
Copyright © Harvard Medical School, 2018. All Rights Reserved.

References
• Stuenkel et al. Treatment of symptoms of the Menopause: An
Endocrine Society Clinical Practice Guideline. J Clin Endocrinol
Metab 2015l 100:3975
• Moyer VA U.S. Preventive Services Task Force. Menopausal
hormone therapy for the primary prevention of chronic conditions:
U.S. Preventive Services Task Force recommendation statement.
Ann Intern Med. 2013;158(1):47.
• Manson JE, Chlebowski RT. Menopausal hormone therapy and
health outcomes during the intervention and extended
poststopping phases of the Women's Health Initiative randomized
trials. JAMA. 2013;310(13):1353.
• Hodis HN, Mack WJ. Vascular Effects of Early versus Late
Postmenopausal Treatment with Estradiol. N Engl J Med.
2016;374(13):1221
• Boardman HM, Hartley L et al. Hormone therapy for preventing
cardiovascular disease in post-menopausal women. Cochrane
Database Syst Rev. 2015 CD002229

1755
Copyright © Harvard Medical School, 2018. All Rights Reserved.

Evaluation of the Patient


with Menstrual Irregularities

Maria A. Yialamas, MD

Associate Program Director, Internal Medicine Residency


Department of Medicine, Brigham and Women’s Hospital
Assistant Professor of Medicine, Harvard Medical School

I have no conflicts of interest to report.

1756
Copyright © Harvard Medical School, 2018. All Rights Reserved.

Female Hypothalamic-Pituitary-Gonadal Axis

Hypothalamus

GnRH
Pituitary

LH E2,
FSH Progesterone,
Inhibin A & B

Ovary

150
Hormone Secretion in the
40
Normal Menstrual Cycle
LH (IU/L)

FSH (IU/L)

100

20
50
Length 25-35 days
Luteal phase 12-14 days
0 0 Follicular phase variable
300 20
P4 (ng/mL)
E2 (pg/mL)

200

10

100

0 0
-20 Menses -10 -5 0 5 10

Days Centered to Ovulation Welt et al, JCEM 1999

1757
Copyright © Harvard Medical School, 2018. All Rights Reserved.

Case #1: History


• 19 year old woman presents to your clinic for
evaluation because she has had no menses
for 8 months.

• Her menarche was at age 13, and she had


regular menses until 8 months ago.

• About one year ago, she increased her


aerobic exercise with a 20 lb weight loss.

Case #1: History


• She has not had any headaches, vision
changes, hot flushes, or night sweats.

• She has no history of an eating


disorder.

• Her thyroid review of systems are


negative.

1758
Copyright © Harvard Medical School, 2018. All Rights Reserved.

Case #1: Physical Exam


• BMI 18 BP 100/60 P60
• Skin with no hirsutism, acne, or alopecia.
• Visual fields full.
• Thyroid size is normal. No nodules.
• No galactorrhea.
• Pelvic exam normal.

What is her diagnosis and


what labs would you order?

Amenorrhea
• Primary Amenorrhea
– Absence of menses by age 16

• Secondary Amenorrhea
– Absence of menses for 3 months

Pathophysiologic considerations are the


same for both.

Incidence of genetic & anatomic


abnormalities higher with primary
amenorrhea.

1759
Copyright © Harvard Medical School, 2018. All Rights Reserved.

Etiologies

• Pregnancy

• Uterine or Outflow Tract Disorders

• Ovulatory Disorders

Causes of Primary Amenorrhea

Hypothalamus – Hypothalamus 27%

– Pituitary 2%

– PCOS 7%
Pituitary
LH – Ovary 43%
E2
FSH Inhibin A &
B – Uterus/Outflow Tract 19%
Ovary

1760
Copyright © Harvard Medical School, 2018. All Rights Reserved.

Causes of Secondary Amenorrhea

Hypothalamus – Hypothalamus 36%

– Pituitary 15%

Pituitary – PCOS 30%


LH E2
FSH Inhibin A & B – Ovary 12%

Ovary – Uterus/Outflow Tract 7%

Diagnostic Lab Evaluation

• βhCG (rule out pregnancy!)


• FSH (test for ovarian insufficiency)
• Prolactin
• TSH

1761
Copyright © Harvard Medical School, 2018. All Rights Reserved.

Case #1: Labs


• FSH 3.2 IU/L
• Prolactin 6.3 ng/mL
• TSH 2.6 µU/mL
• βhCG neg

• Provera challenge was negative for a


withdrawal bleed.

What is her diagnosis?

Hypothalamus

Pituitary

LH E2,
FSH Progesterone,
Inhibin A & B

Ovary

1762
Copyright © Harvard Medical School, 2018. All Rights Reserved.

Hypothalamic & Pituitary Etiologies


• Hypothalamic Amenorrhea (HA)
• Hyperprolactinemia
• Tumors and Destructive/Infiltrative Lesions
• Genetic: IHH, Kallmann syndrome
• Cranial Irradiation
• Others: hypo- or hyperthyroidism, excess
cortisol
• Pituitary tumor (mass effect)
• Lymphocytic hypophysitis
• Pituitary infarction
• Empty-sella syndrome

Hypothalamic Amenorrhea

• Etiology
–Energy Output > Energy Input
• Weight loss
• Eating Disorders
• Excessive exercise
–Stress
• Psychological
• Physical

1763
Copyright © Harvard Medical School, 2018. All Rights Reserved.

Hypothalamic Amenorrhea
• Leptin
Peripheral signal indicating sufficient
energy stores for reproduction.
Baseline After 2 Weeks r-metHuLeptin

8
* *
* 30

Leptin (ng/mL)
LH (mIU/mL)

6
* *
* 20
4 *

2 10

0 0
0 1 2 3 4 5 6 7 8 9 101112 0 1 2 3 4 5 6 7 8 9 101112
7PM Time (hr) 7AM 7PM Time (hr) 7AM

Welt et al., 2004

Management Questions
• Does this patient need a MRI?
– Headaches or neurological symptoms
– Elevated prolactin
– History unclear
– Primary amenorrhea

• How about a Bone Mineral Density Scan?


– amenorrhea >6 months

1764
Copyright © Harvard Medical School, 2018. All Rights Reserved.

Hypothalamic Amenorrhea: Recovery

Percentage Recovered
100

80

60

40

20

0
Eating Idiopathic Stress/
Disorder n=9 Weight Loss
n=15 n=6 (Perkins et al., 2001)

Hypothalamic Amenorrhea:
Treatment

• Weight gain, decrease exercise

• Oral contraceptives or
hormone replacement therapy

• Calcium and Vitamin D

1765
Copyright © Harvard Medical School, 2018. All Rights Reserved.

Case #2

• 27 yo with 3 months amenorrhea


• Moderate exercise, no eating disorders
• Insomnia
• No hirsutism, mild acne
• History of hypothyroidism on thyroid
hormone replacement

Case #2

• Family history of early menopause


• Physical exam unremarkable
• Labs:
–FSH 56 IU/L, TSH 3.7 µU/mL, neg
βhCG, prl 10.8 ng/mL
• Diagnosis?
• Further workup?

1766
Copyright © Harvard Medical School, 2018. All Rights Reserved.

Hypothalamus

Pituitary

LH E2,
FSH Progesterone,
Inhibin A & B

Ovary

Primary Ovarian Insufficiency


• Elevated FSH, age < 40 yrs
• Repeat testing important: follicular phase
• Causes:
– Turner’s syndrome
– X chromosome deletions, translocations
– Fragile X premutations
– Autoimmune
– Chemotherapy or radiation therapy
– Galactosemia, FSH/LH receptor mutations,
blepharophimosis, BMP 15 mutations

1767
Copyright © Harvard Medical School, 2018. All Rights Reserved.

Primary Ovarian Insufficiency:


Diagnostic Tests

• Karyotype
– Age <30 yrs, familial cases, ?everyone
– Turner’s features
• Fragile X Premutation screen
• Anti-thyroid and anti-adrenal antibodies

• Anti-ovarian antibodies poor test


• Ovarian biopsy is not helpful

Primary Ovarian Insufficiency:


Treatment
• Oral contraceptive pills or hormone
replacement therapy

• Calcium and vitamin D

1768
Copyright © Harvard Medical School, 2018. All Rights Reserved.

Case #3: History


• 28 year old woman presents to your clinic for
evaluation of irregular menses

• She describes irregular menses since age of


menarche which was at 14 years of age

• She has also had problems with increased


hair growth on her upper lip and chin

Case #3: History


• She does not have hot flashes, night sweats,
galactorrhea, or positive thyroid review of
systems

• She is on no meds currently and has a family


history of type 2 diabetes.

1769
Copyright © Harvard Medical School, 2018. All Rights Reserved.

Case #3: Physical Exam


• BMI of 29
• Skin with hirsutism of the upper lip, chin, and
sides of her face. No acanthosis.
• No clitoromegaly.

What labs/studies would you order?

Case #3: Labs


• Laboratory testing reveals normal
– TSH
– Prolactin
– FSH
– Androgen levels (total testosterone)

What is the diagnosis?

1770
Copyright © Harvard Medical School, 2018. All Rights Reserved.

Polycystic Ovarian Syndrome

Hypothalamus

Adipose
Tissue

Pituitary

Muscle

Ovary

PCOS: Epidemiology
• 4.7-6.8% of women have PCOS as defined
by the NIH criteria
(Knockenhauer et al., JCEM 1998; Diamanti-Kandarakis,
JCEM, 1999; Asuncion, JCEM 2001)

• Most common cause of female infertility (50-


60%)

• May be the most endocrinopathy in young


women

1771
Copyright © Harvard Medical School, 2018. All Rights Reserved.

PCOS: NIH Definition


• Menstrual dysfunction
– < 9 menses per year

• Hyperandrogenism
– Hirsutism, acne, alopecia and/or
– Elevated serum androgens

• Exclusion of other diseases (e.g.


hyperprolactinemia, androgen secreting
tumors, CAH)
(NIH Conference, 1990)

PCOS: Rotterdam Definition


• 2 out of 3 of the following must be true:
– Oligo- or anovulation
– Clinical and/or biodchemical signs of
hyperandrogenism
– Polycystic Ovaries

Exclusion of other etiologies (e.g.


hyperprolactinemia, CAH, androgen
secreting tumors)

(ESHRE/ASRM sponsored PCOS Consensus Workshop Group,Fert and Ster, 2004)

1772
Copyright © Harvard Medical School, 2018. All Rights Reserved.

Polycystic ovary (PCO):


multiple small (2 – 9
mm) follicles
peripheral distribution
increased stromal
volume
ovaries usually
enlarged
(Adams et al, 1985,
Fert and Ster, 2004)

Normal ovary:
fewer follicles
random distribution
no increased stroma

( images courtesy of Judy Adams)

Polycystic Ovarian
Morphology (PCOM)
• 100% of women with PCOS have PCOM
(Taylor et al JCEM 1997)

• 23% of normally cycling women will have PCOM


(Polson et al, Lancet, 1988)

• Present in other ovulatory disorders


(hyperprolactinemia, late onset CAH, adolescence
(Azziz R. JCEM 2006)

1773
Copyright © Harvard Medical School, 2018. All Rights Reserved.

Clinical Manifestations
• Oligo- or anovulation
• Hyperandrogenism
• Infertility
• Insulin Resistance

Treatment of PCOS
Hyperandrogenism
• Weight loss
• Cosmetic measures
• Hormonal therapy

1774
Copyright © Harvard Medical School, 2018. All Rights Reserved.

Treatment of PCOS
Hyperandrogenism Oligo-amenorhea

• Weight loss • Weight Loss

• Cosmetic measures • Hormonal therapy

• Hormonal therapy • Metformin

Treatment of PCOS
Hyperandrogenism Oligo-amenorhea
• Weight loss • Weight Loss

• Cosmetic measures • Hormonal therapy

• Hormonal therapy • Metformin

Infertility
• Weight Loss
• Metformin
• Ovulation Induction/IVF

1775
Copyright © Harvard Medical School, 2018. All Rights Reserved.

Treatment of PCOS
Hyperandrogenism Oligo-amenorhea
• Weight loss • Weight Loss

• Cosmetic measures • Hormonal therapy

• Hormonal therapy • Metformin

Infertility Insulin Resistance


• Weight Loss • Weight Loss/ Exercise
• Metformin • Metformin
• Ovulation Induction/IVF • Monitor BPs, lipids

Treatment of PCOS
Hyperandrogenism Oligo-amenorhea
• Weight loss • Weight Loss
• Cosmetic measures • Hormonal therapy
• Hormonal therapy • Metformin

Infertility Insulin Resistance


• Weight Loss • Weight Loss/ Exercise
• Metformin • Metformin
•Ovulation Induction/IVF • Monitor BPs, lipids

1776
Copyright © Harvard Medical School, 2018. All Rights Reserved.

Summary
• Hypothalamic Amenorrhea
– Diagnosis
– Management (MRI, BMD)
– Treatment (weight gain, OCPs, HRT)

• Primary Ovarian Insufficiency


– Definition
– Diagnostic evaluation (karyotype, Fragile X)
– Treatment options (OCPs, HRT)

Summary
• PCOS
– Epidemiology
– Diagnosis (NIH versus Rotterdam criteria)
– Clinical Manifestations (anovulation,
hyperandrogenism, fertility, insulin
resistance)
– Treatment options (weight loss, OCPs,
spironolactone, metformin)

1777
Copyright © Harvard Medical School, 2018. All Rights Reserved.

Question #1
24 year old woman with a 6 month history of
amenorrhea comes in for evaluation. Her
thyroid review of systems are negative. She
does not have hot flushes, night sweats, or
galactorrhea. She is on no medications.
Physical exam is unremarkable. hCG
negative. FSH and TSH normal. Prolactin is
slightly elevated at 30 ng/mL (<18 ng/mL) and
confirmed on repeat evaluation.

Question #1
What is the next best step?

A) Treat with bromocriptine/cabergoline

B) Treat with an oral contraceptive pill

C) Give a progesterone challenge

D) Obtain a pituitary MRI

E) Repeat the prolactin in 3 months. No treatment for


now.

1778
Copyright © Harvard Medical School, 2018. All Rights Reserved.

Question #1
What is the next best step?

A) Treat with bromocriptine/cabergoline

B) Treat with an oral contraceptive pill

C) Give a progesterone challenge

D) Obtain a pituitary MRI

E) Repeat the prolactin in 3 months. No treatment for


now.

Question #2
34 year old woman with a 4 month history of
amenorrhea comes to see you for evaluation.
Her menses had occurred every 2 months
before they stopped. Her exercise routine is
unchanged; she runs about 25 miles per
week. She has had no hot flushes or night
sweats. Her thyroid review of systems are
negative. Physical exam reveals some
terminal hair growth of her face. TSH, FSH,
and prolactin are normal. hCG negative.

1779
Copyright © Harvard Medical School, 2018. All Rights Reserved.

Question #2
What would you do next?

A) Medroxyprogesterone challenge
B) Treat with OCPs
C) Treat with metformin
D) MRI of the pituitary gland
E) Pelvic ultrasound

Question #2
What would you do next?

A) Medroxyprogesterone challenge
B) Treat with OCPs
C) Treat with metformin
D) MRI of the pituitary gland
E) Pelvic ultrasound

1780
Copyright © Harvard Medical School, 2018. All Rights Reserved.

References

• Gordon CM, et al. 2017 Functional hypothalamic amenorrhea:


An Endocrine Society Clinical Practice Guideline. JCEM
102:1413.

• Legro RS, et al. 2013 Diagnosis and Treatement of Polycystic


Ovarian Syndrome: An Endocrine Society Clinical Practice
Guideline. JCEM 98:4565.

• Nelson LM. 2009 Clinical practice. Primary ovarian insufficiency.


NEJM 360:606.

I have no conflicts of interest to report.

1781
Copyright © Harvard Medical School, 2018. All Rights Reserved.

Osteoporosis and Metabolic


Bone Diseases
Carolyn Becker MD
Master Clinician Educator
Division of Endocrinology, Diabetes and Hypertension
Brigham and Women’s Hospital
Associate Professor of Medicine
Harvard Medical School

Disclosures
• Royalties from UpToDate for 2 chapters on
Premenopausal Osteoporosis ($1800/yr)

1782
Copyright © Harvard Medical School, 2018. All Rights Reserved.

What Does the Primary Care Clinician


Need to Know About Bone in 2018?
• Calcium and vitamin D: anything new?
• Fracture risk factors to watch out for: falls,
meds, mental health, hyponatremia
• 2017 ACP Guidelines: the good and the bad
• Update on denosumab: the good, the bad,
and the ugly
• Managing osteoporosis over the long-term:
have we learned anything?

What Does the Primary Care Clinician


Need to Know About Bone in 2018?
• Calcium and vitamin D: anything new?
• Fracture risk factors to watch out for: falls,
meds, mental health, hyponatremia
• 2017 ACP Guidelines: the good and the bad
• Update on denosumab: the good, the bad,
and the ugly
• Managing osteoporosis over the long-term:
have we learned anything?

1783
Copyright © Harvard Medical School, 2018. All Rights Reserved.

Calcium and Vitamin D: A Summary


• Ca + D supplementation leads to a modest
reduction in fracture risk; Ca alone is not effective
• Vitamin D supplementation may reduce falls
• Population-level intervention with Ca + D has not
proven to be an effective public health strategy
• Side effects of Ca supplementation include kidney
stones and GI upset
• Increased CVD risk from Ca supplementation is not
convincingly supported by current evidence

Harvey NC et al. 2017 Osteoporos Int 28(2):447

Who Should Receive Calcium and


Vitamin D Supplementation?
• Those at high risk for calcium and/or vitamin
D insufficiency (frail elderly, institutionalized,
lack of sunlight exposure, malabsorption,
malnutrition, obese, teenage girls, etc)
• Those receiving treatment for osteoporosis
CHECK 25OHD LEVELS IN ALL OF THESE
GROUPS AND REPLETE TO ~30 ng/mL

Harvey NC et al. 2017 Osteoporos Int 28(2):447

1784
Copyright © Harvard Medical School, 2018. All Rights Reserved.

Checking Vitamin D Levels…


• For elderly without osteoporosis or high
fracture risk, can simply follow Institute of
Medicine (IOM) recommendations and give
800 IU cholecalciferol (vitamin D3) daily

Repleting Vitamin D
• Vitamin D3 (cholecalciferol) is preferred for
chronic therapy; can use D2 (ergocalciferol)
for “loading dose” in severe deficiency
• 800 - 1000 IU D3 daily will increase 25OHD by
~ 10 - 16 ng/ml
• Higher doses needed for obese, those with
malabsorption, or those with increased
metabolism (eg. anticonvulsants)

1785
Copyright © Harvard Medical School, 2018. All Rights Reserved.

What Does the Primary Care Clinician


Need to Know About Bone in 2018?
• Calcium and vitamin D: anything new?
• Fracture risk factors to watch out for: falls,
meds, mental health, hyponatremia
• 2017 ACP Guidelines: the good and the bad
• Update on denosumab: the good, the bad,
and the ugly
• Managing osteoporosis over the long-term:
have we learned anything?

Current Guidelines for Ordering


Screening DXA
• Women age 65 yrs
and older
• Men age 70 and older
• Postmenopausal
women and men age
50-69 with risk
factors
• Post-fragility fracture

National Osteoporosis Foundation 2013 Clinicians Guide to Prevention and Treatment


of Osteoporosis

1786
Copyright © Harvard Medical School, 2018. All Rights Reserved.

Standard Clinical Risk Factors for


Osteoporosis Screening
• Caucasian/Asian • Rheumatoid arthritis
• Older age • Hypogonadism (no rx)
• Previous adult fracture • Inflammatory bowel
• Female gender disease; malabsorption
• Low body weight • Prolonged immobility
• Parental hip fracture • Organ transplantation
• Smoking • Hyperthyroidism
• Alcohol (3 or more/d) • COPD
• Glucocorticoids (5 mg pred • Diabetes (Type 1)
or > x 3 mos or more)

Kanis JA et al 2013 Osteoporos Int 24:23

Standard Clinical Risk Factors for


Osteoporosis Screening
• Caucasian/Asian • Rheumatoid arthritis
• Older age • Hypogonadism (no rx)
• Previous adult fracture • Inflammatory bowel
• Female gender disease; malabsorption

ARE WE
Low body weight
MISSING SOME IMPORTANT
• Prolonged immobility
• Parental CLINICAL
hip fracture RISK FACTORS?
• Organ transplantation
• Smoking • Hyperthyroidism
• Alcohol (3 or more/d) • COPD
• Glucocorticoids (5 mg pred • Diabetes (Type 1)
or > x 3 mos or more)

Kanis JA et al 2013 Osteoporos Int 24:23

1787
Copyright © Harvard Medical School, 2018. All Rights Reserved.

Osteoporotic Fractures in the US

>50% OF OSTEOPOROTIC
FRACTURES
OCCUR DUE TO A FALL!

Shuler FD et al. 2012 Orthopedics 35(9):798

To Reduce Fractures, We Need to Reduce


Falls, Increase Bone Strength, or Both

1788
Copyright © Harvard Medical School, 2018. All Rights Reserved.

We Have Under-Emphasized
Identifying Risk Factors for Falls
• Dementia
• Frailty
• Poor visual acuity
• Gait instability
• Postural hypotension
• Peripheral neuropathy
• Neurodegenerative disorders
• Osteoarthritis of hips, knees
• Diabetes
Strongest Predictor = Previous Fall!
• Vitamin D deficiency
• Medications

Screening for Falls During the Annual


Physical Exam
• Any falls in the past year?
• If yes, how many, where did they occur, and
any injuries?
• Any unsteadiness while standing or walking?
• Any fears or worries about falling?

1789
Copyright © Harvard Medical School, 2018. All Rights Reserved.

Screening for Falls


• Medication review
• Cognitive screen
• Feet and footwear
• Use of assistive device
• Visual acuity assessment

Moncada LVV, Mire LG. 2017 Am Fam Physician 96(4):240

Fall-Focused Physical Exam


• Vital signs: including orthostatic BP
• Gait, strength, and balance testing (go to
“YouTube” for demonstrations).
– Timed Up and Go test
– 4-Stage Balance Test
– 30-Second Chair Stand

1790
Copyright © Harvard Medical School, 2018. All Rights Reserved.

• > 68,000 patients with mental disorders or on


psych meds underwent baseline DXA and
FRAX® calculations and were followed x 7
years
• Observed hip fracture rates were much
higher than FRAX-predicted hip fracture rates
for this population

Bolton JM et al. 2017 JAMA Psychiatry 74(6):641

1791
Copyright © Harvard Medical School, 2018. All Rights Reserved.

FRAX MAY UNDERESTIMATE ACTUAL


HIP FRACTURE RATE BY 50% FOR THOSE
WITH DEPRESSION OR SSRI USE

Bolton JM et al. 2017 JAMA Psychiatry 74(6):641

N = 447,169

• SSRIs
• Benzodiazepines THESE MEDICATIONS
WERE
• Anticonvulsants
HIGHLY ASSOCIATED
• Hypnotics WITH FALLS
• Antipsychotics

1792
Copyright © Harvard Medical School, 2018. All Rights Reserved.

Limitations of DXA and FRAX®


DXA FRAX®
• Does not predict • Does not include falls
falls • Underestimates
fracture risk (eg. in DM
• Does not measure
and mental illness)
bone quality
• Assumes glucocorticoid
• Underestimates dose 2.5 – 7.5 mg/d
fracture risk (eg. in • Does not account for
those with diabetes) other high-risk meds
• Low sensitivity (AIs, SSRIs, PPIs, etc)

Can We Tweak FRAX® to Make it A


Better Predictor?
• For fallers, multiply FRAX® score by 1.3
• For mental disorder or use of psychotropic
drugs, multiply FRAX® score by 1.5
• For glucocorticoid dose > 7.5 mg/d, multiply
FRAX® by 1.15 at spine and 1.2 at hip
• For those with diabetes add 10 years to age
then recalculate FRAX® or, check off box for
“Rheumatoid Arthritis” and recalculate
Bolton JM et al. 2017 JAMA Psychiatry 74(6):641
Kanis JA et al. 2011 Osteoporos Int 22:2395
McCloskey EV et al. 2016 Curr Opin Rheumatol 28:433

1793
Copyright © Harvard Medical School, 2018. All Rights Reserved.

• Matched case control study from a large


US healthcare system with over 2.9
million lives
• Multiple variables were compared
including presence of hyponatremia
(Na < 135 mEq/L)

Slide from Verbalis JG, Clinical Endocrine Update 2017, Chicago

1794
Copyright © Harvard Medical School, 2018. All Rights Reserved.

Slide from Verbalis JG, Clinical Endocrine Update 2017, Chicago

Loss of osmolytes, Hyponatremia stimulates


eg. glutamate, a osteoclasts to mobilize
neurotransmitter sodium from bone;
associated with gait AVP may also play a role

Negri AL, Ayus JC. 2017 Rev Endocr Metab Disord 18:67

1795
Copyright © Harvard Medical School, 2018. All Rights Reserved.

Implications for the Clinician


• All patients with osteoporosis, fractures, or
unsteady gait should be screened for
hyponatremia
• If Na < 135 is found, an effort should be
made to stop the offending agent (eg.
thiazide or SSRI) or find another underlying
cause
• Look for osteoporosis in outpatients with
hyponatremia
Negri AL, Ayus JC. 2017 Rev Endocr Metab Disord 18:67

What Does the Primary Care Clinician


Need to Know About Bone in 2018?
• Calcium and vitamin D: anything new?
• Fracture risk factors to watch out for: falls,
meds, mental health, hyponatremia
• 2017 ACP Guidelines: the good and the bad
• Update on denosumab: the good, the bad,
and the ugly
• Managing osteoporosis over the long-term:
have we learned anything?

1796
Copyright © Harvard Medical School, 2018. All Rights Reserved.

• In postmenopausal women with osteoporosis,


– Use alendronate, risedronate, zoledronate, or
denosumab to reduce risk of hip or vertebral
fractures
– Menopausal estrogen, estrogen + progestin, or
raloxifene are not recommended
• Treat osteoporotic women with pharmacologic
therapy for 5 years

• BMD monitoring during the 5-year


pharmacologic treatment period is not
recommended
• For men with osteoporosis, offer
bisphosphonates to reduce risk of vertebral
fractures

1797
Copyright © Harvard Medical School, 2018. All Rights Reserved.

• ACP guidelines over-simplify the process


• They exclude anabolic therapy and SERMs
(raloxifene) for postmenopausal osteoporosis
• For men, they exclude denosumab and teriparatide
• By recommending therapy for 5 years, they fail to
promote an individualized, patient-centered
approach

• Treating to a target (eg. T-score > -2.5 at total hip) is


better than a strict “end-date”
• Lack of DXA monitoring during 5 years of treatment
is problematic and will miss noncompliance, non-
response to therapy, etc.
• They don’t address concerns about stopping
denosumab (to follow)

1798
Copyright © Harvard Medical School, 2018. All Rights Reserved.

Case
• A 75 year old woman comes for follow-up.
Recently, she fell and fractured her wrist and
humerus. Everything healed well.
• Screening DXA at age 65 showed
osteoporosis. She started alendronate but
stopped after 2 pills and refused follow-up
BMD. She doesn’t like medications.
• PMH: HTN
• SH: non-exerciser; no bad habits
• Meds: lisinopril 5 mg daily

Case 2 (cont)
• On exam, BP 128/80. Wt 110 lbs, BMI 22. She
has poor tandem gait and is unable to stand
on 1 foot. Rest of exam: Negative
• Current DXA shows T-score -3.0 at spine, -3.2
at the femoral neck and -2.4 at the total hip.
WHAT WOULD BE YOUR APPROACH
TO THIS PATIENT TO REDUCE HER RISK
OF FUTURE FRACTURES?

1799
Copyright © Harvard Medical School, 2018. All Rights Reserved.

Rule Out Secondary Causes of Low


Bone Mineral Density
• CBC • 24-hour urine calcium
• Complete chemistry • Intact-PTH
screen • SPEP, ESR, CRP
• Celiac antibodies
– lytes, Ca, phos, • 24-hr urine free cortisol
creat, alk phos, • Fe/TIBC, ferritin
albumin • Tryptase
• 25(OH)Vitamin D • Homocysteine
• Connective tissue screen
• Testosterone (men) • Bone biopsy
• TSH (when indicated)

Results
• Labs:
– 25(OH)-vitamin D 15 ng/mL (30 – 50
ng/mL)
– Everything else, including SPEP, is normal
• You begin cholecalciferol 2000 IU daily and
will recheck levels in 12 weeks

1800
Copyright © Harvard Medical School, 2018. All Rights Reserved.

Next: Reduce Her Risk for Falls


• Correct vitamin D deficiency
• Take a good fall history, visual assessment
and home evaluation
• Prescribe physical therapy for gait, balance,
and strength training and tai chi

1801
Copyright © Harvard Medical School, 2018. All Rights Reserved.

Compared to Usual Care,


Injurious Falls Can Be Reduced With…
• Exercise (particularly strengthening and
balance, gait training); consider PT
• Vision assessment and treatment
• Environmental assessment and modification
• Calcium and vitamin D supplementation

RECOMMENDATIONS MUST BE TAILORED


TO THE INDIVIDUAL

Next: Increase Bone Strength


• You recommend restarting alendronate but
she worries about side effects from BPs

1802
Copyright © Harvard Medical School, 2018. All Rights Reserved.

Counseling Patients About


Osteonecrosis of the Jaw
• Dental check-up before initiation: if major
surgery required (implant or extraction),
delay start of bisphosphonate.
• If dental surgery needed during BP therapy,
stop the drug only if the dentist insists (data
do not show that this makes a difference)

Adler RA et al. 2016 J Bone Miner Res 31(1):16

Counseling Patients About


Bisphosphonates and AFFs
• 70% of patients get aching pain in femur or
groin prior to complete fracture
• In general, risk for AFF is exceedingly low
(< 0.1%) when BP is used according to
guidelines in suitable candidates (duration ≤
5 years)
• Stress fractures of femur can be picked up
ahead of time by bone scan, MRI, radiograph,
or DXA in symptomatic or worried patients

Adler RA et al. 2016 J Bone Miner Res 31(1):16-35

1803
Copyright © Harvard Medical School, 2018. All Rights Reserved.

85 yo F on Risedronate x 6 Years
with Right Anterior Thigh Pain

Bush and Chew 2009; Radiol Case Rep 4(1)

Bisphosphonates: Monitoring and


Duration of Treatment
• Annual clinical assessment
• Treat with oral bisphosphonate x 5 years
• Treat with IV bisphosphonate x 3 years
• Repeat DXA 1-2 years after starting oral BP
and at completion (for oral and IV)

1804
Copyright © Harvard Medical School, 2018. All Rights Reserved.

Bisphosphonates: Monitoring and


Duration of Treatment
• Annual clinical assessment
• Treat with oral bisphosphonate x 5 years
• Treat with IV bisphosphonate x 3 years
• Repeat DXA 1-2 years after starting oral BP
THEN REASSESS!
and at completion (for oral and IV)

Returning to the Patient….


• She increases dietary calcium to avoid taking
Ca supplements
• She takes 2000 IU cholecalciferol daily and
after 12 weeks, raises her 25OHD to 32
ng/mL; you reduce her dose to 1000 IU daily
• She refuses all bisphosphonates (both oral
and intravenous), refuses to give herself daily
injections (teriparatide), but agrees to take
denosumab 60 mg SQ q 6 months.

1805
Copyright © Harvard Medical School, 2018. All Rights Reserved.

Five Years Later…Success!


• At age 80, after 5 years of therapy, she asks if
she can stop denosumab. She has had no
additional fractures and her BMD has
improved significantly
• T-scores are now -2.3, -2.8, and -2.0 at the
spine, femoral neck, and total hip,
respectively.

Five Years Later…Success!


• At age 80, after 5 years of therapy, she asks if
she can stop denosumab. She has had no
additional fractures and her BMD has
improved
CAN SHEsignificantly
STOP DENOSUMAB AND
GO ON
• T-scores A “DRUG
are now HOLIDAY?”
-2.3, -2.8, and -2.0 at the
spine, femoral neck, and total hip,
respectively.

1806
Copyright © Harvard Medical School, 2018. All Rights Reserved.

Assessing for “Drug Holiday” After


Anti-Resorptive Therapy
1. Does the patient have a history of multiple
major fractures, a hip fracture, or a
vertebral fracture before or during
therapy?
2. Does the patient have a high risk for bone
loss, falls, or fractures in the future (eg.
frailty, frequent falls, glucocorticoids, other
high risk medications or co-morbidities)?
3. Is BMD at the hip ≤ -2.5?
Adler RA et al. 2016 J Bone Miner Res 31(1):16-35

Assessing for “Drug Holiday” After


Bisphosphonate Therapy
1. Does the patient have a history of multiple
major fractures, a hip fracture, or a
vertebral
IF fracture before
THE ANSWER TO ANY or during BP
OF THESE
therapy?
QUESTIONS IS “YES,” THEN
2.
DRUGDoesHOLIDAY
the patientIShave
NOTa RECOMMENDED
high risk for bone
loss, falls, or fractures in the future (eg.
frailty, frequent falls, glucocorticoids, other
high risk medications, co-morbidities)?
3. Is BMD at the hip ≤ -2.5?
Adler RA et al. 2016 J Bone Miner Res 31(1):16-35

1807
Copyright © Harvard Medical School, 2018. All Rights Reserved.

What Does the Primary Care Clinician


Need to Know About Bone in 2018?
• Calcium and vitamin D: anything new?
• Fracture risk factors to watch out for: falls,
meds, mental health, hyponatremia
• 2017 ACP Guidelines: the good and the bad
• Update on denosumab: the good, the bad,
and the ugly
• Managing osteoporosis over the long-term:
have we learned anything?

Denosumab Update

1808
Copyright © Harvard Medical School, 2018. All Rights Reserved.

• Hip and spine BMD continued to increase


throughout 10 years
• Fracture rates in long-term DMAB cohort
remained similar to 1st 3-year rates but there
was no true placebo group for comparison

• Analyzed BMD results of 12 women from


FREEDOM trial who stopped DMAB after 7-
10 consecutive years of taking it
• BMD was measured 18 months after last
dose (12 months after DMAB effect wears
off)

1809
Copyright © Harvard Medical School, 2018. All Rights Reserved.

BMD Changes at Lumbar Spine

↓15%

Popp AW et al. 2018 Calcif Tissue Int [epub]

Popp AW et al. 2018 Calcif Tissue Int [epub]

1810
Copyright © Harvard Medical School, 2018. All Rights Reserved.

Popp AW et al. 2018 Calcif Tissue Int [epub]

• Case series of 9 patients who sustained 50


spontaneous vertebral fractures after
stopping denosumab (within 10 mos after
last injection wore off)
• Associated with marked increase in bone
resorption and rapid loss of BMD

1811
Copyright © Harvard Medical School, 2018. All Rights Reserved.

• Describe 24 postmenopausal women who


experienced 112 vertebral fracture(s) after
stopping denosumab
• Mean # of fractures per person = 4.7 (range
1 – 9); 92% had more than 1 fracture
• All fractures occurred within 2-10 months
after the next injection was due

Spinal CT Scan Several Months Before


(a) and After (b) Vertebral Fractures

Tripto-Shkolnik L et al. 2018 Calcif Tissue Int [epub]

1812
Copyright © Harvard Medical School, 2018. All Rights Reserved.

What To Do?
• After 5 years of DMAB, reassess fracture risk
• If fracture risk remains high, based on BMD,
multiple vertebral fractures, or high FRAX®
score, consider continuing DMAB for 10 years
or switch to different agent
• If and when DMAB is to be stopped, consider
infusion of zoledronate or 1-2 years of oral
bisphosphonate to prevent rapid loss of BMD

Tsourdi E et al. 2017 Bone 105:11-17

What Does the Primary Care Clinician


Need to Know About Bone in 2018?
• Calcium and vitamin D: anything new?
• Fracture risk factors to watch out for: falls,
meds, mental health, hyponatremia
• 2017 ACP Guidelines: the good and the bad
• Update on denosumab: the good, the bad,
and the ugly
• Managing osteoporosis over the long-term:
have we learned anything?

1813
Copyright © Harvard Medical School, 2018. All Rights Reserved.

Long-Term Management of
Postmenopausal Osteoporosis
ABALO

Meier C et al. 2017 Swiss Med Wkly147:w14484

Long-Term Management of
Postmenopausal Osteoporosis
ABALO

WHAT TO DO WITH THOSE FOR WHOM


BISPHOSPHONATES
ARE CONTRAINDICATED OR NOT TOLERATED?

Meier C et al. 2017 Swiss Med Wkly147:w14484

1814
Copyright © Harvard Medical School, 2018. All Rights Reserved.

Take-Home Messages

Osteoporosis
101

Take-Home Messages
1. Prescribe Ca + D for those at high risk for
deficiencies and for those with osteoporosis
2. Reduce fractures with a two-pronged approach:
reduce falls and improve bone strength.
3. Falls are independent risk factors for fracture and
major causes of morbidity and mortality. Do
careful fall assessments by history, medication
review, and physical exam
4. In addition, keep mental health issues,
psychotropic meds, and hyponatremia on your
radar when assessing fracture risk; potentially
reversible!

1815
Copyright © Harvard Medical School, 2018. All Rights Reserved.

Take-Home Messages (cont)


5. Understand the limitations of the new ACP
osteoporosis guidelines
6. Use bisphosphonates as 1st line therapy but
consider denosumab and anabolic therapy
as alternatives
7. Do NOT give a “holiday” from denosumab!

When in Doubt, Stop Worrying and


Get an Endocrine Consult!

1816
Copyright © Harvard Medical School, 2018. All Rights Reserved.

"We come into the world under the


brim of the pelvis and go out through the
neck of the femur"

Sir John Charnley (1911-1982)

THANK YOU!!!!

1817
Copyright © Harvard Medical School, 2018. All Rights Reserved.

Women’s Health:
Take Home Messages

Caren Solomon, MD
Associate Physician, BWH
Associate Professor of Medicine, HMS
Deputy Editor, NEJM

• No disclosures

1818
Copyright © Harvard Medical School, 2018. All Rights Reserved.

Contraception

Contraceptive Effectiveness
• Rates of unintended pregnancy with typical use:
– NO method- 85%
– Diaphragm 12%
– Condom 18%
– Pill/patch/ring 9%
– DepoProvera 6%
– IUD- Copper T 0.8%, LNG-IUS 0.2%
– Etonorgestrel Implant 0.05%

1819
Copyright © Harvard Medical School, 2018. All Rights Reserved.

Long-Acting Reversible Contraception:


IUDs and Implants
• Highly effective (immediately); “forgettable”
• IUDs
– Pregnancy rare (but ectopic pregnancy common if
pregnancy occurs)
– Safe; Most women eligible (including nulliparous, prior h/o
STD)
– Potential complications: expulsion, perforation (rare),
malposition in uterus, PID (risk higher first 20 d after
insertion 1-10/1000 women but after that, rare).
– Types
• Levonorgestrel (Mirena, Skyla, Liletta, Kyleena)
– Overall less bleeding ; often used in women with menorrhagia or
dysmenorrhea; may have hormonal side effects
• Copper IUD (Paragard)
– increased bleeding (espec in first few months after insertion);
more cramping/pain
– Can be inserted within 5 d after single act of unprotected
intercourse as emergency contraception

Long-acting reversible contraception


• Implantable contraception (Etonogestrel (Nexplanon); progestin
subdermal implant )
– Highly effective, use up to 3 years approved (but effective longer)
– Must be inserted by trained provider
– Irregular bleeding common
– Contraindication: current breast cancer
– Potential complications: deep insertion /difficult removal (but
inserter easy to use, infection at insertion site, expulsion (rare)

1820
Copyright © Harvard Medical School, 2018. All Rights Reserved.

Medical Disorders
Complicating Pregnancy

Pre-Existing Hypertension
• Generally good pregnancy outcomes unless
superimposed preeclampsia develops
• Antihypertensive therapy
– generally can be tapered during pregnancy
– Goal SBP 120-160 mm Hg; DBP 80-105 mm Hg
– Stop ACEIs and ARBs prior to conception
– Methyldopa recommended 1st line (long term
outcomes data); or labetolol

1821
Copyright © Harvard Medical School, 2018. All Rights Reserved.

Pre-existing diabetes mellitus


• Birth defect risk directly related to HbA1c at conception
• Other risks: macrosomia, iatrogenic prematurity, birth trauma,
neonatal hypoglycemia; maternal: progression of DM
complications, preeclampsia, increased risk of C-section
• ADA recommends goal of HbA1c < 6.5% at conception
• Goals during pregnancy: fasting blood sugar < 95 mg/dl; 1 h post
prandial < 140 mg/dl; 2 h pp < 120 mg/dl: HbA1c < 6 % (if can be
achieved without a lot of hypoglycemia)
• Treatment
– Human insulin recommended; most experience with NPH and
regular (Category B); Lispro and Aspart also used
– Pump ok
– Insulin requirements typically increase in pregnancy but may
decrease just before delivery

Gestational diabetes mellitus


• DM first diagnosed in pregnancy
• High risk of subsequent Type 2 DM
• Women with history of GDM should be
screened for Type 2 DM

1822
Copyright © Harvard Medical School, 2018. All Rights Reserved.

Hypothyroidism

• Diagnosis made by elevated TSH


• Possible risks: poor fetal growth; premature birth;
pregnancy loss; possible effects on IQ; increased risk
for preeclampsia and placental abruption in mother
• Requirement for thyroid hormone increases in
pregnancy and returns to pre-pregnancy requirement
postpartum
• Titrate thyroid hormone dose to maintain TSH <2.5 at
conception and first 2 trimesters; < 3.5 third trimester

HPV-Related Disease and Vaccination

1823
Copyright © Harvard Medical School, 2018. All Rights Reserved.

HPV
• 80% of people infected over lifetime
• Natural history: 80% clear infection in
12 months
• High risk types: HPV 16, 18, 31, 33, 45,
52, 58

HPV Vaccination
• Recommended starting age 11 or 12
(and through age 26, possibly older but
“off label”.)
• Bivalent (16 and 18), quadrivalent, and
9 v vaccines can be used
• 3 doses
• STILL NEED cervical cancer screening

1824
Copyright © Harvard Medical School, 2018. All Rights Reserved.

Cervical Cancer Screening


• Initiate screening at age 21

• Age 21-29: Pap q 3 years (no HPV screening)


• Age 30-65 (and NOT high risk), may do:
– Combined Pap/HPV q 5 years if both tests
negative
– Paps q 3 years
– HPV alone (Cobras HPV test) q 3 years over age
25

• More frequent screening needed in high risk women


(HIV infection, immunosuppressed, h/o DES
exposure, previously treated CIN2 or 3 or
adenoCA in situ or CA )

Follow-Up of Various Pap Findings


– Absent Endocervical Cells
• Ages 21-29: Routine screening
• Age 30+ : HPV testing
– ASCUS
• If HPV negative, repeat co-testing in 3 years
• If HPV positive- colposcopy
(except ages 21-24, where would repeat PAP in 1 y)
– ASC-H (ie cannot exclude high grade SIL)
• Colposcopy needed

– Endometrial cells
• No further evaluation in asymptomatic premenopausal
women
• If postmenopuasal, need endometrial assessment

1825
Copyright © Harvard Medical School, 2018. All Rights Reserved.

Menopause

Postmenopausal hormone
therapy
• Improves vasomotor symptoms (and this remains
indication for use)

• WHI data
HRT: Increased risks of CHD, stroke, invasive breast
cancer, DVT/PE, urinary incontinence; Reduced risks
of fracture, colorectal cancer
ERT: Increased risk of stroke, reduced risk of fx
Both increased risk of dementia among women 65+

1826
Copyright © Harvard Medical School, 2018. All Rights Reserved.

Postmenopausal hormone therapy:


Endocrine Society Prescribing
Recommendations
• Individualize therapy based on clinical features and
patient preference
• Contraindications: h/o DVT/PE, breast CA, CVD, high
risk endometrial cancer/unexplained vaginal
bleeding, liver disease
• Assess CVD risk (10y) and breast cancer risk (5 y)
before initiate; avoid where risk high (> 10% and >
3% , respectively)

Management of symptoms
• Vasomotor
– ERT/HRT- at lowest effective doses/generally not
more than 5 years (though longer may be OK in
low risk highly symptomatic women)
– Lifestyle- keep cool, weight control, exercise, don’t
smoke, avoid excessive alcohol..
– Phytoestrogens- not generally effective in RCTs
and potential concerns re estrogen agonist effects
– Other medications –
• SSRIs, SNRIs (only paroxetine FDA approved for
vasomotor symptoms; recommended that paroxetine be
avoided in women taking tamoxifen)
• Gabapentin
• Clonidine (but use limited by side effects)

1827
Copyright © Harvard Medical School, 2018. All Rights Reserved.

Management of symptoms
• GU (vulvovaginal atrophy)
– Local estrogen (creams, E2 tablets, vaginal ring)
– Lubricants
– Ospemifene: SERM approved for treatment of
dypareunia in postmenopausal women (potential
adverse effects: hot flashes, DVT/PE..)

Menstrual Irregularities

1828
Copyright © Harvard Medical School, 2018. All Rights Reserved.

Amenorrhea
• Types
– Primary (Absence of menses by age 16)
– Secondary (Absence of menses for 3
months)
• Causes
– Pregnancy, Uterine or Outflow Tract
Disorders,Ovulatory Disorders
– genetic and anatomic abnormalities more
likely with primary amenorrhea
• Eval
– βHCG, FSH, PRL, TSH

Hypothalamic Amenorrhea

• Causes
– Energy Output > Energy Input
• Wt loss, eating disorders, excessive exercise
– Stress
• Psychological, Physical
• Eval: r/o other cause, MRI, BMD
• Treatment
– Weight gain, decrease exercise
– Oral contraceptives/HRT (?)
– Adequate calcium, vitamin D

1829
Copyright © Harvard Medical School, 2018. All Rights Reserved.

Premature Ovarian Insuffiency


• Elevated FSH, age < 40 yrs
• Causes
–Turner’s syndrome ; X chromosome deletions,
translocations; Fragile X premutations
– Autoimmune, Chemotherapy or radiation therapy, other
• Evaluation
•Karyotype, Fragile X premutation screen, anti-thyroid and anti-adrenal
antibodies

•Treatment
–OCPs/HRT; calcium/vit D

PCOS
• Rotterdam definition
– 2 out of 3 of the following:
– Oligo- or anovulation
– Clinical and/or biochemical signs of
hyperandrogenism
– Polycystic Ovaries
Exclusion of other causes (e.g. hyperprolactinemia,
CAH, androgen secreting tumors)
• Other common features:
– obesity; insulin resistance; infertility

1830
Copyright © Harvard Medical School, 2018. All Rights Reserved.

PCOS Management
• Weight loss /exercise
• OCP
• Metformin
• Ovulation induction/IVF
• Hair removal/spironolactone
• Follow up of glucose, lipids, bp

Osteoporosis and
Metabolic Bone Disease

1831
Copyright © Harvard Medical School, 2018. All Rights Reserved.

Calcium and vitamin D


• Supplementation with both may modestly reduce
fracture risk
• Adverse effects (calcium) :GI upset, kidney stones;
not strong evidence to support increased CVD risk
• Supplementation recommended in those with high
risk for deficiency( frail elderly, malabsorption…) and
those receiving osteoporosis treatment
• 25OH vit D levels do not need to be checked in
elderly persons without osteoporosis or high fracture
risk: vit D3 800 IU daily recommended

Bone density
• T-score
BMD compared with “young normal” adults; ( number
of standard deviations (SD) above or below the
mean); Used to dx osteopenia (-1 to -2.5)
osteoporosis (below -2.5)
• Z-score
BMD compared with persons of same sex and age.
A low Z-score indicates possible secondary cause of
osteoporosis.

, BMD recommended in women age 65+ , and men age


70+ without risk factors; earlier with risk factor or
history of fracture

1832
Copyright © Harvard Medical School, 2018. All Rights Reserved.

Risk Factors for Osteoporosis in Women


• Caucasian/Asian ALSO:
• Older age H/o falls (or risk factors for falls)
• Prior fx Mental illness or psychotropic meds
• Low weight Chronic hyponatremia
• Fam hx of fx
• Smoking
• Excess alcohol
• Glucocorticoids
• RA
• Hypogonadism
• IBD/malabsorption
• Organ tx
• Hyperthyroidism
• COPD
• Type 1 DM

Overall management strategy

• Minimize falls
– Exercise (strength and balance, gait
training), vision assessment/treatment,
environmental assessment/management,
calcium/vit D recommended
• Increase bone strength

1833
Copyright © Harvard Medical School, 2018. All Rights Reserved.

Treatment of Postmenopausal
Osteoporosis
• ACP recommends:
– use bisphosphonate or denosumab (and NOT to
use estrogen +/- progestin or raloxifene)
– treat for 5 years (without BMD monitoring)

– BUT there is controversy


• ? better to treat to target (T score > -2.5) /Others
recommend assessment to determine if bisphosphonate
holiday appropriate
• DXA may indicate lack of response or ?noncompliance
• Stopping denosumab at 5 y associated with rapid bone
loss , rebound associated fxs reported

1834
Copyright © Harvard Medical School, 2018. All Rights Reserved.

Women’s Health Board Review

Kathryn M. Rexrode, MD, MPH


Chief, Division of Women’s Health
Department of Medicine
Brigham and Women’s Hospital

Associate Professor of Medicine


Harvard Medical School

Disclosure: None

1835
Copyright © Harvard Medical School, 2018. All Rights Reserved.

Question 1
A 32-year-old woman, G1P1, comes in for a visit 6
months postpartum.
• She is fatigued and has lost few of the 30 lbs she gained
during pregnancy.
• She takes a multivitamin, no other medications.
• On physical examination:
– Weight is 150 lb, height is 5’2”.
– The thyroid feels slightly enlarged, without nodules, and is
nontender.
• CBC is normal.
• TSH is 24 mIU/L.

Question 1
Which of the following is TRUE?
A. Subacute thyroiditis is the most likely diagnosis.
B. This condition is likely to recur in subsequent
pregnancies.
C. TPO antibodies are likely to be negative.
D. You should wait for spontaneous resolution
rather than treating with thyroxine.
E. She should have a thyroid ultrasound.

1836
Copyright © Harvard Medical School, 2018. All Rights Reserved.

Question 1: Answer
Which of the following is TRUE?
A. Subacute thyroiditis is the most likely diagnosis.
B. This condition is likely to recur in subsequent
pregnancies.
C. TPO antibodies are likely to be negative.
D. You should wait for spontaneous resolution
rather than treating with thyroxine.
E. She should have a thyroid ultrasound.

Question 1: Discussion
• This patient has postpartum thyroiditis, a variant of autoimmune thyroiditis,
which is characterized by 3 phases:
– An initial hyperthyroid phase (within 6 months postpartum, lasting up to 2
months) due to leakage of thyroid hormone from an inflamed thyroid gland
– Followed by a hypothyroid phase (typically occurring up to 10 months
postpartum, and lasting 3–6 months), and then, in most cases,
– Return to euthyroidism.
• TPO antibodies are characteristically positive, and recurrence is common
following subsequent pregnancies.
• Symptomatic hypothyroidism should be treated with thyroxine, which should
not be required for more than 6 months unless the patient has developed
permanent hypothyroidism.
• Thyroid ultrasound may be useful in the evaluation of a thyroid nodule but is
not indicated in the evaluation of thyroiditis.
• Subacute thyroiditis is a painful inflammation of the thyroid that often is
described following upper respiratory infection.

1837
Copyright © Harvard Medical School, 2018. All Rights Reserved.

Question 2
A 28-year-old woman comes to establish care.
• She has a long history of oligomenorrhea and hirsutism and was
diagnosed with PCOS.
• Last menstrual period was 4 months ago, which is not unusual
for her. She takes no medications.
• Physical exam: weight is 160 lb, height 5’3”. Blood pressure is
normal.
– Slight terminal hair growth on moustache and sideburns
above her umbilicus, and around her nipples.
– Pelvic exam is limited by body habitus but appears to be
within normal limits.
• Records indicate normal prolactin and TSH levels, normal level
of fasting 17-OH progesterone, and slightly elevated total
testosterone level.

Question 2
All of the following are true EXCEPT:
A. This condition is associated with increased risk for
glucose intolerance or diabetes.
B. Risk for endometrial hyperplasia or cancer is
increased.
C. The finding of polycystic ovaries on pelvic ultrasound
is highly sensitive and specific for the diagnosis.
D. Luteinizing hormone (LH) levels are not required to
make the diagnosis.
E. Spironolactone may be useful in treatment of
associated hirsutism.

1838
Copyright © Harvard Medical School, 2018. All Rights Reserved.

Question 2: Answer
All of the following are true EXCEPT:
A. This condition is associated with increased risk for
glucose intolerance or diabetes.
B. Risk for endometrial hyperplasia or cancer is increased.
C. The finding of polycystic ovaries on pelvic
ultrasound is highly sensitive and specific for the
diagnosis.
D. Luteinizing hormone (LH) levels are not required to make
the diagnosis.
E. Spironolactone may be useful in treatment of associated
hirsutism.

Question 2: Discussion
• PCOS is a diagnosis of exclusion and is clinically diagnosed by the
combination of chronic anovulation and androgen excess not explained by
another endocrine disorder (such as late-onset congenital adrenal
hyperplasia, hyperprolactinemia, androgen-secreting tumor.)
• Although a polycystic appearance of the ovaries is generally present, this has
also been identified in 25% of women without other features of PCOS, and
this finding is considered neither sufficient nor necessary for the diagnosis.
• LH levels are typically elevated (with increased LH/FSH ratio), but this is not
necessary for the diagnosis and need not be routinely measured.
• A clear association has been observed between PCOS and insulin resistance,
and studies have documented an increased prevalence of glucose intolerance
and diabetes in affected women, even independent of associated obesity,
which is common but not always present in affected women.
• Women with PCOS also have increased risk for endometrial hyperplasia and
cancer.

1839
Copyright © Harvard Medical School, 2018. All Rights Reserved.

Question 3
A 48-year-old woman reports irregular menstrual
cycles for the past year.
• Last menstrual period 9 weeks ago
• Hot flashes for past 2 years, affecting sleep
• No significant past medical history
• No family history of blood clots or breast cancer
• No current medications.
• Physical exam is unremarkable:
– Blood pressure normal
– Normal pelvic exam and breast exam

Question 3
Which of the following statements is FALSE?

A. A follicle-stimulating hormone (FSH) level should be


checked to confirm menopause.
B. Low-dose hormone therapy could be considered.
C. A history of deep venous thrombosis would be a
contraindication to the use of postmenopausal hormone
therapy.
D. Combined therapy with low dose estrogen and progestin
would be preferable to an estrogen only regimen.

1840
Copyright © Harvard Medical School, 2018. All Rights Reserved.

Question 3: Answer
Which of the following statements is FALSE?

A. A follicle-stimulating hormone (FSH) level should be


checked to confirm menopause.
B. Low-dose hormone therapy could be considered.
C. A history of deep venous thrombosis would be a
contraindication to the use of postmenopausal hormone
therapy.
D. Combined therapy with low dose estrogen and progestin
would be preferable to an estrogen only regimen.

Question 3: Discussion
• Menopause is strictly defined as cessation of menses for ≥12 months,
but menopausal symptoms and changes in menstrual cycle pattern often
begin well before menses cease.
• A high FSH is characteristic of menopause, but checking FSH levels is not
indicated routinely, as an FSH level is an unreliable indicator of
impending menopause in perimenopausal women.
• Postmenopausal hormone therapy is very useful for managing hot
flushes. In a woman with an intact uterus, estrogen therapy should be
accompanied by progestin therapy to prevent development of
endometrial hyperplasia.
• When postmenopausal hormone therapy is used, short-term use is
recommended, rather than indefinite use.
• Contraindications to hormone therapy include venous
thromboembolism, history of breast cancer, coronary heart disease,
unexplained vaginal bleeding, and active liver disease.

1841
Copyright © Harvard Medical School, 2018. All Rights Reserved.

Question 4
This patient decides at first to take nothing for her
symptoms but returns a year later:
• She has persistent hot flashes and no menses for
the past 6 months.
• She is interested in postmenopausal hormone
therapy.
• Physical exam is normal.
• Mammogram is negative.

Question 4
Which of the following statements is FALSE?
A. Hormone therapy is associated with increased risk
for gallstones.
B. Hormone therapy increases the risk for deep venous
thrombosis/pulmonary embolism.
C. Progestins may have negative effects on mood.
D. Vaginal bleeding is rare after the first 3 months on
combined hormone replacement.
E. Hormone therapy increases the risk for stroke.

1842
Copyright © Harvard Medical School, 2018. All Rights Reserved.

Question: Answer
Which of the following statements is FALSE?
A. Hormone therapy is associated with increased risk
for gallstones.
B. Hormone therapy increases the risk for deep venous
thrombosis/pulmonary embolism.
C. Progestins may have negative effects on mood.
D. Vaginal bleeding is rare after the first 3 months
on combined hormone replacement.
E. Hormone therapy increases the risk for stroke.

Question 4: Discussion
• Patients should be educated regarding potential adverse effects of hormone
therapy. Side effects of estrogen include nausea, headache, and heavy
bleeding, whereas progestins may have adverse effects on mood and may
cause breast tenderness.
• Bleeding is common on combined estrogen/progestin therapy. Bleeding is
usually predictable on cyclical regimens, whereas unpredictable intermittent
bleeding is common the first several months on daily combined regimens.
• CEE/medroxyprogesterone increases risks for DVT/PE, stroke, breast cancer,
gallstones, and dementia (in women 65 years or older).
• Coronary events were increased early after initiation of
CEE/medroxyprogesterone in randomized trials of secondary prevention and
among women generally without history of heart disease.
• Postmenopausal hormone therapy is not indicated for the prevention of
cardiovascular disease.

1843
Copyright © Harvard Medical School, 2018. All Rights Reserved.

Question 5
A 24-year-old woman complains of irregular
menstrual cycles.
• She reports a 30-lb weight gain over the past 3
years, which she has attributed to a sedentary job.
• She takes no medications.
• Physical exam: weight 180 lb, height 5’6”.
– Mild hirsutism and acne on the face and back.
– Abdomen is obese, with pale striae.

Question 5
Which of the following conditions is
inconsistent with this presentation?
A. Late-onset congenital adrenal hyperplasia
B. Polycystic ovary syndrome
C. Cushing’s syndrome
D. Turner syndrome
E. Androgen-secreting tumor

1844
Copyright © Harvard Medical School, 2018. All Rights Reserved.

Question 5: Answer
Which of the following conditions is
inconsistent with this presentation?
A. Late-onset congenital adrenal hyperplasia
B. Polycystic ovary syndrome
C. Cushing’s syndrome
D. Turner syndrome
E. Androgen-secreting tumor

Question 5: Discussion
• This patient is demonstrating symptoms and signs consistent with
androgen excess, including irregular menstrual cycles, acne, and
hirsutism.
• Possible causes of androgen excess include polycystic ovary syndrome,
late-onset congenital hyperplasia, Cushing’s syndrome, and an
androgen secreting tumor.
• Turner syndrome (XO karyotype) is a cause of primary amenorrhea
and is associated with other characteristic features, including short
stature, failure to develop secondary sexual characteristics, and
somatic abnormalities (e.g., webbed neck, shield-like chest); androgen
excess is not a feature of Turner syndrome.

1845
Copyright © Harvard Medical School, 2018. All Rights Reserved.

Question 6
A 62-year-old woman comes to establish primary care.
• Completed menopause at age 52
• No prior hormone therapy.
• History of right tibia fracture while skiing 10 years ago and
hypertension.
• Current medication: hydrochlorothiazide 25 mg daily.
• Smokes cigarettes, ½ pack/day. She does not drink alcohol. She
swims regularly for exercise.
• No family history of hip fracture.
• Physical examination: weight 114 lb, height 5’4”.
– Blood pressure is 128/80 mm Hg.
– Rest of the exam unremarkable.

Question 6
Which of the following is NOT a risk factor for
osteoporosis in this woman?
A. Postmenopausal status
B. Cigarette smoking
C. Her weight
D. Her prior fracture
E. Use of hydrochlorothiazide

1846
Copyright © Harvard Medical School, 2018. All Rights Reserved.

Question 6: Answer
Which of the following is NOT a risk factor for
osteoporosis in this woman?
A. Postmenopausal status
B. Cigarette smoking
C. Her weight
D. Her prior fracture
E. Use of hydrochlorothiazide

Question 6: Discussion
• Risk factors for osteoporosis include age, estrogen
deficiency, cigarette smoking, lean body habitus, personal
history of fracture, family history of osteoporosis in a first-
degree relative, excessive alcohol intake, physical inactivity,
Caucasian race, and inadequate intake of calcium.
• A history of dementia, falls, or frailty also increases fracture
risk.
• Although some medications (e.g., glucocorticoids,
aromatase inhibitors) increase the risk for osteoporosis,
hydrochlorothiazide reduces urinary calcium excretion and
has been associated with reduced fracture risk.

1847
Copyright © Harvard Medical School, 2018. All Rights Reserved.

Question 7
All of the following statements are true for the
management of this patient EXCEPT:
A. She should consume 1200 mg calcium daily.
B. Drinking 2 cups of milk daily will give her adequate
vitamin D.
C. Weight-bearing exercise is recommended.
D. Calcium carbonate supplements should be taken with
meals.
E. Swimming would not be expected to increase her
bone density.

Question 7: Answer
All of the following statements are true for the
management of this patient EXCEPT:
A. She should consume 1200 mg calcium daily.
B. Drinking 2 cups of milk daily will give her
adequate vitamin D.
C. Weight-bearing exercise is recommended.
D. Calcium carbonate supplements should be taken with
meals.
E. Swimming would not be expected to increase her
bone density.

1848
Copyright © Harvard Medical School, 2018. All Rights Reserved.

Question 7: Discussion
• Approaches recommended to optimize bone health include adequate
intake of calcium (1200 mg recommended daily in a postmenopausal
woman), and vitamin D (≥ 400-800 IU daily).
• Calcium carbonate (e.g., in Tums, Os-Cal, Caltrate) is reportedly better
absorbed with meals.
• In contrast, calcium citrate (e.g., Citracal) can be taken at any time; the
latter is recommended in patients taking proton pump inhibitors and is
better tolerated by some women but is also more expensive.
• Vitamin D is added to milk, but only 100 IU per 8-oz serving. A
standard multivitamin will provide 400 IU vitamin D daily.
• Weight-bearing exercise is recommended. Swimming is not associated
with an increase in bone density.

Question 8
Bone density of the spine shows T-score –2.6 and
Z-score –1.1.
Which of the following statements is incorrect?

A. She has osteoporosis.


B. Osteoarthritis of spine could falsely increase her
bone density.
C. Her Z-score compares her to young normal women.
D. Bone mineral density (BMD) is the single best
predictor of fracture.
E. This Z-score would not suggest the need for a
workup for secondary causes of osteoporosis.

1849
Copyright © Harvard Medical School, 2018. All Rights Reserved.

Question 8: Answer
Bone density of the spine shows T-score –2.6 and
Z-score –1.1.
Which of the following statements is incorrect?

A. She has osteoporosis.


B. Osteoarthritis of spine could falsely increase her bone
density.
C. Her Z-score compares her to young normal women.
D. Bone mineral density (BMD) is the single best predictor of
fracture.
E. This Z-score would not suggest the need for a workup for
secondary causes of osteoporosis.

Question 8: Discussion
• The T-score in a bone density report represents a comparison with
“peak” bone density of young normal women
• The Z-score represents a comparison with age-matched women.
• The T-score is used to make the diagnoses of osteopenia or
osteoporosis:
– Osteopenia = T-score between –1 and –2.5 standard deviations below
peak
– Osteoporosis = T-score < –2.5 standard deviations below peak.
• A Z-score below –2 suggests bone loss out of proportion for age
– May be used to identify women more likely to have secondary causes
of osteoporosis.
• Osteophytes or a compression fracture may falsely elevate bone
density readings, and such affected areas should be deleted from
bone density analysis.

1850
Copyright © Harvard Medical School, 2018. All Rights Reserved.

Question 9
Which of the following statements is TRUE regarding
anti-resorptive therapy?
A. Raloxifene therapy would be expected both to improve
bone density and to reduce hot flashes.
B. Estrogen therapy is considered first line treatment for
osteoporosis.
C. Neither alendronate nor risedronate may be taken with
food.
D. Raloxifene does not increase risk for blood clots.
E. Routine dental work should be deferred in patients taking
bisphosphonates.

Question 9: Answer
Which of the following statements is TRUE
regarding anti-resorptive therapy?
A. Raloxifene therapy would be expected both to improve
bone density and to reduce hot flashes.
B. Estrogen therapy is considered first line treatment for
osteoporosis.
C. Neither alendronate nor risedronate may be taken
with food.
D. Raloxifene does not increase risk for blood clots.
E. Routine dental work should be deferred in patients taking
bisphosphonates.

1851
Copyright © Harvard Medical School, 2018. All Rights Reserved.

Question 9: Discussion
• Postmenopausal estrogen therapy reduces the risk of fracture, but due to risks
associated with long-term use, is not recommended as a standard therapy for
osteoporosis.
• Raloxifene (a SERM) reduces the risk of vertebral fractures but not hip
fractures. It may worsen hot flashes and increases risk for DVT/PE.
• Oral bisphosphonates should be taken on an empty stomach, first thing in the
morning, 30 minutes prior to eating, to facilitate absorption (and one should
remain upright after taking ).
• Osteonecrosis of the jaw has been reported among patients taking
bisphosphonates.
– Most reports are in patients using high doses intravenously for metastatic
bone disease, although they are reported in osteoporosis patients.
• Routine dental care should not be withheld in patients taking bisphosphonates.
• Atypical femur fractures have also been associated with bisphosphonate use,
although rare, and fractures overall are reduced with bisphosphonate use.

Question 10
A 48-year-old woman who has been your patient for
several years complains of constipation and abdominal
pain.
• She has seen 2 outside gastroenterologists in the past year.
• Colonoscopy, barium enema, endoscopy, and abdominal CT scan were
all negative.
• Otherwise healthy except for a fracture of the radius from a fall down the
stairs the preceding year.
• Married, without children.
• Physical exam remarkable only for ecchymoses on the back and right
arm.

1852
Copyright © Harvard Medical School, 2018. All Rights Reserved.

Question 10
The most appropriate next step would be to:

A. Repeat a colonoscopy at your institution.


B. Ask her whether she has ever been hurt or
threatened in her relationship.
C. Ask her generally about how she is doing, but avoid
asking directly about domestic violence.
D. Call her husband and discuss the situation with him.

Question 10: Answer


The most appropriate next step would be to:

A. Repeat a colonoscopy at your institution.


B. Ask her whether she has ever been hurt or
threatened in her relationship.
C. Ask her generally about how she is doing, but avoid
asking directly about domestic violence.
D. Call her husband and discuss the situation with him.

1853
Copyright © Harvard Medical School, 2018. All Rights Reserved.

Question 10: Discussion


• The possibility of domestic violence should be considered
in all women, regardless of background.
• Gastrointestinal complaints are common in women who
have experienced domestic violence.
• Unexplained fractures or bruising are more obvious clues,
but there are often no outward signs, and abuse may be
psychological rather than physical.
• It is recommended that all women be screened with a
straightforward question asking about any history of
being hurt or threatened.

Question 11
A 37-year-old woman comes for evaluation of a lump she
discovered in the left breast 1 month earlier.
• G1P1, menarche at 14 years. Regular menses.
• Last menstrual period occurred 1 week ago.
• She drinks 4 cups of coffee daily.
• Family history is negative for breast cancer. Her mother
has fibrocystic breast disease.
• Physical examination: well appearing.
– 1.5-cm mass palpable in the upper outer quadrant of the
left breast, slightly tender to palpation.
– No axillary adenopathy.
• Mammogram is negative.

1854
Copyright © Harvard Medical School, 2018. All Rights Reserved.

Question 11
Which of the following would be the most
appropriate next step?
A. Reassure her. No intervention is indicated.
B. Schedule repeat mammogram in 4–6 months.
C. Tell her to stop coffee and other caffeine intake and
return in 4–6 months for reexamination.
D. Order an ultrasound; if this is negative, no further workup
is required.
E. Order an ultrasound; referral should be made for biopsy
unless the lump is consistent with a simple cyst.

Question 11: Answer


Which of the following would be the most
appropriate next step?

A. Reassure her. No intervention is indicated.


B. Schedule repeat mammogram in 4–6 months.
C. Tell her to stop coffee and other caffeine intake and
return in 4–6 months for reexamination.
D. Order an ultrasound; if this is negative, no further workup
is required.
E. Order an ultrasound; referral should be made for
biopsy unless the lump is consistent with a simple
cyst.

1855
Copyright © Harvard Medical School, 2018. All Rights Reserved.

Question 11:Discussion
• A palpable breast mass requires further evaluation
regardless of patient age or mammogram results.
• Although coffee has been associated with fibrocystic
breast disease, a palpable discrete mass should not
be attributed to this or other benign etiologies
without appropriate workup.
• Ultrasound would be the next step; biopsy would be
indicated for findings other than a simple cyst in this
woman.

Question 12
A 30-year-old G0P0 with a 10-year history of Type 1
DM is interested in becoming pregnant.
• History of nonproliferative retinopathy: last eye exam 2
years ago.
• Checks blood sugars once daily.
• Current Medications: NPH 15 units twice daily, lispro 10
units with meals, and prenatal vitamin.
• Her blood pressure is 124/80 mm Hg; the rest of the
examination is unremarkable.
• Labs: HbA1c 9.0, creatinine 1.3 mg/dL. There is trace
protein on urine dipstick.

1856
Copyright © Harvard Medical School, 2018. All Rights Reserved.

Question 12
All of the following would be recommended prior to
conception EXCEPT:

A. An angiotensin-converting enzyme inhibitor should be


started to minimize progression of renal disease in
pregnancy.
B. She should increase her frequency of blood sugar
monitoring.
C. She should be referred to ophthalmology.
D. Blood sugar control should be tightened to achieve a
normal hemoglobin A1c.
E. She should have a prescription for glucagon.

Question 12: Answer


All of the following would be recommended prior to
conception EXCEPT:

A. An angiotensin-converting enzyme inhibitor should


be started to minimize progression of renal disease
in pregnancy.
B. She should increase her frequency of blood sugar
monitoring.
C. She should be referred to ophthalmology.
D. Blood sugar control should be tightened to achieve a
normal hemoglobin A1c.
E. She should have a prescription for glucagon.

1857
Copyright © Harvard Medical School, 2018. All Rights Reserved.

Question 12: Discussion


• Tight glycemic control is recommended prior to conception in women with
diabetes; risk of congenital anomalies increases with increasing first-
trimester hemoglobin A1c levels.
• Target hemoglobin A1c entering pregnancy should be near 6%.
• Eyes should be checked prior to pregnancy, as proliferative retinopathy
may progress during pregnancy.
• Hypoglycemia is common in the first trimester; women should be aware of
symptoms and treatment of hypoglycemia and have glucagon available.
• Angiotensin-converting enzyme (ACE) inhibitors and angiotensin receptor
blockers are contraindicated in pregnancy.
– Use in 1st trimester has been associated with major congenital anomalies,
including cardiac and central nervous system defects.
– Use in 2nd and 3rd trimesters is associated with oligohydramnios,
intrauterine growth retardation, anuria, renal failure, and death.

Question 13
A routine Pap smear in a 42-year-old woman shows atypical
cells. She is in a monogamous relationship and has had
normal Pap smears in previous years.
Which of the following would be MOST appropriate?

A. Treat empirically with doxycycline and repeat Pap in 3 months.


B. This is a normal finding in a perimenopausal woman and does not
require follow-up.
C. Endometrial sampling should be done to exclude endometrial
cancer.
D. Perform human papilloma virus (HPV) testing for high-risk subtypes.

1858
Copyright © Harvard Medical School, 2018. All Rights Reserved.

Question 13: Answer


A routine Pap smear in a 42-year-old woman shows atypical
cells. She is in a monogamous relationship and has had
normal Pap smears in previous years.
Which of the following would be MOST appropriate?

A. Treat empirically with doxycycline and repeat Pap in 3 months.


B. This is a normal finding in a perimenopausal woman and does not
require follow-up.
C. Endometrial sampling should be done to exclude endometrial
cancer.
D. Perform human papilloma virus (HPV) testing for high-risk
subtypes.

Question 13: Discussion


• Atypical cells are a common finding on Pap smears.
• Bacterial infection is sometimes an underlying cause
of atypical cells and should be treated if there is good
reason to suspect this, but not in asymptomatic low-
risk women.
• HPV screening for high-risk subtypes is indicated when
atypical cells are found; when positive, colposcopy is
indicated.

1859
Copyright © Harvard Medical School, 2018. All Rights Reserved.

Question 14
A 32-year-old woman, G1P0, is 16 weeks pregnant.
• Current symptoms include palpitations and weight loss.
• Physical exam is notable for pulse 110.
– She has lid lag but no appreciable exophthalmos.
– The thyroid gland is symmetrically enlarged to about
1½ times normal size.
• TSH is <0.05 mIU/L, T4 is 22.

Question 14
Which of the following is incorrect?

A. PTU can be used in the first trimester of pregnancy.


B. A thyroid uptake should be performed to confirm the
diagnosis.
C. A beta blocker could be used for symptoms.
D. Thyroid-stimulating immunoglobulin (TSI) would
likely be elevated.
E. This condition is likely to improve with treatment over
the course of pregnancy.

1860
Copyright © Harvard Medical School, 2018. All Rights Reserved.

Question 14: Answer


Which of the following is incorrect?

A. PTU can be used in the first trimester of pregnancy.


B. A thyroid uptake should be performed to confirm
the diagnosis.
C. A beta blocker could be used for symptoms.
D. Thyroid-stimulating immunoglobulin (TSI) would
likely be elevated.
E. This condition is likely to improve with treatment over
the course of pregnancy.

Question 14: Discussion


• The presentation is most consistent with Graves’ disease, which may
present early in pregnancy.
• Thyroid-stimulating immunoglobulins (TSI) are typically detectable but
need not be checked clinically.
• If antithyroid drug therapy is needed in the 1st trimester, PTU is considered
the preferred drug, as methimazole is associated with a rare scalp defect,
aplasia cutis.
• Outside of the 1st trimester, methimazole is the preferred drug, given
reports of liver toxicity and liver failure associated with PTU.
– The minimal dose necessary to keep T4 levels upper normal or slightly above
the normal range is recommended to minimize drug exposure of the fetus (as
this crosses the placenta).
• Beta blockers can be used for symptom control in pregnancy.
• Thyroid uptake testing or treatment with radioactive iodine is strictly
contraindicated in pregnancy.
• The disease tends to remit with treatment over pregnancy, and thyroid
function should be followed closely to avoid overtreatment.

1861
Copyright © Harvard Medical School, 2018. All Rights Reserved.

Question 15
A 22-year-old woman comes for contraceptive counseling. All
of the following are true EXCEPT:
A. The risks associated with use of oral contraceptive pills (OCPs)
outweigh the benefits for women with a history of coronary
heart disease or stroke.
B. OCP use is associated with a reduced risk for ovarian cancer.
C. Women who are at average risk of STDs (without current
cervicitis) are considered appropriate candidates for current
IUDs.
D. Currently used OCPs are associated with a three-fold increase
in breast cancer risk.

Question 15: Answer


A 22-year-old woman comes for contraceptive
counseling. All of the following are true EXCEPT:
A. The risks associated with use of oral contraceptive pills (OCPs)
outweigh the benefits for women with a history of coronary
heart disease or stroke.
B. OCP use is associated with a reduced risk for ovarian cancer.
C. Women who are at average risk of STDs (without current
cervicitis) are considered appropriate candidates for current
IUDs.
D. Currently used OCPs are associated with a three-fold
increase in breast cancer risk.

1862
Copyright © Harvard Medical School, 2018. All Rights Reserved.

Question 15: Discussion


• Risks of OCPs include DVT/PE, even with low-dose preparations; the
risk is reported to be higher with OCPs containing desogestrel versus
progestins such as levonorgestrel.
• Myocardial infarction and stroke are rare risks; the risk is higher in
women who smoke (especially older women) or who have
uncontrolled hypertension.
– A history of cardiovascular disease is a contraindication to use of OCPs.
• OCP users have been reported to have lower risk of ovarian cancer.
• Current OCPs have been associated with a small (about 20%)
increase in breast cancer risk.
• Long acting reversible contraceptives are highly effective ; their use
is associated with reduced risk of unintended pregnancy in sexually
active young women (versus short acting methods). Most women are
eligible for IUDs, except for those with cervicitis at time of insertion.

Question 16
You are paged by a 32-year-old woman who is worried
about pregnancy after having had unprotected
intercourse 36 hours prior. Her LMP was 16 days ago.

Which of the following is TRUE?


A. Loestrin (20 µg EE), 2 now and 2 more in 12 hours, is
appropriate for use as emergency contraception.
B. Levonorgestrel, 1.5 mg as a single dose, is appropriate
for use as emergency contraception.
C. It is too late to use emergency contraception.
D. Emergency contraception is not warranted at this time in
the cycle.

1863
Copyright © Harvard Medical School, 2018. All Rights Reserved.

Question 16: Answer


You are paged by a 32-year-old woman who is worried about
pregnancy after having had unprotected intercourse 36
hours prior. Her LMP was 16 days ago.
Which of the following is TRUE?
A. Loestrin (20 µg EE), 2 now and 2 more in 12 hours, is
appropriate for use as emergency contraception.
B. Levonorgestrel, 1.5 mg as a single dose, is
appropriate for use as emergency contraception.
C. It is too late to use emergency contraception.
D. Emergency contraception is not warranted at this time in
the cycle.

Question 16: Discussion


• Emergency contraception reduces rate of pregnancy when given within 72
to 120 hours of unprotected intercourse.
• Effective emergency contraception therapies include:
– Plan B: levonorgestrel 1.5 mg single dose
– Ulipristal acetate 30 mg.
– The Yuzpe regimen, which is combination OCP to provide 100 µg ethinyl
estradiol initially and then again at 12 hours: e.g., 2 Ovral (50 µg ethinyl
estradiol each), 2 doses 12 hours apart (now uncommonly used)
• Levonorgestrel is usually the preferred method of emergency contraception,
as it is available without a prescription (and is associated with a better side
effect profile than the Yuzpe regimen). A prescription is required for
ulipristal acetate.
• Intercourse during the luteal phase (based on LMP) does not rule out
possible pregnancy; emergency contraception is still appropriate in this
setting.
• Although the efficacy of emergency contraception falls with increasing time
after intercourse, it is still reasonable to use within 5 days of unprotected
intercourse.

1864
Copyright © Harvard Medical School, 2018. All Rights Reserved.

Question 17
Which of the following strategies is consistent with
current guidelines for cervical cancer screening:

A. Pap testing alone every 3 years (if testing normal) in


woman in her 40s.
B. Combined Pap and HPV testing every 5 years (following
normal test) starting at age 21.
C. Pap testing annually starting at age 18 in sexually active
woman.
D. No screening in women who have had HPV vaccination.

Question 17: Answer


Which of the following strategies is consistent with
current guidelines for cervical cancer screening:

A. Pap testing alone every 3 years (if testing normal) in


woman in her 40s.
B. Combined Pap and HPV testing every 5 years (following
normal test) starting at age 21.
C. Pap testing annually starting at age 18 in sexually active
woman.
D. No screening in women who have had HPV vaccination.

1865
Copyright © Harvard Medical School, 2018. All Rights Reserved.

Question 17: Discussion


• Pap smears are recommended starting at age 21 (not before).

For average risk women:


• Ages 21-29: Pap smears recommended q3 years (if normal) ;
no HPV testing for screening purposes.
• Age 30-65: can screen with PAP and HPV q 5 years (preferred)
or PAP smear q 3 years (accepted) if normal
• Age > 65: no further Pap smears if > 3 consecutive normal Pap
tests , or > 2 consecutive negative Pap/HPV tests past 10 years
(latter in past 5 y; ) or hysterectomy for benign disease.

• These do not apply to high risk women.


• Screening recommendations not altered by HPV vaccination.

Question 18
A 35-year-old woman presents for her initial
primary care visit.
• Past medical history only notable for gestational diabetes
during her pregnancy 3 years ago.
• At her 6-week postpartum visit, she completed an oral
glucose tolerance test (OGTT) and was told that her
diabetes in pregnancy had completely resolved.
• She has not seen another physician since her delivery 3
years ago.

1866
Copyright © Harvard Medical School, 2018. All Rights Reserved.

Question 18
Which of the following is TRUE?

A. Gestational diabetes is unlikely to recur in a subsequent


pregnancy.
B. Since she had a normal postpartum OGTT, her risk for
developing diabetes in the future is no higher than that of a
woman whose pregnancy was not complicated by GDM.
C. She is at increased risk for developing Type 2 diabetes,
compared with the general population.
D. Sulfonylureas are recommended to reduce future risk of
diabetes.

Question 18: Answer


Which of the following is TRUE?

A. Gestational diabetes is unlikely to recur in a subsequent


pregnancy.
B. Since she had a normal postpartum OGTT, her risk for
developing diabetes in the future is no higher than that of a
woman whose pregnancy was not complicated by GDM.
C. She is at increased risk for developing Type 2 diabetes,
compared with the general population.
D. Sulfonylureas are recommended to reduce future risk of
diabetes.

1867
Copyright © Harvard Medical School, 2018. All Rights Reserved.

Question 18: Discussion


• Women with gestational diabetes (based on ≥ 2 abnormal values on OGTT
performed at 24-28 weeks gestation) often have normal glucose levels
after delivery.
• Women should complete a 75-gram OGTT 6-12 weeks postpartum to
assess for persistent glucose abnormalities, such as impaired fasting
glucose, impaired glucose tolerance or overt diabetes.
• Even with normal postpartum OGTT results, women have increased risk for
progressing to pre-diabetes or overt diabetes
• Glycemic status should be monitored long-term in these women by
periodic assessment of fasting blood sugar, HbA1c, or 75 g OGTT:
– At a minimum, monitoring every 3 years is recommended
– Annual screening recommended in women with impaired fasting glucose or
impaired glucose tolerance.
• Based on data from the Diabetes Prevention Program, lifestyle modification
and metformin are effective in preventing progression to diabetes in the
future; sulfonylureas are not used for diabetes prevention.

1868
Copyright © Harvard Medical School, 2018. All Rights Reserved.

BOARD REVIEW PRACTICE #2

David D. Berg, MD
Fellow in Cardiovascular Medicine
Department of Medicine
Brigham and Women’s Hospital
Harvard Medical School

No Disclosures

1869
Copyright © Harvard Medical School, 2018. All Rights Reserved.

Case 1
A 23 year-old gentleman with a history of mild asthma presents for evaluation of
intermittent dysphagia for solid foods. He denies heartburn. He has not lost
weight. He reports that one year ago he underwent an upper endoscopy for
removal of pieces of steak. He has taken omeprazole 40 mg BID for the last
month but his symptoms have persisted.

His physical examination is unremarkable. An upper endoscopy reveals circular


rings in the mid esophagus. A biopsy shows a dense eosinophilic infiltrate.

Which of the following is the most appropriate first line therapy?


A. Increase the omeprazole to 80 mg twice daily
B. Esophageal Dilation
C. Topical swallowed fluticasone
D. Oral nifedipine
E. Botulinum toxin injection into the lower esophageal sphincter

Case 1 – The correct answer is C


Eosinophilic esophagitis
• Diagnostic criteria:
– Symptoms of esophageal dysfunction
(dysphagia, food impaction)
– Biopsy: >15 eosinophils/HPF
– Poor response to high dose PPI or
Normal pH in distal esophagus
Endoscopic View of Eosinophilic Esophagitis
UpToDate.com, courtesy of Dr. Eric Libby

• Diagnosis more common in men > women


• First line therapy: topical corticosteroids, 4–6 weeks
• Fluticasone, Budesonide
• Other therapies: elimination diets, leukotriene antagonists
• Dilation reserved for fixed strictures failing medical therapy

Gastroenterology 2007; 133:1342-1363

1870
Copyright © Harvard Medical School, 2018. All Rights Reserved.

Case 2
A 22 year old woman sustained a compound tibial fracture during a motor
vehicle accident requiring surgery. Her hospital course was complicated by a
lower extremity deep venous thrombosis. She has no personal or family
history of PE/DVT and a hypercoaguable work up is negative. She is not
taking any medication.

She is started on rivaroxaban 15 mg BID for 21 days followed by 20 mg once


daily.

How long should she be anticoagulated?


A. 1 months
B. 3 months
C. 12 months
D. Lifelong anticoagulation

Case 2 – The correct answer is B

Anticoagulation duration for VTE:


• Provoked:
– OCP use, surgery / prolonged
immobilization, trauma, pregnancy
– 3 months
• Unprovoked or recurrent:
– Extended or lifelong therapy Ann Intern Med. 2007;146(3):211-222

– Duration depends on patient history and Phlegmasia


preference cerulea dolens

Thrombolytics +/-
Chest. 2016;149:315-352. thrombectomy

1871
Copyright © Harvard Medical School, 2018. All Rights Reserved.

Case 3
A 64 year old woman presents for new primary care visit with complaints
of increasing dyspnea on exertion and fatigue.

On physical examination, the patient appears pale. She has no jugular


venous distention or heart murmurs. Her lungs are clear. Liver and spleen
are not palpable and she has no lower extremity edema.

Laboratories include WBC count 10,000/ml, Hct 23%, MCV 65 fL, RDW
20%, Plts 1,006,000/ml, LDH 158, ESR 24.

What is the most likely cause of the thrombocytosis?

A. Iron deficiency
B. Subacute bacterial endocarditis
C. Acute myocardial infarction
D. Autoimmune hemolytic anemia
E. Essential thrombocythemia

Case 3 – Correct Answer is A


Thrombocytosis

Primary Secondary
Clonal Reactive
Platelet count
Acute inflammation, does not
Bone Marrow Disorder infection, chronic illness,
post-splenectomy determine
primary vs
secondary cause
OFTEN associated with RARELY associated with
vasomotor, thrombotic, vasomotor, thrombotic,
bleeding complications; bleeding complications;
Splenomegaly No organomegaly

Iron deficiency causes


Am J Med 1994;96:247-53
thrombocytosis due to stimulation
NEJM 2004;350:1211-19 of platelet precursors by EPO

1872
Copyright © Harvard Medical School, 2018. All Rights Reserved.

Case 4
A 42 year-old gentleman with HIV infection (CD4 cell count
188/ul) presents with new headaches.

CT scan of the head reveals two ring enhancing lesions with mass
effect.

Which of the following is LEAST likely to be the etiology of this


finding?
A. Toxoplasmosis
B. Primary central nervous system lymphoma
C. Progressive multifocal leukoencephalopathy
D. Tuberculosis
E. Staphylococcus

Case 4 – The correct answer is C


CNS Lesions in Patients with HIV Infection

CNS lesions WITH Toxoplasmosis


mass effect CNS lymphoma
(ring enhancing) Tuberculosis

CD4 < 200


CNS lesions PML
WITHOUT mass effect HIV Encephalopathy
(non-enhancing) CMV Encephalitis

Brain tumors
CD4 > 500 Metastases

Clin Infect Dis 2002; 34:103


Mandell 2005: 1583-1601

1873
Copyright © Harvard Medical School, 2018. All Rights Reserved.

Case 5
A 55 year-old gentleman with HCV cirrhosis presents for follow up
after an upper endoscopy reveals medium-sized esophageal varices.
He has no history of GI bleeding.

On examination, his blood pressure is 110/55 with a pulse of 85


bpm. His abdominal examination reveals splenomegaly. No shifting
dullness or fluid wave is present.

Which of the following is indicated for primary prophylaxis?


A. Norfloxacin
B. Nadolol
C. Transjugular intrahepatic portosystemic shunt (TIPS)
D. Nadolol and variceal banding
E. Isosorbide mononitrate

Case 5 – The correct answer is B


Primary variceal bleeding prophylaxis

Patients Screening SBP Norfloxacin


with Endoscopy
cirrhosis

Medium
or Large Non-
Varices selective BB
Non-
selective BB
+ With Without OR
Bleed Bleed
Variceal
Ligation Variceal
TIPS Ligation
Hepatology 2007; 46:922-938

1874
Copyright © Harvard Medical School, 2018. All Rights Reserved.

Case 6
A 50 year-old woman with Marfan syndrome presents with substernal
CP radiating to her back. Physical exam reveals a II/VI early diastolic
murmur. An ECG is obtained:

Case 6

What diagnostic study would you obtain first?

A. Transthoracic echocardiogram
B. Computed tomography angiography of the chest
C. Cardiac catheterization
D. Lung ventilation-perfusion (V/Q) scan
E. Vasodilator SPECT

1875
Copyright © Harvard Medical School, 2018. All Rights Reserved.

Case 6 – The correct answer is B


Imaging of Acute Aortic Dissection
• Aortic dissection • Dissection can cause STEMI due
– Substernal chest pain, to extension of the dissection
radiating to back flap into the coronary arteries
– New diastolic murmur (R > L)

• Risk factors • Diagnostic tests


– Connective tissue disease – CTA chest / aorta
– Hypertension – Trans-esophageal
echocardiogram (TEE)
– Smoking
– [MRA]
– Atherosclerosis
– Pregnancy • Vasodilator SPECT evaluates
– Trauma myocardial ischemia, not
dissection
– Preexisting aortic aneurysm

Circulation 2003; 108:628-35; 772-8

Case 7
A 58 year-old woman with HTN on HCTZ and atenolol presents
with RLQ abdominal pain. Abdominal CT reveals a low-
attenuation, homogenous adrenal mass (2.7 cm).

Her abdominal pain resolves without intervention after several


hours and she has no further complaints.

What should be the first step in your evaluation?


A. Fine needle biopsy
B. Magnetic resonance imaging (MRI)
C. Repeat CT scan
D. Plasma and urine hormone evaluation
E. Surgical exploration and resection

1876
Copyright © Harvard Medical School, 2018. All Rights Reserved.

Case 7 – The correct answer is D


Adrenal Incidentaloma

Adrenal Incidentaloma Evaluation


• Adrenal mass >1cm •First step: hormonal testing
• Incidence 4-6% •If hormonal testing is negative,
• Most are nonfunctional, FNA can be performed to
benign adenomas evaluate for metastatic disease
• But rarely can be: •Never biopsy or resect without
– Cortisol-secreting adenoma ruling out a hormonally active
– Aldosteronoma tumor
– Pheochromocytoma
•MRI gives clues regarding
– Adrenocortical carcinoma etiology, but does NOT rule out a
– Metastastic lesion functioning tumor

NEJM 2007; 356: 601

Case 8
A 54 year-old woman presents to her PCP with new-onset
headache. The pain is retro-orbital, unilateral (right side only), and
has been progressive over the course of three weeks with
intermittent responsiveness to acetaminophen. She denies visual
changes, fevers, chills, jaw claudication, or weakness. Her exam is
normal, including thorough neurological exam.
What is the most appropriate next step in management?
A. Oxycodone
B. Head MRI/MRA
C. Amitriptyline
D. Sumatriptan
E. Referral to physical therapy

1877
Copyright © Harvard Medical School, 2018. All Rights Reserved.

Case 8 – The correct answer is B


Brain Imaging for Headaches
• Headache warning signs
– ‘First or worst’
– Increased frequency, increased severity
– New-onset headache after age 50
– New-onset with history of cancer / immunodeficiency
– Mental status changes
– Fever, neck stiffness, and meningeal signs
– Focal neurologic deficits (if not migraine with aura)
• Treatment for migraines:
– Abortive therapies: NSAIDs, triptans, anti-emetics
– Prophylaxis: amitriptyline, beta blockers, antiepileptics

• Less effective: opioids & physical therapy

Martin, et al., National Headache Foundation 2004


L.S. Medina et al / Neuroimag Clin N Am 13 (2003) 225–235

Case 9
A 31 year-old monogamous woman with no past medical
history presents for routine Pap smear. The cytological result is
negative for intraepithelial lesion or malignancy. Reflex DNA
testing for high-risk human papillomavirus (HPV) is performed
and is negative. She has no history of abnormal Pap Smears.

What is the next step in management?


A. Repeat cytology with HPV co-testing in 6 months
B. Repeat cytology with HPV co-testing in one year
C. Repeat cytology with HPV co-testing in two years
D. Repeat cytology with HPV co-testing in three years
E. Repeat cytology with HPV co-testing in five years

1878
Copyright © Harvard Medical School, 2018. All Rights Reserved.

Case 9 – The correct answer is E


Cervical Cancer Screening Guidelines (USPSTF)

• Women age ≥ 30 years


– Cytology (Pap smear) every three years
OR
– Cytology AND HPV co-testing every five years

• Women age 21-29 years


– Cytology (Pap smear) every three years
– Screening with HPV co-testing has limited utility in
this population (should be avoided)

Annals of Internal Medicine, 2012 June;156(12):880-891

Case 10
A 56 year-old male from Western Massachusetts presents to
his primary care physician with a fever and rash x 2 weeks. He
has no other symptoms. His Lyme ELISA and Western Blot tests
are positive.

What is your next BEST step in


management?

A. Doxycycline 200mg x1
B. Ceftriaxone 2g IV daily x3wks
C. Amoxicillin 500mg TID x2wks
D. Doxycycline 100mg BID x2wks
E. Azithromycin 500mg x1d,
F. then 250 mg x4d

1879
Copyright © Harvard Medical School, 2018. All Rights Reserved.

Case 10 – The correct answer is D


Treatment of Early Lyme Disease
• Erythema Migrans
– Within 7-14 days after tick detaches
– Most have 1 lesion; 25% have multiple
• Serologic testing
– Only 40% sensitive
– Not required if classic erythema migrans rash
• Treatment of choice: Doxycycline (10-21 days)
– Also treats anaplasmosis; MISSES babesiosis
– In pregnancy & pediatrics: Penicillin or Amoxicillin
• Treatment for advanced disease: 3rd generation
cephalosporin
– Includes advanced heart block, meningitis, neuritis

NEJM 2006;354: 2794-801.


NEJM 2001; 345: 115-125. NEJM 2001; 345: 79-84.

Case 11
A 68 year-old woman with asthma presents to her PCP with right foot
paresthesias, progressive right foot drop, and erythematous rash over
both lower extremities. Five months prior, she had a productive cough
and was found to have patchy infiltrates on CXR, prompting treatment
with antibiotics. Her labs are notable for WBC count 9,600/ml (58%
neutrophils, 9% lymphocytes, and 31% eosinophils), Hct 36.2, plts 271
K/ml.

Which of the following diseases is most likely to be the cause of the


patient’s presentation?

A. Granulomatosis with polyangitis (formerly Wegener’s)


B. Temporal arteritis
C. Schistosomiasis
D. EGPA (formerly Churg-Strauss)
E. Hodgkin’s disease

1880
Copyright © Harvard Medical School, 2018. All Rights Reserved.

Case 11 – The correct answer is D


Eosinophilic Granulomatosis with Polyangiitis
(formerly Churg-Strauss)
• Classic Triad = Asthma, sinus disease, peripheral eosinophilia
• Diagnosis:
- Asthma (particularly late onset)
- Mononeuropathy (multiplex) or polyneuropathy
- Rash: variable pattern (present in ~60%)
- Paranasal sinus abnormalities
- Allergic rhinitis, recurrent sinusitis, nasal polyposis
- Eosinophilia: >10% or > 1500 eosinophils/ml
- Migratory or transient pulmonary opacities on CXR
• Diagnosis supported by p-ANCA or anti-MPO antibodies
• Biopsy may show extravascular accumulation of eosinophils / granulomas,
eosinophilic pneumonia, small-vessel vasculitis

Semin Arthritis Rheum. 2003;33:106-14


Semin Respir Crit Care Med. 2006;27:148-57

Case 12
A 19 year-old woman presents with 3 days of vaginal discharge. She is
sexually active with one partner. Her LMP was 4 days ago. Exam
shows temp 99, BP 100/60 and HR 90. Her pelvic exam shows copious
mucopurulent discharge from a red, inflamed cervix. She has
tenderness on palpation of the cervix but no adnexal or uterine
tenderness. A GC/chlamydia nucleic acid amplification probe returns
positive for N. gonorrhoeae.

What is the next management step?

A. Ceftriaxone 125 mg IM x1
B. Ceftriaxone 250 mg IM x1
C. Ceftriaxone 125 mg IM x1 + Azithromycin 1 gram PO x1
D. Ceftriaxone 250 mg IM x1 + Azithromycin 1 gram PO x1
E. Doxycycline 100 mg PO BID x 7 days

1881
Copyright © Harvard Medical School, 2018. All Rights Reserved.

Case 12 – The Answer is D


Treatment of Gonococcal Infections

• CDC treatment guidelines 2015 recommend:


– Ceftriaxone 250 mg IM (increased from 125 mg in 2011 guidelines)
– AND Azithromycin or doxycycline (dual therapy regardless of chlamydial
coinfection)

• Rationale for increased CTX dose and dual therapy:


– Reports of clinical treatment failures
– Some isolates had decreased susceptibility to cephalosporins
• Increased minimal inhibitory concentration (MIC)
• Cephalosporin-resistant strains are likely tetracycline-resistant as well

2015 STD Treatment Guidelines, www.CDC.gov


J Antimicrob Chemother. 2010;65(10):2141

Case 13
A 34 year-old woman is seen in the ED with confusion, malaise, nausea.
Past medical history is notable for allergic rhinitis. Medications include
loratadine and flonase. On exam, the patient is alert and oriented to self
only. She is noted to have jaundice and bilateral lower extremity
petechiae.

Labs are notable for hematocrit 21, platelets 45,000/ml, reticulocyte


count 15%, and LDH 1,500 U/L. Coagulation studies are normal.

Peripheral smear shows:

1882
Copyright © Harvard Medical School, 2018. All Rights Reserved.

Case 13
Which of the following is the BEST next step?

A. Perform direct antiglobulin (Coombs) test


B. Check ANA
C. Obtain transthoracic echocardiogram
D. Give Intravenous immune globulin
E. Initiate plasma exchange

Case 13 – The Correct Answer is E


Management of primary TMA syndromes
Smear: microangiopathy (schistocytes >5/hpf), rare platelets
• Primary TMA syndromes include TTP,
HUS, and others
• Clinical features:
• MAHA
• Thrombocytopenia
• Neurologic abnormalities
• GI symptoms
Peripheral blood smear with schistocytes (arrows)
• AKI (less common with TTP)
• Low ADAMST13 activity (<10%) is hallmark of TTP
• Early empiric initiation of plasma exchange to treat
presumptive TTP is critical to decrease morbidity and mortality

NEJM 2006: 354(18): 1927

1883
Copyright © Harvard Medical School, 2018. All Rights Reserved.

Case 14
A 45 year-old man with a history of dyspepsia presents to the office
with new dysphagia. He initially developed dyspepsia with occasional
heartburn several years ago and these symptoms responded to
omeprazole 20 mg daily. Two years ago, his symptoms recurred so his
omeprazole dose was increased to 40 mg daily, which again helped his
symptoms. He now presents with difficulty swallowing solid foods.

Which of the following is the next best step?

A. Change the proton pump inhibitor to twice per day


B. Add a night-time dose of a H2 blocker
C. Refer for a barium swallow study
D. Refer for an upper endoscopy
E. Treat the patient for Helicobacter pylori infection

Case 14 – The correct answer is D


Endoscopy for Dyspepsia
• Dyspepsia: chronic / recurrent upper abdominal pain
• Alarm symptoms need early endoscopy
– New onset at > 55 years old
– Family history of upper GI cancer
– Weight loss
– GI bleeding or unexplained iron-deficiency anemia
– Progressive dysphagia or odynophagia
– Persistent vomiting
– Palpable mass or lymphadenopathy

• If younger than 55 years and no alarm features:


– H. pylori test and treat, PPI therapy if still symptomatic

• If alarm features:
– Early endoscopy with biopsy for H. pylori
– Low positive predictive value, high negative predictive value for cancer

Gastroenterology. 2005;129:1756-80

1884
Copyright © Harvard Medical School, 2018. All Rights Reserved.

Case 15
A 24 year-old man presents for an annual physical. On exam,
he is noted to have a harsh III/VI systolic crescendo-
decrescendo murmur best appreciated at the LLSB. The
murmur does not radiate to the carotids and is increased
with Valsalva maneuver. A prominent S4 is heard. What is the
most likely underlying pathology?
A. Congenital aortic stenosis
B. Marfan syndrome
C. Hypertrophic cardiomyopathy
D. Early-onset hypertension
E. Rheumatic heart disease

Case 15 – The correct answer is C


Differentiating systolic murmurs on exam
• Hypertrophic cardiomyopathy
– Systolic crescendo-decrescendo murmur
related to LVOT obstruction and mitral
regurgitation from SAM
– Generally does not radiate to carotids, and
is increased with Valsalva
– HCM is an autosomal dominant disorder of
the cardiac sarcomere with variable
penetrance
– Increased risk of sudden cardiac death

• Aortic Stenosis
– Systolic crescendo-decrescendo murmur,
radiates to carotids, decreased with
Valsalva
– Can be related to bicuspid, calcific, or Hypertrophic cardiomyopathy
rheumatic heart disease

Circulation 2006; 113: e858-862


JACC 2000; 36:2212
N Engl M Med 2004;350:1320-7.

1885
Copyright © Harvard Medical School, 2018. All Rights Reserved.

Case 16
A 22 year-old male college football player with no PMH
presents to the emergency room for evaluation of a small
abscess on his neck. The collection is incised and drained,
and gram stain demonstrates gram positive cocci in clusters.

What treatment would you prescribe?

A. Oral vancomycin
B. Dicloxacillin
C. Oral trimethoprim-sulfamethoxazole
D. Oral penicillin
E. Intravenous nafcillin

Case 16 – The correct answer is C


Treatment of community acquired MRSA
• Community acquired MRSA (CA MRSA)
– Most common organism in skin and soft tissue infections in ED
– Outbreaks reported in athletes, military, IVDU, MSM
– Should be considered in anyone

• CA MRSA is NOT susceptible to penicillins


– Cannot use penicillin, dicloxacillin, or nafcillin

• CA MRSA in stable patients can be treated with:


– Clindamycin, trimethoprim-sulfamethoxazole, long-acting tetracycline
(minocycline or doxycycline), linezolid

• Severe infections with CA MRSA (parenteral agents):


– IV vancomycin, daptomycin, ceftaroline, linezolid
– No role for oral vancomycin besides C. difficile infections

NEJM 2006;355:666-674

1886
Copyright © Harvard Medical School, 2018. All Rights Reserved.

Case 17
A 56 year-old man with heart failure with reduced ejection fraction
is admitted with cough, fever, and a RLL infiltrate on CXR. He is
diagnosed with community acquired pneumonia and started on
levofloxacin. On hospital day 2, the following rhythm is seen on
telemetry and the patient is unresponsive.

JAMA 1990; 264: 2788

Case 17
What is the next best step in management?
A. Defibrillation
B. Synchronized cardioversion
C. Intravenous magnesium
D. Amiodarone
E. Isoproterenol

JAMA 1990; 264: 2788

1887
Copyright © Harvard Medical School, 2018. All Rights Reserved.

Case 17 – The correct answer is A


Management of Torsades de Pointes (TdP)
• TdP: Polymorphic VT in setting of prolonged QT
– Congenital (LQTS genetic variants)
– Acquired
• Drugs (e.g. fluoroquinolones, macrolides, antipsychotics)
• Electrolyte abnormalities (e.g. hypokalemia, hypomagnesemia)
• Bradyarrhythmias (long-short sequences)
• Treatment for unstable VT/VF: Immediate defibrillation
• Intravenous magnesium
– Treats and prevents recurrent TdP
– Not useful for terminating hemodynamically unstable TdP
• Isoproterenol and RV pacing
– Prevents bradycardia, QT dispersion, EADs that predispose to TdP
– Not useful for terminating hemodynamically unstable TdP

JAMA 2003; 289: 2041


Am Heart J 2007;153:89129

Case 18
A 42 year-old alcoholic man with no other PMH is admitted with
nausea and vomiting after a recent drinking binge.

Lab studies reveal an anion gap metabolic acidosis and urine


ketones. Blood glucose is 100 mg/dl.

What is the appropriate first step in management?


A. Intravenous insulin
B. Intravenous steroids
C. Intravenous dextrose
D. Intravenous normal saline
E. Intravenous folate

1888
Copyright © Harvard Medical School, 2018. All Rights Reserved.

Case 18 – The correct answer is D


Alcoholic ketoacidosis
• AKA is caused by:
– Decreased carbohydrate intake
– Alcohol-induced inhibition of gluconeogenesis
– Alcohol-induced stimulation of lipolysis

• Presents as AGMA with ketonemia/ketonuria in an alcoholic


without diabetes

• Initial treatment is IV fluid resuscitation


– Insulin not given because glucose not elevated
– Dextrose given only after thiamine administration to avoid
Wernicke’s encephalopathy
– Folate repletion is important for malnourished or alcoholic
patients, but not for acute treatment of AKA
Emerg Med J. 2006;6:416
Am J Med 1991; 91:119

Case 19
A 52 year-old man is seen in urgent care after he was found to have
2+ blood on a urinalysis performed as part of a life insurance
evaluation. He is a former smoker but otherwise has no PMH, no
prior chemical exposures, and no family history of cancer. You repeat
a urinalysis and examine his urine sediment under a microscope. He
has 6 RBCs per high-powered field, which are normal-appearing.
What is the most appropriate evaluation?
A. Check IgA level
B. Obtain CT urogram
C. Refer to nephrology for a renal biopsy
D. Refer to urology for cystoscopy
E. Obtain CT urogram AND refer for cystoscopy

1889
Copyright © Harvard Medical School, 2018. All Rights Reserved.

Case 19 – The correct answer is E


Evaluation of microscopic hematuria
• Microscopic hematuria: ≥3 RBCs / HPF

• If dysmorphic RBCs or RBC casts are present:


–Suspect glomerular source
–Consider renal biopsy

• If RBCs are normal-appearing:


–Suspect non-glomerular source
• Ureteral bleeding (nephrolithiasis)
• Bladder bleeding (bladder cancer or cystitis)
–Initial evaluation should include CT urogram
–Obtain CTU and cystoscopy if age > 35 or at risk for bladder cancer
NEJM 2003; 348: 2330-8

Case 20
A 50 year-old woman presents to the ED complaining of a sore throat
and concern that a fish bone is stuck in her throat. She had a URI three
weeks prior but is now feeling well. Her vitals signs are stable with a
heart rate of 80 and blood pressure 126/76. Her anterior neck is tender
on exam. Direct laryngoscopy is normal.

Labs are notable for a TSH 0.03 mIU/L and ESR 80 mm/hr.

What is the most appropriate treatment?

A. Propylthiouracil 150 mg BID


B. Atenolol 50 mg daily
C. No treatment, recheck TSH in 4-6 weeks
D. 131I ablative therapy

E. Methimazole 10 mg TID

1890
Copyright © Harvard Medical School, 2018. All Rights Reserved.

Case 20 – The correct answer is C


Management of subacute thyroiditis
• Pain or tenderness of the thyroid on exam helps distinguish
different forms of thyroiditis

• Painless thyroiditis is an autoimmune process


– Includes Hashimoto’s, postpartum, drug-induced thyroiditis
– Often results in a chronically hypothyroid state

• Painful subacute thyroiditis often follows a URI


– Beta blockers are useful only for symptoms or tachycardia

• 131I
ablative therapy is reserved for Graves disease, toxic
multinodular goiter, or thyroid cancer

NEJM 2003; 348: 2646-55


BMJ 2006; 332: 1369-73

Case 21
A 35 year-old male presents to urgent care with a two-week history of
purulent rhinorrhea and right-sided maxillary sinus pain. His symptoms
have persisted despite acetaminophen, fluticasone nasal spray, and
saline irrigation. He takes HCTZ 25 mg daily and amlodipine 5 mg daily
for hypertension, and has no known drug allergies. Exam is notable for
a temperature of 101.5ºF, copious rhinorrhea, and tenderness with
percussion over the maxillary sinuses. Cardiac and pulmonary exams
are unremarkable.

What is the most appropriate next step in management?

A. Amoxicillin 500mg TID x5-7 days


B. Amoxicillin/Clavulanate 875/125mg BID x5-7 days
C. Amoxicillin/Clavulanate 2g/125mg BID x10 days
D. Doxycycline 200mg QD x5-7 days
E. Levofloxacin 500mg daily x5-7 days

1891
Copyright © Harvard Medical School, 2018. All Rights Reserved.

Case 21 – The correct answer is B


Acute Bacterial Rhinosinusitis
• Clinical Presentation
– Persistent symptoms (≥ 7-10 days)
– Severe symptoms
• Fever ≥102ºF
• Purulent nasal discharge
• Facial pain lasting ≥ 3 days
– Initial improvement, then worsening of symptoms
• Treatment: amoxicillin-clavulanate is first-line (IDSA 2012)
– High-dose amoxicillin-clavulanate for patients with risk factors such as:
• Age >65yrs
• Immunocompromised
• Recent antibiotic use
• Recent hospitalization
• High prevalence of resistant S. pneumoniae
– Doxycycline and levofloxacin for patients with penicillin allergy
Clin Infect Dis. 2012;54(8):e72-112

Case 22
A 30 year-old female presents to clinic complaining of bilateral breast
pain. She states that the pain is worse toward the end of her cycles and
also in the evening. She has had the pain for 3 months and describes it as
an aching, diffuse pain. Her exam is notable for a BMI of 28, and
pendulous breasts without erythema, masses or discharge. She has no
lymphadenopathy or skin changes. She is concerned she may have cancer
and tells you that her maternal grandmother had breast cancer at age 76.

What is the best next step in management?

A. Bilateral mammography
B. Bilateral breast MRI
C. Bilateral breast ultrasound
D. BRCA testing
E. Reassurance and recommendation for a more supportive bra

1892
Copyright © Harvard Medical School, 2018. All Rights Reserved.

Case 22 – The correct answer is E


Cystic Mastalgia

• Cyclic mastalgia
– Thought to be related to pain in Cooper’s ligament from
inadequate support of pendulous breasts

• Further testing with imaging and/or BRCA testing would


be overly aggressive

Journal of Reproductive Medicine 2005 Dec;50(12):933-9

Case 23
A 35 year-old man presents to his PCP for a routine physical
exam. His only PMH includes seasonal allergies for which he
uses fluticasone nasal spray. He does not smoke cigarettes or
drink alcohol. There is no family history of colorectal cancer
although his mother did have two adenomatous polyps at
age 55.

When should he undergo his first screening colonoscopy?

A. Now
B. Age 40 years
C. Age 45 years
D. Age 50 years

1893
Copyright © Harvard Medical School, 2018. All Rights Reserved.

Case 23 – The correct answer is B


Colorectal cancer screening
• For average risk patients:
– Colonoscopy beginning at age 50

• If history of adenomatous polyps in a 1st degree relative:


• Screening colonoscopy beginning at age 40*
OR
• Screening colonoscopy 10 years before the diagnosis of
adenomatous polyps in the relative*
(*whichever comes first)

Gastroenterology 2008: 134 1570-1595

1894
Copyright © Harvard Medical School, 2018. All Rights Reserved.

Pulmonary Disease Overview:


Selected Topics
Christopher H. Fanta, M.D.

Pulmonary and Critical Care Medicine


Brigham and Women’s Hospital
Partners Asthma Center
Harvard Medical School

Financial Conflicts of Interest

None.

1895
Copyright © Harvard Medical School, 2018. All Rights Reserved.

Selected Topics

• Hemoptysis

• Bronchiectasis and bronchiolitis

• Solitary pulmonary nodule

Hemoptysis: Case Example

• 56-year-old man presents for evaluation of a


lingering chest cold.

• He had the onset of cough and chest


congestion 8 weeks ago. He expectorated
discolored phlegm with blood streaks for
several days. His cough persists, productive of
small amounts of blood mixed with white
phlegm.

1896
Copyright © Harvard Medical School, 2018. All Rights Reserved.

Hemoptysis: Case Example (cont.)

• He smokes one pack of cigarettes per day and


takes one aspirin daily.
• His chest exam reveals a few scattered rhonchi.

Hemoptysis: Case Example (cont.)

• His chest X-ray is normal.

1897
Copyright © Harvard Medical School, 2018. All Rights Reserved.

Hemoptysis: Initial Work-Up

• Chest radiograph
• Sputum analysis

In massive hemoptysis
• Adequacy of oxygenation/ventilation
• Coagulation studies
• Hemoglobin/hematocrit

Hemoptysis: Differential Diagnosis

• Neoplastic: primary lung cancer, metastases


• Infectious: TB, abscess, bronchiectasis,
necrotizing pneumonia
• Immunologic: vasculitis, capillaritis
• Cardiovascular: PE, AVM, mitral stenosis
• Inflammatory: chronic bronchitis, trauma

1898
Copyright © Harvard Medical School, 2018. All Rights Reserved.

Chest X-rays in Hemoptysis:


Case Histories

Case 1: Four weeks of cough with discolored


phlegm intermittently mixed with
blood; fevers, night sweats, and
significant weight loss.

1899
Copyright © Harvard Medical School, 2018. All Rights Reserved.

Case 1: Further Work-Up/Treatment

Dx: Isolation room; face mask (outpatient)


Sputum for AFB smear and culture

Rx: Anti-tuberculosis therapy with 3-4 drugs

Chest X-rays in Hemoptysis:


Case Histories

Case 2: Several days of hemoptysis with


progressive shortness of breath; dark-
colored urine and creatinine of
2.5 mg%.

1900
Copyright © Harvard Medical School, 2018. All Rights Reserved.

Case 2. Further Work-Up/Treatment

Dx: Serologies (ANCA, anti-GBM antibody, ANA)


Bronchoscopy with BAL
Biopsy of kidney or lung

Rx: High-dose systemic corticosteroids +


immunosuppressives
Plasmapheresis

1901
Copyright © Harvard Medical School, 2018. All Rights Reserved.

Chest X-rays in Hemoptysis:


Case Histories

Case 3: Expectoration of blood-streaked


sputum preceded by chronic a.m.
cough in a heavy cigarette smoker.
Exam is notable for clubbing and
obvious weight loss.

1902
Copyright © Harvard Medical School, 2018. All Rights Reserved.

Case 3. Further Work-Up/Treatment

Dx: Sputum cytology


Fiberoptic bronchoscopy

Rx: Radiation therapy, chemotherapy


(Possible brachytherapy or endobronchial
laser therapy)

Chest X-rays in Hemoptysis:


Case Histories

Case 4: Hemoptysis and pleuritic chest pain on


the third post-operative day.

1903
Copyright © Harvard Medical School, 2018. All Rights Reserved.

1904
Copyright © Harvard Medical School, 2018. All Rights Reserved.

Case 4. Further Work-Up/Treatment

Dx: Chest CT angiogram


Ventilation-perfusion lung scan
(Conventional pulmonary angiogram)

Rx: Anticoagulation

Which of the Following Is the Most


Common Cause of Recurrent/Persistent
Hemoptysis in a Cigarette Smoker with
a Normal CXR?
A. Lung cancer
B. Chronic bronchitis
C. Tuberculosis
D. Aspiration
E. Pulmonary embolism

1905
Copyright © Harvard Medical School, 2018. All Rights Reserved.

Which of the Following Is the Most


Common Cause of Recurrent/Persistent
Hemoptysis in a Cigarette Smoker with
a Normal CXR?
A. Lung cancer
B. Chronic bronchitis
C. Tuberculosis
D. Aspiration
E. Pulmonary embolism

Hemoptysis with a Negative Chest X-ray:


Incidence of Lung Cancer
Authors No. of Pts No. (%) with Lung Cancer
Rath et al. 17 1 (6)
Zavala 55 9 (16)
Weaver et al. 15 0 (0)
Kalenbach et al. 32 7 (22)
Gong et al. 42 3 (7)
Dreisin et al. 19 2 (11)
Donlan et al. 72 0 (0)
Peters et al. 26 0 (0)
Poe et al. 196 12 (6)
Lederle et al. 106 6 (6)
O’Neil et al. 119 6 (5)
TOTAL 699 46 (7)

1906
Copyright © Harvard Medical School, 2018. All Rights Reserved.

Hemoptysis With a Normal CXR:


Risk Factors for Lung Cancer

• Past or present cigarette smoker

• Age >40 years

• Hemoptysis that continues beyond 1 week

Hemoptysis with a Normal CXR:


Pursuing Further Evaluation

• Chest CT scan

• Fiberoptic bronchoscopy

• (Serial chest radiographs)

1907
Copyright © Harvard Medical School, 2018. All Rights Reserved.

Hemoptysis with a Normal Chest X-ray:


Complementary Roles of Bronchoscopy
and Chest CT Scanning

Among 50 patients with hemoptysis and


a normal or non-localizing chest X-ray,
a definitive diagnosis was established
in 17 (34%) patients:

- CT made Dx in 15 (30%)
- FB made diagnosis in 5 (10%)
Tak S, et al. Australas Radiol 1999; 43:451.

Massive Hemoptysis

Definition: 600 ml of blood/24 hr

500 cc
125 cc

1908
Copyright © Harvard Medical School, 2018. All Rights Reserved.

Causes of Massive Hemoptysis

• Cancer of the lung


• Tuberculosis (active and inactive)
• Bronchiectasis
• Lung abscess
• Mycetoma
• Pulmonary vasculitis

1909
Copyright © Harvard Medical School, 2018. All Rights Reserved.

Non-Specific Treatments of
Massive Hemoptysis
• Provide adequate oxygenation and ventilation
• Position with bleeding lung dependent
• Interventional pulmonology:
• Balloon-tipped catheter placement for tamponade
• Laser or argon plasma photocoagulation
• Interventional radiology:
• Bronchial artery embolization
• Potential complication: spinal artery infarction
• Thoracic surgery: Surgical resection

Obstructive Lung Diseases

• Asthma
• Chronic Bronchitis and Emphysema
• Bronchiectasis (including cystic fibrosis)
• Bronchiolitis
• Upper airway obstruction

1910
Copyright © Harvard Medical School, 2018. All Rights Reserved.

Case History

• 66-year-old woman was referred for evaluation of


her “asthma.“
• She had a 10-year history of intermittent wheeze,
cough and exertional dyspnea, made worse by
respiratory tract infections, cold air and exercise.
Her medications were theophylline and inhaled
albuterol.

Case History (cont.)

• Chest exam was notable for inspiratory


rhonchi at the right base. Spirometry revealed
a mild restrictive pattern.
• Sputum culture on several occasions grew H.
influenzae.

1911
Copyright © Harvard Medical School, 2018. All Rights Reserved.

1912
Copyright © Harvard Medical School, 2018. All Rights Reserved.

Clinical Presentations of Bronchiectasis:


Historical Features

• Chronic productive cough


• Recurrent hemoptysis
• Recurrent bronchitis/pneumonia
• Persistent radiographic “pneumonia”
• Associations: sinusitis, infertility

Clinical Presentations of Bronchiectasis:


Physical Examination

• Focal inspiratory crackles


• Low-pitched wheezing (“rhonchi”)
• Clubbing

1913
Copyright © Harvard Medical School, 2018. All Rights Reserved.

Microbiology of Bronchiectasis

• Bacterial
• Staphylococcus
• Hemophilus
• Pseudomonas
• Mycobacterial (MAI)
• Fungal (Aspergillus)

1914
Copyright © Harvard Medical School, 2018. All Rights Reserved.

Chest CT Appearance in
Bronchiectasis

• Lack of bronchial tapering


• Bronchial dilatation (internal diameter >1.5 x
diameter of accompany vessel)
• Visualization of bronchi in lung periphery
(within 1 cm of pleura)
• Bronchial wall thickening may be present

1915
Copyright © Harvard Medical School, 2018. All Rights Reserved.

Bronchiectasis with
mucoid impaction

1916
Copyright © Harvard Medical School, 2018. All Rights Reserved.

Etiologies of Bronchiectasis

• Localized
• Residuum of pneumonia
• Distal to a focal airway obstruction
• Widespread
• Cystic fibrosis
• Primary ciliary dyskinesia
• Hypogammaglobulinemia
• Alpha-1 antitrypsin deficiency

Etiologies of Bronchiectasis:
Other Observations

• AIDS
• Rheumatoid arthritis
• Bronchiolitis
• Non-tuberculous mycobacterial infection
• Allergic bronchopulmonary aspergillosis

1917
Copyright © Harvard Medical School, 2018. All Rights Reserved.

Non-Tuberculous Mycobacterial
(NTM) Pulmonary Infection
• In non-immunocompromised host:
• Bronchiectasis and tree-in-bud nodules
• Predominantly middle and lower lobes
• Predilection for middle-aged and older thin
women (“Lady Windermere’s syndrome”)
• Dx: positive culture from sputum x2 or BAL
• Rx: macrolide, ethambutol, rifampin (>1 year)

1918
Copyright © Harvard Medical School, 2018. All Rights Reserved.

Allergic Bronchopulmonary
Aspergillosis (ABPA)
• Susceptible hosts: asthma and cystic fibrosis
• Aspergillus colonization of airways with
intense allergic response
• Central bronchiectasis
• Very high serum IgE and positive specific IgE
and IgG to aspergillus; eosinophilia
• Rx: steroids plus antifungal therapy (azoles)

1919
Copyright © Harvard Medical School, 2018. All Rights Reserved.

Etiologies:
Special Syndromes

Congenital anatomic defects


Munier-Kuhn syndrome (tracheobronchomegaly)
Yellow nails syndrome (lymphatic hypoplasia)
Young’s syndrome (bronchiectasis and
azospermia)
Williams-Campbell syndrome (bronchial cartilage
deficiency)

Therapeutic Options in
Bronchiectasis

• Antibiotics
• Clearance of secretions
• Bronchodilators
• Mucolytics
• Mechanical vibration techniques
• Other: azithromycin, corticosteroids

1920
Copyright © Harvard Medical School, 2018. All Rights Reserved.

Randomized Trial of Suppressive


Antibiotics (in Cystic Fibrosis)

• 520 patients with cystic fibrosis randomized to


inhaled tobramycin 300 mg BID vs. placebo
every other month for 6 months.

Outcomes: Lung function (FEV1)


Density of Ps. aeruginosa in
sputum
Need for hospitalization/intravenous
antibiotics
Ramsey BW, et al., N Engl J Med 1999; 340:23-30

Randomized Trial of Nebulized


Tobramycin (in Cystic Fibrosis)
Change in Lung Function

Ramsey BW, et al., N Engl J Med 1999; 340:23-30

1921
Copyright © Harvard Medical School, 2018. All Rights Reserved.

Other Therapeutic Options in


Bronchiectasis

• Bronchodilators
• Clearance of secretions
• Chest physiotherapy and postural drainage
• Vibratory PEP device
• External electric vibrator
• Pneumatic vest
• Exercise

Cough Assist Devices

Flutter Acapella

Aerobika

1922
Copyright © Harvard Medical School, 2018. All Rights Reserved.

Azithromycin in Bronchiectasis

Azithromycin 500 mg
(or placebo) 3 days/week

Lancet 2012; 380:660-7.

Azithromycin in Bronchiectasis

Azithromycin 500 mg
(or placebo) 3 days/week

Lancet 2012; 380:660-7.

1923
Copyright © Harvard Medical School, 2018. All Rights Reserved.

Complications of Bronchiectasis

• Hemoptysis
• Infection with resistant organisms
• Respiratory failure
• Other: weight loss, mycetoma

Bronchiolitis: Definition

Acute or chronic cellular inflammation


of the bronchiolar walls.

Acute: Viral (e.g., RSV), mycoplasma

Chronic: Follicular bronchiolitis


Mineral dust bronchiolitis
Cigarette smoke respiratory bronchiolitis
Diffuse panbronchiolitis (Japan)
Bronchiolitis obliterans

1924
Copyright © Harvard Medical School, 2018. All Rights Reserved.

1925
Copyright © Harvard Medical School, 2018. All Rights Reserved.

Bronchiolitis Obliterans: Etiologies


• Toxic fume inhalation (e.g., nitrogen oxides in
silo filler’s lung disease; diacetyl in workers
exposed to artificial butter flavoring for popcorn)
• Post-infectious (e.g., viral, mycoplasma)
• Immune-mediated
• Rheumatoid arthritis
• Ulcerative colitis
• S/P transplantation (e.g., bone marrow, lung)

Bronchiolitis: Clinical Features

• Hx: dyspnea, non-productive cough


• P.E.: inspiratory crackles; mid-inspiratory
squeak; expiratory wheezes
• CXR: hyperinflation; +small patchy
parenchymal infiltrates
• CT: areas of relative oligemia (“mosaic
oligemia”); “tree-in-bud” appearance

1926
Copyright © Harvard Medical School, 2018. All Rights Reserved.

1927
Copyright © Harvard Medical School, 2018. All Rights Reserved.

Bronchiolitis: Treatment

• Corticosteroids (systemic/inhaled)
• Bronchodilators
• (Post-transplant: extracorporeal
photopheresis; etanercept; montelukast)
• Supplemental oxygen as needed
Bronchiolitis obliterans is often refractory to
therapy.

1928
Copyright © Harvard Medical School, 2018. All Rights Reserved.

Evaluation of SPN

In a 50-year-old cigarette smoker, the


next step in the evaluation of this
pulmonary nodule should be
percutaneous needle biopsy:

A. True
B. False

1929
Copyright © Harvard Medical School, 2018. All Rights Reserved.

Evaluation of SPN

In a 50 year-old cigarette smoker, the


next step in the evaluation of this
pulmonary nodule should be
percutaneous needle biopsy:

A. True
B. False

Solitary Pulmonary Nodule:


Etiologies

• Benign
• Malignant

1930
Copyright © Harvard Medical School, 2018. All Rights Reserved.

Solitary Pulmonary Nodule:


Aphorism

“When in doubt, cut it out.”

Solitary Pulmonary Nodule:


Radiographic Features Definitive for
Benign Disease

• Calcification (exception: small eccentric


focus of calcium within nodule)

• Absence of increase in size over 2 years*

*exception: ground-glass (“sub-solid”)


nodule

1931
Copyright © Harvard Medical School, 2018. All Rights Reserved.

(Bronchoalveolar Cell Carcinoma)


Adenocarcinoma In Situ or
Minimally Invasive Adenocarcinoma

Solitary Pulmonary Nodule:


Clinical Features Favoring
Benign Disease

• Age <40 yrs


• Non-smoker
• Residence in area endemic for fungal
infections

1932
Copyright © Harvard Medical School, 2018. All Rights Reserved.

Solitary Pulmonary Nodule:


Radiographic Features Favoring
Benign Disease

• Smooth margin
• Round edge
• Presence of satellite lesions
• Small size (<6 mm on chest CT scan)

Solitary Pulmonary Nodule:


Use of Biopsy Techniques to Identify
Benign Disease

• Percutaneous needle aspiration (or


navigational bronchoscopy) is the preferred
technique, but yield for a specific benign
etiology is low, and a non-specific benign
result is unreliable in excluding malignancy
(false negatives >20%).

1933
Copyright © Harvard Medical School, 2018. All Rights Reserved.

Video-Assisted Thoracoscopic
Surgery (VATS)

• Three one-inch chest incisions, plus


chest tube drainage
• Average length of hospital stay:
2-3 days
• Pre-resection mediastinoscopy is routine
at many centers

Solitary Pulmonary Nodule:


Novel Techniques to Identify Benign
Disease

• Positron emission tomography (PET)


scanning with fluorodeoxyglucose
• Contrast-enhanced CT scan
• Radiolabeled peptide analogs of somatostatin
(depreotide)

1934
Copyright © Harvard Medical School, 2018. All Rights Reserved.

PET Scanning
for Solitary Pulmonary Nodules

Rationale:
• Malignant tumors have increased metabolic activity
• 18F-2-fluoro-2-deoxyglucose (FDG) is a radiotracer
taken up by malignant tumors but not by
metabolically inactive tissue.
• Intensity of FDG uptake in a lung nodule is
compared with background activity.

1935
Copyright © Harvard Medical School, 2018. All Rights Reserved.

1936
Copyright © Harvard Medical School, 2018. All Rights Reserved.

PET Scanning
For Solitary Pulmonary Nodules

Results:
for nodules >8 mm in diameter
• Sensitivity: ~95%
• Specificity: ~85%
• Negative predictive value: ~95%
• False negatives: Bronchoalveolar carcinoma

Pre-Operative Assessment

• Exclude distant metastases


• Brain, bone, liver, adrenals

• Exclude extrathoracic primary malignancy


• Breast, thyroid, and rectal exams; urinalysis

• Assess adequacy of cardiopulmonary reserve


• PFTs; possible ABGs; ETT if indicated

1937
Copyright © Harvard Medical School, 2018. All Rights Reserved.

PET Scanning
For Assessing Distant Metastases

Comparison of conventional staging work-up vs.


conventional work-up plus PET-CT scan in NSCLC:
• Significant reduction in futile thoracotomies;
• No difference in justifiable thoracotomies;
• No difference in mortality.

Fischer B, et al., NEJM 2009; 361:32-9.

Prospective Monitoring of Solitary


Pulmonary Nodule in Low-Risk Patients

• Serial chest X-rays or CT scans


• Surgical resection of enlarging nodule
• Compulsive follow-up!

1938
Copyright © Harvard Medical School, 2018. All Rights Reserved.

Screening for Lung Cancer:


Early Lung Cancer Action Project (ELCAP)

• 1000 cigarette smokers (> 10 pack-years),


> age 60, and fit to undergo thoracic surgery

• Screened with helical low-dose chest CT scans

• 233 persons (23%) had noncalcified nodule(s)


detected on CT scan (vs. 68 persons [7%] on CXR)

Henschke Cl et al., Lancet 1999; 354:99-105.

Screening for Lung Cancer:


Early Lung Cancer Action Project (ELCAP)

• 27 of the 233 persons with nodules had


malignancy (12% of those with nodules,
3% of the total screened population)

• 23/27 (85%) of the malignant tumors were


Stage 1
Henschke Cl, et al., Lancet 1999; 354:99-105.

1939
Copyright © Harvard Medical School, 2018. All Rights Reserved.

Prognosis for Stage I Lung Cancers


Detected by CT Screening

IELCAP
investigators.
NEJM 2006;
355:1763-71.

Screening for Lung Cancer:


National Lung Screening Trial
• 50,000 current and former smokers enrolled
between 9/02 and 2/04.
• Randomized to low-dose screening chest CT
scans vs. routine CXRs annually for 3 years.
• Annual follow-up through 2009.
• Primary outcome: Is there a > 20% decrease
in lung cancer mortality with CT screening?
-- www.nci.nih.gov/nlst

1940
Copyright © Harvard Medical School, 2018. All Rights Reserved.

NLST: Preliminary Results

Arm Person years Lung Lung cancer Reduction in Value of Efficacy


(py) cancer mortality per lung cancer test boundary
deaths 100,000 py mortality (%) statistic

CT 144,097.6 354 245.7 20.3 –3.21 –2.02

CXR 143,363.5 442 308.3

Deficit of lung cancer deaths in CT arm exceeds that expected by chance

NEJM 2011; 10.1056/NEJMoa1102873

NLST: Results

National Lung Screening Trial Research Team.


NEJM 2011; 365:395-409.

1941
Copyright © Harvard Medical School, 2018. All Rights Reserved.

Who Should Be Screened?


(Yearly Low-Dose Chest CT Scans Until Age 77)

• Age 55-80 years;


• Current or former (within 15 years) cigarette
smoker of at least 30 pack-years;
• Absence of life-threatening co-morbidity;
• With proper pre-screening counseling, including
smoking cessation counseling, and follow-up of
detected nodules by experienced team.
-- recommendations of ALA, ACS, and CMS.

Radiographic Follow-Up of SPN:


Fleischner Society 2017 Guidelines

Does not
apply to:
• Patients
who have a
known
cancer.
• Immuno-
suppressed
patients.
• Lung cancer
screening

MacMahon H, et al. Radiology 2017 Feb 23:161659.


doi: 10.1148/radiol.2017161659. [Epub ahead of print].

1942
Copyright © Harvard Medical School, 2018. All Rights Reserved.

Risk Factors for


Lung Cancer:
MacMahon • Tobacco use
H, et al. • Family history of
Radiology lung cancer
2017 Feb • Upper pulmonary
23:161659. lobe location of
nodule
• Presence of
doi: emphysema or
10.1148/ pulmonary fibrosis
radiol. • Older Age
2017161659 • Female gender
[Epub ahead
of print].

Summary Recommendations
Conclusions:
“In general, all SPNs should be considered
malignant until proven otherwise.
Resection is the treatment of choice for all
patients who are surgical candidates after
approriate preoperative evaluation,
including those with indeterminate
nodules.” ACCP Evidence-Based Guidelines:
Diagnosis and Management of Lung Cancer.
Chest 2003; 123 (suppl):1S-337S.

1943
Copyright © Harvard Medical School, 2018. All Rights Reserved.

References
Hemoptysis:
• Dudha M, Lehrman S, Aronow WS, et al. Hemoptysis: diagnosis
and treatment. Compr Ther 2009; 35:139-49.
Bronchiectasis:
• Barker AF. Bronchiectasis. N Engl J Med 2002; 346:1383-93.
Bronchiolitis:
• Barker AF, et al. Obliterative bronchiolitis. N Engl J Med 2014;
370:1820-8.
Solitary Pulmonary Nodule:
• Fischer B, Lassen U, Mortensen J, et al. Preoperative staging of lung
cancer with combined PET-CT. N Engl J Med 2009; 361:32-9.
• National Lung Screening Trial Research Team. Reduced lung-
cancer mortality with low-dose computed tomographic screening.
N Engl J Med 2011; 365:395-409.

Financial Conflicts of Interest

None.

1944
Copyright © Harvard Medical School, 2018. All Rights Reserved.

BRIGHAM AND Harvard


WOMEN’S HOSPITAL
Medical School

Interstitial Lung Disease

Hilary J. Goldberg, MD, MPH


Division of Pulmonary and Critical Care Medicine
Brigham and Women’s Hospital

Financial disclosures

Clinical Coordinating Center Member for multicenter


trial of pirfenidone in RA-ILD
– Genentech, Inc.

1945
Copyright © Harvard Medical School, 2018. All Rights Reserved.

Overview

Presentation and Evaluation


Idiopathic Interstitial Pneumonias
– IPF
– CPFE
Autoimmune Lung Disease
Granulomatous Lung Disease:
– Sarcoidosis
– Hypersensitivity Pneumonitis

Presentation

Presenting symptoms
SOB
Cough
Chest pain
Duration of symptoms

Presenting physical signs


Hypoxia
Tachypnea
Inspiratory crackles
Clubbing
Cor pulmonale

1946
Copyright © Harvard Medical School, 2018. All Rights Reserved.

Imaging
Radiographic features:

Pattern:
Reticular
Nodular
Combined: Reticulonodular

Location:
Upper lobe
Lower lobe
Homogenous vs. heterogenous

CT features

1947
Copyright © Harvard Medical School, 2018. All Rights Reserved.

CT features

Pulmonary function testing


Restrictive ventilatory
defect:
Symmetric decrease
in FEV1 and FVC
Decrease in TLC
Decreased diffusion
capacity

Mixed restriction and


obstruction

1948
Copyright © Harvard Medical School, 2018. All Rights Reserved.

Further evaluation

Bronchoscopy: Patients with atypical


– Rule out infection features (age <45,
– Cell counts extensive ground glass,
– Transbronchial sparing of bases, nodular
biopsies: pattern, absence of
Rule out other causes honeycombing, atypical
Granulomatous PFT findings) should be
disease considered for biopsy if
feasible
Surgical lung biopsy

Neither

Diagnosis

AJRCCM 2011; 183: 788

1949
Copyright © Harvard Medical School, 2018. All Rights Reserved.

The Idiopathic Interstitial Pneumonias

ATS/ERS Joint Statement, AJRCCM 2002; 165: 277

The Idiopathic Interstitial Pneumonias

Am J Resp Crit Care Med 2013; 188: 733

1950
Copyright © Harvard Medical School, 2018. All Rights Reserved.

The Idiopathic Interstitial Pneumonias

Am J Resp Crit Care Med 2013; 188: 733

The Idiopathic Interstitial Pneumonias

CRP Diagnosis Histology


Idiopathic pulmonary Usual interstitial Pneumonia
fibrosis
Nonspecific interstitial Nonspecific interstitial
pneumonitis pneumonitis
Acute interstitial Diffuse alveolar damage
pneumonia
Desquamative Desquamative interstitial
interstitial pneumonitis pneumonitis
Respiratory bronchiolitis Respiratory Bronchiolitis
– interstitial lung
disease

1951
Copyright © Harvard Medical School, 2018. All Rights Reserved.

Pathophysiology of Idiopathic
Pulmonary Fibrosis

Annals of Internal Medicine, vol 134, 2001

Idiopathic Pulmonary Fibrosis –


Potential Risk Factors

Cigarette smoking
Environmental
Microbial agents
GERD
Genetic factors

ATS/ERS/JRS/ALAT Statement;
AJRCCM 2011; 183: 788
Respirology 2015; 20: 1010

1952
Copyright © Harvard Medical School, 2018. All Rights Reserved.

ATS/ERS criteria for diagnosis

Exclusion of other known causes of ILD


Presence of UIP pattern on HRCT in patients not
subjected to surgical lung biopsy
Specific combinations of HRCT and surgical lung biopsy
pattern in patients subjected to surgical lung biopsy

ATS/ERS/JRS/ALAT Statement; AJRCCM 2011; 183: 788-824

Differential diagnosis of Idiopathic


Pulmonary Fibrosis
Other idiopathic interstitial pneumonias
Hypersensitivity pneumonitis
Sarcoidosis
Cryptogenic Organizing Pneumonia
Eosinophilic pneumonia
Malignancy
Infection

1953
Copyright © Harvard Medical School, 2018. All Rights Reserved.

Radiologic features

Usual Interstitial Pneumonitis Alternate Diagnosis


Subpleural, basilar Upper or mid-lung
predominance predominance
Reticular pattern Peribronchovascular
Honeycombing with or distribution
without traction Ground glass,
bronchiectasis micronodules, cysts
Mosaicism

ATS/ERS/JRS/ALAT Statement; AJRCCM 2011; 183: 788-824

HRCT features of Idiopathic Pulmonary


Fibrosis

1954
Copyright © Harvard Medical School, 2018. All Rights Reserved.

Histologic features

Usual Interstitial Pneumonitis Alternate Diagnosis


Subpleural, paraseptal Hyaline membranes
distribution Granuolmas
Fibrosis, architectural Organizing pneumonia
distortion, honeycombing Inflammatory infiltrate
Temporal and spatial Airway centered changes
heterogeneity
Fibroblastic foci

ATS/ERS/JRS/ALAT Statement; AJRCCM 2011; 183: 788-824

Histologic features of Usual Interstitial


Pneumonia

1955
Copyright © Harvard Medical School, 2018. All Rights Reserved.

Histologic features of Usual interstitial


Pneumonia

Acute exacerbation of interstitial lung


disease

Am J Resp Crit Care Med 2016; 194: 265

1956
Copyright © Harvard Medical School, 2018. All Rights Reserved.

Acute exacerbation of interstitial lung


disease

Am J Resp Crit Care Med 2016; 194: 265

Acute exacerbation of interstitial lung


disease

Chest 2008; 134: 1265


Chest 2008; 134: 844

1957
Copyright © Harvard Medical School, 2018. All Rights Reserved.

Acute exacerbation Idiopathic


Pulmonary Fibrosis

Am J Respir Crit Care Med 2007; 176: 636

Idiopathic Pulmonary Fibrosis -


Therapy
Recommendation against the following:
– Corticosteroid monotherapy
– Colchicine
– CSA
– Combined corticosteroid and immune-modulator
therapy
– INF alpha
– Bosentan
– Etanercept
ATS/ERS/JRS/ALAT Statement; AJRCCM 2011; 183: 788-824

1958
Copyright © Harvard Medical School, 2018. All Rights Reserved.

Idiopathic Pulmonary Fibrosis -


therapy
No medical therapies proven to prolong survival
The only therapy demonstrated to prolong survival is
lung transplantation
– Endothelin receptor antagonists
– Anticoagulation
– N-Acetyl-Cysteine
– IL-13 modulation
– Other
Recently published results demonstrate effects on lung
function, composite end points (and mortality)
Newly FDA Approved Medications
– Pirfenidone
– TK inhibitors - nintedanib NEJM 2005; 353(21): 2229-2242

N-Acetyl-Cysteine

Precursor of antioxidant
glutathione
Randomized, placebo
controlled trial of NAC in
addition to “standard
therapy”

NEJM 2005; 353: 2229 - 42

1959
Copyright © Harvard Medical School, 2018. All Rights Reserved.

PANTHER
Triple therapy vs.
placebo
Triple therapy arm
closed early:
Increased
mortality (11% vs
1%)
Increased
hospitalization
(29% vs 8%)
More SAE’s (31%
vs. 9%)
Eur Respir J 2012; 39: 805–806
NEJM 2012; 366: 1968

PANTHER

N Eng J Med 2014; 370: 2093

1960
Copyright © Harvard Medical School, 2018. All Rights Reserved.

Pirfenidone
Anti-inflammatory
and antifibrotic

Inhibits production
and activity of TGF-
beta

Animal studies:
Inhibits TGF-beta
and pro-
collagen/collagen
gene and mRNA
expression

Inhibits fibroblast
mitogenesis

Expert Opin Investig Drugs. 2010


Feb; 19(2): 275–283
N Eng J Med 2014; 370: 2083

Impact of pirfenidone in setting of


declining FVC

Thorax 2016; 71: 429

1961
Copyright © Harvard Medical School, 2018. All Rights Reserved.

Impact of pirfenidone in setting of


declining FVC

Thorax 2016; 71: 429

Nintedanib
Competitive
inhibitor of FGFR-1,
VEGFR-2, PDGFR
alpha and beta

Reduced
proliferation,
migration and
survival of
fibroblasts

Potentially also
attenuates
angiogenesis

Eur Resp J 2015; 45: 1434


N Eng J Med 2014; 370: 2071

1962
Copyright © Harvard Medical School, 2018. All Rights Reserved.

Nintedanib

N Eng J Med 2014; 370: 2071

Idiopathic Pulmonary Fibrosis -


prognosis
Prognosis
3 year 50% mortality
only 10-25% of patients respond to therapy

ATS/ERS Joint Statement, AJRCCM 2002; 165: 277

1963
Copyright © Harvard Medical School, 2018. All Rights Reserved.

Idiopathic Pulmonary Fibrosis – natural


history

ATS/ERS/JRS/ALAT Statement; AJRCCM 2011; 183: 788-824

Combined Emphysema and Pulmonary Fibrosis

Chest 2009; 136: 10

1964
Copyright © Harvard Medical School, 2018. All Rights Reserved.

Autoimmune ILD

Lancet 2012; 380: 689

Sarcoidosis pathophysiology

AJRCCM 1999; 160: 736


Am J Resp Crit Care Med 2011; 183:573

1965
Copyright © Harvard Medical School, 2018. All Rights Reserved.

Sarcoidosis – Systemic Disease

Organ involvement:
Lung (90%)
Skin (dermatitis, EN, LP)
Eyes (ant/post uveitis)
RES (liver, spleen)
Cardiac (Arrhythmias; cardiomyopathy)
Upper airways (Chronic sinusitis)
Hypercalcemia
Renal (stones, granulomas)
Neurologic (Psych/Thalamic pain)

Sarcoidosis – Pulmonary Staging

CXR Abnormalities:
stage I - hilar adenopathy (80% remit)
stage II - adenopathy + interstitial (60%)
stage III - interstitial (30%)
stage IV - end stage upper lobe fibrosis
More acute presentation = better prognosis

1966
Copyright © Harvard Medical School, 2018. All Rights Reserved.

Sarcoidosis – Stage I

Sarcoidosis – Stage II

1967
Copyright © Harvard Medical School, 2018. All Rights Reserved.

Sarcoidosis - Pathology

Sarcoidosis -
Syndromes
Specific Syndromes
Lofgrens
syndrome - E.
nodosum, fever,
hilar adenopathy.
Heerfordt’s
syndrome -
uveitis, parotitis,
fever

N Eng J Med 2013; 369: 458

1968
Copyright © Harvard Medical School, 2018. All Rights Reserved.

Diagnosis

Am J Resp Crit Care Med 2011; 183:573

Sarcoidosis – Indications for Treatment

Indications for treatment


Symptomatic pulmonary disease
DOE
Cough
Airway sarcoid
Worsening PFTs (DLco)
Cardiac involvement
Renal disease
Posterior uveitis
Neurologic involvement
Lupus pernio
Hypercalcemia

1969
Copyright © Harvard Medical School, 2018. All Rights Reserved.

Sarcoidosis - Treatment

Am J Resp Crit Care Med 2011; 183:573

Hypersensitivity Pneumonitis

Can result from various antigen exposures


Leads to variable presentations
Lack of consensus on diagnostic criteria

Arch Pathol Lab Med 2008; 132: 195 – 198


AJRCCM 2005; 171: 792-798

1970
Copyright © Harvard Medical School, 2018. All Rights Reserved.

Hypersensitivity Pneumonitis -
Allergens

Arch Pathol Lab Med 2008; 132: 195 – 198

Hypersensitivity pneumonitis– Clinical


Presentation
Dyspnea
Cough
Crackles
Acute, subacute, or chronic
Allergen may be difficult to identify

Arch Pathol Lab Med 2008; 132: 195 – 198

1971
Copyright © Harvard Medical School, 2018. All Rights Reserved.

Hypersensitivity Pneumonitis -
Evaluation

Detailed history
(Serum precipitans)
Pulmonary function tests
High resolution CT scan
Bronchoscopy
Surgical lung biopsy
Arch Pathol Lab Med 2008; 132: 195 – 198

Hypersensitivity pneumonitis -
Imaging
Upper lobe predominant
Interstitial prominence
Can present in fibrotic phase

Arch Pathol Lab Med 2008; 132: 195 – 198

1972
Copyright © Harvard Medical School, 2018. All Rights Reserved.

Hypersensitivity pneumonitis -
Imaging

Hypersensitivity
pneumonitis -
Pathology

Interstitial pneumonitis
Cellular bronchiolitis
Non-necrotizing
granulomas

Arch Pathol Lab Med 2008; 132: 199-203

1973
Copyright © Harvard Medical School, 2018. All Rights Reserved.

Hypersensitivity pneumonitis –
Management
Identification and elimination of offending allergen
Minimization of exposure
Systemic corticosteroids

Arch Pathol Lab Med 2008; 132: 195 – 198

Approach to Interstitial lung disease

Confirm diagnosis of ILD


CXR, HRCT scan, Bronchoscopy, OLBx, “clinical
picture”
Assess function
PFTs, exercise oximetry testing, QOL tool (SF-36)
Treatment plan
disease specific plus vaccines, exercise, nutrition,
etc.

1974
Copyright © Harvard Medical School, 2018. All Rights Reserved.

Question 1
A 38 yo Caucasian woman reports to your office with joint
discomfort as well as a painful, red rash on her legs for the past 2
weeks. She denies fevers, chills, and weight loss but states that
she has had some sweats. A lung exam reveals no adventitous
sounds. Pulmonary function tests are normal and a chest x-ray
displays bilateral lymphadenopathy without pulmonary
parenchymal abnormalities. The most appropriate course of
action at the present time is which of the following:
A) Refer for lung biopsy as these findings represent an atypical
presentation of an interstitial lung disease
B) Start 40mg of prednisone per day
C) After alternative diagnoses such as heart failure and indolent
infection are ruled out, consideration for lung transplant referral
should be made
D) Schedule follow-up in 4-6 weeks for –reassessment

Question 1
A 38 yo Caucasian woman reports to your office with joint
discomfort as well as a painful, red rash on her legs for the past 2
weeks. She denies fevers, chills, and weight loss but states that
she has had some. A lung exam reveals no adventitous sounds.
Pulmonary function tests are normal and a chest x-ray displays
bilateral lymphadenopathy without pulmonary parenchymal
abnormalities. The most appropriate course of action at the
present time is which of the following:
A) Refer for lung biopsy as these findings represent an atypical
presentation of an interstitial lung disease
B) Start 40mg of prednisone per day
C) After alternative diagnoses such as heart failure and indolent
infection are ruled out, consideration for lung transplant referral
should be made
D) Schedule follow-up in 4-6 weeks for –reassessment

1975
Copyright © Harvard Medical School, 2018. All Rights Reserved.

Question 2
A 61 yo man with a 40 pack year smoking history as well as
hyperlipidemia presents to his primary care physician with
reports of progressive dyspnea on exertion and a dry cough
for the past 2 years. Once an active man, able to walk an
infinite distance on a flat surface, he now notes feeling short
of breath with 3 flights of stairs and while playing with his
grandchildren. His cough is non-productive and is not
associated with various environmental exposures. He is a
former banker. On exam, he is noted to have an oxygen
saturation of 98% on room air and 97% after walking 200
yards. He has fine crackles at the bases of his lungs and no
clinical evidence of volume overload. His basic lab values are
normal. Pulmonary function tests show a reduced total lung
capacity. A CT scan of the chest shows bibasilar, subpleural
reticular findings without infiltrate or lymphadenopathy.

Question 2

The most appropriate course of action at the present time is which


of the following:
A) Refer for lung biopsy as these findings represent an atypical
presentation of an interstitial lung disease
B) Start 40mg of prednisone a day
C) After alternative diagnoses such as heart failure and indolent
infection are ruled out, consider for lung transplant referral
D) Refer the patient for pulmonary rehabilitation

1976
Copyright © Harvard Medical School, 2018. All Rights Reserved.

Question 2
The most appropriate course of action at the present time is which
of the following:
A) Refer for lung biopsy as these findings represent an atypical
presentation of an interstitial lung disease
B) Start 40mg of prednisone a day
C) After alternative diagnoses such as heart failure and indolent
infection are ruled out, consider for lung transplant referral
D) Refer the patient for pulmonary rehabilitation

Consider appropriateness for pirfenidone/nintedanib and/or


referral for lung transplant

Summary

Interstitial lung disease has a variety of causes


A detailed history and targeted evaluation is warranted
IPF is the most common IIP and carries a poor prognosis
New therapies are emerging to slow disease
progression
Other IIPs may be more responsive to immune
suppressive agents

1977
Copyright © Harvard Medical School, 2018. All Rights Reserved.

References

Travis, WD et al, 2013, “An Official American Thoracic Society/European


Respiratory Society Statement: Update of the International
Multidisciplinary Classification of the Idiopathic Interstitial Pneumonias,”
Am J Respir Crit Care Med Vol 188, Iss. 6, pp 733–748

Collard, HR, et al, “Acute Exacerbation of Idiopathic Pulmonary Fibrosis:


An International Working Group Report,” Am J Respir Crit Care Med Vol
194, Iss. 3, pp 265–275

Freemer, M. & King, T.E., Jr. 2001, "The ACCESS Study. Characterization of
Sarcoidosis in the United States", American Journal of Respiratory and
Critical Care Medicine, vol. 164, no. 10, pp. 1754-1755.

Valeyre, D., Prasse, A., Nunes, H., et al, “Sarcoidosis,” Lancet, Vol 383,
Iss. 9923, pp 1155-1167

Madison, JM, 2008, “Hypersensitivity Pneumonitis: Clinical Perspectives,”


Archives of Pathologic and Laboratory Medicine, vol 132, pp. 195-198

Financial Disclosures

Site PI for multi-center clinical trials sponsored by:


– Gilead, Inc.
– Biogen Idec, Inc.

Clinical Coordinating Center Member for multicenter


trial of pirfenidone in ILD
– Genentech, Inc.

1978
Copyright © Harvard Medical School, 2018. All Rights Reserved.

Craig P. Hersh, MD, MPH


Associate Physician, Channing Division of Network Medicine and
Division of Pulmonary and Critical Care Medicine,
Brigham and Women’s Hospital
Associate Professor of Medicine, Harvard Medical School

BRIGHAM AND
WOMEN’S HOSPITAL
HARVARD
MEDICAL SCHOOL

Consultant:
◦ 23andMe
Grant support:
◦ Boehringer-Ingelheim
◦ Novartis

1979
Copyright © Harvard Medical School, 2018. All Rights Reserved.

COPD: Definition and pathophysiology


Assessment of COPD Patients
Treatment of stable COPD
Exacerbations
Summary and Review Questions

COPD: Definition and pathophysiology


Assessment of COPD Patients
Treatment of stable COPD
Exacerbations
Summary and Review Questions

1980
Copyright © Harvard Medical School, 2018. All Rights Reserved.

COPD is a common, preventable and treatable


disease that is characterized by persistent
respiratory symptoms and airflow limitation
that is due to airway and/or alveolar
abnormalities usually caused by significant
exposure to noxious particles or gases.

Global Initiative for Chronic Obstructive Lung Disease 2017


www.goldcopd.org

COPD is a common, preventable and treatable


disease that is characterized by persistent
respiratory symptoms and airflow limitation
that is due to airway and/or alveolar
abnormalities usually caused by significant
exposure to noxious particles or gases.

Global Initiative for Chronic Obstructive Lung Disease 2017


www.goldcopd.org

1981
Copyright © Harvard Medical School, 2018. All Rights Reserved.

NHANES 3 Mannino, MMWR Aug 2002


◦ 12 million diagnosed with COPD
◦ Additional 12 million with undiagnosed airflow
obstruction
State-by-state:
◦ BRFSS 2011

www.cdc.gov/brfss/

COPD is a common, preventable and treatable


disease that is characterized by persistent
respiratory symptoms and airflow limitation
that is due to airway and/or alveolar
abnormalities usually caused by significant
exposure to noxious particles or gases.

Global Initiative for Chronic Obstructive Lung Disease 2017


www.goldcopd.org

1982
Copyright © Harvard Medical School, 2018. All Rights Reserved.

Normal spirometry

Airflow obstruction

www.nhlbi.nih.gov

http://www.swiss-exped.ch/content/ge/research_reports/pneumologie/pneumologie_en.html

1983
Copyright © Harvard Medical School, 2018. All Rights Reserved.

COPD is a common, preventable and treatable


disease that is characterized by persistent
respiratory symptoms and airflow limitation
that is due to airway and/or alveolar
abnormalities usually caused by significant
exposure to noxious particles or gases.

Global Initiative for Chronic Obstructive Lung Disease 2017


www.goldcopd.org

Normal

Kim V, Proc Am Thorac Soc 2008;5:478 Courtesy of histology-world.com

1984
Copyright © Harvard Medical School, 2018. All Rights Reserved.

Kim V, Proc Am Thorac Soc 2008;5:478

Normal

Kim V, Proc Am Thorac Soc 2008;5:478 Courtesy of histology-world.com

1985
Copyright © Harvard Medical School, 2018. All Rights Reserved.

Crapo J, ed. Atlas of COPD, 2009

Crapo J, ed. Atlas of


COPD, 2009

1986
Copyright © Harvard Medical School, 2018. All Rights Reserved.

COPD is a common, preventable and treatable


disease that is characterized by persistent
respiratory symptoms and airflow limitation
that is due to airway and/or alveolar
abnormalities usually caused by significant
exposure to noxious particles or gases.

Global Initiative for Chronic Obstructive Lung Disease 2017


www.goldcopd.org

1987
Copyright © Harvard Medical School, 2018. All Rights Reserved.

Global Strategy for Diagnosis, Management and Prevention of COPD

Risk Factors for COPD

Genes

Infections

Socio-economic
status

Aging Populations
© 2014 Global Initiative for Chronic Obstructive Lung Disease

1988
Copyright © Harvard Medical School, 2018. All Rights Reserved.

Lange P, NEJM 2015;373:111

COPD is a common, preventable and treatable


disease that is characterized by persistent
respiratory symptoms and airflow limitation
that is due to airway and/or alveolar
abnormalities usually caused by significant
exposure to noxious particles or gases.

Global Initiative for Chronic Obstructive Lung Disease 2017


www.goldcopd.org

1989
Copyright © Harvard Medical School, 2018. All Rights Reserved.

COPD: Definition and pathophysiology


Assessment of COPD Patients
Treatment of stable COPD
Exacerbations
Summary and Review Questions

Global Strategy for Diagnosis, Management and Prevention of COPD

Assessment of COPD

Assess symptoms
Assess degree of airflow
limitation using spirometry
Assess risk of exacerbations

Assess comorbidities
© 2014 Global Initiative for Chronic Obstructive Lung Disease

1990
Copyright © Harvard Medical School, 2018. All Rights Reserved.

Symptoms Physical Exam


Cough Tripod posture
Sputum Skin: cyanosis
Dyspnea Breathing: tachypnea, pursed
Wheeze lip, prolonged expiration,
Chest tightness accessory muscle use
Weight loss Barrel Chest
Muscle weakness Breath Sounds: distant,
wheezes
Edema
Cardiac: distant, increased
Depression P2, JVD, edema
Cachexia

http://www.catestonline.org/

1991
Copyright © Harvard Medical School, 2018. All Rights Reserved.

Mahler DA, Chest 1988;93:580

Global Strategy for Diagnosis, Management and Prevention of COPD

Classification of Severity of Airflow


Limitation in COPD*
In patients with FEV1/FVC < 0.70:

GOLD 1: Mild FEV1 > 80% predicted

GOLD 2: Moderate 50% < FEV1 < 80% predicted

GOLD 3: Severe 30% < FEV1 < 50% predicted

GOLD 4: Very Severe FEV1 < 30% predicted

*Based on Post-Bronchodilator FEV1


© 2014 Global Initiative for Chronic Obstructive Lung Disease

1992
Copyright © Harvard Medical School, 2018. All Rights Reserved.

Hurst JR, NEJM 2010;


363:1128.

www.goldcopd.org

1993
Copyright © Harvard Medical School, 2018. All Rights Reserved.

COPD Tuberculosis
Asthma ◦ CXR with infiltrate
◦ Earlier onset Obliterative Bronchiolitis
◦ Atopic history ◦ Young, non-smokers
CHF ◦ Occupational, CVD
◦ Crackles, edema ◦ Air trapping on CT
◦ Restriction on PFTs Diffuse Panbronchiolitis
◦ Male, nonsmokers
Bronchiectasis
◦ Sinusitis
◦ Sputum
◦ Centrilobular nodules
◦ Abnormal X-ray/CT

Lung volumes, diffusing capacity


◦ hyperinflation
Arterial Blood Gas
◦ Recommended in severe COPD
◦ O2 sat gives no information about pCO2
6-minute walk test
◦ Evaluate disability, rehab assessment
Testing for α1-antitrypsin deficiency
◦ Serum levels
◦ Genotyping or protein phenotyping

1994
Copyright © Harvard Medical School, 2018. All Rights Reserved.

Flattened diaphragms
Reduced lung markings
Narrow cardiac
silhouette
Cachexia

Low sensitivity and


specificity
Alternative diagnoses

Muller, Thorax 2002;57:982

Age 47, FEV1 20% predicted

Age 42,
Age 42,FEV
FEV1 38% predicted
1 38% predicted Age 47, FEV1 20% predicted

Hersh CP, COPD 2007;4:331-7

1995
Copyright © Harvard Medical School, 2018. All Rights Reserved.

Crapo J, ed.,
Atlas of COPD, 2009

Pulmonary hypertension
◦ Echocardiogram if signs of right heart failure
Coronary artery disease
◦ Low threshold for evaluation
Lung cancer
◦ CT screening may reduce mortality
Osteoporosis, Depression, GERD, Anemia,
etc.
◦ Usual clinical assessments

National Lung Screening Trial, NEJM 2011;365:395


Vanfleteren LE, Lancet Respir Med 2016

1996
Copyright © Harvard Medical School, 2018. All Rights Reserved.

COPD: Definition and pathophysiology


Assessment of COPD Patients
Treatment of stable COPD
Exacerbations
Summary and Review Questions

Global Strategy for Diagnosis, Management and Prevention of COPD

Manage Stable COPD: Goals of Therapy

Relieve symptoms
Improve exercise tolerance Reduce
symptoms
Improve health status

Prevent disease progression


Reduce
Prevent and treat exacerbations risk
Reduce mortality
© 2015 Global Initiative for Chronic Obstructive Lung Disease

1997
Copyright © Harvard Medical School, 2018. All Rights Reserved.

Smoking Cessation
Smoking Cessation
Smoking Cessation
Supplemental oxygen
Vaccination
◦ Influenza
◦ Pneumococcal polysaccharide-23
◦ Pneumococcal conjugate-13 in adults ≥65
Pulmonary Rehabilitation

Poole PJ, Cochrane Database Syst Rev 2006


Dransfield MT, AJRCCM 2009;180:499

1. ASK: every patient at every visit


2. ADVISE: clear, strong, personalized
3. ASSESS: willingness to quit
4. ASSIST: plan, support, pharmacotherapy
5. ARRANGE: follow-up contact

JAMA 2000;283:3244

1998
Copyright © Harvard Medical School, 2018. All Rights Reserved.

Nicotine replacement therapy


◦ OTC: Patch, Gum, Lozenge
◦ Rx: Inhaler, Nasal spray
Bupropion SR
Varenicline
Combination therapy
◦ Short acting NRT + patch
◦ Bupropion + NRT
◦ Varenicline + NRT
Second-line (off-label)
◦ nortriptyline, clonidine
E-cigarettes???
Patnode CD, Ann Intern Med 2015;163:608
Rigotti N, Lancet Respir Med 2013;1:241
Hartman-Boyce, Cochrane Database Syst Rev 2016

1999
Copyright © Harvard Medical School, 2018. All Rights Reserved.

PaO2 ≤55mmHg or SaO2 ≤88%, or


PaO2 ≤60mmHg or SaO2 ≤89%,
◦ with cor pulmonale, right heart failure or
polycythemia
Benefits for exercise-induced desaturation
are unclear LOTT, NEJM 2016;375:1617

Shortness of
breath

Decreased exercise Reduced


capacity Activity

2000
Copyright © Harvard Medical School, 2018. All Rights Reserved.

Pulmonary rehab
◦ Stable lung disease
◦ Symptomatic dyspnea
Multidisciplinary
6-12 week program, 2-3 times/week
Exercise
◦ Lower body exercise
◦ Upper body exercise
◦ Respiratory Muscle Training
Education
Psychosocial Support

Spruit MA, AJRCCM 2013;188:e13

Reduction in dyspnea
Improved exercise performance
Reduced ER visits and hospitalizations
Improved quality of life
Psychological benefits
Pulmonary rehab is cost-effective
Pulmonary rehab is covered by Medicare
and most private insurers

2001
Copyright © Harvard Medical School, 2018. All Rights Reserved.

Beta2-agonists
Short-acting beta2-agonists
Long-acting beta2-agonists
Anticholinergics
Short-acting anticholinergics
Long-acting anticholinergics
Combination short-acting beta2-agonists + anticholinergic in one inhaler
Combination long-acting beta2-agonist + anticholinergic in one inhaler
Methylxanthines
Inhaled corticosteroids
Combination long-acting beta2-agonists + corticosteroids in one inhaler
Systemic corticosteroids
Phosphodiesterase-4 inhibitors

Bronchodilators are first-line therapy


◦ Strong recommendation for FEV1<60%
Long-acting bronchodilators
◦ LABA or LAMA
Dual bronchodilators: LAMA/LABA
◦ Reduces symptoms and exacerbations compared to
monotherpy
Short-acting bronchodilators in all patients

Qaseem, Ann Intern Med 2011;155:179


Tashkin DP, NEJM 2008;359:1543
Rodrigo GJ, Int J COPD 2017;12:907

2002
Copyright © Harvard Medical School, 2018. All Rights Reserved.

Bronchodilators are first-line therapy


ICS monotherapy is never appropriate
ICS may reduce exacerbation risk in severe
COPD
Triple therapy
ICS are associated with increased risk of
pneumonia
Chronic systemic steroids should be avoided

Calverley PMA, NEJM 2007:356:775


Ernst P, AJRCCM 2007;176:162
Magnussen H, NEJM 2014;371:1285

2003
Copyright © Harvard Medical School, 2018. All Rights Reserved.

www.copdfoundation.org

Calverley, AJRCCM 2007;176:154


Martinez, Lancet 2015;385:857

2004
Copyright © Harvard Medical School, 2018. All Rights Reserved.

MACRO trial
N=1142 HR 0.73 (p<0.001)

1 year

More effective
◦ >65 years old
◦ Ex-smoker
◦ Milder COPD

Albert, NEJM 2011;365:689


Han, AJRCCM 2014;189:1503

Lung volume reduction surgery


◦ Upper lobe predominant
◦ Low exercise capacity
Bronchoscopic lung volume reduction
Lung transplantation

NETT, NEJM 2003;348:2059


*Klooster K, NEJM 2015;373:2325
Davey C, Lancet 2015;386:1066
Yusen R, J Heart Lung Transplant 2015;34:1264

2005
Copyright © Harvard Medical School, 2018. All Rights Reserved.

COPD: Definition and pathophysiology


Assessment of COPD Patients
Treatment of stable COPD
Exacerbations
Summary and Review Questions

Symptoms
◦ dyspnea, sputum volume, purulence
Arterial blood gas
Chest X-ray
ECG
CBC
Blood chemistries
Spirometry: not recommended

www.goldcopd.org

2006
Copyright © Harvard Medical School, 2018. All Rights Reserved.

Supplemental oxygen
Bronchodilators
Systemic corticosteroids
Consider antibiotics
Non-invasive mechanical ventilation
◦ pH≤7.35 and/or PaCO2≥45 mmHg
Fluid status and nutrition
DVT prophylaxis
Co-existing conditions

www.goldcopd.org
Brochard L, NEJM 1995;333:817

Criner GJ, Chest 2015;147:883-893, 894-942

2007
Copyright © Harvard Medical School, 2018. All Rights Reserved.

COPD: Definition and pathophysiology


Assessment of COPD Patients
Treatment of stable COPD
Exacerbations
Summary and Review Questions

COPD includes emphysema, large and


small airway disease
Spirometry is essential for diagnosis
Multidimensional assessment
◦ Symptoms
◦ Airflow limitation
◦ Exacerbation risk
◦ Comorbidities

2008
Copyright © Harvard Medical School, 2018. All Rights Reserved.

Tried and true treatments


◦ Smoking cessation
◦ Supplemental oxygen
◦ Pulmonary rehab
Pharmacotherapy
◦ Bronchodilators are first line
◦ LABA and/or LAMA
◦ ICS: add-on/combination
◦ Roflumilast for frequent exacerbations

a) COPD
b) Asthma
c) Pulmonary fibrosis
d) Congestive heart failure
e) All of the above

2009
Copyright © Harvard Medical School, 2018. All Rights Reserved.

a) COPD
b) Asthma
c) Pulmonary fibrosis
d) Congestive heart failure
e) All of the above
COPD is not the only cause of
dyspnea in a smoker

a) COPD
Spirometry is essential for diagnosis.
b) Asthma
Possible, hx of atopy. Spirometry post-BD.
c) Pulmonary fibrosis
Restricted PFTs. Abnormal chest x-ray, CT scan.
d) Congestive heart failure
Hx of CAD. Exam, chest x-ray, echocardiogram.
e) All of the above

2010
Copyright © Harvard Medical School, 2018. All Rights Reserved.

a) LAMA
b) ICS-LABA combination
c) Nicotine replacement therapy
d) All of the above
e) (a) and (c) only

a) LAMA
b) ICS-LABA combination
c) Nicotine replacement therapy
d) All of the above
e) (a) and (c) only

2011
Copyright © Harvard Medical School, 2018. All Rights Reserved.

FEV1 55%

Bronchodilators first-line
◦ LAMA or LABA
◦ LAMA-LABA combination

ICS for frequent exacerbations (Group D)


Smoking cessation for all current smokers
Assessment of comorbidities

2012
Copyright © Harvard Medical School, 2018. All Rights Reserved.

Global Initiative for Chronic Obstructive Lung Disease, Global


Strategy for the Diagnosis, Management and Prevention of
Chronic Obstructive Pulmonary Disease (updated 2018),
available at www.goldcopd.org
COPD Foundation, www.copdfoundation.org
Criner GJ et al., Prevention of acute exacerbations of COPD.
Chest 2015;174:894-942.
Qaseem A, et al., Diagnosis and Management of Stable
Chronic Obstructive Pulmonary Disease. Ann Intern Med
2011;155:179-191.

2013
Copyright © Harvard Medical School, 2018. All Rights Reserved.

Sleep Apnea:
Diagnosis & Treatment
Intensive Review of Internal Medicine
Lawrence J. Epstein, MD
Assisiant Clinic Director
Division of Sleep and Circadian Disorders,
Department of Medicine
Brigham and Women’s Hospital
Instructor in Medicine
Harvard Medical School

Disclosure
Consultant
American Academy of Sleep Medicine
CareCore National
AIM Specialty Health

2014
Copyright © Harvard Medical School, 2018. All Rights Reserved.

Sleep Apnea

Sleep Apnea is
Common

Dangerous

Easily recognized

Treatable

2015
Copyright © Harvard Medical School, 2018. All Rights Reserved.

Types of Sleep Disordered


Breathing
Apnea
Cessation of airflow > 10 seconds
Hypopnea
Decreased airflow > 10 seconds
associated with either :
Arousal
Oxyhemoglobin desaturation

Apnea Patterns
Obstructive Mixed Central

Airflow

Respiratory
effort

2016
Copyright © Harvard Medical School, 2018. All Rights Reserved.

Central Sleep Apnea


Central Alveolar Hypoventilation
High-Altitude Periodic Breathing
Cheyne-Stokes Ventilation
Congestive Heart Failure (CHF)
40% of those with LVEF < 40%
Causes sleep fragmentation and hypoxemia
Treatment: CHF tx, oxygen, theophylline,
acetazolamide, Positive Airway Pressure

Obstructive Sleep Apnea is


A disorder of sleep and breathing due to
repetitive collapse of the upper airway
Causes
Awake symptoms
Impaired performance
Cardiovascular complications

2017
Copyright © Harvard Medical School, 2018. All Rights Reserved.

Prevalence of Obstructive Sleep


Apnea
30-60 year olds
25

20

Percent of
Population 15
Male
Female
10 U.S. Pop

0
AHI > 5 SAS Asthma

Adapted from Young T et al. N Engl J Med 1993;328.

Obesity Trends* Among U.S. Adults


BRFSS, 1990, 1999, 2009
(*BMI ≥30, or about 30 lbs. overweight for 5’4” person)

1990 1999

2009

No Data <10% 10%–14% 15%–19% 20%–24% 25%–29% ≥30%

Source: Behavioral Risk Factor Surveillance System, CDC

2018
Copyright © Harvard Medical School, 2018. All Rights Reserved.

Pathophysiology of Apnea

Pathophysiology of Sleep Apnea


Awake: Small airway + neuromuscular compensation

Loss of neuromuscular Sleep Onset Hyperventilate:


compensation correct hypoxia
+ & hypercapnia
Decreased pharyngeal
muscle activity Airway opens

Airway collapses
Pharyngeal muscle
activity restored
Apnea
Arousal from
Hypoxia & Increased sleep
Hypercapnia ventilatory
effort

2019
Copyright © Harvard Medical School, 2018. All Rights Reserved.

Clinical Consequences

Sleep Apnea

Sleep fragmentation,
Hypoxia/Hypercapnia

Excessive daytime Cardiovascular


sleepiness Complications

Morbidity
Mortality

Consequences
Excessive Daytime Sleepiness Cardiovascular
Increased motor vehicle Systemic hypertension
crashes
Cardiac arrhythmias
Increased work-related
accidents Myocardial ischemia
Poor job performance
Cerebrovascular disease
Decreased quality of life
Pulmonary Hypertension/
cor pulmonale

2020
Copyright © Harvard Medical School, 2018. All Rights Reserved.

Consequences: Mortality
18 Year Follow-up: All Cause Mortality
No CPAP Treatment

N = 1396

Adjusted for age,


BMI, Htn, CV
disease

Young et al. Sleep


2008;31:1071

Consequences:
Hypertension

Shepard JW Jr. Med Clin North Am 1985;69

2021
Copyright © Harvard Medical School, 2018. All Rights Reserved.

Cardiovascular Consequences:
Hypertension
Prospective Study of Association
Between OSA and Hypertension
3 Adjusted for
2.5 age, sex
BMI, neck
2
Odds circ., cigs.,
Ratio 1.5 ETOH,
1 baseline Htn
0.5
0
0 0.1 - 4.9 5 - 14.9 > 15
Apnea-Hypopnea Index (AHI)
Adapted from Peppard PE et al. N Engl J Med 2000;342.

Sleep Apnea Risk Factors


Obesity
Increasing age
Male gender
Anatomic abnormalities of upper airway
Family history
Alcohol or sedative use
Smoking
Associated conditions

2022
Copyright © Harvard Medical School, 2018. All Rights Reserved.

Risk Factor: Obesity


80
>4% Arterial saturation dipa h-1
70

60

50

40

30

20

10

0
70 80 90 100 110 120 130 140
% Predicted normal neck circumference

Adapted from Davies RJ et al. Eur Respir J 1990;3

Risk Factor: Age & Gender

35

30

25

% with 20
AHI > 5 Female
15 Male

10

0
30-39 Yrs 40-49 Yrs 50-60 Yrs

Adapted from Young T et al. N Engl J Med 1993;328.

2023
Copyright © Harvard Medical School, 2018. All Rights Reserved.

Risk Factor: Menopause


Odds Ratios:
AHI > 5/hr
Peri: 1.2 (0.7, 2.2)
Post: 2.6 (1.4, 4.8)
AHI > 15/hr
Peri: 1.1 (0.5, 2.2)
Post: 3.5 (1.4, 8.8)

Young T, et al, Am J Respir Crit Care Med 2003; 167:1183

Evaluation for OSA


Routine Health Pt Complains of High Risk
Maintenance Exam (PCP) Symptoms (PCP/SS) Screenings (PCP)

Sleep Disorders
Symptoms?

Yes

Sleep Evaluation
(PCP/SS)

No Yes
Evaluate for Other OSA
Symptoms? Sleep Study
Sleep Disorders

Adapted from Epstein et al. J Clin Sleep Med 2009;5(3):263-276

2024
Copyright © Harvard Medical School, 2018. All Rights Reserved.

Routine Health Maintenance


Evaluations
Screening questions about OSA
Is the patient obese?
Is the patient retrognathic?
Does the patient complain of daytime
sleepiness?
Does the patient snore?
Does the patient have hypertension?
A positive response to any of these questions
should trigger a comprehensive sleep evaluation
Epstein et al. J Clin Sleep Med 2009;5(3):263-276

Diagnosis: History
Snoring (loud, chronic)
Nocturnal gasping and choking
Ask bed partner (witnessed apneas)
Automobile or work related accidents
Personality changes or cognitive problems
Risk factors
Excessive daytime sleepiness
Sleep Apnea: Is Your Patient at Risk? NIH Publication, No 95-3803

2025
Copyright © Harvard Medical School, 2018. All Rights Reserved.

Diagnosis: Assessing Daytime


Sleepiness
Often unrecognized by patient
Ask family members
Must ask specific questions
Fatigue vs. sleepiness
Auto crashes or near misses
Sleep in inappropriate settings
- Work

- Social situations

Diagnosis: Physical Examination


Upper body obesity / thick neck
> 17” males
> 16” females
Hypertension
Obvious airway abnormality

2026
Copyright © Harvard Medical School, 2018. All Rights Reserved.

Physical Examination

http://www.ispub.com/ispub/ija/volume_13_number_1_fig2.jpg
Guilleminault C et al. Sleep Apnea
Syndromes. New York: Alan R. Liss, 1978

Case History: MVA victim


History
“Passed out” while driving
Doesn’t remember what happened in car but
was sleepy on drive prior to accident
Falls asleep frequently at work & movies
Wife won’t sleep in same room due to snoring
Exam
BP 160/95, 5’10”, 270 lbs, BMI = 38.7,
18”neck circumference
Crowded oropharynx
What should be done next?

2027
Copyright © Harvard Medical School, 2018. All Rights Reserved.

MVA Victim: Next step?


a. Electrophysiologic study
b. Sleep deprived EEG
c. Overnight sleep study
d. 24 hour Holter monitor
e. Brain MRI

Answer: Overnight Sleep Study


Most likely etiology
Documents presence and severity of OSA
Signs and symptoms poorly predict
disease severity
Appropriate therapy dependent on
severity
Other causes of daytime sleepiness

2028
Copyright © Harvard Medical School, 2018. All Rights Reserved.

Testing Options
In-laboratory full night polysomnography
Split night studies
Home diagnostic systems
Oximetry to full polysomnography

In-Lab Polysomnogram

2029
Copyright © Harvard Medical School, 2018. All Rights Reserved.

Home-based Test

Redline S et al. Chest 1991;100

Home-Based,
Limited Channel Tests
Advantages
Don’t require stay in sleep laboratory
Accurate for moderate to severe OSA
May cost less
Disadvantages
Reliability unknown in patients with other
medical comorbidities
Can’t detect other sleep disorders
Higher failure rate
Don’t know who takes test with most devices

2030
Copyright © Harvard Medical School, 2018. All Rights Reserved.

Therapeutic Approach
Behavioral
Medical
Surgical

Behavioral Interventions
Encourage patients to:
Lose weight
Avoid alcohol and sedatives
Avoid sleep deprivation
Avoid supine sleep position
Stop smoking

2031
Copyright © Harvard Medical School, 2018. All Rights Reserved.

Weight Loss and Sleep Apnea


6
5
4
3
2
1
0
-1
-2
-3
-4
-20 to -10 to -5% to +5 to +10%
Adapted from <-10% <-5% <+5 +10% to +20
Peppard PE et al.
JAMA 2000;284. Change in Body Weight

Medical Interventions
Positive airway pressure
Continuous (CPAP)
Bilevel PAP
Automatic titrating PAP (AutoPAP)
Oral appliances
Expiratory resistance valves
Other (limited role)
Medications
Oxygen

2032
Copyright © Harvard Medical School, 2018. All Rights Reserved.

Positive Airway Pressure

CPAP Benefits: Blood Pressure


Effect of Therapuetic vs. Subtherapeutic
CPAP on BP

Becker et al., Circulation 2003;107:68-73

2033
Copyright © Harvard Medical School, 2018. All Rights Reserved.

Benefits of CPAP: Mortality


Observational Trial of Long-term Cardiovascular Outcomes from OSA

Marin et al Lancet 2005; 365: 1046–53

Benefits of CPAP: Performance


35

30

25

20

15

10

0
Before CPAP After CPAP No Apnea
(n=6) (n=6) (n=12)
Adapted from Findley L et al. Clin Chest Med 1992;13.

2034
Copyright © Harvard Medical School, 2018. All Rights Reserved.

CPAP Compliance
Patient report: 75%
Objectively measured use
> 4 hrs for > 5 nights/week: 46%
Maintenance programs improve compliance
Intensive compliance programs: 65-80%

CPAP Compliance
CMS and most payers require demonstration of
compliance for reimbursement
By 3 months patient must
Show objective evidence of compliance
> 4 hrs/night for 70% of nights
Show subjective improvement
Eval by MD between 31-90 days from start
Ongoing compliance required for supplies
Compliance monitoring and management needs to
be part of any OSA treatment program

2035
Copyright © Harvard Medical School, 2018. All Rights Reserved.

Oral Appliances
Indications
Snoring and apnea (not severe)
Efficacy
Variable
Side effects
TMJ discomfort, dental misalignment,
and salivation

Oral Appliance: Mechanics

2036
Copyright © Harvard Medical School, 2018. All Rights Reserved.

Expiratory Resistance Valves


Mild-moderate OSA
Variable efficacy

Surgical Alternatives
Bypass upper airway
Tracheostomy

Reconstruct upper airway

2037
Copyright © Harvard Medical School, 2018. All Rights Reserved.

Surgical Alternatives
Reconstruct upper airway
Nasal operation
Tonsillectomy
Uvulopalatopharyngoplasty (UPPP)
Laser-assisted
uvulopalatopharyngoplasty (LAUP)
Radiofrequency tissue volume reduction
Palatal implants
Genioglossal advancement
Maxillomandibular advancement

Uvulopalatopharyngoplasty
(UPPP)

2038
Copyright © Harvard Medical School, 2018. All Rights Reserved.

Staged Surgical Procedures

Electrical Nerve Stimulation

Strollo PJ et al.
N Engl J Med
2014;370:139-49.

2039
Copyright © Harvard Medical School, 2018. All Rights Reserved.

Electrical Nerve Stimulation


Prospective, multicenter,
single group cohort study
N = 126 with moderate
– severe OSA
Results
66% responders
29% with AHI < 5
Symptom improvement
Adverse events
21% Serious
57% procedure-related
67% device-related
Role ? (BMI <32, AHI 20-50) Strollo PJ et al. N Engl J Med 2014;370:139-49.

Case: Persistent Sleepiness


47 yo male diagnosed with moderate OSA
and treated with CPAP
After 3 months on CPAP c/o continued
daytime sleepiness
Reports sleeping 8 hrs/night
No change in weight
Still sleepy while driving
What should you do next?

2040
Copyright © Harvard Medical School, 2018. All Rights Reserved.

Case: Persistent Sleepiness


Send for surgical evaluation
Send for oral appliance
Send for bariatric surgery
Check CPAP compliance
Start on modafinil (Provigil)

Answer: Check CPAP Compliance


Reasons for lack of improvement
Noncompliance – most common
Poor sleep habits
Alcohol and sedative use
Depression
Weight gain
Nonapneic sleep disorder
Persistent or recurrent symptoms
Consider referral to sleep specialist

2041
Copyright © Harvard Medical School, 2018. All Rights Reserved.

PAP Compliance Monitoring


Monitor objective adherence – Tracking Systems

Sleep Apnea
Dangerous

Common

Easily recognized

Treatable

2042
Copyright © Harvard Medical School, 2018. All Rights Reserved.

Sleep Apnea

Disclosure
Consultant
American Academy of Sleep Medicine
CareCore National
AIM Specialty Health

2043
Copyright © Harvard Medical School, 2018. All Rights Reserved.

References
Epstein L et al. Clinical guideline for the evaluation,
management and long-term care of obstructive sleep apnea in
adults. J Clin Sleep Med 2009;5:263-76.
Collop et al. Clinical guideline for the use of unattended
portable monitors in the diagnosis of obstructive sleep apnea in
adult patients. J Clin Sleep Med 2007;3:737-47.
White DP. Pathogenesis of obstructive and central sleep apnea.
Am J Respir Crit Care Med. 2005;172:1363-70.
Young T et al. The occurrence of sleep-disordered breathing
among middle-aged adults. N Engl J Med 1993;328:1230–35
Marin et al Long-term cardiovascular outcomes in men with
obstructive sleep apnoea-hypopnoea with or without treatment
with continuous positive airway pressure: an observational study.
Lancet 2005; 365: 1046–53

2044
Copyright © Harvard Medical School, 2018. All Rights Reserved.

Asthma
Elliot Israel, M.D.
Director of Clinical Research
Pulmonary & Critical Care Division
Division of Allergy and Immunology
Department of Medicine
Brigham and Women’s Hospital
Professor of Medicine
Harvard Medical School

I disclose the following relationships in the past year:

• AstraZeneca Consultant
• Entrinsic Health Solutions Consultant
• Genentech Clinical Research Grant
• GlaxoSmithKline Consultant
• Merck Consultant
• Novartis Consultant & Clinical
Research Grant
• 4D Pharma Consultant
• Pneuma Respiratory Consultant
• Regeneron Pharmaceuticals Consultant
• Sanofi Consultant & Clinical
Research Grant
• Vorso Corp Consultant

2045
Copyright © Harvard Medical School, 2018. All Rights Reserved.

Asthma

• Effect of Asthma
• Pathobiology and classification of severity
• Therapy including new rx and controversies
• Subcategories eg Exercise, Aspirin-induced
• Difficult to control asthma
• Asthma Exacerbations

GOALS
• Understand who needs controller therapy or
intensification of therapy
• Understand how to move up therapy
• Recognize when referral is necessary and
understand differential and aggravating
factors
• Understand high risk asthma characteristics

2046
Copyright © Harvard Medical School, 2018. All Rights Reserved.

Definition of Asthma
Chronic inflammatory disorder of the airways
Characterized by:
— Airflow limitation,
• reversible either spontaneously or with treatment
— Airway inflammation
— Increased responsiveness to a variety of stimuli

One Year in the USA with


Asthma
• Over 30 million people had a diagnosis of asthma
• 20 million people said they currently had asthma
• 60% of them had an asthma attack in a year
• In one year almost 2 million had to be cared for an
emergency room
• Half a million required hospitalization
• Almost 4,000 died from their asthma
• The 2015 the estimated economic cost was 82 billion
dollars

2047
Copyright © Harvard Medical School, 2018. All Rights Reserved.

Asthma

• Reversible airway obstruction


• Airway hyperreactivity
• Airway inflammation

Inflammatory Changes in
Chronic Asthma
• Mucus secretion
• Inflammatory cell
infiltration
• Edema INFLAMED
• Smooth muscle
constriction &
NORMAL
hypertrophy
• BM thickening and
subepithelial
collagen

2048
Copyright © Harvard Medical School, 2018. All Rights Reserved.

Inflammation in Asthma
Desquamation

BM

Eosinophils

Normal Asthma

Diagnosis
• Compatible history of wheezing,
shortness of breath, or cough
– Reversible airway obstruction
• FEV1 improved at least 12% post-bronchdilator
OR
– Airway reactivity – methacholine, mannitol,
exercise, hypertonic saline
• Exclusion of other causes of symptoms

2049
Copyright © Harvard Medical School, 2018. All Rights Reserved.

Goal of Asthma Therapy:


Achieve Control
Reduce Impairment
• Prevent chronic and troublesome symptoms
• Require infrequent use of inhaled SABA (≤2 days/week)
• Maintain (near) “normal” pulmonary function
• Maintain normal activity levels
• Meet patients’ expectations of, and satisfaction with, asthma care

Reduce Risk
• Prevent recurrent exacerbations
• Minimize need for emergency department visits or hospitalizations
• Prevent progressive loss of lung function
• Provide optimal pharmacotherapy, with minimal or no adverse effects

NHLBI. National Asthma Education and Prevention Program. Expert Panel Report 3. Available at:
http://www.nhlbi.nih.gov/guidelines/index.htm. Accessed 2.8.07.

Classifying Asthma Severity & Initiating


Treatment in Youths >12 Years & Adults
Classification of Asthma Severity
(Youths >12 years of age & adults)
Components of Severity
Persistent
Intermittent
Mild Moderate Severe
>2 days/week
Symptoms <2 days/week Daily Throughout the day
but not daily
Often
Nighttime awakenings <2/month 3-4/month >1/week but not nightly
7/week
Short-acting beta2-agonist
>2 days/week
use for symptom control <2 days/week Daily Several times per day
but not >1/day
(not prevention of EIB)
Impairment Interference with
None Minor limitation Some limitation Extremely limited
normal activity
• Normal FEV1
between • FEV1 <60%
exacerbations • FEV1 <80% pred • FEV1 >60-80% but <80%
predicted
Lung function • FEV1/FVC pred
• FEV1 >80% • FEV1/FVC reduced
predicted normal • FEV1/FVC reduced 5%
>5%
• FEV1/FVC normal

Exacerbations 0-2/year >2 in 1 year


Risk (consider frequency and Frequency and severity may fluctuate over time for patients in severity category
severity) Relative annual risk of exacerbations may be related to FEV1

Step 1 Step 2 Step 3 Step 4 or 5

Recommended Step for Initiating and consider short course of


systemic oral corticosteroids
Therapy
In 2-6 weeks, evaluate level of asthma control that is achieved, and adjust therapy accordingly.
NHLBI. National Asthma Education and Prevention Program. Expert Panel Report 3. Available at:
http://www.nhlbi.nih.gov/guidelines/index.htm. Accessed 2.8.07.

2050
Copyright © Harvard Medical School, 2018. All Rights Reserved.

Rule of 2’s for Severity and Control

• Mild Persistent or Lack of Control


— Nighttime awakenings >2/mo
— SABA use for sxs (not pre-exercise) >2/wk
— Sx >2 wk
— ACT / ACQ <20 / >1.5
— Lung function Reduced by >20%
— Exacerbations >2/yr
• Moderate
— Daily or weekly as appropriate
— FEV1 >40% reduction

Stepwise Approach for Managing Asthma


in Patients > 12 Years of Age
Persistent Asthma: Daily Medication
Intermittent
Consult with asthma specialist if step 4 care or higher is required.
Asthma
Consider consultation at step 3.

STEP 6
STEP 5 PREFERRED •Step up if
STEP 4 needed
PREFERRED (first, check
PREFERRED High-dose ICS
STEP 3 adherence,
+ LABA + oral environmental
Medium-dose High-dose ICS
ICS + LABA corticosteroid control and
PREFERRED + LABA
STEP 2 comorbid
Medium-dose AND conditions)
ALTERNATIVE AND
ICS
PREFERRED OR
STEP 1 Low-dose ICS Low-dose ICS Medium-dose Consider ASSESS
+ LABA ICS + either Consider CONTROL
Omalizumab
PREFERRED ALTERNATIVE LTRA, Omalizumab
ALTENATIVE
Theophylline for patients
for patients •Step down
Low-dose ICS or Zileuton who have
who have
SABA PRN Cromolyn, + either LTRA,
allergies
allergies if possible
Nedocromil, Theophylline or
LTRA or Zileuton (and asthma is
Theophylline well-controlled
at least 3
months)
Patient Education and Environmental Control at Each Step
• Quick-Relief Medication for All Patients:
• SABA as needed for symptoms. Intensity of treatment depends on severity of symptoms: up to 3 treatments at
20-minute intervals as needed. Short course of systemic oral corticosteroids may be needed.
• Caution: Increasing of β-agonist or use >2x/week for symptoms control indicates inadequate control and the
need to step up treatment.

NHLBI. National Asthma Education and Prevention Program. Expert Panel Report 3: page 517. Available at:
http://www.nhlbi.nih.gov/guidelines/index.htm. Accessed 2.8.07.

2051
Copyright © Harvard Medical School, 2018. All Rights Reserved.

Asthma Control Test™ (ACT)


1. In the past 4 weeks, how much of the time did your asthma keep you Score
from getting as much done at work, school or at home?

2. During the past 4 weeks, how often have you had shortness
of breath?

3. During the past 4 weeks, how often did your asthma symptoms
(wheezing, coughing, shortness of breath, chest tightness or pain) wake
you up at night, or earlier than usual in the morning?

4. During the past 4 weeks, how often have you used your rescue
inhaler or nebulizer medication (such as albuterol)?

5. How would you rate your asthma control during the past
4 weeks?

Asthma Control Test is a trademark of QualityMetric Incorporated.


Copyright 2002, QualityMetric Incorporated. Patient Total Score

Control on ACT or ACQ

• ACT
— 20 or more
• ACQ
— <1.0
— A 0.5 change is felt to be enough to make IS1
a
change in therapy
• Therefore 1.5 is inadequately controlled

2052
Copyright © Harvard Medical School, 2018. All Rights Reserved.

Slide 19

IS1 Partners Information Systems, 4/24/2011

2053
Copyright © Harvard Medical School, 2018. All Rights Reserved.

MEDICATIONS

Short-Acting Beta2-Agonists

Most effective medication for relief of acute


bronchospasm
More than 2 times a week (other than pre-exercise)
suggests inadequate asthma control
Regularly scheduled use is not generally
recommended
May lower effectiveness
May increase airway hyperresponsiveness
May be particulary problematic for those who
are Arg/Arg at the 16th amino-acid position

2054
Copyright © Harvard Medical School, 2018. All Rights Reserved.

Albuterol-HFA

MDI inhalers contain propellants,


traditionally CFCs.
Environmentally-friendly propellants (HFAs)
have replaced CFCs.
For albuterol, these are:
ProAir-HFA, Proventil-HFA, and Ventolin-HFA (as
of yet, no generic albuterol-HFA)

Three Things to Know about


Albuterol-HFA Inhalers

1. They are equally effective as albuterol-


CFC.
2. They have a different feel; a less
forceful, less cold “plume” of
medication.
3. The plastic holder needs to be cleaned
weekly to prevent plugging of
medication.

2055
Copyright © Harvard Medical School, 2018. All Rights Reserved.

Long-Acting Beta2-Agonists

Not a substitute for anti-inflammatory therapy


Not appropriate for monotherapy
Not for acute symptoms or exacerbations
May be beneficial when added to inhaled corticosteroids to
allow lower ICS dose
BLACK BOX WARNING based on data that suggest that their
use (particularly without ICS, but possibly even with them) can
increase the risk of severe asthma exacerbations and deaths
especially in Blacks.
Due to the above, the FDA has suggested that patients
controlled on combination therapy undergo an attempt to have
their LABA eliminated after they achieve control

ICS

• Most effective controller medications


• Use with a spacer reduces local side-effects such as
thrush and dsyphonia
• Long-term high doses are associated with small
increases in
— Osteoporosis -Glaucoma
— Cataracts -Dermal thinning esp elderly
• Prn use with symptoms has been shown to be as
effective as regular use in patients with mild-
persistent asthma in adults and children
• Prn use in combination with LABA, when combined
with regular bid use, appears to produce good
symptom control but is NOT APPROVED IN THE
USA and actively discouraged by the FDA.

2056
Copyright © Harvard Medical School, 2018. All Rights Reserved.

Exacerbation Rates
(symptoms triggering course of inhaled or oral cs)

Boushey & ACRN, NEJM, 2005

GENERAL RULE

Hi DOSE TREATMENT

4 PUFFS A DAY OF HIGHEST


FORMULATION

2057
Copyright © Harvard Medical School, 2018. All Rights Reserved.

ICS/LABA
Currently Approved

• salmeterol/fluticasone (Advair)

• formoterol/budesonide (Symbicort)

• formoterol/mometasone (Dulera)

• formoterol/fluticasone (AirDuo)

• Formoterol/fluticasone (Generic)

Super long-acting beta-agonist


combinations
• Fluticasone furoate 100/vilanterol 25
• Fluticasone furoate 200/vilanterol 25
— Combined long-acting ICS and super-long
acting (LA)BA.
• Only approved in 18 yo and above
• Dose equivalency
— 1 puff 100/25 qd = 1 puff bid FP250/Salm 50
BID
— 1 puff 200/25 qd = 1 puff bid FP500/Salm50
BID
• Not yet approved in asthma
ICS/LAMA/LABA once a day for COPD

2058
Copyright © Harvard Medical School, 2018. All Rights Reserved.

Leukotriene Modifiers

Indications
- Aspirin-exacerbated respiratory disease
- Exercise-induced asthma
Equal or better long term protection than long acting
beta-agonists
Alternative in guidelines to additional ICS or
LABA
Add-on therapy in severe asthma
• Caveats
— Watch for CSS with steroid tapers

Tiotropium as Add on Therapy

Peters & ACRN, NEJM, 2010

2059
Copyright © Harvard Medical School, 2018. All Rights Reserved.

Anti-IgE

• For poor control on high dose ICS/LABA or


equivalent Step 5 therapy
• Qualifications – IgE 30 to 700 and a positive
skin test or RAST to perennial allergen
• Efficacy – reduces exacerbations by ¼ to ½
— FEV1 increases 4%
— Not all patients respond
• Toxicity – rare anaphylaxis
— Questionable increases in cancers

Type 2 & Non-Type 2


Inflammation

Israel & Reddel, NEJM, 2017

2060
Copyright © Harvard Medical School, 2018. All Rights Reserved.

Anti-IL5 Antibody Therapy


• Block IL5 which is responsible for the
development and migration of eosinophils
• Mepolizumab and reslizumab attach to the
IL5 cytokine and block its ability to stimulate
the IL5 receptor
• Benralizumab attaches to the IL5 receptor
and blocks signaling but also is cytotoxic to
cells bearing the IL5 receptor

Anti-IL5
(Mepolizumab, Reslizumab,
Benralizumab)
• Reduce eosinophils
• Reduce exacerbations by >50% in patients
with >2 exacerbations/year and h/o blood
eosinophils
• Mepolizumab and reslizumab have been
shown to reduce OCS
• Variable and less effect on FEV1 and
symptoms
• Use in patients with persistent exacerbations
despite compliance with high dose
ICS/LABA and blood eosinophils >300

2061
Copyright © Harvard Medical School, 2018. All Rights Reserved.

Modeling Suggests that Mepo’s Greatest


Effect is on Patients with 3 or more
Exacerbations or Very High Eosinophils

Pavord, Lancet, 2012

ADMINISTRATION
• Mepolizumab
– Age 12 or older
– 100 mg SC q 4 weeks
– Solution must be used within 8 hours
– Package insert does not assert improvement in
FEV1
• Reslizumab
– Age 18 and above
– 3 mg/kg IV infusion over 20-45 min
– Package insert warns about anaphylaxis (0.3%)
– Labeling consistent with improvement in FEV1
• Benralizumab
– 30 mg SC Q4 weeks for first 3 doses and then q8
weeks

2062
Copyright © Harvard Medical School, 2018. All Rights Reserved.

Bronchial Thermoplasty

— Reduces symptoms and reduces


asthma exacerbations counted after
bronchoscopy
— Significant up front morbidity
— Severe asthma task force
recommends it only be performed in
the context of a clinical trial or registry

Use of Exhaled Nitric Oxide

• Level of >30 ppb is consistent with


persistent eosinophilic inflammation
• Markedly reduced by use of ICS
• Persistently high FeNO despite therapy is
c/w non-compliance or pathobiology
resistant to therapy
• May be a good predictor of response to
therapy for patients considered for biologic
aimed at Type 2 process (Anti-
IgE/IL5/IL4/IL13

2063
Copyright © Harvard Medical School, 2018. All Rights Reserved.

Therapies Not Yet Approved


• Anti-IL4/13 (Dupilumab)
— Improves lung function
— Reduces exacerbations
— Reduces OCS dose
— Very effective to treat atopic dermatitis
— Eosinophils may not be a requirement but may
be exacerbations and reversibility despite
compliance with high dose ICS/LABA

Patients with Greater T2 Markers


Had Reduced Asthma
Exacerbations w/ Dupilumab

•Castro et al , NEJM, 2018

2064
Copyright © Harvard Medical School, 2018. All Rights Reserved.

Recommendations Regarding Allergen


Immunotherapy

Consider allergen immunotherapy


for asthma patients when:
Clear relationship exists between symptoms
and unavoidable exposure
Symptoms are seasonal
Strong rhinitis component

Stepwise Approach for Managing Asthma


in Patients > 12 Years of Age
Persistent Asthma: Daily Medication
Intermittent
Consult with asthma specialist if step 4 care or higher is required.
Asthma
Consider consultation at step 3.

STEP 6
STEP 5 PREFERRED •Step up if
STEP 4 needed
PREFERRED (first, check
PREFERRED High-dose ICS
STEP 3 adherence,
+ LABA + oral environmental
Medium-dose High-dose ICS
ICS + LABA corticosteroid control and
PREFERRED + LABA
STEP 2 comorbid
Medium-dose AND conditions)
ALTERNATIVE AND
ICS
PREFERRED OR
STEP 1 Low-dose ICS Low-dose ICS Medium-dose Consider ASSESS
+ LABA ICS + either Consider CONTROL
Omalizumab
PREFERRED ALTERNATIVE LTRA, Omalizumab
ALTENATIVE
Theophylline or Anti-IL5 in
Low-dose ICS or Zileuton
or Anti-IL5 in appropriate •Step down
SABA PRN Cromolyn, + either LTRA, appropriate patients if possible
Nedocromil, Theophylline or patients
LTRA or Zileuton (and asthma is
Theophylline well-controlled
at least 3
months)
Patient Education and Environmental Control at Each Step
• Quick-Relief Medication for All Patients:
• SABA as needed for symptoms. Intensity of treatment depends on severity of symptoms: up to 3 treatments at
20-minute intervals as needed. Short course of systemic oral corticosteroids may be needed.
• Caution: Increasing of β-agonist or use >2x/week for symptoms control indicates inadequate control and the
need to step up treatment.

NHLBI. National Asthma Education and Prevention Program. Expert Panel Report 3: page 517. Available at:
http://www.nhlbi.nih.gov/guidelines/index.htm. Accessed 2.8.07.

2065
Copyright © Harvard Medical School, 2018. All Rights Reserved.

Specific Sub-Types

Managing Exercise-Induced
Bronchospasm (EIB)

Management Strategies
— Short-acting inhaled beta2-agonists used shortly
before exercise last 2 to 3 hours
— Salmeterol may prevent EIB for 10 to 12 hours
• 50% or greater tachyphylaxis occurs if used regularly
— LT modifiers provide long acting inhibition
— Cromolyn and nedcromil are also acceptable
— A lengthy warmup period before exercise may
preclude medications for patients who can
tolerate it
— Long-term-control therapy, if appropriate

2066
Copyright © Harvard Medical School, 2018. All Rights Reserved.

Aspirin-Exacerbated Respiratory
Disease (AERD)

• Characterized by leukotriene overproduction


in response drugs that block cox-1
–Appears to be related to downregulation of
PGE2 which “brakes” leukotriene production
• Most common in patients with rhinosinusitis
–-Usually occurs in adulthood
• Prevalence 5-25% with increased prevalence
in those with more severe asthma
0498

AERD (Rx)
• Use LT modifiers as part of treatment regimen
since they affect leukotriene mediated effects
• Treat rhinosinusitis aggressively
• Specific Cox-2 inhibitors are generally safe in
moderate doses
• Acetaminophen safe in most patients
although some may have reactions to high
doses
• Aspirin desensitization if poor control
• Anti-IL5 may work
0498

2067
Copyright © Harvard Medical School, 2018. All Rights Reserved.

Asthma-COPD Overlap (ACO)


• Evolving terminology for patients who
have a h/o asthma and also smoked
(some would also define it as asthma
patients with limited reversibility)
• May respond better to anti-cholinergics
• May not be able to restore their lung
function

Neutrophilic or Non-Type 2
Asthma
• More than half of asthma patients have
asthma that involves inflammation
mediated by Type 2 cytokines (IL4,5,
and 13) ≽ IgE/Eosinophils
• A significant minority may have
neutrophilic or paucigranulocytic
inflammation
– May be less responsive to steroids

2068
Copyright © Harvard Medical School, 2018. All Rights Reserved.

Examining Factors
Contributing to Asthma Severity

– Poor adherence/technique
– Rhinitis/sinusitis
– Gastroesophageal reflux (only in
those that have symptomatic reflux)
– Drugs (NSAIDs, beta-blockers)
– Environmental allergens
– Occupational exposures
– Sulfite sensitivity

Differential Dx of Persistent
Wheezing Poorly Responsive to
Therapy
• VCD
– Flattening of the FV loop
– Chinking of the vocal cords on fiberoptic exam
– Frequently in depressed patients
– Almost always in setting of some degree of
airway hyperresponsiveness
• Central airway obstruction
– Mass
– Collapsible airways

2069
Copyright © Harvard Medical School, 2018. All Rights Reserved.

Differential Dx of Persistent
Wheezing Poorly Responsive to
Therapy

• Aspirated foreign body


• Constrictive bronchiolitis
– Rheumatoid Arthritis, Ulcerative Colitis
• Bronchiectasis
• CHF

EXACERBATIONS

2070
Copyright © Harvard Medical School, 2018. All Rights Reserved.

Treatment
• Increased beta-agonists up to every 20
minutes
• 40-60 mg of prednisone for 5-10 days

Emergency Dept Referral

• Beta-agonist effects not lasting more


than an hour
• Unable to complete sentences
• Lower threshold for high-risk patients

2071
Copyright © Harvard Medical School, 2018. All Rights Reserved.

Definition of “High-Risk”

• Newly diagnosed asthma


• On daily prednisone prior to admission
• >2 E.D. visits in last 6 months
• >1 prior hosp’ns in last 12 months
• Ever intubated for asthma
• Severe psychosocial problems
• Drug addiction
• Lower socio-economic status

Antibiotics for Acute,


Severe Asthma
• Purulent-appearing sputum often
contains eosinophils rather than
neutrophils
• During attacks, transtracheal aspirates
reveal no greater prevalence of bacterial
pathogens than among controls without
respiratory disease
• Empiric antibiotics of no proven benefit
in hospitalized patients with acute
attacks

2072
Copyright © Harvard Medical School, 2018. All Rights Reserved.

Achieving Asthma Control

The Five-Point Plan:


A. Making the correct diagnosis
B. Modifying environmental inciters
C. Medications to control asthma
D. Plan for dealing with asthmatic attacks
E. Specialist consultation

Indications for Specialist


Consultation

• Uncertainty regarding diagnosis;


• Failure to achieve good asthma control;
• Frequent need for systemic steroids;
• Frequent ED visits or hospitalizations;
• Unacceptable medication side-effects.

2073
Copyright © Harvard Medical School, 2018. All Rights Reserved.

Points to Remember
• Rules of two’s for initiation of controller and for step up
• Escalation pattern
– (2) ICS (or LTRA) → (3) ↑ ICS or ICS/LABA → (4) Mod Dose
ICS/LABA → (5) Hi Dose ICS/LABA
• Referral if require Hi Dose ICS/LABA for control or preventing
exacerbation
– Aggravating factors
– Differential Diagnosis
• EIB
– LTRA or intermittent salmeterol
– Warm-up
– Treat underlying severity
• AERD – LTRA and referral if poor control
• IgE >30 or Eos >300 may be candidates for biologics especially
with 2 or more exacerbations per year
• High risk patients

Question #1
Which of the following is NOT indicative of
poor asthma control?

A. As needed reliever medication 3x/wk


B. Reliever medication use pre-exercise
4x/week
C. Night-time awakening 3x/mo
D. Three exacerbations/yr

2074
Copyright © Harvard Medical School, 2018. All Rights Reserved.

Answer Question #1
• B – use of bronchodilators pre-exercise
is not considered a index of poor control

• All the others exceed the thresholds for


control which in each case is no more
than 2 times

Question #2
Which of the following is NOT required for
consideration for anti-IgE therapy?

A. IgE > 30
B. Allergic rhinitis
C. A positive skin test or RAST
D. Poor response to level 5 therapy

2075
Copyright © Harvard Medical School, 2018. All Rights Reserved.

Answer Question #2
• B – patients do not need to have allergic
rhinitis

• They do need to have an IgE>30, a


positive skin test or RAST to a perennial
aeroallergen
• In order to consider this therapy they
should have failed high dose ICS and a
LABA

Severe Asthma Program

State of the Art Multidisciplinary


Evaluation and Treatment of
Patients with Severe Asthma
•Pulmonary •GI
•Allergy •Psychiatry
•ENT •Alternative Medicine

2076
Copyright © Harvard Medical School, 2018. All Rights Reserved.

Short List of References


• -National Institutes of Health. Expert Panel Report 3
(EPR3): Guidelines for the diagnosis and
management of asthma. Bethesda, Maryland:
National Institutes of Health National Heart Lung and
Blood Institute. Publication Number 08-4051.
Accessed: May 2010, 2007 July. Report No.:
Publication Number 08-4051 Contract No.:
Publication Number 08-4051.
• -Kerstjens HA, Engel M, Dahl R, et al. Tiotropium in
asthma poorly controlled with standard combination
therapy. N Engl J Med 2012;367:1198-207.
• -Pavord ID, Korn S, Howarth P, et al. Mepolizumab
for severe eosinophilic asthma (DREAM): a
multicentre, double-blind, placebo-controlled trial.
The Lancet 2012;380:651-9.

I disclose the following relationships in the past year:

• AstraZeneca Consultant
• Entrinsic Health Solutions Consultant
• Genentech Clinical Research Grant
• GlaxoSmithKline Consultant
• Merck Consultant
• Novartis Consultant & Clinical
Research Grant
• 4D Pharma Consultant
• Pneuma Respiratory Consultant
• Regeneron Pharmaceuticals Consultant
• Sanofi Consultant & Clinical
Research Grant
• Vorso Corp Consultant

2077
Copyright © Harvard Medical School, 2018. All Rights Reserved.

06/20/17 additions
• 4 puffs a day high dose
• Air duo and generic slide 30
• FeNO update on >20. Slide 37
• Fixed type on LABA on IL4/13 slide 38
• Correction to anti-ige #2 perennial allergen
• Fixed up points to remember
• Fixed FeNO
• Inserted Type 2
• Inserted ACO
• Added il5 to naepp diagram
• Updated cost

06/18 Addition

• T2 and non –T2 diagram


• Dupilumab forest plot
• 2105 costs

2078
Copyright © Harvard Medical School, 2018. All Rights Reserved.

Pleural Effusions: a case-


based review
Scott Schissel, MD, PhD
Chief, Department of Medicine
Brigham and Women’s Faulkner Hospital

Division of Pulmonary and Critical Care Medicine


Brigham and Women’s Hospital
Assistant Professor of Medicine
Harvard Medical School

Disclosure

• None

2079
Copyright © Harvard Medical School, 2018. All Rights Reserved.

Outline
• Normal pleural anatomy and function
• Mechanisms of pleural fluid accumulation
– Transudates vs. Exudates
• Evaluating pleural effusions
– Imaging
– Thoracentesis
– Pleural fluid analysis
• Diagnosis and management of common exudative
effusions
• Evaluating the exudative effusion of unknown
etiology

Normal Pleural Anatomy


Parietal Pleura
Visceral Pleura • Basal pleural fluid
volume: ~2.0 ml/hr
Pleural Space – Daily production:
~25 ml/day

• Drainage capacity:
~15 ml/hour
– Daily absorption
~350 ml/day

2080
Copyright © Harvard Medical School, 2018. All Rights Reserved.

First Case
• 67 year old man with
dyspnea and cough x 2
months
• Exam: poor chest excursion
on the R with absent BS
• US of the R chest: small
pleural effusion
• Pleural Fluid Analysis:
– PF protein 3.9 (serum 7.9)
– LDH 200 (serum 250)
– Glucose 98
– pH 7.48
– GS, Cultures AFB smear NEG
– Cytology NEG

First Case

The next step in managing this patient is:


A. Perform a large volume thoracentesis
B. Place an indwelling pleural catheter
C. Thoracoscopy with pleural biopsy
D. Perform a bronchoscopy
E. B and D

2081
Copyright © Harvard Medical School, 2018. All Rights Reserved.

Pleural Effusions from “trapped lung”

Pleural
effusion

Mass

7
7

Pleural Fluid Origins: Trapped Lung

Parietal Visceral
Pleural
Space
NL
Phydrostatic Serous

Fluid
Ppleura=
-25 cmH20
NL Phydrostatic

2082
Copyright © Harvard Medical School, 2018. All Rights Reserved.

Entrapped v. Trapped Lung

Causes of TRANSUDATIVE Pleural


Effusions
• Congestive Heart Failure (40%)
• Cirrhosis
• Nephrotic Syndrome
• Trapped Lung (also can be exudative)
• Pulmonary Embolism (also can be exudative)
• Myxedema
• Urinothorax
• CSF leak

10

2083
Copyright © Harvard Medical School, 2018. All Rights Reserved.

Causes of EXUDATIVE Pleural Effusions

• Parapneumonic / • Uremia
empyema (25%) • Post-cardiac injury /
• Malignancy (12%) surgery
• PE (10%) • Asbestos
• Tuberculosis • Chylothorax
• Pancreatitis • Intra-abdominal
• RA, SLE Abscess
• Meig’s Syndrome

11

A Quick Interesting Case…


• 57 year old woman with a h/o HTN, HFpEF, HLD and
nephrolithiasis present with dyspnea x 3 weeks
• CXR revealed a large RIGHT pleural effusion only
• Pleural Fluid Analysis:
– PF protein 2.0 (serum 7.9)
– LDH 90 (serum 250)
– Glucose 92
– pH 6.9
– GS, Cultures AFB smear NEG
– Cytology NEG

The cause of this effusion is…?


A URINOTHORAX, secondary to renal
fornix rupture

2084
Copyright © Harvard Medical School, 2018. All Rights Reserved.

Diagnostic Evaluation of Pleural Effusions


• Identifying the etiology of a pleural effusion
requires:
• CLINICAL information
– To suggest an underlying diagnosis
• Radiographic findings
– Infiltrate, mass, lymphadenopathy
• Pleural fluid analysis
– Transudate v. Exudate ->
• Cell count and differential, cytology, culture, etc….

13

Pleural Effusions: When to Tap


Pleural effusion

Substantial fluid?
(>10 mm thick on lateral decubitus CXR)

No Yes

Observation CHF?

No Yes

Thoracentesis Asymmetry, chest pain, fever?

Yes -> No
Thoracentesis

Diuresis,
Observation

Effusion >
3 days ->
Thoracentesis

2085
Copyright © Harvard Medical School, 2018. All Rights Reserved.

The Value of the Lateral Film


AP film Lateral film

Effusion!
Up to 500 ml can be “hidden” on an AP film

15

Value of the Lateral Decubitus CXR


AP film Lat Decubitus film

Thickness of effusion <10 mm likely too small to tap

16

2086
Copyright © Harvard Medical School, 2018. All Rights Reserved.

Imaging Pleural Effusions


Chest Ultrasound

Effusion

Lung

Gastric air
17

Large Volume Thoracentesis:


OK to remove > 1 liter ??
185 pts with 1L -> 3.5L removed
• 1 pt (0.5%) had symptomatic re-expansion
pulmonary edema (RPE) [1.4L removed]
• 4 pts (2.2%) had radiographic RPE only
• RPE did not correlate with pleural fluid volume, end-
expiratory pleural pressure, or symptoms during the
procedure
• No clear guidelines, but RPE is rare and strict
adherence to limiting thoracenteses to 1L is not
supported by data
Feller-Kopman et al 2007 Ann Thoracic Surg:

18

2087
Copyright © Harvard Medical School, 2018. All Rights Reserved.

Transudate or Exudate?:
Light’s criteria
• Distinguish transudate from exudate

• One or more of the following defines an


exudate:
– Pleural fluid (PF) protein : Serum protein>0.5
– PF LDH : Serum LDH>0.6
– PF LDH > 2/3 upper limit normal serum LDH

19

Transudate or Exudate?
• If you truly suspect a transudate (e.g. a
“diuresed” CHF-related effusion), check…

– Serum - PF protein >3.1 gm/dL


– Serum– PF albumin gradient > 1.2 gm/dL

– Then = TRANSUDATE

*Light,
20 RW. Clin Chest Med 2013; 34: 21-26

2088
Copyright © Harvard Medical School, 2018. All Rights Reserved.

Transudate or Exudate*?
Test Sensitivity* Specificity*
PF:serum protein>0.5 98% 83%

PF:serum LDH>0.6 86% 84%

PF LDH>2/3 nl serum 90% 82%

Serum-PF alb < 1.2 87% 92%

21

Next Case!
• 37 year old woman with 2 months of dry
cough and dyspnea on exertion with mild R
anterior pleuritic chest pain

• Past Medical History


– Migraines
– HTN
– Anemia
– Metromenorragia
– Ectopic thyroid, hypothyroidism

22

2089
Copyright © Harvard Medical School, 2018. All Rights Reserved.

Chest X-ray

23

Pleural Fluid Analysis


Pleural Fluid:
LDH: 459
pH: 7.37
glucose: 72
WBC: 900 Diff: 27N 28M 40L 5 mesothelial cells

RBC: too numerous to count


Fluid hematocrit: 22%

24

2090
Copyright © Harvard Medical School, 2018. All Rights Reserved.

Grossly Bloody Pleural Fluid


Fluid Hematocrit Cause
<1% Not significant

1-20% Cancer>>
(hemorrhagic process) PE>Trauma>empyema
20 – 50% Hemorrhagic process
v. Hemothorax
>50% circulating HCT Hemothorax

25

Spontaneous Bloody Effusions and Hemothorax


BLOODY Pleural effusion
No Trauma
No Procedures

Pleural Fluid HCT < 50% Pleural Fluid HCT > 50%
of peripheral blood HCT of peripheral blood HCT
(frank hemothorax)

Etiologies CT Angiogram
Lung Ca or MRA
PE - pulmonary infarction
Tuberculosis

Hemorrhagic Empyema Vascular NON-vascular


Uremia Anomalies
Coagulopathy
Mesothelioma

AVM's Endometriosis Chest wall


Neurofibromatosis Pleural Metastases Bony anomaly
Aneurysms Angiosarcomas "Exostoses"
26
(intercostal, IMA) Thymoma / Thymic cysts

2091
Copyright © Harvard Medical School, 2018. All Rights Reserved.

Spontaneous Bloody Effusions and Hemothorax


BLOODY Pleural effusion
No Trauma
No Procedures

Pleural Fluid HCT < 50% Pleural Fluid HCT > 50%
of peripheral blood HCT of peripheral blood HCT
(frank hemothorax)

Etiologies CT Angiogram
Lung Ca or MRA
PE - pulmonary infarction
Tuberculosis

Hemorrhagic Empyema Vascular NON-vascular


Uremia Anomalies
Coagulopathy
Mesothelioma

AVM's Endometriosis Chest wall


Neurofibromatosis Pleural Metastases Bony anomaly
Aneurysms Angiosarcomas "Exostoses"
(intercostal, IMA) Thymoma / Thymic cysts

Spontaneous Bloody Effusions and Hemothorax


BLOODY Pleural effusion
No Trauma
No Procedures

Pleural Fluid HCT < 50% Pleural Fluid HCT > 50%
of peripheral blood HCT of peripheral blood HCT
(frank hemothorax)

Etiologies CT Angiogram
Lung Ca or MRA
PE - pulmonary infarction
Tuberculosis

Hemorrhagic Empyema Vascular NON-vascular


Uremia Anomalies
Coagulopathy
Mesothelioma

AVM's Endometriosis Chest wall


Neurofibromatosis Pleural Metastases Bony anomaly
Aneurysms Angiosarcomas "Exostoses"
(intercostal, IMA) Thymoma / Thymic cysts

2092
Copyright © Harvard Medical School, 2018. All Rights Reserved.

Back to the case…

• R VATS with pleural


biopsy and wedge
resection
– hemothorax > 1 L
– Endometrial tissue
– lung

29

Thoracic Endometriosis Syndrome (TES):


Clinical Manifestations
• Major manifestations of TES -
– Pneumothorax ~ 80%
– Hemothorax / hemopneumothorax ~ 14%
– Hemoptysis ~ 7%
– Lung nodules ~ 6%

• Other rare manifestations of TES


– Isolated, catamenial chest pain
– Catamenial pneumomediastinum
– Pulmonary vascular invasion
– Pleural-based mass

• Interesting aside –
– Pelvic endometriosis is NOT found in up to 15 – 30% (depending on case
series) of TES patients

30

2093
Copyright © Harvard Medical School, 2018. All Rights Reserved.

TES: treatment and outcome


• Medical therapy – goals are to suppress growth and
maintenance of the endometrium
– Oral contraceptives
– Danazol
– Progestational agents
– GnRH analogs
• Most successful for long-term suppression of catamenial pneumothorax
and hemoptysis
– Medical therapy ALONE often leads to recurrent thoracic
disease
• Catamenial pneumonthorax recurs in 60% of patients on hormonal
therapy at 12 months
• Pleuroscopic surgery / pleurodesis, beneficial adjunct treatment

31

Another Case!

• 72 year old man with 2 days of


– Intense R sided chest pain, dry
cough, chills and dyspnea
• Exam
– T 100.9, Decreased BS right base
• Labs
– Serum WBC 15K (80 poly’s, 10%
bands)
• Lateral Decubitus CXR revealed
a flowing moderate-sized
effusion
• Blood cultures were obtained
and Antibiotics started

32

2094
Copyright © Harvard Medical School, 2018. All Rights Reserved.

Quick Quiz
The most likely infection in this
• Thoracentesis -> case is:
• Pleural Fluid Analysis
– Sero-sanguinous A. S. pneumoniae
– PF protein 3 (serum 4) B. S. milleri
– PF LDH 800 (serum 300) C. H. influenza
– PF pH 7.18 D. S. aureus
– Gram stain and cultures NEG E. A or C

33

Microbiology of COMMUNITY-Acquired
Pleural Infections

S. Pneumoniae Strep Milleri


13% 32%

Staphylococci
11%
Other 18%

Anaerobes H. Flu 3%
16%

Enterobacter
7%

34

2095
Copyright © Harvard Medical School, 2018. All Rights Reserved.

Quick Quiz
The next step in managing this
• Thoracentesis -> effusion is:
• Pleural Fluid Analysis A. Antibiotics and close
– Sero-sanguinous observation, including daily
– PF protein 3 (serum 4) CXRs
– PF LDH 800 (serum 300) B. Thoracentesis to drain the
– PF pH 7.18 pleural space
– Gram stain and cultures NEG C. VATS decortication
D. Chest tube drainage +/-
fibrinolytics to the pleural
space
E. A or D

35

COMPLICATIONS of Parapneumonic Effusions:


why drain?
• Chronic Pleural Infection
• Secondary Lung Abscess
• Bronchopleural Fistula
• Empyema Necessitans
– Pleuro-cutaneous fistula

• Pleural Fibrosis
– Lung entrapment
– Impaired lung function
– Surgical decortication
36

2096
Copyright © Harvard Medical School, 2018. All Rights Reserved.

Who to Drain ?
Pleural Fluid Fluid pH Risk of Drain?
Anatomy Micro Poor
outcome
<10 mm on N/A N/A LOW No
Lat decub
CXR
<½ GS and pH > 7.20 LOW No, BUT
hemithorax Cx NEG need to
AND -> AND -> follow
>½ GS or pH < 7.20 Moderate / YES
hemithorax, Cx + High
loculated, Or ->
thick pleura
Or ->

37

Treating Complex Parapneumonic Effusions / Empyema

• Definitive Pleural Drainage


– Commonly via chest tube
– Serial thoracenteses an alternative, but not well-studied

• Pleural Catheter + Fibrinolytics


– Variably used and STILL requires individual assessment
– Multicenter Intrapleural Sepsis Trial (MIST-1)
• Pleural saline v. streptokinase
• Streptokinase did NOT improve survival, need for surgical intervention,
Chest CT appearance, or lung function
– N Engl J Med 2011 365: 516
• Additional fibrinolytic trial (s)….

38

2097
Copyright © Harvard Medical School, 2018. All Rights Reserved.

Small Drainage Catheters + Fibrinolytic + DNA’se


Treatment of Empyema
• N Engl J Med 2011 365: 516
– Pleural saline v. tPA v. DNAse v. tPA + DNAse x 3
days
– tPA + DNAse group had decreased need for:
• Surgical intervention (4% compared to 39% for placebo); and…
• An improved CXR by day 7 and 30

Important considerations with intra – pleural tPA


~1.8% risk of pleural hemorrhage
Markedly simulates pleural fluid production (up to 6-fold)
Contraindicated in presence of broncho-pleural fistula

39

Treating Complex Parapneumonic Effusions / Empyema


• Surgical Decortication
– Indicated if little clinical or radiographic improvement after 1
week of antibiotics and chest tube drainage +/- pleural lytics
– Required in ~30% of cases
– VATS adequate in 60% of these cases

• Appropriate Antibiotics
– Duration uncertain: 2 weeks minimum, but as long as
necessary for drains to be removed

– Sometimes longer courses required for atypical pathogens


(e.g. actinomyces) or in cases of prolonged pleural drainage

40

2098
Copyright © Harvard Medical School, 2018. All Rights Reserved.

A Surprise Case…
• 53 year old man with 2 months of acute on chronic
exertional dyspnea
• 3+ ankle edema
• Orthopnea & paroxysmal nocturnal dyspnea, worse
than baseline

41

Past Medical History

– HFpEF: LV EF 55% – CKD: stage III


– Atrial Fibrillation – Diabetes (type II)
– HTN
– Dyslipidemia
– Peripheral Vascular
Disease

– Obesity: BMI 50

42

2099
Copyright © Harvard Medical School, 2018. All Rights Reserved.

Case CXR

43

Removed: 1200 cc
Thoracentesis
WBC 1650 (P14 L83 M2)
pH 7.57
Glucose 262
Total Protein 3.9
Albumin 2.3
LDH 159

Gram stain/culture negative


Cytology negative

Cholesterol 77
Triglycerides 1,276
44

2100
Copyright © Harvard Medical School, 2018. All Rights Reserved.

Chylothorax?
Pleural Fluid Triglyceride level (mg/dl)

>110 50-110 <50


Chylothorax Lipoprotein PSEUDO-
analysis Chylothorax
(likely chronic exudate)

+ Chylomicrons ->
CHYLOTHORAX

Traumatic Non Traumatic


Chest trauma Lymphoma, solid tumors, chest XRT
Thoracic surgery Histoplasmosis, MTB, Sarcoid
Chylous ascites
LAM, yellow nail syndrome, Amyloid
L subclavian DVT
Filariasis

45

Pseudo - Chylothorax
• Pseudo - Chylothorax: Cholesterol, phospholipid Complexes = from:
Cell degradation, chronic exudate, empyema

Cholesterol > 250 mg/dL


+ cholesterol crystals (rhomboid)
TGfluid LOW

• Chylothorax:
TG fluid > 110mg/dL
TG fluid > TG serum
Cholesterol fluid < 200 mg/dL
+ chylomicrons

46

2101
Copyright © Harvard Medical School, 2018. All Rights Reserved.

Location of Duct Disruption


Determines Side of Chylothorax

47

Chyle Loss Increases Mortality

Fat
Malnutrition
Vitamins 17-35% mortality
A, D, E, K (limited data)

Proteins 4.5-fold ↑ risk


Immunosuppression of death in
Immunoglobulins
surgical patients
Lymphocytes

Paul et al. J Thorac Cardiovasc Surg. 1985 Feb;89(2):221-7.


48 Shah et al. Ann Thorac Surg. 2012 Mar;93(3):897-903.

2102
Copyright © Harvard Medical School, 2018. All Rights Reserved.

Management
Remove Chyle
↓ Chyle Flow Thoracentesis
No short- or long-chain TG intake Tube Thoracostomy
Octreotide or Somatostatin Pleuroperitoneal Shunt
Pleurovenous Shunt

Close Chyle Leak


Maintain Nutrition Pleurodesis
Duct Embolization
Medium-chain TGs Surgical Duct Ligation
enter portal circulation directly
Total parental nutrition (TPN)

49

Exudative Effusion of Unclear Etiology


• 46 year old man with a h/o myelodysplastic
syndrome, s/p an unrelated allo-stem cell transplant:
complicated by cutaneous and GI graft-versus host
disease

• NOW with dyspnea and progressive right-sided


pleuritic chest pain x 2-3 weeks

• No fevers/chills, known sick contacts, or recent travel


• Medications: prednisone 20 mg daily, tacrolimus,
atovaquone, valganciclovir
50

2103
Copyright © Harvard Medical School, 2018. All Rights Reserved.

Chest X ray

51

Thoracentesis
• Labs:
– Serum: LDH 229, Total Protein 6.5, Albumin 3.7
– Fluid: LDH 226, Total Protein 4.1, Albumin 2.7
– Fluid: pH 7.6, Glucose 208, Amylase 12
– Fluid: ADA 3.1 (usually > 40 U/L in tuberculous pleural
effusions)

• Cultures: AFB, Fungal, Aerobic/Anaerobic,


Actinomyces, Nocardia, PCP all negative
• Cytology + flow cytometry negative for malignancy

52

2104
Copyright © Harvard Medical School, 2018. All Rights Reserved.

Exudative Effusion of Unclear Etiology


• Up to 20% of exudative pleural effusions have no clear
etiology, even after:
– Pleural fluid analysis from thoracentesis and
– Thoracoscopy and pleural biopsy
• Most undiagnosed exudates are from:
– Malignancy (including mesothelioma)
– Chronic empyema (including atypical organisms)
– Tuberculosis
– Rheumatoid Arthritis / inflammatory
– Pulmonary Embolus
– “Diuresed” CHF

53

Exudative Pleural Effusion

"Borderline" Exudate All Other Exudates

Serum:PF Albumin > 1.2 g/dL NEGATIVE


Pleural Fluid NT-proBNP > 2000 pg/mL History
Consider CHF, nephrosis, cirrhosis Microbiology
Cytology

Consider PE (DDIMER, PE-CT)


Consider TB
ADA > 40 U/L or
IFN-gamma > 140 pg/mL

NEGATIVE

Thoracoscopy and
Pleural Biopsy

54

2105
Copyright © Harvard Medical School, 2018. All Rights Reserved.

Pleural Fluid Biomarkers:


new diagnostic tools for idiopathic exudates

Porcel JM. Clin Chest Med 2013; 34: 27–37

55

Back to the case… evaluation for Infection


• Blood cultures negative
• 1,3 Beta D-glucan <31
• Galactomannan 0.15 (negative)
• S pneumo and Legionella Urine Ag negative
• Histoplasma and Blastomyces Urine Ag negative
• Cryptococcal Ag negative
• CMV PCR (blood) negative
• PCP (sputum) negative

56

2106
Copyright © Harvard Medical School, 2018. All Rights Reserved.

Bronchoscopy, Pleuroscopy and


Pleural Biopsy

Methenamine Silver Stain (MSS) 100x Modified AFB Stain 100x

57

Our Case:
after 6 months of antibiotic therapy

58

2107
Copyright © Harvard Medical School, 2018. All Rights Reserved.

Quick FINAL Case!

• 55 yo man with subacute


dyspnea
– History: EtOH
– Exam: Decreased BS R chest,
+ Ascites
– Pleural fluid analysis:
• PF protein 1.0 (serum 3.0)
• PF LDH 100 (serum 300)

• What is the etiology of


the patient’s R-sided
pleural effusion?

Quick FINAL Case: Answer


• Hepatic Hydrothorax
– Occurs in ~7% of pts with ascites
– Usually large R effusion (80% on R)
– Forms due to rifts in the diaphragm
– Can accumulate rapidly
– 20% of HH can form withOUT ascites
– Tap pleural + peritoneal fluid (to r/o infection
– “SBP” - and alleviate symptoms)
– Treat underlying ascites and cirrhosis -> often
difficult…
– TIPS can be affective for refractory HH

2108
Copyright © Harvard Medical School, 2018. All Rights Reserved.

Hepatic Hydrothorax: follow-up


• Spironolactone,
furosemide, IV Albumin,
and octreotide minimally
effective –

• TIPS performed and


discharged on diuretic
regimen

• CXR 3 months later!

Further Reading
1. Light, RW. Pleural Effusion. N Engl J Med 2002; 346:
1971
2. Light, RW. The Light Criteria The Beginning and
Why they are Useful 40 Years Later. Clin Chest Med
2013; 34: 21-26
3. Heffner, JE. Discriminating Between Transudates
and Exudates. Clin Chest Med 2006; 27: 241
4. McGrath EE and Anderson PB. Diagnosis of Pleural
Effusion: A systematic approach. Am J Critical Care
2011; 20: 119
5. Porcel Jose M. Pearls and myths in pleural fluid
analysis. Respirology 2011; 16: 44

62

2109
Copyright © Harvard Medical School, 2018. All Rights Reserved.

Thanks!

63

2110
Copyright © Harvard Medical School, 2018. All Rights Reserved.

Chest X-ray Refresher

Christopher H. Fanta, M.D.

Pulmonary and Critical Care Division


Brigham and Women’s Hospital
Partners Asthma Center
Harvard Medical School

Financial Conflicts of Interest

None.

2111
Copyright © Harvard Medical School, 2018. All Rights Reserved.

Chest X-ray Refresher:


Three Themes
I. Chronic interstitial pneumonitis
II. Obstructive lung diseases
III. Pulmonary infiltrates in the
immunocompromised host

Note: the order of the chest X-rays does not


match the order of the case histories.

LINEAR

NODULAR

LINEAR
AND
NODULAR

HONEY-
COMBING

2112
Copyright © Harvard Medical School, 2018. All Rights Reserved.

2113
Copyright © Harvard Medical School, 2018. All Rights Reserved.

2114
Copyright © Harvard Medical School, 2018. All Rights Reserved.

I. Chronic Interstitial Lung


Disease

The patient presents to you with


progressive dyspnea on exertion of
more than 6 weeks' duration and a
non-productive cough. On chest
examination, you auscultate bilateral
posterior inspiratory crackles. On
further questioning, you obtain the
additional history of:

I. Chronic Interstitial Lung


Disease – Case Histories

A. Work cleaning and insulating boilers


for 30 years.
B. Work as a stone-cutter in a quarry.
for 20 years, now retired for 10 years.
C. Prior episodes of erythema nodosum
and uveitis.
D. Inoperable gastric cancer.

2115
Copyright © Harvard Medical School, 2018. All Rights Reserved.

C. Sarcoidosis

2116
Copyright © Harvard Medical School, 2018. All Rights Reserved.

2117
Copyright © Harvard Medical School, 2018. All Rights Reserved.

Bilateral Hilar Adenopathy:


Differential Dx

• Sarcoidosis
• Berylliosis
• Lymphoma
• Granulomatous disease (e.g., TB)
• Metastatic cancer

Sarcoidosis:
Radiographic stages
• Stage 1: Hilar/mediastinal adenopathy
alone
• Stage 2: Hilar/mediastinal adenopathy
with pulmonary infiltrates
• Stage 3: Pulmonary infiltrates without
hilar/mediastinal adenopathy

2118
Copyright © Harvard Medical School, 2018. All Rights Reserved.

Sarcoidosis: Radiographic
Features

• Myriad radiographic patterns:


linear, tree-in-bud nodularity,
larger nodules, miliary, etc.
• Often upper lobe predominant
• Active disease is PET-positive

2119
Copyright © Harvard Medical School, 2018. All Rights Reserved.

2120
Copyright © Harvard Medical School, 2018. All Rights Reserved.

2121
Copyright © Harvard Medical School, 2018. All Rights Reserved.

A. Asbestosis

A. Asbestosis

2122
Copyright © Harvard Medical School, 2018. All Rights Reserved.

A. Asbestosis

2123
Copyright © Harvard Medical School, 2018. All Rights Reserved.

B. Silicosis with progressive


massive fibrosis

2124
Copyright © Harvard Medical School, 2018. All Rights Reserved.

2125
Copyright © Harvard Medical School, 2018. All Rights Reserved.

2126
Copyright © Harvard Medical School, 2018. All Rights Reserved.

2127
Copyright © Harvard Medical School, 2018. All Rights Reserved.

D. Lymphangitic carcinomatosis

D. Lymphangitic carcinomatosis

2128
Copyright © Harvard Medical School, 2018. All Rights Reserved.

Lymphangitic Carcinomatosis

Histologic type: almost always adeno-


carcinoma. Usual primary sites include:

• Breast
• Lung
• Stomach
• Pancreas
• Thyroid

Three Themes

I. Chronic interstitial pneumonitis


II. Obstructive lung diseases
III. Pulmonary infiltrates in the
immunocompromised host

2129
Copyright © Harvard Medical School, 2018. All Rights Reserved.

Common Obstructive Diseases

• Asthma
• Chronic bronchitis and emphysema
(COPD)
• Bronchiectasis
• Bronchiolitis
• Upper airway obstruction

Other Diseases Manifesting


with Airflow Obstruction

• Sarcoidosis
• Congestive heart failure
• Lymphangioleiomyomatosis (LAM)

2130
Copyright © Harvard Medical School, 2018. All Rights Reserved.

II. Obstructive Lung Diseases

This patient complains of long-


standing shortness of breath, especially
on exercise, and a daily cough. By
reviewing the patient's history, you
learn that:

II. Obstructive Lung Diseases


– Case Histories
A. The patient's father and older brother
died in their 40's of emphysema.
B. The patient has had a chronic
productive cough and recurrent
sinusitis since childhood and now
presents for evaluation of infertility.

2131
Copyright © Harvard Medical School, 2018. All Rights Reserved.

II. Obstructive Lung Diseases


– Case Histories (cont.)
C. The patient, a non-smoker, had
recurrent pneumothoraces in the
past and a pleural effusion that was
said to look "milky" when drained.
D. The patient complains of weight loss,
chronic diarrhea, and sinusitis.
Mucoid pseudomonas has been
grown from the sputum.

2132
Copyright © Harvard Medical School, 2018. All Rights Reserved.

A. Emphysema with alpha-1


antitrypsin deficiency

2133
Copyright © Harvard Medical School, 2018. All Rights Reserved.

2134
Copyright © Harvard Medical School, 2018. All Rights Reserved.

2135
Copyright © Harvard Medical School, 2018. All Rights Reserved.

C. Lymphangioleiomyomatosis

2136
Copyright © Harvard Medical School, 2018. All Rights Reserved.

B. Primary ciliary dyskinesia

Primary Ciliary Dyskinesia


(Immotile Cilia Syndrome)
(Kartagener’s Syndrome)

• Bronchiectasis
• Sinusitis
• Situs inversus (50%)
• Immotile sperm

- Ultrastructural and/or functional


abnormality of cilia

2137
Copyright © Harvard Medical School, 2018. All Rights Reserved.

Normal

Immotile cilia syndrome

Primary Ciliary Dyskinesia: Dx

• Electron microscopy (nasal or bronchial


mucosal biopsy); sperm analysis
• Low nasal nitric oxide level
• Ciliary immunofluorescence
• Genetic testing (“bronchiectasis panel”)

2138
Copyright © Harvard Medical School, 2018. All Rights Reserved.

D. Cystic fibrosis

2139
Copyright © Harvard Medical School, 2018. All Rights Reserved.

D. Cystic fibrosis

Molecular Basis of Cystic Fibrosis

• Deletion of 3 base pairs at codon 508


causes most cases (∆F 508 mutation).

• Gene product: CF transmembrane


conductance regulator (CFTR); functions
as chloride channel in apical cell membrane.

• Impaired chloride ion flux across


respiratory epithelium leads to viscous
airway mucus.

2140
Copyright © Harvard Medical School, 2018. All Rights Reserved.

Three Themes

I. Chronic interstitial pneumonitis


II. Obstructive lung diseases
III. Pulmonary infiltrates in the
immunocompromised host

III. Fever and Pulmonary Infiltrates


in Immunocompromised Hosts
-- Case Histories
A. This patient received a cadaveric renal
transplant two months ago and now
presents with rapidly progressive dyspnea,
tachypnea, and hypoxemia.
B. This patient with acute myelogenous
leukemia received combination chemo-
therapy, including adriamycin to a total
dose of 450 mg. He/she complains of
gradually worsening shortness of breath and
a non-productive cough.

2141
Copyright © Harvard Medical School, 2018. All Rights Reserved.

III. Fever and Pulmonary Infiltrates


in Immunocompromised Hosts
-- Case Histories
C. This patient with Hodgkin's Disease is
eight weeks status post radiation treatment
to a mantle port and to the spleen.
D. This neutropenic patient with acute
leukemia has received broad-spectrum
antibiotics for seven days because of fever
without identifiable source. Now he/she
develops cough, a new fever spike, and an
episode of hemoptysis.

Fever and Pulmonary Infiltrates:


Clinical and Radiographic
Considerations

• Diffuse vs. (multi-)focal infiltrates


• e.g., PCP/viral vs. gram-negative bacterial
• Nature of immunocompromise
• e.g., splenectomy vs. neutropenia
• Time of onset and tempo of illness
• e.g., timing after RTx or BMT;
• e.g., tempo of bacterial vs. mycobacterial
infection.

2142
Copyright © Harvard Medical School, 2018. All Rights Reserved.

Non-infectious Causes of Pulmonary


Infiltrates in Immunocompromised Hosts

• Radiation-induced pneumonitis and fibrosis


• Drug-induced pulmonary infiltrates
• Localized bleeding/diffuse alveolar
hemorrhage
• Pulmonary spread of malignancy
• Cryptogenic organizing pneumonia
(Bronchiolitis obliterans organizing
pneumonia)
• Non-specific interstitial inflammation/fibrosis

2143
Copyright © Harvard Medical School, 2018. All Rights Reserved.

D. Invasive aspergillosis

2144
Copyright © Harvard Medical School, 2018. All Rights Reserved.

C. Radiation pneumonitis

2145
Copyright © Harvard Medical School, 2018. All Rights Reserved.

C. Radiation pneumonitis

Radiation Pneumonitis:
Radiographic Features

• Onset 2-6 months following


completion of radiation therapy
• Defies normal anatomic boundaries
• Matches radiation fields in
distribution

2146
Copyright © Harvard Medical School, 2018. All Rights Reserved.

2147
Copyright © Harvard Medical School, 2018. All Rights Reserved.

B. Interstitial pulmonary edema

Pulmonary Venous Hypertension:


Radiographic Features

• Upper zone vessel dilatation


• Interstitial edema
• Septal (Kerley) lines
• Peribronchial/perivascular edema
• Hilar haze
• Mottling of lung fields
• Alveolar edema
• Pleural effusions/thickening of
fissures

2148
Copyright © Harvard Medical School, 2018. All Rights Reserved.

A. Pneumocystis pneumonia (?)

2149
Copyright © Harvard Medical School, 2018. All Rights Reserved.

Diffuse Ground-Glass Opacities

A. Pneumocystis pneumonia (?)

Diffuse Airspace (Alveolar


Filling) Opacities

• Pneumocystis pneumonia
• Cytomegalovirus (CMV) pneumonia
• Diffuse alveolar hemorrhage
• Others: e.g., pulmonary alveolar
proteinosis; drug-induced
pneumonitis

2150
Copyright © Harvard Medical School, 2018. All Rights Reserved.

Mystery Film: 48 year-old


cigarette smoker with dyspnea.

Question #1:

Which of the following blood tests is most


likely to be helpful in assessment of this
patient?
A. Cystic fibrosis gene mutation
B. ANCA
C. Alpha-1 antitrypsin level
D. Angiotensin converting enzyme
E. Hypersensitivity pneumonitis
antibody panel

2151
Copyright © Harvard Medical School, 2018. All Rights Reserved.

Question #1:

Which of the following blood tests is most


likely to be helpful in assessment of this
patient?
A. Cystic fibrosis gene mutation
B. ANCA
C. Alpha-1 antitrypsin level
D. Angiotensin converting enzyme
E. Hypersensitivity pneumonitis
antibody panel

Question #2:

This patient has cough with thick, green


sputum on a daily basis. The
abnormality at the left lower lobe is most
likely?
A. Bronchoalveolar carcinoma
B. Bronchiectasis
C. Left lower lobe collapse
D. Aspergilloma
E. Loculated pleural effusion

2152
Copyright © Harvard Medical School, 2018. All Rights Reserved.

Mystery Film with Close-up

Question #2:

This patient has cough with thick, green


sputum on a daily basis. The
abnormality at the left lower lobe is most
likely?
A. Bronchoalveolar carcinoma
B. Bronchiectasis
C. Left lower lobe collapse
D. Aspergilloma
E. Loculated pleural effusion

2153
Copyright © Harvard Medical School, 2018. All Rights Reserved.

References
• Goodman LR. Felson’s Principles of Chest
Roentgenology: A Programmed Text (3rd ed.).
Philadelphia: Saunders Elsevier, 2007.
• McLoud TC, Boisell PM. Thoracic Radiology:
The Requisites (2nd ed.). Philadelphia: Mosby
Elsevier, 2010.
• Fraser RS, et al. Fraser and Pare’s Diagnosis of
Diseases of the Chest (4th ed. – 4 volume set).
Philadelphia: Saunders,1999.

Financial Conflicts of Interest

None.

2154
Copyright © Harvard Medical School, 2018. All Rights Reserved.

Pulmonary Function Testing


Scott Schissel, MD, PhD
Chief, Department of Medicine
Brigham and Women’s Faulkner Hospital

Division of Pulmonary and Critical Care Medicine


Brigham and Women’s Hospital

Assistant Professor of Medicine


Harvard Medical School

NO FINANCIAL DISCLOSURES
Scott L. Schissel, M.D.,PhD

2155
Copyright © Harvard Medical School, 2018. All Rights Reserved.

The “Standard” PFTs


• Spirometry
– Measures air FLOW -> assess for airway
obstruction
– Good screening test for RESTRICTION
• Lung Volume Measurement
– ONLY test that proves RESTRICTION in lung
function – can also assess for gas trapping
• Gas Exchange = DLco
– Indirect assessment of parenchymal lung disease,
pulmonary vascular disease, and / or anemia
• Respiratory Muscle Strength Testing
– Maximum inspiratory and expiratory pressures =
“MIP” and “MEP”
3

Spirometry: the MANEUVER


“deep breath in”

inspiration

Tidal Breathing

expiration

“keep blowing out”

2156
Copyright © Harvard Medical School, 2018. All Rights Reserved.

Spirometry: what we measure


TLC

IRV Inspiratory Reserve Volume

FVC
VT Tidal Volume (VT)
Expiratory Reserve Volume (ERV)
ERV

RV Residual Volume

Spirometry: exhaled volume over time

NORMAL OBSTRUCTION
Volume (liters)

Volume (liters)

FVC
FEV1 FVC

FEV0.5

FEV1

Time (seconds) Time (seconds)

2157
Copyright © Harvard Medical School, 2018. All Rights Reserved.

Spirometry: continuous
FLOW versus VOLUME

Exhale

Inhale

Spirometry: the numbers should


be known before looking at them!

Exhale
1
Inhale

2158
Copyright © Harvard Medical School, 2018. All Rights Reserved.

Spirometry Example 1: what’s going on ??

Spirometry Example 1: what’s going on ??

Severe Obstruction
(FEV1 / FVC < 0.7)

What’s “normal”:
>80% of the mean or > 95% CI

2159
Copyright © Harvard Medical School, 2018. All Rights Reserved.

PFT’s: an aside to define “normal”


• Use the 95% confidence interval (CI) to
determine Normal v Abnormal but….
• Still use % predicted (% of the mean) to SCORE
severity of obstruction or restriction
• Why ?

95% CI 95% CI

Grading OBSTRUCTION on Spirometry


• First – is there obstruction?
– FEV1 / FVC ratio <0.70 YES = obstruction present
• Examine FEV1 to grade severity
– 70 – 100% predicted = MILD
– 60 - 69% = MODERATE
– 50 - 59% = MODERATELY SEVERE
– 35 – 49% = SEVERE
– < 35% = VERY SEVERE
• Bronchodilator Response
– 12% and 200 mL change in FEV1 or FVC
– Hold short-acting and long-acting bronchodilators 4 hours
and 12 hours, respectively, prior to testing

2160
Copyright © Harvard Medical School, 2018. All Rights Reserved.

Back to cases…

13

Spirometry Example 2: what’s going on ??

2161
Copyright © Harvard Medical School, 2018. All Rights Reserved.

Spirometry Example 2: what’s going on ??

Fixed Large Airway Obstruction:


Peak flow << FEV1 = RED FLAG

Spirometry Example 2:
severe upper airway stenosis

Sub glottic trachea


1
6

2162
Copyright © Harvard Medical School, 2018. All Rights Reserved.

QUIZ on PFT Patterns of Large Airway


Obstruction
A B C

Variable Variable
FIXED EXTRA-thoracic INTRA-thoracic

Spirometry Example 3: what’s going on ?

Obstructed? Restricted? BOTH??

2163
Copyright © Harvard Medical School, 2018. All Rights Reserved.

In Severe Obstruction FVC is LOW Due to


Trapped Gas at End-Exhalation
TLC

IRV Inspiratory Reserve Volume

VT Tidal Volume (VT) FVC


ERV

Expiratory Reserve Volume (ERV)

RV Residual Volume

So, when the FVC is LOW……


• We JUST need to know whether the RV (or
FRC), the gas left on end-exhalation, is HIGH
or LOW to define RESTRICTION versus pure
OBSTRUCTION

• And, this is the exact principle and utility of


LUNG VOLUME TESTING

20

2164
Copyright © Harvard Medical School, 2018. All Rights Reserved.

Techniques to Measure Lung Volumes:


RV and FRC in Particular!
• Helium Dilution
– Cheaper technology
• Body Plethysmography
– Most accurate

• Nitrogen washout
–Rarely used

21

Body Plethysmography: “body box”


• Patient in closed chamber
• Plethysmograph is vented to
outside
• Shutter closed at end expiration =
FRC
• Patient then pants against closed
shutter / manometer ->
compressing FRC volume ->

• Based on Boyle’s Law:


• P1 * V1 = P2 * V2…..
• Volume of FRC can be calculated

2165
Copyright © Harvard Medical School, 2018. All Rights Reserved.

Spirometry: obstructed, restricted, both??

Lung Volumes example 1

Note the HIGH RV, FRC and LOW IC

2166
Copyright © Harvard Medical School, 2018. All Rights Reserved.

Restrictive Mechanisms
• Parenchymal lung disease / pulm fibrosis
– Increased lung elastance – sets FRC and RV
LOWER since the lung recoils the chest wall
inward
• Neuromuscular disease
– Since FRC is passive, set point between lung
and chest wall recoil relatively UNCHANGED
• Chest wall disease (skeletal, soft tissue =
slceroderma / Pleural fibrosis)
– DEPENDS, but FRC and RV usually LOW

25

Lung Volumes example 2

Severe Restriction: note relatively preserved FRC, RV.


Dx = ALS, diaphragm weakness

2167
Copyright © Harvard Medical School, 2018. All Rights Reserved.

DLco / Gas Exchange Principles


• Oxygen is CONSUMED, thus its diffusion is nearly
impossible to measure
• CO is great since it is freely diffusible, metabolically
inert, binds HgB ~250 x more avidly than O2, our
PaCO should be zero, and it is easy to measure

Vco = DLco (Palv CO – Pcap CO)


Vco = AD (P1- P2) 0 mmHg
T

[DLco]
DLco = Vco / Pa CO ml/min/mmHg

DLco Example 1

Note Obstruction with a LOW DLco and DL/VA = emphysema

2168
Copyright © Harvard Medical School, 2018. All Rights Reserved.

DLco Example 2
1

Note Restriction with a LOW DLco and DL/VA = ILD

Factors and Conditions that Affect the DLco


May DECREASE the DLco May INCREASE the DLco
Poor inspiratory effort Muller maneuver
(inhaled volume should >85% of best (inspire forcefully against a closed airway
FVC) / opposite of valsalva)
Poor CO equilibration during testing Supine position
(breath hold < 10 seconds)
Prior valsalva maneuvers Polycythemia

Elevated carboxyhemoglobin level Left to Right Shunt

Emphysema Alveolar hemorrhage

Interstitial lung disease Asthma

Pulmonary vascular disease Obesity

Lung resection

Increase FiO2
(use of supplemental oxygen)

2169
Copyright © Harvard Medical School, 2018. All Rights Reserved.

Pulmonary Function Tests in OBESITY


Change from normal
Parameter BMI Comments
FVC, TLC, and RV DECREASED Modest, linear decrease by ~0.5% / unit
increase in BMI
Significant, exponential decrease by 3 – 5%
FRC and ERV DECREASED / unit increase in BMI

DLco UNCHANGED to
INCREASED

Jones, RL et al. 2006. Chest, 2006. 130 (3): 827-33

Exhaled Breath Analysis: nitric oxide in asthma


52 yo F with 5 years of intermittent cough and chest
tightness; spirometry is normal

Labs revealed:
Peripheral eosinophils of 6%, absolute count 550

The best next step(s) include:


A. Start empiric albuterol and high dose inhaled fluticasone
B. Perform a methacholine challenge
C. No action since spirometry is normal
D. Measure exhaled nitric oxide level
E. B or D

Answer: E exhaled NO = 75 ppb

2170
Copyright © Harvard Medical School, 2018. All Rights Reserved.

Exhaled Nitric Oxide in Asthma


• An exhaled NO level > 50 ppb has good specificity for
eosinophilic airway inflammation and corticosteroid-
responsive symptoms
Exhaled Nitric Oxide Level

NON
asthmatics
Stable
ASTHMA

“Cut Point” for


corticosteroid
response

Exhaled Nitric Oxide: bottom-line guidelines


• Interpretation of exhaled nitric oxide levels in DIAGNOSIS of
eosinophilic airways disease:
– < 25 ppb NEGATIVE
– 25 – 50 ppb equivocal
– > 50 ppb POSITIVE
• Interpretation of significant CHANGE in nitric oxide levels
– Baseline NO level > 50 ppb = 20% change
– Baseline NO level < 50 ppb = 10 ppb change

• USE of exhaled NO levels to GUIDE asthma therapy very


uncertain / controversial:
– Likely effective as a marker of medication compliance in eosinophilic
asthma
– No clear advantage over other markers of asthma control
– In pregnancy, use of exhaled NO levels may decrease asthma
exacerbations and use of oral corticosteroids

2171
Copyright © Harvard Medical School, 2018. All Rights Reserved.

PFTs: pulling it all


together in 3 cases….

35

Case 1
• 69 yo F with a 40 pack year h/o Tobacco
use with 3 years of progressive DOE
• No cough, sputum, wheeze, or chest pain
• No significant co-morbidities
• Otherwise – had been quite active….

36

2172
Copyright © Harvard Medical School, 2018. All Rights Reserved.

Case 1: spirometry

Case 1: full PFTs


1

2173
Copyright © Harvard Medical School, 2018. All Rights Reserved.

Case 1 Question
• Which of the following processes are
LEAST likely present in this patient?

A. Anemia
B. Interstitial lung disease
C. Asthma
D. Pulmonary vascular disease

39

Case 1: ANSWER (C. Asthma LEAST Likely)

The isolated reduced DLco suggests anemia, pulmonary vascular disease, or


BALANCED obstruction and restriction (emphysema and IPF); asthma does not affect
the DLco

2174
Copyright © Harvard Medical School, 2018. All Rights Reserved.

Case 1: ANSWER
Homogeneous EMPHYSEMA Subpleural ILD

The interstitial fibrosis decreases the hyperinflation of emphysema -> “normalizing”


spirometry and lung volumes BUT BOTH processes DECREASE the DLco and impair
gas exchange!

Case 2
• 44 yo M with a history of “asthma”
presents with 2 years of progressive DOE,
cough and intermittent wheeze
• No significant co-morbidities
• No tobacco history

42

2175
Copyright © Harvard Medical School, 2018. All Rights Reserved.

Case 2: spirometry

Case 2: full PFTs

The FEV1 and FVC do NOT change after bronchodilator treatment

2176
Copyright © Harvard Medical School, 2018. All Rights Reserved.

Case 2 Question
• Which of the following diagnoses is most
consistent with these PFT data?

A. Emphysema
B. Idiopathic pulmonary arterial
hypertension
C. Asthma
D. Sarcoid
45

Case 2: ANSWER (D. Sarcoid)

COMBINED severe obstruction and restriction =


SARCOID

2177
Copyright © Harvard Medical School, 2018. All Rights Reserved.

Case 2: ANSWER
“Bronchocentric” Fibrosis

SARCOID -> airway and parenchymal fibrosis ->


obstruction AND restriction with a low DLco

Case 3
• 47 yo F presents with 2 years of episodic
shortness of breath and DOE
• No associated cough or wheeze
• No significant co-morbidities
• No tobacco use history
• Usually active = plays tennis, but has
DYPNEA with each episode

48

2178
Copyright © Harvard Medical School, 2018. All Rights Reserved.

Case 3: SPIROMETRY

Mean 95% CI Actual % Pred

FVC 3.01 2.20 2.77 92

FEV1 2.52 1.88 1.78 70

FEV1/FVC 0.74 0.70 0.65 --

PEFR 5.81 -- 5.24 90

Case 3 Question
• Which of the following statements is
TRUE?

A. The spirometry data are consistent with a


restrictive ventilatory defect
B. The normal peak expiratory flow rate (PEFR)
excludes a diagnosis of asthma
C. Repeat spirometry with and without a
bronchodilator is indicated, since PEFR can be
normal in asthma
D. The reduced FEV1 and FEV1/FVC but normal PEFR
is most suggestive of COPD

2179
Copyright © Harvard Medical School, 2018. All Rights Reserved.

Case 3: ANSWER (C. Repeat testing +/- a bronchodilator )


Pre bronchodilator Post bronchodilator
Mean 95% CI Actual % Pred Actual % Pred % Change
FVC 3.01 2.20 2.77 92 3.00 99 8
FEV1 2.52 1.88 1.78 70 2.28 90 28
FEV1/FVC 0.74 0.70 0.65 -- 0.76 -- --

PEFR 5.81 -- 5.24 90 5.82 100 11

Pre bronchodilator

Post bronchodilator
Flow (L/sec)

Volume (L)

Case 3: ANSWER
Appreciate that PEAK FLOW can be
NORMAL in ASTHMA

2180
Copyright © Harvard Medical School, 2018. All Rights Reserved.

Selected References
• Miller, MR et al. ATS/ERS Task Force: standardization of lung
function testing. Eur Respir J. 2005. 26: p153-161

• Pellegrino, R et al. ATS/ERS Task Force: interpretative strategies for


lung function tests. Eur Respir J. 2005. 26: p948-968

• MacIntyre, N et al. ATS/ERS Task Force: standardization of the


single-breath determination of carbon monoxide uptake in the lung.
Eur Respir J. 2005. 26: p720-735

• Jones, RL et al. The effects of body mass index on lung volumes.


Chest. 2006. 130: p827 – 833

• Essat, M et al. Fractional exhaled nitric oxide for the management of


asthma in adults: a systematic review. Eur Respir J. 2016. epub

53

2181
Copyright © Harvard Medical School, 2018. All Rights Reserved.

Take Home Messages in


Pulmonary Medicine
Christopher H. Fanta, M.D.
Pulmonary and Critical Care Division
Brigham and Women’s Hospital
Partners Asthma Center
Harvard Medical School

Financial Conflicts of Interest

None.

2182
Copyright © Harvard Medical School, 2018. All Rights Reserved.

Topics Covered This Morning


1. Hemoptysis
2. Bronchiectasis/bronchiolitis
3. Solitary Pulmonary Nodule
4. Interstitial Lung Disease
5. COPD
6. Sleep Apnea Syndrome
7. Asthma
8. Pleural Disease

“Brevity is the Soul of Wit”

The Reduced
Shakespeare
Company
All 37 plays in 97
minutes!

2183
Copyright © Harvard Medical School, 2018. All Rights Reserved.

1. Hemoptysis
P Evaluation should include a chest X-ray; further
work-up depends on the findings on the chest
X-ray.
P A normal chest X-ray does not exclude serious
pulmonary pathology (i.e., lung cancer found in
5-7% of high-risk patients).
P At risk: cigarette smokers; >age 40; persistent
hemoptysis for >1 week.
P Chest CT and bronchoscopy of complementary
value in hemoptysis with negative chest X-ray.

Massive Hemoptysis
P Definition: >600 ml/24 hr; lesser amounts can
also cause respiratory insufficiency.
P Think bronchiectasis, TB, cancer, mycetoma,
vasculitis.
P Conservative measures include oxygenation,
sometimes intubation, positioning with bleeding
side down.
P Management team: thoracic surgery;
interventional pulmonology; and interventional
radiology (bronchial artery embolization).
P Major complication of the latter: spinal infarct.

2184
Copyright © Harvard Medical School, 2018. All Rights Reserved.

2. Bronchiectasis

P Definition: irreversible focal/multifocal


dilation of bronchi, predisposing to
recurrent/persistent bronchial infection.
P Causes: necrotizing infection/inflammation.
P Multifocal bronchiectasis: think
hypogammaglobulinemia, cystic fibrosis,
primary ciliary dyskinesia. Many cases
remain idiopathic.

Bronchiectasis (cont.)

P Diagnosis: CT scan.
P Typical pathogens: staph, hemophilus,
pseudomonas.
P Treatment: bronchodilators, clearance of
secretions, and antibiotics, including the
option of inhaled antibiotics and macrolide.
P Choice of antibiotics guided by sputum
culture.

2185
Copyright © Harvard Medical School, 2018. All Rights Reserved.

Bronchiolitis obliterans
P Definition: chronic cellular inflammation of
bronchioles with characteristic intraluminal
polypoid tissue.
P Causes: viral infection; noxious gas
inhalation; rheumatoid arthritis; ulcerative
colitis; s/p transplant.
P Diagnosis: obstructive lung disease – not
asthma or COPD or bronchiectasis;
occasionally: tree-in-bud small nodules.

“Tree-in-bud” pattern

2186
Copyright © Harvard Medical School, 2018. All Rights Reserved.

Bronchiolitis obliterans (cont.)

P Treatment: bronchodilators,
steroids (inhaled and systemic),
immunosuppressants.
P Often poorly responsive to treatment.

3. Solitary Pulmonary Nodule

P Differential diagnosis: benign or malignant.


P Features of benign nodules: calcification;
lack of growth (over 2 years; exception:
ground-glass [sub-solid] nodules).
P Diagnostic test: PET scan for nodules >8
mm (negative scan indicates benign disease
with 95% certainty;
other 5%: bronchoalveolar Ca).

2187
Copyright © Harvard Medical School, 2018. All Rights Reserved.

Solitary Pulmonary Nodule (cont.)


P Other diagnostic tests: bronchoscopy and
transthoracic needle aspirate unreliable in
excluding malignancy in small peripheral nodules.
P For high-risk patients/larger nodules: surgical
excision.
P For low-risk patients/smaller nodules: serial chest
imaging over at least 2 years.
P Screening for lung cancers with chest CT scans:
20% reduction in lung cancer deaths; current and
former (<15 years) smokers (>30 pack-years)
screened annually from age 55–80 yrs.

4. Interstitial Lung Disease


(Dr. Hilary Goldberg)
P Consider chronic interstitial lung disease:
P Exertional dyspnea with non-productive
cough
P Inspiratory crackles
P Not heart failure
P Obtain additional history:
P environmental/occupational exposures
P collagen-vascular disease

2188
Copyright © Harvard Medical School, 2018. All Rights Reserved.

Interstitial Lung Disease (cont.)


P Confirm chronic interstitial lung disease:
P Reticulonodular infiltrates on CXR, CT scan
P Restriction on PFTs
P exceptions: some sarcoidosis, eosinophilic
granuloma, LAM
P Oxygen desaturation with exercise

Interstitial Lung Disease (cont.)


P Establish diagnosis of chronic interstitial lung
disease:
P Thoracoscopic lung biopsy (VATS)
P Bronchoscopy
P to Dx sarcoidosis, hypersensitivity pneumonitis,
or lymphangitic carcinomatosis
P to exclude infection, malignancy

2189
Copyright © Harvard Medical School, 2018. All Rights Reserved.

Idiopathic Pulmonary Fibrosis vs.


Sarcoidosis
IPF Sarcoidosis

Age of onset >Middle age 20-50 y.o.


Radiographic Bibasilar Lymphadenopathy;
appearance predominance upper lobe
predominance
CT appearance Characteristic Variable: ground-glass
subpleural opacities; peribronchial
honeycombing nodules

Idiopathic Pulmonary Fibrosis vs.


Sarcoidosis

Idiopathic
pulmonary fibrosis

2190
Copyright © Harvard Medical School, 2018. All Rights Reserved.

Idiopathic Pulmonary Fibrosis vs.


Sarcoidosis

Sarcoidosis

Idiopathic Pulmonary Fibrosis vs.


Sarcoidosis
IPF Sarcoidosis

Extra- None (except Myriad, including:


pulmonary clubbing) eyes, liver, skin,
involment calcium metabolism.
Pathology UIP: fibrosis Non-caseating
interspersed with granulomas
normal lung

2191
Copyright © Harvard Medical School, 2018. All Rights Reserved.

Idiopathic Pulmonary Fibrosis vs.


Sarcoidosis
IPF Sarcoidosis

Natural 50% mortality at 3 Progression in small


history years minority
Treatment Pirfenidone or Nothing needed;
nintedanib; prednisone;
supplemental oxygen; methotrexate or
lung transplantation infliximab (Remicade)

5. COPD (Dr. Craig Hersh)


P Definitions:
P Chronic bronchitis: daily cough and sputum
production for at least 3 months out of the
year for at least 2 consecutive years.
P Emphysema: abnormal dilation of airspaces
due to destruction of alveolar walls.
P COPD: some combination of chronic
bronchitis and emphysema, causing airflow
obstruction that is not fully reversible.

2192
Copyright © Harvard Medical School, 2018. All Rights Reserved.

Therapeutic Interventions:
Smoking Cessation

P Smoking cessation counseling


P Nicotine replacement therapy
P Patch, gum, lozenge, inhaler, nasal spray
P Bupropion (Wellbutrin, Zyban)
P Varenicline (Chantix) – a nicotinic acetylcholine
receptor antagonist (and partial agonist)
P (Financial incentives: NEJM 2018; May 23, 2018
DOI: 10.1056/NEJMsa1715757)

2193
Copyright © Harvard Medical School, 2018. All Rights Reserved.

Medical Therapies for COPD


P Bronchodilators (once- or twice-daily)
P Long-acting inhaled beta-agonists (LABA)
P Long-acting muscarinic antagonists (LAMA)

P Inhaled corticosteroids (ICS)


P Do not slow decline in lung function;
P Increase the risk of pneumonia
P Role: frequent exacerbations; eosinopilia

Medical Therapies for Severe COPD

P LABA/ICS (Advair, AirDuo, Breo, Dulera,


Sybicort)
P LABA/LAMA (Anoro, Bevespi, Stiolto, Utibron)
P LABA/LAMA/ICS (Trelegy)

2194
Copyright © Harvard Medical School, 2018. All Rights Reserved.

(My) Current Thinking


• Do not use ICS alone.
• LAMA or LABA/ICS similar as first choice.
• LABA/LAMA provides greater improvement
in lung function than LABA/ICS or LAMA alone.
• Triple therapy (ICS/LABA/LAMA) appropriate for
severe disease with frequent exacerbations,
eosinophilia.

Additional Therapies for COPD

P Supplemental oxygen for the hypoxemic patient


P Improves exercise capacity, reduces pulmonary
hypertension and secondary polycythemia, and
prolongs survival in COPD with resting SaO2 <88%.

P Long-Term Oxygen Treatment Trial (LOTT)


P No long-term benefit from chronic supplemental
oxygen in COPD with SaO2 89-93% and/or oxygen
desaturation (to SaO2 80-90%) with exercise.
(N Engl J Med 2016; 375:1617-27).

2195
Copyright © Harvard Medical School, 2018. All Rights Reserved.

Additional Therapies for COPD (cont.)

• Don’t forget about outpatient pulmonary


rehabilitation.
• Highly selected patients: lung volume reduction.
• Everyone: influenza and pneumococcal vaccines.

6. Sleep Apnea (Dr. Lawrence Epstein)

P Definitions:
P Apnea = cessation of airflow >10 seconds
P Obstructive apnea = apnea despite respiratory
effort
P Central apnea = apnea without respiratory effort
P Mixed apnea = central followed by obstructive
apnea
P Hypopnea = reduction in airflow by >30%
with arousal or O2 desaturation (>3%)

2196
Copyright © Harvard Medical School, 2018. All Rights Reserved.

Prevalence of Sleep Apnea Syndrome


Among working population aged 30-60 yrs:
Men: 4%
Women: 2%
Apnea-hypopnea index (AHI) = # of apneas
and hypopneas per hour of sleep.
AHI 5-15 = Mild OSA
AHI 15-30 = Moderate OSA
AHI >30 = Severe OSA

Consequences of Sleep Apnea


Sleep fragmentation +
recurrent hypoxemia/hypercapnia

Daytime Cardiovascular
hypersomnolence consequences
Decreased productivity Pulmonary HT
Automobile accidents Myocardial ischemia
Cardiac arrhythmias
Systemic HT
Cerebrovasc. disease

Increased morbidity and mortality

2197
Copyright © Harvard Medical School, 2018. All Rights Reserved.

When to Suspect Sleep Apnea


P Snoring (loud, chronic)
P Nocturnal gasping and choking
P Ask bed partner (witnessed apneas)
P Excessive daytime sleepiness
P Automobile- or work-related accidents
P Personality changes or cognitive problems
P Risk factors

Risk Factors for Sleep Apnea


P Obesity (neck size: >17” in men; > 16” in women)
P Increasing age
P Male gender; post-menopausal women
P Anatomic abnormalities of upper airway
P Family history
P Alcohol or sedative use
P Smoking

2198
Copyright © Harvard Medical School, 2018. All Rights Reserved.

Diagnosis of Sleep Apnea Syndrome


P Overnight polysomnogram

Treatment of Sleep Apnea Syndrome


P Behavioral
P Weight loss; avoid alcohol/sedatives; avoid sleeping on
back; quit smoking
P Medical
P CPAP/B-PAP
P Oral appliances
P Nasal expiratory resistance valves
P Surgery
P UP3; mandibular advancement; others
P Tracheostomy

2199
Copyright © Harvard Medical School, 2018. All Rights Reserved.

7. Asthma (Dr. Elliot Israel)


P Step-care approach to asthma treatment:
P Old approach: categorize severity of asthma
P New approach: assess asthma control
P Current impairment domain (within 2-4 weeks)
P Frequency of “rescue” bronchodilator use (goal: <2 days/wk)
P Frequency of nocturnal awakenings (goal: <2 days/mo)
P Lung function (goal: >80%)
P Future risk domain (within past year)
P Number of severe asthma attacks (goal: <1/yr)

Asthma (cont.)
P Stepping up therapy in poorly controlled asthma:
Specialized Rx:
Biologics (anti-IgE, anti-IL-5); BT
Three controllers

ICS+Anti-LT or ICS-LABA

ICS or Anti-LT

Rescue Inhaler Only

2200
Copyright © Harvard Medical School, 2018. All Rights Reserved.

Asthma Therapy
Inhaled corticosteroids:
P Reduce symptoms and prevent asthmatic
exacerbations
P Do not appear to alter the natural history of
asthma (that is, have not been shown to prevent the
development of irreversible airflow obstruction)
P Have dose-dependent systemic absorption,
impacting at high doses over prolonged period:
P Bone density, cataracts, intraocular pressure, and
skin bruising/thinning
P Can be used intermittently in mild, well-controlled
asthma

Inhaled Steroid Preparations


mcg/puff
• Budesonide DPI* Pulmicort 90, 180
• Mometasone MDI-HFA* Asmanex 100, 200
• Mometasone DPI* Asmanex Twisthaler 110, 220
• Beclomethasone DPI Qvar 40, 80
• Fluticasone MDI-HFA Flovent 44, 110, 220
• Fluticasone DPI Flovent Diskus/ArmonAir 50/55, 100/113, 250/232
• Fluticasone furoate DPI* Arnuity 100, 200
• Ciclesonide MDI-HFA* Alvesco 80, 160
• Flunisolide MDI-HFA Aerospan 80
* category B in pregnancy ; available by nebulization
* approved for once-daily dosing * small particle size; pro-drug formulation

2201
Copyright © Harvard Medical School, 2018. All Rights Reserved.

Dosing of Inhaled
Corticosteroids (mcg/day)

Low Medium High

<400 mcg 400 – 800 mcg >800 mcg

Combination ICS and LABA


Combination Brand name Dose per inhalation

Fluticasone propionate Advair DPI 100/50, 250/50, 500/50


+ salmeterol Advair HFA 44/21, 115/21, 230/21
AirDuo DPI 55/14, 113/14, 232/14
Budesonide + Symbicort HFA 80/4.5, 160/4.5
formoterol

Mometasone + Dulera HFA 100/5, 200/5


formoterol
Fluticasone furoate + Breo DPI 100/25, 200/25
vilanterol*
*Once-daily dosing

2202
Copyright © Harvard Medical School, 2018. All Rights Reserved.

Safety Concerns Regarding


Long-Acting Inhaled Beta-Agonists
• Black Box warning regarding salmeterol
and all other LABAs – now removed
WARNING: DATA FROM A LARGE PLACEBO-CONTROLLED US
STUDY THAT COMPARED THE SAFETY OF SALMETEROL
(SEREVENT® INHALATION AEROSOL) OR PLACEBO ADDED
TO USUAL ASTHMA THERAPY SHOWED A SMALL BUT
SIGNIFICANT INCREASE IN ASTHMA-RELATED DEATHS IN
PATIENTS RECEIVING SALMETEROL (13 DEATHS OUT OF
13,176 PATIENTS TREATED FOR 28 WEEKS) VERSUS THOSE ON
PLACEBO (3 OF 13,179) (SEE WARNINGS AND CLINICAL
TRIALS: ASTHMA: SALMETEROL MULTI-CENTER ASTHMA
RESEARCH TRIAL).

Fluticasone + Salmeterol vs.


Fluticasone alone
• Among 11,679 patients randomized to fluticasone (in 3
doses) vs. fluticasone (3 doses) plus salmeterol
• No deaths; only 2 intubations (both in fluticasone
alone group); overall no difference in “serious adverse
events”
• Fewer severe asthma exacerbations in group treated
with fluticasone + salmeterol.

Stempel DA, et al, NEJM 2016; 374:1822.

2203
Copyright © Harvard Medical School, 2018. All Rights Reserved.

Leukotriene Modifiers

Leukotriene Receptor Montelukast (Singulair)


Antagonists:
Zafirlukast (Accolate)

Lipoxygenase Inhibitor: Zileuton (Zyflo)

Arachidonic Acid Pathway


Membrane Phospholipids

Phospholipase A2

Arachidonic Acid Zileuton

Cyclooxygenase 5-lipoxygenase
Prostaglandins Leukotrienes Montelukast
Thromboxanes C4, D4, E4 Zafirlukast

Cysteinyl leukotriene receptor

2204
Copyright © Harvard Medical School, 2018. All Rights Reserved.

Aspirin-Intolerant Asthma
Membrane Phospholipids

Phospholipase A2
Aspirin
NSAIDs Arachidonic Acid
Cyclooxygenase 5-lipoxygenase
Prostaglandins Leukotrienes C4,
Thromboxanes
D4, E4
Cysteinyl leukotriene receptor

8. Pleural Effusions
(Dr. Scott Schissel)
P Mechanisms:
P Hydrostatic imbalance (transudates)
P Besides congestive heart failure:
P Nephrotic syndrome
P Trapped lung (also can be exudative)
P Pulmonary embolism (also can be exudative)
P Myxedema

2205
Copyright © Harvard Medical School, 2018. All Rights Reserved.

Exudates vs. Transudates

P Light’s criteria for an exudate (any one of the following


3 characteristics):
P Pleural fluid to serum total protein ratio >0.5
P Pleural fluid to serum LDH ratio >0.6
P Absolute pleural fluid LDH >2/3 upper limit of
normal of serum LDH

Exudates vs. Transudates (cont.)


P Additional criteria for transudate:
P Serum TP - pleural fluid TP >3.1 gm/dl
P Serum albumin - pleural fluid albumin >1.2 gm/dl
P Pleural fluid NT-proBNP >1500 pg/ml

P Other criterion for exudate:


P Pleural fluid cholesterol: >60 mg/dl

2206
Copyright © Harvard Medical School, 2018. All Rights Reserved.

Pleural Fluid Analysis


P Bloody effusion:
P In the absence of chest trauma, think:
cancer, pulmonary embolism, pneumonia.
P Chylous effusion:
P Chylomicrons present on lipoprotein analysis;
pleural fluid triglycerides >110 mg/dl).
P In the absence of thoracic duct trauma, think
obstruction due to: tumor, granulomatous disease,
LAM, radiation fibrosis, or yellow nails syndrome.

Chylous
effusion

2207
Copyright © Harvard Medical School, 2018. All Rights Reserved.

White Blood Cell Differential


Lymphocyte-predominant: think cancer, TB,
post-CABG (Dressler’s syndrome), collagen-
vascular disease
Neutrophil-predominant: think
parapneumonic, pulmonary embolism,
pancreatitis.
Eosinophilic: not helpful!!

Low Pleural Fluid pH

Seen in: complicated parapneumonic effusion


/ empyema; tuberculous pleuritis; rheumatoid
and lupus pleuritis; and long-standing
malignant pleural effusions.
Also seen with esophageal rupture;
hemothorax; systemic acidosis.
Most useful in management of
parapneumonic effusions.

2208
Copyright © Harvard Medical School, 2018. All Rights Reserved.

Pleural Fluid pH
in Parapneumonic Effusions
P Results from intense inflammation and anaerobic
glucose metabolism leading to production of
lactic acid and CO2.
Pleural Fluid Micro Fluid pH Risk of Drain?
Anatomy Poor
outcome
<½ GS and Cx NEG pH >7.20 LOW No, BUT need
hemithorax and to follow
>½ GS or Cx POS pH <7.20 Moderate YES
hemithorax, or / High
loculated

Unexplained Exudative Effusion


Special studies in certain cases: amylase,
D-DIMER, adenosine deaminase, interferon-
gamma, ANA, LE cells, RF, fibulin-3,
mesothelin, CEA, CA15-3.
Send cytology again.
Thoracoscopic pleural biopsy.
Rare indication for closed needle biopsy.

2209
Copyright © Harvard Medical School, 2018. All Rights Reserved.

Financial Conflicts of Interest

None.

2210
Copyright © Harvard Medical School, 2018. All Rights Reserved.

Evaluation of the dyspneic


patient
David M Systrom, M.D.
Associate Physician
Pulmonary and Critical Care Medicine
Brigham and Women’s Hospital
Asst. Prof.
Harvard Medical School

No disclosures

2211
Copyright © Harvard Medical School, 2018. All Rights Reserved.

Patient MF
21 yo male Harvard Crew
1.5 years SOB, especially w/ intense
training, competition
Patient and mom endorsed “noisy
breathing” during exacerbations
Rapid clearing of sx, EMT’s : “normal
exam”

Patient MF
“Doc make me better, we are going to the
Henley”

2212
Copyright © Harvard Medical School, 2018. All Rights Reserved.

Pt MF
Exam: normal
Spirometry:

Unexplained Dyspnea

2213
Copyright © Harvard Medical School, 2018. All Rights Reserved.

Vocal Cord Dysfunction


AKA laryngeal dyskinesia
Often young women w/ PTSD, many = conversion
reaction
Can be worsened by PNDr, GERD
Key to hx is inspiratory stridor, hoarseness
Often confused w/ asthma, methacholine
challenge may help, ABG’s should be normal
Spirometry: flattened inspiratory F-V loop
Confirmation w/ direct laryngoscopy: panting, full
exhalation: forced inspiration, exercise…
..inappropriate closure of glottis during inspiration
Treatment: voice training, SSRI

Pt WL
51 YO M, mildly obese
Previously 35 min Elliptical, 4 d /week, TM
4.4 mph, 4 deg, 15 min.
In 6/12 hanging an AC unit out a window,
leaning over a window sill, and had sudden R
anterior sharp CP, persistent R shoulder pain
and SOB since, immediate orthopnea
No persistent cough, wheeze, F, C, sweat, B
sx.

2214
Copyright © Harvard Medical School, 2018. All Rights Reserved.

Pt WL
SPIROMETRY (BTPS)
Predicted Pre-BD
Mean 95% CI Actual %Pred
FVC (Lts) 5.24 4.12 2.30 44
FEV1 (Lts) 4.14 3.30 1.70 41
FEV6 (Lts) 5.19 4.23 2.28 44
FEV1/FVC (%) 79 71 74 94

LUNG VOLUMES (PLETHYSMOGRAPHY)


TLC (Lts) 7.41 5.80 4.68 63

DIFFUSING CAPACITY
DLCO Unc (mL/min/mmHg) 32.71 22.69 25.85 79
DLCO Corr (Hb) (mL/min/mmHg) 32.71 22.69 25.85 79
VA@BTPS (Lts) 7.41 6.26 3.93 53
DL/VA (%) 4.43 3.50 6.58 149

RESPIRATORY MUSCLE STRENGTH


MIP 89 34 38
MEP 134 68 51

Pt WL

2215
Copyright © Harvard Medical School, 2018. All Rights Reserved.

Pt WL

Respiratory Muscle Weakness


Acute or chronic DOE, immediate orthopnea
Usually B diaphragmatic dysfxn, but unilateral plus other
can > SOB
Diagnosis:
Restrictive defect may be mild
VC upright > supine (10% fall), MIP
CT chest>r/o mass, LAD
Phrenic nerve conduction, diaphragmatic EMG
Treatment
Time, e.g., trauma, CABG
Weight loss
Elevate HOB
PSG: look for hypopneas>BiPAP
Inspiratory muscle training
IVIg, steroids

2216
Copyright © Harvard Medical School, 2018. All Rights Reserved.

Pt MR
•61 yo running Shoe Co Exec
•Two year decline in running splits 6>8

min miles due to SOB


•No immediate orthopnea, myalgias,

lightheadedness
•Exam routine labs normal

•Spirometry, TTE, CT chest, cardiac

stress normal
•Invasive CPET:

Invasive CPET

VO2max

CaO2
mPAP-PCWP

CvO2

2217
Copyright © Harvard Medical School, 2018. All Rights Reserved.

Invasive CPET Dx Algorithm


Impairment
VO2max < 80%

Pulmonary Mechanical O2 Flux


VEmax/MVV > 0.7 AT < 40%

Central Cardiac Mt Myopathy


Qtmax < 80% Ca-vO2 < [Hb]

Left Heart
PCWPmax > 20 mmHg

Right Heart
mPAPmax > 30 mmHg and
PVRmax >120 dynes .s.-5

Preload Failure
RAPmax < 9mm Hg
All else normal

Pt MR

2218
Copyright © Harvard Medical School, 2018. All Rights Reserved.

Invasive CPET Dx Algorithm


Impairment
VO2max < 80%

Pulmonary Mechanical O2 Flux


VEmax/MVV > 0.7 AT < 40%

Central Cardiac Mt Myopathy


Qtmax < 80% Ca-vO2 < [Hb]

Left Heart
PCWPmax > 20 mmHg

Right Heart
mPAPmax > 30 mmHg and
PVRmax >120 dynes .s.-5

Preload Failure
RAPmax < 9mm Hg
All else normal

EiPAH

2219
Copyright © Harvard Medical School, 2018. All Rights Reserved.

EiPAH

EiPAH

2220
Copyright © Harvard Medical School, 2018. All Rights Reserved.

eiPAH
•Presents w/ unexplained dyspnea
•aCPET: Intermediate exercise phenotype

between normal and resting PAH


•? Early disease vs stable variant

•Missed by TTE and resting RHC

•Ambrisentan responsive, ? Others

•Need clinical trials

Case ED
23 yo college student
In middle school, had DOE running in
field hockey.
Several years orthostatic
lightheadedness, occ. syncope, s/p tilt
table x 3, results equivocal > Rx'd w/
midodrine q 3h.
PMHx: mild asthma, migraines
ROS: pos. myalgias during and post
exercise x years.
Exam and spiro normal

2221
Copyright © Harvard Medical School, 2018. All Rights Reserved.

Pt ED

Case ED
PRE FLUIDS
Rest Heart Rate (bpm) 100
Max.Cardiac Output (L/min)) 8.56 (62%)
Max VO2 (mL/min) 856 (44%)
POST FLUIDS
Rest Heart Rate (bpm) 87
Max Cardiac Output (L/min) 11.67 (85%)
Max. VO2 (mL/min) 1004 (52%)

2222
Copyright © Harvard Medical School, 2018. All Rights Reserved.

Invasive CPET Dx Algorithm


Impairment
VO2max < 80%

Pulmonary Mechanical O2 Flux


VEmax/MVV > 0.7 AT < 40%

Central Cardiac Mt Myopathy


Qtmax < 80% Ca-vO2 < [Hb]

Left Heart
PCWPmax > 20 mmHg

Right Heart
mPAPmax > 30 mmHg and
PVRmax >120 dynes .s.-5

Preload Failure
RAPmax < 9mm Hg
All else normal

Case ED
Follow-up
In 3/13 started pyridostigmine 30 mg
po TID
BP's up off midodrine now used prn
fatigue, lightheaded approximately 1 x
week.
Currently, on good days, can move
around house comfortably, biking 6
min>20 min, limit is fatigue >> SOB

2223
Copyright © Harvard Medical School, 2018. All Rights Reserved.

Preload Failure
Young women, SOB, fatigue,
lightheadedness
Exacerbations after stress: viral, pregnancy
Overlap w/ POTS/OH, Mt myopathy, SFPN
Tilt Table may help, cort stim, consider
structurally impaired venous return, e.g.,
chronic DVT
Rx salt and H20 load, compression stockings,
aerobic exercise, Florinef, SSRI, midodrine,
Mestinon

Pt MP
28-year-old woman with increasing DOE x 5 mos.
A year ago she noted DOE climbing 6-7 stairs, now
on level ground w/ B leg fatigue.

Past medical history:


Childhood asthma
2003 mild aortic stenosis and insufficiency
1999 Raynaud's Disease
1999 Migraine headaches
S/p negative Tilt Table

ROS: No myalgias, arthralgias, cough, wheezing.

2224
Copyright © Harvard Medical School, 2018. All Rights Reserved.

Pt MP
Predicted Measured (% Pred)
Max VO2 (mL/min) 1820 1198 66%
Max VO2 (mL/kg/min) 20.3
VO2 (mL/min) at AT >40% 439 24%
Cardiac Output max (L/min) 13 13.5 104%

Case MP
==================================
= ADVANCED CARDIOPULMONARY GASES =
==================================
-----------------------------------------------------
Time CaO2 CvO2 Ca-vO2 PaO2 PaCO2 pH Lactate
-----------------------------------------------------
REST 18.7 11.8 6.9 106 43 7.39 0.5
-----------------------------------------------------
FW1 18.2 9.0 9.2 95 41 7.38 0.9
-----------------------------------------------------
1 17.2 9.4 7.8 110 36 7.41 0.9
2 17.8 9.7 8.1 105 40 7.41 0.9
3 18.5 9.6 8.9 110 39 7.40 1.1
4 17.9 9.4 8.5 104 39 7.39 2.1
5 17.4 9.2 8.2 116 38 7.38 3.5
-----------------------------------------------------
PEAK 18.0 9.1 8.9 125 36 7.37 5.5

2225
Copyright © Harvard Medical School, 2018. All Rights Reserved.

Invasive CPET Dx Algorithm


Impairment
VO2max < 80%

Pulmonary Mechanical O2 Flux


VEmax/MVV > 0.7 AT < 40%

Central Cardiac Mt Myopathy


Qtmax < 80% Ca-vO2 < [Hb]

Left Heart
PCWPmax > 20 mmHg

Right Heart
mPAPmax > 30 mmHg and
PVRmax >120 dynes .s.-5

Preload Failure
RAPmax < 9mm Hg
All else normal

Mitochondrial Myopathy
Present in adulthood w/ dyspnea > fatigue
Exacerbations after stress: e.g., viral, surgery
Often w/ longstanding, but sometimes
episodic exertional intolerance
w/u may include blood Mt mutation screen,
muscle bx, iCPET
Rx is Vitamin cocktail, including CoEnzyme
Q10

2226
Copyright © Harvard Medical School, 2018. All Rights Reserved.

Advanced CPET Diagnoses


(%, n=225)

7
HFpEF
13 30 eiPAH
rPAH
Mt myopathy
20 22 Normal/Detrained
Preload

Take-Home Messages
a. Unexplained dyspnea is defined as un or
underexplained sx after a thorough hx,
exam, routine labs, full PFT’s, TTE and
chest radiography when appropriate
b. Additional testing might include an ENT
eval, MTC, MIP’s
c. iCPET can rule in or out exercise-
induced PH, preload failure and suggest
a Mt myopathy

2227
Copyright © Harvard Medical School, 2018. All Rights Reserved.

Board Questions
23 yo F presents w/ 18 mos of DOE
following an apparent viral syndrome.
Her DDx includes:

a. eiPAH
b. Preload failure
c. Mt myopathy
d. All of the above
e. b&c

Board Questions
23 yo F presents w/ 18 mos of DOE and
episodic lightheadedness following an
apparent viral syndrome. Her DDx includes:

a. eiPAH
b. Preload failure
c. Mt myopathy
d. All of the above
e. b&c

2228
Copyright © Harvard Medical School, 2018. All Rights Reserved.

Case 1
Preload failure and Mt
myopathies often present
together and in the post-
infectious setting
An autoimmune
pathogenesis is suspected

Board Questions
18 yo F presents w/ two years of DOE and subjective
wheezing. Exam in office is normal. PEFR in the office
and field is repeatedly normal. SABA and ICS/LABA
have not helped. The next step should be:

a. Escalation of asthma controllers, e.g., tiotropium


b. CT chest
c. TTE
d. Cardiopulmonary exercise testing
e. Methacholine challenge

2229
Copyright © Harvard Medical School, 2018. All Rights Reserved.

Board Questions
18 yo F presents w/ two years of DOE and subjective
wheezing. Exam in office is normal. PEFR in the office
and field is repeatedly normal. SABA and ICS/LABA
have not helped. The next step should be:

a. Escalation of asthma controllers, e.g., tiotropium


b. CT chest
c. TTE
d. Cardiopulmonary exercise testing
e. Methacholine challenge

Case 2
Vocal cord dysfunction presents in a similar
fashion to asthma. Exacerbations can be
provoked by emotions and exercise, and
aggravate by post nasal drip and GERD.
Negative MTC essentially rules out asthma.
Clues are subjective and objective stridor, rapid
clearing, flattened inspiratory F-V loop. Dx
confirmed by direct laryngoscopy w/ maneuvers.
Rx = speech therapy.

2230
Copyright © Harvard Medical School, 2018. All Rights Reserved.

References
Morris MJ, et al. Vocal cord dysfunction: etiologies and treatment. Clin
Pulmonary Med. 2006; 13:73–86.
Oldham WM, Lewis GD, Opotowsky AR, Waxman AB, Systrom DM.
Unexplained exertional dyspnea caused by low ventricular filling
pressures: results from clinical invasive cardiopulmonary exercise testing.
Pulm Circ 2016; 6:1, 55-62
Taivassalo T, et al. The spectrum of exercise tolerance in mitochondrial
myopathies: a study of 40 patients. Brain 2003; 126:413-423.
Functional impact of exercise pulmonary hypertension in patients with
borderline resting pulmonary artery pressure. Oliveira RKF Faria Urbina
M, Maron BA, Santos M, Waxman AB, Systrom DM. Pulm Circ 2017.
DOI: 10.1177/2045893217709025
Segrera SA, Lawler L, Opotowsky AR, Systrom DM, Waxman AB. Open
label study of ambrisentan in patients with exercise pulmonary
hypertension. Pulm Circ 2017; 7(2) 1–8
W Huang, S Resch, RKF Oliveira, BA Cockrill, DM Systrom, AB
Waxman. Invasive cardiopulmonary exercise testing in the evaluation of
unexplained dyspnea: Insights from a multidisciplinary dyspnea center.
Eur J Prev Card 2017; 0(00) 1–10 DOI: 10.1177/2047487317709605
Maron BA, Cockrill BA, Waxman AB, Systrom DM. The invasive
cardiopulmonary exercise test. Circulation. 2013;127:1157-1164

2231
Copyright © Harvard Medical School, 2018. All Rights Reserved.

PULMONARY MEDICINE
BOARD REVIEW

Christopher H. Fanta, M.D.


Pulmonary and Critical Care Division
Brigham and Women’s Hospital
Partners Asthma Center
Harvard Medical School

Financial Conflicts of Interest

None.

2232
Copyright © Harvard Medical School, 2018. All Rights Reserved.

Question #1:

A 75-year-old smoker (1 ppd for 40 years,


quitting 15 years ago) presents with
progressive exertional dyspnea over the last 6
months. He reports a non-productive cough,
worse in the mornings, without fever, chest
pain, or hemoptysis. He denies orthopnea,
PND, or swelling of his lower extremities.

Question #1 (Cont.):

He has retired from his work as a lawyer. He


has no unusual exposures at home, no arthritic
complaints or symptoms of Raynaud’s
disease.
His exam is notable for clubbing of the digits
and coarse inspiratory crackles in the lower
lung zones.

2233
Copyright © Harvard Medical School, 2018. All Rights Reserved.

Question #1 (Cont.):
His oxygen saturation is 95% at rest and falls
to 87% with walking.
Spirometry suggests moderate restriction.
Chest X-ray shows increased linear
markings, predominantly in the lower lung
zones. His chest CT scan shows basilar
honeycombing and traction bronchiectasis,
read as “suggestive of usual interstitial
pneumonitis.”

2234
Copyright © Harvard Medical School, 2018. All Rights Reserved.

Question #1
Which of the following actions would you take?

A. Refer to Pulmonary for transbronchial lung biopsy


B. Refer to Thoracic Surgery for thoracoscopic lung
biopsy
C. Begin anti-inflammatory therapy with high-dose
prednisone
D. Begin immunosuppressive therapy with prednisone
and azathioprine
E. Begin anti-fibrotic therapy with pirfenidone or
nintedanib.

Question #1
Which of the following actions would you take?

A. Refer to Pulmonary for transbronchial lung biopsy


B. Refer to Thoracic Surgery for thoracoscopic lung
biopsy
C. Begin anti-inflammatory therapy with high-dose
prednisone
D. Begin immunosuppressive therapy with prednisone
and azathioprine
E. Begin anti-fibrotic therapy with pirfenidone or
nintedanib.

2235
Copyright © Harvard Medical School, 2018. All Rights Reserved.

Question #2
Conditions commonly associated with cystic fibrosis
include each of the following except:

A. Bronchiectasis
B. Sinusitis
C. Airflow obstruction
D. Aspermia
E. Systemic Pseudomonas infections

Question #2
Conditions commonly associated with cystic fibrosis
include each of the following except:

A. Bronchiectasis
B. Sinusitis
C. Airflow obstruction
D. Aspermia
E. Systemic Pseudomonas infections

2236
Copyright © Harvard Medical School, 2018. All Rights Reserved.

Question #3
Typical manifestations of asbestos-related intrathoracic
disease include each of the following except:

A. Fibrocalcific parenchymal disease, predominantly


involving the upper zones of the lung
B. Pleural plaques
C. Malignant mesothelioma
D. Benign pleural effusions
E. Bronchogenic carcinoma

Question #3
Typical manifestations of asbestos-related intrathoracic
disease include each of the following except:

A. Fibrocalcific parenchymal disease, predominantly


involving the upper zones of the lung
B. Pleural plaques
C. Malignant mesothelioma
D. Benign pleural effusions
E. Bronchogenic carcinoma

2237
Copyright © Harvard Medical School, 2018. All Rights Reserved.

2238
Copyright © Harvard Medical School, 2018. All Rights Reserved.

2239
Copyright © Harvard Medical School, 2018. All Rights Reserved.

Question #4:

During December of last year, a 63-year-old


man with a 60 pack-year history of cigarette
smoking noticed an increase in the amount of
his usual sputum production. His sputum
became now yellow-green. Findings on
physical examination were normal. His
temperature was 37.8oC (100oF). A chest X-
ray was obtained and was normal.

Question #4 (Cont.):

A gram stain of sputum showed less than 10


squamous epithelial cells per low-power field
and many neutrophils per high-power field.
The predominant organisms on the gram-
stained smear were gram-negative cocci in
pairs.

2240
Copyright © Harvard Medical School, 2018. All Rights Reserved.

Question #4
You would initiate a course of antibiotic therapy
using which of the following?

A. Ampicillin, orally
B. Amoxicillin-clavulanic acid, orally
C. Procaine penicillin G, intramuscularly twice a day
D. Cefoxatime, intravenously
E. Cephalexin, orally

Question #4
You would initiate a course of antibiotic therapy
using which of the following?

A. Ampicillin, orally
B. Amoxicillin-clavulanic acid, orally
C. Procaine penicillin G, intramuscularly twice a day
D. Cefoxatime, intravenously
E. Cephalexin, orally

2241
Copyright © Harvard Medical School, 2018. All Rights Reserved.

Question #5
An intensive pulmonary rehabilitation program
in patients with chronic obstructive pulmonary
disease has been shown to improve

A. Survival
B. Cardiovascular function
C. Exercise tolerance
D. Expiratory flow rates

2242
Copyright © Harvard Medical School, 2018. All Rights Reserved.

Question #5
An intensive pulmonary rehabilitation program
in patients with chronic obstructive pulmonary
disease has been shown to improve

A. Survival
B. Cardiovascular function
C. Exercise tolerance
D. Expiratory flow rates

2243
Copyright © Harvard Medical School, 2018. All Rights Reserved.

Question #6 (cont.):
Her chest exam reveals only a transient
expiratory rhonchus.
Chest X-ray is read as showing a lingular
pneumonia.
Chest CT scan shows bronchiectasis with
tree-in-bud nodularity in the RML, lingula, and
RLL. The radiologic interpretation raises the
possibility of non-tuberculous mycobacterial
infection.

2244
Copyright © Harvard Medical School, 2018. All Rights Reserved.

Question #6
Based on these findings you recommend:

A. Levofloxacin for 2 week course


B. Initiation of therapy for M. avium-intracellulare with
azithromycin, ethambutol, and rifampin
C. Initiation of therapy for non-tuberculous
mycobacteria with INH, ethambutol, rifampin, and
pyrazinamide
D. Sputum for AFB culture and smear X 3
E. Blood for anti-nuclear cytoplasmic antibody (ANCA)

Question #6
Based on these findings you recommend:

A. Levofloxacin for 2 week course


B. Initiation of therapy for M. avium-intracellulare with
azithromycin, ethambutol, and rifampin
C. Initiation of therapy for non-tuberculous
mycobacteria with INH, ethambutol, rifampin, and
pyrazinamide
D. Sputum for AFB culture and smear X 3
E. Blood for alpha-1 antitrypsin level

2245
Copyright © Harvard Medical School, 2018. All Rights Reserved.

Question #7:

An arterial blood gas analysis shows a PO2


of 40 mm Hg, PCO2 of 80 mm Hg, and a pH of
7.10 in a patient breathing room air.

Question #7
These findings are most likely to be associated
with which of the following disorders?

A. Adult respiratory distress syndrome


B. Status asthmaticus
C. Severe bacterial pneumonia
D. End-stage chronic obstructive pulmonary disease
E. Sedative drug overdose

2246
Copyright © Harvard Medical School, 2018. All Rights Reserved.

pH Changes in Respiratory Acidosis

Acute: ∆ pH = ∆ PCO2 * 0.008

Chronic: ∆ pH = ∆ PCO2 * 0.003

Acute - on -
Chronic: ∆ pH = ∆ PCO2 * 0.005

Case Example

Acute: ∆ pH = 40 * 0.008 = 0.32


Predicted pH = 7.08

Chronic: ∆ pH = 40 * 0.003 = 0.12


Predicted pH = 7.28

Acute - on -
Chronic: ∆ pH = 40 * 0.005 = 0.20
Predicted pH = 7.20

2247
Copyright © Harvard Medical School, 2018. All Rights Reserved.

A-a Gradient for Oxygen

A -aDO2 = PAO2 - PaO2

PAO2 = (PB - 47) * FlO2 - PCO2/R

= (760 - 47) * 0.21 - PCO2/0.8

= 150 - PCO2/0.8

Case Example

PAO2 = (PB - 47) * FlO2 - PCO2/R

= 150 - 80/0.8

= 50

PAO2 - PaO2 = 50 - 40

A-aDO2 = 10 (Normal = 5 - 20)

2248
Copyright © Harvard Medical School, 2018. All Rights Reserved.

Question #7
These findings are most likely to be associated
with which of the following disorders?

A. Adult respiratory distress syndrome


B. Status asthmaticus
C. Severe bacterial pneumonia
D. End-stage chronic obstructive pulmonary disease
E. Sedative drug overdose

Question #8:

A 52-year-old school teacher presents for a


check-up. She has recently moved to the area
to assume a new teaching position. She has
never smoked cigarettes and has been found
healthy at previous annual check-ups.

2249
Copyright © Harvard Medical School, 2018. All Rights Reserved.

Question #8 (Cont.):

Her physical examination and routine


laboratory studies are normal. However, a
routine posteroanterior and lateral chest X-ray
shows a 1-cm x 1.8-cm smooth, well-
demarcated lesion in the right middle lobe.

Question #8
The appropriate initial step for the patient’s
internist would be:

A. Obtain a chest PET-CT scan


B. Order a transthoracic needle biopsy
C. Ask a pulmonologist to perform bronchoscopy
D. Obtain prior CXRs from her former physician
E. Order chest MRI

2250
Copyright © Harvard Medical School, 2018. All Rights Reserved.

Question #8
The appropriate initial step for the patient’s
internist would be:

A. Obtain a chest PET-CT scan


B. Order a transthoracic needle biopsy
C. Ask a pulmonologist to perform bronchoscopy
D. Obtain prior CXRs from her former physician
E. Order chest MRI

Question #9:
A 47-year-old woman presents with a six-week
history of nonproductive cough, moderate exertional
dyspnea, and temperature to 38.3oC (101oF). The
patient has been in good health in the past, although
she has smoked two packs of cigarettes per day for
the last 25 years. The patient has received
clarithromycin 500 mg orally four times a day for 10
days on two occasions without improvement in her
symptoms.

2251
Copyright © Harvard Medical School, 2018. All Rights Reserved.

Question #9 (Cont.):
There is no history of ocular inflammation, skin
rash, or arthritis.
The physical examination shows normal jugular
venous pressure and no peripheral
lymphadenopathy. The intensity of breath sounds is
normal except over the lower lobes bilaterally where
they are significantly reduced. There are also
bibasilar crackles, but no bronchial breathing or
egophony, and no wheezing.

Question #9 (Cont.):
Laboratory studies:
• Hematocrit 31%
• Leukocyte count, 11,100/uL with 18% lymphocytes,
64% polys, 7% band forms,
6% monocytes, and 5% eosinophils
• Serum creatinine 0.8 mg/dl
• Urinalysis is normal
• The chest X-ray shows airspace disease at both lung
bases.

2252
Copyright © Harvard Medical School, 2018. All Rights Reserved.

Question #9
The most likely diagnosis is:

A. Legionnaire’s disease
B. Wegener’s granulomatosis
C. Streptoccoccus pneumoniae pneumonia
D. Idiopathic pulmonary fibrosis
E. Cryptogenic organizing pneumonia (Bronchiolitis
obliterans organizing pneumonia)

2253
Copyright © Harvard Medical School, 2018. All Rights Reserved.

Question #9
The most likely diagnosis is:

A. Legionnaire’s disease
B. Wegener’s granulomatosis
C. Streptoccoccus pneumoniae pneumonia
D. Idiopathic pulmonary fibrosis
E. Cryptogenic organizing pneumonia (Bronchiolitis
obliterans organizing pneumonia)

Differentiating BOOP from


Bronchiolitis Obliterans

Cryptogenic Organizing Constrictive


Pneumonia (BOOP) Bronchiolitis (BO)
Presentation Pneumonia-like Emphysema-like

Chest X-ray Multifocal or diffuse Hyperinflation


pulmonary opacities
Physiology Restrictive Obstructive

Response to Good Poor


Steroid
Treatment

2254
Copyright © Harvard Medical School, 2018. All Rights Reserved.

Questions #10 - 14
10. A majority of cigarette smokers are affected
11. Reduced FEV1/FVC ratio
12. Decreased diffusing capacity (DLCO)
13. Most patients have a deficiency of alpha-1
antitrypsin
14. Montelukast (Singulair®) is useful in treatment
A. Emphysema
B. Asthma
C. Both
D. Neither

Questions #10 - 14
10. A majority of cigarette smokers are affected

A. Emphysema
B. Asthma
C. Both
D. Neither

2255
Copyright © Harvard Medical School, 2018. All Rights Reserved.

Questions #10 - 14
10. A majority of cigarette smokers are affected

A. Emphysema
B. Asthma
C. Both
D. Neither

Questions #10 - 14

11. Reduced FEV1/FVC ratio

A. Emphysema
B. Asthma
C. Both
D. Neither

2256
Copyright © Harvard Medical School, 2018. All Rights Reserved.

Questions #10 - 14

11. Reduced FEV1/FVC ratio

A. Emphysema
B. Asthma
C. Both
D. Neither

Questions #10 - 14

12. Decreased diffusing capacity (DLCO)

A. Emphysema
B. Asthma
C. Both
D. Neither

2257
Copyright © Harvard Medical School, 2018. All Rights Reserved.

Questions #10 - 14

12. Decreased diffusing capacity (DLCO)

A. Emphysema
B. Asthma
C. Both
D. Neither

Questions #10 - 14

13. Most patients have a deficiency of alpha-1


antitrypsin

A. Emphysema
B. Asthma
C. Both
D. Neither

2258
Copyright © Harvard Medical School, 2018. All Rights Reserved.

Questions #10 - 14

13. Most patients have a deficiency of alpha-1


antitrypsin

A. Emphysema
B. Asthma
C. Both
D. Neither

Questions #10 - 14

14. Montelukast (Singulair®) is useful in treatment

A. Emphysema
B. Asthma
C. Both
D. Neither

2259
Copyright © Harvard Medical School, 2018. All Rights Reserved.

Questions #10 - 14

14. Montelukast (Singulair®) is useful in treatment

A. Emphysema
B. Asthma
C. Both
D. Neither

Questions #15 - 18
15. May be characterized by severe and diffuse lung infiltrates
16. Most commonly caused by sepsis and gastric aspiration
17. Positive-pressure ventilation may be an important adjunct
treatment
18. Corticosteroid therapy has been shown to be beneficial when
initiated early

A. Adult respiratory distress syndrome


B. Severe cardiogenic pulmonary edema
C. Both
D. Neither

2260
Copyright © Harvard Medical School, 2018. All Rights Reserved.

Questions #15 - 18

15. May be characterized by severe and diffuse


lung infiltrates

A. Adult respiratory distress syndrome


B. Severe cardiogenic pulmonary edema
C. Both
D. Neither

Questions #15 - 18

15. May be characterized by severe and diffuse


lung infiltrates

A. Adult respiratory distress syndrome


B. Severe cardiogenic pulmonary edema
C. Both
D. Neither

2261
Copyright © Harvard Medical School, 2018. All Rights Reserved.

Questions #15 - 18

16. Most commonly caused by sepsis and gastric


aspiration

A. Adult respiratory distress syndrome


B. Severe cardiogenic pulmonary edema
C. Both
D. Neither

Questions #15 - 18

16. Most commonly caused by sepsis and gastric


aspiration

A. Adult respiratory distress syndrome


B. Severe cardiogenic pulmonary edema
C. Both
D. Neither

2262
Copyright © Harvard Medical School, 2018. All Rights Reserved.

Questions #15 - 18

17. Positive-pressure ventilation may be an


important adjunct treatment

A. Adult respiratory distress syndrome


B. Severe cardiogenic pulmonary edema
C. Both
D. Neither

Questions #15 - 18

17. Positive-pressure ventilation may be an


important adjunct treatment

A. Adult respiratory distress syndrome


B. Severe cardiogenic pulmonary edema
C. Both
D. Neither

2263
Copyright © Harvard Medical School, 2018. All Rights Reserved.

Questions #15 - 18

18. Corticosteroid therapy has been shown to be


beneficial when initiated early
A. Adult respiratory distress syndrome
B. Severe cardiogenic pulmonary edema
C. Both
D. Neither

Questions #15 - 18

18. Corticosteroid therapy has been shown to be


beneficial when initiated early
A. Adult respiratory distress syndrome
B. Severe cardiogenic pulmonary edema
C. Both
D. Neither

2264
Copyright © Harvard Medical School, 2018. All Rights Reserved.

Questions #19 - 22
19. Necrobiotic nodules
20. Occurrence in women only
21. Diabetes insipidus
22. Recurrent aspiration pneumonias

A. Rheumatoid arthritis
B. Scleroderma
C. Langerhans cell histiocytosis (histiocytosis X;
eosinophilic granuloma)
D. Lymphangioleiomyomatosis
E. Idiopathic pulmonary fibrosis

Questions #19 - 22
19. Necrobiotic nodules

A. Rheumatoid arthritis
B. Scleroderma
C. Langerhans cell histiocytosis (histiocytosis X;
eosinophilic granuloma)
D. Lymphangioleiomyomatosis
E. Idiopathic pulmonary fibrosis

2265
Copyright © Harvard Medical School, 2018. All Rights Reserved.

Questions #19 - 22
19. Necrobiotic nodules

A. Rheumatoid arthritis
B. Scleroderma
C. Langerhans cell histiocytosis (histiocytosis X;
eosinophilic granuloma)
D. Lymphangioleiomyomatosis
E. Idiopathic pulmonary fibrosis

Questions #19 - 22

20. Occurrence in women only

A. Rheumatoid arthritis
B. Scleroderma
C. Langerhans cell histiocytosis (histiocytosis X;
eosinophilic granuloma)
D. Lymphangioleiomyomatosis
E. Idiopathic pulmonary fibrosis

2266
Copyright © Harvard Medical School, 2018. All Rights Reserved.

Questions #19 - 22

20. Occurrence in women only

A. Rheumatoid arthritis
B. Scleroderma
C. Langerhans cell histiocytosis (histiocytosis X;
eosinophilic granuloma)
D. Lymphangioleiomyomatosis
E. Idiopathic pulmonary fibrosis

Questions #19 - 22

21. Diabetes insipidus

A. Rheumatoid arthritis
B. Scleroderma
C. Langerhans cell histiocytosis (histiocytosis X;
eosinophilic granuloma)
D. Lymphangioleiomyomatosis
E. Idiopathic pulmonary fibrosis

2267
Copyright © Harvard Medical School, 2018. All Rights Reserved.

Questions #19 - 22

21. Diabetes insipidus

A. Rheumatoid arthritis
B. Scleroderma
C. Langerhans cell histiocytosis (histiocytosis X;
eosinophilic granuloma)
D. Lymphangioleiomyomatosis
E. Idiopathic pulmonary fibrosis

Questions #19 - 22

22. Recurrent aspiration pneumonias


A. Rheumatoid arthritis
B. Scleroderma
C. Langerhans cell histiocytosis (histiocytosis X;
eosinophilic granuloma)
D. Lymphangioleiomyomatosis
E. Idiopathic pulmonary fibrosis

2268
Copyright © Harvard Medical School, 2018. All Rights Reserved.

Questions #19 - 22

22. Recurrent aspiration pneumonias


A. Rheumatoid arthritis
B. Scleroderma
C. Langerhans cell histiocytosis (histiocytosis X;
eosinophilic granuloma)
D. Lymphangioleiomyomatosis
E. Idiopathic pulmonary fibrosis

Questions #23 - 27
Detection and Treatment of
Asymptomatic (Latent) Tuberculous
Infection
• Tuberculosis germs in the lungs without any evidence
for active infection = latent tuberculous infection (LTBI).

• Most cases of active TB are due to reactivation of


LTBI.

• Treatment of LTBI can eradicate TB germs and prevent


reactivation, but carries risk of potentially serious side
effects (in an asymptomatic person).

2269
Copyright © Harvard Medical School, 2018. All Rights Reserved.

Questions #23 - 27
Indications for Treating Latent
Tuberculous Infection

• Household contact
• Recent convertor
• X-ray of inactive TB; never treated with TB drugs
• Special circumstances (e.g., diabetes, HIV, dialysis,
immunosuppressing drugs, major weight loss, silicosis,
recent immigration from an endemic area)
• (Under age 35 years)

Defining a Positive PPD Skin Test

• High risk pop’n: > 5 mm induration

• Moderate risk pop’n: > 10 mm induration

• Low risk pop’n: > 15 mm induration

2270
Copyright © Harvard Medical School, 2018. All Rights Reserved.

Defining a Positive PPD Skin Test


(cont’d)

• High risk:
• HIV disease or immunosuppression;
• Recent close contact; or
• Scarring on CXR c/w inactive disease

Defining a Positive PPD Skin Test


(cont’d)

• Moderate risk:
• Increased risk of exposure
• (e.g., from countries with high TB prevalence;
I.V. drug abusers; homeless; nursing home
residents; health care workers; children
exposed to high-risk adults)

2271
Copyright © Harvard Medical School, 2018. All Rights Reserved.

Defining a Positive PPD Skin Test


(cont’d)

• Moderate risk (cont’d):


• Increased risk of activation
• (e.g., diabetes; chronic renal failure; weight loss
>10% IBW; gastrectomy; lymphoma or
leukemia; lung or head/neck cancer; Rx with
tumor necrosis factor alpha inhibitors)

Influence of Prior Vaccination with


BCG

• It is recommended that a history of prior


vaccination with BCG should be ignored
when deciding about treatment of a positive
PPD skin test.

2272
Copyright © Harvard Medical School, 2018. All Rights Reserved.

Interferon Gamma Release Assays

• IGRAs and TSTs can be used interchangeably;


• BCG does not cause a positive IGRA result; and
• IGRAs are less likely to be positive due to cross-
reactivity with atypical mycobacteria.
Mazurek GH, et al. Updated guidelines for using
interferon gamma release assays to detect
Mycobacterium tuberculosis infection – United
States, 2010. MMWR 2010; 59:RR-5.

Questions # 23 - 27
Treatment of latent tuberculous infection (e.g.,
isoniazid 300 mg daily) should be given in
which of the following cases?

23. A 21-year-old healthcare worker with a TB skin test


reaction of 15 mm induration. Chest X-ray is normal.
The skin test was positive 5 years ago at 10 mm
induration.

A. Yes
B. No

2273
Copyright © Harvard Medical School, 2018. All Rights Reserved.

Questions # 23 - 27
Treatment of latent tuberculous infection (e.g.,
isoniazid 300 mg daily) should be given in
which of the following cases?

23. A 21-year-old healthcare worker with a TB skin test


reaction of 15 mm induration. Chest X-ray is normal.
The skin test was positive 5 years ago at 10 mm
induration.

A. Yes
B. No

Questions # 23 - 27
Treatment of latent tuberculous infection (e.g.,
isoniazid 300 mg daily) should be given in
which of the following cases?

24. A 74-year-old man with no known TB exposure has a


TB skin test of 5 mm induration discovered on routine
testing. A repeat skin test 1 week later shows a 15 mm
induration. A chest X-ray is obtained and is normal.

A. Yes
B. No

2274
Copyright © Harvard Medical School, 2018. All Rights Reserved.

Questions # 23 - 27
Treatment of latent tuberculous infection (e.g.,
isoniazid 300 mg daily) should be given in
which of the following cases?

24. A 74-year-old man with no known TB exposure has a TB


skin test of 5 mm induration discovered on routine
testing. A repeat skin test 1 week later shows a 15 mm
induration. A chest X-ray is obtained and is normal.

A. Yes
B. No

Booster Phenomenon

Person previously exposed to TB who on


initial testing has a negative result but who, on
repeat testing one to a few weeks later, has a
positive test.

2275
Copyright © Harvard Medical School, 2018. All Rights Reserved.

Booster Phenomenon (Cont.)

The initial result is a “false negative” due to


waning cellular immunity; the latter is a true
positive due to “boosting” of the immune
response by the recent re-exposure to
tuberculin protein.

Booster Phenomenon (Cont.)

Booster phenomenon should be


distinguished from a recent conversion.
Screening (e.g., in nursing homes, hospitals)
often uses an initial two-step PPD testing
procedure, with repeat testing 1-3 weeks
after an initial negative result.

2276
Copyright © Harvard Medical School, 2018. All Rights Reserved.

Questions #23 - 27 (cont’d)


Treatment of latent tuberculous infection (e.g.,
isoniazid 300 mg daily) should be given in
which of the following cases?

25. A 45-year-old former IV drug abuser who is HIV


positive has a clear chest X-ray. His TB test is 5 mm
in induration. Prior skin tests are reported as
“equivocal.”

A. Yes
B. No

Questions #23 - 27 (cont’d)


Treatment of latent tuberculous infection (e.g.,
isoniazid 300 mg daily) should be given in
which of the following cases?

25. A 45-year-old former IV drug abuser who is HIV


positive has a clear chest X-ray. His TB test is 5 mm
in induration. Prior skin tests are reported as
“equivocal.”

A. Yes
B. No

2277
Copyright © Harvard Medical School, 2018. All Rights Reserved.

Questions #23 - 27 (cont’d)


Treatment of latent tuberculous infection (e.g.,
isoniazid 300 mg daily) should be given in
which of the following cases?

26. A 53-year-old woman with no known medical illnesses


and a clear chest X-ray has a TB test reaction of 8 mm.
She has never had skin testing and is tested now
because her husband has just had active TB
diagnosed after a 6-mos illness.
A. Yes
B. No

Questions #23 - 27 (cont’d)


Treatment of latent tuberculous infection (e.g.,
isoniazid 300 mg daily) should be given in
which of the following cases?

26. A 53-year-old woman with no known medical illnesses


and a clear chest X-ray has a TB test reaction of 8 mm.
She has never had skin testing and is tested now
because her husband has just had active TB
diagnosed after a 6-mos illness.
A. Yes
B. No

2278
Copyright © Harvard Medical School, 2018. All Rights Reserved.

Questions #23 - 27 (cont’d)


Treatment of latent tuberculous infection (e.g.,
isoniazid 300 mg daily) should be given in
which of the following cases?

27. A 26-year-old homeless man has a TB skin test of


15 mm induration. He has no symptoms but his
chest X-ray shows a left upper lobe infiltrate.

A. Yes
B. No

Questions #23 - 27 (cont’d)


Treatment of latent tuberculous infection (e.g.,
isoniazid 300 mg daily) should be given in
which of the following cases?

27. A 26-year-old homeless man has a TB skin test of


15 mm induration. He has no symptoms but his
chest X-ray shows a left upper lobe infiltrate.

A. Yes
B. No

2279
Copyright © Harvard Medical School, 2018. All Rights Reserved.

Additional Recommendations for


Treatment of Latent Tuberculous
Infection

• INH for 9 months is preferred therapy. Alternatives:


• INH for 6 months;
• Rifampin for 4 months
• INH + rifapentine once weekly for 3 months

• Routine laboratory monitoring not indicated, with


certain exceptions: HIV infection; pregnancy;
chronic liver disease; or regular alcohol use.

Financial Conflicts of Interest

None.

2280
Copyright © Harvard Medical School, 2018. All Rights Reserved.

Women’s Neurology

Mary Angela O’Neal, M.D.


Assistant Professor, Department of Neurology, Harvard Medical School
Director of the Women’s Neurology Program
Director of the Neurosciences clinic

Disclosures

I have no disclosures

2281
Copyright © Harvard Medical School, 2018. All Rights Reserved.

Objectives

Use Cases to:


• Discuss gender specific neurological concerns

• Review the pathophysiology of the neurologic


disorder as it relates to gender
• Describe treatment specific concerns for
women with neurologic disease
MS, Migraine, Stroke in pregnancy, Eclampsia &
Stroke Risk Factors in Women

Women’s Life Cycle

Menarche Pregnancy Menopause

Multiple Sclerosis

Migraine

Alzheimer's
Dementia

Stroke

2282
Copyright © Harvard Medical School, 2018. All Rights Reserved.

Women’s Life Cycle

Menarche Pregnancy Menopause

Eclampsia

Stroke

Issues to discuss when caring for Women in their


Reproductive years

Family planning

Discussion of medication risks in pregnancy

Effects of pregnancy on the underlying


disease

Effects of the underlying disease on


pregnancy

2283
Copyright © Harvard Medical School, 2018. All Rights Reserved.

FDA Pharmaceutical Pregnancy Categories

A Adequate and well controlled human studies have failed to demonstrate a risk to the
fetus in the first trimester of pregnancy ( and there is no risk in later trimesters).
B Animal reproduction studies have failed to demonstrate a risk to the fetus and there
are no adequate and well controlled studies in pregnant women OR Animal studies have
shown an adverse effect, but adequate and well-controlled studies in pregnant women
have failed to demonstrate a risk to the fetus in any trimester.
C Animal reproduction studies have shown an adverse effect on the fetus and there
are no adequate and well-controlled studies in humans, but potential benefits may
warrant use of the drug in pregnant women despite potential risks.
D There is positive evidence of human fetal risk based on adverse reaction data from
investigational or marketing experience or studies in humans, but potential benefits
may warrant use of the drug in pregnant women despite potential risks
X Studies in animals or humans have demonstrated fetal abnormalities and/or there
is positive evidence of human fetal risk based on adverse reaction data from
investigational or marketing experience, and the risks involved in use of the drug in
pregnant women clearly outweigh potential benefits

Reproductive Health and Pregnancy Concerns

• Multiple Sclerosis

• Migraine

2284
Copyright © Harvard Medical School, 2018. All Rights Reserved.

Case 1

A 28 year old women with


relapsing remitting MS
who has stable disease.
She is on interferon beta-
1b and an oral
contraceptive.

She wants to get pregnant.

Questions

• What should I do about my medication?

• How will pregnancy effect my MS?

• What if I have a relapse?

• Is it safe for me to breast feed?

2285
Copyright © Harvard Medical School, 2018. All Rights Reserved.

Immuno- FDA Drug Half Fetal and maternal risks Secretion in breast milk
modulating Class Life
agents
C 10 hours
Interferon β-1-b Spontaneous abortions in animals. not seen in
Minimal
and β-1-a humans
B 7 hours
Glatiramer acetate None reported Minimal
C 25 - 32
Intravenous
days Probably safe in pregnancy Unknown
immunoglobulin
Fingolimod C 6-9 days Teratogenicity seen in animals and humans. No Avoid in lactation
specific pattern observed.

Dimethyl Fumarate C 1 hour increased spontaneous abortion in animals. Not Avoid in lactation
reported in humans
Teriflunomide X 18-19 days Teratogenicity seen in animals; precursor leflunomide Avoid in lactation
is a known human teratogen. No malformations in
humans observed thus far.

Daclizumab C 20 days Embryofetal deaths observed in animals with early Avoid in lactation
exposure. No fetal malformations in humans observed
thus far.

Natalizumab C 7-15 days Yes (at supratherapeutic doses in primates). Transient Avoid in lactation
hematologic abnormalities in late pregnancy exposure
in humans.

Alemtuzumab C 12 days Animals. No human malformations seen, but thyroid Avoid in lactation
monitoring necessary for mother throughout
pregnancy. No evidence for spontaneous abortion or
birth defects.

Rituximab/ C 22 days/26 Animals. No human malformations seen; transient B- Avoid in lactation


days cell depletion in human neonates following pregnancy
Ocrelizumab
exposure

Immune System and Pregnancy

Low Estrogen
MS
Th1 IL-2
IFN
LT
Th

Th2 IL-4
High Estrogen
IL-5
IL-6
IL-10

2286
Copyright © Harvard Medical School, 2018. All Rights Reserved.

Multiple Sclerosis Relapse Rate


Annual RR P value
Year before pregnancy 0.7 --

First trimester 0.5 0.03

Second trimester 0.6 0.17

Third trimester 0.2 <0.001

1-3 months postpartum 1.2 <0.001

4-6 months postpartum 0.9 0.17

7-9 months postpartum 0.9 0.15

10-12 months postpartum 0.6 0.59

Breastfeeding

Who’s at risk?

Effects of breast feeding

Exclusive breastfeeding and the risk of postpartum relapses in women with multiple sclerosis.
Arch Neurol 2009 Aug;66(8):958-63.
Exclusive Breast feeding and the Effect on Postpartum Multiple Sclerosis Relapse. JAMA Neurol 2015 OCT:1132-1138

2287
Copyright © Harvard Medical School, 2018. All Rights Reserved.

Reproductive Health and Pregnancy Concerns

MS

Migraine

Case 2

A 23 year old women with migraine without aura


is now 8 weeks pregnant. Her migraines had been
well controlled with sumatriptan.

She’s now having her usual headaches with nausea


and vomiting 2-3 times a week.

2288
Copyright © Harvard Medical School, 2018. All Rights Reserved.

Treatment

A. Continue the sumatriptan

B. Have her take acetaminophen

C. Try caffeine and metoclopramide

D. Give her oxycodone

2289
Copyright © Harvard Medical School, 2018. All Rights Reserved.

Migraine during Pregnancy

60-70 % migraines undergo remission

Small percent of new onset migraine during


pregnancy

Increased risk of preeclampsia/ eclampsia

Migraine Treatment in Pregnancy

Planning

Symptomatic therapy

Other

2290
Copyright © Harvard Medical School, 2018. All Rights Reserved.

Symptomatic Therapies
Generic Name Level of Risk in Breastfeeding- Hale
Pregnancy Lactation Rating

Acetaminophen B L1

B (D in 3rd trimester) L1-L2


NSAIDS

Metoclopramide B L2

Prochlorperazine C L3

Dihydroergotamine X L4

Magnesium A (D) L1

Triptans C L3

Preventative Medications
Drug Class Generic Name Level of Risk in Breastfeeding
Pregnancy

Beta- blockers Atenolol D Caution


Propranolol C (D at term) Compatible

Gabapentin C Compatible
Antiepileptics Topiramate D Caution
Valproate X Caution

C Compatible
Tricyclics Amitriptyline

2291
Copyright © Harvard Medical School, 2018. All Rights Reserved.

Triptans and Pregnancy

Data from the Sumatriptan/Naratriptan Pregnancy


Registry

Data from Norwegian Mother and Child Cohort

Migraine and Lactation

Triptans considered safe

Several Preventative drugs also


safe

LactMed

2292
Copyright © Harvard Medical School, 2018. All Rights Reserved.

Pregnancy Related Disorders

Ischemic Stroke

Eclampsia

Case 3

A 28-year-old woman G2P1 at 30 weeks gestation


was last seen well at 10 am. She was found on the
ground not speaking or moving her right side at
10:50 am.
On initial exam at noon, she was mute with left
gaze deviation and dense right hemiplegia.

NIHSS* was 15.

2293
Copyright © Harvard Medical School, 2018. All Rights Reserved.

How should you treat this lady?

What imaging is most appropriate?

A. Head CT/ CTA Neck and Brain

B. Brain MRI, Vessel Imaging

Radiation Exposure

DETERMINISTIC EFFECTS

2294
Copyright © Harvard Medical School, 2018. All Rights Reserved.

Radiation Exposure

STOCHASIC EFFECTS

Imaging

Neuroimaging can be performed safely

MRI versus CT

Gadolinium is avoided

2295
Copyright © Harvard Medical School, 2018. All Rights Reserved.

2296
Copyright © Harvard Medical School, 2018. All Rights Reserved.

Use of IV t-PA in Pregnancy

Use of IA t-PA in Pregnancy

2297
Copyright © Harvard Medical School, 2018. All Rights Reserved.

Causes of Ischemic Stroke in Pregnancy

Cardiac emboli
Dissection
Pre-eclampsia/ Eclampsia
Coagulopathy
Cerebral Venous Thrombosis
Reversible Cerebral Vasoconstriction Syndrome
Other

Pregnancy Related Disorders

Ischemic Stroke

Eclampsia

2298
Copyright © Harvard Medical School, 2018. All Rights Reserved.

Case 4

A 35 year old woman G1 P0 at 31 and 5/7 weeks of


gestation was woke with a severe headache. She
began seeing visual spots, and a half hour later she
completely lost her vision.

Shortly thereafter, she developed the worst


headache of her life and blacked out. At the
outside hospital her blood pressure was 170/120.
She was transferred to our hospital.

Red Flags

• Change in headache character or pattern

• Headaches with characteristics of elevated ICP

• New headaches

• Associated with elevated BP

• Unusually severe headache

• Abnormal neurologic exam

• Headaches associated with systemic disorders

2299
Copyright © Harvard Medical School, 2018. All Rights Reserved.

Frequency and Headache Type by Trimester

Migraines

Idiopathic Intracranial Hypertension

Preeclampsia/Eclampsia

Post Dural Puncture Headache

Cerebral Venous Thrombosis

First Trimester Second Trimester Third Trimester Postpartum

Pre-eclampsia/Eclampsia Definition

Diagnosed when a pregnant woman develops


High blood pressure (two separate readings taken
at least six hours apart of 140 or more in systolic
blood pressure and/or 90 or more in diastolic
blood pressure)
300 mg of protein in a 24-hour urine sample
(proteinuria
Onset of seizures and change in mental status
defines eclampsia

2300
Copyright © Harvard Medical School, 2018. All Rights Reserved.

Pathogenesis of Eclampsia

Pre-eclampsia/ Eclampsia

PEE is associated with both significant maternal


and fetal morbidity and mortality
Maternal complications include abruption
placentae, disseminated coagulopathy, acute renal
failure, stroke, hemorrhage, death and long term
cardiovascular morbidity
Fetal complications include premature delivery,
low birth weight, hypoxic neurologic injury, and
death.

2301
Copyright © Harvard Medical School, 2018. All Rights Reserved.

Maladaptation to placental
implantation

Axial Flair MRI

2302
Copyright © Harvard Medical School, 2018. All Rights Reserved.

Treatment of Eclampsia

• Blood Pressure control


– Labetalol

• Magnesium Sulfate
– Prevention of progression from preeclampsia to
eclampsia and eclamptic seizures

Lucas et al. A comparison of magnesium sulfate and phenytoin for the prevention of eclampsia.
N Engl J Med 1995;333:201–5.

Postmenopausal Health

Stroke

2303
Copyright © Harvard Medical School, 2018. All Rights Reserved.

Case 5

A 76 year old woman with a PMH of DM has new


onset atrial fibrillation. She does not smoke and
drinks 1-2 alcoholic beverages a week. She has never
had a TIA or stroke. She is normotensive and not
overweight.

What gender specific questions should be asked?

What is her stroke risk?

Stroke in Women

Third leading cause


of death

Incidence

2304
Copyright © Harvard Medical School, 2018. All Rights Reserved.

Pregnancy complications and long term


stroke risk

Gestational diabetes

Preeclampsia/Eclampsia

Pregnancy induced hypertension

Preeclampsia and stroke risk in later life


Study date Total no. of Design OR - 95% CI
subjects

Lykke-2009 782,287 Retrospective cohort 1.36-1.66 (1.29-2.14)

Funai- 2005 37,061 Retrospective cohort 3.07 (2.18-4.33)

Kestenbaum- 2003 124,141 Case control study 2.53 (1.70-3.77)

Irgens- 2001 626,272 Retrospective cohort Preterm PE 5.0


(2.09-12.35)

2305
Copyright © Harvard Medical School, 2018. All Rights Reserved.

Atrial Fibrillation

Female sex is an independent predictor of stroke in


patients with AF

Higher risk in older women than men

2306
Copyright © Harvard Medical School, 2018. All Rights Reserved.

Women’s Issues in Neurology- expected take home points

Reproductive Health & Pregnancy MS, Migraine


Concerns

Pregnancy Ischemic Stroke, Eclampsia

Postmenopausal Stroke
Health

2307
Copyright © Harvard Medical School, 2018. All Rights Reserved.

Headache
Carolyn Bernstein M.D. FAHS
Asst. Professor of Neurology, Harvard Medical School
Associate Neurologist, Brigham and Women’s Hospital

Dr. Bernstein receives research


support from Palmer Chiropractic, and
consults for Amgen.

2308
Copyright © Harvard Medical School, 2018. All Rights Reserved.

Goals
• Understanding of diagnosis of common types
of headaches
• “red flag” recognition
• Work-up
• Treatment, medication and integrative
therapies
• Preparedness for board questions

According to article in The Telegraph ,


average attention of humans has fallen
to 8 seconds, less than a goldfish

2309
Copyright © Harvard Medical School, 2018. All Rights Reserved.

What is a headache?
• Full Definition of HEADACHE
• 1: pain in the head
• 2: a vexatious or baffling situation or
problem <meetings had become a
giant headache — Franklin Foer>

Primary vs. Secondary


• Idiopathic • “secondary” to
• Non-structural underlying disease
• Rule-out diagnosis • Tumor
• Aneursym
• Infection
• inflammatory

2310
Copyright © Harvard Medical School, 2018. All Rights Reserved.

Psychogenic headache
• Nice summary by Dr. Elizabeth Loder on HCNE
website
• Headache which is associated with psychiatric
origin
• Psychiatric disease may co-exist with other
primary and secondary headaches

Office evaluation of headache—


the essentials
• How long
• Where on head
• Quality of pain
• Positional
• Nausea, vomiting
• Photophobia/phonophobia
• Dizziness, vertigo
• Focal neurologic symptoms, especially visual
• Duration
• Degree of disability
• Family history

2311
Copyright © Harvard Medical School, 2018. All Rights Reserved.

Red Flags
“first or worst”
New and different
LOC
Focal signs
Increasingly worse
ecology

International Headache Society


Criteria
• Website at the end of talk
• Very specific organization
• Clearly defined criteria
• Useful to refer
• Lots of esoterica

2312
Copyright © Harvard Medical School, 2018. All Rights Reserved.

Focus on Primary

Location, location, location

2313
Copyright © Harvard Medical School, 2018. All Rights Reserved.

Migraine
• Unilateral
• Minimum four hours
• Throbbing pain
• Moderate to severe intensity
• Photo/phono
• Nausea or vomiting
• At least five events

Migraine with aura


• At least two attacks
• Fully reversible symptoms
• More than 5 and less than 60 minutes
• Speech, sensory or visual

2314
Copyright © Harvard Medical School, 2018. All Rights Reserved.

Cortical spreading depression—wave of depolarization

2315
Copyright © Harvard Medical School, 2018. All Rights Reserved.

Other migraine variants


• Menstrual
• “complicated”
• Hemiplegic

• Chronic

Tension Type Headache


“hatband headache”
• Ten episodes
• Bilateral
• Pressing/tightening
• Mild to moderate
• 30 min to 7 days
• Variants—infrequent/frequent/chronic
• No nausea or vomiting
• Sound or light sensitivity

2316
Copyright © Harvard Medical School, 2018. All Rights Reserved.

Chronic daily headache—


what is it?

TACs
• SUNCT
• SUNA
• Cluster
• Hemicrania

2317
Copyright © Harvard Medical School, 2018. All Rights Reserved.

SUNCT
• This syndrome is characterised by short-
lasting attacks of unilateral pain that are much
briefer than those seen in any other TAC and
very often accompanied by prominent
lacrimation and redness of the ipsilateral eye.
• 5 to 240 seconds, occuring multiple times/day

SUNA
• Attacks of unilateral orbital, supraorbital or
temporal stabbing or pulsating pain lasting
from 2 seconds to 10 minutes

2318
Copyright © Harvard Medical School, 2018. All Rights Reserved.

Cluster
Clint Eastwood Headache?
15-180 minutes
Unilateral pain, very severe, same distribution
Tearing/rhinorhea/injection/ipsi hydrosis
Meosis/ptosis

Cluster sufferers pace, migraine patients lie in


the dark with ice

hemicrania
• 2 to 30 minutes
• Indomethacin responsive
• Side-locked

2319
Copyright © Harvard Medical School, 2018. All Rights Reserved.

When to image and what to order


• Red flags
• Focal symptoms
• Age
• Positional

• Choosing Wisely article for patients

CT vs. MRI
CT Scan—blood, bone MRI
• Trauma • Acute neurologic changes
• ? SAH • ? Stroke
• Acute neurologic changes • Hydrocephalus, IIH
• ? Tumor
• ? Infection
• Imaging of posterior fossa

2320
Copyright © Harvard Medical School, 2018. All Rights Reserved.

Ask the clinical question


CALL THE RADIOLOGIST

CADASIL
• Cerebral autosomal dominant arteriopathy
with sub-cortical infarcts
• Inherited, notch 3 gene mutation
• Migraines and multiple strokes progressing to
dementia
• Cognitive deterioration, seizures, psychiatric
problems
• Genetic testing is available

2321
Copyright © Harvard Medical School, 2018. All Rights Reserved.

Syndrome
Call-Fleming Syndrome
• Can last for weeks
• Thunderclap headaches
• Focal neurologic signs
• Seizures
• Many potential triggers

2322
Copyright © Harvard Medical School, 2018. All Rights Reserved.

Labs, other tests


• Case specific—arteritis, lyme, infection
• EEG—change in LOC, odd aura
• Cardiac work-up—syncope with headache

2323
Copyright © Harvard Medical School, 2018. All Rights Reserved.

Treatment

Medication vs integrative
• Evidence-based • Evidence-based
• How to monitor • Safety
• Ecology • Cost
• Patient preference • Patient preference

2324
Copyright © Harvard Medical School, 2018. All Rights Reserved.

Guidelines--
• AAN 2012 Migraine is excellent
• Start low and titrate up for preventives
• “minimal effective dose”
• Review contraception if indicated
• Abortives--stratify

Migraine
• Antihypertensives
• Anticonvulsants
• Antidepressants
• antispasmodics
• Onabotulinum toxin

• CGRP monoclonal antibodies!!!


Newly approved, target molecule or receptor

2325
Copyright © Harvard Medical School, 2018. All Rights Reserved.

abortives
• Triptans--formulations
• Ergots—potential for side effects
• Anti-inflammatories
• Anti-nausea

• Little or no role for narcotics/barbiturates

hormonal
• ? Estrogen/progesterone formulations
• Role of HRT

2326
Copyright © Harvard Medical School, 2018. All Rights Reserved.

TTH
• All off-label
• Antidepressants
• Anticonvulsants
• Antispasmodics
• Anti-inflammatories

TACS
• Indomethacin—for hemicrania
• Triptans
• Calcium-channel blockers
• Steroids
• ?lithium

2327
Copyright © Harvard Medical School, 2018. All Rights Reserved.

procedures
• Nerve blocks
• Trigger point injections
• Onabotulinum toxin
• Transcranial magnetic stimulation
• ? Migraine surgery

Migraine headband—FDA approved

2328
Copyright © Harvard Medical School, 2018. All Rights Reserved.

Integrative therapies
• Yoga
• Mindfulness
• Tai Chi
• Herbs
• Vitamins/supplements
• Nutrition
• Acupuncture
• massage

Apps for tracking


• Iheadache
• My migraine
• migrainebuddy

2329
Copyright © Harvard Medical School, 2018. All Rights Reserved.

Case One
• Jane, age 24, grad student
• No medical problems besides headache
• Occurs several times/month
• Throbbing pain, usually behind her right eye
• Preceded by floating wedge shape that glows and
crosses from one side to the other
• Needs dark and quiet room when it happens
• Usually occurs one day before period and mid-
cycle

Diagnosis?
• Migraine with aura
• Menstrual migraine
• TIA
• Tension-type headache

2330
Copyright © Harvard Medical School, 2018. All Rights Reserved.

How would you treat?


• Continuous combined oral contraceptive
• Trial of topiramate
• Abortive plan: triptan plus NSAID
• OTC

Case Two
• Ashley, age 19, college student
• Throbbing pain over forehead and under eyes
• Worse with movement
• Light bothers her
• Slight nausea
• Runny nose, some tearing
• Very anxious about school, headaches, life

2331
Copyright © Harvard Medical School, 2018. All Rights Reserved.

Diagnosis?
• Sinus headache
• Migraine without aura
• Tension-type headache

Headache attributed to Rhinosinusitis


• Diagnostic criteria: Frontal headache accompanied by pain
in one or more regions of the face, ears or teeth and
fulfilling criteria C and D. Clinical, nasal endoscopic, CT
and/or MRI imaging and/or laboratory evidence of acute
or acute-on-chronic rhinosinusitis1;2Headache and facial
pain develop simultaneously with onset or acute
exacerbation of rhinosinusitis. Headache and/or facial
pain resolve within 7 days after remission or successful
treatment of acute or acute-on-chronic rhinosinusitis.
• Notes: Clinical evidence may include purulence in the
nasal cavity, nasal obstruction, hyposmia/anosmia and/or
fever. Chronic sinusitis is not validated as a cause of
headache or facial pain unless relapsing into an acute
stage.

2332
Copyright © Harvard Medical School, 2018. All Rights Reserved.

How would you treat?


• Preventive med
• Antibiotics
• Nasal spray
• Biobehavioral therapies

Case Three
• Tom, age 53, no headache history
• Wakes up at 4 a.m. at least half the month
with dull headache
• Pain lasts at least 15 minutes, some occasional
nausea associated
• NSAIDS are not helpful
• No autonomic symptoms

2333
Copyright © Harvard Medical School, 2018. All Rights Reserved.

Diagnosis
• Cluster
• Migraine
• Tumor
• Aneurysm
• Other

Treatment?

• Verapamil
• Lithium
• Amitryptilene
• coffee

2334
Copyright © Harvard Medical School, 2018. All Rights Reserved.

Case Four
• Christine, age 42
• Works in an office
• Frequent headache of minor to moderate
severity but annoying
• Band around her head, neck pain
• Mild phonophobia, no other symptoms
• Has been present for years
• Exam is normal other than paracervical trigger
points

Diagnosis?
• Migraine without aura
• Chronic Daily headache
• Episodic Tension-type headache
• Chronic Tension-type headache

2335
Copyright © Harvard Medical School, 2018. All Rights Reserved.

How to treat Christine


• Physical therapy
• Amitryptilene
• Muscle relaxants
• Exercise/yoga therapies
• psychotherapy

Case Five
• Anna, age 51
• Pain on top of head in very focal area, circular,
about 4 cm
• Present almost all the time
• Irritated by combing/brushing hair and
washing
• Mild to moderate severity

2336
Copyright © Harvard Medical School, 2018. All Rights Reserved.

Diagnosis
• Vasculitis
• Migraine
• Tumor
• Skin lesion
• Nummular headache

How to treat?
• Gabapentin
• Amitryptilene
• Topiramate
• clonazepam

2337
Copyright © Harvard Medical School, 2018. All Rights Reserved.

Case 6
• 47 yo woman with migraine with both visual
and sensory aura
• Never had previous complications
• Getting married
• Placed on birth control patch by her PCP for
contraception
• No FH

• Leg cramps first—”muscle spasm”


• Shortness of breath
• Right body weakness

2338
Copyright © Harvard Medical School, 2018. All Rights Reserved.

Poor outcome
• Migraine with aura
• Father had coagulopathy—Factor 5 Leiden
thrombophilia—had a number of strokes and
an MI before the age of 50
• ? Other types of birth control
• Maintained on warfarin—had persistent
increased intracranial pressure headache plus
migraines

2339
Copyright © Harvard Medical School, 2018. All Rights Reserved.

Case 7
• Hannah, college student, age 19, LH
• Occasional throbbing headache, has
scintillations followed by pain
• Studying animal science, goes to Zambia for a
semester
• Arrives on a hot day to rural village and goes
running

• Can’t see to the left


• Left arm and face tingling
• Unable to talk
• Throbbing excruciating headache 30 minutes
later

2340
Copyright © Harvard Medical School, 2018. All Rights Reserved.

TRANSFERRED TO LUSAKA

• Given some medication intravenously


• Process cleared
• Had a few headaches with scintillations after

• Came to see me about three months after


arriving home

2341
Copyright © Harvard Medical School, 2018. All Rights Reserved.

• Ecology—was on estrogen containing birth


control
• Really hot and had not been drinking much
water…

What happened?

What would you recommend?

2342
Copyright © Harvard Medical School, 2018. All Rights Reserved.

Websites
• http://www.headaches.org/pdf/MIDAS.pdf
• http://www.americanheadachesociety.org/
• http://www.headaches.org/
• http://www.achenet.org/
• http://itunes.apple.com/us/app/headache-
diary-lite/id309227463?mt=8
• http://www.ncbi.nlm.nih.gov/pubmed/22671
714

2343
Copyright © Harvard Medical School, 2018. All Rights Reserved.

Update is Ischemic Stroke

Galen V. Henderson, MD
Director of the Division of Neurocritical Care
Psychiatry Overview
Department of Neurology
Brigham and Women’s Hospital
Assistant Professor of Neurology
Harvard Medical School

I have no conflicts of interest

2344
Copyright © Harvard Medical School, 2018. All Rights Reserved.

Outline

Epidemiology

Definitions of TIA

Imaging (which type is best)

New time standard in catheter based treatment

Cardiac Echo

2345
Copyright © Harvard Medical School, 2018. All Rights Reserved.

Stroke in the US
• 795 000 people experience a new or
recurrent stroke.
– Approximately 610 000 of these are
first attacks, and 185 000 are recurrent
attacks.
• 137 000 stroke deaths annually in
the United States.
• Leading cause of serious, long-term
disability
• Third leading cause of death in the U.S.;
second leading cause worldwide
• Second-leading cause of hospital
admission among older adults
Stroke. 2011;42:849-877

Estimated Outcomes After


Ischemic Stroke
70%
Estimated %
60% Low
60%
Percentage With O utcom e

53% High
50%

40% 40% 40%


34%
30%
25% 24%
20%

10%

0%
5-yr stroke 5-yr stroke Functional Dementia at
disability (complete
recurrence mortality or partial 52 mo
dependence)

Estimated Outcomes
Adapted with permission from Sacco RL. Neurology. 1997;49(5 suppl 4):S39.

2346
Copyright © Harvard Medical School, 2018. All Rights Reserved.

Original Article
Five-Year Risk of Stroke after TIA or Minor
Ischemic Stroke
Pierre Amarenco, M.D., Philippa C. Lavallée, M.D., Linsay Monteiro Tavares, B.S.T.,
Julien Labreuche, B.S.T., Gregory W. Albers, M.D., Halim Abboud, M.D., Sabrina
Anticoli, M.D., Heinrich Audebert, M.D., Natan M. Bornstein, M.D., Louis R.
Caplan, M.D., Manuel Correia, M.D., Geoffrey A. Donnan, M.D., José M. Ferro, M.D.,
Fernando Gongora-Rivera, M.D., Wolfgang Heide, M.D., Michael G. Hennerici, M.D.,
Peter J. Kelly, M.D., Michal Král, M.D., Hsiu-Fen Lin, M.D., Carlos Molina, M.D., Jong
Moo Park, M.D., Francisco Purroy, M.D., Peter M. Rothwell, M.D., Tomas
Segura, M.D., David Školoudík, M.D., Ph.D., P. Gabriel Steg, M.D., Pierre-Jean
Touboul, M.D., Shinichiro Uchiyama, M.D., Éric Vicaut, M.D., Yongjun Wang, M.D.,
Lawrence K.S. Wong, M.D., for the TIAregistry.org Investigators

N Engl J Med
Volume 378(23):2182-2190
June 7, 2018

Study Overview/Conclusions

• In a follow-up to a 1-year
study involving patients who
had a TIA or minor stroke,
the rate of cardiovascular
events including stroke was
6.4% in the first year and
6.4% in the second through
fifth years.

2347
Copyright © Harvard Medical School, 2018. All Rights Reserved.

Prevalence of Ischemic Stroke


88%
of all strokes
are
ischemic

Ischemic Stroke
88% Intracerebral
Hemorrhage
9%

Subarachnoid
Hemorrhage
3%

American Heart Association. Heart Disease and Stroke Statistics—2005 Update. 2005.

Common Causes – Not a single


disease
•Atherothrombosis •Cardiac source
– Large-vessel – Atrial fibrillation
• Extracranial – Dilated cardiomyopathy
• Aortic
• Cervical ICA •Nonatherosclerotic
• Cervical CCA arteriopathies, eg:
• Intracranial – Vasculitis
• ICA – Migraine
• MCA
• Vertebral artery •Prothrombotic disorders
• Basilar artery
– Small vessel
• Lacunar

Helgason CM et al. Circulation. 1997;96:701-707.

2348
Copyright © Harvard Medical School, 2018. All Rights Reserved.

Risk Factors for Stroke


Modifiable Nonmodifiable
•Hypertension •Age
•Diabetes
•Cardiac disease •Gender
•Atrial fibrillation •Race/ethnicity
•TIA/prior stroke •Heredity
•Metabolic syndrome
•Dyslipidemia
•Cigarette smoking
•Alcohol abuse
•Obesity
•Physical inactivity
•Carotid stenosis

Goldstein L et al. Circulation. 2001;103:163-182.


Broderick J et al. Stroke. 1998;29:415-421.
Brown WV. Clin Cornerstone. 2004;6 Suppl 3:S30-S34.

2349
Copyright © Harvard Medical School, 2018. All Rights Reserved.

INTERSTROKE: Population-attributable
risk for common risk factors
Risk factor Population-attributable
risk, % (99% CI)

Hypertension 34.6 (30.4–39.1)


Smoking 18.9 (15.3–23.1)
Waist-to-hip ratio (tertile 2 vs tertile 1) 26.5 (18.8–36.0)
Dietary risk score (tertile 2 vs tertile 1) 18.8 (11.2–29.7)
Regular physical activity 28.5 (14.5–48.5)
Diabetes 5.0 (2.6–9.5)
Alcohol intake 3.8 (0.9–14.4)
Cardiac causes 6.7 (4.8–9.1)
Ratio of apolipoprotein B to A1 24.9 (15.7–37.1)
(tertile 2 vs tertile 1)
Psychological factors
•Stress 4.6 (2.1–9.6)
•Depression 5.2 (2.7–9.8)
*For the protective factor of physical activity, the population-attributable risks are provided for individuals who
do not participate in regular physical activity.

O'Donnell MJ et al. Lancet 2010; available at: http://www.thelancet.com.

Sleep Apnea

• OSA increased stroke rate 2-5x


• Prevalence of OSA in patient with
stroke/TIA up to 70%
• Screening
– Sleep questionnaire
– Nocturnal polysomnographic diagnostic testing

2350
Copyright © Harvard Medical School, 2018. All Rights Reserved.

Sleep Apnea
Epworth Sleepiness Scale
0=would never doze or sleep
1=Slight chance Chance of Dozing or Sleeping
2=Moderate chance Sitting and reading
3=High chance Watching TV
Sitting inactive in a public area
Being a passenger in a motor vehicle
for an hour or more
Sitting and talking to someone
Sitting quietly after lunch (no alcohol)
Stopped for a few minutes in traffic
while driving

Total score: up to “9” WNL

Prevalence of Stroke by Age


14

12
% of population

10 Men
8 Women

6
4
2
0
20–24 25–34 35–44 45–54 55–64 65–74 75+
Age range (years)

AHA. Heart Disease and Stroke Statistics—2015 Update.

2351
Copyright © Harvard Medical School, 2018. All Rights Reserved.

Total Number of Elderly by Age


Group: 1900 to 2050
Millions
80

60

40 65 or older

20 85 or older

0
1900 1950 2000 2050
Projected

Note: data for the years 2000 to 2050 are middle-series projections of the population.
Reference population: these data refer to the resident population.
US Census Bureau. Decennial Census Data and Population Projections, 2003.

Transient Ischemic Attacks


(TIAs)

Historic Definition
Temporary focal brain or retinal
deficits caused by vascular
disease that resolve within 24
hours

2352
Copyright © Harvard Medical School, 2018. All Rights Reserved.

New Definition of TIA

TIA is a brief episode of


neurologic dysfunction caused
by focal brain or retinal ischemia,
with clinical symptoms typically
lasting less than one hour, and
without evidence of acute
infarction
N Engl J Med, Vol. 337, Nov 21, 2002, 1713-1717.
Stroke, Vol 37, 2006, 577-617.
Stroke. 2009;40:2276.

Short-term Prognosis after


Emergency Department Diagnosis of TIA

30.0%
Inclusion criteria: TIA by ED physicians
25.1%
Outcome events Objective: Short-term risk of stroke
25.0% after ED diagnosis
Outcome
Measures: Risk of stroke and other
20.0% events during the 90
days
after index TIA
15.0% 12.7%
10.5% Johnston SC. et al. JAMA 2000;
10.0% 284: 2901-2906

5.0% 2.6% 2.6%

0.0%
Total Stroke Recurrent CV event Death
TIA

2353
Copyright © Harvard Medical School, 2018. All Rights Reserved.

ABCD2 of TIA
• Patients with TIA score points for each of the following factors:
• Age 60 years (1 point)

• Blood pressure 140/90 mm Hg on first evaluation (1 point)

• Clinical symptoms of focal weakness with the spell (2 points) or


speech impairment without weakness (1point)

• Duration 60 minutes (2 points) or 10 to 59 minutes (1 point)

• Diabetes (1 point).

• 2-day risk of stroke:


• 0% for scores of 0 -1
• 1.3% for 2 -3
• 3, 4.1% for 4-5
• 8.1% for 6-7 Stroke. 2009;40:2276.

Admit to the Hospital?

• Reasonable to hospitalize patients with TIA


if they present within 72 hours of the event
and any of the following criteria are
present:

–ABCD2 score of 3 or greater

–ABCD2 score of 0-2 and uncertainty that


diagnostic workup can be completed within
2 days as an outpatient
Stroke. 2009;40:2276.

2354
Copyright © Harvard Medical School, 2018. All Rights Reserved.

Working up TIA

• Neuroimaging evaluation within 24 hours of symptom


onset.
• MRI, including DWI, is the preferred brain
diagnostic imaging modality.

• Noninvasive imaging of the cervicocephalic vessels


should be performed routinely as part of the evaluation

• Noninvasive testing of the intracranial vasculature


reliably excludes the presence of intracranial stenosis

• Patients with suspected TIA should be evaluated as


soon as possible after an event

• ECG/ECHO/Holter Stroke. 2009;40:2276

Imaging approach to acute


stroke
• Four P’s
– Parenchyma
• Assess early signs of acute stroke
• Rule out haemorrhage
– Pipes
• Assess extracranial circulation (carotid & vertebral) and
intracranial circulation for evidence of intravascular thrombus
– Perfusion
• Assess Cerebral blood volume (CBV), Cerebral blood flow (CBF)
and Mean transit time (MTT)
– Penumbra
• Assess tissue at risk of dying if ischemia continues without
recanalization of intravascular thrombus

2355
Copyright © Harvard Medical School, 2018. All Rights Reserved.

Evaluation of Tissue Status:


Noncontrast Head CT
Advantages Disadvantages

• Almost universally •Often normal in


available hyperacute phase

• Rapid •Insensitive to lacunar


and posterior fossa
• High sensitivity for strokes
detection of
hemorrhage (100%
ICH, 90% SAH)

Evaluation of Tissue Status:


Multimodal MRI (including DWI)
Advantages •Disadvantages
– More sensitive to – Not universally
acute ischemia available

– More sensitive to – Longer scanning time


posterior fossa lesions
– Patient
– More sensitive to contraindications (e.g.
small vessel, lacunar pacemaker)
lesions

2356
Copyright © Harvard Medical School, 2018. All Rights Reserved.

MRI - Tissue Status: Ischemia

CT

DWI

Evaluation of Vessel Status


1. CT Angiography

2. MR Angiography

3. Ultrasound Techniques

4. Catheter Angiography

2357
Copyright © Harvard Medical School, 2018. All Rights Reserved.

CT Angiography
•Requires injection of intravenous contrast
agent

•New generation helical scanners allow rapid


evaluation of aortic arch, neck, and intracranial
vessels with 1 injection

•80-100% accuracy compared with catheter


angiography

•Disadvantages: iodinated contrast agent,


radiation exposure

CTA: Carotid Stenosis

2358
Copyright © Harvard Medical School, 2018. All Rights Reserved.

CTA: MCA Stenosis

MR Angiography
•Noninvasive means to evaluate neck and intracranial
vessels

•Time of flight technique may overestimate stenoses

•Not reliable in identifying distal or branch


intracranial occlusions

•Sensitivity and specificity 70-100% compared to


catheter angiography

•Power-injector, contrast-enhanced techniques –


increased sensitivity

•Subject to limitations of standard MRI

2359
Copyright © Harvard Medical School, 2018. All Rights Reserved.

MR Angiography

Neck MRA: Right Intracranial MRA:


Carotid Stenosis Left ICA Occlusion

Evidence-based guideline: The role of diffusion and


perfusion MRI for the diagnosis of acute ischemic stroke

• DWI should be considered superior to


noncontrast CT scan for the diagnosis of
acute ischemic stroke in patients
presenting within 12 hours of symptom onset
(Level A).
• There is insufficient evidence to support or
refute the value of PWI in diagnosing acute
ischemic stroke (Level U).
• Baseline DWI volume should be considered
useful in predicting baseline clinical
stroke severity and final lesion volume
in anterior-circulation stroke syndromes (Level
B).

NEUROLOGY 2010;75:177-185

2360
Copyright © Harvard Medical School, 2018. All Rights Reserved.

Role of CT in Acute stroke –


CT perfusion imaging
• MTT perfusion map is the most sensitive indicator of
stroke
• CBF and CBV more spicific in distinguishing ischemia from
infarction

MTT CBF CBV

2361
Copyright © Harvard Medical School, 2018. All Rights Reserved.

CT vs MRI in acute stroke


• CT is widely available and fast
–Noncontrast CT, CT angiography and CT
perfusion can be performed in under 15
minutes
–Superior to MRA in evaluating the vessels
• Less artefacts and better quantification of
lesions
• MRI stroke protocol takes longer
–Conventional MRI, DWI, MRA and PWI
–No radiation
–Can be performed without contrast
• Arterial spin labelling, time-of-flight
• The two modalities are equally useful for
evaluating acute stroke
–Equivalent depiction of the penumbra

Whics scan to order?


• Office practice MRI/MRA
• In ED and symptomatic CT/CTA
• In ED and asymptomatic MRI/MRA

• When there is a consideration of


thrombolysis IV/IA Either

• IV TPA < 4.5 hours CT is min.

2362
Copyright © Harvard Medical School, 2018. All Rights Reserved.

Acute Ischemic Stroke:


ASA/AAN/ACCP Guidelines
• Pharmacotherapies
– tPA (tissue plasminogen activator) within 3 hours of stroke
onset

– Aspirin for acute stroke (within 48 hours of symptom onset):


160 to 325 mg/day) (Grade 1A) to reduce stroke mortality and
decrease morbidity; ONLY if no contraindications or if patient
will not be given rtPA1

– Heparin and low molecular weight heparin (LMWH): not


indicated and may increase bleeding complications;

– LMWH and heparinoids may be considered for DVT


prophylaxis in at-risk patients1

• Early consultation by neurologist or stroke team critical2


1. Coull BM et al. Stroke. 2002;33:1934-1942.
2. Adams HP et al. Stroke. 2003;34:1056-1083.

IV tPA, the “Gold Standard”


• Systemic “Clot Buster”
• FDA Approved for the treatment of AIS in 1996
• Only 8% of ischemic stroke patients are eligible
for IV tPA
• Narrow time window
*Non FDA-Approved Therapies
• Risk of cerebral and systemic hemorrhage
• Achieves early reperfusion in only 13-50% of
large vessel occlusions
National Institute of Neurological Disorders and Stroke rt-PA Stroke Study Group. Tissue plasminogen activator for acute ischemic stroke. N Engl J Med.
1995;333:1581-1587. Kleindorfer DO, Broderick JP, et al. Emergency department arrival times after acute ischemic stroke during the 1990s. Neurocrit Care.
2007;7(1):31-5. del Zoppo GJ, Poeck K, Pessin MS, et al. Recombinant tissue plasminogen activator in acute thrombotic and embolic stroke. Ann Neurol
1992;32:78-86. 6. Bhatia R, Hill MD, Shobha N, et al. Low rates of acute recanalization with intravenous recombinant tissue plasminogen activator in ischemic
stroke: realworld experience and a call for action. Stroke 2010;41:2254-8.

2363
Copyright © Harvard Medical School, 2018. All Rights Reserved.

Meta-analysis Shows a Strong Correlation Between


Revascularization and Good Patient Outcomes
70%

60% 58.1%

50%
% of Patients

41.6%
40%

30% 24.8%

20% 14.4% 13.7% 12.5%


10%

0%
Good Outcome 90-Day Mortality SICH *
(mRS 0-2)

Revascularized Non-revascularized
*Differences in sICH were not statistically significant between the
revascularized and non-revascularized groups
Rha JH, Saver JL. The impact of recanalization on ischemic stroke outcome:
a meta-analysis. Stroke. 2007 Mar;38(3):967-73.

35-40% of Ischemic Strokes are Considered


“Large Vessel”
• This subset of ischemic stroke comprises blockages in the:
– Internal Carotid Artery (ICA)
– Middle Cerebral Artery (MCA)
– Vertebral / Basilar Artery

• Patient prognosis with these types of stroke is poor

Vessel Mortality Rate


ICA 53%1
MCA 27%2
Basilar Artery 89-90%3
1. Jansen O, et al.
2. Furlan A et al. PROACT II Trial
3. Brückmann H et al.

2364
Copyright © Harvard Medical School, 2018. All Rights Reserved.

Turning Point
The Era of Stent-Retrievers

Technological advances
• Stent-retriever technology for safe, reliable performance

• Significant improvement in revascularization and patient


outcomes vs older technology, proven in randomized
clinical trials*
*Non FDA-Approved Therapies

Trial Summary
mRS 0-2
Imaging Required TICI 2b/3
to Confirm Device(s) Used in Revascularization
Trial Intervention Control Odds Ratio
Occlusion Prior to Intervention Arm Rate in the
Randomization? Intervention Arm Arm Arm (95% CI)

IA Lytic (138), Merci 38% ICA


0.02
Retriever® (95), EKOS 44% M1 40.8% 38.7%
IMS III No (-0.06 to
(22), Penumbra (54), 44% M2 (N=415) (N=214)
0.09)
Solitaire FR (5) 23% multi M2
24% pen 21% pen 26% pen
Merci Retriever®,
(n=34) (n=34) (n=34)
MR RESCUE No EKOS, IA Lytic, NS
27% nonp 17% nonp 10% nonp
Penumbra
(n=30) (n=30) (n=20)

97% Stent
33% 19% 2.16
MR CLEAN Yes Retrievers, 2% other 58.7% (N=196)
(N=233) (N=267) (1.39-3.38)
Mechanical

72.4% 53.0% 29.3% 1.8


ESCAPE Yes 86% Stent Retriever
(n=156) (n=164) (n=147) (1.4-2.4)
88.0% 60.2% 35.5% 2.75
SWIFT PRIME Yes 100% Stent Retriever
(n=83) (n=98) (n=93) (1.53,4.95)
86.2% 71% 40% 4.2
EXTEND-IA Yes 100% Stent Retriever
(n=29) (n=35) (n=35) (1.3-13)

2365
Copyright © Harvard Medical School, 2018. All Rights Reserved.

Goal of Ischemic Stroke Treatment


Before Intervention

After Successful Intervention

2366
Copyright © Harvard Medical School, 2018. All Rights Reserved.

Original Article
Thrombectomy 6 to 24 Hours after Stroke with a
Mismatch between Deficit and Infarct
Raul G. Nogueira, M.D., Ashutosh P. Jadhav, M.D., Ph.D., Diogo C. Haussen, M.D.,
Alain Bonafe, M.D., Ronald F. Budzik, M.D., Parita Bhuva, M.D., Dileep R.
Yavagal, M.D., Marc Ribo, M.D., Christophe Cognard, M.D., Ricardo A. Hanel, M.D.,
Cathy A. Sila, M.D., Ameer E. Hassan, D.O., Monica Millan, M.D., Elad I. Levy, M.D.,
Peter Mitchell, M.D., Michael Chen, M.D., Joey D. English, M.D., Qaisar A.
Shah, M.D., Frank L. Silver, M.D., Vitor M. Pereira, M.D., Brijesh P. Mehta, M.D.,
Blaise W. Baxter, M.D., Michael G. Abraham, M.D., Pedro Cardona, M.D., Erol
Veznedaroglu, M.D., Frank R. Hellinger, M.D., Lei Feng, M.D., Jawad F.
Kirmani, M.D., Demetrius K. Lopes, M.D., Brian T. Jankowitz, M.D., Michael R.
Frankel, M.D., Vincent Costalat, M.D., Nirav A. Vora, M.D., Albert J. Yoo, M.D., Ph.D.,
Amer M. Malik, M.D., Anthony J. Furlan, M.D., Marta Rubiera, M.D., Amin
Aghaebrahim, M.D., Jean-Marc Olivot, M.D., Wondwossen G. Tekle, M.D., Ryan
Shields, M.Sc., Todd Graves, Ph.D., Roger J. Lewis, M.D., Ph.D., Wade S.
N Engl JM.D.,
Smith, M.D., Ph.D., David S. Liebeskind, Med Jeffrey L. Saver, M.D., Tudor G.
Jovin, M.D.,Volume 378(1):11-21
for the DAWN Trial Investigators
January 4, 2018

Study Overview

• Among patients with occlusion of a


large intracerebral vessel who had
a clinical deficit that was
disproportionately severe relative to
the infarct volume, 90-day
outcomes for disability were better
with late thrombectomy plus
standard care than with standard
care alone.

2367
Copyright © Harvard Medical School, 2018. All Rights Reserved.

Imaging Inclusion Criteria:


• < 1/3 MCA territory involved, as evidenced
by CT or MRI
• Occlusion of the intracranial ICA and/or
MCA-M1 as evidenced by MRA or CTA
• Clinical Imaging Mismatch (CIM) defined as
one of the following on MR-DWI or CTP-rCBF
maps:
– 0-<21 cc core infarct and NIHSS ≥ 10 (and age
≥ 80 years old)
– 0-<31 cc core infarct and NIHSS ≥ 10 (and age
< 80 years old)
– 31 cc to <51 cc core infarct and NIHSS ≥ 20
(and age < 80 years old)

Distribution of Scores on the Modified Rankin Scale at 90 Days.

Nogueira RG et al. N Engl J Med 2018;378:11-21

2368
Copyright © Harvard Medical School, 2018. All Rights Reserved.

Subgroup Analyses of the First Primary End Point.

Nogueira RG et al. N Engl J Med 2018;378:11-21

• For every 100 patients treated with


endovascular therapy, 49 will have a
less disabled outcome as a result of
treatment, including 36 who will be
functionally independent.

2369
Copyright © Harvard Medical School, 2018. All Rights Reserved.

Conclusions

• Among patients with acute stroke


who had last been known to be well
6 to 24 hours earlier and who had a
mismatch between clinical deficit
and infarct, outcomes for disability
at 90 days were better with
thrombectomy plus standard care
than with standard care alone.

PHANTOM-S: The Pre-Hospital Acute Neurological Therapy and


Optimization of Medical Care in Stroke Patients - Study "PHANTOM-S"

Lancet Neurol. 2012 May;11(5):397-404. doi: 10.1016/S1474-


4422(12)70057-1. Epub 2012 Apr 11.

2370
Copyright © Harvard Medical School, 2018. All Rights Reserved.

Telestroke
• AAN and ASA are supporters of
bipartisan legislation which was
recently introduced in Congress, the
Furthering Access to Stroke
Telemedicine (FAST) Act (H.R. 2799).

• This bipartisan bill expands access to


stroke telemedicine (also called
“telestroke”) treatment in
Medicare. Similar legislation (S.1465)
has been introduced in the Senate.

Lowering Blood Pressure

2371
Copyright © Harvard Medical School, 2018. All Rights Reserved.

ASA Treatment Guidelines: Ischemic Stroke Not


Eligible for Thrombolytic Therapy

BP Level
Treatment
(mm Hg)
SBP <220
OR No treatment unless end-organ involvement
DBP <120
SBP >220
OR Nicardipine or labetalol to 10% -15% ↓ in BP
DBP <121-140

DBP >140 Nitroprusside to 10% -15% ↓ in BP

ASA = American Stroke Association; IS = ischemic stroke; SBP = systolic blood pressure; DBP = diastolic blood pressure.
Adams HP, et al. Stroke. 2007;38:1655-1711.

ASA Treatment Guidelines: Ischemic Stroke


Eligible for Thrombolytic Therapy

BP Level (mm Hg) Treatment


Pretreatment Labetalol (may repeat once), nitropaste, or
SBP >185 or nicardipine
DBP >110 If BP not reduced and maintained,
do not administer rt-PA
During and after
rt-PA
SBP 180-230
OR Labetalol
DBP 105-120
SBP >230
Nicardipine or labetalol
OR
If BP not controlled, consider nitroprusside
DBP 121-140
rt-PA = recombinant tissue plasminogen activator.
Adams HP, et al. Stroke. 2007;38:1655-1711.

2372
Copyright © Harvard Medical School, 2018. All Rights Reserved.

Blood Pressure and Stroke


What to Conclude
• All studies support detection and aggressive treatment of
blood pressure for both primary and secondary
prevention

• Reduction of stroke by 35%-40% possible1

• Thiazide-type diuretic recommended as first therapeutic


agent1
• ACEI and ARBs are more effective in reducing
progression of renal disease and are recommended as
first-choice medications for patients with diabetes
1. Chobanian AV et al, and the National High Blood Pressure Education Program Coordinating Committee.
JAMA. 2003;289:2560-2572.
2. Stroke, Vol 37, 2006, 577-617.
3. Schrader J. Stroke. 2003;34:1199-1703.

Antiplatelets

2373
Copyright © Harvard Medical School, 2018. All Rights Reserved.

In patients with a history of noncardioembolic ischemic


stroke or TIA, we recommend long-term treatment with:

• Aspirin (75-100 mg once daily)


• Clopidogrel (75 mg once daily)
• Aspirin/extended-release dipyridamole (25 mg/200 mg
bid)
• Cilostazol (100 mg bid)
Over
• No antiplatelet therapy (Grade 1A)
• Oral anticoagulants (Grade 1B)
• Combination of clopidogrel plus aspirin (Grade 1B)
• Triflusal (Grade 2B)

CHEST. 2012;141(2_suppl):1S-1S. doi: 10.1378/chest.1412S1

Which antiplatelet is better?


• Of the recommended antiplatelet regimens, we
suggest clopidogrel or aspirin/extended
release dipyridamole over aspirin (Grade 2B)
or cilostazol (Grade 2C)

• Remarks: With long-term use (> 5 years), the


benefit of clopidogrel over aspirin in
preventing major vascular events may be
offset by a reduction in cancer-related
mortality with regimens that contain aspirin.

CHEST. 2012;141(2_suppl):1S-1S. doi: 10.1378/chest.1412S1

2374
Copyright © Harvard Medical School, 2018. All Rights Reserved.

Original Article
Clopidogrel with Aspirin in Acute Minor Stroke or
Transient Ischemic Attack
Yongjun Wang, M.D., Yilong Wang, M.D., Ph.D., Xingquan Zhao, M.D., Ph.D., Liping
Liu, M.D., Ph.D., David Wang, D.O., F.A.H.A., F.A.A.N., Chunxue Wang, M.D., Ph.D.,
Chen Wang, M.D., Hao Li, Ph.D., Xia Meng, M.D., Ph.D., Liying Cui, M.D., Ph.D.,
Jianping Jia, M.D., Ph.D., Qiang Dong, M.D., Ph.D., Anding Xu, M.D., Ph.D., Jinsheng
Zeng, M.D., Ph.D., Yansheng Li, M.D., Ph.D., Zhimin Wang, M.D., Haiqin Xia, M.D., S.
Claiborne Johnston, M.D., Ph.D., for the CHANCE Investigators

N Engl J Med
Volume 369(1):11-19
July 4, 2013

Primary objective of the CHANCE trial

• To assess the efficacy of a 3-month regimen of


clopidogrel-aspirin (300 mg load followed by
75 mg/day) vs. aspirin alone on reducing the 3-
month risk of new stroke (ischemic or
hemorrhagic) when initiated within 24 hours of
symptom onset in patients with high-risk TIA
or minor stroke.

N Engl J Med Volume 369(1):11-19 July 4, 2013

2375
Copyright © Harvard Medical School, 2018. All Rights Reserved.

CHANCE trial
• Age ≥ 40 years;

• Either:
Non-disabling ischemic
stroke(NIHSS≤3), or
TIA with moderate-to-high risk of
stroke recurrence (ABCD2 score ≥ 4).

• Study drug can be given within 24 h of


symptom onset.
N Engl J Med Volume 369(1):11-19 July 4, 2013

2376
Copyright © Harvard Medical School, 2018. All Rights Reserved.

CHANCE Trial Summary


• TIA and minor ischemic stroke are a treatable
emergency

• Clopidogrel with a 300 mg load plus aspirin reduces


subsequent stroke risk compared to aspirin alone.

• Clopidogrel-aspirin is safe in this setting with no


increase in bleeding.

• Even more aggressive interventions after acute TIA


and minor stroke may be indicated but require clinical
trials.

N Engl J Med Volume 369(1):11-19 July 4, 2013

2377
Copyright © Harvard Medical School, 2018. All Rights Reserved.

O r i g i na l A r t i c l e

Clopidogrel and Aspirin in Acute Ischemic Stroke and High-Risk


TIA
S. Claiborne Johnston, M.D., Ph.D., J. Donald Easton, M.D., Mary
Farrant, M.B.A., William Barsan, M.D., Robin A. Conwit, M.D.,
Jordan J. Elm, Ph.D.,
Anthony S. Kim, M.D., Anne S. Lindblad, Ph.D., and Yuko Y.
Palesch, Ph.D., for the Clinical Research Collaboration,
Neurological Emergencies Treatment Trials Network, and the
ABSTRPOINT
ACT Investigators

BACKGROUND
Combination antiplatelet therapy with clopidogrel and aspirin may reduce the rate of recurrent stroke during
the first 3 months after a minor ischemic stroke or tran- sient ischemic attack (TIA). A trial of combination
antiplatelet therapy in a Chinese population has shown a reduction in the risk of recurrent stroke. We tested
this com- bination in an international population.
METHODS
In a randomized trial, we assigned patients with minor ischemic stroke or high-risk TIA to receive either
clopidogrel at a loading dose of 600 mg on day 1, followed by 75 mg per day, plus aspirin (at a dose of 50 to 325
mg per day) or the same range of doses of aspirin alone. The dose of aspirin in each group was selected by the
site investigator. The primary efficacy outcome in a time-to-event analysis was the risk of a composite of major
ischemic events, which was defined as ischemic stroke, myocardial infarction, or death from an ischemic
vascular event, at 90 days.
RESULTS
A total of 4881 patients were enrolled at 269 international sites. The trial was halted after 84% of the
anticipated number of patients had been enrolled because the data and safety monitoring board had
determined that the combination of clopidogrel and aspirin was associated with both a lower risk of major
ischemic events and a higher risk of major hemorrhage than aspirin alone at 90 days. Major ischemic events
occurred in 121 of 2432 patients (5.0%) receiving clopidogrel plus aspirin and in 160 of 2449 patients (6.5%)
receiving aspirin plus placebo (hazard ratio, 0.75; 95% confidence interval [CI], 0.59 to 0.95; P=0.02), with
most events occurring dur- ing the first week after the initial event. Major hemorrhage occurred in 23 patients
(0.9%) receiving clopidogrel plus aspirin and in 10 patients (0.4%) receiving aspirin plus placebo (hazard ratio,
2.32; 95% CI, 1.10 to 4.87; P= 0.02).
CONCLUSIONS
In patients with minor ischemic stroke or high-risk TIA, those who received a com- bination of clopidogrel and
aspirin had a lower risk of major ischemic events but a higher risk of major hemorrhage at 90
1
days than those
n e ngl j med nejm.org
who received aspirin alone. (Funded by the National Institute of Neurological Disorders and Stroke; POINT
ClinicalTrials.gov number, NCT00991029.)

A Primary Efficacy
Outcome
10
100
90 9

8
80
7 Aspirin
6.5
70
Patients with Event (%)

6
60 Clopidogrel plus aspirin 5.0
5

50 4

40 3
No. of Patients No. with Event
2 Aspirin 2449 160
30 Clopidogrel plus Aspirin 2432 121
1 Hazard ratio, 0.75 (95% CI, 0.59–
20 0.95) P=0.02
0
0 7 30 76 90
10

0
0 7 30 76 90
Days since Randomization
No. at Risk
Aspirin 24492269 2153 2105 1365
Clopidogrel plus aspirin 24322279 2178 2113 1445

BPrimary Safety Outcome: Major Hemorrhage


100 10

9
90
No. of Patients No. with Event
8 Aspirin 2449 10
80 Clopidogrel plus Aspirin 2432 23
7
Hazard ratio, 2.32 (95% CI, 1.10–
70 4.87) P=0.02
6
Patients with Event (%)

60 5

50 4

3
40
2
30 Aspirin Clopidogrel plus aspirin
1 0.9
20 0.4
0
0 7 30 76 90
10

0
0 7 30 76 90
Days since Randomization
No. at Risk
Aspirin 2449 2372 2271 2230 1448
Clopidogrel plus aspirin 2432 2336 2256 2192 1505

Figure 2. Primary Efficacy and Safety Outcomes.


Shown are the percentages of patients with the primary efficacy outcome (a composite of ischemic stroke, myocar- dial
infarction, or death from ischemic vascular causes) (Panel A) and the primary safety outcome of major hemor- rhage (Panel
B). Inset graphs show the same data on an expanded y axis.

n e ngl j med nejm.org 7


The New England Journal of Medicine
Downloaded from nejm.org by GALEN HENDERSON on June 13, 2018. For personal use only. No other uses without permission.
Copyright © 2018 Massachusetts Medical Society. All rights reserved.

2378
Copyright © Harvard Medical School, 2018. All Rights Reserved.

Effect of statins on all strokes, fatal


stroke, and hemorrhagic stroke

Studies Relative risk


reduction
All stroke (total) 0.82 (0.77–0.87)
•All stroke (primary-prevention studies) 0.81 (0.75–0.87)
•All stroke (secondary prevention: SPARCL, HPS, LIPID, 0.88 (0.78-0.99)
and CARE)
Fatal stroke (total) 0.87 (0.73–1.03)
•Fatal stroke (primary-prevention studies) 0.90 (0.76–1.05)
•Fatal stroke (secondary prevention: SPARCL) 0.59 (0.36–0.97)
Hemorrhagic stroke (total) 1.03 (0.75–1.41)
•Hemorrhagic stroke (primary-prevention studies) 0.81 (0.60–1.08)
•Hemorrhagic stroke (secondary prevention: SPARCL 1.73 (1.19–2.50)
and HPS)

Amarenco P, Labreuche J. Lancet Neurol 2009; 8:453-463.

SPARCL
Effects of High-dose Atorvastatin After
Stroke or TIA

Placebo
Stroke or TIA (%)

Atorvastatin 80 mg qd

HR, 0.77 (95% CI, 0.67–0.88); P<0.001

Years Since Randomization


No. at Risk
Atorvastatin 2365 2148 2023 1933 1837 871 119
Placebo 2366 2132 1998 1871 1780 803 126

SPARCL=Stroke Prevention by Aggressive Reduction in Cholesterol Levels.


SPARCL Investigators. N Engl J Med. 2006;355:549-559.

2379
Copyright © Harvard Medical School, 2018. All Rights Reserved.

ACC/AHA Guidelines for the Treatment of Blood


Cholesterol in Primary Prevention

• Recommendations based on the 10


year risk for cardiovascular disease
• Shifts away from specific cholesterol
goals
• Estimated risk dictates intensity of
statin Rx: high risk mandates high
intensity statin Rx
• Atorvastatin 10 mg is moderate
intensity statin Rx and 40 t0 80 mg is
high intensity
• 10 year Risk calculator

Brain Attack Coalition


Types of Stroke Care Centers
• Primary Stroke Centers • Comprehensive Stroke
Center
– The primary stroke center
stabilizes and provides – The comprehensive stroke
emergency care to acute center provides complete
stroke patients. care to patients
experiencing the most
– Primary stroke centers complex strokes that
ultimately transfer patients require specialized testing
to a comprehensive stroke and other interventions.
center or admit patients for
further care, based on level – These centers typically
of need. include tertiary care
centers or hospitals with
appropriate infrastructure.

2380
Copyright © Harvard Medical School, 2018. All Rights Reserved.

Primary Stroke Center vs. Comprehensive


Stroke Center
Component Primary Stroke Center Comprehensive Stroke Center
Education Community, EMS, & 1 hr Nursing/Staff Education Increased Community, EMS Nursing/Staff Education; Referring hospital teaching

Performance Measures Ischemic Stroke focus of 10 quality measures Ischemic + Hemorrhagic Stroke measurement and reporting + 56 other elements of data

Patient Care Follow-up N/A Patient call within 7 days of discharge


Modified ranking scall for all intervention patients 90-days post discharge
Patient satisfaction data collection

Stroke Research N/A Patient centered stroke research w/ IRB approval


Participation from advanced practice nurse

Volume Requirements N/A 20 Aneurysm cases annually


15 endovascular procedures annually
25 tPA patients annually

Ultrasound, angiography, CT, Ultrasonography,


Advanced Imaging Access to Imaging MR/MRA, TCD, TEE/TTE

Stroke Director, Emergency MD, Neuro-


Stroke Director, Program Manager, access to Interventionalist, Neuroradiologist, CV
MD Staffing Neurology and Neurosurgery neurosurgeon, vascular surgeon

Doctorate-prepared NP with leadership,


Advances Practice Nursing N/A education, and research responsibilities

Dedicated Neuro-ICU, IR suite to accommodate


two complex stroke patients; EMS coordination
Other Assets/Resources N/A and documentation

Questions for the future


• Which out-of-hospital stroke scale to use for identifying large
artery occlusions?

• Would telemedicine directed by a remote stroke physician be


a better triage technique?

• Should primary stroke centers be bypassed to transport


patients to comprehensive centers, even if it means delaying
the start of intravenous tissue plasminogen activator (tPA)?

• How much delay in bypass is acceptable?

• Would a mobile stroke unit with computed tomographic

2381
Copyright © Harvard Medical School, 2018. All Rights Reserved.

Impact of PROTECT pilot phase on


Treatment Rates at Discharge
Pre PROTECT

100
90
P 80
e 70
r 60
c 50
e 40
n 30
t 20
10
0
Antithrombotic Statin ACEI/ARB Thiazide

--Ovbiagele et al, Stroke 2003

2382
Copyright © Harvard Medical School, 2018. All Rights Reserved.

Evidence of Better Outcomes in


Stroke Centers
• Stroke unit trialists’ collaboration meta-analysis
– OR death: 0.82 (0.69, 0.98)
– OR death/inst: 0.76 (0.64, 0.90)
– OR death/dep: 0.71 (0.61, 0.84)

• In-hospital death less frequent in facilities with


vascular neurologist; adjusted OR=0.49, P<0.0001

• Trend toward fewer deaths in facilities with dedicated


stroke team available by pager

• JCAHO credentialing of stroke centers ensures that


patients receive proper care
– www.jointcommission.org/
Indredavik B, et al. Stroke. 1999;30:917-923. Goldstein LB, et al. Neurology. 2003;61:792-796.
Indredavik B, et al. Stroke. 1997;28:1861-1866. Alberts MJ, et al. JAMA. 2000;283:3102-3109.
Indredavik B, et al. Stroke. 2000;31:2989-2994. Gillum LA, et al. Stroke. 2001;32:2137-2142.

Work-up of TIA and Ischemic Stroke


All Patients •Lipids
•Brain Imaging
•Neurovascular Selected Patients
imaging – Hepatic functions
•Blood glucose – Toxicology
•Serum electrolytes – Blood alcohol level
•CBC w/ Platelets – Pregnancy
•PT/PTT/INR – Hypercoagulable w/u
•12 lead EKG/ROMI – EEG
•Holter monitoring – LP
•TTE/TEE
•Supplemental O2
•Fever reduction

2383
Copyright © Harvard Medical School, 2018. All Rights Reserved.

Original Article
Atrial Fibrillation in Patients with Cryptogenic
Stroke
David J. Gladstone, M.D., Ph.D., Melanie Spring, M.D., Paul Dorian, M.D., Val
Panzov, M.D., Kevin E. Thorpe, M.Math., Judith Hall, M.Sc., Haris Vaid, B.Sc., Martin
O'Donnell, M.B., Ph.D., Andreas Laupacis, M.D., Robert Côté, M.D., Mukul
Sharma, M.D., John A. Blakely, M.D., Ashfaq Shuaib, M.D., Vladimir Hachinski, M.D.,
D.Sc., Shelagh B. Coutts, M.B., Ch.B., M.D., Demetrios J. Sahlas, M.D., Phil
Teal, M.D., Samuel Yip, M.D., J. David Spence, M.D., Brian Buck, M.D., Steve
Verreault, M.D., Leanne K. Casaubon, M.D., Andrew Penn, M.D., Daniel
Selchen, M.D., Albert Jin, M.D., David Howse, M.D., Manu Mehdiratta, M.D., Karl
Boyle, M.B., B.Ch., Richard Aviv, M.B., Ch.B., Moira K. Kapral, M.D., Muhammad
Mamdani, Pharm.D., M.P.H., for the EMBRACE Investigators and Coordinators

N Engl J Med
Volume 370(26):2467-2477
June 26, 2014

Study Overview

• In this study, patients with


cryptogenic stroke who were
randomly assigned to undergo
intensive ECG monitoring for 30
days had a higher incidence of
detected atrial fibrillation (16%)
than those assigned to receive
standard 24-hour monitoring (3%).

2384
Copyright © Harvard Medical School, 2018. All Rights Reserved.

Incremental Yield of Prolonged ECG Monitoring for the Detection of Atrial Fibrillation in
Patients with Cryptogenic Stroke or TIA.

Gladstone DJ et al. N Engl J Med 2014;370:2467-2477

Conclusions

• Among patients with a recent cryptogenic


stroke or TIA who were 55 years of age or
older, paroxysmal atrial fibrillation was
common.

• Noninvasive ambulatory ECG monitoring for


a target of 30 days significantly improved the
detection of atrial fibrillation by a factor of
more than five and nearly doubled the rate of
anticoagulant treatment, as compared with
the standard practice of short-duration ECG
monitoring.

2385
Copyright © Harvard Medical School, 2018. All Rights Reserved.

Discharged with:
•Blood pressure control
–Diabetics ACEI/ARBs

•Antiplatelets

•Statins

•Lifestyle changes

2386
Copyright © Harvard Medical School, 2018. All Rights Reserved.

Summary

Epidemiology

Definitions of TIA

Imaging (which type is best)

New time standard in catheter based treatment

Cardiac Echo

I have no conflicts of interest

2387
Copyright © Harvard Medical School, 2018. All Rights Reserved.

Question 1
Mr. Jones has 3 hours of sudden onset
dysarthria and arm/hand weakness
and then symptoms completely
resolve.
Is this a:

A. Stroke
B. TIA
C. RIND
D. Complicated migraine

Answer to Question 1
• Then answer is A.

• The definition of TIA is < 1 hour and


negative imaging

• Reversible ischemic neurologic deficit


is no longer a valid term

• Migraine symptoms are not sudden

2388
Copyright © Harvard Medical School, 2018. All Rights Reserved.

Question 2
What is the most practical cerebral
imaging study within 3 hours of
stroke signs and symptoms?

A. CT of brain
B. CT of brain/CT angiogram of head
and neck
C. MRI of brain
D. MRI of brain/MRA of head and neck

Answer to Question 2
• The answer is B.

• CT/CTA scans seem to to be the most


practical cerebral imaging study in the
EDs, ICUs, and for other inpatients.

2389
Copyright © Harvard Medical School, 2018. All Rights Reserved.

Question 3
Mr. Jones 83 years old right handed
male has a NIHSS of 12, had a
witnessed onset of his stroke and is
within 2 hours with a neg. CT

A. Too old to consider to give TPA


B. Give ASA only
C. If there are no protocol exclusions,
give TPA
D. Give Clopidogrel only

Answer to Question 3
• The answer is C.

• In giving IV TPA within 3 hours, there


is no age cutoff.

• ASA or Clopedigril should be given


ASAP only if the patient is not a TPA
candidate

2390
Copyright © Harvard Medical School, 2018. All Rights Reserved.

Question 4
Which is the most appropriate
antiplatelet therapy for
noncardioembolic stroke?

A. Aspririn
B. Clopidogrel
C. Asp/dyp combination
D. Asp/Clopidogrel
E. Any listed above

Answer to Question 4
• The answer is E.

• Ischemic stroke is a very


heterogeneous disease and the
treatment regimen depends on the
etiology of the patient’s ischemic
event.

2391
Copyright © Harvard Medical School, 2018. All Rights Reserved.

Question 5
How do I workup the patient with the
diagnosis of TIA?

A.Cardiac Echo
B.EKG/Holter
C.Brain imaging (CT or MRI)
D.Vascular imaging (CTA or MRA)
E.All of these above

Answer to Question 5
• The answer is E.

• Echo to r/o PFO and to evaluate


ejection fraction.

• EKG and Holter to r/o Afib.

• We always need brain imaging and


neurovascular imaging

2392
Copyright © Harvard Medical School, 2018. All Rights Reserved.

Thank you for your Attention

2393
Copyright © Harvard Medical School, 2018. All Rights Reserved.

Seizure Disorders

Tracey A. Milligan, MD, MS, FAAN


Distinguished Clinician
Psychiatry Overview
Vice Chair for Education
Edward B Bromfield Epilepsy Center
Department of Neurology
Brigham and Women’s Hospital
Assistant Professor of Neurology
Harvard Medical School

No Disclosures

2394
Copyright © Harvard Medical School, 2018. All Rights Reserved.

Epilepsy is Common: 1 in 26
Diagnosis US Prevalence US New Cases/yr
Migraine 28 million

Alzheimer’s Dementia 5.4 million 454,000

Stroke 4.4 million 700,000

Epilepsy 3.4 million 300,000 / 200,000


Parkinson’s disease 500,000 50,000

Multiple sclerosis 400,000 10,000

https://www.cdc.gov/epilepsy/data/index.html

Definitions

• Seizure: the clinical


manifestation of an
abnormal and excessive
excitation of a population
of cortical neurons

• Epilepsy: a tendency
toward recurrent seizures
unprovoked by systemic or
neurologic insults

2395
Copyright © Harvard Medical School, 2018. All Rights Reserved.

Symptomatic Seizures
• Close temporal relationship to
acute brain injury
or
• Simultaneously due to metabolic,
toxic, infections, inflammatory
process

• Clinically and on EEG look like


epileptic seizures (typically GTC)
• Different pathogenesis, therapy,
prognosis

Organ Dysfunction
Cardiac
• focal seizures from cardioembolic stroke
• generalized tonic-clonic or nonconvulsive or
myoclonic seizures from global cerebral ischemia
after cardiac arrest

Renal
• Common in acute uremia
• 7-10 days after onset of renal failure
• Typically bilateral tonic-clonic, but can be focal
• Uncommon in chronic renal insufficiency
• Treat by correcting metabolic abnormalities and
blood pressure if hypertensive encephalopathy
• Increased risk with penicillin

Hepatic
• Uncommon in acute hepatic encephalopathy and
chronic liver disease
• Check for hypoglycemia as a cause
• Acute intermittent porphyria – frequently
associated with seizures and epilepsy

2396
Copyright © Harvard Medical School, 2018. All Rights Reserved.

• Alcohol dependence:
10.2% among excessive
drinkers (>8 drinks per
week in 30 days) and
10.5% among binge
drinkers (>4 drinks per
occasion
• 50% will experience
symptoms of
alcohol withdrawal
upon reduced
alcohol intake
• 2 million episodes of
ETOH withdrawal per year
in US
• 5-10% of
patients have a
seizure
• 8% of all patients
Alcohol Withdrawal admitted to the hospital
• 16-31% of patients in ICU
Seizures • Up to 31% of trauma
patients

Uusaro A, Parviainen I, Tenhunen JJ, et al. The proportion of intensive care unit admissions related to
alcohol use: a prospective cohort study. Acta Anaesthesiol Scand. 2005;49:1236-1240

The Alcohol Withdrawal Syndrome

Victor, M. & Adams, R.D. Res Publ Assn Nerv Ment Dis 32, 526-573 (1953).

2397
Copyright © Harvard Medical School, 2018. All Rights Reserved.

Alcohol Withdrawal
Seizures
Typically occur after years of drinking
and multiple episodes of withdrawal.
Onset 12-48 after last drink
Generalized tonic-clonic
Multiple, within 6 hours
Post-ictal drowsiness < 1 hour
Treat with benzodiazepine or
barbiturate (dosed by CIWA)

Victor and Brausch; Epilepsia 1967; 1-20

Antibiotics
• Unsubstituted penicillins
• 4th generation
cephalosporins
(cefepime)
• Imipenem
• Ciprofloxacin
• In combination with
renal dysfunction, brain
lesions and epilepsy
• Isoniazid (pyridoxine
deficiency – treat with
B6 and benzodiazepine)

2398
Copyright © Harvard Medical School, 2018. All Rights Reserved.

Bupropion
• Incidence is correlated with dose
o Incidence is 0.4% at doses of 300-450mg/day
but increases by 100 fold in doses > 600 mg/day
o Sustained release formulation has lower
incidence due to lower peak plasma
concentrations
o By comparison
-0.1% with SSRIs
- 0.4-2 % with TCAs
• Anorexia and bulimia further increase the risk
(a randomized trial in which bupropion was
given to 55 nondepressed presents induced a
GTC in 4 patients – 7 percent).

Davidson J J Clin Psyciatry 1989

Sodium
• Hyponatremia
• Symptoms relate to rate
of change rather than
absolute value
• Hypernatremia
Electrolyte • Seizure may occur during
rehydration
Abnormalities Calcium
• Hypocalcemia
• Focal seizures in 20% of
patients, accompanied by
altered mental status and
tetany
• Hypercalcemia
• Infrequently causes
seizures
Magnesium
• Hypomagnesemim (< 0.8
mEq/L)
• Multifocal and generalized
seizures

2399
Copyright © Harvard Medical School, 2018. All Rights Reserved.

Conceptual Definition of Epilepsy

Epilepsia, 46(4):470-472, 2005


Blackwell Publishing, Inc. © 2005 International League Against Epilepsy

Epileptic Seizures and Epilepsy: Definitions Proposed By the International


League Against Epilepsy (ILAE) and the International Bureau for Epilepsy
(IBE).

Fisher RS, van Emde Boas W, Blume W, Elger C, Genton P, Lee P, Engel J Jr.

Epilepsy is a disorder of the brain characterized by an enduring predisposition


to generate epileptic seizures, and by the neurobiologic, cognitive,
psychological, and social consequences of this condition. The definition of
epilepsy requires the occurrence of at least one epileptic seizure.

ILAE OFFICIAL
REPORT

Epilepsy is a disease of the brain defined by any of the following conditions

1. A least two unprovoked (or reflex) seizures occurring >24 h apart

2. One unprovoked (or reflex) seizure and a probability of further seizures similar to
the general recurrence risk (at least 60%) after two unprovoked seizures, occurring
over the next 10 years

3. Diagnosis of an epilepsy syndrome

Epilepsy is considered to be resolved for individuals who had an age-dependent epilepsy syndrome but
are now past the applicable age or those who have remained seizure-free for the last 10 years, with no
seizure medicines for the last 5 years. Can Dx and Tx for Epilepsy after
ONE seizure

2400
Copyright © Harvard Medical School, 2018. All Rights Reserved.

Classification System
• Designed to provide greater diagnostic specificity
for treatment and research
• Change in terminology used to describe seizure
type and to describe etiology

2401
Copyright © Harvard Medical School, 2018. All Rights Reserved.

Instruction manual for the ILAE 2017 operational classification of seizure types

Epilepsia
8 MAR 2017 DOI: 10.1111/epi.13671
http://onlinelibrary.wiley.com/doi/10.1111/epi.13671/full#epi13671-fig-0002

ILAE classification of the epilepsies: Position paper of the ILAE Commission for
Classification and Terminology

Epilepsia
8 MAR 2017 DOI: 10.1111/epi.13709
http://onlinelibrary.wiley.com/doi/10.1111/epi.13709/full#epi13709-fig-0001

2402
Copyright © Harvard Medical School, 2018. All Rights Reserved.

Case 1: Status Epilepticus

• 36 y.o. man with a PMH of a penetrating head injury 10 years


previously resulting in behavioral and cognitive deficits
• While having dinner with his mother he has a “fit” and has 2
seizures later that night
• He does not receive treatment and 3 months later has a severe
seizure
• He receives treatment, but he continues to have seizures and
dies 2 days later

Status Epilepticus

Treatment
Bloodletting

2403
Copyright © Harvard Medical School, 2018. All Rights Reserved.

The year is 1860


The patient is Phineas Gage

Status epilepticus

• 30 minutes of either:
– Continuous seizure activity
– Repetitive seizures without recovery in between
Epilepsy Foundation of America –
JAMA 1993

• 5 minutes or more of continuous clinical and/or electrographic


seizure activity or recurrent seizure activity without recovery
between seizures

Brophy, Gretchen M., et al. "Guidelines for the evaluation and management of
status epilepticus." Neurocritical care 17.1 (2012): 3-23.

2404
Copyright © Harvard Medical School, 2018. All Rights Reserved.

How do you treat Status Epilepticus?

Use algorithm

Lowenstein, DH, Alldredge, BK Status Epilepticus


N Engl J Med 1998 338: 970-976

2405
Copyright © Harvard Medical School, 2018. All Rights Reserved.

Status Epilepticus Pearls

• A seizure lasting 5 minutes is a medical emergency


• Treat while looking for cause
• Faster treatment is more effective treatment
• 1st line benzodiazepine
• 2nd line fosphenytoin

2406
Copyright © Harvard Medical School, 2018. All Rights Reserved.

Case 2: Epilepsy in the Elderly

A 75 year old woman is brought to the physician for episodes of


confusion. She has weekly episodes of feeling unwell and staring
into space for 30-60 seconds followed by mild confusion. She
has a history of hypertension, tobacco use and was recently
diagnosed with lung cancer. Her examination is normal. Which of
the following is the most likely diagnosis?

A. Generalized seizure disorder


B. Focal motor seizure
C. Dementia
D. Psychogenic non-epileptic seizure
E. Absence seizure
F. Focal seizure with impaired awareness

Classification System: 2 Seizure Categories

Focal Generalized
originate within originate at some
networks limited point within, and
rapidly engaging,
to one bilaterally distributed
hemisphere (may networks (can include
be discretely cortical and
localized or more subcortical structures,
but not necessarily
widely include the entire
distributed) cortex)

2407
Copyright © Harvard Medical School, 2018. All Rights Reserved.

Videos
Generalized Absence Seizures (=Petit Focal Impaired Awareness (=Complex
Mal) Partial)
• Children • Can occur at any age
• Easily controlled • 2/3 controlled
• EEG shows generalized spike wave • EEG shows focal discharges
discharges • MRI is required, may be abnormal
• MRI is normal • Variable prognosis
• Excellent prognosis

Focal impaired awareness evolving to bilateral convulsions

2408
Copyright © Harvard Medical School, 2018. All Rights Reserved.

Seizure Classification

Focal Generalized

Focal Aware Focal Impaired


Awareness
(simple partial)
(complex partial)
Evolve to bilateral convulsions

And others
Absence Tonic Clonic Myoclonic (clonic, tonic,
atonic)

International League Against Epilepsy (ILAE)


Idiopathic Genetic
Symptomatic
Structural/Metabolic
Cryptogenic
Generalized and Focal will not apply
to electroclinical syndrome
FOCAL = network limited to 1
hemisphere
GENERALIZED = bilaterally
distributed networks

2409
Copyright © Harvard Medical School, 2018. All Rights Reserved.

Practice Guidelines
• EEG - after a first seizure an EEG should be
considered as part of the routine
neurodiagnostic evaluation because it has
substantial yield and has value in determining
risk of seizure recurrence (Level B).

• Neuroimaging - brain imaging using CT or


MRI should be considered as part of the
neurodiagnostic evaluation of adults
presenting with an apparent unprovoked first
seizure (Level B).
AAN Practice Parameter 2007

Which of the following are true?

A. A normal EEG excludes the diagnosis of epilepsy


B. The EEG will show generalized seizure activity
C. Focal slowing on the EEG helps to localize the anatomic origin
of the seizures
D. A normal MRI excludes seizures as a diagnosis

2410
Copyright © Harvard Medical School, 2018. All Rights Reserved.

EEG

• Assess for epileptiform discharges (30%,


increases to 80-90% with 3 EEGs)
• Help diagnose presence, type, and location
of epilepsy
• Negative EEG does not rule out epilepsy
• Sleep deprived EEG more sensitive
(increase yield by 30%)
• EEG after spell or seizure is more sensitive
(51% vs 30%)
• Extended monitoring: video EEG or
ambulatory EEG

MRI in New-Onset Seizures


• 77% of 993 Consecutive patients in a 1st seizure
clinic (14-94 yoa) had an MRI
– 28% had potentially epileptogenic lesions (1/2 gliosis or
encephalomalacia) (1/2 developmental abnormalities,
vascular, tumors, MTS)
• Highest in those with focal seizures (53%) (p < 0.001)
– Gliosis or Encephalomalacia (49%)
– Tumors (15%)
– Cavernomas (9%)
– MTS (9%)
• Only 16% of those with potentially epileptogenic lesions
had epileptiform EEG abnormalities
Hakami T, et al. Neurology 2013
– 22% had other MRI abnormalities unrelated to seizures

2411
Copyright © Harvard Medical School, 2018. All Rights Reserved.

Late-onset epilepsy

Common Causes

Cerebrovascular disease >50%

Neurodegenerative disease 10-20%

Intracerebral tumors 10-30%

Traumatic brain injury 20%

Brodie et al, Lancet Neurol 2009; 8: 1019–30

Age and Etiology of Epilepsy

When etiology is known:


Older Adults (>60 years)

Adapted from Hauser WA. Epilepsia. 1992;33(suppl 4):S6-S14.

2412
Copyright © Harvard Medical School, 2018. All Rights Reserved.

Many Antiepileptic Drugs (AEDs)


Older Medications Newer Medications
(1st Generation) (2nd Generation)
Year Trade FDA Trade
Generic Name Generic Name
Introduced Name Approval Name
Bromides 1857 —
Felbamate 1993 Felbatol®
Gabapentin 1993 Neurontin®
Phenobarbital 1912 Luminal®
Lamotrigine 1994 Lamictal®
Phenytoin Sodium 1956 Dilantin® Topiramate 1996 Topamax®
Ethosuximide 1960 Zarontin® Tiagabine 1997 Gabitril®
Diazepam 1963 Valium® Levetiracetam 1999 Keppra®
Carbamazepine 1968 Tegretol®
Oxcarbazepine 2000 Trileptal®
Zonisamide 2000 Zonegran®
Lorazepam 1977 Ativan® Pregabalin 2005 Lyrica®
Valproic acid 1978 Depakene® Lacosamide 2008 Vimpat®
Divalproex 1983 Depakote® Rufinamide 2008 Banzel®
sodium
Ezogabine 2011 Potiga®
Carbamazepine 1986 Epitol®
Clobazam 2011 Onfi®
Diazepam 1997 Diastat®
Perampanel 2012 Fycompa®
Carbamazepine 1997 Carbatrol®
Eslicarbazepine 2014 Aptiom®
Phenytoin Sodium 1998 Phenytek® Brivaracetam 2016 Briviact®

Many Antiepileptic Drugs (AEDs)

Older Medications Newer Medications


(1st Generation) (2nd Generation)

Trade
Generic Name Generic Name FDA Approval
Name

Gabapentin 1993 Neurontin®


Phenobarbital

Lamotrigine 1994 Lamictal®

Phenytoin Sodium
Topiramate 1996 Topamax®

Carbamazepine Levetiracetam 1999 Keppra®

Oxcarbazepine 2000 Trileptal®


Divalproex sodium
Zonisamide 2000 Zonegran®

Pregabalin 2005 Lyrica®

Lacosamide 2008 Vimpat®

2413
Copyright © Harvard Medical School, 2018. All Rights Reserved.

Summary AEDs in the Elderly


• Lamotrigine most effective and tolerable
• Levetiracetam not studied in RCT but seems effective in
retrospective studies
• Oxcarbazepine and Carbamazepine not well tolerated
• General recommendation:
• Slow titration to Lamotrigine to 50mg BID or Levetiracetam to
500mg BID
• Elderly patients tend to achieve seizure control at lower AED
dosages and lower serum AED levels

Buchsbaum et al, Arch Neurol. 2010;67(4):408

A. J. Rowan et al. Neurology 2005;64:1868-1873

Epilepsy in the Elderly- Pearls

• Epilepsy most commonly occurs in the elderly


• New onset epilepsy is focal epilepsy
• EEG and MRI may be normal
• Monotherapy – low dose lamotrigine, levetiracetam,
gabapentin
• Newer AEDs have less side effects and fewer drug-drug
interactions
• Have a high index of suspicion for episodes of altered
awareness, confusion, falls, loss of consciousness

2414
Copyright © Harvard Medical School, 2018. All Rights Reserved.

Case 3 – First Seizure?

• Emily is brought to the ED after a witnessed GTC


• She was with a friend at a coffee shop the morning after a
concert
• At the concert, she drank ETOH
• She did not sleep that night
• Her friend noticed jerking movements and then Emily had a
GTC
• Emily feels tired and sore and doesn’t remember anything
• Her examination is normal

After 1 unprovoked seizure: 21%-45% risk of


another in 2 years (especially after the first year)

Lower Risk with AED


Higher Risk
• Decreases AR by 35%
• Prior brain lesion or insult
(2y)
(e.g., stroke or trauma)
• QOL (and risk of harm)
• Epileptiform EEG
may not be affected
• Significant brain-imaging
• Long term px not
abnormality
affected
• Nocturnal seizure
• ADE: 7-31% (usually
mild and reversible)
Treat after second seizure because of higher risk (57% by 1 year and 73% by 4 years)

2415
Copyright © Harvard Medical School, 2018. All Rights Reserved.

Syncope vs Seizure vs Psychogenic

2416
Copyright © Harvard Medical School, 2018. All Rights Reserved.

Clinical Feature Syncope Seizure

Syncope vs. Seizure


Loss of consciousness Typical Common

Episode duration Seconds Minutes

Involuntary movements Common Typical

Triggers Frequent Rare

nausea, blurred vision, feeling


Preceding Symptoms hot, tinnitus, palpitations sensory, motor, psychic auras

Amnesia for event, Amnesia for event plus confusion,


Post-Ictal somnolence, headache somnolence, headache

EEG Slow waves, flattening Focal or generalized spike-wave

2417
Copyright © Harvard Medical School, 2018. All Rights Reserved.

Psychogenic Nonepileptic Seizures

• 10-45% of refractory epilepsy


(referral centers); minority have both NES and ES
• Females>males
• Psychiatric mechanism —
dissociation, conversion, abuse
• Often requires video-EEG monitoring

49

Psychogenic Nonepileptic Seizures

Clinical Feature Psychogenic Nonepileptic Seizures Epilepsy

Trigger Frequent Rare

Onset Often gradual Usually sudden

May stop and stop, pelvic thrusting, back arching, erratic movements Usually synchronized
Movements
and absence of stereotypy and stereotyped

Eyes Closed Open

Lateral tongue bite Rare Common

Self-injury Rare Common

Incontinence Rare Common

Post-ictal confusion Rare Common

Duration lengthy (hours) 1-2 minutes

Serum prolactin usually normal usually elevated

2418
Copyright © Harvard Medical School, 2018. All Rights Reserved.

Pseudo-pseudoseizure
(Frontal Lobe Epilepsy)
• When it might be a seizure, but sounds bizarre, think frontal lobe
epilepsy
• Complex behaviors with motor agitation, strong emotional feelings,
repetitive motor activity involving pelvic thrusting, pedaling, thrashing
• Often accompanied by vocalizations or laughter/crying
• Often bizarre and misdiagnosed as psychogenic
• Tend to be frequent and brief, with less post-ictal confusion
• Clues: stereotyped semiology, occurrence during sleep, brief in
duration, abnormal MRI

2419
Copyright © Harvard Medical School, 2018. All Rights Reserved.

Self-Reported Symptoms
Less Frequent More Frequent

Syncope PNES Epilepsy


After: I feel drained or sleepy
I want to know what happens when I have blacked out
My attacks come on without any warning
During: I have no idea what is happening around me
After: I feel very upset
After: I feel very confused
In: I feel lightheaded, like I might pass out
After: my muscles ache
My attacks make me feel very low or empty
My attacks come over me
When I feel an attack coming on I try to fight it
In: sounds are distorted
In: my vision goes dim or dark
In: I feel hot or cold
After: I feel relieved
In: I am conscious but I can’t react to things
In: my mouth goes very dry
My attacks are associated with emotional stress
I am aware of a trigger for my attacks
During: my heart pounds and I feel shaky and sweaty
During: I feel very frightened
During: I can see or hear the people around me
My head spins in my attacks
During: I do not recognize my friends or family
In: everything seems to move away from me

Reuber, M et al. 2016. Neurology

1. Was it really a seizure?


(open ended questions, eye-witness, pre- & post-ictal clues)

2. Is it a provoked seizure?
(alcohol or drugs, etc.)

Key 3. Is it the first seizure?


Questions (myoclonus, prior auras, altered awareness, photosensitive)

Guiding 4. Localization/Type of seizure


the
History 5. Cause of seizure

6. Epilepsy syndrome?

2420
Copyright © Harvard Medical School, 2018. All Rights Reserved.

Epilepsy Syndromes
• Defined typical characteristics in addition to seizure
type
• Age, location, EEG, treatment, prognosis
• Examples include: Childhood absence epilepsy
(CAE), juvenile myoclonic epilepsy (JME), and
benign rolandic epilepsy (BRE)

• Spectrum of action (broad vs.


narrow)
• Parenteral
Anti-Epileptic administration/loading options
• Pharmacokinetics and drug
Drug (AED) interactions
• Inducers: carbamazepine,
Considerations phenytoin, phenobarbital
• Inhibitors: valproate,
felbamate
• Protein bound: valproate,
phenytoin

• Concomitant disease
• Birth control

Broad Spectrum – use when you don’t • Likely adverse effects


know the type or cause of seizure – • Efficacy
levetiracetam, lamotrigine, valproate, • Cost
topiramate, zonisamide

2421
Copyright © Harvard Medical School, 2018. All Rights Reserved.

Idiosyncratic: usually systemic/allergic

• Rashes
• Organ toxicity
AEDs: Dose-related: usually CNS

Adverse • Dizziness, drowsiness, ataxia


• Cognitive slowing, depression (may also be

Effects idiosyncratic)

Chronic: drug-induced diseases

• Osteoporosis

DO NOT START:
Carbamazepine
351% Oxcarbazepine
Phenytoin
increased risk
Lamotrigine
(Eslicarbazepine)

2422
Copyright © Harvard Medical School, 2018. All Rights Reserved.

Bone Health
Increased risk of fracture in
Decrease in bone density people with epilepsy
• Phenytoin
• Primidone
• Phenobarbital
• Carbamazepine
• Valproic acid

Significant bone loss in men too


Supplement with vitamin D and more frequent evaluation of bone
mineral density
Neurology 2003; 61: S2-17; Adnress DL et al,
Arch Neurol, 2002

Women With
Epilepsy
OCP’s less effective: phenytoin,
phenobarbital, carbamazepine,
oxcarbazepine, eslicarbazepine,
topiramate
OCP’s lower lamotrigine level and
can cause breakthrough seizures
• Folic acid 1 mg daily
• Check vitamin D level
• Increased risk of osteopenia with
phenytoin, phenobarbital,
carbamazepine, valproate

2423
Copyright © Harvard Medical School, 2018. All Rights Reserved.

Teratogenic Risk
Profiles of
Antiepleptic Drugs

Pennell PB. Neurotherapeutics 2016.

Driving
• Epilepsy may account for 0.02% to
0.04% of reported car crashes
• Required seizure-free intervals vary
greatly among jurisdictions (typically
3 to 12 months)
• Mandatory physician reporting:
CA, OR, PA, DE, NV, NJ
• State driver licensing laws available
at
http://www.epilepsyfoundation.org
• Discuss driving with patient and
document in medical record

2424
Copyright © Harvard Medical School, 2018. All Rights Reserved.

Start with possibility it was not a


seizure
Look for provoking causes

50% of patients with 1st seizure


1st Seizure have epilepsy

Pearls Family hx, neuro exam, MRI, EEG


are key for prognosis
Start AED in some cases

Counsel regarding safety and no


driving

• No seizure for >5 years


• Favorable factors
• control on one
Discontinuing drug at low dose
• no unsuccessful
AEDs attempts at
withdrawal
• Normal neurologic
exam, EEG,
“benign
syndrome”
• Consider risks/benefits
(e.g. driving,
Epilepsypredictiontools.info pregnancy, job issues)
• Always taper

Lamberink HJ et al. Lancet Neurology 2017; 16(7):523

2425
Copyright © Harvard Medical School, 2018. All Rights Reserved.

Other Treatment Options


Laser Ablation Vagus Nerve Stimulation

Responsive Neurostimulation

Cannabidiol

SUDEP: Sudden unexpected


death in epilepsy

• Morbidity and mortality:


• Falls, drowning, co-morbid
conditions
• SUDEP (1/500-1/1000 adults per
year; 1/4500 children per year)
• 27 X that in the general population
and the cause of death in 2-18% of
patients, but less than half of
epilepsy related deaths
• Patients randomized to placebo –
10X more likely to have SUDEP

2426
Copyright © Harvard Medical School, 2018. All Rights Reserved.

Depression and
Death
• More than 1 of every 3 people
with epilepsy have depression
• 3-4X risk of suicide
• newly diagnosed (5X),
• hx psychiatric illness (29X)
• Prior suicide attempt -> 5X risk of
subsequent epilepsy
• Substance abuse, psychosis,
bipolar disorder, schizophrenia,
depression independently
associated with new-onset
epilepsy
Leestma et al. Ann Neurol 1989;26(2):195-203 ;Fricker,
1998;Walczak et al. Neurology 2001;56(4):519-525;
Martin et al Epilepsia 2014; Fazel S et al. 2013. Lancet;
382:1646-54

Epilepsy is a common disease with different


causes and manifestations

Other disorders can mimic seizures, including


convulsive syncope and psychogenic seizures

History is key to accurate diagnosis


Summary
of Key Diagnostic studies for all patients: MRI, EEG

Points
There are many therapeutic options

Education and safety issues are important


considerations

2427
Copyright © Harvard Medical School, 2018. All Rights Reserved.

Exam question:

• A 36 year old man presents with seizures x 1 day. He has a


history of a penetrating head injury 10 years ago and resultant
behavioral and cognitive disorders. He had his first seizure 3
months ago. He did not receive any treatment. His current
seizure began last night and he has had several seizures
without regaining consciousness. Which of the following is the
best treatment?
A. Lorazepam IV
B. Lorazepam + phenytoin
C. Clonazepam
D. Clonazepam + fosphenytoin

Exam Question 2

• A 29-year-old woman with juvenile myclonic epilepsy comes to


the physician for prepregnancy counseling. Her last seizure
was 8 years ago. She takes levetiracetam 500 mg twice daily
and an oral contraceptive. Which of the following is correct
regarding her management?
• A. Use high estrogen OCP since efficacy is decreased with
levetiracetam
• B. Discontinue AED treatment
• C. Prescribe folate
• D. Advise consideration of adoption given risk of birth defect

2428
Copyright © Harvard Medical School, 2018. All Rights Reserved.

• No disclosures

References

• Brophy, Gretchen M., et al. "Guidelines for the evaluation and


management of status epilepticus." Neurocritical care 17.1 (2012):
3-23.
• Krumholz, A., et al. "Practice Parameter: Evaluating an apparent
unprovoked first seizure in adults (an evidence-based review) Report
of the Quality Standards Subcommittee of the American Academy of
Neurology and the American Epilepsy Society." Neurology 69.21
(2007): 1996-2007.
• Meador KJ. To Stop or Not to Stop the AED? Epilepsy Currents.
2008;8(4):90-91. doi:10.1111/j.1535-7511.2008.00250.
• Laxer, Kenneth D., et al. "The consequences of refractory epilepsy
and its treatment." Epilepsy & Behavior 37 (2014): 59-70.
• Pugh, Mary Jo, and Katharine K. McMillan. "Guidelines and Quality
Standards for Adults with Epilepsy." Neurologic clinics 34.2 (2016):
313-325.

2429
Copyright © Harvard Medical School, 2018. All Rights Reserved.

Board Review: Neurology


Mary A. O’Neal, MD
Clinical Director of the Neurosciences Center
Director of the Women’s Neurology Program, BWH
Assistant Professor of Neurology, Harvard Medical School

No disclosures

2430
Copyright © Harvard Medical School, 2018. All Rights Reserved.

Question 1:

A 47 year-old hiker has pain in the left buttock


that radiates to her left lateral foot. The pain is
not affected by position or movement.

Question 1 (cont’d):

She also says that her left foot tingles as do


parts of her right foot and thigh, left fingers, and
the left anterior part of her chest.

2431
Copyright © Harvard Medical School, 2018. All Rights Reserved.

Question 1 (cont’d):

One month ago, a physician treated her for a


right Bell palsy and arthritis.

Question 1 (cont’d):
Which of the following would be most helpful in
establishing a diagnosis?

A. Electromyography and nerve conduction velocity


measurements
B. Somatosensory evoked responses
C. Magnetic resonance imaging of the spine and spinal
cord
D. Lumbar puncture
E. Lumbar roentgenography

2432
Copyright © Harvard Medical School, 2018. All Rights Reserved.

Question 1:
The answer is D

Question 1 Answer
Which of the following would be most helpful in establishing a diagnosis?
A. Electromyography and nerve conduction velocity measurements
B. Somatosensory evoked responses
C. Magnetic resonance imaging of the spine and spinal cord

D. Lumbar puncture
E. Lumbar roentgenography

Lyme disease can present with cranial neuropathies (typically the facial nerve, may
be bilateral, occurs in 5-10% of untreated patients) and inflammatory radiculopathy
(may mimic a mechanical radiculopathy with pain and sensorimotor symptoms).
Lumbar puncture will rule out other inflammatory, infectious or neoplastic etiologies
and will be abnormal and diagnostic in Lyme disease. CSF will typically show a
lymphocytic pleocytosis. EMG/NCS and MRI may be abnormal, but would not
provide a definitive diagnosis. An X-ray of the lumbar spine (lumbar roetenography)
would not be helpful.

2433
Copyright © Harvard Medical School, 2018. All Rights Reserved.

Question 2:

A 78 year-old normotensive woman realizes


suddenly that she cannot see to her left and that
her left hand tingles.

Question 2 (cont’d):

Two years previous, she had a stroke which left


her right arm and right leg weak. The family
reports that during the past year she has
become unreliable in daily responsibilities, and
often forgets people’s names.

2434
Copyright © Harvard Medical School, 2018. All Rights Reserved.

Question 2 (cont’d):

Computed tomographic scan shows a recent,


well-circumscribed homogeneous right parietal-
temporal hemorrhage; an old, slit-like cavity in
the left medial frontal lobe under the cortex; and
moderate ventricular dilatation and cortical
sulcal widening.

Question 2 (cont’d):

The most likely diagnosis is:

A. Hemorrhage into a brain tumor


B. Cerebral amyloid angiopathy
C. Embolization of cardiac origin with hemorrhagic
infarction
D. Multiple cerebral aneurysms
E. Recurrent head trauma

2435
Copyright © Harvard Medical School, 2018. All Rights Reserved.

Question 2:
The answer is B

Question 2 answer

The most likely diagnosis is:


A. Hemorrhage into a brain tumor

B. Cerebral amyloid angiopathy


C. Embolization of cardiac origin with hemorrhagic infarction
D. Multiple cerebral aneurysms
E. Recurrent head trauma

Her acute symptoms of a left hemifield vision loss and left hand
somatosensory complaints are explained by the recent right parietal-
temporal hemorrhage. Her prior stroke resulted in right sided weakness and
is explained by the lesion in the left frontal lobe. A slit-like cavity is seen as a
residual finding after a hemorrhage. Therefore she has had at least 2
intracerbral hemorrhages and has a possible dementia (family’s report of
one year of being unreliable). ICH is the most recognized complication of
CAA.

2436
Copyright © Harvard Medical School, 2018. All Rights Reserved.

Question 3:

After skiing, a 27 year-old woman developed


pain in the left mastoid region that radiated to
the left occiput and neck.

Question 3 (cont’d):

The next day she became dizzy, staggered, and


felt pain in the left forehead and eye.

2437
Copyright © Harvard Medical School, 2018. All Rights Reserved.

Question 3 (cont’d):

Physical examination shows normal blood


pressure, but with abnormal neurologic findings
including:
decreased pin perception on the left side of the face and
the right limbs and trunk;
a left Horner syndrome;
rotatory nystagmus;
weakness of the left palate and pharynx;
clumsiness and uncoordination of the left limbs.

Question 3 (cont’d):

The most likely diagnosis is:

A. Embolization to the basilar artery with pontine infarction


B. Pontine hemorrhage
C. Dissection of the left vertebral artery
D. Dissection of the left internal carotid artery
E. Artherosclerotic occlusion of the internal carotid artery

2438
Copyright © Harvard Medical School, 2018. All Rights Reserved.

Question 3:
The answer is C

Question 3 answer
The most likely diagnosis is:
A. Embolization to the basilar artery with pontine infarction
B. Pontine hemorrhage

C. Dissection of the left vertebral artery


D. Dissection of the left internal carotid artery
E. Artherosclerotic occlusion of the internal carotid artery

Vertebral artery dissection can present with ipsilateral occipital or neck


pain. Her examination is consistent with a left lateral medullary
syndrome (Wallenburg syndrome). This area of the medulla is supplied
by the PICA (posterior inferior cerebellar artery), a branch of the
vertebral artery. The lateral meduallary syndrome consists of: crossed
sensory findings with ipsilateral loss of pain and temperature to the
face and cotralateral to the body, ipsilateral horner syndrome,
ipsilateral ataxia, weakness of the larynx and pharynx leading to
dysphagia, dysarthria, vertigo.

2439
Copyright © Harvard Medical School, 2018. All Rights Reserved.

Question 4:

A 64 year-old man has progressive spasticity of


gait, impotence, and urinary frequency. He has
occasional headaches at the vertex.

Physical examination shows increased tone in


the bilateral lower extremities, 3+ patella and
ankle reflexes and bilateral extensor plantar
reflexes.

Question 4 (cont’d):
Which of the following is the next best step in his
care?

A. Physical therapy
B. Magnetic resonance imaging of the neck
C. EMG and NCV
D. CT of the lumbar spine
E. Cerebral angiogram

2440
Copyright © Harvard Medical School, 2018. All Rights Reserved.

Question 4:
The answer is B

Question 4 answer:
Which of the following is the next best step in his care?
A. Physical therapy

B. MRI of the neck


C. EMG and NCV
D. CT of the lumbar spine
E. Cerebral angiogram

MRI of the neck is the test of choice to assess for cervical


myelopathy. Cervical cord compression can lead to progressive
spasticity in gait, sexual dysfunction, and spastic bladder (urinary
frequency). The most common etiology of spinal cord dysfunction
in the elderly is cervical spondylosis. Signs of cervical canal
narrowing can be seen on plain films, CT, MRI, and CT
myelogram. EMG/NCV are rarely helpful, but can be used when
assessing for peripheral nerve pathology.

2441
Copyright © Harvard Medical School, 2018. All Rights Reserved.

Question 5:

An 80 year-old woman with mitral valve disease


and chronic atrial fibrillation becomes suddenly
confused during a family dinner.

Question 5 (cont’d):

She is awake and alert, and her motor function


appears intact and symmetric. She speaks in
long sentences unconnected to the events of the
evening or the questions asked of her.

2442
Copyright © Harvard Medical School, 2018. All Rights Reserved.

Question 5 (cont’d):

She uses many word substitutions and


nonsense words. She appears unable to
understand questions put to her by family
members.

Question 5 (cont’d):

The most likely diagnosis is:

A. Complex partial seizure


B. Transient global amnesia
C. Dominant hemisphere stroke
D. Nondominant hemisphere stroke
E. Delirium

2443
Copyright © Harvard Medical School, 2018. All Rights Reserved.

Question 5:
The answer is C

Question 5 answer
The most likely diagnosis is:
A. Complex partial seizure
B. Transient global amnesia

C. Left (dominant) hemisphere stroke


D. Right (nondominant) hemisphere stroke
E. Delirium

She has risk factors for stroke (AF) and has sudden onset of deficits. She speaks
fluently but uses word substitutions (paraphasias) and nonsense words
(neologisms). She has impaired comprehension. This description is classical for
Wernicke’s type aphasia and localizes to the left hemisphere involving the posterior
temporal lobe (Wernicke’s area). There are often no motor deficits because the
motor pathways are spared. Sometimes there is a visual field cut on the right due
to involvement of the optic radiations. A lesion involving the optic radiations in the
temporal lobe will result in a superior quadrantonopsia. Over 95% of right handed
people and the majority of left handed people have language in the left hemisphere.
Therefore it is called the dominant hemisphere (short for dominant hemisphere for
language).

2444
Copyright © Harvard Medical School, 2018. All Rights Reserved.

Question 6:

A 48 year-old man comes to the physician


complaining of right leg pain and numbness for the
past 3 weeks. The pain is on the upper, lateral
thigh and is burning and stinging in quality. The
pain radiates to the groin and down to the knee.
He has a PMH of obesity and has not seen a
doctor for many years.

Question 6 (cont’d):
On examination, he is obese and otherwise well-
appearing. Strength is 5/5 bilaterally. There is decreased
sensation in the right lateral thigh to temperature and
pinprick. There is decreased sensation to pinprick,
temperature and vibration in a bilateral stocking distribution
to the mid-calves bilaterally. Reflexes are 1+ at the patella
and 0 at the Achilles and bilateral flexor plantar reflexes
bilaterally.

2445
Copyright © Harvard Medical School, 2018. All Rights Reserved.

Question 6 (cont’d):
Which of the following is the best diagnostic test to obtain at
this time?

A. Hemoglobin A1C
B. Vitamin B12
C. EMG
D. MRI of the lumbar spine
E. CT of the pelvis

Question 6:
The answer is A

2446
Copyright © Harvard Medical School, 2018. All Rights Reserved.

Question 6 answer:
Which of the following is the best diagnostic test to obtain at this time?

A. Hemoglobin A1C
B. Vitamin B12
C. EMG
D. MRI of the lumbar spine
E. CT of the pelvis

The patient is presenting with meralgia paresthetica, a


neuropathy of the lateral femoral cutaneous nerve. This nerve
can be compressed by tight fitting clothing, often in association
with weight gain. Weight loss can also lead to this condition,
presumably by decreasing the fat padding around this nerve. The
diagnosis is made clinically in the setting of characteristic sensory
changes with normal motor function. It is more common in
patients with diabetes mellitus. This patient has signs of a
peripheral neuropathy as well that may be due to undiagnosed
diabetes. A B12 deficiency causes a myelopathy as well as
neuropathy and would not present in this fashion.

Question 7:

Ten days ago, a 22 year-old man had an


inoculation of tetanus toxoid in his right arm after
removal of a splinter.

2447
Copyright © Harvard Medical School, 2018. All Rights Reserved.

Question 7 (cont’d):

He now has severe pain in the right shoulder


and arm and parasthesias in the right hand.

Question 7 (cont’d):

Physical examination shows severe weakness of


muscles around the right shoulder girdle and
absence of the right biceps reflex.

2448
Copyright © Harvard Medical School, 2018. All Rights Reserved.

Question 7 (cont’d):

Passive range of movement of the shoulder is


normal. Sensory examination is normal, and
other reflexes in the arms and legs are normal.

Question 7 (cont’d):

The most likely diagnosis is:

A. Brachial neuritis
B. Herniated C-7 disk
C. Epidural cervical spinal abscess
D. Cervical spinal cord tumor
E. Rotator cuff injury

2449
Copyright © Harvard Medical School, 2018. All Rights Reserved.

Question 7:
The answer is A

Question 7 answer:
The most likely diagnosis is:

A. Brachial neuritis
B. Herniated C-7 disk
C. Epidural cervical spinal abscess
D. Cervical spinal cord tumor
E. Rotator cuff injury

“Parsonage-Turner syndrome” is characterized by inflammation of the brachial


plexus. It typically presents with severe shoulder pain and then as the pain is
improving, there is weakness in the deltoid and periscapular muscles. It is often
idiopathic, but may be autoimmune and triggered by immunization. There is typically
good recovery and treatment is supportive with physical therapy and pain control.
There is no role for immunosuppression. A C7 disc herniation would lead to
weakness in the triceps and wrist extensors and the biceps reflex would not be
affected.

2450
Copyright © Harvard Medical School, 2018. All Rights Reserved.

Question 8:

A 60 year-old man with a long history of back


pain recently began feeling weakness and
tingling in his legs when he walks more than a
half a block. The symptoms disappear when he
sits.

Question 8 (cont’d):

He has no symptoms when doing bicycling-like


exercises supine on his bed even after 30
minutes.

2451
Copyright © Harvard Medical School, 2018. All Rights Reserved.

Question 8 (cont’d):

Except for an absent left ankle reflex, neurologic


examination is normal. Foot and femoral pulses
are normal.

Question 8 (cont’d):

The most likely diagnosis is:

A. Aortic atherosclerosis with claudication


B. Polyneuropathy
C. Herniated lumbar L-5 disk
D. Lumbar spinal stenosis
E. Cervical spondylitic myelopathy

2452
Copyright © Harvard Medical School, 2018. All Rights Reserved.

Question 8:
The answer is D

Question 8 answer:
The most likely diagnosis is:
A. Aortic atherosclerosis with claudication
B. Polyneuropathy
C. Herniated lumbar L-5 disk

D. Lumbar spinal stenosis


E. Cervical spondylitic myelopathy

“Neurogenic claudication”
Lumbar stenosis can be asymptomatic, associated with low back pain, cause
symptoms and signs of focal nerve root compression, or give rise to neurogenic
claudication Neurogenic claudication refers to pain and discomfort in the low back,
buttocks, and legs that occurs after walking and is relieved by sitting. Relief of
symptoms with flexion of the spine explains why it is often easier to walk up an
incline than on a level surface, and forms the basis of the bicycle test. A patient with
neurogenic claudication will be able to cycle (spine flexed), but will not be able to
walk erect (spine extended) for an equivalent time. A patient with vascular
claudication is expected to have the same tolerance for both activities.

2453
Copyright © Harvard Medical School, 2018. All Rights Reserved.

Question 9:

A 25 year-old previously healthy man is found


unconscious in his apartment. There is no
evidence of trauma.

Question 9 (cont’d):

On examination, he is responsive to voice and


painful stimulation. There is no evidence of
meningeal irritation. The pupils are 3 mm and
unreactive.

2454
Copyright © Harvard Medical School, 2018. All Rights Reserved.

Question 9 (cont’d):

There is no inducible eye movements by the


doll’s eye maneuver or irrigation of a tympanic
membrane with ice water.

Question 9 (cont’d):

The blood pressure is 90/70 mm Hg, and the


pulse rate is 54/min. Respiratory function is
depressed.

2455
Copyright © Harvard Medical School, 2018. All Rights Reserved.

Question 9 (cont’d):

The most likely diagnosis is:

A. Sedative drug overdose


B. Subarachnoid hemorrhage
C. Intracranial mass
D. Brain stem stroke
E. Narcotic overdose

Question 9:
The answer is E

2456
Copyright © Harvard Medical School, 2018. All Rights Reserved.

Question 9 answer
The most likely diagnosis is:
A. Sedative drug overdose
B. Subarachnoid hemorrhage
C. Intracranial mass
D. Brain stem stroke

E. Narcotic overdose

The clues to this diagnosis are that he has small pupils and
has depressed respiratory rate. This patient has impaired brain
stem function, which can occur temporarily with drug
overdose. Both sedative and narcotic drug overdose can lead
to absent oculovestibular reflexes, but pupils are not small with
a sedative drug overdose.

Question 10:

A 35 year-old woman, who had a renal


transplant 4 years ago for renal failure due to
membranous glomerulonephritis, is hospitalized
because of progressive left sided weakness and
dysarthria.

2457
Copyright © Harvard Medical School, 2018. All Rights Reserved.

Question 10 (cont’d):

She has been treated with prednisone and


cyclosporine. A test for the human
immunodeficiency virus is negative.

Question 10 (cont’d):

MRI of the brain shows, in the right frontal lobe,


a focal area of abnormal signal, that spared the
cortical gray matter.

2458
Copyright © Harvard Medical School, 2018. All Rights Reserved.

Question 10 (cont’d):

There is minimal mass effect and no contrast


enhancement. There are also similar smaller
lesions throughout the white matter.

Question 10 (cont’d):

The most likely diagnosis is:

A. Multiple sclerosis
B. Glioma
C. Embolic stroke
D. Progressive multifocal leukoencephalopathy
E. Primary central nervous system lymphoma

2459
Copyright © Harvard Medical School, 2018. All Rights Reserved.

Question 10:
The answer is D

Question 10 answer
The most likely diagnosis is:
A. Multiple sclerosis
B. Glioma
C. Embolic stroke

D. Progressive multifocal leukoencephalopathy


E. Primary central nervous system lymphoma

The patient is immunosuppressed leaving her vulnerable to reactivation of the JC


virus. This disease is usually associated with AIDS, but is also seen patients
immunocompromised due to other reasons. This virus causes destruction of the
CNS white matter by infecting the oligodendrocytes. With acute lesions of multiple
sclerosis, glioma and with lymphoma there is often enhancement due to break
down of the blood brain barrier. Additionally, MS requires 2 separate episodes of
demyelination in time and space. A stroke affects the gray matter as well as the
white matter.

2460
Copyright © Harvard Medical School, 2018. All Rights Reserved.

Question 11:

The diagnosis can be established by:

A. Measuring beta2-microglobulin in cerebrospinal fluid


B. Electroencephalography
C. Arteriography
D. Magnetic resonance imaging
E. Brain biopsy

Question 11:
The answer is E

2461
Copyright © Harvard Medical School, 2018. All Rights Reserved.

Question 11 answer:
The diagnosis can be established by:
A. Measuring beta2-microglobulin in cerebrospinal fluid
B. Electroencephalography
C. Arteriography
D. Magnetic resonance imaging

E. Brain biopsy

A diagnosis of PML can be made via CSF


with polymerase chain reaction (PCR) for
the JC virus or by brain biopsy. PCR of the
CSF has been shown to be highly specific
(92-99%) and sensitive (74-93%) for the
detection of JC virus in patients with PML.
Brain biopsy has a sensitivity of 74-92%
and a specificity of 92-100%.

Question 12:

A 60 year-old businessman suddenly becomes


confused at a meeting. He has no perceivable
motor impairment and recognizes his
colleagues.

2462
Copyright © Harvard Medical School, 2018. All Rights Reserved.

Question 12 (cont’d):

He continually asks the same questions about


the subject matter under consideration at the
meeting.

Question 12 (cont’d):

One week later, the patient is normal, but unable


to remember the events of the meeting.

2463
Copyright © Harvard Medical School, 2018. All Rights Reserved.

Question 12 (cont’d):

The most likely diagnosis is:

A. Hysterical fugue state


B. Pulmonary embolism
C. Stroke syndrome
D. Transient global amnesia
E. Complex partial seizure

Question 12:
The answer is D

2464
Copyright © Harvard Medical School, 2018. All Rights Reserved.

Question 12 answer
The most likely diagnosis is:
A. Hysterical fugue state
B. Pulmonary embolism
C. Stroke syndrome

D. Transient global amnesia


E. Complex partial seizure

TGA is characterized by <24 hours of inability to form new memories


(anterograde amnesia), which cannot be attributed to other causes such as stroke
or seizure, with normal level of consciousness and otherwise normal cognitive
functioning and neurologic examination. It most commonly affects the middle-
aged or elderly. The underlying cause of TGA is unclear, but there appears to be
reversible dysfunction of the hippocampus possibly due to ischemia or
excitotoxicity.

Question 13:

Four years after having lumpectomy and


radiation treatment for breast carcinoma, a 45
year-old woman develops pain and weakness of
the left leg that spreads over a period of 1 week
to involve the right leg.

2465
Copyright © Harvard Medical School, 2018. All Rights Reserved.

Question 13 (cont’d):

She also has local back pain in the midthorax


and a circumferential band-like sensation.

Question 13 (cont’d):

In the past day, she has become incontinent of


urine after brief urgency, and her genitalia are
numb.

2466
Copyright © Harvard Medical School, 2018. All Rights Reserved.

Question 13 (cont’d):

The patient weighs 160 kg (352 lbs). Reflexes


are 3+ with unsustained ankle and knee clonus;
toes are extensor and the legs occasionally jerk
into a flexed posture.

Question 13 (cont’d):

The most likely diagnosis is:

A. Intramedullary metastasis
B. Epidural metastasis
C. Carcinomatous meningitis
D. Metastasis to the sagittal sinus
E. Radiation necrosis of the spinal cord

2467
Copyright © Harvard Medical School, 2018. All Rights Reserved.

Question 13:
The answer is B

Question 13 answer
The most likely diagnosis is:
A. Intramedullary metastasis

B. Epidural metastasis
C. Carcinomatous meningitis
D. Metastasis to the sagittal sinus
E. Radiation necrosis of the spinal cord

Cord compression at the thoracic level


In a patient with bilateral leg weakness, it is crucial to check for a
sensory level, bowel and bladder dysfunction, and saddle
anesthesia. This presentation is a neurologic emergency. The most
common metastases leading to cord compression are prostate,
breast and lung cancer. Metastasis usually arises in the posterior
aspect of vertebral body with later invasion of epidural space. The
most common complaint is pain, and two thirds of patients will have
motor signs at time of diagnosis of spinal epidural metastasis.

2468
Copyright © Harvard Medical School, 2018. All Rights Reserved.

Question 14:

A 25 year-old woman had lost the vision in the


right eye for 6 weeks at age 18. The loss was
attributed to “nerves” when her father died.

Question 14 (cont’d):

Her visual acuity has been as bad as 20/400, but


is now 20/20 with 20/15 acuity in the left eye.

2469
Copyright © Harvard Medical School, 2018. All Rights Reserved.

Question 14 (cont’d):

She has been having recurrent 20- to 30-minute


episodes of dimming of vision related to exercise
and hot showers. She has no other neurologic
symptoms.

Question 14 (cont’d):
Which of the following statements about this
patient is (are) true?

A. She has a larger pupil on the right


B. She probably has a normal visually evoked potential
C. She has a right afferent pupillary defect
D. She has definite multiple sclerosis
E. She has an optic nerve tumor

2470
Copyright © Harvard Medical School, 2018. All Rights Reserved.

Question 14:
The answer is C

Question 14 answer
Which of the following statements about this patient is (are) true?
A. She has a larger pupil on the right
B. She probably has a normal visually evoked potential

C. She has a right afferent pupillary defect


D. She has definite multiple sclerosis
E. She has an optic nerve tumor

She has a right afferent pupillary defect. She had an initial


presentation consistent with R optic neuritis, which is associated with
ipsilateral afferent pupillary defect. 20-30% of the time optic neuritis is
the presenting sign of multiple sclerosis (MS). Up to 50% of patients
with MS will develop an episode of optic neuritis.

Uhthoff's phenomenon, episodic transient worsening of symptoms


with increased body temperature (due to exertion, hot weather, hot
showers), occurs in isolated optic neuritis and in MS.

2471
Copyright © Harvard Medical School, 2018. All Rights Reserved.

Question 14 (cont’d):
Six months later she returns after having an
episode of right leg weakness and numbness
that resolved. MRI of the brain at that time
showed an enhancing lesion of the left
periventricular white matter. She begins
treatment for MS, but develops fever, chills,
malaise, muscle aches and fatigue.

Question 14 (cont’d):
Which of the following is the most likely treatment
that was initiated?

A. Intravenous methylprednisolone
B. Rituximab
C. Glatiramer
D. Interferon beta
E. Natalizumab

2472
Copyright © Harvard Medical School, 2018. All Rights Reserved.

Question 14:
The answer is D

Question 14 answer
Which of the following is the most likely treatment
that was initiated?
A. Intravenous methylprednisolone
B. Rituximab – nausea, vomiting, dizziness, h/a, pruritus,
asthenia, shivering
C. Glatiramer – post-injection reactions (systemic & local)
D. Interferon beta
E. Natalizumab – depression, nausea/GI, PML

Interferon beta frequently causes flu-like side effects and this is the
most common side effect requiring intervention. These side effects
usually appear early in therapy, last about 1 day after injection and may
subside over time. NSAIDs and acethaminophen are used to manage
symptoms.

2473
Copyright © Harvard Medical School, 2018. All Rights Reserved.

Question 15:

A 60 year-old mechanic fractured his left tibia.


Two weeks later he develops a severe, constant
ache in his leg.

Question 15 (cont’d):

The leg becomes pale with some cyanotic


mottling, feels cold and sweaty, and movement
is limited. The toenails are brittle and short.

2474
Copyright © Harvard Medical School, 2018. All Rights Reserved.

Question 15 (cont’d):

The most likely diagnosis is:

A. Tarsal tunnel syndrome


B. Occlusion of the tibial artery
C. Psychophysiologic disorder
D. Complex regional pain syndrome
E. Dermatophyte infection

Question 15:
The answer is D

2475
Copyright © Harvard Medical School, 2018. All Rights Reserved.

Question 15 answer
The most likely diagnosis is:
A. Tarsal tunnel syndrome
B. Occlusion of the tibial artery
C. Psychophysiologic disorder
D. Complex regional pain syndrome
E. Dermatophyte infection

Reflex symptathetic dystrophy (RSD)/ Complex regional pain syndrome (CRPS). CRPS
is a chronic progressive disease characterized by severe pain, swelling and changes in
the skin. The cause of this syndrome is currently unknown. Precipitating factors include
injury and surgery, although there are documented cases that have no demonstrable
injury to the original site. The syndrome is diagnosed by: the presence of an initiating
noxious event or a cause of immobilization (type 1) or after nerve injury (type 2),
continuing pain, allodynia (perception of pain from a nonpainful stimulus), or hyperalgesia
disproportionate to the inciting event, evidence at some time of edema, changes in skin
blood flow, or abnormal sudomotor activity in the area of pain.

Question 16:

The diagnosis of brain death requires


documentation of each of the following except:

A. Isolectric electroencephalogram
B. Pupillary unreactivity
C. Absence of eye movements
D. Apnea
E. Lack of receptivity and responsivity

2476
Copyright © Harvard Medical School, 2018. All Rights Reserved.

Question 16:
The answer is A

Question 16 answer:
The diagnosis of brain death requires documentation of each of the
following except:

A. Isolectric electroencephalogram
B. Pupillary unreactivity
C. Absence of eye movements
D. Apnea
E. Lack of receptivity and responsivity

EEG is not necessary to diagnose brain death, although it can


be used as an augmentative tool in certain conditions.

2477
Copyright © Harvard Medical School, 2018. All Rights Reserved.

Question 17:

A 24 year old woman comes to the physician


complaining of headaches for the past 6 months.
The headaches are bilateral and pulsatile. They
last 3-10 hours and are associated with nausea
and photophobia. She has missed several days of
work this past month due to headaches. She has
a past medical history of asthma, depression and
insomnia. She takes lorazepam prn and an oral
contraceptive.

Question 17:

Her optic fundi are shown below. Her neurologic


examination is normal.

Non-contrast MRI of the brain was performed last


week when she was seen in the emergency
department for a headache and it was normal.

2478
Copyright © Harvard Medical School, 2018. All Rights Reserved.

Question 17:

Which of the following is the next best step in her


care? Order or prescribe:

A. Repeat MRI of the brain with gadolinium


B. Lumbar puncture
C. Sertraline
D. Amitriptyline
E. Propranolol

Question 17:
The answer is D

2479
Copyright © Harvard Medical School, 2018. All Rights Reserved.

Question 17 answer:
Which of the following is the next best step in her care? Order or prescribe:
A. Repeat MRI of the brain with gadolinium
B. Lumbar puncture
C. Sertraline

D. Amitriptyline
E. Propranolol

She meets criteria for migraine without aura, despite the bilateral
pain. She is having frequent enough headaches (>2/mo that are
disabling) to benefit from a prophylactic agent. Amitriptyline is
considered a first-line agent for this purpose and would be preferable
to propranolol given her history of depression, asthma and insomnia.
Sertraline is less efficacious. Fundoscopic photos are of normal
discs. There is no indication for MRI or LP.

Papilledema

2480
Copyright © Harvard Medical School, 2018. All Rights Reserved.

Questions 18 & 19:

For each numbered word, phrase or statement,


select the one lettered heading that is most
closely associated with it. Each lettered heading
may be selected once, more than once, or not at
all.

Question 18:
A. Recurrent vertigo attacks lasting 1. Gentamicin
seconds occurring most frequently ototoxicity
when turning in bed at night without 2. Benign
tinnitus and with normal hearing positional vertigo
B. Single vertigo episodes lasting weeks 3. Vestibular
without decrease in hearing neuritis
C. Recurrent vertigo attacks lasting hours 4. Wallenberg
with tinnitus, and unilateral hearing loss syndrome (lateral
medullary
D. Vertigo and dysequilibrium lasting
infarction)
years with bilateral hearing loss
5. Meniere’s
E. An attack of vertigo with hiccups, facial
disease
numbness, and Horner syndrome
followed by months of dizziness

2481
Copyright © Harvard Medical School, 2018. All Rights Reserved.

Question 18:
The answers are 1-D, 2-A, 3-B, 4-E, and 5-C

Question 18 answer:
A. Recurrent vertigo attacks lasting 1. Gentamicin
seconds occurring most frequently ototoxicity - D
when turning in bed at night without 2. Benign
tinnitus and with normal hearing - 2 positional vertigo
B. Single vertigo episodes lasting weeks -A
without decrease in hearing - 3 3. Vestibular
C. Recurrent vertigo attacks lasting hours neuritis - B
with tinnitus, and unilateral hearing loss 4. Wallenberg
5 syndrome (lateral
D. Vertigo and dysequilibrium lasting medullary
years with bilateral hearing loss - 1 infarction) - E
E. An attack of vertigo with hiccups, facial 5. Meniere’s
numbness, and Horner syndrome disease - C
followed by months of dizziness -4

2482
Copyright © Harvard Medical School, 2018. All Rights Reserved.

Question 19:

A. Guillain-Barre syndrome 1. Treated with


(acute inflammatory plasmapheresis or IVIG
polyneuritis)

2. Treated with
B. Chronic inflammatory
demyelinating corticosteroids
polyneuropathy (CIDP)
3. Complicated by
C. Both respiratory dysfunction

D. Neither

Question 19:
The answers are 1-C, 2-B, and 3-A

2483
Copyright © Harvard Medical School, 2018. All Rights Reserved.

Question 19 answers:

A. Guillain-Barre syndrome 1. Treated with


(acute inflammatory plasmapheresis or IVIG
polyneuritis)
Both
B. Chronic inflammatory
demyelinating 2. Treated with
polyneuropathy (CIDP) corticosteroids
CIDP only
C. Both

D. Neither 3. Complicated by
respiratory dysfunction
AIDP only

Question 20
A 28-year-old woman G2P1 at 30 weeks gestation
was last seen well at 10 am. She was found on
the ground not speaking or moving her right side at
10:50 am.
On initial exam at noon, she was mute with left
gaze deviation and dense right hemiplegia.

NIHSS* was 15.

2484
Copyright © Harvard Medical School, 2018. All Rights Reserved.

What is the next best step? Order or


Prescribe?
A. Brain MRI

B. Head CT with contrast

C. Head CT without contrast

D. IV tPA

Question 20:
The answer is C

2485
Copyright © Harvard Medical School, 2018. All Rights Reserved.

Question 20:

The treatment of stroke during pregnancy should be, in


most cases, the same as in the non-pregnant state. The
mother’s health is the most important factor in maintaining
the health of the fetus.

Computerized tomography (CT) imaging raises concern


about fetal exposure to ionizing radiation. The level of
radiation exposure from most diagnostic imaging is far
below that known to cause fetal anomalies. Therefore, if
needed for accurate diagnosis, CT is appropriate.

Good Luck!

2486
Copyright © Harvard Medical School, 2018. All Rights Reserved.

Update in Geriatrics

Suzanne E. Salamon, MD
Associate Chief Clinical Geriatrics
Beth Israel Deaconess Medical Center
Division of Gerontology
Assistant Professor ,Harvard Medical School

No Disclosures

2487
Copyright © Harvard Medical School, 2018. All Rights Reserved.

Structure
• Title and Reference
• Background which led to study
• Study structure
• Results
• Conclusion

Topics
• Is the high dose flu shot really more effective?
• The new shingles vaccine
• Is vaginal estrogen safe?
• Methylphenidate (Ritalin) for Alzheimer’s
• Magnesium for leg cramps?
• What is effective for preventing falls?
• Can we prevent dementia?
• Thyroid-when to treat-or not
• Myths about getting older

2488
Copyright © Harvard Medical School, 2018. All Rights Reserved.

Vaccines Recommended for People > 65

• Flu (every year)


• Pneumonia (once each for most people)
PPSV 23
PVC or Prevnar 13
• TDaP once, then Td every 10 years
• Shingles

2489
Copyright © Harvard Medical School, 2018. All Rights Reserved.

Comparative effectiveness of high-dose vs


standard-dose influenza vaccination on numbers
of US nursing home residents admitted to
hospital: a cluster-randomised trial

Lancet Respir Med 2017 Jul 20

Gravenstein et al

Background
• The US Advisory Committee on Immunization
Practices recommends that all>6 mos old get flu
shot by October.
• Flu season usually starts in December or early
Jan., Average flu season lasts ≈13 weeks
• Flu vaccine available in US by end of
July,though most get flu shots starting in
September (peak shots in Oct.)
• Maximum vaccine effectiveness seen shortly after
vaccination (takes ≈ 14 days to mount protective
response) then declines in
effectiveness/antibodies of about 7% month
• On average, flu vaccine is 30% effective-this year

2490
Copyright © Harvard Medical School, 2018. All Rights Reserved.

Background
• 2 vaccines designed specifically for people 65 and older:
1-The high dose vaccine (IIV3-HD, Fluzone) -4 times the amount of antigen
as the regular flu shot.(IIV3-SD) stronger immune response (higher antibody
production).
-Clinical trial of >30,000 participants showed that adults >65 years who got
high dose vaccine had 24% fewer influenza infections than those who received the
standard dose flu vaccine. (NEJM 2014;371:635 Aug 14,2014.DiazGrados et al)
-Approved for use in the United States since 2009.

2 -The adjuvanted flu vaccine, Fluad designed to create a stronger immune


response to vaccination.
- Canadian observational study of 282 persons > 65 during the 2011-12
season, Fluad was 63% more effective in preventing illness than regular-dose
unadjuvanted flu shots.
- No randomized studies comparing Fluad with Fluzone High-Dose.
- Available for the first time in the US during the 2016-2017 season.

CDC does not recommend 1 flu shot over the other

High dose twice as expensive as standard dose ($ 30 vs $50)

The Study
• Done to compare high-dose trivalent influenza
vaccine (IIV3-HD) with a standard-dose
vaccine(IIV3-SD) in reducing hospital admissions of
nursing home residents in the USA.

• Randomized 823 US nursing homes (38,000 pts) to


standard vs high dose vaccine

2491
Copyright © Harvard Medical School, 2018. All Rights Reserved.

Results
• Incidence of respiratory-related hospital
admissions significantly lower in facilities
where residents received high-dose influenza
vaccines than in those that received standard-
dose influenza vaccines
• 3-4% over 6 months(HD) vs 3-9% over 6
months (SD)
• Statistically significant, though not
dramatic during flu season 2013-2014.

Conclusion

• When compared with standard-dose vaccine,


high-dose influenza vaccine can reduce risk of
respiratory-related hospital admissions from
nursing home residents aged 65 years and
older.

2492
Copyright © Harvard Medical School, 2018. All Rights Reserved.

Efficacy of the Herpes Zoster Subunit Vaccine in


Adults 70 Years of Age or Older

NEJM 2016;375: 1019-1032

Cunningham et al

Background
• Herpes zoster (shingles) results from reactivation of
latent varicella-zoster virus (VZV)/chickenpox,
presents as vesicular, painful dermatomal rash.
• In US, estimated 1 million cases of shingles a year
• One in three people will get shingles during their
lifetime.
• One in five people who have shingles will develop
post-herpetic neuralgia.

2493
Copyright © Harvard Medical School, 2018. All Rights Reserved.

Background
• Increased incidence with age
• T cell immunity decreases with age
• Postherpetic neurlagia (PHN) is most common
complication with chronic neuropathic pain
• 1st vaccine, Zostavax, is a live attenuated
vaccine (2006)
• Effectiveness is less in older adults:
70% in adults 50-59 41% in adults 70-79
64% in adults 60-69 18% in adults >80 YO

2494
Copyright © Harvard Medical School, 2018. All Rights Reserved.

Background, cont.
• CDC recommended zoster vaccine in 2006 for those
>60 yrs
• Now a new shingles vaccine was approved 10/2017
• Shingrix-nonlive, subunit (HZ/su) vaccine that
combines a protein found on the surface of the VZV
that causes shingles, with an adjuvant system, to
enhance the immune response to the antigen
• This study is 2nd to test safety and efficacy

The Study
• Randomized, placebo-controlled phase 3 trial
conducted in 18 countries
• Adults 70 years of age and older
• Participants received 2 doses of HZ/su or placebo
(1:1 ratio), I.M. 2-6 months apart.
• 6950 vaccine, 6950 placebo, mean age 75.6 YO
• Assessed vaccine efficacy against herpes zoster and
PHN

2495
Copyright © Harvard Medical School, 2018. All Rights Reserved.

Results
• Mean follow-up period 3.7 years
• Shingles occurred in 23 HZ/su recipients and 223
placebo recipients
• Vaccine efficacy ages 70-79: 90%
• Vaccine efficacy > 80 YO 89.1%
• Vaccine efficacy against PHN 88.8% (mostly because
of decreased incidence of shingles)
• Side effects of injection-site and systemic reactions
(fatigue) more common,( 79% vs 29 %), transient,
not considered serious

Risk of Development of Herpes Zoster after Vaccination.

Cunningham AL et al. N Engl J Med 2016;375:1019-1032

2496
Copyright © Harvard Medical School, 2018. All Rights Reserved.

Risk of Development of Postherpetic Neuralgia after Vaccination in the Pooled Population.

Cunningham AL et al. N Engl J Med 2016;375:1019-1032

Shingrix®: Adverse Reactions

Cunningham AL et al. N Engl J Med. 2016; 375:1019-1032

2497
Copyright © Harvard Medical School, 2018. All Rights Reserved.

Conclusion
• Adjuvanted subunit HZ/su vaccine reduced the risk of
herpes zoster and postherpetic neuralgia among adults
>70 Y.O. without substantial safety concerns
• Requires 2 injections, given 2-6 months apart
• Side effects, not dangerous, more common than
present vaccine
• October 2017, FDA approved Shingrix for adults
>50, (even those with prior shingles, previous
vaccination, no h/o chicken pox)

Addendum
• Immunocompromised persons. As with ZVL, the
ACIP recommends the use of RZV in persons taking
low-dose immunosuppressive therapy (e.g., <20
mg/day of prednisone or equivalent or using inhaled
or topical steroids) and persons anticipating
immunosuppression or who have recovered from an
immunocompromising illness

• Whereas RZV is licensed for all persons aged ≥50


years, immunocompromised persons and those on
moderate to high doses of immunosuppressive
therapy were excluded from the efficacy studies and
thus, ACIP has not made recommendations regarding
the use of RZV in these patients

2498
Copyright © Harvard Medical School, 2018. All Rights Reserved.

Breast cancer, endometrial cancer, and


cardiovascular events in participants who used
vaginal estrogen in the Women’s Health
Initiative Observational Study

Menopause:January 2018,
Vol 25,Issue 1,p.11-20
Crandall et al.

Background
• Vaginal atrophy after menopause -vaginal dryness, pain &
bleeding during sex, itching, irritation, burning, and discharge,
urinary symptoms, recurrent UTI
• Up to 45% of postmenopausal women ,but few seek help
• Due to decreased estrogen
• Underreported and undertreated.
• NAMS recommends non-hormonal vaginal
lubricants (Replens,K-Y,Astrogllide) and increased sexual
activity. OK for mild, but not moderate or severe sx.
• Systemic hormones OK for menopause symptoms, but long-
term therapy not recommended

2499
Copyright © Harvard Medical School, 2018. All Rights Reserved.

Background
• Increased risk for deep vein thrombosis, pulmonary embolism,
coronary heart disease, and endometrial and breast cancer.1
• Long-term systemic hormone therapy no longer recommended
if considered solely for the treatment of vaginal atrophy
• Local vaginal estrogen therapy –increased blood flow to
uterine lining,epithelial thickness,secreations,reverses
atrophy,decreases symptoms
• BUT-is it safe?
• This study done to determine association between vaginal
estrogen and risk of coronary heart disease (CHD), invasive
breast cancer, stroke, pulmonary embolism, hip fracture,
colorectal cancer, endometrial cancer, or death from any cause.

Vaginal Estrogen Formulations

• Vaginal ring-every 3 months. Slow,steady


release of estrogen
• Estrogen cream-1-2 times/week
• Estrogen tablets 1-2 times/week

2500
Copyright © Harvard Medical School, 2018. All Rights Reserved.

• Estradiol Vaginal Insert


• Dosage Form: vaginal insert
• WARNING: ENDOMETRIAL CANCER, CARDIOVASCULAR DISORDERS,
BREAST CANCER and PROBABLE DEMENTIA
• Estrogen-Alone Therapy
• -Endometrial CancerThere is an increased risk of endometrial cancer in a woman with a
uterus who uses unopposed estrogens. Cardiovascular Disorders and Probable Dementia
• -The Women’s Health Initiative (WHI) estrogen-alone substudy reported increased risks of
stroke and deep vein thrombosis (DVT) in postmenopausal women (50 to 79 years of age)
The WHI Memory Study (WHIMS) estrogen-alone ancillary study of WHI reported an
increased risk of developing probable dementia in postmenopausal women 65 years of age or
older during 5.2 years of treatment with daily CE (0.625 mg)-alone, relative to placebo In the
absence of comparable data, these risks should be assumed to be similar for other
doses of CE and other dosage forms of estrogens.
• Breast Cancer
• The WHI estrogen plus progestin substudy also demonstrated an increased risk of invasive
breast cancer [see Warnings and Precautions (5.3), and Clinical Studies (14.2)].
• In the absence of comparable data, these risks should be assumed to be similar for other
doses of CE and MPA, and other combinations and dosage forms of estrogens and
progestins.
• Estrogens with or without progestins should be prescribed at the lowest effective doses and
for the shortest duration consistent with treatment goals and risks for the individual woman.

The Study
• Prospective observational cohort study
• Data from participants of the Women's Health
Initiative Observational Study
• Recruited at 40 US clinical centers, aged 50 to 79
years at baseline
• Did not use systemic estrogen therapy during follow-
up (n = 45,663, median follow-up 7.2 years).
• Collected data regarding incident CHD, invasive
breast cancer, stroke, pulmonary embolism, hip
fracture, colorectal cancer, endometrial cancer, death,
and self-reported use of vaginal estrogen (cream,
tablet).

2501
Copyright © Harvard Medical School, 2018. All Rights Reserved.

Results
• Among women with an intact uterus, the risks of
stroke, invasive breast cancer, colorectal cancer,
endometrial cancer, and pulmonary
embolism/deep vein thrombosis were not
significantly different between vaginal estrogen
users and nonusers
• Risks of CHD, fracture, all-cause mortality, were
lower in users than in nonusers
• Among hysterectomized women, the risks were
not significantly different in users versus nonusers
of vaginal estrogen

Conclusion

• The risks of cardiovascular disease and cancer


were not elevated among postmenopausal
women using vaginal estrogens, providing
reassurance about the safety of treatment.

2502
Copyright © Harvard Medical School, 2018. All Rights Reserved.

Methylphenidate for Apathy in Community-


Dwelling Older Veterans With Mild Alzheimer’s
Disease: A Double-Blind, Randomized, Placebo-
Controlled Trial

The American Journal of Psychiatry


Published online Sept.15, 2017
Padala et al

2503
Copyright © Harvard Medical School, 2018. All Rights Reserved.

Background
• Methylphenidate (Ritalin) (MP)has been around since
1950’s-1960’s.
• Psychostimulant, similar to amphetamine
• Works by inhibiting uptake of dopamine and
norephinephrine, and some serotonin, increasing level
of dopamine leading to increased alertness
• Despite its availability and use in children with
ADHD for decades, it is rarely used in the older
population because of concerns of side effects and
DEA schedule II controlled substance classification

Clinical Applications
• DEPRESSION:
-Wallace et al studied 13 patients,mean age 72, with
major depression in double blind study using MP
BID (8 a.m & noon). Improved depression scores
within 8 days, no adverse effects.(Am J Psych:1995)
• Fast-acting, few side effects
• Can be used together with SSRI while SSRI is
reaching therapeutic levels

2504
Copyright © Harvard Medical School, 2018. All Rights Reserved.

Combined citalopram and methylphenidate


improved treatment response compared to either
drug alone in geriatric depression: a randomized
double-blind, placebo-controlled trial

Am J Psychiatry 2015 Jun 1; 172 (6):561-569

Lavretsky et al

• POSTSTROKE RECOVERY:
-Studies note improvements in mood, ADL’s and
motor functioning with MP following stroke

-Similar studies showed improvement in stroke


recovery with SSRI, separate from post-stroke
depression

2505
Copyright © Harvard Medical School, 2018. All Rights Reserved.

• IMPROVED MOTOR FUNCTION with MP in


Parkinson’s disease
-Several studies showed that a single dose of MP
improved gait speed, postural stability, freezing in older
people with PD (Shorer.J Gerontol A Biol Sci Med Sci.2013;29:15-17)

• PALLIATIVE CARE AND HOSPICE


- Decreased depression and fatigue (Hardy.Am J Geriatr
Pharmacother.2009;7:34-59). Doses 10-20 mg/day

-Main side effects: dry mouth, agitation, insomnia

Background
• Apathy is a common behavioral problem in
Alzheimer’s disease.
• Leads to functional impairment, higher service
utilization, higher caregiver burden, and increased
mortality.
• The authors’ objective was to study the effects of
methylphenidate on apathy in Alzheimer’s disease.

2506
Copyright © Harvard Medical School, 2018. All Rights Reserved.

The Study
• 12-week, prospective, double-blind, randomized,
placebo-controlled trial (methylphenidate versus
placebo) in community-dwelling veterans (N=60)
with mild Alzheimer’s disease.
• The primary outcome for apathy (Apathy
Evaluation Scale–Clinician) and secondary
outcomes for cognition (Mini-Mental State
Examination), functional status (activities of daily
living, instrumental activities of daily living)
• Improvement and severity measured at baseline
and at 4, 8, and 12 weeks.

Results
• Participants were all men (77 years old, SD=8).
• Methylphenidate group had significantly greater
improvement in apathy than the placebo group at
4 weeks, 8 weeks, and 12 weeks.
• At 12 weeks, there was also greater improvement
in cognition, functional status, caregiver burden,
CGI scores, and depression in the
methylphenidate group compared with the
placebo group.

2507
Copyright © Harvard Medical School, 2018. All Rights Reserved.

Conclusion

• Methylphenidate improved apathy in a group


of community-dwelling veterans with mild
Alzheimer’s disease.
• Methylphenidate also improved cognition,
functional status, caregiver burden, CGI
scores, and depression.

Effect of Magnesium Oxide Supplementation on


Nocturnal Leg Cramps

Jama Intern Med 2017:177(5) 617-623

Maor et al

2508
Copyright © Harvard Medical School, 2018. All Rights Reserved.

Background
• Nocturnal leg cramps (NLC) are painful contractions
of muscles occurring at rest, mostly at night
• Up to 60% adults report having NLC
• Although may be due to electrolyte imbalance,
hemodialsis, other conditions, most are idiopathic.
• Quinine is only treatment with moderate evidence in
reducing muscle cramps, but US FDA issued a
warning in 2010 against quinine due to risk of life-
threatening reactions
• Tonic water contains no more than 83 mg of quinine
per liter—a much lower concentration than the 500 to
1,000 mg in the therapeutic dose of quinine tablets.

2509
Copyright © Harvard Medical School, 2018. All Rights Reserved.

Background
• Magnesium commonly recommended for NLC, but
study of magnesium citrate did not decreased leg
cramps
• Magnesium oxide increased intracellular magnesium
levels more than mag citrate.
• This study was done to determine if Magnesium
oxide deceased leg cramps.

The Study
• 94 individuals, each with at least 4 NLC
during the screening 2 week period, mean age
65, randomly assigned to magnesium oxide or
placebo.
• Treated for 4 weeks
• Primary outcomes: difference in mean number
of NLC per week.
• Secondary outcomes: severity and duration of
NLC, quality of life, quality of sleep

2510
Copyright © Harvard Medical School, 2018. All Rights Reserved.

Results
• There was no difference between the severity
and duration of NLC, quality of life or quality
of sleep.

• Both groups had slightly lower number of


NLC (placebo effect)

Conclusion
• Oral magnesium oxide was not superior to placebo
for older adults with NLC.
• Decrease number of NLC in both groups is probably
a placebo effect that may explain the wide use of
magnesium for NLC

2511
Copyright © Harvard Medical School, 2018. All Rights Reserved.

Comparisons of Interventions for Preventing


Falls in Older Adults

Tricco,A. et al

JAMA 2017 Nov 7;318:1659

2512
Copyright © Harvard Medical School, 2018. All Rights Reserved.

Background
• 2010 National Institute of Aging data showed 2-year
prevalence of falls among those >65 YO was 36%
• Falls cause injury, death, anxiety, depression
• Difficult to determine effectiveness of fall prevention
programs
• This study was done to evaluate all available fall-
prevention interventions for older people to determine
which are most effective.

The Study
• 283 RCTs comparing falls interventions with
“usual” care to determine what works to
decrease incidence of falls

2513
Copyright © Harvard Medical School, 2018. All Rights Reserved.

Results
• (159,910 participants), mean age 78
• When compared with usual care,these interventions
were associated with reductions in injurious falls:
- exercise
-combined exercise and vision evaluation and
treatment
-combined, exercise, vision, and environmental
assessment modification
-Calcium + vitamin D

Conclusion
• Exercise alone and various combinations of
interventions were associated with lower risk
of injurious falls compared with usual care.

• Choice of fall preventions intervention


depends on patient and caregiver preferences.

2514
Copyright © Harvard Medical School, 2018. All Rights Reserved.

2515
Copyright © Harvard Medical School, 2018. All Rights Reserved.

Can We Prevent Dementia?

Annals of Internal Medicine

December 19, 2017

2516
Copyright © Harvard Medical School, 2018. All Rights Reserved.

Background
• 47 million people with Alzheimer disease and related dementias
(ADRD) worldwide
• Incidence in US is declining, prevalence increasing as population
ages
• Dementia-related costs exceed those of heart disease and cancer.
• Since we have no cure, preventing or delaying the onset of dementia
is a public health priority.
• Is there a “magic bullet” for preventing dementia?

• What is the evidence for preventing dementia by cognitive training


physical activity, O.T.C. supplements and prescription medication,?

• December 19, 2017 Annals of Internal Medicine published 4


evidence-based reviews of trials of preventive interventions of
adults without dementia to see if any interventions did indeed
prevent dementia.

Background

2517
Copyright © Harvard Medical School, 2018. All Rights Reserved.

Does Cognitive Training Prevent Cognitive


Decline?: A Systematic Review

Annals Internal Medicine


2 January 2018 Vol: 168, Issue 1
Butler et al

Background
• Fear of Alzheimer disease and related dementias
(ADRD) considered by many to be worse than death ,
driving a growing “brain-training” industry.
• Structured activities to stimulate brain, - cognitive
training exercises-marketed to otherwise healthy
adults and persons with recent diagnosis of mild
cognitive impairment (MCI)
• promoted to slow or prevent cognitive decline,
including dementia, but effectiveness is highly
debated.
• This study reviewed the evidence for cognitive
training on the occurrence of dementia

2518
Copyright © Harvard Medical School, 2018. All Rights Reserved.

The Study
• Looked at studies 2009-2017, found 34
randomized trials of cognitive training
interventions lasting from 2 weeks to 6
months, then followed them for up to 2 years.
• Most measured outcomes of test performance
(memory, processing speed)

Results
• Improvments in test performance of the domain
tested (eg memory)which improved in that area
rather than measuring progression to dementia

• Evidence was insufficient to determine whether


cognitive training exercises prevented mild cognitive
impairment or dementia

• Often plaques and tangles start to form many years


earlier, so these trainings were started late.

2519
Copyright © Harvard Medical School, 2018. All Rights Reserved.

Conclusion

• Cognitive training seemed to improve test


performance only in the domain trained

• Evidence was insufficient that training


prevented or delayed cognitive decline

Physical Activity Interventions in Preventing


Cognitive Decline and Alzheimer-Type
Dementia: A Systematic Review

Ann Intern Med Dec.19, 2017

Brasure, et al

2520
Copyright © Harvard Medical School, 2018. All Rights Reserved.

The Study
• Reviewed several electronic databases 2009-2017 that
lasted 6 months or longer, enrolled adults without
clinically diagnosed cognitive impairments and
compared dementia outcomes between physical
activity interventions and inactive controls

• Follow-up 6 months to 2 years

• Looked at studies of aerobic training, resistance


training, tai chi, diet

Results
• Insufficient evidence that physical activity
intervention is effective in preventing
cognitive decline.

2521
Copyright © Harvard Medical School, 2018. All Rights Reserved.

Conclusion
• Physical activity interventions begun after
decades of high-risk behavior likely are
insufficient to reduce dementia risk
• However regular physical activity may need to
begin earlier in life and be sustained as a
lifestyle.
• Physical activity may slow cognitive decline
by decreasing dementia due to vascular factors

Over-the-Counter Supplement Interventions to


Prevent Cognitive Decline, MCI, and Alzheimer-
Type Dementia: A Systematic Review

Annals of Internal Medicine


December 19, 2017
Brasure, et al

2522
Copyright © Harvard Medical School, 2018. All Rights Reserved.

Background
• Fear of dementia has lead to a growing industry of
OTC supplements intended to boost brain health and
prevent or slow cognitive decline.

• Alzheimer’s Research and Prevention Foundation


says “you should definitely take a high potency
multiple vitamin and mineral capsule” as well as:
coenzyme Q10,alpha lipoic acid, ginkgo biloba,
phosphatidylserine, Omega-3’s,acetyl-L-carnitine
vitamin C 2000 mg/day vitamin E

Background
• >60% older adults use OTC supplements
• 2015 Americans spent $37 billion on OTC
supplements, $91 million on ginkgo biloba
• Do they work?
• This review summarizes the evidence on
efficacy of OTC supplements in preventing or
delaying cognitive or Alzheimer-type dementia

2523
Copyright © Harvard Medical School, 2018. All Rights Reserved.

The Study
• On line search of studies 2009-2017 of people
with normal cognition.
• Followed people for a minimum of 6 months
• Measured neuropsych testing for dementia or
MCI (mild cognitive impairment)

Results
• 56 studies covering 13 categories of OTC treatments:
• -Omega Fatty Acids-up to 6 years: no better than placebo
• -Soy: No difference from placebo
• -Ginkgo biloba: No difference
• -B Vitamins (Folate,B6,B12)-no benefit
• -Vitamin D + Calcium-no benefit
• -Vitamin E-no difference in incidence of dementia at 10 years
• - Vitamin C(500 mg) or Beta Carotene: Vit C +/-
• -Multivitamins-no difference over 5 years

2524
Copyright © Harvard Medical School, 2018. All Rights Reserved.

Conclusion
• Small number of OTC supplements evaluated for
potential effects on dementia
• Most OTC interventions studied have no proven
benefit in preventing or delaying dementia in older
adults
• Supplements may work better in persons with low
levels of nutrient or vitamin (baseline deficiencies
reported only in B vitamins
• Evidence is insufficient for clinicians to recommend
any of the OTC supplements to patients with normal
cognition or MCI for preventing dementia
• Few trials are currently ongoing

Pharmacologic Interventions to Prevent


Cognitive Decline, MCI, and Alzheimer’s
Dementia

Annals of Internal Medicine


Dec. 19,2017
Fink, et al

2525
Copyright © Harvard Medical School, 2018. All Rights Reserved.

Background

• Several medications have been suggested to


possibly prevent or reduce the risk of
Alzheimer dementia
• This review was done to assess the most
current evidence about the efficacy and safety
of drug therapy in preventing or delaying
dementia

The Study
• The authors search Medline and other large
data bases 2009-2017 looking at prescription
drugs to see if any decrease risk of dementia

- dementia medications -blood pressure meds


- diabetes meds -lipid-lowering meds
- NSAID’s - hormone therapy
- Testosterone

2526
Copyright © Harvard Medical School, 2018. All Rights Reserved.

Results
• Cholinesterase inhibitors- in pts with 1 apolipoprotein
E4 allele, less progression to Alz.disease (not stat.sig)
• Antihypertensives-no difference from placebo
• Diabetes meds-insufficient evidence for prevention
• Lipid-lowering meds-no difference from placebo
• NSAID’s-no difference
• Estrogen only-none or slight increase dementia
• Estrogen + progesterone-increased risk stroke, CAD,
breast cancer, PD
• SERM’s-no risk reduction
• Testosterone-no decrease risk

Conclusion

• Evidence did not support use of


antihypertensives, statins, NSAID’s, aspirin,
diabetes drugs or cholinesterase inhibitors for
preventing Alzheimer’s dementia in adults
with normal cognition or mild cognitive
impairment

2527
Copyright © Harvard Medical School, 2018. All Rights Reserved.

Except in extreme cases:


• 1. Thyroid medication does not alleviate most
symptoms.

• 2. People with higher TSH live longer

• 3. Suppressed/low TSH (i.e. higher free


thyroid levels, from overactive thyroid or
meds) leads to increased incidence of
osteoporosis and fractures

2528
Copyright © Harvard Medical School, 2018. All Rights Reserved.

Thyroid Hormone Therapy for Older Adults with


Subclinical Hypothyroidism

N Engl J Med 2017; 376:2534-2544


Stott, et al

Background

• Subclinical hypothyroidism defined as high


TSH with normal free T4
• TSH, Normal T4
• Occurs in 8-18% adults >65 YO
• Most have no symptoms
• Should the elevated TSH be treated?

2529
Copyright © Harvard Medical School, 2018. All Rights Reserved.

The Study
• Double-blind, randomized, placebo-controlled trial
• 737 adults >65 YO with elevated TSH 4.6-19.99 with
normal free T4
• Half got levothyroxine of 50 µg daily (25 if weight
<50 kg or had CAD), half got placebo
• Primary outcome:
• -Changes in hypothyroid symptoms(feeling cold,
constipation, weight gain, slow movements, dry skin,
fatigue)

Results

• Mean age 74 years


• Mean TSH 6.4
• At 1 year, no difference in the 2 groups in
hypothyroid symptoms.

2530
Copyright © Harvard Medical School, 2018. All Rights Reserved.

Conclusion

• Levothyroxine provided no apparent benefits


in older persons with subclinical
hypothyroidism in 396 patient age 75

Association of Thyroid Function With Life


Expectancy With and Without Cardiovascular
Disease

JAMA Intern Med 2017 Sep 18


Bano et al

2531
Copyright © Harvard Medical School, 2018. All Rights Reserved.

Background
• Subclinical hypothyroidism characterized by
abnormal TSH and normal T4

• Both clinical and subclinical thyroid dysfunction


associated with increased heart disease and mortality

Background
• Overactivity of the thyroid gland is thought to have a
negative effect on overall health.

• Higher FT4 associated with an increased heart rate

• This study was done to investigate life expectancy


(LE) and cardiovascular disease (CVD) among
thyroid status categories ( ie, hypothyroidism,
euthyroidism, hyperthyroidism)

2532
Copyright © Harvard Medical School, 2018. All Rights Reserved.

The Study
• 7785 adults (mean age 65)without known
thyroid disease (TSH and FT4 within normal
reference ranges) divided into 3 groups
according to TSH and FT4 levels

• Followed for 8 years for fatal or nonfatal


adverse CVD events

Results
• During median follow-up of 8 years, adverse CVD
events occurred in 10% of patients and 17% died
overall.
• Group with highest Free T4 (FT4) had 32% higher
risk for CVD and 50% more likely to die than people
in lowest group
• People in group with lowest FT4 lived 3 years longer
than people in highest FT4 group
• Group with highest TSH 20% less likely to die than
other groups

2533
Copyright © Harvard Medical School, 2018. All Rights Reserved.

Higher thyrotropin (TSH) and lower free thyroxine (Free T4) =longer life
expectancy

Conclusion
• People with higher TSH and low-normal FT4 five up
to 3.5 years longer than those with high normal FT4.

• Helpful in knowing how to treat patient who believe


that higher thyroid replacement makes them feel
better.

2534
Copyright © Harvard Medical School, 2018. All Rights Reserved.

Subclinical Thyroid Dysfunction and Fracture Risk

JAMA. 2015;313 (20):2055-2065


Blum, et al

• Subclinical hyperthyroidism was associated with an


increased risk of fractures, with highest risk in those
with suppressed TSH <0.10 mIU/L

• No association between subclinical hypothyroidism


and fractures

Why Everything you think About Aging May Be


Wrong

Wall Street Journal

Anne Tergesen
Nov. 30, 2014

2535
Copyright © Harvard Medical School, 2018. All Rights Reserved.

• Stereotype of later life as time of loneliness,


depression, decline, but research shows that in many
way, life gets better as we get older
• Moods and sense of well-being often improve with
age
• Expertise deepens, wisdom flourishes
• Many challenges, health related
• Several myths about aging:

Myths about Aging


1.Depression is more prevalent in Old Age
-Research shows emotional well-being improves until
the 70’s, when it levels off, even among centenarians
2. Cognitive Decline is Inevitable.
-Except in dementia, knowledge and wisdom help
people more in real-life than on tests.

3. Older Workers are Less Productive.


-People >55 YO make up 22% of the US labor force
-No relationship between age and job performance

2536
Copyright © Harvard Medical School, 2018. All Rights Reserved.

4. Loneliness is More Likely


-Older people report better marriages, better
friendships, higher rate of close ties

5. Creativity Declines with Age


Mark Twain, Cezanne, Frank Lloyd Wright, Robert
Frost, Virginia Woolf, Julia Childs, Laura Ingalls
Wilder, Benjamin Franklin, Mother Theresa, Ghandi,
Grandma Moses

2537
Copyright © Harvard Medical School, 2018. All Rights Reserved.

2538
Copyright © Harvard Medical School, 2018. All Rights Reserved.

NY Times Feb 11, 2017

• By Claire Cain Miller NEW YORK TIMES


• Women much more likely to work into 60s,
70s today, studies show
• 30% women 65-69 are working. 18% women
age 70-74 work

2539
Copyright © Harvard Medical School, 2018. All Rights Reserved.

Summary
• High-dose flu vaccine reduces respiratory hospitalizations from
N.H.residents > 65 years more than standard dose
• Shingrix vaccine prevents shingles in 90%,even in 90 YO
• Vaginal estrogen appears to be safe
• Methlyphenidate helps depression/apathy in Alz.patients
• Magnesium not effective for leg cramps,but ?placebo effect
• Exercise, vision, environmental factors,Ca+D decrease falls
• So far, no “magic bullet” to prevent Alzheimers]dementia
• Suppressed TSH may cause more problems than elevated TSH
• Growing older may not be as bad as you think!

2540
Copyright © Harvard Medical School, 2018. All Rights Reserved.

Obesity Management

Florencia Halperin, MD, MMSc


Co-Director, Center for Weight Management and Metabolic Surgery
Chief, Division of Endocrinology, Brigham and Women’s Faulkner Hospital
Division of Endocrinology, Brigham and Women’s Hospital
Instructor in Medicine, Harvard Medical School

None

2541
Copyright © Harvard Medical School, 2018. All Rights Reserved.

At the end of this class, participants will be able to:


Apply the approach that obesity is a disease, not a
behavioral problem
Understand how to formulate an effective obesity
treatment program with multiple components
Know indications for approved obesity treatments,
including lifestyle interventions, pharmacotherapy, and
surgery
Be familiar with recent obesity management guidelines

Jensen MD Circulation 2013; Apovian CM


JCEM 2015;Garvey WT Endocr Prac 2016

2542
Copyright © Harvard Medical School, 2018. All Rights Reserved.

Body Mass Index (BMI): kg / m2


Overweight: BMI 25.0 – 29.9
Obesity: BMI > 30.0
Class I: BMI 30.0 – 34.9
Class II: BMI 35.0 – 39.9
Class III: BMI > 40

Obesity is defined as a chronic, relapsing, multi-factorial,


neurobehavioral disease, wherein and increase in body fat
promotes adipose tissue dysfunction and abnormal fat mass
physical forces, resulting in adverse metabolic,
biochemical and psychosocial consequences
–American Society of Bariatric Physicians

Lizarbe B Front Neurogen 2013

2543
Copyright © Harvard Medical School, 2018. All Rights Reserved.

OBESITY

Neurobehavioral Psychological

Economic Endocrine

Immune
Developmental

Epi/Genetics Environment

Risk, not just BMI, should drive treatment decisions


Link weight loss goals to health goals
Earlier intervention
Chronic disease management
Individualized treatment approach(es)

2544
Copyright © Harvard Medical School, 2018. All Rights Reserved.

DIETARY INTERVENTIONS
PHARMACOTHERAPY

Placebo

Lorcaserin 10 QD

Lorcaserin 15 QD
EXERCISE INTERVENTIONS

Lorcaserin 10 BID

Gardner IJO 2012; Church PLoS


ONE 2009; Smith S NEJM 2010

DPP: Reduction in progression from IGT to T2DM

31% Reduction
(7.8 vs. 11%/yr)
58% Reduction
(4.8 vs. 11%/yr)

6.7%
Weight
Loss

DPP Research Group NEJM 2002;


Hamman Diabetes Care 2006

2545
Copyright © Harvard Medical School, 2018. All Rights Reserved.

Look AHEAD, 10 yrs: No difference in CV death, nonfatal


MI, CVA, hospitalization angina Look AHEAD Group NEJM 2013

Da Qing: 577 IGT, RCT 6 yrs, 23 yr follow up, CV death HR


0.59, all cause mortality HR 0.71 Li G Lancet Diab Endocrin 2014

Mediterranean Diet: 7000, HR 0.7 of major CV events


Estruch R NEJM 2013

2546
Copyright © Harvard Medical School, 2018. All Rights Reserved.

Broaching the subject: Counseling


Diet
Behavior Modification
Physical Activity
Medications
Surgery

Talk about it! Document it!


Obesity - a complex, multi-factorial disease

Initiating the conversation


◦ “We have not discussed your weight recently. What are your
thoughts about your weight and health at this time?”
Patient preferred term: “Weight”

Focus non-weight outcomes/health improvements with


5-10% weight loss

Wadden Obes Res 2003; AHA/ACC/TOS Guidelines Circulation 2013

2547
Copyright © Harvard Medical School, 2018. All Rights Reserved.

Motivational Interviewing:
◦ Patient-centered, collaborative counseling style
◦ Encourage personal reasons for change
◦ Explore ambivalence about change (behavioral change is a
process)
◦ Help identify and overcome individual barriers to success
◦ Can be effective in brief encounters
Core Practices
◦ Express empathy
◦ Reflective listening
◦ Collaboration – guiding and negotiating, not dictating
◦ Shared decision-making
◦ Support self-efficacy and autonomy

The clinician and patient should agree on whether weight loss


is appropriate and assess if the patient is prepared to
undertake the measures necessary to succeed:
◦ “How prepared are you to make changes in your diet, to be
more physically active, and to use behavior change
strategies such as recording your weight and food intake?”

If not prepared, counseling likely counterproductive


Periodically re-assess interest and readiness
Discuss avoiding additional weight gain to prevent risks
Evaluate and treat CVD risk factors

AHA/ACC/TOS Guidelines Circulation 2013

2548
Copyright © Harvard Medical School, 2018. All Rights Reserved.

“While weight loss treatment is ongoing, manage risk


factors such as hypertension, dyslipidemia, and other
obesity-related conditions.

This includes monitoring the patient’s requirements for


medication change as weight loss progresses,
particularly for anti-hypertensive and diabetes
medications that can cause hypoglycemia.”

AHA/ACC/TOS Guidelines Circulation 2013

5-10%
in 6 mos … BUT HOW?

AHA/TOS (2013)

2549
Copyright © Harvard Medical School, 2018. All Rights Reserved.

Dr. You

Caloric restriction
Portion controlled foods
Frequent follow up visits
Self monitoring
◦ Weight, diet, physical activity
Physical activity (exercise and NEAT)
Referral to a high-intensity comprehensive program
(including commercial programs, e.g. Weight
Watchers)

6 months: 6 kg (7%) weight loss


2 years: 3-4 kg weight loss
Irrespective of macronutrient composition

Sacks FM NEJM 2009


Bottom line for weight loss: Caloric restriction and
adherence (not macronutrient composition)

2550
Copyright © Harvard Medical School, 2018. All Rights Reserved.

1,200-1,500 kcal/d for women


1,500-1,800 kcal/d men
OR 500-750 kcal/d energy deficit

One of the evidence-based diets that restricts certain


food types (e.g. high-carb, high-fat) in order to create
an energy deficit by reduced food intake

AHA/ACC/TOS Guidelines Circulation 2013

Frozen diet meals


Liquid meals
Bars
Portion controlled
Calorie controlled
Convenient
Inexpensive
As part of a sensible
Wadden TA Obesity 2009
Well-planned menu

2551
Copyright © Harvard Medical School, 2018. All Rights Reserved.

Number of sessions attended (Look AHEAD)

Wadden TA Obesity 2009

Food, exercise, weight


Highest adherence to completing food intakes
greatest weight loss

www.wellocracy.com

2552
Copyright © Harvard Medical School, 2018. All Rights Reserved.

Low fitness independent predictor of mortality for any BMI

Exercise alone: limited effect on weight loss


Exercise plus diet: augments loss (modest)
Weight loss maintenance: More is better (>300 mins/week)

Pedometer use
◦ NEAT
◦ Increases
steps/d (~30%)
◦ Best predictor of
increased activity:
Step goal

Wadden TA Circulation 2012

Shaw K Cochrane 2006; Saris WH Obes Rev 2003; Bravata DM JAMA 2007

The most effective behavioral weight loss treatment is


a high intensity comprehensive program
◦ In-person
◦ High-intensity (i.e., ≥14 sessions in 6 months)
◦ Individual or group sessions by trained interventionist

The principal components include:


◦ Prescription of a moderately-reduced calorie diet
◦ Program of increased physical activity
◦ Use of behavioral strategies to facilitate adherence

AHA/ACC/TOS Guidelines Circulation 2013

2553
Copyright © Harvard Medical School, 2018. All Rights Reserved.

What are the approved weight loss medications?

When is it appropriate to consider use?

Which meds for which patients?

Obesity drugs
alter
physiology
not just
behavior(s)

Potential Mechanisms of Action


• Decrease appetite/cravings
• Decrease leptin resistance
• Increased energy expenditure
• Increase adherence by mitigating Ioannides-Demos LL J Obes 2011;
biological or genetic factors York DA. Nutrition 2000; Liu et al. Cell 2015

2554
Copyright © Harvard Medical School, 2018. All Rights Reserved.

*Above placebo
Weight
Name Mechanism Side Effects Dose Other
Loss*
Adrenergic/ 15-37.5
Phentermine 5% ↑HR, ↑BP Generic
CNS mg QAM
3.75/2 mg QMO U HCG;
Phentermine/
Adrenergic/ ↑HR, ↑BP,
(14d) 1mo chem↓CO2;
Topiramate Cognitive Not in CAD, CVA
7-9% CNS 7.5/46 mg
(Qsymia) Teratogenic in last 6mo
QAM
5-HT2c
Lorcaserin 10 mg
3.5% receptor Headache Not with SSRI
(Belviq) BID
agonist
Orlistat Lipase 60-120 Vitamin
3% Steatorrhea
(Alli,Xenical) Inhibitor mg QAC deficiencies

Naltrexone/ Nausea 8/90 mg: Not with other


Buproprion 4% CNS Constipation 2 tabs BID bupropion,
(Contrave) Headache (titration) opioids

Liraglutide GLP-1 Nausea, 3 mg SC


8-9%
(Saxenda) agonist/CNS Diarrhea QD

Adjunct to diet and exercise


Indication: BMI >30; BMI >27 with co-morbidities

If never participated in a comprehensive lifestyle


intervention program, undertake such a program prior

If unable to lose or sustain weight loss with


comprehensive lifestyle intervention and meets BMI
criteria, adjunctive therapies may be considered
◦ Maximized behavioral approaches
◦ Weight plateau

AHA/ACC/TOS Guidelines Circulation 2013

2555
Copyright © Harvard Medical School, 2018. All Rights Reserved.

Assessment monthly for first 3 mo, then every 3 mo

If effective (weight loss 5% at 3 mo) and safe,


recommend medication be continued
If ineffective (weight loss <5% at 3 mo) or
safety/tolerability issues, recommend medication be
discontinued and alternative medications or
approaches be considered

Apovian CM JCEM 2015

Effectiveness
Contra-indications
◦ Based on medical conditions or med-med interactions
Possible benefit for multiple medical conditions
◦ Headache prevention (topiramate)
◦ Diabetes/Pre-diabetes (liraglutide)
Cost
Patient Preference

2556
Copyright © Harvard Medical School, 2018. All Rights Reserved.

0
-1
-2 -2.8 -2.8 -2.8
Weight (kg)

-3 -3.6 -3.6 -3.3


-3.8
-4 -4.5
-5 -5.2 Cheapest
-5.6
-6 phentermine
-7 Most Effective
-8 -8.8 Phen+Top,
-9 Liraglutide
FDA-Approved Off-Label
-10

Witkamp RF Pharm Res 2011;Gadde K Arch Int Med 2013;


Powell AG Clin Pharm Ther 2011; Torgerson JS Diab Care 2004; Smith
NEJM 2010; Garvey WT AJCN 2012.

RCT, Liraglutide 3 mg vs placebo for DM prevention


BMI >27 (co-morbidities) or >30, pre-diabetes, n=2254
50% completed to week 160

At week 160:
HR 0.21 (95% CI 0.13–0.34) 6%

2%

LeRoux C Lancet Feb 2017

2557
Copyright © Harvard Medical School, 2018. All Rights Reserved.

Clinical Factors Avoid/Use Caution


Increased Seizure Risk Naltrexone/Bupropion
Kidney Stones Phentermine/Topiramate, Orlistat
Glaucoma Phentermine/Topiramate
Uncontrolled Hypertension Naltrexone/Bupropion
Phentermine
CAD/CVA Phentermine
Renal Insufficiency (mod-severe) Use half-dose: Phentermine/Topiramate,
Naltrexone/Bupropion
Avoid: Liraglutide, lorcaserin
Hepatic Insufficiency (mod-severe) Use half-dose: Phentermine/Topiramate
Use quarter dose: Naltrexone/Bupropion
Caution: Liraglutide, lorcaserin
SSRI use Caution: lorcaserin

Indications: BMI >40; BMI >35 with co-morbidities

(LAGB) (RYGB) (SG)


2011 2014
Total 158,000 193,000
RYGB 36.7% 26.8%
LAGB 35.4% 9.5%
SG 17.8% 51.7%
Ponce J SOARD 2015

2558
Copyright © Harvard Medical School, 2018. All Rights Reserved.

LAGB SG RYGB
Weight Loss
40-45% EBW 50-60% EBW 60-70% EBW
(2 yrs)
Length of
1 hour 1 hours 2 hours
Surgery
Time in Hospital 1 day 2 days 2 days
Risk of Death <0.05 % 0.1-0.3 % 0.3-0.5 %
Reversal of Yes, if medically
No Very Difficult
Procedure necessary
Inadequate loss; Dumping
Other Issues
Band removal syndrome

RCT

Absolute Difference in HbA1c (%)

 Lower in Surgery Lower in Control

5 Year Follow Up Data


Mingrone G et al Lancet 2015
Schauer PR et al NEJM 2017
Muller-Stich Ann Surg 2015

2559
Copyright © Harvard Medical School, 2018. All Rights Reserved.

Obesity is a disease, not a behavioral problem


One size does not fit all: long-term strategy/relationship
Goal of medical interventions is 5-10% lost in 6 mos
Calorie restriction, Behavioral Strategies, Exercise
Meds: adjunct to lifestyle therapy, re-assess effect and
need for (dis)continuation
Surgery: RYGB, SG highly effective in “diabesity”
Lots of new guidelines to review!

1. Jensen MD, et al. 2013 AHA/ACC/TOS Guideline for the Management of


Overweight and Obesity in Adults. Circulation. 2014 Jun 24;129(25 Suppl
2):S102-38.
2. Apovian CM, et al. Pharmacological Management of Obesity: An Endocrine
Society Clinical Practice Guideline. J Clin Endocrinol Metab. 2015
Feb;100(2):342-62.
3. Garvey WT, et al. American Association of Clinical Endocrinologists and
American College of Endocrinology comprehensive clinical practice
guidelines for medical care of patients with obesity. Endocr Pract.
2016;22(Suppl 3):1-203
4. Mechanik, JI, et al. Clinical practice guidelines for the perioperative nutritional,
metabolic, and nonsurgical support of the bariatric surgery patient--2013
update: cosponsored by American Association of Clinical Endocrinologists, The
Obesity Society, and American Society for Metabolic &amp; Bariatric Surgery.
Obesity (Silver Spring). 2013 Mar;21 Suppl 1:S1-27
5. Heymsfield SB and Wadden TA. Mechanisms, Pathophysiology, and
Management of Obesity. N Engl J Med 2017; 376:254-266

2560
Copyright © Harvard Medical School, 2018. All Rights Reserved.

2561
Copyright © Harvard Medical School, 2018. All Rights Reserved.

A. Guidelines suggest prescribing caloric restriction for


weight loss: 1,200 -1,500 kcal/day for women and 1,500-
1,800 kcal/day for men
B. Guidelines suggest that a low-carbohydrate, low glycemic
index approach is preferred for diabetes risk reduction
and longer term weight loss maintenance
C. Self-monitoring by tracking food intake is associated with
successful weight loss, but monitoring weight is
associated with worse outcomes
D. Physical activity can augment weight loss, but beyond
150 mins/week little added benefit is observed

A
At this time, available evidence suggests that caloric
restriction and adherence (not a specific
macronutrient composition) is what matters the
most for weight loss
Monitoring weight, food intake and exercise are
recommended
Studies support that the more physical activity, the
more weight lost and maintained

2562
Copyright © Harvard Medical School, 2018. All Rights Reserved.

A. Discontinuation of the medication once the patient


has lost 5% of initial body weight
B. Discontinuation of the medication after 3 mos
(regardless of effect)
C. Discontinuation if the patient does not lose 5% of
initial body weight at 3 mos
D. Change to use as needed once the patient loses 5%
of initial body weight - to lower exposure/risks

C. Discontinuation of weight loss medications is


recommended if the patient does not lose 5% of initial
body weight at 3 mos
Close monitoring of patients on weight loss
medications is essential
There is a lot of biologic variability in response to
pharmacologic (and other) treatment approaches, so if
one approach is not working for an individual, it should
be re-addressed.
At all visits there should be an emphasis on
medications as adjunct to diet and exercise

2563
Copyright © Harvard Medical School, 2018. All Rights Reserved.

End of Life

Lisa Soleymani Lehmann, MD, PhD, MSc


VA National Center for Ethics in Health Care
Associate Professor of Medicine
Harvard Medical School
LzsLehmann@gmail.com

Disclosures

No Disclosures

2564
Copyright © Harvard Medical School, 2018. All Rights Reserved.

Disclaimer

The views expressed in this presentation do not


represent those of the Federal Government,
Veterans Health Administration, or the National
Center for Ethics in Health Care.

Overview
• Advance care planning (ACP)
– Understand the utility of advance directives
– A process that should focus on patient’s goals
and values
• Explore how to effectively communicate with
patients about goals of care and values
• Identify common pitfalls in communication
and how to avoid them
• Provide a framework for ACP conversations

2565
Copyright © Harvard Medical School, 2018. All Rights Reserved.

What is Advance Care Planning (ACP)?

• Advance care planning is a process that


involves making decisions about the care
patients would want to receive if they
become unable to speak for themselves
– An advance directive is one outcome of process
• The decisions should be based on their
personal values, preferences, and discussions
with their loved ones and doctors

National Hospice and Palliative Care Organization (NHPCO)

Clinical Indications for ACP


• Routine indications
– Discussing poor prognosis
– Discussing treatment with low probability of success
– MD wound not be surprised if the patient died in 6-12
months
– Reviewing health care maintenance with patient
• Urgent indications
– Imminent death
– Patient talks about wanting to die
– Recently hospitalized for severe progressive illness
– Severe suffering and poor prognosis

T Quill, JAMA 2000;284(19): 2502-7

2566
Copyright © Harvard Medical School, 2018. All Rights Reserved.

Benefits of Advance Care Planning


• Promotes patient autonomy
– Gives voice to patients’ preferences even when
they cannot speak
– Gives patients peace of mind that their
preferences will be respected
• Benefits for healthcare providers
– Avoids future confusion and conflict
• Benefits for family
– Decreases family angst about end-of-life decisions
– Diminishes family guilt if treatment is limited

Clinical Outcomes &


Advance Care Planning

• Decreased ICU admission


• Improved quality of life
• Increased hospice
• Improves family bereavement outcomes

-Wright AA et al. JAMA. 2008;300(14):1665-7.


-Temel JS et al. N Engl J Med 2010;363:733-42.
-Wright AA et al. JAMA. 2016;315(3):284-292.

2567
Copyright © Harvard Medical School, 2018. All Rights Reserved.

Patients Want to Discuss


End-of-Life Preferences
• 80% of patients want to talk about end-of-life
(EOL) wishes
– Only 7% had a doctor discuss it
• 82% say it is important to put their wishes
in writing
– Only 23% have actually done so
• 60% believe it is extremely important not to
burden family with tough EOL decisions
– 56% have not communicated their EOL wishes
-Californians’ Attitudes Toward End-of-Life Issues. 2011.

A Step-Wise Approach to ACP


1. Solicit patient’s understanding of his/her illness
2. Assess if the patient has a health care agent
- Encourage communication between patient and agent
3. Elicit patient’s goals and values
4. Offer a prognosis
-Share information on outcomes
5. Make a recommendation based on goals & values
6. Summarize
7. Document goals and values
8. Apply the directives when indication arises

2568
Copyright © Harvard Medical School, 2018. All Rights Reserved.

Beyond Advance Directives:


Discussing Patients’ Goals of Care

• Cure
• Aggressive treatment aimed at life
prolongation at all costs
• Treatment aimed at restoring prior level of
function
• Symptom control or comfort care

Questions to Elicit Goals of Care

2569
Copyright © Harvard Medical School, 2018. All Rights Reserved.

Questions to Elicit Patient’s Values


• Is there anything I should know about your beliefs
or wishes that would help me be sure you get the
care you want?

• When you think about the future, what do you


hope for? Is there any thing you want to avoid?

• When you think about the possibility that you may


get sicker, what worries you the most?

• If your health gets worse, what are the most


important things to you?

2570
Copyright © Harvard Medical School, 2018. All Rights Reserved.

Prognosis: Survival to Hospital


Discharge after CPR in the Elderly

• Medicare data on 433, 985 patients who


underwent in-hospital CPR
• Patients >65 years
• Overall survival 18.3%
-Ehlenbach WJ, et al. NEJM 2009; 361:22-31.

Prognosis: Survival in Cancer


Patients Undergoing CPR

- Reisfield GM, et al. Resuscitation. 2006;71:152-160.

2571
Copyright © Harvard Medical School, 2018. All Rights Reserved.

Outcomes of In-hospital CPR


United States 2000-2009
• All patients > 18 years of age
• Survival increased from 20% to 29%
• Discharge home decreased 36% to 24%
• Discharge to hospice or long term care increased
– Hospice 0.4% to 7%
– Long term care 1% to 9%
• Neurologic compromise increased by 38%
• Feeding tube use increased by 28%
• Ventilator use increased by 58%

-Kazaure, Roman, and Sosa. Resuscitation. 2013

ePrognosis
• Estimating prognosis for elders
• Repository of geriatric prognostic indices
• Guide for clinicians about mortality outcomes
• Internally validated in 218,088 nursing home
residents
• www.eprognosis.ucsf.edu
-JAMA 2010;304 (17):1929-1935.

2572
Copyright © Harvard Medical School, 2018. All Rights Reserved.

Make a Recommendation
• As with other treatment decisions, it is appropriate
to offer your recommendations

• Base on clinical situation, goals of care & patient


values

• Offers guidance and relieves patients & families of


some of the burden of decision-making

• Does not undermine patient autonomy

Common Pitfalls
• Physicians:
– Talk too much
– Fail to respond to patient emotions
– Use jargon
– Misses opportunities for empathic connection
– Fail to elicit patient’s values and goals
– Jumping straight to code status and treatment
preferences
– Offering forms and reading material without
discussion

-JGIM 1995;10:436-442.

2573
Copyright © Harvard Medical School, 2018. All Rights Reserved.

Questions to Guide Conversation


• What is your understanding of your illness?
• How much information do you want about what is likely to
happen in the future with your illness?
• Who would you want to make decisions for you if you cannot
communicate?
• What are your goals for medical care?
• What are your biggest fears and worries?
• If you become sicker, how much are you willing to go through in
order to have the possibility of more time?
• Are there certain health situations you would find unacceptable
or make your life not worth living? E.g. feeding tube, unable to
care for self, etc.
• Have you discussed your preferences with your family?

Video of Goals of Care


Conversation
Conducting a Goals of Care Conversation with
a Patient

8:06 Elicit values


10:14 Goals of care

2574
Copyright © Harvard Medical School, 2018. All Rights Reserved.

Advance Care Planning: Summary Points


• More than just filling out a form
• A process of understanding values and goals
• Begin with what the patient understands
• Listen more and talk less
• Share prognostic information & data on CPR
outcomes
• Include family in conversations
• Make a recommendation & document

Question 1
You have a patient with advanced multiple sclerosis
who has developed renal failure secondary to diabetes. The
patient is DNR. She presents to the ED unconscious with a K
of 8 meq/L.

Which is the most appropriate next step?


a. No intervention because she is DNR
b. Discuss a reversal of the DNR order and dialyze
c. Proceed with dialysis
d. Give sodium polystyrene sulfonate (Kalexate, Kayexalate,
Kionex) until you can find the health care agent to discuss
dialysis

2575
Copyright © Harvard Medical School, 2018. All Rights Reserved.

Question 1
You have a patient with severe advanced multiple sclerosis
who has developed renal failure secondary to diabetes. The
patient is DNR. She presents to the ED unconscious with a K
of 8 meq/L.

Which is the most appropriate next step?


a. No intervention because she is DNR
b. Discuss a reversal of the DNR order and dialyze
c. Proceed with dialysis
d. Give sodium polystyrene sulfonate (Kalexate,
Kayexalate, Kionex) until you can find the health care
agent to discuss dialysis

Question 1
Correct answer is proceed with dialysis.
• “Do-Not-Resuscitate” (DNR) is specifically defined
as refraining from cardiopulmonary resuscitative
efforts
• A DNR order should prompt a conversation about
the patient’s goals of care and raise the question
of whether she intended comfort care only
• DNR does not mean do not treat
• Hyperkalemia is life threatening. Sodium
polystyrene sulfonate (Kalexate, Kayexalate,
Kionex)is an inferior therapy for the long-term
management of renal failure

2576
Copyright © Harvard Medical School, 2018. All Rights Reserved.

Question 2
A 29 year old man sustained a C1 and C2 spinal fracture
during a boxing champing championship 3 months ago. He
is paralyzed from the neck down and is ventilator dependent.
He is fully alert and understands his condition. He requests
removal from the ventilator and understands that he will die
as a result.

The most appropriate next step is:


a. Assess for depression and if no evidence remove the
ventilator as requested
b. Obtain a court order to continue the ventilator
c. Seek family consensus on removing the ventilator
d. Seek approval of the health care agent

Question 2
A 29 year old man sustained a C1 and C2 spinal fracture
during a boxing champing championship 3 months ago. He
is paralyzed from the neck down and is ventilator dependent.
He is fully alert and understands his condition. He requests
removal from the ventilator and understands that he will die
as a result.

The most appropriate next step is:


a. Assess for depression and if no evidence remove the
ventilator as requested
b. Obtain a court order to continue the ventilator
c. Seek family consensus on removing the ventilator
d. Seek approval of the health care agent

2577
Copyright © Harvard Medical School, 2018. All Rights Reserved.

Question 2
Correct answer is assess for depression and remove the
ventilator.
• Any adult patient with the mental capacity to understand
his medical condition and the implications of withdrawal
of treatment has the right to do what he wants to his own
body
• There is no ethical distinction between withholding and
withdrawing life sustaining treatment
• Patients are frequently depressed following a high c-spine
injury
• The patient is alert so no consent of the family or health
care agent is necessary

Resources
• http://www.ethics.va.gov/goalsofcaretraining.asp
• www.makingyourwishesknown.com
• Online decision aid to create personalized advance directive
• MyDirectives.com
• Universal advance digital directive
• www.knowyourwishes.com
• End of life discussion compass guide
• MOLST (MA Medical Orders for Life Sustaining Treatment)
• Written instructions from a clinician to other health professionals
based on patient preferences
• theconversationproject.org
• A guide with prompts to facilitate EOL conversations
• https://www.prepareforyourcare.org

2578
Copyright © Harvard Medical School, 2018. All Rights Reserved.

References
1. Dying in America: Improving Quality and Honoring Individual
Preferences Near the End of Life. Institute of Medicine. 2014.
2. Clayton JM, Hancock KM, Butow PN, Tattersall MH, Currow
DC, Adler J, et al. Clinical practice guidelines for
communicating prognosis and end-of-life issues with adults
in the advanced stages of a life-limiting illness, and their
caregivers. Med J Aust. 2007;186(12 Suppl):S77, S9, S83-108.
3. Dingfield LE, Kayser JB. Integrating Advance Care Planning
Into Practice. CHEST 2017; 151(6):1387-1393.
4. Gehlbach TH et al. Code status orders and goals of care in the
medical ICU. Chest 2011;1394:802-809.
5. Dalal S, Bruera E. End-of-Life Care Matters: Palliative Cancer
Care Results in Better Care and Lower Cost. The Oncologist.
2017;22:361-368.

2579
Copyright © Harvard Medical School, 2018. All Rights Reserved.

Psychiatry Overview
General Internal Medicine
Board Review
Ann L. Pinto, MD PhD
Staff Physician
Departments of Internal Medicine and Primary Care
Brigham and Women’s Hospital
Instructor, Harvard Medical School

• No disclosures

2580
Copyright © Harvard Medical School, 2018. All Rights Reserved.

Question 1
• A 65 yo woman comes to the office because her
brother has been diagnosed with stage IV lung
cancer. She wants to be screened for lung cancer
herself. She smoked one pack of cigarettes daily for
40 years and quit 8 years ago.
• Which of the following discussion points about lung
cancer screening in asymptomatic current or former
heavy smokers is true ?

Question 1 (con’t.)
A. Yearly screening chest x-rays reduce lung cancer
mortality.
B. Annual low dose CT scanning (LDCT) results in a
20% relative reduction in lung cancer mortality.
C. Most lung cancers detected by LDCT are stage III or
IV.
D. 5% of LDCT scans had positive findings
E. 50% of the positive screens in LDCT represent
cancer.

2581
Copyright © Harvard Medical School, 2018. All Rights Reserved.

Answer: B
Lung cancer screening
• National Lung Screening Trial
• Enrolled 53,000 asymptomatic high risk smokers
– Ages 55-64
– Greater than 30-pack-year history of smoking
– Currently smoking or quit within the last 15 years
• Annual low dose CT screening for 3 years versus
annual screening by x-ray
• Median follow-up 6.5 years; study terminated early
due to benefit.

Answer: B
• Results of National Lung Screening Trial
– 20% relative reduction in lung cancer mortality.
– 6.7% reduction in all cause mortality.
– 24% of scans had positive findings, 95% of which
were NOT cancer.
– Most cancers (70%) were stage I or II.

NEJM 2013; 368:1980

2582
Copyright © Harvard Medical School, 2018. All Rights Reserved.

Question 2
• 42 yo male complains of fatigue, low libido and
erectile dysfunction. His exam is notable for
gynecomastia. Total testosterone 180 mg/dL, normal
FSH and LH. What do you recommend as a next step?
A. Testosterone replacement therapy
B. Pituitary evaluation
C. Sildenafil
D. Relationship counseling
E. Semen analysis

Answer: B
Secondary hypogonadism
• Secondary hypogonadism: reproducibly low
testosterone with inappropriately low/normal FSH
and LH indicate a pituitary problem.
• Evaluation should include:
• Prolactin/other tests of pituitary function
• Iron studies (r/o hemochromatosis)
• Pituitary MRI
• Exclude excessive exercise/eating d/o.
• No semen analysis unless fertility is desired.
Lancet 2014; 383:1250-63.

2583
Copyright © Harvard Medical School, 2018. All Rights Reserved.

Question 3
• 83 yo F with an ischemic cardiomyopathy and EF15%
is admitted with pulmonary edema for the fourth
time this year. Despite aggressive diuresis, she
remains volume overloaded with declining renal
function.
• She wants to go home but her son disagrees and
wants aggressive care. You recommend a palliative
care consultation.

Question 3 (con’t.)
• Which of the follow is true regarding palliative care in
patients with advanced heart failure?
A. Patients should be referred when curative
therapies have been exhausted.
B. Palliative care requires discontinuation of active
treatment.
C. The onset of functional decline in heart failure
correlates strongly with 6 month prognosis.
D. Palliative care focuses on the psychosocial needs
of pts and families in addition to physical needs.

2584
Copyright © Harvard Medical School, 2018. All Rights Reserved.

Answer: D
Palliative care/advanced HF
• Heart failure has an unpredictable clinical trajectory,
unlike cancer.
• Palliative care focuses on maximizing QOL and does
not preclude therapies designed to prolong survival
• Can be initiated at any time.
• Shared decision making: patient’s goals and wishes
• Educates patients and families about the future and
encourages advance care planning.
• Eases transition to hospice care when needed.

Question 4
• 65 yo professor presents after a colleague found him
wandering, unable to find his office. His wife reports
several falls, 2 episodes where he thought he saw an
another person at the dinner table with them, and
that he thrashes around violently in his sleep. Only 1
year ago he was awarded a major prize for his
research.
• On exam he is orthostatic with slowed speech, and
some limb rigidity. No tremor. He has marked
difficulty with clock drawing. B12, TSH, RPR are nl.

2585
Copyright © Harvard Medical School, 2018. All Rights Reserved.

Question 4
• Based on these findings, the most likely diagnosis is:
A. Alzheimer’s disease
B. Parkinson’s disease
C. Lewy body dementia
D. Multi-infarct dementia
E. Normal pressure hydrocephalus

Answer: C
Lewy body dementia
• Early deficits in visuospatial and executive functioning
• Typically shorter course than AD, often with rapid decline.
• Visual hallucinations - highly specific for Lewy body
• REM sleep disorder: “acting out dreams”
• Parkinsonian symptoms, especially bradykinesia and
stiffness
• Onset of dementia and Parkinsonian symptoms typically
within 1 year of each other
• Cognitive fluctuations with variable attention/alertness
• Autonomic dysfunction (orthostasis -> falls are common)

2586
Copyright © Harvard Medical School, 2018. All Rights Reserved.

Question 5
• 27 yo M presents to the ER with a food impaction.
He notes worsening heartburn and difficulty
swallowing that has not responded to OTC
omeprazole. His medical history is significant only for
eczema. What is the most likely diagnosis?
A. Candida esophagitis
B. Achalasia
C. Diffuse esophageal spasm
D. GERD
E. Eosinophilic esophagitis

Answer: E
Eosinophilic esophagitis
• Male preponderance (M:F 3:1)
• Peak incidence: Childhood or 3rd-4th decade
• Patients typically have a history of atopy
• Most common presentation: dysphagia for solids
• Food impaction is common (33-50% of patients)
• Diagnosis: EGD with biopsy showing eosinophils PLUS lack of
response to PPI.
• Treatment:
– Swallowed inhaled corticosteroids
– Dietary modification
• Relapse is common if treatment is discontinued

Am. J. Gastroenterology 2013; 108:679-692

2587
Copyright © Harvard Medical School, 2018. All Rights Reserved.

Question 6
• 58-year-old M presents with burning neck pain that
radiates down his L arm. Despite high-dose NSAIDs,
his pain is 8/10 in severity. He has a history of
hypertension and chronic headache. Medications
include imipramine, lisinopril and trazodone. You
diagnose a cervical radiculopathy, refer him for PT
and prescribe tramadol for pain. 4 hours later he is
brought to the ER with altered mental status,
agitation and fever to 104oF. On exam he has dilated
pupils, hyperreflexia and spontaneous clonus.

Question 6 (con’t.)
• Based on this history, the most likely diagnosis
is:
A. Intracerebral hemorrhage
B. Bacterial meningitis
C. Anticholinergic toxicity
D. Serotonin syndrome
E. Neuroleptic malignant syndrome

2588
Copyright © Harvard Medical School, 2018. All Rights Reserved.

Answer: D
Serotonin syndrome
• Precipitated by use of serotonergic drugs
– Not just SSRIs!
– SSRI/SNRIs, MAOIs, TCAs, trazodone, opiates including
tramadol, drugs of abuse, some antibiotics (linezolid,
cipro)
• Symptoms
– fever, AMS, rigidity, spontaneous clonus, hyperreflexia,
autonomic instability
– usually within 24 hr of initiation/dose change of culprit
drug, commonly <6 hours
– can vary from mild to life-threatening and are often
missed/attributed to other causes
• Treatment: discontinue medication/supportive care

Question 7
• A 68-year-old male presents for follow-up
after a hospital admission for decompensated
CHF. His medical history is notable for CAD
with an ischemic cardiomyopathy and EF of
30%. He notes dyspnea with mild exertion and
worsening lower extremity edema.

2589
Copyright © Harvard Medical School, 2018. All Rights Reserved.

Question 7 (con’t.)
• Which of the following medications does not
have a mortality benefit in patients with class
III CHF?
A. Aliskiren
B. Candesartan
C. Metoprolol
D. Enalapril
E. Spironolactone

Answer: A
Management of CHF
• Aliskiren+enalapril led to more adverse effects vs.
enalapril alone without decrease in death or HF
hospitalization. Aliskiren alone did not meet criteria for
non-inferiority vs. enalapril. (ATMOSPHERE 2016)
• ACE-inhibitors improve mortality in pts with symptomatic
and asymptomatic LV dysfunction (CONSENSUS, SOLVD)
• Beta-blockers improve survival in pts with systolic
dysfunction, usually after stabilization on a diuretic and
an ACE-inhibitor (MERIT-HF)
• ARB use in ACE-inhibitor intolerant pts improves survival
in pts with CHF/reduced EF (CHARM-Alternative)
• Spironolactone has a mortality benefit in pts with Class III
or IV heart failure (RALES)

2590
Copyright © Harvard Medical School, 2018. All Rights Reserved.

Question 8
• A 19-year-old female with a history of IUD
placement 3 weeks ago presents with
complaints of crampy lower abdominal pain
and dyspareunia. She is afebrile. Her exam is
notable for cervical friability and bilateral
adnexal tenderness. Pregnancy test is
negative.

Question 8 (con’t.)
• What treatment to you recommend?
A. Azithromycin 2 g po ×1
B. Ciprofloxacin 500 mg b.i.d. + metronidazole 500
mg b.i.d. ×14 days
C. Ceftriaxone 250 mg IM + doxycycline 100 mg
b.i.d. + metronidazole 500 mg b.i.d. ×14 days
D. Drug regimen in answer C plus removal of the
IUD.

2591
Copyright © Harvard Medical School, 2018. All Rights Reserved.

Answer: C
Pelvic inflammatory disease
• Ascending infection of the female upper genital tract
• Major cause of infertility and ectopic pregnancy
• Polymicrobial infection: C. trachomatis and N.
gonorrhoeae, gram-negatives, anaerobes, and
streptococci
• Presentation: lower abdominal or pelvic pain
– Dyspareunia, intermenstrual or post-coital bleeding
– Vaginal discharge, dysuria also common
• Negative endocervical GC/chlamydia testing does not
rule out upper tract infection and treatment
regimens need to cover these

Answer: C
Pelvic inflammatory disease
• Treat if any one: uterine, adnexal or cervical motion
tenderness - microbial testing is not needed.
• Ceftriaxone 250 mg IM + doxycycline 100 mg bid x 14 days
(covers gonorrhea and chlamydia respectively)
• Replacing doxycycline, above, with azithromycin 2 g x1 is an
option for patients for whom compliance may be an issue.
• Increasing resistance of N. gonorrhoeae to fluoroquinolones
precludes use.
• Need for anaerobic coverage not definitive, BUT most
guidelines recommend addition of metronidazole (also covers
Trichomonas).
• IUDs do not need to be removed unless failure to improve.
• Always screen for HIV and other STDs in pts with PID.

2592
Copyright © Harvard Medical School, 2018. All Rights Reserved.

Question 9
A 33-year-old female presents to establish
care. Her medical history is significant for
Hodgkin’s lymphoma diagnosed at age 16 and
treated with ABVD (adriamycin, bleomycin,
vinblastine, dacarbazine) plus mantle
irradiation. She has been free of disease since
completion of therapy. She feels entirely well
and has no complaints.

Question 9 (con’t.)
• Which of the following screening tests is
indicated in this 31-year-old woman?
A. Annual mammograms
B. Annual breast MRIs
C. Echocardiogram every other year
D. Annual TSH
E. All of the above

2593
Copyright © Harvard Medical School, 2018. All Rights Reserved.

Answer: E
Hodgkin’s lymphoma survivorship
• General principles of pediatric cancer survivorship
– Childhood cancer survivors are at increased risk of
treatment-related toxicities – both malignant and
non-malignant
– Risk varies with treatment regimen
– Imperative to know specifics of treatment and
manage accordingly
– High index of suspicion for serious illness even in
young patients

Answer E
Hodgkin’s lymphoma survivorship
• 18-fold increased risk of secondary malignancy
– Risk of cancer: 26% at 30 years post tx; higher in women
– 13% of female survivors will get breast cancer by age 40
• Cardiovascular complications from mediastinal
irradiation increased with anthracycline treatment
– Accelerated atherosclerosis primary cause of death
– Important valvular disease, LV dysfunction, arrhythmias
• Thyroid dysfunction in as many as 30% of survivors
• Infertility/premature menopause common
• High incidence of anxiety and PTSD

2594
Copyright © Harvard Medical School, 2018. All Rights Reserved.

Answer E
Hodgkin’s lymphoma survivorship
• Female patients with chest irradiation
– Mammograms AND breast MRI annually from age 25 or 8
years post treatment (whichever is latest)
• Radiation (chest) + anthracycline treatment
– Echocardiogram/EKG at least q2yr
• Thyroid testing annually
• If pelvic or abdominal radiation (not this patient)
– Colonoscopy at 35 or 15 yr post treatment
• Don’t forget immunizations especially if s/p
splenectomy

• Clinical guidelines for follow up according to


specific treatment regimens: Children’s
Oncology Group Long-Term Follow-up
Guidelines for Survivors of Childhood,
Adolescent and Young Adult Cancer – great
resource
• www.survivorshipguidelines.org

2595
Copyright © Harvard Medical School, 2018. All Rights Reserved.

Question 10
• 46 yo M presents for follow-up of poorly controlled
hypertension. He takes lisinopril 40 mg, HCTZ 25 mg,
and amlodipine 10 mg. He is compliant with his
medications. BP is 152/92. What would be the best
choice of an additional medication to control his BP?
A. Aliskiren
B. Spironolactone
C. Doxazosin
D. Bisoprolol
E. Valsartan

Answer: B
Resistant hypertension
• Hypertension despite use of 3 anti-hypertensives of
different classes, one of which is a diuretic.
• PATHWAY-2 study (2015): for patients with resistant
hypertension (pts were on ACE/ARB, CCB and
diuretic), spironolactone was most effective
additional agent vs bisoprolol or doxazosin
• ACC/AHA 2017 hypertension guidelines:
spironolactone (or eplerenone) is the preferred
medication for resistant hypertension.
• Avoid dual blockade of renin-angiotensin system
(increased risk of AKI, hyperkalemia)
• Consider evaluation for secondary causes of HTN.
Lancet 2015; 386:2059
J Am Coll Cardiol 2018;71:e127-e248.

2596
Copyright © Harvard Medical School, 2018. All Rights Reserved.

Question 11
24 yo woman presents with dysuria and frequency
without fever, back pain or vaginal discharge. Exam
is notable only for mild suprapubic tenderness.
Which of the following is NOT recommended as first
line therapy?
A. Fosfomycin 3 g po x1
B. Ciprofloxacin 250 mg bid x 3 days
C. Bactrim DS 1 po bid x 3 days
D. Nitrofurantoin 100 mg po bid x 5 days

Answer: B
Acute uncomplicated cystitis
• Culture not necessary for uncomplicated cases
• Complicated UTI: male, pregnant or with structural
abnormalities
• 80% E.coli, 10% other GNR, remainder gram +
• Base treatment on local resistance, allergy, cost
• Hospital resistance data skew community estimates
• FQ highly effective but reserve use for more
serious infections – can cause collateral damage
• FQ use linked with infection with FQR GNRs, MRSA
• Tendinitis, QT-prolongation
IDSA guideline acute cystitis/pyelonephritis
Clinical Infectious Diseases ; 2011 ; 52 : e103 -e120

2597
Copyright © Harvard Medical School, 2018. All Rights Reserved.

Question 12
• Which of the following patients should be treated for
latent TB infection?
A. 58-year-old male starting hemodialysis with PPD
10 mm
B. 27-year-old student from Uganda with PPD 5 mm
C. 34-year-old injection drug user with cough,
weight loss
D. 24-year-old suburban teacher with PPD 10 mm
E. 45-year-old healthy spouse of patient with active
pulmonary TB and negative PPD

Answer: A
Latent TB infection
• 11 million people in US have LTBI; 5-10% will
reactivate if not treated
• Reactivation most common cause of new TB in US
• Goal: to screen those at high risk for reactivation:
– Recent PPD conversion
– HIV-positive
– Patients on immunosuppresion
– Patients on hemodialysis
– Close contacts of patients with active TB

2598
Copyright © Harvard Medical School, 2018. All Rights Reserved.

Answer: A
Latent TB infection
• Who should be treated for LTBI?
• PPD 5 mm:
– HIV positive,
– immunosuppressed,
– close contacts of pts with TB,
– patients with an abnormal CXR suggesting healed TB
• PPD 10 mm:
– IV drug users
– Homeless or residents/employees of congregate housing
– Pts with hematologic or head and neck cancer
– Patients on dialysis
– Recent arrivals from high incidence areas

Answer: A
Latent TB infection
Specific answer for this case:
A. Hemodialysis patients are at high risk for reactivation
and should be treated
B. Borderline positive test and likely acquired from early
exposure; low risk of reactivation
C. Symptoms of active pulmonary TB and should be tested
with sputum AFB testing
D. Low risk patients should not be tested but if they are,
should not be treated unless PPD>15 mm
E. This patient tested negative and does not need
treatment now; however, she is at high risk of
contracting TB and should be retested 8-10 weeks after
last exposure

2599
Copyright © Harvard Medical School, 2018. All Rights Reserved.

Question 13
• A 68-year-old male complains of urinary
urgency, frequency and awakening 3 times a
night to urinate. He denies hesitancy, dribbling
or weak stream. He has cut back on caffeine
and alcohol with no improvement. On exam,
his prostate is mildly enlarged, smooth and
symmetrical. Post void residual is 30 mL.
Urinalysis is unremarkable; PSA and renal
function are normal.

Question 13 (con’t.)
The best first choice of medication for him
would be:
A. Tamsulosin
B. Oxybutynin
C. Finasteride
D. Sildenafil
E. Bladder botulinum toxin

2600
Copyright © Harvard Medical School, 2018. All Rights Reserved.

Answer: B
Male lower urinary tract symptoms
• Lower urinary tract symptoms
– Bladder storage phase symptoms:
• Urgency, frequency , nocturia, involuntary loss of urine
– Bladder outlet obstruction symptoms:
• Hesitancy, incomplete emptying (elevated post void
residual), weak stream, dribbling
• This patient has an enlarged prostate but no
symptoms of bladder obstruction
• First choice for overactive bladder without bladder
outlet obstruction is an anti-cholinergic

Question 14
• Disease X has a prevalence of 10% in your
clinic. If you have a test for X that is 90%
sensitive and 90% specific, what fraction of
patients with a positive test truly have X?
A. 50%
B. 67%
C. 75%
D. 90%

2601
Copyright © Harvard Medical School, 2018. All Rights Reserved.

Answer: A
Biostatistics
With Without
disease disease
Positive 9 9 18
test
Negative 1 81 82
test
10 90 100
• SENSITIVITY: % patients with disease who test positive: 9/10 = 90%
• SPECIFICITY: % patients without disease who test negative: 81/90 = 90%
• POSITIVE PREDICTIVE VALUE: % patients with positive test who have the
disease: 9/18 = 50%

Biostatistics:
• Sensitivity quantifies avoiding false negatives.
Therefore a negative result from a high
sensitivity test is likely to be a true negative
and rules out disease
• Specificity quantifies avoiding false positives.
Therefore a positive result from a high
specificity test is likely to be a true positive
and rules in disease
• Neither are dependent upon prevalence

2602
Copyright © Harvard Medical School, 2018. All Rights Reserved.

Question 15
• 64 yo male with HTN, diabetes, obesity
presents with his 4th episode of acute gout
this year despite dietary modification. He is
anxious to treat his current symptoms and to
prevent further attacks. Exam today is notable
for erythema and exquisite tenderness of the
L 1st MTP joint and adjacent toe. No tophi are
noted. Renal function is normal.

Question 15 (con’t)
What is the best plan to treat his current
symptoms and prevent further attacks?
A. Indomethacin until flare resolves then
discontinue indomethacin and start allopurinol
B. Colchicine and allopurinol indefinitely
C. Febuxostat indefinitely
D. Colchicine indefinitely
E. Colchicine until flare resolves then add
allopurinol; discontinue colchicine after 3
months overlap, continue allopurinol indefinitely

2603
Copyright © Harvard Medical School, 2018. All Rights Reserved.

Answer: E
Management/prophylaxis of gout
• An acute attack should be treated within 24 hours
– NSAIDS
– Colchicine
– Oral corticosteroids (may consider intra-articular
steroids if 1-2 large joints affected)
• If a patient is already on urate-lowering therapy
(ULT), that should be continued without interruption
during treatment of an acute attack
• Consider starting ULT if ≥ 2 attacks/yr

Answer: E
Management/prophylaxis of gout
• Urate lowering therapy (ULT) should not be initiated
until the acute flare has resolved
• Dose of ULT can be titrated up every 4 weeks; goal
serum urate <6 mg/dL
• Initiation of ULT can precipitate a gout flare and
should not be prescribed without inflammatory
prophylaxis (NSAIDs, colchicine).
• Anti-inflammatory prophylaxis should be continued
after initiation of ULT and continued for 3 months
after target serum urate is reached

2604
Copyright © Harvard Medical School, 2018. All Rights Reserved.

Management/prophylaxis of gout
• Colchicine should not be used indefinitely due
to possible development of serious toxicities
(esp. in CKD) such as neuromyopathy.
• HCTZ can raise, and losartan decrease, uric
acid levels – modification of antihypertensives
may be useful.
• Caution using allopurinol in some Asian
populations.

Question 16a
• A 24 yo male camp counselor from
Massachusetts presents with 3 days of
intermittent fever as high as 104o, malaise,
myalgias and headache. Exam reveals a
temperature of 101.7 and a palpable spleen
tip.
• Lab testing: wbc 3.3, hct 32.3, plt 84, ALT 135,
AST 106, total bilirubin 2.8, direct bilirubin 0.3,
cr 0.9.

2605
Copyright © Harvard Medical School, 2018. All Rights Reserved.

Question 16a (con’t.)


• The most likely diagnosis is:
A. Tick-borne relapsing fever
B. Lyme disease
C. Rocky mountain spotted fever
D. Human granulocytic anaplasmosis
E. Babesiosis

Answer: E
Babesiosis
• Causative organism: Babesia microti in USA.
• Co-infection with Lyme common
• Asymptomatic to severe/life-threatening infection
• Asplenic/immunocompromised pts are at high risk
• Symptoms: fever, sweats, myalgias, arthralgias
• Laboratory testing: transaminitis, hemolytic anemia,
thrombocytopenia are common
• Confirmatory testing: PCR or thin smear showing
“Maltese cross” inclusions

2606
Copyright © Harvard Medical School, 2018. All Rights Reserved.

Question 16b
PCR confirms your diagnosis of babesiosis. What
is the best treatment for him?
A. Doxycyline
B. Cefuroxime
C. Amoxicillin-clavulanic acid
D. Clindamycin plus quinine
E. Azithromycin plus atovaquone

Answer: E
Babesiosis treatment
• Mild disease: atovaquone + azithromycin
• Severe disease (consider if elderly, immuno-
suppressed (medication/splenectomy/HIV+)):
clindamycin + quinine
• Doxycycline (used to treat Lyme) does not
cover babesiosis.

2607
Copyright © Harvard Medical School, 2018. All Rights Reserved.

Question 17
• 32 yo F presents for preconception counseling.
She has hypertension for which she takes
HCTZ. What is the best plan for managing her
hypertension given an anticipated pregnancy?
A. Continue HCTZ
B. Replace HCTZ with spironolactone
C. Replace HCTZ with nifedipine
D. Replace HCTZ with lisinopril
E. Replace HCTZ with atenolol

Answer: C
Chronic hypertension in pregnancy
• Chronic HTN: HTN that predates pregnancy.
• Pre-eclampsia: HTN associated with
– Proteinuria (no longer essential for diagnosis)
– Platelets <100,000
– Abnormal LFTs (transaminases ≥ 2x ULN)
– Renal impairment/pulmonary edema/visual
changes
• Gestational HTN: HTN that begins after the 20th
week of pregnancy without end-organ damage

2608
Copyright © Harvard Medical School, 2018. All Rights Reserved.

Chronic HTN in pregnancy


• First line choices in pregnancy: nifedipine, labetalol
and methyldopa.
• Diuretic use is controversial in absence of edema; not
favored
• Atenolol has possible increased risk of IUGR: avoid
• Avoid medication that affects the renin-angiotensin
system (Ace I, ARB, mineralocorticoid antagonists)
due to increased risk of fetal renal abnormalities,
IUGR, fetal death
ACOG Guidelines for Hypertension in
Pregnancy www.ACOG.org

Question 18
• A 48 yo male with hypertension, hyperlipidemia and
diabetes is found to have persistent mild elevation of
transaminases. His only medication is lisinopril. He
rarely drinks alcohol.
• BMI is 35 with notable central adiposity. There are
no stigmata of advanced liver disease. Hepatitis
serologies are negative, transferrin saturation is
normal and testing for autoimmune liver disease is
negative. Ultrasound shows diffuse hepatic steatosis.

2609
Copyright © Harvard Medical School, 2018. All Rights Reserved.

Question 18 (con’t.)
In addition to weight loss, what is the most
appropriate pharmacologic intervention?
A. Pioglitazone
B. Vitamin E
C. Atorvastatin
D. Ursodeoxycholic acid
E. Metformin

Answer: C
NAFLD spectrum
• Non-alcoholic fatty liver (20% US population)
– No inflammation, low risk of progression to cirrhosis
• NASH (non-alcoholic steatohepatitis) (1.5%-6.5%)
– + inflammation, +/- fibrosis
– 20% will progress to cirrhosis
• NASH cirrhosis
– 2-3% will develop HCC
• Risk factors for more serious disease: DM, high BMI/visceral
adiposity, metabolic syndrome
• Increased mortality: CVD is most common cause
– This patient should have aggressive risk factor reduction and statins should
NOT be avoided

2610
Copyright © Harvard Medical School, 2018. All Rights Reserved.

Answer: C
NAFLD spectrum
• Medication should be reserved for patient
with biopsy-proven NASH with fibrosis
• Pioglitazone improves histology in NASH
• long term safety concerns (heart failure, bone loss)
• Vitamin E improves histology in non-diabetic
patients
• possible increase in all cause mortality, prostate cancer
• Metformin, ursodeoxycholic acid have not been
shown to have benefit
AASLD guidelines:
Hepatology 2018;67:328-357

Question 19
• A 35-year-old female presents with complaints of
fever and ankle pain. Her exam is notable for clear
lungs, ankle swelling and dusky, tender nodules on
her lower extremities. A chest x-ray shows bilateral
hilar adenopathy. You diagnose her with acute
sarcoidosis. Which of the following statements
about sarcoidosis is true?

2611
Copyright © Harvard Medical School, 2018. All Rights Reserved.

Question 19 (con’t)
A. Sarcoidosis most commonly affects the lungs and
pulmonary function tests typically show obstruction.
B. Sarcoidosis frequently does not require treatment and it
is unclear if treatment helps prognosis.
C. Atrial fibrillation is the most common presentation of
cardiac sarcoidosis
D. A biopsy showing caseating granulomas is required for
diagnosis
E. Sarcoidosis is most commonly seen in white Americans

Answer: B
Sarcoidosis
• Idiopathic granulomatous disease
• 3-4x more common in blacks in the U.S.
• Biopsy: non-caseating granulomas.
• Can affect any organ system; lungs, skin, eyes most
common. Cardiac and CNS sarcoid have highest
morbidity.
• Clinical course ranges from asymptomatic to life-
threatening.

2612
Copyright © Harvard Medical School, 2018. All Rights Reserved.

Sarcoidosis
• Lungs most commonly affected (95%). PFTs are
frequently normal; most common abnormality is
restrictive lung disease.
• Most common cardiac symptom is heart block; VT is
the most common arrhythmia.
• Lofgren’s syndrome (case presentation) does not
require biopsy for diagnosis and typically does not
require treatment.
• Many cases do not require treatment and it is not
clear if treatment improves survival.

Question 20
• A 65 yo female presents with 2 weeks of progressive
knee pain that began after an evening of dancing at a
wedding. Pain is worse with climbing stairs. On exam
there is swelling and tenderness about 2 inches
below the joint, medially. The most likely diagnosis is:
A. Meniscal tear
B. Osgood-Schlatter syndrome
C. Iliotibial band syndrome
D. Anserine bursitis
E. Patellar teninditis

2613
Copyright © Harvard Medical School, 2018. All Rights Reserved.

Answer: D
Anserine bursitis
• Overuse injury – often develops after new activity
• Inflammation of bursa located between proximal
anteromedial tibia and conjoined tendon (medial
hamstring tendons)
• Risk factors: tight hamstrings, valgus knee
deformity, flat feet, female gender, obesity,
repetitive activities, OA
• Treatment: Icing, stretches, NSAIDs

Question 21
• A 67 yo male presents for follow up after a
spell of L arm weakness that lasted 30 minutes
then resolved spontaneously. His medical
history is significant for HTN and
hyperlipidemia. His neurologic exam is
completely normal. MRI brain/MRA head and
neck showed no infarct or hemodynamically
significant stenosis. Echocardiogram was
normal and 30 day event monitor showed no
arrhythmia.

2614
Copyright © Harvard Medical School, 2018. All Rights Reserved.

Question 21 (con’t.)
• Which of the following is the best option for
long-term secondary prevention of stroke in
this patient who has had a TIA?
A. Warfarin
B. Ticlopidine
C. ASA
D. Cilostazol
E. ASA + clopidogrel

Answer: C
Secondary prevention of stroke
• Antiplatelet agents are useful for secondary
prevention; current AHA/ASA guidelines recommend:
– ASA (low doses as effective as high doses)
– ASA + dipyridamole (Aggrenox)
– Clopidogrel
– On average, risk of stroke reduced by 22% with these
agents
• Long term use of ASA + clopidogrel led to an
increased risk of bleeding with no reduction in
vascular risk (MATCH trial)

2615
Copyright © Harvard Medical School, 2018. All Rights Reserved.

Answer: C
Secondary prevention of stroke
• Warfarin should not be used in patients with a non-
cardioembolic TIA/stroke
• Ticlopidine is effective for prevention but is
associated with significant side effects (severe
neutropenia, rash, diarrhea) that make it less
desirable.
• Cilostazol has shown some promise in early trials in
Asia but there is insufficient evidence to recommend
it at this time.

Question 22
• 44 yo male presents with pain/stiffness in his hands.
He also notes some erectile dysfunction. He has
recently been diagnosed with diabetes and is being
evaluated for abnormal liver function tests. On exam
he has bony enlargement of 2nd, 3rd MCPs but no
synovitis. What test is most likely to be diagnostic?
A. Anti-nuclear antibody
B. Transferrin saturation
C. Anti-cyclic citrullinated peptide antibody
D. Rheumatoid factor
E. No laboratory testing needed

2616
Copyright © Harvard Medical School, 2018. All Rights Reserved.

Answer: B
Hemochromatosis
• Iron overload as a result of mutations in HFE gene;
affects 1 in 200-500 people in US
• Symptoms:
– Early: arthropathy, fatigue, impotence
– Late: diabetes, cirrhosis, cardiomyopathy
• Polyarticular/symmetric arthritis: classic finding is
bony enlargement of 2nd/3rd MCPs without synovitis
• Screen: Fe/TIBC (>60% men, >50% women); positive
screens should be followed by genetic testing.

Question 23
• A 21 yo college student presents to your office
complaining of fever, cough, sneezing and
watery eyes. His medical history is
unremarkable and his immunizations are up to
date. He takes no medications and has no
allergies. Exam is significant for a temperature
of 104o, conjunctival injection, clear lungs.
You also note white spots on a red background
on the buccal mucosa.

2617
Copyright © Harvard Medical School, 2018. All Rights Reserved.

Koplik spots
CDC Public Health Image Library

Question 23 (con’t.)
• Which of the following is true regarding this case?
A. Diagnosis is clinical
B. Antiviral medication reduces the duration of
illness
C. Encephalitis is the most common cause of
associated death
D. The patient should be placed on contact
precautions
E. Post-exposure prophylaxis with ciprofloxacin
should be provided for susceptible contacts.

2618
Copyright © Harvard Medical School, 2018. All Rights Reserved.

Answer: A
Measles (rubeola)
• Incubation period: 10-12 days post-exposure
• Prodrome: fever (often high), cough, coryza,
conjunctivitis, Koplik spots (pathognomonic)
• Maculopapular rash: 2-4 days after onset of fever
• Rash starts at head and proceeds downwards to
trunk and then extremities
• Contagious 4 days before/4 days after rash develops
• Attack rate for susceptible patients is 90%
• 1 dose MMR 90% protective
• 2 doses MMR 97% protective

Answer: D
Measles (rubeola)
• In US in 2017: 118 cases in 18 states and D.C.
• Diagnosis is clinical with confirmation by PCR of
nasopharyngeal swab/serologies
• Treatment is supportive – anti-virals do not help
• Patients should be placed in respiratory isolation
• 30% of cases have complications
– Most common: otitis media, diarrhea
– Pneumonia most common cause of death
– Rare: encephalitis, seizure, death (0.2%)

2619
Copyright © Harvard Medical School, 2018. All Rights Reserved.

Answer: D
Measles (rubeola)
• Susceptible contacts should be vaccinated
within 72 hours of exposure
• IVIG for high risk/immunocompromised up to
6 days post exposure
• Report to local health authority or CDC
immediately
• NO ASSOCIATION between MMR vaccine and
autism

Question 24
Potential drawbacks to randomized controlled
trials include all but the following?
A. Internal validity
B. External validity
C. Selective dropout
D. Ethicality

2620
Copyright © Harvard Medical School, 2018. All Rights Reserved.

Answer: A
Trial design
Internal validity: confidence that we can place
in the cause and effect relationship in a
scientific study. Major design goal of RCT.
External validity: generalizability of results to
other populations.
Selective dropout: loss of particular, non-
random participants in a study.
Ethicality: potential conflict between research
goal and responsibility to individual patient.

2621
Copyright © Harvard Medical School, 2018. All Rights Reserved.

BIOSTATISTICS BOARD REVIEW

JULIE E. BURING, ScD


Senior Epidemiologist
Division of Preventive Medicine, Department of Medicine
Brigham and Women’s Hospital
Professor of Medicine
Harvard Medical School

No Disclosures

1
2622
Copyright © Harvard Medical School, 2018. All Rights Reserved.

Topics To Be Covered

• Screening (sensitivity, specificity,


predictive value, biases)
• Measuring data
• Frequency (incidence, prevalence)
• Association (ratio, difference)
• Interpretation of data
• Types of studies
• Valid statistical association

SCREENING

2
2623
Copyright © Harvard Medical School, 2018. All Rights Reserved.

Question 1

100 women over the age of 50 received mammograms


at a mobile breast cancer screening unit.
27 women had findings suspicious for malignancy on
the mammogram: 19 of these women were confirmed
as having breast cancer by biopsy (true positives).
1 woman had a negative mammogram but in the
subsequent year developed breast cancer, and is
assumed to have had the disease at the time of
screening (false negative).
What is the sensitivity of the mammogram? the
specificity? the predictive value of a positive test?

“truth”
Disease Status
Yes No

+ a b a+b
Screening
Test
- c d c+d

a+c b+d
T+ a
Sensitivity = =
Dx+ a+c
T- d
Specificity = Dx- = b + d

3
2624
Copyright © Harvard Medical School, 2018. All Rights Reserved.

Breast Cancer
+ -

+ 19 8 27
Mammogram
- 1 72 73

20 80 100
T+ 19
Sensitivity = Dx+ = = 95%
20

T- 72
Specificity = Dx- = 80 = 90%

“truth”
Disease Status
Yes No

+ a b a+b
Screening
Test
- c d c+d

a+c b+d
Dx+ a
Predictive Value (+) = PV(+) = = a+b
T+
Dx- d
Predictive Value (-) = PV (-) = = c+d
T-

4
2625
Copyright © Harvard Medical School, 2018. All Rights Reserved.

Breast Cancer
+ -

+ 19 8 27
Mammogram
- 1 72 73

20 80 100

Dx+ 19
PV (+) = = = 70%
T+ 27

Dx- 72
PV (-) = T- = 73 = 99%

What influences sensitivity,


specificity, and predictive value?

5
2626
Copyright © Harvard Medical School, 2018. All Rights Reserved.

Question 2

In a Mantoux tuberculosis screening program in a


high risk population, a positive test was defined as
10 mm of induration. If guidelines change, and a
positive test is now defined as only 5 mm of
induration, which of the following will be true?
More than one answer may be correct.
A. Sensitivity will increase
B. Specificity will decrease
C. Positive predictive value will increase
D. False positives will increase
E. False negatives will decrease

“Criterion of positivity” influences both


sensitivity and specificity of the screening test:

Criterion of positivity sensitivity


specificity

Criterion of positivity sensitivity


specificity

PV+ affected by sensitivity and specificity, but


increased mainly by increase in underlying
disease prevalence (like screening a higher risk
group).

6
2627
Copyright © Harvard Medical School, 2018. All Rights Reserved.

Question 2
In a Mantoux tuberculosis screening program in a
high risk population, a positive test was defined as
10 mm of induration. If a positive test is now
defined as only 5 mm of induration (i.e., ↓ the
criterion of positivity), which of the following will
be true? More than one answer may be correct.
A. Sensitivity will increase
B. Specificity will decrease
C. Positive predictive value will increase
D. False positives will increase
E. False negatives will decrease

Correct answers: A, B, D, E

What are issues in the interpretation


of screening results?

7
2628
Copyright © Harvard Medical School, 2018. All Rights Reserved.

Question 3

A randomized trial was conducted to evaluate the effectiveness of a


new screening program for colon cancer.
Among those patients whose cancers were detected by the screening
program, average age at diagnosis was 54 years and average age at
death was 60 years: thus, average survival from diagnosis to death
was 6 years.
For patients whose cancers were detected by clinical symptoms,
average age at diagnosis was 56 years and average age at death was
60 years: thus, average survival from diagnosis to death was 4 years.
The investigators reported a statistically significant 2 year increase in
survival from colon cancer associated with screening.
This mistaken conclusion is most likely due to:
A. The play of chance
B. Confounding by stage of disease at diagnosis
C. Lead time bias
D. Length bias

Lead Time

Death
screen symptoms

60 years
54 years

60 years
56 years

Incorrect Conclusion: 2 year increase in survival with


screening. Really just advanced diagnosis by 2 years.
Problem: Lead time bias (Correct answer “C”)
Solution: Compare age-specific (ex. at age 60) mortality
rates

8
2629
Copyright © Harvard Medical School, 2018. All Rights Reserved.

MEASURING DATA

Measuring Data

• Measures of disease frequency


• Measures of association
• How do we refer to these measures?
• How do we calculate these measures?

9
2630
Copyright © Harvard Medical School, 2018. All Rights Reserved.

Question 4

In each statement below, data are presented


based on the Framingham Study of coronary
heart disease. Chose the measure which best
describes each of these statements.
A. Prevalence measure
B. Incidence measure
C. Standardized morbidity ratio
D. Age-specific measure
E. Age-adjusted measure

number of existing cases at a point in time


Prevalence =
total population

number of new cases during a period of time


Incidence =
population at risk

Prevalence and incidence can be overall, or can be


category-specific (e.g. age-specific)

observed cases
Standardized morbidity ratio:
expected cases

10
2631
Copyright © Harvard Medical School, 2018. All Rights Reserved.

1. At the initial examination, 17 persons per


1,000 had evidence of coronary heart disease:

1. At the initial examination, 17 persons per 1,000


had evidence of coronary heart disease:

Prevalence measure

11
2632
Copyright © Harvard Medical School, 2018. All Rights Reserved.

3. During the first eight years of the study,


45 persons developed coronary heart disease
per 1,000 persons who entered the study free
of coronary heart disease:

3. During the first eight years of the study,


45 persons developed coronary heart disease
per 1,000 persons who entered the study free
of coronary heart disease:

Incidence measure

12
2633
Copyright © Harvard Medical School, 2018. All Rights Reserved.

4. At the initial examination, 31 persons aged


45 to 62 had coronary heart disease per
1,000 persons examined in this age group:

4. At the initial examination, 31 persons aged


45 to 62 had coronary heart disease per
1,000 persons examined in this age group:

Age-specific prevalence measure

13
2634
Copyright © Harvard Medical School, 2018. All Rights Reserved.

How do we calculate incidence


and prevalence?

Question 5

At the beginning of 2013, 800 people diagnosed


with diabetes lived in small town, which had a
mid-year population estimated at 10,000.
During that year, 200 new cases of diabetes were
diagnosed in the town and 40 people died of
complications of diabetes.

14
2635
Copyright © Harvard Medical School, 2018. All Rights Reserved.

• during 2013, 200 new cases diagnosed


• estimated midyear population = 10,000
• on 1/1/13, 800 diabetics in city
• during 2013, 40 died from diabetes

A. Incidence measure per 1,000 population


during 2013:

• in 2013, 200 new cases diagnosed


• estimated midyear population = 10,000
• on 1/1/13, 800 diabetics in city
• during 2013, 40 died from diabetes

A. Incidence measure per 1,000 during 2013:


200 20
I= 10,000 = 1,000

15
2636
Copyright © Harvard Medical School, 2018. All Rights Reserved.

• in 2013, 200 new cases diagnosed


• estimated midyear population = 10,000
• on 1/1/13, 800 diabetics in city
• during 2013, 40 died from diabetes

B. Prevalence measure per 1,000 on 1/1/13:

• in 2013, 200 new cases diagnosed


• estimated midyear population = 10,000
• on 1/1/13, 800 diabetics in city
• during 2013, 40 died from diabetes

B. Prevalence measure per 1,000 on 1/1/13:


80
P = 800 =
10,000 1,000

16
2637
Copyright © Harvard Medical School, 2018. All Rights Reserved.

• in 2013, 200 new cases diagnosed


• estimated midyear population = 10,000
• on 1/1/13, 800 diabetics in city
• during 2013, 40 died from diabetes

C. Prevalence measure per 1,000 on 12/31/13:

• in 2013, 200 new cases diagnosed


• estimated midyear population = 10,000
• on 1/1/13, 800 diabetics in city
• during 2013, 40 died from diabetes

C. Prevalence measure per 1,000 on 12/31/13:

800+200-40 96
P = = 1,000
10,000

17
2638
Copyright © Harvard Medical School, 2018. All Rights Reserved.

• in 2013, 200 new cases diagnosed


• estimated midyear population = 10,000
• on 1/1/13, 800 diabetics in city
• during 2013, 40 died from diabetes

D. Mortality due to diabetes per 1,000 general


population during 2013:

• in 2013, 200 new cases diagnosed


• estimated midyear population = 10,000
• on 1/1/13, 800 diabetics in city
• during 2013, 40 died from diabetes

D. Mortality (differentiated from case-fatality)


due to diabetes per 1,000 during 2013:
40 4
M = =
10,000 1,000

18
2639
Copyright © Harvard Medical School, 2018. All Rights Reserved.

E. If the prevalence of diabetes in 2013 is


greater than that in 1950, this could be due
to:
A. A change in the incidence rate
B. A change in the duration of the
disease
C. Either (A) or (B) or both

The proportion of the population that has a


disease at a point in time (prevalence)
depends on both the rate of development of
the disease in the population (incidence) as
well as the duration of the disease from onset
to termination (death or cure). Under steady
state, P = I x D.
Thus, a change in prevalence can reflect a
change in incidence, a change in duration, or
both. (Answer “C“: either A or B or both).

19
2640
Copyright © Harvard Medical School, 2018. All Rights Reserved.

How do we calculate measures of


association?

Question 6

The table below gives the annual mortality rates


from lung cancer and coronary heart disease
among cigarette smokers and nonsmokers in a
cohort study of British male physicians.

Annual Mortality Rates per 100,000

Lung Coronary
Cancer Heart Disease
Cigarette smokers 140 669
Nonsmokers 10 413

20
2641
Copyright © Harvard Medical School, 2018. All Rights Reserved.

RELATIVE RISK (RR): Incidence in the


exposed group divided by Incidence in
nonexposed group: RR = Ie / Io
• Measure of the strength of association
• If no association, RR = 1
ATTRIBUTABLE RISK AMONG EXPOSED:
ARe = Ie - Io
• Assuming exposure causes outcome,
measure of public health impact among
the exposed
• If no association, AR = 0

Mortality rate/100,000
Coronary
Lung Cancer Heart Disease
Smokers 140 669
Nonsmokers 10 413

Ie 140/100,000
RR (Lung Cancer) = = = 14
Io 10/100,000

Those who smoke have 14x the risk of dying


from lung cancer as nonsmokers.

21
2642
Copyright © Harvard Medical School, 2018. All Rights Reserved.

Mortality rate/100,000
Coronary
Lung Cancer Heart Disease
Smokers 140 669
Nonsmokers 10 413

Ie 669/100,000
RR (CHD) = = = 1.6
Io 413/100,000

Those who smoke have 1.6x the risk (or 60%


increased risk) of dying from CHD, compared to
nonsmokers.

Mortality rate/100,000
Coronary
Lung Cancer Heart Disease
Smokers 140 669
Nonsmokers 10 413

ARe (Lung Cancer) = Ie- Io = 140/100,000 - 10/100,000


= 130/100,000 smokers
Assuming smoking causes lung cancer, for every
100,000 smokers, 130 cases of lung cancer are due
to their smoking, or could be eliminated if smoking
were eliminated.

22
2643
Copyright © Harvard Medical School, 2018. All Rights Reserved.

Mortality rate/100,000
Coronary
Lung Cancer Heart Disease
Smokers 140 669
Nonsmokers 10 413

ARe (CHD) = Ie-Io = 669/100,000 - 413/100,000


= 256/100,000 smokers
Assuming smoking causes heart disease, for every
100,000 smokers, 256 cases of CHD are due to their
smoking, or could be eliminated if smoking were
eliminated.

Based on these data, is smoking a stronger risk


factor for lung cancer or CHD?

Lung cancer, since the RR for lung cancer is


greater than for heart disease.

23
2644
Copyright © Harvard Medical School, 2018. All Rights Reserved.

But if smoking were eliminated, could more


deaths be eliminated among smokers from lung
cancer or CHD?

Coronary heart disease, since the ARe for heart


disease (256/100,000 smokers) is greater than
for lung cancer (130/100,000 smokers). This
reflects that the baseline risk of CHD mortality
(413/100,000) is much higher than lung cancer
mortality (10/1000,000); thus even a small
decreased risk could affect a much larger
number of people on an absolute scale.

Number Needed to Treat

24
2645
Copyright © Harvard Medical School, 2018. All Rights Reserved.

Nonfatal MI or
Death from CHD

Pravastatin 212 1869 2081

Placebo 274 1804 2078

486 3673 4159

• RR = Ie = 212/2081 = 0.77
Io 274/2078
Those on pravastatin have a 33% lower risk of nonfatal
MI or death from CHD.
• ARe = Ie- Io = 212/2081 - 274/2078 = - 0.03
Note negative sign: 3 per 100 of the nonfatal MIs or
CHD deaths in the placebo group could have been
prevented by use of pravastatin, assuming causality.

• NNT = Number needed to treat = 1


ARe
= 1
0.03
= 33
We would need to treat 33 patients over 5 years
(median duration of follow-up) in order to prevent
one nonfatal MI or CHD death.

25
2646
Copyright © Harvard Medical School, 2018. All Rights Reserved.

INTERPRETATION OF DATA

• Types of studies
• Valid statistical association

Question 7
The association between low birth weight and
maternal smoking during pregnancy was
studied by obtaining smoking histories from
women at the time of their first prenatal visits
and comparing subsequent birth weights
among women with various smoking histories
What type of study is this?
A. Case-control study
B. Cohort study
C. Randomized clinical trial
D. Cross-sectional survey

26
2647
Copyright © Harvard Medical School, 2018. All Rights Reserved.

Case Control

Exp D
Cohort

Exp D
Clinical Trial (investigator allocates)

Exp D

Cross-sectional Survey

Exposure, Outcome

The association between low birth weight and


maternal smoking during pregnancy can be
studied by obtaining smoking status at time of
their first prenatal visit (EXPOSED VS
NONEXPOSED) and then comparing subsequent
birth weight (OUTCOME) with these smoking
histories. What type of study is this?
A. Case-control study
B. Cohort study
C. Randomized clinical trial
D. Cross-sectional survey

Correct answer: B

27
2648
Copyright © Harvard Medical School, 2018. All Rights Reserved.

Question 8
A case-control study was undertaken to evaluate the
relationship between maternal smoking during
pregnancy and low birth weight. A total of 350
mothers of low birth weight babies and 400 mothers
of normal weight babies were interviewed. Of the
mothers of low birth weight babies, 200 reported
smoking during the pregnancy, while 200 of the
mothers of the normal weight babies also reported
such a history.
What is the magnitude of the association between
smoking and birth weight? Is the observed
association valid?

Birth Weight
low normal

yes 200 200


a b
Smoking
no 150 200
c d
350 400

Measure of association in a case-control study is the odds


ratio (OR), which is a valid estimate of the relative risk:
ad
OR = bc

28
2649
Copyright © Harvard Medical School, 2018. All Rights Reserved.

Birth Weight
low normal

yes 200 200


a b
Smoking
no 150 200
c d
350 400

ad 200 (200)
A. OR = = = 1.3
bc 200 (150)
Mothers who smoke during pregnancy have 1.3 times the
risk, or a 30% increased risk, of having low birth weight
baby. Magnitude of association.

Validity of an Association

• To determine if an association observed in a


study is internally valid, we need to rule out
alternative explanations for the findings:
Chance
Bias
Confounding

• Versus external validity – generalizability of


study results

29
2650
Copyright © Harvard Medical School, 2018. All Rights Reserved.

• Chance: role of sampling variability.

• Bias: any source of systematic error in


the determination of the association.

• Confounding: mixture of effect between


the association under study and a third
variable, which may be responsible in
part or totally for the association seen.

1. In comparing the association between the two


groups (OR = 1.3), the p-value is found to be
0.20. The correct interpretation of this result is:
1. The null hypothesis is rejected.
2. The association is statistically significant.
3. The association did not occur by chance.
4. The association is compatible with the null
hypothesis.
5. Sampling variation is an unlikely explanation
of the association.

30
2651
Copyright © Harvard Medical School, 2018. All Rights Reserved.

The p-value is the probability that the observed


data (or data more extreme) would occur due to
the effects of chance alone, given that the null
hypothesis is true (H0, that there is truly no
difference, no association between the two
groups).

• If P <0.05, reject H0, association is statistically


significant at 0.05 level.

• If P >0.05, cannot reject H0, association is not


statistically significant at 0.05 level.

• P-value does not mean due to chance, or


mean chance is ruled out - relates to
likelihood of chance being an explanation of
the findings.

31
2652
Copyright © Harvard Medical School, 2018. All Rights Reserved.

1. In comparing the association between the two


groups, the p-value is found to be 0.20. The
correct interpretation of this result is:
NOTE: P-value GREATER THAN 0.05
1. The null hypothesis is rejected.
2. The associaton is statistically significant.
3. The association did not occur by chance.
4. The association is compatible with the
null hypothesis of no association.
5. Sampling variation is an unlikely
explanation of the association.

2. It is suggested that smoking mothers of


low-birth weight babies would tend to deny
such an activity due to feelings of guilt,
compared to nonsmoking mothers of low
birth weight babies, or smoking mothers of
normal weight babies. This would be an
example of the effects of:
1. Chance
2. Selection bias
3. Recall bias
4. Confounding

32
2653
Copyright © Harvard Medical School, 2018. All Rights Reserved.

2. It is suggested that smoking mothers of


low-birth weight babies would tend to
deny such an activity due to feelings of
guilt. This would be an example of the
effects of:
1. Chance
2. Selection bias
3. Recall bias – those with
outcome tend to recall/report exposures
differently than those without
4. Confounding

3. The effect of the situation described in (B)


(smoking mothers of low-birth weight babies
tend to deny such an activity) would be to
result in an observed relative risk which is
an:
1. Underestimate of the true relative risk.
2. Overestimate of the true relative risk.
3. The same as the true relative risk.

33
2654
Copyright © Harvard Medical School, 2018. All Rights Reserved.

3. The effect of the situation described in (B)


(smoking mothers of low-birth weight babies
tend to deny such an activity) would be to
result in an observed relative risk which is
an:
1. Underestimate of the true relative risk:
smoking won’t look as bad as it actually is.
2. Overestimate of the true relative risk.
3. The same as the true relative risk.

4. It was also observed that mothers of low


birth weight babies tended to be younger
than the mothers of the normal-weight
children. Moreover, smoking rates are
known to be higher in younger women in
this population. This would be an example
of the effects of:
1. Chance
2. Selection bias
3. Recall bias
4. Confounding

34
2655
Copyright © Harvard Medical School, 2018. All Rights Reserved.

SMOKING LOW BIRTH WEIGHT

MATERNAL AGE (3rd factor)

Correct answer: 4 - Confounding


If maternal age (the confounder) is uncontrolled,
will overestimate true association between
smoking and low birth weight.

TAKE HOME MESSAGE

• Limited number of necessary concepts in


Biostatistics for the Boards

• TRUST YOUR INSTINCTS

35
2656
Copyright © Harvard Medical School, 2018. All Rights Reserved.

No Disclosures

References
1. Fletcher RH, Fletcher SW. Clinical Epidemiology, The
Essentials. 5th Edition. Lippincott Williams and
Wilkins. 2012.
2. Glantz SA. Primer of Biostatistics. 7th Edition. McGraw-
Hill. 2011.
3. Hulley SB, Cummings SR, Browner WS, Grady DG,
Newman TB. Designing Clinical Research. 4rd Edition.
Wolters Kluwer. 2013.

36
2657
Copyright © Harvard Medical School, 2018. All Rights Reserved.

Morning Report 2018

Maria A. Yialamas, MD
Associate Program Director, Internal Medicine Residency
Department of Medicine, Brigham and Women’s Hospital
Assistant Professor of Medicine, Harvard Medical School

• I have no disclosures to report.

2658
Copyright © Harvard Medical School, 2018. All Rights Reserved.

Image of the Day


• 25 year old gentleman comes to see you for
better control of his diabetes.

• His past medical history is notable for type 2


diabetes first diagnosed 6 years ago and
hypertension diagnosed 3 years ago.

• Initially, he was placed on metformin but due to


gradually poorer control of his diabetes, he was
started on insulin.

Image of the Day


• Current regimen:
– Metformin 1000 mg bid
– Insulin glargine 20 U qhs

• Very motivated, carefully follows an ADA diet


and exercises on a regular basis.

• Frustrated that his lifestyle changes are not


making a difference in his diabetes control. His
blood sugars are in the 200s again.

2659
Copyright © Harvard Medical School, 2018. All Rights Reserved.

What would you do next?

2660
Copyright © Harvard Medical School, 2018. All Rights Reserved.

Signs and Symptoms of


Acromegaly
• Enlarged jaw • Voice deepening
• Enlarged, swollen • Diabetes
hands and feet • Hypertension
• Coarse facial features • LVH
• Nose enlargement • Organomegaly
• Spreading of teeth • Colonic polyps and
• Joint pains colon cancer
• Skin tags • Sleep apnea
• Macroglossia

2661
Copyright © Harvard Medical School, 2018. All Rights Reserved.

Diagnosis of Acromegaly
• Insulin growth factor -1 (IGF-1)

• If IGF-1 elevated
– OGTT with growth hormone levels
– Suppression of GH to < 1ng/mL in normals

Patient Course
• IGF-1 and Growth Hormone levels were
elevated

• Imaging confirmed a pituitary tumor

• Transphenoidal surgery with cure

2662
Copyright © Harvard Medical School, 2018. All Rights Reserved.

Secondary Causes of
Hyperglycemia
• Acromegaly
• Cushing’s Syndrome
• Pheochromocytoma
• Adrenal Insufficiency
• Hyperthyroidism
• Glucagonoma
• Somatostatin-secreting tumors
• Pancreatic disease
• Medications (i.e. glucocorticoids, atypical
antipsychotics, HIV protease inhibitors)

Case
62 year old housewife and mother of four children came
in for evaluation of hair loss.

“If I go bald, I will kill myself.”

She was in good general health.

She did have a history of hypothyroidism. Positive


thyroid peroxidase antibodies. Euthyroid on replacement.

Tonsillectomy. Appendectomy. Scarlet fever as a child.

2663
Copyright © Harvard Medical School, 2018. All Rights Reserved.

First noticed symptoms 3 weeks ago-fair amount


of hair in her comb.

Two days prior, she saw her hairdresser who


told her “you have huge bald spots” and asked
her if she was ill.

No weight loss, fevers, chills, sweats, rash,


weight change, change in urinary or bowel
habits, bruising, change in skin pigmentation.

Her husband and children were well. Her


mother had died of breast cancer at age
56 and her father from pneumonia and
Alzheimer’s disease at age 82.

2664
Copyright © Harvard Medical School, 2018. All Rights Reserved.

BP 105/65 HR 64 RR 10

Several 2 to 6 cm bald spots with broken off


hairs at the edges.

No hair loss over her body.

Normal female escutcheon.

No thyroid nodules.

No lymphadenopathy.

Breast, lung, cardiovascular, and


abdominal exams were unremarkable.

Stool was brown and guaiac negative.

Gait, speech, and cranial nerves all intact.

2665
Copyright © Harvard Medical School, 2018. All Rights Reserved.

• WBC 3100/µL (62 P, 28 L, 7 M, 3 E)


• Hematocrit 36%
• Platelets 224, 000/µL

• Kidney and liver function tests normal.

• What additional tests would you want to


perform?

• Iron 18 µg/dL (40-159)


• TIBC 380 µg/dL (250-450)
• Transferrin saturation 5%

• B12 111 pg/mL (250-900)


• MMA 0.6 µmol/L (0-0.40)

2666
Copyright © Harvard Medical School, 2018. All Rights Reserved.

Hair Growth Cycle


• Anagen
– Growth phase
– 2-3 years
– 80-90% of follicles

• Catagen
– Involution phase
– 2-3 weeks
– 1-3% of follicles

• Telogen
– Resting phase
– 3-4 months
– Hair shed at end of telogen (75/day)
– Mature root sheath or club at proximal end
– 5-10% of follicles

Alopecia
• Scarring
– Associated fibrosis, inflammation, and loss of
hair follicles
– Smooth scalp with decreased # of follicular
openings

• Non-scarring
– Hair shafts gone but follicles preserved
– Potentially reversible

2667
Copyright © Harvard Medical School, 2018. All Rights Reserved.

Non-scarring and Scarring Hair Loss

Shapiro J. N Engl J Med 2007;357:1620-1630

Scarring Alopecia
• Discoid lupus
• Sarcoidosis
• Lichen Planus

2668
Copyright © Harvard Medical School, 2018. All Rights Reserved.

Non-scarring Alopecia
• Androgenetic telogen effluvium
• Alopecia areata
• Tinea capitis
• Traumatic alopecia
• SLE
• Secondary syphilis
• Hypothyroidism, hyperthyroidism
• Hypopituitarism
• HIV
• Deficiencies of protein, iron, biotin, zinc
• Medications

Androgenetic Alopecia
• Most common type of hair loss: 30-40% of adult men
and women

• Genetically determined shortening of anagen phase


(shorter, thinner hair shafts)

• Increased sensitivity of the hair follicles to androgens or


increased levels of androgens

• Anterior/midline scalp

• Polygenic inheritance/variable penetrance

2669
Copyright © Harvard Medical School, 2018. All Rights Reserved.

Androgenetic Alopecia

Shapiro J. N Engl J Med 2007;357:1620-1630

Androgenetic Alopecia
• Women less than men
– Fewer androgen receptors in hair follicles
– Less 5 α-reductase activity ( T DHT)
– Increased aromatase ( T E2)

• Rule out androgen excess in women


– Acne
– Hirsutism
– Male escutcheon
– Clitoromegaly

2670
Copyright © Harvard Medical School, 2018. All Rights Reserved.

Telogen Effluvian
• Most common cause of diffuse hair loss

• Diffuse shedding of normal hairs (clubbed hairs)

• Often follows major illness or postpartum

• Pull test
– Grasp around 30 hairs. If more than 5 club hairs come out –
abnormal

• Thyroid disease, iron deficiency, lupus

• Usually regrows in 3-4 months

Alopecia: Pull Test

Shapiro J. N Engl J Med 2007;357:1620-1630

2671
Copyright © Harvard Medical School, 2018. All Rights Reserved.

Alopecia Areata
• Common (1/1,000)

• T cell infiltrates around hair follicles

• Associated with other autoimmune diseases (vitiligo,


thyroiditis, and pernicious anemia)

• Smooth, circular discrete area of complete hair loss

• Black dots of hair broken a few mm from scalp at edge of


lesion

Alopecia Areata
• May have hair loss on body and/or pitted
nails

• Plucked hairs look like exclamation points


(narrow, rather than clubbed at the base)

• Develops over a few weeks, regrows in a


few months (90%)

2672
Copyright © Harvard Medical School, 2018. All Rights Reserved.

Clubbed Hairs

Springer et al, AFP, 2003

Alopecia Evaluation
• TSH
• Iron/TIBC
• Rule out trauma
• Meds: warfarin, heparin, PTU, vitamin A,
lithium, beta blockers, clonidine,
amphetamines
• ANA, VDRL
• Vitamin B12

2673
Copyright © Harvard Medical School, 2018. All Rights Reserved.

EKG of the day

Brugada Syndrome
•Elevated J point, ST segment in V1-2
•Brugada Syndrome vs. Brugada Pattern
•Inverted T Type 1; upright T in Types 2-3.
•Polymorphous VT/sudden death
•Often familial
•Channelopathy – usually Na+
•Occasionally can be provoked by Type 1 anti-
arrhythmics, stress
•See similar ST-T abnormalities in lateral leads in
hypothermia, hypercalcemia.

2674
Copyright © Harvard Medical School, 2018. All Rights Reserved.

Case
59 year old man with an extensive cardiac
history including CAD and VT, who presented
with epistaxis.

He had been quite ill over the last 4 weeks.

4 weeks previously
• The patient was hospitalized for AICD
interrogation due to multiple discharges.

• During his 10-day hospital stay both a


fractured lead and malfunctioning
epicardial patch were replaced via
thoracotomy.

2675
Copyright © Harvard Medical School, 2018. All Rights Reserved.

4 weeks previously
• Post-operative fevers were treated with 5
days of vancomycin, cefazolin and
levofloxacin. All cultures were negative,
and antibiotics were discontinued.

• The patient was still having low grade


temperatures when he was discharged.

Presentation
• In the post-discharge period, the patient
experienced malaise, fatigue, anorexia with an 8
lb weight loss, and diarrhea.

• He reported no tactile fevers.

• One day prior to admission, he developed a


right-sided headache, without visual changes,
nuchal rigidity, altered speech or local
weakness.

2676
Copyright © Harvard Medical School, 2018. All Rights Reserved.

Day of admission
• On the day of admission, the patient developed
new epistaxis that did not resolve with local
compression. He also noticed blood in the
sclera of his right eye.

• On route to the ED, family members noted left


leg and right arm shaking associated with mild
confusion but no loss of consciousness.

Additional ROS
• No bladder or bowel incontinence.
• No chest pain, orthopnea or palpitations.
• No head trauma.

2677
Copyright © Harvard Medical School, 2018. All Rights Reserved.

Past Medical History


• DM2
• CAD (s/p MI and CABG • CVA in 1997
in 1996) • Hypertension
• ICM (EF=40%) • Hypercholesterolemia
• VT (s/p AICD placement • Obesity
in 1999, revised 4 weeks
previously)
• Diverticulitis

Medications
• Lisinopril 40 mg po qd
• Atenolol 50 mg po qd
• Amiodarone 200 mg po qd
• Warfarin 5mg po qd
• Aspirin 325 po qd
• Ranitidine 150 mg po bid

2678
Copyright © Harvard Medical School, 2018. All Rights Reserved.

Allergies: NKDA

Soc Hx: Lived in the North End with a stable


male partner. No known TB risk factors. No
tobacco, intravenous drugs, or significant
alcohol use.

Fam Hx: Father- lung cancer, Mother - stroke

Physical Exam
T-101.4oF P-80 BP-146/60 RR-20
O2 sats - 96% on 2L oxygen by NC

General: Awake, diaphoretic, soaked through his


clothes, confused but followed simple
commands.

Skin: Multiple ecchymoses. Mild incisional


erythema, but no evidence of external chest
wound breakdown or drainage. Poor skin
turgor.

2679
Copyright © Harvard Medical School, 2018. All Rights Reserved.

Physical Exam (cont’d)


• HEENT: Right conjunctival hemorrhage.
EOMI.

• Neck: supple. JVP flat.

• Lungs: Bibasilar crackles.

• CV: RRR. 2/6 SEM. No peripheral edema.

• Neuro: Tremulous. Reflexes 3-4+.

Labs

10.5 139 107 55


9.5 243 132
31.2 4.4 27 1.1

MCV- 87

PTT- 55.6 Ca-9.5 Mg-1.6


INR – 12.7 PO4-4.4 TP-5.9
Alb-4.7 CKMB (-)

2680
Copyright © Harvard Medical School, 2018. All Rights Reserved.

Studies
CXR - Cardiomegaly, no infiltrates, AICD in place

ECG - Old anterior-septal MI. No change from


previous study

Head CT - Subdural hematomas affecting bilateral


frontal areas; right internal capsule CVA (likely
old)

Hospital Course
• FFP and vitamin K therapy were initiated.

• Persistent fevers developed with Temps 101-102oF.


Blood, sputum and stool samples were obtained for
microbiologic study. Pan CT scans showed no source on
infection.

• ID and Neurology consultations were obtained.

• Phenytoin was initiated for question of seizure-like


activity.

2681
Copyright © Harvard Medical School, 2018. All Rights Reserved.

Hospital Course (cont’d)


• The patient remained febrile.

• Blood cultures, lumbar puncture,


echocardiography and serologies for HIV and
syphilis failed to explain the fevers.

• By hospital day 5, severe hypernatremia


developed (Na 163) with worsening mental
status.

• Temp was 104.5. RR 40

What would you do next?

2682
Copyright © Harvard Medical School, 2018. All Rights Reserved.

• >38.00C for 3 days

Roth, AFP, 2003

Roth, AFP, 2003

2683
Copyright © Harvard Medical School, 2018. All Rights Reserved.

Rare Miscellanous Causes


• Atrial myxoma
• Familial Mediterranean Fever
• Histiocytosis
• Pheochromocytoma
• Whipple’s disease
• Thyroiditis and Thyrotoxicosis

Hospital Course
• TFTs ordered:
• TSH <0.01 (0.5 -5 uU/mL)
• T4 18.5 (4.5-10.9 ug/dL)
• THBI 2.51 (0.77-1.23)
• FT4I 46.4 (4.5-10.9)
• T3 343 (60-181 ng/dL)

• Concern was for amiodarone induced


hyperthyroidism

2684
Copyright © Harvard Medical School, 2018. All Rights Reserved.

Treatment
• Methimazole 30 mg per rectum q6 hrs was
initiated, but later changed to
propythiouracil (PTU) 400 mg pngt q 6 hrs.

• Methylprednisolone 60 mg iv q12 hrs

• Iopanoic acid 1 gm qd

Treatment
• Pulmonary aspiration was felt to have
occurred in the setting of delirium and
agitation.

• Despite anti-thyroid therapy, the patient


developed hyperpyrexia (T=107oF) and
increased clonus.

• A diagnosis of Thyroid Storm was made,


and the patient underwent
thyroidectomy.

2685
Copyright © Harvard Medical School, 2018. All Rights Reserved.

Follow-up
• The patient’s temp dropped quickly after
the thyroidectomy.

• The patient was discharged after a 3 week


hospital course.

• At follow-up, he had returned to his


previous level of function.

Thyroxine and Amiodarone

2686
Copyright © Harvard Medical School, 2018. All Rights Reserved.

Amiodarone
• 75 mg of iodine per 200 mg tablet

• 10% of iodine released as free iodide daily

• Average daily iodine intake in US is 0.15-0.30


mg.

• ½ life is 100 days

(Daniels, JCEM, 2001; Basaria and Cooper Am J Med, 2005)

Amiodarone and Thyroid Function

(Basaria and Cooper, Am J Med, 2005)

2687
Copyright © Harvard Medical School, 2018. All Rights Reserved.

Chronic Amiodarone Treatment

(Melmed et al. JCEM, 1981)

Probability of Thyroid Disease with


Amiodarone Treatment

(Trip MD et al., Am J Med, 1991)

2688
Copyright © Harvard Medical School, 2018. All Rights Reserved.

Amiodarone and Hyperthyroidism

(Daniels, JCEM, 2001)

Severe Hyperthyroidism -
Treatment
• Thyroid gland
– Inhibit synthesis – PTU, Methimazole
– Inhibit TH release – Iodine, Lithium
– Surgery

• Peripheral Effects of TH
– Inhibit T4 to T3 conversion – PTU, steroids, iopanoic acid,
propranolol
– Removal of excess thyroid hormone – plasmapheresis

• Systemic Decompensation
– Treat hyperthermia – Tylenol, cooling
– Correct dehydration and nutritional deficits – fluids, electrolytes
– Supportive therapy – pressors, treat CHF, steroids

2689
Copyright © Harvard Medical School, 2018. All Rights Reserved.

Anticoagulation and Hyperthyroidism


• Hyperthyroidism increases metabolism of
vitamin K-dependent clotting factors

• Altered protein binding of warfarin

• Enhanced affinity of warfarin for its


receptors

(Chute et al., Endo Practice, 1997)

Lab of the Day


• 36 year old woman with diabetes, liver disease,
and HIV comes to see you in follow-up.

• She says that her blood glucose has been very


elevated – consistently in the 250s.

• She believes this is due to poor diet.

• You counsel her regarding diet and exercise,


refer her to a nutrionist, and order a hemoglobin
A1c.

2690
Copyright © Harvard Medical School, 2018. All Rights Reserved.

Lab of the Day (cont’d)


• Her HgB A1c returns at 6.8%.

• You are surprised at this hemoglobin A1c.


What value did you expect?

Hemoglobin A1c (%) Average blood glucose


(mg/dL)
6 135

7 170

8 205

9 240

10 275

(Hemoglobin A1c x 33) - 60

2691
Copyright © Harvard Medical School, 2018. All Rights Reserved.

Lab of the Day (cont’d)


• Why is there a discrepancy?
– Glucometer
– Timing of testing
– Assay

– Treatment of iron, folate, or vitamin B12


deficiency
– Hemolysis

Hemoglobin A1c
• Red blood cells are freely permeable to glucose

• Glucose irreversibly attaches to the hemoglobin

• The hemoglobin A1c represents 3 months of


blood sugar readings because of the lifespan of
red blood cells

• Increases in red blood cell turnover falsely low


hemoglobin A1c

2692
Copyright © Harvard Medical School, 2018. All Rights Reserved.

Our patient
• Hct 36%
• Iron studies, folate, and B12 all normal and not on recent
treatment for deficiencies

• LDH 285 (107-235 U/L)


• Haptoglobin <8 (30-200 mg/dL)

• She has hemolysis – probably from her liver disease.

• How could you check her blood sugar control?

Fructosamine
• Measure of glycosolated end products

• Her level was 538 (0-285 umol/L).

2693
Copyright © Harvard Medical School, 2018. All Rights Reserved.

Case
44 year old private investor presented to
his PCP with low libido of one year’s
duration.

He has been married for seven years. Had


his first child one year ago.

The decline in his libido was gradual.

He is healthy and active, loves outdoor activities and


building ships.

Increasing fatigue over the past year has limited his time
outdoors.

Does not feel depressed. Has no other signs or


symptoms of a systemic illness including no fevers,
chills, headaches, weight changes, changes in his hair or
skin texture, bowel movements.

His only medical problem is mild hand arthritis.

2694
Copyright © Harvard Medical School, 2018. All Rights Reserved.

What more would you want to know?

Low libido
• Psychiatric: depression, anxiety disorder
• Medications: SSRIs, anticonvulsants, antihypertensive
medications
• Systemic illness
• Recreational drugs
• Androgen deficiency
• Thyroid disease
• Hyperprolactinemia
• Erectile dysfunction

2695
Copyright © Harvard Medical School, 2018. All Rights Reserved.

Normal pubertal development.

In addition to the low libido, he has noticed


decreased morning erections and decreased
ejaculatory volume.

No change in frequency of shaving, no breast


enlargement, or change in testicular size.

No headaches, vision changes.

No medications.

No tobacco, rare ETOH, no illicit drug use.

Family history unremarkable for thyroid


disease, delayed puberty, infertility.

2696
Copyright © Harvard Medical School, 2018. All Rights Reserved.

• Ht 6’4”, Wt 162 lb, BP 128/82, P 84, RR 10

• Gen: non-eunochoidal
• HEENT: PERRL, EOMI, Visual fields full
• Thyroid: normal in size and without nodules.
• No gynecomastia
• Lung, cardiovascular, and abdominal exams normal.
• GU: testes 10 cc bilaterally, nl phallus
• Hair: No change in male distribution of body hair
• Neuro: normal muscle bulk and strength.

Labs
• Testosterone 36 ng/dL (270-1190)

• TSH and prolactin normal

What would you do next?

2697
Copyright © Harvard Medical School, 2018. All Rights Reserved.

Male Hypothalamic-Pituitary-Gonadal Axis

Hypothalamus

GnRH
Pituitary

LH
FSH Testosterone
Estradiol
Inhibin B
Testes

Male Hypothalamic-Pituitary-Gonadal Axis

Normal Secondary Primary

LH
FSH

2698
Copyright © Harvard Medical School, 2018. All Rights Reserved.

Labs
• Testosterone 40 ng/dL
• LH 1.5 U/L (2.4-5.9)
• FSH 1.3 (0.9-15.0)

What would you do next?

Further Evaluation
• MRI of the pituitary gland revealed no
masses.

• Bone density scan: spine T score -2.5

• Semen analysis – azoospermia

• CBC Hct 36.1%

2699
Copyright © Harvard Medical School, 2018. All Rights Reserved.

Secondary Hypogonadism:
Acquired

• Benign tumors & cysts • Critical Illness


• Malignant tumors • Chronic systemic illness
• Infiltrative diseases • Glucocorticoids
• Infections • Chronic opiate use
• Pituitary apoplexy • Adult-onset
• Trauma Hypogonadotropic
• Radiation Hypogonadism
• Hyperprolactinemia

Infiltrative Diseases
• Hemochromatosis
• Sarcoidosis
• Eosinophilic granulomatosis

• Fe = 219 ug/dL (40-159 ug/dL)


• TIBC = 257 ug/dL (250-450 ug/dL)
• Transferrin saturation = Fe/TIBC = 85.2%
• Ferritin = 2435 ug/L (20-400 ug/L)

• HFE testing: C282Y homozygote

2700
Copyright © Harvard Medical School, 2018. All Rights Reserved.

Hemochromatosis and
hypogonadism

• Secondary
– Deposition in the pituitary gland

• Primary – much less common


– Deposition in the testes

Hemochromatosis and recovery


of function
• Little, if any, data

• Most say that if >40 years old, gonadal function


does not recover

(Cundy et al., Clin Endo, 1993)

2701
Copyright © Harvard Medical School, 2018. All Rights Reserved.

Summary
• Secondary causes of diabetes
• Acromegaly
• Alopecia evaluation
• Brugada Syndrome
• Fever of unknown origin
• Amiodarone and the thyroid
• Extreme hyperthyroidism/thyroid storm
• Potential inaccuracies in hemoglobin A1c
• Hypogonadism
• Hemochromatosis

References
• Bhasin S et al. Testosterone therapy in men with androgen deficiency syndromes: an
Endocrine Society clinical practice guideline. J Clin Endocrinol Metab 2010; 95: 2536.

• Brugada J et al. Right bindle-branch block and St-segment elevation in leades V1


through V3: a marker for sudden death in patients withour demonstrable stuctural
heart dissease. Circulation 1998; 97: 457.

• Daniels GH. Amiodarone-induced thyrotoxicosis. J Clin Endocrinol Metab 2001; 86: 3.

• Klubo-Gwiezdzinska J, Wartofsky L. Thyroid emergencies. Med Clin Norh Am 2012;


96: 385.

• Shapiro J. Clinical practice: har loss in women. NEJM 2007; 357: 1620.

2702
Copyright © Harvard Medical School, 2018. All Rights Reserved.

• I have no disclosures to report.

2703
Copyright © Harvard Medical School, 2018. All Rights Reserved.

General Internal Medicine


Cases and Questions
Lori Wiviott Tishler, MD
VP Medical Affairs
Commonwealth Care Alliance
Division of General Medicine
Brigham and Women’s Hospital
Assistant Professor of Medicine, Harvard Medical School

• Nothing to declare
• No conflicts of interest

2704
Copyright © Harvard Medical School, 2018. All Rights Reserved.

Case One

High Risk Breast Cancer


Screening

A healthy 38 year old woman


presents to your office for an
annual. Her 46 year old sister
was recently diagnosed with
breast cancer. She wonders,
“Should I have an MRI?”

2705
Copyright © Harvard Medical School, 2018. All Rights Reserved.

•Past Medical History: benign breast biopsy at


28, 2 normal pregnancies at 32 and 34.
Menarche at age 11. Regular periods.
•Medications: IUD
•Family history: sister as above, mother with
post menopausal breast cancer at 65,
maternal aunt with ovarian cancer. CAD on
the paternal side, and thyroid disease in a
3rd sibling.

•Social Hx: She’s married with 2 kids,


works as a teacher, active in her
synagogue. No dv, alcohol, or street
drugs. No tobacco.

•Her physical exam is completely


normal.

2706
Copyright © Harvard Medical School, 2018. All Rights Reserved.

What is her lifetime risk of


breast cancer?
A. 8%
B. 15%
C. 20%
D. 35%
E. 40%

What is her lifetime risk of


breast cancer?
A. 8%
B. 15%
C. 20%
D. 35%
E. 40%

2707
Copyright © Harvard Medical School, 2018. All Rights Reserved.

Risk Assessment
While in the office, you go to
www.cancer.gov/bcrisktool
and answer the following
questions

NCI Risk Model


Gail Model
• Race/Ethnicity White
• History of breast cancer,DCIS,LCIS No
• Age 38
• Age at Menarche 7-11
• Age at first live birth >30
• First Degree relatives >1
• Biopsies? Yes
• How many 1
• Any with atypical hyperplasia No

2708
Copyright © Harvard Medical School, 2018. All Rights Reserved.

Caveats
Underestimates risk for people who are
gene positive.

Developed for populations, not individuals

Does not factor in other risks,including


breast density, chest wall radiation.

What is the recommended


screening for our patient?

Annual clinical breast exam and:


A. Annual mammogram starting at 35?
B. Mammograms every 6 months starting at 35?
C. Mammogram one year; MRI the next?
D. Both MRI and mammogram every 6 months
E. Mammogram and MRI every year.

2709
Copyright © Harvard Medical School, 2018. All Rights Reserved.

What is the recommended screening for


our patient?

A. Annual mammogram starting at 35?


B. Mammograms every 6 months
starting at 35?
C. Mammogram one year; MRI the next?
D. Both MRI and mammogram every 6
months
E. Annual mammogram and annual MRI,
6 months apart

Key Points
These recommendations are mammogram and MRI for women with a lifetime
risk of breast cancer of 20% or more.

Consider referring for genetic counseling for people at high risk.

USPSTF recommendations would not apply to her because of her high risk.

Keep your eyes on data around breast density and new risk calculators, e.g
https://tools.bcsc-scc.org/BC5yearRisk/calculator.htm (not lifetime risk)

2710
Copyright © Harvard Medical School, 2018. All Rights Reserved.

My Mother-in-Law

On her way out of the office, she stops to


ask you about whether her mother-in-
law, now 86 with progressive dementia
and Parkinson’s Disease should still have
mammograms. Her mobility and
comprehension are severely limited and
she’s been living in a skilled nursing
facility.

Screen Mother-in-Law?

1.It’s up to the family and her mom


2.It’s up to the family and her mom,
but I recommend it!
3.It’s up to the family and her mom,
but I really suggest that it’s not
necessary

2711
Copyright © Harvard Medical School, 2018. All Rights Reserved.

Screen Mother-in-Law?

1.It’s up to the family and her mom


2.It’s up to the family and her mom,
but I recommend it!
3.It’s up to the family and her mom,
but I really suggest that it’s not
necessary

When to Stop Screening?

There is no good evidence for screening after the age of 74.

What is the woman’s overall health status?

What is her preference?

Does she have a 5-10 year life expectancy?

How would she act upon a cancer diagnosis?

2712
Copyright © Harvard Medical School, 2018. All Rights Reserved.

Case Two
E-Cigarettes

“I’m smoking…Again”
But this time, I’m trying e-cigs. I
think they are better for me. What
do you think?

By the way, what’s a Juul? My


daughter mentioned that kids are
using them in high school.

2713
Copyright © Harvard Medical School, 2018. All Rights Reserved.

What do you think?

2714
Copyright © Harvard Medical School, 2018. All Rights Reserved.

Since you don’t think the “uh, I don’t know


would be a good answer,” you turn to your
desktop resource of choice and learn that
there is little good literature on e-cigs, but
that they function by delivering a nicotine-
containing vapor that contains fewer toxins
that a regular cigarette, that they are not FDA
approved, and that no one really knows if
they help people to quit smoking.

Are E-Cigarettes or Varenicline better at


24 weeks

A. E Cigarettes
B. Varenicline

2715
Copyright © Harvard Medical School, 2018. All Rights Reserved.

Are E-Cigarettes or Varenicline better


for sustained quitting in 24 weeks

A. E-cigs
B. Varenicline

E-Cigarettes

About 20.3% of current smokers who had tried to quit in


the last year were current e-cigarette users.

One study with e-cigarette users (only 40 patients) did


decrease smoking after 24 weeks

Varenicline for 24 weeks significantly increased quit rate


(37.8% vs. 12.5 for placebo)

JAMA Int Med, May 2014

2716
Copyright © Harvard Medical School, 2018. All Rights Reserved.

In addition to trying to figure out the


best method to quit, the patient is
very concerned about weight gain
after quitting smoking. She wonders
what the average weight gain is:

How much weight can I


expect to gain?
A. The average weight gain is
11 lbs over 5 months
B.16 lbs over 5 months
C.30 lbs in a year
D.20 lbs in a year

2717
Copyright © Harvard Medical School, 2018. All Rights Reserved.

How much weight can I


expect to gain?
A. The average weight gain
is 11 lbs over 5 months
B.16 lbs over 5 months
C.30 lbs in a year
D.20 lbs in a year

Weight Gain After Quitting


•It’s a reality.
•One of the most common reasons
for relapse
•Benefits usually outweigh the
downside
•Average is about 11 lbs

2718
Copyright © Harvard Medical School, 2018. All Rights Reserved.

Cases 3 and 4
Red Eyes

Case 3
50 year old man comes in with a huge
red area in his eye. He is in good health
and recently carried many boxes to the
attic. No pain in the eye. Vision is
20/20. He’s worried that this occurred
because he’s missed his
antihypertensive meds over the last few
days.

2719
Copyright © Harvard Medical School, 2018. All Rights Reserved.

What is Your Diagnosis?


A. Severe Conjunctivitis
B. Subconjunctival Hemorrhage
C. Herpes Keratitis
D. Hypertensive Urgency
E. Acute Angle Closure Glaucoma

2720
Copyright © Harvard Medical School, 2018. All Rights Reserved.

What is Your Diagnosis?


A. Severe Conjunctivitis
B. Subconjunctival Hemorrhage
C. Herpes Keratitis
D. Hypertensive Urgency
E. Acute Angle Closure Glaucoma

Case 4
50 year old woman with RA
comes in to see you. Her
disease has been active lately
and she is not feeling well.
Since yesterday, she has had
photophobia and two painful,
red eyes.

2721
Copyright © Harvard Medical School, 2018. All Rights Reserved.

On Exam
She’s clearly in pain, worse
with pressure to the eye

Photophobia on exam

ESR is 100

Phenylephrine drops don’t


clear it.

What is Your Diagnosis?


A. Herpes Keratitis
B. Acute Angle Closure Glaucoma
C. Scleritis
D. Episcleritis
E. Rheumatoid Eye

2722
Copyright © Harvard Medical School, 2018. All Rights Reserved.

What is Your Diagnosis?


A. Herpes Keratitis
B. Acute Angle Closure Glaucoma
C. Scleritis
D. Episcleritis
E. Rheumatoid Eye

Episcleritis vs. Scleritis


• Acute in onset, may be • Striking, highly
local or diffuse symptomatic course
• Associated with
• Resolves without rheumatologic disease in
treatment 50% of patients
• Vision is not affected • Painful, photophobia
• Phenylephrine drops • Threatens vision
leads to transient • Opthalmologic and
resolution. Sclera systemic treatment
normal underneath warranted urgently

2723
Copyright © Harvard Medical School, 2018. All Rights Reserved.

Case 5
Geriatrics: Falls in the Elderly

Case 5
92 year old community
dwelling elderly woman
brought by her 73 year old
daughter because she fell –
Again.

2724
Copyright © Harvard Medical School, 2018. All Rights Reserved.

Quickly ascertaining that her only injuries


are wounded pride and a sore bottom,
you review her history. The fall occurred
at home when she got up from the chair
and headed for the bathroom. She lives
alone, doesn’t drive, ignores her walker.
She takes multiple meds including
atenolol, hctz, lisinopril, insulin, and
metformin.

At night she takes amitryptiline


for her diabetic neuropathy.
She also drinks, though she’s
never admitted this to you
and you’ve learned this from
her daughter.

2725
Copyright © Harvard Medical School, 2018. All Rights Reserved.

On a Limited Physical Exam,


Which is Most Helpful?
A. Full Neurological Exam
B. Functional Exam
C. Vital Signs
D. Cardiac Exam
E. Joint Exam

On a Limited Physical Exam,


Which is Most Helpful?
A. Full Neurological Exam
B. Functional Exam
C. Vital Signs
D. Cardiac Exam
E. Joint Exam

2726
Copyright © Harvard Medical School, 2018. All Rights Reserved.

Functional Evaluation: The


Get Up and Go Test
Record the time that it takes for a
patient at risk to get up from a chair,
walk 10 feet, and return to the chair.
If this takes more than 30 seconds,
they have impaired mobility and are
at greater risk for a fall.

Results
You do the “up and go” test and she
scores 31 seconds, a marker of impaired
mobility. You send the visiting nurses in
to do a home safety assessment, stress
the importance of using her walker at all
times, stop the elavil and try something
else for her neuropathy. You address
the alcohol again as well.

2727
Copyright © Harvard Medical School, 2018. All Rights Reserved.

What is the Likelihood of


Recurrent Falls?
Of the 1/3 of elderly people who fall
every year,
A. 20%
B. 30%
C. 40%
D. 50%
E. 60%
have multiple falls.

What is the Likelihood of Recurrent


Falls?

Of the 1/3 of elderly people who fall every


year,
A. 20%
B. 30%
C. 40%
D. 50%
E. 60%
have multiple falls.

2728
Copyright © Harvard Medical School, 2018. All Rights Reserved.

• Fell in her bathtub and


sustained subdural. Age 88.

• Died from traumatic brain injury


after a fall at his Manhattan
home. Age 85

2729
Copyright © Harvard Medical School, 2018. All Rights Reserved.

Falls in the Elderly


Are responsible for 70% of accidental deaths in people 75
and over.

Increases with age and transcends ethnic groups

Cause significant morbidity, including decline of functional


status, risk for hospitalization

Cause serious injuries in 5-15% of falls. Hip fracture occurs


in 1-2% of falls.

Risk Factors
Intrinsic Extrinsic
• Muscle weakness
•Poor lighting
• Gait and balance
dysfunction •Clutter
• Visual impairment •Environmental
• Cognitive impairment obstacles
• Orthostatic hypotension •Bad shoes
• Meds

2730
Copyright © Harvard Medical School, 2018. All Rights Reserved.

Drinking in Elderly Patients in the ED

Lifetime alcohol abuse was 24%

14% had a drinking problem in the last year.

Elderly patients with GI complaints had a much higher rate


of alcohol issues (22%) than those who fell.

Physicians detected only 21% of the cases of current abuse


of alcohol.

Case 6
Chronic Pain

2731
Copyright © Harvard Medical School, 2018. All Rights Reserved.

A 69 Year Old Man with Back Pain

JM is a 69 year old woman with back


pain. She has had many studies
which show minimal reversible
problems. She has tried multiple
treatment modalities and adjunctive
therapy. She gets some relief, but
still suffers.

PMH: HTN and gout


Social: No tob, 3 drinks/week,
married, well-supported by
family. No history of substance
abuse.

2732
Copyright © Harvard Medical School, 2018. All Rights Reserved.

Trial of Narcotics?
You’ve been burned in the past
by people seeking narcotics.
What questions will be most
helpful in determining whether
she has a high risk of developing
problematic behavior around
these meds?

Opioid Risk
A. Family history of Substance Abuse
B. Personal History of Substance
Abuse
C. Preadolescent Sexual Abuse
D. A&B
E. All of the above.

2733
Copyright © Harvard Medical School, 2018. All Rights Reserved.

Opioid Risk
A. Family history of Substance Abuse
B. Personal History of Substance
Abuse
C. Preadolescent Sexual Abuse
D. A&B
E. All of the above

Opioid Risk
Family history of substance abuse
• Greatest for prescription drugs

Personal history of substance abuse


• Greatest for prescription drugs

Age between 16 and 45

Preadolescent sexual abuse (women)

Certain Psychiatric Diseases

2734
Copyright © Harvard Medical School, 2018. All Rights Reserved.

Opioid Addiction
Low incidence of iatrogenic addiction in
low risk groups

About 10% of the general population is at


risk for addictive disorders

Ask about risk factors!

Addiction Problem?
Our patient has been on a stable dose of
long and short-acting narcotics. You’ve
followed your state’s PMP and she is
obtaining meds appropriately. One day, she
comes in, obviously in pain, telling you she
strained his back helping her grandson learn
to swim, used up all her breakthrough
meds, borrowed some from her spouse,
and needs more (of a specific medicine).

2735
Copyright © Harvard Medical School, 2018. All Rights Reserved.

What should you do?


A. Her behavior is worrisome. Consider
weaning all narcotics.
B. She’s in violation of his contract. No
more meds.
C. Refill the meds, just this one time.
Make it clear to herthat he’s violating his
contract.
D. Refill the meds, she’s clearly in pain and
has been up front with you.

What should you do?


A. Her behavior is worrisome. Consider
weaning all narcotics.
B. She’s in violation of his contract. No
more meds.
C. Refill the meds, just this one time.
Make it clear to her that he’s violating
his contract.
D. Refill the meds, she’s clearly in pain
and has been up front with you.

2736
Copyright © Harvard Medical School, 2018. All Rights Reserved.

Pseudoaddiction
• Evidence of Physical Distress
• Change in frequency of drug use or unsanctioned
dose escalation
• Drug hoarding
• Requesting specific drugs
• Anxiety
• Openly acquiring drugs from others
• Asking for more meds or reluctance to change
regime
• Behavior stops with dose/med change

Friendly Reminder: Naloxone saves


lives

2737
Copyright © Harvard Medical School, 2018. All Rights Reserved.

Cases 7 & 8
Workplace Related Medicine

The patient following JM

Fell in her work at a local


discount store about 7-8 weeks
ago. She has been complaining
of pain in her lower back ever
since then.

2738
Copyright © Harvard Medical School, 2018. All Rights Reserved.

Treatment with NSAIDS and


muscle relaxants did not help.
Imaging was negative and PT,
likewise, did not make a
difference. Her symptoms are
out of proportion to her injury
and to your exam.

“I don’t think I can go back to


work yet, doctor,” she says
tearfully. “I’m just not ready.”

2739
Copyright © Harvard Medical School, 2018. All Rights Reserved.

What do you tell her now?


A. Tell her to stop malingering and get back to work.
B. Screen for and treat depression
C. Work with her and her company to come up with a
transition back to work plan.
D. Inquire about pending litigation
E. B, C, and D

What Do You Tell Her Now?

A. Tell her to stop malingering and get back to


work.
B. Screen for and treat depression
C. Work with her and her company to come up
with a transition back to work plan.
D. Inquire about pending litigation
E. B, C, and D

2740
Copyright © Harvard Medical School, 2018. All Rights Reserved.

Returning to Work after an Injury

Vast majority of people get better, go


back to work.

Patient factors include perception of


pain, injury, depression, lawsuits
pending, other secondary gain

Returning to Work after an Injury

Work factors include inability to provide transitional


employment.

MD factors include our own discomfort with when patients


are stable to return to work, desire to help a bad situation

Only 50% of workers ever return to work if they have been


out >6 months.

2741
Copyright © Harvard Medical School, 2018. All Rights Reserved.

Lead Astray?
A patient who has lead exposure in
his work as an instructor in the
police academy firing range comes
in to follow up after a
hospitalization department for a left
sided facial droop and a right-sided
hemiparesis.

While you discuss his recovery


from what appears to be a
CVA, his wife tells you that
she is sure that lead poisoning
caused his MCA stroke.

2742
Copyright © Harvard Medical School, 2018. All Rights Reserved.

Which is the Correct Response?

• A. She’s probably right. Check a lead level


immediately.
• B. She’s probably wrong. Lead exposure only
causes peripheral neuropathies.
• C. She’s probably wrong. It is uncommon for
occupational exposures to cause focal
neurological problems such as strokes.
• D. She’s right. Lead can cause encephalopathy,
why not strokes?

Which is the Correct Response?

• A. She’s probably right. Check a lead level


immediately.
• B. She’s probably wrong. Lead exposure only
causes peripheral neuropathies.
• C. She’s probably wrong. It is uncommon for
occupational exposures to cause focal
neurological problems such as strokes.
• D. She’s right. Lead can cause encephalopathy,
why not strokes?

2743
Copyright © Harvard Medical School, 2018. All Rights Reserved.

And, yet, it’s worth noting

Prevalence of CHD and stroke in


adults younger than 55 is highest
in those with service and blue
collar jobs, particularly
accommodation and food service.
(Nat’l Health Interview Survey).

Risks may include stress, shift


work, exposures to second hand
smoke and other particulates.

Case 9

After Gastric Bypass


Surgery

2744
Copyright © Harvard Medical School, 2018. All Rights Reserved.

A 45 year old female patient


Presents to your clinic to
establish care. She is new to
you. Past medical history is
notable for diabetes, which she
proudly tells you is “in
remission” after gastric bypass
surgery 5 years ago.

•She followed up with her


surgeon for a year or two, but
since then has not really been
seen by a physician. She has
maintained her weight loss.

2745
Copyright © Harvard Medical School, 2018. All Rights Reserved.

•She currently takes no


medicines, no herbal
supplements, and no vitamins.
•The remainder of her history and
exam are unremarkable.

What Labs Would You Order?


A. None, she’s fine
B. B12, A1C, CBC
C. B12, A1C, CBC, Vitamin D
D. B12, A1C, CBC, Chem-7
E. B12, A1C, Chem-7

2746
Copyright © Harvard Medical School, 2018. All Rights Reserved.

What Labs Would You Order?

A. None, she’s fine


B. B12, A1C, CBC
C. B12, A1C, CBC, Vitamin D
D. B12, A1C, CBC, Chem-7
E. B12, A1C, Chem-7

Late Complications of Gastric Bypass

Most common are anemia and b12 deficiency (30%) if not replaced.

Incisional hernia in 10%

Depression/Emotional disturbance (5-10%)

Rare ulcers at the anastomosis site (1-2%)

Very rare electrolyte abnormalities, sbo,

Cholelithiasis

2747
Copyright © Harvard Medical School, 2018. All Rights Reserved.

Case 10:Case 10

Unilateral Hearing Loss

Can You Talk to My Good Ear?

A 36 year old man calls your office with a complaint


that he woke up and can’t hear out of his left ear
very well. He has no history of ear problems, no
recent infections. He did travel recently on an
airplane and wonders if it’s because of the plane
trip. Your astute triage nurse had him hum and the
hum sound did not lateralize. He has no vertigo, but
he has tinnitus and a little bit of ear pain.

2748
Copyright © Harvard Medical School, 2018. All Rights Reserved.

What Do You Do Now?

A. Don’t worry. It’s the plane, take a


decongestant and call me in the
morning if you’re not better.
B. Don’t worry. See if you can come in
this week and we’ll take out the wax.
C. Worry a little. Maybe he has an otitis.
Add him on later or tomorrow.
D. Worry. See him today or refer to ENT.

What do you do now?


A. Don’t worry. It’s the plane, take a
decongestant and let me know if not
better.
B. Don’t worry. See if you can come in
this week and we’ll take out the wax.
C. Worry a little. Maybe he has an otitis.
Add him on later or tomorrow.
D. Worry. See him today or refer to ENT.

2749
Copyright © Harvard Medical School, 2018. All Rights Reserved.

Sudden Sensorineural Hearing


Loss
Rapid hearing loss. Usually noted over 12 hours or on
awakening in the morning.

Usually (but not always) unilateral

More than 90% of patients have tinnitus

Often patients have a sense of ear fullness

Etiologies for SSNHL

Myriad, but most commonly


autoimmune, microvascular, or viral
cochleitis.

Depending on the series, tumor


ranges from 3-30%. MRI to evaluate
the retrocochlear space is indicated.

2750
Copyright © Harvard Medical School, 2018. All Rights Reserved.

Treatment

No agreement on the best protocols.

Most ENT will treat with high dose glucocorticoids (60-


80mg/day) for 10 days. Some studies suggest faster healing.

Some data about intratympanitic steroids as well.

Subset of patients may improve with antivirals.

Prognosis

Generally, the prognosis is good.


• Better if it is high or low frequency loss and not across the
board.
• Better for younger patients

May take up to 4 months and not be complete

May be worse in people with vertigo

2751
Copyright © Harvard Medical School, 2018. All Rights Reserved.

Case 12: Transgender Medicine

You have a new patient coming to see


you. She is a transwoman, meaning she
was assigned male at birth (AMAB), but
has chosen to live as a woman since her
young adulthood. She has been taking
estrogen and testosterone blockers since
her late teens and is now 50. She has not
undergone gender affirmation surgery.

2752
Copyright © Harvard Medical School, 2018. All Rights Reserved.

What cancer screening is most appropriate


for this 53 year old woman?
A. Breast exam, mammogram, pap,
PSA
B. Breast exam, mammogram, colon
cancer screening
C. Breast exam, mammogram, colon
cancer screening, PSA
D. Breast exam, PSA

What Cancer Screening is Most Appropriate


for this 53 year old woman?

A. Breast exam, mammogram, pap,


PSA
B. Breast exam, mammogram, +/-
DRE
C. Breast exam, mammogram, DRE,
and PSA
D. Breast exam, PSA

2753
Copyright © Harvard Medical School, 2018. All Rights Reserved.

I’ve never seen a transgender patient

A phone survey in MA found a self-reported prevalence of .5%

Provide sensitive care

Empathic, non judgmental, don’t assume sexuality based on gender

Honor the patients gender identity (pronouns)

Provide care for the anatomy that is present.

Beyond the PHQ-2


Don’t assume that your patient is depressed, but DO be aware that
depression, anxiety, substance abuse, and trauma are more common in the
transgender population. No clear recommendations, but be aware:

16% have h/o Substance use disorder

Over 50% meet criteria for depression

May be increased risk of suicidal ideation

2754
Copyright © Harvard Medical School, 2018. All Rights Reserved.

Resources for Transgender Care

• https://oi.mgh.harvard.edu/pcoi/primary_care_guidelines/Transgend
er.asp#surg
• UCSF Center of Excellence for Transgender Health
• National LGBT Health Education Center

Best of Luck on the Boards!

• Nothing to declare
• No conflicts of interest

2755
Copyright © Harvard Medical School, 2018. All Rights Reserved.

Psychiatry Overview
Treating Opioid Use Disorder
Sarah E. Wakeman, MD, FASAM
Medical Director, Mass General Hospital Substance Use Disorder Initiative
Program Director, Mass General Addiction Medicine Fellowship
Clinical Co-Lead, Partners Healthcare Substance Use Disorder Initiative
Medical Director, RIZE Massachusetts
Assistant Professor of Medicine, Harvard Medical School

Disclosures
• None

2756
Copyright © Harvard Medical School, 2018. All Rights Reserved.

Drug overdoses now


leading cause of
death for Americans
under 50

Increase in
Opioid
Prescribing Was
Correlated with
Overdose & Rx
OUD

Paulozzi LJ, Jones C, Mack K, Rudd R. Vital signs: overdoses of prescription opioid pain
relievers—United States, 1999–2008. MMWR Morb Mortal Wkly Rep 2011;60:1487–92.

2757
Copyright © Harvard Medical School, 2018. All Rights Reserved.

Ongoing Death
Toll Due to
Heroin/Fentanyl

Dowell D. JAMA. Published online October 11, 2017. doi:10.1001/jama.2017.15971

National Opioid-Related Inpatient


Hospitalizations and ED Visits

2758
Copyright © Harvard Medical School, 2018. All Rights Reserved.

Prevalence of Substance Use Disorder

SAMHSA National Survey on Drug Use and Health. Retrieved from http://iusbirt.org/wp-content/uploads/2014/09/NSDUH14-0904_infographic.jpg

Case 1: Cindy
• New patient: 48 year-old woman with history of low
back pain previously on chronic opioid therapy with
extended release oxycodone

• Chart reports of early refill requests, lost prescriptions

• Toxicology positive for opiates

• PE notable for rhinorrhea, piloerection, myadriasis

• Comes to see you requesting oxycodone

2759
Copyright © Harvard Medical School, 2018. All Rights Reserved.

Case 1: Cindy
• What is the diagnosis?
1. Undertreated pain
2. Diversion
3. Opioid use disorder
4. All of the above

Differential diagnosis
• Aberrant drug-related behavior
1. Undertreated pain
2. Diversion
3. Substance use disorder
• Diagnostic tools
– Clinical history
– SUD dx criteria
– PMP
– Toxicology

2760
Copyright © Harvard Medical School, 2018. All Rights Reserved.

Case 1: Back to Cindy


• Describes initial back injury followed by escalating oxycodone use.
Transitioned to daily injection heroin use since last doctor stopped
prescribing oxycodone.

• Adult children worried, lost job due to missing work due to


withdrawal symptoms.

• Has experienced several non-fatal overdoses.

• “I’ve been to detox too many times to count, it never helps. I hate
AA meetings. I tried buprenorphine a couple of times and it helped
but taking that every day is just replacing one addiction with
another. Sure I wish I could stop using but heroin is the only thing
that makes me feel ok and helps with the pain.”

Patterns of Opioid use in the US

Cicero N Engl J Med 2015; 373:1789-1790

2761
Copyright © Harvard Medical School, 2018. All Rights Reserved.

DSM-5 Opioid Use Disorder


• Taking more or using longer than intended
• Not being able to cut down or stop
• Spending more time obtaining, using, or recovering
• Craving
• Role failure
• Use despite relationship problems
• Important activities given up
• Recurrent use in hazardous situations
• Use despite negative impact on physical or mental health
• Tolerance
• Withdrawal

2-3 mild, 4-5 moderate, 6 or more severe

Case 1: Cindy
• What is the diagnosis?
1. Undertreated pain
2. Diversion
3. Opioid use disorder
4. All of the above

2762
Copyright © Harvard Medical School, 2018. All Rights Reserved.

Case 1: Cindy
• What is the diagnosis?
1. Undertreated pain
2. Diversion
3. Opioid use disorder
4. All of the above

DSM-5 Opioid Use Disorder


Taking more or using longer than intended
Not being able to cut down or stop
Spending more time obtaining, using, or recovering
Craving
Role failure
Use despite relationship problems
Important activities given up
Recurrent use in hazardous situations
Use despite negative impact on physical or mental health
Tolerance
Withdrawal

2-3 mild, 4-5 moderate, 6 or more severe

2763
Copyright © Harvard Medical School, 2018. All Rights Reserved.

Case: Can We Help?


• You tell Cindy that she has a severe opioid use
disorder. What do you do next?
1. Give her a list of detox facilities to call.
2. Tell her that she is pre-contemplative. Advise her
to come back when she is ready to change.
3. Ask about her prior experiences with treatment
and consider buprenorphine initiation.
4. Offer her oxycodone for her back pain.

Case: Can We Help?


• You tell Cindy that she has a severe opioid use
disorder. What do you do next?
1. Give her a list of detox facilities to call.
2. Tell her that she is pre-contemplative. Advise her
to come back when she is ready to change.
3. Ask about her prior experiences with treatment
and consider buprenorphine initiation.
4. Offer her oxycodone for her back pain.

2764
Copyright © Harvard Medical School, 2018. All Rights Reserved.

Motivation: Neither Fixed nor


Linear

Ambivalence in Normal

2765
Copyright © Harvard Medical School, 2018. All Rights Reserved.

Listening for Change Talk


• “I’ve been to detox too many times to count,
it never helps. I hate AA meetings. I tried
buprenorphine and it helped but taking that
every day is just replacing one addiction with
another. Sure I wish I could stop using but
heroin is the only thing that helps with the
pain.”

Strengthening Change Talk


• Elaborate
– “Why did you decide to make that change?”

• Affirm
– “It takes a lot of strength to make those changes.”

• Reflect
– “It sounds like you are ready to stop using.”

2766
Copyright © Harvard Medical School, 2018. All Rights Reserved.

Responding to Change Talk


• “I’ve been to detox too many times to count,
it never helps. I hate AA meetings. I tried
buprenorphine and it helped but taking that
every day is just replacing one addiction with
another. Sure I wish I could stop using but
heroin is the only thing that helps with the
pain.” You are determined to stop using
heroin. What made you decide to
try medication treatment with
buprenorphine and go to detox
and AA meetings in the past?

“As we have seen repeatedly in the history of


medicine, science is one of the strongest allies in
resolving public health crises. Ending the opioid
epidemic will not be any different.”

2767
Copyright © Harvard Medical School, 2018. All Rights Reserved.

Understanding Addiction
• “The question is frequently asked: Why does a man
become a drug addict? The answer is that he usually
does not intend to. Junk wins by default. I tried it as a
matter of curiosity. I drifted along taking shots when
I could score. I ended up hooked. You don’t decide to
be an addict. One morning you wake up sick and
you’re an addict. ”
William S. Burroughs, Junky (1953)

Natural History of Opioid Use


Disorder

Using to feel good

Needing to use more to


feel normal

Using to keep from getting


sick

2768
Copyright © Harvard Medical School, 2018. All Rights Reserved.

Understanding Addiction
• Primary, chronic brain
disease characterized by
compulsive drug seeking
and use despite harmful
consequences

• Involves cycles of
recurrence and remission

• 40-60% genetic
American Society of Addiction Medicine. April 12, 2011. www.asam.org
NIDA. August, 2010. http://www.drugabuse.gov/publications/science-addiction

Substance Use Disorder is a Chronic, but


Treatable Illness
Decreased Heart Metabolism Decreased Brain
in Coronary Artery Disease Metabolism in Substance
High
Use Disorder

Low
Healthy heart Diseased Heart Healthy Brain Diseased Brain

Slide Courtesy of NIDA

2769
Copyright © Harvard Medical School, 2018. All Rights Reserved.

Visualizing Recovery

Volkow et al. J. Neurosci., December 1, 2001, 21(23):9414–9418

Treatment Outcomes as Good as for


Other Chronic Diseases

NIDA. Principles of Drug Addiction Treatment. 2012. McLellan et al., JAMA, 284:1689-1695, 2000 .

2770
Copyright © Harvard Medical School, 2018. All Rights Reserved.

• No cure
Similar to • Goal is to prevent acute
Management and chronic complications

of Diabetes • Individualized treatment


or HIV plans and goals

• Treatment includes:
– Medication
– Lifestyle changes
– Regular monitoring
for complications
– Behavioral support

Lessons
from
HIV/AIDS
• In the 1990s lifesaving
medication available;
fundamentally altered
the epidemic

• Sharp & sustained


declines in mortality

• Focus shifted to
adherence,
engagement in care

Sepkowitz KA. N Engl J Med 2001; 344:1764-1772

2771
Copyright © Harvard Medical School, 2018. All Rights Reserved.

Access & Adherence to Lifesaving Medication Can Fundamentally Alter


the Overdose Epidemic

Carrieri et al. Clinical Infectious Diseases, Volume 43, Issue Supplement_4, 15 December 2006, S197–S215

Medication
Methadone
Buprenorphine
Naltrexone

Psychosocial Interventions
What is Cognitive behavioral therapy
Effective Motivational enhancement therapy
Treatment? Contingency management

Recovery Supports
Recovery coaching
Mutual help organizations

2772
Copyright © Harvard Medical School, 2018. All Rights Reserved.

Medication Treatment Improves


Retention, Abstinence, Survival

Buprenorphine Maintenance
75% retained in treatment
75% abstinent by toxicology

Detoxification + counseling
0% retained in treatment
20% died

Kakko et al. Lancet. 2003 Feb


22;361(9358):662-8

Goal of Medications for


Addiction Treatment

1 2 3 4
Relieve Block effects Reduce Restore
withdrawal of other cravings normal reward
symptoms opioids pathway

2773
Copyright © Harvard Medical School, 2018. All Rights Reserved.

Pharmacology of Treatments

Antagonist
(naltrexone)

Pharmacology of Treatments

Stable level of opioid


effect experienced as
neither intoxication
nor withdrawal, but as
“normal”

Antagonist
(naltrexone)

2774
Copyright © Harvard Medical School, 2018. All Rights Reserved.

Opioid Detoxification Ineffective

Chutuape et al. Am J Drug Alcohol Abuse. 2001


Feb;27(1):19-44.

What is Effective Treatment?

Treatment A

Treatment B

Kakko et al. Lancet. 2003 Feb 22;361(9358):662-8

2775
Copyright © Harvard Medical School, 2018. All Rights Reserved.

Opioid Agonist Therapy is Effective


Treatment

Buprenorphine Maintenance
75% retained in treatment
75% abstinent by toxicology

Detoxification
0% retained in treatment
20% died

Kakko et al. Lancet. 2003 Feb


22;361(9358):662-8

Treatment Retention Higher

Sees et al. JAMA. 2000;283(10):1303-1310.

2776
Copyright © Harvard Medical School, 2018. All Rights Reserved.

Opioid Use Lower

Mattick et al. Cochrane Database of Systematic


Reviews 2009, Issue 3. Art. No.: CD002209.

Mortality Decreased
All cause mortality rates (per 1000
person years):

•In methadone treatment: 11.3


•Out of methadone treatment: 36.1

•In buprenorphine treatment: 4.3


•Out of buprenorphine treatment: 9.5

Overdose mortality rates:

•In methadone treatment: 2.6


•Out of methadone treatment: 12.7

•In buprenorphine treatment: 1.4


•Out of buprenorphine treatment: 4.6

Sordo et al. BMJ 2017;357:j1550

2777
Copyright © Harvard Medical School, 2018. All Rights Reserved.

Buprenorphine vs. XR-NTX?

• The difference in opioid relapse-free survival between XR-NTX and BUP-NX

• Relapse events were greater for XR-NTX (65%) than for BUP-NX (57%)

• Only 72% of XR-NTX group started med (53% if randomized during detox)

• 94% of group assigned to buprenorphine started

Lee JD et al. The Lancet , Volume 391 , Issue 10118 , 309 - 318

Treatment Selection:
Belief versus Science
“We as a society… think [people with addiction]
should just get off drugs and by strenuously
hauling up on their own bootstraps should
stay off no matter what.
Policymakers and some clinicians continue to
promote detoxification as ‘treatment,’ even
though detoxification does nothing to help
people stay off drugs.”

Ling W. J Neuroimmune Pharmacol (2016)


11:394–400

2778
Copyright © Harvard Medical School, 2018. All Rights Reserved.

Medications for Addiction Treatment Work

Back to Cindy
• She has now been on buprenorphine/naloxone for 12
months and is doing well. She no longer meets criteria
for opioid use disorder, has started working, and
reconnected with family.

• How long does she need to stay on bup/nlx?


1. Medication treatments are best utilized as a bridge to
abstinence, she should begin a taper.
2. Medication treatments may be required long term or
even throughout life; so long as she is benefitting there is
no need to taper.
3. She should begin a taper after 2 years of treatment if she
continues to do well.

2779
Copyright © Harvard Medical School, 2018. All Rights Reserved.

Back to Cindy
• She has now been on buprenorphine/naloxone for 12
months and is doing well. She no longer meets criteria
for opioid use disorder, has started working, and
reconnected with family.

• How long does she need to stay on bup/nlx?


1. Medication treatments are best utilized as a bridge to
abstinence, she should begin a taper.
2. Medication treatments may be required long term or
even throughout life; so long as she is benefitting there
is no need to taper.
3. She should begin a taper after 2 years of treatment if she
continues to do well.

Long-term Treatment

• “In most cases, treatment will be required in


the long term or even throughout life. Such
long-term treatment, common for many
medical conditions, should not be seen as
treatment failure, but rather as a cost-
effective way of prolonging life and improving
quality of life, supporting the natural and
long-term process of change and recovery.”

World Health Organization http://apps.who.int/iris/bitstream/10665/43948/1/9789241547543_eng.pdf

2780
Copyright © Harvard Medical School, 2018. All Rights Reserved.

Long Term Outcomes with Buprenorphine

Engagement in agonist therapy associated with abstinence at Month 42:


Patients on agonist therapy: 79.6% abstinent
Not on agonist therapy: 50.8% abstinent

Weiss et al. Drug Alc Depend. 2015;150:112-9.

Treatment Takes Time

Hser et al. Addiction. 2016 Apr;111(4):695-705.

2781
Copyright © Harvard Medical School, 2018. All Rights Reserved.

Treatment Effective in Primary Care

No difference in self reported opioid use, opioid abstinence, study


completion, or cocaine abstinence between the 2 groups
Fiellin DA et al. Am J Med 126:1 2013

Hospitals Have Opportunity to Initiate


Treatment
• Initiating methadone in hospital:
– 82% present for follow-up addiction care

• Buprenorphine vs. detox among inpatients:


– Bupe: 72.2% enter into treatment after discharge
– Detox : 11.9% enter treatment after discharge

• Buprenorphine vs. referral in ED:


– Bupe: 78% engaged in treatment at 30 days
– Referral: 37% engaged in treatment at 30 days

J Gen Intern Med. Aug 2010; 25(8): 803–808; JAMA Intern Med 2014 Aug;174(8):1369-76.; D'Onofrio et al. JAMA 2015 Apr 28;313(16):1636-44

2782
Copyright © Harvard Medical School, 2018. All Rights Reserved.

Lack of Access to Treatment

Ann Fam Med 2015;13:23-26.

Despite Scientific Advances, Huge Gaps in Care

“[The] profound gap between the science of addiction and


current practice… is a result of decades of marginalizing
addiction as a social problem rather than treating it as a
medical condition. Much of what passes for “treatment”
of addiction bears little resemblance to the treatment of
other health conditions.”
Addiction Medicine: Closing the Gap between Science and Practice
www.casacolumbia.org

2783
Copyright © Harvard Medical School, 2018. All Rights Reserved.

The Need for Change


“For nearly a century, physicians were indoctrinated with the
societal attitude that addicts brought upon themselves the
suffering they deserve. Even after we began to regard addicts
as having a disease, our policies continued to reflect our
attitude: addicts are sick, they need help, but they also sin, so
do not help them too much. Until the correct mindset is
restored in the physician, the mere availability of an effective
medication will not make a difference. To put it another way,
for buprenorphine to succeed clinically, physicians themselves
must first change before they can help patients change their
lives.”

Ling. J Neuroimmune Pharmacol (2016) 11:394–400

Turning The Tide

2784
Copyright © Harvard Medical School, 2018. All Rights Reserved.

Evidence • Immediate access to all


types of medication for
Based Care OUD treatment

Checklist • Treatment based on


clinical need & patient
preference/experience
(i.e. not a one-size-fits-all
approach)

• Focus is on retaining
patients in care

Prevention

• Judicious opioid prescribing


• Address risk factors for
development of OUD

Systems Treatment

Based • Immediate access to medication for


OUD in all settings
• Reduce stigma
Checklist
Harm Reduction

• Naloxone
• Syringe exchange
• Safe consumption sites

2785
Copyright © Harvard Medical School, 2018. All Rights Reserved.

Take Home Points


• Fatal overdoses, the leading cause of death for
Americans under 50, are preventable

• Opioid use disorder is a diagnosable and


treatable disease

• Medications are the cornerstone of effective


treatment and can be successfully provided in
primary care and initiated in hospitals/EDs

Thank you!
• swakeman@partners.org
• @DrSarahWakeman

2786
Copyright © Harvard Medical School, 2018. All Rights Reserved.

References
• Carrieri et al. Clinical Infectious Diseases, Volume 43, Issue Supplement_4, 15 December 2006, S197–S215
• Chutuape et al. Am J Drug Alcohol Abuse. 2001 Feb;27(1):19-44.
• Cicero N Engl J Med 2015; 373:1789-1790
• Clark RE et al. J Subst Abuse Treat. 2015 Oct;57:75-80
• D'Onofrio et al. JAMA 2015 Apr 28;313(16):1636-44
• European Monitoring Centre for Drugs and Drug Addiction (2015)
• Fiellin DA et al. Am J Med 126:1 2013
• Hser et al. Addiction. 2016 Apr;111(4):695-705.
• Hutchinson et al. Ann Fam Med 2015;13:23-26.
• Kakko et al. Lancet. 2003 Feb 22;361(9358):662-8
• Katz J. 2017. https://www.nytimes.com/interactive/2017/06/05/upshot/opioid-epidemic-drug-overdose-deaths-are-rising-
faster-than-ever.html?_r=0
• Lee JD et al. The Lancet , Volume 391 , Issue 10118 , 309 - 318
• Leibschutz et al JAMA Intern Med 2014 Aug;174(8):1369-76
• Ling W. J Neuroimmune Pharmacol (2016) 11:394–400
• Mattick et al. Cochrane Database of Systematic Reviews 2009, Issue 3. Art. No.: CD002209.
• McLellan et al., JAMA, 284:1689-1695, 2000.
• Murthy V. https://addiction.surgeongeneral.gov/surgeon-generals-report.pdf
• Sees et al. JAMA. 2000;283(10):1303-1310.
• Sepkowitz KA. N Engl J Med 2001; 344:1764-1772
• Shanahan et al. J Gen Intern Med. Aug 2010; 25(8): 803–808
• Sordo et al. BMJ 2017;357:j1550
• Volkow N Engl J Med. May 31, 2017DOI: 10.1056/NEJMsr1706626
• Weiss et al. 2016. https://www.hcup-us.ahrq.gov/reports/statbriefs/sb219-Opioid-Hospital-Stays-ED-Visits-by-State.jsp
• Weiss et al. Drug Alc Depend. 2015;150:112-9.
• World Health Organization http://apps.who.int/iris/bitstream/10665/43948/1/9789241547543_eng.pdf

2787
Copyright © Harvard Medical School, 2018. All Rights Reserved.

Hyperlipidemia
Scott Kinlay, MBBS, PhD

Associate Chief Cardiovascular Division & Director of Vascular


Medicine, VA Boston Healthcare System
Associate Physician, Brigham & Women’s Hospital
Associate Professor in Medicine, Harvard Medical School

Disclosures
• Research Grants:
– VA CSP, VA MERIT, DoD
• Consultant:
– Colorado Prevention Center DSMB
• Speakers Bureau: None
• Advisory Committees:
– ACC PAD Council
• Ownership/ Other Financial Interest: None

2788
Copyright © Harvard Medical School, 2018. All Rights Reserved.

Learning Objectives
• Review effects of diet on lipids and
cardiovascular risk
• Review value and limitations of statin
therapy
• Review the implementation of current lipid
guidelines to reduce cardiovascular risk
• Review the new PCSK9 agents

Decreasing Cardiovascular Risk


Healthy Lifestyles
• Some lipid effects
• Non-lipid effects
– Vascular compliance
– Cardiac mechanics
– Muscle metabolism

2789
Copyright © Harvard Medical School, 2018. All Rights Reserved.

Fats and Coronary Artery Disease


• 1960-1980 Coronary Epidemic
• Response focused on fat as a
the causal “nutrient”
• Recommended dietary
allowances for saturated fat
and dietary cholesterol
• The food industry response
was predictable…

Low Fat Processed Foods

An Explosion of Processed
Foods with Refined Sugar
Instead of Fat

2790
Copyright © Harvard Medical School, 2018. All Rights Reserved.

Fat versus Sugar?


• Fat is a lubricant and more satiating
• Sugar is less satiating so the tendency is to
eat more more calories obesity

Modulators of High COH Diet Effects

Fiber
Whole Grain Content
Content

Glycemic
Index
Liquid versus
Accessibility
Solid COH
of Starch
and Sugars

Mozaffarian D. Circulation 2016; 133: 187

2791
Copyright © Harvard Medical School, 2018. All Rights Reserved.

Low Carb & Weight Loss Diets


• Atkins – 4 Phase Diet
– Very low COH, unlimited protein/fat
• Weight Watchers
– Calorie restriction by Physical Activity/Behav
Modification,
• Zone Diet and South Beach (3 Phase) Diets (similar)
– Low glycemic COH, Low-fat proteins, some monounsat FA
• Paleo Diet
– High protein, fruits, nuts, vegetables, organ meats, lard
– Low or no dairy, grains, sugars, legumes

Effects of Diets on Lipids


Short Term < 12 Mths Long Term > 12 Mths
Atkins ↓ 0-20% TG ↓ 10% TG
↑ 0-10% HDL ↑ 20% HDL
↑ 0-10% LDL ↔ 0% LDL
Atkins versus
Weight Watchers TG Less decline No difference
HDL Less increase
LDL about the same
Zone Both decrease TG No difference
No change HDL
LDL 10% lower
Paleo: Uncertain data
Effects on CV Risk Unknown
Atallah R. Circ Cardiovasc Qual Outcomes 2014; 7: 815

2792
Copyright © Harvard Medical School, 2018. All Rights Reserved.

Low Fat or Low Carb for Weight Loss?


609 Subjects in the DIETFITS RCT

Healthy Low Fat


Healthy Low Carb

-40 -30 -20 -10 0 10 20


12 Month Weight Change, kg
Note: Both Healthy Diets
Gardner CD, et al. JAMA 2018; 319: 667

Other Diets
• DASH Diet
– Designed to lower BP, but also lowers CVD
• Gluten-free diets
– For celiac disease, but no other evidence of benefit
• Ketogenic diet
– Low COH (± protein) to induce ketosis & weight loss
• Vegetarian diet
– Associated with low CVD if low in saturated fats
• Mediterranean diet

2793
Copyright © Harvard Medical School, 2018. All Rights Reserved.

“Mediterranean Diet”
• Food-based not nutrient-based diet
• High intake: fruits, nuts, non-starchy vegetables,
legumes, monounsat fats (olive/canola)
• Modest intake: whole grains, fish,chicken,
• Low intake: red meat, refined grains, starches, sugars

Life Magazine
Greece January 1948.
“Lunch! bread, one onion,
and olives”

PREDIMED: Mediterranean Diet


Primary Endpoint: Myocardial Infarction, Stroke, CV Death

Control Diet

Med Diet + Nuts

Med Diet +
Extra Virgin Olive Oil

Estruch R, et al. NEJM 2013; 368: 1279

2794
Copyright © Harvard Medical School, 2018. All Rights Reserved.

Low Fat Nutrient or Food-Based Diet?


Women’ Health Predimed
Initiative Low Fat “Mediterranean” Diet

Mozaffarian D. Circulation 2016; 133: 187


Estruch R, et al. NEJM 2013; 368: 1279
Howard BV, et al. JAMA 2006; 295: 655

Low Calorie Vegetarian vs Mediterranean


118 Subjects in the CARDIVEG RCT

Body Weight (kg) BMI Fat Mass (kg)

No Difference in Weight Loss

Opposite Changes in LDL and Trigs, but Small Differences


140
LDL p=0.01
140
Trigs p=0.01 Both
Trigs (mg/dL)
LDL (mg/dL)

130 130 Low


120 120
Calorie

110 110 Soti F, et al.


Circulation 2018,
100 100 Feb 26
T0 T2 T0 T2

2795
Copyright © Harvard Medical School, 2018. All Rights Reserved.

US Dietary Guidelines Advisory


Committee 2015
• The overall body of evidence identifies that a
healthy dietary pattern is
– Diets higher in vegetables, fruits: Strong Evidence
– Diets higher whole grains: Moderate to Strong
Evidence
– Diets higher low- or non-fat dairy, seafood, legumes,
and nuts: Moderate Evidence
– Diets with some alcohol (only among adults), but
increased risks of accidents: Moderate Evidence
– Diets lower in red and processed meat: Strong
Evidence
– Diets low in sugar-sweetened foods and drinks and
refined grains: Strong Evidence
http://www.health.gov/dietaryguidelines/2015-scientific-report/02-executive-summary.asp

Does Changing to a Healthy Diet Work?


20% Increase in 3 Healthy Diet Scores
48K Women in the Nurses Study & 25K Men Health Prof Fup Study

Lower Risk of Death

Decreased risk of death with a healthier diet with greater


decrease in risk with increasing years
Sotos-Prieto M, et al. NEJM 2017; 377: 143

2796
Copyright © Harvard Medical School, 2018. All Rights Reserved.

Features of a Healthy Diet


…our findings support the 2015 Dietary
Guidelines Advisory Committee that it is not
necessary to conform to a single diet plan to
achieve healthy eating patterns

These three dietary patterns, although


different…. Common food groups in each score
contributing most to improvements were
whole grains, vegetables, fruits, and fish….

Sotos-Prieto M, et al. NEJM 2017; 377: 143

LDL Lowering Medications


Important Adjunct to Healthy
Lifestyle in Higher Risk Groups
• Familial Hypercholesterolemia
• Clinically Evident Atherosclerosis
– CAD, PAD, CVD
• Diabetes Mellitus
• Multiple Risk Factors

2797
Copyright © Harvard Medical School, 2018. All Rights Reserved.

Decrease in LDL and Decrease in Risk


Relative Risk of Major Vascular Events

Relative Risk Reduction, %


Between-Group Difference in LDL-C, mmol/L
Silverman MG, et al. JAMA 2016;316:1289

LDL Lowering Therapies


All LDL lowering
therapies increase the LDL-R
expression of LDL
receptors on the liver
to increase LDL uptake,
Statins
and drive down serum Bile salt Binders
Ezetimibe
LDL levels. PCSK9 inhibitors

2798
Copyright © Harvard Medical School, 2018. All Rights Reserved.

Familial Hypercholesterolemia
Disorders of the LDL Receptor
• Autosomal dominant
• Strong association - premature CAD
• > 200 LDL receptor mutations:
– Phenotype influences course of disease
• Founder effect in certain populations:
– French Canadians, Lebanon
• Clinical diagnosis: Xanthomas, high LDL, FH

Homozygous vs Heterozygous FH
Homozygous Heterozygous
Mutant Alleles Both One
Frequency 1 : 1 million 1 : 500
LDL Receptor Activity < 2% 2 – 25%
Cutaneous Xanthomas as Child Xanthomas as Adult
LDL >> 190 mg/dL* > 190 mg/dL*
Myocardial Infarct Childhood – Early 40% by 50 years age
Adult
Treatment Apheresis, Drugs ± Apheresis
Liver Transplant, Genetic Counselling?
Drugs
* LDL 190 mg/dL ≈ 5.0 mmol/L

2799
Copyright © Harvard Medical School, 2018. All Rights Reserved.

Dutch Lipid Clinic Network Score


Patient History and Physical
Premature CAD (<55 men, < 60 women) 2
Premature cerebral or PVD 1
Tendon xanthoma 6
Arcus cornealis < 45 years old 4
First-degree relative
Premature CVD or LDL > 95th centile 1
Xanthoma, arcus cornealis, or children w LDL > 95th cent 2
LDL Cholesterol
> 330 mg/dL (8.5 mmol/L) 8
250-329 mg/dL (6.5-8.4 mmol/L) 5
190-249 mg/dL (5.0-6.4 mmol/L) 3
155-189 mg/dL (4.0-4.9 mmol/L) 1
DNA functional mutation of LDLR, apoB, or PCSK9 gene 8
Nordestgaard BG, et al. EHJ 2013;34:3478-3490a.

2800
Copyright © Harvard Medical School, 2018. All Rights Reserved.

Dutch Lipid Clinic Network Score


Diagnosis Total Points
Definite FH >8
Probable FH 6–8
Possible FH 3-5
Unlikely FH <3

Statins &
Cardiovascular Risk

They Work…

2801
Copyright © Harvard Medical School, 2018. All Rights Reserved.

Statins Work In….


• Acute coronary syndromes
• Stable coronary syndromes
• Coronary equivalent syndromes
• Diabetes
• Hypertension and other risk factors
• Peripheral artery disease
• Multiple risk factors with moderate to high
risk of cardiovascular disease over 10yrs

Statins and Primary vs Secondary


~175K Subjects
Prevention in 27 Trials
Rel Risk per 40 mg/dL
reduction in LDL
“Primary”
Prevention
Lower Risk

Higher Risk
Major
Vasc Event
“Secondary” RR ↓25%
Prevention
Lower Risk

Higher Risk Major


Vasc Event
RR ↓20%
CTT Collaborators. Lancet. 2012;380:581.

2802
Copyright © Harvard Medical School, 2018. All Rights Reserved.

How to Use Statins

Absolute LDL Change at 1 Year and


Percentage Reduction in Events
Major Coronary Events
50%
Reduction in Events

35%

25%

10%

0%

20% 20 40 60 80 mg/dL
(0.5) (1.0) (1.5) (2.0) (mmol/L)
Reduction in LDL
Cholesterol Treatment Trialists (CTT) Collaborators. Lancet 2005; 366:1267

2803
Copyright © Harvard Medical School, 2018. All Rights Reserved.

35% Relative Reduction Yields a Higher


Absolute Risk Reduction
For Higher Risk Patients
ACS Absolute
Benefit
SAP
from
CAD Equivalent Treatment
ASx Multi Risk Factor

ASx 0-1 Risk Factor

2804
Copyright © Harvard Medical School, 2018. All Rights Reserved.

2805
Copyright © Harvard Medical School, 2018. All Rights Reserved.

2806
Copyright © Harvard Medical School, 2018. All Rights Reserved.

35% Relative Reduction Yields a Higher


Absolute Risk Reduction
For Higher Risk Patients
ACS Absolute
Benefit
SAP
from
CAD Equivalent Treatment
ASx Multi Risk Factor

ASx 0-1 Risk Factor

2807
Copyright © Harvard Medical School, 2018. All Rights Reserved.

Adverse Effects Statins


• Myopathy
– Myositis↔Rhabdo
– Myalgia
• Cancer (No)
• Diabetes Mellitus Bosch X, et al. NEJM 2009; 361: 62

• Cognitive Changes
• Hepatitis (so rare only test if symptoms/signs)
• Use in the Elderly

Myositis
• Myositis (1:100)
– Muscle weakness/pain, ↑CK 5-10x ULN Symptoms/ CK
– Without renal involvement Rapidly
• Rhabdomyolysis (1:10,000) Reversible on
– Profound weakness, ↑CK > 10x ULN Statin
– Renal involvement and/or myoglobinuria Withdrawal

• Statin-Induced Necrotizing
Autoimmune Myopathy (1:100,000) Persistence of
– Profound proximal weakness Symptoms/ CK
– Ab to HMG CoA Reductase on Statin
– ELISA test & Consider immunotherapy Withdrawal

2808
Copyright © Harvard Medical School, 2018. All Rights Reserved.

Risk Factors for Myositis


• Hypothyroidism
• Vit D deficiency?
• High dose statins (esp. simvastatin)
• Advanced age
• Lower BMI
• Cytochrome P450 Inhibition
– Increases serum statin concentration but unpredictable
effect on risk of myositis
• Rare NSLCO1B1 polymorphisms
– Decrease statin uptake by the liver

Factors that Decrease CYP-450 3A4


PotenXally ↑ StaXn ConcentraXons but Side-Effects Unpredictable

• Gemfibrozil (esp with simvastatin)


• Cyclosporine
• Macrolide antibiotics
• Azole antifungals
• ART drugs in HIV (esp protease
inhibitors)
• Grapefruit juice
– Inhibits GI but not Liver CYP-450

2809
Copyright © Harvard Medical School, 2018. All Rights Reserved.

Other Potential Causes of CK↑


• Hypothyroid (also increases lipids)
• Idiopathic Hyper-CK-emia
– Benign elevation of CK (100’s to 1000’s)
• Injury or Excessive Exercise
– Focal muscle tear vs Diffuse myositis
• Alcoholism
– Pressure necrosis and dehydration
• Other Inflammatory Myopathies*
– Polymyositis, Dermatomyositis (heliotrope)
– Also present with ↑CK and Myopathic EMG
* Check out Clinical Problem Solving: NEJM 2016: 374: 1774
http://www.nejm.org/doi/full/10.1056/NEJMimc1508369

Statin Myalgia Without Myositis


• Muscle pain and aching
• CK normal or < 4 x ULN
• STOMP trial: 420 statin naïve patients
– 10% in atorva 80/d group vs 5% in placebo group
– So incidence probably about 5%
• Symptoms started on average 1 mth after
starting statin
• Resolved within 2 weeks of stopping drug
• Returned within 4 weeks of re-challenge
• Coenzyme Q10 not effective in one RCT
STOMP: Parker BA, et al. Circulation 2013; 27: 96
Taylor BA, et al. Atherosclerosis 2015; 238: 329

2810
Copyright © Harvard Medical School, 2018. All Rights Reserved.

GAUSS-3 Statin Intolerance


• 491 patients intolerant of Atorva 10mg/d or
another statin
• Rechallenged with Atorva 20mg/d or Placebo
Muscle Symptoms
50%
43%
40%

30% 27%
20% 17%
10%
10%

0%
Statin Only Placebo Only Statin & Placebo Neither
Nissen SE. JAMA 2016; 315: 1580

Management of Statin Myopathy


• Stop Statin until symptoms resolve
• Check for contributing factors
– Hypothyroidism, Vit D deficiency
– Drug interactions – e.g. new drugs
• Reassure patients of the safety of statins
– Try at least 2 other statins or lower doses
– Use alternate day dosing ( e.g. atorva/rosuva)
• Non-statin alternatives
– Ezetimibe 10mg/d (20% LDL lowering)
– PCSK9 inhibitors if FH or CVD “not at goal”

2811
Copyright © Harvard Medical School, 2018. All Rights Reserved.

Case: HyperCK and Myopathy


• 75 yr old African American man seen for lymphedema
• Right CEA 2014, DM on diet, HT, HL
• Rosuvastatin 40mg/d since 2013
• July 2015/ Feb 2016: Doing well
• Mar 2016: ER visit for palpitations, no muscle aches
but CK 1083. Rosuva stopped. TSH/VitD/ESR normal
• Apr 2016: On questioning he had mild muscle aches
and difficulty climbing stairs over 1 year “old age” –
symptoms gone now off statin
• Jun 2016: Started 10mg Rosuva /day
• Oct 2016: No muscle symptoms on Rosuv 5mg/d

Statin, CK and LDL


Date Rosuva Muscle Sx CK LDL
mg/d U/L mg/dL
Nov 2012 0 153
Feb 2016 40 Yes (retro) 44
Mar 2016 40 Yes 1084 42
(ER Visit) Stopped
May 2016 0 No 1036 120
Jun 2016 10 No 1296 72
Oct 2016 5 No 1334 94

2812
Copyright © Harvard Medical School, 2018. All Rights Reserved.

Case: Myopathy on High Dose Statin


• 46 yr old man with moderate renal a. stenosis
• Strong FH: Father SCD 45yrs, Mother CAD
• Non-alcoholic fatty liver treated with simva and
prava with mild increase in LFTs – statins stopped
• Oct 2015: Atorva 20/d Myalgias stopped
• Apr-Jun 2016: Dose ranging Rosuva
• Oct 2016: Rosuva 5 mg/d & doing well
– AST: 10/2015=27 10/2016=31
– ALT: 10/2015=48 10/2016=37

Statin Dose, Symptoms, Lab Work


Date Statin LDL mg/dL Myalgias CK
mg/d
Aug 2015 0 177 No
Oct 2015 A 20 77 Yes
Feb 2016 0 172 No ETT OK
11min
April 2016 R 10 71 No 91
May 2016 R 20 65 Yes 730
May 2016 0
Jun 2016 R5
Oct 2016 R5 85 No 251
Plan: If Symptoms and CK remain stable, consider adding Ezetimibe

2813
Copyright © Harvard Medical School, 2018. All Rights Reserved.

Statins and Diabetes


Jupiter Study: Rosuva vs Placebo
Major Risk Factors for Diabetes Mellitus

No major risk factors One or more risk factors

HR = 1.28 (1.07 – 1.54), p=0.01

HR = 0.99 (0.45 – 2.21), p=0.99

Years of Follow-up
Ridker PM, et al. Lancet 2012; 380: 565

Statins and Diabetes: Impact on CVD


Jupiter Study: Rosuva vs Placebo
Major Risk Factors for Diabetes Mellitus

No major risk factors One or more risk factors

HR = 0.67 (0.55 – 0.81)


p=0.0001
HR = 0.67 (0.53 – 0.85)
p=0.0007

Years of Follow-up
Ridker PM, et al. Lancet 2012; 380: 565

2814
Copyright © Harvard Medical School, 2018. All Rights Reserved.

~175K Subjects
Statins and Cancer
Rel Risk per 40 mg/dL
reduction in LDL

Cancer Incidence

Cancer Death

CTT Collaborators. Lancet. 2012;380:581.

Statins and Cognition


• High LDL a risk factor for stroke, multi-infarct
dementia, and Alzheimer's disease
• 60 cases reported to the FDA
of reversible cognitive
dysfunction with statins
• Observational meta-analyses
fail to show an effect of statins
on cognition
• In animal models, statins
decrease neuroinflammation
and amyloid-β concentrations Thompson PD, et al. JACC 2016; 67: 2395

2815
Copyright © Harvard Medical School, 2018. All Rights Reserved.

25 Statin RCTs and Cognition


Statins Better Statins Worse
Domain N (n)
Global 5 (26,515)
Attention 7 (732)
Executive 7 (26,926)
Memory 8 (26,850)
Processing Speed 10 (6,630)
Working Memory 3 (83)

All Domains 14 (27,643)

Standardized Mean Difference Ott BR, et al. J Gen Intern Med 2015; 30: 348

Statins in the Elderly Over 75 Years


• Clinical trials mostly < 75 yrs
Over 75 yrs age group
• Large number of CVD events
• Many trials support continued statins in
patients > 75 yrs
• But, acute pharmacokinetics/dynamics
– Very old have higher serum statin concentrations
– Cmax 50% higher, AUC 25% higher
– ?higher risk of side effects with maximum doses

2816
Copyright © Harvard Medical School, 2018. All Rights Reserved.

Statins Over 75 Years of Age

Good Life Expectancy Poor Life Expectancy


Clinically Evident CAD Muscle Weakness
Diabetes, PVD, isch CVA Frailty or Dementia
Drug Interactions
Consider Intermediate Dose

Where Statins May Not


Work

2817
Copyright © Harvard Medical School, 2018. All Rights Reserved.

RCTs No Benefit From Statins


• End-Stage Renal Disease on Dialysis 2+yrs
– AURORA (Rosuva), 4D (Atorva)
– InSHARP (Simva/Ezet) benefit, but some pre-HDx
• Aortic Stenosis
– Saltire (Atorva), SEAS (Simva/Ezet), Astronomer (Rosuva)
• CHF no CAD
– CORONA (Rosuva), GISSI-HF (Rosuva) all Negative
• ARDS/ COPD
– ARDS & COPD Clinical Research Networks - No benefit
• CABG
– STICS No benefit Rosuva 20mg for post-op AFib or MI
• Pre-PCI in ACS versus within 24 hours
– SECURE-PCI No benefit
Fellstrom B et al. N Engl J Med 2009;360:1395-1407 SALTIRE: Cowell JS, et al. NEJM 2005; 352: 2389
Kjekshus J et al. N Engl J Med 2007;357: 2248 SEAS: Rossebø AB, et al. NEJM 2008; 359: 1343
Astronomer: Chan KL, et al. Circulation 2010; 121: 306
Zheng Z, et al. N Engl J Med 2016; 374: 1744 Berwanger O, et al. JAMA 2018. March 11

Therapies for Low HDL and


High TGs

2818
Copyright © Harvard Medical School, 2018. All Rights Reserved.

Fibrates, Niacins, Omega-3


Fibrates: ACCORD Niacin: AIM-HIGH

No Significant Differences
In CV Events When Used With Statins
The ACCORD Study Group. N Engl J Med 2010;362:1563-1574 AIM-HIGH Investigators. NEJM 2011; 365: 2255

Niacin: HPS2-THRIVE Omega-3: ORIGIN

HPS2-THRIVE Investigators. NEJM 2014; 371: 203 ORIGIN Trial Investigators. NEJM 2012; 367: 309

CETP Inhibitors Raise HDL 2-3x


Torcetrapib: ILLUMINATE Dalcetrapib: Dal-OUTCOMES

Barter PJ, et al. NEJM 2007; 357: 2109 Schwartz GG, et al. NEJM 2012; 367: 2089

Evacetrapib: ACCELERATE
No Significant Differences
(or Worse)
In CV Events When Used
With Statins
Lincoff AM, et al.. NEJM 2017; 376: 1933

2819
Copyright © Harvard Medical School, 2018. All Rights Reserved.

Role for Non-LDL Lowering

• No role in combination with statins


• Maybe a role in statin intolerant subjects
• Gemfibrozil: Helsinki, VA-HIT
• Niacin: CDP, HATs
• Omega 3 FA: GISSI, small studies
• Better drugs available

Questions
• 52 yr old man with HIV tolerating highly active anti-retroviral
therapy (HAART) without any symptoms. He presents for a
routine follow-up including renewal of his medications. His
fasting lipid levels are:

– Total Cholesterol 253


– LDL cholesterol 170
– HDL cholesterol 41
– Trigs 210

• The most appropriate management is:


– Start simvastatin 40 mg/d Higher risk of HAART drug interaction
– Start fenofibrate Particularly with protease inhibitors
A: – Start atorvastatin 20 mg/d
– Strict dietary change to lower saturated fats and recheck lipids in 6
months Unlikely to achieve required lipid lowering

2820
Copyright © Harvard Medical School, 2018. All Rights Reserved.

Ezetimibe and Statins

IMPROVE-IT: Ezetimibe & Simva


Ezetimibe/Simva 10/40mg versus Simva 40-80mg
HR = 0.936 (95%CI: 0.89, 0.99)
p = 0.016 Statin Alone
ΔLDL
26% vs 43%
Event Rate (%)

Statin & Ezetimibe

Years Since Randomization Cannon C, et al. NEJM 2015

2821
Copyright © Harvard Medical School, 2018. All Rights Reserved.

Implications of IMPROVE-IT
Reduction in Major Vascular Events (%)
Implies the main effect
of statins and non-
statins is through LDL
lowering

Regenerates interest in
LDL targets?

Supports novel LDL


lowering therapies

Reduction in LDL (mmol/L) Cannon C, et al. NEJM 2015


Jarcho JA, Keaney, JF. NEJM 2015

The 2013 AHA/ACC


Lipid Guidelines

http://circ.ahajournals.org/content/early/2013/11/11/01.cir.0000437738.63853.7a.citation

DOI: 10.1056/NEJMms1314569

2822
Copyright © Harvard Medical School, 2018. All Rights Reserved.

Principles: What Do The RCTs Say?


• Who to Treat:
– Patients with Clinically Evident CVD
• ACS, Stable Angina, Prior MI
• HPS, PROVE-IT TIMI22, MIRACL, TNT, CARE, LIPID
– Patients with Diabetes Mellitus & LDL > 70mg/dL
• HPS, CARDS, CTT
– No Diabetes and 10 yr CVD Risk > 7.5%
• WOSCOPS, AFCAPS, ASCOT-LLA, JUPITER
– Patients with LDL > 190mg/dL (i.e. FH)
• Excluded from most RCTs due to high risk of
premature CVD

Principles: What Do The RCTs Say?


• How to Treat:
– Higher risk patients with high intensity statin
therapy to reduce LDL > 50%
• Atorva 40-80mg/d or
• Rosuva 20-40mg/d
– Moderate risk patients with moderate intensity
statin therapy to reduce LDL 30-50%
• Atorva 10-20mg/d, Rosuva 5-10mg/d,
• Simva 20-40mg/d, Prava 40-80mg/d, Lova 40mg/d
• Fluva ext rel 80mg/d, Fluva 40mg bid, Pitava 2-4mg/d

2823
Copyright © Harvard Medical School, 2018. All Rights Reserved.

Measuring CK, LFTs, Lipids


• Baseline pre-statin
– Measure CK only if there is an increased risk of statin
myopathy (history of statin myopathy, family history
of myopathy, drugs with potential interactions)
– Measure baseline LFTs and lipids
• Measurements after statin therapy
– No need to measure LDL (maybe for adherence)
– Remeasure CK only if muscle symptoms
– Remeasure LFTs only if suspicion of hepatotoxicity
(fatigue, jaundice, anorexia, weight loss)

iPhone App Android App


https://itunes.apple.com/us/app/ascvd- https://play.google.com/store/apps/
risk-estimator/id808875968?mt=8 details?id=org.acc.cvrisk&hl=en

http://www.acc.org/tools-and-practice-
support/mobile-
resources/features/2013-prevention-
guidelines-ascvd-risk-estimator

2824
Copyright © Harvard Medical School, 2018. All Rights Reserved.

New Algorithm
Patients > 21 Years of Age Without
- CHF (NYHA class II, III, IV)
- ESRD with Hemodialysis

Diabetes Mellitus No DM
Clinical Athero
40-75 yrs & LDL 40-75 yrs & LDL LDL > 190 mg/dL
CVD
70-189 mg/dL 70-189 mg/dL

High Intensity Calculate 10-yr Calculate 10-yr High Intensity


Statin CVD Risk CVD Risk Statin

CV Risk < 7.5%:


Moderate
CV Risk > 7.5%
Intensity Statin
Moderate - High
CV Risk > 7.5%
Intensity Statin
High Intensity
Statin
Keaney JF, et al. DOI: 10.1056/NEJMms1314569

Triglycerides and Pancreatitis


• 2013 AHA/ACC Lipid Guidelines
– Suggest specific treatment if > 500 mg/dL and
there is concern of increased risk of
atheroscerosis events
• 2012 Endocrine Society Clinical Guidelines
Committee
– Risk of pancreatitis increases above 1000 mg/dL
and consider treatment with concentrations
higher than this

Endocrine Society Clinical Guidelines Committee. JClinEndocrMetab 2012; 97: 2969

2825
Copyright © Harvard Medical School, 2018. All Rights Reserved.

Novel Therapies for LDL Lowering


• PCSK9 Monoclonal Ab Inhibitors
– Alirocumab (Praluent) Available
for Use
– Evolocumab (Repatha)
• PCSK9 RNA Interference Drugs
– Inclisiran – 6mth action in phase 1 trial
• Drugs Interfering with Apo B synthesis
– Lomitapide (↑LFTs, hepaXc steatosis)
– Mipomersen (Monoclonal Ab – injection site
reacXon, ↑LFTs, hepaXc steatosis

PCSK9 & LDL-Receptor Expression


• PCSK9 binds to the
LDL receptor
• LDL receptor degrades
• Prevents LDL receptor
recycling

• PCKS9 inhibitors block


PCSK9 binding to LDL
receptors
• Increase LDL receptor
recycling and
Fuster V. NatureRevCard 2014; 11: 671
expression

2826
Copyright © Harvard Medical School, 2018. All Rights Reserved.

ODYSSEY LONG TERM: Alirocumab


LDL Cholesterol Response: Primary Endpoint 24 months
LDL Cholesterol (mg/dL)

120

60

0
0 24 36 78
Week
Robinson JG, et al. NEJM 2015; 372: 1489

FOURIER: Evolocumab
LDL Cholesterol Response Over 3 Years
LDL Cholesterol (mg/dL)

100 Placebo + Statin

mean LDL = 96 mg/dL


(2.5 mmol/L)

50 Evolocumab + Statin

mean LDL = 30 mg/dL


(0.8 mmol/L)
0
0 1 2 3
Years
Sabatine MS, et al. NEJM 2017; 376: 1713

2827
Copyright © Harvard Medical School, 2018. All Rights Reserved.

FOURIER: Evolocumab
MACE: CV Events, Hospitalization UAP, Revascularization

Sabatine MS, et al. NEJM 2017; 376: 1713

What Happens with an Average LDL


of 30mg/dL (0.8 mmol/L)?

2828
Copyright © Harvard Medical School, 2018. All Rights Reserved.

FOURIER: Adverse Events


10%
8.1%
7.7%
8%
6% 5.0% 4.8%

4% p < 0.001 3.1% 2.9%


2.1%
2% 1.6% 1.6% 1.5%

0.1% 0.1%
0%
Injectn Site Allergic Reactn Muscle Event Rhabdomyolysis New Diabetes Neurocognitive
Reactn

Evolocumab Placebo

Sabatine MS, et al. NEJM 2017; 376: 1713

EBBINHAUS Substudy of FOURIER


Cambridge Neuropsychological Test Automated Battery

• 1974 Subjects
• 40-85 Years

No Difference in Cognitive Function Over 19 months

Giugliano R, et al.
NEJM 2017; 377: 633

2829
Copyright © Harvard Medical School, 2018. All Rights Reserved.

FDA Recommended Doses

• Alirocumab (Praluent)
– 75 mg subcut every 2 weeks
– 150 mg s/c every 2 weeks if inadequate response
• Evolocumab (Repatha)
– 140 mg s/c every 2 weeks, OR
– 420 mg s/c every month (3 x 140mg over 30 min)
• Measure LDL at 4-8 weeks and 6 monthly

Cost of PCSK-9 Inhibitors

• Alirocumab and Evolocumab cost $14,600/ person/ year


• If 100% reduction in CVD events
– 3 events prevented per 100 patients = $60,000
– Treatment of 100 patients = $1,460,000
– Net cost = $1,400,000 or $14,000 per patient
• If 5% of patients eligible, insurance premiums would
need to increased $124 per person
• Cost-effectiveness only acceptable if annual cost drops
to ≈ $4,500 per person
Schulman KA, et al. N Engl J Med 2015; 373:1591
Kazi DS, et al. JAMA 2016;11004 Nov 2016

2830
Copyright © Harvard Medical School, 2018. All Rights Reserved.

PCSK9: Prior Authorization

PCSK9 Access
> 45, 000 Patients
Prescribed PCSK9i in
Symphony Health
Solutions

• 53% Rejected
• 16% Approved
but not Filled
• 31% Approved
and Filled

Navar AM. JAMACard 2017; 2: 1217

2831
Copyright © Harvard Medical School, 2018. All Rights Reserved.

PCSK9 Access
As Copay Increases
Patients Filling Prescriptions, %

Less Likely to Fill Prescription

Number of Patients
Copay Range, $0 - > $500

Navar AM. JAMACard 2017; 2: 1217

Prescribing PCSK9 Inhibitors


Cost Usually Covered by 3rd Party Payers
• Heterozygous FH
– Abnormal gene or LDL > 250 with xanthomas or a
family history of premature ASCVD or Dutch Score>8
– AND, High dose atorva/rosuva and ezetimibe not
resulted in a > 50% reduction in LDL
– AND, Counseled on healthy lifestyle, other RF changes
• Homozygous FH
– Homozygous gene abnormality or untreated LDL > 500
mg/dL or max statin and ezetimibe LDL > 300 mg/dL
– AND, Counseled on healthy lifestyle, other RF changes

2832
Copyright © Harvard Medical School, 2018. All Rights Reserved.

Prescribing PCSK9 Inhibitors


More Difficult Cases for 3rd Party Payer Coverage
• Patients with ASCVD requiring further LDL
lowering
– Large trials yet to establish benefit
– Unclear what threshold defines “need for further
LDL lowering”, ? LDL > 100 mg/dL
– FDA concerned providers will skip high dose
statin/ezetimibe regimens
• Patients intolerant of statins
– Difficult to determine as a many patients with
symptoms tolerate some statin on rechallenge

Summary
• Lifestyle changes are fundamental
• New lipid guidelines emphasize statin therapy
of two intensities
• Omit LDL goals and non-statin therapies
among statin-tolerant subjects
• Avoid statins where they are of no benefit
• PCSK9 Inhibitors are powerful novel agents
but limited in their availability

2833
Copyright © Harvard Medical School, 2018. All Rights Reserved.

Question 1
A 59 year old woman presents to your outpatient clinic
with a history of diabetes mellitus 5 years treated with
diet and metformin and hypertension controlled by an
ACE inhibitor and HCTZ. She stopped smoking 2 years
ago. Her fasting LDL is 115 with triglycerides of 260 and
an HDL of 43 on simvastatin 10mg/d. Which of the
following therapies are indicated to reduce her CAD risk
A. Replace Simvastatin with Atorvastatin 40mg/d or
Rosuvastatin 20mg/d
B. Add Niacin 500mg/d escalating to 2g/d
C. Add Fish oil 1g/d
D. Increase Simvastatin to 20mg/d

Answer Q1
CAD risk equivalent with treated diabetes and
hypertension and an ex-smoker. Mildly elevated LDL,
elevated TGs and low HDL
A. Replace Simvastatin with Atorvastatin 40mg/d or
Rosuvastatin 20mg/d

Current guidelines recommend an intensive statin dose.


The recent randomized trials show no benefit from
adding niacin or fibrates to statin therapy. Doubling the
dose of simvastatin would decrease LDL by less than
10% and have a smaller effect on reducing CAD risk.
Statin therapy also reduces TGs

2834
Copyright © Harvard Medical School, 2018. All Rights Reserved.

Question 2
A 43 year old single man presents for a health checkup
after a screening cholesterol at a shopping center was
reported as high. He has no CAD symptoms, and no
diabetes, hypertension and is a non-smoker. His weight
has increased by 35 lb since college. His LDL is 135 with
triglycerides of 249 and an HDL of 43. Initial therapy
would include
A. Atorvastatin 40mg/d or Rosuvastatin 20mg/d
B. Simvastatin 20mg/d
C. An exercise prescription and dietary advice
D. Niacin 500mg/d

Answer Q2
Young man with no CAD risk equivalents. Bad Lifestyle is
the likely culprit for his weight gain and dyslipidemia.

C. An exercise prescription and dietary advice


Drug therapy is not recommended at this level of risk.
Lifestyle changes are recommended to reduce his
lifetime risk of ASCVD.
Myamericanheart.org/cvriskcalculator

2835
Copyright © Harvard Medical School, 2018. All Rights Reserved.

Question 3
A 69 year old woman man presents for an annual
physical exam. She has no symptoms and has a regular
exercise program of walking 30 minutes each morning.
Her blood pressure is 130/65 her BMI is 23. Her fasting
lipid panel shows an LDL of 135 mg/dL with triglycerides
of 195 and an HDL of 54. The appropriate dietary advice
would include which of the following
A. High in vegetables and fruit, and low in red meat
B. High in red meat and salad, and low in whole grains
C. High in sugar sweetened foods, and low in whole grains
D. High in coconut foods and oils, and low in wheat grains

Answer Q3
Current dietary guidelines support a “Mediterranean”
diet high in vegetables and fruits and low in red meat.

A. High in vegetables and fruit, and low in red meat

Answer B is similar to “Paleo” diets, which are high in animal protein and low
in carbohydrate. These are associated with short-term weight loss but may
have detrimental effects on the lipid panel particularly in non-obese
individuals.
Answer C is typical of the diets of industrialized countries and adverse to
health
Answer D is a variation on some “Paleo” diets which emphasize coconut based
foods and oils. Coconut oil is a saturated fat and can increase LDL levels. The
long-term health outcomes of diets high in coconut are unknown.
http://www.health.gov/dietaryguidelines/2015-scientific-report/02-executive-summary.asp

2836
Copyright © Harvard Medical School, 2018. All Rights Reserved.

References
• Review of Diets and Diet Guidelines. Mozaffarian D. Circulation 2016; 133: 187
• PREDIMED. Mediterranean Diet Trial. Estruch R, et al. NEJM 2013; 368: 1279
• US Dietary Guidelines Advisory. http://www.health.gov/dietaryguidelines/2015-
scientific-report/02-executive-summary.asp
• New AHA/ ACC Guidelines:
– http://circ.ahajournals.org/content/early/2013/11/11/01.cir.0000437738.63
853.7a.citation
– Keaney J, et al. NEJM 2014; 370: 275
• Old Guidelines
– ATPIII. JAMA. 2001;285:2486-2487. Grundy S, et al. Circulation 2004; 110:
227
• Tall A. NEJM 2006; 354: 1310
• FOURIER Trial of Evolocumab. Sabatine MS, et al. NEJM 2017; 376: 1713

2837
Copyright © Harvard Medical School, 2018. All Rights Reserved.

Update in Sepsis

Rebecca M. Baron, M.D.


Assistant Professor of Medicine, Harvard Medical School
Associate Physician, Brigham and Women’s Hospital
Pulmonary and Critical Care Division

Conflict of Interest
No Conflicts
Thanks to Chanu Rhee for allowing me to
steal some of his slides.

2838
Copyright © Harvard Medical School, 2018. All Rights Reserved.

Case

A 47 yo woman with alcoholic cirrhosis is brought to


your ER with fevers, confusion, shortness of breath,
and worsening ascites. Temp is 37°C, SBP is 50
mmHg, HR 150 bpm, RR 40/min, CVP 4 mm Hg, and
O2 sat 90% (RA). CXR shows diffuse infiltrates, and
peritoneal fluid returns with a leukocyte count of
1000/µL (95% polys).

Question #1:

Does this patient have sepsis?

By what criteria?

2839
Copyright © Harvard Medical School, 2018. All Rights Reserved.

What is Sepsis (2001-15)?


Systemic Sepsis: SIRS +
Inflammatory Infection
Response Syndrome
(SIRS): Severe Sepsis:
Temp >38°C or <36°C Sepsis+ Organ
Heart Rate > 90 bpm Dysfunction
Resp Rate > 20/min
WBC >10000, <4000, Septic Shock:
or Bandemia>10% Sepsis+Refractory
Hypotension

Organ Dysfunction +/- Hypotension


(2001-2015)

2840
Copyright © Harvard Medical School, 2018. All Rights Reserved.

• 19-person SCCM/ESICM task force


• Meetings, Delphi processes, EHR records
• Endorsed by 31 Professional Societies
• Prior perceived shortcomings from prior:
• Focus on inflammation
• Continuum sepsis shock
• Vagueness of SIRS
• Severe sepsis is ‘redundant’

• SEPSIS: (>10% mortality)


• Life-threatening organ dysfunction
• Caused by dysregulated response to infection
• Increase SOFA score of ≥2
• SHOCK: (>40% mortality)
• Vasopressors for MAP≥65 mmg Hg
• Lactate>2 mmol/L
• In absence of hypovolemia

2841
Copyright © Harvard Medical School, 2018. All Rights Reserved.

SOFA Score: 6 Organ Systems, 0-4 Points


Points 0 1 2 3 4
<200 + mechanical <100 + mechanical
PaO2/FiO2 ≥400 <400 <300
ventilation ventilation

Platelets ≥150 <150 <100 <50 <20

Bilirubin <1.2 1.2-1.9 2.0-5.9 6.0-11.9 >12.0


Dopamine 5.1-15 Dopamine >15
Dopamine or or
Blood MAP MAP <5 Epinephrine <0.1 Epinephrine >0.1
Pressure ≥70 <70 or or or
Dobutamine Norepinephrine Norepinephrine
<0.1 >0.1
Glasgow
Coma 15 13-14 10-12 6-9 <6
Scale
3.5-4.9 >5.0
Creatinine <1.2 1.2-1.9 2.0-3.4 or or
<500cc urine/d <200cc urine/d

Vincent et al, Intensive Care Med 1996

qSOFA
• Out of hospital, ED, Ward settings
• Worse outcomes predicted from sepsis with 2 of:
• Respiratory Rate ≥ 22/min
• Altered mental status (GCS ≤ 13)
• SBP ≤ 100 mmHg
• Ongoing inquiry as to its validation
• LESS SENSITIVE but MORE SPECIFIC than SIRS
for sepsis screening.

2842
Copyright © Harvard Medical School, 2018. All Rights Reserved.

Limitations
Lab testing required (lactate, SOFA) not
feasible in low-resource settings
Prospective validation underway for
qSOFA score vs. other metrics
No use of biologic markers of sepsis to
help define subgroups or predictors
Practicality of definition for screening
patients vs. billing vs. epidemiologic vs.
research tools, etc.
In Part, from: Abraham E. JAMA 2016;315:757-759

SSC’s approach: essentially no different than before!

Step 1: Screen for Infection


• SIRS may be a sign of infection
Step 2: Screen for Organ Dysfunction
• Use organ dysfunction to target initial treatment and qSOFA to
target closer monitoring for deterioration from sepsis
Step 3: If hypotensive or lactate >=2.0 30 ml/kg crystalloid, with
reassessment of volume responsiveness or tissue perfusion

My bottom line: DON’T MISS SEPSIS. SCREEN with


SIRS, PROGNOSTICATE with qSOFA, Dx SEPSIS w SOFA

2843
Copyright © Harvard Medical School, 2018. All Rights Reserved.

Case, Question #2
A 47 yo woman with alcoholic cirrhosis is brought to
your ER with fevers, confusion, shortness of breath,
and worsening ascites. Temp is 37°C, SBP is 50
mmHg, HR 150 bpm, RR 40/min, CVP 4 mm Hg, and
O2 sat 90% (RA). CXR shows diffuse infiltrates, and
peritoneal fluid returns with a leukocyte count of
1000/µL (95% polys). You initiate your sepsis bundle.
Initial management of hemodynamics should entail
use of:
a. Vasopressin
b. Norepinephrine
c. Norepinephrine + Furosemide
d. Intravenous fluids
e. Dobutamine

Case, Question #2
A 47 yo woman with alcoholic cirrhosis is brought to
your ER with fevers, confusion, shortness of breath,
and worsening ascites. Temp is 37°C, SBP is 50
mmHg, HR 150 bpm, RR 40/min, CVP 4 mm Hg, and
O2 sat 90% (RA). CXR shows diffuse infiltrates, and
peritoneal fluid returns with a leukocyte count of
1000/µL (95% polys). You initiate your sepsis bundle.
Initial management of hemodynamics should entail
use of:
a. Vasopressin
b. Norepinephrine
c. Norepinephrine + Furosemide
d. Intravenous fluids
e. Dobutamine

2844
Copyright © Harvard Medical School, 2018. All Rights Reserved.

Question #2:

What are we supposed to be


doing with fluids in sepsis?

Sepsis: Source Control

EARLY, BROAD, EMPIRIC antibiotic therapy


Not the time to be elegant!
Think of sources needing SURGICAL
INTERVENTION
Catheter / device Remove it
Soft tissue abscess Drain it
Empyema Chest tube
Cholangitis ERCP
Endocarditis Abx/Valve replacement
Septic arthritis Joint debridement
Narrow therapy after 48 – 72H of cultures

2845
Copyright © Harvard Medical School, 2018. All Rights Reserved.

Septic Shock: EARLY Goal-Directed Therapy


2001- 2014

*Treatment began in E.D.: 6-hours


*A-line, Central line insertion
*Monitored: Mean arterial BP (MAP)
Central venous pressure (CVP)
Central venous O2 sat (SCVO2)
NEJM 2001; 345:1368 Hematocrit

EARLY Goal-Directed Therapy


INTERVENTION

500cc IVF Q 30 min!

NL arterial pH
NL base deficit Norepinephrine
NL lactate
ScvO2 > 70%

Dobutamine: dose
increased 2.5 mcg Q
15 min!

2846
Copyright © Harvard Medical School, 2018. All Rights Reserved.

EARLY Goal-Directed Therapy


HOW’D THEY DO THAT??

EARLY Goal-Directed Therapy


IT WORKED!!

However, what are the key components and broader applicability of EGDT?

2847
Copyright © Harvard Medical School, 2018. All Rights Reserved.

1341 Patients, Compared:


1. EGDT, vs.
2. Protocolized care without CVC, inotropes,
blood, vs.
3. Usual care
NO DIFFERENCE in Primary outcome of
60d in-hospital mortality or other outcomes

ProCESS*: What does it mean?


Mortality in usual care group substantially
lower than in EGDT in first trial (18% vs.
46%), thus “usual care” has evolved.
Severe sepsis without septic shock wasn’t
studied in ProCESS.
This trial and general practice supports
early antibiotics and fluids and uncertain re:
CVC for everyone, PRBCs**, dobutamine.
Targeted fluid resuscitation now the goal.

*ARISE, PROMISE trials, NEJM 2014-5 **TRISS trial, NEJM 2014: restrictive strategy

2848
Copyright © Harvard Medical School, 2018. All Rights Reserved.

CMS: (3h: plus


LA, cx, Abx)
(6h: f/u LA)

Surviving Sepsis Guidelines 2016, ICM 2017

From: Will This Hemodynamically Unstable Patient Respond to a Bolus of


Intravenous Fluids?
JAMA. 2016;316(12):1298-1309. doi:10.1001/jama.2016.12310

Figure Legend:

Effect of Increase in Preload on Stroke Volume of Ventricles With Normal and Decreased
Contractility. Frank-Starling curves illustrate that the effect of a given increase in preload
on stroke volume is dependent both on ventricular contractility and on baseline preload.
Copyright © 2016 American Medical
Date of download: 9/26/2017
Association. All rights reserved.

2849
Copyright © Harvard Medical School, 2018. All Rights Reserved.

Starting now at an ICU near you:


CLOVER trial design (PETAL Network)

What ‘flavor’ of fluid should we be


using in sepsis?
NO: Hetastarch
Mostly NO: Albumin
Yes: Crystalloid…moving toward
favoring LR over saline?

NEJM March 1, 2018

2850
Copyright © Harvard Medical School, 2018. All Rights Reserved.

FLUIDS in SEPSIS BOTTOM LINE:

Bolus crystalloid, but don’t overdo it.

Find your favorite way(s) to target


resuscitation.

Case, Question #3
You have administered broad-spectrum antibiotics to
treat presumed spontaneous bacterial peritonitis,
give her supplemental O2 (now saturating 90% on
100% FM), and after fluid resuscitation with 3L of
crystalloid, her HR comes down to 100 bpm, her SBP
has risen to 65 mm Hg with a mean arterial pressure
(MAP) of 50 mm Hg, and the CVP is 13 mm Hg. You
next order:
a. Vasopressin
b. Norepinephrine
c. Norepinephrine + Furosemide
d. 1 Unit of PRBCs
e. Dobutamine

2851
Copyright © Harvard Medical School, 2018. All Rights Reserved.

Case, Question #3
You have administered broad-spectrum antibiotics to
treat presumed spontaneous bacterial peritonitis,
give her supplemental O2 (now saturating 90% on
100% FM), and after fluid resuscitation with 3L of
crystalloid, her HR comes down to 100 bpm, her SBP
has risen to 65 mm Hg with a mean arterial pressure
(MAP) of 50 mm Hg, and the CVP is 13 mm Hg. You
next order:
a. Vasopressin
b. Norepinephrine
c. Norepinephrine + Furosemide
d. 1 Unit of PRBCs
e. Dobutamine

Question #3:

What’s up with pressors?

2852
Copyright © Harvard Medical School, 2018. All Rights Reserved.

Action of Vasoactive Catecholamines


2001- 2008

β1: HR, Contractility


β2: Vasodilation

Vasoconstriction

Chest 2007; 132:1678-1687; NEJM 2010; Lancet 2007* (Epi 2nd-line)

Vasopressin as a ‘Pressor’
Multi-center RCT with
~800 patients
No significant mortality
benefit in adding
vasopressin to
norepinephrine
Less sick people did
better with addition of
vasopressin
Widespread use as an
adjunctive support with
norepinephrine

VASST Trial
Russell et al. NEJM 2008; 358:877-887, Feb 28, 2008.

2853
Copyright © Harvard Medical School, 2018. All Rights Reserved.

New kid on the block:

Multi-center RCT with 344 patients (mostly sepsis; but NOT


low output states)
Add-on to conventional pressors increased MAP to 75
within 3h (69% vs. 23%, p<0.001)
Similar rates of adverse events
Intriguing as a new class of vasopressors

Surviving Sepsis Guidelines 2016, CCM 2017

2854
Copyright © Harvard Medical School, 2018. All Rights Reserved.

Question #2:

What are we supposed to be


doing with steroids in sepsis?

RELATIVE Adrenal Insufficiency


Basic Background
“High-dose” corticosteroids DO NOT improve
sepsis outcomes1

Absolute adrenal insufficiency in sepsis is RARE

What about RELATIVE adrenal insufficiency?


2002-2008: Cort-stim to select ‘nonresponders’
based upon a subgroup analysis

1NEJM 1984 311: 1137; Crit Care Med 1995 23: 1430; JAMA 2002

2855
Copyright © Harvard Medical School, 2018. All Rights Reserved.

Multi-center, double-blinded, placebo RCT

251 vs. 250 patients: Hydrocortisone or


Placebo q6h for 5d, then tapered 6d.

Primary outcome: Death 28 days.

2856
Copyright © Harvard Medical School, 2018. All Rights Reserved.

Low-Dose Steroids in Sepsis, 2018:

3800 patients, RCT 1241 patients, RCT


Less sick patients Activated Protein C
HC continuous infusion removed
No fludrocortisone Sicker patients
No mortality benefit 90d Included fludrocortisone
Faster resolution of shock 90d mortality benefit
Fewer blood transfusions Similar infections though
with steroids (unclear why) more viral with HC/FC
More hyperglycemia
with HC
March 1, 2018

Steroids in Sepsis Bottom Line?


“It is unlikely that in the near future sufficiently powered
trials will provide us with better data. Thus, clinicians
will have to use these data and subsequent meta-
analyses to decide how best to treat patients with
septic shock. Estimating 90-day mortality at the
bedside is not practical. It is likely that some
practitioners caring for a patient with a deteriorating
condition who is receiving escalating doses of
vasopressors, in whom other core interventions have
been instituted (i.e., appropriate antibiotics and
adequate volume resuscitation and source control), will
consider that the short-term benefits of low-dose
hydrocortisone may exceed any risks (e.g.,
antiinflammatory effects) as an added therapy in
selected patients.”
NEJM Editorial 2018

2857
Copyright © Harvard Medical School, 2018. All Rights Reserved.

Surviving Sepsis Guidelines 2016, CCM 2017

Adjunctive Sepsis Care

Tight Glucose Control2


Target glucose ≤180 mg/dl rather than tighter control.
Low Tidal Volume Ventilation in ARDS3
Judicious Fluid Management in Acute Lung Injury4
Protocolized Central Line Insertion and Care and
attention to ICU ‘bundles’ and prophylactic care5,6,7

1NEJM 2008;358:111; 2NEJM 2009;360:1283; 3NEJM 2000;342:1301;


4NEJM 2006;354:2564; 5NEJM 2006;355:2725; 6JAMA 2009;301:1231; 7CCM 2016

2858
Copyright © Harvard Medical School, 2018. All Rights Reserved.

Take-Home Points
Early Identification of Patients with Sepsis*
Source Control and Early Broad-Spectrum
Antibiotics
Early fluid resuscitation, primarily with crystalloid
(“Early Goal Directed Therapy” targeted fluids)
Norepinephrine as First Pressor of Choice
Consider Addition of Vasopressin to
Norepinephrine
Consider Epinephrine as 2nd-line Agent for
Refractory Hypotension to Norepinephrine
Limited role for Dopamine and Neosynephrine
as initial agents of choice; Angiotensin II is new.
*Note New Guidelines Consensus Guidelines, Crit Care Med 2016

Take-Home Points, Cont’d


Activated Protein C is no longer available.
Consider low-dose hydrocortisone for patients with
sepsis-induced refractory hypotension despite fluids and
pressors (without use of ACTH stim test to guide
decision-making) – perhaps in the sicker patients.
Uncertain goal of glucose control, but target ≤180 mg/dL
(instead of tighter control) suggested until more data
available. Hourly monitoring of glucose levels while on
an insulin drip is critical, and avoid hypoglycemia.
Attention to standard ICU care (e.g., ventilator bundle,
DVT and GI prophylaxis, central line care, etc).

CCM Consensus Guidelines, 2016

2859
Copyright © Harvard Medical School, 2018. All Rights Reserved.

Sepsis: Summary
NO rhAPC

To ICU High Risk of


Death Improving

Respiratory Check Cx’s ->


Start LD HC(??)
Support narrow Abx

0h 6h 24h 48h
Shock LOW Risk of Shock
starts Death resolving?

Antibiotics NO
? Debridement “Early Goal
NOT Septic? Source
Directed Rx”
Known?
Fluid End-of-life
Resuscitation No better discussion? Steroids?

Conflict of Interest
No Conflicts
Thanks to Chanu Rhee for allowing me to
steal some of his slides.

2860
Copyright © Harvard Medical School, 2018. All Rights Reserved.

Selected Key References


Rhodes A, et al. Surviving Sepsis Campaign: International Guidelines for
Management of Sepsis and Septic Shock. Crit Care Med 2017; 45:486.
Annane D, et al. Hydrocortisone plus Fludrocortisone for adults with septic
shock. NEJM 2018; 378: 809.
Venkatesh B, et al. Adjunctive glucorticoid therapy in patients with septic
shock. NEJM 2018; 378: 797.
Khanna A, et al. Angiotensin II for the treatment of vasodilatory shock.
NEJM 2017; 377: 419.
Bentzer P, et al. Will this hemodynamically unstable patient respond to a
bolus of intravenous fluids? JAMA 2016; 12: 1298.
Singer M, et al. The 3rd international Consensus Definitions for Sepsis and
Septic Shock (Sepsis-3). JAMA 2016; 315: 801
The ProCESS Investigators. A randomized trial of protocol-based care for
early septic shock. NEJM 2014; 370: 1683.

Suggested Algorithm for PCT Use at BWH

2861
Copyright © Harvard Medical School, 2018. All Rights Reserved.

Suggested Algorithm for PCT Use at BWH, Cont’d

Oxygen Supply vs. Demand

Rivers, E. P. et al. CMAJ 2005;173:1054-1065

2862
Copyright © Harvard Medical School, 2018. All Rights Reserved.

Other “goals” of therapy


That can be monitored?

1818 patients : crystalloid vs.


crystalloid+20% albumin
Primary outcome: 28d mortality
NO DIFFERENCE in 28d mortality or other
secondary outcomes

2863
Copyright © Harvard Medical School, 2018. All Rights Reserved.

776 patients with septic shock: MAP 80-85


vs MAP 65-70 mm Hg
Primary endpoint 28d mortality
NO DIFFERENCE in 28d mortality, 90d
mortality, or adverse events
More new atrial fibrillation in high target group
Chronic HTN: less RRT in high target group

Multi-center, RCT, 1679 patients with shock


(858 patients DA; 821 NE)
Primary endpoint: 28d Mortality

No Significant Difference
(DA: 52.5%, NE: 48.5%)

2864
Copyright © Harvard Medical School, 2018. All Rights Reserved.

Secondary and Subgroup Analyses:

1. DA Increased arrhythmias

2. DA Increased 28d mortality in


cardiogenic shock subgroup

“Vasopressin Deficiency” in Sepsis?

2865
Copyright © Harvard Medical School, 2018. All Rights Reserved.

Norepinephrine vs. Vasopressin

About 400 septic shock subjects


Vaso/steroids vs. vaso vs. NE/steroids vs. NE
No difference in kidney failure free days at 28d
No clear renal protective effect demonstrated

Role for Epinephrine?

330 Patients with Septic Shock:


NE + Dobut (if needed) vs. Epi

Primary Outcome: 28d Mortality

No Difference
Similar secondary outcomes and adverse
events rates, as well

2866
Copyright © Harvard Medical School, 2018. All Rights Reserved.

Relative Adrenal Insufficiency


The ACTH-Stim Test Story
Annane et al. JAMA 2002 288: 862
300 septic patients
149 Patients: Placebo
151 Patients: Hydrocortisone 50 mg IV Q6h +
Fludrocortisone 50 µg PO QD x 7 days
All got:
Baseline, random cortisol levels
250 µg ACTH stimulation test
“Nonresponder”: Increase cortisol < 10 µg/dL
“Responder”: Increase cortisol ≥ 9 µg/dL

Relative Adrenal Insufficiency

+ STEROIDS

+ Relative Adrenal Insufficiency

+ STEROIDS

– Relative Adrenal Insufficiency


60

2867
Copyright © Harvard Medical School, 2018. All Rights Reserved.

Popular Topics for the Boards

Kathleen J. Haley, MD
Associate Physician
Department of Medicine, Division of Pulmonary and Critical Care
Brigham and Women’s Hospital
Assistant Professor of Medicine
Harvard School of Medicine

Disclosures
• None

2868
Copyright © Harvard Medical School, 2018. All Rights Reserved.

Themes
• Medical statistics
• Decisional Capacity
• Iatrogenic problems
• Prolonged critical illness
• Weakness in ICU patients
• Disputes among surrogates/no HCP

Allow Me to Introduce You…


Mark Twain
There are three kinds of lies: lies, d—d lies,
and statistics

Thomas Bayes (professional clergyman and


amateur mathematician) – formulated
Bayes theorem

References for Images:


Mark Twain. The Biography Channel website. 2013.
Available at: http://www.biography.com/people/mark-twain-9512564.
Thomas Bayes: Celebrating Statisticians: Thomas Bayes
Available at: http://www.blogs.sas.com

2869
Copyright © Harvard Medical School, 2018. All Rights Reserved.

Case 1
• You are the attending for a 75yo woman admitted with diarrhea,
fever, tachycardia and leukocytosis
– Her symptoms started 1 week after completing a course of
antibiotics for a UTI

• Fortuitously, Grand Rounds this week reviewed C. difficile


– Approximately 60% antibiotic-associated
diarrhea is caused by C. difficile

• Your hospital has recently changed its laboratory assay for C. difficile
–The new assay has a sensitivity of 70% and a
specificity of 95%

• Your patient’s test comes back negative

Case 1
• What does the negative test mean?
– A. Specificity must be put into the context of the underlying
prevalence. Since both are high, the negative predictive value is
also high – the patient is unlikely to have C. difficile.
– B. A specificity of 95%, means that there is a 95% chance that
the negative test result is a true negative regardless of the
disease prevalence. The negative predictive value is therefore
95%
– C. The Negative predictive value is 68%.
– D. The population prevalence is 60%, so a specificity of 95%
gives a negative predictive value of 57%.
– E. The likelihood ratio of a negative test (LR-) in this assay is 4.9

2870
Copyright © Harvard Medical School, 2018. All Rights Reserved.

Case 1
• What does the negative test mean?
– A. Specificity must be put into the context of the
underlying prevalence. Since both are high, the
negative predictive value is also high – the patient is
unlikely to have D. difficile.
– B. A specificity of 95%, means that there is a 95%
chance that the negative test result is a true negative
regardless of the disease prevalence.
– C. The Negative predictive value is 68%.
– D. The population prevalence is 60%, so a specificity
of 95% gives a negative predictive value of 57%
– E. The likelihood ratio of a negative test (LR-) in this
assay is 4.9

Calculating Predictive Values


• Positive Predictive Value
– Probability of X (usually a disease) in all patients having a
positive test result
– True Positives/(True positives + False positives)
• Negative Predictive Value
– Probability of being without X (usually a disease) in all patients
with negative test result
– True Negatives/(True Negatives + False negatives)
• In our patient:
– Actual incidence of disease = 60%
– Test sensitivity = 70%
– Test specificity = 95% Disease
– For 100 patients: + - Totals
• 38/(38 + 18) = 0.68
Test + 42 2 44
Test - 18 38 56
60 40 100

2871
Copyright © Harvard Medical School, 2018. All Rights Reserved.

Medical Statistics: Definitions


• Sensitivity: Positive test in pts having X
– High sensitivity means few false negatives
– True Positive/(True positive + False negative)
• Specificity: Negative test in pts without X
– High specificity means few false positives
– True Negative/(True negative + False positive)
• Predictive Values: Probability that the test result
predicts the disease (or lack thereof)
• Odds Ratio and Likelihood Ratio
– Tests the test
– Useless tests have OR = 1 and LR = 1

More Definitions
• “Real-World” Example
• Absolute Risk
– Dabigatran vs. warfarin in
– Difference between patients with atrial fibrillation1
proportions of patients with
and without a characteristic • Annual incidence of serious
developing condition X bleeding in patients on 110 mg
dabigatran was 2.71%, vs.
• Number Needed to Treat (or 3.36% in patients on warfarin
Harm)
– Absolute risk reduction with
– 1/Absolute Risk dabigatran was 0.65%
• Relative Risk (Risk Ratio) – Number needed to treat =
– Comparison of the 1/0.0065 = 154
proportion of patients with – Relative Risk of serious
and without a characteristic bleed with dabigatran =
who develop condition X 0.0271/0.0336 = 0.81
• Confidence Intervals – The confidence intervals
– Distance away from 1 were 0.69 – 0.93, favoring
indicates strength of effect dabigatran over warfarin

Reference:
1. S.J. Connolly et al., NEJM 2009, vol. 361, pp. 1139 - 1151

2872
Copyright © Harvard Medical School, 2018. All Rights Reserved.

Another Consideration
• The Best Medical Journal has just published a non-
inferiority study on a new antihypertensive regimen
compared to a commonly used regimen.
– No overall superiority was shown compared to the usual therapy.
– Patients over 70 years old had significant decreases in both
systolic and diastolic BP on the new regimen.
– What is the significance of this?

• This is an example of “Post Hoc” Analysis


– Be careful! If the subgroup wasn’t included in the original
design, it can’t be relied upon for analysis
– Instead, it is “hypothesis generating” and requires further
testing

General Board Point


• There will be several questions
involving medical statistics

2873
Copyright © Harvard Medical School, 2018. All Rights Reserved.

Case 2
• You are the attending physician for a 74 year old woman who was
admitted 3 weeks ago with severe sepsis. Her comorbidities include
vascular dementia with mild functional impairment. Her course has
had multiple complications. Her adult daughter, who is her health
care proxy, her husband state that they feel that the patient would
want to continue full aggressive treatment of her illness.

Case 2
• You are the attending physician for a 74 year old woman who was
admitted 3 weeks ago with severe sepsis. Her comorbidities include
vascular dementia with mild functional impairment. Her course has
had multiple complications. Her adult daughter, who is her health
care proxy, her husband state that they feel that the patient would
want to continue full aggressive treatment of her illness.
• This morning you receive an email from the night nurse, who writes
that, for the past week, the patient has been having episodes of
agitation during which she pulls at her IV lines and tracheostomy
tube. This behavior is new, and the night nurse is convinced that
the patient is trying to refuse medical treatment.

2874
Copyright © Harvard Medical School, 2018. All Rights Reserved.

Case 2
• You are the attending physician for a 74 year old woman who was
admitted 3 weeks ago with severe sepsis. Her comorbidities include
vascular dementia with mild functional impairment. Her course has
had multiple complications. Her adult daughter, who is her health
care proxy, her husband state that they feel that the patient would
want to continue full aggressive treatment of her illness.
• This morning you receive an email from the night nurse, who writes
that, for the past week, the patient has been having episodes of
agitation during which she pulls at her IV lines and tracheostomy
tube. This behavior is new, and the night nurse is convinced that
the patient is trying to refuse medical treatment.
• When you ask the patient about this report, she turns away. When
her daughter visits, you mention the nurse’s concerns. The
daughter tells the patient, “We need you. You want to keep going,
don’t you?” The patient does not nod, but does squeeze her
daughter’s hand in response.

Case 2, continued
• Patient’s current exam
– VS Temp 99.2, HR 105, BP 115/65, RR 24, O2 sat
94% on FiO2 35%
– Chest: Bibasilar rales; Heart: Irreg Irreg; Abd: soft,
+BS; Ext: 1+ pedal edema; Neuro: Awake,
responds appropriately to Y/N questions, follows 1-
step commands; strength 3 - 4/5 bilat upper and
lower ext, MAE

2875
Copyright © Harvard Medical School, 2018. All Rights Reserved.

Case 2, continued
• Patient’s current meds include
– metoprolol
– inhaled albuterol/ipratropium
– ceftriaxone
– famotidine
– dalteparin
– trazodone
– lorazepam

Case 2, continued
• Of the following, the best next course of action
would be:
- A Ethics consult since the HCP is not acting on the
patient’s wishes
- B Assessing the patient’s ability to communicate and
her decision-making capacity
- C Continue aggressive therapy since that is the
guidance from the HCP, and patient’s dementia
precludes her from participating in this decision
- D Evaluating the patient for causes of delirium
- E Consulting psychiatry to evaluate for depression

2876
Copyright © Harvard Medical School, 2018. All Rights Reserved.

Case 2, continued
• Of the following, the best next course of action
would be:
- A Ethics consult since the HCP is not acting on the
patient’s wishes
- B Assessing the patient’s ability to communicate and
her decision-making capacity
- C Continue aggressive therapy since that is the
guidance from the HCP, and patient’s dementia
precludes her from participating in this decision
- D Evaluating the patient for causes of delirium
- E Consulting psychiatry to evaluate for depression

Case 2 - Explanation
• At this point, there is not enough information to evaluate
whether patient’s HCP is or is not acting on the patient’s
previously expressed wishes.
• It is not clear from the evidence presented what is
causing her nocturnal symptoms. The absence of
daytime symptoms do not rule out delirium.
• Ambivalence is common among patient who have a long
and burdensome course of therapy.
• Determining a critically ill patient’s decisional capacity is
difficult and time consuming; psychiatric consultation can
help in this assessment.
– Bedside assessment by Confusion Assessment Method-ICU
(CAM-ICU; Ely et al., Crit Care Med 2001)

2877
Copyright © Harvard Medical School, 2018. All Rights Reserved.

Decisional Capacity – Take Home


Points
• Intertwined with informed consent
• Requires
– Understanding and remembering choices
– Understanding consequences
– Being able to rationally consider choices
• May not arrive at the “right” choice
– Being able to indicate a choice
– A careful bedside assessment can be used
• Sliding scale
• Diagnosis of dementia per se does not rule out
decisional capacity

General Point for the Boards


• Be prepared for ethics and decisional
capacity questions - look for an option that
preserves a patient’s autonomy

2878
Copyright © Harvard Medical School, 2018. All Rights Reserved.

Case 3
• Your patient is a 68yo man with recurrent lung cancer who was
admitted to your ICU 2 days ago.
• He had been in his USOH and tolerating an outpatient
chemotherapy regimen (pembrolizumab and cyclophosphamide)
until 3 weeks PTA
– He had a sick contact (grandson with cold)
– His PCP started levofloxacin for possible pneumonia
• His symptoms include NP cough, fever, dyspnea
• His dyspnea progressed, and he required intubation last evening
– Bronchoscopy with lavage performed, Gram stain shows
neutrophils but no organisms
• His AM exam shows dependent rales, regular HR with a 2/6 systolic
murmur at the mid-axillary line, no JVD, no pedal edema
• Which would be the most reasonable next step?

Case 3
• A. Perform a CT angiogram to exclude PE
• B. Broaden antimicrobial therapy to
include anti-fungal (Aspergillus) coverage
• C. Float a PA line to exclude pulmonary
edema as a contributing factor
• D. Add solumedrol to treat possible drug-
induced pneumonitis
• E. Perform a non-contrast CT to evaluate
for lymphangitic spread of tumor

2879
Copyright © Harvard Medical School, 2018. All Rights Reserved.

Case 3
• A. Perform a CT angiogram to exclude PE
• B. Broaden antimicrobial therapy to
include anti-fungal (Aspergillus) coverage
• C. Float a PA line to exclude pulmonary
edema as a contributing factor
• D. Add solumedrol to treat possible
drug-induced pneumonitis
• E. Perform a non-contrast CT to evaluate
for lymphangitic spread of tumor

Case 3 - Explanation
• While the patient has a malignancy, the
subacute course would be atypical for PE
• The patient has not been severely
neutropenic for a prolonged period, and so
is not at risk for a fungal pneumonia
• The reported exam is not typical
pulmonary edema

2880
Copyright © Harvard Medical School, 2018. All Rights Reserved.

Case 3, Explanation
• Both cyclophosphamide and pembrolizumab have been
associated with drug-induced pneumonitis
• The incidence of pneumonitis with checkpoint inhibitors
is approximately 3 - 5%
– The incidence is higher, up to 10% with checkpoint inhibition
therapies combined with anti-CTLA4 therapy
– Approximately 25% of the patients developing pneumonitis will
have severe pneumonitis (grade 3 or 4)
• Treatment is to stop the offending drug and add steroids

General Points for the Boards


• Use all the clues provided in the question stem
– The set up may be tricky, but the needed information
will be there (somewhere)
• Be on the watch for a choice that is associated with an
iatrogenic problem

2881
Copyright © Harvard Medical School, 2018. All Rights Reserved.

Case 4
• Your patient, an 79 year old woman, passed away this morning. Her family
(spouse, 2 sons, and a daughter, who is her healthcare agent) was at her
bedside at the time of her death.
• She was admitted 12 weeks ago following a motor vehicle collision during
which she sustained a femur fracture. Her fracture was repaired by
Orthopedics with a ORIF. Her course was complicated by DVT on post-op
day 3 and prolonged respiratory failure. After her initial post-op course, she
was transferred to the medical intensive care unit (MICU) for management
of her prolonged respiratory failure. Her course in the MICU has been
significant for atrial fibrillation, ventilator associated pneumonia, urosepsis,
and delirium.
• Based on patient’s long-standing, consistent statements that she would not
want prolonged support by machines, her goals of care were changed to a
comfort-focused approach yesterday.
• You approach her family regarding consent for autopsy.
• The patient’s husband does not want an autopsy, but all of the children,
including her daughter who is the healthcare agent, want an autopsy

What is true regarding consent for autopsy for this patient?

Case 4

- A. The autopsy cannot proceed since a patient’s family


must give unanimous consent.
- B. Since the patient had several iatrogenic
complications during her hospitalization, the case must
be referred to the medical examiner.
- C. Since the patient’s admission was preceded by a
motor vehicle accident, the case must be referred to
the medical examiner.
- D. The autopsy can proceed since the health care
proxy has given permission.
- E. The autopsy cannot proceed since the next of kin
has refused permission.

2882
Copyright © Harvard Medical School, 2018. All Rights Reserved.

Case 4

- A. The autopsy cannot proceed since a patient’s family must give


unanimous consent.
- B. Since the patient had several iatrogenic complications during
her hospitalization, the case must be referred to the medical
examiner.
- C. Since the patient’s admission was preceded by
a motor vehicle accident, the case must be referred
to the medical examiner.
- D. The autopsy can proceed since the health care proxy has given
permission.
- E. The autopsy cannot proceed since the next of kin has refused
permission.

Case 4 - Answer
• The medical examiner must be notified since the
admission was precipitated by trauma.
– State laws for medical examiner notification vary
– State laws differ regarding permission for autopsy
– Some states allow HCP to consent, if explicitly
instructed in advance directive
– Most laws regarding autopsy permission follow
inheritance and property laws

2883
Copyright © Harvard Medical School, 2018. All Rights Reserved.

Autopsy Permission/Roles of HCP


– Take Home Points
• Authority for granting permission for
autopsy varies state by state
– Only a few states grant this to the HCP
• States frequently put constraints on this permission
– Generally the next of kin gives permission
• Medical examiner always has the authority
to perform an autopsy.
• The authority of the HCP is for decisions
regarding medical treatment.

General Point for the Boards


• Be prepared for a couple of questions on
health care proxies, including limits to their
authority, when and how a health care
proxy makes decisions, and who is eligible
to be a health care proxy.

2884
Copyright © Harvard Medical School, 2018. All Rights Reserved.

Case 5

• You are the attending for a 62yo woman who is


recovering from sepsis due to pyelonephritis.
– Her initial presentation was significant for
hypotension requiring pressors and altered mental
status.
• She was intubated for airway protection.
– Unfortunately, she aspirated during intubation, which
led to pneumonia and ARDS.
• Today is her tenth day on the ventilator, and she
still requires substantial support.
– You are meeting today with her family to discuss
possible tracheostomy placement.

Case 5
• What is true about patients who survive ARDS?
– A The use of sedatives with amnestic effects can minimize the
symptoms of later post-traumatic stress disorder.
– B Early enteral feeding is important to maintaining adequate
muscle mass in patients requiring prolonged ventilator
support. Unfortunately, full feeding goals (calories and/or
protein) are frequently not achieved due to GI intolerance.
– C Despite near normalization of their pulmonary function tests,
ARDS survivors report significant decrements in their
emotional and physical functioning for up to 5 years following their
illness.
– D Critical illness associated neuropathy/myopathy is strongly
associated with statin use.
– E With the exception of patients with spinal cord injuries,
patients with trauma-associated ARDS have a better
prognosis than patients with medical causes of ARDS

2885
Copyright © Harvard Medical School, 2018. All Rights Reserved.

Case 5 - Answer
• What is true about patients who survive ARDS?
– A The use of sedatives with amnestic effects can minimize the
symptoms of later post-traumatic stress disorder.
– B Early enteral feeding is important to maintaining adequate
muscle mass in patients requiring prolonged ventilator
support. Unfortunately, full feeding goals (calories and/or
protein) are frequently not achieved due to GI intolerance.
– C Despite near normalization of their pulmonary function tests,
ARDS survivors report significant decrements in their
emotional and physical functioning for up to 5 years following
their illness.
– D Critical illness associated neuropathy/myopathy is strongly
associated with statin use.
– E With the exception of patients with spinal cord injuries,
patients with trauma-associated ARDS have a better
prognosis than patients with medical causes of ARDS

Prolonged Critical Illness


• Prolonged critical illness is associated with significant
morbidity and healthcare costs
• Recent studies of survivors of ARDS show persistent
physical and neuropsychiatric limitations for up to 5
years after discharge from the acute hospital
– Near-normal pulmonary function tests by 1-year
– 6-minute walk test below range expected for age
– Over half of ARDS survivors report significant anxiety and/or
depression up to 5 years after their acute illness
• Predictors of decreased functioning after ARDS
– Age greater than 52
– Critical illness-associated weakness
– Comorbid disease

M. Herridge et al. NEJM 2011; vol. 364 pp.1293 – 1304

2886
Copyright © Harvard Medical School, 2018. All Rights Reserved.

General Point for the Boards


• Be prepared for questions highlighting the
substantial personal and societal costs of
prolonged critical illness

Case 6
• You are the attending for a 56yo
man, h/o DM and asthma, who
was admitted 21 days ago for
pneumonia following H1N1
influenza.
• His initial hospital course included
an emergent traumatic intubation
• Hospital course was significant for
refractory hypoxemia requiring
paralysis (48hr) and ECMO.
• He gradually improved, and was
decannulated 2 days ago.

2887
Copyright © Harvard Medical School, 2018. All Rights Reserved.

This morning…
• On exam, he is afebrile with stable VS.
• Mental status exam shows him to be
awake, and nodding appropriately to Y/N
questions.
• His chest shows basilar crackles, and his
heart is regular and without murmurs.
• On neurologic exam, your patient cannot
lift either his arms or legs, although he can
move his fingers and toes to command.

The least likely cause would be


- A. Medications
- B. Low potassium
- C. Sequelae of traumatic intubation
- D. Undiagnosed premorbid neuromuscular
disease
- E. Critical illness associated weakness

2888
Copyright © Harvard Medical School, 2018. All Rights Reserved.

The least likely cause would be


- A. Medications
- B. Low potassium
- C. Sequelae of traumatic intubation
- D. Undiagnosed premorbid neuromuscular
disease
- E. ICU-Acquired weakness

ICU-Acquired Weakness
(ICU-AW)
• Approximately 75,000 patients annually develop
ICU-AW (Fan et al., AJRCCM, 2014)
• Patients with ICU-AW have 30% higher acute
hospital costs (Hermans et al. AJRCCM 2014)
than ICU patients without weakness
– This doesn’t count costs of rehabilitation,
readmission, post-rehabilitation support
• Patients with ICU-AW have higher mortality
compared to non-weak ICU patients
– Higher post-ICU mortality in first year: 28 vs 11%
– Possible higher in-hospital mortality (inconsistent
study results)

2889
Copyright © Harvard Medical School, 2018. All Rights Reserved.

Weakness Increases Mortality??


• Unknown mechanisms, probably multiple
– Difficulty swallowing
– Poor cough
– Prolonged ventilation
– Poor balance
– Difficult to move in bed so increased
decubitus ulcer incidence
– Possible persistent inflammatory process

ICU-Acquired Weakness
• Can occur very early in course
– Generally seen after 1 wk in ICU
• Common
– Clinically significant findings in up to 2/3 patients requiring
mechanical ventilator for over 1 week
– Depending on study >80% of patients on mechanical ventilation
have evidence
• Persists
– 6 minute walk test only 70% age-predicted maximum in ARDS
survivors 5yrs after acute illness (Herridge et al., NEJM 2003)
• Subtypes
– Myopathy
– Neuropathy
– Combined myopathy/neuropathy

2890
Copyright © Harvard Medical School, 2018. All Rights Reserved.

Risk Factors
• Age
• Sepsis
• Duration of organ failure
• Mechanical Ventilation
• Premorbid functional status
• Female gender (inconsistent)
• Medication (inconsistent)
– Aminoglycosides
– Neuromuscular blockers
– Glucocorticoids

Weakness in ICU patients -


Etiologies
• Primary neuromuscular disease
– Increases likelihood of critical illness
– Critical illness can unmask neuromuscular disease
• New, but separate, event complicating critical illness
– New CVA, embolism due to endocarditis, spinal cord ischemia
• Complication of therapy
– Meds
• Metabolic derrangements
– Hypokalemia
• Due to critical illness
– ICU-Acquired Weakness (ICU-AW)

2891
Copyright © Harvard Medical School, 2018. All Rights Reserved.

Diagnosis - Approach
• Differential can be summarized by MUSCLES
(Maramattom BV and Wijdicks EF, Critical Care Medicine 2006, vol.
34, pp 2835 – 2841)
– M: Meds – steroids, amiodarone, NMB,
aminoglycosides, lasix
– U: Undiagnosed primary neuromuscular disease
– S: Spinal cord problem such as ischemia
– C: Critical illness associated weakness
– L: Loss of muscle such as rhabodmyolysis
– E: Electrolytes: low K, low phos, high Mg
– S: Systemic illness: Hypthyroidism, adrenal
insufficiency, porphyria

General Board Point


• Weakness following critical illness is
common, and has a broad differential.

2892
Copyright © Harvard Medical School, 2018. All Rights Reserved.

Case 7
• You are on service at a Boston teaching hospital
• One of your patients is a 58yo diabetic woman
who was admitted 2wks ago with urosepsis
• Her initial shock has resolved, and she is
tolerating PS 12/5 for vent support
• Her course has been complicated by ATN
She is oliguric, and becoming volume overloaded

Renal is advising starting dialysis

Family Meeting
• You review your patient’s situation
• She has no written healthcare proxy
• Present at the meeting
– Husband (separated)
• She saw a news program on dialysis a year ago and said she’d never want it
– Sister
• Reports a recent conversation where the patient wanted her to be the HCP
• She states that her sister would definitely want dialysis because “she is a
fighter”
– 2 brothers
• No opinion regarding patient’s wishes, but both confirm she was
independent prior to current illness and enjoyed her quality of life
• During the meeting
– Her husband states he should be the HCP
– Her sister presents a living will from 5yr ago
• Specifies no mechanical support if terminal illness

2893
Copyright © Harvard Medical School, 2018. All Rights Reserved.

What is true about decision-making


for this patient?
- A. As the legal next of kin, patient’s husband is her HCP
- B. Her sister should be her health care proxy, since
that reflects the patient’s most recently stated wishes
- C. The patient’s advance directive should be used to
guide decision-making
- D. A legal guardian may be required for this patient
- E. Since there is no HCP, decisions require a consensus
among her family members

What is true about decision-making


for this patient? - Answer
- A. As the legal next of kin, patient’s husband is her HCP
- B. Her sister should be her health care proxy, since
that reflects the patient’s most recently stated wishes
- C. Her advance directive should be used to guide
decision-making
- D. A legal guardian may be required for this patient
- E. Since there is no HCP, decisions require a consensus
among her family members

2894
Copyright © Harvard Medical School, 2018. All Rights Reserved.

Explanation
• The Heath Care Proxy (HCP) is a legally designated role
– Paperwork requirement varies by state
• The default decision maker for patients lacking a HCP is determined
by state law
• Advance directives can be useful guides as to the patient’s wishes, if
– The patient anticipates the clinical situation at hand
– Can be more useful if general guidance provided
• “Five Wishes”, “Thinking Ahead”
– Some states allow durable orders for resuscitation preferences
• POLST/MOLST/LaPOST
• Even if there is no HCP, and no state-determined hierarchy of
surrogate decision makers, a consensus decision among people
who care for the patient provides a good basis for most clinical
decisions.
– Sometimes, a legal guardian is needed if no consensus

Healthcare Proxy – Take Home


Points
• A Health Care Proxy (HCP) has the same authority as a
decisionally capable patient
– Treatment decisions, not necessarily research (varies by state)
• Healthcare proxy uses “substituted judgment” as a basis
for decisions
– Most often uses “known or probable” wishes
– Other standards: “best interests”, “clear and convincing
evidence”
• State-specific guidelines if no written proxy
– Most states use “next of kin” as default proxy
– 7 states do not have defined hierarchy
• Authority applies only to healthcare matters
– Significant limits in some states
– Sometimes can be specifically directed to consent to autopsy

Adapted from C. Sabatino and E. Wood, “Decision-Making and Advance Directives –


Nuts & Bolts, National Aging and Law Conference, December 2010

2895
Copyright © Harvard Medical School, 2018. All Rights Reserved.

General Point for Boards


• Be prepared for questions regarding end
of life issues, ethics, surrogate decision
making
– Since state law guides these areas, look for
general principles
– Especially autonomy

Summary
• Statistical analyses are an integral part of evaluating medical data.
• The health care proxy (HCP) uses substituted judgment to make
decisions regarding medical therapy for an incapacitated patient. The
role of HCP ceases with the patient’s death. If there is no HCP
surrogate decision making is guided by state law.
• Decisional capacity is a continuum. Bedside testing can be helpful.
Impaired cognition does not always preclude decisional capacity.

• Iatrogenic injury is a favorite target for the boards – be on the lookout


for drug-associated syndromes, such as drug-induced pneumonitis.

• Survivors of prolonged critical illness experience many physical and


neuropsychiatric sequelae for several years after their acute illness.
• Weakness following critical illness is common, and can be severe.
It is an independent predictor of poor outcome.

2896
Copyright © Harvard Medical School, 2018. All Rights Reserved.

Disclosures
• None

References
• Medical Statistics
– J. A. Knottnerus et al. British Medical Journal 2002,
vol. 324, pp. 477 – 480.
• Advance Directives and Medical Decision
Making
– R.S. Olick, Chest 2012, vol. 141, pp. 232 – 238.
• Checkpoint-Associated Pneumonitis
– J Naidoo et al. J Clin Oncol 2017, vol. 35, pp709-717.
• Prolonged Critical Illness
– M. S. Herridge et al. NEJM 2011, vol. 364, pp.1293 –
1304.

2897
Copyright © Harvard Medical School, 2018. All Rights Reserved.

GOOD LUCK WITH YOUR


BOARDS !!

2898
Copyright © Harvard Medical School, 2018. All Rights Reserved.

Mechanical Ventilation
Basic to Advanced Concepts

Kathleen J. Haley, M.D.


Associate Physician
Department of Medicine, Division of Pulmonary and Critical Care
Brigham and Women’s Hospital
Assistant Professor of Medicine
Harvard School of Medicine

Disclosures

• None

2899
Copyright © Harvard Medical School, 2018. All Rights Reserved.

Outline
• Indications
• Mechanical Ventilation Modes and Variables
• Lung Mechanics
• Complications of Mechanical Ventilation
• Liberation from Mechanical Ventilation
• Non-invasive Ventilation

Historical Perspective

2900
Copyright © Harvard Medical School, 2018. All Rights Reserved.

Indications for Mechanical Ventilation


• Hypoxemia
– Inability to achieve
adequate oxygenation
• Hypercapnea
– Inability to maintain
adequate alveolar
ventilation
• Reduce Work of Breathing
• Airway protection
– Intact laryngeal reflexes
are better

Goals of Mechanical Ventilation

• Maintain adequate oxygenation (pO2)


• Guarantee adequate alveolar ventilation (pCO2)
• Avoid harm
• Optimize patient comfort
• Extubate as soon as possible!!

2901
Copyright © Harvard Medical School, 2018. All Rights Reserved.

Important Variables
• Pressure
Compliance
(∆V/∆P)
• Volume Resistance • Triggering
(∆P/∆F)

• Flow • Cycling

• Time

Triggering
• Triggering – what causes the
ventilator to begin the
inspiratory phase
– Time – vent cycles at a
frequency determined by the
control rate
– Pressure – the vent senses
the pt’s inspiratory effort by
way of a decrease in the
baseline pressure
– Flow – vent senses the pt’s
inspiratory effort by way of a
deflection in the continuous
biased flow

2902
Copyright © Harvard Medical School, 2018. All Rights Reserved.

Cycling
• Cycling is how the ventilator switches from the
inspiratory phase to the expiratory phase.

– Time – such as in pressure control ventilation


– Flow – such as in pressure support ventilation
– Volume – such as in volume control ventilation

Modes of Ventilation:
Too many choices!!!!!!
AutoFlow

Auto Mode

VS SC

PPS
PCV
ECMO

2903
Copyright © Harvard Medical School, 2018. All Rights Reserved.

Commonly Used
Modes of Mechanical Ventilation
• Assist Control (AC)
– Can set either volume or pressure as the independent variable
– Respiratory drive not needed
– Patient-initiated breaths receive full support (volume or inspiratory pressure)
– When AC used without other description, refers to AC/volume-controlled
• Pressure Controlled (PCV) – usually done with AC mode
– Set both the inspiratory pressure and inspiratory time
– Allows control of the inspiratory:expiratory ratio
• Pressure Support Ventilation (PSV)
– Patient must have a reliable respiratory drive (spontaneous mode)
• Intermittent Mandatory Ventilation (IMV)

What Does AC Look Like?

2904
Copyright © Harvard Medical School, 2018. All Rights Reserved.

What Does IMV Look Like?

What Does PSV Look Like?

2905
Copyright © Harvard Medical School, 2018. All Rights Reserved.

Understanding
Modes of Mechanical Ventilation

Mode Name What is set? Initiation


(triggering)
Assist Control (AC) Volume Patient or timer

IMV Volume Patient or timer

Pressure Control (PCV) Pressure Patient or timer

Pressure Support (PSV) Pressure Patient

Understanding
Modes of Mechanical Ventilation

Mode Name What is set? Initiation Termination


(cycling)
AC/VC Volume Patient or timer Volume

IMV/VC Volume Patient or timer Volume

PCV Pressure Patient or timer Timer

PSV Pressure Patient Flow Rate

2906
Copyright © Harvard Medical School, 2018. All Rights Reserved.

Understanding Mechanical Ventilation


Other parameters to set:

Mode FiO2 Insp Flow Rate PEEP Inspiratory Time

AC Yes Yes Yes No (indirect)

IMV Yes Yes Yes No (indirect)

PCV Yes No Yes Yes

PSV Yes No Yes No

Assist Control/Volume Control


Ventilation
• Most commonly used mode of ventilation
• Pros:
– All breaths assisted
– Precise control of tidal volume and minute ventilation
• Important role in ventilation of unstable patients
• Con:
– Airway pressure is not controlled during inspiration
– Not a weaning mode

2907
Copyright © Harvard Medical School, 2018. All Rights Reserved.

Pressure Support Ventilation (PSV)

• Pro:
– Patient control of respiration
– Very comfortable for patients
– Excellent weaning mode of ventilation
• Cons:
– No guarantee of minute ventilation
– No backup respiratory rate
• Risk of apnea

Pressure Control Ventilation (PCV)

• Pro:
– Control of pressure during inspiratory phase of respiratory
cycle
– Easy to set ratio of inspiration and expiration
– Useful in cases of severe hypoxemia (inverse ratio
ventilation)
– High flows can be more comfortable in fibrotic lung
processes
• Cons:
– No guarantee of minute ventilation

2908
Copyright © Harvard Medical School, 2018. All Rights Reserved.

Lung mechanics and why we care

Lung Mechanics

Note: distending pressure is the total


pressure of the entire respiratory system.

2909
Copyright © Harvard Medical School, 2018. All Rights Reserved.

The ventilator as a diagnostic tool:


Clinical case

• 25 y.o. male with a narcotic overdose.


Intubated in emergency room for airway
protection and hypercarbia.
• Initial Vent Settings
AC, Rate=10, TV=500ml, PEEP=5, Flow=
60L/min, FiO2=1.0
• PIP=20 cm H20, Plateau=15 cm H20
• Determine compliance and resistance:

Compliance=
∆volume/∆pressure=
Tidal volume/plateau-PEEP
500/(15-5)=50
Normal: 50-100 ml/cm H20

2910
Copyright © Harvard Medical School, 2018. All Rights Reserved.

Resistance=
∆pressure/flow=
(PIP – plateau)/flow
(20 – 15)/1 = 5 cmH2O/L/sec
Normal: 5 -12 cmH2O/L/sec

Respiratory Mechanics
Elevated Peak Inspiratory Pressure

Resistance Compliance
Problem Problem

Problem with obstruction Problem with the


of the flow of gas into the stiffness of the lung
lung related to the tubes
or airway

2911
Copyright © Harvard Medical School, 2018. All Rights Reserved.

Elevated Peak Airway Pressures

• Increased resistance • Decreased compliance

Water/kink of ventilator tubing Pneumonia


Obstructed ETT/secretions ARDS
Asthma/bronchospasm Severe CHF
COPD Effusion
Pneumothorax
Right mainstem intubation

Complications of Mechanical Ventilation

• Ventilator-induced lung injury


• Ventilator associated pneumonia
• Oxygen toxicity
• Decreased cardiac preload
• Gastric stress ulceration

2912
Copyright © Harvard Medical School, 2018. All Rights Reserved.

Ventilator Strategies May Contribute to Lung Injury

Slutsky AS, Ranieri


VM. N Engl J Med
2013;369:2126-2136.

Complications of Mechanical Ventilation

(a) Capillary stress fracture with incipient (b) Higher power view of stress fracture
extravasation of erythrocyte. showing exposure of collagen filaments.

Crit Care. 2003 Dec;7(6):435-44. Epub 2003 Oct 17.

2913
Copyright © Harvard Medical School, 2018. All Rights Reserved.

Complications of mechanical ventilation:


(Tension) Pneumothorax

Strategies to Avoid Barotauma and Cyclic Recruitment:


Lung-Protective Ventilation

AACN Clin Issues. 2001 May;12(2):234-46.

2914
Copyright © Harvard Medical School, 2018. All Rights Reserved.

• Small tidal volume (6ml/kg) versus large tidal volume


(12ml/ kg)
• Inclusion criteria
– Mechanical ventilation
– paO2/FiO2 < 300
– Bilateral pulmonary infiltrates
– No evidence of LA hypertension
• PCWP < 18mm Hg

Main Outcome
Variables

The Acute Respiratory Distress Syndrome Network. N Engl J Med 2000;342:1301-1308

2915
Copyright © Harvard Medical School, 2018. All Rights Reserved.

Lung Protective Ventilation

• Low tidal volume


– 6 mL/kg
– Maximum plateau pressure
<30 cm H20
– FiO2 and PEEP adjusted to
keep PaO2 55-80 mm Hg

• PEEP
– Optimal PEEP remains
unclear
– Adjust to optimize
compliance (“best PEEP”)
– Use PEEP to get adequate
oxygenation

Auto (“intrinsic”)PEEP
Pressure will build in the chest if there is not adequate expiratory time to empty
each tidal volume

2916
Copyright © Harvard Medical School, 2018. All Rights Reserved.

Intrinsic or “Auto” PEEP

• Usually only clinically relevant in those with


obstructive lung disease (e.g. COPD)
• Can cause:
– Ventilator dyssynchrony
– Increased work of breathing
– Barotrauma
– Hypotension

How to treat auto-PEEP


• Decrease minute ventilation
– Decrease TV
– Decrease RR
• Increase expiratory time
– Increase inspiratory flow rate
• Increase extrinsic PEEP to facilitate synchrony
• Decrease airway resistance
– Optimize inhaled bronchodilators

2917
Copyright © Harvard Medical School, 2018. All Rights Reserved.

Prevention of
Ventilator-Associated Events, Including Infectious
Ventilator-Associated Events
• Recommended • Not Recommended
– Noninvasive ventilation – Selective gut
– Orotracheal intubation decontamination
– Ventilator circuit change for new – Early tracheostomy
patient or when soiled
– Hand washing • Controversial
– Closed endotracheal suction sustem – Chlorhexidine as the oral
– Continuous aspiration of subglottic cleaning agent
secretions
– Minimize sedation
– Oral hygiene
– Elevation of the head of bed
– Extubation as soon as feasible

The exit strategy


“Weaning” is rarely necessary
• Assess patient readiness
for extubation every day
– Hemodynamically stable
– Able to protect airway
– FiO2 < 60%
– PEEP < or = 7.5 cm H2O

• Spontaneous breathing
trial paired with “sedation
vacation”

AJRCCM 1994; 150:896

2918
Copyright © Harvard Medical School, 2018. All Rights Reserved.

Spontaneous awakening trial (SAT) paired with spontanous breathing trial (SBT)

Girard Lancet 2009

Non-invasive mechanical ventilation

Indications Contraindications
• Heart failure • Copious Secretions
• Hypercapnic respiratory • Altered mental status
failure (COPD • Need for secure airway
exacerbation)

2919
Copyright © Harvard Medical School, 2018. All Rights Reserved.

Summary
• Indications: guarantee adequate
– Ventilation, oxygenation, airway protection
• Goals: guarantee adequate
– Ventilation, oxygenation, comfort, avoid harm
• Variables
– Pressure, volume, flow, time. Triggering/cycling
• Mechanics
– Compliance and Resistance
• Complications
– Baro- and Volu- trauma, IVAC
• Lung protective strategy
– Tidal volume 6 ml/kg, plateau pressure < or = 30 mmHg, “optimal”
PEEP
• The exit strategy: the sooner, the better

Case 1.

25 y.o. gentleman with testicular cancer is admitted to the ICU


immediately after orchiectomy. Patient has no prior pulmonary
history.
Initial ventilator settings are FiO2=0.30, Mode = AC/VC, Rate =4,
TV=350, PEEP=5. Patient’s observed respiratory rate is 4.
Initial ABG reveals pH=7.24, pCO2=60, pO2=104

Which of the following ventilator changes will best correct the


respiratory acidosis?

a) Change mode to intermittent mandatory ventilation (IMV)


b) Change mode to pressure support ventilation (PSV)
c) Increase the respiratory rate
d) Decrease the FiO2

2920
Copyright © Harvard Medical School, 2018. All Rights Reserved.

Case 2.

60 y.o. woman (lifetime nonsmoker) is admitted to the ICU with


hypoxemic respiratory failure secondary to legionella pneumonia.
Patient has no prior pulmonary history.
Initial ventilator settings are FiO2=0.50, Mode = AC/VC, Rate =12,
TV=400, PEEP=5. Patient’s observed respiratory rate is 16.
Initial ABG reveals pH=7.38, pCO2=42, pO2=50

Which of the following ventilator changes will not correct the


hypoxemia?

a) Change mode to intermittent mandatory ventilation (IMV)


b) Increase the PEEP
c) Increase the FiO2
d) Change mode to PCV and prolong the inspiratory time

Disclosures
• None

2921
Copyright © Harvard Medical School, 2018. All Rights Reserved.

References

• Pham T et al. Mayo Clinic Proc. 2017; 92(9): 1382:1400.


• Carney D, DiRocco J, Nieman G. Crit Care Med. 2005;33(3
Suppl):S122-8.Med 2000;342:1301-1308.
• Habashi NM. Crit Care Med. 2005;33(3 Suppl)S228-40.
• Slutsky, AS. Chest 1999;116:9-15.
• Kompas, M. Am J Respir Crit Care Med 2015; vol 192: 1420-
1430

Thank you

2922
Copyright © Harvard Medical School, 2018. All Rights Reserved.

Highlights of the Pain, Agitation,


Delirium,
Immobility, and Sleep (PAD-IS)
Guidelines 2018

Gerald L. Weinhouse, MD
Associate Physician
Pulmonary and Critical Care Division, Dept of Medicine
Brigham and Women’s Hospital
Assistant Professor of Medicine
Harvard Medical School

Disclosures

• No financial disclosures

• I was a member of the PAD-IS committee

2923
Copyright © Harvard Medical School, 2018. All Rights Reserved.

Why organize a talk around these


guidelines?
• Extensively analyzed review of relevant literature
with an aim to guide clinical practice.

• Addresses 5 core clinical issues relevant to most


ICU patients and focused on outcomes

• International group of experts

• Patients included as collaborators and co-authors

Pain
• Pain is complex and influenced by psychological
and demographic variables

• Severe pain is associated with negative outcomes

• Implementation of assessment-driven
standardized pain management improves
outcomes

• Gold standard: self-report

2924
Copyright © Harvard Medical School, 2018. All Rights Reserved.

Pain
• Among those able to self-report, the Numeric
Rating Scale (0-10) is the best and is valid and
feasible

Pain
• Among those
unable to self-
report and in
whom behavior
is observable,
the Behavioral
Pain Scale(BPS)
in intubated and
non-intubated
(BPS-NI) and
Critical Care Pain
Observation Tool
are most reliable
and valid.

2925
Copyright © Harvard Medical School, 2018. All Rights Reserved.

Pain
• Proxy report:
– Family can be involved and their assessment is
closer to the patients’ then nurses’ and physicians’
but agreement between patient and family is still
only moderate.

• Physiologic measures:
– Vital signs are not valid indicators; can only be
used as cues to begin further assessment.

Pain—Pharmacologic management
• Opioids are the mainstay; however…

– Adjuncts to decrease pain and opioid


consumption, ie acetaminophen, low-dose
ketamine, neuropathic pain medications.

– Not for routine use:


• IV Lidocaine
• NSAID’s

2926
Copyright © Harvard Medical School, 2018. All Rights Reserved.

Pain—non-pharmacologic management

• Supported by the literature:


– Massage
– Music
• Not well-supported:
– Hypnosis
– Cybertherapy (virtual reality)

• Protocol vs no protocol (usual care)

Agitation/Sedation

• Light vs deep sedation in mechanically


ventilated critically ill patients

2927
Copyright © Harvard Medical School, 2018. All Rights Reserved.

Critical Care Medicine 2018;46:850-859.

Agitation/Sedation
• How do we coordinate sedation use with the
need to expeditiously liberate patients from
mechanical ventilation?

2928
Copyright © Harvard Medical School, 2018. All Rights Reserved.

Agitation/Sedation

Daily sedative interruption = nurse-driven protocolized targeted sedation

Agitation/Sedation:

Pharmacology Monitoring
• Bispectral index (BIS) for
• Propofol or dexmedetomidine deep sedation or
versus benzodiazepines neuromuscular blockade

Other
• Restraints: case by case
• Propofol versus risk/benefit assessment
Dexmedetomidine

2929
Copyright © Harvard Medical School, 2018. All Rights Reserved.

Delirium
Modifiable risk factors Non-Modifiable risk factors

• Benzodiazepine use • Age


• Blood transfusions • Dementia
• Prior coma
• Pre-ICU emergency or
trauma
• High APACHE score

Delirium
• A syndrome of acute (develops in hours to
days) alteration in consciousness accompanied
by change in cognition or perceptual
disturbance which fluctuates over time.

• Independently associated with outcomes in


the critically ill.

2930
Copyright © Harvard Medical School, 2018. All Rights Reserved.

Delirium
• Regular assessment with a validated
assessment tool is recommended

Delirium
Has not been consistently
Is associated with: associated with:

• Cognitive impairment at 3 • PTSD


and 12 months post- • ICU length of stay
discharge • Depression
• Functional dependence
• Mortality

Post intensive care syndrome (PICS): new or worsening impairments in


physical,cognitive, or mental health status arising after critical illness and
persisting beyond acute care hospitalization.

2931
Copyright © Harvard Medical School, 2018. All Rights Reserved.

Delirium--Pharmacology
• Prevention?
– Prophylactic use of medication to
prevent delirium is not supported by
the existing literature. It remains
somewhat controversial.
[haloperidol, atypical antipsychotic,
HMG-CoA reductase inhibitor
(statin), ketamine]

• Treatment?
– No pharmacologic treatment has
consistently demonstrated efficacy to
treat delirium or subsyndromal
delirium. For the mechanically
ventilated patient where agitation is
interfering with weaning/extubation
dexmedetomidine may be the best
sedative.

Delirium—Treatment

Multi-component, non-pharmacologic intervention


focused on risk reduction (i.e. prevention)

• Improve sleep
• Improve wakefulness (reduce sedation)
• Reduce immobility
• Reduce visual/hearing impairment

2932
Copyright © Harvard Medical School, 2018. All Rights Reserved.

Immobility
(Rehabilitation/Mobilization)

• Highlighted in the 2013 PAD guidelines as


beneficial as part of a delirium management
strategy

• A set of interventions designed to optimize


functioning and reduce disability.

• Rehabilitation or mobilization is recommended


for critically ill adults.

Immobility
• Ok to begin Ok if:
rehab/mobilization if HR 60-130/min
stability even if stability SBP 90-180 mmHg
is achieved with DBP 60-100 mmHg
vasoactive infusions or RR 5-40/min
mechanical ventilation. SpO2 > or = 88%
FiO2 < 0.6 and PEEP < 10 mmHg
• Rehab can be done in or Airway is secured
out of bed

2933
Copyright © Harvard Medical School, 2018. All Rights Reserved.

Sleep

• Why is sleep now part of the PAD-IS


guidelines?

– Sleep is considered a potentially modifiable risk


factor influencing recovery in critically ill adults.

– Whether it is a primary cause of some poor


outcomes or represents collateral damage from
brain failure in critical illness can be debated.

2934
Copyright © Harvard Medical School, 2018. All Rights Reserved.

Sleep and ICU outcomes

• Adversely affects ICU quality of life

• ? Delirium
– Sleep interventions lessen delirium occurrence

Flannery et al. Crit Care Med 2016

Sleep
• Should we/could we
monitor sleep in
critically ill adults?

But it is still worthwhile to


ask patients or caregivers
how they slept.

2935
Copyright © Harvard Medical School, 2018. All Rights Reserved.

Sleep
Non-pharmacologic
intervention

Mechanical ventilation

Aromatherapy,
acupressure, music

Noise and light reduction

Sleep
Pharmacologic interventions for
the sole purpose of improving
sleep.

Melatonin:
Propofol:
Dexmedetomidine:

2936
Copyright © Harvard Medical School, 2018. All Rights Reserved.

What about…
• Benzodiazepines and benzo-receptor agonists
– Not well studied for this indication, too many problems,
known negative effects on sleep architecture

• Antidepressants
– No data

• Antipsychotics
– No data

• Antihistamines
– Please don’t

Sleep
• Should a sleep-promoting protocol be used to improve
sleep in critically ill adults?

4 studies:
One RCT, 3 observational

Most were combinations of non-


pharmacologic strategies

Pooled analysis of the observational studies


showed reduction in delirium.

2937
Copyright © Harvard Medical School, 2018. All Rights Reserved.

ABCDEF
The ABCDEF bundle is the evidence
based framework created as a tool to
implement the PAD guidelines.

• A Assess, prevent, manage • Pain


pain
• B SAT coupled with SBT
• C Choice of analgesia, • Agitation
sedation
• D Delirium assessment, • Delirium
prevention, management
• E Early mobility and exercise • Immobility
• F Family engagement and
empowerment
• [? G Get sleep]

Take Home Messages

1. Pain is assessed regularly with validated tools and managed with


opioids along with adjuncts such as acetaminophen to decrease
opioid use
2. Agitation is managed with light sedation either with daily
interruption or nurse-driven protocol
3. Delirium is regularly assessed but there is no generally approved
pharmacologic therapy. Risk reduction/prevention is best
4. Immobility is treated with rehab/mobilization as long as the patient
is stable even if stability is maintained with mechanical ventilation
or vasopressor support
5. Sleep is best achieved by non-pharmacologic means such as noise
and light reduction via a sleep-promoting protocol

2938
Copyright © Harvard Medical School, 2018. All Rights Reserved.

Question #1
• A 65 yo man is admitted to the ICU in respiratory failure from
pneumococcal pneumonia. He is mechanically ventilated but agitated and
dyssynchronous with the mechanical ventilator. Which of the following is
not recommended:

• 1. Adjust the mechanical ventilator so he is more synchronous; maybe he


will be less agitated

• 2. Begin a continuous infusion of Propofol titrated as high as necessary to


make him stop moving so he will synchronize with the ventilator

• 3. Use opioids if he is assessed to be in pain

• 4. Assess him for delirium.

Question #2
• The patient in Question #1 was stabilized and 6 days after admission was
thought possibly to be ready to extubate. Which of the following is true?

• 1. His mechanical ventilatory support should be slowly weaned while he


remains on his sedation

• 2. He should be placed on an infusion of dexmedetomidine to help him


sleep at night so he will be more awake in the morning to give him the
best chance to extubate

• 3. Sedation should be held and he should have a “spontaneous breathing


trial” to see if he is ready for extubation

• 4. He cannot be extubated if he is delirious

2939
Copyright © Harvard Medical School, 2018. All Rights Reserved.

Acute Heart Failure and Cardiogenic Shock

Akshay S. Desai MD, MPH
Advanced Heart Disease Section
Cardiovascular Division
Brigham and Women’s Hospital 
Boston, MA

Disclosures: Dr. Desai has been a paid consultant to St. Jude Medical, Inc.

Disclosures

• Dr. Desai has received honoraria for 
consulting from Novartis, AstraZeneca, 
Abbott, Relypsa, Signature Medical, and 
DalCor Pharma

• Dr. Desai has received research grants from 
Novartis

2940
Copyright © Harvard Medical School, 2018. All Rights Reserved.

Heart Failure is a Clinical Diagnosis

Congestion at Rest
No Yes Signs/symptoms
of congestion
No Warm & Dry Warm & Wet  Orthopnea/PND
Low  JVD
Perfusion  Ascites
at Rest  Edema
 Rales (rare in HF)
Yes Cold & Dry Cold & Wet

Possible evidence of low perfusion


 Narrow pulse pressure  Cool extremities
 Sleepy/obtunded  Hypotension with ACE inhibitor
 Low serum sodium  Renal dysfunction (one cause)
Stevenson LW. Eur J Heart Fail. 1999;1:251

No Difference in Outcomes between Care guided 
by PA Catheter and Clinical Assessment

Hemodynamic Optimization with PAC guidance was safe and associated with 
improvements in quality of life and reductions in MR, but had no impact on mortality

Routine Use of Invasive Hemodynamic Heart Monitoring Not Recommended

ESCAPE Investigators. JAMA 2005; 294: 1625

2941
Copyright © Harvard Medical School, 2018. All Rights Reserved.

When to consider PA Catheter? 

• Uncertain fluid status, perfusion, systemic or pulmonary vascular 
resistance
• Clinically significant hypotension or worsening renal function with 
empiric therapy
• Evaluation of candidacy for advanced therapies such as VAD or 
transplant
• Presumed cardiogenic shock
• Severe clinical decompensation with uncertain relative 
contributions from elevated filling pressures, hypoperfusion,  
abnormal vascular tone
• Apparent inotrope dependence or symptoms that persist despite 
adjustment of recommended therapies

Hemodynamic Profiles in Acute HF

Congestion at Rest
No Yes
Warm & Dry Warm & Wet
No PCWP normal PCWP elevated
Low CI normal CI normal
Perfusion (compensated)
at Rest Cold & Dry Cold & Wet
Yes PCWP low/normal PCWP elevated
CI decreased CI decreased

Stevenson LW. Eur J Heart Fail. 1999;1:251

2942
Copyright © Harvard Medical School, 2018. All Rights Reserved.

Hemodynamic Profiles in Acute HF

Congestion at Rest
No Yes
Warm & Dry Warm & Wet
No PCWP normal PCWP elevated
Low CI normal CI normal
Perfusion (compensated)
at Rest Cold & Dry Cold & Wet
Yes PCWP low/normal PCWP elevated
CI decreased CI decreased

Stevenson LW. Eur J Heart Fail. 1999;1:251

Impact of Vasoactive 
Medications on PV Loops

Normal PV Loop  Vasoconstrictor/Vasodilator Positive/Negative Inotrope

SV

2943
Copyright © Harvard Medical School, 2018. All Rights Reserved.

Intravenous Vasodilator Drugs for Heart


Failure

Half‐Life  Cardiac  Systemic  Pulmonary  Central  Mean 


(mins) Output  Vascular  Vascular  Venous  Arterial 
(CO) Resistance  Resistance  Pressure Pressure 
(SVR) (PVR) (CVP) (MAP)

Nitroglycerine 3 ↑ ↓ ↓ ↓↓↓ ↓
Nitroprusside 1‐2 * ↑↑↑ ↓↓↓ ↓↓ ↓ ↓↓↓

Nesiritide 18 ↑↑ ↓↓ ↓↓ ↓↓ ↓↓

2013 ACC/AHA/HFSA HF Guidelines (IIb, A):  Parenteral Vasodilators May be 
considered an adjuvant to diuretic therapy for stable patients with heart failure

Intravenous Intropic Drugs

Drug α1 β1 β2 HR CO SVR PVR


Dobutamine (2‐10 µg/kg/min)  + +++ ++ ↔↑ ↑↑ ↓ ↔

Dopamine

1‐2 µg/kg/min — — — ↔ ↔ ↔ ↔

2‐10 µg/kg/min + ++ — ↑ ↑↑ ↔↑ ↔

10‐20 µg/kg/min +++ ++ — ↑↑ ↔↑ ↑↑ ↔

Milrinone (0.1‐0.75 µg/kg/min) PDE 3 inhibitor ↑ ↑↑ ↓↓ ↓

Levosimendan Calcium Sensitizer ↑ ↑↑ ↓↓ ↓

2944
Copyright © Harvard Medical School, 2018. All Rights Reserved.

Mortality at 6 months by IV 
medication use

0.8
Survival

0.6
Mortality with Inotropes increased
relative to vasodilators in acute HF
0.4
Use only as bridge to definitive
Inotrope therapy or for palliation in medically
0.2
Vasodilators refractory patients
Neither
0
0 30 60 90 120 150 180

Days

Elkayam U, et al. Am Heart J 2007; 153: 98-104

IV Milrinone In HF Patients: OPTIME‐CHF

N=951 pts, milrinone @ 0.5 µg/kg/min vs. placebo, 60 d F/U

12
*
% Patients During Hospitalization

10

8
Placebo
6
* Milrinone
4 †

0
MI A Fib VT/VF  BP Death

Cuffe et al. JAMA 2002;287:1541-7.

2945
Copyright © Harvard Medical School, 2018. All Rights Reserved.

ACC/AHA/HFSA 2013 Guidelines

Yancy C, et al. Circulation 2013; 128: e240

Cardiogenic Shock:
Definition
Clinically, a syndrome of diminished cardiac
output and vital organ hypoperfusion in the
face of adequate intracardiac filling pressures
Operationally,
Marked and persistent  SBP < 80‐90 mm Hg
hypotension (> 30 mins) 
Reduced Cardiac Index  CI < 2.0‐2.2 L/min/m2

Normal or Elevated Cardiac  PCWP > 18 mm Hg
Filling Pressure

Forrester JS, et al. N Engl J Med 1976; 295:1404-1


Hollenberg, et al. Ann Int Med 1999; 131:47-99

2946
Copyright © Harvard Medical School, 2018. All Rights Reserved.

Cardiogenic Shock: Etiology

• Acute MI
– Post‐MI complications
• Chronic heart failure
• Myocarditis
• Tako‐tsubo cardiomyopathy
• Hypertrophic cardiomyopathy
• Acute valvular regurgitation
• Aortic dissection
• Tamponade
• Pulmonary embolism
Circulation 2008;117:686-97

Pathophysiology of
Cardiogenic Shock
Myocardial infarction

Myocardial dysfunction
Systemic
Inflammatory Systolic Diastolic
response
syndrome
(IL-6, TNF-, NO) LVEDP
Cardiac output Pulmonary congestion
Stroke volume

Systemic Hypotension
perfusion

Coronary Perfusion
pressure
Hypoxemia
Ischemia

Compensatory Progressive
vasoconstriction myocardial
dysfunction

Reynolds HR, Hochman JS Circulation 2008; 117: 686-97. DEATH

2947
Copyright © Harvard Medical School, 2018. All Rights Reserved.

Question 1
A 60 year‐old man presents to the EW with 2 hours of crushing substernal chest pain radiating 
to his left arm, nausea, and diaphoresis. 
On examination, his BP is 82/60 mm Hg, heart rate is 110 bpm, and oxygen saturation is 95% on 
4L of oxygen. He is in severe respiratory distress and has marked jugular venous distension, 
bilateral rales, an S3 gallop, and cold clammy extremities. 
Electrocardiogram reveals sinus tachycardia with ST elevation in the anterolateral leads and ST 
depression in the inferior leads. The patient is given aspirin, clopidogrel, nitroglycerin, 
heparin, and IV fluids. He remains hypotensive despite dopamine. 
Urgent Coronary angiography is planned. Which of the following additional steps is most 
appropriate? 

A. Initiate low dose beta‐blocker
B. Add a phosphodiesterase inhibitor
C. Insert an intra‐aortic balloon pump
D. Administer thrombolytic therapy
E. Endomyocardial biopsy

Grading Shock Severity
Increasing severity of cardiogenic shock
90.0%
Pre-shock ‘Mild’ ‘Moderate’ 80.0%
‘Refractory’
80.0% Need
for
70.0% MCS

60.0%
50.0% 42.0%
40.0%
Mortality

30.0% 21.0%
20.0%
10.0% 7.5%
3.0%
0.0%

Kar, et al. Circulation 2012; 125: 1809-1817.


Inotrope Dose

2948
Copyright © Harvard Medical School, 2018. All Rights Reserved.

Potential Benefits of MCS in 
Cardiogenic Shock
• Maintain End‐Organ Perfusion
• Reduce Intracardiac Filling Pressures
• Reduce ventricular volumes, wall stress, and myocardial 
oxygen consumption
• Augment coronary perfusion
• Limit Infarct Size

Rihal, et al. J Am Coll Cardiol 2015; 65: e7-26.

Considerations in Device 
Selection

• Etiology of Shock
• Anticipated Duration of Support
• Left Ventricular Support, Right Ventricular Support, or Both?
• What is the Goal?
• Bridge to Recovery
• Bridge to Durable VAD
• Bridge to Transplant
• Bridge to Decision

2949
Copyright © Harvard Medical School, 2018. All Rights Reserved.

Options for Temporary MCS
LV Support Only*
IABP Impella TandemHeart ECMO
Percutaneous

Less Support More support

TandemHeart Abiomed Centrimag ECMO


Surgically Implanted

BVS 5000

Intra‐Aortic Balloon Counterpulsation (IABP)

•Traditional mainstay of mechanical therapy for 
cardiogenic shock

•Inflates in diastole 
• Augments coronary and peripheral perfusion
•Deflates in systole
• Reduces afterload
•Effective in acute hemodynamic stabilization, 
though no definitive impact on mortality in shock
•Patients with a good hemodynamic response to 
IABP experience better outcome2

Willerson JT, et al. Am J Med 1975; 58: 183-91


Ramanathan K, et al. Circulation 2003; 108(suppl I): I-672

2950
Copyright © Harvard Medical School, 2018. All Rights Reserved.

Physiological Effects of IABP

Cardiac Index  40% Cardiac Output  500 ml/min


(L/min/M2)
Heart Rate  7 bts/min
Arterial Lactate  42%
(mmol/L) Systolic BP  20 mmHg

Coronary  34% Diastolic BP  30 mmHg


Blood Flow
(M2/100g/min)

Sheidt, NEJM 1973; Mueller, Circ 1972

Use of IABP
• Consider for • Contraindications
– patients with STEMI when  – Significant AI
cardiogenic shock is not  – Severe PAD
rapidly reversed with  – Abdominal aortic aneurysm
pharmacologic therapy 
– Aortic Dissection
– inotrope‐refractory heart 
failure in patients with 
cardiomyopathy
– Medically refractory 
ventricular tachycardia
– Mechanical MI complication 
(VSD, Papillary muscle 
rupture)

2951
Copyright © Harvard Medical School, 2018. All Rights Reserved.

Question 2

You are called to the CCU to see a 47 y/o man 
being supported with an IABP following an AMI 
with cardiogenic shock. The nurse notes a change 
in the IABP waveform. What manipulation should 
be made to correct the timing:

a. Nothing, the timing is optimal
b. Increase the inflation delay
c. Increase the inflation time
d. Reduce the inflation time

IABP Timing

Unassisted
Diastolic and
Systolic Pressures Augmented Pressure
in Early Diastole

ECG

Lower end-diastolic
Aortic and systolic
Pressure pressures on
subsequent beat

Inflation at dicrotic notch


(onset diastole)
Deflation at End-Diastole

Krishna and Zacharowski, Contin Educ Anaesth Crit Care Pain 2009;9:24-28

2952
Copyright © Harvard Medical School, 2018. All Rights Reserved.

Suboptimal Timing
Early Inflation Late Inflation

Use R‐wave 
(ECG) trigger 
rather than 
Early Deflation Late Deflation pressure 
waveform 
trigger in 
patients with 
arrhythmias

Krishna and Zacharowski, Contin Educ Anaesth Crit Care Pain 2009;9:24-28

IABP SHOCK-2 Trial: Primary


Outcome
30-Day Mortality

HR 0.96, 95% CI: 0.79 – 1.17, p=0.69
Control: 41.3%
IABP: 39.7%

Thiele et al, NEJM 2012

2953
Copyright © Harvard Medical School, 2018. All Rights Reserved.

Trends in MCS Use

Kapur and Esposito, Heart Failure Clinics 2015

Percutaneous MCS Devices
TandemHeart Impella

21F LA- 15F FA Access 12F-21F


Centrifugal, 3.5-4 l/min Flow Axial 2.5-5.0 l/min
Yes Anticoagulation Yes
Short-term Approved Short-term

2954
Copyright © Harvard Medical School, 2018. All Rights Reserved.

Device Comparison
IABP Impella TandemHeart
Ease of Implantation
  X
Support LV only LV, (RV) LV, RV, BiV
Mode of Cannulation Percutaneous Percutaneous Percutaneous or
Surgical
Cannula Size 7F 12-14F 21F
Trigger ECG/Arterial Pressure Asynchronous Asynchronous
Cardiac Output 0.5 L/min 2.5-4.0 L/min 4-6 L/min
Augmentation
Pulsatile Flow Yes No No

Duration of Support Short up to 7 days up to 14 days


Limb Ischemia + ++ +++
Hemolysis + ++ ++
Insertion site bleeding + ++ ++
Adapted from Myat, et al. JACC Cardiovasc Interv 2015; 8: 229-44.

IABP vs Percutaneous LVAD
Cardiac Index

LVAD IABP Cardiac Index P(heterogeneity) = 0.22


meansd meansd Mean Difference I2 = 34.0%

Thiele et al. 2.30.6 1.80.4 0.55 (0.23 ; 0.87)

Burkhoff et al. 2.20.6 2.10.2 0.16 (-0.14 ; 0.46)

Seyfarth et al. 2.20.6 1.80.7 0.36 (-0.16 ; 0.88)

Pooled 0.35 (0.09 ; 0.61)

-2 -1 0 1 2
Favors IABP Favors LVAD

Cheng et al: EHJ 2009; 30:2102

2955
Copyright © Harvard Medical School, 2018. All Rights Reserved.

IABP vs Percutaneous LVAD
Pulmonary Capillary Wedge Pressure

LVAD IABP Pulmonary Wedge Pressure P(heterogeneity) = 0.01


meansd meansd Mean Difference I2 = 76.6%

Thiele et al. 165 227 -5.6 (-9.2 ; -2.1)

Burkhoff et al. 164 253 -8.4 (-11.0 ; -5.8)

Seyfarth et al. 195 206 -1.0 (-5.2 ; 3.2)

Pooled -5.3 (-9.4 ; -1.2)

-20 -10 0 10 20
Favors LVAD Favors IABP

Cheng et al: EHJ 2009; 30:2102

IABP vs Percutaneous LVAD 


30 Day Mortality 

Cheng et al: EHJ 2009; 30:2102

2956
Copyright © Harvard Medical School, 2018. All Rights Reserved.

Question 3
30‐year old female at 37 weeks’ gestation of a previously uncomplicated pregnancy presents 
with a 3‐day history of pleuritic chest pain. Upon admission, due to concern for fetal distress 
she is taken to urgent Caesarean section without obstetrical complication. On closure of the 
abdomen, she suffers complete cardiovascular collapse with PEA arrest. Cardiopulmonary 
resuscitation is initiated with recovery of spontaneous circulation within 20 minutes. Blood 
pressure is 80/60 with PaO2 40 mm Hg despite an FIO2 of 1.0. the extremities are mottled and 
clammy. You are called to discuss options for urgent mechanical circulatory support for 
stabilization. 

Which of the following is the best strategy?

A. Intra‐aortic Balloon Pump
B. Veno‐Venous ECMO
C. Veno‐Arterial ECMO
D. Percutaneous Right Ventricular Assist Device
E. Percutaneous Left Ventricular Assist Device

Extra‐Corporeal Membrane Oxygenation

• Centrifugal, non-pulsatile pump for blood


propulsion linked to a membrane oxygenator for
gas exchange

• 2 broad configurations
• Veno-Venous  Oxygenation/pulmonary support
• Veno-Arterial  Cardiopulmonary Support

• Cannulation at bedside without fluoroscopic


guidance

• ≥ 4.5 L/min depending on cannula size


• Inflow: RA via femoral vein (18-31 F)
• Outflow: Descending aorta via femoral artery (15-22F)

• Anticoagulation required (ACT 180-250)

Cheng R, et al. Ann Thorac Surg 2014; 97(2): 610-16


Kapur and Esposito, Heart Failure Clinics 2015

2957
Copyright © Harvard Medical School, 2018. All Rights Reserved.

ECMO Configurations

Veno-Venous ECMO Veno-Arterial ECMO


(Pulmonary Blood Flow Intact) (Cardiopulmonary Bypass)

ECMO: Accepted Indications

• Hypoxemic respiratory failure with PaO2/FiO2 of <100 mmHg 
despite optimization of the ventilator settings, including the tidal 
V‐V ECMO

volume, positive end‐expiratory pressure (PEEP), and inspiratory to 
expiratory (I:E) ratio
• Avoid in those mechanically ventilated for > 7 days
• Hypercapnic respiratory failure with an arterial pH less than 7.20

• Refractory cardiogenic shock
V‐A ECMO

• Cardiac arrest
• Failure to wean from cardiopulmonary bypass after cardiac surgery
• As a bridge to either cardiac transplantation or placement of a 
ventricular assist device

2958
Copyright © Harvard Medical School, 2018. All Rights Reserved.

ECMO vs. Mechanical Ventilation for 
Respiratory Failure/ARDS

Munshi, et al. Ann Am Thorac Soc 2014; 11(5):802–810

ECMO vs. Mechanical Ventilation for 
Respiratory Failure/ARDS

Overall, no definitive mortality benefit seen

Potential benefit in subgroup of studies examining


VV ECMO in patients with H1N1-related ARDS

Munshi, et al. Ann Am Thorac Soc 2014; 11(5):802–810

2959
Copyright © Harvard Medical School, 2018. All Rights Reserved.

V‐A ECMO in Cardiogenic
Shock/Cardiac Arrest
• Efficacy
• No Randomized Trials, only Observational Studies
• Median Survival to Hospital Discharge: 41% (IQR 13‐78%)
• Best median survival in myocarditis (73%)
• Benefit in cardiac arrest greatest if door to ECLS times < 30 minutes

– Concerns
• LV/Aortic Stasis 
• Pulmonary Hemorrhage
• Coronary/Cerebral Hypoxia

• Complications in practice
• Kidney Injury (56%)
• Bleeding (41%)
• Significant Infection (30%)
• Lower extremity ischemia (17%)
• Neurologic Complications (13%)

Cheng R, et al. Ann Thorac Surg 2014; 97(2): 610-16

Options for RV failure

• TandemHeart RVAD / BiVAD
• Impella RP (investigational)
• Surgical (Centrimag) RVAD / BiVAD
• Veno‐Venous ECMO (isolated RV failure)
• RA‐LA ECMO (Pulmonary + RV failure)
• Veno‐arterial ECMO (biventricular failure)

2960
Copyright © Harvard Medical School, 2018. All Rights Reserved.

Increasing severity of cardiogenic shock

Pre-shock ‘Mild’ ‘Moderate’ ‘Refractory’


IABP Impella 5.0
90.0% Impella CP Centrifugal LVAD 80.0%
ECMO
80.0%
70.0%
60.0%
Mortality

50.0% 42.0%
40.0%
30.0% 21.0%
20.0%
10.0% 7.5%
3.0%
0.0%

Kar, et al. Circulation 2012; 125: 1809-1817.


Inotrope Dose

Selection of Percutaneous MCS for 
Refractory CS

Kar, et al. Circulation 2012; 125: 1809-1817.

2961
Copyright © Harvard Medical School, 2018. All Rights Reserved.

Take Home Points

• Options for management of cardiogenic shock are 
expanding
• For medically refractory patients, early institution of 
mechanical circulatory support is key
• Device selection must be tailored to the needs of the 
individual patient and local expertise
• Consider ultimate goals of care before transitioning 
to mechanical support
• Patients that stabilize on mechanical support may be 
candidates for durable VAD as destination therapy or 
bridge to cardiac transplant

Thank You!

www.brighamandwomens.org/heart 

2962
Copyright © Harvard Medical School, 2018. All Rights Reserved.

Critical Care: Take Home Messages


and Clinical Pearls

Elizabeth Gay, MD
Associate program director, Brigham and Women’s Hospital Pulmonary
and Critical Care Medicine Fellowship
Assistant professor of medicine, Harvard Medical School

• No disclosures

2963
Copyright © Harvard Medical School, 2018. All Rights Reserved.

Case 1
A 38 year old man with a history of a traumatic splenectomy presents with
fever and confusion. He was well until a few hours before presentation,
when he suddenly felt light-headed, became febrile and per his wife, not
his usual self. The day before he had taken his young daughter to the park.

• PMH: Spleen removed after shrapnel injury during Iraq tour


• No allergies, no medications
• Married to a woman he met while in Iraq and lives with her and their
four year old daughter. Discharged from Army and working in
management for construction firm. No travel recently.
• ROS: No headaches, rash, abdominal pain, cough, diarrhea, nausea

Physical exam
T38.9, RR 25, HR 130, BP 90/65, SaO2 94% on room air
General- ill appearing, mild increase in work of breathing
Oropharynx- clear
Chest- a few basilar crackles
CV- tachycardic, regular, no mrg
Abd- soft, NT
Ext- no edema
No rashes
Oriented to self and hospital, but missed year
Neurologic exam non-focal, no meningeal signs

2964
Copyright © Harvard Medical School, 2018. All Rights Reserved.

Question 1
Which of the following is the next best step in care for this
patient?

A) Obtain blood cultures and CXR


B) Obtain blood cultures and CXR, and start broad spectrum antibiotic
coverage
C) Obtain blood cultures, CXR, and lumbar puncture, then start broad
spectrum antibiotic coverage
D) Obtain blood cultures and CXR, then start broad spectrum antibiotic
coverage and 2 L of normal saline

Teaching points
• Recent studies have called into question the need for care
bundles around sepsis, but strong data supports that early
antibiotics decrease mortality.
• Whether or not early fluids are important is harder to tease
out, but the bulk of the evidence suggests that early
resuscitation is important.
• A qSOFA score of 2 suggests increased mortality and in this
patient without a spleen, sepsis is an emergency.

2965
Copyright © Harvard Medical School, 2018. All Rights Reserved.

Case 1 continued. . .
• Labs return with lactate of 5, Cr of 2, mildly elevated AST and ALT, wbc of
20 with left shift, plt of 102. CXR is clear.

• He receives antibiotic therapy with vancomycin and piperacillin-


tazobactam and 4 L of fluid, but rapidly becomes more hypotensive.
Within 2 hours he is on 30 mcg/min of norepinephrine, .04 U/min of
vasopressin, 400 mcg/min of phenylephrine and epinephrine is being
hung.

• Bedside ultrasound does not show hydronephrosis, nor gallbladder


pathology. Common bile duct is not dilated. Echo shows hyperdynamic LV
without compromise. Detailed skin exam does not reveal any lesions.

Question 2
What is the next best step in care?

A) Broaden antibiotics to meropenem and gentamycin


B) Obtain pan-CT scan
C) Add dobutamine
D) Start stress dose steroids

2966
Copyright © Harvard Medical School, 2018. All Rights Reserved.

Teaching points
• Patient risk factors for drug resistant organisms, not severity of illness,
should determine antibiotic choice.
• Refractory septic shock merits best possible work-up for surgical source.
Ultrasound at the bedside may be the safest option if a patient is too
unstable to travel.
• Some septic patients may have additional element of cardiogenic shock,
but without evidence for this, adding an inotrope may worsen
hypotension.
• Stress dose steroids may decrease duration of shock (and potentially have
a mortality benefit in some select patient population). This is a
reasonable option in refractory shock. There is no need to check a cortisol
stimulation test. You can also consider fludracortisone in addition to
hydrocortisone 50 mg IV q6 hours.

Case conclusion
• The patient died of refractory shock 6 hours
after arrival in ICU. All of his blood cultures
grew pneumococcus. He had not been
vaccinated after his splenectomy.

2967
Copyright © Harvard Medical School, 2018. All Rights Reserved.

Case 2
A 71 year old man with a history of chronic lymphocytic
leukemia (never requiring treatment) presents with abdominal
pain and is found to have pancreatitis, thought secondary to a
gallstone which had passed. He is admitted to the general ward
and given a 1 L NS bolus on arrival. Overnight he develops a new
oxygen requirement and is given furosemide. The next morning
on rounds he is noted to be more confused, with increased work
of breathing. Stat labs reveal a lactate of 4, increasing
leukocytosis and Hb of 17, as well as Cr of 2.5 (from 1.2
baseline). K is 3. Urine output is 10 cc/hour.

Physical Exam
T38.1 HR 125 BP 101/65 RR 30 SAO2 92% on 40% NRB
General- alert, but confused, diaphoretic, accessory muscle use
Chest- diminished at the bases with some proximal rhonchi
CV- tachycardic, regular, no mrg
Abd- distended, tense, non-tender, hypoactive bowel sounds
Ext- trace edema, warm
Oriented x 2, neurologic exam grossly non-focal

2968
Copyright © Harvard Medical School, 2018. All Rights Reserved.

Question 1
Which of the following is the best initial fluid management
strategy?
A) Start NS at 200 cc/hour
B) Bolus 50 grams of albumin
C) Bolus 2 L of NS
D) Bolus 2 L of LR

Teaching points
• Correction of hypovolemia in severe pancreatitis likely
improves outcomes and may decrease extent of necrosis.
• Too much fluid may predispose to pulmonary edema, bowel
wall edema, and abdominal compartment syndrome.
• There is a lack of data to suggest that colloids should be
preferred over crystalloids in sepsis or pancreatitis.
• LR may be associated with less renal injury than NS.

2969
Copyright © Harvard Medical School, 2018. All Rights Reserved.

Case continued
The patient is admitted to the ICU and intubated for respiratory
distress. He does not require high ventilator settings, but PEEP is
set at 12 to help with significant atelectasis. Bladder pressure is
14. Over the next two days in the ICU, he receives a total of 6 L
of LR. Lactate normalizes and HR is the 80s with a systolic blood
pressure of 120. He requires 40% fio2 on the ventilator.
Over the next day, urine output decreases to 25 cc an hour and
Cr increases to 3, but electrolytes are in order. UA shows muddy
brown casts. He continues to spike fevers to 102. CT abdomen
shows more necrosis around pancreas but no fluid collection.
Bladder pressure remains 12 to 14.

Question 2
Which of the following is the next best strategy for management
of this patient’s renal failure?
A) Start continuous venovenous hemofiltration (CVVH)
B) Start a furosemide drip, aiming to increase urine output to
50 cc/hour
C) Continue to follow expectantly
D) Start intermittent hemodialysis

2970
Copyright © Harvard Medical School, 2018. All Rights Reserved.

Teaching points
• Evidence to date does not support that routine early initiation
of renal replacement therapy provides a benefit over waiting
for traditional indications for dialysis.
• Diuresis in ATN likely does not change the course of the
disease so can be attempted as needed for clinical
management of volume overload or electrolyte disarray.

Case 3
A 49 year old man is transferred to your ICU for refractory
hypoxemic respiratory failure. He has no clear preceding
pulmonary diagnosis, but had been noted to have abnormal
chest imaging 6 months before presentation. His current
presentation was marked by a febrile illness, then rapid
development of bilateral infiltrates requiring intubation. He has
received antibiotics and steroids at the outside hospital.

2971
Copyright © Harvard Medical School, 2018. All Rights Reserved.

Physical exam
T36 HR 75 BP 110/68, breathing with vent , SaO2 92%
Vent settings: AC, R16, TV 540 (8 cc/kg IBW), fio2 95%, PEEP 5
ABG 7.24/paCO2 70/paO2 62
Plateau 30, Peak 35
General- intubated, sedated on propofol
ETT in place
Chest- bilateral harsh breath sounds and some basilar crackles
CV- rrr, S1, 2, no mrg
Ext- warm, no edema, no rashes

http://courses.washington.edu/med620/images/mv_c3fig1.jpg

2972
Copyright © Harvard Medical School, 2018. All Rights Reserved.

Question 1
What is the next best step in ventilator management?
A) Increase RR to 20
B) Increase TV to 600
C) Decrease TV to 450, increase RR to 20
D) Decrease TV to 250

Teaching point
• The primary management for ARDS remains a low tidal
volume strategy, targeting a plateau pressure less than 30,
pa02 of 60 and pH of 7.15 or higher.
• We don’t know if higher PEEP is better, so a bedside
evaluation of each patient remains the best strategy.
Complications from high PEEP include hypotension from
decrease in preload and barotrauma.

2973
Copyright © Harvard Medical School, 2018. All Rights Reserved.

Case continued. . .
• TV is decreased to 5 cc/kg and with increase in RR, pH remains
around 7.2 with pa02 of 60
• He is started on a cisatracurium drip.
• At attempt at diuresis leads to hypotension and is aborted.
• Overnight he becomes extremely hypoxemic after a turn, with
pa02 of 52 on blood gas. CXR is unchanged. Mechanics show
further decrease in compliance.

Question 3
What is the next best step in care?
A) Change to pressure control ventilation
B) Change to high frequency oscillatory ventilation
C) Prone positioning
D) Call ECMO team

2974
Copyright © Harvard Medical School, 2018. All Rights Reserved.

Teaching points
• For refractory ARDS, prone positioning may provide a survival
benefit in patients with the most severe disease.
• ECMO is an appropriate consideration for an otherwise
healthy patient with single organ failure and should be
considered early.
• Other modes of ventilation have not been shown to be
helpful and in the case of HFOV may be harmful in adults.

References
• Venkatesh, Balasubramanian, et al. "Adjunctive Glucocorticoid Therapy in Patients with Septic
Shock." New England Journal of Medicine (2018)

• Semler, Matthew W., et al. "Balanced crystalloids versus saline in critically ill adults." New England
Journal of Medicine378.9 (2018).

• Chaudhuri, Dipayan, et al. "Early Renal Replacement Therapy Versus Standard Care in the ICU: A
Systematic Review, Meta-Analysis, and Cost Analysis." Journal of intensive care medicine (2017)

• Guérin C, Reignier J, Richard JC, et al. Prone positioning in severe acute respiratory distress
syndrome. N Engl J Med (2013)

2975
Copyright © Harvard Medical School, 2018. All Rights Reserved.

Critical Care Board Review


Elizabeth Gay, MD

Associate program director, Harvard-Brigham and Women’s


Hospital Fellowship in Pulmonary and Critical Care Medicine
Assistant Professor of Medicine, Harvard Medical School
Boston, MA

Disclosures

• None

2976
Copyright © Harvard Medical School, 2018. All Rights Reserved.

Case #1
A 68 year-old man with a history of CAD, COPD, and diastolic
heart failure presents with worsening dyspnea for two days,
associated with wheezing. No chest pains or palpitations.

T 36, P 110/min, BP 100/95 mmHg, RR26


Oxygen saturation 95% on non-rebreather, 40% fiO2
Mild JVD, no HJR, +accessory muscle use
Chest- bilateral wheezes and rales throughout both lungs
Abd- soft, NT
CV- tachycardic, regular, no mrg, nl S1, 2
Ext- Warm, + symmetric LE edema

Case #1

2977
Copyright © Harvard Medical School, 2018. All Rights Reserved.

Case continued. . .

• In the ED he is given broad spectrum


antibiotics, albuterol and ipratropium
nebulizers, and IV solumedrol.
• They then increase the FiO2 to 60%.
• He is still unable to speak in full sentences.
• ABG: 7.25/paC02 55/pa02 89

Acute: Increase paCO2 by 10,


decrease pH by .08. Interpret this ABG.
Chronic: Increase by 10,
decrease by .03

Question 1: What would be the most


appropriate intervention at this time?

• A. Intubate and begin assist control ventilation


• B. Decrease the FiO2
• C. Apply non-invasive positive pressure ventilation
• D. Decrease the rate of IV fluid administration
• E. Begin bicarbonate IV

2978
Copyright © Harvard Medical School, 2018. All Rights Reserved.

Indications for non-invasive positive pressure


ventilation:

– Acute-on-chronic respiratory failure (exacerbation of COPD)

– Patients with acute asthma

– Acute congestive heart failure (not caused by an acute myocardial


infarction)

– Patients who are immunocompromised with pneumonia

– To facilitate weaning from invasive ventilation in difficult to wean


patients with COPD

– Post operative patients

– Obesity hypoventilation, neuromuscular weakness

– Palliative care for dyspnea

Question 1: Explanation
NIV contraindications:

– Unstable: ischemia, hypotension, arrhythmia, upper GIB,


profuse emesis, etc
– Agitated, altered MS
– Unable to protect airway
– Swallowing impairment
– Excessive secretions
– Recent upper airway or GI surgery

2979
Copyright © Harvard Medical School, 2018. All Rights Reserved.

Question 1: Explanation

• Goals of NIV:
– Ventilation: reduce PaCO2
– Improve oxygenation
– Reduce work of breathing
– Reduce dyspnea

Question 1 Answers: What would be the most


appropriate intervention at this time?
• A. Intubate and begin assist control ventilation.
NIV may allow you to avoid intubation.
• B. Decrease the FiO2
Although there is no need to over-oxygenate, decreasing fio2 in the face of hypoxemia
will not improve the situation.
• C. Apply non-invasive positive pressure ventilation
• D. Decrease the rate of IV fluid administration
This may be reasonable, but is unlikely to quickly correct respiratory failure.
• E. Begin bicarbonate IV
Creating a metabolic alkalosis will decrease the need for ventilation, but risks volume
overload and doesn’t address the underlying respiratory failure.

10

2980
Copyright © Harvard Medical School, 2018. All Rights Reserved.

NIV in the ICU

• Predictors of failure in hypoxemic respiratory failure:


– No improvement in PaO2/FiO2 in 1-2 hours
– Older patients (> 40!)
– APACHE >34
– ARDS
– Pneumonia: Observational studies suggest NIV NOT useful
in hypoxemia respiratory failure secondary to CAP in the
absence of COPD
– Multi-organ failure

11

NIV versus standard Rx

Nava et al, 2009 Lancet

2981
Copyright © Harvard Medical School, 2018. All Rights Reserved.

Case #1: The story continues. . .

• Despite NIV, the patient does not seem to be


improving.
• Labs have now come back and troponin is elevated,
ECG unchanged.
• BP 85/40 mmHg, Pulse 105/min
• SaO2 93% on bi-level 10/5 cmH2O with 100 % FiO2

13

CXR- ARDS

What
condition
appears to
be
developing?

2982
Copyright © Harvard Medical School, 2018. All Rights Reserved.

Question 2: What is the next best step in


management?

A. Transfer to CCU
B. Increase IVF due to low BP
C. Intubate the patient
D. Add sedation to improve synchrony with NIV

15

Question 2: Explanation

Failure of NIV indicated by:


No improvement in blood gas.
No improvement or worsening respiratory
distress.
Worsening mental status.
Worsening hemodynamics.

16

2983
Copyright © Harvard Medical School, 2018. All Rights Reserved.

Question 2 Answers: What is the next best


step in management?

A. Transfer to CCU
This may be reasonable but should not delay intubation.
B. Increase IVF due to low BP
You should re-evaluate volume status before increasing fluid
administration in the setting of developing ARDS.
B. Intubate the patient
C. Add sedation to improve synchrony with NIV
NIV is a spontaneous mode of ventilation and adding sedation may
decrease respiratory drive and lead to worsening acidosis.

17

Case #2
A 67 year-old man with a history of hypertension,
peripheral vascular disease, and chronic renal
insufficiency is post-operative day two from a
femoral-popliteal bypass graft. He becomes
increasingly restless and combative with nursing.
Social history notable for a 45 pack-year history of
smoking cigarettes and 1-2 drinks with dinner,
perhaps more on weekends. He is a high level
executive in the financial industry.
Current medications: Fentanyl PCA, Metoprolol 12.5 mg
BID, furosemide 20 mg IV daily

18

2984
Copyright © Harvard Medical School, 2018. All Rights Reserved.

Case #2 continued

BP 140/95 mmHg, HR 90/min, RR 20, Sa02 95% on RA


Uncooperative, belligerent, paranoid ideation
No jvd, MM dry
Chest- clear throughout
CV- tachycardic, regular, no mrg
Ext- trace symmetric edema, bypass site is clean and dry with no
evidence of bleeding
Neuro- won’t follow commands, pupils equal and reactive,
normal and symmetric reflexes

19

Question 1 Answers: What is the most


likely diagnosis?

A. Alcohol withdrawal
B. Non-alcohol related delirium
C. Medical disorder (infection, stroke, etc)
D. Psychiatric disorder (psychosis)
E. Insufficient information

20

2985
Copyright © Harvard Medical School, 2018. All Rights Reserved.

Case #2 Explanation:
Key points about ICU delirium
Agitation ≠ Delirium
• 3 delirium phenotypes
– Hyperactive
– Hypoactive
– Mixed

• Delirium is defined (DSM IV) by:

– Disturbance in consciousness
– Change in cognition or development of perceptual disturbance not
accounted for by pre-existing or evolving dementia
– Disturbance develops over hours-days and fluctuates in severity

21

Case #2 Explanation:
Core points about ICU delirium
Predisposing factors:
• Advanced age
• Dementia
• Functional impairment in activities of daily living
• High medical comorbidity
• History of alcohol abuse
• Male gender
• Sensory impairment (blindness, deafness)

22

2986
Copyright © Harvard Medical School, 2018. All Rights Reserved.

ICU Delirium

• Differential diagnosis
– Psychiatric disorders: schizophrenia, depression,
mania, dementia
– Medical disorders: endocrinopathy, infection, CNS
injury, sepsis, renal failure, hypoxia, hypoglycemia,
electrolyte imbalance
– Substances: intoxication, adverse effect,
withdrawal

23

ICU Delirium

• Implications:
– 3 times higher risk of death by 6 months
– $15k to $25k higher hospital costs
– 5 fewer ventilator free days (days alive and off
vent)
– 9 times higher incidence of cognitive impairment
at hospital discharge

Ely et al, JAMA 2004

24

2987
Copyright © Harvard Medical School, 2018. All Rights Reserved.

ICU Delirium

• Diagnosis based on a • Treatment: the 4 “P’s”


combination of: • Prevention
• History* • Prevention
• Assessment tool • Prevention
• Exclusion of • ?Dexmedetomidine
medical/psychiatric • Psychotropics
illness

• Avoid Polypharmacy

Question 1: What is the most likely


diagnosis?
A. Alcohol withdrawal

This is a possible diagnosis here, but you need more information both to understand
drinking history and to rule out other medical problems. He does not have clear
signs of autonomic hyperactivity which are typically associated with alcohol
withdrawal.
B. Non-alcohol related delirium
You need to rule out a medical disorder and obtain more history of about alcohol use.
C. Medical disorder (infection, stroke, etc)
You would want more information (labs) to rule out a new medical complication.
D. Psychiatric disorder (psychosis)
A new onset psychosis would be unlikely.
E. Insufficient information

26

2988
Copyright © Harvard Medical School, 2018. All Rights Reserved.

Case #3

A 28 year-old woman in her 34th week of pregnancy


presents with acute dyspnea. This started suddenly
one morning when she was doing some chores in her
house. She has some pleurisy but no cough,
palpitations, or fevers.

• Pregnancy has been uneventful until now


• No PMH
• Medications: Prenatal vitamins

27
27

Case 3 continued. . .
BP 95/70 mmHg, RR 20, Pulse 100/min, SaO2 97% on
RA
Alert, mild increase in work of breathing
There is no JVD or HJR
Chest- clear throughout
CV- tachycardic, regular, soft systolic murmur
Abd- gravid, soft
Ext- warm, no c/c/e
No rashes
Oriented x 4

28

2989
Copyright © Harvard Medical School, 2018. All Rights Reserved.

Question 1: Which of the following is the


most likely diagnosis?

A. Asthma exacerbation
B. Perinatal cardiomyopathy
C. Anxiety
D. Pulmonary embolism

29

Case #3

Causes of Maternal Mortality


– Pulmonary embolism 20%
– Hypertensive disorders (including eclampsia) 15%
– Ectopic pregnancy 12%
– Bleeding 11%
– CVA 10%
– Anesthesia 8%
– Cardiomyopathy 5%

30

2990
Copyright © Harvard Medical School, 2018. All Rights Reserved.

Case #3

• Pulmonary Embolism
– Leading cause of peri-partum maternal mortality
– AA>Caucasians
– Risk factors: older age, c-section
– DVT present in 0.3% of all deliveries
– Factors V, VII, VIII, IX, X, XII and fibrinogen are all
increased during pregnancy

31

Case #3 continued. . .

• Despite heparin gtt and supplemental oxygen, her


condition worsens.
• BP 80/40 mmHg, P120/min, RR 35, SaO2 88% on 100
% non-rebreather
• Lungs: clear throughout
• Echo at bedside: Dilated and hypokinetic RV, RVSP
50
• Doppler US shows femoral DVT
What is the next best step in management?

32

2991
Copyright © Harvard Medical School, 2018. All Rights Reserved.

Case #3

• INTUBATE
– Hypoxia can be harmful to the fetus
– Maternal SaO2 > 95%
– TREAT the Patient!
• Sedatives, antibiotics, other medications generally ok to
use

33

Thrombolysis?

Ahearn, Gregory S., et al. Archives of Internal Medicine 162.11 (2002): 1221-1227.

2992
Copyright © Harvard Medical School, 2018. All Rights Reserved.

Case #4
An 81 year-old man with a history of CAD and CRI is admitted with pleuritic
chest pain, fevers and cough of 2 days duration. He is also mildly confused
per his wife.

T-101.8 BP 100/60 mmHg HR 110/min RR 32 SaO2 91% on RA


General- frail, thin, mild respiratory distress, oriented x 2
No jvd, MM dry
Chest- Rales upper right and mid lung zones
CV- tachycardic, regular, no mrg
Ext- no edema, warm
Initial labs:
– WBC-12K with left shift
– Electrolytes normal except for Cr 1.5 (baseline)
– Blood glucose 260 mg/dl

35

Case #4

2993
Copyright © Harvard Medical School, 2018. All Rights Reserved.

Case continued. . .

• You diagnosis the patient with community


acquired pneumonia and start ceftriaxone and
azithromycin after obtaining sputum and
blood cultures. He is admitted to the ICU for
close monitoring.

What would you recommend for glucose


control? What is your target blood sugar?

37

Question 1: Which of the following is the most accurate


statement about glucose control in the critically ill
patient?
A. Tight glucose control (80-110 mg/dl) is associated with a mortality
benefit compared with a goal of < 180 mg/dl.
B. Hyperglycemia in the critically ill is associated with increased mortality.
C. Tight glucose control is associated with fewer days on mechanical
ventilation.
D. Episodes of hypoglycemia in patients managed with tight glucose
control are insignificant if the protocols are done properly.
E. Tight glucose control is only beneficial for critically ill diabetics.

38

2994
Copyright © Harvard Medical School, 2018. All Rights Reserved.

Case #4 Explanation:
Glycemic control in the critically ill
• Targeting < 180 mg/dl resulted in
lower mortality then 80-110
mg/dl.
• Severe hypoglycemia, < or = 40
mg/dl, was observed in the 80-
110 mg/dl group (6.8%) vs the <
180 mg/dl (0.5%).
• In this trial, the absolute risk of
death at 90 days increased by 2.6
percentage points; hence, the
number needed to harm = 38.
• Presently, tight or “intensive”
glucose control is not
recommended.

Question 1 Answers: Which of the following is the most


accurate statement about glucose control in the
critically ill patient?
A. Tight glucose control (80-110 mg/dl) is associated with a mortality benefit compared with
a goal of < 180 mg/dl.
More recent studies have not shown improved mortality with tight control.
B. Hyperglycemia in the critically ill is associated with increased mortality.
C. Tight glucose control is associated with fewer days on mechanical ventilation.
Glucose control has not been shown to change days on the ventilator.
D. Episodes of hypoglycemia in patients managed with tight glucose control are insignificant if
the protocols are done properly.
Even in studies with detailed protocols, hypoglycemia was an important issue.
E. Tight glucose control is only beneficial for critically ill diabetics.
Even in patients with diabetes the data does not support tight control during critical illness.

40

2995
Copyright © Harvard Medical School, 2018. All Rights Reserved.

Case #5

A 41 year-old man presents with dyspnea and chest


tightness. Shortly after arrival to the ED he is
intubated for respiratory distress.

Past medical history: obesity (BMI 39), DM2, asthma

41

Case continued. . .

• Physical Exam
T36 BP 160/75 mmHg HR 125/min RR26 Sa02 92 on 50% fiO2
General- intubated, over-breathing the vent and mildly agitated
Large neck, difficult to appreciate JVD
Chest- poor air movement, no wheezes
CV- tachycardic, regular, no mrg
Ext- symmetric trace edema

42

2996
Copyright © Harvard Medical School, 2018. All Rights Reserved.

Question 1: Which of the following is the most accurate


statement about the role of this patient’s obesity in the
ICU?

A. His tidal volume should be based on his current weight


rather than his ideal body weight.
B. His ICU mortality is higher then someone of normal weight
C. He is likely to have more days on mechanical ventilation
then someone of normal weight.
D. His weight is not relevant to his ICU care.

43

Case #5 Explanation:
Obesity and prognosis in the critically ill

• No effect on mortality
until BMI < or = 18.5
or > or = 40
• Effects on: Duration
of Mechanical
Ventilation and LOS
in post-op patients.

Crit Care Med 2008;38:151

2997
Copyright © Harvard Medical School, 2018. All Rights Reserved.

Question 1 Answers: Which of the following is the most


accurate statement about the role of this patient’s
obesity in the ICU?

A. His tidal volume should be based on his current weight rather than his ideal
body weight.
Lung size doesn’t change with increasing BMI, so you should use ideal body weight.
B. His ICU mortality is higher then someone of normal weight
Between a BMI of 30 and 39, mortality does not appear to be higher and may actually
be improved in some populations (obesity paradox).
C. He is likely to have more days on mechanical ventilation then someone of
normal weight.
D. His weight is not relevant to his ICU care.
Obesity is important for ICU care, influencing things like procedures, imaging studies
and drug dosing.

45

Summary points
• Case #1:
– NIV may prevent intubation for COPD exacerbations.
– Co-morbidities or worsening of respiratory function obligate
intubation.
– Myocardial infarction and hemodynamic instability are absolute
contraindications to NIV.
• Case #2:
– Agitation with cognitive dysfunction has a broad differential and is
best diagnosed with a good history, exclusion of reversible
medical/psychological causes, and application of validated assessment
tools.
• Case #3:
– Pulmonary embolism is a leading cause of maternal mortality.
– In general, critical illness in pregnancy should not be treated
differently than in non-pregnant adults.

46

2998
Copyright © Harvard Medical School, 2018. All Rights Reserved.

Summary Points
• Case #4:
– Hypoglycemia is associated with poor outcomes for critically ill
patients but so are aggressive attempts at glycemic control.

• Case #5:
– Obesity is not necessarily associated with higher ICU mortality but
may be associated with longer time on mechanical ventilation.

47

2999
Copyright © Harvard Medical School, 2018. All Rights Reserved.

Board Review Practice 3


Sanjay Divakaran, M.D.
Fellow, Division of Cardiovascular Medicine
Department of Medicine
Brigham and Women’s Hospital
Clinical Fellow in Medicine
Harvard Medical School
sdivakaran@bwh.harvard.edu

Disclosures:

None

3000
Copyright © Harvard Medical School, 2018. All Rights Reserved.

CASE 1
A 70-year-old woman presents with pain in her hands and wrists for 9 months.
Her hands are stiff in the morning for 15 minutes. She has pain with sewing and
typing. She has not noticed any joint swelling. Her vital signs are normal. Her
bilateral proximal interphalangeal joints are tender to palpation and have bony
enlargements. The first carpometacarpal joints are also tender and have bony
squaring bilaterally. Her metacarpal squeeze test is negative. The remainder of
the exam is normal.

Which of the following studies should be done to establish the diagnosis?

A) ANA
B) Uric acid
C) Radiography of the hands
D) Rheumatoid factor
E) No additional studies are needed

The correct answer is E


• This patient with first carpometacarpal joint tenderness and squaring has
osteoarthritis.
• While symmetric and polyarticular arthritis seem to suggest rheumatoid arthritis,
the joints involved are not consistent with rheumatoid arthritis, and there are no
clinical indicators of joint inflammation.
• The DIP joints are almost always involved in degenerative arthritis, and rarely in
RA. PIP joints can be involved in either. The MCP joints are involved almost
exclusively RA.
• At the wrist, involvement of the first carpometacarpal joint is almost always a
sign of osteoarthritis, while involvement of ulnar styloid is almost always a sign
of rheumatoid arthritis.
• Morning stiffness less than 30 minutes indicates degenerative rather than
inflammatory arthritis.
• ACR criteria for diagnosing OA of the hands:
– Hand pain, aching, or stiffness, plus 3 of…
• Hard tissue enlargement of two or more of 10 selected joints
• Fewer than three swollen metacarpophalangeal joints
• Hard tissue enlargement of two or more DIP joints
• Deformity of two or more of 10 selected joints

Arthritis Rheum 1990;33:1601-10.

3001
Copyright © Harvard Medical School, 2018. All Rights Reserved.

Osteoarthritis Rheumatoid Arthritis


Courtesy of Dr. Sara Tedeschi McGonagle D et al. Rheumatology 2008;47:1278-1285.

CASE 2
A 68-year-old man with a history of hypertension and gout presents for his annual
exam. He was a past smoker for 20 years but quit 30 years ago. He drinks one glass
of red wine daily. He exercises regularly. He has no specific complaints. He gets his
influenza vaccination annually and he received his pneumococcal vaccine 3 years
ago. He had a normal colonoscopy 7 years ago. He is on amlodipine and
allopurinol. His vital signs are normal and his physical exam including
cardiopulmonary, abdomen, prostate, and peripheral pulses are all unremarkable.

Which of the following screening tests is most appropriate for this patient based
on most evidence of benefit?

A) CT Coronary Calcium Imaging


B) Prostate Specific Antigen
C) Thyroid Stimulating Hormone
D) Abdominal Ultrasound
E) Exercise Treadmill Test

3002
Copyright © Harvard Medical School, 2018. All Rights Reserved.

The correct answer is D


• USPSTF recommends that men between the ages of 65 and 75 with any
current or past history of smoking undergo a one time screening for AAA
with an abdominal ultrasound.
• Several studies have demonstrate survival benefit of screening including a
population based study of over 67,800 men aged 65 and 74 that were
randomized to AAA screening with surgery for those found to have AAA
>5.4cm vs. no screening that showed AAA related-mortality was reduced by an
average of 42% (95% CI, 22%-58%) in the screened population compared with
the non-screened population. Several studies have shown no benefit in male
non-smokers and in women.
• CT coronary calcium imaging has insufficient data to determine whether the
association of coronary calcium with coronary artery disease risk warrants
screening in asymptomatic men.
• The ability of prostate cancer screening tests to prolong life is uncertain.
• There is not enough evidence to recommend routine screening for thyroid
disease. However, screening high risk populations such as patients with
diabetes, down syndrome, and postmenopausal women may be justified.
Lancet 2002;360(9345):1531-9.
Ann Intern Med 2005;142:203-11.

CASE 3
A 20-year-old woman with a history of systemic lupus erythematosus
diagnosed two years earlier presents to the emergency department with
fatigue and fevers to 100.5 for several days.

Home medications include metoprolol succinate 25mg daily, lisinopril 10mg


daily, prednisone 15mg daily, hydroxychloroquine 200mg daily, azathioprine
50mg daily, and dapsone 100mg daily.

Initial evaluation reveals a young woman in no acute distress. Vital signs are
notable for a temperature of 100.5, heart rate of 60, blood pressure 110/70,
respiratory rate of 16, and an oxygen saturation of 80% on room air. Oxygen
saturation increases to 88% with a non-rebreather mask. Chest X-ray and CT
scan of the chest are unremarkable. CBC reveals a WBC 5.42, Hemoglobin 10
(at her baseline), Plt 273. Chemistry panel is unremarkable. Arterial blood gas
shows a pH of 7.38, PaCO2 32, and PaO2 527 on 100% oxygen via the non-
rebreather mask.

3003
Copyright © Harvard Medical School, 2018. All Rights Reserved.

CASE 3 (cont.)
The next best step in management is:

A) Endotracheal intubation and mechanical ventilation


B) Transfuse 2 units PRBCs
C) Treat with hyperbaric oxygen
D) Stop the dapsone
E) Treat with trimethoprim sulfamethoxazole and prednisone

The correct answer is D.


• This is a case of methemoglobinemia caused by dapsone.

• Methemoglobin is a form of hemoglobin that does not bind oxygen.


– It is normally present at <1% due to the activity of anti-oxidant enzymatic
pathways.

• Acquired methemoglobinemia is caused by exogenous oxidizing drugs, including


trimethoprim, sulphonamides, dapsone, local anesthetics, and aniline dyes.
– Most people are asymptomatic except for cyanosis, and possibly headache
and fatigue.
– Severe methemoglobinemia can cause shortness of breath, mental status
changes, dysrhythmias, seizures, coma, and death.

• The diagnosis should be suspected in the setting of a normal PaO2 despite a low
peripheral oxygen saturation.
– Patients may or may not have cyanosis.

• Treatment: stop the offending medication.


– Severe cases may be treated with IV methylene blue.

3004
Copyright © Harvard Medical School, 2018. All Rights Reserved.

CASE 4
A 37-year-old woman with no significant past medical history presents to the
emergency department with 2 days of days of nausea, vomiting, and
abdominal pain. Her only medication is acetaminophen, which she has been
taking for low back pain. She has not been taking any calcium supplements.

Labs reveal calcium 15.4, phosphate 4.9, and creatinine 1.6. Her PTH level is
low and her PTH-RP is undetectable.

All of the following would be appropriate initial therapy for her hypercalcemia
in the acute setting EXCEPT:

A) Hydration with intravenous normal saline


B) Zolendronate 4mg IV
C) Furosemide 40mg IV
D) Calcitonin 4 units/kg IM

The correct answer is C.


•The recommended approach for the treatment of severe hypercalcemia
(calcium > 14 mg/dL) includes:
– Hydration with normal saline.
– Bisphosphonates (zolendronate or pamidronate).
– Calcitonin.

• Loop diuretics such as furosemide (Choice C) are no longer recommended.


– Risks:
• Electrolyte abnormalities.
• Volume depletion resulting from over-diuresis.

• Loop diurectics should be considered in cases of fluid overload in order to


restore euvolemia.

LeGrand SB, Leskuski D, Zama ISO. Ann Intern Med. 2008;149(4):259.

3005
Copyright © Harvard Medical School, 2018. All Rights Reserved.

CASE 4 – Part II
CT scan of the chest, abdomen, and pelvis reveals diffuse lytic lesions in the
spine, pelvis, long bones, and ribs. No other abnormalities are noted.
Additional workup demonstrates a normal serum protein electrophoresis,
normal serum angiotensin converting enzyme level, and a mildly elevated
LDH of 450.

The most likely diagnosis is:


A) Multiple myeloma
B) Sarcoidosis
C) Metastatic breast cancer
D) Diffuse large B-cell lymphoma
E) Langerhans Cell Histiocytosis

The correct answer is C.


• All of the choices listed can cause lytic bone lesions and hypercalcemia.

• The most likely diagnosis in a 37 year old woman with no other CT findings is metastatic
breast cancer.
– CT of the chest may miss breast lesions.

• The median age at diagnosis for multiple myeloma (Choice A) is 66 years, and only ~2% of
patients are younger than 40 years old.

• Sarcoidosis (Choice B) of bone occurs in approximately 5% of patients with the disease,


but this usually occurs in advanced disease and rarely without pulmonary manifestations.

• Diffuse large B-cell lymphoma (Choice D) is a rare cause of bony lesions. The CT scan
would have been expected to show lymphadenopathy as well.

• Langerhans cell histiocytosis (Choice E) is a rare disorder (2 in 1 million) that may present
with bony lesions. Other presenting symptoms may include rash, diabetes insipidus,
lymphadenopathy, ataxia, and memory difficulty.

3006
Copyright © Harvard Medical School, 2018. All Rights Reserved.

CASE 5
A 32-year-old man with no significant past medical history
presents with low-grade fevers, anorexia, headache, and neck
stiffness of 4 days’ duration, which started shortly after a dental
procedure. The night prior to presentation he had one episode
of emesis and a worsening posterior headache. This morning, his
wife noticed that he seemed “not quite himself” and was
“walking into walls,” prompting her to bring him into the
Emergency Department. In the ED, he undergoes the following
head imaging.

Non Contrast Head CT

3007
Copyright © Harvard Medical School, 2018. All Rights Reserved.

Axial T2 Flair MRI

CASE 5 (cont.)
What is the most likely diagnosis and best next management
choice?

A) Meningitis; Treatment with ceftazidime, vancomycin, and


micafungin
B) Meningitis; Treatment with ceftriaxone, vancomycin, and
ampicillin
C) Brain abscess; Treatment with ceftazidime, vancomycin,
micafungin, and acyclovir
D) Ruptured brain abscess; Treatment with ceftriaxone,
vancomycin, and metronidazole with neurosurgery consultation
if symptoms do not improve with 24h of antibiotics
E) Ruptured brain abscess; Treatment with ceftriaxone,
vancomycin, and metronidazole with emergency neurosurgery
consultation

3008
Copyright © Harvard Medical School, 2018. All Rights Reserved.

The correct answer is E.


• The patient’s initial clinical syndrome of fevers, anorexia,
headache, and neck stiffness is consistent with a diagnosis of
meningitis.
• An abrupt change in symptomatology and focal neurological
symptoms are an indication for emergent head imaging (non-
contrast head CT).
• The head CT demonstrates a brain abscess with likely
surrounding vasogenic edema.
• The MRI demonstrates pus within the ventricle, indicating
rupture of the brain abscess, a neurological emergency with a
very high mortality rate
• Emergency neurosurgical consultation and broad spectrum
antibiotics are indicated in this case.

Mathisen and Johnson, Clinical Infectious Diseases, Vol. 25, No. 4, Oct., 1997

CASE 5 (cont.)
Which of the following is not indicated in the
management/workup of this patient?

A) Blood culture
B) Consideration of ventricular drainage
C) Serial lumbar punctures to evaluate opening pressure
D) TTE with bubble study
E) Careful physical examination of the sinuses and
tympanic membranes

3009
Copyright © Harvard Medical School, 2018. All Rights Reserved.

The correct answer is C.


• Blood cultures (choice A) can be helpful in identification of causative
organism of central nervous system infections by hematogenous spread.

• Emergency neurosurgical consultation for consideration ventricular drainage


(choice B) is indicated, as intracranial pressure can lead to brainstem
herniation.

• Serial lumbar punctures (choice C) to evaluate opening pressure are not


indicated in the management of these patients.
– Serial lumbar punctures are used in the management of cryptococcal meningitis.

• TTE with bubble (choice D) is indicated, as patients with cyanotic heart


disease have a higher incidence of CNS infections.
– The presence of a right-to-left intracardiac shunt in a patient with a brain abscess may be
an indication for closure of the cardiac defect.

• Careful physical examination of the sinuses and tympanic membranes


(choice E) is indicated, as the possibility of direct spread of infection from the
paranasal/frontal sinuses or from otitis media must be evaluated.

CASE 6
A 30-year-old woman with ulcerative colitis and autoimmune hepatitis
complicated by cirrhosis, ascites, and esophageal varices presents with
dyspnea and left-sided back pain. Abdominal ultrasound shows minimal
ascites and chest X-ray reveals the following:

3010
Copyright © Harvard Medical School, 2018. All Rights Reserved.

CASE 6 (cont.)
Which of the following would not be appropriate in the
evaluation and management of this pleural effusion?

A) Thoracentesis
B) Chest tube
C) Diuretics
D) TIPS (transjugular intrahepatic portosystemic
shunt)
E) Evaluation for liver transplantation

The correct answer is B.


• This patient most likely has a pleural effusion due to cirrhosis and ascites.
- Secondary to diaphragmatic defect(s), which can be microscopic.
- 85% right-sided, 13% left-sided, and 2% bilateral.

• Thoracentesis (choice A) should be performed to assess for other possible


etiologies of the pleural effusion and to rule out infection.

• Diuretics (choice C) can be used to manage the pleural effusion.

• TIPS (choice D) is used to manage refractory pleural effusions.

• Patients with hepatic hydrothorax should be evaluated for liver


transplantation (choice E).

• A chest tube (choice B) should not be placed in the setting of hepatic


hydrothorax as it can cause massive protein and electrolyte depletion,
infection, renal failure, and bleeding.
Aliment Pharmacol Ther 2004; 20 :271

3011
Copyright © Harvard Medical School, 2018. All Rights Reserved.

CASE 7
A 56-year-old woman with hypertension presents to the Emergency
Department with left lower quadrant abdominal pain and no bowel
movements for several days. Her medications include hydrochlorothiazide
and omeprazole. Her vital signs, including her blood pressure, are normal.
Her physical examination is not revealing. Abdominal CT suggests
constipation. The CT also shows a 3.5cm right adrenal lesion.

All of the following tests for the evaluation of this adrenal lesion are
appropriate except?

A) Dexamethasone suppression test


B) Plasma aldosterone and renin
C) 24 hour urine fractionated metanephrines and catecholamines
D) Cosyntropin stimulation test
E) Adrenal protocol CT or MRI

The correct answer is D.


• This is an adrenal nodule incidentally discovered during the course of
abdominal imaging performed for other reasons in a patient with mild
hypertension.

• Appropriate laboratory evaluation includes:


-Dexamethasone suppression test to evaluate for Cushing Disease (choice
A).
- 24-hour urine fractionated metanephrines and catecholamines (or
plasma free metanephrines) to evaluate for pheochromocytoma (choice
C).
- Plasma aldosterone and renin if the patient has hypertension (choice B).

• An adrenal-protocol CT or MRI (choice E) should be performed to identify


benign features or those concerning for malignancy.

A cosyntropin stimulation test (choice D) evaluates for adrenal insufficiency


and is not a standard part of the evaluate of an adrenal lesion.
Young WF. NEJM 2007; 356: 601

3012
Copyright © Harvard Medical School, 2018. All Rights Reserved.

CASE 8
A 42-year-old man with a history of morbid obesity status-post
bariatric surgery with 75 pound weight loss presents for a follow-up
visit. He complains of 5 years of progressive gait instability and
numbness and weakness in his distal extremities. He now has
trouble holding a cup and his handwriting is deteriorating. His family
history is unremarkable. He takes high vitamin supplements
including B-complex and zinc.

On examination, he has an unsteady gait, Romberg sign, spasticity in


the bilateral lower extremities, bilateral hyperreflexia, and Babinski
sign bilaterally. There is impaired vibration and position sense in the
feet. Pain and temperature sensation in the lower extremities are
normal. His labs reveal leukopenia, neutropenia, normocytic anemia,
high serum zinc, and low ceruloplasmin. Levels of Vitamin B12,
folate, homocysteine, and methylmalonic acid are normal.

CASE 8 (cont.)
Which of the following is the most likely cause of
his condition?

A) Vitamin B12 deficiency


B) Paraneoplastic polyneuropathy
C) Copper deficiency
D) Vitamin B6 toxicity
E) Lead toxicity

3013
Copyright © Harvard Medical School, 2018. All Rights Reserved.

The correct answer is C.


• The patient has findings suggestive of progressive impairment of the
corticospinal tracts and posterior columns of the spinal cord (subacute
combined degeneration).
– In a patient with history of bariatric surgery, deficiencies of either copper
or Vitamin B12 should be suspected.
– However, the Vitamin B12, methylmalonic acid, and homocysteine levels
in this case are normal, which rules out Vitamin B12 deficiency (choice A).

• Copper deficiency (choice C) affects the corticospinal tract (hyperreflexia and


Babinski sign) and posterior column (impaired vibration sensation).
– Zinc competes with copper for intestinal absorption, and the patient is
taking supplemental zinc.

• There are no signs or symptoms to suggest an underlying neoplasm (choice B).

• Vitamin B6 toxicity (choice D) causes peripheral neuropathy, but not upper


motor neuron signs. Toxicity is very rare at doses less than 100mg daily.

• Adults with lead poisoning (E) choice can have many symptoms, including a
peripheral neuropathy that manifests as wrist or ankle weakness; however,
lead toxicity does not cause upper motor neuron signs.

Neurology 2007;68(21):1843-1850.

CASE 9
A 56-year-old woman presents with dyspnea on exertion and fatigue
for 2 months. She has a history of hypertension and Stage 4 chronic
kidney disease. She has no nausea, vomiting, anorexia, or chest pain.
Her weight is stable and she is adherent to a renal diet. Her
medications include furosemide and lisinopril. Her health care
maintenance is up-to-date, including a recent colonoscopy.

On examination, her BP is 118/62. BMI is 24. Her conjunctivae are


pale. Her cardiac and lung exams are unremarkable. She has 1+
lower extremity edema. Labs reveal hemoglobin 9.6, MCV 92, eGFR
18, ferritin 190, iron 57, TIBC 257, and transferrin saturation 22%.
Urinalysis reveals 1+ protein. Her stool is guaiac negative, and
treatment with erythropoietin is begun. Four weeks later, her fatigue
and exercise tolerance has improved and her hemoglobin is now
12.6 with transferrin saturation 21%.

3014
Copyright © Harvard Medical School, 2018. All Rights Reserved.

CASE 9 (cont.)
What is the most appropriate next step in the
management of this patient?

A) Stop erythropoietin
B) Stop lisinopril
C) Change lisinopril to HCTZ
D) Add IV ferrous gluconate
E) Schedule EGD

The correct answer is A.


• Therapy with erythropoietin has been shown to effectively treat anemia
in patients with CKD and may reduce fatigue and the need for blood
transfusions.

• Normalizing hemoglobin levels with erythropoiesis stimulating agents in


patients with anemia due to CKD is associated with increased risk of
death, non-fatal MI, stroke, CHF and thrombosis.
– The FDA recommends that erythropoietin should not be used once
hemoglobin is higher than 12.
– The rate of hemoglobin increase should not exceed 1g/dL over 2 weeks.
– Therefore, treatment with the erythropoiesis stimulating agent should be
stopped (choice A).

• Iron saturation of >20% is adequate. IV iron (choice D) is not necessary.


Oral iron may be beneficial, as increased red cell production over time
may deplete iron stores in the future.

• ACE inhibitors can slow the progression of chronic kidney disease,


particularly those associated with proteinuria, and should therefore not
be stopped (choices B and C).
Lancet 2007;369(9559):381–388.

3015
Copyright © Harvard Medical School, 2018. All Rights Reserved.

CASE 10
A 44-year-old man is brought to the emergency department after brief loss of
consciousness at work lasting for approximately 30 seconds. He has had a 5-
day history of dyspnea on exertion and chest pain. On the morning of
presentation, he had difficulty walking to work because of shortness of breath
and worsening chest pain.

On presentation, his ECG demonstrates 1mm ST segment elevations in leads I,


II, and aVL. Laboratory values are notable for a troponin-T of 0.22, normal CK
and CK-MB, WBC 4.1, Hgb 7, plt 11. The LDH is elevated at 1300, and PT and
PTT are normal.

A peripheral blood smear is shown.

(Image source: library.med.utah.edu)

CASE 10 (cont.)
The most appropriate initial treatment for this
condition is:

A) Platelet transfusion
B) Cardiac catheterization
C) Rituximab
D) IVIG
E) Plasma exchange

3016
Copyright © Harvard Medical School, 2018. All Rights Reserved.

The correct answer is E.


• This is a case of thrombotic thrombocytopenic purpura (TTP), given the concurrence of
microangioapathic hemolytic anemia and thrombocytopenia without an alternative explanation.
– It is now rare to encounter all five manifestations of the classic pentad of TTP
(microangiopathic hemolytic anemia, thrombocytopenia, renal failure, neurologic changes,
and fevers).
– Plasma exchange (choice E) should be initiated even if there is uncertainty about the
diagnosis of TTP, since the risks of progressive TTP generally outweigh the risks of
exchange.

• Platelet transfusion (Choice A) is contraindicated in TTP, as it may lead to progressive


consumption of infused platelets and production of thrombi, causing worsening neurologic
symptoms and/or renal failure.

•Cardiac catheterization (Choice B) might be reasonable for this patient later in his course, but is
not the most appropriate initial treatment. The ischemic signs and symptoms are most likely due
to microthrombi in the coronary circulation, which are treated by exchange.

• Rituximab (Choice C) may be used in addition to plasma exchange for refractory or recurrent
TTP, but at present is not indicated for up-front treatment of de novo TTP.

•IVIG (Choice D) may also be used as an adjunctive therapy in TTP, but not as an alternative to
plasma exchange.

CASE 11
A 42-year-old man presents to the emergency department for evaluation one
week after an episode of severe left-sided chest pain in the setting of cocaine
use. The chest pain persisted for approximately 24 hours, and then resolved.
He has not had any further chest pain, is currently chest pain free, and has no
exam features of heart failure.

His ECG in the emergency department is shown below:

Cardiac biomarkers are notable for a normal CK and CK-MB, and an elevated
troponin-T at 13.2.

3017
Copyright © Harvard Medical School, 2018. All Rights Reserved.

CASE 11 (cont.)

CASE 11 (cont.)
A 42-year-old man presents to the emergency department for evaluation one
week after an episode of severe left-sided chest pain in the setting of cocaine
use. The chest pain persisted for approximately 24 hours, and then resolved.
He has not had any further chest pain, is currently chest pain free, and has no
exam features of heart failure.

His ECG in the emergency department is shown below:

Cardiac biomarkers are notable for a normal CK and CK-MB, and an elevated
troponin-T at 13.2.

3018
Copyright © Harvard Medical School, 2018. All Rights Reserved.

CASE 11 (cont.)
The best next step in management is:

A) Echocardiogram
B) Anticoagulation with heparin
C) Urgent cardiac catheterization
D) Clopidogrel
E) Pharmacologic stress test

The correct answer is A.


• The clinical history of chest pain in the setting of cocaine use one week ago and his ECG
suggests a missed anterior MI.
– Following a myocardial infarction, cardiac biomarkers peak in 18-24 hours.
• CK and CK-MB remain elevated for 48 hours, while troponins may remain elevated for 10
days.
• The pattern of biomarkers here is consistent with a missed MI one week ago.

• The ECG demonstrates ST elevations in leads V1-V4, suggestive of an anterior ST-elevation


MI. There are also Q-waves in leads V1-V4.

• The presence of anterior Q-waves plus persistent ST-elevations with a clinical story of a
cocaine-induced MI one week ago is suggestive of a left ventricular aneurysm.
– An echocardiogram should be performed to look for the presence of a ventricular
aneurysm and ventricular thrombus (choice A).
– Left ventricular aneurysms are common complication of anterior MIs.
– Left ventricular aneurysms are treated with afterload reduction and anticoagulation.

• Late catheterization of STEMI (after 24-48hrs) (Choice C) should only be done for severe
heart failure, electrical or hemodynamic instability, or persistent ischemia.

3019
Copyright © Harvard Medical School, 2018. All Rights Reserved.

CASE 12
A 28-year-old woman with no significant past medical history presents with
nausea and vomiting after completing her first marathon. She was able to
complete the marathon and thereafter immediately rehydrated. She took
four 200mg ibuprofen tablets and was at a post-marathon party when she
started to feel ill, saying unusual things to her friends such as, “I made a
terrible mistake” and “I am drowning.” Her friends brought her to the
emergency department. On examination, she is tired appearing and mildly
confused, and has an otherwise non-focal neurological exam. Her jugular
venous pressure is 6 cmH2O

What is the best next step in the workup and management of this patient?

A) Administration of 1L intravenous normal saline


B) Oral rehydration with an electrolyte replacement sports drink
C) STAT electrolyte panel
D) Administration of hypertonic saline at a rate of 1cc/kg/hr
E) Measurement of an ibuprofen level

The correct answer is C.


• This patient likely has symptomatic, acute hyponatremia, likely caused
by extreme loss of hypotonic fluid (sweat) from exercise and
replacement of the hypotonic fluid with free water.

• The first step in this emergency situation should be to obtain a STAT


electrolyte panel to confirm hyponatremia and to help guide further
treatment (choice C).

• In the setting of euvolemia, rehydration with normal saline and PO


hydration is inadequate to correct hyponatremia and can result in
worsening hyponatremia (choices A and B).

• Empiric administration of hypertonic saline should be avoided in a


clinical setting where labs can be obtained rapidly as correcting
hyponatremia too quickly can lead to an osmotic demyelination
syndrome or central pontine myelinolysis (choice D).

• The patient’s signs and symptoms are inconsistent with ibuprofen


overdose (choice E).

3020
Copyright © Harvard Medical School, 2018. All Rights Reserved.

CASE 13
A 36-year-old woman with depression, mild asthma, and obesity
presents with three weeks of a non-productive cough. She also
has paroxysms of coughing and post-tussive vomiting. She
denies significant wheezing. She works at a day care. She got her
vaccinations as a child. Vital signs, lung exam, complete
metabolic panel, and chest X-ray are unremarkable.

The best treatment at this time would be?

A) Albuterol inhaler
B) Azithromycin
C) Prednisone
D) Anti-tussive agents
E) Admission to the hospital for IV antibiotics

The correct answer is B.


• The clinical presentation suggests pertussis, which can be a cause
of persistent cough in adults, even those who received vaccinations as
child.
-Paroxysms of coughing and post-tussive vomiting are
suggestive of pertussis.
- The clinical course of pertussis in adults is usually less severe
than in children.
- Treatment for pertussis with a macrolide antibiotic (i.e.
azithromycin, choice B) is advised within 4 weeks of symptom
onset in order to contain the spread of infection.

• Albuterol inhaler (choice A) and prednisone (choice C) might be used


for an asthma attack, but are not indicated for this patient.

• There is no clinical indication for hospitalization or IV antibiotics for


this patient (choice E).

3021
Copyright © Harvard Medical School, 2018. All Rights Reserved.

CASE 14
A 25-year-old man who recently moved to the US from France (and had not
seen physicians there) presents with a headache, and on imaging is found to
have a superior sagittal sinus thrombosis. On examination, he is noted to be
tall and thin, and to have pectus excavatum and arachnodactyly. No
murmurs are appreciated on cardiac auscultation.

Which of the following tests would you expect to be abnormal in this


patient?

A) Activated protein C resistance


B) Fibrillin-1 mutation
C) Protein S activity
D) Homocysteine level
E) Prothrombin G20210A mutation

The correct answer is D.


• The patient likely has homocystinuria, which can be diagnosted by an elevated
homocysteine level in the blood (choice D). These patients have a clinical phenotype
that can be similar to Marfan Syndrome, but there are some key differences:
- Patients with homocystienuria can be tall and thin, and have pectus excavatum and
arachnodactyly, similar to Marfan Syndrome. However, they do not have the
cardiovascular problems associated with Marfan Syndrome.
- Patients with homocystinuria may have associated venous thromobsis, which is
not present in Marfan Syndrome.
- Patients with homocystinuria have associated neuro-psychiatric disorders, venous
thrombosis, osteoporosis, and skin hypopigmentation.
- Patients with homcystinuria have downward lens dislocation while those with
Marfan Syndrome have upward lens dislocation.
- Most patients with Marfan Syndrome have a mutation in Fibrillin-1 (choice B).

• He was likely born in an environment where newborn screening for homocystinuria


(which now commonly exists) was not done.

• Factor V Leiden (resistance to activated protein C), prothrombin gene (G20210A)


mutation, and protein S deficiency are causes of a prothrombotic state, but do not
explain his other physical findings.
http://emedicine.medscape.com/article/1115062-overview

3022
Copyright © Harvard Medical School, 2018. All Rights Reserved.

CASE 15
A 38-year-old woman with a history of diffuse cutaneous systemic sclerosis
presents with lower extremity edema for one week. Her baseline blood pressures
are 120-140/70-80. She is on nifedipine and omeprazole.

On examination, she is afebrile. Her HR is 98 and BP is 170/100. She has skin


thickening over the face, hands, arms, chest and abdomen. There are
telangiectasias on her face and palms. Her cardiac examination is notable for an
S4. Her lungs are clear. She has 2+ lower extremity edema. Laboratory evaluation
reveals hemoglobin 9.8, platelets 95, BUN 40, creatinine 2.4, and albumin 3.4.
Urinalysis shows 2+ protein and 10 RBCs/high-powered field. A peripheral blood
smear shows 2+ schistocytes.

Along with admitting the patient to the hospital, what is the most appropriate
next step in management?

A) Increase the nifedipine dose


B) Begin captopril
C) Start oral labetalol
D) Begin IV methylprednisolone
E) Start oral prednisone

The correct answer is B.


• The patient has scleroderma and now has hypertension, lower
extremity edema, renal failure with proteinuria, and
microangiopathy consistent with scleroderma renal crisis.

• The drug of choice for scleroderma renal crisis is an ACE


inhibitor with rapid titration to reduce blood pressure.
– ACE inhibitors are the most effective medication to
preserve or improve renal function in scleroderma renal
crisis.
– ACE inhibitors have shown a significant reduction in
mortality in this setting.

• Corticosteroids are not indicated (choices D and E) for


scleroderma renal crisis.

Rheum Dis Clin North Am 2003;29:315-333.

3023
Copyright © Harvard Medical School, 2018. All Rights Reserved.

CASE 16
A 36-year-old man presented to his primary care physician with a week-long
history of severe pain in his left Achilles tendon. Over the past few days, he
has also developed pain and swelling in his fingers and toes (see photographs
below). He has been having difficulty walking and bearing weight. Of note,
two weeks ago, he developed a week-long course of diarrhea accompanied by
chills and sweats following a weekend camping trip.

CASE 16 (cont.)
The most appropriate treatment is:

A) Ceftriaxone 1g IV
B) Methylprednisolone 1000mg IV
C) Prednisone 60mg PO
D) Indomethicin 50mg PO
E) Observation

3024
Copyright © Harvard Medical School, 2018. All Rights Reserved.

The correct answer is D.


• This is a presentation of reactive arthritis, which presents as an asymmetric mono/oligo-
arthritis, predominantly of lower extremity joints.
–Classically, it also presents with enthesitis (inflammation of the insertion of ligaments,
tendons, joint capsule, or fascia to bone -- typically the Achillies tendon) and dactylitis
(“sausage digits”).
–Extra-articular involvement may include urethritis, conjunctivitis, uveitis, oral ulcers, and
rashes.

• Reactive arthritis may occur following genitourinary or enteric infections caused by Chlamydia
trachomatis, Yersinia, Salmonella, Shigella, Campylobacter, and possibly Clostridium difficile.

• Treatment for reactive arthritis is with NSAIDs, such as indomethicin, for at least two weeks.

• Gonococcal arthritis (for which ceftriaxone, choice A, would be an appropriate treatment) may
present with the abrupt onset of a mono- or oligo-arthritis, but typically does not cause sausage
digits, and frequently presents with a rash.

• Intravenous (Choice B) or oral (Choice C) steroids may be used to treat numerous rheumatologic
conditions, but are not the treatment of choice for reactive arthritis.

• Observation (Choice E) is not the best choice given the severity of symptoms in this case.

CASE 17
A 39-year-old woman of Greek descent presents to the emergency
department after experiencing a brief loss of consciousness while at work.
Workup in the emergency department reveals a WBC 4, Hgb 6.5, Plt 207.

She notes that she had a viral syndrome one week ago, which subsequently
resolved. She has no history of bleeding. She recently moved into a new
house three months ago. She notes that she has had a propensity to chew ice
for the past one year. She has no family history of anemia.

Additional workup reveals: MCV 55, Iron < assay, Ferritin 1, TIBC 400, ESR 8.
Normal haptoglobin, LDH, B12, and folate levels.

Blood smear shows microcytic, hypochromic red blood cells of varying


shapes.

Of note, CBC three years ago showed a Hgb of 9.5 with an MCV of 85.

3025
Copyright © Harvard Medical School, 2018. All Rights Reserved.

CASE 17 (cont.)
The most likely diagnosis is:

A) Thalassemia
B) Iron deficiency anemia
C) Lead toxicity
D) Hemolysis
E) Anemia of chronic inflammation

The correct answer is B.


• This is a case of profound iron-deficiency anemia, as evidenced by microcytic anemia
with very low ferritin level.
• Common causes of microcytic anemia include iron deficiency, thalassemia, and chronic
inflammation (i.e. anemia of chronic disease). Rarer causes include copper deficiency,
lead poisoning, and sideroblastic anemia.
• Iron deficiency may have an insidious onset, with typical presenting symptoms including
fatigue, weakness, exercise intolerance, headache, and irritability. Additional signs and
symptoms include tongue pain, dry mouth, pica/pagophagia, and restless leg syndrome.
• Thalassemia (Choice A) typically presents with very low MCVs, as in this case, but iron
stores should be normal to increased. The prior MCV of 85 also makes thalassemia highly
unlikely, as it is an inherited disorder.
•Lead poisoning (Choice C) may cause microcytic anemia. Basophilic stippling is often
(but not always) evident on peripheral blood smear. Other manifestations include
abdominal pain, joint and muscle aches, memory problems, and irritability.
• The normal haptoglobin and LDH make the diagnosis of hemolytic anemia (Choice D)
less likely.
• Anemia of chronic disease (Choice E) typically presents with low iron and low TIBC, but
an increased ferritin. The ESR would also be expected to be elevated.

3026
Copyright © Harvard Medical School, 2018. All Rights Reserved.

CASE 18
A 19-year-old man with no significant past medical history presents with a
new, non-productive cough of five months duration. He was seen by his PCP
when the cough began, was diagnosed with bronchitis, and was treated with
a course of antibiotics. His symptoms did not improve, the cough worsened,
and it is now accompanied by dyspnea and wheezing. He feels dyspneic on
exertion and often coughs with exertional activity. Examination is notable for
an oxygen saturation of 91% on RA and scattered expiratory wheezes. WBC is
15.6 with an absolute eosinophil count of 1890/µL. Review of systems is
notable for a 25-pound weight loss in the last five months. His social history is
notable for marijuana use.

What is the most likely diagnosis?


A) Asthma
B) Chronic bronchitis
C) Eosinophilic granulomatosis with polyangiitis (Churg-Strauss Syndrome)
D) Allergic bronchopulmonary aspergillosis (ABPA)
E) Invasive aspergillosis

The correct answer is D.


• ABPA (choice D) should be suspected in a patient with new onset asthma as
an adult in the presence of peripheral eosinophilia.

• While this constellation of symptoms can also be seen in eosinophilic


granulomatosis with polyangiitis (Churg-Strauss Syndrome), the historical
feature of marijuana use heavily sways the diagnosis in favor of ABPA, as
marijuana is often contaminated with Aspergillus.

• Eosinophilic granulomatosis with polyangiitis (choice C) is characterized by


sinus disease, asthma, end-organ damage from peripheral and tissue
eosinophilia, including cardiac dysfunction and neuropathy.

• Chronic bronchitis (choice B) is unlikely to explain the patient’s eosinophilia


or end-expiratory wheezing.

• Invasive aspergillosis (choice E) is usually seen in patients with underlying


immunocompromise and is associated with fevers and occasionally with frank
hemoptysis or blood tinged sputum.

3027
Copyright © Harvard Medical School, 2018. All Rights Reserved.

CASE 18 – Part II
All of the following are reasonable next steps in the workup and
treatment of this patient except?

A) Start prednisone at a dose of 0.5-1.0 mg/kg/day


B) Check strongyloides serology
C) Check IgE level and consider initiation of omalizumab
D) Encourage patient to stop marijuana usage
E) Check for FIP1L1-PDGFRα translocation

The correct answer is C.


• The treatment for ABPA includes:
– Cessation of offending agents (choice D).
– Oral steroids (choice A).
• It is important to rule-out occult stronglyoidiasis (choice B) in a
patient with eosinophilia prior to the initiation of steroids, so as to
avoid precipitating life-threatening strongyloides hyperinfection
syndrome.

• The test for the presence of FIP1L1-PDGFRα translocation (choice E) is a


molecular diagnostic tool that is used in the diagnosis of chronic eosinophilic
leukemia.

• Omalizumab (choice C) is a monoclonal antibody directed at IgE that is


FDA-approved for the treatment of refractory asthma in the setting of an
elevated IgE level. It is not currently indicated in the treatment of ABPA.

3028
Copyright © Harvard Medical School, 2018. All Rights Reserved.

CASE 19
A 64-year-old man with hypertension presents with three days of right upper
quadrant abdominal pain. Labs show ALT 927, AST 1048, alkaline phosphatase
132, total bilirubin 7.8, direct bilirubin 5.7, WBC 8.62, Hct 42.5, platelets 270,
albumin 3.5, and INR 1.1. He denied acetaminophen ingestion and chronic alcohol
use. Hepatitis A IgM, hepatitis B surface antigen, and hepatitis B core IgM are
negative. Hepatitis C viral load is 5,041,727 with genotype 1B. Anti-neutrophil
antibody, anti-smooth muscle antibody, and testing for herpes simplex virus,
cytomegalovirus, and Epstein-Barr virus are negative. RUQ ultrasound with
Doppler shows no cirrhosis or hepatomegaly, and patent portal and hepatic veins
with normal flow. He later admitted to recent IV heroin use. Which of the
following is the best management of this patient at this time?

A) Monitor hepatitis C viral load in 3 months and consider treatment if the


virus has not not cleared
B) Tenofovir
C) Ledipasvir/sofosbuvir
D) N-acetylcysteine
E) Prednisolone

The correct answer is A.


• This patient likely has acute hepatitis C virus infection, given the recent exposure to IV
drugs, the positive viral load, and the lack of another cause for his hepatitis.

• Hepatitis C accounts for 20% of acute hepatitis in the US, but is often asymptomatic
(approximately 25% of patients present with symptoms). The RNA viral load becomes
detectable from days up to 8 weeks after infection, while the IgG serology is detectable
after 8 weeks. There is no IgM for hepatitis C.

• There is limited evidence for the treatment of acute hepatitis C, but the general
recommendations are to monitor the viral load again at 3 months (choice A), as those
patients who present with symptomatic acute hepatitis C infection are more likely to
clear the infection without treatment within the first few months.

• If treatment for chronic HCV infection is subsequently indicated, then


ledipasvir/sofosbuvir would be an option at that time (choice C).

• Tenofovir (choice B) is a treatment for hepatitis B. N-acetylcysteine is a treatment for


acetaminophen hepatotoxicity (choice D). Prednisolone (choice E) is a treatment for
alcoholic hepatitis.

J Hepatology 2005; 42: S108

3029
Copyright © Harvard Medical School, 2018. All Rights Reserved.

CASE 20
A 27-year-old woman originally from Brazil who is 25 weeks pregnant (G1P0)
presents with dyspnea, blood tinged-sputum, and pleuritic chest pain. Her HR is
86 bpm, BP is 126/73 mmHg, and O2 saturation is 82% on RA. Examination reveals
diffuse rales bilaterally and a difficult-to-auscultate, low-pitched diastolic rumble
at the apex. Electrocardiogram shows sinus tachycardia. A chest X-ray shows
diffuse bilateral infiltrates. Echocardiography reveals a normal ejection fraction, a
diffusely thickened mitral valve, moderate-to-severe mitral stenosis, and elevated
pulmonary artery systolic pressures. She is intubated for respiratory support. Fetal
ultrasound is reassuring.

The most appropriate regimen for medical management is:


A) Beta-blocker and gentle diuresis
B) Digoxin
C) Dopamine
D) ACE inhibitor
E) Hydralazine and nitrates

The correct answer is A.


• This patient likely had rheumatic fever as a child resulting in rheumatic heart
disease and mitral stenosis, which was asymptomatic until her pregnancy.

• Medical management of symptomatic mitral stenosis in pregnancy involves


beta-blockade (to slow the heart rate, which improves diastolic filling) and
gentle diuresis. If this is not adequate, percutaneous mitral valvuloplasty can
be considered.

• Positive inotropes (such as digoxin, choice B, and dopamine, choice C) should


be avoided in mitral stenosis.

• Vasodilators (such as ACE inhibitors, choice D, and hydralazine and nitrates,


choice E) are not first line agents in the setting of decompensated mitral
stenosis.

• The safety of medications during pregnancy should also be considered.


Carabello BA. Circulation. 2005;112:432-437.
Bonow RO et al. J Am Coll Cardiol. 2006 Aug 1;48(3):e1-148.

3030
Copyright © Harvard Medical School, 2018. All Rights Reserved.

CASE 21
A 29-year-old woman presents to the emergency department
with sore throat, fever, and recurrent hematuria.

She was in her usual state of health until one month ago, when
she developed a sore throat and a fever of 101 F. The following
day, she noticed frank blood in her urine and went to the
emergency department. She was diagnosed with a presumed
urinary tract infection and given a seven day course of
cephalexin. After several days of antibiotics, her fevers resolved
and her urine cleared. She remained in good health until one
week ago, when she again developed fever, sore throat and
bloody urine, and returned to the emergency department. She
notes that she had a similar episode about three years ago.

CASE 21 (cont.)
Urinalysis is notable for 3+ blood and 2+ protein.
Urine sediment shows 493 dysmorphic RBCs. No casts are seen.
Laboratory values are notable for a creatinine of 3.4. Complement
components C3 and C4 are normal.
Renal ultrasound shows no evidence of obstruction, hydronephrosis, or
perinephric fluid collection.

Which of the following is the most likely diagnosis:

A) Post-streptococcal glomerulonephritis
B) IgA nephropathy
C) Carcinoma of the bladder
D) Urinary tract infection
E) Nephrolithiasis

3031
Copyright © Harvard Medical School, 2018. All Rights Reserved.

The correct answer is B.


• IgA nephropathy is the most common cause of primary glomerulonephritis in the
developed world. It usually presents with recurrent gross hematuria, usually less
than five days after an upper respiratory infection or bacterial tonsillitis, though it
may present with microscopic hematuria and mild proteinuria.

• Post-streptococcal GN (Choice A) is an immune-complex disease from specific


nephritogenic strains of Group A streptococcus. It usually occurs one to three
weeks after pharyngitis and three to six weeks after skin infection. Children ages
5-12 and adults over 60 years of age are at highest risk. Complements are usually
decreased, and this may persist for four to eight weeks. Renal function resolves
within three to four weeks; hematuria may persist for three to six months. Unlike
IgA nephropathy, it post-streptococcal GN rarely recurs.

• Proteinuria and dysmorphic RBCs suggest glomerular (rather than extraglomerular)


pathology, making the remaining choices unlikely.
– Carcinoma of the bladder (Choice C) typically affects older patients, with a
mean age at diagnosis of around 70.
– Urinary tract infections (Choice D) and nephrolithiasis (choice E) would be
expected to present with additional signs and symptoms.

Selected References
• Altman R, Alarcón G, Appelrouth D, Bloch D, Borenstein D, Brandt K, et al.
The American College of Rheumatology criteria for the classification and
reporting of osteoarthritis of the hand. Arthritis Rheum 1990; 33: 1601–10

• Ashton HA, Buxton MJ, Day NE, et al; Multicentre Aneurysm Screening
Study Group. The Multicentre Aneurysm Screening Study (MASS) into the
effect of abdominal aortic aneurysm screening on mortality in men: a
randomised controlled trial. Lancet. 2002 Nov 16;360(9345):1531-9

• Fleming C, Whitlock EP, Beil T, Lederle F. Screening for abdominal aortic


aneurysm: a best-evidence systematic review for the U.S. Preventive
Services Task Force. Ann Intern Med 2005;142:203-11.

• LeGrand SB, Leskuski D, Zama ISO. Narrative review: furosemide for


hypercalcemia: an unproven yet common practice. Ann Intern Med.
2008;149(4):259.

• Young WF. The Incidentally Discovered Adrenal Mass. N Engl J Med 2007;
356:601-6

3032
Copyright © Harvard Medical School, 2018. All Rights Reserved.

Board Review Practice 3


Sanjay Divakaran, M.D.
Fellow, Division of Cardiovascular Medicine
Department of Medicine
Brigham and Women’s Hospital
Clinical Fellow in Medicine
Harvard Medical School
sdivakaran@bwh.harvard.edu

3033
Copyright © Harvard Medical School, 2018. All Rights Reserved.

Joslin Diabetes Center


Diabetes 2008: From Research to Clinical Practice
Hypertension, Renal Disease, and Cardiovascular Disease in Diabetes:
Current Approach to Diagnosis and Treatment

Diabetes Update 2018

ROBERT C. STANTON, MD
Chief of Kidney and Hypertension Section
Joslin Diabetes Center
Associate Professor of Medicine
Harvard Medical School

Disclosure

I have been a Consultant to Janssen


Pharmaceuticals on the Global Renal
Advisory Board.

3034
Copyright © Harvard Medical School, 2018. All Rights Reserved.

Joslin Diabetes Center


Diabetes 2008: From Research to Clinical Practice
Hypertension, Renal Disease, and Cardiovascular Disease in Diabetes:
Current Approach to Diagnosis and Treatment
Learning Objectives
Understand the Risks of Diabetes

Review the World of Medications for Diabetes

Understand the Emerging Roles of the Newer Medications –GLP1-RAs


and SGLT2 Inhibitors

Know the Role of Metformin, Lactic Acidosis Risk, and How to Dose
Metformin

Appreciate the Role of New Diabetes Technologies

Know Current Recommendations for Management of Type 2 Diabetes

- Glucose control has greatest


impact on Microvascular
disease

- Macrovascular impact of
glucose control takes longer, is
only modest, but is real; Other
approaches are more potent

3035
Copyright © Harvard Medical School, 2018. All Rights Reserved.

Joslin Diabetes Center


Diabetes 2008: From Research to Clinical Practice
Hypertension, Renal Disease, and Cardiovascular Disease in Diabetes:
Current Approach to Diagnosis and Treatment

Quality of Diabetes Care in the US


Data from:
CDC’s National
health interview
survey;
National hospital
discharge
survey;
US Renal Data
System; National
Vital statistics
System

What is the A1c Goal?

American College of Physicians Recommends 7-8% for


Most Non-Pregnant Adults with Type 2 Diabetes. And to
lower Medications if A1c is <6.5%

The American Diabetes Association Recommends <7%


for most people. But <6.5% or as high as 8% may be
appropriate.

The Best Approach: Aim for <7% but Individualize goals!


Riddle et al Diabetes Care 41:1121-1124, 2018

3036
Copyright © Harvard Medical School, 2018. All Rights Reserved.

Joslin Diabetes Center


Diabetes 2008: From Research to Clinical Practice
Hypertension, Renal Disease, and Cardiovascular Disease in Diabetes:
Current Approach to Diagnosis and Treatment

Which of the following medications are currently


most recommended as initial drugs for treatment
of newly diagnosed type 2 diabetes.

A) Metformin, Sulfonylureas, DPP-4 inhibitors


B) Metformin, SGLT-2 inhibitors, GLP-1 Receptor Agonists
C) Metformin, GLP-1 Receptor Agonists, Insulin
D) Metformin, TZDs, DPP-4 Inhibitors

Which of the following medications are currently most


recommended as initial drugs for treatment of newly
diagnosed type 2 diabetes.
A) Metformin, Sulfonylureas, DPP-4 inhibitors
B) Metformin, SGLT-2 inhibitors, GLP-1 Receptor Agonists
C) Metformin, GLP-1 Receptor Agonists, Insulin
D) Metformin, TZDs, DPP-4 Inhibitors

The answer is B. In fact, any of these medications could be used


depending on the presenting blood glucose levels and
glycohemoglobin A1c. Current guidelines recommend metformin.
And there is increased support for starting with SGLT-2 Inhibitors or
GLP-1 Receptor Agonists.

3037
Copyright © Harvard Medical School, 2018. All Rights Reserved.

Joslin Diabetes Center


Diabetes 2008: From Research to Clinical Practice
Hypertension, Renal Disease, and Cardiovascular Disease in Diabetes:
Current Approach to Diagnosis and Treatment

Garber et al Endocrine Practice 24:91-120, 2018

Lifestyle Modification in the Diabetes


Prevention Program Outcomes Study
16-session curriculum with individual sessions
Goal – 7% weight loss
• Healthy, low-fat, low-calorie diet
• 150 min per week of moderate-intensity physical activity
• After the first 24 weeks, individual and group sessions were used
to reinforce the lifestyle modification behaviours
DPPOS – Quarterly Sessions for All Participants
• Supplementary sessions for DPP Lifestyle Participants
• Twice a Year Visits.

Nathan et al The Lancet: Diabetes and Endocrinology 3:866-875, 2015

3038
Copyright © Harvard Medical School, 2018. All Rights Reserved.

Joslin Diabetes Center


Diabetes 2008: From Research to Clinical Practice
Hypertension, Renal Disease, and Cardiovascular Disease in Diabetes:
Current Approach to Diagnosis and Treatment
Lifestyle Intervention or Metformin Prevents
Development of Diabetes – 15 Year Follow Up

Placebo Metformin

Lifestyle Intervention

Nathan et al The Lancet: Diabetes and Endocrinology 3:866-875, 2015

Garber et al Endocrine Practice 24:91-120, 2018

3039
Copyright © Harvard Medical School, 2018. All Rights Reserved.

Joslin Diabetes Center


Diabetes 2008: From Research to Clinical Practice
Hypertension, Renal Disease, and Cardiovascular Disease in Diabetes:
Current Approach to Diagnosis and Treatment
Multiple Complex Pathophysiological
Abnormalities in T2DM
pancreatic
incretin insulin
effect secretion
pancreatic
glucagon
- secretion
gut
carbohydrate
delivery & HYPERGLYCEMIA
absorption

This image cannot currently be display ed.

+
hepatic renal peripheral
glucose glucose glucose
production excretion uptake
Adapted from: Inzucchi SE, Sherwin RS in: Cecil Medicine 2011

Multiple Pathophysiologically-Based
Therapies for T2DM
GLP-1R Insulin
agonists pancreatic
incretin Glinides insulin
SUs secretion
effect
DPP-4 Amylin pancreatic
inhibitors mimetics glucagon
- secretion DA
agonists
gut A G I s
carbohydrate
delivery & HYPERGLYCEMIA
absorption

Metformin TZDs
This image cannot currently be display ed.

Bile acid
-
sequestrants
+ SGLT-2
inhibitors
hepatic renal peripheral
glucose glucose glucose
production excretion uptake
Adapted from: Inzucchi SE, Sherwin RS in: Cecil Medicine 2011

3040
Copyright © Harvard Medical School, 2018. All Rights Reserved.

Joslin Diabetes Center


Diabetes 2008: From Research to Clinical Practice
Hypertension, Renal Disease, and Cardiovascular Disease in Diabetes:
Current Approach to Diagnosis and Treatment
Major Pathophysiologically-Based
Therapies for T2DM
GLP-1R Insulin
agonists pancreatic
incretin insulin
SUs secretion
effect
DPP-4 pancreatic
inhibitors glucagon
- secretion
gut
carbohydrate
delivery & HYPERGLYCEMIA
absorption

Metformin TZDs
This image cannot currently be display ed.

+ SGLT-2
inhibitors
hepatic renal peripheral
glucose glucose glucose
production excretion uptake
Adapted from: Inzucchi SE, Sherwin RS in: Cecil Medicine 2011

GLP-1R Insulin
agonists “insulin
SUs providers”
DPP-4
inhibitors
“incretin
enhancers”

Metformin
“insulin TZDs
sensitizers”

SGLT-2
inhibitors “glucose
excreter”

3041
Copyright © Harvard Medical School, 2018. All Rights Reserved.

Joslin Diabetes Center


Diabetes 2008: From Research to Clinical Practice
Hypertension, Renal Disease, and Cardiovascular Disease in Diabetes:
Current Approach to Diagnosis and Treatment

Hypoglycemia Risk

Any Medication May Cause Hypoglycemia but of Particular


Risk are Sulfonylureas and Insulin

3042
Copyright © Harvard Medical School, 2018. All Rights Reserved.

Joslin Diabetes Center


Diabetes 2008: From Research to Clinical Practice
Hypertension, Renal Disease, and Cardiovascular Disease in Diabetes:
Current Approach to Diagnosis and Treatment

Mechanism of Action of Sulfonylureas

Sola et al Archives of Medical Science 11:840-848, 2015

Hypoglycemic Risk of Sulfonylureas Increases as


GFR Declines (as Compared to Metformin)

Van Dalem et al British Medical Journal 354:i3625, 2016

3043
Copyright © Harvard Medical School, 2018. All Rights Reserved.

Joslin Diabetes Center


Diabetes 2008: From Research to Clinical Practice
Hypertension, Renal Disease, and Cardiovascular Disease in Diabetes:
Current Approach to Diagnosis and Treatment

All of the glucose lowering medication classes


below have been shown to lower cardiovascular or
stroke risk in prospective randomized trials except
one. Identify the exception.

A) SGLT-2 Inhibitors
B) GLP-1 Receptor Agonists
C) Thiazolidinedione
D) Metformin

3044
Copyright © Harvard Medical School, 2018. All Rights Reserved.

Joslin Diabetes Center


Diabetes 2008: From Research to Clinical Practice
Hypertension, Renal Disease, and Cardiovascular Disease in Diabetes:
Current Approach to Diagnosis and Treatment
All of the glucose lowering medication classes below have
been shown to lower cardiovascular or stroke risk in
prospective randomized trials except one. Identify the
exception.
A) SGLT-2 Inhibitors
B) GLP-1 Receptor Agonists
C) Thiazolidinedione
D) Metformin
The answer is D) metformin. SGLT-2 inhibitors and GLP-1 receptor
agonists have been shown to significantly lower cardiovascular risk in
multiple trials. Pioglitazone has been show to lower stroke and non-
fatal MI risk in one study. Metformin has never been shown to lower
cardiovascular or stroke risk.

Thiazolidinediones
Available since 1997.
Pioglitazone is the TZD mostly available

Mechanism: PPAR-γ activation, increased peripheral glucose


uptake, decrease lipolysis.

Dosing: daily, takes weeks-months for full effect, max effective


dose = max dose.

A1c lowering: 1-2%


Pros: efficacy, metabolic effects, daily dosing, no
hypoglycemia, ?preservation of beta-cell function.
Cardiovascular benefit shown in Proactive trial
Cons: cost, weight gain, edema/CHF, CV controversy,
fractures, urologic cancers?

Kernan et al. New England Journal of Medicine 374:1321–1331, 2016


Bril et al. Clinical Gastroenterology and Hepatology 16:558-566, 2018

3045
Copyright © Harvard Medical School, 2018. All Rights Reserved.

Joslin Diabetes Center


Diabetes 2008: From Research to Clinical Practice
Hypertension, Renal Disease, and Cardiovascular Disease in Diabetes:
Current Approach to Diagnosis and Treatment

Thiazolidinediones
Available since 1997.
Pioglitazone is the TZD mostly available

Mechanism: PPAR-γ activation, increased peripheral glucose


uptake, decrease lipolysis.
• Pioglitazone lowered progression to liver fibrosis in Nonalcoholic
Steatohepatitis (NASH)
Dosing: in weeks-months
daily, takes people withfor
Type 2 Diabetes
full effect, (Cusi, 2018)
max effective
dose = max dose.
• Lowered risk of stroke in Non-Diabetic Patients(Kernan, 2016)
• Prevented A1c
progression from Prediabetes to Diabetes (Kernan, 2016)
lowering: 1-2%
Pros: efficacy, metabolic effects, daily dosing, no
hypoglycemia, ?preservation of beta-cell function.
Cardiovascular benefit shown in Proactive trial
Cons: cost, weight gain, edema/CHF, CV controversy,
fractures, urologic cancers?

Kernan et al. New England Journal of Medicine 374:1321–1331, 2016


Bril et al. Clinical Gastroenterology and Hepatology 16:558-566, 2018

Thiazolidinediones

The Principal Risks are Edema and Weight Gain

Association Studies have Suggested Increased Bone Loss


and Fracture Risk
• (Possibly due to Decreased Osteoblast Activity)

Low Hypoglycemia Risk

Wang et al Scientific Reports &;1717; 2017


Riche and King Pharmacotherapy 30:716-727, 2010

3046
Copyright © Harvard Medical School, 2018. All Rights Reserved.

Joslin Diabetes Center


Diabetes 2008: From Research to Clinical Practice
Hypertension, Renal Disease, and Cardiovascular Disease in Diabetes:
Current Approach to Diagnosis and Treatment

TZDs and Edema

Wang et al Scientific Reports &;1717; 2017

3047
Copyright © Harvard Medical School, 2018. All Rights Reserved.

Joslin Diabetes Center


Diabetes 2008: From Research to Clinical Practice
Hypertension, Renal Disease, and Cardiovascular Disease in Diabetes:
Current Approach to Diagnosis and Treatment
GLP-1 Source

Muskiet et al Nature Reviews Nephrology 13:605-628, 2017

GLP-1 Agonists/DPP4 Inhibitors


Gut Derived Hormone GLP-1 Has Multiple Actions Including
Increasing Insulin Secretion

DPP4 is a Peptidase that Cleaves GLP-1

• Main Antidiabetic Action of DPP4 Inhibition is thought to


be due to Increasing GLP-1, but DPP4 has many
substrates so there may well be many other actions.

Mann et al New England Journal of Medicine 377:839-848, 2017


Thomas et al Diabetes Therapy 7:439-454, 2016

3048
Copyright © Harvard Medical School, 2018. All Rights Reserved.

Joslin Diabetes Center


Diabetes 2008: From Research to Clinical Practice
Hypertension, Renal Disease, and Cardiovascular Disease in Diabetes:
Current Approach to Diagnosis and Treatment

GLP-1 Actions

Muskiet et al Nature Reviews Nephrology 13:605-628, 2017

Liraglutide Reduced CV Risk


(LEADER Trial)

Marso et al New England Journal of Medicine 375:311-322, 2016

3049
Copyright © Harvard Medical School, 2018. All Rights Reserved.

Joslin Diabetes Center


Diabetes 2008: From Research to Clinical Practice
Hypertension, Renal Disease, and Cardiovascular Disease in Diabetes:
Current Approach to Diagnosis and Treatment

Semaglutide Reduced CV Risk


(SUSTAIN-6 Trial)

Marso et al New England Journal of Medicine 375:311-322, 2016

3050
Copyright © Harvard Medical School, 2018. All Rights Reserved.

Joslin Diabetes Center


Diabetes 2008: From Research to Clinical Practice
Hypertension, Renal Disease, and Cardiovascular Disease in Diabetes:
Current Approach to Diagnosis and Treatment

Effects of SGLT2 Inhibitors

*Changes Cellular
Fuel Metabolism

van Bommel et al Clinical J. Amer. Soc. Nephrol. 12:700-710, 2017


*Mudaliar et al Diabetes Care:1115-1122, 2016

EMPA-REG Improved 3-Point MACE


(CV related death, non-fatal MI and non-fatal stroke)

N=7070,
14% rel. risk
DM and reduction
CV
disease

Zinman et al New England Journal of Medicine 373:2117-2128, 2015


]

3051
Copyright © Harvard Medical School, 2018. All Rights Reserved.

Joslin Diabetes Center


Diabetes 2008: From Research to Clinical Practice
Hypertension, Renal Disease, and Cardiovascular Disease in Diabetes:
Current Approach to Diagnosis and Treatment
EMPA-REG: Improved Hospitalization for Heart Failure

Zinman et al New England Journal of Medicine 373:2117-2128, 2015

EMPA-REG Improved Cardiovascular Mortality

38% rel. risk


reduction

38
Zinman et al New England Journal of Medicine 373:2117-2128, 2015
.Re):131.
2016 Dec;16(12):131.

3052
Copyright © Harvard Medical School, 2018. All Rights Reserved.

Joslin Diabetes Center


Diabetes 2008: From Research to Clinical Practice
Hypertension, Renal Disease, and Cardiovascular Disease in Diabetes:
Current Approach to Diagnosis and Treatment

Canagliflozin Decreases CV Events:


3 Point MACE (CANVAS Trial)

Neal et al New England Journal of Medicine 377:644-657, 2017

SGLT2 Inhibitors Side Effects

The Principal Side Effects are Urinary Tract Infections and


Genital Mycotic Infections

3053
Copyright © Harvard Medical School, 2018. All Rights Reserved.

Joslin Diabetes Center


Diabetes 2008: From Research to Clinical Practice
Hypertension, Renal Disease, and Cardiovascular Disease in Diabetes:
Current Approach to Diagnosis and Treatment

SGLT2 Inhibitors Side Effects


Amputations - CANVAS Trial (Canagliflozin) Reported a Rate of
6.3/1000 patient-years for amputations (primarily toe and metatarsal)
as compared to 3.4/1000 in Control Group1
• Amputations occurred in those with preexisting peripheral vascular disease

Diabetic Ketoacidosis – Recent Publication Reports Double the Rate


of DKA in Initiation of SGLT2 Inhibitors as Compared to DPP4. But
overall rates were very low2

Bone Loss and Fracture Risk has been Reported – But overall Risk
is Unclear at this time3

1) Neal et al New England Journal of Medicine 377:644-657, 2017


2) Fralick et al New England Journal of Medicine 376:2300-2302, 2017
3) Tang et al Diabetes, Obesity, and Metabolism 18:1199-1206, 2016

Are SGLT-2 Inhibitors Renoprotective?

3054
Copyright © Harvard Medical School, 2018. All Rights Reserved.

Joslin Diabetes Center


Diabetes 2008: From Research to Clinical Practice
Hypertension, Renal Disease, and Cardiovascular Disease in Diabetes:
Current Approach to Diagnosis and Treatment

Effects of SGLT2 Inhibitors

*Changes Cellular
Fuel Metabolism

van Bommel et al Clinical J. Amer. Soc. Nephrol. 12:700-710, 2017


*Mudaliar et al Diabetes Care:1115-1122, 2016

Empagliflozin (SGLT2 Inhibitor) Slowed Decline in


eGFR
Normal Rate of GFR Decline: 0.5 – 1 ml/year

Wanner et al NEJM 375:323-334, 2016

3055
Copyright © Harvard Medical School, 2018. All Rights Reserved.

Joslin Diabetes Center


Diabetes 2008: From Research to Clinical Practice
Hypertension, Renal Disease, and Cardiovascular Disease in Diabetes:
Current Approach to Diagnosis and Treatment
Empagliflozin Decreased Glomerular Hyperfiltration in 8 Week Study in People
with Type 1 Diabetes Possibly by Altering Tubulo-Glomerular Feedback

Cherney et al Circulation 129:587-597, 2014

Thoughts about SGLT2 Inhibitors and Diabetic


Kidney Disease
SGLT2 Inhibitors May be RenoProtective
• (Secondary Analysis of EMPA-REG Cardiovascular Study)
• (Secondary Analysis of CANVAS Cardiovascular Study)
Outstanding Issues:
• Does Empagliflozin really prevent the decline in eGFR due to normal
aging as it appeared to do in EMPA-REG Study?
• Is it Safe to Have High Levels of Glucose in the Nephron for Many
Years? – No answer for 10 Years or More
• CREDENCE Study - Canagliflozin (To be Completed 2019) will answer
some of these questions
• New Empagliflozin and Dapagliflozin Studies: Chronic Kidney Disease
with and without Type 2 Diabetes Mellitus

3056
Copyright © Harvard Medical School, 2018. All Rights Reserved.

Joslin Diabetes Center


Diabetes 2008: From Research to Clinical Practice
Hypertension, Renal Disease, and Cardiovascular Disease in Diabetes:
Current Approach to Diagnosis and Treatment

Metformin

Metformin is Considered the First Drug of Choice for People


with Type 2 Diabetes Mellitus

Phenformin was taken off the market due to lactic acidosis


in 1978

Until 2016, Recommendations had been to Stop Metformin


at Serum Creatinine (>1.5 mg/dl for men and >1.4 mg/dl for
women – about an eGFR of 60 ml/min) to prevent lactic
acidosis
Garber et al Endocrine Practice 23:207-238, 2017
DeFronzo et al Metabolism 65:20-29, 2016

3057
Copyright © Harvard Medical School, 2018. All Rights Reserved.

Joslin Diabetes Center


Diabetes 2008: From Research to Clinical Practice
Hypertension, Renal Disease, and Cardiovascular Disease in Diabetes:
Current Approach to Diagnosis and Treatment
Metformin Mechanism of Action

Rena et al Diabetologia 60:1577-1585, 2017

Biochemistry of Lactate Production

DeFronzo et al Metabolism 65:20-29, 2016

3058
Copyright © Harvard Medical School, 2018. All Rights Reserved.

Joslin Diabetes Center


Diabetes 2008: From Research to Clinical Practice
Hypertension, Renal Disease, and Cardiovascular Disease in Diabetes:
Current Approach to Diagnosis and Treatment
Metformin Mechanism of Lactic Acidosis

Rena et al Diabetologia 60:1577-1585, 2017

Arguments For Metformin Use in Patients to


an eGFR of 30 ml/min
Metformin is an Excellent Antidiabetic Medication with
Multiple Beneficial Actions and Few Side Effects

Lactic Acidosis is Very Rare


– For example Systematic Review of about 150,000 patients –
Risk was 3.3/100,000 (Sulfonlyureas had 4.8 cases/100,000)

Bakris and Molitch Diabetes Care 39:1287-1291, 2016


Lu et al Annals of Pharmacology 47:1488-1497, 2013

3059
Copyright © Harvard Medical School, 2018. All Rights Reserved.

Joslin Diabetes Center


Diabetes 2008: From Research to Clinical Practice
Hypertension, Renal Disease, and Cardiovascular Disease in Diabetes:
Current Approach to Diagnosis and Treatment

Risk of Lactic Acidosis is Very Low

Inzucchi et al JAMA 312: 2668-2675, 2014

Arguments Against Metformin Use in Patients


with an eGFR of <45 ml/min
Metformin Associated Lactic Acidosis (MALA) is Very
Serious and Often Leads to Death

Conditions Predisposing to MALA are those Associated


with an Acute Decline in eGFR (Dehydration, Sepsis,
Acute Kidney Injury)
These Conditions are Common and Unpredictable

Kalantar-Zadeh and Kovesdy Diabetes Care 39:1281-1286, 2016

3060
Copyright © Harvard Medical School, 2018. All Rights Reserved.

Joslin Diabetes Center


Diabetes 2008: From Research to Clinical Practice
Hypertension, Renal Disease, and Cardiovascular Disease in Diabetes:
Current Approach to Diagnosis and Treatment

Metformin–CKD Prescribing
Guidelines (April 2016)
• Obtain eGFR before starting metformin and annually, more
frequently in those at risk for renal impairment (e.g., elderly).
• Metformin contraindicated in patients with an eGFR <30.
• Starting metformin in patients with an eGFR between 30-45 not
recommended.
• If eGFR falls <45, assess the benefits and risks of continuing
treatment. D/C if eGFR falls <30.
• Hold metformin at the time of / before iodinated contrast
procedure if eGFR 30-60; if h/o liver disease, alcoholism, or heart
failure; or if intra-arterial contrast. Recheck eGFR 48 hrs after
procedure and restart if renal function stable.
http://www.fda.gov/Drugs/DrugSafety/ucm493244.htm (accessed 4-8-16)

Recommendations
Metformin Can be Safely Used to eGFR of 30 ml/min
• If GFR is <45 ml/min maximal recommended dose is 500 mg twice
a day

Hold Metformin in Situations of eGFR Decline (Dehydration,


AKI, etc)

Hold Metformin in Situations of Increased Risk for eGFR


Decline (e.g. Contrast Dye Studies)

Stanton American Journal of Kidney Disease 66:193-195, 2015

3061
Copyright © Harvard Medical School, 2018. All Rights Reserved.

Joslin Diabetes Center


Diabetes 2008: From Research to Clinical Practice
Hypertension, Renal Disease, and Cardiovascular Disease in Diabetes:
Current Approach to Diagnosis and Treatment

Metformin
GLP-1 RA
SGLT-2i

DPP-4i
TZDs
SU/GN

Garber et al Endocrine Practice 24:91-120, 2018

Garber et al Endocrine Practice 24:91-120, 2018

3062
Copyright © Harvard Medical School, 2018. All Rights Reserved.

Joslin Diabetes Center


Diabetes 2008: From Research to Clinical Practice
Hypertension, Renal Disease, and Cardiovascular Disease in Diabetes:
Current Approach to Diagnosis and Treatment

Garber et al Endocrine Practice 24:91-120, 2018

Advances in Insulin Therapy

1. Newer basal insulins (early data show less


hypo), that are also available in concentrated
formulations (e.g., glargine U-300, degludec U-
100, U-200). Note: PD/PK change with U-300
2. Combination basal insulin + GLP-1 receptor agonist
fixed combinations (e.g. glargine + lixisenatide,
degludec + liraglutide) – insulin dose is the one
titrated. (Less hypo, no weight gain, lower insulin dose)

3. Combination basal insulin + fast acting dose


– ex: iDegAsp – one injection daily to cover
main meal + basal
4. Inhaled insulin : outcomes needed

3063
Copyright © Harvard Medical School, 2018. All Rights Reserved.

Joslin Diabetes Center


Diabetes 2008: From Research to Clinical Practice
Hypertension, Renal Disease, and Cardiovascular Disease in Diabetes:
Current Approach to Diagnosis and Treatment

Advances in Insulin Therapy


5. Increasing utilization of
continuous glucose monitoring
(CGM, or ‘sensors’) to guide
therapy in those on insulin pumps
and/or intensive insulin regimens
(mainly T1DM.). Can now reduce
need for fingersticks
6. Significant advances in the
‘artificial’ or ‘bionic’ pancreas that
links a CGM sensor to an insulin
(& glucagon?) pump (T1DM.) First
‘hybrid closed-loop pump’ available
now in US (Medtronic 670G device)

CGM and Insulin Pump Placement

https://www.niddk.nih.gov/health-information/diabetes/overview/managing-diabetes/continuous-
glucose-monitoring

3064
Copyright © Harvard Medical School, 2018. All Rights Reserved.

Joslin Diabetes Center


Diabetes 2008: From Research to Clinical Practice
Hypertension, Renal Disease, and Cardiovascular Disease in Diabetes:
Current Approach to Diagnosis and Treatment

Insulin Pump Improves Glucose Control

Hirsch and Skylar


http://diabetesmanager.pbworks.com/w/page/17680318/The%20Management%20of%20Type%201%20Di
abetes%20, 2009

CGM Provides A Large Amount of Data

Danne et al Diabetes Care 317:1631-1640, 2017

3065
Copyright © Harvard Medical School, 2018. All Rights Reserved.

Joslin Diabetes Center


Diabetes 2008: From Research to Clinical Practice
Hypertension, Renal Disease, and Cardiovascular Disease in Diabetes:
Current Approach to Diagnosis and Treatment

CGM Use Improved A1c (DIAMOND Trial)


24 Week Study

Beck et al JAMA 317:371-378, 2017

CGM and Insulin Pump Placement

https://www.niddk.nih.gov/health-information/diabetes/overview/managing-diabetes/continuous-
glucose-monitoring

3066
Copyright © Harvard Medical School, 2018. All Rights Reserved.

Joslin Diabetes Center


Diabetes 2008: From Research to Clinical Practice
Hypertension, Renal Disease, and Cardiovascular Disease in Diabetes:
Current Approach to Diagnosis and Treatment

CGM and Insulin Pump Placement

Atkinson et al The Lancet 383:69-82.2014

Diabetes Update 2018 Summary

Lifestyle Modifications and Metformin remain the mainstay of


treatment.
Trials with GLP-1RAs and SGLT-2 Inhibitors showing significant
cardiovascular risk reduction are making these likely second line
agents and possibly first line.
Weigh the risks & benefits of each agent to design a treatment regimen
individualized to each patient.
Major advances in technology are having a large impact on insulin
therapy, especially in T1DM

3067
Copyright © Harvard Medical School, 2018. All Rights Reserved.

Joslin Diabetes Center


Diabetes 2008: From Research to Clinical Practice
Hypertension, Renal Disease, and Cardiovascular Disease in Diabetes:
Current Approach to Diagnosis and Treatment

Selected References

Garber et al Consensus Statement by the American Association of Clinical Endocrinologists


and American College of Endocrinology on the Comprehensive Type 2 Diabetes Management
Algorithm Summary – 2018 Executive SummaryEndocrine Practice 24:91-120, 2018

Inzucchi SE, et al. Management of hyperglycemia in type 2 diabetes, 2016: a patient-centered


approach: Diabetes Care. 38:140-149, 2015

Zinman et al Empagliflozin, Cardiovascular Outcomes, and Mortality in Type 2 Diabetes New


England Journal of Medicine 373:2117-2128, 2015

Marso et al Liraglutide and Cardiovascular Outcomes in Type 2 Diabetes New England Journal
of Medicine 375:311-322, 2016

Riddle et al A1C Targets Should Be Personalized to Maximize Benefits While Limiting Risks
Diabetes Care 41:1121-1124, 2018

3068
Copyright © Harvard Medical School, 2018. All Rights Reserved.

Joslin Diabetes Center


Diabetes 2008: From Research to Clinical Practice
Hypertension, Renal Disease, and Cardiovascular Disease in Diabetes:
Current Approach to Diagnosis and Treatment

Diabetes: Managing Common


Complications
ROBERT C. STANTON, MD
Chief of Kidney and Hypertension Section
Joslin Diabetes Center
Associate Professor of Medicine
Harvard Medical School

Disclosures

I have served as a consultant to Janssen


Pharmaceuticals in the past 12 months.

3069
Copyright © Harvard Medical School, 2018. All Rights Reserved.

Joslin Diabetes Center


Diabetes 2008: From Research to Clinical Practice
Hypertension, Renal Disease, and Cardiovascular Disease in Diabetes:
Current Approach to Diagnosis and Treatment

Learning Objectives

Know the Complications of Diabetes Mellitus

Understand the Importance and Goals of Blood Sugar and Blood


Pressure Control for Managing Complicatios

Know How to Screen for and Manage Diabetic Complications

- Glucose control has greatest


impact on Microvascular
disease

- Macrovascular impact of
glucose control takes longer, is
only modest, but is real; Other
approaches are more potent

- Tailoring regimens to reduce


CV disease is the next frontier
in diabetes management

3070
Copyright © Harvard Medical School, 2018. All Rights Reserved.

Joslin Diabetes Center


Diabetes 2008: From Research to Clinical Practice
Hypertension, Renal Disease, and Cardiovascular Disease in Diabetes:
Current Approach to Diagnosis and Treatment

Complications of Diabetes in the United States

#1 cause of blindness in Developed Countries


#1 cause of end-stage kidney disease in U.S. (and soon
the World)– 53% of all new cases
20-30% of people with diabetes develop kidney disease
~60% to 70% of people with diabetes have mild to severe
forms of nervous system damage
Major factor leading to lower-extremity amputations
Cardiovascular disease is the cause of death in 75% to
80% of people with type 2 diabetes
Afkarian et al JAMA 2016;316(6):602-10
USRDS.ORG 2018 Annual Data Report
Diabetes Care: Standards of Medical Care - 20187

Which one of the goals below are recommended


current best practices to prevent diabetic
complications for all patients.

A) BP <130/80, A1c <8%, RAAS Inhibitor


B) BP<140/80, A1c <7%
C) BP<130/80, A1c <6%, RAAS Inhibitor
D) BP<130/80, A1c <7%

3071
Copyright © Harvard Medical School, 2018. All Rights Reserved.

Joslin Diabetes Center


Diabetes 2008: From Research to Clinical Practice
Hypertension, Renal Disease, and Cardiovascular Disease in Diabetes:
Current Approach to Diagnosis and Treatment

Which one of the groups below reflect current best


practices to prevent diabetic complications.

A) BP <130/80, A1c <8%, RAAS Inhibitor


B) BP<140/80, A1c <7%
C) BP<130/80, A1c <6%, RAAS Inhibitor
D) BP<130/80, A1c <7%

The answer is D. Current goals (although there is


some controversy) are BP<130/80 and A1c of <7%.
There is no unique role for RAAS inhibitors
although there may be good reasons to start with
them or add them early on for BP control.

No Primary Prevention of Development of


Microalbuminuria by Enalapril or Losartan

Mauer et al N Engl J Med.; 361:40-51, 2009

3072
Copyright © Harvard Medical School, 2018. All Rights Reserved.

Joslin Diabetes Center


Diabetes 2008: From Research to Clinical Practice
Hypertension, Renal Disease, and Cardiovascular Disease in Diabetes:
Current Approach to Diagnosis and Treatment

Biopsy Study Shows No Primary Prevention of


Mesangial Expansion by Enalapril or Losartan

Mauer et al N Engl J Med.; 361:40-51, 2009

Comparison of Prevention Trials for Type 2


Diabetes

Trials Showing
Prevention Benefits had
Higher Baseline BPs

Bilous et al Ann Intern Med.151:11-20, 2009

3073
Copyright © Harvard Medical School, 2018. All Rights Reserved.

Joslin Diabetes Center


Diabetes 2008: From Research to Clinical Practice
Hypertension, Renal Disease, and Cardiovascular Disease in Diabetes:
Current Approach to Diagnosis and Treatment

Diabetic Complications: Blood Sugar Control

25 Year Follow Up of DCCT/EDIC (Type 1 Diabetes) –


50% Reduction of Microalbuminuria in Original
Intensively Treated Group

De Boer
Diabetes Care
37: 24-30,
2014

3074
Copyright © Harvard Medical School, 2018. All Rights Reserved.

Joslin Diabetes Center


Diabetes 2008: From Research to Clinical Practice
Hypertension, Renal Disease, and Cardiovascular Disease in Diabetes:
Current Approach to Diagnosis and Treatment
25 Year Follow Up of DCCT/EDIC (Type 1 Diabetes) –
50% Reduction of Microalbuminuria in Original
Intensively Treated Group

De Boer
Diabetes Care
37: 24-30,
2014

30 Year Follow Up of DCCT/EDIC –


32% Reduction in MACE (CV Outcomes) in
Original Intensively Treated Group

DCCT/EDIC Study Group. Diabetes Care 39: 686-693, 2016

3075
Copyright © Harvard Medical School, 2018. All Rights Reserved.

Joslin Diabetes Center


Diabetes 2008: From Research to Clinical Practice
Hypertension, Renal Disease, and Cardiovascular Disease in Diabetes:
Current Approach to Diagnosis and Treatment
10 Year Follow Up of UKPDS (Type 2 Diabetes)

Holman et al New England Journal of Medicine 359: 1577-1589, 2008

10 Year Follow Up of UKPDS (Type 2 Diabetes)

Holman et al New England Journal of Medicine 359: 1577-1589, 2008

3076
Copyright © Harvard Medical School, 2018. All Rights Reserved.

Joslin Diabetes Center


Diabetes 2008: From Research to Clinical Practice
Hypertension, Renal Disease, and Cardiovascular Disease in Diabetes:
Current Approach to Diagnosis and Treatment
10 Year Follow Up of UKPDS (Type 2 Diabetes)

Holman et al New England Journal of Medicine 359: 1577-1589, 2008

A1c Goal

Aim for <7% but Individualize goals!

UKPDS vs ACCORD and ADVANCE

• ACCORD/ADVANCE Participants were 8 and 12 years older than


UKPDS and were Considered to be at High Risk

• UKPDS were newly diagnosed

3077
Copyright © Harvard Medical School, 2018. All Rights Reserved.

Joslin Diabetes Center


Diabetes 2008: From Research to Clinical Practice
Hypertension, Renal Disease, and Cardiovascular Disease in Diabetes:
Current Approach to Diagnosis and Treatment

Diabetic Complications: Blood Pressure


Control

Blood Pressure Goal

JNC 8 Recommended <140/90


(As did Many Other Organizations Until Recently)

Goal Blood Pressure IS:


<130/80

3078
Copyright © Harvard Medical School, 2018. All Rights Reserved.

Joslin Diabetes Center


Diabetes 2008: From Research to Clinical Practice
Hypertension, Renal Disease, and Cardiovascular Disease in Diabetes:
Current Approach to Diagnosis and Treatment
ACCORD Blood Pressures

Accord Study Group. N Engl J Med;362:1575-1585, 2010

ACCORD Trial: No Benefit of Tight BP


Control on Cardiovascular Outcomes

Accord Study Group. N Engl J Med;362:1575-1585, 2010

3079
Copyright © Harvard Medical School, 2018. All Rights Reserved.

Joslin Diabetes Center


Diabetes 2008: From Research to Clinical Practice
Hypertension, Renal Disease, and Cardiovascular Disease in Diabetes:
Current Approach to Diagnosis and Treatment

Low Blood Pressures Appear to Increase


Cardiovascular Mortality

Cooper-DeHoff RM et al JAMA 304: 61-68, 2010

SPRINT – Systolic Blood Pressure Intervention Trial - (Improved CV


Outcomes in Intensive BP Group (<120 mmHg) vs Less Intense
Control (<140 mmHg) in Subset with CKD

COMBINED CV OUTCOMES
SPRINT Participant
Characteristics:

>75 yo

Increased CV Risk

No Diabetes ALL CAUSE MORTALITY


Mellitus

Cheung et al J Am Soc Nephrol 28: 2812-2823, 2017

3080
Copyright © Harvard Medical School, 2018. All Rights Reserved.

Joslin Diabetes Center


Diabetes 2008: From Research to Clinical Practice
Hypertension, Renal Disease, and Cardiovascular Disease in Diabetes:
Current Approach to Diagnosis and Treatment
Analysis of ACCORD Study (Type 2 DM Participants) Study Using
SPRINT Criteria Show Improved CV Outcomes in Intensive BP
Group (<120 mmHg) vs Less Intense Control (<140 mmHg)

COMBINED CV OUTCOMES:
CV Death, Nonfatal MI, Nonfatal Stroke

Buckley et al Diabetes Care September 25, EPUB, Ahead of Print, 2017

ALL OFFICE BLOOD PRESSURES (INCLUDING


THOSE TAKEN IN MY OFFICE) ARE WRONG
PROPER WAY TO TAKE BLOOD PRESSURE:

RESTING FOR AT LEAST 5 MINUTES

USE PROPER CUFF SIZE

USE BARE ARM RESTING AT HEART LEVEL

TAKE MULTIPLE MEASUREMENTS AND AVERAGE


RESULTS

CHECK ORTHOSTATIC BLOOD PRESSURE


Drawz and Ix J Am Society of Nephrology October 19 EPUB, Ahead of Print, 2017
DeBoer et al Diabetes Care 40:1273-1284, 2017

3081
Copyright © Harvard Medical School, 2018. All Rights Reserved.

Joslin Diabetes Center


Diabetes 2008: From Research to Clinical Practice
Hypertension, Renal Disease, and Cardiovascular Disease in Diabetes:
Current Approach to Diagnosis and Treatment
What is the Initial Drug Choice for Hypertension
Control in Diabetes?
Individualize!!

It will take at least 2 likely 3 or more medications to reach


goal

ADA Recommends Using a Medication that Lowers CV Risk


– ACE Inhibitors, ARBs, Dihydropyridine Calcium Channel
Blockers, or Thiazide-Like Diuretics. But if there is
increased urine albumin then start with ACE-I or ARB

Choose a medication on the following:


• Cost
• Age
• Co-morbidities
• Side Effects
Diabetes Care 41:Supplement 1, 2018

Diabetic Kidney Disease

3082
Copyright © Harvard Medical School, 2018. All Rights Reserved.

Joslin Diabetes Center


Diabetes 2008: From Research to Clinical Practice
Hypertension, Renal Disease, and Cardiovascular Disease in Diabetes:
Current Approach to Diagnosis and Treatment

Quality of Diabetes Care in the US


Data from:
CDC’s National
health interview
survey;
National hospital
discharge
survey;
US Renal Data
System;
National Vital
statistics System

29

Numbers of Patients on Dialysis-USA: 1992

www.usrds.org

3083
Copyright © Harvard Medical School, 2018. All Rights Reserved.

Joslin Diabetes Center


Diabetes 2008: From Research to Clinical Practice
Hypertension, Renal Disease, and Cardiovascular Disease in Diabetes:
Current Approach to Diagnosis and Treatment

Numbers of Patients on Dialysis-USA: 2002

www.usrds.org

vol 2 Figure 1.9 Map of the adjusted prevalence of ESRD, by Health


Service Area, in the U.S. population, 2011-2015*

Data Source: Special analyses, USRDS ESRD Database. Standardized for age, sex, and race. The standard population was
the U.S. population in 2011. *Three Health Service Areas were suppressed because the ratio of unadjusted rate to
adjusted rate or adjusted rate to unadjusted rate was greater than 3. Values for cells with 10 or fewer patients are
suppressed. Abbreviation: ESRD, end-stage renal disease.

2017 Annual Data Report


32
Volume 2, Chapter 1

3084
Copyright © Harvard Medical School, 2018. All Rights Reserved.

Joslin Diabetes Center


Diabetes 2008: From Research to Clinical Practice
Hypertension, Renal Disease, and Cardiovascular Disease in Diabetes:
Current Approach to Diagnosis and Treatment
Figure 1.16 Trends in adjusted* prevalence (per million) of ESRD, by primary cause of
ESRD, in the U.S. population, 1996-2014

DIABETES

HYPERTENSION

GLOMERULONEPHRITIS

CYSTIC KIDNEY DISEASE

Data Source: Reference Table B.2(2) and special analyses, USRDS ESRD Database. *Point prevalence on December 31 of each
year. Adjusted for age, sex, and race. The standard population was the U.S. population in 2011. Abbreviation: ESRD, end-stage
renal disease.

2016 Annual Data Report, Vol 2, ESRD, Ch 1 33

vol 2 Figure 1.11 Trends in adjusted prevalence of ESRD, by race, in


the U.S. population, 2000-2015
NATIVE HAWAIIAN/PACIFIC ISLANDER

AFRICAN-AMERICAN

AMERICAN INDIAN/ALASKA NATIVES


ASIAN
WHITE

Data Source: Reference Table B.2(2) and special analyses, USRDS ESRD Database. Point prevalence on December 31 of
each year. Standardized for age and sex. The standard population was the U.S. population in 2011. Abbreviations NH/PI:
Native Hawaiian/Pacific Islander; AI/AN: Americans Indian/Alaska Natives; ESRD, end-stage renal disease.

2017 Annual Data Report


34
Volume 2, Chapter 1

3085
Copyright © Harvard Medical School, 2018. All Rights Reserved.

Joslin Diabetes Center


Diabetes 2008: From Research to Clinical Practice
Hypertension, Renal Disease, and Cardiovascular Disease in Diabetes:
Current Approach to Diagnosis and Treatment
vol 2 Figure 1.12 Trends in the adjusted prevalence of ESRD, by
Hispanic ethnicity, in the U.S. population, 2000-2015

HISPANIC

NON- HISPANIC

Data Source: Reference Tables B.1, B.2(2). Point prevalence on December 31 of each year. Standardized for age, sex,
and race. The standard population was the U.S. population in 2011. Abbreviation: ESRD, end-stage renal disease.

2017 Annual Data Report


35
Volume 2, Chapter 1

Diabetic Kidney Disease: 2015 Data


About 14.8% of the U.S. population has chronic kidney disease (CKD) as
defined as GFR of < 60 mL/min or elevated urine albumin level

Most CKD patients die of heart disease before reaching end stage kidney
disease

About 700,000 patients are on dialysis or have a kidney transplant

Death rates are 15-20% per year for dialysis patients

Medicare spent $37 billion in 2015 on ESRD , which is about 7.3% of the
Medicare budget. And an additional $50 billion was spent on CKD
Patients

www.usrds.org

3086
Copyright © Harvard Medical School, 2018. All Rights Reserved.

Joslin Diabetes Center


Diabetes 2008: From Research to Clinical Practice
Hypertension, Renal Disease, and Cardiovascular Disease in Diabetes:
Current Approach to Diagnosis and Treatment

Excess Mortality in Type 2 Diabetes is Due to


Kidney Disease

Afkarian et al J Am Soc Nephrol 24: 302–308, 2013

Diagnose Kidney Disease Early:


Measure eGFR and Spot Urine
Albumin/Creatinine Ratio
(microalbumin test)
Yearly
Use Nephelometry to Measure
Microalbumin as Dipsticks are Relatively
Insensitive and May Miss Low (but
significant) Increases in the Urine
Abumin Level

3087
Copyright © Harvard Medical School, 2018. All Rights Reserved.

Joslin Diabetes Center


Diabetes 2008: From Research to Clinical Practice
Hypertension, Renal Disease, and Cardiovascular Disease in Diabetes:
Current Approach to Diagnosis and Treatment

Value of Graphing GFR


Creatinine was 1.0 in 2004 and now 1.5
GFR Declined from 60 to 35 over 9 Years

-2.8 ml/min/year

People with Diabetes and Kidney


Disease May NOT have Diabetic
Kidney Disease

3088
Copyright © Harvard Medical School, 2018. All Rights Reserved.

Joslin Diabetes Center


Diabetes 2008: From Research to Clinical Practice
Hypertension, Renal Disease, and Cardiovascular Disease in Diabetes:
Current Approach to Diagnosis and Treatment

Reasons to Consider Kidney Diseases Other


than Diabetes in People with Diabetes
Type 1 Diabetes Duration of <5 years
Absence of Diabetic Retinopathy (especially in Type 1 DM
Persistently Low Hemoglobin A1c
Active Urine Sediment (especially many RBCs or WBCs)
Rapidly Declining GFR
Rapidly Increasing or Very High Urine Protein Level

Any of These May Be an Indication for Kidney Biopsy

Liang et al PLOS ONE 1 May 2013 | Volume 8 | Issue 5 e64184

Causes of Non-Diabetic Kidney Disease in


Patients with Diabetes

Clin J Am Soc Nephrol 8: 1718–1724, 2013

3089
Copyright © Harvard Medical School, 2018. All Rights Reserved.

Joslin Diabetes Center


Diabetes 2008: From Research to Clinical Practice
Hypertension, Renal Disease, and Cardiovascular Disease in Diabetes:
Current Approach to Diagnosis and Treatment
Albuminuria/Proteinuria
Normal total protein renal excretion is about 200
mg/day (up to 20 mg is albumin) – Most of the
protein comes from tubular cells – Intact Albumin
is from Glomerular Filtration

Normal Albumin Excretion by Nephelometry is an


Albumin/Creatinine Ratio of <30 mg/g

Microalbuminuria – 30–300 mg/g

Macroalbuminuria – >300 mg/g

Increasing Albuminuria and Decreasing GFR


Are Associated with Increased
Cardiovascular and Renal Events in Type 2
Diabetes

Ninomiya T et al J. Am. Soc. Neph. 20:1813-1819 (2009)

3090
Copyright © Harvard Medical School, 2018. All Rights Reserved.

Joslin Diabetes Center


Diabetes 2008: From Research to Clinical Practice
Hypertension, Renal Disease, and Cardiovascular Disease in Diabetes:
Current Approach to Diagnosis and Treatment

Albuminuria appears to be a marker of


generalized inflammation and
endothelial dysfunction.

Prevention of Diabetic Kidney Disease

Blood Glucose Control


• Goal is A1c <7%

Blood Pressure Control


• Goal is <130/80

Stop Smoking

Measure eGFR and urine albumin/creatinine ratio at least


Yearly

3091
Copyright © Harvard Medical School, 2018. All Rights Reserved.

Joslin Diabetes Center


Diabetes 2008: From Research to Clinical Practice
Hypertension, Renal Disease, and Cardiovascular Disease in Diabetes:
Current Approach to Diagnosis and Treatment
Treatment of Diabetic Kidney Disease
Blood Pressure Goal - <130/80

Blood Sugar Control - A1c < 7.0%

Lower Urine Albumin – Use RAAS Inhibitors


(Especially if >300mg/g)

Stop Smoking

Measure eGFR and urine albumin/creatinine ratio at


Every Visit

All of the following are effects of ACE Inhibitors


and Angiotensin Receptor Blockers except one.
Identify the exception.

A) Decrease Blood Pressure


B) Lower Urine Albumin Level
C) Increase eGFR
D) Increase Potassium

3092
Copyright © Harvard Medical School, 2018. All Rights Reserved.

Joslin Diabetes Center


Diabetes 2008: From Research to Clinical Practice
Hypertension, Renal Disease, and Cardiovascular Disease in Diabetes:
Current Approach to Diagnosis and Treatment

All of the following are effects of ACE Inhibitors


and Angiotensin Receptor Blockers except one.
Identify the exception.

A) Decrease Blood Pressure


B) Lower Urine Albumin Level
C) Increase eGFR
D) Increase Potassium
ACE Inhibitors and ARBs lower GFR
(increase creatinine) by causing vasodilation
of the efferent arteriole

The greater the initial decline in GFR after


starting Losartan, the slower the rate of
decline in long term GFR

Holtkamp FA et al Kidney International 80:282-287, 2011

3093
Copyright © Harvard Medical School, 2018. All Rights Reserved.

Joslin Diabetes Center


Diabetes 2008: From Research to Clinical Practice
Hypertension, Renal Disease, and Cardiovascular Disease in Diabetes:
Current Approach to Diagnosis and Treatment

Use Of ACE Inhibitors/ARBS

There is no unique role for ACE inhibitors/ARBs for


prevention of diabetic kidney disease.

For patients with diabetic kidney disease - All patients


with increased urine albumin (generally a urine
albumin/creatinine ratio >300 mg/g) should be on an
ACE-inhibitor or an Angiotensin Receptor Blocker even if
the patient has excellent blood pressure

Eye Diseases and Diabetes

Nonproliferative diabetic retinopathy (NPDR)


Proliferative diabetic retinopathy (PDR)
Glaucoma at earlier age
Cataract formation at earlier age
Retinal detachment
Macular edema more common

Best prevention: A1C, BP, LDL in target; yearly dilated eye


exams and early intervention
Recent Study – 60% of People with Diabetes do not have
Yearly Exams
Beaser RS and the Staff of Joslin Diabetes Center. Joslin’s Diabetes Deskbook: A Guide
for Primary Care Providers, 3rd Edition. Boston: Joslin Diabetes Center, 2014.
Murchison et al. Nonadherence to Eye Care in People with Diabetes. Abstract Presented
at American Academy of Ophthalmology Annual Meeting 2016.

3094
Copyright © Harvard Medical School, 2018. All Rights Reserved.

Joslin Diabetes Center


Diabetes 2008: From Research to Clinical Practice
Hypertension, Renal Disease, and Cardiovascular Disease in Diabetes:
Current Approach to Diagnosis and Treatment

Diabetes and Neuropathy

Symptoms are usually in the feet first, and can consist of:
• Numbness, paresthesias to severe pain
• Mild weakness
• Loss of position and vibratory sensation
• Sleep disturbance, depression
Examine Feet for Signs of Neuropathy – Use a filament
Treatment is Mainly for Pain
• ADA Recommends pregabalin or duloxetine as initial treatment

Beaser RS and the Staff of Joslin Diabetes Center. Joslin’s Diabetes Deskbook: A Guide for
Primary Care Providers, 3rd Edition. Boston: Joslin Diabetes Center, 2014.
Diabetes Care 41:Supplement 1, 2018

Diabetes and Foot Problems:


Factors Increasing Risk
High blood glucose levels
Prior ulcerations or amputations
PVD, neuropathy, retinopathy
Smoking
Structural deformities
Can’t see, feel, or reach the foot

Best Prevention: 50% of amputations can be


prevented by protecting the feet, daily foot
care, daily foot checks, referrals for problem
feet
Beaser RS and the Staff of Joslin Diabetes Center. Joslin’s Diabetes
Deskbook: A Guide for Primary Care Providers, 3rd Edition. Boston: Joslin
Diabetes Center, 2014.

3095
Copyright © Harvard Medical School, 2018. All Rights Reserved.

Joslin Diabetes Center


Diabetes 2008: From Research to Clinical Practice
Hypertension, Renal Disease, and Cardiovascular Disease in Diabetes:
Current Approach to Diagnosis and Treatment
Filament Exam

Boulton et al Diabetes Care 31:1679-1685, 2008

Other Complications

Sexual dysfunction in men and women


• Marker for high risk of heart disease in men,
associated with vascular problems
Gastroparesis
• Slowing of the emptying of the stomach
Autonomic dysfunction
• Orthostatic hypotension

3096
Copyright © Harvard Medical School, 2018. All Rights Reserved.

Joslin Diabetes Center


Diabetes 2008: From Research to Clinical Practice
Hypertension, Renal Disease, and Cardiovascular Disease in Diabetes:
Current Approach to Diagnosis and Treatment
Smoking and Diabetes
People who smoke are 40%–60 % more likely
to get diabetes than people who do not smoke
Smoking with diabetes
• Increases the risk of severe kidney damage
and nerve problems
• Greatly increases the risk of heart attack,
stroke, and foot amputations
• Increases insulin resistance and blood
glucose levels
• Worsens gum disease and tooth loss

Willi, et al. JAMA. 2007;298:2654-2664. | CDC, Smoking and Diabetes


http://www.cdc.gov/tobacco/campaign/tips/diseases/diabetes.html?mobile=false | Demmer, et al. Diabetes Care.
2012;35:2036-2042

Management Issues for Macrovascular


Disease: CV Disease, Stroke, and Peripheral
Vascular Disease in Type 2 Diabetes
Dyslipidemia (low HDL, high triglycerides)
Hypertension (2½ times more common in diabetes)
Nephropathy
Obesity/sedentary lifestyle
Altered coagulation, platelet function, and fibrinolysis
Hyperinsulinemia/hyperproinsulinemia/insulin resistance
Cigarette smoking
Hyperglycemia
Bierman EL. Arterioscler Thromb. 1992;12:647-656.

3097
Copyright © Harvard Medical School, 2018. All Rights Reserved.

Joslin Diabetes Center


Diabetes 2008: From Research to Clinical Practice
Hypertension, Renal Disease, and Cardiovascular Disease in Diabetes:
Current Approach to Diagnosis and Treatment

Reducing Risk: Changing One Risk Factor Helps

Controlling A1C BP < 130/80 LDL-C Microalbumin Smoking


< 7% < 100
Reduction in

Heart/ related
Circulation

Nephropathy
related

Retinopathy
related related

Neuropathy
related

Copyright © 2017 by Joslin Diabetes Center. All Rights Reserved.

Office Management of Diabetic Complications Summary

Blood pressure control <130/80


• Check Orthostatic Blood Pressure

Blood sugar control (A1c <7%)


Manage Cardiovascular Risk Factors – Screen for CVD/PVD
Yearly Dilated Eye Exams
Calculate eGFR and Measure Urine Albumin/Creatinine Ratio
at least Yearly
Foot Exam at least Yearly – Use Filament
Screen for Neuropathy, Sexual Dysfunction, and
Gastroparesis
Stop smoking

3098
Copyright © Harvard Medical School, 2018. All Rights Reserved.

Joslin Diabetes Center


Diabetes 2008: From Research to Clinical Practice
Hypertension, Renal Disease, and Cardiovascular Disease in Diabetes:
Current Approach to Diagnosis and Treatment

Selected References
Standards of Medical Care. Diabetes Care Volume 14 (Supplement
1), 2018
www.USRDS.org
Buckley LF et al. Intensive versus standard blood pressure control
in SPRINT-eligible participants of ACCORD-BP. Diabetes Care
40:1733–1738, 2017
De Boer et al Kidney Disease and Related Findings in the Diabetes
Control and Complications Trial/Epidemiology of Diabetes
Interventions and Complications Study. Diabetes Care 37:24-30,
2015
Holman et al 10-Year Follow-up of Intensive Glucose Control in
Type 2 Diabetes. New England Journal of Medicine 359: 1577-1589,
2008

3099
Copyright © Harvard Medical School, 2018. All Rights Reserved.

Thyroid Disease
Psychiatry Overview
Matthew Kim, M.D.
Clinical Director
Division of Endocrinology, Diabetes and Hypertension
Brigham and Women’s Hospital
Assistant Professor
Harvard Medical School

No disclosures to report

3100
Copyright © Harvard Medical School, 2018. All Rights Reserved.

Topics
• Hypothyroidism
• Hyperthyroidism
• Non‐thyroidal illness
• Thyroiditis
• Thyroid nodules


TRH

+
T4 T3

TSH

T4 T3
+

3101
Copyright © Harvard Medical School, 2018. All Rights Reserved.

T4

1, 5’ ‐ Deiodinase

T3

0.03% T4 0.1% T3

T4 T3
T4 T3
T4 T3
Thyroxine‐Binding Thyroxine‐Binding
Globulin T4 Globulin T3

99.97% T4 99.9% T3
T4 T3
T4 T3

Thyroxine‐Binding
T4 T3
Prealbumin
T4 Albumin T3
Albumin T4 T3

3102
Copyright © Harvard Medical School, 2018. All Rights Reserved.

Hypothyroidism

Hypothyroidism: Etiologies
• Iodine deficiency
• Autoimmune thyroiditis 95%
• Post‐ablative hypothyroidism
• Post‐surgical hypothyroidism 4%
• Drug‐mediated inhibition
• Insufficient TRH and/or TSH secretion
• Congenital absence of thyroid tissue < 1%
• Resistance to thyroid hormone

3103
Copyright © Harvard Medical School, 2018. All Rights Reserved.

Autoimmune Thyroiditis
• Hashimoto’s thyroiditis, chronic lymphocytic 
thyroiditis
• Marked female predominance, increasing 
prevalence with age 
• Associated with increased rates of infertility and 
miscarriage
• Association with other autoimmune disorders
– Type 1 diabetes
– Addison’s disease
– Vitiligo
– Premature gray hair

Hypothyroidism: Diagnosis
1. Check a TSH level
• Normal = euthyroid
• Elevated = primary hypothyroidism
• Low = possible secondary hypothyroidism

3104
Copyright © Harvard Medical School, 2018. All Rights Reserved.

Hypothyroidism: Diagnosis
2. If the TSH level is elevated, check a total T4
or free T4 level
• Elevated TSH: low total T4 and/or free T4 = 
primary hypothyroidism
• Elevated TSH: normal total T4 and free T4 = 
subclinical (mild) hypothyroidism

Hypothyroidism: Diagnosis
3. If the TSH level is low and you suspect 
hypothyroidism, check a free T4 level
• Low TSH: low free T4 = secondary 
hypothyroidism

3105
Copyright © Harvard Medical School, 2018. All Rights Reserved.

Hypothyroidism: Diagnosis
4. If no obvious underlying cause for primary 
hypothyroidism is evident, check anti‐thyroid 
peroxidase and/or anti‐thyroglobulin 
antibody titers
• Elevated anti‐thyroid peroxidase and/or anti‐
thyroglobulin antibody titers = autoimmune 
thyroiditis

Hypothyroidism: Treatment
• Levothyroxine (Synthroid®, Levoxyl®, Unithroid®, 
Levothroid®, Tirosint®)
• Taken once daily
• Starting dose
– Healthy young adults  0.8 mcg/lb daily
– Age > 60 25‐50 mcg daily
– Cardiac ischemia 12.5‐25 mcg daily
• Reduce daily dose by 25% when administered IV
• Check a TSH level 6 weeks after starting or 
adjusting a dose

3106
Copyright © Harvard Medical School, 2018. All Rights Reserved.

Hypothyroidism: Treatment
• Subclinical (mild) hypothyroidism
– Indications for treatment are controversial 
– Usually treated if
• TSH > 10‐15 mIU/L
• Concomitant hypercholesterolemia
• Pregnancy
– Doses usually need to be increased during early 
stages of pregnancy
– Try to maintain TSH levels between 0.5‐2.5 mIU/L 
during the first trimester

Hypothyroidism: Treatment
• Agents that block absorption
– Iron sulfate
– Sucralfate
– Bile acid resins (Welchol®, cholestyramine)
• Agents that increase metabolism
– Rifampin
– Phenytoin
– Phenobarbital
– Carbamazepine

3107
Copyright © Harvard Medical School, 2018. All Rights Reserved.

Hyperthyroidism

Hyperthyroidism: Definitions
• Thyrotoxicosis – A systemic syndrome 
characterized by exposure to excessive levels 
of thyroid hormone
• Hyperthyroidism – A systemic syndrome 
characterized by exposure to excessive levels 
of thyroid hormone produced by overactive 
functional thyroid tissue

3108
Copyright © Harvard Medical School, 2018. All Rights Reserved.

Thyrotoxicosis

Hyperthyroidism

Hyperthyroidism: Etiologies
• Graves’ disease 88%
• Toxic adenoma
• Toxic multinodular goiter 11%
• Iodine exposure
• Struma ovarii
< 1%
• TSH‐secreting pituitary adenoma

3109
Copyright © Harvard Medical School, 2018. All Rights Reserved.

Non‐Hyperthyroid Thyrotoxicosis: 
Etiologies
• Subacute thyroiditis
• Autoimmune thyroiditis

Graves’ Disease
• Marked female predominance
• Commonly presents between 15‐35 years of 
age
• Complications
– Thyroid eye disease 
(Graves’ ophthalmopathy)
– Dermopathy (pretibial myxedema)
– Acropachy

3110
Copyright © Harvard Medical School, 2018. All Rights Reserved.

Toxic Adenoma
• Hyperthyroidism caused by growth of a single 
autonomously functioning hyperplastic 
thyroid nodule
• Usually large

3111
Copyright © Harvard Medical School, 2018. All Rights Reserved.

Toxic Multinodular Goiter
• Hyperthyroidism caused by growth of multiple 
autonomously functioning hyperplastic 
thyroid nodules
• May develop in the setting of a previously 
euthyroid multinodular goiter
• Nodules may vary in size
• Hyperthyroidism may be precipitated by 
exposure to iodine (Jod‐Basedow 
phenomenon) 

3112
Copyright © Harvard Medical School, 2018. All Rights Reserved.

Hyperthyroidism: Diagnosis
1. Check a TSH level
• Normal = euthyroid
• Low = thyrotoxicosis

3113
Copyright © Harvard Medical School, 2018. All Rights Reserved.

Hyperthyroidism: Diagnosis
2. If the TSH level is low and you suspect 
hyperthyroidism, check a total T4 or free T4
level and a total T3 level
• Low TSH: elevated total T4, free T4, or total T3 = 
thyrotoxicosis
• Low TSH: normal total T4, free T4, and total T3 = 
subclinical (mild) thyrotoxicosis

Hyperthyroidism: Diagnosis
3. Check for clinical evidence of complications 
of Graves’ disease
• Goiter
• Symptoms and signs of thyroid eye disease
• Dermopathy
• Acropachy

3114
Copyright © Harvard Medical School, 2018. All Rights Reserved.

Hyperthyroidism: Diagnosis
4. If no complications of Graves’ disease are 
clinically evident, consider checking anti‐TSH 
receptor antibodies or radionuclide testing 
with a thyroid uptake and scan

Anti‐TSH Receptor Antibodies
• Thyroid stimulating immunoglobulin (TSI)
– Bioassay that measures activation of cells that 
express TSH receptors
• Thyrotropin‐binding inhibitory 
immunoglobulin (TBII)
– Immunoassay that detects antibodies that bind to 
TSH receptors

3115
Copyright © Harvard Medical School, 2018. All Rights Reserved.

Radionuclide Testing
• Thyroid uptake study
– Used to measure level of activity
– High uptake (usually > 25%) = increased iodine uptake 
and organification consistent with hyperthyroidism
– Low uptake = non‐hyperthyroid thyrotoxicosis
• Thyroid scan
– Generates images that reflect distribution of activity
– Can help to distinguish between causes of 
hyperthyroidism

Graves’ disease

3116
Copyright © Harvard Medical School, 2018. All Rights Reserved.

Toxic adenoma

Toxic multinodular goiter

3117
Copyright © Harvard Medical School, 2018. All Rights Reserved.

Hyperthyroidism: Management
• Beta blockers
– Atenolol: 25‐50 mg daily
– Propranolol LA: 60‐80 mg daily
• Antithyroid drugs
• Radioactive iodine
• Thyroid surgery

Antithyroid Drugs
• Methimazole
– Agent of choice in most cases
– Can be taken once daily
– Started at a dose of 5‐40 mg daily
– Recheck thyroid hormone levels 3‐4 weeks after 
starting or changing a dose
– Adverse effects
• Common ‐ pruritis, rash
• Rare ‐ agranulocytosis, hepatotoxicity, vasculitis

3118
Copyright © Harvard Medical School, 2018. All Rights Reserved.

Antithyroid Drugs
• Methimazole
– Graves’ disease
• 30‐60% chance of remission after 12‐18 months of 
treatment
• 50% chance of relapse within 18 months of stopping 
treatment
– Toxic adenoma and toxic multinodular goiter
• Require continuous treatment
• Doses may need to be increased over time

Antithyroid Drugs
• When to use propylthiouracil
– Prior to conception and during the first trimester 
of pregnancy
• Methimazole has been associated with aplasia cutis
• Consider switching to methimazole during the second 
trimester
– In cases of thyroid storm
• Acts to block peripheral conversion of T4 to T3
– When a patient who is allergic to methimazole 
declines radioactive iodine treatment or surgery

3119
Copyright © Harvard Medical School, 2018. All Rights Reserved.

Radioactive Iodine 
• Iodine‐131
– May be given immediately or after a period of 
treatment with antithyroid drugs
– Can’t be given while pregnant or breastfeeding
– May cause transient inflammation and  
thyrotoxicosis
– Can take up to 2‐6 months to work
– Delay conception until 6 months out from 
treatment due to risk of fetal thyroid damage

Radioactive Iodine 
• Graves’ disease
– Dose based on uptake and gland weight
– Treatment may exacerbate thyroid eye disease
• Toxic adenoma
– Usually shrinks size without impairing function of 
normal tissue
• Toxic multinodular goiter
– May relive compressive symptoms caused by 
substernal extension

3120
Copyright © Harvard Medical School, 2018. All Rights Reserved.

Thyroid Surgery
• Graves’ disease
– In cases of severe hyperthyroidism (thyroid storm)
– When a patient who is allergic to antithyroid drugs 
can’t be or refuses to be treated with radioactive 
iodine
• Toxic multinodular goiter
– When there is substernal extension causing 
significant compressive symptoms

Non‐Thyroidal Illness

3121
Copyright © Harvard Medical School, 2018. All Rights Reserved.

Non‐Thyroidal Illness
• “Euthyroid sick syndrome”
• Transient changes in hypothalamic‐pituitary 
function, thyroid hormone secretion, and 
deiodinase activity that occur in the setting of 
acute physiologic stress
• Combinations of changes may suggest 
underlying thyrotoxicosis and/or 
hypothyroidism

Non‐Thyroidal Illness
• TSH
– Initially suppressed due to decreased 
hypothalamic‐pituitary secretion
– May rebound to high levels during recovery
• T4
– Decreased due to diminished secretion
• T3
– Decreased due to inhibition of deiodinase activity

3122
Copyright © Harvard Medical School, 2018. All Rights Reserved.

TSH
reverse T3
Normal

T4
T3

Mild Moderate Severe Recovery

Thyroiditis

3123
Copyright © Harvard Medical School, 2018. All Rights Reserved.

Subacute Thyroiditis
• Onset is often preceded by a nonspecific viral 
illness
• Patients present with pain localized to the 
thyroid gland which may radiate upwards to 
the neck and jaw
• Thyroid gland may be slightly enlarged and 
exquisitely tender to palpation
• Inflammation may spread from one lobe to 
the other

3124
Copyright © Harvard Medical School, 2018. All Rights Reserved.

Subacute Thyroiditis
• Low thyroid uptake
• Lab tests may reveal an elevated ESR
• Thyrotoxic phase lasting 1‐4 weeks caused by 
release of thyroid hormone, followed by a 
resolving hypothyroid phase lasting 1‐3 
months

3125
Copyright © Harvard Medical School, 2018. All Rights Reserved.

Subacute Thyroiditis
• High dose ibuprofen, aspirin, celecoxib, or 
prednisone 20‐40 mg daily for pain control
• Symptoms may flare up as doses are tapered
• Beta blockers to attenuate thyrotoxic 
symptoms
• Temporizing treatment with levothyroxine 
during hypothyroid phase

Autoimmune Thyroiditis
• Lymphocytic thyroiditis, painless thyroiditis, or 
silent thyroiditis
• May develop in 5‐8% of all women following 
pregnancy (postpartum thyroiditis)
• Usually asymptomatic
• Thyroid gland may be slightly enlarged, but is 
not tender
• Most cases resolve completely within 6 
months, though 10% may recur

3126
Copyright © Harvard Medical School, 2018. All Rights Reserved.

Autoimmune Thyroiditis
• Low thyroid uptake
• ESR is normal
• Thyrotoxic phase caused by release of thyroid 
hormone (90% of cases), followed by a 
resolving hypothyroid phase (50% of cases) 
• Beta blockers to attenuate thyrotoxic 
symptoms
• Temporizing treatment with levothyroxine 
during hypothyroid phase

Checkpoint Inhibitors
• Immunomodulatory antibodies that inhibit 
programmed cell death receptor 1 and 
receptor ligand 1 (PD‐1, PD‐L1) and cytotoxic 
T‐lymphocyte‐associated antigen 4 (CTLA‐4)
• Immunologic enhancement can trigger new‐
onset autoimmune thyroiditis
• May present with severe thyrotoxicosis 
followed by rapid progression to overt 
hypothyroidism

3127
Copyright © Harvard Medical School, 2018. All Rights Reserved.

Checkpoint Inhibitors

Thyrotoxicosis Hypothyroidism
Atezolizumab (Tecentriq®) 8.0% 13.2%
Nivolumab (Opdivo®) 3.2% 7.0%
Ipilimumab (Yervoy®) 1.7% 3.8%
Pembrolizumab (Keytruda®) 0.6% 3.9%

Thyroid Nodules

3128
Copyright © Harvard Medical School, 2018. All Rights Reserved.

Thyroid Nodules
• 54,000 new cases of thyroid cancer in the U.S. 
in 2018
• 300,000 new palpable thyroid nodules 
detected annually
• Incidental nodules noted on 13% of carotid 
doppler studies
• Discrete nodules identified on 67% of thyroid 
ultrasounds

Thyroid Nodules: Evaluation
• TSH level
– Assess functional status
• Thyroid ultrasound
– Confirm presence 
– Characterize
– Detect additional non‐palpable nodules
– Identify lymphadenopathy

3129
Copyright © Harvard Medical School, 2018. All Rights Reserved.

TSH and thyroid ultrasound

Low TSH Normal TSH High TSH

Anti‐thyroid 
Toxic adenoma Thyroid scan
antibodies

Toxic multinodular  Autoimmune 
Graves’ disease Nodule
goiter thyroiditis

Cold nodule Fine needle aspiration biopsy

Thyroid Nodules: FNA Criteria
• > 1.0 cm
– Hypoechoic nodules
– Nodules with suspicious features
• > 1.5 cm
– Isoechoic nodules
– Hyperechoic nodules
• > 2.0 cm 
– Spongiform nodules

3130
Copyright © Harvard Medical School, 2018. All Rights Reserved.

Thyroid Nodules: FNA Criteria
• Suspicious features
– Irregular margins
– Microcalcifications
– Coronal height > width
– Extrathyroidal extension
– Extrusion through a rim of calcification

3131
Copyright © Harvard Medical School, 2018. All Rights Reserved.

Bethesda System for Reporting Thyroid 
Cytopathology

Malignant
Benign 0 ‐ 3%
Atypia of undetermined significance 10 ‐ 30%
Suspicious for a follicular neoplasm 25 ‐ 40%
Suspicious for malignancy 50 ‐ 75%
Malignant 97 ‐ 99%

3132
Copyright © Harvard Medical School, 2018. All Rights Reserved.

Thyroid Nodules: Management
• Benign
– Monitor with serial imaging
• Atypia of undetermined significance
– Repeat biopsy +/‐ genetic profiling (Afirma®, ThyroSeq®)
• Suspicious for a follicular neoplasm
– Repeat biopsy +/‐ genetic profiling
• Suspicious for malignancy 
– Hemithyroidectomy or total thyroidectomy
• Malignant
– Total thyroidectomy

Question 1
A 33 year‐old male is noted to have palpable 
enlargement of the right side of his thyroid. 
Ultrasound reveals a 3.1 cm nodule with smooth 
borders. Lab tests show TSH 0.1 mU/L (0.5 ‐ 5.2 
mU/L) and T4 11.5 µg/dL (4.6 ‐ 10.7 µg/dL). He 
reports a history of symptomatic palpitations and 
weight loss of 5 lbs over the course of 3 months, 
despite an increase in his appetite. He is not taking 
any medications and has not noted any problems 
with dysphagia or dysphonia. 

3133
Copyright © Harvard Medical School, 2018. All Rights Reserved.

Question 1
What should you do next?
A. Perform a fine needle aspiration biopsy of the 
right sided nodule
B. Administer a 15 mCi dose of I‐131
C. Refer the patient to a thyroid surgeon
D. Start methimazole at a dose of 5 mg daily
E. Check a radioiodine scan and uptake

Question 1
What should you do next?
A. Perform a fine needle aspiration biopsy of the 
right sided nodule
B. Administer a 15 mCi dose of I‐131
C. Refer the patient to a thyroid surgeon
D. Start methimazole at a dose of 5 mg daily
E. Check a radioiodine scan and uptake

3134
Copyright © Harvard Medical School, 2018. All Rights Reserved.

Question 2
A 74 year‐old male with a history of hypertension and 
hypercholesterolemia is hospitalized after presenting with 
a three month history of progressive fatigue and dyspnea 
on exertion. His weight is 164 lbs, his pulse is 44 bpm, and 
lab tests show CPK 528 U/L (22 ‐ 198 U/L), TSH 65 mU/L 
(0.5 ‐ 5.2 mU/L), free T4 0.2 ng/dL (0.9 ‐ 1.7 ng/dL), and 
T4 2.3 µg/dL (4.6 ‐ 10.7 µg/dL). A nuclear stress test 
reveals findings consistent with ischemia. Coronary 
angiography reveals diffuse three vessel disease that is 
not amenable to stenting. A cardiologist recommends 
that he undergo coronary artery bypass surgery.

Question 2
What would you recommend?
A. Start levothyroxine at a dose of 125 mcg daily
B. Administer a 60 mcg intravenous dose of 
levothyroxine daily 
C. Check anti‐thyroid peroxidase and anti‐
thyroglobulin antibodies
D. Start levothyroxine at a dose of 12.5 mcg daily
E. Defer treatment with thyroid hormone 
replacement until after he has undergone 
revascularization

3135
Copyright © Harvard Medical School, 2018. All Rights Reserved.

Question 2
What would you recommend?
A. Start levothyroxine at a dose of 125 mcg daily
B. Administer a 60 mcg intravenous dose of 
levothyroxine daily 
C. Check anti‐thyroid peroxidase and anti‐
thyroglobulin antibodies
D. Start levothyroxine at a dose of 12.5 mcg daily
E. Defer treatment with thyroid hormone 
replacement until after he has undergone 
revascularization

Key Points
• The TSH level is the most sensitive index of 
thyroid function
• Graves’ disease is usually diagnosed clinically
• Pregnancy is associated with increased 
levothyroxine dose requirements
• Treatment of subclinical thyroid disease is 
optional in most cases
• Fine needle aspiration is the most informative 
approach to the evaluation of thyroid nodules 

3136
Copyright © Harvard Medical School, 2018. All Rights Reserved.

References
• Durante C et al. The Diagnosis and Management 
of Thyroid Nodules. JAMA. 2018 Mar 6;319(9): 
914‐924
• Peeters RP. Subclinical Hypothyroidism. N Engl J 
Med. 2017 Jun 29;376(26):2556‐2565
• Smith TJ, Hegedüs L. Graves' Disease. N Engl J 
Med. 2016 Oct 20;375(16):1552‐1565
• Samuels MH. Subacute, silent, and postpartum 
thyroiditis. Med Clin N Am. 2012. 96(2):223‐33

3137
Copyright © Harvard Medical School, 2018. All Rights Reserved.

ADRENAL DISORDERS
Anand Vaidya, MD MMSc
Director, Center for Adrenal Disorders
Division of Endocrinology, Diabetes, & Hypertension
Brigham and Women’s Hospital
Assistant Professor of Medicine, Harvard Medical School

Disclosures

None

3138
Copyright © Harvard Medical School, 2018. All Rights Reserved.

Learning Objectives

1. Review the role of the adrenal gland in physiology and


pathophysiology

2. Evaluation of adrenal insufficiency

3. Evaluation of an adrenal mass

4. Biochemical work up for hyperaldosteronism,


hypercortisolism, and pheochromocytoma

TUTORIAL VIDEOS:
ADRENAL PHYSIOLOGY: https://www.youtube.com/watch?v=bM6rhEuOtBM
ADRENAL INSUFFICIENCY: https://www.youtube.com/watch?v=SgckxKvccKo
PRIMARY ALDOSTERONISM: https://www.youtube.com/watch?v=db9v9kNIiXU
PHEOCHROMOCYTOMA: https://www.youtube.com/watch?v=0tZ8kJ6dN3A

Take Home Points


• The adrenal glands synthesize and secrete hormones that play a vital
role in hemodynamic homestasis. Understanding the physiology of
adrenal hormone feedback and the relative interpretations of adrenal
testing is crucial for accurate assessments.

• Carefully timed cortisol and ACTH measurements can help decipher


whether the HPA axis is functioning appropriately

• The prevalence of adrenal masses increases with age, and their


incidental discovery has increased.

• All adrenal masses should be carefully evaluated for malignant potential


and hormone hyperfunction

• Consider involving endocrine/adrenal colleagues at any time in the


evaluation

3139
Copyright © Harvard Medical School, 2018. All Rights Reserved.

Case 1
• 28yoF presents to ER 6 weeks after having a baby
• Cannot breastfeed well
• Presents with progressive fatigue, dizziness, orthostasis,
salt craving, hyperpigmentation, anorexia, and weight loss
• BP=60/40 mmHg
• IV saline (8L) and BP improves

• Cortisol 0.80 mcg/dL


• (60mins after 250 mcg cosyntropin)= 1.0 mcg/dL
• ACTH>1000pg/mL

Question 1
The most likely diagnosis is:

A) Primary adrenal insufficiency


B) Acute secondary adrenal insufficiency
C) Chronic secondary adrenal insufficiency
D) Ectopic ACTH syndrome
E) Cushing’s disease

3140
Copyright © Harvard Medical School, 2018. All Rights Reserved.

Question 1
This most likely diagnosis is:

A) Primary adrenal insufficiency


B) Acute secondary adrenal insufficiency
C) Chronic secondary adrenal insufficiency
D) Ectopic ACTH syndrome
E) Cushing’s disease

Adrenal Cortex: Steroidogenesis


Cholesterol
Zona Glomerulosa Zona Fasciculata Zona Reticularis

Pregnenolone 17-Hydroxypregnenolone DHEA

Progesterone 17-Hydroxyprogesterone Androstenedione

11-Deoxycorticosterone 11-Deoxycortisol Testosterone

Corticosterone Cortisol Estradiol

Aldosterone

3141
Copyright © Harvard Medical School, 2018. All Rights Reserved.

Hypothalamic-Pituitary-Adrenal Axis

P
Glucocorticoid
Receptor

Target
Organ
Cell
Adrenal
Mineralocorticoid
Receptor

The Glucocorticoid Receptor


Physiologic Activation of the GR

Increase Blood Glucose


• Increase gluconeogenesis
• Promote protein breakdown
• Augment epinephrine-mediated lipolysis

Increase Cardiovascular tone


•Increase pressor response to NE, AngII
•Stimulate synthesis of Epi
•Inhibit synthesis of NO

Immune Modulation

CNS Effects
•Appetite
•Mood
•Sleep/wake

Promote fetal tissue and development

Involved in parturition

Many other magical things…

3142
Copyright © Harvard Medical School, 2018. All Rights Reserved.

The Mineralocorticoid Receptor

Physiologic Activation of the MR


• Sodium reabsorption
• Expansion of plasma volume
• Urinary potassium secretion
• Urinary hydrogen secretion

Hypothalamic-Pituitary-Adrenal Physiology:
NEWTON’s 3rd LAW: For every action, there is an equal and opposite reaction (feedback)

3143
Copyright © Harvard Medical School, 2018. All Rights Reserved.

Hypothalamic-Pituitary-Adrenal Physiology

H CRH

Glucocorticoid
Receptor
P
Cortisol
POMC
• ACTH
• (MSH)

Cortisol
Glucocorticoid
Receptor
Adrenal
Mineralocorticoid
Receptor
11βOH-steroid dehydrogenase 2 Target
Inactivates cortisol to cortisone Organ
Cell

Hypothalamic-Pituitary-Adrenal Physiology

H CRH

ACTH

Glucocorticoid
Receptor
Adrenal Mineralocorticoid
Receptor
AGT Target
Ang-II Aldosterone Organ
Renin Ang-I ACE K+ Cell

3144
Copyright © Harvard Medical School, 2018. All Rights Reserved.

The HPA axis

1) Cortisol secretion is entirely dependent on ACTH

2) Aldosterone is not dependent on ACTH. It is regulated


in part by:
• Angiotensin II (Renin-angiotensin system)
• K+ balance
• ACTH

3) HPA axis responds to:


• Diurnal variation/clock
• “stress”: ACTH & cortisol secretion is augmented
“relative” to the degree of stress

Hypothalamic-Pituitary-Target Organ Physiology:

25
Cortisol (µg/dL)

20

15

10

0
2400 0400 0800 1200 1600 2000 2400

TIME

3145
Copyright © Harvard Medical School, 2018. All Rights Reserved.

Testing the HPA axis

STATIC TEST (entire HPA axis):


•Morning cortisol (and ACTH)
•An “appropriate” AM cortisol should ideally be >15-18 µg/dL, reflecting
a morning peak, but values > 10-12 µg/dL when pre-test probability is
low

Vaidya et al. NEJM 2010; 362(6): e16

Testing the HPA axis

STATIC TEST (entire HPA axis):


•Morning cortisol (and ACTH)
•An “appropriate” AM cortisol should ideally be >15-18 µg/dL, reflecting
a morning peak, but values > 10-12 µg/dL when pre-test probability is
low

PROVOCATIVE TEST (adrenal glands):


•250 µg cosyntropin stimulation test

Vaidya et al. NEJM 2010; 362(6): e16

3146
Copyright © Harvard Medical School, 2018. All Rights Reserved.

Primary Adrenal Insufficiency


(Addison’s Disease)
Manifestations/Characteristics:
H CRH
Labs:
• low cortisol
P • high ACTH
• low aldosterone
-hyperkalemia, hyponatremia,
ACTH hypovolemia

Physical Exam:
Cortisol
• fatigue/lethargy/anorexia Glucocorticoid
Aldosterone
• hypotension/orthostasis/salt craving Receptor
Adrenal • weight loss Mineralocorticoid
• hyperpigmentation Receptor
• abdominal pain
• many many more Target
Organ
Cell

Primary Adrenal Insufficiency


(Addison’s Disease)
Response to Cosyntropin:
H CRH

• sub-optimal
P EXAMPLE:
60 mins following
Morning
250 µg cosyntropin
ACTH Cosyntropin Cortisol (µg/dL) 1.8 2.1
ACTH (pg/mL) 1100

Glucocorticoid
Cortisol Receptor
Adrenal
Mineralocorticoid
Receptor

Target
Organ
Cell

3147
Copyright © Harvard Medical School, 2018. All Rights Reserved.

Primary Adrenal Insufficiency (Addison’s)

Causes:
• Autoimmune
• Infiltrative infections (TB, fungal)
• Hemorrhage
• Infiltrative malignancy

Medications:
• Anti-fungal medications
• Heparin
• Etomidate

ACUTE Secondary Adrenal Insufficiency

H CRH
Manifestations/Characteristics:

Labs:
P • low basal cortisol
• inappropriately low ACTH
• ± hyponatremia
ACTH
• Normal K and aldosterone regulation

Physical:
Cortisol
• completely normal Glucocorticoid
Adrenal • mild, progressive, fatigue at baseline Receptor
• severe fatigue, orthostasis, Mineralocorticoid
Receptor
hypotension, in situations of stress
Target
Organ
Cell

3148
Copyright © Harvard Medical School, 2018. All Rights Reserved.

ACUTE Secondary Adrenal Insufficiency

Response to Cosyntropin:
H CRH

• NORMAL
P EXAMPLE:
60 mins following
Morning
250 µg cosyntropin
ACTH Cosyntropin Cortisol (µg/dL) 2.2 26.0
ACTH (pg/mL) 10

Cortisol Glucocorticoid
Receptor
Adrenal
Mineralocorticoid
Receptor

Target
Organ
Cell

CHRONIC Secondary Adrenal Insufficiency

With chronic ACTH deficiency, adrenal cortex


H CRH
(ZF) will atrophy, and will progressively
respond less to cosyntoprin stimulation
EXAMPLE:
P 60 mins following
Morning
250 µg cosyntropin
Cortisol (µg/dL) 2.2 4.5
ACTH Cosyntropin
ACTH (pg/mL) 10

Glucocorticoid
Cortisol Receptor
Adrenal
Mineralocorticoid
Receptor

Target
Organ
Cell

3149
Copyright © Harvard Medical School, 2018. All Rights Reserved.

Secondary Adrenal Insufficiency

Causes:
• Pituitary mass: adenoma or metastatic lesion
• Pituitary infection
• Pituitary infiltration (granulomatous disease, iron)
• Pituitary trauma

Medications:
• Glucocorticoids
• Megesterol
• Opioids

Case 1

• Diagnosed with primary adrenal insufficiency (Addison’s)


• Treated with IV saline (8L)
• IV hydrocortisone initially
• Transitioned to PO hydrocortisone/fludrocortisone

3150
Copyright © Harvard Medical School, 2018. All Rights Reserved.

Case 2
• 46 year old pre-menopausal woman was in a car accident.

• No known medical conditions

• Brought to ER and complained of some abdominal pain. Had


a rapid unenhanced Abdominal CT that revealed no
hemorrhage or other injuries

• Incidental discovery of a 2.2 cm R adrenal mass, with 5 HU


unenhanced density

What, if anything, do you tell her about the


incidental adrenal mass??

Incidentally Disovered Adrenal Masses


• Adrenal tumors are incidentally discovered in 1-10% of adults.

• A minority represent malignant entities (primary adrenal malignancy or extra-


adrenal metastasis)

• The majority are determined to be benign and “nonfunctional” and therefore


are considered to pose no health risk.

• In contrast, ~10-15% of adrenal tumors autonomously secrete adrenal


hormones. These “functional” tumors are associated with an increased risk
for cardiometabolic outcomes, such as CV disease, diabetes, and
osteoporosis/fracture.

• Therefore, all incidentally discovered adrenal tumors should be carefully


evaluated to determine whether they are: 1) malignant and/or 2) functional.

Nieman LK. JCEM 2010; Young et al. NEJM 2007; Zeiger et al. Endocr Pract 2009; Fassnacht EJE 2016

3151
Copyright © Harvard Medical School, 2018. All Rights Reserved.

Differential Diagnosis of Adrenal Mass

NON-FUNCTIONAL FUNCTIONAL

BENIGN

MALIGNANT

Differential Diagnosis of Adrenal Mass

NON-FUNCTIONAL FUNCTIONAL
(85-95%) (5-15%)
Adrenocortical Adenoma
Myelolipoma
Neuroblastoma

BENIGN Ganglioneuroma
(~90-95%)
Cyst
Hemorrhage
Infection (fungal, tuberculous)
Hemangioma

MALIGNANT
(~5%)

3152
Copyright © Harvard Medical School, 2018. All Rights Reserved.

Differential Diagnosis of Adrenal Mass

NON-FUNCTIONAL FUNCTIONAL
(85-95%) (5-15%)
Adrenocortical Adenoma Adrenocortical Adenoma
Myelolipoma Aldosterone producing
Neuroblastoma Cortisol producing

BENIGN Ganglioneuroma Micro- or Macro-nodular Disease


(~90-95%)
Cyst Aldosterone producing
Hemorrhage Cortisol producing
Infection (fungal, tuberculous) Pheochromocytoma
Hemangioma

MALIGNANT
(~5%)

Differential Diagnosis of Adrenal Mass

NON-FUNCTIONAL FUNCTIONAL
(85-95%) (5-15%)
Adrenocortical Adenoma Adrenocortical Adenoma
Myelolipoma Aldosterone producing
Neuroblastoma Cortisol producing

BENIGN Ganglioneuroma Micro- or Macro-nodular Disease


(~90-95%)
Cyst Aldosterone producing
Hemorrhage Cortisol producing
Infection (fungal, tuberculous) Pheochromocytoma
Hemangioma
Adrenocortical carcinoma Adrenocortical carcinoma
MALIGNANT
Metastatic cancer from a non-
(~5%) Pheochromocytoma
adrenal primary

3153
Copyright © Harvard Medical School, 2018. All Rights Reserved.

General Diagnostic Approach


1. Is there evidence for malignancy?
2. Is there adrenal hormone excess?

Clinical Biochemical Radiographic


Phenotype Phenotype Phenotype

History and physical exam Laboratory evaluation for Radiographic evidence


for evidence of hormone evidence of adrenal supportive of a benign or
excess or malignancy hormone excess malignant mass

Clinical Phenotype
Overt Overt Overt
Cortisol Excess Catecholamine Excess Aldosterone Excess
• Obesity/weight gain • Episodic symptoms • Hypertension
• Lipodystrophy o Hypertension • Hypokalemia
oCentral adiposity o Palpitations • Alkalosis
oSupraclavicular fat pads o Anxiety/Panic
oDorsocervical fat pad o Sweats/Tremors
oRounded face o Headache
• Hyperglycemia/Diabetes o Arrhythmia
• Hypertension
• Insomnia
• Mood disorder/Psychosis
• Osteoporosis
• Immunesuppression
• Platelet dysfunction
• Hypercoagulable state
• Myopathy
• Atrophic skin

3154
Copyright © Harvard Medical School, 2018. All Rights Reserved.

Case 2 – Clinical Phenotype

• No symptoms
• No signs to suggest hypercortisolism,
pheochromocytoma, hyperaldosteronism, or Clinical
hirsutism. Phenotype

• No evidence of weight loss, abdominal


distention, or androgen excess, to suggest
metastatic cancer or hyperfunctioning
adrenocortical carcinoma
Unrevealing

Case 2 – Radiographic Phenotype


Normal comparison Patient’s non-contrast CT

2.2cm right adrenal mass


5 Hounsfield units
Round, homogenous

3155
Copyright © Harvard Medical School, 2018. All Rights Reserved.

Question 2
A 2.2 cm adrenal nodule with an unenhanced density of 5 HU
on CT is most suggestive of:

A)Myelolipoma
B)Adrenocortical adenoma
C)Pheochromocytoma
D)Metastatic lung cancer to the adrenal gland
E)Adrenocortical carcinoma

Question 2
A 2.2 cm adrenal nodule with an unenhanced density of 5 HU
on CT is most suggestive of:

A)Myelolipoma
B)Adrenocortical adenoma
C)Pheochromocytoma
D)Metastatic lung cancer to the adrenal gland
E)Adrenocortical carcinoma

3156
Copyright © Harvard Medical School, 2018. All Rights Reserved.

Case 2 – Radiographic Phenotype


Likely Potentially
Characteristic
Benign Malignant

Size < 4 cm > 4-6 cm

Attenuation on
<10 HU > 10 HU
unenhanced CT
Contrast washout on
CT protocol at 15
Absolute>60% Absolute<60% Radiographic
minutes
Relative >40% Relative<40%
Phenotype
MRI chemical shift
suggestive of lipid-rich Yes No
content

FDG avidity on PET No Yes

Irregular Borders No Yes

Heterogeneous content No Yes BENIGN: Suggestive of


Necrosis No Yes adrenocortical adenoma
Calcifications No Yes

Rate of Growth < 1cm/y >1cm/y

Miller. Nature Rev Endo 2014


Vaidya. Scientific American Medicine 2015

Case 2 – Biochemical Phenotype

Suggested screening biochemical evaluation for adrenal masses:

Condition Patients Test Abnormal Value


1 mg Nonfunctional: ≤1.8 mcg/dL
Autonomous
ALL Dexamethasone Possible: 1.9-5.0 mcg/dL
cortisol secretion
Suppression Test Autonomous: 5.0 mcg/dL

Serum aldosterone
Primary HTN and/or • Suppressed PRA
to plasma renin
Aldosteronism hypokalemia • ARR>20-25
activity ratio (ARR)

Plasma (or urinary)


Pheochromocytoma ALL (almost) fractionated >2-4x ULRR
metanephrines
Adrenal androgen Hirsutism or DHEAS
Higher than ULN
excess virilization Total Testosterone

Fassnacht EJE 2016

3157
Copyright © Harvard Medical School, 2018. All Rights Reserved.

Case 2 – Biochemical Phenotype


• 1mg DST #1 => cortisol: 8.0 µg/dL
• 1mg DST #2 => 7.8 µg/dL, ACTH<5 pg/mL
• 8mg DST => 8.1 µg/dL, ACTH<5 pg/mL Biochemical
Phenotype
• 24h Urine Free Cortisol: 45 µg/24h (<45)

•Midnight Salivary Cortisol:


3.7, 3.9, 4.6, 4.3 nmol/L (<4.3)

• Random ACTH: 5 pg/mL


• Plasma metanephrines: normal
Autonomous
• Aldosterone/PRA: not suggestive cortisol
• DHEAS: normal secretion?

Case 2 – Clinical Diagnosis

Benign adrenocorticol adenoma

Autonomous cortisol secretion

No clinical signs of hypercortisolism

Should surgery be recommended?

3158
Copyright © Harvard Medical School, 2018. All Rights Reserved.

Case 2 – Outcome
• BP = 122/75 mmHg
• Fasting Blood Glucose = 99 mg/dL
• HbA1c = 5.8%
• Bone Mineral Density:
• Spine T= -3.2
• Femoral Neck T= -2.2
• Total Hip T= -2.0
• INDIVIDUALIZED DECISION: Laparoscopic R adrenalectomy
• Peri-operative IV hydrocortisone considered, but not given
• Pathology revealed 2.5 cm adrenal cortical adenoma
• Post-op AM cortisol 4 mcg/dL, ACTH<10 pg/mL (asymptomatic)
• 1 week post-op, morning cortisol = 17 µg/dL

Suggested Diagnostic Algorithm for Incidentally Discovered Adrenal Mass


Adrenal Mass

Clinical Phenotype

(+) Biochemical Phenotype (-/+)


Confirm Overt Hormone
Excess with Clinical
Radiographic Phenotype
Syndrome

Radiographic Phenotype Suspicious Benign Appearing


>4-6cm, >10 HU, contrast avid, <10HU, <4cm, non-contrast avid,
and Localization heterogeneous homogeneous

Surveillance Considerations:
Consider surgery If initially “nonfunctional”:
((Consider alternative imaging: • No strong evidence for repeated biochemical
CT with washout, MRI)) testing
• Repeat biochemical testing if worsening
If Unilateral: Consider surgery comorbidities (HTN, DM, low BMD)
?Metastases or infection: Biopsy If autonomous cortisol secretion without
•Growth>0.5cm/year or +20%
•Suspicious radiographic features clinical syndrome:
Unsure? =>Surveillance: repeat • Individualized consideration for surgery
•New or worsening hormonal excess imaging in 3-6 months based on comorbidities and other factors.
• Repeat biochemical testing annually
•No firm evidence for radiographic
surveillance

3159
Copyright © Harvard Medical School, 2018. All Rights Reserved.

Adrenal Tumors with Autonomous Cortisol Secretion

&

Risk for CVD, Diabetes, Skeletal Disease

Comorbidities associated with adrenal tumors with


autonomous and “subclinical” cortisol secretion

Comorbidities
Hypertension
Glucose intolerance/type 2 diabetes
Obesity
Dyslipidemia
Osteoporosis/Vertebral Fracture

Fassnacht et al. EJE 2016

3160
Copyright © Harvard Medical School, 2018. All Rights Reserved.

Autonomous Cortisol Secretion

Stable “nonfunctional”
DST≤1.8 mcg/dL

Proportion with Cardiovascular Mortality (%)


Proportion with Cardiovascular Disease (%)

Worsening Stable,
≤1.8 => 1.8-5.0 but Any Autonomous Cortisol
or DST 1.8-5.0 mcg/dL
1.8-5.0 => >5.0 DST >5.0 mcg/dL

Stable,
Worsening
but Any Autonomous Cortisol
DST 1.8-5.0 mcg/dL
≤1.8 => 1.8-5.0
DST >5.0 mcg/dL or
1.8-5.0 => >5.0

Stable “nonfunctional”
DST≤1.8 mcg/dL

Di Dalmazi et al. Lancet Diabetes & Endo 2014

Autonomous Cortisol Secretion

Most deaths due to cardiovascular or infectious causes

DeBono et al. JCEM 2014


Morelli et al. JCEM 2015

3161
Copyright © Harvard Medical School, 2018. All Rights Reserved.

Autonomous Cortisol Secretion

• Participants with adrenal incidentalomas who developed incident subclinical


vertebral fractures detected on BMD and higher 1mg DST (2.7 vs 2.0 mcg/dL).
• Autnomous cortisol secretion associated with ~10-fold higher risk of incident (~3y
follow-up) vertebral fracture.

DeBone et al. JCEM 2014


Morelli et al. JCEM 2015

50
Adjusted HR: 2.36 (1.45, 3.84)
45 Absolute Risk: 15.6 % (6.9, 24.3)
Composite Diabetes (%)

Adrenal Tumor with


40
Subclinical Hypercortisolism
35 32.0% (1mg DST 1.9-5.0 mcg/dL)
Incident

30 27.3%
“Non-Functional”
25
Adrenal Tumor
20 (1mg DST ≤ 1.8 mcg/dL)

15 11.7%
No Adrenal Tumor
10

0
0 5 10 15 20

Years of Follow-Up

3162
Copyright © Harvard Medical School, 2018. All Rights Reserved.

Lopez et al. Ann Int Med 2016

Continuum of Cardiometabolic Risk

Adrenal Tumor
With Overt
Cushing Syndrome
Adrenal tumor
with Autonomous Cortisol
Secretion but no Cushing
syndrome

“Nonfunctional”
adrenal tumor

Morphologically Normal
with no hormone excess

Prevalence of Condition

Should Intervention be Performed?

HTN
Improved
SBP

DM
Improved FBG

Bancos et al. EJE 2016

3163
Copyright © Harvard Medical School, 2018. All Rights Reserved.

Case 3

• 42yoF healthy woman


• New HTN 130-140/80
Clinical
• Spontaneous headache => BP 200/100 Phenotype
• Over several months treated with:
– Lisinopril+amlodipine+labetolol: BP still
140/90
• Hypokalemia (2.3 mmol/L) on more than
one occasion Concerning for excess
aldosterone

No clinical signs of
hypercortisolism

Hyperaldosteronism: who to screen?

• Severe or Resistant Hypertension


– BP>150/100 mmHg x3; or >140/90 mmHg on 3 medications; or <140/90 mmHg on 4+
medications

• HTN + spontaneous or diuretic-induced hypokalemia

• HTN + adrenal mass

• HTN + sleep apnea

• HTN and family member with PA

• HTN + Family history of early-onset HTN or cerebrovascular


accident (<40yrs)

Funder et al. JCEM 2016

3164
Copyright © Harvard Medical School, 2018. All Rights Reserved.

Case 3 – Biochemical Phenotype


Diagnostic Testing for Adrenal Aldosterone Excess:

Goal Test Method Desired Value


Screening Aldosterone-to-Renin Peripheral blood • suppressed
Ratio (ARR) Plasma renin
activity (PRA)
• ARR>20-25
Confirmation Salt Suppression Test Oral salt x4-5 days Non-suppressible
to achieve 24h aldosterone
UNa>200mmol/24h

IV saline
Localization CT imaging CT: surgical CT: N/A
planning
AVS AVS: lateralization
AVS: lateralization or non-
of hormone function lateralization

Case 3 – Biochemical Phenotype

Biochemical
• Aldosterone 22 ng/dL (on anti-HTNives) Phenotype
• PRA <0.6 ng/mL/h
• ARR >>>40

Highly suggestive of
Primary Aldosteronism

3165
Copyright © Harvard Medical School, 2018. All Rights Reserved.

Case 3 – Biochemical Phenotype

Oral Salt Suppression Test:


Biochemical
Phenotype
• 24h Urine Na: 378mmol/24h
• 24h Urine Aldosterone: 25.5 mcg/24h

Primary
Aldosteronism

Case 3 – Radiographic Phenotype

3166
Copyright © Harvard Medical School, 2018. All Rights Reserved.

Suggested Diagnostic Algorithm


Adrenal Mass

1. Clinical Phenotype

2. Biochemical Phenotype
(+)
Confirm Aldosterone
Excess

3. Radiographic Phenotype Bilateral Medical therapy


and Localization Disease with MRA

•Improve BP and reduce


Unilateral Disease Surgery anti-HTN medications
•Lower CVD risk

Case 3 Outcome

• Laparoscopic L adrenalectomy

• Pathology revealed 2.4 cm adrenal cortical adenoma

• Post-op, BP 105/60 mmHg on lisinopril 10mg daily; no


hypokalemia

3167
Copyright © Harvard Medical School, 2018. All Rights Reserved.

Case 3 – Clinical Diagnosis

• Primary Aldosteronism
• Benign L adrenal cortical adenoma (Conn’s
tumor)

Case 4

• 28yo healthy, athletic, woman


• Runs 4-6 miles a day Clinical
• Excessive sweating Phenotype
• Anxiety
• BP 140/90mmHg (new)

• No palpitations, chest pain, tremors, etc Concerning for


pheochromocytoma?

3168
Copyright © Harvard Medical School, 2018. All Rights Reserved.

Who should you evaluate for Pheo-PGL?


1. Paroxysmal constellation of symptoms/signs, or episodic “spells”, strongly
suggestive of catecholamine excess
• palpitations, sweating, anxiety/panic, elevations in blood pressure,
headache/visual symptoms, pallor

2. Provocation/precipitation of symptoms by culprit medications


• dopamine antagonists, beta-blockers, sympathomimetics, opioids,
NERI/SSRI, MAO inhibitors, glucocorticoids

3. Incidentally discovered adrenal mass (>10 HU on CT)

4. Known or suspected hereditary predisposition for syndromic Pheo-PGL

5. Known history of prior Pheo-PGL

Lenders et al. JCEM 2014

Case 4 – Biochemical Phenotype

Desired Value in true


Test catecholamine- Comment
producing tumors
Plasma fractionated
metanephrines Excellent Screening tests
(metanephrine, normetanephrine) >2-4x ULRR in (LC-MS/MS)
clinically apparent
24h urinary fractionated functional tumor Sensitivity: 95-100%,
metanephrines (and creatinine) Specificity: ~85-90%

~10-15% of individuals who do not


have a catecholamine-producing
tumor will have “positive” biochemistry

~100% negative predictive value

Lenders et al. JCEM 2014

3169
Copyright © Harvard Medical School, 2018. All Rights Reserved.

Case 4 – Biochemical Phenotype

Biochemical
• Plasma metanephrines: 20 (<62)
• Plasma normetanephrines 615 (<145) Phenotype

• 24h urine NE: 1508 (<135)


• 24h urine Normetanephrines 3156 (<1050)
• 24h urine Epi: 2 (<20)
• 24h urine metanephrines: 151 (<460)
Pheochromocytoma

Case 4 – Radiographic Phenotype

RIGHT
2 cm mass
35 HU

T2 hyperintense
on MRI

3170
Copyright © Harvard Medical School, 2018. All Rights Reserved.

Suggested Diagnostic Algorithm for Suspected or Known Adrenal Mass


Adrenal Mass

Clinical Phenotype

(+) Biochemical Phenotype (-/+)


Confirm Overt Hormone
Excess with Clinical
Radiographic Phenotype
Syndrome

Radiographic Phenotype Suspicious Benign Appearing


>4-6cm, >10 HU, contrast avid, <10HU, <4cm, non-contrast avid,
and Localization heterogeneous homogeneous

Surveillance Considerations:
Consider surgery If initially “nonfunctional”:
((Consider alternative imaging: • No strong evidence for repeated biochemical
CT with washout, MRI)) testing
• Repeat biochemical testing if worsening
If Unilateral: Consider surgery comorbidities (HTN, DM, low BMD)
?Metastases or infection: Biopsy If autonomous cortisol secretion without
•Growth>0.5cm/year or +20%
•Suspicious radiographic features clinical syndrome:
Unsure? =>Surveillance: repeat • Individualized consideration for surgery
•New or worsening hormonal excess imaging in 3-6 months based on comorbidities and other factors.
• Repeat biochemical testing annually
•No firm evidence for radiographic
surveillance
AACE guidelines 2009, Young NEJM 2007, Vaidya. Scientific American Medicine 2015

Case 4 Outcome

• Alpha-blocked and hydrated in preparation for surgery


• Laparoscopic R adrenalectomy

• Pathology revealed pheochromocytoma

• Post-op: sweats, anxiety, hypertension all resolved

3171
Copyright © Harvard Medical School, 2018. All Rights Reserved.

Case 4 – Clinical Diagnosis

• Pheochromocytoma

Case 5
• 37yo healthy woman had an abdominal CT for RLQ
pain
• No cause of pain found

• Incidental RIGHT 2.8cm Adrenal mass (<10HU), LEFT


~3cm adrenal mass (18HU)

• Was told that these are likely benign “adenomas” and


to follow up in 1 year.

3172
Copyright © Harvard Medical School, 2018. All Rights Reserved.

Case 5 – Radiographic Phenotype

R: 2.8 cm, <10 HU L: 3.0 cm, 18 HU

What would you recommend?

Suggested Diagnostic Algorithm for Suspected or Known Adrenal Mass


Adrenal Mass

Clinical Phenotype

(+) Biochemical Phenotype (-/+)


Confirm Overt Hormone
Excess with Clinical
Radiographic Phenotype
Syndrome

Radiographic Phenotype Suspicious Benign Appearing


>4-6cm, >10 HU, contrast avid, <10HU, <4cm, non-contrast avid,
and Localization heterogeneous homogeneous

Surveillance Considerations:
Consider surgery If initially “nonfunctional”:
((Consider alternative imaging: • No strong evidence for repeated biochemical
CT with washout, MRI)) testing
• Repeat biochemical testing if worsening
If Unilateral: Consider surgery comorbidities (HTN, DM, low BMD)
?Metastases or infection: Biopsy If autonomous cortisol secretion without
•Growth>0.5cm/year or +20%
•Suspicious radiographic features clinical syndrome:
Unsure? =>Surveillance: repeat • Individualized consideration for surgery
•New or worsening hormonal excess imaging in 3-6 months based on comorbidities and other factors.
• Repeat biochemical testing annually
•No firm evidence for radiographic
surveillance
AACE guidelines 2009, Young NEJM 2007, Vaidya. Scientific American Medicine 2015

3173
Copyright © Harvard Medical School, 2018. All Rights Reserved.

Case 5
• She returned 1.5 years later
• Has new hirsutism on face and upper chest
• Repeat imaging was performed

Case 5 – Radiographic Phenotype

R: Unchanged L: 4.3 cm, 30 HU


2.8 cm, <10 HU Larger
Heterogeneous
Nodular
Fat stranding

3174
Copyright © Harvard Medical School, 2018. All Rights Reserved.

Case 5
• Underwent a radical L adrenalectomy

• Pathology revealed a high grade stage I


Adrenocortical carcinoma…

Case 5 – Clincial Diagnosis


• STAGE I, HIGH GRADE, ADRENOCORTICAL CARCINOMA

• Adrenal carcinomas are rare, highly aggressive, fast growing, and can
produce multiple adrenal hormones

• Only known cure is early recognition and radical surgery

• 50-70% are metastatic at time of presentation

• Median survival < 12 months from time of diagnosis; can be prolonged


with cytoreduction (surgery/IR)or adjuvant mitotane/EDP therapy

• Morbidity and mortality associated with rapid tumor growth and spread,
but also uncontrolled hormone excess states (i.e. Cushing’s syndrome,
mineralocorticoid excess).

3175
Copyright © Harvard Medical School, 2018. All Rights Reserved.

Take Home Points


• The adrenal glands synthesize and secrete hormones that play a vital
role in hemodynamic homestasis. Understanding the physiology of
adrenal hormone feedback and the relative interpretations of adrenal
testing is crucial for accurate assessments.

• Carefully timed cortisol and ACTH measurements can help decipher


whether the HPA axis is functioning appropriately

• The prevalence of adrenal masses increases with age, and their


incidental discovery has increased.

• All adrenal masses should be carefully evaluated for malignant potential


and hormone hyperfunction

• Consider involving endocrine/adrenal colleagues at any time in the


evaluation

Supplemental References
TUTORIAL VIDEOS:
ADRENAL PHYSIOLOGY: https://www.youtube.com/watch?v=bM6rhEuOtBM
ADRENAL INSUFFICIENCY: https://www.youtube.com/watch?v=SgckxKvccKo
PRIMARY ALDOSTERONISM: https://www.youtube.com/watch?v=db9v9kNIiXU
PHEOCHROMOCYTOMA: https://www.youtube.com/watch?v=0tZ8kJ6dN3A
Fassnacht et al. Management of adrenal incidentalomas: European guidelines.
European Journal of Endocrinology 2016

Young WF, Jr. Clinical practice. The incidentally discovered adrenal mass.
New England Journal of Medicine 2007;356:601-10.

Vaidya A, Hamrahian AH, Auchus RJ. Genetics of Primary Aldosteronism. Endocrine Practice 2015; 21(4): 400-5.

Funder JW, et al. Case Detection, Diagnosis, and Treatment of Patients with Primary Aldosteronism: An Endocrine Society
Clinical Practice Guideline.
Journal of Clinical Endocrinology and Metabolism 2016

Lenders JWM, Duh QY, Eisenhofer G, et al. Pheochromocytoma and Paraganglioma: An Endocrine Society Clinical Practice
Guideline.
Journal of Clinical Endocrinology and Metabolism 2014; 99: 1915-1942

Bornstein et al. Diagnosis and Treatment of Primary Adrenal Insufficiency: An Endocrine Society Clinical Practice Guideline.
Journal of Clinical Endocrinology and Metabolism 2015

3176
Copyright © Harvard Medical School, 2018. All Rights Reserved.

ADRENAL DISORDERS
Anand Vaidya, MD MMSc
Director, Center for Adrenal Disorders
Division of Endocrinology, Diabetes, & Hypertension
Brigham and Women’s Hospital
Assistant Professor of Medicine, Harvard Medical School

3177
Copyright © Harvard Medical School, 2018. All Rights Reserved.

Pituitary Disorders

Ursula B. Kaiser, M.D.


Chief, Division of Endocrinology, Diabetes,
and Hypertension
Brigham and Women’s Hospital
Professor of Medicine
Harvard Medical School

Disclosures

BMS, Ferring – research grant funding


NovoNordisk – scientific advisory board
Chiasma – clinical trial investigator

3178
Copyright © Harvard Medical School, 2018. All Rights Reserved.

Learning Objectives

Lecture is a general neuroendocrine review.


Goal is to discuss:
Neuroendocrine physiology
Diagnostic approach and management of
pituitary disorders

Outline

I. Pituitary Physiology
II. Causes of Pituitary Disease
III. Approach to Evaluation and Management of
Pituitary Disease
A. Pituitary Hormone Excess
B. Pituitary Hormone Deficiency
C. Mass Effects

3179
Copyright © Harvard Medical School, 2018. All Rights Reserved.

Pituitary Gland
Anterior Pituitary Posterior Pituitary
Adenohypophysis Neurohypophysis
80% of the gland 20% of the gland
Derived from Rathke’s pouch Direct extension of the
(oral ectoderm) hypothalamus
Comprised of 5 cell types Axon terminals from SON and
Secretes 6 hormones PVN of hypothalamic neurons
Controlled by neuropeptides Hormone produced in
from the hypothalamus & hypothalamus, stored in
feedback from target organs pituitary

Pituitary Physiology
Anterior Pituitary Posterior Pituitary
Hypothalamic Hypothalamus
Releasing/Inhibiting
Supraoptic nucleus
Neuropeptide
Paraventricular nucleus

Axons
Anterior Pituitary
Hormone
Posterior Pituitary

Target
Organ
AVP Oxytocin

3180
Copyright © Harvard Medical School, 2018. All Rights Reserved.

Approach to Pituitary Disorders

Evaluate:

Pituitary hyperfunction
Baseline and “Suppression tests”

Pituitary hypofunction
Baseline and “Stimulation tests”

Mass effects

Approach to Pituitary Disorders

Posterior Pituitary
Overproduction of AVP
• Syndrome of Inappropriate Antidiuretic
Hormone Secretion (SIADH)

Underproduction of AVP
• Diabetes Insipidus (DI)
- Central (pituitary)
- Nephrogenic
- Primary polydipsia

3181
Copyright © Harvard Medical School, 2018. All Rights Reserved.

Clinical Case 1

39 year old female presented for pituitary


evaluation of headaches
MRI scan revealed 2x3.2 cm sellar mass,
extending suprasellarly to the optic
chiasm and into the L cavernous sinus.

Normal Adenoma Normal Adenoma

Differential Diagnosis of Sellar/Parasellar Lesions

Benign Tumors Granulomatous, Infectious, and


Pituitary adenoma (carcinoma) Inflammatory
Meningioma Lymphocytic hypophysitis
Cell Rest Tumors Abscess
Craniopharyngioma Sarcoidosis
Rathke’s cleft cyst Tuberculosis
Epidermoid Eosinophilic granulomatosis
Chordoma Mycoses
Lipoma Metastatic Tumors
Colloid cyst Vascular Lesions
Primitive Germ Cell Tumors Hematologic Malignancies
Germinoma Miscellaneous
Teratoma Empty sella syndrome
Dysgerminoma Arachnoid cyst
Oligodendroglioma
Ependymoma
Astrocytoma

3182
Copyright © Harvard Medical School, 2018. All Rights Reserved.

Pituitary Adenomas: Epidemiology

Pituitary adenomas are the 3rd most common brain tumor.


They account for 10-15% of all intracranial tumors.
MRI studies 14.4%
Autopsy series 12-22.5%

They are classified according to size.


Microadenomas < 10mm
Macroadenomas > 10 mm

Ezzat Cancer 101: 613, 2004


Daly JCEM 91: 4769, 2006

Pituitary Tumorigenesis

Lactotrope Gonadotrope Somatotrope Corticotrope Thyrotrope

Daly et al. JCEM 91:4769, 2006

3183
Copyright © Harvard Medical School, 2018. All Rights Reserved.

Patient Evaluation
History:
Questions regarding endocrine hypo- or hyper-function.
Think of anterior & posterior pituitary.
Hyperfunction: Hypofunction:
Cushings syndrome Adrenal insufficiency
Hyperthyroidism Hypothyroidism
GH excess GH deficiency
Prolactin excess Hypogonadism

Neurological symptoms: headache, visual disturbance.

Standard Pituitary Laboratory Tests

Thyroid
TSH, free T4

Reproductive
Prolactin
FSH, LH, testosterone (men) or estradiol (women)

GH
Critical to assess prolactin prior
IGF-I
proceeding to surgery
Adrenal Extra tests required if GH or ACTH
Cortisol excess is suspected

3184
Copyright © Harvard Medical School, 2018. All Rights Reserved.

Clinical Case 2-1

24 yr old woman G2P2, menses never


resumed after d/c OCP’s. ROS: HA’s, no
visual or other neuro c/o. + fatigue,
depression, and cold intolerance. Can’t
seem to lose weight she gained with last
pregnancy. + galactorrhea x 6 mos.
Meds: none.
What should you look for on exam?
PE: normal. What is the differential diagnosis?

Clinical Case 2-2

Labs What is the single most


E2 11; LH 6; FSH 5 important test to order
to exclude a physiologic
IGF-1 nl 183, GH 1.0 cause of elevated
α-subunit 0.8 (nl) prolactin?
cortisol 8.2 Hematocrit
Ferritin
FT4 1.0, TSH 1.5 Pregnancy test
Prolactin 283 Chest Xray

3185
Copyright © Harvard Medical School, 2018. All Rights Reserved.

Clinical Case 2-3

Labs What is the single most


E2 11; LH 6; FSH 5 important test to order
to exclude a physiologic
IGF-1 nl 183, GH 1.0 cause of elevated
α-subunit 0.8 (nl) prolactin?
cortisol 8.2 Hematocrit
Ferritin
FT4 1.0, TSH 1.5 Pregnancy test
Prolactin 283 Chest Xray
HCG negative

Clinical Case 2-4

Labs How do you interpret


E2 11; LH 6; FSH 5 the lab values?
IGF-1 nl 183, GH 1.0 What is the next step?
α-subunit 0.8 (nl)
What is the diagnosis?
cortisol 8.2
FT4 1.0, TSH 1.5 What are the treatment
Prolactin 283 options?
HCG negative

3186
Copyright © Harvard Medical School, 2018. All Rights Reserved.

Clinical Presentation of
Hyperprolactinemia

Galactorrhea **
Hypogonadism **
Oligo/amenorrhea
Infertility
Erectile dysfunction
Growth arrest / delayed puberty

Mass effects if tumor is large

DDx: Hyperprolactinemia
Physiologic states:
Suckling
Pregnancy, Lactation, Exercise, Stress, Sleep TRH DA

Medications E2
Lactotrope
Primary hypothyroidism
Systemic disorders: Prolactin
Neurogenic chest wall lesion, renal failure, cirrhosis,
seizures
Hypothalamic-pituitary stalk damage Breast
Radiation, infiltrations, cysts, tumors, trauma
Prolactinoma
Idiopathic
Macroprolactinemia

3187
Copyright © Harvard Medical School, 2018. All Rights Reserved.

Medications and Hyperprolactinemia


More Common Less Common
DA receptor blockers Antihypertensives
Phenothiazines, haloperidol, Methyldopa, reserpine,
resperidone, olanzepine verapamil
Domperidone, metoclopromide
Antidepressants
DA synthesis inhibitors Tricyclics, SSRIs (minimal)
Methyldopa
Opiates
Estrogens
Antihistamines
Cimetidine

Prolactinomas

Account for 30-40% of pituitary tumors


More common in women (3 fold higher) than men
Premenopausal women usually present earlier with clinical
symptoms and microadenomas.
Postmenopausal women and men usually present later with
macroadenomas

3188
Copyright © Harvard Medical School, 2018. All Rights Reserved.

Prolactinomas: Treatment Options

General Treatment Options:


Medical therapy with DA agonists
– Cabergoline (preferred), bromocriptine

Transsphenoidal resection
– Second line therapy in most cases

Radiation
– Generally reserved for resistant or aggressive tumors

Melmed et al. JCEM. 96: 273-288, 2011

Clinical Case 3-1

57 yr old male with multiple medical problems


who recently presented to PCP with SOB,
CP. Found to have new onset CHF, hypoxia
– LVEF 25%, cardiac cath normal.

PMH: Rheumatoid arthritis, gout, nephrolithiasis, colon polyps,


carpal tunnel syndrome, COPD.

Meds: Captopril, allopurinol, ASA, prednisone 5mg/d,


methotrexate.

3189
Copyright © Harvard Medical School, 2018. All Rights Reserved.

Clinical Case 3-2


PE: 126/88 P82.
HEENT: PERRLA, EOMI, VF full to confrontation, Fundoscopic exam
normal. Large tongue. Upper/lower dentures.
Neck: moderate symmetrical thyromegaly, no nodules palpated.
Lungs: bibasilar rales. RRR +S3
Abd: obese, o/w normal.
Ext: Large doughy hands. Shoe size 13, wide. No active joint inflammation.
Multiple skin tags.

What diagnosis are you considering?


What lab tests would you like?

Clinical Features of Acromegaly


Soft tissue hypertrophy
Arthritis / carpal tunnel
syndrome
Increased head, hand, foot
size.
Organomegaly
– Cardiomegaly with CHF
Metabolic Disturbances
– Diabetes mellitus
Obstructive sleep apnea
Dental malocclusion
Colon polyps/cancer
Increased mortality

3190
Copyright © Harvard Medical School, 2018. All Rights Reserved.

Clinical Case 3-3


Labs:
GH 2.4; IGF-1 985 (high)
FT4 = 0.7 (normal 0.7-2.7), TSH 0.4
FSH 11, LH 7.4, testosterone 234 (low normal)
Prolactin 7 (normal)

What diagnosis are you considering now?


Would you order any radiologic tests at this time?

Clinical Case 3-4

What test can be used to confirm GH


hypersecretion?
A. Midnight salivary GH
B. Oral glucose tolerance test for GH
suppression
C. Urinary IGF-1
D. Serum IGFBP-3

3191
Copyright © Harvard Medical School, 2018. All Rights Reserved.

Clinical Case 3-4

What test can be used to confirm GH


hypersecretion?
A. Midnight salivary GH
B. Oral glucose tolerance test for GH
suppression
C. Urinary IGF-1
D. Serum IGFBP-3

Acromegaly: Treatment
Surgery
Medical Therapy
Somatostatin analogs
– Octreotide LAR
– Lanreotide
– Pasireotide
Cabergoline
Pegvisomant

Radiation Goal is “biochemical cure”:


Normal IGF-I
Conventional
Normal GH suppression
Radiosurgery

3192
Copyright © Harvard Medical School, 2018. All Rights Reserved.

Clinical Case 4-1


50 yr old woman referred to PE: 176/90 P72
evaluate weight gain. Moon face, dorsocervical
Reports rapid 50lb wt gain. and supraclavicular fat pads
No HA, visual, neuro Thyroid 30g pebbly
complaints.
CTA B, RRR
“Dr. you must do something
about this weight!!!” Central obesity
PMH: hypothyroidism, HTN, Abd: +purple striae
“borderline DM.” 1+ edema
FMH: obesity, thyroid Neuro: normal
disease
Meds: L-T4, lisinopril, ASA What would you recommend?

Clinical Features of Cushing’s


Syndrome

Central obesity
Skin changes
Hirsutism
Menstrual irregularities
Hypertension, CAD
Muscle weakness
Clinical
Osteoporosis
presentation
can vary Mood disturbances
dramatically

3193
Copyright © Harvard Medical School, 2018. All Rights Reserved.

Clinical Case 4-2


You are concerned about Cushing’s syndrome in
this patient. What is the best first step in
evaluating this patient?
A. Screen for hypercortisolism
B. Obtain a pituitary MRI
C. Obtain a CT scan of the adrenal glands
D. Perform inferior petrosal sinus sampling to
determine if the source of excess cortisol is an
ACTH producing pituitary adenoma

Clinical Case 4-2


You are concerned about Cushing’s syndrome in
this patient. What is the best first step in
evaluating this patient?
A. Screen for hypercortisolism
B. Obtain a pituitary MRI
C. Obtain a CT scan of the adrenal glands
D. Perform inferior petrosal sinus sampling to
determine if the source of excess cortisol is an
ACTH producing pituitary adenoma

3194
Copyright © Harvard Medical School, 2018. All Rights Reserved.

Obesity and Cushing’s Syndrome

Many signs/symptoms/co-morbidities overlap and are


common in the general population:
Obesity / weight gain
Hypertension
DM
Sleep apnea / sleep disorders
Depression / “stress”

This is a rare disease.


The challenge is deciding who to screen for CS.

Screening for Cushing’s Syndrome


Endocrine Society Clinical Guidelines
Patients with “unusual” features for age
Osteoporosis, hypertension, etc

Patients with multiple and progressive clinical


features of Cushing’s syndrome
Children with decreasing height percentile and
increasing weight
Patients with adrenal incidentalomas
Nieman L et al. JCEM 93(5): 1526-40. 2008.

3195
Copyright © Harvard Medical School, 2018. All Rights Reserved.

Cushing’s Syndrome
Step 1: Document syndrome of hypercortisolism
Screening tests for hypercortisolism include:
– 24 hour urine free cortisol (x2)
– Late night salivary cortisol levels (x2)
– 1 mg overnight dexamethasone test

Step 2: Determine whether it is ACTH dependent or


independent.

Step 3: Localize tumor and remove.

Thyrotropinomas
Very rare! (approx. 0.5-1% of pituitary adenomas)
Clinical presentation:
Hyperthyroidism, goiter
– Patients often treated previously with thyroidectomy / I131
70% present with macroadenomas

Diagnosis:
– elevated T4, T3
– Inappropriately NORMAL or elevated TSH *******
– elevated α-subunit, molar ratio α-subunit/TSH>1

Treatment:
– Surgery (treatment of choice), somatostatin analogs

3196
Copyright © Harvard Medical School, 2018. All Rights Reserved.

“Nonfunctioning” Adenomas
Appear clinically inactive.
Often secrete α−subunit, FSHβ or LHβ subunit or intact
gonadotropins.

One third of all pituitary tumors.

Often present with mass effect symptoms only and


no evidence of hormonal overactivity.

Some patients with large tumors present with


panhypopituitarism.

Treatment of choice is surgery

Pituitary Adenomas:
Therapeutic Considerations

Treat symptoms related to mass effects.


Restore or preserve vision
Neurologic improvements – cranial nerves, headaches

Correct pituitary hyperfunction.


Aim for biochemical cure

Medical hormone replacement for hypopituitarism.

3197
Copyright © Harvard Medical School, 2018. All Rights Reserved.

Perioperative Management
Preoperative Evaluation: Long Term Management
Assess pituitary function Patients typically evaluated
Replace as needed 1, 6, 12 weeks post-
Thyroid & cortisol most important operatively.
Stress-dose glucocorticoids if MRI typically repeated at 12
necessary
week visit to serve as new
baseline.
Early Inpatient Management: Follow-up annually or as
dictated by clinic status.
Assess for complications:
Hormone assessment
Neurologic status
MRI
Endocrine
Diabetes insipidus Long term assessment of
SIADH hormone status and tumor
Adrenal insufficiency recurrence required
Woodmansee WW et al. AACE Clinical Review Endoc Prac. 21:832-838. 2015.

Hypopituitarism
Management
Treatment based on correcting hormone deficiencies.
Adrenal - hydrocortisone or prednisone. Use lowest dose possible.
– Stress dose coverage
– Mineralocorticoid replacement not necessary.
Thyroid – levothyroxine, after adrenal replacement
– ** remember TSH cannot guide Rx.
Gonadal - Men require testosterone, women may require estrogen-
progestin replacement. Gonadotropins for fertility.
Growth hormone – Need provocative testing. Can treat with rhGH.
Prolactin - no replacement available or required.
Posterior pituitary – desmopressin (DDAVP).
Medical Alert Jewelry

3198
Copyright © Harvard Medical School, 2018. All Rights Reserved.

Pituitary Disease Summary


“Take Home” Messages:
When evaluating patients with pituitary disorders, let pituitary
physiology be your guide. Evaluate:
Pituitary hyperfunction
– GH (acromegaly)
– ACTH (Cushing’s disease – hypercortisolism)
– Prolactin (galactorrhea, menstrual disorders, erectile dysfunction)
– TSH (hyperthyroidism)
Pituitary hypofunction – all hormone systems
Mass effects (headache, visual dysfunction)

Treatment is aimed at restoring normal pituitary function and can


include: surgery, hormone replacement, medications

Thank You

3199
Copyright © Harvard Medical School, 2018. All Rights Reserved.

General References/
Additional Reading
Prolactinomas/ hyperprolactinemia
Melmed S et al. Diagnosis & Treatment of Hyperprolactinemia: An Endocrine Society Clinical
Practice Guideline. J Clin Endocrinol Metab 2011. 96: 273-288.
Klibanski, A. Prolactinomas. N Engl J Med 2010. 362:1219-26.
Acromegaly
Katznelson L. et al. Acromegaly: An Endocrine Society Clinical Practice Guideline. J Clin Endocrinol
Metab 2014. 99: 3933-3951.
Cushing’s Syndrome
Nieman LK et al. The Diagnosis of Cushing’s Syndrome: An Endocrine Society Clinical Practice
Guideline. J Clin Endocrinol Metab 2008. 93: 1526–1540.
Nieman LK et al. Treatment of Cushing’s Syndrome: An Endocrine Society Clinical Practice
Guideline. J Clin Endocrinol Metab 2015. 100: 2807-2831.
Hypopituitarism
Fleseriu M et al. Hormonal Replacement in Hypopituitarism in Adults An Endocrine Society Clinical
Practice Guideline. J Clin Endocrinol Metab 2016. 101: 3888-3921.
Peri-operative Management
Woodmansee WW et al. AACE/ACE Disease State Clinical Review: Postoperative Management
Following Pituitary Surgery. Endocrine Practice. 2015. 21: 832-838.

3200
Copyright © Harvard Medical School, 2018. All Rights Reserved.

Take-Home Messages and Clinical


Pearls
Carolyn Becker MD
Master Clinician Educator
Division of Endocrinology, Diabetes and
Hypertension
Brigham and Women’s Hospital
Associate Professor of Medicine
Harvard Medical School

Disclosures
• Royalties from UpToDate for 2 chapters
on Premenopausal Osteoporosis
($1800/yr)

3201
Copyright © Harvard Medical School, 2018. All Rights Reserved.

Topics to be Reviewed
Diabetes/Obesity
Thyroid
Adrenal
Pituitary
Hypoglycemia

Case 1
• A 54 year old man presents to you for follow up
of T2DM, HTN, hyperlipidemia, and obesity (BMI
44 kg/m2).
• Six months ago, you added sitagliptin to
metformin and glipizide for HgbA1C of 8.2%.
• A1C improved to 7.6% but he wants help with
weight loss. He refuses to consider bariatric
surgery.
• In addition to prescribing a diet and exercise
program, you plan to switch from sitagliptin
to liraglutide 3.0 mg daily to help him lose
weight.

3202
Copyright © Harvard Medical School, 2018. All Rights Reserved.

Current Medications
• Metformin 1000 mg twice daily with meals
• Glipizide 5 mg twice daily with meals
• Sitagliptin 100 mg once daily to be
stopped and switched to liraglutide 3.0 mg
daily
• Lisinopril 20 mg daily
• Atorvastatin 40 mg daily

If You Make No Other Medication Changes


After Switching to Liraglutide, What is the
Patient At Greatest Risk For?
A. Rhabdomyolysis
B. Hypotension
C. Hypoglycemia
D. Hypertriglyceridemia
E. Hyperkalemia

3203
Copyright © Harvard Medical School, 2018. All Rights Reserved.

If You Make No Other Medication Changes


After Switching to Liraglutide, What is the
Patient At Greatest Risk For?
A. Rhabdomyolysis
B. Hypotension
C. Hypoglycemia
D. Hypertriglyceridemia
E. Hyperkalemia

Davies MJ. 2015 JAMA 314:687


Pi-Sunyer X. 2015 NEJM 371:11

Proper Use of GLP-1 Agonists


• GLP-1 agonists (eg. liraglutide) are used at a dose of 1.8
mg daily to improve glycemic control in patients with
T2DM.
• At a dose of 3.0 mg daily, liraglutide also is approved for
weight loss and can be an adjunct to diet and exercise
• However, in patients with T2DM on other glucose-
lowering medications there is a significantly higher risk of
hypoglycemia when liraglutide 3.0 mg is added.
• Hypoglycemic risk was ~25% for either metformin or
pioglitazone and ~43% with a sulfonylurea + liraglutide
3.0 mg daily
• Take home message: reduce sulfonylurea by 50% (or
stop completely) when starting high dose liraglutide
to help in weight loss in patients with T2DM

3204
Copyright © Harvard Medical School, 2018. All Rights Reserved.

Case 2
• A 28 year old woman comes to you for help in
losing weight.
• She gained 20 lbs after college due to a more
sedentary lifestyle and increased caloric intake
• She gained another 30 lbs in the past year
despite trying to cut back on food and increase
exercise. Her only medication is paroxetine for
depression.
• She started paroxetine 18 months ago

After Stopping Paroxetine, Which of the


Following Would You Begin for her
Depression?
A. Amitriptyline
B. Mirtazapine
C. Venlafaxine
D. Fluoxetine
E. Sertraline

3205
Copyright © Harvard Medical School, 2018. All Rights Reserved.

After Stopping Paroxetine, Which of the


Following Would You Begin for her
Depression?
A. Amitriptyline
B. Mirtazapine
C. Venlafaxine
D. Fluoxetine
E. Sertraline

Obesity and Anti-Depressants


• The SSRI paroxetine (Paxil) is associated with
the greatest weight gain in this list
• Of the tricyclics, amitriptyline (Elavil) promotes
the greatest weight gain.
• Mirtazapine (Remeron), venlafaxine (Effexor),
and duloxetine (Cymbalta) can all promote
weight gain though milder
• Sertraline (Zoloft), citalopram (Celexa), and
escitalopram (Lexapro) are weight neutral
• Fluoxetine (Prozac) and buproprion (Wellbutrin)
promote weight loss

3206
Copyright © Harvard Medical School, 2018. All Rights Reserved.

When Lifestyle Fails:


Weight Loss Medications for Obesity
• Phentermine (Adipex)
• Phentermine/topiramate (Qysmia)
• Orlistat (Xenical)
• Lorcaserin (Belviq)
• Naltrexone/bupropion (Contrave)
• Liraglutide (Saxenda)

Thyroid Case
• A 30 year old woman presents with mild fatigue,
5-lb weight gain, and occasional constipation
• She is 2 months postpartum and plans to breast-
feed for 1 year
• Her only medication is a prenatal MVI
• FH is positive for hypothyroidism in her mother
• On exam, she appears well, though tired
– Wt 130 lbs (60 kg), BMI 22, pulse 70 and regular
– Thyroid gland: normal in size and consistency
• Labs: TSH 30 mU/L (nl 0.5 – 5.0), free T4 1.2
(nl 0.8 – 1.5)

3207
Copyright © Harvard Medical School, 2018. All Rights Reserved.

What Would You Do Next?

A. Order a radioiodine scan and uptake


B. Begin levothyroxine 50 mcg daily
C. Begin levothyroxine 100 mcg daily
D. Ask her to return for a repeat TSH in 1
month
E. Check anti-thyroid antibodies

What Would You Do Next?

A. Order a radioiodine scan and uptake


B. Begin levothyroxine 50 mcg daily
C. Begin levothyroxine 100 mcg daily
D. Ask her to return for a repeat TSH in 1
month
E. Check anti-thyroid antibodies

3208
Copyright © Harvard Medical School, 2018. All Rights Reserved.

Postpartum Thyroiditis
A.RAI scan and uptake is indicated for evaluation
of thyrotoxicosis, not hypothyroidism, and is
contraindicated during lactation

B. and C. In a symptomatic hypothyroid patient,


levothyroxine 100 mcg/d (or 0.8 mcg/lb) would
be a good starting dose. Given her normal free
T4, a lower starting dose such as 50 mcg/d
might be considered.

Postpartum Thyroiditis
D. In the postpartum setting with minimal
symptoms, no goiter, and normal free T4, the
most likely diagnosis is postpartum thyroiditis
which will resolve 50% of the time without rx.
Monitoring TSH in this case is reasonable.

E. Positive anti-thyroid antibodies can be present


in both postpartum thyroiditis and Hashimoto’s
so will not help distinguish between the 2
possibilities.

3209
Copyright © Harvard Medical School, 2018. All Rights Reserved.

Case 3
• A 60 year old Caucasian woman admitted
with acute abdominal pain and sepsis is
found to have a ruptured appendix.
• She is treated with antibiotics and
emergency appendectomy via
laparotomy and does well intraoperatively.
• Over the next 48 hours, she experiences
nausea, vomiting and hypotension
requiring intravenous saline and pressors.

Case (cont)
• Her PMH is positive for arthritis of both
knees and spinal stenosis. She denies any
history of oral prednisone use.
• ROS: She noted some fatigue and mild
decreased appetite over the past 2 months
months.
• PE: she has generalized obesity with mild
facial plethora

3210
Copyright © Harvard Medical School, 2018. All Rights Reserved.

Case (cont)
• Labs as outpatient prior to surgery: Normal
complete metabolic panel, TSH, free T4
and FSH 55 (c/w menopause)
• Now: serum Na+ 130, K+ 3.8, serum
cortisol 1.8 mcg/dL, ACTH < 10 pg/mL.
• After 250 mcg of cosyntropin IV, her
serum cortisol rises from 2.0 to 3.5 mcg/dL
at 60 minutes.

Which Diagnosis is Most Likely to Explain


Her Adrenal Insufficiency?
A. Bilateral adrenal hemorrhage
B. Perioperative opiate exposure
C. Acute pituitary infarction
D. Exogenous glucocorticoids
E. Addison’s disease

3211
Copyright © Harvard Medical School, 2018. All Rights Reserved.

Which Diagnosis is Most Likely to Explain


Her Adrenal Insufficiency?
A. Bilateral adrenal hemorrhage
B. Perioperative opiate exposure
C. Acute pituitary infarction
D. Exogenous glucocorticoids
E. Addison’s disease

Exogenous Glucocorticoids
• This patient had received multiple injections of
methylprednisolone into her spine and several
joints over the past 15 months.
• When asked about taking “prednisone”, she did
not associate this with her steroid injections.
• Following her last injection 4 months ago, she
had mild symptoms of steroid withdrawal but
compensated until her acute sepsis, surgery and
anesthesia when she decompensated.

3212
Copyright © Harvard Medical School, 2018. All Rights Reserved.

Exogenous Glucocorticoids
• Her labs are consistent with chronic secondary
adrenal insufficiency.
• Opiates can transiently suppress the HPA axis
but cortisol post-cosyntropin should have
stimulated normally.
• ACTH would be HIGH in both adrenal
hemorrhage and Addison’s as these cause
primary adrenal insufficiency
• Acute pituitary infarction should have impacted
the other anterior pituitary hormones and
cosyntropin stimulation would be normal.

ACUTE Secondary Adrenal Insufficiency

H CRH
Manifestations/Characteristics:

Labs:
P • low basal cortisol
• inappropriately low ACTH
• ± hyponatremia
ACTH
• Normal K and aldosterone regulation

Physical:
Cortisol
• completely normal Glucocorticoid
Adrenal • mild, progressive, fatigue at baseline Receptor
• severe fatigue, orthostasis, Mineralocorticoid
Receptor
hypotension, in situations of stress
Target
Organ
Cell

3213
Copyright © Harvard Medical School, 2018. All Rights Reserved.

ACUTE Secondary Adrenal Insufficiency

Response to Cosyntropin:
H CRH

• NORMAL
P EXAMPLE:
60 mins following
Morning
250 µg cosyntropin
ACTH Cosyntropin
Cortisol (µg/dL) 2.2 26.0
ACTH (pg/mL) 10
Cortisol
Glucocorticoid
Receptor
Adrenal
Mineralocorticoid
Receptor

Target
Organ
Cell

CHRONIC Secondary Adrenal Insufficiency

With chronic ACTH deficiency, adrenal cortex


H CRH
(ZF) will atrophy, and will progressively
respond less to cosyntoprin stimulation
EXAMPLE:
P 60 mins following
Morning
250 µg cosyntropin
Cortisol (µg/dL) 2.2 4.5
ACTH Cosyntropin
ACTH (pg/mL) 10

Glucocorticoid
Cortisol Receptor
Adrenal
Mineralocorticoid
Receptor

Target
Organ
Cell

3214
Copyright © Harvard Medical School, 2018. All Rights Reserved.

Causes of Hyperprolactinemia
I. Physiologic II. Pharmacologic III. Pathophysiologic
Menstrual cycle Dopamine antagonists Primary hypothyroidism
Pregnancy - phenothiazines Chronic renal failure
Nursing - haloperidol Chest wall lesions
Nipple stimulation - risperidone Polycystic ovary syndrome
Stress - metoclopromide Idiopathic
- domperidone Macroprolactinemia
Amitriptyline Hypothal/pituitary lesions
Antihypertensives Prolactinoma
- methyldopa
- reserpine Other medications?
Verapamil Birth control pills?
Cimetidine Check TSH
Estrogens Stalk compression?

Evaluation of the Non-Diabetic Patient


with Low Glucose & Symptoms
Must get labs when the glucose is LOW
Glucose
Insulin
C-peptide
Sulfonylurea (SFU) screen
Others

3215
Copyright © Harvard Medical School, 2018. All Rights Reserved.

Proinsulin is Cleaved into Insulin and


C-Peptide and Secreted
Proinsulin
B Chain C-peptide A Chain

Insulin C-peptide
A Chain C-peptide
S S +
B Chain

When Glucose is Low (<50)


• INSULIN HIGH
– C-peptide high
• SFU negative insulinoma
• SFU positive sulfonylurea effect
– C-peptide low surreptitious insulin use
• INSULIN LOW
– Liver, heart, or kidney failure; sepsis, ETOH,
Cortisol or GH deficiency, nonpancreatic
tumors, inborn errors of metabolism,
inanition

3216
Copyright © Harvard Medical School, 2018. All Rights Reserved.

Disclosures
• Royalties from UpToDate for 2 chapters
on Premenopausal Osteoporosis
($1800/yr)

THANK YOU!

3217
Copyright © Harvard Medical School, 2018. All Rights Reserved.

Endocrine Board Review


Alexander Turchin, MD, MS
Associate Professor of Medicine, Harvard Medical School
Director of Quality in Diabetes, Division of Endocrinology,
Brigham and Women’s Hospital

Disclosures:
Brio Systems
Eli Lilly
Merck
Monarch Medical Technologies

3218
Copyright © Harvard Medical School, 2018. All Rights Reserved.

Case 1
A 68-year-old man is being treated with heparin for
pulmonary embolism. Four days after admission, he has
sudden onset of severe abdominal / flank pain and
tenderness. He is found to be hypotensive. Labs show
hyponatremia and hyperkalemia, and his hematocrit is
35 percent, as compared with 40 percent at the time of
admission.

The most appropriate next step is to:


A. Measure serum aldosterone
B. Measure serum cortisol
C. Measure serum cortisol before and after
administration of corticotropin (ACTH)
D. Measure urinary cortisol excretion
E. Start total fluid restriction at 1,200 cc / 24 hours

Case 1
A 68-year-old man is being treated with heparin for
pulmonary embolism. Four days after admission, he has
sudden onset of severe abdominal / flank pain and
tenderness. He is found to be hypotensive. Labs show
hyponatremia and hyperkalemia, and his hematocrit is
35 percent, as compared with 40 percent at the time of
admission.

The most appropriate next step is to:


A. Measure serum aldosterone
B. Measure serum cortisol
C. Measure serum cortisol before and after
administration of corticotropin (ACTH)
D. Measure urinary cortisol excretion
E. Start total fluid restriction at 1,200 cc / 24 hours

3219
Copyright © Harvard Medical School, 2018. All Rights Reserved.

Case 1: Explanation
This patient has acute primary adrenal insufficiency.
ACTH stimulation test (C) is the best way to make this
diagnosis.
A random cortisol measurement without ACTH
stimulation (B) can be difficult to interpret.
Decreased production of aldosterone (A) does not
necessarily indicate decreased production of cortisol,
which is potentially life-threatening.
Urinary cortisol excretion (D) is not a reliable test for
diagnosis of adrenal insufficiency.
Fluid restriction (E) is not indicated in treatment of
hyponatremia caused by adrenal insufficiency and can,
in fact, exacerbate the patient’s condition.

Case 2
A 52-year-old woman is brought to the office after falling and
striking her abdomen on the edge of a chair. She had
abdominal pain soon thereafter, but it has subsided. She is
normotensive. Physical examination is unremarkable except
mild abdominal tenderness. CT of the abdomen reveals a 3-
cm hypodense left adrenal mass with smooth borders. Serum
electrolytes are normal.
The most appropriate next step is to:
A. Measure plasma metanephrines
B. Fine needle aspiration of the mass
C. Measure 8 AM serum cortisol following 1 mg
dexamethasone at midnight
D. Both A and C
E. Repeat abdominal CT in six months

3220
Copyright © Harvard Medical School, 2018. All Rights Reserved.

Case 2
A 52-year-old woman is brought to the office after falling and
striking her abdomen on the edge of a chair. She had
abdominal pain soon thereafter, but it has subsided. She is
normotensive. Physical examination is unremarkable except
mild abdominal tenderness. CT of the abdomen reveals a 3-
cm hypodense left adrenal mass with smooth borders. Serum
electrolytes are normal.
The most appropriate next step is to:
A. Measure plasma metanephrines
B. Fine needle aspiration of the mass
C. Measure 8 AM serum cortisol following 1 mg
dexamethasone at midnight
D. Both A and C
E. Repeat abdominal CT in six months

Case 2: Explanation
Subclinical Cushing syndrome and pheochromocytoma
are relatively common in patients with adrenal
incidentalomas and must be ruled out (D).
Fine needle aspiration of an adrenal mass (B) can only
detect a metastatic lesion and is not indicated in a
patient without a known primary malignancy.
A repeat CT (E) to ensure that the nodule is not growing
will be helpful but is not the first priority – ruling out
hormonal overproduction is more urgent.

3221
Copyright © Harvard Medical School, 2018. All Rights Reserved.

Case 3
A 24-year-old veterinary student has had symptoms of
hypoglycemia before breakfast for several months.
Laboratory studies early one morning reveal the
following:
Serum glucose 28 mg/dl
Serum insulin 65 µU/ml (normal, 5-15)
Serum C-peptide 0.1 ng/ml (normal, 0.5-3.0)
Serum cortisol 27 µg/dl (normal, 8-25)
The most likely cause of these results is
A. Adrenal insufficiency
B. Non-islet-cell tumor
C. Insulinoma
D. Surreptitious administration of insulin
E. Surreptitious ingestion of glyburide

Case 3
A 24-year-old veterinary student has had symptoms of
hypoglycemia before breakfast for several months.
Laboratory studies early one morning reveal the
following:
Serum glucose 28 mg/dl
Serum insulin 65 µU/ml (normal, 5-15)
Serum C-peptide 0.1 ng/ml (normal, 0.5-3.0)
Serum cortisol 27 µg/dl (normal, 8-25)
The most likely cause of these results is
A. Adrenal insufficiency
B. Non-islet-cell tumor
C. Insulinoma
D. Surreptitious administration of insulin
E. Surreptitious ingestion of glyburide

3222
Copyright © Harvard Medical School, 2018. All Rights Reserved.

Case 3: Explanation
Elevated insulin and low C-peptide levels (D) are
consistent with exogenous insulin administration.
Adrenal insufficiency (A) is ruled out by a normal cortisol
level.
Hypoglycemia caused by a non-islet cell tumor (B) is
characterized by low insulin and elevated IGF-2 level.
Both an insulinoma (C) and sulfonylurea overdose (E)
would result in elevated insulin as well as C-peptide
levels.

Case 4
A 32-year-old man comes to see you for work-up of
infertility after he was found to have a low sperm count
and low testosterone. He is significantly taller than both
of his parents. His testicles are small and firm.
What is the best next step in the diagnostic workup:

A. Measure serum follicle stimulating hormone (FSH)


B. Measure serum luteinizing hormone (LH)
C. Order a pituitary MRI
D. Measure serum prolactin level
E. Examine his karyotype

3223
Copyright © Harvard Medical School, 2018. All Rights Reserved.

Case 4
A 32-year-old man comes to see you for work-up of
infertility after he was found to have a low sperm count
and low testosterone. He is significantly taller than both
of his parents. His testicles are small and firm.
What is the best next step in the diagnostic workup:

A. Measure serum follicle stimulating hormone (FSH)


B. Measure serum luteinizing hormone (LH)
C. Order a pituitary MRI
D. Measure serum prolactin level
E. Examine his karyotype

Case 4: Explanation
Based on the physical examination, the index of
suspicion is high that the patient has Klinefelter
syndrome (XXY). While this syndrome is characterized
by primary hypogonadism leading to elevation of FSH
(A) and LH (B), neither test is diagnostic. Karyotype (E)
is the best way to make the diagnosis. Prolactin (C) is
not affected in Klinefelter syndrome and there is no
anatomic pituitary pathology that could be visible on the
MRI.

3224
Copyright © Harvard Medical School, 2018. All Rights Reserved.

Case 5
A 71-year-old man comes to see you for follow-up. He
complains of gradually progressive burning pain in both feet
that is worse at rest and is relieved with walking; it is
interfering with his sleep. He has had type 2 diabetes for
over 20 years, poorly controlled over most of this time, but
more recently with A1c in upper 6s. His physical examination
reveals intact pulses but absent ankle reflexes in both feet.
Any of the medications below would be appropriate to offer
him except:
A. Oxycodone 5 mg every 6 hours as needed
B. Amitriptyline (Elavil) 25 mg at bedtime
C. Pregabalin (Lyrica) 50 mg every 8 hours
D. Capsaicin cream 3-4 times per day
E. Duloxetine (Cymbalta) 60 mg daily

Case 5
A 71-year-old man comes to see you for follow-up. He
complains of gradually progressive burning pain in both feet
that is worse at rest and is relieved with walking; it is
interfering with his sleep. He has had type 2 diabetes for
over 20 years, poorly controlled over most of this time, but
more recently with A1c in upper 6s. His physical examination
reveals intact pulses but absent ankle reflexes in both feet.
Any of the medications below would be appropriate to offer
him except:
A. Oxycodone 5 mg every 6 hours as needed
B. Amitriptyline (Elavil) 25 mg at bedtime
C. Pregabalin (Lyrica) 50 mg every 8 hours
D. Capsaicin cream 3-4 times per day
E. Duloxetine (Cymbalta) 60 mg daily

3225
Copyright © Harvard Medical School, 2018. All Rights Reserved.

Case 5: Explanation
This patient has painful diabetic neuropathy, as
evidenced by the quality of the pain, improvement with
activity and evidence of neuropathy (absent ankle
reflexes) on physical examination, as well as history of
poorly controlled diabetes.
While painful diabetic neuropathy can be self-limiting,
his symptoms are severe and are interfering with his
sleep, so treatment would be advised. Pregabalin (C),
and duloxetine (E) are FDA approved for treatment of
painful diabetic neuropathy, and there is strong evidence
for efficacy for amitriptyline (B) and capsaicin (D).
Opioids such as oxycodone (A) are not recommended
as the first line treatment of painful diabetic neuropathy.

Case 6
A 44-year-old woman has had weakness and nervousness
for several months. She also has noted occasional
palpitations and has lost 5 lbs. Her pulse rate is 108
beats/minute. She has mild eyelid retraction and a tremor of
her hands, but no thyroid enlargement or nodules.
Her serum TSH is 0.01 µU/ml (normal, 0.4-4.0) and serum
free thyroxine concentration is 2.0 ng/dl (normal, 0.8-1.6). Her
thyroid radioiodine uptake at 24 hours is 52 percent (normal,
15-35) with diffuse pattern. Pregnancy test is negative.
The next step is to:
A. Measure serum C-reactive protein
B. Administer I-123 radioiodine isotope
C. Administer I-131 radioiodine isotope
D. Start propylthiouracil
E. Start methimazole

3226
Copyright © Harvard Medical School, 2018. All Rights Reserved.

Case 6
A 44-year-old woman has had weakness and nervousness
for several months. She also has noted occasional
palpitations and has lost 5 lbs. Her pulse rate is 108
beats/minute. She has mild eyelid retraction and a tremor of
her hands, but no thyroid enlargement or nodules.
Her serum TSH is 0.01 µU/ml (normal, 0.4-4.0) and serum
free thyroxine concentration is 2.0 ng/dl (normal, 0.8-1.6). Her
thyroid radioiodine uptake at 24 hours is 52 percent (normal,
15-35) with diffuse pattern. Pregnancy test is negative.
The next step is to:
A. Measure serum C-reactive protein
B. Administer I-123 radioiodine isotope
C. Administer I-131 radioiodine isotope
D. Start propylthiouracil
E. Start methimazole

Case 6: Explanation
This patient has mild Graves disease. First line
treatment for Graves disease are thionamides, and
specifically methimazole (E).
Propylthiouracil (D) is associated with an increased risk
of liver failure.
Measurement of C-reactive protein level (A) does not
assist in management of Graves disease.
Treatment with I-131 isotope (C) usually leads to
permanent hypothyroidism and is therefore not
recommended as first line choice in most cases.
When radioiodine is used, I-131 isotope is preferred to
I-123 (B) because the former emits a large fraction of its
radiation as beta rays (electrons) which have a short
penetration depth and do not affect organs other than
the thyroid.

3227
Copyright © Harvard Medical School, 2018. All Rights Reserved.

Case 7
A 32-year-old woman has had erratic menstrual periods
since adolescence and amenorrhea for about 4 months. She
has had mild facial hirsutism for more than 10 years. She
recently gained about 5 pounds, and has had less energy
than in the past. She takes no medications. She has mild
facial hirsutism, but no striae, central adiposity, hot flashes, or
galactorrhea. Her prolactin is measured to be 52 ng/mL (nl 4-
30), and the pregnancy test is negative.
The most appropriate next step is to order:
A. Ovarian ultrasonography
B. Serum follicle stimulating hormone (FSH)
C. Pituitary MRI
D. Serum testosterone
E. Serum TSH

Case 7
A 32-year-old woman has had erratic menstrual periods
since adolescence and amenorrhea for about 4 months. She
has had mild facial hirsutism for more than 10 years. She
recently gained about 5 pounds, and has had less energy
than in the past. She takes no medications. She has mild
facial hirsutism, but no striae, central adiposity, hot flashes, or
galactorrhea. Her prolactin is measured to be 52 ng/mL (nl 4-
30), and the pregnancy test is negative.
The most appropriate next step is to order:
A. Ovarian ultrasonography
B. Serum follicle stimulating hormone (FSH)
C. Pituitary MRI
D. Serum testosterone
E. Serum TSH

3228
Copyright © Harvard Medical School, 2018. All Rights Reserved.

Case 7: Explanation
Mild hyperprolactinemia associated with secondary
amenorrhea can be due to primary hypothyroidism (E)
as low free T4 stimulates increased TRH production
from the hypothalamus which stimulates secretion of
both TSH and prolactin from the pituitary.
Ovarian ultrasonography (A) and measurement of
testosterone levels (D) are not helpful in diagnosing the
etiology of hyperprolactinemia.
FSH (B) will be suppressed in patients with
hypothalamic amenorrhea. She does not have hot
flashes so premature ovarian insufficiency is unlikely
and it would not be associated with hyperprolactinemia.
Hypothyroidism is more common than pituitary masses
and should be ruled out before a pituitary MRI (C) is
considered.

Case 8
A 68-year-old woman was found unresponsive at home by
her daughter. In the Emergency Department, her temperature
was 103.2, oxygen saturation 70% on room air, blood
pressure 90/40 and heart rate 115. When given oxygen she
was sleepy, but arousable. Her thyroid exam was normal. She
was hospitalized and treated with intravenous fluids and
antibiotics.
On day 2 TFTs were drawn because of persistent sinus
tachycardia. TSH was 0.15 µU/ml (nl 0.5-5.0) and free
thyroxine was 0.6 ng/dL (nl 0.8-1.6). The best next step is:
A. Pituitary MRI
B. Thyroid ultrasound
C. Thyroid I-123 scan and uptake
D. Levothyroxine 100 mcg daily
E. Re-evaluate in 4-6 weeks

3229
Copyright © Harvard Medical School, 2018. All Rights Reserved.

Case 8
A 68-year-old woman was found unresponsive at home by
her daughter. In the Emergency Department, her temperature
was 103.2, oxygen saturation 70% on room air, blood
pressure 90/40 and heart rate 115. When given oxygen she
was sleepy, but arousable. Her thyroid exam was normal. She
was hospitalized and treated with intravenous fluids and
antibiotics.
On day 2 TFTs were drawn because of persistent sinus
tachycardia. TSH was 0.15 µU/ml (nl 0.5-5.0) and free
thyroxine was 0.6 ng/dL (nl 0.8-1.6). The best next step is:
A. Pituitary MRI
B. Thyroid ultrasound
C. Thyroid I-123 scan and uptake
D. Levothyroxine 100 mcg daily
E. Re-evaluate in 4-6 weeks

Case 8: Explanation
This patient’s thyroid function tests are consistent with
both sick euthyroid syndrome and secondary
hypothyroidism. However, in a setting of critical illness in
a patient without a known pituitary lesion, sick euthyroid
syndrome is much more likely, and expectant
management (E) is recommended.
Pituitary MRI (A) would only be indicated if the patient’s
thyroid hormone levels do not normalize as she recovers
from her illness.
Thyroid ultrasound (B) and radioiodine scan and uptake
(C) are not helpful in diagnosing the etiology of
hypothyroidism.
Levothyroxine supplementation (D) has not been shown
to be of benefit in patients with sick euthyroid syndrome.

3230
Copyright © Harvard Medical School, 2018. All Rights Reserved.

Case 9
A 67-year-old woman comes for a follow-up visit two years
after initiating alendronate (Fosamax) for treatment of
osteoporosis. She takes calcium 500 mg twice daily and
vitamin D 800 units daily. She takes alendronate on Sunday
mornings together with the rest of her medications. She walks
a mile 5 days a week. Her 25-OH-D level is 32 ng/mL
(normal). Two years ago her T-score in the left hip was -2.6. A
week ago follow-up bone densitometry showed a 6%
decrease in the left hip (significant). The best next step is:
A. Ask her to skip the morning calcium on Sundays
B. Double her calcium dose
C. Double her vitamin D dose
D. Add raloxifene (Evista)
E. Add ibandronate (Boniva)

Case 9
A 67-year-old woman comes for a follow-up visit two years
after initiating alendronate (Fosamax) for treatment of
osteoporosis. She takes calcium 500 mg twice daily and
vitamin D 800 units daily. She takes alendronate on Sunday
mornings together with the rest of her medications. She walks
a mile 5 days a week. Her 25-OH-D level is 32 ng/mL
(normal). Two years ago her T-score in the left hip was -2.6. A
week ago follow-up bone densitometry showed a 6%
decrease in the left hip (significant). The best next step is:
A. Ask her to skip the morning calcium on Sundays
B. Double her calcium dose
C. Double her vitamin D dose
D. Add raloxifene (Evista)
E. Add ibandronate (Boniva)

3231
Copyright © Harvard Medical School, 2018. All Rights Reserved.

Case 9: Explanation
This patient has a failure of treatment with an oral
bisphosphonate. She takes alendronate together with
calcium which can decrease bisphosphonate absorption.
Therefore the first step is to make sure calcium and
alendronate are taken separately (A).
She already takes adequate calcium (B) and vitamin D
(C) doses. Increasing these is unlikely to be of clinical
benefit and could have adverse effects.
Neither raloxifene (D) nor ibandronate (E) have been
shown to reduce the risk of hip fractures.

Case 10
A 62-year-old man comes for follow-up of diabetes. He
used to be treated with metformin 1000 mg bid and glipizide
10 mg bid but two years ago glipizide was stopped and
glargine (Lantus) insulin started. He now takes 30 units of
glargine at night. He wakes up from hypoglycemia 2-3 times a
week but his daytime glucose ranges between 150-220
mg/dL. His A1C is 7.5%. The best next step is:
A.Stop glargine and restart glipizide
B.Take glargine in the morning instead of at night
C.Decrease glargine and add a rapid acting insulin before
every meal
D.Stop glargine and start detemir (Levemir) insulin at night
E.Ask him to eat a snack before going to bed

3232
Copyright © Harvard Medical School, 2018. All Rights Reserved.

Case 10
A 62-year-old man comes for follow-up of diabetes. He
used to be treated with metformin 1000 mg bid and glipizide
10 mg bid but two years ago glipizide was stopped and
glargine (Lantus) insulin started. He now takes 30 units of
glargine at night. He wakes up from hypoglycemia 2-3 times a
week but his daytime glucose ranges between 150-220
mg/dL. His A1C is 7.5%. The best next step is:
A.Stop glargine and restart glipizide
B.Take glargine in the morning instead of at night
C.Decrease glargine and add a rapid acting insulin before
every meal
D.Stop glargine and start detemir (Levemir) insulin at night
E.Ask him to eat a snack before going to bed

Case 10: Explanation


This patient has excessive basal insulin dose as
evidenced by nocturnal hypoglycemia; his
hyperglycemia is likely post-prandial. Therefore basal
insulin dose should be decreased and prandial insulin
added (C).
Replacing glargine with glipizide (A) will further increase
his daytime glucose levels.
Administering glargine in the morning instead of at night
(B) is unlikely to make a difference.
Replacing glargine with evening detemir (D) will not
reduce nocturnal hypoglycemia.
Bedtime snack (E) will reduce nocturnal hypoglycemia
but will not improve his overall glycemic control.

3233
Copyright © Harvard Medical School, 2018. All Rights Reserved.

Case 11
A 57-year-old woman is evaluated for severe hypertension
resistant to treatment with three anti-hypertensive
medications (ACE inhibitor, calcium channel blocker, and
HCTZ) and hypokalemia. She is found to have serum
aldosterone 24 ng/dL (nl 4.0 – 21.0) and plasma renin activity
0.2 ng/mL/hr (nl 0.6 – 3.0). Plasma metanephrine and
normetanephrine levels are normal. A 24 hour urine
aldosterone level is 20 mcg with urine sodium of 220 mEq.
Abdominal CT shows a 2-cm benign appearing nodule in the
right adrenal gland. The best next step is:
A. Abdominal MRI
B. Repeat the CT in 6 months
C. Refer to an experienced surgeon for right adrenalectomy
D. Adrenal vein sampling
E. Stop all anti-hypertensives and repeat biochemical tests

Case 11
A 57-year-old woman is evaluated for severe hypertension
resistant to treatment with three anti-hypertensive
medications (ACE inhibitor, calcium channel blocker, and
HCTZ) and hypokalemia. She is found to have serum
aldosterone 24 ng/dL (nl 4.0 – 21.0) and plasma renin activity
0.2 ng/mL/hr (nl 0.6 – 3.0). Plasma metanephrine and
normetanephrine levels are normal. A 24 hour urine
aldosterone level is 20 mcg with urine sodium of 220 mEq.
Abdominal CT shows a 2-cm benign appearing nodule in the
right adrenal gland. The best next step is:
A. Abdominal MRI
B. Repeat the CT in 6 months
C. Refer to an experienced surgeon for right adrenalectomy
D. Adrenal vein sampling
E. Stop all anti-hypertensives and repeat biochemical tests

3234
Copyright © Harvard Medical School, 2018. All Rights Reserved.

Case 11: Explanation


This patient has primary hyperaldosteronism. However,
adrenal nodules are common in this age group, while
excess aldosterone production could have a different
source (e.g. bilateral adrenal hyperplasia). Therefore
confirmation of laterality of aldosterone production (D) is
the first step and adrenalectomy (C) is premature.
Abdominal MRI (A) will not provide any additional
information.
Deferral of definitive treatment (B) of primary
hyperaldosteronism is not appropriate.
The diagnosis of primary hyperaldosteronism is
definitive and no further biochemical workup (E) is
needed.

Case 12
A 41-year-old man comes to your office complaining of
progressive erectile dysfunction over the last several years.
Evaluation shows testosterone 1,200 pg/ml (nl 1,800 – 6,900)
and prolactin of 68 ng/ml (nl 4-23), confirmed by dilution.
Pituitary MRI shows a 2.5-cm intrasellar mass consistent with
pituitary adenoma. He denies headaches; his neurological
examination is normal and visual fields are intact. Morning
cortisol is 12 mcg/dl, IGF-1 is normal, and LH and FSH are
low. The best next step is:
A.Refer to a neurosurgeon
B.Start bromocriptine (Parlodel)
C.Start cabergoline (Dostinex)
D.Start testosterone patch
E.Repeat pituitary MRI in 6 months

3235
Copyright © Harvard Medical School, 2018. All Rights Reserved.

Case 12
A 41-year-old man comes to your office complaining of
progressive erectile dysfunction over the last several years.
Evaluation shows testosterone 1,200 pg/ml (nl 1,800 – 6,900)
and prolactin of 68 ng/ml (nl 4-23), confirmed by dilution.
Pituitary MRI shows a 2.5-cm intrasellar mass consistent with
pituitary adenoma. He denies headaches; his neurological
examination is normal and visual fields are intact. Morning
cortisol is 12 mcg/dl, IGF-1 is normal, and LH and FSH are
low. The best next step is:
A.Refer to a neurosurgeon
B.Start bromocriptine (Parlodel)
C.Start cabergoline (Dostinex)
D.Start testosterone patch
E.Repeat pituitary MRI in 6 months

Case 12: Explanation


This patient has a non-secretory pituitary
macroadenoma and should be referred to a
neurosurgeon for resection (A) to prevent further
complications, such as optic chiasm compression.
Deferral of treatment (E) is not appropriate.
Prolactin elevation is mild and thus due to pituitary stalk
compression rather than production by the tumor.
Consequently treatment with bromocriptine (B) or
cabergoline (C) is not the appropriate therapy.
Gonadotropin production may recover after resection of
the tumor, and therefore testosterone supplementation
(D) is premature.

3236
Copyright © Harvard Medical School, 2018. All Rights Reserved.

Case 13
A 56-year-old previously healthy woman reports intense
flushing (her face turns purple) that’s been going on for 6
months. She underwent menopause 4 years ago and was
asymptomatic at the time. She also complains of frequent
watery bowel movements. She takes no medications. She
has one of the episodes during her visit and her blood
pressure falls to 102/61 down from 127/74. The best next
step is:
A.Magnetic resonance imaging of the brain
B.24-hour urine collection for catecholamines and
metanephrines
C.24-hour urine collection for 5-hydroxyindoleacetic acid (5-
HIAA)
D.24-hour urine collection for tryptase
E.Start estrogen supplementation

Case 13
A 56-year-old previously healthy woman reports intense
flushing (her face turns purple) that’s been going on for 6
months. She underwent menopause 4 years ago and was
asymptomatic at the time. She also complains of frequent
watery bowel movements. She takes no medications. She
has one of the episodes during her visit and her blood
pressure falls to 102/61 down from 127/74. The best next
step is:
A.Magnetic resonance imaging of the brain
B.24-hour urine collection for catecholamines and
metanephrines
C.24-hour urine collection for 5-hydroxyindoleacetic acid
(5-HIAA)
D.24-hour urine collection for tryptase
E.Start estrogen supplementation

3237
Copyright © Harvard Medical School, 2018. All Rights Reserved.

Case 13: Explanation


This woman’s symptoms are most consistent with
carcinoid syndrome; therefore 24-urine collection for
5-HIAA (C) is the best way to confirm the diagnosis.
Pituitary tumors do not typically cause flushing and thus
pituitary MRI (A) will not be helpful.
Pheochromocytomas (B) rarely cause flushing and are
not usually associated with diarrhea.
Serum (rather than urine) tryptase (D) measurement can
be used to diagnose systemic mastocytosis.
New onset of flushing several years after the
menopause is unlikely be due to estrogen deficiency (E).

Case 14
A 25-year-old man comes in for routine physical. He had
craniopharyngioma resection at the age of 12 and has been
taking levothyroxine ever since. His current dose is 112 mcg
daily. Blood tests show TSH of 0.05 µU/ml (nl 0.5-5.0) and
free thyroxine 0.7 ng/dL (nl 0.8-1.6). The best next step is:
A. Decrease levothyroxine to 88 mcg daily
B. Increase levothyroxine to 125 mcg daily
C. Radioactive iodine uptake
D. Pituitary MRI
E. Re-evaluate in 6 months

3238
Copyright © Harvard Medical School, 2018. All Rights Reserved.

Case 14
A 25-year-old man comes in for routine physical. He had
craniopharyngioma resection at the age of 12 and has been
taking levothyroxine ever since. His current dose is 112 mcg
daily. Blood tests show TSH of 0.05 µU/ml (nl 0.5-5.0) and
free thyroxine 0.7 ng/dL (nl 0.8-1.6). The best next step is:
A. Decrease levothyroxine to 88 mcg daily
B. Increase levothyroxine to 125 mcg daily
C. Radioactive iodine uptake
D. Pituitary MRI
E. Re-evaluate in 6 months

Case 14: Explanation


This patient with a known pituitary lesion that has been
surgically resected has secondary hypothyroidism. In
these circumstances, TSH is not a useful indicator of
thyroid hormone adequacy. His free thyroxine levels are
low and his levothyroxine dose should be increased (B)
rather than decreased (A). Deferral of treatment (E) is
not appropriate.
As hyperthyroidism is not suspected, radioactive iodine
uptake (C) will not be helpful.
His pituitary disease is well established and there is no
reason to suspect any changes that could be revealed
by a pituitary MRI (D).

3239
Copyright © Harvard Medical School, 2018. All Rights Reserved.

Case 15
A 71-year-old man returns to see you for follow-up of type 2
diabetes. He also has hypercholesterolemia, heart failure,
hypertension, osteoarthritis, remote history of pancreatitis and
bladder cancer and family history of medullary thyroid cancer.
His current medications include glipizide 10 mg bid and
simvastatin 20 mg qhs. His hemoglobin A1c is 8.2% and
fasting blood glucose 130-150 mg/dL. The best next step is to
add the following medication to glipizide:
A. metformin 1000 mg daily
B. liraglutide (Victoza) 0.6 mg SQ daily
C. sitagliptin (Januvia) 100 mg daily
D. pioglitazone (Actos) 15 mg daily
E. glargine (Lantus) insulin 15 units SQ qhs

Case 15
A 71-year-old man returns to see you for follow-up of type 2
diabetes. He also has hypercholesterolemia, heart failure,
hypertension, osteoarthritis, remote history of pancreatitis and
bladder cancer and family history of medullary thyroid cancer.
His current medications include glipizide 10 mg bid and
simvastatin 20 mg qhs. His hemoglobin A1c is 8.2% and
fasting blood glucose 130-150 mg/dL. The best next step is to
add the following medication to glipizide:
A. metformin 1000 mg daily
B. liraglutide (Victoza) 0.6 mg SQ daily
C. sitagliptin (Januvia) 100 mg daily
D. pioglitazone (Actos) 15 mg daily
E. glargine (Lantus) insulin 15 units SQ qhs

3240
Copyright © Harvard Medical School, 2018. All Rights Reserved.

Case 15: Explanation


In this patient metformin (A) and pioglitazone (D) are
contraindicated because of history of heart failure,
liraglutide (B) and sitagliptin (C) are contraindicated
because of history of pancreatitis, and pioglitazone (D) is
also contraindicated because of history of bladder
cancer. Liraglutide (B) is additionally contraindicated
because of family history of medullary thyroid cancer.
Therefore glargine insulin (E) is the only remaining
treatment option.

Case 16
A 65-year-old woman comes to see you as a new patient.
She has type 2 diabetes, hypertension and hypothyroidism.
She takes metformin XR 2000 mg daily, glimepiride 4 mg
daily, lisinopril 20 mg daily, and diltiazem XR 240 mg daily.
Her fasting lipid profile shows LDL cholesterol of 120 mg/dL,
HDL cholesterol 51 mg/dL, and triglycerides 167 mg/dL. The
best next step is:
A. Start ezetimibe (Zetia) 10 mg daily
B. Start niacin extended release 500 mg daily
C. Start fenofibrate (Tricor) 145 mg daily
D. Start simvastatin (Zocor) 40 mg daily
E. Start atorvastatin (Lipitor) 20 mg daily

3241
Copyright © Harvard Medical School, 2018. All Rights Reserved.

Case 16
A 65-year-old woman comes to see you as a new patient.
She has type 2 diabetes, hypertension and hypothyroidism.
She takes metformin XR 2000 mg daily, glimepiride 4 mg
daily, lisinopril 20 mg daily, and diltiazem XR 240 mg daily.
Her fasting lipid profile shows LDL cholesterol of 120 mg/dL,
HDL cholesterol 51 mg/dL, and triglycerides 167 mg/dL. The
best next step is:
A. Start ezetimibe (Zetia) 10 mg daily
B. Start niacin extended release 500 mg daily
C. Start fenofibrate (Tricor) 145 mg daily
D. Start simvastatin (Zocor) 40 mg daily
E. Start atorvastatin (Lipitor) 20 mg daily

Case 16: Explanation


This woman needs to lower her LDL (guidelines
recommend at least moderate intensity statin therapy for
most patients with diabetes).
Niacin (B) and fenofibrate (C) have not been
unequivocally shown to decrease the incidence of
cardiovascular events in patients with elevated
cholesterol, while ezetimibe (A) benefits are relatively
weak; they are therefore not recommended as first line
treatment.
Simvastatin (D) has significant drug-drug interactions
with many calcium channel blockers and is
contraindicated in doses greater than 10 mg daily in
patients who take diltiazem.
Consequently atorvastatin (E) is the best treatment
choice.

3242
Copyright © Harvard Medical School, 2018. All Rights Reserved.

Case 17
A 53-year-old woman comes to see you for follow-up of
type 2 diabetes. She also has hypertension and obstructive
sleep apnea. She has struggled with obesity for many years
and her current BMI is 37.5 kg/m2. She takes metformin 2,000
mg/day. Her A1c is 9.2%. Her blood pressure is well
controlled on lisinopril 20 mg daily, amlodipine 10 mg daily
and hydrochlorothiazide 25 mg daily. She has tried multiple
weight loss diets over the years but has always regained the
weight she loses. The best next step is:
A. Start glipizide 10 mg daily
B. Start glargine (Lantus) insulin 15 units at bedtime
C. Recommend that she consider a bariatric procedure
D. Start phentermine 30 mg daily
E. Start empagliflozin (Jardiance) 10 mg daily

Case 17
A 53-year-old woman comes to see you for follow-up of
type 2 diabetes. She also has hypertension and obstructive
sleep apnea. She has struggled with obesity for many years
and her current BMI is 37.5 kg/m2. She takes metformin 2,000
mg/day. Her A1c is 9.2%. Her blood pressure is well
controlled on lisinopril 20 mg daily, amlodipine 10 mg daily
and hydrochlorothiazide 25 mg daily. She has tried multiple
weight loss diets over the years but has always regained the
weight she loses. The best next step is:
A. Start glipizide 10 mg daily
B. Start glargine (Lantus) insulin 15 units at bedtime
C. Recommend that she consider a bariatric procedure
D. Start phentermine 30 mg daily
E. Start empagliflozin (Jardiance) 10 mg daily

3243
Copyright © Harvard Medical School, 2018. All Rights Reserved.

Case 17: Explanation


This woman needs to improve her glycemic control. She
is obese and has several reversible complications of
obesity; it would therefore be ideal for her to also lose
weight.
A bariatric procedure (C), such as a Roux-en-Y gastric
bypass or a sleeve gastrectomy, will achieve both of
these goals. Glipizide (A) and glargine (B) will lead to
weight gain. Phentermine (D) will achieve modest weight
loss, but is only recommended for short-term use and
will not improve her glucose control. Empagliflozin (E)
will only result in modest weight loss and is unlikely to
bring her A1c below 7.0%.

GOOD LUCK!

54

3244
Copyright © Harvard Medical School, 2018. All Rights Reserved.

Disclosures:
Brio Systems
Eli Lilly
Merck
Monarch Medical Technologies

55

REFERENCES

1. Williams Textbook of Endocrinology, 13th edition.

2. Werner & Ingbar's The Thyroid: A Fundamental and

Clinical Text, 10th edition.

3. American Diabetes Association. Standards of

Medical Care in Diabetes – 2018. Diabetes Care

2018; Volume 41; Supplement 1.

3245
Copyright © Harvard Medical School, 2018. All Rights Reserved.

Rheumatoid Arthritis: Diagnosis and Treatment

Paul F Dellaripa MD
Brigham and Women’s Hospital
Boston MA
Associate Professor of Medicine
Harvard Medical School

Disclosures
• Genentech
• Up to Date
• Bristol Myers Squibb

3246
Copyright © Harvard Medical School, 2018. All Rights Reserved.

Rheumatoid Arthritis
• RA is a systemic diseases that primarily affects joints and untreated
or undertreated disease can lead to significant pain, disability and
higher risk of death
• RA has extra-articular effects in the lung and other organs, and a
higher risk of cardiovascular disease
• Recognition of early disease offers the opportunity at early
intervention and control of disease and better functional outcomes
as well as the potential for lower mortality
• There are many treatment options in 2018 but early intervention with
defined goals and targets of therapy while minimizing risk is key

What is RA ?

3247
Copyright © Harvard Medical School, 2018. All Rights Reserved.

But RA is also this!

RA: clinical findings


• Female predominance (3:1)
• General peak of onset is 25-50 years
• Although can occur at any age
• Symmetric synovitis
• Small joint involvement is most common
• Hands especially wrists, MCP and PIP
• Cervical spine disease can occur;atlanto-axial instability or superior migration of the dens.
• LS spine is spared
• Disease may start or remain asymmetrical
• Extra-articular involvement
• Lung - Interstitial lung disease
• Higher risk of lymphoma
• Cardiovascular
• Vasculitis
• Pericarditis
• Coronary artery disease

3248
Copyright © Harvard Medical School, 2018. All Rights Reserved.

Other diagnosis to consider


• Seronegative spondyloarthritis (Psoriatic arthritis, ankylosing
spondylitis, reactive, IBD related)
• Infection (viral, Lyme, less common septic)
• SLE
• Polymyalgia rheumatica
• Sarcoid
• Crystalline disease (gout/pseudogout)
• Vasculitis

2010 Classification Criteria from ACR/EULAR


• Who should be tested?
• Patients with synovitis of at least one joint which has no other explanation
• Point system
• ≥6/10 points required to make the diagnosis of RA
• Points given based on
• Number of joints involved
• Serology (Rheumatoid factor and CCP/ACPA)
• Acute phase reactants
• Duration of symptoms

3249
Copyright © Harvard Medical School, 2018. All Rights Reserved.

Re-defining RA: 2010 Classification


Criteria from ACR/EULAR
Score
Descriptor
A. Joint involvement (use highest applicable category)
1 large joint 0
2-10 large joints 1
1-3 small joints (with or without large joint involvement) 2
4-10 small joints (with or without large joint involvement) 3
>10 joints (at least 1 small joint) 5
B. Serology (high-positive: ≥ 3x upper limit normal for the test)
Negative RF and negative ACPA 0
Low-positive RF or low-positive ACPA 2
High-positive RF or high-positive ACPA 3
C. Acute phase reactants
Normal CRP and ESR 0
Abnormally elevated CRP or ESR 1
D. Duration of symptoms
<6 weeks 0
≥6 weeks 1

RA and increased mortality risk


Age-Standardized Mortality 1976-2012 in NHS
200

200
150

150
10,000)

All RA
10,000

Seropositive RA
non-RA
(per

Seronegative RA
100
Deaths per

all-RA
100
sero+ RA
Non-RA
Deaths

sero- RA
50

50

0
0

1976 1980 1984 1988 1992 1996 2000 2004 2008


Calendar Sparks et al, presented at ACR 2014
Calendaryear
year
Sparks et al, Arthritis Care Res, 2016

3250
Copyright © Harvard Medical School, 2018. All Rights Reserved.

Mortality and RA – why the increased risk?


• Increased risk of cardiovascular disease (CVD)
• 2008 meta-analysis looking at 24 studies (111,758 patients) suggests 50%
increased risk of CVD deaths in RA
• Study by Wasko et al examining 5625 patients for 25 years
• Found 70% decreased risk of mortality in those taking methotrexate >1year
• This may be related to decreased CAD risk in patients who start
methotrexate

Avina-Zubieta et al, Arth Care Res, 2008


Wasko et al, Arth Rheum 2014

Mortality of RA-ILD

R
A

RA-ILD

Olson AJRCCM 201111


Bongartz Arth Rheum 2010
Park AJRCCM 2007
Kim Chest 2009

3251
Copyright © Harvard Medical School, 2018. All Rights Reserved.

What happens when you delay treatment?


• 598pts from EARLY RA LEIDEN COHORT of 1674 pts
• Consecutive pts enrolled between 1993-2006
• Study evaluated assessment delay and disease outcome defined as
rate of joint destruction and sustained DMARD free remission
• In pts with RA 69% were assessed >12 weeks after symptoms/referral
was associated with less DMARD free remission and more xray
progression over 6 yrs
• Those pts seen within 12 wks of symptom onset had greater DMARD
free remission and less progression (Van der Linden Arthritis Rheum
2010; 62:3537-3546)

Long-term impact of delay in assessment of patients with early arthritis

Arthritis & Rheumatism


Volume 62, Issue 12, pages 3537-3546, 30 NOV 2010 DOI: 10.1002/art.27692
http://onlinelibrary.wiley.com/doi/10.1002/art.27692/full#fig2

3252
Copyright © Harvard Medical School, 2018. All Rights Reserved.

Reductions in Radiographic Progression in Early Rheumatoid Arthritis Over Twenty-


Five Years: Changing Contribution From Rheumatoid Factor in Two Multicenter UK
Inception Cohorts (Carpenter L et al. Arthritis Care Res 69(12);1809-17 2017)

• 2 longitudinal inception cohorts in the UK


• 1465 pts between 1986-2001 (ERAS) and 1236 pts between 2002-
2013 (ERAN)
• RA- within 3yrs of symptoms onset
• XRAY – lower erosion scores at baseline in newer cohort ERAN and
lower progression rates, progression in 74% of ERAS and 27% ERAN
• DMARD use major factor in decline of progression, adequate
treatment may remove the predictive value of RF on xray progression

Reductions in Radiographic Progression in Early Rheumatoid Arthritis Over Twenty-Five Years: Changing
Contribution From Rheumatoid Factor in Two Multicenter UK Inception Cohorts

Arthritis Care & Research


Volume 69, Issue 12, pages 1809-1817, 6 NOV 2017 DOI: 10.1002/acr.23217
http://onlinelibrary.wiley.com/doi/10.1002/acr.23217/full#acr23217-fig-0001

3253
Copyright © Harvard Medical School, 2018. All Rights Reserved.

Reductions in Radiographic Progression in Early Rheumatoid Arthritis Over Twenty-Five Years: Changing
Contribution From Rheumatoid Factor in Two Multicenter UK Inception Cohorts

Arthritis Care & Research


Volume 69, Issue 12, pages 1809-1817, 6 NOV 2017 DOI: 10.1002/acr.23217
http://onlinelibrary.wiley.com/doi/10.1002/acr.23217/full#acr23217-fig-0003

So we know RA is bad for you and once you have


made the diagnosis you should treat ASAP….Then
what?
How can you assess risk to identify those with who will develop more
severe disease?
Based on that risk assessment, what agents should you start and how
to assess for efficacy?
In simplest terms, the goal is to start therapy within 3
months of disease onset if not sooner and methotrexate is
the preferred DMARD in the vast majority of patients.

3254
Copyright © Harvard Medical School, 2018. All Rights Reserved.

What constitutes poor prognosis?


• Functional limitation
• Extraarticular disease
• Rheumatoid factor positivity or presence of anticyclic citrullinated
peptide (CCP) antibodies
• Bony erosions documented radiographically

How do we do assessments?
• Disease Activity Score 28 joints (DAS28) =DAS28-ESR = ( 0.56 *
sqr(TJC)) + (0.28 * sqr(SJC)) + ( 0.7 * ln(ESR)) + (0.014 * GH)
• Simplified Disease Activity Index (SDAI)= SJC + TJC +PGA + EGA + CRP
• Clinical Disease Activity Index (CDAI) =SJC + TJC + PGA + EGA
• MD HAQ score.

• The patient-reported outcome measures include:


• Routine Assessment of Patient Index Data 3 (RAPID3)
• Patient Activity Scale (PAS) and PAS-II

3255
Copyright © Harvard Medical School, 2018. All Rights Reserved.

RAPID 3: patient centered measure of


function, pain and global status. ( Pincus et al
2009)
• RAPID3 (routine assessment of patient index data 3) is a pooled index
of the 3 patient-reported American College of Rheumatology
rheumatoid arthritis (RA) Core Data Set measures: function, pain, and
patient global estimate of status.
• Each of the 3 individual measures is scored 0 to 10, for a total of 30/3.
• Disease severity may be classified on the basis of RAPID 3 scores: >4 =
high; 2.1-4 = moderate; 1.1-2 = low; < or =1 = remission.
• RAPID3 scores are correlated with the disease activity score 28
(DAS28) and clinical disease activity index (CDAI)

2015 American College of Rheumatology Guideline for the Treatment of Rheumatoid Arthritis

Arthritis & Rheumatology


Volume 68, Issue 1, pages 1-26, 6 NOV 2015 DOI: 10.1002/art.39480
http://onlinelibrary.wiley.com/doi/10.1002/art.39480/full#art39480-fig-0003

3256
Copyright © Harvard Medical School, 2018. All Rights Reserved.

Methotrexate
• Very effective – standard of care as first choice in most cases of RA for
over 30 years
• Can be given orally or subcutaneously.
• Found to:
• Change natural history of RA
• Decrease extra-articular manifestations
• Increase QOL and even potentially survival

Methotrexate: potential toxicities


• Gastrointestinal discomfort (most common)
• Liver enzyme abnormalities (role of fibroscan)
• Hematologic abnormalities
• Opportunistic infections
• Effect on reproduction ( need to stop for at least 3 months before
attempting conception)
• Rare complications:
• Lung toxicity ( less than 1%)
• Nephrotoxicity
• ? Malignancy ( lymphoma)

3257
Copyright © Harvard Medical School, 2018. All Rights Reserved.

Methotrexate – what if patient cannot


tolerate MTX ?
• Leflunomide; mechanism of action similar to MTX, daily oral dose,
similar level of efficacy as MTX; need to monitor LFTs;teratogenic
• Sulfasalazine: slow onset of action, not used as a single agent but
often in triple therapy combination
• Hydroxychloroquine; milder agent, often used as part of combination
therapy, ocular toxicity risk.
• Targeted synthetic DMARDs (tofacitinib)

What to ensure your patient has before


starting immunosuppressive therapy
• Lab baseline
• LFTs, creatinine, CBC
• Hepatitis B and C screening
• TB testing (PPD or T spot/quantiferon gold)
• Inactivated vaccines
• Influenza
• Pneumonia (PCV13 and PPSV23)
• Live vaccines
• Most contraindicated when on immunosuppressive therapy
• If given prior to initiation must wait at least one month before starting therapy
• Exception is zoster
• Safe to give: azathioprine (<3mg/kg), methotrexate (<0.4mg/kg) or those taking prednisone <20 mg/d
• New vaccine available but information on side effect in rheumatic disease pts not available

3258
Copyright © Harvard Medical School, 2018. All Rights Reserved.

If someone doesn’t respond to MTX alone


then what?
• Combination therapy either other conventional DMARDs with MTX
• Biologic agent (TNFi or other)
• Combination of biologic agent with MTX
• Combination of conventional and synthetic DMARD

Population Early RA (< 3 yrs)


Type of study Randomized, double-blind trial

Treatment Groups -(IE) Immediate MTX plus etanercept


-(IT) Immediate triple therapy (MTX plus SSA plus HCQ)
-(SE) Step-up from MTX to MTX plus etanercept
-(ST) Step-up from MTX to triple therapy
Step-up if DAS28 > 3.2 at week 24

Length of study 2 years


Primary Outcome DAS28-ESR values from week 48 to week 102
Conclusion No difference in DAS28-ESR between 48-102 weeks in any group, IE and IT (equally)
more effective than SE and ST at week 24
Less radiographic progression in SE group as compared to ST

3259
Copyright © Harvard Medical School, 2018. All Rights Reserved.

• Based on the RACAT study


• Etanercept-MTX combination would result in 0.15
additional quality adjusted life year, but this would
cost 77,290 dollars

Anti-TNF Therapy
• 5 FDA approved TNF modulators in RA
• All have similar effects—with roughly a response rate of 60-70%
• Work in early disease and late disease—clinical outcomes better in
early disease possibly related to the presence of more inflammation
• Stabilize radiographic progression
• While can be used as monotherapy, often and most effectively used
in combination with MTX or other conventional DMARD and there are
multiple studies to support their efficacy (see next slide) .

3260
Copyright © Harvard Medical School, 2018. All Rights Reserved.

(A) Sustained remission (sREM), sustained low disease activity (sLDA) and American College of
Rheumatology (ACR)50 response at Week 52. in patients with early RA and poor prognostic
factors

P Emery et al. Ann Rheum Dis 2017;76:96-104

©2017 by BMJ Publishing Group Ltd and European League Against Rheumatism

Anti-TNF Therapy – adverse events


• Cutaneous
• Injection site reactions
• Diffuse rashes
• Infectious
• Sepsis, septic joints, pneumococcal infections
• Tuberculosis
• Cardiac
• Increased risk class III-IV CHF
• Neurologic
• Demyelination
• Oncologic
• Worry in the past has been increased risk of lymphoma and
other malignancies

3261
Copyright © Harvard Medical School, 2018. All Rights Reserved.

Malignancy and anti-TNF


• Skin cancers may be increased in patients taking anti-TNF but recent study shows
no identifiable increased risk of melanoma with anti-TNF (Ann Rheum Dis 2017;
76:386). We still recommend once yearly skin checks by dermatology in patients
on these medications
• Lymphoma and TNF inhibitors: Large cohort studies with no clear evidence of
increased risk (Arthritis Rheum 2006;54(9):2757-2764)
• Risk of malignancy in general is lower with TNFi and cDMARD compared to MTX
and abatacept and RTX (Solomon D Semin Arthritis Rheum. 2014 Feb;43(4):489-
97)

Rare potential side effects


• Pancytopenia and aplasia
• Autoimmune manifestations
• Granulomatous pulmonary disease
• Vasculitis
• SLE-type glomerulonephritis
• Psoriasis

3262
Copyright © Harvard Medical School, 2018. All Rights Reserved.

Absolute contraindications to
TNF-Blockers

• Heart failure: CHF III/IV


• Active/latent Tuberculosis
• Active infection
• Demyelinating disease - MS/optic neuritis
• Consider
• Malignancy – speak with oncologist prior to initiating

Abatacept – mechanism of action:blocks


activation of co-stimulatory pathway

3263
Copyright © Harvard Medical School, 2018. All Rights Reserved.

Abatacept
• Monthly infusions or weekly injections
• Similar risks to anti-TNF but less TB risk
• May be used with or without methotrexate
• Usually after TNF “failure”—response rate 50%
• Onset 4-16 weeks

Genovese M. N Engl J Med 2005;


353(11):1114-23.
1Kremer JM, et al. Ann Intern Med.

2006;144:865-876.

Rituximab

• B cell depletion though may have other effects on immune system


• Works about 50% in anti-TNF failures
• Onset 6-12 weeks-may have long term effect
• 2 infusions—1000mg vs 500mg
• For use mainly in seropositive pts

• Taylor R ,Lindorfer M
• Nature Rev 2006

3264
Copyright © Harvard Medical School, 2018. All Rights Reserved.

Rituximab: Adverse events


• Infections including PML ( due to JC virus) , HepB, others
• Rashes including psoriasis
• Low cell counts ( WBC)
• Low IgG for long durations--?significance
• Need to check Hep B core IgM prior to starting RTX

Tocilizumab
• Monoclonal antibody that blocks IL-6r
• 50% response rate in TNF failures
• Onset 2-12 weeks
• Impressive improvements in CRP, QOL, fatigue
• Adverse events:
• Infections
• ? GI perforation - avoid in patients with history of diverticulitis
• Lipid, leukocyte and liver test abnormalities

3265
Copyright © Harvard Medical School, 2018. All Rights Reserved.

Mechanism of Tocilizumab
(Kim GW et al Archives of Pharmacal Research 2015)

Gerd R Burmester et al. Ann Rheum Dis 2016;75:1081-1091

©2016 by BMJ Publishing Group Ltd and European League Against Rheumatism

3266
Copyright © Harvard Medical School, 2018. All Rights Reserved.

Tofacitinib:a new synthetic DMARD


• First oral biologic agent targets Janus Kinase molecule 1 and 3
• Indication- Active moderate to severe RA, inadequate response or
intolerance to MTX
• Monotherapy or in combination with MTX
• Combination of tofacitinib (5 mg twice daily) with MTX (15 to 25 mg
per week) similar in efficacy to adalimumab plus MTX and appears
more efficacious than tofacitinib alone (Fleischmann R et al Lancet.
2017;390(10093):457)

JAK inhibition

3267
Copyright © Harvard Medical School, 2018. All Rights Reserved.

JAK inhibitors-potential complications


• Infections
• Opportunistic infections
• Herpes Zoster ( risks inc steroid use, age, Asia , duration of use)
• Lipid abnormalities
• Neutropenia
• Anemia
• Gastrointestinal perforation
• Increase serum creatinine –
• Clinical significance?
• LFTs
• Malignancy

Corticosteroids in RA: there is controversy


• Use in early disease to control inflammation is reasonable though
long term use is fraught with hazard and significant adverse events
including bone fragility, hypertension,infection, glaucoma
• Some advocate for use of low dose steroids (5-7.5 mg) in addition to
DMARD and biologic use
• Our goal is to limit use of CS to lowest possible dose or none at all.

3268
Copyright © Harvard Medical School, 2018. All Rights Reserved.

If remission is achieved, can you with withdraw or lower


doses of biologics? The answer is you may be able to lower dose but
most patients will flare if you withdraw drug entirely (Smolen J et al Lancet
381:2013)
• PRESERVE trial:604 (72·4%) of 834 enrolled patients who achieved
remission on MTX/etanercept were eligible for the double-blind period, of
whom 202 were assigned to 50 mg etanercept plus methotrexate, 202 to
25 mg etanercept plus methotrexate, and 200 to placebo plus
methotrexate.
• At week 88, 166 (82·6%) of 201 patients who had received at least one
dose of 50 mg etanercept and one or more DAS28 evaluations had low
disease activity, compared with 84 (42·6%) of 197 who had received
placebo (mean difference 40·8%, 95% CI 32·5–49·1%; p<0·0001).
• Additionally, 159 (79·1%) of 201 patients given 25 mg etanercept had low
disease activity at week 88 (mean difference from placebo 35·9%, 27·0–
44·8%; p<0·0001).

Phase III Study Evaluating Continuation, Tapering, and Withdrawal of


Certolizumab Pegol After One Year of Therapy in Patients With Early
Rheumatoid Arthritis (Weinblatt ME et al. Arthritis Rheumatol 2017 Oct; 69(10): 1937–1948)

3269
Copyright © Harvard Medical School, 2018. All Rights Reserved.

Take home points


• Rheumatoid arthritis is a treatable disease with improved outcomes
when the disease is treated early
• Methotrexate is the cornerstone of therapy in most patients
• There are a variety of combinations of therapies that involve
conventional DMARDS, biologics and synthetic DMARDS that can be
used in combination with methotrexate in those with poor prognostic
factors.
• Attention to vaccine use and risk for infection is paramount
• Early referral to a rheumatologist is encouraged.

Question 1
• A 44 year-old man who is doing well on methotrexate 25mg/week for
18 months develops a cough, fever and malaise. CXR reveals a diffuse
infiltrate and small pleural effusions.

3270
Copyright © Harvard Medical School, 2018. All Rights Reserved.

Question 1
The next interventions should all be performed EXCEPT:
• A. Cessation of methotrexate
• B. Chest CT scan
• C. Emergency Bronchoscopy
• D. Broad Spectrum antibiotics and steroids
• E. Sputum for culture/sensitivity and methenamine silver as well as
1,3 beta glucan

Question 1: Answer C
• Emergency bronchoscopy
• While this may be needed eventually, other interventions should be
initiated first. If opportunistic infection ( PJP, fungal) is suspected then
bronchoscopy may be warranted. A transbronchial biopsy is useful if
granulomatous disease is suspected.

3271
Copyright © Harvard Medical School, 2018. All Rights Reserved.

Question 2
• A 50 yo female with a history of MS and sulfa allergy presents with 2
months of joint pain in her hands and feet. She is RF and CCP + high
titer , CRP 20. The patient is started on Methotrexate and increased
to 25 mg (orally then changed to subcutaneous) and 4 months later
has a modest improvement in her pain and function. Her RAPID 3
score went from 8 to 5. and CRP of 8. Her liver enzymes have been
normal except with one elevation less than 2X normal which is now
no longer present

What therapeutic options would you


consider to be the best option at this time?
• A. Begin triple therapy with MTX/HCQ/SSZ
• B. Add a TNF inhibitor
• C. Add leflunomide
• D. Add abatacept sq weekly

3272
Copyright © Harvard Medical School, 2018. All Rights Reserved.

The best answer is D


• In this pt with RA with a higher risk for poor prognosis and MS and
partial response to MTX , adding an additional agent makes sense.
• In this case TNF inhibitors are not recommended in pts with MS,
triple therapy cannot be used due to sulfa allergy
• leflunomide which can be used in combination with MTX but does
have a higher risk of liver inflammation and there were liver enzyme
abnormalities in this patient
• In this case, the best available option is to add abatacept.

Question 3
• A 50 yo female developed painful swollen joints in the wrists, hands,
knees and feet 1 year ago, high titer CCP antibody. X-rays show small
erosions in the MCPs of the hands. She is initially started on
methotrexate and then adalimumab is added with remission within 9
months. She is doing well but doesn’t like taking all of these
medications and wants to know if she can get off of them soon

3273
Copyright © Harvard Medical School, 2018. All Rights Reserved.

How would you advice this patient?


• A. you have severe erosive RA and you will be on these meds or
similar for the rest of your life
• B. Given that you are in remission, we can begin to taper one med at
a time and you should be able to get off both meds
• C. stop the methotrexate
• D. Withdrawal of medication in RA can be done but the rate of flare is
high (>60-70%) though some patients may be able to maintain
remission with less medication.

The correct answer is D


• Withdrawal of medication in RA is challenging in many patients
though recent data (3 trials so far) suggest patients may tolerate
lower doses of their medication.

3274
Copyright © Harvard Medical School, 2018. All Rights Reserved.

References
• Wasko MC, Dasgupta A, Hubert Het al. Propensity-adjusted association of methotrexate with overall survival in
rheumatoid arthritis. 2013 Feb;65(2):334-42.
• Singh JA, Saag KA, Bridges SL, et al . American College of Rheumatology Guideline for the Treatment of Rheumatoid
ArthritisArthritis & Rheum 2015; 68(1) :1-26
• O’Dell J, Mikuls T, Taylor TH et al. Therapies for Active Rheumatoid Arthritis after Methotrexate Failure N Eng J Med 2013
2013 :369(4) ;307
• Weinblatt ME et al. Phase III Study Evaluating Continuation, Tapering, and Withdrawal of Certolizumab Pegol After One
Year of Therapy in Patients With Early Rheumatoid Arthritis Arthritis Rheumatol 2017 Oct; 69(10): 1937–1948
• Solomon DH, Kremer JM, Fisher M et al. Comparative cancer risk associated with methotrexate, other non-biologic and
biologic disease-modifying anti-rheumatic drugs.2014 Feb;43(4):489-97
• Fleischmann R , Mysler E, Hall S et al Lancet. Efficacy and safety of tofacitinib monotherapy, tofacitinib with methotrexate,
and adalimumab with methotrexate in patients with rheumatoid arthritis (ORAL Strategy): a phase 3b/4, double-blind,
head-to-head, randomised controlled trial.Lancet 2017 Jul 29;390(10093):457-468.

3275
Copyright © Harvard Medical School, 2018. All Rights Reserved.

Update in Rheumatic Diseases:


Scleroderma/Sjögrens/Myositis

Laura L Tarter MD

Assistant Professor of Medicine


Harvard Medical School
Division of Rheumatology
Brigham and Women’s Hospital

Disclosures

None

3276
Copyright © Harvard Medical School, 2018. All Rights Reserved.

Outline
1. Scleroderma
2. Sjögren’s syndrome
3. Myositis

• Clinical manifestations and diagnosis


• Treatment options

Systemic Sclerosis (SSc/Scleroderma)


• Complex systemic disease that affects the
skin, lung, heart, GI tract, and kidney
• Incidence = 19 cases / million
• Pathophysiology
• Vascular and endothelial dysfunction
• Autoimmunity
• Fibroblastic activation and proliferation

3277
Copyright © Harvard Medical School, 2018. All Rights Reserved.

Scleroderma = Hardening of the Skin


• Localized scleroderma
• Morphea

• Linear scleroderma

• Systemic sclerosis
• Diffuse
• Limited (CREST)
• Sine scleroderma
• Overlap Syndrome

Katz KA, Derm Online

Limited Systemic Sclerosis: CREST


• Calcinosis
• Raynaud’s
• Esophageal Dysmotility
• Sclerodactyly
• Telangiectasias

ACR Image Bank

3278
Copyright © Harvard Medical School, 2018. All Rights Reserved.

Limited vs Diffuse SSc


Limited Diffuse

Skin thickening Face, forearms, lower Face, trunk, entire


legs, hands and feet arms/legs, hands and
feet; tendon friction rubs
Raynaud’s phenomenon Progressive disease Rapid onset following RP:
following onset of RP months to 3 years
Autoantibodies Association with anti- Association with anti-Scl
centromere Ab (50-60%) 70 ab (30%)

Internal organ Late: primarily pulmonary Early: ILD (75%),


involvement HTN (10-30%), myocardial disease,
esophageal dysmotility diffuse GI involvement,
scleroderma renal crisis
(10-15%)
Prognosis 10-yr survival > 70% 10-yr survival 40%

Skin Thickening

3279
Copyright © Harvard Medical School, 2018. All Rights Reserved.

Puffy Hands of Early SSc

Sclerodactyly

ACR Image Bank

3280
Copyright © Harvard Medical School, 2018. All Rights Reserved.

Mauskopf Facies

Natural History of Skin Tightening in SSc

Medsger TA et al. Rheum Dis N Am 2003

3281
Copyright © Harvard Medical School, 2018. All Rights Reserved.

Raynaud’s Phenomenon

• Not an infrequent initial presentation of CTDs


• Episodic, reversible digital skin color change
• White to blue to red

• Well-demarcated

• Due to vasospasm

• Usually cold-induced

Raynaud’s Phenomenon

3282
Copyright © Harvard Medical School, 2018. All Rights Reserved.

Raynaud’s Phenomenon

Raynaud’s Phenomenon
• Primary Raynaud’s
• Common in young women (<age 30)
• Often have + family history
• ANA mostly negative
• Secondary Raynaud’s
• Onset in > age 35
• Digital ulcers, pitting scars in fingers
• Abnormal capillary micrscopy
• Presence of autoantibodies

3283
Copyright © Harvard Medical School, 2018. All Rights Reserved.

Secondary Raynaud’s Phenomenon


• Connective tissue diseases
• SSc, SLE, MCTD, Undifferentiated CTD,
Sjogren’s syndrome, Dermatomyositis
• Occlusive arterial disease
• Atherosclerosis, Antiphospholipid Syndrome,
Buerger’s disease
• Vascular injury
• Frostbite, vibratory trauma
• Vinyl chloride, bleomycin, amphetamines,

cocaine
• Hyperviscosity/ cold-reacting proteins

Digital Ulcer in SSc

3284
Copyright © Harvard Medical School, 2018. All Rights Reserved.

Nailfold Capillaroscopy: Periungual Changes

SSc: Autoantibodies
• ANA + in nearly all cases
• Often nucleolar
• Anti-centromere
• 50-60% limited SSc
• Anti-Scl-70 (anti-topoisomerase ab)
• 30% diffuse SSc
• Anti-RNA polymerase III
• Associated with increased risk of renal crisis
• Other antibodies: anti-DNA polymerase, anti-U3-
RNP, anti-PM-Scl
• Often seen in overlap disease

3285
Copyright © Harvard Medical School, 2018. All Rights Reserved.

SSc: Internal Organ Involvement


• Interstitial lung disease
• Leading cause of mortality
• Occurs in 75% diffuse SSc
• Often within the first 4 years

• Pulmonary hypertension
• More common in limited SSc (10-30%)
• Usually occurs years into illness
• Concomitant pulmonary hypertension and ILD
• Extremely challenging clinical scenario

• Increased rate of malignant lung neoplasms

SSc: Internal Organ Involvement


• Renal Disease
• 60-80% diffuse SSc in autopsy studies
• Microalbuminuria, HTN, mild Cr rise ~ 50%
• Renal crisis 10-15% (early diffuse SSc)

• Cardiac Involvement
• Congestive heart failure, myocardial fibrosis
• Pericarditis (10-20%)
• Arrhythmias
• GI Involvement (90%)
• Esophageal hypomotility/ LES incompetence
• Gastroparesis
• Watermelon stomach (vascular ectasia in the antrum)

3286
Copyright © Harvard Medical School, 2018. All Rights Reserved.

Interstitial Lung Disease in SSc

SSc: Esophageal Dysmoliity (?Role in ILD)

3287
Copyright © Harvard Medical School, 2018. All Rights Reserved.

SSc Treatment: ILD


• Current standard of care
• Modest improvement lung function/skin scores with
Cyclophosphamide
• MMF

• Anti-fibrotics in IPF: FDA approved 2014


• Pirfenidone
• Nintedanib
• Ongoing trials in CTD ILD
• Numerous other trials in CTD ILD

SSc Treatment: Ther Has Been Progress!


• Renal: ACE inhibitors
• PAH: Prostacyclin, Endothelin antagonists,
Phosphodiesterase inhibitors
• Reflux: PPI high dose, anti reflux surgery
• Raynaud’s: Endothelin antagonists, Prostacyclin,
Phosphodiesterase inhibitors, Surgery, Botox
• Lung transplant

3288
Copyright © Harvard Medical School, 2018. All Rights Reserved.

Sullivan KM et al., SCOT Study Investigators


January 4, 2018

Key Points in Scleroderma


• ILD remains the leading cause of mortality
• Treatment options have improved
• Raynaud’s that develops > age 35 raises
concern for a rheumatic disease
• The presence of concomitant Raynaud’s and
GERD should raise suspicion for limited SSc
• Pulmonary HTN is a complication of long
standing limited SSc; screening is essential as
treatments are available that improve morbidity
and possibly mortality

3289
Copyright © Harvard Medical School, 2018. All Rights Reserved.

Sjögren’s Syndrome (SS)


• Autoimmune disorder characterized by
salivary and lacrimal gland dysfunction
• Decreased production of tears and saliva

• Primary and secondary


• Prevalence primary SS = 2-10 per 10,000
• Pathophysiology
• Proliferation and infiltration of lymphocytes in
exocrine glands
• Autoantibody production and the role of B cells in
the pathophysiology of this disease are the
source of ongoing investigation

Sicca Syndrome Manifestations


• Keratoconjunctivitis sicca
• Ocular dryness

• Corneal injury

• Xerostomia
• Oral dryness, dysphagia

• Dental caries, thrush

• Nasal dryness and epistaxis


• Vaginal dryness
• Dyspareunia

• Candidiasis

3290
Copyright © Harvard Medical School, 2018. All Rights Reserved.

Classification Criteria for Primary SS


Item Weight/Score
Labial salivary gland with focal lymphocytic 3
sialadenitis and focus score of > 1 foci/4 mm2
Anti-SSA/Ro positive 3

Ocular Staining Score >5 in at least 1 eye 1

Schirmer’s test <5 mm/5 min in at least 1 eye 1

Unstimulated whole saliva flow rate <0.1 1


mL/min
Score of >4 needed to establish dx
* exclusions: active hepatitis, head/neck xrt, sarcoidosis AIDS,
amyloidosis, IgG4-related disease
Shiboski CH et al., Ann Rheum Dis 2017

SS: Parotid Gland Enlargement

3291
Copyright © Harvard Medical School, 2018. All Rights Reserved.

SS: Corneal Abrasions

SS: Salivary Hypofunction

3292
Copyright © Harvard Medical School, 2018. All Rights Reserved.

Primary SS: Extraglandular Disease

• Peripheral neuropathy
• Vasculitis
• Interstitial lung disease (LIP, NSIP)
• Synovitis
• Risk factors for more aggressive disease:
+RF, low C4, cryoglobulinemia
• Increased risk of lymphoma

50 yo with Known SS with 8 lb Weight Loss


Lung biopsy c/w Marginal Zone Lymphoma

3293
Copyright © Harvard Medical School, 2018. All Rights Reserved.

SS: Associated Conditions


• Connective tissue diseases
• SLE
• RA
• Systemic sclerosis

• Hypothyroidism
• Cryoglobulinemia
• Autoimmune hepatitis

Sjögren’s Syndrome: Treatment


• Exocrine gland dysfunction
• Xerostomia: oral hygiene and agents to
stimulate salivary secretion
(pilocarpine/muscarinic agonist )
• KCS: cellulose products to augment tear
replacement and topical cyclosporine
• Treatment of extraglandular disease is difficult
• Trials are ongoing, mostly in the area of B cell
directed therapy
• Hydroxychloroquine may be useful in those with
fatigue and arthralgias

3294
Copyright © Harvard Medical School, 2018. All Rights Reserved.

Key Points About Sjögren’s Syndrome


• Higher risk of non-Hodgkins lymphoma in
primary Sjögrens (up to 44 fold or 5% lifetime)
• Most cases are secondary to other rheumatic
diseases
• Treatment geared towards managing sicca
symptoms and promoting oral hygiene
• Treatment of extraglandular disease is
challenging
• Patients with low titer ANA, fatigue, and eye or
mouth dryness are a significant challenge
diagnostically

Inflammatory Myopathies
• Group of autoimmune disorders
• Common feature = immune-mediated muscle injury
• Usually present with muscle weakness and
elevated muscle enzymes (CK/aldolase)
• Disorders include
• Dermatomyositis (DM)
• Polymyositis (PM)
• Overlap myositis (with another systemic
rheumatic disease)
• Inclusion body myositis (IBM)
• Necrotizing autoimmune myositis (NAM)
• Histopathologic and clinical distinctions
Senecal et al., Arth Rheum 2017

3295
Copyright © Harvard Medical School, 2018. All Rights Reserved.

Inflammatory Myopathies: DDx


• Inflammatory myopathies • Amyloid myopathy
• Drug-induced myopathies • Sarcoid myopathy
• Steroids • Metabolic myopathies
• Statins • Disorders of carbohydrate
• Colchicine and lipid metabolism
• Hydroxychloroquine • Hypothyroidism
• Alcohol • Electrolyte disturbances
• Zidovudine • Hyper/hyponatremia
• Infections • Hypokalemia
• Viral • Hypophosphatemia
• Toxoplasmosis • Hypocalcemia
• Trichinosis • Neurologic disorders
• Bacterial pyomyositis • Myasthenia gravis
• Systemic vasculitis • Motor neuron disease
• PAN, GPA, eGPA • Muscular dystrophy

Inclusion Body Myositis v


Idiopathic Inflammatory Myopathy
Inclusion Body Myositis IIM

Sex Male > female Female > male

Age Usually > 50 Common before 50

Onset Slowly progressive Acute or sub-acute

Weakness Distal and asymmetric Proximal and symmetric


muscle weakness
EMG Myopathic and neuropathic Myopathic changes
changes
Muscle biopsy Mononuclear cell infiltrates Inflammation, fiber necrosis
and vacuoles containing
amyloid
Response to Generally poor Generally good
immunosuppression

3296
Copyright © Harvard Medical School, 2018. All Rights Reserved.

IIM (Photomicrograph)

Inclusion Body Myositis (Photomicrograph)

3297
Copyright © Harvard Medical School, 2018. All Rights Reserved.

DM and PM: Clinical Features


• Proximal muscle weakness
• > 90% PM patients
• 50-60% DM patients at presentation; skin
features may precede weakness
• Amyopathic DM
• Skin findings
• Classic DM
• Not found in PM

DM: Gottron’s Papules

3298
Copyright © Harvard Medical School, 2018. All Rights Reserved.

DM: Heliotrope Rash

3299
Copyright © Harvard Medical School, 2018. All Rights Reserved.

DM: Poikiloderma (Shawl and V Signs)

DM: Nailbed

3300
Copyright © Harvard Medical School, 2018. All Rights Reserved.

DM and PM: Clinical Features


• ILD (10%)
• Cardiac disease
• Myocarditis—frequently subclinical
• 3-4x increased risk MI
• Esophageal disease
• Weakness of striated muscle of upper 1/3rd of
esophagus aspiration

ILD and Pneumomediastinum

3301
Copyright © Harvard Medical School, 2018. All Rights Reserved.

DM and PM: Autoantibodies


~ 80% ANA +
Myositis-specific Clinical syndrome Prevalence
autoantibodies
Antisynthetase Antisynthetase syndrome 20%
antibodies,
including anti-Jo-1
Anti-signal Severe myopathy, aggressive disease that 5%
recognition particle may be difficult to control
(SRP)
Anti-Mi-2 Acute onset DM, classic skin findings, good 7-30%
prognosis

Anti-MDA-5 Rapidly progressive ILD, cutaneous


ulceration involving Gottron’s papules,
arthritis, alopecia, oral ulcers, amyopathic

Antisynthetase Syndrome
• Fever
• Raynaud’s
• Inflammatory arthritis
• Mechanics hands
• ILD (can be severe)

3302
Copyright © Harvard Medical School, 2018. All Rights Reserved.

Treatment Regimens in IIM


• Corticosteroids (often with DMARD)
• DMARDs
• Methotrexate
• Calcineurin inhibition (Tacrolimus)
• Azathioprine
• Mycophenolate
• Cyclophosphamide (mostly in ILD)
• Rituximab (recent trial efficacy unclear but may
be useful in antisynthetase syndrome)
• IVIG (refractory cases)
• Abatacept (T cell co-stimulatory inhibitor)

Summary
• Scleroderma
• Recognize clinical patterns
• Major morbidity/mortality = lung disease
• There are newer treatment options!
• Sjögrens syndrome
• Recognize clinical manifestations
• Sometimes marked by systemic disease
• Higher risk for lymphoma
• Inflammatory myopathy
• Recognize clinical patterns
• Newer antibodies may help with diagnosis
• Major morbidity = lung disease

3303
Copyright © Harvard Medical School, 2018. All Rights Reserved.

Board Question #1
• 53 yo female with 20 year hx of Raynaud’s
develops fatigue and dyspnea over the
preceding 6 months
• On Nifedipine
• BP 115/80
• Exam notable for prominent P2
• PFTs show DLCO 48% predicted (low)
• 02 sat is 96% rest, 93% with activity (abnormal)
• Echo shows mild TR, PASP 48 mmHg

Board Question #1

What is the appropriate next test for


this patient?

• A. CT angiogram of the chest


• B. HRCT of the chest
• C. Pulmonary artery catheterization
• D. Exercise stress test

3304
Copyright © Harvard Medical School, 2018. All Rights Reserved.

Board Question #1

• Correct answer: C
• The longstanding history of Raynaud’s
raises the question of a CTD. Given the
decline in DLCO and echo findings, PAH
remains the greatest concern.

Board Question #2

• 51 yo man with diffuse cutaneous SSc is


admitted with new onset hypertension
associated with anemia and
thrombocytopenia
• On admission: BP 180/105; skin
thickening over face, chest, hands, legs;
lungs clear; heart RRR normal S1 S2; 1+
edema in legs

3305
Copyright © Harvard Medical School, 2018. All Rights Reserved.

Board Question #2

• Hgb 9.8 Plt 101K Cr 1.4


• UA 2+ protein, no casts
• Smear: 2+ schistocytes
• Started on captopril 6.25mg every 8hrs
• Captopril escalated to 25 mg every 8hrs
• 3 days later, BP 140/95, Cr now 2.1
• UA 2+ protein

Board Question #2

Which of the following is the most


appropriate next step?

• A. Discontinue captopril, begin nifedipine


• B. Continue to increase the captopril
• C. Start plasmapheresis
• D. Angiography to assess for RAS
• E. Order the RNA polymerase III ab

3306
Copyright © Harvard Medical School, 2018. All Rights Reserved.

Board Question #2

• Correct answer: B
• This patient has systemic sclerosis with
diffuse skin disease and is at significant
risk for renal crisis which is the case here.
Despite the continued increase in
creatinine, the ACE inhibitor should be
continued.
• The RNA poly III ab confers increased risk
for renal crisis

References
• Bournia VK, Vlachoyiannopoulus PG, Selmi C, Moutsoupoulus HM Gerswin
ME. Recent advances in the treatment of systemic sclerosis. Clin Rev
Allergy Immunol 2009:36(2-3):176-200
• Tashkin DP, Elashoff R, Clements PJ et al. for the Scleroderma Lung Study.
Cyclophosphamide versus placebo in scleroderma lung disease. N Eng J
Med 2006;354:2655-2666
• Tashkin DP, Roth M, Clements PJ et al. Mycophenolate in Scleroderma:SLS
II. Lancet 2016;4(9):708-719
• Coghlan JG, Pope J, Denton C. Assessment of endpoints in pulmonary
arterial hypertension associated with connective tissue disease. Curr Opin
Pulm Med 2010:16(suppl) S27-34.
• Meijer JM et al. Effectiveness of rituximab treatment in primary Sjogrens
syndrome: a randomized, double blind placebo controlled trial. Arthritis
Rheum 2010:62(4):960
• Dalakas M. Inflammatory Muscle disease. N Eng J Med. 2015:372:1734-47
• King TE, Jr., Bradford WZ, Castro-Bernardini S, et al. A phase 3 trial of
pirfenidone in patients with idiopathic pulmonary fibrosis. N Engl J Med
2014;370:2083-92.
• Richeldi L, du Bois RM, Raghu G, et al. Efficacy and safety of nintedanib in
idiopathic pulmonary fibrosis. N Engl J Med 2014;370:2071-82.

3307
Copyright © Harvard Medical School, 2018. All Rights Reserved.

Vasculitis/GCA/PMR
• Internal Medicine Board Review
• Paul F Dellaripa MD, Rheumatology,
Brigham and Women’s Hospital, Boston
MA
• Associate Professor of Medicine, Harvard
Medical School

Disclosures
• Up to Date
• Genentech
• Bristol Myers Squibb

3308
Copyright © Harvard Medical School, 2018. All Rights Reserved.

Vasculitis: Summary
• A heterogeneous group of disorders
characterized by vascular inflammation
leading to vessel occlusion and tissue
ischemia and necrosis.
• Difficult but improving treatment options
• Pattern recognition is key to early
diagnosis and early therapeutic
intervention.

Vasculitis:Outline
• Polyarteritis nodosa: abdominal pain, skin ulcers,
neuropathy
• Microscopic polyangiitis:pulm/renal syndrome, MPO
ANCA
• Granulomatosis with polyangiitis (Wegeners
granulomatosis): upper and lower respiratory tract,
pulm/renal syndrome, PR3 ANCA predominate
• Eosinophilic Granulomatosis with Polyangiitis
(Churg Strauss):asthma, eosinophilia, pulmonary
infiltrates
• Cryoglobulinemic vasculitis:cutaneous vasculitis
• Behcets:oral and/or genital ulcers, rash, uveitis
• Takayasus arteritis: pulseless syndrome affected
females.
• Giant Cell Arteritis;headache, jaw claudication, visual
loss, PMR

3309
Copyright © Harvard Medical School, 2018. All Rights Reserved.

Vasculitis:Classification
•Small vessels (venules, arterioles) • Small and medium muscular
arteries
–Drug-induced and serum
sickness – Classic PAN
–Ig A vascullitis ( Henoch- – Microscopic polyangiitis
Schönlein purpura) – GPA (Wegener’s
–Cryoglobulinemia granulomatosis)
–Vasculitis associated with – EGPA( Churg-Strauss
systemic rheumatic diseases vasculitis)
–Vasculitis associated with – Kawasaki syndrome
malignancy – Rheumatoid vasculitis
–Hypocomplementemic – SLE
urticarial vasculitis
–Vasculitis associated with • Large arteries
infections – Giant cell or temporal
arteritis
– Takayasu arteritis

Vessel Size and Clinical Disease


Correlates
• Large vessel (GCA, Takayasu)
• Medium vessel (PAN, Kawasaki) PAN is
REALLY RARE
• Small vessel (ANCA disease (GPA (WG) or
MPA, IgA/HSP, Goodpastures,
SLE/RA/CTD,cryoglobulinemia)
• Small and Medium (Buergers, Cogan’s,
Primary CNS vasculitis, sometimes ANCA
vasculitis )
• Any or all types of blood vessels (Behcet’s,
Relapsing Polychondritis, Cogan’s)

3310
Copyright © Harvard Medical School, 2018. All Rights Reserved.

When to suspect vasculitis: clinical


features
• Multisystem disease

• Unexplained constitutional signs and symptoms

• Skin lesions (palpable purpura)

• Ischemic vascular changes (gangrene, claudication, Raynaud’s


• phenomenon, livedo)

• Glomerulonephritis

• Mononeuritis multiplex

• Myalgia, arthralgia/arthritis

• Abdominal (intestinal angina) or testicular pain

Hard to classify vasculitis


(Lamprecht, Clin Exp Rheum 2011)
• Goodpastures syndrome (DAH, glomerulopnephritis)
• Behcet’s disease ( oral/genital ulcers, arthritis,vasculitis)
• IgG4 related systemic disease (autoimmune pancreatitis,
chronic sclerosing sialadenitis, orbital inflammatory pseudotumour,aortitis,and
retroperitoneal fibrosis)

• Cogan's syndrome (vestibulitis, hearing loss, vasculitis)


• Primary CNS vasculitis
• Intestinal vasculitis
• Chronic periaortitis
• Thromboangiitis obliterans (Burger’s disease)

3311
Copyright © Harvard Medical School, 2018. All Rights Reserved.

Conditions that mimic systemic


vasculitis
• Atheroembolic disease

• Cardiac myxoma

• Thrombotic disorders
– Anti-phospholipid antibody syndrome
– Thrombotic thrombocytopenic purpura
• Drug-induced vascular damage
– Ergot derivatives
– Cocaine
– Amphetamines
• Infective endocarditis

What is this ?
• Raynauds/acral
necrosis
• Antiphospholipid Ab
• Endocarditis
• Small vessel
vasculitis (including
RA)
• Medium vessel
vasculitis

3312
Copyright © Harvard Medical School, 2018. All Rights Reserved.

Pathogenesis
• Immune complex deposition ( SLE, cryos,
HSP)
• Ab vs vascular structures ( antiGBM)
• Ab against no vascular structures (ANCA)
• Cell mediated tissue injury, Th1, IL-6,
TH17 (GCA, TA)

ACR 1990 criteria for classification of


polyarteritis nodosa (1-4)
• Must have at least 3 of the 10 criteria present.
– Weight loss > 4 kg
– Livedo reticularis
– Testicular pain or tenderness
– Myalgias, weakness, or leg tenderness
– Mononeuropathy or polyneuropathy
– Diastolic BP > 90
– Elevated BUN/creatinine
– Hepatitis B virus
– Arteriographic abnormality
– Biopsy of small or medium artery containing PMN
• Sensitivity 82.2% and Specificity 86.6%

3313
Copyright © Harvard Medical School, 2018. All Rights Reserved.

Real world considerations for


PAN
• Middle aged male
• Fever, weight loss, and abdominal pain
• Elevated inflammatory markers
• Hypertension
• Neuropathy
• No glomerulonephritis
• Can be associated with Hepatitis B

3314
Copyright © Harvard Medical School, 2018. All Rights Reserved.

Polyarteritis nodosa: wrist drop

PAN: sural nerve

3315
Copyright © Harvard Medical School, 2018. All Rights Reserved.

Case
• 26 yo presented in the spring 2010 with
right flank pain, CT scan showed a right
renal infarct. Pt treated with
anticoagulation after an unremarkable
evaluation for hypercoagulability and then
developed a hematoma. He had mild
fatigue but otherwise was well.
• An MRI/A was performed

3316
Copyright © Harvard Medical School, 2018. All Rights Reserved.

3317
Copyright © Harvard Medical School, 2018. All Rights Reserved.

ANCA associated vasculitis


• Include Granulomatosis with Polyangittis (Wegener's
Granulomatosis ), microscopic polyangitiis and
EGPA (Churg Strauss) and drug induced vasculitic
syndromes (PTU, allopurinol, levamisole tainted
cocaine)
• Morbidity associated with these diseases typically
related to renal failure due to do glomerulonephritis
or pulmonary hemorrhage/respiratory failure
• Mortality often related to disease progression but
more often treatment associated infection. In
fact,vasculitis that appears to be worsening
should be considered an infection until proven
otherwise.

ANCA Associated Vasculitis


• C-ANCA with PR3 reactivity most commonly found in
GPA (Wegener’s ), though p-ANCA-MPO has been
noted in 10% of cases of WG
• P-ANCA-MPO most often seen in microscopic
polyangitiis. Occasionally ANCA and anti-GBM can
be concurrent
• Very high titer ANCA (esp MPO ) raises possibility of
drug induced vasculitis (including cocaine)
• In initial treatment, the ANCA type is irrelevant,
though mortality is higher with PR3 ANCA
• ANCA is useful diagnostically but does not
necessarily predict relapse

3318
Copyright © Harvard Medical School, 2018. All Rights Reserved.

Anticytoplasmic autoantibodies

Granulomatosis with Polyangiitis


(formerly Wegener's Granulomatosis)
• Classic patterns include upper respiratory
symptoms, sinusitis, epistaxis, hearing loss, otitis
media (remember, adults otherwise rarely get otitis
media)
• Lower respiratory symptoms ( bronchitis,
hemoptysis due to capillaritis)
• Glomerulonephritis
• Mononeuritis multiplex, cranial neuropathy, orbital
pseudotumor
• Constitutional symptoms (fever, weight loss,fatique)

3319
Copyright © Harvard Medical School, 2018. All Rights Reserved.

Immunohistopathology: ANCA
reuslts in a pauci-immune
patholgy
• GPA(Wegeners): scant or no immune
deposits
• SLE: clumpy immune complex
deposition
• Goodpastures:linear IgG deposition
along the glomerular basement
membrane

3320
Copyright © Harvard Medical School, 2018. All Rights Reserved.

3321
Copyright © Harvard Medical School, 2018. All Rights Reserved.

3322
Copyright © Harvard Medical School, 2018. All Rights Reserved.

3323
Copyright © Harvard Medical School, 2018. All Rights Reserved.

Microscopic polyangiitis
• Present with GN as major clinical
finding
• Mononeuritis multiplex
• Alveolar hemorrhage
• Sometimes difficult to distinguish from
GPA(WG)
• ANCA in pANCA pattern (MPO ab)

Eosinophilic Granulomatosis with


Polyangiitis ( Churg Strauss
Syndrome)
• Asthma
• Eosinophilia
• Non fixed pulmonary infiltrates
• Paranasal sinus abnormality
• Extravascular eosinophils (biopsy)
• Mononeuropathy or polyneuropathy

3324
Copyright © Harvard Medical School, 2018. All Rights Reserved.

Clinical features of CSS


• Phase I: adult onset asthma, most require
steroids for control
• Phase II: eosinophilic infiltrates in the lung
and GI tract. Lung infiltrates are typically
peripheral, patchy and asymmetrical.
• Phase III:constitutional symptoms and
systemic disease, peripheral neuropathy, CNS
vasculitis, mesenteric ischemia, cutaneous
vasculitis
• cardiac involvement with myocarditis is
the leading cause of death.

3325
Copyright © Harvard Medical School, 2018. All Rights Reserved.

3326
Copyright © Harvard Medical School, 2018. All Rights Reserved.

What could this be?


• PAN ( necrotic lesions, often nodular)
• Cryoglobulinemia
• Cocaine associated ANCA disease (
typically skin disease)

ACR classification criteria:


Takayasu arteritis (13,14)
• Must have at least 3 of the 6 criteria present.
– Age < 40 years at disease onset
– Claudication of extremities
– Decreased brachial artery pulse
– BP difference > 10 mm Hg between arms
– Bruit over subclavian arteries or aorta
– Arteriogram abnormality: occlusion or narrowing in
aorta or main branches
– Think young female, pulseless, constitutional
symptoms!

3327
Copyright © Harvard Medical School, 2018. All Rights Reserved.

Takayasu’s arteritis

Criteria for diagnosis of Behçet’s


disease (15,16)
• Recurrent oral ulceration plus two of the
following:
– Recurrent genital ulceration
– Eye lesions (anterior/posterior uveitis or cells
in vitreous or retinal vasculitis)
– Skin lesions (E. Nodosum, pseudofolliculitis,
papulopustular lesions or acneiform nodules)
– Positive pathergy test
• Sensitivity 91% and specificity 96%

3328
Copyright © Harvard Medical School, 2018. All Rights Reserved.

Behçet’s syndrome: ulceration,


tongue

Folliculitis in Behcet’s

3329
Copyright © Harvard Medical School, 2018. All Rights Reserved.

Ulcerations on scrotum in
Behcet’s

Cryoglobulinemia
• 3 types ( I, II, III) with Type II ( hepatitis C)
and III (CTDs)
• Immune complex mediated vasculitis with
complement consumption (low C4)
• Mixed cryo associated with both IgG and IgM
• IgM is often monoclonal and specific for Fc of
the IgG ( presence of rheumatoid factor)
• Nerve, skin and rarely renal and lung
involved.
• Need to treat the Hep C infection long term
to control the vasculitis

3330
Copyright © Harvard Medical School, 2018. All Rights Reserved.

Cryoglobulinemia: ear
necrosis

Cryoglobulinemia

3331
Copyright © Harvard Medical School, 2018. All Rights Reserved.

Treatment regimens:Vasculitis
• May need to start Treatment prior to having a clear
diagnosis. Always rule out infection!
• Rituximab effective in ANCA associated disease; non
inferior compared to CYC in both new onset and relapsing
disease (RAVE trial NEJM 2010) with steroids
• Oral cyclophosphamide 2mg/kg/day adjusted for renal
function in severe cases or intravenous CYC .5 mg/m2 q3-4
weeks , which may be safer
• Plasmapheresis: may be useful in ANCA associated
disease/cryoglobulinemia where standard regimen is not
effective or renal disease is rapidly progressive (renal
failure Cr >5.7 perhaps in DAH)
• Corticosteroids and now IL-6 inhibition in Takayasu
• In Behcets TNFi like infliximab and adulimumab.
• Other therapies include mycophenolate, azathioprine,
methotrexate,abatacept
• Antiviral therapy in Hep C associated cryoglobulinemia

Board Question
• 74 yo male is hospitalized with diffuse
alveolar hemorrhage over a period of a
few days. He had a prodrome of malaise
and arthralgia for several weeks.
• In the ICU his serum creatinine is 7.4
mg/dl, urinalysis shows 3+protein, many
RBCs, scattered red cell casts.
• He is intubated, with copious bloody
secretions evident from the ETT.

3332
Copyright © Harvard Medical School, 2018. All Rights Reserved.

Which of the following antibodies is the


patient most likely to have

• A. AntiSm antibodies
• B. AntiGBM antibodies
• C. p-ANCA MPO (myeloperoxidase)
• D. c-ANCA PR3
• E all of the above

Correct answer
• E is correct
• This case is an example of the
pulmonary renal syndrome. This clinical
pattern can be seen in SLE, ANCA
associated disease, and Goodpastures
and so all of the antibody testing are
reasonable.

3333
Copyright © Harvard Medical School, 2018. All Rights Reserved.

Board Review
• 45yo male with long standing asthma
• New onset fever, fatigue , skin rash and
worsening dyspnea.
• Using albuterol inhaler more frequently and
requiring more oral steroids and was recently
started on a leukotriene antagonist.
• Complains of diffuse abdominal pain
• CXR shows bilateral patchy infiltrates, and
WBC count is 15,000/ul with 25% eos.

Which of the following is the


correct next step
• A. Increase inhaled steroids
• B. Begin plasmapheresis
• C. Increase the prednisone from 10 mg
to 20 mg orally per day
• D Begin intravenous corticosteroids.
• E. perform an open lung biopsy

3334
Copyright © Harvard Medical School, 2018. All Rights Reserved.

• Correct answer is D
• This patient has Eosinophilic
Granulomatosis with Polyangiitis ( CSS) .
• He has severe systemic manifestations
with mononeuritis and should be treated
with intravenous steroids in high doses.
• The decision to use steroid sparing agents
or cytotoxic therapy depends on severity
of disease

Giant Cell Arteritis


• Systemic vasculitis of large and medium vessels
of unknown etiology characterized by
transmural inflammation, granuloma formation,
and luminal occlusion of the cranial blood
vessels (16).
• Annual incidence is North America is 19-32 per
100,000 in persons > 50 yrs
• Cell mediated immune processes are important
in this disorder. IL-6, IL17, interferon gamma,
IL1B and TH-17 play important but varying roles
in pathogenesis and clinical manifestations.

3335
Copyright © Harvard Medical School, 2018. All Rights Reserved.

GCA: Clinical manifestations


• Headache (scalp tenderness, including
posterior)
• Jaw claudication due to facial artery
involvement ( sometimes rarely presenting as
trismus)
• Visual loss due to retinal and posterior ciliary
artery vasculitis. ( blurriness, diploplia)
• Permanent visual loss noted in up to 15% of
cases
• Though rare, there is a long term risk of thoracic
aneurysm

Giant Cell Arteritis:other


manifestations
• Weight loss
• Fever ( including FUO)
• Polymyalgia rheumatica
• Ischemia of the head, neck or extremities
due to occlusive disease of the carotid,
subclavian and vertebral arteries
• Carotidynia
• Cough

3336
Copyright © Harvard Medical School, 2018. All Rights Reserved.

Giant Cell arteritis: Diagnosis


three of the following five criteria was associated with a 94% sensitivity and 91%
specificity)

• Age greater than or equal to 50 years at time of


disease onset
• Localized headache of new onset
• Tenderness or decreased pulse of the temporal
artery
• Erythrocyte sedimentation rate greater than 50
mm/h (Westergren)
• Biopsy which includes an artery, and reveals a
necrotizing arteritis with a predominance of
mononuclear cells or a granulomatous process
with multinucleated giant cells

Headache in GCA
• Scalp pain
• Location can be temporal, posterior,
occipital
• “My hair hurts to brush”
• Tongue pain
• Ear, nose pain, jaw pain, trismus
• “throat pain”

3337
Copyright © Harvard Medical School, 2018. All Rights Reserved.

Retinal ischemia due to GCA

3338
Copyright © Harvard Medical School, 2018. All Rights Reserved.

Aortic dissection: a rare


complication of GCA

Diagnosis: GCA
• Laboratory markers: ESR, CRP, alkaline
phosphatase, IL-6 though inflammatory
markers may be normal or low in a small
number of cases
• Whenever there is a reasonable suspicion of
GCA, a temporal artery biopsy should be
performed. The morbidity of a biopsy is low
• It is preferable to obtain a biopsy prior to starting
therapy but pathologic findings can persist for up
to several weeks after starting steroids, so
therapy should not be delayed if a biopsy cannot
be obtained promptly
• Ultrasound emerging as a diagnostic test
though false negative and positives have
been noted

3339
Copyright © Harvard Medical School, 2018. All Rights Reserved.

3340
Copyright © Harvard Medical School, 2018. All Rights Reserved.

Treatment of GCA
• Prednisone 40-80 mg per day for 4 weeks
• Reduce to 20 mg per day by third month
• Reduce by 5 mg every 2-4 weeks and then at 10 mg
reduce by 1 mg every 4-6 weeks, total therapy up to 2
years
• Benefit of intravenous corticosteroids with visual
symptoms is unclear but reasonable to consider
• IL-6 inhibition: FDA approved for GCA with shortened
course of steroids ( Stone et al NEJM 2017)
• Methotrexate as steroid sparing not effective
• No evidence that TNF blockade is effective
• Low dose ASA may reduce ischemic events in GCA
• Th1 and Th-17 blockade:(Ustekinumab) ?role in
refractory GCA (Conway et al Ann Rheum Dis 2016)

Board question
• 82 year old healthy female ( no meds) is
evaluated for a 2 week history of headache and
neck pain. She also complains of achiness of the
shoulders, neck , and lower back. She had two
episodes of blurriness in her right eye transiently
last week but none now.
• On exam the scalp is diffusely tender,
carotidynia noted , ROM limited due to pain in
shoulders. Vision normal.Reflexes normal in
UEs.
• ESR 24 mm/h, CRP 1.0 Hgb 11.7 CK 150

3341
Copyright © Harvard Medical School, 2018. All Rights Reserved.

Which of the following is the most


appropriate next step at this time?
• A MRI of head
• B EMG
• C Prednisone 1 mg/kg per day
• D Prednisone 15 mg per day
• E Doppler US of the carotid artery
• F Temporal artery biopsy

• Correct answer C
• This patient has a significant risk for GCA
and should be treated immediately
regardless of access to biopsy
• The low ESR and CRP can be seen in
GCA and should not dissuade treatment
and further diagnostic testing

3342
Copyright © Harvard Medical School, 2018. All Rights Reserved.

Pearls : GCA
• GCA is a strong consideration in the elderly with new
onset headache, neck ache , visual changes or
unexplained fatigue or anemia
• Jaw claudication is the most specific sign of GCA,
followed by visual loss and TA tenderness
• A more robust inflammatory response is correlated with a
lower risk for visual loss.
• Patients with a low or normal ESR and CRP can have
GCA If one biopsy is negative, the additional yield of a
contralateral biopsy is modest but is reasonable to
consider performing.
• A rising ESR in a treated for GCA does not necessarily
suggest that GCA is returning
• IL-6 inhibition is the first FDA approved treatment in GCA

Polymyalgia Rheumatica
• Age>50
• 1 month duration of morning stiffness in 3
or more areas ( shoulders, hips, thighs
and neck)
• ESR >40 mm/hr
• Exclusion of other diseases

3343
Copyright © Harvard Medical School, 2018. All Rights Reserved.

3344
Copyright © Harvard Medical School, 2018. All Rights Reserved.

PMR: Differential diagnosis


• Rheumatoid arthritis
• Polymyositis
• Rotator cuff tendonitis
• Parkinson's disease
• Giant cell arteritis
• Fibromyalgia
• Malignancy

PMR treatment
• Corticosteroids should result in substantial and often
gratifying improvement in symptoms in low dose ( <20
mg/day prednisone). If response is underwhelming
reconsider diagnosis
• With resolution of symptoms and normalization of ESR (
typically 1-2 months) , taper steroids by 2.5 mg every 2-4
weeks until 10mg/day and then taper by 1 mg per month.
Typical duration of treatment is up to 2 years or longer.
Relapse is not uncommon.
• Unknown benefits though anectodes for DMARDsand
biologics (hydroxychloroquine, MTX or Tocilizumab)

3345
Copyright © Harvard Medical School, 2018. All Rights Reserved.

PMR treatment
• In patients with high clinical suspicion of
disease where prednisone is not effective,
consider the use of methylprednisolone, as
some patients do not metabolize prednisone.
• Other agents, including MTX,
hydroxychloroquine, other DMRDS and now IL-6
inhibition may be helpful but limited trials

PMR pearls
• Synovitis of the hands can occur in PMR
• A normal ESR does not exclude the diagnosis
• 15-20% of patients with PMR will develop GCA
• Consider a temporal artery biopsy if persistent
constitutional symptoms or inflammatory
markers remain high.

3346
Copyright © Harvard Medical School, 2018. All Rights Reserved.

Vasculitis: Take home Points


• A heterogeneous group of disorders
characterized by vascular inflammation
leading to vessel occlusion and tissue
ischemia and necrosis.
• Emerging, more targeted treatment
options on the horizon
• Pattern recognition is key to early
diagnosis, early therapeutic intervention
and limiting end organ damage.

References
• Stone JH et al. Rituximab versus cyclophosphamide for ANCA-associated
vasculitis. N Engl J Med. 2010;363(3):221.

• Yates M, Watts RA Bajema IM et al. EULAR/ERA-EDTA recommendations for


the management of ANCA-associated vasculitis. Ann Rheum Dis 2016;75:1583-
1594

• Guivellin L et al Rituximab versus azathioprine for maintenance in ANCA-


associated vasculitis. N Eng J Med. 2014;371(19):1771-80

• Stone JH et al. Trial of tocilizumab in giant cell arteritis. N Eng J Med


2017;377:317-328

• Alibaz-Oner F, Aydin SZ, Direskeneli H. Recent advances in Takayasu arteritis


• Eur J Rheumatol.2015;2(1):24-30

• Hatemi G et al. Behçet's syndrome: a critical digest of the 2014-2015 literature.


Clin Exp Rheumatol. 2015 33(6 Suppl 94):S3-14

3347
Copyright © Harvard Medical School, 2018. All Rights Reserved.

Soft Tissue Rheumatism

Elinor Mody, MD
Chief, Division of Rheumatology
Reliant Medical Group

Some problems are difficult, but diagnosing


and treating most causes of joint pain are not!

3348
Copyright © Harvard Medical School, 2018. All Rights Reserved.

Common areas of pain

• Shoulder
• Elbow
• Wrist
• Hip
• Knee
• Ankle

Some definitions

• Tendon: The collagenous portion of a muscle at the


origin and insertion.
• Enthesis: The attachment of a tendon to bone.
• Bursa: Two membranes and the lubricated space
between, that allows near-frictionless motion of soft
tissue over bone.
• Ligament: Collagenous tissue connecting two bony
structures.
• Fascia: A layer or sheet of fibrous tissue that
separates or covers various soft tissues or organs.

3349
Copyright © Harvard Medical School, 2018. All Rights Reserved.

Tendinitis

• History
Pain with active motion
Weakness
May elicit an overuse history

• Exam
Little discomfort with passive ROM
Isometric contraction is always tender.
Palpable tendons usually tender

Bursitis

• History
Pain with active motion
Weakness not prominent
May elicit an overuse history

• Exam
Passive ROM more uncomfortable than tendinitis
Isometric contraction is tender
Palpable bursae tender

3350
Copyright © Harvard Medical School, 2018. All Rights Reserved.

Arthritis

• History
Pain with active motion
Weakness only related to pain
AM stiffness
• Exam
Passive ROM as uncomfortable as AROM
Isometrics not tender (the joint isn’t moving)
Crepitus in OA
Joint aspirate often inflammatory

Shoulder: Common causes of shoulder pain

• Tendinitis: Rotator cuff (+/- Ca2+), biceps


• Bursitis: Subacromial and subdeltoid bursae
• Adhesive capsulitis
• Arthritis: Glenohumeral, AC
• Neuropathy: Cervical, notalgia
• Dislocation/trauma: GH, AC separation, Fx
• Other: Cellulitis, pneumonia, cardiac

3351
Copyright © Harvard Medical School, 2018. All Rights Reserved.

Shoulder: Normal anatomy

Shoulder: Normal anatomy

3352
Copyright © Harvard Medical School, 2018. All Rights Reserved.

Shoulder: Normal anatomy

“Empty Can Test

» Tests Supraspinatus
» Arm abducted 90 degrees in pronation
» Downward pressure applied

3353
Copyright © Harvard Medical School, 2018. All Rights Reserved.

Napoleon Test

» Tests subscapularis
» Press belly with elbow at 90 degrees
» Pain or inability to perform suggests
subscapularis tendonitis/derangement

External Rotation Lag

» Tests Infraspinatus
» Hold arm in external rotation with elbow at
90 degrees
» Inability to maintain suggests infraspinatus
tendonitis/derangement

3354
Copyright © Harvard Medical School, 2018. All Rights Reserved.

Hawkins- Kennedy

» Tests for subacromial impingement


» Hold arm at 90 degrees flexion, with elbow
at 90 degrees.
» Forcibly internally rotate shoulder
» Pain with this maneuver suggests
impingement

Shoulder: Subacromial approach

3355
Copyright © Harvard Medical School, 2018. All Rights Reserved.

Shoulder: Posterior approach

Shoulder: Normal arthrogram

3356
Copyright © Harvard Medical School, 2018. All Rights Reserved.

Shoulder: Chondrocalcinosis and AC


osteophytes

Other therapies

• Physical therapy
• Systemic Steroids or NSAIDs
• Disease-specific therapy, e.g. colchicine for
crystal disease
• MR imaging indicated primarily to guide
surgery. Not needed for diagnosis or
treatment.

3357
Copyright © Harvard Medical School, 2018. All Rights Reserved.

Elbow

• Lateral epicondylitis (actually an enthesitis)


• Olecranon bursitis
• Arthritis is uncommon

Elbow: Lateral epicondylitis

3358
Copyright © Harvard Medical School, 2018. All Rights Reserved.

Elbow: Tennis elbow strap

Elbow: Olecranon bursitis

3359
Copyright © Harvard Medical School, 2018. All Rights Reserved.

Elbow: Olecranon bursitis

Other therapies

• Physical therapy
• Steroids or NSAIDs
• Disease-specific therapy, e.g. colchicine for
crystal disease, antibiotics for infection, etc.
• Steroid injection of olecranon bursa should
be avoided, due to risk of infection

3360
Copyright © Harvard Medical School, 2018. All Rights Reserved.

Wrist

• Fracture: Colle’s
• Arthritis: Crystal
• Median neuropathy: carpal tunnel syndrome
• deQuervain’s tenosynovitis

Carpal tunnel syndrome

• Often overweight, middle-aged patient


• May have history of overuse/misuse
• Night pain, numbness in median distribution
• Tinel’s & Phalen’s signs
• EMG/NCS only necessary prior to surgery
• Therapeutic options include splint, rest, steroid
injection, surgery

3361
Copyright © Harvard Medical School, 2018. All Rights Reserved.

Wrist: rest

Wrist: splint

3362
Copyright © Harvard Medical School, 2018. All Rights Reserved.

Wrist: Aspiration/Injection

Hip

• Bursitis
• Tendinitis
• Groin “pull”
• Arthritis
• Referred from spine

3363
Copyright © Harvard Medical School, 2018. All Rights Reserved.

Hip: Bursitis

Hip: Trochanteric bursitis

3364
Copyright © Harvard Medical School, 2018. All Rights Reserved.

Hip: Trochanteric bursa injection

Knee

• Trauma: meniscal or ligamentous tear, Fx


• Arthritis: Inflammatory and OA
• Bursitis: Pre-patellar, pes anserine, etc.

The knee is the first joint we’ve discussed


today where routine plain films are often
helpful!

3365
Copyright © Harvard Medical School, 2018. All Rights Reserved.

Knee: Normal Anatomy

Knee: Normal Anatomy

3366
Copyright © Harvard Medical School, 2018. All Rights Reserved.

Knee: Chondrocalcinosis

Knee: Osteoarthritis

3367
Copyright © Harvard Medical School, 2018. All Rights Reserved.

Ankle
• “Sprain” = traumatic ligamentous tear
• Fracture
• Arthritis
– OA is uncommon in the ankle
– Crystal arthritis is common
• Tendinitis
– Peroneus longus et brevus
– Tibialis posterior/FHL
– Tibialis anterior
• Neuropathy, e.g. Tarsal tunnel syndrome

Ankle: Normal

3368
Copyright © Harvard Medical School, 2018. All Rights Reserved.

Ankle & Foot: Normal

Ankle: Aspiration/Injection

3369
Copyright © Harvard Medical School, 2018. All Rights Reserved.

Summary

• Joint, tendon, and bursal pain are among the most


common reasons patients seek medical attention.
• A diagnosis is frequently evident after a proper
history and exam. Usually, a film or other test is
NOT required for accurate diagnosis and treatment.
• Embrace the principle of local therapy for a local
problem.

3370
Copyright © Harvard Medical School, 2018. All Rights Reserved.

SLE and the Antiphospholipid Syndrome

Susan Y. Ritter MD, PhD


Associate Physician
Division of Rheumatology, Immunology and Allergy
Department of Medicine
Brigham and Women’s Hospital
Instructor in Medicine
Harvard Medical School

Disclosures

• Spouse is employee of Pfizer

3371
Copyright © Harvard Medical School, 2018. All Rights Reserved.

Overview
• SLE – different presentations and severity
• Classification criteria
• Cutaneous and drug induced
• Treatment
• Antiphospholipid Syndrome

SLE Epidemiology
• Prevalence ~1 in 1000
• Most common in females in reproductive
years
• 9:1 female : male ratio
• More common in Black, Latino and Asian

3372
Copyright © Harvard Medical School, 2018. All Rights Reserved.

What is Lupus?
• Great imitator
• Many different organ-systems may be involved
• Disease occurs in the presence of
autoantibodies—most commonly ANA

Many faces of SLE


• 51 yo white male—glomerulonephritis (GN), + ANA, +
dsDNA, seizures

• 32 yo black female—arthritis, discoid rash, pleurisy, +


ANA, + Ro

• 52 yo female— thrombocytopenia, hemolytic anemia,


renal disease, +ANA, + APLA

• 27 yo female –fevers, class V GN, low WBC, + ANA, +


dsDNA, low complements, arthritis

3373
Copyright © Harvard Medical School, 2018. All Rights Reserved.

What is shared?
• Positive autoantibodies
• Clinical manifestations suggestive of SLE

Classification Criteria for SLE


4 are necessary, and 1 should be + ANA
• Malar Rash • Neurologic: seizures or psychosis
• Discoid Rash
• Photosensitivity • Hematologic: anemia, leuko or
• Apthous ulcers lymphopenia, thrombocytopenia

• Arthritis • Immunologic: anti-DNA, anti-


Smith, antiphospholipid antibody
• Serositis (pericarditis or pleuritis)
• Positive ANA
• Renal (proteinuria or casts)

3374
Copyright © Harvard Medical School, 2018. All Rights Reserved.

Other common symptoms


• Fatigue
• Headaches
• Malaise
• Cognitive impairment “lupus fog”
• Myalgias

Case 1
• 33 yo woman comes to see you.
• She has mouth sores and a rash over the
bridge of her nose that looks like this:

3375
Copyright © Harvard Medical School, 2018. All Rights Reserved.

3376
Copyright © Harvard Medical School, 2018. All Rights Reserved.

Case 1: continued
• Her PCP sent off an ANA that was positive at a
titer of 1:320 and she had dsDNA at 45 units.
• Does she have SLE?

Case 1:
• Malar rash
• Photosensitivity
• Aphthous ulcers
• Positive ANA
• Positive anti-dsDNA

3377
Copyright © Harvard Medical School, 2018. All Rights Reserved.

Other skin manifestations of SLE

Discoid lupus

3378
Copyright © Harvard Medical School, 2018. All Rights Reserved.

Cutaneous Lupus: Discoid Lupus


• Patient have defined plaques that become
thickened over time. Can scar and leave
hypopigmented lesions
• 10-15% of patients develop systemic lupus—in
those that are ANA positive
• In SLE patients, 10-15% will have discoid lesions
• Treatment: sun avoidance, antimalarials,
dapsone, immunosuppressive agents,
thalidomide

Cutaneous Lupus: SCLE (subacute


cutaneous lupus erythematosus)
• More common in Caucasians
• 75% are women
• Lesions occur in sun-exposed areas
• Two types of lesions:
– Papulosquamous
– Polycyclic annular

3379
Copyright © Harvard Medical School, 2018. All Rights Reserved.

SCLE

Other Skin Findings in SLE


• Alopecia
• Vasculitis
• Raynaud’s
• Bullous lesions
• Urticarial lesions
• Panniculitis
• Nail lesions

3380
Copyright © Harvard Medical School, 2018. All Rights Reserved.

Alopecia

Raynaud’s

3381
Copyright © Harvard Medical School, 2018. All Rights Reserved.

Case 2
• 40 yo man whom you have been following for
several years with SLE (malar rash, positive
serologies and renal disease) complains of
chest pain when he takes a deep breath
• Is this related to lupus?

Pulmonary Manifestations of SLE


• Pleuritis
• Lupus pneumonitis
• Chronic interstitial lung disease
• Pulmonary hemorrhage (high mortality)
• Shrinking lung syndrome (secondary to
diaphragmatic paralysis and lung disease)

3382
Copyright © Harvard Medical School, 2018. All Rights Reserved.

Case 3
• 35 yo female whom you have followed for SLE
for many years (joint symptoms, renal disease,
positive serologies, requiring steroids) has
read on the internet that she is at risk for
having a heart attack
• How do you counsel her?

Cardiovascular Manifestations of SLE


• Accelerated atherosclerosis
– Disease + drugs
• Pericarditis
• Valvular heart disease—usually in conjunction
with the antiphospholipid syndrome
• Coronary vasculitis—rare
• Myocarditis—rare

3383
Copyright © Harvard Medical School, 2018. All Rights Reserved.

Case 4
• 32 yo woman referred from her PCP with one
month of joint swelling affecting her PIPs and
MCPs. She has a history of psoriasis and her
PCP believes she has psoriatic arthritis.

Case 4: continued
• Despite being placed on NSAIDs and low dose
steroids, she continues to feel unwell.
• Calls your office with fevers 102, feels terrible
• Initial labs reveal a WBC 2.8.
• You decide to send off an ANA which returns
at 1:1280 and dsDNA is 84.

3384
Copyright © Harvard Medical School, 2018. All Rights Reserved.

Jaccoud’s arthropathy

Musculoskeletal manifestations of SLE


• 95% will have musculoskeletal complains
• Arthralgias and arthritis
• Septic arthritis
• Osteonecrosis—risk increases with steroid use > 20
mg per day
• Myositis—more commonly seen in patients with
Mixed Connective Tissue Disease (MCTD)
• Myopathy—secondary to steroids—proximal muscle
weakness

3385
Copyright © Harvard Medical School, 2018. All Rights Reserved.

Case 5
• 25 yo woman referred to you with anemia and
thrombocytopenia
– Hemocrit 25
– Platelet count 90,000
• She has a history of pleuritis
• Positive ANA and false positive VDRL

Hematologic manifestations of SLE


• Anemia—usually Coomb’s positive
• Leukopenia—WBC < 4000
• Lymphopenia—Lymphocytes < 1500
• Thrombocytopenia—Platelets < 100,000

3386
Copyright © Harvard Medical School, 2018. All Rights Reserved.

Case 6
• 29 yo woman with history of mild SLE
(arthritis, malar rash, positive serologies)
presents with edema, proteinuria, and RBC in
her urine
• What do you do?

Renal disease in SLE


• Proteinuria > 500 mg protein/24 hr urine
• Presence of casts

3387
Copyright © Harvard Medical School, 2018. All Rights Reserved.

WHO Classification
• Class I: Normal or Minimal Change
• Class II: Mesangial glomerulonephritis
• Class III: Focal proliferative glomerulonephritis
• Class IV: Diffuse proliferative
glomerulonephritis
• Class V: Membranous glomerulonephritis
• Class VI: Global sclerosis > 90% sclerosed
lesions

Activity vs. Chronicity


• Look for the degree of activity in the involved
renal specimen
• High disease activity implies better response
to therapy
• Chronic lesions are scarred, and will not
improve with immunosuppression

3388
Copyright © Harvard Medical School, 2018. All Rights Reserved.

Case 7
• 25 yo woman with history of SLE (hematologic
and arthritis) presents with altered mental
status

Neuropsychiatric Manifestations of SLE

Neurologic Psychiatric
• CVA • Psychosis
• Seizures • Cognitive disorder
• Transverse myelitis
• Pseudo dementia
• Optic neuritis
• Meningitis • Functional
• Headaches
• Organic brain syndromes
• Neuropathies
• Associated with
antiphospholipid antibodies

3389
Copyright © Harvard Medical School, 2018. All Rights Reserved.

Other SLE manifestations


• Sjogren’s Syndrome
• GI abnormalities
– Abdominal pain
– Anorexia
– Peritonitis
– Pancreatitis
– Hepatitis
• Secondary fibromyalgia

Case 8
• You are asked to see a 38 yo woman with a
history of a rash over the bridge of her nose
which she has been told is acne rosacea and
some joint achiness
• She has an ANA sent that is positive at 1:40
• Does she have lupus?

3390
Copyright © Harvard Medical School, 2018. All Rights Reserved.

Few notes about ANA test


• ANA test was not designed as a screening test
• Up to 20% of healthy adults, particularly women, have
a low titer ANA and do not go on to develop rheumatic
diseases
• Causes of false positive tests:
– viral infections
– family history of rheumatic disease
– other autoimmune disease like GI or thyroid disease
• Once a patient has a positive ANA, it does not need to
be retested, unless symptoms change and there is an
increased suspicion for a rheumatic disease.

Autoantibodies in SLE
• ANA found in 95% of patients
• Anti-dsDNA in 40-80% of patients
• Anti-Sm 25% of patients
• Anti-histone—seen in drug induced SLE
• Anti-Ro(SSA), Anti-La(SSB)– Sjogren’s, SCLE
• Anti-RNP—Mixed Connective Tissue disease
• False positive VDRL—antiphospholipid
antibody

3391
Copyright © Harvard Medical School, 2018. All Rights Reserved.

Choosing Wisely Campaign


Don’t test ANA sub-serologies without a positive ANA and
clinical suspicion of immune-mediated disease.

• Tests for anti-nuclear antibody (ANA) sub-serologies are


usually negative if the ANA is negative.
• Exceptions include anti-Jo1, which can be positive in
some forms of myositis, or occasionally anti-SSA, in the
setting of lupus or Sjögren’s syndrome.
• Broad testing of autoantibodies should be avoided;
instead the choice of autoantibodies should be guided by
the specific disease under consideration.

www.rheumatology.org/FiveThings

Case 9
• 63 yo man started on procainamide for an abnormal
heart rhythm.
• He develops joint pain and a skin rash
• Work-up reveals a positive ANA and anti-histone
antibody

3392
Copyright © Harvard Medical School, 2018. All Rights Reserved.

Drug Induced SLE


• Patients present with lupus-like illness
• Usually arthritis, rash and serositis
• Positive ANA and anti-histone antibody
• Rare to have renal, neuropsychiatric or
vasculitic disease
• Often responds to drug withdrawal, NSAIDs or
low dose prednisone

Drugs Involved in Drug-Induced SLE

3393
Copyright © Harvard Medical School, 2018. All Rights Reserved.

Drugs Involved in Drug-Induced SLE


Common Rare
• Procainamide • Beta-blockers
• Hydralazine • D-Penicillamine
• INH
• Quinidine
• PTU
• Hydantoins
• Trimethadione
• Chlorpromazine

General Treatment Advice in SLE


• Sun avoidance and protection
• Diet
• Exercise
• Smoking cessation

3394
Copyright © Harvard Medical School, 2018. All Rights Reserved.

Lupus Flare
• A flare is a measurable increase in disease
activity in one or more organ systems
involving new or worse clinical signs and
symptoms and/or laboratory measurements.
It must be considered clinically significant by
the assessor and usually there would be at
least consideration of a change or an increase
in treatment.

International consensus for a definition of disease flare in lupus. Ruperto et al. Lupus. 2011 Apr;20(5):453-62.

Treatment: Mild Disease


• Symptoms: low grade fever, rash, arthralgia, fatigue

• NSAIDs
• Antimalarials—hydroxychloroquine most common
• Low dose prednisone < 10 mg/day

3395
Copyright © Harvard Medical School, 2018. All Rights Reserved.

Treatment: Severe Disease


• Steroids: 0.5-1 mg/kg/day—renal, CNS
• Cyclophosphamide—monthly pulses 500-1000
mg/m2 or biweekly for 12 weeks—renal, CNS
• Azathioprine—renal
• Mycophenolate mofetil—renal, skin
• Tacrolimus –renal, skin
• Methotrexate – arthritis, skin
• Belimumab– steroid resistant serositis and
arthritis

Other therapies (off label)


• Cyclosporine A
• Rituximab
• Abatacept– joint symptoms, in trials for renal
disease
• Bone marrow transplant

3396
Copyright © Harvard Medical School, 2018. All Rights Reserved.

Summary of SLE
• SLE is a multi-system autoimmune disorder
• SLE can look like many different disease entities
• To diagnose SLE, the ANA should be positive
• A positive ANA dose NOT make a diagnosis of SLE
• Direct treatment towards the underlying system
involved
• All patients unless contraindicated should be offered
hydroxychloroquine
• Can use steroids and other immunosuppressives
• Biologics other than belimumab are under
investigation

Antiphospholipid Antibody
Syndrome

3397
Copyright © Harvard Medical School, 2018. All Rights Reserved.

When to think about antiphopholipid


antibodies?
• Early miscarriages
• Thrombocytopenia
• Stroke/TIA in younger patients
• DVT/PE
• Patients with lupus

Antiphospholipid Antibody Syndrome:


Major criteria
• Presence of an anticardiolipin antibody (IgG or
IgM) and/or lupus anticoagulant on 2 separate
occasions 12 weeks apart PLUS one of the
following clinical events:
• Arterial thrombotic event
• Venous thrombotic event
• Recurrent pregnancy losses
– 2 or more 1st trimester losses
– 2nd trimester loss or severe intrauterine growth
restriction

3398
Copyright © Harvard Medical School, 2018. All Rights Reserved.

Other clinical features


(not part of criteria)
• Thrombocytopenia
• Coomb’s positive hemolytic anemia
• Livedo reticularis
• Raynaud’s phenomenon
• Migraines and cognitive dysfunction
• Valvular vegetations or thickening
• Renal disease—thrombotic microangiopathy

Associated medical conditions


• Sneddon’s syndrome: Strokes and livedo
reticularis in young women
• Evan’s syndrome: Thrombocytopenia and
Coomb’s positive hemolytic anemia
• CVAs and MIs in individuals under 40
• SLE
• Catastrophic APS (CAPS)—sudden multisystem
occlusive disease

3399
Copyright © Harvard Medical School, 2018. All Rights Reserved.

Antiphospholipid Antibody Tests


• Lupus Anticoagulant
• Anti-Cardiolipin
• Beta-2 glycoprotein I

Lupus Anticoagulant
• In vitro prolongation of clotting test
– In vivo it is a pro-coagulant
• Activated PTT, platelet neutralizing procedure,
dilute Russell Viper Venom time are used
• Lupus anticoagulant should be confirmed by
adding phospholipid and normalizing the test
result

3400
Copyright © Harvard Medical School, 2018. All Rights Reserved.

3 Possibilities
Lupus Anticoagulant Confirmatory What dose it mean?
- Negative
+ - Negative
+ + Positive

Anticardiolipin Antibodies
• ELISA test: standardized using international
standard—GPL units
• All subsets: IgG, IgM, IgA, IgD can be seen, but
IgG is the most clinically relevant
• Clinically relevant titer is ≥ 40 units

3401
Copyright © Harvard Medical School, 2018. All Rights Reserved.

Anti-beta-2 glycoprotein I
• ELISA test
• IgG and IgM forms
• Clinically relevant titer is ≥ 40 units
• dRVVT (used with LAC) is sensitive to the
presence of anti-beta-2-glycoprotein I
• Closely correlated with thrombotic events

Antibodies in Various Patient


Populations
• Healthy controls: 1-2%
• Recurrent miscarriage population: 5-10%
• SLE: 20-40%
• SLE with livedo or Raynaud’s: 80%
• Stroke, MI under age 40: 20%
• HIV infected: IgM 50-60%

3402
Copyright © Harvard Medical School, 2018. All Rights Reserved.

Who should be evaluated for


antiphospholipid antibodies?
• SLE patients
• Patients under age 40 with CVA, MI, no
obvious risk factors
• Recurrent venous or arterial clots
• Women with recurrent first trimester
pregnancy losses, or second trimester loss

Treatment: Patients with documented


clot + antibodies
• Lifelong anticoagulation—generally warfarin
• Target INR 2.5
• Recurrent episodes, severe arterial episodes,
target INR 3-3.5

• There is insufficient data regarding the efficacy


of direct oral anticoagulants in this condition.

3403
Copyright © Harvard Medical School, 2018. All Rights Reserved.

Treatment: Antibodies in the absence


of clinical events
• Some advocate the use of prophylactic aspirin
therapy in those with positive antibodies. This
therapy has NOT been shown to decrease the
risk of clotting episodes except in patients
with co-existing SLE

Catastrophic Antiphospholipid
Syndrome (CAPS)
• Consider when multiple clots over 7 days
– Renal failure, diffuse alveolar hemorrhage, adrenal
hemorrhage, encephalopathy can be seen
– Can look similar to other thrombotic
microangiopathies
• Treatment with anticoagulants,
corticosteroids, IVIG and plasma exchange
– Some use Rituximab or Eculizumab (off label) if
refractory

3404
Copyright © Harvard Medical School, 2018. All Rights Reserved.

Summary of APS
• The antiphosholipid syndrome is defined as:
– Arterial clots, venous clots or obstetrical
complications in the presence of an
antiphospholipid antibody
– The antibody testing needs to be positive on 2
separate occasions at least 12 weeks apart
– Treatment for the arterial or venous complications
is life-long anticoagulation

Question 1
• 23 F with a history of malaise, facial rash and
achiness. Appropriate work-up includes:
• A) ANA
• B) CBC w/ diff, LFTs, creatinine and urinalysis
• C) dsDNA
• D) All of the above

3405
Copyright © Harvard Medical School, 2018. All Rights Reserved.

Answer
• B) CBC, LFTs, renal function screen

• As a first pass for the evaluation of suspected


SLE, a complete history and physical should be
performed. Appropriate laboratory testing
includes a CBC with differential, liver function
tests, creatinine and urinalysis. Avoid sending
an ANA and/or dsDNA unless there is a strong
suspicion of SLE.

Question 2
• 43 F presents with a DVT with no clear
precipitant. PMHx is notable for 2 first trimester
miscarriages and one second trimester
miscarriage. Appropriate testing includes:
• A) Lupus anti-coagulant
• B) Anticardiolipin antibody
• C) VDRL
• D) A, B, and C
• E) A and B only

3406
Copyright © Harvard Medical School, 2018. All Rights Reserved.

Answer
• E) A and B should be sent

• The history is suggestive of antiphospholipid


syndrome
• Send off both the lupus anticoagulant and
anticardiolipin antibody when you have a high
suspicion for the antiphospholipid antibody
syndrome
• VDRL is not a good screening assay for this
disorder

References
• Classification criteria for systemic lupus erythematosus: a
review. Petri M, Magder L. Lupus. 2004;13(11):829

• Guidelines for referral and management of systemic lupus


erythematosus in adults. American College of Rheumatology
Ad Hoc Committee on Systemic Lupus Erythematosus
Guidelines. Arthritis Rheum. 1999 Sep; 42(9): 1785-96

• Diagnosis and Management of the Antiphospholipid


Syndrome. Garcia D and Erkan D. NEJM. 2018 May 24; 378
(21): 2010-2019.

3407
Copyright © Harvard Medical School, 2018. All Rights Reserved.

Mono-articular Joint Complaints

Derrick J. Todd, M.D., Ph.D.


Associate Physician
Department of Rheumatology, Immunology, and Allergy
Brigham and Women’s Hospital
Instructor of Medicine
Harvard Medical School

Disclosures
• UpToDate: Author, Reviewer
• Optum: Consultant

3408
Copyright © Harvard Medical School, 2018. All Rights Reserved.

“When an arthritis
patient walks in the
front door, I feel like
leaving by the back…”

Objectives

1. Approach to the patient with monoarthralgia

2. Septic arthritis in detail

3. Microcrystalline arthritis in detail

4. Interspersed: Case-based questions

3409
Copyright © Harvard Medical School, 2018. All Rights Reserved.

Question 1:
58 y.o. alcoholic man presents to clinic with rapid
severe knee pain, swelling, and altered gait. He
has psoriatic arthritis, on adalimumab. No other
complaints. Imaging as shown. Joint aspiration:
30 cc non-bloody cloudy fluid, crystals (shown),
50,000 WBC/mm3, 95% PMN.
What is the most appropriate next step?
A. Contact orthopedics for “washout”
B. Send home with Rx for cefalexin
C. Intra-articular steroid injection
D. Wait for gram stain results before A-C

Knee Radiograph

3410
Copyright © Harvard Medical School, 2018. All Rights Reserved.

Crystal Analysis

Question 2:
A 42 year old otherwise healthy woman presents
with mild left knee discomfort but lots of swelling.
The swelling has been progressing for weeks,
without much pain or any associated systemic
symptoms. Joint aspiration: 90 cc non-bloody
slightly cloudy fluid with 12,000 WBC/mm3, 65%
PMN, no crystals, and negative Gram stain.
Which of the following would be most likely to
provide a diagnosis?
A. Serum uric acid level
B. Serum lyme serology
C. Serum ANA test
D. MRI of the affected knee

3411
Copyright © Harvard Medical School, 2018. All Rights Reserved.

Making a Diagnosis

Clinical Assessment

Diagnostic
Laboratory
Imaging
Testing

3412
Copyright © Harvard Medical School, 2018. All Rights Reserved.

Clinical Assessment
1. Pathophysiology
• Inflammatory or non-inflammatory

2. Anatomy
• Articular or not
• Joint distribution

3. Chronology
• Acute/explosive (hours or days)
• Sub-acute (weeks)
• Chronic/insidious (months or years)

4. Co-morbidities and risk factors

Acute Inflammatory Monoarthritis

Pain, warmth,
swelling, and
erythema,
involving only
one joint

3413
Copyright © Harvard Medical School, 2018. All Rights Reserved.

Acute Monoarthritis: Likely Causes


1. Septic arthritis
• Staph and Strep species (>90%)
• Neisseria species
• Other gram negatives
2. Microcrystalline disease
• Acute gouty arthritis
• Acute pseudo-gouty arthritis
3. Systemic rheumatic disease of one joint
• Spondyloarthritis (PsA, ReA, AS, IBD)
• Rheumatoid arthritis
4. Trauma or hemarthrosis

Acute Monoarthritis: Presentation

1. Acute onset: days, not weeks


2. Joint is swollen, red, warm, and painful.
3. Worry about septic if there is involvement of:
• The knee
• A prosthetic joint
• Proximal joint: shoulder or hip
• Axial joint: TMJ, SC, SI, pubis, spine

3414
Copyright © Harvard Medical School, 2018. All Rights Reserved.

Exam: Confirm “Mono” “Arthritis”

1. Perform a complete joint exam


2. How do you assess for effusion?
3. Examination should distinguish arthritis from
peri-arthritis (bursitis or tendonitis)
4. Why is active versus passive ROM important?
5. Search for infection sources and for tophi.

It’s swollen, now what?


1. Arthrocentesis (pre-antibiotics)
2. What do you need?
• Syringes, needles, and tubes
• Someone who knows how to do it.
• Universal precautions
• Prep: cleanser, lidocaine, gauze, and chuck
• Give anesthesia time!
3. Any word of advice?
4. What do you send it for?

3415
Copyright © Harvard Medical School, 2018. All Rights Reserved.

Fluid WBC Analysis

Not Sterile Septic


Normal Inflamed Inflamed Inflamed
WBC <200 Up to 1K 1K – 100K >30K
% PMN <25% 25 – 50% 25 – 90% >90%

Fluid Crystal Analysis

3416
Copyright © Harvard Medical School, 2018. All Rights Reserved.

Fluid Crystal Analysis


Calcium Pyrophosphate Dihydrate
(Pseudogout)

Monosodium Urate
(Gout)

Interpretation of Inflamed Fluid

Crystal Analysis
Negative Positive
Gram Stain

Negative

Positive

3417
Copyright © Harvard Medical School, 2018. All Rights Reserved.

Interpretation of Inflamed Fluid

Crystal Analysis
Negative Positive
Gram Stain

Negative

Positive Septic Joint

Interpretation of Inflamed Fluid

Crystal Analysis
Negative Positive
Gram Stain

Negative Crystal
Disease
Positive Septic Joint

3418
Copyright © Harvard Medical School, 2018. All Rights Reserved.

Interpretation of Inflamed Fluid

Crystal Analysis
Negative Positive
Gram Stain

Negative Crystal
Disease
Positive Septic Joint Crystals AND
Septic Joint

Interpretation of Inflamed Fluid

Crystal Analysis
Negative Positive
Gram Stain

Negative Many Options: Crystal


RA,PsA,Lyme Disease
Positive Septic Joint Crystals AND
Septic Joint

3419
Copyright © Harvard Medical School, 2018. All Rights Reserved.

Empiric Treatment

Crystal Analysis
Negative Positive
Gram Stain

Negative Consider Crystal


Empiric Abx Therapy
Positive Empiric Abx Empiric Abx
Gout Therapy

Additional Workup
1. Blood cultures
2. CBC with diff
3. Metabolic profile
4. Imaging (especially if history of trauma or prior
prosthesis is present)
5. Consider serologic workup for RA or lyme
6. Hunt for other sources of infection

3420
Copyright © Harvard Medical School, 2018. All Rights Reserved.

Risk Factors for Septic Arthritis


1. Immune-suppressed patient
• Medications: steroids, chemotherapy, DMARDs
• Co-morbidities: diabetes mellitus, cirrhosis, renal
disease, asplenism, inherited immunodeficiency, HIV
2. Prosthetic joint: recent or established
3. Recent sepsis or invasive procedure
4. Intravenous drug user
5. Pre-existing damaged joints
6. Coagulopathy

Management: Septic Arthritis


1. Antibiotics
• Empiric therapy: 3rd gen cephalosporin + vancomycin
• Tailored therapy is ideal (beta-lactam)
• Inferior alternatives: aminoglycosides, quinolone
2. Serial drainage with WBC analysis
3. Review all joints daily
4. Get orthopedics on board if…
• Any prosthetic joint is involved
• Any joint other than the knee
• The knee in a young person
• Refractory to drainage and Abx
• Osteomyelitis is present

3421
Copyright © Harvard Medical School, 2018. All Rights Reserved.

Myth-Busting: Gout 2018

Basic Principles of Gout


1. Clinical gout results from the convergence of
two processes
• Chronic hyperuricemia, with crystal deposition
• Inflammatory response to the crystals
• NO SINGLE MEDICATION ADDRESSES BOTH
2. There are different stages of gout
• Clinically silent hyperuricemia
• Intermittent acute gouty arthritis/tendinitis/bursitis
• Chronic gouty arthropathy (with or without tophi)

3422
Copyright © Harvard Medical School, 2018. All Rights Reserved.

Uric Acid
1. Uric acid level stratifies a patient’s risk for
developing clinical gout over a lifetime
• Hyperuricemia is not diagnostic of gout
• Normal uric acid does not exclude gout
2. Many factors affect a spot serum uric acid
• Hydration and dietary status
• Acute changes in renal function
• Medications (esp. diuretics)
• Active gout attack
3. Uric acid should also be used as a target of
treatment in a patient with established gout.
• Goal: uric acid <6 mg/dL, or <5 mg/dL if tophi

Hyperuricemia: Risk Factors


1. Un-modifiable risk factors
• Family history of gout or hyperuricemia
• Age and male gender
• Co-morbidities: cardiac disease, renal insufficiency,
heme malignancy, inherited metabolic defects

2. Modifiable risk factors


• Diet: alcohol (beer), shellfish, organ meat, red meat,
heavy dairy, high-fructose corn syrup
• Medications: diuretics, cyclosporine, HAART
• Obesity
• Lead exposure

3423
Copyright © Harvard Medical School, 2018. All Rights Reserved.

Acute Gouty Arthritis


1. Acute inflammatory arthritis/tendinitis/bursitis
• Usually mono- or oligo-articular
• Aspirate: inflammatory fluid with uric acid crystals
• DON’T FORGET ABOUT PSEUDOGOUT!
2. Triggers are mostly innocuous or unpredictable
• Recent trauma or repetitive overuse
• Systemic inflamed state: surgery, infection
• Fluctuations in uric acid: fluid shifts, diuretics, or
someone messed with the allopurinol
• Dietary indiscretion?
3. Treatment: Target the inflammation!

Pseudogout Arthritis: Risk Factors


1. History of pseudogout
2. Presence of chondrocalcinosis may be helpful
3. Concurrent trauma or inflamed state
4. Usually limited to the elderly
5. Altered metabolic states
• Hemochromatosis
• Hyper-PTH
• Thyroid abnormalities
• Low phosphate or magnesium
• Acromegaly

3424
Copyright © Harvard Medical School, 2018. All Rights Reserved.

Chondrocalcinosis

Acute Crystalline Arthritis: Basics

Management principles for gout or pseudogout


1. Early treatment is the most successful
2. It gets better on its own with time
3. Rest, ice, and analgesics help almost everyone
4. Try to avoid the orthopedics service
5. Don’t mess with the allopurinol in patients with gout

3425
Copyright © Harvard Medical School, 2018. All Rights Reserved.

Acute Crystalline Arthritis: Medications


1. NSAIDS: assess co-morbidities and risks
2. Corticosteroids: low doses are safe and effective
• Intra-articular: directed therapy
• Oral: 20 mg daily, tapered by 5 mg every 4 days
• Quick: IM/IV methylprednisolone 40 mg x1, then oral
3. Oral colchicine
• NEVER give intravenously
• 0.6 mg BID-TID for 1-2 days, then QD-BID
• CAUTION in renal compromise
• CAUTION with medication interactions
• Do NOT dose to the point of diarrhea…

Acute Crystalline Arthritis: Helpful Facts


1. Uric acid level is not always helpful in acute gout.
2. Distribution of involvement may be helpful:
• Most common site: Peripheral extremities
• Can include tendon sheaths and bursae
• Rarely occurs centrally: SI joints, pubis, or discs
3. May co-exist with septic arthritis or other forms of
crystalline arthritis (CPPD)
4. In bad polyarticular disease, patients can
demonstrate features of sepsis.
5. Management of hyperuricemia is often
unnecessary in the acute setting.

3426
Copyright © Harvard Medical School, 2018. All Rights Reserved.

Chronic/Recurrent Gout

THE GOUCH

3427
Copyright © Harvard Medical School, 2018. All Rights Reserved.

Chronic/Recurrent Gout
1. A disease of hyperuricemia
• Treating only inflammatory episodes is effective for
symptomatic gout but insufficient for chronic disease
• Hyperuricemia associated with many disease states:
chronic arthropathy, chronic kidney disease,
cardiovascular disease
2. When to direct treatment at hyperuricemia?
• All patients with clinical gout should be encouraged to
modify risk factors: weight, diet, and possibly diuretics
• Indicators for anti-hyperuricemic medication: tophi,
erosive arthropathy, multiple attacks, high recurrence
risk, uric acid nephropathy, or nephrolithiasis

Role of Imaging in Diagnosis of Gout


1. Radiography
• Rarely diagnostic for
gout, but can be
suggestive
• Useful to determine
extent of gouty
arthropathy

3428
Copyright © Harvard Medical School, 2018. All Rights Reserved.

Role of Imaging in Diagnosis of Gout


1. Radiography
– Rarely diagnostic for gout, but can be suggestive
– Useful for determining extent of any gouty arthropathy

2. Ultrasound (MSKUS)
– Advantages: relatively inexpensive; no radiation;
assess multiple sites; sensitive to gouty changes
– Disadvantages: operator dependent

I Got a Bridge to Sell Ya’

3429
Copyright © Harvard Medical School, 2018. All Rights Reserved.

MSKUS: Subclinical Tophus

MSKUS: Subclinical Tophus

3430
Copyright © Harvard Medical School, 2018. All Rights Reserved.

MSKUS: Subclinical Tophus

MSKUS: Subclinical Tophus

3431
Copyright © Harvard Medical School, 2018. All Rights Reserved.

Negative Birefringence

Role of Imaging in Diagnosis of Gout


1. Radiography
– Rarely diagnostic for gout, but can be suggestive
– Useful for determining extent of any gouty arthropathy

2. Ultrasound
– Advantages: relatively inexpensive; no radiation;
assess multiple sites; sensitive to gouty changes
– Disadvantages: operator dependent

3. Dual energy CT scan (DECT)


– Advantages: standardized; can assess entire
foot/ankle; can see “through” bone
– Disadvantages: cost; radiation; availability; familiarity

3432
Copyright © Harvard Medical School, 2018. All Rights Reserved.

Dual-Energy CT

http://www.dsct.com/index.php/dual-energy-imaging

Gout: Dual-Energy CT

A Huppertz 2014, Rheumatol Int

3433
Copyright © Harvard Medical School, 2018. All Rights Reserved.

Medications for Chronic Gout


1. Treat hyperuricemia!
• Explain that treatment is usually “for life”
• Goal uric acid: <6 mg/dL, or <5 mg/dL if tophi present
• Almost all compliant patients can achieve this!
• Early in treatment: patients at high risk for gout attack
• Almost always co-prescribe an anti-inflammatory such
as low-dose colchicine, typically for months.
2. The agents
• Xanthine oxidase inhibitors: allopurinol, febuxostat
• Uricosurics: probenacid, losartan, lesinurad (new)
• Recombinant uricase: pegloticase, rasburicase

Allopurinol: Myth-busting
1. Properly used, allopurinol is the most effective and
safest agent for almost all patients.
• Inadequate dose is most likely reason for treatment failure
• Allopurinol is generally safe, even at high doses.
• Start low and go slow, especially in CKD.
• I will titrate to >300 mg daily, even in patients with CKD.
• Rapid change in allopurinol can trigger gout attack!
2. Allopurinol toxicity
• Renally excreted, and only very rarely nephrotoxic
• Changes in LFTs or blood counts can be dose limiting
• Rash is rare, but can portend a more serious reaction
• Hypersensitivity: SJS, TEN, DRESS, DIL, ANCA vasculitis
• HLA-B*5801 high-risk allele (Korean, Han Chinese, Thai)

3434
Copyright © Harvard Medical School, 2018. All Rights Reserved.

Conclusions: Gout
1. Try to secure a crystal-proven diagnosis
2. Non-invasive imaging modalities help diagnose
gout and determine extent of disease burden
3. Treat inflammatory phase of acute gouty
arthritis with anti-inflammatory agents
• NSAIDs, corticosteroids, colchicine, ice, rest
4. Use anti-hyperuricemic agents when indicated
• Tophi, erosions, nephrolithiasis, multiple attacks
• Allopurinol, febuxostat, probenecid, pegloticase
5. Overlap #3 and #4, often for months
6. PLEASE don’t mess with my allopurinol

Question 1:
58 y.o. alcoholic man presents to clinic with rapid
severe knee pain, swelling, and altered gait. He
has psoriatic arthritis, on adalimumab. No other
complaints. Imaging as shown. Joint aspiration:
30 cc non-bloody cloudy fluid, crystals (shown),
50,000 WBC/mm3, 95% PMN.
What is the most appropriate next step?
A. Contact orthopedics for “washout”
B. Send home with Rx for cefalexin
C. Intra-articular steroid injection
D. Wait for gram stain results before A-C

3435
Copyright © Harvard Medical School, 2018. All Rights Reserved.

Question 1:
58 y.o. alcoholic man presents to clinic with rapid
severe knee pain, swelling, and altered gait. He
has psoriatic arthritis, on adalimumab. No other
complaints. Imaging as shown. Joint aspiration:
30 cc non-bloody cloudy fluid, crystals (shown),
50,000 WBC/mm3, 95% PMN.
What is the most appropriate next step?
A. Contact orthopedics for “washout”
B. Send home with Rx for cefalexin
C. Intra-articular steroid injection
D. Wait for gram stain results before A-C

Question 2:
A 42 year old otherwise healthy woman presents
with mild left knee discomfort but lots of swelling.
The swelling has been progressing for weeks,
without much pain or any associated systemic
symptoms. Joint aspiration: 90 cc non-bloody
slightly cloudy fluid with 12,000 WBC/mm3, 65%
PMN, no crystals, and negative Gram stain.
Which of the following would be most likely to
provide a diagnosis?
A. Serum uric acid level
B. Serum lyme serology
C. Serum ANA test
D. MRI of the affected knee

3436
Copyright © Harvard Medical School, 2018. All Rights Reserved.

Question 2:
A 42 year old otherwise healthy woman presents
with mild left knee discomfort but lots of swelling.
The swelling has been progressing for weeks,
without much pain or any associated systemic
symptoms. Joint aspiration: 90 cc non-bloody
slightly cloudy fluid with 12,000 WBC/mm3, 65%
PMN, no crystals, and negative Gram stain.
Which of the following would be most likely to
provide a diagnosis?
A. Serum uric acid level
B. Serum lyme serology
C. Serum ANA test
D. MRI of the affected knee

3437
Copyright © Harvard Medical School, 2018. All Rights Reserved.

Comments on Lyme Testing


1. Too big a topic for this talk
2. Proper sequence: Start with a screening
ELISA, and then do a Western blot only as a
confirmatory test.
3. Lyme result can be negative in early disease
(immunologic window), but are invariably
positive in established disease.
4. Avoid fishing expeditions and “specialty lyme
laboratories”, especially in patients without
clinical features of Lyme disease.
5. Be prepared to interpret a positive result!

When to Call Us
1. Any rheumatology patient
2. Any complex patient
3. The exam is equivocal
4. You would like to have procedural oversight or
to review synovial fluid yourself.
5. Tap is dry, but exam suggests fluid is present
6. At 2 A.M., remember that ortho is in-house ☺

3438
Copyright © Harvard Medical School, 2018. All Rights Reserved.

Summary
1. History and exam should prove that acute
monoarthritis is “acute” “mono” and “arthritis.”
2. A hot joint is septic until proven otherwise.
3. Arthrocentesis with synovial fluid analysis is the
procedure of choice and guides empiric therapy.
4. Don’t mess with the allopurinol.

Disclosures
None

3439
Copyright © Harvard Medical School, 2018. All Rights Reserved.

Additional Reading for William Osler


1. Baker DG, Schumacher HR. “Acute monoarthritis” N
Engl J Med. 1993;329(14):1013-1020.
2. Schlesinger N. “Diagnosing and treating gout: A review
to aid primary care physicians.” Postgrad Med.
2010;122(2):157–61.
3. Sharff KA, Richards EP, and Townes JM. “Clinical
Management of Septic Arthritis.” Current
Rheumatology Reports. 2013;15(6):332.
4. Wilson JF. “In the clinic. Gout.” Ann Intern Med.
2010;152(3):ITC21. Erratum Ann Intern Med.
2010;152(7):479–80.

3440
Copyright © Harvard Medical School, 2018. All Rights Reserved.

Take home messages


Rheumatology

Paul F Dellaripa MD

Disclosures
• Up To Date
• Genentech
• Bristol Myers Squibb

3441
Copyright © Harvard Medical School, 2018. All Rights Reserved.

Teaching point:Distinguishing Bursitis


vs arthritis on examination
• The ability to fully extend the knee with limited
flexion indicates the process is outside the joint (
ie bursa) . In bursitis, intrabursal pressures
increase with flexion and decrease with
extension.
• In a intra-articular process (ie synovitis in a joint)
the patient lacks the ability to fully extend the
knee because intra-articular pressure increases in
extension and is lowest in semiflexion.

Acute Monoarthritis: Likely causes


• Septic arthritis (Staph, Strep mostly, Neisseria,
Gram negatives less likely)
• Micro crystalline disease ( gout, pseudogout)
• Systemic rheumatic disease of one joint or
starting in one joint (RA, spondyloarthritis)
• Trauma or hemarthrosis

3442
Copyright © Harvard Medical School, 2018. All Rights Reserved.

Synovial fluid analysis:


interpretation of cell count/diff

• <1-2,000 wbc/cc: noninflammatory fluid


– O.A., trauma, other
• >2,000 wbc/cc: inflammatory fluid
– DDx is extensive; basically includes all
the systemic rheumatic diseases

Synovial fluid analysis:


interpretation of cell count/diff [cont.]
• >30-100,000 wbc/cc:
suggestive of septic arthritis
• But may also be seen with
gout and pseudogout and occasionally with
R.A. and so-called “reactive arthritis”

3443
Copyright © Harvard Medical School, 2018. All Rights Reserved.

Synovial fluid analysis: what to send-make


every attempt to obtain!
• 1. Gram stain, C&S
• 2. Cell count and differential
(anticoagulated tube)
• 3. Analysis for crystals by compensated
polarized microscopy
(anticoagulated tube;
crystals will “keep” for hours)
---------------------------------------------------------------
(Glucose, total protein, LDH, etc.
don’t add anything)

Treatment of rheumatoid arthritis


• MTX - 1st line DMARD for most patients
- weekly dosing 15-25 mg PO or SQ
- folic acid 1 mg daily to reduce side
effect risks or leucovorin weekly 5-15mg
- CBC, LFT, Cre monitoring
• “treat to target” (frequent monitoring +
medication adjustment) improves
longterm outcomes
• optimize treatment with current agent
before making changes

3444
Copyright © Harvard Medical School, 2018. All Rights Reserved.

Treatment of rheumatoid arthritis


• second line agents (typically added to Mtx):
- TNF inhibitors
- IL-6 inhibitors
- B cell depletion (Rituximab)
- T cell co-stimulatory blockade
- oral Jak inhibitors
- traditional DMARD (Sulfasalazine,
Hydroxychloroquine “triple therapy”)
• role of corticosteroids:
- useful adjunct at low doses (<10 mg)
Singh Arthritis
Rheumatol 2016
- many side effects

Singh Arthritis Rheumatol 2016

3445
Copyright © Harvard Medical School, 2018. All Rights Reserved.

Take home message in RA

• Remember that there are new criteria for the diagnosis of RA


• Maximizing MTX is still a cornerstone of Rx in RA and likely lowers
mortality in RA
• Patients starting on DMARDs like MTX or leflunomide or any
biologic need to have an assessment of Hep B and C status.
• Assess for prior history of TB prior to DMARD or biologic therapy
• Pneumovax/flu vaccine advocated for such patients.
• Consideration for timing of vaccinations such as zoster are
important before beginning drugs for RA
• The Use of triple therapy may become more prominent compared
to MTX/biologic combination based on recent data

Spondyloarthritis
• psoriatic arthritis
• IBD
• Reactive arthritis
• ankylosing spondylitis
• Role of TNF inhibitors in
axial disease
• Newer agents now
approved that affect
TH17 pathway ( IL-17
and IL-12/23 inhibition)

3446
Copyright © Harvard Medical School, 2018. All Rights Reserved.

Adequate NSAID
trial:
- lack of response
(or intolerance) to
≧2 different NSAIDs
over 1 month
- incomplete
responses to ≧2
different NSAIDs
Ward Arthritis Rheum 2016 over 2 months

SLE and Autoantibodies


• ANA- found in >95% of patients
• Anti-ds-DNA- 40% -80% of patients
• Anti-Sm- 25% of patients
• Anti-histone- seen in drug induced SLE
• Anti-Ro, Anti-La - Sjogren’s, SCLE
• Anti-RNP- Mixed Connective Tissue Disease
• False positive VDRL- an anti-phospholipid antibody

3447
Copyright © Harvard Medical School, 2018. All Rights Reserved.

The Antiphospholipid Syndrome is


defined as:
• Arterial clots, venous clots or obstetrical
complications in the presence of an
antiphospholipid antibody. The antibody
testing needs to be positive on two separate
occasions at least 12 weeks apart.
• Treatment for the arterial or venous
complications is life-long anti-coagulation

Patients who should be evaluated for


antiphospholipid antibodies
• SLE patients
• Patients under age 40 with CVA, MI no obvious risk
factors
• Recurrent venous or arterial clots
• Women with recurrent first trimester pregnancy
losses, or second trimester loss
• Some advocate the use of prophylactic aspirin
therapy in those with positive antibodies. This
therapy has not been shown to decrease the risk of
clotting events except in patients with co-existing
SLE.

3448
Copyright © Harvard Medical School, 2018. All Rights Reserved.

Osteoarthritis:treatment options
• Acetominophen
• NSAIDS, tramadol (careful in the elderly, interaction tramadol with
SSRI)
• Steroid Injections Evidence based data: Steroid injection for osteoarthritis of the hip: a
randomized, double-blind, placebo-controlled trial. (Lambert RG, et al Arthritis Rheum. 2007;56(7):2278)
• Splinting of small joint like the CMC joint of the thumb can be very
useful
• Viscosupplemenation (controversial)
• Chondroitan (data not supportive)
• Arthroscopic lavage/debridement of the knee ( data not
supportive)
• Narcotics (avoid)
• Weight loss and knee OA *

Patient has features of secondary OA. JSN narrowing at MCPs

•cartilage calcification

• OA type changes at wrist and


MCPs with joint space narrowing

Chondrocalcinosis------->>
The Differential Diagnosis of Secondary OA:
Hemochromatosis
Hypothyroidism
Hyperparathyroidism
Rarely : Wilson’s Disease; Ochronosis

18

3449
Copyright © Harvard Medical School, 2018. All Rights Reserved.

Uric acid

• The serum uric acid (SUA) ranges widely in


gout, from “normal” to elevated
• A very low SUA (< 4-5 mg %)
makes diagnosis less likely
• SUA may drop by 1-2 mg %
during an acute attack

Clinical presentation: gouty arthritis


• Typical presentation:
- acute
- monoarticular or oligoarticular
- lower extremity joints
• 1st MTP: involved in 50% of initial attacks;
95% involvement in recurring gout
• BUT: any joint and tendon in the body
lined with synovium may be affected

3450
Copyright © Harvard Medical School, 2018. All Rights Reserved.

A caveat about gout treatment


• Important: don’t treat the serum uric acid during an acute attack of
gout-usually
• If the SUA is changing rapidly (up or down), risk of acute gout is
increased
– so never start hypouricemic Rx (e.g., allopurinol)
in the midst of an acute attack—but…
– obvious corollary: if a patient is already on alllopurinol and
flares, and has been on allopurinol don’t stop it
– Add on Colchicine for acute disease is reasonable in addition to
steroids/NSAIDS

Chronic Rx: maximal dose of allopurinol is not known


Febuxostat: used when intolerant to allopurinol
Uricase: used only where severe burden of tophi
Anakindra: very useful in recalcitrant cases, but expensive.

Clinical presentation: acute pseudogout

• Older adults
• Typically presents as an acute monarticular
arthritis, most commonly at the knee,
shoulder.
– big toe (gout) :: knee (pseudogout)
• Can be febrile
• Chondrocalcinosis is common

3451
Copyright © Harvard Medical School, 2018. All Rights Reserved.

Chondrocalcinosis

Pseudogout [cont.]

– remember chondrocalcinosis is a
radiologic diagnosis
– and pseudogout is a
clinical diagnosis

3452
Copyright © Harvard Medical School, 2018. All Rights Reserved.

Raynauds phenomenon
• Primary Raynauds common in young women (teens
and twenties), may have a family history of this as
well, ANA mostly negative.
• Onset of Raynauds in adults after the age of 35
concerning for the development of a rheumatic
syndrome and is termed secondary Raynauds.
Digital ulcers, pitting scars in fingers, abnormal
capillary microscopy and presence of autoantibodies
suggest the development of an underlying rheumatic
syndrome.

Key points in Scleroderma


• Recognize the presence of concomitant Raynauds and
GERD as a possible clinical manifestation of underlying
limited scleroderma and therefore the longterm risk
of pulmonary hypertension
• ILD occurs in diffuse SSc>limited SSc but can occur in
both
• Avoid steroids in SSc especially high dose, in SSc pts. (
increased risk of renal crisis)
• Pulmonary hypertension is a complication of long
standing scleroderma of mostly the limited variant and
treatment that improve morbidity and possible
mortality are available now.
• Newer treatments and clinical trials underway

3453
Copyright © Harvard Medical School, 2018. All Rights Reserved.

Newer classification in myositis


• Inclusion body myositis (IBM)
• DM ( prob or definite)
• PM (prob or definite)
• Amyopathic DM
• Nonspecific myositis
• Necrotizing myopathy
• If you have a pt being treated for “inflammatory
myositis” and they are not improving consider the
possibility of inclusion body myositis

Fibromyalgia

Non-inflammatory
Pain without objective findings
Sleep disturbance
Mood disorders
Normal labs
Polymyositis

Painless weakness
Proximal>Distal
CK elevated
EMG - insertional irritability

28

3454
Copyright © Harvard Medical School, 2018. All Rights Reserved.

Antisynthetase syndrome
Dermatomyositis: “mechanic’s hands”

• Fever
• Raynauds
• Inflammatory Arthritis
• Mechanics hands
• ILD (can be severe)
• Myositis
• Typically associated with Jo-1, PL-12, PL-
7 ab amongst others recently discovered

ANCA associated vasculitis


• Include Granulomatosis with Polyangittis (Wegener's
Granulomatosis ), microscopic polyangitiis and EGPA
(Churg Strauss) and drug induced vasculitic syndromes
(PTU, allopurinol, levamisole tainted cocaine)
• Morbidity associated with these diseases typically related
to renal failure due to do glomerulonephritis or pulmonary
hemorrhage.
• Mortality often related to disease progression but more
often treatment associated infection. In fact,vasculitis
that appears to be worsening should be considered
an infection until proven otherwise.
• Rituximab established therapy for ANCA vasculitis but
other agents such as MTX, Azathioprine or abatacept
may be used in limited cases or as maintenance
therapy

3455
Copyright © Harvard Medical School, 2018. All Rights Reserved.

GCA Pearls
• GCA is a strong consideration in the elderly with new onset
headache, neck ache , visual changes or unexplained fatigue or
anemia
• Jaw claudication is the most specific sign of GCA, followed by visual
loss and TA tenderness
• A more robust inflammatory response is correlated with a lower
risk for visual loss. (33)
• Patients with a low or normal ESR and CRP can have GCA If one
biopsy is negative, the additional yield of a contralateral biopsy is
modest but is reasonable to consider performing. Treat if needed—
have about 2 weeks to change pathology
• A rising ESR in a treated for GCA does not necessarily suggest that
GCA is returning
• Ultrasound can help but a negative study does not rule out the
diagnosis
• Tociluzimab now FDA approved for GCA and can allow more rapid
taper of steroids ( Stone et al NEJM 2017)

PMR pearls
• Synovitis of the hands can occur in PMR-a
subset of these patients evolve into RA
• A normal ESR does not exclude the diagnosis
• 15-20% of patients with PMR will develop GCA
• Consider a temporal artery biopsy where
persistent constitutional symptoms or
inflammatory markers remain high.

3456
Copyright © Harvard Medical School, 2018. All Rights Reserved.

Final case: 40 yo male Bilateral ankle pain and


painful red nodules on shins..what is the next
test to order

Lofgrens syndrome: Acute form of sarcoid, typically


responds to NSAIDS, rarely steroids and rarely progress
to chronic sarcoidosis

3457
Copyright © Harvard Medical School, 2018. All Rights Reserved.

Rheumatology Board Review

Joerg Ermann, MD
Instructor in Medicine, Associate Physician
Brigham and Women’s Hospital
Harvard Medical School

Disclosures

• Consulting/Scientific Advisory Boards:


Boehringer Ingelheim, Eli Lilly, Novartis
• Research Grants:
Boehringer Ingelheim, Pfizer

3458
Copyright © Harvard Medical School, 2018. All Rights Reserved.

Question 1

A 38 yo woman with a recent diagnosis of seropositive


rheumatoid arthritis has been treated with oral
Methotrexate 12.5 mg weekly for the past 4 months. She
has had only modest improvement. According to your
notes, the number of swollen joints has dropped from 15
to 11 and the number of tender joints has dropped from
18 to 13.
Labs:
RF and anti-CCP positive
Hct 36.1
ESR 50, CRP 14.1 mg/l

What would you recommend?

A. add a TNF inhibitor


B. begin Rituximab infusions
C. switch MTX to Tofacitinib
D. increase MTX to 25 mg weekly
E. add prednisone 20 mg daily

3459
Copyright © Harvard Medical School, 2018. All Rights Reserved.

What would you recommend?

A. add a TNF inhibitor


B. begin Rituximab infusions
C. switch MTX to Tofacitinib
D. increase MTX to 25 mg weekly
E. add prednisone 20 mg daily

Singh Arthritis Rheumatol 2016; 68:1-26

3460
Copyright © Harvard Medical School, 2018. All Rights Reserved.

Treatment of rheumatoid arthritis

• Methotrexate
- 1st line DMARD for most patients
- weekly dosing 15-25 mg PO or SQ
- folic acid 1 mg daily reduces side effects
- CBC, LFT, Crea monitoring
• T2T = “treat to target” improves longterm outcomes
• optimize treatment with current agent before changing
drugs

Singh Arthritis Rheumatol 2016; 68:1-26

Treatment of rheumatoid arthritis

• second line agents, typically added to Methotrexate:


- cytokine inhibitor (TNF, IL-6)
- B cell depletion
- T cell co-stimulatory blockade
- oral Jak inhibitors
- conventional DMARDs (Hydroxychloroquine,
Sulfasalazine)
• role of corticosteroids:
- useful adjunct at low doses (≦10 mg)
- many side effects

Singh Arthritis Rheumatol 2016; 68:1-26

3461
Copyright © Harvard Medical School, 2018. All Rights Reserved.

Question 2

Since the introduction of monoclonal antibodies into


clinical practice ~20 years ago, biologics have
revolutionized the care of patients with rheumatic
diseases, cancer and other illnesses.
Within the next few years, the patents for several of the
original biologics will expire opening the market to
biosimilars produced by competing manufacturers.
Which of the following statement regarding biosimilars is
correct?

Compared with the reference product,


biosimilars…

A. are structurally identical


B. have similar efficacy
C. are ~90% cheaper
D. are cheaper but may not be as effective
E. all of the above

3462
Copyright © Harvard Medical School, 2018. All Rights Reserved.

Compared with the reference product,


biosimilars…

A. are structurally identical


B. have similar efficacy
C. are ~90% cheaper
D. are cheaper but may not be as effective
E. all of the above

Biosimilars

• identical amino acid sequence as originator (reference)


drug but structure may differ slightly due to post-
translational modifications
• biosimilars ≠ generic drugs
• must be highly similar to an approved biological
product (originator) in terms of structure, function,
quality, clinical efficacy and safety
• at least 1 clinical trial in 1 indication → extrapolation to
other indications

Lyman NEJM 2018; 378:2036-44

3463
Copyright © Harvard Medical School, 2018. All Rights Reserved.

Question 3

A 52 yo female with seropositive rheumatoid arthritis


comes for her annual physical examination. She was
diagnosed with the disease 10 years ago and is currently
taking 20 mg methotrexate PO weekly and injects a TNF
inhibitor every other week.
She denies any significant joint problems (joint swelling,
pain, morning stiffness) and feels generally well.
She plans to travel to South America later in the year. You
review her vaccination records.

Which of the following vaccines should be avoided


in this patient?

A. Hepatitis A
B. Hepatitis B
C. Td booster
D. yellow fever
E. recombinant Zoster vaccine

3464
Copyright © Harvard Medical School, 2018. All Rights Reserved.

Which of the following vaccines should be avoided


in this patient?

A. Hepatitis A
B. Hepatitis B
C. Td booster
D. yellow fever
E. recombinant Zoster vaccine

Vaccination of patients on
immunosuppressive drugs

• live-attenuated vaccines are contraindicated


- MMR
- live-attenuated influenza
- yellow fever
- zoster live vaccine (recombinant zoster vaccine is
safe)
• all patients on immunosuppressive drugs should
receive
- annual recombinant influenza
- pneumococcal vaccine (PCV13 + PPSV23), ideally
prior to initiation of immunosuppression
- other vaccines as indicated by age or other
circumstances
https://www.cdc.gov/vaccines/hcp/acip-recs/general-recs/immunocompetence.html

3465
Copyright © Harvard Medical School, 2018. All Rights Reserved.

Question 4

A 28 yo kindergarten teacher presents with 1 week of


pain, swelling and morning stiffness in her hands and
knees. 3 weeks ago, she had a cold with nasal discharge,
fever, malaise, and muscle aches. Several children in her
class had similar symptoms accompanied by an
erythematous rash on the cheeks.
Exam:
Mild soft tissue swelling of several PIPs and MCPs.
Minimally swollen + tender wrists and knees bilaterally.
Labs:
CBC unremarkable, ESR 40
ANA 1:40, RF 24 IU

What initial treatment would you


recommend?

A. Naproxen 500 mg BID


B. Hydroxychloroquine 400 mg daily
C. Methotrexate 15 mg qweek + folic acid 1 mg daily
D. Prednisone 20 mg daily
E. Doxycycline 100 mg BID for 1 week

3466
Copyright © Harvard Medical School, 2018. All Rights Reserved.

What initial treatment would you


recommend?

A. Naproxen 500 mg BID


B. Hydroxychloroquine 400 mg daily
C. Methotrexate 15 mg qweek + folic acid 1 mg daily
D. Prednisone 20 mg daily
E. Doxycycline 100 mg BID for 1 week

What is the most likely diagnosis?

• presentation highly suggestive of Parvovirus B-19


infection (viral arthritis)
- acute mild symmetric synovitis
- exposure to sick children with facial rash
• generally a self limited disease, improvement over
several weeks, minimal intervention required
• RF may be mildly elevated,
ANA of 1:40 is generally not significant

Marks Clin Med (London) 2016; 16:129-34

3467
Copyright © Harvard Medical School, 2018. All Rights Reserved.

Question 5

A 25 yo female medical student presents to the ED with


fever, chills and a diffuse rash. This started the day after
she returned from Haiti where she had spent one month
volunteering in a health clinic. Within hours, she also
developed severe widespread joint pain (affecting her
PIPs, MCPs, wrists, MTPs, ankles, knees, hips).
Exam:
T 103°F, HR 130/min, BP 130/71, O2 98% on room air
diffuse, blanchable erythematous rash on trunk and limbs,
swollen MCPs and MTPs, multiple tender joints
Laboratory:
CRP elevated, Wbc 3.6, Hct 30, Plt 140
LFT WNL

Which of these viruses could be the cause


for the patient’s symptoms?

A. Zika
B. Chikungunya
C. Borrelia
D. O’nyong’nyong
E. Ross River

3468
Copyright © Harvard Medical School, 2018. All Rights Reserved.

Which of these viruses could be the cause


for the patient’s symptoms?

A. Zika
B. Chikungunya
C. Borrelia
D. O’nyong’nyong
E. Ross River

Chikungunya arthritis

• consider viral arthritis in context of travel history


Chikungunya - Caribbean, Central Africa, SE Asia
O’nyong'nyong - East Africa
Ross River - Australia
• arbovirus (arthrophod-born) transmitted by mosquitoes
→ acute disease with fever, rash, arthritis, myalgias,
typically self-limited
• confirmation by antibody testing (IgM, IgG)
• treatment - NSAIDs

Waymouth Semin Arthritis Rheum 2013; 42:273-8, Weaver NEJM 2015; 372:1231-9, Sutaria Curr Opin Rheumatol 2018; 30:256

3469
Copyright © Harvard Medical School, 2018. All Rights Reserved.

Origin, spread and distribution of


Chikungunya virus and its vectors

Weaver NEJM 2015; 372:1231-9

Question 6

A 72 yo female presents with 6 weeks of stiffness in the


neck and both shoulder. She has shoulder pain when
dressing herself in the morning. She also reports pain at
night and sleeps poorly. Symptoms improve as the day
progresses but then return in the early evening. No
significant PMH except for HTN.
Exam:
Mildly decreased passive ROM in the shoulders. Mild OA
changes in the hands. No synovitis. No muscle weakness.
Labs: CBC unremarkable, ESR 12 mm/h

3470
Copyright © Harvard Medical School, 2018. All Rights Reserved.

What is the most likely diagnosis?

A. polymyositis
B. early rheumatoid arthritis (RA)
C. fibromyalgia
D. polymyalgia rheumatica (PMR)
E. metabolic myopathy

What is the most likely diagnosis?

A. polymyositis
B. early rheumatoid arthritis (RA)
C. fibromyalgia
D. polymyalgia rheumatica (PMR)
E. metabolic myopathy

3471
Copyright © Harvard Medical School, 2018. All Rights Reserved.

Polymyalgia rheumatica (PMR)

• clinical diagnosis
• pain and morning stiffness in neck, shoulders, hips
age >50, +/- constitutional symptoms
• shoulder girdle (60%)
hip girdle (5%)
both (35%)
• Labs: ESR and/or CRP elevated in 80%
anemia 15%
• associated with GCA (headache, visual loss, FUO)

Kermani Lancet 2013; 381:63-72, Helfgott Arthritis Rheum 1996; 39:304-7

Question 7

An 80 yo woman was hospitalized two days ago for upper


GI bleeding. She was found to have a peptic ulcer and
was placed on omeprazole plus IV normal saline for
hydration.
Today, she presents with acute painful swelling of the
right knee. PMH is significant for OA of the hands and
knees, for which she took Ibuprofen at home.
Exam:
T 100.0
Heberden + Bouchard nodes in both hands
warmth and a large effusion in the right knee
knee aspiration → 60 ml of yellow cloudy fluid
(+ intracellular rhomboid positively birefringent crystals)

3472
Copyright © Harvard Medical School, 2018. All Rights Reserved.

What is the likely cell count in the synovial


fluid of this patient?

A. 500 cells/µl
B. 500 cells/ml
C. 15,000 cells/µl
D. 15,000 cells/dl
E. 15,000 cells/ml

What is the likely cell count in the synovial


fluid of this patient?

A. 500 cells/µl
B. 500 cells/ml
C. 15,000 cells/µl
D. 15,000 cells/dl
E. 15,000 cells/ml

3473
Copyright © Harvard Medical School, 2018. All Rights Reserved.

What is the likely cell count in the synovial


fluid of this patient?

A. 500 cells/µl
B. 500 cells/ml
C. 15,000 cells/µl
D. 15,000 cells/dl
E. 15,000 cells/ml

Synovial fluid analysis

• informative tests:
- cell count + differential
- Gram stain + culture
- crystal analysis
• not informative: albumin, protein, glucose
• “inflammatory” >2,000 cells/µl
• septic arthritis more likely with
- high cell count (>50,000 cells/µl)
- high neutrophil fraction (>95%)

Courtney Best Pract Res Clin Rheumatol 2009; 23:161-92

3474
Copyright © Harvard Medical School, 2018. All Rights Reserved.

Question 8: What is the serum uric acid target


for patients on uric acid lowering drugs?

A. less than 4 mg/dl


B. less than 5 mg/dl
C. less than 6 mg/dl
D. less than 7 mg/dl
E. less than 8 mg/dl

Question 8: What is the serum uric acid target


for patients on uric acid lowering drugs?

A. less than 4 mg/dl


B. less than 5 mg/dl
C. less than 6 mg/dl
D. less than 7 mg/dl
E. less than 8 mg/dl

3475
Copyright © Harvard Medical School, 2018. All Rights Reserved.

Allopurinol dosing

• treat to target: serum uric acid <6 mg/dl in all patients


• alternative target:
<5 mg/dl in severe gout for faster dissolution of
crystals
• Flare prophylaxis during initiation!
(e.g. low dose Colchicine)
• patients may require >300 mg daily to achieve target
• repeat uric acid test 1 month after dose adjustments

Khanna Arthritis Care Res 2012; 64:1431-46, Richette Ann Rheum Dis 2017; 76:29-42

Allopurinol toxicity

• clinical features:
- rashes (mild → Stevens-Johnson syndrome)
- LFT abnormalities
- Allopurinol hypersensitivity syndrome
• low starting dose reduces risk for severe toxicity
• consider screening for HLA-B*58:01 in Asians
• alternatives
- desensitization
- Febuxostat
- uricosuric drugs

Khanna Arthritis Care Res 2012; 64:1431-46

3476
Copyright © Harvard Medical School, 2018. All Rights Reserved.

Question 9

A 68 yo man with HTN and tophaceous gout returns for


follow-up. He was started on Allopurinol + Colchicine
6 months ago. Following Allopurinol titration (current dose
400 mg daily), his serum uric acid has been <6.0 mg/dl
for the last 3 months without gout flare.
Exam:
Moderate-sized tophus over the left elbow, two small
tophi on DIPs. No swollen or tender joint.
Lab:
uric acid 4.9 mg/dl
CBC, CMP unremarkable

Which of the following is the most


appropriate next step in management?

A. add Probenecid
B. change Allopurinol to Febuxostat
C. reduce the Allopurinol dose
D. discontinue Colchicine
E. continue current gout medications

3477
Copyright © Harvard Medical School, 2018. All Rights Reserved.

Which of the following is the most


appropriate next step in management?

A. add Probenecid
B. change Allopurinol to Febuxostat
C. reduce the Allopurinol dose
D. discontinue Colchicine
E. continue current gout medications

Khanna Arthritis Care Res 2012; 64:1431-46, Richette Ann Rheum Dis 2017; 76:29-42

Flare prophylaxis during initiation of uric


acid lowering therapy

• recommended for all gout patients, continue until no


evidence of gout activity (no gout flare in 3 months, no
tophi)
• minimum duration
- 6 months
- 3 months after achieving serum target (if no tophus)
- 6 months after achieving serum target (if tophi were
present)
• options
- Colchicine (renally dosed)
- low dose NSAIDs
- low dose Prednisone (≦10 mg daily)

Khanna Arthritis Care Res 2012; 64:1447–61

3478
Copyright © Harvard Medical School, 2018. All Rights Reserved.

Question 10

A 45 yo male presents with painful swelling of the 3rd and


4th toe of the right foot and a rash under the soles of his
feet for 4 weeks. He denies fever, recent dysuria,
diarrhea, eye problems, or a h/o psoriasis or back pain.
He is sexually active. He takes no medications.
Exam:
Dactylitis of the right 3rd and 4th toes. Yellow-brown
nodular/hyperkeratotic crusty rash under both feet.
Laboratory + Imaging:
CRP 8 mg/l
CBC unremarkable
X-ray foot - soft tissue swelling

3479
Copyright © Harvard Medical School, 2018. All Rights Reserved.

Which of the following is the most


appropriate diagnostic test?

A. anti-CCP
B. ANA
C. urine PCR test for Chlamydia trachomatis
D. HLA-B27
E. X-ray of the pelvis

Which of the following is the most


appropriate diagnostic test?

A. anti-CCP
B. ANA
C. urine PCR test for Chlamydia trachomatis
D. HLA-B27
E. X-ray of the pelvis

3480
Copyright © Harvard Medical School, 2018. All Rights Reserved.

Reactive arthritis

• within 4 weeks after UTI or infectious diarrhea


Chlamydia, Yersinia, Salmonella, Shigella,
Campylobacter
• synovial fluid: Gram-stain and culture negative
other microbial analysis also often negative
• self-limited (50%) or chronic (50%)
- asymmetric oligoarthritis, enthesitis, dactylitis
- sacroiliitis
- keratoderma blennorrhagica, circinate balanitis
• treatment with NSAIDs (+ DMARDs if chronic)
no role for antibiotics

Carter Best Pract Res Clin Rheumatol 2011; 25:359-74

HLA-B27

• strongly associated with SpA


- ankylosing spondylitis ~90%
- reactive arthritis 30-70%
- psoriatic arthritis 40-50%
• prevalence of HLA-B27+ in US
6.1% overall
7.5% non-Hispanic White
4.6% Mexican American
1.1% non-Hispanic Black
• HLA-B27 never makes a diagnosis of SpA

Khan Curr Rheumatol Rev 2010; 12:337-41, Reveille Arthritis Rheum 2011, 64:5, 1407-11

3481
Copyright © Harvard Medical School, 2018. All Rights Reserved.

Question 11

A 28 yo male presents with chronic pain in the low back


and buttocks for about 3 years. The pain is worse in the
morning and associated with stiffness. Symptoms
improve with exercise. Ibuprofen provides some relief.
The patient denies peripheral joint pain or swelling,
rashes, GI complaints, eye symptoms. A sibling has
psoriasis.
Exam:
Tenderness to palpation over the SI joints, otherwise
unremarkable. ROM in the lumbar spine is normal.
Labs and Imaging:
HLA-B27 negative, CRP 7 mg/l
No evidence for sacroiliitis on pelvic X-ray.

What is the most appropriate diagnostic


test?

A. X-ray cervical, thoracic, lumbar spine


B. CT pelvis
C. U/S of SI joints
D. MRI of the SI joints
E. 99mTc bone scan

3482
Copyright © Harvard Medical School, 2018. All Rights Reserved.

What is the most appropriate diagnostic


test?

A. X-ray cervical, thoracic, lumbar spine


B. CT pelvis
C. U/S of SI joints
D. MRI of the SI joints
E. 99mTc bone scan

Axial spondyloarthritis

• includes patients with AS + ‘non-radiographic’ disease


• characteristic features
- chronic back pain (>3 months), onset before age 45
- inflammatory back pain
- association with HLA-B27, psoriasis, IBD, uveitis
- good response to NSAIDs
- M:F = 1:1 (2-3:1 in AS)
• initial tests - CRP, HLA-B27, X-ray pelvis
MRI of SI joints if X-ray negative
• recognize and refer to Rheumatology for evaluation

Rudwaleit Ann Rheum Dis 2009; 68:777-83, Sieper Arthritis Rheum 2013; 65:543-551

3483
Copyright © Harvard Medical School, 2018. All Rights Reserved.

2009 ASAS classification criteria for axial SpA


(in patients with back pain ≥3 months and age at onset <45 years)

Sacroiliitis on imaging HLA-B27


plus or plus
≥1 SpA feature ≥2 other SpA features

• inflammatory back pain Sacroiliitis on imaging


• arthritis • active (acute) inflammation
• enthesis (heel) on MRI highly suggestive of
• uveitis sacroiliitis associated with
• dactylitis SpA
• psoriasis OR
• Crohn’s disease/ulcerative • definite radiographic
colitis sacroiliitis according to mNY
• good response to NSAIDs criteria
• family history for SpA
sensitivity 82.9%, specificity 84.4% (overall)
• HLA-B27 sensitivity 66.2%, specificity 97.3% (imaging
• elevated CRP arm)

Rudwaleit Ann Rheum Dis 2009;


68:777-83

Bilateral sacroiliitis on MRI (STIR)

3484
Copyright © Harvard Medical School, 2018. All Rights Reserved.

Question 12

A 29 yo male with ankylosing spondylitis diagnosed


2 years ago presents with worsening low back and
buttock pain over the last 6 months. Morning stiffness
lasts 3 hours. ROS is negative except for fatigue.
Symptoms had previously been well-controlled with
physical therapy and Naproxen.
A switch to Indomethacin 50 mg TID one month ago was
without benefit.
Exam:
Tenderness to palpation over the SI joints.
Schober test 4 cm. Otherwise unremarkable.
Labs:
CRP 10 mg/l

What is the most appropriate addition


to this patient’s medical therapy?

A. Tramadol
B. Methotrexate
C. Sulfasalazine
D. Adalimumab
E. Prednisone

3485
Copyright © Harvard Medical School, 2018. All Rights Reserved.

What is the most appropriate addition


to this patient’s medical therapy?

A. Tramadol
B. Methotrexate
C. Sulfasalazine
D. Adalimumab
E. Prednisone

Management of ankylosing spondylitis

• first line: NSAIDs and physical therapy


• inadequate NSAID response:
- no response/intolerance to ≥ 2 NSAIDs over 1 month
- incomplete response to ≥ 2 NSAIDs over 2 months
• second line: TNF inhibitors, IL-17A antagonists
• no role for systemic corticosteroids or conventional
DMARDs (e.g. Methotrexate)

Ward Arthritis Rheumatol 2016; 68:282-98, van der Heijde Ann Rheum Dis 2017; 76:978-
991

3486
Copyright © Harvard Medical School, 2018. All Rights Reserved.

Question 13
A 42 year-old man is evaluated for pain and swelling of
his left wrist. This started suddenly four days ago. No
history of trauma.
The patient has insurance through ‘HMO Cheap’. In
addition to radiographs, they will only pay for only one
blood test.

Which of the following tests should be


ordered?

A. transferrin saturation
B. serum calcium
C. rheumatoid factor
D. ANA
E. serum uric acid

3487
Copyright © Harvard Medical School, 2018. All Rights Reserved.

Which of the following tests should be


ordered?

A. transferrin saturation
B. serum calcium
C. rheumatoid factor
D. ANA
E. serum uric acid

• pertinent information:
- chondrocalcinosis
- acute wrist arthritis (pseudogout)
- degenerative changes in wrist and MCP2/3
• presentation c/w CPPD + secondary osteoarthritis
• differential diagnosis:
- hemochromatosis
- hyperparathyroidism
- hypothyroidism
- acromegaly
- diabetes mellitus
- Wilson’s disease, Ochronosis (rare)

Husar-Memmer Curr Rheumatol Rep 2013; 16:393

3488
Copyright © Harvard Medical School, 2018. All Rights Reserved.

Question 14

A 45 yo high school teacher is evaluated for low back


pain that started insidiously 3 weeks ago and has
become worse over the last week. It is now constant,
worse with movements and does not respond to OTC
ibuprofen. He denies a history of trauma, malignancy,
fever, weight loss, numbness, tingling or weakness in the
legs, bladder or bowel symptoms. He does not use IV
drugs.
Exam:
uncomfortable, reduced ROM in L spine
sensation, strength, DTR in legs are symmetric and WNL
straight-leg-raise test → lumbar pain

What is the most appropriate next step?

A. X-ray lumbar spine


B. MRI lumbar spine
C. corticosteroid taper
D. bedrest for 3 days, then graded exercise program
E. change NSAID

3489
Copyright © Harvard Medical School, 2018. All Rights Reserved.

What is the most appropriate next step?

A. X-ray lumbar spine


B. MRI lumbar spine
C. corticosteroid taper
D. bedrest for 3 days, then graded exercise program
E. change NSAID

Nonspecific low back pain

• pain without evidence for a serious underlying


condition (e.g. cancer, infection, cauda equina
syndrome), spinal stenosis, radiculopathy, or other
specific spinal cause (e.g. compression fracture, AS)
• imaging not recommended
• management
- education + reassurance
- superficial heat, other non-pharmacological
interventions
- NSAIDs or muscle relaxants

Chou Ann Int Med 2007; 147:478-91, Qaseem Ann Int Med 2017; 166:514-530

3490
Copyright © Harvard Medical School, 2018. All Rights Reserved.

Question 15

A 44 yo colleague sees you for left shoulder pain. She is


an avid tennis player and has had pain in the left deltoid
area for the past 3 weeks. The pain is worse with
movements and has prevented her from playing tennis.
Exam:
Subacromial tenderness. The AC joint is nontender.
Full passive ROM in the glenohumeral joint. Resisted
shoulder abduction causes pain, there is no weakness.
Resisted forearm flexion is unremarkable.
X-ray: normal

What is the diagnosis?

A. supraspinatus tendonitis
B. rotator cuff tear
C. bicipital tendonitis
D. calcific tendonitis
E. AC joint sprain

3491
Copyright © Harvard Medical School, 2018. All Rights Reserved.

What is the diagnosis?

A. supraspinatus tendonitis
B. rotator cuff tear
C. bicipital tendonitis
D. calcific tendonitis
E. AC joint sprain

cervical radiculopathy

AC joint calcific tendonitis


pathology frozen shoulder
rotator cuff impingement syndrome
(supraspinatus tendonitis, subacromial bursitis)
supraspinatus tear
OA of the glenohumeral joint
crystal-induced arthritis

bicipital tendonitis
referred pain
(heart,
gall bladder)

Beach NEJM 2016; 375:e24

3492
Copyright © Harvard Medical School, 2018. All Rights Reserved.

Question 16

A 42 yo female with long standing asthma maintained on


oral corticosteroids for frequent flares complains about a
new problem. She is having increasing difficulties
climbing stairs, getting up from a chair has become
difficult. She denies upper extremity weakness, joint pain,
muscle pain, stiffness. Her breathing is at baseline.
Exam: moon face, abdominal striae, unable to get up
from chair without using her hands
Labs: CBC, CPK, TSH, ESR are WNL
You suspect she has corticosteroid-induced myopathy.

Which statement is correct regarding


corticosteroid-induced myopathy?

A. muscle pain and stiffness are typical


B. serum CPK may be raised 2-3 times normal
C. patients often describe difficulty rising from chairs
D. shoulder girdle symptoms are generally greater than
hip girdle symptoms
E. the development of this condition is independent of
the corticosteroid dose

3493
Copyright © Harvard Medical School, 2018. All Rights Reserved.

Which statement is correct regarding


corticosteroid-induced myopathy?

A. muscle pain and stiffness are typical


B. serum CPK may be raised 2-3 times normal
C. patients often describe difficulty rising from chairs
D. shoulder girdle symptoms are generally greater than
hip girdle symptoms
E. the development of this condition is independent of
the corticosteroid dose

Classification of myopathies associated


with systemic disorders

• Endocrine myopathies
• Inflammatory myopathies
• Paraneoplastic myopathy
• Myopathy from infectious disease
• Drug- and toxin-induced myopathies
• Critical illness myopathy
• Metabolic myopathies

Chawla Front Neurol 2011; 2:49

3494
Copyright © Harvard Medical School, 2018. All Rights Reserved.

Corticosteroid myopathy

• Can develop an any time following the initiation of


corticosteroid therapy.
• More common with higher doses of corticosteroids and
longer term use of the drug.
• Commonest complaint is difficulty rising from chairs.
Patients note weakness (typically LE > UE ).
No pain or stiffness. Serum CPK is not elevated.

Chawla Front Neurol 2011; 2:49

Question 17

A 25 yo female presents with episodic painful color


changes in her hands. This started after she moved to
Boston three years ago to attend graduate school.
Episodes are triggered by cold exposure. Typically, her
fingers turn white and later blue.
She has no relevant past medical history and takes no
medications except for birth control. She feels chronically
fatigued. ROS is otherwise unremarkable (no joint pain or
swelling, rashes, photosensitivity, dry eyes or mouth,
mucosal lesions, SOB, weakness).
Physical examination is unremarkable.

3495
Copyright © Harvard Medical School, 2018. All Rights Reserved.

What is the next step in evaluating this


patient?

A. perform nailfold capillary microscopy


B. check ANA
C. check antiphospholipid antibodies
D. check scleroderma antibodies (Scl-70, anti-
centromer)
E. no additional tests needed

What is the next step in evaluating this


patient?

A. perform nailfold capillary microscopy


B. check ANA
C. check antiphospholipid antibodies
D. check scleroderma antibodies (Scl-70, anti-
centromer)
E. no additional tests needed

3496
Copyright © Harvard Medical School, 2018. All Rights Reserved.

normal

pathological

Wigley NEJM 2016; 375:556-65

Question 18: Which of the following is not included in


the classification criteria for anti-phospholipid
syndrome?

A. h/o venous thromboembolism


B. h/o premature delivery due to pre-
eclampsia/eclampsia
C. positive anti-cardiolipin antibody test
D. positive anti-β2 microglobulin antibody test
E. presence of lupus anticoagulant

3497
Copyright © Harvard Medical School, 2018. All Rights Reserved.

Question 18: Which of the following is not included in


the classification criteria for anti-phospholipid
syndrome?

A. h/o venous thromboembolism


B. h/o premature delivery due to pre-
eclampsia/eclampsia
C. positive anti-cardiolipin antibody test
D. positive anti-β2 microglobulin antibody test
E. presence of lupus anticoagulant

Classification criteria for definite anti-phospholipid syndrome (APS)

Miyakis J Thromb Haemost 2006; 4:295-306, Schreiber Nat Rev Dis Primers 2018; 4:17103

3498
Copyright © Harvard Medical School, 2018. All Rights Reserved.

Question 19

A 52 yo man presents with several episodes of


hemoptysis over the past day.
He had been well until 3 weeks ago when he developed
myalgia, arthralgia, epistaxis, and diminished hearing.
One week ago he developed a rash and weakness in his
right hand.
Past medical history is otherwise unremarkable. The
patients takes no medications.

Exam:
T 100.4, BP 152/100, HR 72, RR 24
bilateral conjunctival injection, tender maxillary sinuses
decreased hearing in both sides
diffuse rhonchi
reduced grip strength right hand
palpable purpura over the lower extremities
Labs:
Wbc 12,300
ESR 84
Crea 2.1
UA: 3+ protein, 50 Rbc, 20 Wbc, mixed cellular casts

3499
Copyright © Harvard Medical School, 2018. All Rights Reserved.

Which of the following tests is most likely to


establish the diagnosis?

A. anti-dsDNA
B. cryoglobulins
C. complement levels (C3, C4)
D. ANCA
E. urine electrophoresis

Which of the following tests is most likely to


establish the diagnosis?

A. anti-dsDNA
B. cryoglobulins
C. complement levels (C3, C4)
D. ANCA
E. urine electrophoresis

3500
Copyright © Harvard Medical School, 2018. All Rights Reserved.

ANCA in the Diagnosis of Vasculitis

• useful test, especially when strongly positive,


c-ANCA is more specific than p-ANCA
• indirect immunofluorescence: p-ANCA c-ANCA
ELISA: anti-MPO anti-PR3
• ANCA-associated vasculitides:
- GPA (Wegener’s): c-ANCA, rarely p-ANCA
- MPA (Microscopic polyangiitis): p-ANCA
- EGPA (Churg-Strauss): c-ANCA or p-ANCA

Jennette Arthritis Rheum 2013; 65:1-11

3501
Copyright © Harvard Medical School, 2018. All Rights Reserved.

Question 20

An 84 yo male without significant PMH presents with


fatigue and malaise for 6 weeks. He tires easily and has
difficulties completing routine daily activities. He reports a
4 lb weight loss. ROS is negative for fever, night sweats,
SOB, chest pain, changes in urinary or bowel habits. He
denies joint pain, stiffness, headaches, visual complaints.
Exam:
Tired looking but otherwise unremarkable.
Labs:
Wbc 5.0, Hct 28.4, MCV 90, Plt 186
ESR 120

What would you do next?

A. biopsy of a single temporal artery


B. biopsy of both temporal arteries
C. trial of Prednisone 60 mg daily
D. trial of Prednisone 15 mg daily
E. serum protein electrophoresis

3502
Copyright © Harvard Medical School, 2018. All Rights Reserved.

What would you do next?

A. biopsy of a single temporal artery


B. biopsy of both temporal arteries
C. trial of Prednisone 60 mg daily
D. trial of Prednisone 15 mg daily
E. serum protein electrophoresis

• pertinent information
- normocytic anemia and high ESR
- fatigue, not stiffness
- no headache or visual changes
- normal exam
• What are the features of PMR/GCA?
- age >50
- shoulder and hip girdle stiffness + pain
- cranial/visual symptoms in GCA
- fatigue is not the primary issue
• need to r/o dysproteinemia, myeloma
no role for empirical corticosteroids in this case

Weyand NEJM 2014; 371:50-7

3503
Copyright © Harvard Medical School, 2018. All Rights Reserved.

Question 21: The Anti-Ro antibody is


associated with all of the following
EXCEPT:

A. Sjogren’s syndrome
B. neonatal lupus
C. subacute cutaneous lupus
D. diffuse proliferative glomerulonephritis
E. congenital heart block

Question 21: The Anti-Ro antibody is


associated with all of the following
EXCEPT:

A. Sjogren’s syndrome
B. neonatal lupus
C. subacute cutaneous lupus
D. diffuse proliferative glomerulonephritis
E. congenital heart block

3504
Copyright © Harvard Medical School, 2018. All Rights Reserved.

Anti-Ro/SSA antibodies are associated


with:

• Sjogren’s syndrome
• congenital heart block
• other forms of neonatal lupus
e.g. cutaneous lesions and thrombocytopenia
• subacute cutaneous lupus
• SLE with glomerulonephritis is not typically associated
with this antibody!

Yoshimi Clin Dev Immunol 2012: 606195

Question 22

A 69 yo woman is admitted to the hospital with increasing


shortness of breath on exertion.
The patient has a long history of Raynaud’s and was told
to have scleroderma several years ago. She has lost 30 lb
over the last year in the setting of dysphagia. Facial
telangiectasia and sclerodactyly are clearly present.
While obtaining the history you consider the differences
between diffuse and limited cutaneous systemic sclerosis.

3505
Copyright © Harvard Medical School, 2018. All Rights Reserved.

Which of the following is a characteristic feature


of limited cutaneous systemic sclerosis?

A. skin involvement of proximal limbs and trunk


B. interstitial lung disease
C. pulmonary hypertension
D. increased risk for scleroderma renal crisis
E. anti-Scl70 (topoisomerase) antibodies

Denton Lancet 2017; 390:1685-99

Which of the following is a characteristic feature


of limited cutaneous systemic sclerosis?

A. skin involvement of proximal limbs and trunk


B. interstitial lung disease
C. pulmonary hypertension
D. increased risk for scleroderma renal crisis
E. anti-Scl70 (topoisomerase) antibodies

Denton Lancet 2017; 390:1685-99

3506
Copyright © Harvard Medical School, 2018. All Rights Reserved.

Question 23

A 23 year old female college student is seen in the ER for


acute bilateral ankle swelling and pain over both shins for
5 days. Symptoms have been getting worse and she has
now difficulties walking. She denies any trauma or recent
travel. No significant PMH.
Exam:
T 100.1, BP 110/65, P 85 reg, RR 18
bilateral swelling of ankles, diffusely tender
bilateral lesions over the tibia, painful to palpation
Labs:
Hct 35, ESR 35

3507
Copyright © Harvard Medical School, 2018. All Rights Reserved.

What test would you order next?

A. chest X-ray
B. skin biopsy
C. ACE level
D. RF
E. urinalysis and ANCA

What test would you order next?

A. chest X-ray
B. skin biopsy
C. ACE level
D. RF
E. urinalysis and ANCA

3508
Copyright © Harvard Medical School, 2018. All Rights Reserved.

Lofgren syndrome
Lofgren Acta Med Scan 1952

• acute presentation of sarcoidosis


- erythema nodosum
- bilateral hilar lymphadenopathy
- ankle arthritis
• complete triade - 95% specific for sarcoidosis
(DD coccidioidomycosis in endemic areas)
• 1st line treatment - NSAIDs

O’Regan Ann Intern Med 2012; 156

Courtesy of Dr Christoph Berliner, Radiopaedia.org, rID: 22067

3509
Copyright © Harvard Medical School, 2018. All Rights Reserved.

Question 24: What do rheumatoid arthritis,


psoriatic arthritis, AS and SLE have in common?

A. largely overlapping genetic risk factors


B. similar prevalence (~1%)
C. more common in women than in men
D. TNF inhibitors are first-line therapy
E. increased risk for cardiovascular disease

Question 24: What do rheumatoid arthritis,


psoriatic arthritis, AS and SLE have in common?

A. largely overlapping genetic risk factors


B. similar prevalence (~1%)
C. more common in women than in men
D. TNF inhibitors are first-line therapy
E. increased risk for cardiovascular disease

3510
Copyright © Harvard Medical School, 2018. All Rights Reserved.

Patients with inflammatory rheumatic diseases


have an increased risk for cardiovascular
disease

Nurmohamed Nat Rev Rheumatol 2015; 11:693-704

Questions?

3511
Copyright © Harvard Medical School, 2018. All Rights Reserved.

Board Review Practice


Images
Part 2
Ajay K. Singh, MBBS, FRCP
Brigham and Women’s Hospital
Harvard Medical School

Disclosure
• GSK
Consultant

3512
Copyright © Harvard Medical School, 2018. All Rights Reserved.

Question 14
This patient pulled out a tick
from her skin 30 minutes after
a trek in a conservation area in
Barnstable in Cape Cod.. What
is the most likely diagnosis?

A. Bed bug bite


B. Deer tick bite
C. Kissing bugs
D. Deadly Brown Recluse
Spiders
E. Flea bite

http://lymediseaseguide.org/lyme-disease-rash

Question 14
This patient pulled out a tick
from her skin 30 minutes after
a trek in a conservation area in
Barnstable in Cape Cod.. What
is the most likely diagnosis?

A. Bed bug bite


B. Deer tick bite
C. Kissing bugs
D. Deadly Brown Recluse
Spiders
E. Flea bite

http://lymediseaseguide.org/lyme-disease-rash

3513
Copyright © Harvard Medical School, 2018. All Rights Reserved.

Deadly Brown Recluse Spiders


Fleas are insects that form the order
Siphonaptera. They are wingless, with
mouthparts adapted for piercing skin and
cimicids or, loosely, bed bugs sucking blood.

Body louse, female Blattodea is an order of insects that contains


the cockroaches and the termites. Formerly

Deer Tick

https://en.wikipedia.org/wiki/Lyme_disease

3514
Copyright © Harvard Medical School, 2018. All Rights Reserved.

http://cid.oxfordjournals.org/content/43/9/1089.f
ull

Lyme Facts
• Lyme disease is the most common tickborne infection in both North
America and Europe.
• In the US - Lyme disease is caused by Borrelia burgdorferi, transmitted
by bite from tick species Ixodes scapularis and Ixodes pacificus.
• Clinical manifestations - commonly: skin, joints, nervous system, and
heart.
• Early cutaneous infection with B. burgdorferi is called erythema migrans
-most common clinical manifestation of Lyme disease.
• I. scapularis may also be infected with and transmit Anaplasma
phagocytophilum (previously referred to as Ehrlichia phagocytophila)
and/or Babesia microti, the primary cause of babesiosis.
Abite from an I. scapularis tick may lead to the development of Lyme
disease, human granulocytic anaplasmosis (HGA, formerly known as
human granulocytic ehrlichiosis), or babesiosis as a single infection or, less
frequently, as a coinfection.
HGA and babesiosis should be included in the differential diagnosis of
patients who develop fever after an Ixodes tick bite in an area where these
infections are endemic

3515
Copyright © Harvard Medical School, 2018. All Rights Reserved.

Treatment of Early Lyme


• Early localized or early disseminated Lyme disease
associated with erythema migrans (in the absence of
specific neurologic manifestations or advanced atrioven-
tricular heart block (A-I))
Doxycycline (100 mg twice per day) x14 d
Amoxicillin (500 mg 3 times per day) x14-21
Cefuroxime axetil (500 mg twice per day) 14–21 days l
• Doxycycline has the advantage of being effective for
treatment of HGA (but not for babesiosis), which may
occur simultaneously with early Lyme disease.
• Doxycycline is relatively contraindicated during
pregnancy or lactation and in children !8 years of age.

http://lymediseaseguide.org

3516
Copyright © Harvard Medical School, 2018. All Rights Reserved.

Question 15
A 62 year old woman on dialysis undergoes a total
parathyroidectomy for tertiary hyperparathyroidism
(PTH 1200 pg/mL). Post-operatively (≈6 hrs post op)
she complains of tingling around her lips and around the
mouth. Approximately 9 hours later she has carpopedal
tetany. Both Trousseau’s and Chvostek’s signs are
positive. An ECG is done. Which one of the following is
most consistent with her likely serum calcium level

A B C

3517
Copyright © Harvard Medical School, 2018. All Rights Reserved.

Question
• A
• B
• C

✔A B C

SOURCE:
http://www.angelfire.com/un/al6a/presentation/REsearch/electrolyte_and_metabolic_abnorm.
htm

3518
Copyright © Harvard Medical School, 2018. All Rights Reserved.

ECG Changes in Hypo-and Hypercalcemia

• Hypercalcemia and hypocalcemia predominantly alter the


action potential duration.
• Hypercalcemia shortens the ventricular action potential
duration by shortening phase 2 of the action potential.
Hypocalcemia prolongs phase 2 of the action potential.
• Hypercalcemia – shorter QT interval (ST segment portion)
• Hypocalcemia – longer QT interval
• Severe hypercalcemia (e.g., serum Ca2+ ³15 mg/dl) can also
be associated with decreased T wave amplitude, sometimes
with T wave notching or inversion.
• Hypercalcemia sometimes produces a high takeoff of the ST
segment in leads V1 and V2 and can thus simulate acute
ischemia.
• Hypocalcemia generally does not cause T-wave changes
because it does not affect phase 3 of the action potential.
RuDusky, BM Chest Journal 2001

Question 16

What is the diagnosis?

A.Lead toxicity
B.Copper toxicity
C.Zinc toxicity
D.Mercury toxicity

3519
Copyright © Harvard Medical School, 2018. All Rights Reserved.

https://en.wikipedia.org/wiki/Wilson%27s_disease

What is the diagnosis?

A.Lead toxicity
Copper toxicity
A.Zinc toxicity
B.Mercury toxicity

Wilson’s Disease
• Wilson's disease, also called Wilson disease or hepatolenticular
degeneration
• Autosomal recessive disease, mutation in Wilson’s disease protein
gene (ATP7B), accumulation of copper in tissues – accumulation
in liver, brain.
• Also eyes (Kayser-Fleischer rins – copper deposition in
Descemet’s membrane); kidneys (type 2 RTA), heart
(cardiomyopathy)
• Named after Samuel Wilson a British neurologist who first
described the condition in 1912
• A single abnormal copy of the gene is present in 1 in 100 people,
who do not develop any symptoms (carrier)
• If a child inherits the gene from both parents, the child may
develop Wilson's disease. Symptoms usually appear between the
ages of 6 and 20 years, but cases in much older people have been
described. Wilson's disease occurs in 1 to 4 per 100,000 people

3520
Copyright © Harvard Medical School, 2018. All Rights Reserved.

SOURCE: http://www.eurowilson.org/en/living/guide/what/index.phtml

Question 17
A 48 year old white male with chronic kidney disease
(CKD) with a serum creatinine of 1.5 mg/dL presents to
you complaining of weakness and fatigue and an
episode of palpitations. The patient is on furosemide 40
mg BID, lisinopril 20 mg QD, atorvostatin 20 mg QD and
aspirin 81 mg QD. You do an ECG and draw labs.
Based on the ECG, what is the most likely diagnosis?
A.) Patient has hypomagnesemia, Mg 1.4 mg/dL
B.) Patient has hypokalemia, K 2.0 mEq/L
C.) Patient has hyperkalemia, K 6.8 mEq/L
D.) Patient has hypocalcemia, Ca 7.1 mg/dL

3521
Copyright © Harvard Medical School, 2018. All Rights Reserved.

https://cardiologyboardreview.wordpress.com/page/2/

Question 17
A 48 year old white male with chronic kidney disease
(CKD) with a serum creatinine of 1.5 mg/dL presents to
you complaining of weakness and fatigue and an
episode of palpitations. The patient is on furosemide 40
mg BID, lisinopril 20 mg QD, atorvostatin 20 mg QD and
aspirin 81 mg QD. You do an ECG and draw labs.
Based on the ECG, what is the most likely diagnosis?
A.) Patient has hypomagnesemia, Mg 1.4 mg/dL
B.) Patient has hypokalemia, K 2.0 mEq/L
C.) Patient has hyperkalemia, K 6.8 mEq/L
D.) Patient has hypocalcemia, Ca 7.1 mg/dL

3522
Copyright © Harvard Medical School, 2018. All Rights Reserved.

SOURCE:
https://fluidandelectrolyteimbalances.wordpress.com/2013/04/19/potassi
um/

Hypokalemia
• PR interval may be prolonged
• Flat T wave
• Prominent U wave
• ST depression
• QT prolongation, but often difficult to
measure as T wave flattens

3523
Copyright © Harvard Medical School, 2018. All Rights Reserved.

Question 18
A 55-year-old man presented with a 2-
year history of painful jaw enlargement
and progressively ill-fitting dentures.
The serum level of alkaline
phosphatase and bone-specific alkaline
phosphatase were elevated (154 IU per
liter [normal level, <120] and 92 IU per
liter [normal range, 15 to 41],
respectively). He has an elevated
alkaline phosphatase and a normal
Q: serum creatinine. Which one of the
following tests would confirm the
diagnosis?
A. Bone scan
B. Insulin-like growth factor-1 level
C.Serum calcium
D.Abdominal ultrasound
E.Testing the function of the facial nerve
SOURCE: Patel MB, et al N Engl J Med 2008; 358:625

A. Bone scan
Q:
Paget's disease, acromegaly, and
renal osteodystrophy are among
the causes of jaw enlargement,
Answer: visible in this image. An elevated
alkaline phosphatase makes
Paget's disease the most likely
diagnosis; the diagnosis can be
confirmed with a bone scan and a
mandibular biopsy. Treatment with
a bisphosphonate normalized the
serum level of alkaline
phosphatase. Earlier diagnosis
and treatment might have limited
further mandibular hypertrophy
and pain

3524
Copyright © Harvard Medical School, 2018. All Rights Reserved.

Paget’s Disease of the Bone


• Chronic disorder that can result in enlarged and misshapen
bones; named after Sir James Paget; affects 1.5-8% of
population; presentation >age 55
• Caused by the excessive breakdown and formation of bone,
followed by disorganized bone remodeling.
• Affected bones weaken, resulting in pain, misshapen bones,
fractures and arthritis in the joints near the affected bones.
Rarely, can develop into a primary bone cancer known as
Paget's sarcoma.
• Often Paget's disease is localized to only a few bones in the
body. The pelvis, femur, and lower lumbar vertebrae are the
most commonly affected bones.
• Early diagnosis and treatment is important, after age 40,
siblings and children of someone with Paget's disease should
have an alkaline phosphatase blood test every two or three
years.
https://en.wikipedia.org/wiki/Paget%27s_disease_of_bone

Question 19
A 30 year patient returns with
fever after a visit trekking in
several sub-saharan countries.
He has high fever (39.5oC), low
normal blood pressure and
lethargy. He complains of a stiff
neck and photophobia. The
rash began 2 days before the
visit to the ED and 8 days after
his return.
The most likely diagnosis is:
A.) Erythema multiforme
B.) Meningococal meningitis
C.)Henoch Schonein purpura
D.) Microscopic polyangitis

3525
Copyright © Harvard Medical School, 2018. All Rights Reserved.

A 30 year patient returns with


fever after a visit trekking in
several sub-saharan countries.
He has high fever (39.5oC), low
normal blood pressure and
lethargy. He complains of a stiff
neck and photophobia. The
rash began 2 days before the
visit to the ED and 8 days after
his return.
The most likely diagnosis is:
A.) Erythema multiforme
B.) Meningococal meningitis
C.)Henoch Schonein purpura
D.) Microscopic polyangitis

Erythema multiforme (EM) is an


acute, self-limited, and sometimes
recurring skin condition that is
considered to be a type IV
hypersensitivity reaction associated
with certain infections, medications,
and other various triggers.
SOURCE: http://emedicine.medscape.com/article/1122915-overview
https://en.wikipedia.org/wiki/Erythema_multiforme#/media/File:Erythema_multiforme_minor_of_the_hand.j
pg
HSP – is a systemic vasculitis with IC’s
depositing with IgA
In the skin, the disease causes
palpable purpura
Also, kidney and joint involvement and
abdominal pain (small bowel vasculitis

Classic Triad: purpura, abdominal pain


and arthritis
SOURCE:
http://www.cfp.ca/content/54/8/1117/F1.expansion

3526
Copyright © Harvard Medical School, 2018. All Rights Reserved.

Microscopic polyangiitis (MPA)


is vasculitis of small vessels:
small vessels, including
arterioles, capillaries, and
venules.
Consitutional features: Fever,
anorexia, weight loss, fatigue

Glomerular involvement: RPGN


SOURCE: Rheum: myalgias, arthralgias
https://o.quizlet.com/tA5nW9BXhJwEPC1fHclg
BA_m.jpg and arthritis
GI: Abdominal pain and GI
bleeding
Lung hemorrhage
Peripheral neuropathy

Meninogococcal Meningitis
• Gram-negative diplococcus Neisseria meningitidis
• Presents with intense headache, fever, N/V, Photophoba, stiff neck,
lethargy, altered mental state, rash
• Meningococcal septicemia - rapid circulatory collapse and a hemorrhagic
rash, is a more severe, but less common, form of meningococcal disease.
• cerebrospinal fluid (CSF) usually confirms the presence of meningitis.
Typical CSF abnormalities in meningitis include the following:
Increased opening pressure (>180 mm water)
Pleocytosis of polymorphonuclear leukocytes (white blood cell [WBC]
counts between 10 and 10,000 cells/µL, predominantly neutrophils)
Decreased glucose concentration (< 45 mg/dL)
Increased protein concentration (>45 mg/dL)
• Culture of CSF and blood specimens - To identify N meningitidis and the
serogroup of meningococci, as well as to determine the bacterium’s
susceptibility to antibiotics
• Polymerase chain reaction (PCR) assay - For confirmation of the diagnosis

http://emedicine.medscape.com/article/1165557
-overview

3527
Copyright © Harvard Medical School, 2018. All Rights Reserved.

Meningococcal belt

SOURCE: Aguado T, et al Nature Reviews Microbiology 3, 10-11 (January


2005)

Question 20
A 19-year-old man presented with a 10-month
history of Raynaud's phenomenon, fever,
abdominal pain, and hypertension. On
examination, he had multiple subcutaneous
nodules on his forehead, and his blood
pressure was 150/100 mm Hg. Laboratory
findings included a normal urine sediment, an
elevated erythrocyte sedimentation rate, mild
anemia, and leukocytosis, with negative tests
for antineutrophil cytoplasmic autoantibody,
hepatitis B surface antigen, and hepatitis C
antibody. What diagnosis is suggested by the
findings on his angiogram?

A. Takayasu's arteritis
B. Wegener's granulomatosis
C. Paraganglioma
D. Systemic lupus erythematosus
E. Polyarteritis nodosa
SOURCE: Das CJ , et al .N Engl J Med 2006; 355:2574

3528
Copyright © Harvard Medical School, 2018. All Rights Reserved.

E. Polyarteritis nodosa

The angiogram reveals multiple


microaneurysms involving the
parenchymal branches of the hepatic
artery, splenic artery, renal artery. Biopsy
of a subcutaneous nodule revealed
necrotizing inflammation of medium-
sized arteries, and vasculitis in renal
biopsy specimens — findings consistent
with a diagnosis of polyarteritis nodosa.
The patient was treated with
corticosteroids, resulting in the resolution
of his symptoms, including the skin
nodules, fever, and abdominal pain.
Multiple aneurysmal dilatations up to 1
cm in diameter, involving the bifurcations
and branches of small- and medium-size
arteries, are characteristic of polyarteritis
nodosa.

Polyarteritis nodosa
• Classic polyarteritis nodosa (PAN or c-PAN) - systemic
vasculitis characterized by necrotizing inflammatory lesions
that affect medium-sized and small muscular arteries,
preferentially at vessel bifurcations, resulting in
microaneurysm formation, aneurysmal rupture with
hemorrhage, thrombosis, and, consequently, organ ischemia
or infarction.
• Kussmaul and Maier first described PAN in 1866.
• Affects skin (see the image below), joints, peripheral nerves,
the gut, and the kidney. affects skin joints, peripheral nerves,
the gut, and the kidney

SOURCE: http://emedicine.medscape.com/article/330717-overview

3529
Copyright © Harvard Medical School, 2018. All Rights Reserved.

Question 21
These lesions were neither
pruritic nor painful. What is
the diagnosis?

A. Pyoderma gangenosus
B. Phlegmasia cerulea dolens
C. Pretibial myxedema
D. Necrobiosis lipoidica
diabeticorum
Q: E. Erythema nodosum

Slide courtesy of Graham McMahon, BWH

Q:
These lesions were neither
pruritic nor painful. What is
the diagnosis?
Answer:

D. Necrobiosis lipoidica
diabeticorum

This patient was diagnosed


with necrobiosis lipoidica
diabeticorum.

3530
Copyright © Harvard Medical School, 2018. All Rights Reserved.

Necrobiosis lipoidica
• Necrobiosis lipoidica is a disorder of collagen degeneration
with a granulomatous response, thickening of blood vessel
walls, and fat deposition.
• The main complication of the disease is ulceration, usually
occurring after trauma. Infections can occur but are
uncommon.
• There have been rare reported cases of squamous cell
carcinomas developing in chronic lesions of necrobiosis
lipoidica.
• The condition was first described in 1929, by Oppehhein, who
called it dermatitis atrophicans lipoidica diabetica; in 1932,
however, the disease was renamed necrobiosis lipoidica
diabeticorum (NLD), by Urbach.
• In 1935, Goldsmith reported the first case in a nondiabetic
patient.
SOURCE: http://emedicine.medscape.com/article/1103467-overview

Question 22

58 year old woman presents with bright red blood per rectum,
fatigue and dyspnea. PMH- recurrent episodes of spontaneous
epistaxis. Mother has similar history. Labs – severe iron deficiency
anemia.
The most likely cause is:
A.) Essential telangiectasia
B.) Osler-Weber-Rendu syndrome
C.) Scleroderma
D.) Neurofibromatosis

3531
Copyright © Harvard Medical School, 2018. All Rights Reserved.

58 year old woman presents with bright red blood per rectum,
fatigue and dyspnea. PMH- recurrent episodes of spontaneous
epistaxis. Mother has similar history. Labs – severe iron deficiency
anemia.
The most likely cause is:
A.) Essential telangiectasia
B.) Osler-Weber-Rendu syndrome
C.) Scleroderma SOURCE: Perez-Belmonte LM, et al, NEJM N Engl J
Med 2015; 373:e15
D.) Neurofibromatosis http://www.nejm.org/doi/pdf/10.1056/NEJMicm1414035

Osler-Weber-Rendu Syndrome

• Osler-Weber-Rendu disease (OWRD) - rare autosomal dominant disorder that


affects blood vessels throughout the body (causing vascular dysplasia) and results in
a tendency for bleeding. (also known as hereditary hemorrhagic telangiectasia [HHT]
• HHT is manifested by mucocutaneous telangiectases and arteriovenous
malformations
• Lesions can affect the nasopharynx, central nervous system (CNS), lung, liver, and
spleen, as well as the urinary tract, gastrointestinal (GI) tract, conjunctiva, trunk,
arms, and fingers
• Recurrent and severe epistaxis is the most common presentation, frequently leading
to severe anemia that necessitates transfusion, GI bleeding is also prevalent.
• diagnosis of HHT is made clinically on the basis of the Curaçao criteria (1999):
Epistaxis
• Telangiectasias
• Visceral lesions
• Family history (a first-degree relative with HHT)

3532
Copyright © Harvard Medical School, 2018. All Rights Reserved.

Question 23

25-year-old man with human immunodeficiency virus (HIV) infec-


tion who was receiving highly active antiretroviral therapy presented with a
1-week history of diplopia and headache. The CD4 count was 218 cells per
Q:
cubic millimeter, and the viral load was 50,000 copies per milliliter. The
neurologic examination revealed an inability to abduct the right eye with
horizontal gaze: What is the likely diagnosis?
A. Internuclear ophthalmoplegia
B. Left fourth cranial nerve palsy
C. Left sixth cranial nerve palsy
D. Right fourth cranial nerve palsy
E. Right sixth cranial nerve palsy

SOURCE: Dhillon, WS et al N Engl J Med 2010; 362:e52

E. Right sixth cranial nerve palsy


(VIth cranial nerve palsy or Abducens nerve palsy)

The neurologic examination reveals an inability to abduct the


right eye with horizontal gaze to the right, a finding that is
consistent with an isolated right abducens nerve palsy.

Causes: Vasculopathi (HTN, DM), trauma, idiopathic


cavernous sinus mass, raised intracranial pressure, giant cell
arteritis

3533
Copyright © Harvard Medical School, 2018. All Rights Reserved.

Question 24
This 41 year old patient
with a history of alcohol
abuse developed alopecia
with fine, brittle scalp
hair, diarrhea, and angular
cheilitis. Measurement of
which one of the
following metals is most
Q: likely to be diagnostic?

A. Chromium
B. Copper
C. Manganese
D. Selenium
E. Zinc

Slide courtesy of Graham McMahon, BWH

E. Zinc

This scaly erythematous


eruption that preferentially
Answer:
involved the distal
extremities and the
perineum is most
consistent with
acrodermatitis
enteropathica, confirmed
by measurement of the
Acquired Acrodermatitis Enteropathica zinc level. The patient's
symptoms resolved after
zinc supplementation.

• SOURCE: Wang LC et al N Engl J Med 2005;


352:1121http://www.nejm.org/doi/full/10.1056/NEJMicm030844?vie
wType=Print&viewClass=Print

3534
Copyright © Harvard Medical School, 2018. All Rights Reserved.

Question 25
A 19-year-old woman presented to
the emergency department with a
10-day history of intermittent
odynophagia, voice changes, and
fever. Before her visit to the
emergency department, she was
treated with azithromycin and
prednisone for pharyngitis and,
subsequently, with 2 days of
penicillin and a tapered dose of
prednisone. She was otherwise
Q: healthy. Examination revealed
bilateral swelling of the soft palate
with a midline uvula pushed
anteriorly What is the diagnosis?
A. Ludwig's angina
B. Glossopharyngeal nerve palsy
C. Pharyngeal gonorrhea
D. Bilateral peritonsillar
abscesses
E. Infectious mononucleosis

D. Bilateral peritonsillar
Q: What is the diagnosis? abscesses

Bilateral swelling of the


soft palate is visible with a
Answer:
midline uvula pushed
anteriorly. A CT of the
neck after the
administration of
intravenous contrast
material showed bilateral
peritonsillar abscesses
This is most consistent
with a diagnosis of
bilateral peritonsillar
abscesses.

SOURCE: Fiechtl, JF et al N Engl J Med 2008; 358:e27


http://www.nejm.org/doi/pdf/10.1056/NEJMicm072980

3535
Copyright © Harvard Medical School, 2018. All Rights Reserved.

Hospital Medicine:
What’s New in the Literature

Psychiatry Overview Christopher Roy, MD


Director of Hospital Medicine
Division of General Medicine, Department of Medicine
Brigham and Women’s Hospital
Assistant Professor of Medicine
Harvard Medical School

I have no disclosures

3536
Copyright © Harvard Medical School, 2018. All Rights Reserved.

Case 1
68 year-old female smoker admitted with cough
and purulent sputum, wheezing, and dyspnea
consistent with an acute exacerbation of COPD.
She is afebrile and chest radiograph is normal.

On hospital day #3, she has stabilized and is off


oxygen, remains afebrile. Your medical student
tells you that her procalcitonin (Vidas assay) is
0.15 mcg/L.

Case 1
You should:

a. Continue azithromycin
b. Repeat blood cultures
c. Add ceftriaxone IV
d. Stop azithromycin and discharge home
e. Ask the medical student to prepare a
presentation on procalcitonin for the team

3537
Copyright © Harvard Medical School, 2018. All Rights Reserved.

Case 1
You should:

a. Continue azithromycin
b. Repeat blood cultures
c. Add ceftriaxone IV
d. Stop azithromycin and discharge home
e. Ask the medical student to prepare a
presentation on procalcitonin for the team

Procalcitonin: Overview
• Biomarker of bacterial infection
• Shown to help safely curb antibiotic utilization in
ICU patients with suspected or documented
infection/sepsis, and respiratory tract infections
• FDA approved for:
– Sepsis diagnostic aid (2006)
– Assessment of 28-day sepsis mortality (2016)
– NEW: Guide to antibiotic therapy initiation /
discontinuation in respiratory infections, and
discontinuation in sepsis (2017) (Vidas assay)

3538
Copyright © Harvard Medical School, 2018. All Rights Reserved.

Procalcitonin and Acute Respiratory


Infection
• Cochrane review/metaanalysis of 32 RCTs,
6708 patients who had acute respiratory
infection, randomized to PCT guided therapy
vs usual care
• Mortality was lower in the PCT group (8.6% vs
10%) and antibiotic exposure was 2.4 days
lower. Treatment failure was similar
• Bottom line: PCT algorithms might be useful in
ARI to reduce abx usage and mortality
Cochrane Database of Systematic Reviews,
Published online October 12, 2017

Suggested Algorithm for PCT Use

3539
Copyright © Harvard Medical School, 2018. All Rights Reserved.

Cautions with Procalcitonin


• False positives: inflammation: major
trauma/burns/surgery/pancreatitis
• False negatives: early sepsis or contained
infection, repeat within 6-24 hrs if abx withheld
• Severely immunocompromised patients were
excluded, as were patients with cystic fibrosis or
pregnancy
• Should never replace clinical judgment (clinical
judgment override in sepsis trials used >50%)

Newer trials suggesting PCT not so


great…
• AE COPD study from France
• ProACT study NEJM May 2018

3540
Copyright © Harvard Medical School, 2018. All Rights Reserved.

COPD and Procalcitonin


• Antibiotics are indicated for most acute
exacerbations of COPD requiring
hospitalization
• Triggers of AE COPD are often not bacterial
• Could PCT be used to distinguish bacterial
trigger from viral, environmental etc and
decrease antibiotic utilization?

www.goldcopd.com

AE COPD and Procalcitonin


• RCT of 302 patients admitted to ICU with AE
COPD in France
• PCT guided antibiotic rx vs. usual care
• Failed to safely reduce antibiotic use
• Initiation of antibiotics improved 3 month
survival regardless of PCT level
• Bottom line: don’t use PCT algorithm to
reduce abx for AE COPD

Intensive Care Med (2018) 44:428–437

3541
Copyright © Harvard Medical School, 2018. All Rights Reserved.

• Use of PCT guided rx algorithm for suspected lower resp


infection did not result in less use of antibiotics vs usual care
• Included 1656 pts with final dx of COPD flare, asthma, acute
bronchitis, pneumonia presenting to ED
• “Real world” study, PCT algorithm did not beat usual
care/clinical gestalt Published online NEJM May 20, 2018

Case 2
78 year-old male assisted living resident is
admitted with pneumonia. At baseline he uses
a walker to ambulate. You begin his admission
orders and consider choices for his activity
orders:
a. Ambulate with assistance
b. Fall precautions
c. OOB as tolerated
d. Bedrest
e. PT evaluation

3542
Copyright © Harvard Medical School, 2018. All Rights Reserved.

Case 2
78 year-old male assisted living resident is
admitted with pneumonia. At baseline he uses
a walker to ambulate. You begin his admission
orders and consider choices for his activity
orders:
a. Ambulate with assistance
b. Fall precautions
c. OOB as tolerated
d. Bedrest
e. PT evaluation

Case 2
• Hospitalized elders
are OOB only 4% of
waking hours with
usual care
• A lack of early
mobilization to
contribute to
functional decline
and frailty,
prolonged delirium,
longer LOS, lower
likelihood of
discharge to home
• Structured program
to increase early
mobilization could
reduce these risks

Creditor et al Ann Int Med 1993;


118:219-23

3543
Copyright © Harvard Medical School, 2018. All Rights Reserved.

Let’s MOVE ON
• 10,000 elderly patients admitted
to 14 acute care hospitals in
Ontario
• Quasi experimental time series
approach with visual audits to
assess mobilization rates
• No added resources,
implemented with existing care
team
– Assessed for early mobilization
within 24 hours of admission
– Mobilized 3 times a day
– Progressive and scaled, tailored
to patient’s abilities

From: Outcomes of Mobilisation of Vulnerable Elders in Ontario (MOVE ON): a multisite interrupted time
series evaluation of an implementation intervention to increase patient mobilisation
Age Ageing. 2017;47(1):112-119. doi:10.1093/ageing/afx128
Age Ageing | © The Author 2017. Published by Oxford University Press on behalf of the British Geriatrics Society.This is an
Open Access article distributed under the terms of the Creative Commons Attribution Non-Commercial License
(http://creativecommons.org/licenses/by-nc/4.0/), which permits non-commercial re-use, distribution, and reproduction in any
medium, provided the original work is properly cited. For commercial re-use, please contact journals.permissions@oup.com

3544
Copyright © Harvard Medical School, 2018. All Rights Reserved.

MOVE ON
• 10% increase in patients out of bed
• Significant decrease in median LOS (-6.1 days)
in the intervention and post intervention
phases
• No change in % discharged to home

Case 3
68 year-old female is brought by her husband
to the ED for an episode of syncope. She has a
history of HTN and hyperlipidemia. Syncope
was sudden in onset and there was no
prodrome. She awoke within seconds and was
alert, denied chest pain, dyspnea, or
palpitations. Physical exam including VS, O2
saturation, and 12 lead ECG were normal.

3545
Copyright © Harvard Medical School, 2018. All Rights Reserved.

Case 3
• In addition to cardiac monitoring and
echocardiogram, you should consider:
a. EEG
b. Head CT
c. D-dimer
d. CTA of the coronary arteries
e. Carotid non-invasive studies
f. None of the above

Case 3
• In addition to cardiac monitoring and
echocardiogram, you should consider:
a. EEG
b. Head CT
c. D-dimer
d. CTA of the coronary arteries
e. Carotid non-invasive studies
f. None of the above

3546
Copyright © Harvard Medical School, 2018. All Rights Reserved.

Pulmonary Embolism in
Patients with Syncope
• PESIT study
• 11 hospitals in Italy
• 560 pts with first episode of
syncope
• PE ruled out in 59% with
low prob Wells score and
neg d-dimer
• Prevalence of PE in
remaining pts was 42.2%, in
the entire cohort it was
17.3% (about 1 in 6)
• But higher risk cohort than
syncope pts admitted in
U.S.?
n engl j med 375;16 October 20, 2016

But is the prevalence of PE in patients


with syncope really that high??
• Retrospective observational study of 5 databases in
4 different countries over 6.75 years
• 1,671,944 patients with ED visit for syncope
• Prevalence of PE at ED or hospital discharge and
within 90 days
• Prevalance of PE only 0.06% for all patients and
0.15% for hospitalized patients, and only 0.14-
0.35% at 90 days
• Conclusion: PE was rarely identified, and not all
patients need evaluation for PE
JAMA Int Med. Published online Jan 29, 2018

3547
Copyright © Harvard Medical School, 2018. All Rights Reserved.

Case 4
55 year old female presents with fevers, flank
pain, nausea, vomiting, and confusion. Exam is
notable for T 102.5, HR 130, BP 75/50 R 32.
There is R flank tenderness and she is lethargic
and confused. Urinalysis shows >100 WBC and
4+ bacteria.

Case 4
In addition to starting aggressive volume
resuscitation and IV antibiotics for presumed
urosepsis, which of the following is indicated:
a. Vitamin D, thiamine, and
hydrocortisone
b. Normal saline instead of lactated
ringers for resuscitation
c. Hydrocortisone plus fludrocortisone

3548
Copyright © Harvard Medical School, 2018. All Rights Reserved.

Case 4
In addition to starting aggressive volume
resuscitation and IV antibiotics for presumed
urosepsis, which of the following is indicated:
a. Vitamin D, thiamine, and
hydrocortisone
b. Normal saline instead of lactated
ringers for resuscitation
c. Hydrocortisone plus fludrocortisone

Corticosteroids in Sepsis:
the Final Word?
• Over the past 55 years (!) there have been multiple small
studies and metaanalyses of CS in sepsis, with
equivocal/disparate findings
• 2 very large studies this year, probably will be the last word on
this
• Australian study (ADRENAL) 3,658 pts, (1/3 surgical), with abd
infection or pneumonia as source, received cont infusion of
HC (200 mg/d)
• French study (APROCCHSS) of 1,241 pts (80% medical) most
with pneumonia, rec’d HC 50mg q6h + fludrocortisone 50 mcg
qd Venkatesh et al. NEJM. 2018; 378:797-808.
Annane et al. NEJM. 2018;378:809-18.

3549
Copyright © Harvard Medical School, 2018. All Rights Reserved.

Corticosteroids in Sepsis
• Both studies found that HC shortened the
duration of septic shock (faster resolution and
more pressor-free days)
• The French study showed a reduction in 90d
mortality (43% vs 49.1%, p=0.03) in the HC +
fludrocort arm
• The Australian study showed no 90d mortality
benefit
• Bottom line: treat the sickest pts with HC and
(possibly) fludrocortisone
Venkatesh et al. NEJM. 2018; 378:797-808.
Annane et al. NEJM. 2018;378:809-18.

Face-off: Lactated ringers


vs Normal Saline

NEJM 2018; 378(9)

3550
Copyright © Harvard Medical School, 2018. All Rights Reserved.

Lactated ringers vs normal saline


• In non-critically ill patients presenting to the
ED, use of LR did not increase hospital-free
days but did decrease major adverse kidney
events (4.7% vs 5.6% adj OR 0.82, p=0.01)
• In critically ill patients presenting to the ICU,
use of LR lowered major adverse kidney
events (14.3% vs 15.4%, OR 0.90, p=0.04) and
30d in-hospital mortality (10.3% vs 11.1%,
p=0.06)
NEJM 2018; 378(9)

Vitamin C???

3551
Copyright © Harvard Medical School, 2018. All Rights Reserved.

• 47 pts in single center


• Retrospective
before/after study design
• Cocktail of Vit C 1.5 mg
q6h, HC 50mg q6h,
Thiamine 200mg q12h
• 9% vs 40% mortality (!!!)
• Too good to be true?
CHEST. 2017; 151(6):1229-1238

Case 5
65 yo male is admitted for elective cervical
laminectomy. Perioperatively he receives
prophylactic cefazolin and develops severe
diarrhea on POD#3, WBC 7 26, and fever. C.
diff is sent and positive.

3552
Copyright © Harvard Medical School, 2018. All Rights Reserved.

Case 5
What is the best treatment regimen:

a. Metronidazole 500 mg orally q8h


b. Fidaxomicin 200mg orally bid
c. Vancomycin 500 mg orally q6h
d. IVIG
e. Fecal Microbiome Transplant

Case 5
What is the best treatment regimen:

a. Metronidazole 500 mg orally q8h


b. Fidaxomicin 200mg orally bid
c. Vancomycin 500 mg orally q6h
d. IVIG
e. Fecal Microbiome Transplant

3553
Copyright © Harvard Medical School, 2018. All Rights Reserved.

New 2018 IDSA Guidelines for C diff

• Fidaxomicin
recommended
alternative to
vanco orally
• Metronidazole
downgraded, and
is only weakly
recommended for
mild disease if
fidaxomicin and
vanco unavailable
Clin Infect Dis. 2018 Feb 15

Case 6
You are asked to consult on a 45 year-old male
admitted to the trauma service with pelvic and
rib fractures after a MVA for an elevated TnT. He
developed substernal chest discomfort in the
setting of being transferred to a chair and ECG
was normal but troponin T was 38. The hospital
recently changed to a high sensitivity TnT assay.

3554
Copyright © Harvard Medical School, 2018. All Rights Reserved.

Case 6
What would be your next best diagnostic step?

a. Coronary angiography
b. Dobutamine MIBI
c. CT angiography
d. No further testing required

Case 6
What would be your next best diagnostic step?

a. Coronary angiography
b. Dobutamine MIBI
c. CT angiography
d. No further testing required

3555
Copyright © Harvard Medical School, 2018. All Rights Reserved.

High sensitivity TnT

Hochholzer W, Morrow DA, Giugliano RP, AHJ 2010

Many causes of hsTnT elevation

3556
Copyright © Harvard Medical School, 2018. All Rights Reserved.

Many types of MI
• Type I: Ischemic due
to plaque rupture
(ACS)
• Type 2: Ischemic due
to supply/demand
mismatch
• Type 3: Sudden
cardiac death
• Type 4: Procedure
related (PCI)
• Type 5: CABG related

Even normal people have a number

3557
Copyright © Harvard Medical School, 2018. All Rights Reserved.

Use of HEART Score in Suspected ACS

Partners (BWH/MGH) ED Pathway

3558
Copyright © Harvard Medical School, 2018. All Rights Reserved.

Case 7
45 year old male with DM and HTN admitted
with cholecystitis and has a laparoscopic
cholecystectomy. You are called on POD #2
because his hsTnT (sent because of post-op ECG
changes) is elevated at 38. On exam he is
sedated on fentanyl PCA but denies SOB or CP.
Vital signs have been stable and he did not have
extremes of BP in OR. Rest of exam is normal
and ECG shows non-specific ST-T wave changes
in the lateral leads.

Case 7
Which of the following statements is true?
a) Elevation of hsTnT after non-cardiac surgery has no bearing on
30d mortality
b) Patients with myocardial injury after non-cardiac surgery (MINS)
usually have ischemic symptoms
c) There are no published studies of effective treatment of MINS
d) Elevation of hsTnT has similar prognostic value as elevation of a 4th
generation troponin assay
e) I’ve never heard of MINS so I can’t answer the question

3559
Copyright © Harvard Medical School, 2018. All Rights Reserved.

Case 7
Which of the following statements is true?
a) Elevation of hsTnT after non-cardiac surgery has no bearing on
30d mortality
b) Patients with myocardial injury after non-cardiac surgery (MINS)
usually have ischemic symptoms
c) There are no published studies of effective treatment of MINS
d) Elevation of hsTnT has similar prognostic value as elevation of a
4th generation troponin assay
e) I’ve never heard of MINS so I can’t answer the question

Myocardial injury after non-cardiac


surgery (MINS)
• Elevated postop TnT (4th or 5th generation) is a marker for worse
prognosis and increased mortality after non-cardiac surgery
• MINS includes MI and isolated ischemic troponin elevation that
occur within first 30 days after surgery
• MINS does not include non-ischemic myocardial injury
– sepsis, rapid AF, PE, chronically elevated troponin
• MINS affects ≥8 million adults worldwide annually
• MINS is independently associated with increased risk of CV events
and death over first 2 years after surgery
• Without measuring troponin, most MINS will be missed as most do
not have sx
• ?treatment

3560
Copyright © Harvard Medical School, 2018. All Rights Reserved.

Elevated hsTnT after non-cardiac surgery


predicts increased 30d mortality

And most patients had no ischemic symptoms!


Devereaux JAMA 2017 317(16):1642-51

Dabigatran in
myocardial injury after
noncardiac surgery
Dr. PJ Devereaux on behalf of MANAGE Investigators
Population Health Research Institute, Hamilton, Canada

3561
Copyright © Harvard Medical School, 2018. All Rights Reserved.

MANAGE Trial design


• Multicenter/multinational RCT of 1754 patients with MINS
within 35 days
– randomized to dabigatran 110mg bid or placebo
– Trial stopped early due to loss of funding and slow enrollment
• Partial 2X2 factorial design
– patients not already on PPI
• randomize to omeprazole or placebo
• Primary outcome: major vascular complication (vascular
mortality, nonfatal MI, non-hemorrhagic stroke, peripheral
arterial thrombosis, amputation, and VTE)

Primary efficacy outcome


Outcome Dabigatran Placebo HR P
n=877 n=877 (95% CI) value
no. (%) no. (%)
composite of vascular death
and nonfatal MI, 97 (11) 133 (15) 0.72 0.012
non-hemorrhagic stroke, (0.55-0.93)
peripheral arterial
thrombosis, amputation, and
symptomatic VTE
• There was no significant effect of omeprazole study drug on results of dabigatran primary
efficacy analysis (interaction P=0.79)
• But, study drug was stopped in >40%, and use of ASA and statin was lower than average
• More studies needed

3562
Copyright © Harvard Medical School, 2018. All Rights Reserved.

Case 8
23 year-old female with sickle cell anemia is
admitted with pain crisis. On prior admissions
for the same issue she has been treated
successfully with a high-dose PCA. 5 minutes
after ordering the PCA, pharmacy calls you to
ask if you could switch her to bolus dosing and
maximize non-opioid adjuvants given the
national opioid shortage.

National Shortage of
Injectable Opioids

3563
Copyright © Harvard Medical School, 2018. All Rights Reserved.

Background
July 10, 2017
• As of July 2017, national shortage
of IV administered opioids.
• Medications in short supply
include hydromorphone, fentanyl,
remifentanil, sufentanil,
meperidine, methadone, and
morphine
• All hospitals experiencing the
same shortages, though shortages
may be more severe in some
regions and may vary over time
• Could take over a year for the
supply chain issues to work out

Courtesy of John Fanikos, PharmD

Courtesy of John Fanikos, PharmD

3564
Copyright © Harvard Medical School, 2018. All Rights Reserved.

Courtesy of John Fanikos, PharmD

OPIOID SPARING STRATEGIES

Is my patient on a PCA and able Is my patient on a Is my patient on a opioid


to take oral medications? PCA and NPO? infusion and NPO?
Optimize short acting opioids:* Optimize opioids:* Optimize opioids:*
• D/C PCA and transition to oral opioids (tablet or elixir) • D/C PCA and transition to IV boluses of opioid • Consider discontinuing infusion and transitioning to
• Consider Fentanyl TD IV boluses of opioid
Optimize use of long acting opioids if needed: • Consider Fentanyl TD
• Oxycontin, MS Contin, Fentanyl TD Consider additional adjuvants:**
• If Methadone is considered, consult palliative, postop or chronic pain • Acetaminophen IV Consider additional adjuvants:**
service • Ketorolac • Acetaminophen IV
• Clonidine patch • Ketorolac
Consider additional adjuvants:** • Lidocaine patch • Clonidine patch
• Acetaminophen PO or IV • Lidocaine patch
• Ibuprofen PO or Ketorolac iv or other NSAID Consider creative use of local anesthetics:
• Gabapentin • Can patient have a nerve block? Consider creative use of local anesthetics:
• Clonidine patch • Can patient get an epidural? • Can patient have a nerve block?
• Lidocaine patch • Can patient get an epidural?
Consider opioid sparing IV infusions:
Consider creative use of local anesthetics: (POPS, Chronic, Palliative Pain Service approval Consider opioid sparing IV infusions:
• Can patient have a nerve block? required if out of ICU) (POPS, Chronic, Palliative Pain Service approval
• Can patient get an epidural? • Dexmedetomidine infusion (ICU only) required if out of ICU)
• Lidocaine infusion • Dexmedetomidine infusion (ICU only)
Consider opioid sparing IV infusions: • Ketamine infusion • Lidocaine infusion
(POPS, Chronic, Palliative Pain Service approval required if out of ICU) • Ketamine infusion
• Lidocaine infusion Consider consulting Pain, Palliative Care, or
• Dexmedetomidine infusion (ICU only) Pharmacy Consider consulting Pain, Palliative Care, or Pharmacy
• Ketamine infusion • If pain is difficult to manage • If pain is difficult to manage
• If you need help with conversion to oral opioids • If you need help with conversion to oral opioids
Consider consulting Pain, Palliative Care, or Pharmacy *If your patient is NOT in an acute pain crisis and is able to
• If pain is difficult to manage tolerate POs, IV opioids are NOT available. BWH Department of Anesthesiology, Perioperative and Pain
• If you need help with conversion to oral opioids **Always review contraindications for your patients prior to Medicine, 2018
starting any of these medications. Ver 2018.3 Last update 3/12/2018

Courtesy of John Fanikos, PharmD

3565
Copyright © Harvard Medical School, 2018. All Rights Reserved.

Case 9
82 year-old female with chronic AF, CHF with
EF 25%, HTN, h/o TIA, PVD now admitted to
ortho service for elective THR. You are asked
to comment on management of her
anticoagulation periop. She is on warfarin 2.5
mg daily and her INR is 2.0.

Case 9
With regards to her warfarin anticoagulation, you
should recommend:

a. Stop/reverse warfarin preop and use low molecular


weight heparin or UF heparin bridge pre and post op
b. Stop/reverse warfarin preop and use oral
rivaroxaban as a bridge pre and post op
c. Stop/reverse warfarin and resume postop without a
bridge

3566
Copyright © Harvard Medical School, 2018. All Rights Reserved.

Case 9
With regards to her warfarin anticoagulation, you
should recommend:

a. Stop/reverse warfarin preop and use low molecular


weight heparin or UF heparin bridge pre and post op
b. Stop/reverse warfarin preop and use oral
rivaroxaban as a bridge pre and post op
c. Stop/reverse warfarin and resume postop without a
bridge

Case 9
• CHADS2 score: 5 (CHF, HTN, Age>=75, h/o
TIA)=high risk, 12.5% stroke rate

• CHA2DS2-VASc score: 8 (CHF, HTN, Age>=75,


Vasc dz, TIA, female sex) =high risk=>10% risk
of stroke, 15.2% risk of stroke, TIA, systemic
embolism

3567
Copyright © Harvard Medical School, 2018. All Rights Reserved.

N Engl J Med 2015; 373:823-833

_____

High risk underrepresented

Journal of the American College of Cardiology Jan 2017, 23217;


DOI:10.1016/j.jacc.2016.11.024

3568
Copyright © Harvard Medical School, 2018. All Rights Reserved.

But bridging rarely needed for DOACS…

JACC Jan 2017, 23217; DOI:10.1016/j.jacc.2016.11.024

JACC Jan 2017, 23217;


DOI:10.1016/j.jacc.2016.
11.024

3569
Copyright © Harvard Medical School, 2018. All Rights Reserved.

JACC Jan 2017, 23217;


DOI:10.1016/j.jacc.2016.
11.024

Case 10
63 year-old female with HTN, DM, CHF and
recent DVT/PE on rivaroxaban presents with
rapidly expanding swelling and pain of her R
thigh. On exam she is lethargic, afebrile, HR 130
BP 85/60. Her R thigh is tensely swollen and
ecchymotic and tender. Distal R pulses are
diminished. Hct 26. CT demonstrates a large
anterior compartment hematoma.

3570
Copyright © Harvard Medical School, 2018. All Rights Reserved.

Case 10
In addition to volume resuscitation and transfusion
of pRBC, the best next step would be:
a) Fresh frozen plasma 10-15 ml/kg
b) Idarucizumab 5g IV
c) 4 factor PCC 50 units/kg IV
d) aPCC 50 units/kg IV
e) Andexanet alfa 400 mg IV bolus followed by 480
mg infusion over 2 hours

Case 10
In addition to volume resuscitation and transfusion
of pRBC, the best next step would be:
a) Fresh frozen plasma 10-15 ml/kg
b) Idarucizumab 5g IV
c) 4 factor PCC 50 units/kg IV
d) aPCC 50 units/kg IV
e) Andexanet alfa 400 mg IV bolus followed by 480
mg infusion over 2 hours

3571
Copyright © Harvard Medical School, 2018. All Rights Reserved.

Tomaselli GF. J Am Coll Cardiol.2017 Dec


19;70(24):3042-3067. PMID: 29203195

Tomaselli GF. J Am Coll Cardiol.2017 Dec


19;70(24):3042-3067. PMID: 29203195

3572
Copyright © Harvard Medical School, 2018. All Rights Reserved.

New reversal agents in development


• Andexanet alfa
– Similar structure to Factor Xa and binds Factor Xa
inhibitors
– Being studied currently in ANNEXA-4 trial
– Interim analysis presented at ACC in March
• 132 patients presenting with major bleeding while taking
Factor Xa inhibitor (apixaban, rivaroxaban, edoxaban,
enoxaparin)
• 88% reduction in anti-Factor Xa activity, excellent or very
good hemostasis in 83%
• Safety: at 30d 12% died, 11% had thrombotic event
• Ciraparantag (PER977)
– another direct and indirect Fxa inhibitor binding
molecule, still in very early stages of development.
One dose fully reverses for 24 hrs.

Thank You!

3573
Copyright © Harvard Medical School, 2018. All Rights Reserved.

Emerging Topics for the Boards: Bringing Health Equity


Research into Your Practice

Intensive Review of Internal Medicine


Cheryl R. Clark MD, ScD
Director, Health Equity Research & Intervention
Center for Community Health and Health Equity
Division of General Internal Medicine and Primary Care
Brigham and Women’s Hospital
Assistant Professor of Medicine, Harvard Medical School

Disclosures
• No Disclosures

3574
Copyright © Harvard Medical School, 2018. All Rights Reserved.

Today’s Presentation
• Review frameworks and current literature for
translating health disparities research into practice
– Definitions: differences vs. disparities vs. inequities
– Topic updates:
• Addressing social determinants of health in clinical settings
• Supporting “Cultural Humility” in clinical practice
• Managing implicit bias in clinical care

– Examples of effective clinical interventions in practice

Take Home Points


• Movement toward addressing social factors in
clinical care settings, including new tools and
incentives

• New data to understand needs of diverse groups

• There are strategies we can use to improve


disparities and inequities in clinical settings

3575
Copyright © Harvard Medical School, 2018. All Rights Reserved.

Definitions

Difference vs. Disparities vs. Inequities

Question 1:
Using the National Academy of Medicine (IOM)
framework from Unequal Treatment, which of
the following is a health care disparity?
A. Differences in prescribing adjuvant estrogen therapy
between black and white women
B. Differences in opioid prescribing for chronic pain by race or
ethnicity
C. Differences in patient preferences for skilled nursing facility
use post-hospitalization by race or ethnicity
D. Differences in opioid prescribing for long-bone fracture
by race or ethnicity

3576
Copyright © Harvard Medical School, 2018. All Rights Reserved.

Question 1:
Using the National Academy of Medicine (IOM)
framework from Unequal Treatment, which of
the following is a health care disparity?
A. Differences in prescribing adjuvant estrogen therapy
between black and white women
B. Differences in opioid prescribing for chronic pain by race or
ethnicity
C. Differences in patient preferences for skilled nursing facility
use post-hospitalization by race or ethnicity
D. Differences in opioid prescribing for long-bone fracture
by race or ethnicity

“Disparities” and “Inequities”

3577
Copyright © Harvard Medical School, 2018. All Rights Reserved.

National Academy of Medicine (IOM)


Framework
Disparity:
Differences not
due to clinically
appropriate care
or patient
preferences

Contributors: lack of
interpreters, time
pressure,
payment models,
insurance
coverage

Question 1:
A. Differences in prescribing adjuvant estrogen therapy
between black and white women
*Need more information about tumor data to determine
whether this is clinically appropriate
B. Differences in opioid prescribing for chronic pain by
race or ethnicity
*Overuse of opioid prescribing
C. Differences in patient preferences for skilled nursing
facility use post-hospitalization by race or ethnicity
D. Differences in opioid prescribing for long-bone
fracture by race or ethnicity

3578
Copyright © Harvard Medical School, 2018. All Rights Reserved.

Disparities in Prescribing Opioids to


Control Acute Pain
Table. Percentage of Emergency Department Visits at Which an Opioid was
Prescribed, by Race/Ethnicity, National Hospital Ambulatory Medical Care Survey
(NHAMCS) 1993-2005

Weighted Proportion of Visits at Which an Opioid


Analgesic Was Prescribed, % (95% Confidence Interval)

Asian/ P
Type of Pain No. White Black Hispanic Other Value
Any 156,729 31 (32-32) 23(22-24) 24 (23-26) 28 (26-30) <.001
Long-bone 4348 52 (50-55) 45 (39-50) 51 (44-57) 43 (32-54) 0.02
fracture
Nephrolithiasis 2215 72 (69-75) 56 (44-68) 68 (61-76) 67 (52-82) 0.02
No injury or 161,224 9.9 (9.4-10) 6.6 (6-7) 6.4 (5.8-6.9) 7.2 (6-8) <.001
pain

Source: Pletcher et al. JAMA 2008

What Can We Do to Address Disparities?


• Use population health data
to identify differences and
clinical needs

• Standardize care to
minimize uncertainty

• Expand diversity of the care


team to check bias and
discrimination

• Incentivize change through


payment models

Anderson et al. Health Affairs 2018

3579
Copyright © Harvard Medical School, 2018. All Rights Reserved.

Eliminating Disparities in Practice


Example: Kaiser Permanente
Population health medical record data review to
identify patients not meeting quality metrics
Single pill combination therapy improved adherence
Medical assistants also followed up with patients two
to four weeks after medication adjustments
– Hypertension control increased from 44% to 90%
– Eliminated racial disparities in hypertension, LDL and
glycated hemoglobin control

Sources: Ayanian et al. NEJM 2014


Jaffe et al. Journal of Clinical Hypertension 2016

Question 2
According to the American College of Physicians April
2018 position paper on Addressing Social Determinants
of Health, which of the following is most likely to
address a health inequity?
A. A physician-led press conference describing clinical data
on patient blood lead levels
B. Opening a grocery store to improve access to fresh
foods in an underserved neighborhood
C. Screening patients for social needs at every clinic visit
D. Placing low-income housing near highways to improve
transportation access

3580
Copyright © Harvard Medical School, 2018. All Rights Reserved.

Question 2
According to the American College of Physicians
April 2018 position paper on Addressing Social
Determinants of Health, which of the following is
most likely to address a health inequity?
A. A physician-led press conference describing clinical
data on patient blood lead levels
B. Opening a grocery store to improve access to fresh
foods in an underserved neighborhood
C. Screening patients for social needs at every clinic visit
D. Placing low-income housing near highways to improve
transportation access

What Are Social Determinants Of Health?

• World Health Organization Framework:


– Circumstances in which people are born, grow, live, work,
and age

– Systems put in place to deal


with illness.
– Circumstances and systems
shaped by economics,
social policies, and politics

3581
Copyright © Harvard Medical School, 2018. All Rights Reserved.

What is a Health Inequity?


World Health Organization Framework
Definition of Health Inequities: • Equity from the start
– Maternal and child health
Human Rights and Social Justice
• Healthy places and people
• Differences between individuals – Environmental conditions
and groups that are avoidable
and unfair • Fair employment and safe work
– Living wages, occupational conditions
• Arise from social and economic • Social safety net protections
conditions within and between – Protected standard of living
societies • Health in all policies
– Make the healthy thing the easy thing
• Differences in illness are to do
contingent on social and • Fair financing
economic actions taken to – Progressive tax system
prevent and respond to illness • Rights of women
• Collect and monitor data

Multiple Dimensions of Risk

Source: Closing the Gap in a Generation


Amended from Solar & Irwin, 2007

3582
Copyright © Harvard Medical School, 2018. All Rights Reserved.

Social Determinants of Health:


Multiple Dimensions of Risk
• Affect patients across the life course
• Affect patients via psychosocial, behavioral and
material pathways
– Require multiple or “multi-level” intervention
components, “upstream” investments, and systems
changes

• Physicians can observe inequities in practice and


address them in significant ways

Question 2
Which of the following is most likely to address a health inequity?
A. A physician-led press conference describing clinical data on patient blood lead levels
B. Opening a grocery store to improve access to fresh foods in
an underserved neighborhood
- Simply increasing access to stores may not be impactful without
multi-level interventions to support behavior change
C. Screening patients for social needs at every clinic visit
- Systems for intervention are needed to act on screening results;
tailor timing of screening to clinic resources and patient needs
D. Placing low-income housing near highways to improve
transportation access
-potential childhood asthma risk associated with automobile
exhaust near high-traffic areas
Source: American College of Physicians Annals of Int. Med 2018

3583
Copyright © Harvard Medical School, 2018. All Rights Reserved.

Question 2
Which of the following is most likely to address a
health inequity?
A. A physician-led press conference describing clinical
data on patient blood lead levels
- ACP highlights social, environmental and public health actions as
contributors to patient health, and underscores the role of clinical
data and physician advocacy in addressing determinants of health
B. Opening a grocery store to improve access to fresh foods in an
underserved neighborhood
C. Screening patients for social needs at every clinic visit
D. Placing low-income housing near highways to improve
transportation access

Physician as Advocate

Pediatrician Dr. Mona Hanna-Attisha giving a press conference to recommend


issuing a water advisory due to high lead blood levels found in her patient panel

Story: https://www.nytimes.com/2018/06/09/opinion/sunday/flint-water-pediatrician-detective.html
Press Conference: https://youtu.be/6tELb594WTw
Timeline: https://www.cnn.com/2016/01/20/health/flint-water-crisis-timeline/index.html

3584
Copyright © Harvard Medical School, 2018. All Rights Reserved.

What Can We Do to Eliminate


Inequities in Clinical Care?
ACP statement of 9 recommendations
• Implement public policy interventions to promote health equity
• Health care professionals should be knowledgeable about
screening and identifying social determinants in practice
• Use team-based care approaches
• Invest in evidenced-based programs and social services
• Support research and include diverse participants in research
• Support “health in all policies” in community planning
• Use electronic health record systems to improve health
• Adjust payment and performance metrics for social risk
• Screen and collect social data to aid evidence-driven decisions

Source: American College of Physicians Annals of Int. Med 2018

Example in Practice
Social Screening and Referral System
in Primary Care
• Adapted a social screening tool to document
social needs (e.g. housing instability, food
insecurity)
• Built a screening survey in the EHR to facilitate
data collection
• Used the EHR to refer patients for services

Source: Gold et al. JABFM August 2017

3585
Copyright © Harvard Medical School, 2018. All Rights Reserved.

Topic Updates

• Social determinants of health in clinical settings


• Cultural humility
• Implicit bias in clinical care

Question 3:
Which of the following are currently available tools and
incentives to assist clinicians in addressing social
determinants of health (SDOHs) in clinical settings?
A. New delivery system and payment models, including
Medicaid Accountable Care Organizations (ACOs)
B. Electronic Health Record-integrated screening tools
C. An evidence base of practical strategies to guide
intervention in clinical practice
D. All of the above

3586
Copyright © Harvard Medical School, 2018. All Rights Reserved.

Question 3:
Which of the following are currently available tools and
incentives to assist clinicians in addressing social
determinants of health (SDOHs) in clinical settings?
A. New delivery system and payment models, including
Medicaid Accountable Care Organizations (ACOs)
B. Electronic Health Record-integrated screening tools
C. An evidence base of practical strategies to guide
SDOH intervention in clinical practice
D. All of the above

Delivery/Payment Model: Medicaid


Accountable Care Organizations (ACOs)
- Financial incentive to reduce
total costs of care for populations
- Federal authority: 1115
demonstration waiver
Some state models require
screening and intervening on SDOHs
Models encourage or require
partnerships with community-
based organizations
States with active Medicaid ACOs
Waivers allow payment for select States pursuing Medicaid ACO
services to address SDOHs programs
Sources: Center for Health Care Strategies, Inc.
Resource: https://sirenetwork.ucsf.edu/ (Gottlieb et al.)

3587
Copyright © Harvard Medical School, 2018. All Rights Reserved.

Question 4:
Compared to Cis-gender populations, gender
minority populations (including transgender and
gender non-conforming groups) are more likely
to:
A. Have fair or poor self-rated health
B. Be unemployed or out of the labor force
C. Both A and B
D. We do not have data collected to understand issues
related to gender minority populations

Question 4:
Compared to Cis-gender populations,
gender minority populations
(including transgender and gender
non-conforming groups) are more
likely to:
A. Have fair or poor self-rated health
B. Be unemployed or out of the labor
force
C. Both A and B Sexual and Gender Minority
(SGM)-related questions
D. We do not have data collected to available for use by states
understand issues related to gender from 2014-present
minority populations
Source: Streed et al. JAMA Int Med 2017

3588
Copyright © Harvard Medical School, 2018. All Rights Reserved.

Collecting Data
Demonstrates Cultural Humility
Definition of “Cultural Humility” in medical education:
Commitment to life-long (1) learning about aspects of
cultural identity, (2) addressing power dynamics, and
(3) developing partnerships with patients and their
advocates
In practice
• Attitude: Being willing to say “I don’t know” and being
interested in asking individuals what is important to them
• Action: Collecting and analyzing population data to plan for
resources that address patients’ needs and assets.
Source: Tervalon and Murray-García 1998
Resource: https://www.hospitalmedicine.org/practice-
management/the-5-rs-of-cultural-humility/

2015 CMS Equity Plan for Medicare


• Priority 1: Expand • Resources:
collection, reporting, and
analysis of standardized
data
– Sexual orientation and
gender identity (SOGI) data
– Race, ethnicity and
language (REaL) data
– Disability status

Sources: https://www.cms.gov/About-CMS/Agency-Information/OMH/OMH_Dwnld-
CMS_EquityPlanforMedicare_090615.pdf

https://aspe.hhs.gov/basic-report/hhs-implementation-guidance-data-collection-standards-race-
ethnicity-sex-primary-language-and-disability-status

3589
Copyright © Harvard Medical School, 2018. All Rights Reserved.

Question 5:
There is compelling evidence that a provider’s
implicit bias influences which of the following
outcomes or processes?
A. Clinical outcomes
B. Doctor-patient communication
C. Patient adherence
D. Process measures of quality care delivery

Question 5:
There is compelling evidence that a provider’s
implicit bias influences which of the following?
(Data are emerging on other outcomes)

A. Clinical outcomes
B. Doctor-patient communication
C. Patient adherence
D. Process measures of quality care delivery
Source: Mania et al. Soc Sci Med 2018

3590
Copyright © Harvard Medical School, 2018. All Rights Reserved.

Definition of Implicit Bias

Definition:
“Unexamined cultural stereotypes that are
automatically activated in ways that bypass
deliberate thought and influence one’s judgement
in unintended and unacknowledged ways”

Source: Chapman et al JGIM 2013

Data on Impact of Implicit Bias

• All 7 studies that examined the impact of


implicit bias on communication showed that
stronger provider bias was associated with
poorer patient-provider communication

Source: Mania et al. Soc Sci Med 2018

3591
Copyright © Harvard Medical School, 2018. All Rights Reserved.

Communication Example: Hagiwara et al.


Soc. Sci & Medicine 2016
• IAT used to measure

Physician Talk-Time Ratio


implicit bias for
physicians (n=14),
perceived
discrimination scale in
patients (n = 112)
Physician Implicit Bias
• Physician bias
influenced talk-time
ratio

What Do We Do to Address Implicit


Bias?

• Attention to
communication

• Awareness of implicit
bias and other
cognitive biases

3592
Copyright © Harvard Medical School, 2018. All Rights Reserved.

Culturally and Linguistically


Appropriate Services (CLAS)
• Standard 4. Health care
organizations must offer
and provide language
assistance services,
including bilingual staff and
interpreter services, at no
cost to each
patient/consumer with
limited English proficiency
at all points of contact, in a
timely manner during all
hours of operation.

Professional Interpreters
Improve Outcomes
• Professional interpreters vs. ad hoc
interpretation:
– Reduction in communication errors

– Increase in patient comprehension

– Equalize health care utilization & clinical


outcomes

– Increase satisfaction with communication


Source: Karliner et al. Health Services Research 2007

3593
Copyright © Harvard Medical School, 2018. All Rights Reserved.

Recognizing Implicit Bias is Difficult

Source: Green et al. JGIM 2007

Cognitive Biases in Health Care


(Diagnostic biases)
• Ascertainment bias: a physician’s
thinking is shaped by prior
expectations
• Visceral bias: our feelings affect
decision making
• Fundamental attribution error:
we may blame patients for
illnesses, rather than investigating
contributing circumstances
Sources: Croskerry, Academic Medicine 2003,
The Joint Commission October 2016

3594
Copyright © Harvard Medical School, 2018. All Rights Reserved.

Emerging: What to do About


Cognitive Bias
• Structural interventions
– Standardizing care
– Explicit communication
– Reducing time pressures
• Awareness of Cognitive Bias
– See Crosskerry, Academic Medicine (2003)

• Mindfulness practice
– Lovingkindess and empathy for self
– Curiosity and compassion for patients

Summary
• Health disparities and inequities are multi-factorial,
and include social and medical contributors
• There is a movement toward addressing social
determinants of health in clinical care. Physicians are
increasingly incentivized to have the skills, knowledge,
attitudes and systems in place to address these issues.
• Cultural humility and work to acknowledge implicit
bias contribute to good data and good communication
needed to eliminate disparities and inequities

3595
Copyright © Harvard Medical School, 2018. All Rights Reserved.

References and Resources


• Anderson, Andrew C., et al. "Promoting Health Equity And
Eliminating Disparities Through Performance Measurement
And Payment." Health Affairs 37.3 (2018): 371-377.

• Social Interventions Research & Evaluation Network (SIREN):


https://sirenetwork.ucsf.edu/
• Tervalon, Melanie and Jann Murray-García. "Cultural
humility versus cultural competence: A critical distinction in
defining physician training outcomes in multicultural
education." Journal of health care for the poor and
underserved 9.2 (1998): 117-125.

• Implication Association Test for implicit bias:


https://implicit.harvard.edu/implicit

3596
Copyright © Harvard Medical School, 2018. All Rights Reserved.

General Dermatology Review


Clinical Pearls for Common Problems
Manisha Thakuria, MD
Co-Director, Merkel Cell Carcinoma Center of Excellence
Department of Dermatology,
Brigham & Women’s Hospital
Center for Cutaneous Oncology,
Dana-Farber/Brigham & Women’s Cancer Center
Instructor of Dermatology, Harvard Medical School

Disclosures

• No Disclosures

3597
Copyright © Harvard Medical School, 2018. All Rights Reserved.

Lecture Goals

• Recognize and treat common rashes


• Recognize common benign skin lesions
• Recognize skin cancers
• Know when to refer

Top 10 Problems in Dermatology

• Rashes: • Lesions
– Acne – Benign growths
– Fungi / Tinea – Actinic keratoses
– Seborrheic – BCC
dermatitis – SCC
– Psoriasis – Melanoma
– Eczema

3598
Copyright © Harvard Medical School, 2018. All Rights Reserved.

Common Rashes

ACNE

3599
Copyright © Harvard Medical School, 2018. All Rights Reserved.

Acne: Epidemiology

• Typically presents first


in children/teens, and
resolves by age 25
• However, 12% of
women and 3% of
men have acne until
their 40s
• First acne outbreak
not uncommonly
occurs between age
20-35

Acne: Classification

• Classification is based • Important to also note:


on the morphology: – Scarring
– Comedonal – Hormonal distribution
• Open comedones -
“blackheads”
• Closed comedones -
“whiteheads”
– Inflammatory
• Papules, pustules
– Nodulocystic
• Nodules, cysts

Photo courtesy of Elizabeth Buzney, MD

3600
Copyright © Harvard Medical School, 2018. All Rights Reserved.

Acne: Morphology

inflammatory papules

open comedones

closed comedones

pustule
Photo courtesy of Elizabeth Buzney, MD

Acne: Morphology
open comedones on the nose

Photo courtesy of Elizabeth Buzney, MD

3601
Copyright © Harvard Medical School, 2018. All Rights Reserved.

Acne: Morphology
inflammatory and cystic acne with scarring

Photo courtesy of Elizabeth Buzney, MD

Photo courtesy of Harley Haynes, MD

Acne: Morphology
nodulocystic acne with
scarring

Photo courtesy of Harley Haynes, MD

3602
Copyright © Harvard Medical School, 2018. All Rights Reserved.

Acne: Pathogenesis

• Acne is multifactorial!
– Keep this in mind – most pts require more than one
treatment modality

• Key Components:
1. Defective keratinization
- Results in plugging of the hair follicle, giving rise to comedones
2. Androgens
- Stimulate sebaceous glands oil production
3. Bacteria (Propionibacterium acnes)
- Lives in the follicle, turns oils into free fatty acids, causing inflammation
4. Inflammation
- In response to keratinous debris, FFAs, and P. acnes

Acne: General Treatments Principles

• Main classes of therapy:


– Retinoids - topical and systemic
– Antibiotics - topical and systemic
– Hormonal - systemic only
– Misc - keratolytics, intralesional steroids,dapsone
• Goal:
– #1 – prevent scarring
– #2 – clear skin

3603
Copyright © Harvard Medical School, 2018. All Rights Reserved.

Acne: Treatments Principles

• Combination therapies are most effective


• Treatment is based on Morphology & Severity…
Comedonal Inflammatory Nodulocystic
(black-/whiteheads) (papular, pustular)
Mild Topical retinoid Topical abx
+/- BP + BP
+/- Topical retinoid
Moderate Topical retinoid Oral abx Oral abx
+ BP + BP + Topical retinoid
+ Topical retinoid + BP
Severe (Or if Derm Derm Derm Referral
Presence of Referral Referral (Oral isotretinoin)
Scarring)

• …and also Distribution


– hormonal distribution? Y OCPs, spironolactone
– can pt reach affected body areas? N oral treatments

Acne: Consider the Distribution

3604
Copyright © Harvard Medical School, 2018. All Rights Reserved.

Acne: Topical Retinoids

• Retinoids:
– Tretinoin cream / gel 0.025%, 0.05%, 0.1%
• name brands Retin-A, Retin-A Micro, Atralin, Renova
– Adapalene cream / gel / lotion 0.1% and 0.3%
• Better for those with drier, more sensitive skin
• name brand Differin
– Tazarotene
• Strongest, difficult for most pts to tolerate
• Sig: Apply pea-sized at night, start 1-2x/week and
advance to QHS as tolerated
– SE: redness*, irritation/itching*, scaling*, +/-
photosensitivity
*usually a result of advancing too quickly rather than a medication
intolerance/allergy
– Pea-sized amount sufficient for entire face

Acne: Topical Antibiotics

• Clindamycin 1% lotion, solution, gel, foam


• Erythromycin 2% solution, gel
• Azelaic acid 15% or 20% gel
– Naturally occurring saturated dicarboxylic acid
– Bacteriocidal against P acnes
– Useful in pts with post-inflammatory hyperpigmentation

• Benzoyl Peroxide washes, creams, gels, lotions


– Has both antibacterial and comedolytic properties
– Use in combo with abx above - prevents resistance
– Use in the AM, to avoid oxidizing retinoids (applied PM)
– Pts w/ truncal acne: Keep an extra BP wash in the shower
– SE: bleaching of hair/clothes, dryness, irritation

3605
Copyright © Harvard Medical School, 2018. All Rights Reserved.

Acne: Treatment with Oral Antibiotics

• Tetracyclines are the antibiotic of choice:


– Minocycline – dizziness, dyspigmentation, drug-ind SLE
– Doxycycline – photosensitivity
– Know your SE, counsel appropriately – all can cause GI
upset, pill esophagitis – take with a full glass of water, stay
upright, and ok to take with food if needed for tolerability
• Other options: cephalosporins, macrolides, sulfa
• Strategies:
– Try to taper off oral antibiotics as pt improves
switch to topical regimen as maintenance
– If no benefit after 3 mo at full dose, switch to another abx
– If no benefit after at least 6 mo of 2 different abx
consider isotretinoin

Acne, Simplified

• Most common type: mixed inflamm. & comedonal


• Most common regimen (worth memorizing):
– BPO 2-5% wash otc 1-2x/week in the mornings
– Mild non-medicated wash otherwise BID
– Clindamycin lotion BID
– Tretinoin 0.025% cream QHS, start slowly (e.g. 1-2x/week)
• Unless very dry skin Adapalene 0.1%
• If pain / scarring / nodulocystic
lesions:
– Start oral antibiotics, refer to derm
• Hormonal distribution
– Consider OCPs or spironolactone

3606
Copyright © Harvard Medical School, 2018. All Rights Reserved.

Acne: Last Thoughts

• Unless your patient is extremely oily at baseline,


most patients will do better with creams and
lotions, rather than solutions and gels, as many
of the topical therapies for acne have xerosis as a
side effect

• Tell patients new meds may take 6-8 wks to work


– Retinoids may cause initial flaring
– Oral antibiotics have faster onset

SUPERFICIAL
FUNGAL & YEAST
INFECTIONS

3607
Copyright © Harvard Medical School, 2018. All Rights Reserved.

Superficial Fungal Infections

• Three groups of fungi can cause superficial


infections of the skin (epidermis):
– Dermatophytes
– Candida spp. (yeast)
– Malassezia spp. (yeast)

• “Tinea” = Dermatophytoses = superficial fungal


infection of keratinized tissues, e.g., stratum
corneum, hair, and nails
– Three genera:
• Microsporum
• Trichophyton
• Epidermophyton

Tinea: Diagnosis

• KOH prep – most cost effective means of dx


– You’ll need: 15-blade, alcohol swab, and slide
– Wipe area w/ alcohol swab
– Scrape until sufficient scale on slide, then cover
scale w/ coverslip
• Edge of plaque is highest yield
– Apply a few drops of KOH at edge to get
underneath coverslip
– Light a match! Hold under slide for a few
seconds
– 10x scan to find hyphae, 40x for detail if needed

3608
Copyright © Harvard Medical School, 2018. All Rights Reserved.

Tinea: KOH prep


parallel lines longer than surrounding keratinocytes

branching

Tinea: Another KOH prep

First, scan at 10x…

Then, zoom in to 40x to


precisely identify structures
Photos courtesy of Adam Lipworth, MD

3609
Copyright © Harvard Medical School, 2018. All Rights Reserved.

Photo courtesy of Dermquest.com

Tinea Pedis

• “Athlete’s foot”
• Most common fungal
infxn
• Risk factor for Photo courtesy of Faloonb

development of
cellulitis
• Three clinical
patterns:
– Interdigital
– Moccasin (aka chronic
hyperkeratotic)
– Vesiculobullous
Photo courtesy of Adam Lipworth, MD

Tinea Pedis: Moccasin type

• Often associated with


nail involvement
• Look for the “One
hand, two feet”
syndrome

Photos courtesy of Janice Lin, MD

3610
Copyright © Harvard Medical School, 2018. All Rights Reserved.

Tinea Manus

Look for fine scale in the creases – great place to KOH!

Tinea Manus

3611
Copyright © Harvard Medical School, 2018. All Rights Reserved.

Tinea: Treatment

• Hygiene (T. pedis):


– Dry feet: cotton socks, frequent changes, powders
– Shoes: alternate shoes worn, consider sandals
• Topical antifungals:
– Use at least for 2 weeks, not just until appears resolved
– Many to choose from:
• Imidazoles – clotrimazole, miconazole, econazole,
oxiconazole (note, not ketoconazole)
• Allylamines – terbinafine, butenafine, naftifine
• Ciclopirox
• Discuss, recurrences are common, but should
clear with treatment

Tinea Corporis

• Annular
erythematous scaly
patches, often with
advancing scaling
edge
• Topicals generally
sufficient, unless:
– Heavily hair bearing
areas
– Large surface area
involved
– Animal source
Photos courtesy of Ruth Ann Vleugels, MD

3612
Copyright © Harvard Medical School, 2018. All Rights Reserved.

Tinea Corporis

• Annular
erythematous scaly
patches, often with
advancing scaling
edge
• Topicals generally
sufficient, unless:
– Follicular involvement
– Large surface area
– Animal source
• DDx: nummular
eczema
Photos courtesy of Adam Lipworth, MD

Tinea Cruris

• Scaling erythematous
to hyperpigmented
patch on upper and
inner thighs
– Dry looking
– Central clearing
– Raised scaly border
• Scrotum/Penis rarely
involved
• Topicals effective
• DDx: inverse psoriasis

3613
Copyright © Harvard Medical School, 2018. All Rights Reserved.

Tinea Cruris / Corporis (in pt with AIDS)

Photo courtesy of Adam Lipworth, MD

Tinea Faciei

• Erythematous
plaque with
distinctive
edge-active
scale,
asymmetrical

Photo courtesy of Grook Da Oger

3614
Copyright © Harvard Medical School, 2018. All Rights Reserved.

Case 1

• Frances is a 52 yo
female with a history
of obesity and
diabetes, who
presents with a 3
month history of
itching, burning, red
inframammary rash,
not relieved by triple
antibiotic ointment.
It’s starting to really
worsen now that it’s
finally warm outside.

Case 1

• What is the most likely


cause of Francis’s
rash?
A – Candida intertrigo
B – Inverse psoriasis
C – Seborrheic dermatitis
D – Tinea corporis

3615
Copyright © Harvard Medical School, 2018. All Rights Reserved.

Case 1

• What is the most likely


cause of Frances’s
rash?
A – Candida intertrigo
B – Inverse psoriasis
C – Seborrheic dermatitis
D – Tinea corporis

Case 1

• What is the most likely


cause of Frances’s
rash?
A – Candida intertrigo
Note presence of satellite lesions, itching
and burning, made worse by sweating

B – Inverse psoriasis
Good location, and can look similar.
Plaques lack satellite lesions, and are
often not as bright red.
C – Seborrheic dermatitis
Wrong distribution
D – Tinea corporis
Expect to see raised rim, usually with
scale.

3616
Copyright © Harvard Medical School, 2018. All Rights Reserved.

Candida Intertrigo

• Red moist plaques, often with satellite lesions


Candida intertrigo
– distinguish from inverse psoriasis
• Often more sore than itchy
• Promoted by:
– Diabetes
– Obesity
– Heat inc sweating
– Occlusive clothing inc sweating Inverse psoriasis

– Limited mobility inc sweating

Candida Intertrigo: Treatment Principles

• Can always be treated topically


• Powders don’t work well for treatment
– Penetration poor
– But are great for dz maintenance
• Creams are preferred
• Ointments are a good choice when skin is
macerated or starting to break down

3617
Copyright © Harvard Medical School, 2018. All Rights Reserved.

Candida Intertrigo: Treatment

• Polyenes: nystatin
– notably comes in an ointment – prevent maceration
• Azoles: ketoconazole*, clotrimazole,
econazole, miconazole
• Ciclopirox
– has mild anti-inflammatory properties, and has activity
against Gram positive and Gram negative bacteria that
often co-exist
• Keep it dry to keep it from coming back!
– Corn starch
– Antifungal powders: miconazole, nystatin,
undecylenic acid, tolnaftate, 12% benzoic acid

Seborrheic Dermatitis

• Common, affects 2-5%


• Rash is caused by
inflammatory reaction to
Malassezia globosa

Photo courtesy of Roymishali

Photo courtesy of Harley Haynes, MD

3618
Copyright © Harvard Medical School, 2018. All Rights Reserved.

Seborrheic Dermatitis: Clinical

• Erythematous patches
and thin plaques with
overlying yellowish dry
or greasy scales
• In areas w/ increased
sebaceous gland activity
– Scalp (“dandruff”)
– Face: eyebrows,
nasolabial folds, ears
– Trunk: chest / sternal area
• Worse in pts with HIV

Seborrheic Dermatitis: Treatment

• Antidandruff shampoos (scalp, chest)


– Ketoconazole 1% (otc) or 2% (Rx), selenium sulfide,
zinc pyrithione, ciclopirox
– Lather, leave shampoos on x 10 minutes, rinse, repeat
QOD till clear
• Antidandruff creams (face)
– Ketoconazole 2%, ciclopirox
• +/- a low potency topical steroid
– Hydrocortisone 2.5%, desonide, fluocinolone
solution
• Calcineurin inhibitors (3rd line, concurrent rosacea)
– Pimecrolimus cream, tacrolimus ointment

3619
Copyright © Harvard Medical School, 2018. All Rights Reserved.

Tinea Versicolor

• Misnomer - Not a true


dermatophyte infxn!
• Malassezia furfur
– (formerly Pityrosporum
ovale)
• Well-demarcated
hypopigmented, tan,
pink, or salmon
patches on the trunk
• Scale is inducible by
rubbing w/ finger/blade
Photo courtesy of Adam Lipworth, MD

The Many Colors of Tinea Versicolor


Hypopigmented

Hyperpigmented

Pink or Salmon-colored

Photo courtesy of Margaret Kopelman, MD

3620
Copyright © Harvard Medical School, 2018. All Rights Reserved.

Tinea Versicolor: Treatment

• Shampoos:
– E.g. selenium sulfide, ketoconazole, pyrithione zinc
– Cover larger area
– Leave on 5-10 minutes, then rinse
• Creams:
– E.g. ketoconazole, clotrimazole, miconazole,
econazole
– Easier for limited disease
• Recurrences are common
– Plan to retreat in summer months
• Scale resolves quickly; dyspigmentation weeks to
months

PSORIASIS
&
ECZEMA

3621
Copyright © Harvard Medical School, 2018. All Rights Reserved.

Psoriasis

• Classification by body
surface area (BSA)
– Mild: <5%
– Moderate: 5-10%
– Severe: >10%
• 20% of patients have
moderate to severe disease
– Generally requires
phototherapy or systemic
medications to clear

Psoriasis - Not only a skin disease

• Psoriatic Arthritis 7-42%


– Destructive arthropathy
• Obesity (OR 1.66) &
Metabolic Syndrome
• Cardiovascular risks
– Independent, non-modifiable
risk factor for MI and stroke
– Increased incidence of
arrhythmia
• Malignancy risks
– NMSCs, lymphoma, lung

3622
Copyright © Harvard Medical School, 2018. All Rights Reserved.

Eczema

• Not just one disease!


– Atopic dermatitis
– Contact dermatitis
– Asteatotic eczema
– Dyshidrotic eczema
– Neurodermatitis
– Nummular dermatitis
– Stasis dermatitis

Eczema and Psoriasis

• Some pearls:
– Consider surface area you are treating:
• 30 g tube covers the entire body of an adult once
• Clobetasol >50g/wk – pt at risk of HPA suppression
• Estimate the BSA affected:
1 palm = 1% BSA
(pt’s palm, not yours)

• Then estimate how much you’ll need to give:


1 fingertip unit = ½ g treats 2% BSA (2 palms)

So, if you’re treating a 4% BSA rash (1g/treatment)


with a BID medication (2g/day), and planning 2 weeks
of treatment, dispense 30g

3623
Copyright © Harvard Medical School, 2018. All Rights Reserved.

Eczema and Psoriasis

• Rule of Nines
– Quick way to estimate
BSA in adults
– Divide body into 9%
sections:
• Head – 9%
• Chest – 9%
• Abdomen – 9%
• Back – 9% x 2
• Upper ext – 9%
• Lower ext – 9% x 2
• Groin – 1%

Eczema and Psoriasis

• More pearls…
– Consider the base: What? There’s a difference!?!
• Ointments are greasy, but have fewer potential
irritants, and are generally more potent
• Creams rub in better, don’t mess up clothes as
much, and may be preferred by many patients
• Lotions, gels may be better for acne-prone skin,
hair-bearing areas
• Solutions, foams easier to use in the scalp

3624
Copyright © Harvard Medical School, 2018. All Rights Reserved.

Eczema and Psoriasis

• And more pearls…


– Consider the base:
The Effect of Vehicle on Potency:
Generic name Betamethasone dipropionate Potency Class
Trade name Diprolene ointment 0.05% 1
Diprolene gel 0.05% 1
Diprolene cream 0.05% 2
Trade name Diprosone ointment 0.05% 2
Diprosone cream 0.05% 3
Diprosone lotion 0.05% 5

• Emphasize intermittent therapy to avoid


tachyphylaxis, atrophy, etc – “burst and taper”

Common Dermatologic Growths

• Benign neoplasms Butchart Gardens in Vancouver

– Seborrheic keratoses
– Cherry angiomas
– Sebaceous hyperplasia
• Actinic keratoses
• Basal cell carcinoma
• Squamous cell carcinoma
• Moles and melanoma

Photo courtesy of NorwegianMarcus on wikipedia

3625
Copyright © Harvard Medical School, 2018. All Rights Reserved.

BENIGN NEOPLASMS
OF SKIN

Seborrheic Keratoses

• Common, benign
lesions
• Appear in 30s
• Usually in sun-
exposed distribution

3626
Copyright © Harvard Medical School, 2018. All Rights Reserved.

Seborrheic Keratoses

• Waxy, sometimes velvety, sometimes


verrucous, “stuck-on” appearance,
very well-demarcated

Cherry Angiomas

• Most common
acquired vascular
proliferation
– Dilated, congested
capillaries and post-
capillary venules
• Bright red, dome
shaped papules
• Trunk + extremities
• Appear in teens, 20s,
and beyond
– Increase during
pregnancy

3627
Copyright © Harvard Medical School, 2018. All Rights Reserved.

Sebaceous Hyperplasia

• Not a true neoplasm fd


fdfd afsd
• Benign enlargement of
sebaceous lobule
around follicular
infundibulum
• Yellowish soft papules,
+/- telangectasias,
central dell
• Central / upper face

ACTINIC KERATOSES

3628
Copyright © Harvard Medical School, 2018. All Rights Reserved.

Actinic Keratoses

• AKA, solar keratoses,


senile keratoses
• Precancerous lesions
for SCC
– Similar rates of
transformation as
cervical intraepithelial
neoplasia
– Emphasize, low risk!

Actinic Keratoses: Presentation

• Head, neck, upper


trunk, forearms/hands
• Typical patient:
– Elderly, chronic sun
exposure, light skin
• Rough, erythematous,
scaly thin papule
– Scale can be gritty, white,
yellow, hyperkeratotic, or
a “horn”
Tenderness on palpation,
esp if indurated bx for
invasive SCC

3629
Copyright © Harvard Medical School, 2018. All Rights Reserved.

Actinic Keratoses: Treatments

• Consider number, location, thickness, and your pt


– Local therapies – e.g. cryotherapy
• Preferred when lesions are few or scattered; pt not interested in
field treatments, or not expected to be compliant
• Advantages: quick procedure, short down time (~1-2 week),
physician control
• Disadvantages: painful, can miss pre-clinical lesions, can
inappropriately treat something (e.g. amelanotic melanoma)
– Field therapies – e.g. 5-FU, imiquimod, PDT

Cryotherapy

• AKs
– Single freeze-thaw cycle of 8-10 seconds
• Hypertrophic AKs may require longer or multiple
treatments
• Atrophic AKs or areas of thinner skin may require less
– 1-2mm freeze margin around the lesion

3630
Copyright © Harvard Medical School, 2018. All Rights Reserved.

Actinic Keratoses: Treatments

• Consider number, location, thickness, and your pt


– Local therapies – e.g. cryotherapy
• Preferred when lesions are few or scattered; pt not interested in
field treatments, or not expected to be compliant
• Advantages: quick procedure, short down time (~1-2 week),
physician control
• Disadvantages: painful, can miss pre-clinical lesions, can
inappropriately treat something (e.g. amelanotic melanoma)
– Field therapies – e.g. 5-FU, imiquimod, PDT
• Advantages: treat wide area, prevent new lesions from arising,
“new skin”, ?cosmetic benefits, patient control
– With proper counseling, always a safe option, minimal
scarring
• Disadvantages: longer down time (usually about 1 month), need
for counseling, patient control

Actinic Keratoses: Field Therapies

• 5-fluorouracil
– Generic 5% cream, 2% & 5% solution
– Brands – Efudex 5% cream, Carac 0.5% cream
– Micromedex directions:
• 2 or 5% cream or solution BID x 2-6 wks
• 0.5% microsphere formulation daily x 4 weeks
– My directions: generic 5% cream 1-2x/day x ~3 weeks
• Titrate to erythema, slight irritation
• Counsel patients on expected side effects
• Apply petrolatum to soothe irritated skin
• If too unpleasant – call for ANTIDOTE!
– Topical steroid Rx
• Imiquimod, ingenol mebutate, PDT (photodynamic
therapy) – refer

3631
Copyright © Harvard Medical School, 2018. All Rights Reserved.

Actinic Keratoses: Topical 5-Fluorouracil

– Post-2.5 weeks of
Efudex BID
– Patient presented with
erythema, crusting,
shallow erosions, pain,
and itching. Distressed!
– Management:
• Petrolatum liberally
• Class V topical steroid (in
an ointment base) for 1-2
weeks

Case 2

• Fred is a 86 year old Caucasian gentleman with a history


of multiple skin cancers. He comes in with a 6+ month
history of a slowly growing, pearly lesion below,
occasionally bleeding.

What genetic mutation is associated


with this skin lesion?

A – BRAF
B – CDKN2A gene
C – MLH1/MSH2 DNA mismatch repair
D – Sonic hedgehog / patched (PTCH1)

3632
Copyright © Harvard Medical School, 2018. All Rights Reserved.

Case 2

• Fred is a 86 year old Caucasian gentleman with a history


of multiple skin cancers. He comes in with a 6+ month
history of a slowly growing, pearly lesion below,
occasionally bleeding.

What genetic mutation is associated


with this skin lesion?

A – BRAF
B – CDKN2A gene
C – MLH1/MSH2 DNA mismatch repair
D – Sonic hedgehog / patched (PTCH1)

Case 2

• What genetic mutation is associated with the skin


lesion below? Lesion is a BCC.
A – BRAF
acquired mutation in this gene is the most common event
leading to melanoma, present in ~2/3 of melanomas
Inhibitors: vemurafenib and dabrafenib
B – CDKN2A gene
most common cause of inherited melanoma
C – MLH1/MSH2 DNA mismatch repair
defect leads to Muir Torre syndrome (HNPCC)
D – Sonic hedgehog / patched (PTCH1)

3633
Copyright © Harvard Medical School, 2018. All Rights Reserved.

Basal Cell Carcinoma: Genetics


• BCCs are caused by loss of function mutations in PTCH (tumor
suppressor gene) and activating mutations in smoothened (oncogene)
– Aquired - from DNA damage to basal keratinocytes from UV
– Heritable - Basal Cell Nevus Syndrome (aka Gorlin Syndrome)
• mutation in PTCH
• 100s of BCCs, broad nasal root, palmar pits, and jaw cysts
• Vismodegib, Sonidegib
– hedgehog pathway
inhibitors
– Recently approved for
treatment of advanced
and metastatic BCC
– Blocks smoothened
receptor

Illustration courtesy of Peter Znamenskiy

Nonmelanoma skin cancers

• NMSCs:
– Basal cell carcinoma (75-80%)
– Squamous cell carcinoma
– (Merkel cell carcinoma (<<<1%, although rising incidence))
• 1/5 Americans will develop skin cancer in their
lifetime
• Incidence increases with age
• Other risk factors: Fair skin*, UV exposure,
ionizing radiation, chemical exposures,
occupation, immunosuppression
– Chronic, long term UV SCC
– Intermittent, intense episodes of burning BCC

3634
Copyright © Harvard Medical School, 2018. All Rights Reserved.

BASAL CELL CARCINOMA

BCC: Epidemiology

• Most common type of cancer


– > 1 million cases annually
– 25% of all cancers in the US
• Risk factors:
– UV exposure
• Intermittent holiday > chronic occupational
– Fitzpatrick Type I and II skin
• i.e., light skin/eyes, always burns, zero to minimal ability to tan
– Northern European ancestry
– Immunosuppression
• Patients with BCC at higher risk for melanoma

3635
Copyright © Harvard Medical School, 2018. All Rights Reserved.

BCC: Presentation

• Most commonly on head and neck


• Variable appearance depending on type
– Classic nodular (60%) – favors the face
• Ulcerated variants
• Pigmented variants
– Superficial – favors the trunk/ext
– Morpheaform – scarlike
• Historical Clues:
– Slow growing – develop over months to years
– Friable, bleed easily

BCC Presentation

3636
Copyright © Harvard Medical School, 2018. All Rights Reserved.

BCC Types:
Nodular Superficial

Pigmented Nodular Ulcerated Morpheaform

BCC Types:
Nodular

Nodular

3637
Copyright © Harvard Medical School, 2018. All Rights Reserved.

BCC Types:
Pigmented Nodular

BCC Types:

Morpheaform

Ulcerated
Superficial

3638
Copyright © Harvard Medical School, 2018. All Rights Reserved.

Ulcerated BCCs

SQUAMOUS CELL
CARCINOMA

3639
Copyright © Harvard Medical School, 2018. All Rights Reserved.

Squamous Cell Carcinoma In Situ

• Bowen’s disease, aka


SCC in situ
– Scaly erythematous
patch or thin plaque,
sometimes crusted
– DDx: AKs, superficial
BCC, SCC, psoriasis,
nummular eczema

Squamous Cell Carcinoma In Situ

• Bowen’s disease, aka


SCC in situ
– Scaly erythematous
patch or thin plaque,
sometimes crusted
– DDx: AKs, superficial
BCC, SCC, psoriasis,
nummular eczema

3640
Copyright © Harvard Medical School, 2018. All Rights Reserved.

Squamous Cell Carcinoma In Situ

• Bowen’s disease, aka


SCC in situ
– Scaly erythematous
patch or thin plaque,
sometimes crusted
– DDx: AKs, superficial
BCC, SCC, psoriasis,
nummular eczema

Squamous Cell Carcinoma

• Invasive SCC
– Erythematous keratotic
papule/nodule
– +/- Tenderness
– More rapid growth
– Immunosuppression
– Higher risk for
metastases/death

3641
Copyright © Harvard Medical School, 2018. All Rights Reserved.

Squamous Cell Carcinoma

How Much Risk:


- 2.0 - 5.8% risk of nodal mets
- 2.1% risk of disease specific
death
Risk Factors:
– Tumor diameter > 2 cm
– Invasion beyond fat
– Poor differentiation
– Perineural invasion
– Ear, temple, anogenital

JAMA Dermatology May 2013

Squamous Cell Carcinoma


Bateman’s purpura
(actinic purpura, formerly
senile purpura)

3642
Copyright © Harvard Medical School, 2018. All Rights Reserved.

Case 3

Lucy, a 60 year old Caucasian woman with a history


of breast cancer, presents for an 8 week history
of an enlarging nontender, nonpruritic 8mm
lesion on the left calf. She is an avid runner.
The most likely diagnosis is:

A – Basal cell carcinoma


B – Melanoma
C – Seborrheic keratosis
D – Squamous cell carcinoma

Case 3

Lucy, a 60 year old Caucasian woman with a history


of breast cancer, presents for an 8 week history
of an enlarging nontender, nonpruritic 8mm
lesion on the left calf. She is an avid runner.
The most likely diagnosis is:

A – Basal cell carcinoma


B – Melanoma
C – Seborrheic keratosis
D – Squamous cell carcinoma

3643
Copyright © Harvard Medical School, 2018. All Rights Reserved.

Case 3

Lucy, a 60 year old Caucasian woman with a history of


breast cancer, presents for an 8 week history of an
enlarging nontender, nonpruritic 8mm lesion on the left
calf. She is an avid runner. The most likely diagnosis is:
A – Basal cell carcinoma
Although you can have pigmented varieties, this is growing too rapidly
B – Melanoma
Note the Asymmetry, Border irregularity, Color variation, Diameter over
6mm, and Evolution (growth) in a pt with sun exposure history
C – Seborrheic keratosis
Doesn’t have classic, “stuck-on” appearance
D – Squamous cell carcinoma
Lacks scale and has pigment

Case 3, continued

• Lucy’s pathology:
– Melanoma, Breslow’s depth 1.6mm (intermediate)
• Ulceration: Absent
• Mitoses: 1 mit per sq mm
• Vascular/Lymphatic Invasion: Absent
– Treatment:
• Sentinel lymph node biopsy and wide local excision
– SLNB was negative
• Follow-up:
– Routine skin & lymph node monitoring by Dermatology
– Routine self-skin exams (monthly)
– Sun protection

3644
Copyright © Harvard Medical School, 2018. All Rights Reserved.

Case 3, continued

• 1 year after her treatment,


Lucy returns urgently to your
office, because she suddenly
noted this lesion “overnight.”

• The first thing you do is:


A – Ask to take a closer look
B – Reassure her. You don’t know what it is,
but melanoma never strikes twice in the
same patient.
C – Reassure her. This is a classic cherry
angioma.
D – Tell her to call her dermatologist
immediately because she has a deadly
melanoma.

Case 3, continued

• 1 year after her treatment,


Lucy returns urgently to your
office, because she suddenly
noted this lesion.

• You:
A – (Astutely) Ask to take a closer look
B – Reassure her. You don’t know what it is,
but melanoma never strikes twice in the
same patient.
C – Reassure her. This is a classic cherry
angioma.
D – Tell her to call her dermatologist
immediately because she has a deadly
melanoma.

3645
Copyright © Harvard Medical School, 2018. All Rights Reserved.

Case 3, continued

• Talon Noir – “black heel”


– hemorrhage in the stratum
corneum caused by trauma
– perfectly defined border
– no actual pigment, only blood

c/o Virtual Grand Rounds 2.0

c/o Virtual Grand Rounds 2.0

MELANOMA

3646
Copyright © Harvard Medical School, 2018. All Rights Reserved.

Melanoma: Epidemiology

• Increasing incidence:
– 1/1500 – born in 1935
– 1/600 – born in 1960
– 1/150 – born in 1980
– 1/62 – born in 2006
• 1/34 – if including in situ melanoma
• Represents 5% of all new cancer cases*
– 5th most common cancer type*
– Most common cancer type in Caucasian women 25-29
• Accounts for 79% of all skin cancer deaths
– 1 American dies from melanoma every hour
– ~9700 in 2014
*excluding BCC and SCC

Melanoma: Risk Factors

• Genetic factors
– Sun-sensitive genotype
• “Skin type”: ability to tan / likelihood of sunburn
– Specific melanoma genes
• First degree relative with melanoma
• Multiple family members with melanoma, regardless of degree
• Families with dysplastic nevi + 2 members with melanoma
• Environmental factors
– Intermittent exposure hypothesis
• Just 1 blistering sunburn in childhood more than doubles lifetime
risk of developing melanoma!
– Tanning bed use
• Whener et al 2014: 13% of adults, 43% of college students
admitted to using a tanning bed in the past year.

3647
Copyright © Harvard Medical School, 2018. All Rights Reserved.

Melanoma: Risk Factors

• Phenotypic characteristics
– Blue or green eyes
– Red or blond hair
– >100 typical nevi
– Atypical nevi (any)
– Large congenital nevus (>20cm in adults)
– Fitzpatrick Skin Type I or II
• Burn easily
• Tan rarely or never

Melanoma: Diagnosis

• ABCDs, revamped:
– A – Asymmetry
– B – Borders irregular, notched, scalloped,
poorly defined
– C – Colors, varying shades
– D – diameter > 6mm
– D – Different
• The “ugly duckling” sign
– E – Evolving
• Changing in size, shape, color, or development of
symptoms

3648
Copyright © Harvard Medical School, 2018. All Rights Reserved.

Melanoma: Examples With Dermoscopy

Melanoma: Examples With Dermoscopy

3649
Copyright © Harvard Medical School, 2018. All Rights Reserved.

Melanoma: Diagnosis

PATHOLOGIC DIAGNOSIS:
MALIGNANT MELANOMA, invasive to a depth of 0.5 mm, anatomic level III/early IV.
Mitotic rate 1 per sq. mm

Melanoma: Diagnosis

PATHOLOGIC DIAGNOSIS:
MELANOMA, with nevoid cytomorphology, invasive to a depth of 0.38 mm,
anatomic level II, with regression-like stromal changes. No dermal mitoses seen.

3650
Copyright © Harvard Medical School, 2018. All Rights Reserved.

Melanoma: Diagnosis
June 2011 September 2011 - X
August 2012

(-) family hx/o melanoma


(+) hx/o tanning booth exposure (approx 10
lifetime)
Melanoma 1.35mm, 1 mitosis/mm2
Stage 1B

Case 4

• You referred Andrea to


Plastic Surgery for removal
of a small, firm, persistently
tender growth on her right
upper arm.
• She is a healthy 28 yo
woman with a history of
fibroids and no other
medical problems. The
following pathology report
is sent back to you:
“Pilar leiomyoma.”

3651
Copyright © Harvard Medical School, 2018. All Rights Reserved.

Case 4
You referred Andrea to Plastic Surgery
for removal of a small, firm, persistently • Pilar leiomyoma
tender growth on her right upper arm.
She is a healthy 28 year old woman • The next best step in
with a history of multiple fibroids and no management is:
other medical problems. The following
pathology report is sent back to you:
“Pilar leiomyoma.”
A – Do nothing, benign
lesion.
B – Excise other similar
lesions
C – Re-excision with
narrow margins
D – Refer to Genetics

Case 4

• Pilar leiomyoma
• The next best step in
management is:

A – Do nothing, benign
lesion. True, benign, however…
B – Excise other similar
lesions Can consider, but often can
be managed medically
C – Re-excision with
narrow margins Not necessary
D – Refer to Genetics
Reed’s Syndrome

3652
Copyright © Harvard Medical School, 2018. All Rights Reserved.

Case 4: Reed’s Syndrome


You referred Andrea to Plastic Surgery
for removal of a small, firm,
• Pilar leiomyoma
persistently tender growth on her right – Benign, smooth muscle growths
upper arm. She is a healthy 28 year old arising from arrector pili muscles
woman with a history of multiple – Medical management if patient
fibroids and no other medical
has multiple
problems. The following pathology
report is sent back to you: • Calcium channel blockers
“Pilar leiomyoma.” • Gabapentin
• Phenoxybenzamine
• Reed’s syndrome
– AD genetic condition
– Multiple cutaneous and uterine
leiomyomas
– Defect in fumarate hydratase
gene
– ~20% of patients develop renal
cell carcinoma

Case 4: Take Home Point

• Cutaneous leiomyomas (piloleiomyoma) + uterine


leiomyomas (fibroids) Genetic referral
fumarate hydratase defect (Reed’s synd)
early detection of RCC
save lives
pass the boards
• It is impossible to be familiar with every diagnosis

• There’s no shame in
consulting “Dr. Google”!

3653
Copyright © Harvard Medical School, 2018. All Rights Reserved.

Parting Tips for Skin Success

• Patient education is key


– Most treatment failures are a result of noncompliance
– Set patient expectations
• Set your own expectations
– Get comfortable with a few medications of each type
– Decide what you feel most comfortable managing
– Anticipate when you’re out of your comfort zone and
need to refer
• It is impossible to be familiar with every diagnosis
– Know your resources, including the neighborhood BCD
• Don’t forget your ABCDEs!

Supplemental Reference Slide

• Schmults CD et al. JAMA Derm; 149(5):541-7.


• Whitmore SE et al. JAMA Derm; 2001;44:775-80.
• Wehner MR et al. JAMA Derm. 2014;():.
doi:10.1001/jamadermatol.2013.6896.
• Habif. Clinical Dermatology: A Color Guide to Diagnosis
and Treatment. 5th edition.
– 1 volume text, ideal for the Internal Medicine clinician
• Bolognia, Jorizzo, Schaffer. Dermatology. 3rd edition
– Detailed text, a favorite of most dermatology trainees
• www.aad.org
– Excellent resource for clinicians and patients
• www.merkelcell.org
– If you are interested in what I do, outside of Gen Derm

3654
Copyright © Harvard Medical School, 2018. All Rights Reserved.

Disclosures

• No Disclosures

3655
Copyright © Harvard Medical School, 2018. All Rights Reserved.

Allergy/Immunology Overview

David E. Sloane, M.D., Ed.M.


Brigham and Women’s Hospital
Dana Farber Cancer Institute
West Roxbury VA Medical Center
Harvard Medical School

No Disclosures

3656
Copyright © Harvard Medical School, 2018. All Rights Reserved.

Allergy/Immunology
• Immunology background
• Allergic Rhino-conjunctivitis
• Allergic Asthma
• Angioedema and Urticaria
• Anaphylaxis
• Drug Hypersensitivity and Desensitization
• Food Allergy and Oral Immunotherapy
• Mastocytosis and Mast Cell Activation Syndromes
• Common Variable Immunodeficiency

Our world….

http://voices.nationalgeographic.com/2014/01/07/dung-beetles-use-the-sun-to-navigate/

So why do we not all die of sepsis?

3657
Copyright © Harvard Medical School, 2018. All Rights Reserved.

The Immune System as a


Matter Processing Network
Where it is from:

How bad it is:

The Mast Cell + IgE Paradigm

Slide courtesy of Dr. Tse Wen Chang

3658
Copyright © Harvard Medical School, 2018. All Rights Reserved.

Mast Cell Mediators

Simons FER et al. Risk assessment in anaphylaxis: Current and future approaches. J Allergy Clin Immunol 2007;120:S2-24.

Allergic Rhino-Conjunctivitis
• Symptoms: • Signs:
• Sneezing • Clear bilateral nasal
• Nasal congestion discharge
• Runny nose • pale and edematous
• Nasal itching turbinate mucosa
• Ocular itching, • conjunctival injection
• Increased tearing • clear to while ocular
discharge
• Palatal itching,
• hyper-lacrimation
• Ear blockage
• Ear itching

3659
Copyright © Harvard Medical School, 2018. All Rights Reserved.

Allergic Rhino-Conjunctivitis
• Importance: • Seasonal Allergens:
• Prevalence approximately 15-20% • pollens from trees, grass,
of the population weeds
• Relationship between AR and • Perennial Allergens:
asthma
• dust mites, cat/dog dander
• In one study, 28% of patients with
asthma had AR and 17% of • Diagnosis:
patients with AR had asthma • prick/epicutaneous and
intradermal skin test
• DDx:
• Infectious rhinitis (PMN’s vs.
Eos in smear), cholinergic
rhinitis

Allergic Rhino-Conjunctivitis: Treatment

• Environmental Control/Allergen Avoidance


• Medications:
• Intranasal Steroids: e.g., fluticasone, budesonide
• Intranasal Antihistamine: e.g., azelastine
• Non-sedating antihistamines:
• E.g., Loratidine, Desloratidine, Fexofenadine, Cetirizine,
Levocetirizine,
Decongestants = α adrenergic agonists
• Mast cell Stabilizers: e.g., Cromolyn
• Ocular Agents: e.g., Olpatidine, Ketotifen
• Intranasal Anticholinergics: e.g., Ipratropium
• Combined antihistamine and anti-PAF agents rupatadine

3660
Copyright © Harvard Medical School, 2018. All Rights Reserved.

Immunotherapy for Allergic


Rhino-Conjunctivitis and Asthma
Immunotherapy: Allergen Specific “Vaccine”
beneficial in randomized trials for pollens, Alternaria, dust mites, and cat
dander allergens (Durham NEJM 1999, 2008; Ross RN 2000)
high affinity and specificity pollen-biding IgG4 with IgE blocking ability (James
LK JACI 2012)
generation of Th0 and T reg responses.

Modified/Recombinant hypoallergenic proteins and


chimeras (cat Fel d1/Fc) (Saxon JACI 2011)

Anti-IgE : approved 3/2003 (omalizumab) for treatment of asthma


(NEJM 1999)
moderate to severe persistent allergic asthma
FEV1 < 80%, specific allergen IgE

Sublingual/oral Immunotherapy 2015


•Targeted at children/adolescents and patients
with fear of needles
•Tablets with recombinant major allergens
taken daily or during high pollen season for
grass, ragweed, and dust mites
•Ragweed Tablet in 1871 patients was effective
in decreasing symptoms by 27% during pollen
season (Nelson H Allergy 2013)

3661
Copyright © Harvard Medical School, 2018. All Rights Reserved.

Asthma
• A syndrome, not single disease
– Allergic
– Exercise induced
– Cough variant
• Endotypes or subtypes may co-occur
• Newer biologic therapies
– Omalizumab = anti-IgE for allergic
– Meoplizumab, Reslizumab = anti-IL-5 for eosinophilic
predominant
– Benralizumab = anti-IL-5Rα chain
– (Lebrikizumab = anti-IL-13 in patients with elevated serum periostin
concentrations)

Asthma and Aspirin Sensitivity

AERD = Aspirin Exacerbated Respiratory Disease (Samter’s triad/tetrad):


• Asthma (often steroid dependent)
• Nasal polyposis (loss of smell)
• Aspirin and NSAIDS intolerance (decreased FEV1>12% upon exposure)
• Chronic rhino-sinusitis

Pathobiology:
– NOT IgE mediated
• LT overproduction because PGE2 inhibition of 5-LO pathway (and thus LT
production) is insufficient; COX-1 inhibition further decreases PGE2 generation. LT’s
such as LTC4 produced by mast cells and eos;
• In some patients, eos overexpress LTC4 synthase
• Platelets aggregate with eos, pmns, monos;
• Low concentrations of PGE2.

3662
Copyright © Harvard Medical School, 2018. All Rights Reserved.

AERD
Management:
- Leukotriene blockade (5-LO, LTR antagonists)
- Surgery (polyps, sinuses)
- Aspirin desensitization

Case 1
• A 23-year-old man was evaluated for asthma
and bilateral pulmonary infiltrates. He had had
asthma since childhood. Although he had
occasionally received oral corticosteroids for
asthmatic exacerbations as a child, he had
never required hospitalization for asthma. Of
note is that in the past two years he had been
treated for pneumonia on two occasions. His
current medications included inhaled
corticosteroids and an oral antihistamine.

3663
Copyright © Harvard Medical School, 2018. All Rights Reserved.

Allergic Bronchopulmonary Aspergillosis


Criteria:
Asthma
Pulmonary infiltrates/central bronchiectasis
Elevated total serum IgE >1000 ng/ml
Peripheral eosinophilia
Positive skin test and RAST (IgE) to Aspergillus
Precipitins against Aspergillus

Treatment: Oral steroids, consider anti-fungals

Associations: cystic fibrosis and HLA DR2


.

3664
Copyright © Harvard Medical School, 2018. All Rights Reserved.

Urticaria and Angioedema


• Urticaria
Acute < 6 weeks , Chronic > 6 weeks
• Urticaria :
pruritic geographic macular lesions with central
clearing of short duration (<24h) and variable size
• Angioedema :
swelling ± pain, associated to urticaria in 40-50% of
cases

Acute Urticaria
A cause is found in 20% of cases:

• Drugs: aspirin and NSAIDS, Abx


• Foods: egg, milk, peanut and nuts, seafood,
shellfish
• Infections (viral/bacterial: HBV and HCV)
• Contact Allergies (animal dander and saliva; pollen
from trees, grass, weeds)

3665
Copyright © Harvard Medical School, 2018. All Rights Reserved.

Chronic Urticaria
(a cause is found in < 10% of cases)
Physical Urticarias
•Symptomatic Dermatographism
•Pressure (delayed)
•Exercise Induced
•Cold, solar, aquagenic, vibratory

Autoimmune :
IgG Anti-IgE Receptor (35-40%) or Anti-IgE (5-10%) autologous serum skin
test positive (CIU index)

Hashimoto’s Thyroiditis
elevated anti-peroxidase (TPO) and anti-microsomal antibodies, Graves
disease

Connective tissue disorders, Malignancy


SLE, leukocytoclastic vasculitis, multiple myeloma, plasmocytoma,
cryoglobulinemias

Urticaria: Treatment
• H1 antagonists nonsedating→sedating
• H2 antagonists ranitidine, famotidine,
cimetidine
• Doxepin (combined H1 and H2 antagonist)
• Leukotriene antagonists Montelukast
• Corticosteroids (alternate day ≤ 20mg)
• Others: Colchicine, Dapsone,
Hydroxychloroquine, Sulfasalazine, Cyclosporine,
Plasmapheresis, IVIG, Levothyroxine
Omalizumab Anti-IgE (Chronic idiopathic urticaria/Cold-
induced urticaria, Boyce 2007 JACI, Kaplan 2009, Maurer
2013 NEJM)

3666
Copyright © Harvard Medical School, 2018. All Rights Reserved.

Less itch

Omalizumab in CIU :
150 mg to 300 mg
every 4 weeks

Less hives

Maurer et al NEJM 2013

Case 2
22 y male with recurrent
episodes of abdominal pain
and lip swelling, not pruritic,
no associated hives, and not
responding to corticosteroids
or anti-histamines. Two other
family members have similar
episodes and one uncle died of
asphyxiation at early age

3667
Copyright © Harvard Medical School, 2018. All Rights Reserved.

Angioedema (non mast cell mediated)


Hereditary
• C1inh deficiency : Type I
decreased level due to point/frame shift mutations of the SERPIN gene
• C1inh dysfuntion: Type II
normal serum concentration but non-functional protein
C1Inh/C1Q Nl,
Type III:
Factor XII mutation
Females, familial, menstrual periods (estr/prog)
Acquired
• Inhibitor(s) of C1inh : anti-idiotypic Abs
B cell lymphomas
Excessive consumption (malignancies, connective
tissue diseases SLE, HIV )

Treatment of HAE

From http:/www.ajmc.com/journals/supplement/2013/ace010_13jun_hae_ce/ace010_13jun_lumry1_s103to10
accessed 20 November 2015

3668
Copyright © Harvard Medical School, 2018. All Rights Reserved.

Hereditary and Acquired Angioedema


• Symptoms
episodic swelling of the head, face, neck, extremities and GI (abdominal
pain, nausea and vomiting responsive to fluids and narcotics)
• Diagnosis
C4 C1INH C1q C2
Hereditary ↓ nl/↓ nl +/-
Acquired ↓ nl/↓ ↓ +/-
• Therapy :
Purified C1inh (FDA 2009): Cinryze, Berinert
Kallikrein inhibitor (Kalbitor, ecallantide, 10 mg sq X3)
Bradykinin receptor 2 Inh (Icatabant) 30 mg sq
(Cicardi et al. 2010, NEJM)
androgenic steroids (danazol, stanazolol)
epsilon amino caproic acid, tranexamic acid, FFP

Case 3: Is this anaphylaxis?


• 46 y o m from India eating at a Chinese restaurant, on no meds,
mild intermittent asthma, avoids seafood (fish allergy)
• Felt itchy and flushed after a bite of beef,
• developed urticaria, SOB, and severe wheezing
• Patient collapse within 15 min, 911 called
• 5 attempts fail to intubate due to laryngeal edema
• Epi given, dead upon arrival ED (45 mi)
• Tryptase post mortem sample: 32 ng/ml (NL: 15ng/ml)

3669
Copyright © Harvard Medical School, 2018. All Rights Reserved.

Case 4: is this anaphylaxis?


• 42 y o healthy male stung by wasp while working in yard
• Within 15 min has severe flushing, dizziness and syncope
• 911 called: BP 50/? Mm Hg
• Epinephrine IM is administered by EMT
• Full Recovery
• Tryptase 42 ng/ml in ER
• Tryptase baseline 2 months later 20 ng/ml
• Skin test positive for Hymenoptera venom

Anaphylaxis Definitions
• “A systemic, immediate hypersensitivity reaction
caused by immunoglobulin E (IgE) mediated
immunologic release of mediators from mast cells
and basophils.” (Lieberman, 2003)

• Anaphylaxis = “a severe , potentially fatal systemic


allergic reaction that occurs suddenly after contact
with an allergy-causing substance.” (Second Symposium, JACI
2006;117:391-397)

• “A serious allergic reaction that is rapid in onset and


might cause death” (Simons, 2010)

3670
Copyright © Harvard Medical School, 2018. All Rights Reserved.

Anaphylaxis Clinical Manifestations


1. Acute onset of an illness (min to hrs) with involvement of the skin and/or
mucosae (hives, pruritus, flushing) and at least one of the following
1. Respiratory compromise (dyspnea, wheeze, bronchospasm, stridor, reduced
PEF, hypoxemia)
2. Reduced BP or symptoms (syncope)
2. Two or more of the following that occur rapidly after exposure to a likely
allergen for that patient (min to hrs)
1. Skin and/or mucosae
2. Respiratory compromise
3. Reduced BP
4. GI symptoms (colic, diarrhea)
3. Reduced BP after exposure to a known allergen for that patient (min to hrs)

Table I from Sampson et al JACI 2006 117:391-397.

Frequency of Manifestations
• Cutaneous 90%
– Urticaria and Angioedema 85-90%
– Flushing 45-55%
– Pruritus 2-5%
• Respiratory 40-60%
– Dyspnea and Wheeze 45-50%
– Laryngeal Angioedema 50-60%
– Rhinitis 25-20%
• Dizziness, syncope, hypotension 30-35%
• Abdominal (n, v, colic, diarrhea) 25-30%
• Misc (HA 5-8%, SSCP 4-6%, Sz 1-2%)

Liberman P et al JACI 2005;115(3):S483-S523

3671
Copyright © Harvard Medical School, 2018. All Rights Reserved.

TRYPTASE

TRYPTASE

Swartz 1987 NEJM, 2006; Stevens 2007, Prieto Garcia 2013

Anaphylaxis: Epidemiology
Fatal Anaphylaxis:
Previously “Severe Anaphylaxis” (Sampson Asthma 1992, NEJM, 2006)
Allergy to: Peanuts, Tree Nuts, Fish or Shellfish (transgenic foods)
Patients on ß-Blockers/ACE-inhs

Non Fatal Anaphylaxis:


1/2700 hospitalizations
repeated antibiotics exposure (CF)
0.5% Hymenoptera Stings
1/10,000 RCM Exposure
1% PCN and antibiotics beta lactams and others
1/5,000-25,000 General Anesthetic Exposure
1/3,000-5,000 Hemodialyses

3672
Copyright © Harvard Medical School, 2018. All Rights Reserved.

Anaphylaxis: Pathobiology
IgE /Mast Cell mediated
Foods: Peanuts, Tree Nuts, Seafood, Eggs, Milk
Allergen Extracts, Vaccines, antisera
Hymenoptera Venom: Bees, Wasps, Yellow Jackets, Hornets, Fire Ants
Hormones (Progesterone Autoimmune Dermatitis), Enzymes
Monoclonal Abs :anti-TNFa, anti-CD20
Chemotherapy: platins, taxenes
Beta lactams, ASA and other NSAIDS (COX1>COX2), vancomycin (red person syndrome)
Radio Contrast Media, opiates (direct mast cell activators)
Anesthetics: Curare Derivatives
Dialysis membranes

Anaphylaxis: Diagnosis
Acute :
Tryptase : total >11ng/ml, mature > 1 ng/ml
N-Methyl Histamine in 24h urine collection
Prostaglandin D2 metabolites (PG 11-β-F2-α) in urine
Retrospective :
Antigen-Specific IgE
Specific IgE in serum (in vitro)
Skin Testing (in vivo)
Basophil activation FACS: CD69/CD203

3673
Copyright © Harvard Medical School, 2018. All Rights Reserved.

Management of
Anaphylaxis
Epinephrine IM (not sq) 0.3-0.5cc
recumbent position, quadriceps
Observation for a Minimum of 6 Hours
Obtain a serum Tryptase
Oxygen
Anti-histamines H1 and H2,
Steroids: single dose (IV or oral)
Delayed, protracted anaphylaxis may occur 6 to 24 hours; late phase or
secondary reaction requires repeat Epi in 16-36% of patients.
If ß Blockade: Glucagon 5 - 15 µg/min IV (after trying epi)
ACE inhibitors are implicated in severe/refractory
Education
Allergy evaluation
Auto injectable epinephrine

World Allergy Organization anaphylaxis guidelines: summary. Simmons, F, et al. JACI, 2011;
127: 587-593

3674
Copyright © Harvard Medical School, 2018. All Rights Reserved.

Epi: Take the Right Route

Exercise Induced Anaphylaxis


Associated with food allergy in >30% cases (wheat)
• Specific IgE, Skin Test Food
Management:
Discontinue Exercise Earliest Symptom : Flushing, Pruritus
• Limit Exercise on Hot, Humid Days,
• Exercise with a companion
• Avoid Exercise 4-6 hrs Post Prandial
• Avoid Exercise Post Allergy Immunotherapy
• Avoid Beta-Blockers and ACE Inhibitors
• Use Epinephrine promptly
• Have a Medi-Alert Bracelet

3675
Copyright © Harvard Medical School, 2018. All Rights Reserved.

Drug Hypersensitivity
Common Drugs:
PCN and related Abx: IgE-mediated
- Cross reactivity with cephalosporins (10 % first generation, 1-2% 3th -4th
generation)
aztreonam is non- crossreactive (except ceftazidime)
ASA and NSAID’s: COX-1/COX-2 blockade, universal cross-reactivity
ACE-I (Kininase II): Bradykinin Mediated angioedema
Sulfonamides: no cross-reactivity with non-antibiotic
medications (NEJM 2007)
Diagnosis:
• Skin test (Pre-Pen for PCN: FDA 2010)
• Challenge
Management
Avoidance, MediAlert Bracelets
Desensitization (Antibiotics, Aspirin,Chemotherapy, Mo)

MRGPRX2 Human Receptor for Peptidergic Drugs


Mrgprb2 mediates mast cell
responsiveness and side effects
of peptidergic therapeutic drugs
(Icatibant, ciprofloxacin,
rocuronium, atracuronium).

a, Percentage of responding cells


from wild-type (WT) and
Mrgprb2MUT (MUT) peritoneal mast
cells after drug

b, Left, representative images of


Evans blue stained extravasation
15 min after intraplantar injection
of icatibant. Right, quantification of
Evans blue leakage into the paw
after 15 min.

c, Total histamine release from


wild-type (red diamonds) and
Mrgprb2MUT (black squares) mice
after incubation with named
substances.

Note that no significant difference


between wild-type and
Mrgprb2MUT cells
was found at any dose of anti-IgE
antibody.

Nature 2015

3676
Copyright © Harvard Medical School, 2018. All Rights Reserved.

Mrgprb2 mediates mast cell


responsiveness and side effects of
small molecules therapeutic drugs.

a, Structures of 48/80 and a cyclized


variant. The THIQ motif is highlighted in
blue.

b, Structures of representative members of


all NMBD classes. THIQ
motifs are highlighted in blue. Note that
only succinylcholine lacks a bulky
hydrophobic group.

c, Percentage of responding cells from wild-


type(WT) and Mrgprb2MUT(MUT) peritoneal
mast cells after application of various
NMBDs.

d, Structure of ciprofloxacin,
with the motif common to all
fluoroquinolones highlighted in blue.

e, Percentage of responding cells


from wild-type and Mrgprb2MUT peritoneal
mast cells after fluoroquinolone

f, Changes in n body temperature after


intravenous injection of ciprofloxacin (1.5
mg in125 ml saline) at time 0. n 54 mice
per genotype.

Drug Desensitization
First line therapy for all patients
Prolonged life span (cancer)
Increased QOL (chronic inflammatory diseases)
Protection against anaphylaxis
• Is desensitization for everybody (risks/indications)?
• Can all drugs be desensitized (chemo, MoAbs, antibiotics, ASA)?
• What are the indications (symptoms)?
• What are the contraindications (severe cutaneous, beta blockers)?
• Are they safe (fatalities)?
• What are the outcomes (effectiveness)?

3677
Copyright © Harvard Medical School, 2018. All Rights Reserved.

Castells et al 2008, Sloane et al 2016

PAIN

Safety of Rapid Desensitizations 413 cases and 2177 cases


Castells et al. JACI 2008
Sloane et al JACI IP 2016

Mild Reaction: 27%


(111/413)

Severe Reaction: 6%
No Reaction: 67% (24/413)
(278/413)

94 % of cases with mild or no reactions


No deaths

3678
Copyright © Harvard Medical School, 2018. All Rights Reserved.

Paclitaxel Desensitization
Feldweg A et al Gyn Onc 2005

Step Solution Rate Time (min) Administered dose Cumulative dose (mg)
1 1 1 15 0.0003 0.0003
2 1 2 15 0.0006 0.0009
3 1 5 15 0.0015 0.0024
4 1 10 15 0.0030 0.0054
5 2 2 15 0.0060 0.0060
6 2 5 15 0.0150 0.0210
7 2 10 15 0.0300 0.0510
8 2 20 15 0.0600 0.1110
9 3 5 15 0.1500 0.2610
10 3 10 15 0.3000 0.5610
11 3 20 15 0.6000 1.1610
12 3 40 15 1.2000 2.3610
13 4 10 15 2.9764 5.3374
14 4 20 15 5.9528 11.2902
15 4 40 15 11.9056 23.1957
16 4 75 186 276.8043 300.0000
-----------------------------------------
Total time = 411 minutes

Non-IgE Hypersensitivity Reactions to


Medications
• DRESS syndrome:
eosinophlia, rash, systemic symptoms, LAN, LFT elevations
anti-convulsants: cross-reactivity is high (HLA-B* 1502)
phenytoin, phenobarbital, carbamazepine
role of HHV6-7 reactivation
• Delayed Maculopapular Rashes: EM/SJS/TEN
sufonamides, beta-lactams
Abacavir: fever, rash, systemic involvement (HLA-B*5701)
Quinolones: universal cross-reactivity

3679
Copyright © Harvard Medical School, 2018. All Rights Reserved.

Specific HLA Alleles in Drug


Hypersensitivity
•Abacavir (reverse transcriptase termination
HIV) :
HLA-B 57-01 exclusive

•Carbamazepine
(anti-convulsant):
HLA-B 15:02

Carbamazepine induced
Stevens-Johnson Syndrome
Toxic Epidermal Necrolysis

Abacavir Hypersensitivity

3680
Copyright © Harvard Medical School, 2018. All Rights Reserved.

The New York Times Magazine


June 7th, 2009
•“Mommy, I’m afraid”(impending sense of doom)
• The mother, a 32 yo W, was found lying
unconscious in a public bathroom in a pool of
bloody stool after feeling, hot, dizzy and
having a sensation of “fluttering heart.”
• She told the ER doctors that her only medical
problem was occasional panic attacks, flushing
and a rash.

Urticaria Pigmentosa

Darier’s sign

3681
Copyright © Harvard Medical School, 2018. All Rights Reserved.

Systemic Mastocytosis
Diagnostic Criteria
Major Criteria :
multifocal infiltrates of 15 or more mast cells
in bone marrow and/or extracutaneous organs
Minor Criteria:
1. > 25% spindle shaped mast cells
2. c-kit mutations (codon D816V)
3. aberrant expression of CD2 and CD25
4. Tryptase >20 ng/ml

Systemic Mastocytosis
Spindle shape mast cells
Mast cell aggregates

Tryptase staining

KIT mutation D816V


Aberrant CD25 and CD2

3682
Copyright © Harvard Medical School, 2018. All Rights Reserved.

Classification of Disease associated with Mast


Cell Activation (Akin, Metcalf, Valent JACI 2010)

Non Clonal Mast Cell Activation


Syndrome
Hamilton et al JACI 2011

Go back to case 4

3683
Copyright © Harvard Medical School, 2018. All Rights Reserved.

Common Variable Immunodeficiency


• Sinusitis (>2) , Pneumonia (>2), UTI (>3-4), Deep seated
infections
• Low IgG, IgA, IgM (– 2 SD)
• Poor response to vaccines : Pneumovax, H.Influenza,
Hepatitis, Strep pneumo (Prevnar-13)
• Gammaglobulin replacement:
IVIG or SQ 400mg/kg q 3-4 weeks
(TACI defect, association with autoimmune diseases and
lymphoma)

Summary Pearls for the IM Boards


• Allergic Rhino-conjunctivitis: IT
• Asthma/AERD: ASA desensitization
• Allergic Bronchopulmonary Aspergillosis: IgE
• Angioedema and Urticaria : C4, TPO
• Anaphylaxis: TRYPTASE
• Adverse Drug Reactions/Desensitization: Pre-Pen
• Mastocytosis: c-KIT D816V mutation
• Common Variable Immunodeficiency: IgG

3684
Copyright © Harvard Medical School, 2018. All Rights Reserved.

Board Question 1
• An 18 yo patient presents with an episode of
angiodema of face and abdominal pain. He has no
itching or associated hives. Which test would be
most appropriate?
a) Serum tryptase
b) Serum C4
c) CT abdomen
d) CBC

Board Question 1
• An 18 yo patient presents with an episode of
angiodema of face and abdominal pain. He has no
itching or associated hives. Which test would be
most appropriate?
a) Serum tryptase
b) Serum C4
c) CT abdomen
d) CBC
Explanation: The clinical presentation of angioedema without
associated pruritus, in conjunction with this family history, suggests
hereditary angioedema (HAE). The best single test for HAE is the
serum C4 concentration.

3685
Copyright © Harvard Medical School, 2018. All Rights Reserved.

Board Question 2
• A 50 yo male with CAD presents to the ER with
hypotension , tachycardia, SOB and
generalized hives after a wasp sting. The most
important action in the acute setting is:
a) Perform skin test to Hymenoptera venom
b) Obtain a serum tryptase
c) Administer epinephrine
d) Administer venom immunotherapy

Board Question 2
• A 50 yo male with CAD presents to the ER with
hypotension , tachycardia, SOB and
generalized hives after a wasp sting. The most
important action in the acute setting is:
a) Perform skin test to Hymenoptera venom
b) Obtain a serum tryptase
c) Administer epinephrine
d) Administer venom immunotherapy
Explanation: While all these measures are appropriate, the most
pressing issue acutely is patient safety. Timely administration of
epinephrine is life saving, while a delay in administration is the most
important risk factor for death from anaphylaxis.

3686
Copyright © Harvard Medical School, 2018. All Rights Reserved.

References
• Castells et al JACI 2008 , 2012 Desensitizations
• Boyce et al JACI 2009 Xolair use in Urticaria, Maurer
2013
• Escribano et all 2009 Mastocytosis
• Durham et al NEJM 2008 Immunotherapy for AR
• Cunningham Rundles 2007 Common Variable
immunodefiency
• Simons FE 2011 : World Allergy Organization
anaphylaxis guidelines: J Allergy Clin Immunol
• LEAP peanut allergy study NEJM 2015
• Mast Cell Anaphylactoid receptor Nature 2015

No Disclosures

3687
Copyright © Harvard Medical School, 2018. All Rights Reserved.

Depression Update
Russell G. Vasile MD
Director, Affective Disorders Consultation Service,
Department of Psychiatry
Beth Israel Deaconess Medical Center
Associate Professor of Psychiatry
Harvard Medical School

Psychiatry Overview

Disclosures

No Disclosures

3688
Copyright © Harvard Medical School, 2018. All Rights Reserved.

Learning Objectives

• Understand the spectrum of depressive


disorders and treatment implications

• Appreciate recent developments in the


psychopharmacologic management of
depression, including refractory
depression

• Enhance skills in suicide risk assessment

Emerging Issues - 2018


• Suicide Risk and antidepressants – the
role of occult bipolar depression
• Treatment Resistant Depression –
Augment, Switch, Combine?
• Novel neuroleptic/antipsychotic
medications as antidepressant boosters
• What role for TMS and brain stimulation
techniques?

3689
Copyright © Harvard Medical School, 2018. All Rights Reserved.

Prevalence of Mood Disorders

1 Year (%) Lifetime (%)


Major Depressive
10.3 17.1
Episode
Manic Episode 1.3 1.6

Dysthymia 2.5 6.4

Any Mood Disorder 11.3 19.3

Kessler RC, et al. Arch Gen Psychiatry. 1994;51:8.

DSM- 5 Classification
of Mood Disorders
Mood disorders

Bipolar disorders Depressive disorders

Bipolar I CyclothymicBipolar Major Dysthymic Depressive


disorder disorder disorder depressive disorder disorder
NOS disorder NOS

Bipolar II Single Recurrent


disorder episode

3690
Copyright © Harvard Medical School, 2018. All Rights Reserved.

Screening for Mood Disorders


• Patient Health Questionnaire 2 (PHQ-2);longer
version PHQ-9
• Quick Inventory of Depressive Symptoms
• Beck Depression Rating Scale
– Self-report
• Hamilton Rating Scale for Depression
– Clinician administered
• Young Mania Rating Scale
• Columbia Classification Algorithm for Suicide
Assessment ( C-CASA)

PHQ-2
• Little interest or pleasure in doing things
• Feeling down and depressed or hopeless

0 - Not at all
1 - Several days
2 - More than half the days
3 - Nearly every day

Score of 6 or more correlates 78% with major


depression.

3691
Copyright © Harvard Medical School, 2018. All Rights Reserved.

Subtypes of
Depressive Disorders
• Bipolar/Unipolar
– Rapid Cycling as a sub-variant
• Melancholic/Non-Melancholic
• Psychotic/Non-Psychotic
• Agitated/Retarded
• Responsive/Non-responsive
• Atypical
– Rejection Sensitive Dysphoria
• Seasonal Affective Disorder

DSM-5 Criteria for


Melancholic Features
A. Either of the following, occurring during the
most severe period of the current episode:
1. Loss of pleasure in all, or almost all, activities
2. Lack of reactivity to usually pleasurable stimuli
B. Three or more of the following:
1) Distinct quality of depressed mood (ie, different from
feelings experienced after loved one’s death)
2) Depression regularly worse in the morning
3) Early morning awakening
4) Marked psychomotor retardation or agitation
5) Significant anorexia or weight loss
6) Excessive or inappropriate guilt

3692
Copyright © Harvard Medical School, 2018. All Rights Reserved.

Recurrence is Common
Rate of recurrence per episode

100 90
80
Recurrence Rate (%)

70
60 50
40

20

0
After 1 After 2 After 3
Depressive Episode(s)

DSM-IV: 1994;341-342.

3693
Copyright © Harvard Medical School, 2018. All Rights Reserved.

Detecting Bipolar Disorder


• History of mood elevation
• Family history of bipolar disorder
• Sub-syndromal mood elevation
• Mixed states/racing thoughts
• Antidepressant associated mania
• Response to mood stabilizers
• No biological markers established

NIMH Collaborative Depression Study


-Depression as the Unmet-Need

Wks depressed Wks manic


BP I 31% 10%
n=135

BP II 52% 1.4%
n=71

10 years follow-up; BP II-- greater chronicity and comorbidity


Judd et al., Arch Gen Psych 2002; 59:- 530 537

3694
Copyright © Harvard Medical School, 2018. All Rights Reserved.

Factors Affecting the Choice


of Antidepressant
• Side effect profile

• History of previous response

• Family history of response

• Safety in overdose

• Differential effects in subtypes


of depression

3695
Copyright © Harvard Medical School, 2018. All Rights Reserved.

Selecting Among
Antidepressants
• Initial choices – SSRI/SNRI
– Half-life considerations
– Activation – sedation
• Secondary options
– Tricyclic antidepressants
– Mirtazapine
– Bupropion
• MAO-inhibitors
– Phenelzine, Tranylcypromine

Treatment Recommendations
• In first episode patients
– Use maximum tolerated dosage, Continue
treatment for 6 months following remission.
Discontinue medications gradually over 2-4 weeks.

• In patients with recurrent depression


– Maintenance treatment for patients with three or
more episodes of depression, or two severely
disabling episodes.

3696
Copyright © Harvard Medical School, 2018. All Rights Reserved.

Differences between the SSRIs


• Half-life of fluoxetine much greater than paroxetine

• Inhibition of P450 2D6 enzymes much greater with


paroxetine and fluoxetine as compared to sertraline or
citalopram

• Inhibition of P450 34A


– Fluvoxamine delays clearance of alprazolam

• Inhibition of P450 1A2, 3A4, 2C9


– Fluvoxamine delays clearance of warfarin,
theophylline,propranolol, clozapine

SSRI Side Effects


• Jitters, fatigue, insomnia, headache
• Sexual dysfunction, erectile dysfunction,
anorgasmia
• Weight gain may occur
• Rare risk of GI bleeding possibly due to
inhibition of platelet function
• Inappropriate SIADH in the elderly
• Switch to mania, agitation
• Osteoporosis, risk of non-vertebral fracture in
elderly (Diem et al, Arch Intern Med, 2007)

3697
Copyright © Harvard Medical School, 2018. All Rights Reserved.

Serotonin Syndrome
• GI – cramping, diarrhea, bloating
• Neurological – Tremor, dysarthria
• Cardiovascular –tachycardia, hypertension
• Psychiatric – confusion, mania, restless

• Medications that may contribute:


– Isoniazid, Linezolid
– Tramadol, Dextromorphan, St. John’s Wort

SSRI Withdrawal
• CNS Symptoms
– Sleep disturbance, vivid dreams
– Anxiety, restlessness
– Headache

• Parasympathetic Symptoms
– Sweating, sialorrhea
– Nausea, vomiting, cramps, diarrhea

3698
Copyright © Harvard Medical School, 2018. All Rights Reserved.

Citalopram
• Highly selective SSRI; Minimal
NE/Dopamine activity
• Dosage range 10 – 40 mg; QT
prolongation FDA warning
• Metabolized by liver; No active metabolites
• SSRI side effects generally well tolerated
• Rare side-effects – hyponatremia, SIADH
• No in-patient depression studies
conducted

Escitalopram (Lexapro)
• Active S- enantiomer of racemic citalopram – twice as
potent as the racemic mixture
• Absorption unaffected by food
• Plasma peak at 5 hours –steady state one week
• 10 mg/day standard dosage
• Little effect on CYP isoenzymes
• Relatively few medication interactions
• In large studies, 10 or 20 mg/day of Escitalopram were
as effective as 40 mg/day of citalopram.
• Theoretical advantages over citalopram, possibly fewer
side effects.

3699
Copyright © Harvard Medical School, 2018. All Rights Reserved.

Venlafaxine
• SNRI – serotonin effects at lower dosages;
• Norepinephrine effects at higher dosages
• Dosage range – 75 mg – 300 mg
• Monitor BP at higher dosage range
• FDA approved for generalized anxiety and major
depression.
• Probable greater antidepressant efficacy at
higher dosage
• Extended release formulation available

Wellbutrin (Bupropion)
• Dopaminergic/Noradrenergic agonist
• Stimulating antidepressant -75 -375 mg qd
– Sustained and Extended Release options
• No sexual dysfunction
• Contraindicated in patients with seizures
• Effective in ADHD
• Avoid concurrent use of stimulants
• Insomnia may be a side-effect
• Limited anti-anxiety properties; may cause
anxiety

3700
Copyright © Harvard Medical School, 2018. All Rights Reserved.

Duloxetine (Cymbalta)
• SNRI Dosage range 20-30 mg bid; 60-90
mg in refractory patients
• Avoid use in patients with renal or hepatic
impairment
• May increase anti-arrythmic blood levels
• May be useful in pain syndromes
• Side –effects – dry mouth, nausea,
constipation, dizziness, fatigue
• Avoid in pregnancy

Mirtazapine
• Alternative to SSRI/SNRI
• Less sexual dysfunction
• Sedation and weight gain side-effects
• Anti-anxiety properties
• Dosage range 15-60 mg qd
• Excellent for sleepless, underweight
patients, including elderly
• NE and 5HT1 agonist

3701
Copyright © Harvard Medical School, 2018. All Rights Reserved.

Heterogeneity of Treatment
Resistant Depression
• Bipolar Depression and Latent Bipolar
• Axis II Co-morbidity
• Substance Abuse
• Anxiety Disorders
• Trauma, Abuse and Psychosocial Crisis
• Occult Medical Disorders
• Undiagnosed Sleep Apnea
• Schizoaffective, Schizophrenia Spectrum
Adapted from Fagiolini and Kupfer, Biol Psychiatry 2003;53:640-648

Standard MAOIs are


Particularly Effective for:
• Atypical Depression
• Resistant Depression
• Elderly Depressives (maintenance effects)
• Social Phobia
• Neuroticism
• Interpersonal Hypersensitivity
• Phobic Avoidance

3702
Copyright © Harvard Medical School, 2018. All Rights Reserved.

Augmentation Medication
Strategies for TRD
• Lithium – best established (7/9 controlled studies)
Lower blood levels -0.4-0.6 meq/l effective
• T3 – 25-50 mcg. Efficacy established with TCAs
• Novel neuroleptics – Aripiprazole (Abilify), 5-10
mg (FDA approved)
• Olanzapine-Fluoxetine combination in bipolar
depression

Switch Strategies for TRD


• Changes within class – uncontrolled
studies show variable response rates
• SSRI to TCA – 40% response in open
study (SSRI to nortriptylene)
• SSRI to Venlafaxine (43% remission)
• SSRI to Mirtazapine – open study after 8
weeks, 48% respond

3703
Copyright © Harvard Medical School, 2018. All Rights Reserved.

Atypical Antipsychotics in TRD


• Aripiprazole Augmentation; Brexpiprazole
augmentation dosage 2-4 mg qd
• SSRI augmentation with risperidone or
ziprasidone-may facilitate response
• Lurasidone and quetiapine for bipolar
depression; Cariprazine dosage 1.5 mg qd
• Olanzapine – fluoxetine combination –
large meta analysis demonstrates efficacy
in TRD

Lurasidone for Bipolar I Depression

• Lurasidone dosage 20-120 mg qd


• Studied as adjunctive agent to lithium or
Valproate in bipolar depression
• Studied as monotherapy for bipolar
depression – 53% response to low dose
therapy (20-60 mg qd); 51% response
for high dose (80-120) versus 30% for
placebo
• Also effective utilized adjunctively with
lithium or Depakote

3704
Copyright © Harvard Medical School, 2018. All Rights Reserved.

Lurasidone for bipolar depression

• Side effects –Nausea, somnolence, headache,


akasthesia
• Treatment emergent suicidality 9% versus 6%
for placebo in adjunctive study; 14 % in
monotherapy for all three groups.
• Minimal changes in lipids, weight and glycemic
control
• Significantly more patients achieved remission
in the lurasidone treated group
• Treatment emergent hypomania not significant

Electroconvulsive Therapy
• Key indications – Psychotic Depression, Suicidal Press,
Food Refusal, Treatment Refractory Depression,
Parkinson’s Disease, Refractory Manic Excitement.

• Medical Concerns – 4 deaths/100,000 treatments.


Cardiac risks- recent myocardial infarction, unstable
angina, hypertension, atrial fibrillation, post-
cerebrovascular accident, intracranial aneurysm or
space-occupying lesions or other causes of increased
intracranial pressure.

• Post ECT maintenance – High rate of relapse particularly


with psychotic depression; Post ECT medications -
nortriptylene and lithium, MAO-I, Maintenance ECT.

3705
Copyright © Harvard Medical School, 2018. All Rights Reserved.

Suicidal Behavior and Depression


• 20-40% of patients with an affective disorder
exhibit nonfatal suicidal behaviors, including
thoughts of suicide1
• Estimates associate 16,000 suicides in the US
annually with depressive disorder2
• 18% of those with a history of major depressive
disorder (MDD) attempt suicide2
• 15% of patients with severe primary MDD of at
least 1 month’s duration eventually commit
suicide2
1. Review of Psychiatry. Washington, DC: American Psychiatric Press; 1988;7:353-385
2. Am J Psychiatry. 1988;145:1351-1357.

Risk Factors for Suicide


• Previous Attempts
• Psychosis
• Major Depression or Bipolar Diagnosis
• Alcohol or Drug Abuse
• Losses, deaths, shame, poverty
• Social isolation, unmarried, homosexual
• Lack of access to clinical care
• Access to firearms, toxins, medicines
• History of violence or impulsivity
• Prominent anxiety or agitation

3706
Copyright © Harvard Medical School, 2018. All Rights Reserved.

Suicide Risk Assessment


• Press
– The sense of urgency to die
• Perturbation
– The degree of stress imposed on the patient
• Pain
– The subjective sense of anguish
• Risk-Rescue Assessment
– Previous attempts-assessment of lethality
• Guns, jumping, hanging, drowning are particularly
malignant
– Previous attempts-assessment of attempts to obtain help
• Psychosis
– Command hallucinations
• Personality Disorder, Depression, Substance Abuse

From Schneidman, Multidimensional Approach to Suicide, In Suicide:


Understanding and Responding, IU Press, 1989

Antidepressants and Suicide Risk


• Increased risk of suicidal ideation and behavior
in children, adolescents and adults age 24 and
younger
• No effect on suicidal ideation or behavior in
adults age 25-65
• Reduction in suicidal ideation and behavior in
adults 65 and older.
• Risk appears secondary to emerging hypomania
or agitation

Stone et al. BMJ 2009:339 2880


Liberon Neuropsychopharmacology:35 1619

3707
Copyright © Harvard Medical School, 2018. All Rights Reserved.

Summary – Key Points


• Assess the Depressive Spectrum –
Appreciate the longitudinal history and co-
morbidity
• Watch for latent bipolarity
• Careful follow-up to assess early side-
effect experience –particularly in young
adults and adolescents
• Watch for latent psychosis in the elderly
• Follow-up phone calls after beginning
antidepressants - Outreach

Question 1
• Which of the following is not an appropriate
treatment for bipolar depression?
A. Lurasidone
B. Lithium Carbonate
C. Buproprion as a stand alone medication
D. Low dose anti-depressant covered by a mood
stabilizer anti-manic medication.
E. Quetiapine in a dosage of 300 mg per day.

ANSWER : C
Antidepressants can trigger mania if not combined
with a mood stabilizer.

3708
Copyright © Harvard Medical School, 2018. All Rights Reserved.

Question 2

• What is the most rapid and effective


treatment for psychotic depression with
command hallucinations in the elderly?
• A. Antidepressant medication
• B. Antipsychotic medication
• C. Electroconvulsive therapy
• D. Antidepressant combined with
antipsychotic medication
• ANSWER: C
• Antidepressant medications may take 2-4 weeks to
become effective.

References

• Chandler V: Google and suicides; what can we learn about the use of the internet to
prevent suicides. Public Health 154 (2018) 144-150

• McIntyre RS et al: Treatment-resistant depression: Definitions, review of the evidence


and algorithmic approach. Journal of Affective Disorders 156 (2014); 1-7

• Gobbi et al. Antidepressant combination versus antidepressant plus second-


generation antipsychotic augmentation in treatment-resistant unipolar depression.
International Clinical Psychopharmacology 2018, 33:34-43.

• Nelson, J. Craig: Adjunctive Ziprasidone in Major Depression and the Current Status
of Adjunctive Atypical Antipsychotics. Am J Psychiatry 2015;172: 1176-1178

• Loebel et. Al.: Lurasidone as adjunctive therapy with lithium or valproate for the
treatment of bipolar depression; A randomized, double blind, placebo-controlled
study. Am J Psychiatry 2014; 171:169-177

3709
Copyright © Harvard Medical School, 2018. All Rights Reserved.

Disclosures

No Disclosures

3710
Copyright © Harvard Medical School, 2018. All Rights Reserved.

Diagnostic Errors in Medicine


Gordon D. Schiff MD
Associate Director Center for Patient Safety Research and Practice
Brigham and Women's Hospital Div. General Medicine

Safety Director – Harvard Center for Primary Care


Academic Improvement Collaborative

Associate Professor of Medicine Harvard Medical School

Financial Conflicts/Disclosures
• None relevant to talk

• Commercial
– None related (Medaware software evaluation)
• Other/Grant Funding
– CRICO Malpractice Grants–Diagnostic Errors/Pitfalls
– Gordon & Betty Moore Foundation- Diagnostic Error Projects
– SIDM/PCORI Research Mentor honorarium
– AHRQ –HIT Safety Grant –Drug Indications
– Gold Foundation- Boundaries Issues
2

3711
Copyright © Harvard Medical School, 2018. All Rights Reserved.

7 Key Points
1. Dx Errors frequent, important, yet underappreciated
2. Growing interest, recognition
Especially 2o NAM Improving Dx 2016 Report
3. Traditional approaches limited
Low leverage, misapplied, often counterproductive
4. HIT: important role in preventing as well as causing
5. Metrics: elusive, illusive; Need for new Culture
6. More Conservative Dx: not counter to missing Dx
7. Patients: ↑ engagement
Enhanced role understanding uncertainty, co-producing dx

3712
Copyright © Harvard Medical School, 2018. All Rights Reserved.

21% Experienced Medical Error


IHI/NPSF 2017
Survey

3713
Copyright © Harvard Medical School, 2018. All Rights Reserved.

Misdiagnosis
Leading
Type of Error

Patient
Identified
Factors

3714
Copyright © Harvard Medical School, 2018. All Rights Reserved.

Patient
Identified
Factors

10
Schiff et al JAMA Intern Med 2013

3715
Copyright © Harvard Medical School, 2018. All Rights Reserved.

3716
Copyright © Harvard Medical School, 2018. All Rights Reserved.

Diagnosis-Related Closed Claims

Top Allegation Categories

13 Selection: N=10693 closed claims from 2013-2017

Diagnosis-Related Closed Claims

Top Allegation Details

Selection: N=3466 closed claims from 2013-2017 with a Diagnosis-Related allegation

14

3717
Copyright © Harvard Medical School, 2018. All Rights Reserved.

Diagnosis-Related Closed Claims

Injury Severity

Selection: N=3466 closed claims from 2013-2017 with a Diagnosis-Related allegation


Footnote: Injury severity based on National Association of Insurance Commission (NAIC) codes.

15

Diagnosis-Related Closed Claims

Top Specialties

Selection: N=3466 closed claims from 2013-2017 with a Diagnosis-Related allegation

16

3718
Copyright © Harvard Medical School, 2018. All Rights Reserved.

Diagnosis-Related Closed Claims

Top Risk Management Categories

17

Top Risk Management Details – Diagnosis-Related Closed Claims


Clinical Judgment

Clinical Judgment – Percentage of Claims

21% 19% 17% 10% 8%


Narrow Inadequate / Misinterpretation Failure to Failure to
Diagnostic Inappropriate of Diagnostic Adequately Obtain
Focus Testing Studies Assess Specialty
Patient’s Consult or
Condition Referral

Selection: Closed PL claims from 2013-2017, N=1840 with a Diagnosis-Related allegation and a Clinical Judgment
Risk Management issue

18

3719
Copyright © Harvard Medical School, 2018. All Rights Reserved.

Schiff & Graber Diagnosis Errors in Acute Care Setting. Principles and Practice of Hospital Medicine McGraw Hill 2012

Top Risk Management Details – Diagnosis-Related Closed Claims


Clinical Systems

Clinical Systems – Percentage of Claims

17% 16% 13% 10% 8%


Failure to Manage Insufficient Results/Specimen Lack of Failure to Inform
Patient’s Follow- Studies (e.g., Lost, Misfiled, or Coordination of Patient of Test
up Care Imaging) MD Unaware of Care Results
Results

Selection: Closed PL claims from 2013-2017, N=550 with a Diagnosis-Related allegation and a Clinical Systems
Risk Management issue

20

3720
Copyright © Harvard Medical School, 2018. All Rights Reserved.

IOM Report
September
2015

8 IOM Goals to Improve Diagnosis and


Reduce Diagnostic Error
GOAL 1 Facilitate more effective teamwork in the diagnostic process among
health care professionals, patients, and their families

GOAL 2 Enhance health care professional education and training in the


diagnostic process

GOAL 3 Ensure that health information technologies support patients and


health care professionals in the diagnostic process

GOAL 4 Develop and deploy approaches to identify, learn from, and reduce
diagnostic errors and near misses in clinical practice

3721
Copyright © Harvard Medical School, 2018. All Rights Reserved.

8 IOM Goals to Improve Diagnosis and


Reduce Diagnostic Error
GOAL 5 Establish a work system and culture that supports the diagnostic
process and improvements in diagnostic performance

GOAL 6 Develop a reporting environment and medical liability system that


facilitates improved diagnosis through learning from diagnostic errors
and near misses

GOAL 7 Design a payment and care delivery environment that supports the
diagnostic process

GOAL 8 Provide dedicated funding for research on the diagnostic process and
diagnostic errors

3722
Copyright © Harvard Medical School, 2018. All Rights Reserved.

What is a Diagnosis Error?

Adverse
Outcomes Diagnostic
Process
Failures
Delayed,
Missed,
Misdiagnosis
Modified from
Schiff Advances in Patient Safety AHRQ 2005,
Schiff & Leape Acad Med 2012

3723
Copyright © Harvard Medical School, 2018. All Rights Reserved.

Marshal Wolf
Brigham

27

Sherlock Holmes
Dr. Gregory House

28

3724
Copyright © Harvard Medical School, 2018. All Rights Reserved.

Don Berwick

Former President and CEO


Institute for Healthcare
Improvement (IHI)
Former Director Centers for
Medicare & Medicaid Services

29

Genius diagnosticians make great stories,


but they don't make great health care.
The idea is to make accuracy reliable,
not heroic

Don Berwick
Boston Globe 7/14/2002
30

3725
Copyright © Harvard Medical School, 2018. All Rights Reserved.

2 Key Improvement Concepts

• Situational Awareness
• Safety Nets

Diagnostic Risk
Situational Awareness
• Specialized type of situational awareness
• High reliability organizations/theory
– High worry anticipation of what can go wrong
– Preoccupied w/ risks recognizing/preventing
• Appreciation diagnosis uncertainty, limitations
– Limitations of tests, systems’ vulnerabilities
– Knowing when “over head” need for help
• Making failures visible
• Don’t miss diagnoses, red flag symptoms
• Diagnostic pitfalls – potentially useful construct

32

3726
Copyright © Harvard Medical School, 2018. All Rights Reserved.

• Perhaps the most important distinguishing feature of


high-reliability organizations is their collective
preoccupation with the possibility of failure. They
expect to make errors and train their workforce to
recognize and recover them. They continually
rehearse familiar scenarios of failure and strive hard
to imagine novel ones. Instead of isolating failures,
they generalize them. Instead of making local repairs,
they look for system reforms

Reason Human error: models and management West J Med. 2000;

What is a Diagnostic Pitfall?

Need for/Value of Situational Awareness

3727
Copyright © Harvard Medical School, 2018. All Rights Reserved.

GENERIC TYPES of PITFALLS


• Disease A repeatedly mistaken for Disease B
• Bipolar disease mistaken for depression
• Failure to appreciate test/exam limitations
• Pt w/ breast lump and negative mammogram and/or ultrasound
• Atypical presentation
• Addison’s disease presenting with cognitive difficulties
• Presuming chronic disease accounts for new symptoms
• Lung cancer: failure to pursue new/unresolving pulmonary sx in patient
with pre-existing COPD
• Overlooking drug, other environmental cause
• Pancreatitis from drug; carbon monoxide toxicity fail to consider
• Failure to monitor evolving symptom
• Normal imagining shortly after head injury, but chronic subdural
hematoma later develops

BREAST CANCER PITFALLS: MALPRACTICE CASES

Pitfall N Example
1. Family History - Failure to obtain family history of breast cancer
4
Issues - Under-weighing family history of breast cancer
- Underestimating risk of BC in young symptomatic
2. Atypical patients
- Fast-growing cancers arising during MMG interval
Presentation/
6 - Under-weighing complaints of patients with
Cognitive psychiatric diagnoses
Challenges - Prioritizing chronic medical or social issues over
screenings in complex patients
- Lump felt to be benign on physical exam
3. False Negative
2 - Bias in wanting to reassure patient, due to low
Physical Exam likelihood of BC
- Fibrocystic breast tissue can obscure underlying BC
in MMG
4. Fibrocystic/Dense - Not recognizing changes in breast density over time
9
Breast Dilemmas - Failure to investigate unilateral fibrocystic changes
- Failure to investigate breast lump with FNA in
patient with dense breasts and negative U/S

3728
Copyright © Harvard Medical School, 2018. All Rights Reserved.

BREAST CANCER PITFALLS: MALPRACTICE CASES

Pitfall N Example
5. Screening vs.
- Ordering/performing a screening MMG, rather than
Diagnostic 2
a diagnostic MMG
Mammogram Order
- False negative MMG in pt with fibrocystic breasts
- Failure to reevaluate breast complaints in light of
previously negative MMG
6. False Negative
9 - Misreading of MMG by radiologists
Mammogram - Failure to follow-up on nipple retraction observed
on MMG, attributing it to imaging technique
- Falsely reassuring negative “additional views”
7. False Negative - Falsely reassuring negative U/S in pts with breast
2
Ultrasound lump
- Failure to refer to breast surgeon
8. Surgical Referral 4 - Breast lump appearing benign to surgeon palpation
- Patient failure to follow-up on referral

Schiff et al. Unpublished data Coverys/CRICO Closed Claims review 2016

BREAST CANCER PITFALLS: MALPRACTICE CASES

Pitfall N Example
9. Biopsy Performance/ - Inability to recognize missed sampling due to
1
Interpretation bleeding/complications and failure to repeat biopsy
- Failure to order diagnostic imaging studies (MMG
10. Failure to Order
2 and U/S)
Further Studies - Failure to recommend excisional biopsy
11. Diffusion of - Failure to document/ensure pt was receiving
screening MMGS and breast exams
Responsibility/ 4
- Failed coordination/communication between PCP
Coordination Issues and GYN
- Failure to follow-up on resolution of mastitis
- Failure to pursue etiology of persistent galactorrhea
- Pursuing lymphoma as cause of lymphadenopathy
12. Other Symptoms 8
- Axillar lymphadenopathy lost due to fact that not
incorporated into BIRADS coding (revised now)
- Failure to work up persistent painful cyst

Schiff et al. Unpublished data Coverys/CRICO Closed Claims review 2016

3729
Copyright © Harvard Medical School, 2018. All Rights Reserved.

Diagnostic Situational Awareness Model

Diagnostic Risk
Safety Nets
• Recognizing inherent uncertainties/risks, build
in mitigation, protections, recovery structures
and processes
• Proactive, systematic follow-up, feedback via
closed loop systems
• Major role for HIT to hard-wire
– To automate, ensure reliability, ease burden on
staff/memory, ensure loops closed and outliers
visible

40

3730
Copyright © Harvard Medical School, 2018. All Rights Reserved.

41

El-Kareh
Schiff
BMJ QS 2013

42

3731
Copyright © Harvard Medical School, 2018. All Rights Reserved.

43

Clinical Documentation

CYA

3732
Copyright © Harvard Medical School, 2018. All Rights Reserved.

Canvass for
Your
Assessment

-Differential Diagnosis
-Weighing Likelihoods
-Etiology Canvass for
-Urgency
-Degree of Your
certainty
Assessment

3733
Copyright © Harvard Medical School, 2018. All Rights Reserved.

Open Loop System

Water goes on the


same time each day,
regardless of whether
it is raining or lawn is
flooded

Schiff A J Med 2008


47

48

3734
Copyright © Harvard Medical School, 2018. All Rights Reserved.

49

55/338 (16%) not improved


of whom only 21 (38%)
had contacted any clinician

50

3735
Copyright © Harvard Medical School, 2018. All Rights Reserved.

Feedback –Key Role in Safety


• Structural commitment patient role to play
• Embodies/conveys message: uncertainty, caring,
reassurance, access if needed
• Allows deployment of test of time, more conservative
diagnosis
• Enables differential diagnosis
• Emphasizes that disease is dynamic
• Reinforces culture of learning & improvement
• Illustrates how much disease is self limited
• Makes invisible missed diagnoses visible

51

Examples of Feedback Learning


Feeding back to upstream hospital Tracking persistent mysteries
- spinal epidural abscess
Chart correction by patients
IVR follow-up post urgent care visit
Radiology/pathology
- UAB Berner project
- systematic second reviews
Dedicated Dx Error M&M
2nd opinion cases
- Best Doctors dx changed
Autopsy Feedback
- 7/32 MDs aware disseminated CMV Linking lab and pharmacy data
- to find signal of errors (missed ↑ TSH)
ED residents post admission tracking
Urgent care
Feedback to previous service - call back f/up systems

Malpractice
52
- knock on the door

3736
Copyright © Harvard Medical School, 2018. All Rights Reserved.

Feedback- Challenges
• Effort, time, support required
• Discontinuities
• Can convey non-reassuring message
• Feedback fatigue
• Non-response not always good predictor of
misdiagnosis as multiple confounders
• Tampering – form of availability bias

53

Role for Patient


In Minimizing and Preventing Diagnosis Error and Delay
• Push for timely access • Being patient: time & tests
• Reliable follow-up, continuity • Recruiting family for support
• Keen observer, reporter sx • Respecting limits on staff time,
• Proactive on test results society resources
• Sharing hunches • Agreeing to disagree
• Curiously reading on own • Help in building, maintaining
• Meticulously adhering w/ trust and communication
empiric trial regimens • Getting involved with patient
• Active as co-investigator organizations

54

3737
Copyright © Harvard Medical School, 2018. All Rights Reserved.

Role for Patient


In Minimizing and Preventing Diagnosis Error and Delay
• Push for timely access • Being patient: time & tests
• Reliable follow-up, continuity • Recruiting family for support
• Keen observer, reporter sx • Respecting limits on staff time,
• Proactive on test results society resources
• Sharing hunches • Agreeing to disagree
• Curiously reading on own • Help in building, maintaining
• Meticulously adhering w/ trust and communication
empiric trial regimens • Getting involved with patient
• Active as co-investigator organizations

Key question is:


What will it take at the provider and institutional end
to support these roles and help them flourish? 55
Ricardo Levins Morales Art Studio

56

3738
Copyright © Harvard Medical School, 2018. All Rights Reserved.

Summary Question #1
• Which of the following are examples of
“Situational Awareness” constructs that hold
potential for helping clinicians and patients
anticipate, recognize, and/or prevent diagnostic
errors
A. Red flag symptoms
B. Context-relevant lists of Don’t Miss diagnoses
C. Just-in-time reminders about potential pitfalls
D. All of the above

Question #1 Answer Slide


• Answer is D- All of the above
• Red flag symptoms, context-relevant lists of
don’t miss diagnoses, and reminders about
potential pitfalls can help the clinician in real
time with being more aware of critical factors
to consider or mistakes to try to avoid

3739
Copyright © Harvard Medical School, 2018. All Rights Reserved.

Summary Question #2
Which of the following statements regarding diagnostic
errors is not true:

A. Patient surveys reveal at least 1 in 5 have experienced a


medical error, the majority of which are diagnostic errors
B. The best way to decrease diagnostic errors is to order more
tests to screen for all possible diagnosis
C. Improving patient follow-up by phone to check how symptoms
are evolving is an example enhanced follow-up and feedback
D. Better clinical documentation goes beyond “CYA”, and
includes improving recording of differential diagnosis, degree of
certainty, follow-up contingencies

Question #2 Answer Slide


• Answer is B- Order lots of tests.
This is not a good idea owing to the fact that
unselected tests without proven screening value in
low probability situations/populations are highly
vulnerable to false positive errors.
It is much better to focus on the patient, their history
and more likely diagnoses. This more conservative
and appropriate approach also avoids overdiagnosis
errors and uncovering low risk incidental-omas.
• All the others choices are true.

3740
Copyright © Harvard Medical School, 2018. All Rights Reserved.

Further Reading/Resources
• National Academy of Medicine: Improving Diagnosis in
Health Care Report. Free online viewing. Especially
recommend Executive Summary pp. 1-18.
• AHRQ PSNet Website Topics Safety Target Diagnostic
Errors. Up-to-date collection of articles on Dx Error.
• Society for Improving Diagnosis in Medicine (SIDM)
Website, International Conferences, resources
• Schiff & Ruan. The Elusive and Illusive Quest for Diagnostic
Safety Metrics. Jl of Gen Internal Med 2018
• Schiff. Diagnostic Error: Time for a New Paradigm.
BMJ Quality and Safety 2013

DIAGNOSTIC ERROR IN MEDICINE 11TH


INTERNATIONAL CONFERENCE NEW ORLEANS

3741
Copyright © Harvard Medical School, 2018. All Rights Reserved.

Supplemental
Slides

JGIM 7/18

3742
Copyright © Harvard Medical School, 2018. All Rights Reserved.

Culture of Diagnostic Safety & Improvement

1. Driving out fear so no one afraid to ask questions,


question a diagnosis, share when things go wrong
– Dealing w/ adverse events replacing blame & fear, w/ learning & improvement
2. Organization-wide commitment to improving diagnosis,
learning from diagnosis delays, diagnostic process errors
– Leadership/organizational recognition that misdiagnosis is the
#1 top cause of patient-reported errors
– Aggressive reporting, appreciative investigation, of adverse events
– Relentless curiosity/worry/conferencing: what is wrong with patient; what
might be missing, what can go wrong in system?
– Obsession w/ details of dx process: what can go wrong, limitations of tests

Culture of Diagnostic Safety & Improvement


3. Recognition uncertainty inherent in diagnoses, tests, illness
presentation and evolution; anticipation of common pitfalls
– Situational awareness local, disease specific, literature reported
vulnerabilities/pitfalls.
– Reliable, proactive, follow-up safety nets & feedback systems to detect and protect
– Conservative approaches to testing, imaging
• Enabled by shared decision-making and reliable follow-up

4. Respect human limitations, need for cognitive, process support


– Decreased reliance on human memory, minimizing negative effects of stress,
fatigue, fear, recognizing limited ability to truly multitask.
– Redesign EMRs & communication systems to support cognition, collaborative
diagnosis, and follow-up
5. Enhanced role for patient in co-producing diagnosis
– Working collaboratively to formulate history, diagnosis, monitor course,
raise and research questions

3743
Copyright © Harvard Medical School, 2018. All Rights Reserved.

Evaluation of Cardiac and Pulmonary Risk in the Preop


Patient

Adam C. Schaffer, MD, MPH


Associate Physician, Hospital Medicine Unit, Brigham and
Women‘s Hospital
Instructor, Harvard Medical School

Conflicts of Interest
I have no conflicts of interest to declare

3744
Copyright © Harvard Medical School, 2018. All Rights Reserved.

Clinical Case
76-year-old male with severe COPD, on 3 liters of home
oxygen and chronic prednisone 7.5 mg daily, DMII on
metformin, dyspnea with minimal exertion.
He has no history of MI or CHF. His EKG is essentially
normal.
He has metastatic colon cancer, with a single metastasis
to the brain causing left arm weakness
You are seeing him on the office for a preop evaluation
prior to neurosurgery scheduled for the following week to
resect the metastasis causing the left arm weakness
He underwent successful resection of a colon mass
three years ago

Introduction
The role of the clinician performing preoperative
evaluations is not to provide medical “clearance”
prior to surgery
Instead, the clinician should:
– Provide an assessment of the patient’s cardiac and
other risks going into the procedure
– Decide whether additional preoperative testing, such
as a cardiac stress test, is needed
– When indicated, recommend measures to reduce the
perioperative risk, such as beta blockers and statins
– Assist the surgeon in deciding whether to go forward
with the procedure

3745
Copyright © Harvard Medical School, 2018. All Rights Reserved.

New Preop Guidelines were


Published in 2014

ACC/AHA 2007 Periop Guidelines

Source: Fleisher LA, et al. ACC/AHA 2007 guidelines…. Circulation. Oct 23 2007;116(17):e418-499.

3746
Copyright © Harvard Medical School, 2018. All Rights Reserved.

ACC/AHA 2014 Periop Guidelines

Fleisher LA, Fleischmann KE, Auerbach AD, et al. ACC/AHA Clinical Practice Guideline 2014 ACC/AHA ...Circulation. published
online before print August 1, 2014.

Comparison of the old and the


new guidelines

3747
Copyright © Harvard Medical School, 2018. All Rights Reserved.

Risk assessment in the


New Guidelines
For risk assessment, the new guidelines recommend
estimating the preoperative risk of a major adverse
cardiac event (MACE), which here are defined as death
or MI
The risk of MACE is a function of the both the risk
associated with the procedure and the risk associated
with the patient
If there is a low risk of MACE, which is defined as < 1%,
then one goes to surgery

Risk assessment in the


New Guidelines
The new guidelines suggest three way to
determine if the MACE is < 1% or not:
1. ACS NSQIP Surgical Risk Calculator
(http://www.riskcalculator.facs.org/)
2. Perioperative Cardiac Risk Calculator
(http://www.surgicalriskcalculator.com/miorcardi
acarrest)
3. RCRI score (though one of the two options
above is preferred because they outperform the
RCRI score)

3748
Copyright © Harvard Medical School, 2018. All Rights Reserved.

ACS NSQIP
Surgical Risk Calculator

Cohen ME, Ko CY, Bilimoria KY, et al. Optimizing ACS NSQIP modeling for evaluation of surgical quality and risk: .... Journal of the
American College of Surgeons. Aug 2013;217(2):336-346.e331.

ACS NSQIP
Surgical Risk Calculator

Cohen ME, Ko CY, Bilimoria KY, et al. Optimizing ACS NSQIP modeling for evaluation of surgical quality and risk: .... Journal of the
American College of Surgeons. Aug 2013;217(2):336-346.e331.

3749
Copyright © Harvard Medical School, 2018. All Rights Reserved.

Gupta Perioperative Cardiac


Risk Calculator

Gupta PK, Gupta H, Sundaram A, et al. Development and validation of a risk calculator for prediction of cardiac risk after surgery.
Circulation. Jul 26 2011;124(4):381-387.

Risk Assessment: RCRI


Risk Factor Definition

1. High-risk type of surgery Intraperitoneal, intrathoracic, or suprainguinal


vascular procedures
2. Ischemic heart disease History of MI, positive stress test, current
cardiac CP, nitrate usage, ECG with pathologic
Q waves
3. History of congestive heart failure History of CHF, pulmonary edema, or PND;
rales or S3 on exam; chest x-ray with
pulmonary edema
4. History of cerebrovascular disease History of transient ischemic attack or stroke
5. Insulin therapy for diabetes

6. Preoperative serum creatinine > 2.0


mg/dL

“A patient with 0 or 1 [RCRI] predictor(s) of risk would have a low


risk of MACE. Patients with ≥ 2 predictors of risk would have
elevated risk.”

Source: Lee TH, Marcantonio ER, Mangione CM, et al. Derivation and prospective validation of a simple index for prediction of
cardiac risk of major noncardiac surgery. Circulation. Sep 7 1999;100(10):1043-1049.

3750
Copyright © Harvard Medical School, 2018. All Rights Reserved.

The new preop evaluation guidelines


Under the new
guidelines, if your risk
of MACE is low
(< 1%), then you go
to surgery
If you risk is elevated
(≥ 1%), then you
consider the patient’s
functional capacity

The new preop evaluation guidelines


Under the new
guidelines, if your
functional
capacity is ≥ 4
METs, then you
proceed to
surgery
This is similar to
the old guidelines

3751
Copyright © Harvard Medical School, 2018. All Rights Reserved.

The new preop evaluation guidelines


In a patient whose
MACE risk is ≥ 1%
and whose functional
capacity is < 4 METs,
under the new
guidelines, you would
consider a stress test
if it would alter
decision making

The new preop evaluation guidelines


Consider the following patient: a 78 y.o. M with a
hx of ESRD on HD, MI, HTN, current smoker
with COPD, who is largely wheelchair bound due
to severe OA, is scheduled for a knee
replacement
Under the 2007 guidelines, this patient has an
RCRI score of 2, and is undergoing intermediate
risk surgery, and so no stress test is
recommended

3752
Copyright © Harvard Medical School, 2018. All Rights Reserved.

The new preop evaluation guidelines


Consider the following patient: a 78 y.o. M with a
hx of ESRD on HD, MI, HTN, current smoker
with COPD, who is largely wheelchair bound due
to severe OA, is scheduled for a knee
replacement
Under the 2014 guidelines, this patient has a
MACE risk of ≥ 1% (using his RCRI ≥ 2 or the
risk calculator) and he does not make 4 METs,
thus the guidelines suggest considering a stress
test if it will alter management

Why are we now considering stress tests


in more patients under the new guidelines?
The DECREASE-II Trial (2006) by Poldermans
et al found no benefit to preop stress testing,
with revascularization if positive, in intermediate
risk patients undergoing vascular surgery
The DECREASE-V Pilot Study (2007) by
Poldermans et al found no benefit to preop
revascularization in high-risk patients (with
positive stress tests) undergoing vascular
surgery

3753
Copyright © Harvard Medical School, 2018. All Rights Reserved.

Why are we now considering stress


tests in more patients under the new
guidelines?

Concerns have been raised about the scientific integrity


of the studies published by Poldermans group
The authors of the 2014 Guidelines said evidence from
the DECREASE trials, conducted by Poldermans’s
group, would not form the basis of any recommendations

Preoperative ECGs in the


New 2014 ACC/AHA Guidelines
Class IIa: Preoperative resting 12-lead ECG is
reasonable for patients with known coronary heart
disease, significant arrhythmia, peripheral arterial
disease, cerebrovascular disease, or other significant
structural heart disease, except for those undergoing
low-risk surgery
Class IIb: Preoperative resting 12-lead ECG may be
considered for asymptomatic patients without known
coronary heart disease, except for those undergoing low-
risk surgery
Class III: Routine preoperative resting 12-lead ECG is
not useful for asymptomatic patients undergoing low-risk
surgical procedures

3754
Copyright © Harvard Medical School, 2018. All Rights Reserved.

Perioperative Beta Blockers:


2014 Recommendations
Perioperative beta blockade appears to be of
benefit in selected patients who are at elevated
risk of perioperative cardiac events
Per the ACC/AHA 2014 Periop Guidelines,
there is one class I indication for perioperative
beta-blocker use:
– “Beta blockers should be continued in
patients undergoing surgery who have been
on beta blockers chronically”
What to do in patients who are not
already on beta blockers in controversial

Perioperative Beta Blockers:


The POISE Trial
The PeriOperative ISchemic Evaluation (POISE) Trial enrolled 8351
patients undergoing noncardiac surgery with at least one cardiac
risk factor
Patients were randomized to either placebo or controlled-release
metoprolol (CR metoprolol) 100 mg orally 2 – 4 hours prior to
surgery, a postoperative dose of CR metoprolol based on heart rate
and blood pressure, and then 200 mg of CR metoprolol orally daily
for the next 30 days
The beta blocker arm had a lower rate of the primary outcome (30-
day cardiac events): 5.8% in the beta blocker arm versus 6.9% in
the placebo arm (P=0.04)
However, the total mortality was higher in the CR metoprolol group
(3.1%) than in the placebo group (2.3%) (P=0.03)
The higher mortality in the beta blocker arm was driven by an
increase in the stroke rate in this group
The general view of this trial is that the dose of periop beta blockers
given was too large, and so led to the increased stroke rate

Source: Poise Study Group. Effects of extended-release metoprolol succinate in patients undergoing non-cardiac surgery (POISE
trial)…. Lancet. May 31 2008;371(9627):1839-1847.

3755
Copyright © Harvard Medical School, 2018. All Rights Reserved.

Perioperative Beta Blockers:


How (not) to Use them

“Routine administration of high-dose beta


blockers in the absence of dose titration is not
useful and may be harmful to patients not
currently taking beta blockers who are
undergoing noncardiac surgery”

Source: Fleisher LA, et al. 2009 ACCF/AHA focused update on perioperative beta blockade incorporated into the ACC/AHA 2007
guidelines on perioperative cardiovascular… . Circulation. Nov 24 2009;120(21):e169-276.

Perioperative Beta Blockers:


Retrospective Data
Lindenauer et al. performed a large retrospective
cohort study examining the benefits of periop beta
blockers based on the cardiac risk of the patient
Patients undergoing noncardiac surgery (mainly
orthopedic and abdominal procedures), both routine
and emergent, were included
Patients receiving prophylactic periop beta blockers
were compared with patients not receiving beta
blockers
This study is debated:
– On the one hand, it was quite large (n=663,635)
– On the other hand, it was retrospective, and based on the use
of an administrative database. No charts were reviewed. Beta
blockers started on hospital day 1 or 2 were considered
prophylactic.

Source: Lindenauer PK, et al. Perioperative beta-blocker therapy …. New England Journal of Medicine. Jul 28 2005;353(4):349-361.

3756
Copyright © Harvard Medical School, 2018. All Rights Reserved.

Perioperative Beta Blockers:


Retrospective Data
Patients without cardiac risk factors who got
periop beta blockers seemed to be harmed by
them
Patients with an RCRI of at least 2, and
certainly with an RCRI of 3, appeared to
benefit from beta blockers

Source: Lindenauer PK, et al. Perioperative beta-blocker therapy …. New England Journal of Medicine. Jul 28 2005;353(4):349-361.

Perioperative Beta Blockers:


2014 ACC/AHA Recommendations
The 2014 AHA guidelines are offer mainly IIb
recommendations about when to start periop beta
blockers in those who are not on them
In patients with an RCRI score of 3 or more, it may be
reasonable to begin beta blockers prior to surgery
(class IIb recommendation)
“In patients with a compelling long-term indication for
beta-blocker therapy but no other RCRI risk factors,
initiating beta blockers in the perioperative setting as
an approach to reduce perioperative risk is of
uncertain benefit”
Beta-blocker therapy should not be started on the day
of surgery (class III recommendation)

3757
Copyright © Harvard Medical School, 2018. All Rights Reserved.

Perioperative Beta Blockers:


Which One to Use
One retrospective study from April 2011 examined
perioperative atenolol versus metoprolol
3787 high-risk patients undergoing inpatient surgery at
the San Francisco VAMC were included
1011 patients received atenolol and 2776 patients
received metoprolol
Patients who received atenolol, versus metoprolol, had
lower 30-day mortality (1% versus 3%, P < 0.0008) and
lower 1-year mortality (7% versus 13%, P < 0.0001)
This was thought to be due to the risk of missing doses
of the shorter-acting metoprolol, with resultant
beta-blocker withdrawal effect

Wallace AW, Au S, Cason BA. Perioperative β-blockade: atenolol is associated with reduced mortality when compared to metoprolol.
Anesthesiology. Apr 2011;114(4):824-836.

Periop Beta Blockers


Take Home Points
In patients who are already on beta blockers,
continue them on beta blockers perioperatively
You want to avoid beta blocker withdrawal
In patients not already on beta blockers with an
RCRI score of ≥ 3 “it may be reasonable to
begin beta blockers before surgery”
If beta blocker are being started in preparation
for surgery, you want to start them well ahead of
surgery and not on the day of surgery

3758
Copyright © Harvard Medical School, 2018. All Rights Reserved.

Perioperative Statins
The DECREASE-III trial enrolled 497 patients, age > 40,
at elevated cardiac risk, scheduled to undergo
noncardiac vascular surgery
All patients had to be statin naïve
All patients were on beta blockers
– Patients who were already taking a beta blocker were continued
on this beta blocker
– Patients who were not on a beta blocker were started on one,
and their dose was titrated based on their HR
Patients were randomized to fluvastatin 80 mg daily or a
placebo. This statin was started on average 37 days
prior to surgery and continued for at least 30 days after
surgery

Source: Schouten O, Boersma E, Hoeks SE, et al. Fluvastatin and perioperative events in patients undergoing vascular surgery. New
England Journal of Medicine. Sep 3 2009;361(10):980-989.

DECREASE III Trial

Perioperative Myocardial Ischemia: Perioperative death from CV cause or MI:


10.8% in the statin arm vs. 19.0% in the 4.8% in the statin arm vs. 10.1% in the
placebo arm (P = 0.01) placebo arm (P = 0.03)

Placebo

Placebo

Fluvastatin

Fluvastatin

Source: Schouten O, Boersma E, Hoeks SE, et al. Fluvastatin and perioperative events in patients undergoing vascular surgery. New
England Journal of Medicine. Sep 3 2009;361(10):980-989.

3759
Copyright © Harvard Medical School, 2018. All Rights Reserved.

Perioperative Statins:
2014 ACC/AHA Recommendations
Statins should be continued in patients currently
taking statins and scheduled for noncardiac
surgery (class I)
Perioperative initiation of statin use is
reasonable in patients undergoing vascular
surgery (class IIa)
Perioperative initiation of statins may be
considered in patients with clinical indications
according to GDMT who are undergoing
elevated-risk procedures (class IIb)

Perioperative Aspirin
The POISE 2 Trial, an RCT published in the
NEJM in April 2014, looked at the effect of
perioperative ASA
The trial enrolled 10,010 patients undergoing
noncardiac surgery who were at risk for vascular
complications
Patient within the coronary stent critical periods
were excluded
The primary endpoint was death or nonfatal MI
at 30 days
Devereaux PJ, Mrkobrada M, Sessler DI, et al. Aspirin in patients undergoing noncardiac surgery. New England Journal of
Medicine. Apr 17 2014;370(16):1494-1503.

3760
Copyright © Harvard Medical School, 2018. All Rights Reserved.

Perioperative Aspirin
The patients were stratified by whether they were
already taking ASA (continuation group) or not (initiation
group)
There was no benefit to ASA in the primary outcome or
any of the secondary outcomes
The negative results were the same for the continuation
group and the initiation group
Taking ASA was associated with an increased risk of
major bleeding
Starting at POD#8, there was no significant difference in
the bleeding risk between and ASA and placebo groups

Devereaux PJ, Mrkobrada M, Sessler DI, et al. Aspirin in patients undergoing noncardiac surgery. New England Journal of
Medicine. Apr 17 2014;370(16):1494-1503.

Perioperative Aspirin

Devereaux PJ, Mrkobrada M, Sessler DI, et al. Aspirin in patients undergoing noncardiac surgery. New England Journal of
Medicine. Apr 17 2014;370(16):1494-1503.

3761
Copyright © Harvard Medical School, 2018. All Rights Reserved.

Perioperative Clonidine
As a companion to the periop ASA trial, there was a
parallel periop clonidine trial
The trial, as RCT, enrolled 10,010 patients undergoing
noncardiac surgery who were at risk for vascular
complications
The primary endpoint was death or nonfatal MI at 30
days
There was no benefit to periop clonidine in reducing the
primary endpoint
Patients in the clonidine arm had an increase the risk of
clinically important hypotension and nonfatal cardiac
arrest.
Devereaux PJ, Sessler DI, Leslie K, et al. Clonidine in patients undergoing noncardiac surgery. New England Journal of Medicine.
Apr 17 2014;370(16):1504-1513.

Bridging Anticoagulation
The BRIDGE trial randomized 1884
patients with Afib on coumadin who were
scheduled for an elective procedure to
either bridging with LMWH (dalteparin) or
placebo.
Patients had to have at least 1 of the
CHADS2 risk factors.
The mean CHADS2 score was 2.3

Source: Douketis JD, Spyropoulos AC, Kaatz S, et al. Perioperative Bridging Anticoagulation in Patients with Atrial
Fibrillation. New England Journal of Medicine. June 22, 2015 (published on-line ahead of print).

3762
Copyright © Harvard Medical School, 2018. All Rights Reserved.

Bridging Anticoagulation

Source: Douketis JD, Spyropoulos AC, Kaatz S, et al. Perioperative Bridging Anticoagulation in Patients with Atrial
Fibrillation. New England Journal of Medicine. June 22, 2015 (published on-line ahead of print).

Bridging Anticoagulation

Source: Douketis JD, Spyropoulos AC, Kaatz S, et al. Perioperative Bridging Anticoagulation in Patients with Atrial
Fibrillation. New England Journal of Medicine. June 22, 2015 (published on-line ahead of print).

3763
Copyright © Harvard Medical School, 2018. All Rights Reserved.

Bridging Anticoagulation

Source: Douketis JD, Spyropoulos AC, Kaatz S, et al. Perioperative Bridging Anticoagulation in Patients with Atrial
Fibrillation. New England Journal of Medicine. June 22, 2015 (published on-line ahead of print).

Perioperative Pulmonary Complications


Tested on 5,859 patients
in 63 centers
Respiratory complications
were defined as:
– Respiratory infection or
failure
– Bronchospasm
– Atelectasis
– Pleural effusion
– Pneumothorax
– Aspiration pneumonitis
Score:
– < 26 denotes a 3.4% risk
– 26-45 denotes a 13.0% risk
– >45 denotes a 38.0% risk

Mazo V, Sabate S, Canet J, et al. Prospective external validation of a predictive score for postoperative pulmonary complications.
Anesthesiology. 2014;121(2):219-231.

3764
Copyright © Harvard Medical School, 2018. All Rights Reserved.

Perioperative Pulmonary Risk


Reduction Strategies: Lung Expansion
In patients at elevated risk, such as those undergoing
abdominal surgery, a lung expansion maneuver is
appropriate, and is more effective than no intervention
Options include incentive spirometry, lung expansion
exercises, and continuous positive airway pressure
There is no compelling evidence favoring one lung
expansion intervention over another
Continuous positive airway pressure may be appropriate
in patients who are unable to undergo either incentive
spirometry or lung expansion exercises. CPAP is
advisable in OSA patients.

Source: Lawrence VA, et al. Strategies to reduce postoperative pulmonary complications after noncardiothoracic surgery: systematic
review for the American College of Physicians. Annals of Internal Medicine. Apr 18 2006;144(8):596-608.

Perioperative Pulmonary Risk


Reduction Strategies
Smoking cessation
– May help reduce the incidence of postop pulmonary
complications
– However, smoking cessation immediately (< 8 weeks) prior to
surgery may increase the risk of postop pulmonary complications
Anesthesia techniques need to be considered
– Patients who had residual muscle blockade after receiving the
long-acting neuromuscular-blocking agent pancuronium had an
increased rate of postop pulmonary complications compared to
patients without residual muscle blockade
– Use of either spinal or epidural anesthesia, as compared to
general anesthesia, may also reduce the incidence of postop
pulmonary complications

Source: Lawrence VA, et al. Strategies to reduce postoperative pulmonary complications after noncardiothoracic surgery: systematic
review for the American College of Physicians. Annals of Internal Medicine. Apr 18 2006;144(8):596-608.

3765
Copyright © Harvard Medical School, 2018. All Rights Reserved.

The Timing of Surgery for Hip


Fracture Patients
Retrospective cohort analysis of 42,230 hip fracture
patients in Ontario
Mean age around 80, around 70% female
Primary outcome was 30-d mortality

Source: Pincus D, Ravi B, Wasserstein D, et al. Association Between Wait Time and 30-Day Mortality in Adults Undergoing Hip
Fracture Surgery. JAMA. Nov 28 2017;318(20):1994-2003..

Periop Issues for OSA Patients


Both respiratory and cardiovascular periop
complications are more common in
patients with untreated OSA than
comparable patients without OSA
Prescribing CPAP for patients with OSA
perioperatively reduced the risk of
cardiovascular but not pulmonary
complications

Mutter TC, Chateau D, Moffatt M, Ramsey C, Roos LL, Kryger M. A matched cohort study of postoperative outcomes in
obstructive sleep apnea: could preoperative diagnosis and treatment prevent complications? Anesthesiology. Oct
2014;121(4):707-718.

3766
Copyright © Harvard Medical School, 2018. All Rights Reserved.

Periop Issues for Obese Patients


There are an increasing number of case reports
of obese patients having postop rhabdomyolysis
after surgery
It is likely that the immobilization and weight on
the gluteal muscle results in necrosis
If an obese patient has postop AKI, consider
rhabdomyolysis and check a CK
Obese patients has restrictive lung physiology,
and so consider CPAP in hypoxic obese patients
postop

The Value of
Medical Consultation Itself
A retrospective cohort study, using an
administrative database, included almost
270,000 patients undergoing elective
intermediate- or high-risk noncardiac surgery
38.8% of these patients underwent medical
consultation
Propensity scores were used to produce a
matched-pairs cohort that reduced differences
between patients who did and did not undergo
preoperative consultation

Source: Wijeysundera et al. Outcomes and processes of care related to preoperative medical consultation. Archives of Internal
Medicine. Aug 9 2010;170(15):1365-1374.

3767
Copyright © Harvard Medical School, 2018. All Rights Reserved.

The Value of
Medical Consultation Itself
Undergoing medical consultation was
associated with:
– Increased new beta-blocker usage (95% CI:
2.36-2.65)
– Increased new statin usage (95% CI: 1.34-
1.54)
– Increased preop cardiac stress testing (95%
CI: 2.33-2.47)
– Increased 30-day mortality (95% CI: 1.07-1.25)
– Increased 1-year mortality (95% CI: 1.04-1.12)
Source: Wijeysundera et al. Outcomes and processes of care related to preoperative medical consultation. Archives of Internal
Medicine. Aug 9 2010;170(15):1365-1374.

The Value of
Medical Consultation Itself
Possible conclusions to draw from this study:
– Using propensity scores does not always give you
matched cohorts
– We do not do a good job selecting which patients
should undergo medical consultation
– We do not do a good job with medical consultation
itself

Source: Wijeysundera et al. Outcomes and processes of care related to preoperative medical consultation. Archives of Internal
Medicine. Aug 9 2010;170(15):1365-1374.

3768
Copyright © Harvard Medical School, 2018. All Rights Reserved.

Clinical Case
76-year-old male with severe COPD, on 3 liters of home
oxygen and chronic prednisone 7.5 mg daily, DMII on
metformin, dyspnea with minimal exertion.
He has no history of MI or CHF. His EKG is essentially
normal.
He has metastatic colon cancer, with a single metastasis
to the brain causing left arm weakness
You are seeing him on the office for a preop evaluation
prior to neurosurgery scheduled for the following week to
resect the metastasis causing the left arm weakness
He underwent successful resection of a colon mass
three years ago

Clinical Case

1.8%

3769
Copyright © Harvard Medical School, 2018. All Rights Reserved.

Clinical Case
What actually happened:
– The neurosurgeon cancelled the case
– The patient was scheduled for brain XRT
instead
– Surgery remains on the table as an option
If the patent undergoes surgery, consider cort
stim versus stress dose steroids

Multiple Choice Question 1


A 71-year old male with a history of
hypertension, diabetes mellitus, and atrial
fibrillation on warfarin is scheduled for a
left knee arthroplasty.

3770
Copyright © Harvard Medical School, 2018. All Rights Reserved.

Multiple Choice Question 1


Which one of the following is the preferred
recommendation for handling his anticoagulation
perioperatively?
A. He should receive bridging anticoagulation with low
molecular weight heparin
B. He should receive bridging anticoagulation with
unfractionated heparin
C. He should receive bridging anticoagulation with
fondaparinux
D. His warfarin should be continued
E. No bridging anticoagulation is indicated

Multiple Choice Question 1


The correct answer is (E), no bridging
anticoagulation is indicated. The BRIDGE trial
showed no benefit (and increased bleeding) with
bridging anticoagulation with low molecular
weight heparin in patients with a CHADS2 score
of 2.3, which is similar to this patient (who has a
CHADs 2 score of 2). Bridging with other agents
would be expected to have a similarly
unfavorable risk-benefit profile.

3771
Copyright © Harvard Medical School, 2018. All Rights Reserved.

Multiple Choice Question 2


You are performing a pre-operative
evaluation on a 77-year-old male with a
history of diabetes mellitus on insulin,
ESRD, and hypertension for which the
patient takes metoprolol. He is scheduled
for partial colon resection for recurrent
diverticulitis. His MACE risk is 2.1%. He
can walk up two flights of stairs briskly
without difficulty.

Multiple Choice Question 2


Which one of the following is the most
appropriate preoperative recommendation in this
patient?
A. His metoprolol should be changed to clonidine
B. No preoperative stress test is indicated
C. He should undergo a resting cardiac echo
D. He should undergo a exercise nuclear stress
test
E. He should undergo a pharmacologic nuclear
stress test

3772
Copyright © Harvard Medical School, 2018. All Rights Reserved.

Multiple Choice Question 2


The correct answer is (B), no preoperative stress test is
indicated.
The patient has a MACE risk ≥ 1%, but his functional
capacity is > 4 METs, and so no preop stress test is
indicated. Thus answers (D) and (E) are incorrect. The
patient has no symptoms of valvular or other heart
disease that would support a cardiac echo, making
answer (C) incorrect. Clonidine is not effective in preop
cardiac risk reduction, and so answer (A) is incorrect.

Key References
1. Fleisher LA, Fleischmann KE, Auerbach AD, et al. 2014 ACC/AHA guideline on
perioperative cardiovascular evaluation and management of patients undergoing
noncardiac surgery: a report of the American College of Cardiology/American Heart
Association Task Force on Practice Guidelines. Circulation. Dec 9
2014;130(24):e278-333.
2. Cohen ME, Ko CY, Bilimoria KY, et al. Optimizing ACS NSQIP modeling for
evaluation of surgical quality and risk: patient risk adjustment, procedure mix
adjustment, shrinkage adjustment, and surgical focus. Journal of the American
College of Surgeons. Aug 2013;217(2):336-346.e331.
3. POISE Study Group, Devereaux PJ, Yang H, et al. Effects of extended-release
metoprolol succinate in patients undergoing non-cardiac surgery (POISE trial): a
randomised controlled trial. Lancet. May 31 2008;371(9627):1839-1847.
4. Devereaux PJ, Mrkobrada M, Sessler DI, et al. Aspirin in patients undergoing
noncardiac surgery. New England Journal of Medicine. Apr 17 2014;370(16):1494-
1503.
5. Douketis JD, Spyropoulos AC, Kaatz S, et al. Perioperative Bridging
Anticoagulation in Patients with Atrial Fibrillation. New England Journal of Medicine.
August 27 2015;373(9):823-833.

3773

Das könnte Ihnen auch gefallen